You are on page 1of 1106

SELF HELP

ry to achieve goals and sustain success in life.


There is a story that illustrates this fact.

A very elderly and a young man were immediate neighbors with only a wall separating their
houses,
.
They planted two identical plants, one each in their. respective
.
premises and. beyond to
nurture them. While the young man supplied a lot of water and plenty of fertilizer to his plant,
the elderly man supplied minimum water fertilizer to the plant in his premises.

After a few months, the young man's plant became a beautiful sight with lovely green
leaves. The elderly man plants, though not so beautiful, grew in to a strong structure.

One night there was a storm with heavy winds and rain. The next day, the two men came
to check the condition of their respective plants. The young man was shocked to see that while
his plant was totally uprooted, the elderly man's plant was unscathed and standing straight.
When the young man asked the elderly man how this could have happened, the old man replied,
"Son, you supplied the tree with more than resources it needed and so, it never felt the need to go
deep in to earth for its survival. On the other hand, I gave my plant only minimum supply of
resources so that it could barely survive. So to strengthen itself, it had to spread its roots deep in
to earth and by doing so it developed its own strength. This could enable it to resist the winds
and stand strongly. Increasing one's ability depends on one's own effort and what other do for us
provides only a good base".

This story should be an eye opener to both students and teachers. Remember - A teacher
is only a ladder and not an escalator! Students should utilize the knowledge imparted by the
teacher as a strong base, show the zeal to go deep into the subject and widen the scope of their
knowledge. There is no elevator to success. There are only steps. Climb them. Also the teacher
should always be an inspiring model to the students. All of us should keep this in our mind.

The way to Heaven is only through Hell ! No pains, No gains ! !

*** WITH BEST WISHES TO ALL THE STUDENTS ***


Previous Gl\TE Questions & Solutions (Mech.)
Weightage ( 2016 & 2017)

2016 2017

S.No. Name of the Subject SET 1 SET2 SET3 SET 1 SET2

1M 2M 1M 2M 1M 2M 1M 2M 1M 2M

1 Production SQ SQ 4Q SQ SQ 6Q s 6 2 s

2 IM&OR 0 2Q lQ 2Q lQ 2Q 0 2 2 3

3 FM& Turbo machinery 3Q 3Q 2Q 3Q 2Q 3Q 3 3 3 3

4 Heat Transfer lQ 3Q 3Q 2Q 3Q 2Q 1 2 2 2

s Thermodynamics 3Q 3Q 2Q 4Q 2Q 4Q 3 4 2 4

6 Strength of Materials 3Q 4Q 3Q 4Q 2Q 3Q 4 4 4 1

7 Machine Design 0 lQ 2Q lQ lQ lQ 1 1 1 3

8 Theory of Machines 2Q 3Q lQ 2Q 3Q 2Q 1 3 2 3

9 Engineering Mechanics 2Q 2Q lQ 2Q lQ 3Q 2 1 0 2

10 Material Science lQ 0 lQ lQ 0 0 0 0 2 0

11 General Aptitude SQ SQ 4Q 6Q 6Q 4Q s s s s

12 Engineering Mathematics SQ SQ SQ SQ SQ SQ s 4 s 4
Applications:

-
Power Engineering: Air and gas compressors; vapour and gas power cycles, concepts of
regeneration and reheat.
I.C. Engines: Air-standard Otto, Diesel and dual cycles.
Refrigeration and air-conditioning: Vapour and gas refrigeration and heat pump cycles;
properties of moist air, psychrometric chart, basic psychrometric processes.
turbomachinery:
. . .
Impulse and .reaction principles, velocity diagrams, ' Pelton:wheel, Francis and
. . . . · . . . .

Kaplan turbines. · ·

Materials, Manufacturing and Industrial Engineering

Engineering Materials: Structure and properties of engineering materials, phase diagrams, heat
treatment, stress-strain diagrams for engineering materials.
Casting, Forming and Joining Processes: Different types of castings, design of patterns, moulds
and cores; solidification and cooling; riser and gating design. Plastic deformation and yield criteria;
fundamentals of hot and cold working processes; load estimation for bulk (forging, rolling, extrusion,
drawing) and sheet (shearing, deep drawing, bending) metal forming processes; principles of
powder metallurgy. Principles of welding, brazing, soldering and adhesive bonding.
Machining and Machine Tool Operations: Mechanics of machining; basic machine tools; single and
multi-point cutting tools, tool geometry and materials, tool life and wear; economics of machining;
principles of non-traditional machining processes; principles of work holding, design of jigs and
fixtures. Metrology and Inspection: Limits, fits and tolerances; linear and angular measurements;
comparators; gauge design; interferometry; form and finish measurement; alignment and testing
methods; tolerance analysis in manufacturing and assembly.
Computer Integrated Manufacturing: Basic concepts of CAD/CAM and their integration tools.
Production Planning and Control: Forecasting models, aggregate production planning, scheduling,
materials requirement planning.
Inventory Control: Deterministic models; safety stock inventory control systems.
Operations Research: Linear programming, simplex method, transportation, assignment, network
flow models, simple queuing models, PERT and CPM.

General Aptitude
Numerical Ability: Numerical computation, numerical estimation, numerical reasoning and data
interpretation.
Verbal Ability: English grammar, sentence completion, verbal analogies, word groups, instructions,
critical reasoning and verbal deduction.
GATE Syllabus for Mechanical Engineering
(Other than Engineering Mathematics)

· > Applied Mechanics and Design


Engineering Mechanics: Free-body diagrams and equilibrium; trusses and frames; virtual work;
kinematics and dynamics of particles and of rigid bodies in plane motion; impulse and momentum
(linear
. . . . and angular).an9 energy f()rmulations, colli.sions.
: ' . . ' �. ,, ..
Mechanics of Materials: Stress and strain, elastic constants, Poisson's ratio; Mohr's circle for plane
stress and plane strain; thin cylinders; shear force and bending moment diagrams; bending and
shear stresses; deflection of beams; torsion of circular shafts; Euler's theory of columns; energy
methods; thermal stresses; strain gauges and rosettes; testing of materials with universal testing
machine; testing of hardness and impact strength.
Theory of Machines: Displacement, velocity and acceleration analysis of plane mechanisms;
dynamic analysis of linkages; cams; gears and gear trains; flywheels and governors; balancing of
reciprocating and rotating masses; gyroscope.
Vibrations: Free and forced vibration of single degree of freedom systems, effect of damping;
vibration isolation; resonance; critical speeds of shafts.
Machine Design: Design for static and dynamic loading; failure theories; fatigue strength and the S­
N diagram; principles of the design of machine elements such as bolted, riveted and welded joints;
shafts, gears, rolling and sliding contact bearings, brakes and clutches, springs.

Fluid Mechanics and Thermal Sciences


Fluid Mechanics: Fluid properties; fluid statics, manometry, buoyancy, forces on submerged bodies,
stability of floating bodies; control-volume analysis of mass, momentum and energy; fluid
acceleration; differential equations of continuity and momentum; Bernoulli's equation; dimensional
analysis; viscous flow of incompressible fluids, boundary layer, elementary turbulent flow, flow
through pipes, head losses in pipes, bends and fittings.
Heat-Transfer: Modes of heat transfer; one dimensional heat conduction, resistance concept and
electrical analogy, heat transfer through fins; unsteady heat conduction, lumped parameter system,
Heisler's charts; thermal boundary layer, dimensionless parameters in free and forced convective
heat transfer, heat transfer correlations for flow over flat plates and through pipes, effect of
turbulence; heat exchanger performance, LMTD and NTU methods; radiative heat transfer, Stefan­
Boltzmann Jaw, Wien's displacement Jaw, black and grey surfaces, view factors, radiation network
analysis.
Thermodynamics: Thermodynamic systems and processes; properties of pure substances,
behaviour of ideal and real gases; zeroth and first Jaws of thermodynamics, calculation of work and
heat in various processes; second law of thermodynamics; thermodynamic property charts and
tables, availability and irreversibility; thermodynamic relations.
IP@h'Mlilil'ilii*tdlf1987-2017
Wl(·litilOlijriH*filiiHAl#BIHlil
'.!,' · �

1 ENGINEERING MECHANICS 01-40

2 STRENGTH OF MATERIALS ,, 41-116

3 THEORY OF MACHINES ,, 117-196

4 MACHINE DESIGN 197-242

5 FM&TURBO MACHINERY 243-344

6 HEAT TRANSFER,.,. 345-428

7 THERMODYNAMICS, 429-580

8 PRODUCTION 581- 888

9 IM&OR 889-982

10 MATERIAL SCIENCE 983-1004

11 GENERAL APTITUDE 1005-1096


Foreword
GATE in Mechanical Engineeri��_..., -
- ' L.i:-.,,,"-----

Starting year : 1987 "" --\ '


From 1992- 2002 : Objective and Conventional Questions
From 2003 - 2013 : Objective Questions with multiple choices
2014 onwards : Objective Questions with multiple choices &
Numerical Answer Questions without options

The style, quality and content of the Solutions for previous GATE Questions of
Mechanical Engineering, will encourage the reader, especially the student whether above
average, average or below average to learn the concept and answer the question in the
subject without any tension. However, it is the reader who should confirm this and any
comments and suggestions would be pleasantly received by the Academy.

The student should not miss to go through the solutions for conventional questions
asked prior to 2003, as more concepts are brought out in them that will facilitate to answer
the numerical answer questions, Common data and Linked answer questions , if any, easily.

The student is advised to solve the problems without referring to the solutions. The
student has to analyze the given question carefully, identify the concept on which the
question is framed, recall the relevant equations, find out the desired answer, verify the
answer with the final key such as (a), (b), (c), (d), then go through the hints to clarify his
answer. This will help to face numerical answer questions, better. The student is advised to
have a standard text book ready for reference to strengthen the related concepts, if
necessary. The student is advised not to write the solution steps in the space around the
question. By doing so, he loses an opportunity of effective revision.
As observed in the GATE - 17, number of sets may be possible, being.online exams.
Hence, don't skip any subject. All are the equally important.
It is believed that this book is a Valuable aid to the students appearing for competitive
exams like GATE, JTO, DRDO, ISRO and Other PSUs. This book can also be used by fresh
Teachers in Engineering in improving their Concepts.
Mathematics Previous Questions & Solutions of GATE of all branches are available
in a separate booklet.
With best wishes to all those who wish to go through the following pages.

Y.V. Gopala Krishna Murthy


M.Tech. MIE,
Managing Director,
ACE Engineering Academy
ACE Engineering publications
Engineering Mechanics
(Questions)
Page No. 02

CONTENTS
-
-- --- - --

Chapter Questions Solutions


Name of the Chap· ter
No. Page No. Page No.
- - - -=-��-=--�=-=---=-

01 Engineering Machines I 03 - 20 21-40


Engineering Mechanics
03 The cylinder shown below rolls without
One Mark Questions slipping. In which direction does the friction
force act? Towards which of the following
01. A and B are the end-points of a diameter of points is the acceleration of the point of
a disc rolling along a straight line with a contact A on the cylinder directed?
counter Clock-wise angular velocity as (GATE-ME-93)
shown in the figure. Referring to the
velocity vectors VA and VB shown in the
figure: (GATE-ME-90)

(a) The mass centre


(b) The geometric centre
(c) The point P as marked
(d) None of the above

(a) VA and VB are both correct. 04. A stone of mass m at the end of a string of
(b) VA is incorrect but VB is correct. length l is whirled in a vertical circle at a
(c) VA and VB are both incorrect. constant speed. The tension in the string will
be maximum when the stone is
(d) VA is correct but VB is incorrect
(GATE-ME-94)
(a) at the top of the circle
02 Instantaneous centre of a body rolling with
(b) half way down from the top
sliding on a stationary curved surface lies.
(c) quarter-way down from the top
(GATE-ME-92)
(d) at the bottom of the circle
(a) at the point of contact
(b) on the common normal at the point of
05. A ball A of mass m falls under gravity from
contact
a height h and strikes another ball B of mass
(c) on the common tangent at the point of
m which is supported at rest on a spring of
contact
stiffness k. Assume perfect elastic impact.
(d) at the centre of curvature of the
Immediately after the impact
stationary surface.
(GATE-ME-96)

!M•li@jjji4.jjijmfi1Mi!iMij>Hyderabad I Delhi I Bhopal I Pune I Bhubaneswar I Lucknow I Patna I Bengalund�I Vijayawada I Vmg I Tirupati I Kukatpal]y I Kolkata I
:4: ME Previous Solutions Booklet

(a) the velocity of ball A is 1._�2gh 09. The ratio of tension on the tight side to that
2 on the slack side in a flat belt drive is
( b) the velocity of ball A is zero (GATE-ME-00)
(a) proportional to the product of
(c) the velocity of both balls is 1._�2gh
2 coefficient of friction and lap angle.
(d) none of the above (b) an exponential function of the product
of coefficient of friction and lap angle.
06. A wheel of mass m and radius r 1s m (c) proportional to lap angle
accelerated rolling motion without slip (d) proportional to the coefficient of
under a steady axle torque T. If the friction.
coefficient of kinetic friction is µ, the
friction force from the ground on the wheel 10. A truss consists of horizontal members and
1s (GATE-ME-96) vertical members having length L each. The
(a) µmg (b) T/r members AE, DE and BF are inclined at 45°
(c) zero (d) none of the above to the horizontal. For the uniformly
distributed load P per unit length on the
07. A car moving with uniform acceleration member EF of the truss shown in the figure,
covers 450m in a 5 second interval, and the force in the member CD is
covers 700m in the next 5 second interval. (GATE-ME-03)
The acceleration of the car is
tfUnit length
(GATE-ME-98) ��-�,�� F
(a) 7m/s2
(b) 50m/s2
(c) 25m/s2 (d) 10m/s2

08. A steel wheel of 600 mm diameter on a C


horizontal steel rail. It carries a load of 500
N. The coefficient of rolling resistance is
0.3. The force in newton, necessary to roll PL
(a) (b)PL
the wheel along the rail is (GATE-ME-00) 2
(a) 0.5 (b) 5 (c) 1.5 (d) 150 2PL
(c) Zero (d)
3

yderabad I Delhi I Bhopal I Punc I Bhubancswar I Lucknow I Patna I Bengaiuru I Chennai I Vtjayawada jVizag I TllUpari I Kukatpally I Kolbla
:5: Engineering Mechanics

11. A bullet of mass 'm' travels at a very high (a)Zero


velocity 'V' (as shown in the figure) and (b)490 ��Ml
gets embedded inside the block of mass 'M'
initially at rest on a rough horizontal floor. (d)981 N in tension
The block with the bullet is seen to move a
distance 'S' along the floor. Assuming µ to 13. The time variation of the position of a
be the coefficient of kinetic friction between particle in rectilinear motion is given by
the block and the floor and 'g' the x = 2t 3 + t 2 + 2t. If 'V' is the velocity and
acceleration due to gravity, what is the 'a' acceleration of the particle in consistent
velocity 'V' of the bullet? units, the motion started with
(GATE-ME-03) (GATE-ME-05)
(a)V = 0, a = 0 (b)V = O, a = 2
(c)V = 2, a = O (d)V = 2, a = 2
V )
M
14. A simple pendulum of length of 5m, with a
bob of mass 1kg, is in simple harmonic
+m m motion. As it passes through its mean
(a) M �2µgS (b) M- �2µgS
m m position, the bob has a speed of 5m/s. The
µ(M + m) net force on the bob at the mean position is
(c) �2gS (d) M �2µgS
m m (GATE-ME-OS)
(a)zero (b)2.5N
12. The figure shows a pin-jointed plane truss (c)5N (d)25N
loaded at the point M by hanging a mass of
100 kg. The member LN of the truss is 15. During inelastic collision of two particles,
subjected to a load of (GATE-ME-04) which one of the following is conserved?
(GATE-ME-07)
(a) Total linear momentum only
(b) Total kinetic energy only
(c) Both linear momentum and kinetic energy
(d) Neither linear momentum nor kinetic
energy

!IJl•@j@jjjiU.jj4Aflbiil!ilh+1ydcrabad I Delhi I Bhopal I Punc I Bhubaneswar I Lucknow I Pama I Bcngaluru I Chcnnai I VtjayawadaJ Vizag I Tirupati I KukatpaJ)y I Kolkala I
.,. V ACE : 6: ME_Previous Solutions Booklet
.. ,.....,.�FipmqPmiJiratiooa
...,
• • • •

16. A straight rod length L(t) , hinged at one end 19. A stone with mass of 0. 1kg is catapulted as
freely extensible at tlle � 1 .. , '�nd, rotates shown in the figure. The total force Fx (in N)
through an angle 0(t) abo e hinge. At exerted by the rubber band as a function of
time t, L(t) = l m, L(t) = lm / s , 0(t) =n/4 distance x (in m) is given by Fx = 300x2 • If
the stone is displaced by 0. 1 m from the un­
rad and 0(t) = 1 rad/s. The magnitude of the
stretched position (x = 0) of the rubber band,
velocity at the other end of the rod is
the energy stored in the rubber bad is
(GATE-ME-08)
(a) 1 mis (b) .fi.ml s
(c) F3m / s (d) 2 mis

17. A block weighing 981 N is resting on a


horizontal surface. The coefficient of
friction between the block and the horizontal (GATE-ME-11)
surface is µ = 0.2. A vertical cable attached
to the block provides partial support as (a) 0.0 1 J (b) 0. 1 J
shown in the figure. A man can pull (c) 1 J (d) 10 J
horizontally with a force of 100 N. What
will be the tension, T (inN) in the cable if 20. A circular object of radius r rolls without
the man is just able to move the block to the slipping on a horizontal level floor with the
right? (GATE-ME-09) center having velocity V. The velocity at the
point of contact between the object and the
T ·�
floor is (GATE-14 - SET-1)
(a) 176.2
(b) 196.0 100N
(a) zero
(c) 481.0 0 (b) V in the direction of motion
µ=0.2
(d) 981.0 (c) V opposite to the direction of motion
(d) V vertically upward from the floor
18. The coefficient of restitution of a perfectly
plastic impact is (GATE-ME-11) 2 1. A two member truss ABC is shown in the
(a) Zero (b) 1 figure. The force (in kN) transmitted in
(c) 2 (d) infinite member AB is

!IJl@M!@hiii•ih4RbbiM\hfjj� I I I I I I I I I
yderabad Dellii Bhopal Pune Bhubaneswar Lucknow Patna Benga)wu Chennai jVtjayawadal Viz.ag Tirupari I Kukatpally I Kolkata
.:-.t ACE
...:- 7 ee Mech cs
v.... - ,."vE!·��-�Pu�---
��-���·'\0111
�========�:��:=======�Engin�:·::nng:·�::;��·;=
.,

25. A small ball of mass 1 kg moving with a


lm-1
�0)-------:o B velocity y.:......:.o�undergoes a direct central
impact ·� 1.ationary ball of mass 2 kg.
The impact is perfectly elastic. The speed (in
mis) of 2 kg mass ball after the impact will
(GATE -14-SET-2) be (GATE-15-Set 2)

22. A mass m 1 of 100kg traveling with a


26. A weight of SOON is supported by two
uniform velocity of 5 mis along a line
metallic ropes as shown in the figure. The
collides with a stationary mass m2 of
values of tensions T1 and T2 are respectively.
1000kg. After the collision, both the masses
travel together with the same velocity. The
coefficient of restitution is
(GATE-14- SET-3)
(a) 0.6 (b) 0. 1 (c) 0.01 (d) 0
SOON
23. In a statically determinate plane truss, the (GATE-15-Set 3)
number of joints (j) and the number of (a) 433N and 250N
members (m) are related by (b) 250N and 433N
(GATE-14- SET-4) (c) 353.5N and 250N
(a) j = 2m-3 (b) m = 2j+l (d) 250Nand 353.5N
(c) m = 2j-3 (d) m = 2j-1
27. A rigid ball of weight 100 N is suspended
24. Two identical trusses support a load of 100 with the help of a string. The ball is pulled
N as shown in the figure. The length of each by a horizontal force F such that the string
truss is I .Om; cross-sectional area is 200 makes an angle of 30° with the vertical. The
mm2 , young's modulus E = 2000Pa. The magnitude of force F (in N) is __
force in the truss AB (in N) is ___
(GATE-15- Set 1)
A
F

lOON
(GATE- 16- SET- 1)

yderabad I Dellti I Bhopal I Pune I Bhubaneswar I Lucknow I Patna I Bengaluru I Oiennai I Vliayawadal Vizag I Tirupali I Kukatpal)y I Kolkala
:8: ME Previous Solutions Booklet

28. A point mass M is released from rest and 30. A force F is acting on a bent bar which is
slides down a ,spherica1 h-o'W\- �f radius R ) clamped at one end as shown in the figure.
from a height H as s.."""'___.,.�- the figure
below. The surface of the bowl is smooth
(no friction). The velocity of the mass at the
bottom of the bowl is

The CORRECT free body diagram is


(GATE - 16 - SET - 3)
(a)
(GATE - 16 - SET - 1)
(a) jgH (b) �2gR
(c) �2gH (d) 0
M

29. A point mass having mass M is moving with (b)


a velocity V at an angle 0 to the wall as
shown in the figure. The mass undergoes a
perfectly elastic collision with the smooth
wall and re?O�nds. The total change (final
minus initial) in the momentum of the mass
(GATE - 16 - SET - 2)
(c)
lS

y.J
I -�__,_v_

x, i
0 ___
M

(a) -2MVcos0 j (b) 2MVsin0 ]


(c) 2MVcos0 J (d)-2MVsin0 J

I
)'dcrabad Delhi I Bhopal I Pune I Bhubaneswar I Lucknow I Patna I Benga1uru I Chennai !Vuayawada IVizag I Tirupati I KukalpallyI Kolkata
'!.�
. .t\CE . .
.l'.���PoNialiona
-t==================================
Engineering Mechanics :9:

3 1 . A Particle of unit mass is moving on a plane.


Its trajectory, in polar coordinates, is given
by r(t) = t2 , 0 (t) = t, where t is time. The
kinetic energy of the particle at time t = 2 is T
(GATE - 17 - SET - l)
(b) 1 2 (c) 1 6 (d)24
Spr!ig
(a) 4 scale

m
32. The following figure shows the velocity­
time plot for a particle travelling along a (a)2T/g (b) T( 1 + c47t)/g
straight line. The distance covered by the (c)4T/g (d)None of the above
particle from t = 0 to t = 5 s is ----
m. 02. AB and CD are two uniform and identical
bars of mass 1 0kg each, as shown. The
hinges at A and B are frictionless. The
assembly is released from rest and motion
4

occurs in the vertical plane. At the instant


that the hinge B passes the point B, the angle
between the two bars will be
(GATE-ME-96)
2 3 4 5 6
Time (s) Im
(GATE - 17 - SET - l)

Two Marks Questions

0 1 . A spring scale indicates a tension T in the


right hand cable of the pulley system shown (a)60 degrees (b)37.4 degrees
in Neglecting the mass of the pulleys and (c)30 degrees (d)45 degrees
ignoring friction between the cable and
pulley the mass m is (GATE-ME-95)

jl\lii!@OiiiijjjQJmbiMiiiB!+yderabad I Delhi I Bhopal I Pwic I Bhubaneswar I I..ncknow I Patna I Bengaiuru I Chcnnai IVuayawadaI Vmg I Ttrupati I Kukatpa!Jy I Ko!ka!a I
: 10 : ME Previous Solutions Booklet

03. A rod of length Im is sliding in a comer as 05. A mass 35 kg is suspended from a


shown in. At an instant when... ,the rod makes weightless bar AB which is supported by a

an angle of 60 degrees 1
e horizontal cable CB and a pin at A as shown in. The
plane, the velocity of point A on the rod is pin reactions at A on the bar AB are
l rn/s. The angular velocity of the rod at this (GATE-ME-97 )
instant is (GATE-ME -9 6 ) C

lm/s
275mm
(

(a) Rx = 343.4 N, Ry = 755.4 N


(b) Rx = 343.4 N, Ry = 0
(c) Rx = 755.4 N, Ry = 343.4 N
(a) 2 rad/s (b) 1.5 rad/s
(d) Rx = 755.4 N, Ry = 0
(c) 0.5 rad/s (d) 0.75 rad/s

06. As shown in Figure, a person A is standing


04. Match 4 correct pairs between List I and List
at the centre of a rotating platform facing
II. No credit will be given for partially
person B who is riding a bicycle, heading
correct matching (GATE-ME-9 6 )
East. The relevant speeds and distances are
List- I shown in given figure person, a bicycle,
(a) Collision of particles heading East. At the instant under
(b) Stability consideration, what is the apparent velocity
(c) Satellite motion of B as seen by A? (GATE-ME-99 )
(d) Spinning top

T B: ..,
' V = 8m/s
I

t
List- II
I

1. Euler's equation of motion


co = l rad/sec
2 . Minimum kinetic energy
3 . Minimum potential energy
4. Impulse-momentum principle
(a) 3 mis heading East
5. Conservation of moment of momentum
(b) 3 mis heading West
IM•@i!@hiMiUNMfln5ijjjjj.yderabad I Delhi I Bhopal I Pune I Bhubaneswar I L.icknow I Patna I Bengaluru I Cheruiai I Vtiayawada j V,zag I Tuupali I Kukalpally I Kolkala I
: 11 : Engineering Mechanics

(c)8 mis heading East 09. The figure below shown a pair of pin jointed
(d) 1 3 mis heading East gripper ton !:: holding an object weighting
2000N. he coefficient of friction (µ) at the
Common data Q. 07 & Q. 08 gripping surface is 0. 1 XX is the line of
A reel of mass "m" and radius of gyration "k" is action of the input force and Y - Y is the
rolling down smoothly from rest with one end of line of application of gripping force. If the
the thread wound on it held in the ceiling as pin joint is assumed to be frictionless, the
depicted in the figure below. Considering the magnitude of force F required to hold the
thickness of the thread and its mass negligible in weight is (GATE-ME-04)
comparison with the radius "r" of the hub and the X F -----r-- X
reel mass "m", symbol "g" represents the
acceleration due to gravity. 300 mm

Thread
1 50 mm

(a) 1 000 N (b)2000 N


(c)2500 N (d)5000 N

1 0. An ejector mechanism consists of a helical


(GATE-2003)

07. The linear acceleration of the reel is: compression spring having a spring constant
of k = 98 1 x 1 03 N/m. It is pre-compressed by
1 00mm from its free state. If it is used to
eject a mass of 1 00kg held on it, the mass
mgr 2
(d) will move up through a distance of
(r 2 + k 2 )
.. .. .. .. .. .. .. .. .. .. .. .. .. .. .. (GATE-ME-04)
.. .. .. .. .. .. .. .. .. .. .. .. .. .. .. Mass
.. .. .. .. .. .. .. .. .. .. .. .. .. .. ..
08. The tension in thread is
.. . .. . .. . .. . .. . .. . .. . ..
.. .. .. .. .. .. .. .. .. .. .. .. .. .. ..
,.
mgr mgk
(a) (b)
2 2

(r 2 + k 2 ) (r 2 + k 2 )
mgrk mg (a) 1 00 mm (b)5000 mm
(c) 2 (d)
(r + k 2 ) (r 2 + k 2 ) (c)581 mm (d) 1 000 mm

\( I I 11.!_111l l 1 1 1 1� P11lil11 .1111 H 1 ..., yderabad I Delhi I Bhopal I Pune I Bhubaneswar l Lucknow! Patna I Bengaluru I Chennai I Vtjayawada I Vizag I Tuupati I Kukatpally I Kolkata
1 1. A rigid body shown in the fig.(a) has a mass 13. An elevator (lift) consists of the elevator
of 10kg. It rotates with a , iform angular cage and a counter weight, of mass m each.
velocity 'ro' A balancing mass of 20 kg is The cage and the counter weight are
attached as shown in fig.(b). The percentage connected by chain that passes over a pulley.
increase in mass moment of inertia as a The pulley is coupled to a motor. It is
result of this addition is desired that the elevator should have a
10 kg maximum stopping time of t seconds from a
a peak speed V. If the inertias of the pulley
a
0 and the chain are neglected, the minimum
N

power that the motor must have is


0
0
N

y
0
ll

20 kg
Fig. (a) Fig. (b)
EB
(GATE-ME-04) Chain
(a) 25% (b) 50% l m

(c) 100% (d) 200% /


cage

12. A shell is fired from cannon. At the instant


the shell is just about to leave the barrel, its Counter
/
weight
velocity relative to the barrel is 3mls, while (GATE-ME-OS)
the barrel is swinging upwards with a 1 mV 2
(a) - mV 2 (b)
constant angular velocity of 2 rad/s. The 2 2t
magnitude of the absolute velocity of the mV 2 2mV 2
(c) (d)
shell is (GATE-ME-OS) t t

14. Two books of mass 1 kg each are kept on a


table one over the other. The coefficient of
friction on every pair of containing surfaces
is 0.3. The lower book is pulled with a
(a) 3 mis (b) 4 mis horizontal force F. The minimum value of F
(c) 5 mis (d) 7 mis for which slip occurs between the two books
lS. (GATE-ME-OS)

!IS11i@jjji4ijjj4@biM\hii!� yderabad I Delhi I Bhopal I Pune I Bhubaneswar I Lucknow I Patna I Benga)uru I Chennai I Vuayawada I Vizag I T111Jpali I J(ukatpally I Kolkata
: 13 : Engineering Mechanics

(a) Zero (b) 1 .06 N (a) 520 N and 300 N


(c) 5.74 N (d) 8.83 N (b) 300 N and 520 N
(c) 450 N and 1 50 N
1 5 . A 1 kg mass of clay, moving with a velocity (d) 1 50 N and 450 N
of 1 0 mis, strikes a stationery wheel and
sticks to it. The solid wheel has a mass of 20 1 7. A block of mass M is released from point P
kg and a radius of I m. Assuming that the on a rough inclined plane with inclination
wheel and the ground are both rigid and that angle 0, shown in the figure below. The
the wheel is set into pure rolling motion, the coefficient of friction is µ. If µ < tan0, then
angular velocity of the wheel immediately the time taken by the block to reach another
after the impact is approximately point Q on the inclined plane, where PQ = -S,
(GATE-ME-OS) 1s (GATE-ME-07)

1 0 mis �
O __
1 kg


(a) Zero (b) 1 /3 rad/s

(c) V{io rad/s 2s


3
(d) 1 0/3 rad/s (a)
g cos 0(tan 0 - µ )
2s
1 6. If point A is in equilibrium under the action (b)
g cos 0(tan 0 + µ )
of the applied forces, the values of tensions
2s
TAB and TAc are respectively. (c)
g sin 0(tan 0 - µ )
2s
(d)
g sin 0(tan 0 + µ )

1 8. A circular disk of radius 'R' rolls without


600N slipping at a velocity v. The magnitude of
the velocity at point 'P' (see figure) is
(GATE-ME-06) (GATE-ME-08)
2 1. A 1 kg block is resting on a surface with
coefficient of friction µ = 0. 1. A force of 0.8
V ) N is applied to the block as shown in the
figure. The friction force is
---� (GATE-ME-11)

(a) ,/3 V (b) -JJv


d::!:l
2
(a) Zero (b) 0. 8 N
V 2V
(c) - (d) (c) 0.89 N (d) 1.2 N
2 ,/3
Common Data for Question Q.22 & Q.23
19. Consider a truss PQR loaded at P with a Two steel truss members, AC and BC, each
force F as shown in the figure. The tension having cross sectional area of 100 mm2 , are
in the member QR is (GATE- ME-08) subjected to a horizontal force F as shown in
figure. All the joints are hinged. (GATE-ME-12)
p iF
(a) 0.5 F

(b) 0.63 F

(c) 0.73 F

(d) 0.87 F

20. A uniform rigid rod of mass M and length L


is hinged at one end as shown in the below 22. If F = l kN, magnitude of the vertical
figure. A force P is applied at a distance of reaction force developed at the point B in
2L/3 from the hinge so that the rod swings kN is
to the right. The reaction at the hinge is (a) 0.63 (b) 0.32 (c) 1.26 (d) 1.46

23. The maximum force F in kN that can be


2L/3 applied at C such that the axial stress in any
p L
of the truss members DOES NOT exceed
l OOMPa is
(GATE-ME-09)
(a) 8. 17 (b) l l . 15 (c) 14. 14 (d)22.30
(a)-P (b) 0 (c)P/3 (d) 2P/3

!lfllpj@jjjl41h4Rflftjffimojj� ydcrabadj Delhij Bhopalj Pune j Bhubaneswarj Lucknow j Patna j Bengalwuj Chennai jVgayawadajVu.ag jTuupari I Kukatpally j Kolkata
: 15 : Engineering Mechanics

24. A pin jointed uniform rigid rod of weight W the block attained after 1 0 seconds is
and length L is supported horizontally by an
external force F as shown in the figure
.- (GATE-14-SET-l)

below. The force F is suddenly removed. At 27. A truck accelerates up a 10 ° incline with a
the instant of force removal, the magnitude crate of 1 00 kg. Value of static coefficient of
of vertical reaction developed at the support friction between the crate and the truck
lS (GATE-ME-13) surface is 0.3. The maximum value of
acceleration (in mls2) of the truck such that
___
/4Jr L
�F the crate does not slide down is
(a) zero (b) W/4 (c) W/2 (d) W (GATE-14-SET-2)

25. A block R of mass 100 kg is placed on a 28. A rigid link PQ of length 2 m rotates about
block S of mass 1 50 kg as shown in the the pinned end Q with a constant angular
figure. Block R is tied to the wall by a acceleration of 1 2 rad/s2 . When the angular
massless and inextensible string PQ. I f the velocity of the link is 4 rad/s, the magnitude
coefficient of static friction for all surfaces of the resultant acceleration (in mls2) of the
is 0.4, the minimum force F (in kN) needed end P is ___ (GATE-ME-14-SET-l)
to move the block S is
(GATE-14-SET-l) 29. A body of mass (m) 1 0kg is initially
stationary on a 45 ° inclined plane as shown
in figure. The coefficient of dynamic friction
between the body and the plane is 0.5. The
F
body slides down the plane and attains a
velocity of 20 mis. The distance travelled (in
(a) 0.69 (b) 0.88 (c) 0.98 (d) 1 .37 meter) by the body along the plane is
(GATE-14-SET-3)

/2�=�- - -
26. A block weighing 200N is in contact with a
level plane whose coefficient of static and
kinetic friction are 0.4 and 0.2. respectively.
(/
The block is acted upon by a horizontal
force (in newton) P = 1 Ot, where t denotes
the time in seconds. The velocity (in mis) of

\ ( } } !l;._,111( l 1111� P11hli1 ,il!O[h Fyderabad j Delhi j Bhopal j Pune j Bhubaneswari Lucknow l Patna l BengaJuru j Chennai j V,jayawada j Vu.ag j T,rupati I Kukatpally i Kolkata I
,, ..)...,EF.nginecring
�...). ACE P11blicaricn : 16: ME_Previous Solutions Booklet
,. ����;;;;;;;;;�===========================
30. An annular disc has a mass m, inner radius (a) 1 1.25 tension (b) 1 1.25 compression
R and outer radius 2R. The disc rolls on a (c) 13.5 tension (d) 13.5 compression
flat surface without slipping. If the velocity
of the center of mass is v, the kinetic energy 33. A uniform slender rod (8 m length and 3 kg
of the disc is (GATE-14-SET-3) mass) rotates in a vertical plane about a
horizontal axis 1m from its end as shown in
(a) i_ mV 2 (b) .!..!_ mV 2
16 16 the figure. The magnitude of the angular
acceleration (in rad/s2) of the rod at the
(c) .!l mv 2 (d) � mV 2
16 16 position shown is_ (GATE-14-SET-4)

3 } . A four-wheel vehicle of mass 1000kg moves


uniformly in a straight line with the wheels
I
lm
i 7m
J
revolving at 10 rad/s. The wheel are
identical, each with a radius of 0.2 m. then a 34. A wardrobe (mass 100 kg, height 4m, width
constant braking torque is applied to all the 2m, depth I m), symmetric about the Y-Y
wheels and the vehicle experiences a axis, stands on a rough level floor as shown
uniform deceleration. For the vehicle to stop in the figure. A force P is applied at mid­
in 10s, the braking torque (in N. m) on each height on the wardrobe so as to tip it about
wheel is ___. (GATE-ME-14-SET-3) point Q without slipping. What are the
minimum values of the force (in Newton)
32. For the truss shown in the figure, the forces and the static coefficient of friction µ
F 1 and F2 are 9 kN and 3 kN, respectively. between the floor and the wardrobe,
The force (in kN) in the member QS is respectively ? (GATE-ME-14-SET-4)
(GATE-ME-14-SET-4)
(a) 490.5 and 0.5 ,y
I

2 ln
(b) 981 and 0.5 p
• 4m
All dimensions 2
are in m (c) 1000.5 and 0.15
I
Q
:y
1 .5
(d) 1000.5 and 0.25

!ltlli@jjjiiijj4/mbi@d.jj..?1yderabad l Dclhi l Bhopal l Pune l Bhubaneswar l Uleknowl Patna l Bengaiuru l Cbennai lVtjaya\Wda lVizag l1irupati I Kukatpallyl Kolkala I
: 17 : Engineering Mechanics

35. A ladder AB of length 5 m and weight (W) by 20% . The height (in m)to which the ball
600 N is resting against a wall. Assuming will rise is (GATE-15-Set 1)
frictionless contact at the floor (B) and the
wall (A), the magnitude of the force P (in 38. The initial velocity of an object is 40 mis.
newton) required to maintain equilibrium of The acceleration a of the object is given by
the ladder is the following expression: a = - 0. 1 v
Where v is the instantaneous velocity of the
object. The velocity of the object after 3
seconds will be (GATE -15 -Set 2)

39. For the truss shown in figure, the magnitude


(GATE-14-SET-4) of the force (in kN)in the member SR is

36. For the truss shown in figure, the magnitude


of the force in member PR and the support
reaction at R are respectively

60°
l OO kN

(GATE-15 -Set 2)
(a)10 (b)14.14 (c) 20 (d)28.28

(GATE - 15-Set 1) 40. A bullet spin as the shot is fired from a gun.
(a)122.47 kN and 50 kN For this purpose, two helical slots as shown
(b)70.71 kN and 100 kN in the figure are cut in the barrel. Projections
(c)70.71 kN and 50 kN A and B on the bullet engage in each of the
(d)81.65 kN and l OOkN slots.

37. A ball of mass 0.1 kg, initially at rest, is


dropped from height of 1m. Ball hits the
ground and bounces off the ground. Upon
impact with the ground, the velocity reduces
:XXX :D> 0.5 m
B

ydcrabad I Delhi I Bhopal I Pune l Bhubancswar l Lucknow I Patna l Bcngaluru I Chennai ) Vtjayawada jVu.ag J T=pati I Kukatpally I Kolkata
: 18: ME Previous Solutions Booklet

Helical slots are such that one turn of helix (a) 2W tensile and ./3w compressive
is completed over a distance of 0.5m. If
(b) ./3w tensile and 2W compressive
velocity of bullet when it exits the barrel is
20m/s, its spmmng speed (in rad/s)
(c) ./3w compressive and 2W tensile
1s___ (GATE -15 -Set 3) (d) 2W compressive and ./3w tensile

4 1. A block of mass m rests on an inclined plane 43. A system of particles in motion has mass
and is attached by a string to the wall as center G as shown in the figure. The particle
shown in the figure. The coefficient of static i has mass mi and its position with respect to
friction between the plane and the block is a fixed point O is given by the position
0.25. The string can withstand a maximum vector ri. The position of the particle with
force of 20 N. The maximum value of the respect to G is given by the vector Pi . The
mass (m) for which the string will not break time rate of change of the angular
and the block will be in static equilibrium is momentum of the system of particles about
___kg. G is (The quantity Pi indicates second
Take cos0 = 0.8 and sin 0 = 0.6. derivative of P i with respect to time and
Acceleration due to gravity g = 10 m/s2 likewise for ri).
System boundary

(GATE - 16 - SET - 1)
0
42. A two -member truss PQR is supporting a (GATE - 16 - SET - 2)
load W. The axial forces in members PQ (a) Lri x mi Pi (b) Li Pi X mi �
and QR are respectively
(C) Ljrj X mi � (d) Li Pi X mi Pi
--- L --•I
44. An inextensible massless string goes over a
R frictionless pulley. Two weights of 100 N
and 200 N are attached to the two ends of
(GATE - 16 - SET - 1)
the string. The weights are released from
!lflli!i§hiiiiidQRftbiM\d.jj� yderabad J Delhi J Bhopa) J PuneJ BhubaneswarJ l..ucknow J Patna J Bengaluru J Chcnnai JVuayawada J Vizag JT1IUpari I Kukatpally J Kolkala
: 19 : Engineering Mechanics

rest, and start moving due to gravity. The


tension in the string (in N) is ____
-v2

200N x, 1

l OON The magnitude of the velocity V2 (in mis) at


(GATE - 16 - SET - 3) the end B is ----
(GATE - 16 - SET - 3)
45. A circular disc of radius 100 mm and mass 1
kg, initially at rest at position A, rolls 47. In abrasive water jet machining, the velocity
without slipping down a curved path as of water at the exit of the orifice, before
shown in figure. The speed v of the disc mixing with abrasives, is 800 mis. The mass
when it reaches position B is ____ mis. flow rate of water is 3.4 kg/min. The
A cceleration due to gravity g = 10 m/s 2
abrasives are added to the water jet at a rate
of 0.6 kg/min with negligible velocity.
Assume that at the end of the focusing tube,
abrasive particles and water come out with
equal velocity. Consider that there is no air
in the abrasive water jet. Assuming
conservation of momentum, the velocity (in
mis) of the abrasive water jet at the end of
(GATE - 16 - SET - 3) the focusing tube is ____
(GATE - PI-16)
46. A rigid rod (AB) of length L = ,./2 m is
48. Two disks A and B with identical mass (m)
undergoing translational as well as rotational
and radius (R) are initially at rest. They roll
motion in the x-y plane (see the figure). The
down from the top of identical inclined
point A has the velocity V 1 = i + 2 j m I s.
A A

planes without slipping. Disk A has all of


The end B is constrained to move only along
its mass concentrated at the rim, while Disk
the x direction.
B has its mass uniformly distributed. At the
bottom of the plane, the ratio of velocity of

li1•1i!i§jj@i4jjjNRflti1Mi!iijj.yderabad I Delhi I Bhopal I Pune I Bhubaneswar I Lucknow I Patna! Bengaluru I Chennai I Vliayawada I V,zag I Tirupati I Kukatpal)y I Kolkata I
: 20 : ME_Previous Solutions Booklet

the center of disk A to the velocity of the


center of disk B is (GATE - 17 - SET- l )

(a) .J{ (b) l (c) 1 (d) ..fi

49. The rod PQ of length L = ·..Ji m, and


uniformly distributed mass of M = 10 kg, is
released from rest at the position shown in
the figure. The ends slide along the
frictionless faces OP and OQ. Assume
acceleration due to gravity, g = 10 m/s2 • The
mass moment of inertia of the rod about its
centre of mass and an axis perpendicular to
the plane of the figure is (ML2/12). At this
instant, the magnitude of angular
2
acceleration (in radian/s ) of the rod is

0 Q
(GATE - 17 - SET - 2)

\l L l 11i.;111t t 1 111..; P11hl1t .111c 111" FyderabadJ DelhiJ Bhopal l Punc l Bhubaneswar l I..ucknow l l'alna i Bcnpluru l <llcnnai lVuayawadalVmc ITuupali I Kukalpal)yJ Kolkala I
Solutions
05. Ans: (b)
One Mark Solutions Sol: m A = m 8 = m (given)
Velocity of ball 'A' before impact
01. Ans: (c)
UA :;:: �2gh
Sol:
us = O
Velocity of approach = uA - us = �2gh
by applying linear momentum equation
m u + O = m A VA + m 8 V8
A A
02. Ans: (b) => UA = VA + Vs ------- ( 1)
Sol: Instantaneous center of a body rolling with
For perfect elastic collision , e = 1
sliding on a stationary curve is always on the
. ·. Velocity of approach = velocity of
common normal at the point of contact
separation
=> UA - us = Vs - VA
03. Ans: (b)
=> UA = Vs - VA ---------- (2)
Sol: The frictional force always acts m the
VA + Vs = Vs - VA
opposite direction of motion
=> 2VA = 0 => VA = 0
In this case the acceleration at the point
contact will be pass through the geometric :. U A = VB = �2gh
center
06. Ans: (d)
04. Ans: (d) Sol:
Sol:
Tension,
T = miol + mg cos0 F
For Tmax , 1/,

0 = 0° i.e., vertical
F = Friction force
Tension at bottom mgcos0
I = mr 2
T = mlol + mg
mg 0

Tension at top LMo = 0


T 1 = mfol - mg T-F x r = Io a

!M11ii@jjji4.jjjiAftbii\!i.jjlt yderabad J Delhi J Bhopal J Pune I Bhubaneswar J Lucknow I Patna J Bengaluru I Chennai I Vtjayawada I Vmg I Trrupati I Kukatpally I Kolkata
" "' . ACE
. . .
�-���PnNntions
':. : 22 : ME GATE Previous Solutions

=> T - F x r = mr 2 a
F = mra

:. a =
a

2 m r2
IMA = 0 = Wxa-Px(OB)
Frictional force, F= m r a

= mr x --2 = -
T T Since the depression is actually very small,
2m r 2r
the distance (OB)may be replaced by r;
Hence, Pxr = Wxa
07. A ns: (d )
Here, the horizontal component of the
1
S ol: S = ut + - at 2
Velocity
surface reaction R is equal to P by
2
inspection and is called rolling resistance.
=> 450 = Su + .!_ a x 5 2 The distance 'a' is called coefficient of
2
rolling resistance and is expressed in mm.
again700 = 5v + .!_ a x 5 2 W = 500 N, a = 0.3, r = 300 mm
2
500 x 0.3
Solving these equations, v - u = 50 p= = 0.5 N
300
v-u
=> a = -- = 1 0 m / sec 2
t 09. A ns: (b )
08. A ns: (a ) Sol:
S ol: Rolling resistance occurs because of the
deformation of the surface under a rolling
10. A ns: (a )
load. A wheel of weight W and radius r is
Sol: L FY = O => R A + R 8 = P x L
being pulled out of the depression and over
the point A by the horizontal force P.
Naturally this is a continuous process as the
PL PL
wheel rolls. => R = R =
A 2 ' B 2

!1H11i@jjji4jjjjiRftbiM\!ijjj.� yderabad I Delhi I Bhopal I Pune I Bhubaneswar I Lucknow I Patna I Bengaluru J Chennai I VijayawadaJ Vizag I Tirupati I KukatpallyJ Kolkata
...

� , ....
ti, V • ACE•

�,..,,.,,,. PuNir;atinns
• •
: 23 : Engineering Mechanics

FBD at Point A: Again R = (M + m)g


u2
=> (M + m)- = µ(M + m)gS
2
=> u 2 = 2µgS
A => u = �2µgS -----(1 )
Again
(M+m)u = mV
mV
u = -- ---------- (2)
M+m
mv =
So solving this �2µgS
M+m
M+m �
=> V = --v 2µgS
m

0 => TA C PL 12. A ns: (a )


L Fx TAE cos 45 = -
2
= =
Sol:
L---
K-------.- M
FBD at Point C:

PL C Considering joint L as shown in the figure


TAc Teo - Teo above => rY = 0
= =
2
=> The force in member LN = 0
11. A ns: (a )
Sol: K. E lost by the block with bullet = work 13. A ns: (d )
done to overcome the frictional force Sol: x = 2t 3 + t 2 + 2t
u2 dx
=> (M + m) - = F x S V= = 6t 2 + 2t + 2
2 dt
Where F = Frictional force = µR dv
a = - = 1 2t + 2
u2 dt
=> (M + m) - = µRS At, t = 0 => V = 2 and a = 2
2
\' 'I J 11gll!n r 111g P11hht.1t101h �yderabad I Delhi I Bhopal I Pune I Bhubaneswar I l.ucknow I Patna I Bengaluru I Chennai I Vtjayawada I Vizag l 'firupati I Kukatpally I Kolkara I
: 24 : ME GATE Previous Solutions

14. Ans: (a) 18. Ans: (a)


Sol: At mean position acceleration = 0
19. Ans: (b)

T - (mg + mn.tl) = 0 300 x 3 l o.1


x
Sol: E = YF d = [
=0
:. Fnet 0 3 0
mra,
At mean position net 1
= 100 x -- .= O. l J
force on bob is zero. 1 000
mg .
rnrro2

15. Ans: (a) 20. Ans: (a)


Sol: Instantaneous centre will have zero velocity
16. Ans: (b) because the instantaneous centre is the point
dL of contact between the object and the floor.
Sol: V = = lm / s
r dt
Ld0 21. Ans : 20 kN
V0 = - = l x l = lm / s
dt Sol: Im

0.5m
17. Ans: (c)
Sol: The FBD of the above block shown
T
Lami' s triangle

1----+ l OO N ,w FAB
F ______,,r--......

FED ofthe block


N
l OkN

l:Y = O => N+T-W = O


N = W-T = 98 1 - T
O.
F = µN = 0.2 (98 1 - T) tan 0 = S =:> 0 = tan-1 ( 0 · 5 ) = 26.56 °
1 .0 1
l:X = 0 => 1 00 - F = 0.
From the Lami's triangle
F = 100 = 0.2 (98 1 - T)
=> T = 48 1 N ----10 = Fae ° = ----FAB
sin 26.56 0
sin 90 sin 63 .44 °

\( I l rt�.,llH t 1 1 1 1,., 1 '1 t ! d 1c 1!1(JJI-.. �ydcrabadj Delhi I Bhopal I Punc I Bhubancswarj Lucknow! Patna! Bcngaluru I Chennai j VtjayawadajVm,g j Thupati I Kubtpa))yj Kolkatt I
.:-�.�="t......-
.,
ACE
v ii'....:..-;....
==�
25Publicdioos
=================================
: : Engineering Mechanics

10 1 1 l 1
FAB = x sin 63 .44 = 20kN - m 1 u 21 + - m 2 u 22 = - m 1 v 21 + - m 2 v 22
sin 26.56 2 2 2 2
1O 1 1 1
Fae = x sin 90 = 22.36kN - x l x l 2 2 = - x l x v� + - 2 x v;
sin 26.56 2 2 2
v� + 2v; = 144 .......... (2)
22. Ans: (d)
From equation ( 1) v, = 12 - 2v 2 substituting
Sol: Relative velocity after impact = 0
in equation (2)
Coefficient of restitution
2
Re lativevelocityafter Im pact (12 - 2 v i } + 2 v; = 144
= =0
Re lativevelocity before Im pact 144 + 4v; - 2 x 2 x 12 x v 2 + 2v; = 144
6v; - 48v 2 = 0 => v 2 (6v 2 - 48) = 0
23. Ans: (c)
48
=> v 2 = - = 8 m / s
24. Ans: 100 6

Sol: I FY = 0 26. Ans: (a)


FAB = Fae = F Sol: Adopting Lami ' s Theorem
2 FAB sin 0 = 100 N 500 T1 T2
= =
sin 90 sin 120 sin l 50
2 FAB x sin 30 = 100
T1 = 433 N
100
F = FAB = Fae = -- T2 = 250 N
1
2x-
2
27. Ans: 57.735N (range 55 to 60)
F = FAB = Fae = l OO N
Sol: By Lami ' s theorem T

--- = ---°
25. Ans: 7.8 to 8.2 F 100
°
sin 150 sin 120
Sol: m 1 = lkg m2=2kg m 1 = lkg m2=2kg

0 +O 0
v,
+ 0 => F = 57.735 N
u1 12m/s u 0
=
2
= V2
IOO N
F rom momentum conservation equation 28. Ans: (c)
Sol: By Energy Conservation
1
l x 12 = v 1 + 2 v 2 • • • • • ••• ( 1) mgH = - mv2
2
From kinetic energy conservation equation
=> V = .j2gH

!IJll4jj§mi4@iRdbi@hiiii,)ttyderabad I Delhi I Bhopal I Punc I Bhubaneswar I Lucknow I Patna I Bengaluru I Chcnnai I Vliayawadal Vizag I TJIUpali Kukatpally I Kolkata I
I
: 26 : ME GATE Previous Solutions

29. Ans: (d) Alternate Solution:


Sol: WH/////4'/////////2! d0

L \,
Yr= - = 2t = 4 'Ve= r - = r = t 2 = 4
, ' dr
,
dt
'
dt
Resultant velocity

V= �V,2 + V; = .J4 2 + 4 2 = 4,fi,

U = V cos 0 i + V sin 0 ] Kinetic energy = _!_ mV2 = _!_ x 1 x (4.fi.) 2


2 2
V = V cos 0 i -V sin 0 j
A A

= 1 6 N-m
� mv = m (v-U) =-2 mV sin 0 J
32. Ans: 10
30. Ans: (a)
Sol: Total distance covered = Area under
31. Ans: (c) velocity-time graph
Sol:
,• particle (r,0) 1 1 1
= - x l x l + l x 1 + l x 1 + - x 3x 1 + 2 x 2 + - x 2 x 2
,/ polar coordinates 2 2 2
,,
r, , , ' = 10 m
,,
,,
, ,' 0

dr Two Marks Solutions


r(t=2 sec) = t2 ; V = - = 2t '· V
dt
(t=2) = 2 x 2 = 4 mis; r=2 x 2=4m
d 01. Ans: (c)
e(t) = t· ro = O = lrad/ s·
' dt ' Sol: T 2T T
I = mr = 1 x 4 = 1 6 kg-m2
2 2
t t t
Total kinetic energy = ..!.. mv + _!_ Iro 2 2 m

2 2
T+2T+T = mg
= _!_ X 1 X r 2 + _!_ X 1 6 X 1 2
2 2 4T = mg
= 8 + 8 = 1 6 N-m
m = 4T/g

!1H1i@jjjfojji4Rflftj@j111jjii"yderabad I Delhi I Bhopal I Pune I Bhubaneswar I Lucknow I Patnal Bengaluru I Chennai J Vuayawada I Viz.ag J Trrupati I Kukatpa)Jy I Kolkatl I
-:.-� ACE
E�PnNir:moos : 27 : Engineering Mechanics
� � =====================================
02. Ans: (a) 04. Ans : a - 4, b - 3, c - 1, d - 5
Sol:
05. Ans : (d)
125
Sol: tan0 = -
275
T

=> e = 24.45°
B
- - - - 30° \.- - B

Tsin0 = mg.
I
I

60° :

Tsin24.45 = (35x9.81)
I
I

mg
I

As centre of gravity passes through 'B'.


I

=> T = 829.5 N
There is no moment about 'B' so always the
Rx = Tcos24.45 = 755.4 N
angle between two bars is constant. Ry = O

03. Ans: (a)


06. Ans: (d)
Sol: Va = 1 mis
Sol: Apparent velocity of B w.r.t A
Va = along vertical
= 8 mis (towards East)
Vb = along horizontal

07. Ans: (a)


Sol:
Thread

So instantaneous center of Va and Vb will be mg


perpendicular to A and B respectively
a = linear acceleration, k = radius of gyration
IA = OB = I x cos 0 = I x cos 60 ° = _!_ m
For vertical translation motion
mg - T = ma --- --- ( 1)
IB = OA = l x sin 0 = I x sin 60 ° = .fj m For rotational motion
2
Va = ro x IA T x r = Ia
a
=> ro = _a = 2 rad / sec
V Tr = mk2 a = mk 2 x -

IIJll4j@jjji4jjjdRflbii!jjih+yderabad I Delhi I Bhopal I Pune I Bhubaneswar I LJ.ICknowl Patna I Bengaluru I Chennai I Vtjayawada I Vmg I Tirupati I KukalpaJ)y I Kolkatl I
""'.

, �...." . ACE
. . .
..�&ipwrmgPnblnnma : 28 : ME_GATE_ Previous Solutions

mk. 2 1 1 . A ns: (b )
�T = - x a ------(2)
r2 S ol: 1 1 = l Ox0.22 = 0.4 kg-m2
mk. 2 gr 2 lz = 1 0x0.22+20x0. 1 2= 0.4 + 0.2 = 0.6kg.m2
mg - -2- x a = ma � a = 2 2
r (k + r ) Percentage of increase in mass moment,
0.6 - 0.4
I= x 1 00 � 50%
08. A ns: (c ) 0.4
S ol: Tension in thread,
T = mg - ma 1 2. A ns: (c )
mr g = mgk
2 2 S ol: Velocity of bomb (Vb) = canon
= mg -
k2 + r2 r2 + k2
Vcanon = rco = 2 X 2 = 4 m / S
09. A ns: (d )
S ol: II\
µR
C

R --- --- R

2000 N 0

vb = .J3 2 + 4 2 = 5 m / s
At equilibrium
2000 4
2 µR = 2000 � R = = 10 OOON tan e = -
2 x 0. 1 ' 3
Taking moment about pin e = 53. 1 3
1 0,000 x 1 50 = F x 300
F = 5000 N 1 3. A ns: (c )
S ol: T1 = mg - mx
10. A ns: (b ) T2 = mg + mx
S ol: k = 98 1 x 1 03 Nim, 8 = 100 mm f
(T2 - T1 )dx
Power = ----­
1 t
mgh = - k x 2
2
2mf�x dx 2m r2 mV 2
98 1 x 1 0 x {0. 1) =- - = V dV =
h = _!_ x
3 2
= 5 m = 5000 mm t t Jo t
2 100 X 9.81

\( I } ll..,l!H l l !ll,_.\ P1 dd1l .tlit>lh Fyderabad l DclhiJ Bhopal l Pune J BhubaneswarJ LucknowJPatnaJ Benga!uru J Chennai J VtjayawadaJ Vmg ITuupati I KukatpallyJ Kolkata I
1 4. A ns: (d) 1 5 . A ns: (b)
S ol: �---� S ol:
mV
3
� m1 V r = le ro = - m r 2 ro

2
3 C
=> 10 X 1 X 1 = - X 20 X 1 X 0)
2
W2 = 9.8 1 N
(neglecting mass of the clay)

FBD of block 2 => ro = !Q = _!_ rad / s


30 3

1 6 . A ns: (a)
S ol: TAB
TAc
2
=
9.8 1 N

--- F TAB cos60° = TAC cos30°


FBD of block I
TAB = .fi TAc 600N

TAB sin60° + TAc sin30 ° = 600 N


3 1
- TAC + - TAC = 600
2 2
From FBD of book 2, LY = 0 TAc = 300 N
=> N2 = W2 = 9.81 N TAa = 520 N
F2 = µN2 = 0.3 x 9. 8 1 = 2.943N
17. A ns: (a)
S ol:
From FBD of book 1 , LY = 0
=> N1 = N2+W2 N
= 9.81 + 9.81 = 1 9.62 N
F1 = µN 1 = 0.3 x 1 9.62 = 5.886N
F = F 1 + F2 = 8.83 N '
'' ''
g ''
e(�� Mgsin0
\('} l.!ll.!, 111{ I t I l l � P11hl1t .t!JOII'> �yderabad I Delhi I Bhopal I Pune I Bhubaneswar I Lucknow I Patna I Bengaluru I Chennai I Vgayawada I Vizag I Tuupati I Kukatpally I Kolkala I
: 30 : ME_GATE_ Previous Solutions

mgsin0 - F = ma or
VA = V = rro
mgcos0 = N
F = µN.
=> mgsin0 - µN= ma
=> mgsin0 - µmgcos0 = ma
=> a = g(sin0 - µcos0) Vp =
- - -
VA + Vp0 = V + AP x ro

Again S = ut + _!_ at 2 => S = _!_ at 2 ( :. u = O) = .,Jv 2 + V 2 + 2V 2 cos 60 = .fj V


2 2
8
°"' t - J3! - g(sin 0 � µ cos 0)
19. Ans: (b)
Sol:
2S
=> t = � PRsin30
g cos e(tan e - µ) PQsin45t
I
I
I
I
I I
I I
I I
PQ I
-- --• - - - PR
I

18 Ans: (a) PQcos45 PRcos30


Sol:
Force in member PQ considering joint P
V ) PQ cos45 = PR cos30
PQ = 1 .224 PR
PQ sin45 + PR sin30 = F
0 l .224PR x 0.707 + 0.5PR = F
PR = 0.732 F
Here 'O' is the instantaneous centre
Vp = ro x OP Now, considering joint R
VA Rro=
- - - - - - - - • PRcos30
R + R - OP 2
2 2
In � OAP , cos 1 20° =
2R x R
+
2R 2 - OP 2 PRsin30
- 0.5 = ----
2 R2
OP = .fjR QR = PR cos30 = 0.732F x cos 30
= 0.63F (Tensile)
Vp = .fjR x ro = .fjv

l11i•i!@Oii4.jjiAfll!@jjjjj+ydcrabad J Delhi J Bhopal J Pune J Bhubancswar J Lucknow ! Patna J Bcngaluru J Chcnnai J Vijayawada J Vu.ag J T11Upati J K.ukatpallyJ Kolkata I
: 31 : Engineering Mechanics

20. A ns: (b ) 22. A ns: (a )


S ol: S ol: 1:X = 0 => F = FAcx cos45 + Fee xcos60
LY = 0 => FAC sin45 = Fee sin60
Inertia t=fl
Fsc sin 60
FAC - 1 .224 Fee
force ej--p
. =
Sill 45
(Free body diagram) 1 1 .224 x cos45 x Fee + Feccos60
=

T = lo a. 1 = 0.865 Fee + 0.5Fec,


2L mL2 1
=> P x - = -- x a. Fee = -- = 0.732 kN
3 3 1 .365
2P Vertical force at 'B' = Fecsin60
a. = -
mL = 0.732xsin60 = 0.634 kN

Inertia force = J dm r a.
23. A ns: (b )
- L
= a. m r = a. m- S ol:
FAC
=
F
:::) FAC =0.895F
sin 120 sin 105
= 2P mL = P FAC = Maximum force
mL 2 0.895F 0.895F
=Stress => =l 00
At 'O' Area 1 00
Net reaction = Inertia force - P F = l l . 1 7 kN
=P-P=O

24. A ns: (b )
21. A ns: (b ) S ol :
I a.
S ol: 1 x 9.81 = 9.81N ..,
0.8N w ,I
F
When the support is removed the rod
�Y = O => N = 9. 8 1 N accelerates and undergo rotation about the
Fs = µN = O. l x 9.81 = 0.98 N axis of the hinge. The acceleration 'a.' can
The External force applied = 0.8 N < Fs be found using the dynamic equilibrium.

=> Frictional force = External applied Ia. = W _i


force = 0.8 N
\( I l 11�11 tl 1 1 1 1 1 1,.:, Pt il1l1l ,1t1( 1 1 h Fyderabad I Delhi I Bhopal I Pune I Bhubaneswarl Lucknow I Patna I Benpluru I Chennai I Vuayawada I V,zag I Tirupari I Kukatpally I Kolkata I
... ,. �....
". "
'!i.. .
i�PnJwarioN . .A!.CE . .
: 32 : ME_GATE_ Previous Solutions
' ==========================================
W FBD of block R :
I= {_ (moment of inertia of the rod
g 3 WR = 100x9.81 = 98 1N

about the hinge) rFy = O


N2 = WR =98 1 N
=> a =
W _
_
w.!:-
;, - � ; [·: I - mn F2 = µ N2 �'� N2

g ' 3
= 0.4x 98 1 = 392.4N

For force equilibrium


FBD of block S:
xa p

The inertia force will act m the upward


direction (opposite to motion)
Ws = 1 50 x9.8 1 = 1 47 1 .5 N
R - W + p x a .dx = 0 f Lfy = O
N, N2 + Ws = 98 1 + 1 47 1 .5 = 2452.5 N
0
=

R = W - p-a = W -p-a
p2
f1 = µ N1 0.4x 2452.5= 98 1 N
2
X
=
2 2
Lfx = 0 => F - f 1 - F2 = 0
W £2 3 g
= W - -. - - - F = F, + F2 = 98 1 + 392.4
2 2
= 1373.4 N = 1 .3734 kN
gf f

R = W [ 1- !] = :

( p is the mass per unit length) 26. Ans: 4.905 mis


Sol: µs = 0.4 ; µK = 0.2
pdx. xa is the inertia force on the element
FBD of the block
of length dx is the upward direction.
W = 200 N
25. Ans: (d)
Sol:
P = l Ot

F
N
F

\( I I 11;.!,!IH t I l l l � Pulilu ,1111 1 1 1.., yderabad I Delhi I Bhopal I Pune I Bhubaneswar I Licknow I Patna I Benga)uru I Chennai I Vtjayawada I Vizag I Tirupari I Kukatpa)ly I Kolkata
: 33 : Engineering Mechanics

W.r.t free body diagram of the block : 27. A ns: 1 . 198 m/s2
Fs = µsN ; S ol:
FK = µKN
FBD of the crate
I:Fy O =
fN
N-W = O
N = W= 200 N
Limiting friction or static friction
(Fs) = 0.4 x200 = 80 N
Kinetic Friction Wv JI0°
I

(FK) 0.2 x 200 = 40 N


I
= : W=l00x9.81=981 N

The block starts moving only when the W.r.t. FBD of the crate:
force, P exceeds static friction , F s Wx = Wsin 10° = 98 1 xsin 10° = 1 70.34N
Thus, under static equilibrium Wv = W cos 10° = 98 1 x cos1 0° = 966.09 N
=> IFx = O IFv = 0 => N - Wv = 0
=> P-Fs = 0 => l Ot = 80 N = Wv = 966.09N;
80 F = µN = 0.3x 966.09 =289.828 N
t = - = 8 sec
10 IFx = 0 => P = Wx -F
: . The block starts moving only => P + 289.828 -1 70.34 =O
when t > 8 seconds P = 1 1 9.488 N
P = ma = 1 1 9.488 N
During 8 seconds to 10 seconds of time: 1 1 9.488
=> a = = 1 . l 98 m/s 2
According to Newton's second law of motion 1 00
Force = mass x acceleration
dv 200 dv 28. Ans: a = 40m/s2
(P - FK ) = m x - =::, (l Ot - 40) = - x ­
dt 9.8 1 dt Sol:
10
200
f (1 0t - 40)dt = -f
9.8 1
dv
V

0
8

[5t2 - 40! 1° = 20.387 x V => (1 80-80) = 20.387 x V


Velocity (V) = 4.905 mis

jM11Q@jjji4.jji4jmniM\!ii¥.;1yderabad I Dellii I Bhopal I Pune I Bhubaneswar I Lucknow I Patna I Bengaluru I Chennai I VrjayawadaJ Vu.ag I Thupati J Kuk.atpally I Kolkata I
: 34 : ME GATE Previous Solutions

Tangential acceleration 30. Ans: (c)


aT = r a = 2x 1 2 = 24rn/s2 Sol:
Normal acceleration, aN = r oi
= 2 X 42 = 32 rn/s2
The resultant acceleration
a = �a � + a � = ..J24 2 + 3 2 2 = 40m / s 2 Instantaneous Centre

29. Ans: 57.67 m Mass per unit surface area of disc,


Sol: M
ma =
n[(2R )2 - R 2 ]
F
=
M =
M
w.=Wsin �;.�/ n(4R 2
- R 2)
31tR 2
,, Mass of elementary ring,
,,
, ,,',\,\,' w = cos 45° M 2M
\

, ,, , , ' y
m = --2 x 2m dr = --rdr
31tR 3R 2
\
(
W=mg = 98. IN
,,
/
Moment of inertia of that elementary ring is
\
\

given by,
Wx = W sin 45 = 98. 1 xsin 45 = 69.367 N
2M
Wy = W cos 45 = 69.367 N di = --rdr x r2
3R 2
IFv = O
As such elementary rings vary from r = R to
N -Wv = O
r = 2R, the total moment of inertia of whole
N = Wv = 69.367 N
disc is given by
F = µKN = 0.5 x 69.367 = 34.683N 2R
2M
IFx = 0 (Dynamic Equilibrium I= J r3 dr
R 3R
2
- o· Alembert principle )
Wx - F- ma = O � I = 2M2 [ R ]
4 2R

69.367 - 34.683 - l O xa = 0 3R 4 R
a = 3 .468m I s 2 2M X
= ! [16R 4 - R 4 ]
V = 20rn/s2 ; u = O; a = 3.468rn/s2 3R 4
2

V 2 =2as 2M _!_ 5MR 2


x x 1 5R 4
y 2 = 20 2
= =
3R 2 4 2
S - --- = 57. 67 m
=
2a 2 x 3.468
l@•li!i§jjji4ijjjiRflGi@il!jj.½iyderabad I Delhi I Bhopal I Pune I Bhubaneswar I Lucknow I Patna I Bengalwu I Chennai I Vtjayawada I Vu.ag I Tuupati I Kukatpally I Kolkala I
: 35 : Engineering Mechanics

. . 1 1 32. A ns: (a )
Tota1 kmetlc energy = - mV 2 + - Ico 2
2 2 S ol:
F2 =3 kN
V = rco = 2R m 3m
3m
V
co = - rad/s R
2R
Total Kinetic Energy
1 1 5mR 2 V
2
= mV 2 + x x( )
2 2 2 2R
5mV 2 1 3mV 2
= _!_ mV 2 + =
2 16 16 s ''
1 .5m ',' 3m
31. A ns : 10 N - m ''
S ol: m = 1 000 kg, r = 0.2 m 'X V1
LFy = O
coo = 10 rad/sec , t = 10 sec
V1 +V2 - 9+3 =O
co = coo + at
LMR = O
O = lO + a x lO
=:> Vi x 1 .5 +3 x 3 -9 x 6 = 0
a =- 1 rad/sec 2
=> V1 = 30 kN (t)
1
0 = ro 0 t + - at 2 V2 = -30 +9 - 3 = - 24 kN (t)
2
1 Adopting method of sections -section x-x
= 10 x 1 0 + - {- 1) x 1 0 2 = 100 - 50 adopted and RHS taken
2
20
0 = 50 rad 0 = tan -1 ( · ) = 53. 13 °
1 .5
1
Work done = - m V22 - V12 ) (
2 LFy = 0 (W.r.t. RHS of the section x-x)
V1 + F2 -Vi-Fy = 0
= _!_ mr2 (ro2 - ro� )
2 =:> Fsin 53. 1 3 = 30+ 3-24
F = 1 1 .25 kN (Tension)
TS = _!_ x l OOO x 0.2 2 x (- 1 00)
2 :. Force in member, QS = 1 1 .25 kN (Tension)
T x 50 = 2000 (or ) F1 = Fps sin(53. 1 3)
T = 40 N-m (anti-clock wise) Fps = 1 1 .25 compression
40 -Fps = Fos
Torque per wheel = - = I O N - m
4
:. Fos = 1 1 .25 1 (Tension)

!lfl•i!@jjji4ijjj4AflftiM!jjjjj+yderabad I Delhi I Bhopal I Pune I Bhubaneswarl Luclmow I Patna I Bengaluru I Chennai I Vtjayawada jVizag I T11Upati I .Kukatpally I Kolkata I
: 36 : ME GATE Previous Solutions

33. A ns: 2.0 5 3 rad/s2 35. A ns : 399 to 40 1 N


S ol: S ol:
7m

W=3x9.81=29.43N
3m
M = Ia
M = 29.43 x 3 = 88.29N-m W=600N
p
mf 2 3 X 82 2m � 2m
I = 1 0 + md 2 = -- + md 2 = -- + 3 x 3 2 1�
12 12
= 1 6 + 2 7 = 43kg - m 2

:EFy = 0
M 88 29
a = = · = 2.053 rad / s 2 Ns - W = O
I 43
Ns = 600 N
34. A ns: (a ) :EMA = 0
S ol: Px3 +Wx2 - Nsx 4= 0

,., T I
I
I
I ',
,,
',
I
P = 4NB - 2W
3

1::'._ _ I
I I
I I

4 x 600 - 2 x 600
I I

p 4m
I
I
I
I

P= = 400N
3
'!'__ _ ,/
0 36 . A ns: (c )
IMQ O =
S ol: PQ = QR = 4m
=> Px2 -Wx l = 0 N
Take moments about Q
100 x 9. 8 1
P= 490.5N 1 00 cos 60 x PQ = VR x 4

IFv = O => VR = 50 kN
=:> N = W = 981N
F = µN = P
490.5 = µ X 981 50
480.5 FPR = -- = 70.7 l kN
µ= = 0.5 sin 45
98 1

!lfl1ijjQjjji44jjjiRftbftM!iii!+)Hyc1erabad I Delhi I Bhopal I Pune I Bhubaneswarl Lucknow! Patna ! Bengaiuru I Chennai I Vliayawada IVmg I TU1lpati I Kukatpa11y I Kolkata I
: 37 : Engineering Mechanics

37. A ns: 0. 6 4 At point R,


S ol: Let, V 1 Velocity with which ball hits
=
1:Fy =
O
the ground FRQ =
FRT sin 45
V 1 = fii,h;
FRT = �
sin 45
vi = .J2 x 9.81xl = 4.427m /sec
1:Fx = 0
Velocity with which ball bounces up
20
= 4.427-20% of 4.427 FsR FRT cos 45 = -- x cos 45
=

sin 45
=
3.54 m/sec FsR = 20 kN
v_ 3.54 2
h2 = - 2 = = 0.64m
2g 2 x 9.81 40. A ns: 2 5 1 to 2 5 2

S ol: � = �
38. A ns: 29. 5 to 29.7 21t p

S ol: v0
dv
= 40 m / s ; a =-0.l v => - =-0.l v
e =s , 0 = ro , S = V
dt 21t P
V x 21t 20 x 21t
dv
= J-O.l dt + C 1 => ln v =-0.lt + C 1 ro = -- = ---
J V p 0.5

When t = 0 => v = 40 m / s => ln 40 = C 1 = 3.68 =>ro =


801t rad/sec =
25 1.32 rad/sec

ln v = 3.68 - 0.lt 41. A ns: 5 (range 4.9 5 to 5 .0 5 )


At, t =
3s => V = 29.63 mis S ol:
T = 20 N

39. Ans: (c )
S ol: Take moments about point P
VQ x 3 =
30 x 2
1:Fv 0
VQ = 20 kN
=

=> N =
W cos 0 =
0.8 W
At point Q, f µN
= =
0.2 W
1:Fy =
O 1:Fx =
0
FR0 = V0 => 0.6 W = 20 + 0.2 W
FRQ 20 kN
= => W = 50 N
=> m =
5 kg

jlJ1i4jj§jjjl4.jj4QnndMjj.jj+iyderabad I Delhi I Bhopal I Pune I Bhubaneswar l Lucknow I Palna I Bengahnu I Chennai I Vliayawada j Vmg I Tlrtlpati I Kukatpally I Kolkata I
: 38 : ME GATE Previous Solutions

42. A ns: (b ) 44. A ns: 133. 33 (range 130 to 135 )


270°
Sol: Sol:
TPQ oo

� 60°
TQR

By Lami' s theorem
w
-- = -- = -�- m1a1 = T - 1 00
T
TQR
sin 30 sin 60 sin 270
PQ

1 00
- a1 = T - 1 00 ------ ( 1 )
=> Tpo = J3w (T) g
=> TQR = -2 W= 2 W(C) m2a2 = 200 - T
200
- a2 = 200 - T ------ (2)
43. A ns: (b & d ) g
Sol: The rate of change of the angular 200 1 OO
a = 200 - a - 1 00
momentum of the particles about 'G' is g g
= ! (Lm x p x vJ
300a = 1 00 g

=> a = g
dp dv 3
= L( m- x v. + mp x -)
dt dt :. T = 1 00 a1 + 1 00 [·: a1 = a2]
1
I

dmp dv l OO
=-LV- x -- + Lmp x -i T= x + 1 00 = 1 33.33 N
dt dt
g

g 3

!
1

= -v i x (Lmp) + Lmp x � 45. A ns: 20 (range 19.9 to 20. 1)


1
Sol: - Ico2 - 0 = mgh
2
1 3
- x - mr 2 x co2 = mgh
2 2
- mv 2 = mgh
3
4
v 2 = - gh
4
3
v = �� gh = �� x 1 0 x 30 = 20 m/sec

l11••Qjj§jjjiUI0/4AflbiM\jj.jj� yderabad I Delhi I Bhopal I Pune I Bhubaneswar I Lucknow I Patna I Bengalwu I Chennai J Vliayawada I V!Zag I Tuupari I Kukatpally I Kolkata
: 39 : Engineering Mechanics

46 . A ns: 3 (range 2.9 to 3.1 ) 47. A ns: ( 6 80 )


S ol: V 1 = i + 2j S ol: Vw = 800 mis
IV1I = 2 + 1 2 = ,J"s
�h mw = 3 .4 kg/min
By drawing velocity diagram ma = 0.6 kg/min

0· 6 3.4 0 6 + 3.4 v
xO+ x 800 = ( · )x
60 60 60

45.33 = _!_ x v
15
V2

a = tan -I ( T) = 63 .44
V = 45.33 x 15 = 679.95
= 680 mlsec
By applying sine rule
V2
,Js = --- 48. A ns: (a )
sin 45 sin 7 1 .57 S ol: V = Rro
:. V2 = 3 mlsec

A lternate solution:
B
tJ-----V2

As the rod is rigid the velocity of all the 1 1 1 1


� - mV 2 + -mVA2 = -mVB2 + - mVB2
points on rod along the length must be same. 2 A 2 2 4

V1 = -.J2 2 + 12 = Js m l s

a = tan-1 (�) = 63.44 °

V 1 cos(63 .43 - 45) = V2 cos45


� V2 = 3 mis

!lflli@jjjiiijjjjjmftiM!hMi#yderabad l Delhi I Bhopal I Pune I Bhubaneswar l Lucknow l Patna l Bengaiuru l Chennai jVtjayawadalVizag I Tirupati I Kukatpally l Kolkata I
"' " - �
A ,CE
�··��FnpeeqPnl,li,,mrm
. . : 40 : ME_GATE_ Previous Solutions
� � ===�==================================
49. A ns: 7. 5
S ol:
- - rop
- - - - - - - - ... O(Instant centre)
45 ° ,
/:
I
,d' I fOQ
,' I

d 2 - 2x(½) ' - ½m'


Q Take moment about the I centre
Moment = M = I0 a
10 = Mass moment of inertia with respect to
m.t' 2
Given, L = Jim , m = 10 kg mstant centre O = I cG + md 2 = -- + md2
.

12
mL2
m( j
g = 10 m/s2, I cG = --
12 1 = Ji + m x _!_ = m + m = 4m = 2m
12 2 6 2 6 3
0

Angular acceleration, a = ?
a = Angular acceleration
The rod PQ is in general plane motion while
M = moment with respect to O = W xd 1
falling down.
= m x gx d 1
d1
Ji
= - x cos 45 2m
2 m x g x d1 = - x a
3
1
d1 = - m = d 2 � a = l x 10 x _!_ = 7 .5 rad / sec 2
2 2 2
rOQ = Ji x sin 45 = 1 m = rop

!M11Q@jjj44ijjjj@fliMfojjj� I I I I I I I I I I I
yderabad Delhi Bhopal Pune Bhubaneswarl Lucknow ! Patna Bengaluru Chennai Vyayawada Vizag Tirupati Kukatpally Kolkata
Strength of Materials
(Questions)
Page No. 42

CONTENTS
"- �---�--�r-·- - --�����- - - ��,- - - - - ,, -���� - - --
:' Chapter Questions Solutions
Name o f the Chap ter
:1

- ��� --�' Page No� . ��g _� �o. �


11


01 Simple Stress 43 - 50 5 1 - 56

02 Complex Stress 5 7 - 60 61 - 64

03 SFD & BM D 65 - 67 68 - 70

04 Centroids and Moment of Inertia 7 1 - 72 73 - 73

OS Pure Bending 74 - 76 77 - 80

06 Shear Stress in Beams 8 1 - 81 82 - 82


07 Springs 83 - 84 85 - 85

08 Torsion 86 - 90 9 1 - 95

09 Slopes and Deflections 96 - 99 100 - 104


10 Thin Cylinders 105 - 106 107 - 108
11 Column & Struts 109 - 1 10 111 - 112

12 Propped and Fixed Beams 113 - 113 1 14 - 1 14

13 Strain Energy 115 - 115 116 - 116


Ct Simple stresses
of elasticity of 100 GPa. Assume both the
One Mark Questions materials to be homogeneous and isotropic
and the axial force causes the same amount
01. A large uniform plate containing a rivet-hole of uniform stress in both the rods. The
is subjected to uniform uniaxial tension of stresses developed are with in the
95 MPa. The maximum stress in the plate is proportional limit of the respective
materials. Which of the following
observations is correct? (GATE-ME-03)
95 MPa 1 0 cm (a) Both rods elongate by the same amount
(b) M ild steel rod elongates more than the
cast iron
(c) Cast iron rod elongates more than the
(GATE-ME-92)
mild steel rod
(a) 100 MPa (b) 285 MPa
(d) As the stresses are equal strains are
(c) 190 MPa (d) Indeterminate
also equal in both the rods

02 . A free bar of length 1 m is uniformly heated 04. In terms of Poisson ' s ratio (µ) the ratio of
° °
from 0 C to a temperature of t C. a is the Young ' s Modulus (E) to Shear Modulus (G)
coefficient of linear expansion and E the of elastic materials is (GATE-ME-04)
modulus of elasticity. The stress in the bar is (b) 2 ( 1 - µ)
(a) 2(1 + µ)
(GATE-ME-95)
1 1
(a) atE (b) atE/2 (c) -(1 + µ)
2
(d) - (1 - µ)
2
(c) zero (d) none of the above
05 . A uniform, slender cylindrical rod is made
03 . Two identical circular rods of same diameter of a homogeneous and isotropic material.
and same length are subjected to same The rod rests on a frictionless surface. The
magnitude of axial tensile force. One of the rod is heated uniformly. If the radial and
rod is made of mild steel having the longitudinal thermal stresses are represented
modulus of elasticity of 206 GPa. The other by crr and crz respectively, then
rod is made of cast iron having the modulus (GATE-ME-04)
!li1@4i@jjjl4lm4Aflfij@jj.jj� ydcrabad I Delhi I Bhopal I Punc I Bhubancswar I Lucknow I Patna I Bcngaluru I Chcnnai I Vliayawada I Vmg I Tirupati I Kukatpally l K.olkata
: 44 : Strength of Materials

(a) crr = O ,crz = O (b) O"r ::;t: 0, O"z = 0 08. A circular rod of length 'L ' and area of
(c) crr = O, crz ::;t: O (d) O"r ::;t: 0, O"z ::;t: 0 cross-section 'A ' has a modulus of elasticity
'E ' and coefficient of thermal expansion 'a ' .
06. A steel rod of length L and diameter D, One end of the rod is fixed and other end is
fixed at both ends, is uniformly heated to a free. If the temperature of the rod is
temperature rise of �T. The Young ' s increased by AT, then
modulus is E and the coefficient of linear (GATE-ME-14-SET-l)
expansion is 'a ' . The thermal stress in the (a) stress developed in the rod is EaAT and
rod is (GATE-ME-07) strain developed in the rod is a AT
(a) 0 (b) aA T (b) both stress and strain developed in the
(c) EaA T (d) EaA TL rod are zero
(c) stress developed in the rod is zero and
07. A rod of length L having uniform cross­ strain developed in the rod is aAT
sectional area A is subjected to a tensile (d) stress developed in the rod is EaAT and
force P as shown in the figure below. If the strain developed in the rod is zero
Young ' s modulus of the material varies
linearly from E 1 to E2 along the length of the 09. A metallic rod of 500mm length and 50mm
rod, the normal stress developed at the diameter, when subjected to a tensile force
section - SS is (GATE-ME-13) of 1 OOkN at the ends, experiences an
r--+ s increase in its length by 0.5mm and a
E1 reduction in its diameter by 0.015mm. The
P +-- I---+ p Poisson ' s ratio of the rod material is
(GATE-ME-14-SET-1)
�s
L/2
�1 �, 10. A steel cube, with all faces to deform, has

L Young ' s modulus, E, Poisson ' s ratio, v, and
coefficient of thermal expansion, a. The
P(E, - E 2 ) pressure (hydrostatic stress) developed
(a) !_ (b)
A A(E 1 + E 2 ) within the cube, when it is subjected to a
PE 2 PE, uniform increase in temperature, �T, is
(c) (d)
AE 1 AE 2 given by (GATE-ME-14-SET-2)

!Iii•i!@jjjgq.jjjjJmfilM\i&m.*/ �yderabad I Delhi I Bhopal I Pune I Bhubaneswar I Lucknow I Pama I Bengaluru I Chennai I Vijayawada I Vizag I T1111pati I Kukatpally I Kolkata J
:45: Simple Stresses

a(�T)E 13. The number of independent elastic constants


(a) 0 (b)
1-2v required to define the stress-strain
_ a(�T)E a(�T)E relationship for an isotropic elastic solid is
(c) (d)
1-2v 3(1-2v) (GATE-14-SET-4)

11. The stress-strain curve for mild steel is 14. A rod is subjected to an uniaxial load within
shown in the figure given below. Choose the linear elastic limit. When the change in the
correct option referring to both figure and stress is 200 MPa, the change in the strain is
table (GATE-14-SET-3) 0.001. If the Poisson's ratio of the rod is 0.3
the modulus of rigidity ( in GPa) is___
(GATE-15-Set 2)

15. The room-temperature stress (cr) - strain(e)


curves of four materials P,Q,R and S are
shown in the figure below. The material that
Stress e (%)
behaves as a perfectly plastic material is
Point on Description of the point
cr
the graph
p 1. Upper yield point
Q 2. Ultimate Tensile Strength p
R 3. Proportionality Limit
s 4. Elastic Limit
T 5. Lower Yield Point
(GATE -PI-15)
u 6. Failure
(a) p (b) Q (c) R (d) S
(a) P-1, Q-2, R-3, S-4, T-5, U-6
(b) P-3, Q-1, R-4, S-2, T-6, U-5
16. Consider the following statements:
(c) P-3' Q-4' R-1' S-5' T-2' U-6
(P) Hardness is the resistance of a material
(d) P-4' Q-1' R-5' S-2' T-3' U-6
to indentation.
(Q) Elastic modulus 1s a measure of
12. If the Poisson's ratio of an elastic material is
ductility.
0.4, the ratio of modulus of rigidity to
(R) Deflection depends on stiffness.
Young's modulus is __ (GATE-14-SET-4)
jlJINi@jjj§§mijQflntM\Uii!+1Yderabad I Delhi I BhopalI PuneI Bhubaneswarl Lucknow! Patna I Bengaluru I Oiennai I Vijayawada I V123g I Tirupati I K�y I Kolkata I
1
, ,....... A!CE
... Strength of Materials
� :�Pnbtiatiaos : 46:
==
..,

..' '===========================================
ti • • • •

(S) The total area under the stress-strain 18. The Poisson's ratio for a perfectly
curve is a measure of resilience. incompressible linear elastic material is
Among the above statements, the correct (GATE-17-SET- l)
ones are (GATE -PI-16) (a) 1 (b) 0.5 (c) 0 (d) infinity
(a) P and Q only. (b) Q and S only.
(c) P and R only. (d) R and S only. 19. In the engmeenng stress-strain curve for
mild steel, the Ultimate Tensile Strength
17. A beam is subjected to an inclined (UTS) refers to (GATE-17-SET-1)
concentrated load as shown in the figure (a) Yield stress (b) Proportional limit
below. Neglect the weight of the beam. (c) Maximum stress (d) Fracture stress

I 20. A steel bar is held by two fixed supports as


shown in the figure and is subjected to an
increase of temperature �T = 100°C. If the
The correct Free Body Diagram of the beam coefficient of thermal expansion and
lS (GATE - PI-16) Young's modulus of elasticity of steel are
l lxl 0--6/°C and 200GPa, respectively, the
(A)
magnitude of thermal stress (in MPa)
induced in the bar is---

(B)

(GATE-17-SET-2)
(C)
21. If E is the modulus of elasticity in GPa, G is
the shear modulus in GPa and v is the
Poisson's ratio of a linear elastic and
(D) (i-----"----. isotropic material, the three terms are
related as (GATE-PI-17)
(a)E=G (l-2v) (b)E=2G(l-v)
(c)E=G ( l +2v) (d) E=2G(l +v)

!ltlli@mi4.jjiRffb1Mihfo� yderabad I Delhi I Bhopal I Pune I Bhubaneswar I Lucknow I Patna I BengahnuI Chennai IVtiayawada jV17.3g I Tuupati I Kukatpally I Kolkata
ACE : 47: Simple Stresses
��-�
. ..�&,pw,qPublimiooa
., 'ii! • • • •

Pb/ Pa/
T2 = 2 2
Two Marks Questions (a +b }'
2 2 (a +b )
Pa/ Pb/
( 2}' T2 (
01. Determine the temperature rise necessary to 2 a +b
2
2 a 2 +b 2 )
induce buckling in a 1m long circular rod of
diameter 40 mm shown in the figure below. 03. A 200 x 100 x 50 mm steel block is
Assume the rod to be pinned at its ends and subjected to a hydrostatic pressure of 15
the coefficient of thermal expansion as MPa. The Young's modulus and Poisson's
20x10-6/° C. Assume uniform heating of the ratio of the material are 200 GPa and 0.3
bar. (GATE-ME-93) respectively. The change in the volume
40 mm diameter of the block in mm3 is (GATE-ME-03)
27

[[-+----f
� (a) 85 (b) 90 (c) 100 (d) 110
(

4
_? 1-
m ----t�� 04. The figure below shows a steel rod of 25
mm2 cross sectional area. It is loaded at four
02. Below Fig. shows a rigid bar hinged at A points. K, L, M and N. Assume Estee! = 200
and supported in a horizontal position by GPa. The total change in length of the rod
two vertical identical steel wires. Neglect due to loading is (GATE-ME-04)
the weight of the beam. The tension T I and
lO�� - _ _ ---r- -�N
-- � - -0N---- 00
T2 induced in these wires by a vertical load � L � 25 2 N M

1
+1

P applied as shown are (GATE-ME-94) ·:


1700mm
�1
-----------
500mm 1:"�1�
(a) 1 µm (b)-10 µm
a-:
I
I

(c) 16 µm (d)-20 µm

r/
05. A steel bar of 40 mm x 40 mm square cross­
-1 section is subjected to an axial compressive
--p- -: load of 200 kN. If the length of the bar is 2
p m and E = 200 GPa. The decrement in
(a)T1=T2 = - length of the bar will be (GATE-ME-06)
2
Pa/ Pb/ (a) 1.25 mm (b) 2.70 mm
(b)T1 = T2= 2 2
(a +b }' (a +b )
(c) 4.05 mm (d) 5.40 mm
2 2

\( l I 11g11Ht1111� P11hl1<.1t11ll\', yderabad I Delhi I Bhopal I PuneI Bhubaneswar I Lucknow I PatnaI Bengaluru I Chennai I VtjayawadaI Vizag I Tirupati I Kukatpally I Kolkata
: 48: Strength of Materials

06. A bar having a cross-sectional area of 700 09. A metallic bar of uniform cross-section with
mm 2
is subjected to axial loads at the specific weight of 100 kN/m3 is hung
positions indicated. The value of stress in vertically down. The length and Young's
the segment BC is (GATE-ME-06) modulus of the bar are 100 m and 200 GPa,
respectively. The elongation of the bar, in
63
��-3_kN mm, due to its own weight is ____
kN
5 _E __4_9kN_�E-��
A B C D (GATE - PI-15)
10. A horizontal bar with a constant cross­
(a) 40 MPa (b) 50 MPa section is subjected to loading as shown in
(c) 70 MPa (d) 120 MPa the figure. The Young's modules for the
sections AB and BC are 3E and E,
07. A solid steel cube constrained on all SIX respectively.
faces is heated so that the temperature rises
uniformly by dT. If the thermal coefficient F
of the material is a, young's modulus is E
L
and the Poisson's ratio is u, the thermal
L

stress developed in the cube due to heating For the deflection at C to be zero, the ratio
IS (GATE-ME-12) P/F is (GATE-16-SET-1)
_ a(dT)E 2a(dT)E
(a) (b)-
(1-2u) (1-2u) 11. A hypothetical engmeenng stress-strain
_ 3a(dT)E _ a(dT)E curve shown in the figure has three straight
(c) (d)
(1-2u) 3(1-2u) lines PQ, QR, RS with coordinates P(O,O), Q
(0.2,100), R(0.6, 140) and S(0.8, 130). 'Q' is
08. A 200 mm long, stress free rod at room the yield point, 'R' is the UTS point and 'S'
temperature is held between two immovable the fracture point.
rigid walls. The temperature of the rod is 160 0 .6, 140
°
uniformly raised by 250 C. If the Young's �
140
120
i::1-.
modulus and coefficient of thermal 6 100

expansion are 200 GPa and 1x1o-5/° C,


� 80 I
! 60 l
respectively, the magnitude of the
40 1
Jc,o 20 1 p
0, 0
longitudinal stress(in MPa) developed in the O
0 .2 0 .4 0 .6 0 .8

rod is (GATE-ME-12)
0
Engg. Strain(%)

!ltlli!@jji§§jjjjiRflftiM\ii!jj� yderabad I Delhi I Bhopal I Pune I Bhubaneswar I Lucknow I Patna I Bengalwu I Chennai IV\iayawada IVizag I T1n1pati I Kukatpally I Kolkata
: 49: Simple Stresses

The toughness of the material (in MJ/m3 ) is 200°C, the axial stress developed in the rod
(GATE-16-SET-1) 1s ___MPa.

12. In the figure, the load P = 1 N, length L = 1


m, Young's modulus E = 70 GPa, and the
cross-section of the links is a square with
I-.____,_JI
=:J 250 mm I+- 0.2mm

dimension 10 mm x 10 mm. All joints are Young's modulus of the material of the rod
pin joints. is 200 GPa and the coefficient of thermal
expansion is 10-5 per °C.
(GATE-16-SET-2)

14. A square plate of dimension L x L is


subjected to a uniform pressure load P = 250
MPa on its edges as shown in the figure.
Assume plane stress conditions. The
A
Young's modulus E = 200 GPa.

p
The stress (in Pa) in the link AB is __
(Indicate compressive stress by a negative
sign and tensil e stressby apositive sign)
(GATE-16-SET-2) p L

13. A circular metallic rod of length 250 mm is


placed between two rigid immovable walls p
as shown in the figure. The rod is in perfect The deformed shape 1s a square of
contact with the wall on the left side and dimension L-28. If L = 2 m and 8 = 0.001
there is a gap of 0.2 mm between the rod m, the Poisson's ratio of the plate material is
and the wall on the right side. If the
temperature of the rod is increased by (GATE-16-SET-3)

Delhi I Bhopal I Punc I Bhubancswar I Lucknow I Patna I Bcngaluru I Chennai I Vijayawada I Vizag I Tirupati I Kukatpally I Kolkata I
\( } ( ll�llll l I Ill� P11!1ll( .111, )[I'-
Fyderabad I

,.
, � ACE · Pi� : 50: Strength of Materials
, ....,."�
... EngiomJ

15. A horizontal bar, fixed at one end (x = 0), shown (Figure is not to scale). If g = 10
has a length of 1 m, and cross-sectional area m/s2 , the maximum compression of the
of 100 mm2 • Its elastic modulus varies elastic bar is ___ mm.
along its length as given by E(x) = 100 e x-
GPa, where x is the length coordinate (in m) m = 100kg
along the axis of the bar. An axial tensile H= 10 mm
load of 10 kN is applied at the free end
(x = 1). The axial displacement of the free
end is --- mm. L= lm
(GATE-17 - SET-1)

16. A point mass of 100 kg is dropped onto a


massless elastic bar (cross-sectional area = (GATE-17-SET-1)
100 mm2 , length = 1 m, Young's modulus =
100 GPa) from a height H of 10 mm as

!M•lii@jjjl§@i@fij@j!iih+yderabad!DelhilBhopa)IPunelBhubancswarl Lucknow!Patna!BengalurujChennaijVuayawadajVizag !Tirupati I Knkalpa)ly! Kolk.ala I


SOLUTIONS
06. Ans: (c)
One Mark Solutions Sol: Temperature stress = a(�TXE)

07. Ans: (a)


01. Ans: (c)
Sol: Maximum stress m plate develops across Sol: s
rivet hole, let P be the axial tensile force.
p p
P = 95 x t x 10cm = cr x tx (10 - 5)cm
lO
Maximum stress, cr =95 x =190MPa L/2
5
p
Normal stress = -
A
02. Ans: (c)
Normal stress for determinate structures is
Sol: A bar free to expand due to temperature rise
independent ofyoung's modulus
does not develop any stress.

08. Ans: (c).


03. Ans: (c) Sol: Circular rod free to expand, stress developed
PL
Sol: 8/= is zero. Strain developed is a (�T).
AE
1 09. Ans: 0.3
3/ oc-
E Sol: Length, L = 500mm
As E of cast iron is less than the mildsteel Diameter, D = 50mm
then the change in length is more in cast iron Tensile force, P = 100 kN
Increase in length, 8L = 0.5mm
04. Ans: (a)
Decrease in diameter, 8D = 0.015mm
Sol: E=2G(l +µ)
E/G =2(1+µ)
Poisson's ratio µ - " 1""''
oi)
8t)
- (

' ElrneM (

05. Ans: (a)


(0.015/50) =
Sol: Free to expand means no stresses will µ= 0.3.
(0.5/500)
develop
\( I l 1Il-:,lll( t llll� P11lil1<,lllt)II", �yderabad I Delhi I Bhopal I Pune I Bhubaneswarl Lucknow! Patna I Bengaluru I Chennai I Vtjayawada I Viz.ag I Trrupati I Kukatpally I Kolkata I
: 52: Strength of Materials

·10. Ans: (a) 16. Ans: (c)


Sol: A cube free to deform has no stresses. Sol: Hardness of a material will be measured by
indentation technique.
11. Ans: (c) If a material undergo more strain
(deflection) in elastic region means stiffness
12. Ans: 0.36
is low.
Sol: Poisson's ratio,µ = 0.4
E = 2G(l+ µ )
17. Ans: (b)
G -- 1
= -
E 2(1+µ)
18. Ans: (b)
l Sol: Poisson's ratio of perfectly incompressible
= 0.36
2(1+ 0.4)
(non - dilatant) material is 0.5 (maximum).

13. Ans: (2) 19. Ans: (c)


Sol: Number of independent elastic constants for
isotropic material are two and these two are 20. Ans: 220
E (young's modulus) andµ (Poisson's ratio) Sol: Temperature increase, T = 100°C
Young's modulus ofelasticity,
14. Ans: 76 to 78 E = 200x 103MPa
Sol: cr=200N/mm 2 Coefficient ofthermal expansion,
f
E =0.001 mm/mm a = l lx lf -ol°C
µ= 0.3 Thermal stress, CTt = (at) E
cr=EE = (1 l x 10-6)(100)(200x103)
cr 200 = 220 MPa
E= = =200 x 10 3 N /mm 2
E 0.001
E =2G(l +µ) 21. Ans: (d)
E =.
G= 76.92 x 10 3 MPa
2(1+µ)
= 76.92GPa

15. Ans: (d)

!IS•liiiYhi44ihiiAflblM\iiill4 ydcrabad I Delhi I Bhopal IPune I Bhubaneswarl Lucknow I Patna I Bengaluru I Chennai IVuayawadalVi.zag ITirupati I Kukalpal)yj Kolkata
: 53: Simple Stresses

Pia
S.1m1·1ar1y, T1 =
Two Marks Solutions (a + b 2 )
2

01. 03. Ans: (b)


Sol: Euler's buckling load Sol: For hydrostatic pressure
:rr2
Pe=- 2 EI---- (1)
(jx =(jy =az =a
/

Temperature thrust,
P = (a tE)A. --- (2)
&=a(l-2µ)
For equilibrium, equation 1 = 2 ; E
7t 2 7t 2 7t oV
EI = a x t x E x A � E x -( 40)4 t y= - = t X +t y +t z =
V
3t
£ (1000) 2 64
-
2

3 x l5
= 20 xl0-6 x t x E x 1t x 402 oV = (1-2 x 0.3){200 x 100 x 50}
200 x 10 3

� t = 49.34°C o V = 90 mm 3

02. Ans: (b) 04. Ans: (b)


Sol: "f.MA = 0 Sol:
T2 (b) + T1(a) =P(/) ---- (1) 100 N �lOO N
of, i(b) = of, 2(a) 150 N �150 N
T1 l = Ti f x a
x (b) 50 N �50 N
AE AE
a
T1 =- x T2
b c)/ = o/KL +o/LM +o/MN
Sub P 1 in equation (1) r,e 100 x 500 (-150 Xsoo)
= 3 +
(a2) 25 x 200 x l0 25 x 200 x l0 3
T2 (b) + T2 - - = P( l)
b 50 x 400
+-----3
T2 ( b': a')= P l
= -lOxlO---{; m =-lOµm
25 x 200 x 10

_ Plb
T2 -
(a +b2)
2

l1t11ii@hli4•hl4Wbiiihih� yderabad I Delhi I Bhopal I Pune I Bhubaneswar I Lucknow I Patna I BengaluruI Chennai I VuayawadajVmg I Tirupati I Kukatpally I Kolkata

J-.,.._.��PnNiaaoos
ti, ACE
V • • • • : 54 : Strength of Materials

05. Ans: (a) Strain in X-direction due to temperature rise,


PL
Sol: ol =
s 1 = a(� T)
AE Strain in X-direction due to volumetric
f>l = (2oox 10 }2000) = 3
stress
1.25 mm
40x 40x 200x 10 3 ( ) ( ) ( )
s 1 = -cr -u -cr -u -cr = a�T
E E E
06. Ans: (a) � cr (2u-1) = a�T
Sol: E
� � 63 kN
63 kN �
= ( a�TE ) = -(Ea�T)
A B cr 2u-1 1-2u
FBD of BC:
08. Ans: 500

28 kN +-c=J-. 28 kN Sol: Stress developed due to temperature rise

B C cr = atE
= 1 x 10·5 x 250 x 200 x 103 = 500 MPa

21kN� 21 kN 09. Ans: 2.5


C D = yR 2 = 100x 103 x (100)2 x 1000 =
Sol: 0 2_5mm
2E 2x 200x l 09
Load
cr = - -
Area 10. Ans: 4 (range 3.9 to 4.1)
- 28 X 10 - 40MPa Sol:
3
(]'BC -
700 � (�F
(F:::_P)�P)
3E E
07. Ans: (a)
Sol: OAB + 0Bc = 0
(F-P )L + (FXL) -
-0
A(3E) (A XE)
cr
a (F-P) +F = 0

a
cr F-P + 3F= 0

4F = P� p = 4
O'x = cry = O'z = cr F
\( I l lt:.!,llltt1 111:..:, P11!1lH,l[ICllh ydcrabad I Delhi I Bhopal I Pune I Bhubaneswar I Lucknow! Patna! Bcngaluru I Chennai I Vtjayawada I Vizag ITirupati I Kukatpally I Kolkata
: 55 : Simple Stresses

11. Ans: 0.8 5 (range 0.849 to 0. 851) 14. Ans: 0.2 (range 0.18 to 0.22)
Sol: Toughness = Area under diagram Sol: Ev = Ex + cy + tz
0 ·2 0.4 µ ( cr + cr)
= .!.x x10o +.!. ( )( 100 +140 ) + €
= (cr _ µcr ) + (cr _ µcr ) _
2 100 2 100 V E E E E E
.!. 0·2 Due to plane stress condition, 0
( )( 140 +130 ) Ez =

2 100
t v = ; X 2 X (1-µ )
T = 0.1 + 0.48 + 0.27 = 0.85 M J/m3
( 2 - 2x o.001) - 22
= p
2

12. Ans: 0 (range -1 to 1) --'----- 2 -'--- --x 2 ( 1 - µ )


2 E
Sol: rFx = 0 250
-2x10 3 = ----3 x 2 ( 1 - µ )
-

P sin45° = FB cos45° 200x l 0


FBc = P 200
- 1-µ
-
250
IFy =O p
FAB 0.8 = 1 - µ
:. µ = 0.2
FBc sin45° -P cos45 + FAB = 0
15. Ans: (1. 718)
Sol: I

Strii :x
:. FAB O
1 1,.
=

: . CJ'AB = O � p

:1
-+ldx :

13. Ans: 240 (range 239 to 241) I• L


Sol: R a t = 250x (10-5 X200)
1
= - = 0.5mm > gap Change in length of small strip,
2
Deformation prevented b = Px_x (dx)
A X _ X E XX
PR
0.5 -0.2 = 0.3 = -
=
AE
=> cr(250) Total change in length of bar,
0.3 = L P dx)
200x 103 8 o - r 8 - r ( x -x X
L
t tal - Jo Jo (Ax x )E x-x
cr = 240 MPa -
Px-x P Constant
= =

IM•@i!j§jjjl4@4Rbfiilmih� yderabad I Delhi I Bhopal I Pune I Bhubaneswarl Lucknow! Patna I Bengaluru I Chennai I VgayawadalV17.3g I Tuupati I KukatpaJiyf Kolkata
: 56 : Strength of Materials

Ax-x = A = Constant
Ex-x = 1 00 e-x GPa
p fL dx p fl dx Impact factor
8 to1ai =
A Jo -
l_Oe___
O_ x =_ 9
0_x__
1_ _ Jo e-x
l O_O_A
= 1 000 x 1 000 3 [ l + l + 2 x 1 00 x l 0 x l OO x l O ]
3
3
10 x l0 ( x )l l OO x l OO x 1 0 l OOO x 1 000
= 1 00 X 1 0 -6 X 1 00 X 1 0 9 \e O
3
= l .7 1 82 x l 0- m = _!_ [1 + .J1 + 200 ] = 1 .5 1 77 mm
10
Dtotal = 1 .7 1 8 mm

16. Ans: 1.5177


Sol: m = 1 00 kg ; H = 1 0 mm ;
L = 1 m= 1 000 mm ; E = 1 00 GPa ;
g = 1 0 m/sec2
W = 1 00 x 1 0 = 1 000 N

\(' l I 11�11u l 1 1 1 1g P11hlu .111011, yderabad I Delhi I Bhopal I Punc I Bhubaneswar I Luck nowI PatnaI Bengalwu I C henna i I Vgayawad aI Vi7.ag I Tirupati I Kukatpally! Kol.kata
Complex Stresses
04. The figure shows the state of stress at a
One Mark Questions certain point in a stressed body. The
magnitudes of normal stresses in the x and y
0 1 . An elastic body is subjected to a tensile
directions are 1 00 MPa and 20 MPa
stress X in a particular direction and a
compressive stress Y in its perpendicular respectively. The radius of Mohr's stress

direction. X and Y are unequal in circle representing this state of stress is


magnitude. On the plane of maximum shear (GATE-ME-04)
stress in the body there will be:
(GATE-M E-89)
(a) no normal stress
(b) also the maximum normal stress
( c) the minimum normal stress er Y
(d) both normal stress and shear stress (a) 1 20 (b) 80 (c) 60 (d) 40

02. A cylindrical elastic body subjected to pure


05. If the principal stresses in a plane stress
torsion about its axis develops:
problem are cr 1 = 1 00 MPa, = 40 MPa,
(GATE-ME-89)
cr2

the magnitude of the maximum shear stress


(a) tensile stress in a direction 45 ° to the
(in MPa) will be (GATE-ME-09)
axis
(b) no tensile or compressive stress (a) 60 (b) 50
(c) maximum shear stress along the axis of (c) 30 (d) 20
the shaft
(d) maximum shear stress at 45 ° to the axis 06. The state of plane-stress at a point is given
by crx = 200 MPa, cry = 1 00 MPa and
03. If the two principal strains at a point are 'txy = 1 00 MPa. The maximum shear stress
-6 -6
1 000 x 10 and -600 x 1 0 , then the (in MPa) is (GATE-ME-10)
maximum shear strain is (GATE-ME-96) (a) l l l . 8 (b) 150. 1
6 --{i
(a) 800 X 1 0- (b) 500 X 10 (c) 1 80.3 (d) 223.6
-6 -6
(c) 1 600 X 10 (d) 200 X 1 0

!IJ11i@iii44ijjj$RflblM\ii!h+1ydcrabad l Delhi I Bhopal I Punc I Bhubaneswarl Lucknow I Patna I Bcngaluru I Chcnnai I Vuayawadal Vwg ITuupali I Kukalpallyl Kolkata I
07. A thin plate of uniform thickness is subject 10. The state of stress at a point on an element is
to pressure as shown in the figure below shown in figure (a). The same state of stress
is shown in another coordinate system in
figure (b) .

z 'txy

Under the assumption of plane stress, which


one of the following is correct ?
(GATE-ME-14-SET-2) 'tyy
Fig. (b)
(a) Normal stress is zero in the z-direction Fig. (a)

(b) Normal stress is tensile in the z­


direction The components ('txx, 'tyy, 'txy) are given by
(c) Normal stress is compressive in the z­ (GATE -16-SET-3)
direction (a) ( P / Ji, -P I Ji, O ) (b) (O, 0, P)
(d) Normal stress varies in the z-direction
(c) (P, -P, -P I Ji ) (d) ( 0, 0, P / Ji )

08. If the principal stress values are 120 MPa,


-50MPa and 10 MPa in a given state of 11. The state of stress at a point is crx=cry=crz=
stress, then maximum shear stress in the 0 and 'txy 'tyx=50MPa. The
'txz='tzx='tyz='tzy = =

material, in MPa, is ----- maximum normal stress (in MPa) at that


(GATE - PI-15) point is ___ (GATE-17 -SET - 2)

09. A shaft with a circular cross-section is 12. For a two dimensional state-of-stress
subjected to pure twisting moment. The ratio defined as CTxx = cryy = 'txy = S, the Mohr's
of the maximum shear stress to the largest circle of stress has (GATE-Pl-17)
principal stress is (a) center at (S, 0) and radius S
(GATE-16-SET-2) (b) center at (0,0) and radius S
(a) 2.0 (b) 1. 0 (c) 0.5 (d) 0 (c) center at (S, 0) and radius 0
(d) center at (S/2,0) and radius 2S

lii11i!i§jjji4ijjjiAflGi@jj.jj.yderabadj Delhi I Bhopal j Pune I Bhubaneswar l Lucknow! Patna! Bengaluru I Chennai j Vtjayawada jV,zag j Tuupati I Kukatpallyj Kolkata I
: 59 : Complex Stresses

Comm o n data for 04 & 05.


Two Marks Questions The state of stress at a point P in a two
dimensional loading is such that the Mohr's
01. The three-dimensional state of stress at a circle is a point located at 175 MPa on the
point is given by: (GATE-ME-90) positive normal stress axis.

f
30 1 0 -! OJ
[cr] = 10 0 2 0 MN /m2 04. The maximum and minimum principal
- 1 0 20 0 stresses respectively from the Mohr's circle
The shear stress in the x-y plane at the same are (GATE-ME-03)
point is then equal to (a) + 175 MPa, - 175 MPa
(a) zero MN/m2 (b) -10 MN/m2 (b) + 175 MPa, + 175 MPa
(c) 10 MN/m2 (d) 20 MN/m2 (c) 0,-175 MPa
(d) 0, 0
02 At a point in a stressed body the state of
stress on two planes 45 ° apart is as shown 05. The directions of maximum and minimum
principal stresses at the point "P" from the
below. Determine the two principal stresses
Mohr's circle are (GATE-M E-03)
(GATE-ME-93)
(a) 0, 90° (b) 90°, 0
(c) 45°, 135° (d) All directions

8 MPa 06. The Mohr's circle of plane stress for a point


in a body is shown. The design is to be done
on the basis of the maximum shear stress
3 MPa
theory for yielding. Then, yielding will just
2 MPa
begin if the designer chooses a ductile
material whose yield strength is
03. A shaft subjected to torsion experiences a
pure shear stress 't on the surface. The 't(MPa)
maximum principal stress on the surface
which is at 45° to the axis will have a value
cr(MPa)
(GATE-ME-03)
(a) 't cos 45° (b) 2't cos 45° 1 00
(GATE-ME-04)
-10
° ° °
(c) 't COS 45 2
(d) 2't sin 45 cos 45
!1t11iii§hiiiiiiiNAflfiiMiiiih+YdCrabad I Delhi I Bhopal I Pune I Bhubaneswar I Lucknow I Patna I Bengaluru I Chennai I VijayawadaJVu.ag rnrupari I Kukatpally I Kolkata I
: 60 : Strength of Material

(a) 45 MPa (b) 50 MPa 11. A bar of rectangular cross-sectional area of


(c) 90 MPa (d) 100 MPa 50 mm2 is pulled from both the sides by
equal forces of 100 N as shown in the figure
07. A solid circular shaft of diameter 100 mm is below. The shear stress (in MPa) along the
subjected to an axial stress of 50 MPa. It is plane making an angle 45° with the axis,
further subjected to a torque of 10 KN-m. shown as a dashed line in the figure, is __.
The maximum principal stress experienced ''
''
on the shaft is closest to (GATE-ME-08) ''
'
(a) 41 MPa (b) 82 MPa l OO�
li_J.___�
-� !
'-----�LlQO N
45 (\ '
°

'

(c) 164 MPa (d) 204 MPa ''
''
' (GATE - PI-16)
08. A two dimensional fluid element rotates like
a rigid body. At a point within the element,
12. A rectangular region in a solid is in a state of
the pressure is 1 unit. Radius of the Mohr's
plane strain. The (x,y) coordinates of the
circle, characterizing the state at that point,
comers of the undeformed rectangle are
(GATE-ME-08)
given by P(O,O), Q(4,0), R(4,3), S(0,3). The
lS

(a) 0.5 unit (b) O unit


rectangle is subjected to uniform strains, Exx
(c) 1 unit (d) 2 unit
= 0.001, Eyy = 0.002, Yxy = 0.003. The
09. The state of stress at a point under plane deformed length of the elongated diagonal,
stress condition is upto three decimal places, is __ units.
crxx 40 MPa, cryy=100 MPa & 'txy = 40MPa.
= (GATE-17-SET-l)
The radius of the Mohr's circle representing
the given state of stress in MPa is
(GATE-ME-12)
(a) 40 (b) 50 (c) 60 (d) 100

10. In a plane stress condition, the components


of stress at a point are crx = 20 MPa, cry = 80
MPa and 'txy = 40 MPa.The maximum shear
stress (in MPa) at the point is
(GATE-15-Set 2)
(a) 20 (b) 25 (c) 50 (d) 100

\t I l J1�111l l 1 111� P1ihl1, .11Jt111, �yderabad ! Dellii l BhopaI ! Pune ! Bhubaneswarl UICknow l Patna l Bengaluru ! Chennai l Viiayawada l Vizag j Tuupati I Kukatpallyl KolkataI
SOLUTIONS
03 . Ans: (c)
One Mark Solutions
= 1 000 X 10-6 -(-600 X 10 -6 )
01. Ans: (d)
= 1600x 10-6
Sol: The given state of stress is indicated on the
element , the corresponding Mohr's circle 1s
04. Ans: (c)
also shown.
Sol:
100
2

cr - cr Y
Radius of M ohr circle = "
2
= 100 - (- 20) = 60 MPa
maximum
shear
2

05. Ans: (b)


Sol: G iven o-1 = 100 MPa, o-2 = 40 MPa,
cry crx
Plane of maximum shear 1s subjected to Use, cr3 = 0
normal stress also. l OO - O
50 MPa
2
02. Ans: (a)
Sol: cr = t cr = t 06. Ans: (a)
200 + 100 200 - 100
S0I .• ---- +
2
CTi , CT2 - ( ---- ) +1002
) -�- -) 2 2
cr = t cr = t
= 150 ± 50 .Js
If an element is subjected to pure torsion,
one diagonal is subjected to tensile stress cr, = 261.80MPa , cr 2 = 3 8.19MPa
and another diagonal is subjected to = 261.80-3 8 . 19
= l l l .S MPa
tmax

compressive stress. (Refer figure). 2

\( } I ! l ..,1,1( l l l ! I ..., Pt i l d H ,ill< j[ j', Fyderabad l Delhi l Bbopai l Pune l Bhubaneswar l Lucknowl Palna l Bengaluru l Oicnnai l Vuayawada J V,zag I TinJpali I Kukalpallyl Kolkata I
�··��&gnw:nngPnbliratiom
A ,CE
: 62 : Strength of Material
� ==========================================
., V • � • •

07. Ans: (a) 12. Ans: (a)


Sol: Under plane stress condition stress along Sol:
thickness direction (i.e. Z ) is zero.
( S, S)

08. Ans: 85
Sol: <Jx = 120 MPa , cry =-50 MPa, <Jz = l OMPa
er

_ 120-(-50) _
"Cmax - ---- - 85
2 ( S, -S )
Center � (S, 0) Radius � S
09. Ans: (b)

Sol: -rmax =1
0-1 Two Marks Solutions

-��] l:
01. Ans: (c)
10. Ans: (b)
Sol:
p
0=90°
Sol: cr = [ �
-10 20
:i o

0
=
"C zx
°
0=0
Comparing similar elements
-p p
"C xy = l OMPa

-p
02.
At the plane 0 = 45° (in Mohr circle 0 = 90°) Sol:
<J'J = <J2 = 0
"Cxx = "Cyy = 0
"Cxy=p 8 + 0-y 8 - o-y
� 2 = -- +-- cos(2 x 45)
2 2
11. Ans: 50 -3 x sin(2 x 45)
Sol: The state of stress is pure shear
= 2 = 4 +- +0-3 xl
Hence, cr1 --a2 = "C
(j y

2
:. crmax = 50MPa

!IH1ii@jjji4jjjj4Aflliiidfjj.. ydmbad I Delhi I Bhopal I Pune I Bhubaneswar I Lucknow I Patna I Bengaluru I Chennai I Vtiayawada I V,zag I T1111pari I Kukatpal)y I Kolkata
•t -- .
'!.� I, =
,jj :....�·���-�-�-·��·�========��===========�����;:�
:·.1·�·�-�A:CA.,
.na, ,.. ·. 63 ·. Complex Stresses

Principal stresses 06. Ans: (c)


- )
Sol: r max = a, 0"2 = - l OO - (- l O = - 45MPa
2 2

'
= 8 � 2 ± l; 2 ) + 3 '
:. O"y , = - 45 x 2 = ± 90MPa
= 5 ± 4.243
cr 1 = 9.243 MPa
07. Ans: (b)
and cr2 = 0.757 MPa
Sol: O" = 50 MPa
1 0x l 0 6
03 . Ans: (d) -r = -r =- = 50.95 MPa
max
Sol:
Zp
� (1 00 ) 3
16

08. Ans: (b)


For an element subjects to pure shear stress, Sol: For hydrostatic pressure normal stress in all
principal stresses are ' ± r 'acting along a directions is the same. The shear stress on
diagonal plane. any plane is zero. Therefore radius of Mohr
The maximum principal stress = 't circle is zero.

04. Ans: (b) 09. Ans: (b)


Sol: From Mohr's circle the principal stresses are Sol: Radius of Mohr circle,
0"1 , 0"2 = 1 75 MPa
rMohr circle
1 75 MPa = t00; 40 r + 40 '

05. Ans: (d) = .J30 2 +40 2 = 10.J9 +16 = 50 MPa


Sol: All the planes are principal planes without
shear

Fydcrabadi Delliil Bhopal I Pune I Bhubaneswarj Lucknow! Pama! Bengalurul Chennai !Vuayawada!Vizag I T,rupati I Kukatpal)yi Kolkata I
e0=53. 1 3 = 0.003079 = �L = L 1 -Lo = L1 -5
10. Ans: (c)
Sol: crx = 20 , cry = 80, 'txy = 40 L0 L0 5
L 1 = 5 x 0.003079 + 5 = 5.015 units
• mu =±
(er,;"')' + {<_, f
= ± (20 ; 80 J +(40 ) '

= ± 50 MPa

11. Ans: (1 MPa )


cr ( 100 / 50 )
Sol: =- = -'-------'-- = lMPa
2 2
'tmax

12. Ans: 5. 015


Sol:
S(0,3) R(4,3)
,--------,.

P(O,O) Q(4,0)

Exx = 0.001, Eyy = 0.002, Yxy = 0.003


tan0 = 4/3, 0 = 53.13
Normal strain along the oblique plane with
inclination 0 is

e0 = (
E
xx
+ E YY
2
0.001 +0.002
J
E -E
2
y

0.001-0.002
J
+ ( xx YY cos 20 + ;Y sin 20

e 0=53 . 1 3 = ( )+( )
2 2
0.003
cos(2 x 53.13 ) + sin(2 x 53.13)
2
= 1.5 X 10·3 + 1.399 X 104 + 1.44 X 10·3
= 0.003079
\( I l 1u
.;111t
. l J l llg P11!,!it,1tH,11.., yderabad ! Dellii l Bhopa) !Punej Bhubaneswar l Lucknow l Patna l Bengalwu j Chennai l Vtjayawadaj V17.3g I Tirupati I Kukatpa))y i Kolkata
SFD and BMD
03 . A concentrated load P acts on a simply
L
One Mark Questions supported beam of span L at a distance
3
01. For a simply supporting beam on two end from the left support. The bending moment
supports the Bending Moment is maximum: at the point of application of the load is
(GATE-M E-89) given by (GATE-M E-93)
(a) usually on the supports PL 2PL
(a) (b)
(b) always at mid span 3 3
(c) where there is no shear force PL 2PL
(c) (d)
(d) where the deflection is maximum 9 9

04. A cantilever beam carries the anti­


02. A block of steel is loaded by a tangential symmetric load shown, where w is the peak
force on its top surface while the bottom
O

intensity of the distributed load.


surface is held rigidly. The deformation of Qualitatively, the correct bending moment
the block is due to (GATE-M E-92) diagram for this beam is
F (GATE-M E-OS)

(a) shear only


(b) bending only
()
a P4 (b) �

(c) shear and bending


d
( ) torsion (c)
� (d)
0

!MINM@hiiiih/4RftftiM\hih+Ydcrabad l Delhi l Bhopal


l Pune ! Bhubaneswar l Lucknow l Patna ! Bengaiuru ! Chennai l V\iayawada ! Vmg I T1IUpati I Kukatpally l Kolkala I
05. A simply supported beam PQ is loaded by a clearly labeling the important values. Also
moment of 1 kN-m at the mid-span of the indicate your sign convention.
beam as shown in the figure. The reaction (GATE-ME-96)
forces Rp and RQ at supports P and Q lOOkN I OOkN
respectively are (GATE-ME-11)
B
1 kN-m 0
p Q Re
l .5m 2m Im

(a) l kN downward, 1 kN upward 02. A mass of 35 kg is suspended from a


(b) 0.5kN upward, 0.5 kN downward weightless bar AB which is supported by a
(c) 0.5kN downward, 0.5 kN upward cable CB and a pin at A as shown in Fig.
( d) 1 kN downward, 1 kN downward below. The pin reactions at A on the bar AB
are (GATE-ME-97)
06. For a loaded cantilever beam of uniform
cross-section, the bending moment (in N­
mm) along the length is M(x) = 5x2+ 10x,
where x is the distance (in mm) measured
from the free end of the beam. The B
magnitude of shear force (in N) in the cross­ 275mm
section at x = 1 0mm is---
(GATE - 17 - SET- 2) YLx
(a) Rx = 343.4 N (b) Rx = 343.4 N
Two Marks Questions I Ry = 755.4 N Ry = O

0 1 . Two bars AB and BC are connected by a (c ) Rx = 755.4 N (d ) Rx = 755.4 N


frictionless hinge at B. The assembly is Ry = 343.4 N Ry = O
supported and loaded as shown in Fig. Draw
the shear force and bending moment
diagrams for the combined beam AC,

!lflli!AhiiiihiiAfll!MiHiii+yderabad l DclhilBhopallPunc lBhubancswarl LucknowlPatnalBenplurulOiennailVuayawadalVizag JTuupali I Kukaq,allyj Kolbla I


: 67 : SFD & BMD

03. A beam is made up of two identical bars AB 06. For the overhanging beam shown in figure,
and BC, by hinging them together at B. The the magnitude of maximum bending
end A is built-in (cantilevered) and the end moment (inkN-m) is __
C is simply-supported. With the load P 1 0 k:N/m
acting as shown, the bending moment at A is
(GATE-ME-05) A

t 1111 L
B
r•1
1111111
4
L/2
lp
•I
L

1111
4m 2m
(GATE-15-Set 3)

07. A cantilever beam OP is connected to


(a) Zero (b) PL/2
another beam PQ with a pin joint as shown
(c) 3PL/2 (d) Indeterminate
in figure. A load of 10 kN is applied at the
mid-point of PQ. The magnitude of bending
04. In a simply-supported beam loaded as
movement (inkN-m) at fixed end O is
shown below, the maximum bending
(GATE-1 5-Set 2)
moment in N-m is (GATE-ME-07)

\'�
l O k:N
i Q
1.soornm • : • 500mm .1
9
�---�
1100N •I-
'------. 2m lm •I
-: l+--
lOOrnm (a) 2.5 (b) 5 (c) 10 (d) 25

(a) 25 (b) 30 (c) 35 (d) 60 08. A beam is loaded as shown in the figure

I• 1 m
l O k:N
05. A simply supported beam of length L is �
subjected to a varying distributed load
sin(3mJL)Nm- 1 , where the distance x is 6 k:N
p 0
measured from the left support. The
magnitude of the vertical reaction force in N
at the left support is (GATE-ME-13)
2m 2m 2m

(a) zero (b) L/3n The bending moment, in Nm, at point R is


(c) Lin (d) 2L/n (GATE - PI-15)

\( I J I H.:, 1 1 \l t I till.!, P11lil!{ .111011 .... ydcrabad I Dellii I Bhopal I Punc I Bhubaneswar I Lucknow I Patna I Bcngaluru I Chennai I Vijayawada I Vizag I Tirupati I Kukatpally I Kolka!a
SOLUTIONS
l k:N-m
One Mark Solutions

01. Ans: (c)


Sol: Bending moment is maximum by magnitude Ro
where shear force becomes zero. Rp = 1 k:N (-1-) Ro = 1 k:N (t)

02. Ans: (c) 06. Ans: 110


Sol: The block behaves as a cantilever. It will be Sol: M(x) = 5x2+ lOx
subjected to shear force and bending Shear force
moment. dM
F= = l O x + 10 = 10 x 10 + 10 = 110 N
dx
03. Ans: (d)

i
Sol: Two Marks Solutions

t.
2P !_

p
01.
- 3 Sol:
3 3 3
. 2P I 2Pf
Bendmg moment = -x - = -
3 3 9
l OOkN
t
I OOkN
04. Ans: (d)
Sol: Upto centre of beam there will be steep �I OOkN
increase in BM, later on there is a gradual
increase in BM.
:. The appropriate answer is 'D'. 1 50kN-m

05. Ans: (a)


M lk:N -m
Sol: R P = R Q = = lk:N B MD
1 Im

�ydcrabad
I Delhi I Bhopal I Punc I Bhubaneswar I Lucknow I Patna I Bengaluru I Oicnnai I Vtjayawada I Vizag I 1irupati I Kukatpally I Kolkata I
: 69: SFD & BMD

02. Ans: (d) 04. Ans: (b)


Sol: T Sol:
l OO N
A

35 x 9.81 N 0.5m

e = tan - 125 =
1
) 24.44°
275 100 10 =
(

RA - 40 kN
2 1
=
rFX = O
Rx =T cos 24.44 ---- (1) 100 10 =
RB = + 60 kN
2 1
LFY = 0
: . M c = Max BM = R 8 (0.5m) = 30 kN-m
T sin 24.44 + Ry -(35 x 9.81) = 0 ---- (2)
rM A = O
05. Ans: (b)
T sin0(275) = (35 x 9.81)(275) Sol: The loaded beam is as shown in figure
35x9.81 =
T= 830 N
sin 24.44
SubstituteT in equation (1) T� 116 116 �
//3
;

113 T
Rx= 755.4N. RA � Rs
SubstituteT in equation (2)
Ry = O RA = Ra = hatched area
rl / 6
= sin
L dx
03. Ans: (b) Jo
(37tX)
Sol:
FBD

p �p
2 p 2
A__ /_�i2
a B

= L
3 1t

yderabadl Delhi l Bhopal I Punc l Bhubaneswarl Lucknow l Patna l Bengalurul Olennai I Vuayawada j V17.3g ! Tirupali I Kukatpally i Kolkata
06. Ans: 40
Sol: RQ = Rp + 10
20 kN Rp x 1 + 10 x 0.5 =0
lOkN
Rp = -5 (-Ve indicates, Rp for PQ link is
lm 2m
upward)
30 kN
:. Mo = 5x2 = 10 kN-m

(SFD)

08. Ans: 14000


Sol:
lO kN 6sin30
j
p R 6cos30 s
(BMD)
2m 2m 2m

Take moments about 'A'


RBx4 - 20x6-1Ox4x2 = 0 Rp + Rs = 10 + 6 sin 30 = 13
=> RB = 50 kN
LMp = O
Since RA + RB = 60
=>Rs x 6- 3x4-10x2 - 10 = 0
=> RA = 60 - 50 = 10 kN
=>Rs = 7 kN, RP = 6 kN
Since Shear force at 'B' is zero
Hence maximum bending moment occurs at Bending moment at R = Rs x 2
= 7x2=14 kN-m
'B' that is BMmax = 20 x 2 = 40 kN-m
=14000 N-m
07. Ans: (c)
Sol:

l1t1•i!Mhi4410/4Rdfi1Mijijjj+ydcrabad I Delhi I Bhopal I Pune I Bhubancswar I Lucknow I Patna I Benga)uru I Chennai I Vuayawada j V17.3g I Tirupati I Kukatpally I Kolkata I
Centroids and Moment of
Inertia

One Mark Questions Two Marks Questions

0 1 . The second moment of a circular area about 0 1 . A disc of radius' r 'has a hole of radius 'r/2'
the diameter is given by (D is the diameter) cut-out as shown. The centroid of the
(GATE-ME-03 ) remaining disc (shaded portion) at a radial
4 4 4 4
trD trD trD trD distance from the centre "O" is
(a) (b) (c) (d)
4 16 32 64

02. For the same material and the mass, which


of the following configurations of flywheel
will have maximum mass moment of inertia
about the axis of rotation 00' passing
r r
through the center of gravity. (a) -
2
(c) -
6
(d) .c8
(GATE-15-Set 3)

02. The value of moment of inertia of the


(a) Solid Cylinder (b) Rimmed Wheel
section shown in the figure about the axis­
O' XX is
0
. . . . . . . . .
0 O'

0
..... ....
-- x
(c) Solid Sphere (d) Solid Cube 1 20

I
O' : ) : ) [ : ::[)i [ :)::i : : : [ \ 3=0,: (( : \(::\)\ :: : : /j)
0 - ;.:..�--�-�- �-�--�-� :..·-···
I

60
..···

!IJ•li!i§OiiiijjjiAflbiM\!i.jj� yderabad I Delhi I Bhopal I Pune I Bhubaneswar I Lucknow I Patna! Bengaluru I Qiennai IVtiayawadaJ Vizag ITirupati I Kukalpally I Kolkata
: 72: Strength of Material

All dim en sion s are in ' m m ' 03. The figure shows cross-section of a beam
(GATE -15-Set 3) subjected to bending. The area moment of
(a) 8.5050x106 mm4 inertia (in mm4) of this cross-section about
(b) 6.8850x106 mm4 its base is ----
(c) 7.7625 x106 mm4
(d) 8.5725x106 mm4
Is R4 All dimensions are in mm

(GATE -16-SET-1)

!Iiili!i§hiiiihiMRflnjjjjjjjj� I I I I I I I I I
ydcrabad Delhi Bhopal Pune Bhubaneswar wcknow Patna Bengaluru Chennai j Vtjayawada j Vizag Tuupati I Kukatpally I Kolkata
SOLUTIONS
02. Ans: (b)
Sol:
One Mark Solutions

01. Ans: (d)


{J x
-+-------,.-+- l Omrn

Sol: / = / = .!!_ D 4
X y 64
Bh 3 bh 3
Moment of inertia = --- 2 Ax +-J
(
2
02 . Ans: (b) 12 12
Sol: Radius of gyration is highest for a rim type Moment of inertia
flywheel, hence it has maximum mass 60 X 1203 _ 30 X 303
= 2( +(30 X 30 X 30 2 ]
moment of inertia. 12 12
= 6.885x106 mm4

Two Marks Solutions 03. Ans: 1875.63 (range 1873 to 1879)


Sol:

01. Ans: (c)


Sol: Assuming reference y-axis passing through 8 mmI R4
origin of big circle
- A1 X 1 - A2 X 2
---'-------'----=-----=-
X =
A 1 - A2
10 x103
I= +10x10(5)2
1r r 2 (o )-1r(r/2) 2 (r/2) = R 12

-[
= 2
(-)
1r r - 1r(r/2 ) 2
6
: X ( 84 ) +: X ( 82 ) ( 5 ) J
2

Note: Negative sign indicates centroid lies 4


left of origin. = 1 875.63 mm4

ili11i@jjjQ4ijji@fti@j111n� yderabad I Dellii I Bhopal I Pune I Bhubaneswari Lucknow ! Patna I Bengalwu I Chennai I Vtjayawada jVizag I T,rupati I Kukatpally I Kolkata
cs Pure Bending

I One Mark Qnestions I 03. Consider a beam with circular cross-section


of diameter d. The ratio of the second
moment of area about the neutral axis to the
O1. The beams, one having square cross section section modulus of the area is
and another circular cross-section, are (GAT E-17-SET - l)
subjected to the same amount of bending d nd
moment. If the cross sectional area as well (a) - (b) (c) d (d) 1td
2 2
as the material of both the beams are the
same then (GATE-ME-03)
(a) Maximum bending stress developed m Two Marks Questions
both the beams is the same
(b) The circular beam experience more O 1. A 6 metre long supported wooden beam of
bending stress than the square one rectangular section 10 cm x 20 cm deep is
(c) The square beam experience more strengthened by mild steel plates 0.5 cm x
bending stress than the circular one
10 cm wide at the top and bottom fibre over
(d) As the material is same both beams will the entire length. Find the minimum
experience same deformation
supportable uniformly distributed load
considering failures in steel and wood due to
02. The cross-sections of two solid bars made of
flexure. Weakening of wood due to screws
the same material are shown in the figure.
and weakening of the steel plates due to
The square cross-section has flexural
drilled holes may be ignored.
(bending) rigidity I 1 while the circular cross­
(GATE-ME-87)
section has flexural rigidity Ii. Both sections
I� 100 mm �I r- 5mm
have the same cross-sectional area. The ratio

D O
Ii/h is

(GATE -16-SET-3)
(a) lht (b) 2/rc (c) rc/3 (d) rc/6 th..__ 5 mm

Fydenbad j Delhi j Bhopal j Pune j Bhubaneswar l Lucknow j Patna j Bengalwu j Chennai l Vuayawada l V,zag I Trrupati I Kukatpal)yl Kolkatal
: 75 : Pure Bending

Permissible tensile stress for steel


fO N
= 156.8 N/mm2
Permissible tensile stress for wood III Omm

= 14.89 N/mm2
Young's modulus of mild steel
Im
• Im
•I
l+.i
1 0mm

= 1 .96 x 105 N/mm2


Young's modulus of wood
= 0. 1 1 7 x 105 N/mm2
(a) 60MPa p
02. A tapered cantilever beam of constant (b) 60MPa 17
thickness is loaded as shown in the sketch
below. The bending stress will be:
(c) 400MPa

(GATE-ME-88)
( ) 400MPa
d
p
17
Statement for Linked Answer Q 04 & QOS
A mass less beam has a loading pattern as shown
in figure. The beam is of rectangular cross section
(a) Maximum near the fixed end with a width of 30 mm and height of 100 mm.
(GATE-ME-OS)
(b) Maximum at x = _!_ L
2 3kN/m
(c) Maximum at x = 3. L
3
2m
(d) Uniform throughout the length •�Re

03. A cantilever beam has the square cross


04. The maximum bending moment occurs at
section of 10 mm 10 mm. It carries a
(a) Location B
x

transverse load of 10 N. Considering only


(b) 2675 mm to the right of A
the bottom fibres of the beam, the correct
(c) 2500 mm to the right of A
representation of the longitudinal variation
(d) 3225 mm to the right of A
of the bending stress is (GATE-ME-OS)
!lfl1ih§jjji4ijjj4AflftiM\j/fjj+yderabad I Delhi I Bhopal I Pune I Bhubaneswar! wcknow! Patna I Benga)uru I Chennai I Vtiayawada I Vizag I Tuupari I Kukatpally I Kolkala I
...
f.' -...... .A!
• CE •
'!...
W
��
V •
Publirariooe

: 76: Strength of Materials
� =========================================
05. The maximum magnitude of bending stress F ( 3L-b ) F ( 3L +b )
(a) (b)
(in MPa) is given by 4b3 4b3
(a) 60.0 (b) 67.5 F ( 3L - 4b ) F ( 3L -2b )
(c) (d)
(c) 200.0 (d) 225.0 4b 3 4b 3

Statement for Linked Answer Q06 & Q07


09. A simply-supported beam of length 3L is
A simply supported beam of span length 6m and
subjected to the loading shown in the figure.
75 mm diameter carries a udl of 1 .5 kN/m
(GATE-ME-06) p
L
p

06. What is the maximum value of bending


moment?
(a) 9 kN-m (b) 6.75 kN-m It is given that P = 1 N , L = 1 m and
(c) 81 kN-m (d) 125 kN-m Young's modulus E = 200 GPa. The cross­
section is a square with dimension 100 mm.
07. What is the maximum value of bending
The bending stress (in Pa) at the point A
stress?
located at the top surface of the beam at a
(a) 162.98 MPa (b) 325.95 MPa
distance of l .5L from the left end is
(c) 625.95 MPa (d) 651.90 MPa
(Indicate compressive stress by a negative
sign and tensile stress by a positive sign).
08. For the component loaded with a force F as
(GATE-16-SET-1)
shown in the figure, the axial stress at the
comer point P is (GATE-ME-08)
10. A 1 m x 1 0 mm x 10 mm cantilever beam is
p subjected to a uniformly distributed load per
unit length of 1 00 N/m as shown in the
figure below. The normal stress (in MPa)
due to bending at point P is _____
j Point P

L
(GATE -PI-1 6)

!M•1ijj§jjjl4.fjg@G1Miii!jj+yderabad I Delhi I Bhopal I Pune I Bhubaneswar I Lucknow I Pama I Benga)uru I Chennai I Vlia,yaw.ida I V,zag I TlfUpati I Kukatpally I Kolkata I
SOLUTIONS
03. Ans: (a)
d
One Mark Solutions Sol: = Ymax = 2
I NA

( J
I NA

01. Ans: (b) Ymax


Sol: Mcircle = Msquare = M

Acircle = Asquare

D
: D = a => a = �
2 2
Two Marks Solutions

My
=-
01.
(J b
I
Es = l .96 x 1 0 5 =
M( ¥ Sol: m= 16.75
) = 3 2 M3 = 1 0. 1 8 (�3 ) E 1 0. 1 17 x l 0 5
((J b tircle
nD D
� D4 Equivalent section in terms of steel is given
64
below
1 00mm

200mm
> (<Jb)square
(<Jb)circle b=5.97mm
Circular beam expenences more
stresses than square beam. 5mm

02. Ans: (c)


T
100 x 210 3
94 x 200 3
Sol: a2 = nd2 / 4 I NA = _
12 12
a = d J;. /2 = 77. 1 75 X 1 06 - 62.66 X 1 06
= 14.5 X 1 06 mm4
7t 2
d4
a;,{
.!i = 1 2 -- -1 92
--- If permissible stress in steel = 1 56. 89 MPa
1td¼ 1td 4
64 The stress in timber is
12

64

\( I I r1i._:1111 t 1111� P1 ild1( ,ttll111" �yderabad j Delhi j Bhopal j Punc j Bhubancswarj l.ucknowj Patna j Bcngaluru j Clicnnai j Vtjayawada j Vizag j TlfUpati I Kukatpally j Kolkata I
: 78 : Strength of Materials

105mm � 156.8 MPa 02. Ans: (d)

l OO mm � ? =
156.8x100 (J_) Sol:
105 m
=8.915 MPa < 14.89 MPa
6PL
If permissible stress in wood = 14.89 MPa fd 2
The stress in steel is
100 � 14.89
105 x14.89
105 � ? = x m = 261.87 MPa For beam of uniform strength
100
M = f. z
Therefore, permissible stress in steel should
bd 2
be limited to = 156.8 MPa PL = f.

6PL
b=
From bending equation fd 2
M f
- =-
I y 03. Ans: (a)
Sol: ION
1
M = f. z = 156.8 x --
Ymax i lm

14.5 x10 6
= 156.8 X
105
BMD
M = 21.65 kN-m
M =l OkN - m
The safe udl on the beam 'w' is
w/ 2
- Bending stress at the point of maximum
=M
bending moment,
8M - -- 65 x 1 0 6 f M
8x 2 1 .- = 10 x l 0 = 6 0 M Pa
3
w - - -- =
/2 ( 6000)
2
Z ! O x 10 2
( J
= 6
4.81 N/mm or 4.81 kN/m
As f oc M , bending stress variation along
the length of beam is same as BMD

!lfllii@Oii4iOi4RflGiM\ii!h*)Hyc1erabad j Delhi l Bhopal j Pune l Bhubaneswari Lucknowj Parnaj Bengaluru l Cbennai l Vuayawada l Vi7.ag jT11Upali I Kukalpally j Kolkato I
: 79 : Pure Bending

04. Ans: (c) 05. Ans: (b)


Sol: Sol: Max BM occurs at the point of zero SF
3kN/m
;· BM max • M = R B (1. 5 )-3 (1. 5{1; )
5

2m = 3.375 kN-m
M 3.375x 10 6
l .5 kN ----- fmax =- = 67. 5 MPa
z 30x;oo' J
=

2m 06. Ans: (b)


Sol: 1. 5 kN/m
/
4. 5 kN
LFy = 0
6m �
RA + RB = 3 (2)
Maximum bending moment
LM A = O w, 2 (1. 5 X6)
2
6.75 kN
8 8
=
R B (4)- 3(2)(3) 0
=

4. 5 kN.
07. Ans: (a)
RB =

and RA =
l .5 kN M 6·75 x 10 6
Sol: f max = z = = 162.98MPa

�( 75 )3
From similar triangles 32
1 .5 4. 5
= 08. Ans: (d)
(2 - x) X
Sol: Point P is subjected to axial force and
x = 6 -3x
p M
6 bending. (J'R = CJ' + j = - + -
X = - = 1 .5 A Z
4
F
= --- + -�-�
F(L - b)
1 . 5 m from end B. (2b X2b) (2b 2b ) '
x =
[ � ]
or 2.5 = 2500 mm from end A
. _ F(3L-2b)
• • CT R - 4b 3

�yderabad l Delhi I Bhopal I Pwie I Bhubaneswarl Lucknow I Patnal Bengalwu I Chennail Vtjayawada l Vizag IT=pati I K.ukatpallyl Kolkara I
ACE :.
.. " = :·�
�...E.l!l :nog
F.Dlinm
�· Pn�Ni�on • ========.::== 0 ========�S:tr�e:
�:011:, �::::o:::::: ri:als:
f ate�
8��:�
ngth M

09. Ans: 0 (range -1 to 1) 10. Ans: (300)


Sol:
Sol: M =I 00x l x (�) = 50N-m
p p

M 50x l� .
A
cr = - = ( = 300 MPa (Tension)
L L L Z I0x l0 J
2

rMp = 0 => -� (3L) + P (2L) -P (L) = 0


�(3L) = P(L)
=
p
� +3
MA = �(1.5L) -P(0.5L)
p 3 p
MA = - (- L)-- L
3 2 2
MA =
0 => (jA = O [·: a oc M]

!IHli@ihiiiihiRbGtM\iiiii+yderabad I Delhi I Bhopal I Pwie I Bhubaneswar I Lucknow I Patna I Benga)uru I Chcnnai I Vuayawada I Vmg I Tuupari I Kukatpally I Kolkata I
Shear Stress in Beams
One Mark Questions
Two Mark Questions

01 Which one ofthe following diagrams shows 01. Consider a simply supported beam of length
correctly the distribution of transverse shear 50h, with a rectangular cross-section of
stress across the depth h of a rectangular depth h, and width 2h. The beam carries a
beam subjected to varying bending moment vertical point load P, at its mid-point. The
along its length? ratio of the maximum shear stress to the
(GATE-ME-90) maximum bending stress in the beam is
(GATE-ME-14-SET-3)
(a) (b)
(a) 0.02 (b) 0.10
(c) 0.05 (d) 0.01

02. A hollow shaft (do = 2di where do and di are


the outer and inner diameters respectively)
needs to transmit 20 k W power at 3000
RPM. If the maximum permissible shear
(c)
stress is 30MPa, do is
(GATE-15-Set 2)
(a) 11.29 mm (b) 22.58 mm
(c) 33.87 mm (d) 45.16 mm

�ydcrabad I Delhi I Bhopal I Pune I Bhubaneswarj 1..ucknowl Patna I Bengalwu I Chcnnai I Vuayawada j Vizag I Tirupati I Kukatpal)y I Kolkata I
SOLUTIONS
02. Ans: (b)

One Mark Solutions Sol: do= 2di


d.
K = - = 0.5
1

01. Ans: (b) do

Sol: Shear stress distribution in a rectangular 60P = 60 X 20 X 103


T=
section is parabolic with zero at extremes 21tN 21tx 3000
and maximum at neutral axis. = 63.66 x103 N-mm

t = ----16T
1td� (l - k 4 )

16x 63.66x l 03
30 =
Two Mark Solutions
1tx d� x (l - 0.54 )
01. Ans : (a) do = 22.58 mm

So l: t max = � t avg = % [:]


=
¾[(2:Xh)] = ::,
PL
= M = 42
fmax
Z bd
6

= P(50 h )x 62 = 75P2
4x [2h][h ] 2 h
3P
= 4h = 3x 2 = _3_ = 0 . 02
t max 2

fmax 75P 4x 75 150


2h 2

l11•1ih§Oii4@4Rflbj@h@+yc!erabadi Delhi I Bhopal I Pune I Bhubaneswarl Lucknow! Patna! Bengaluru I Chennai I VuayawadalV17.ag IT1n1pati I Kuk.alpallyi Kolbla I
C7 Springs
02. The spring constant of a helical compression
One Mark Questions spring DOES NOT depend on
(GATE- 16-SET- 1)
01. The figure shows arrangements of springs. (a) coil diameter
They have stiff nesses K, and K2 as marked. (b) material strength
Which of the following arrangements offers (c) number of active turns
2 1 2 (d) wire diameter
a stiffness = (GATE-ME-87)
K K

K 1 +2K 2

(a) (b) Two Marks Questions

01. The deflection of a spring with 20 active


turns under a load of 1000 N is 10 mm. the
spring is made into two pieces each of 10
active coils and placed in parallel under the
same load. The deflection of this system is
(GATE-ME-95)
(a) 20mm (b) 10 mm
(c) 5 mm (d) 2.5 mm

(d) 02. A weighing machine consists of a 2 kg pan


(c)
resting on a spring in this condition, with
the pan resting on the spring, the length of
the spring is 200 mm. When a mass of 20 kg
is placed on the pan, the length of the spring
becomes 100 mm. For the spring, the un­
deformed length L and the spring constant
(stiffness) are
K (GATE-ME-OS)

\( I I 11:..,111t t 1 1 11� P11lilu.1111111 ... �yderabad I Delhi I Bhopal I Pune I Bhubancswarj wcknowl Patna I Bengalwu I Chennai I Vyayawadal Vizag I T"uupari I Kukatpallyj Kolkata I

� ..,. "' ·CE
!., , ,.�" �
.� . .
PnNiramna : 84 : Strength of Materials

(a) L = 220 mm, K = 1862 Nim 04. A helical compression spring made of a wire
(b) L = 210 mm, K = 1960 Nim of circular cross-section is subjected to a
(c) L = 200 mm, K = 19860 Nim compressive load. The maximum shear
(d) L = 200 mm, K = 2156 Nim stress induced in the cross-section of the
wire is 24 MPa. For the same compressive
03. A compression spring is made of music wire load, if both the wire diameter and the mean
of 20 mm diameter having a shear strength coil diameter are doubled, the maximum
and shear modulus of 800 MPa and 80 GPa shear stress (in MPa) induced in the cross­
respectively. The mean coil diameter is section of the wire is -----
20 mm, free length is 40 mm and the (GATE-17-SET - 2 )
number of active coils is 10. If the mean coil
diameter is reduced to 10 mm, the stiffness
of the spring is approximately
(GATE-ME-08)
(a) Decreases by 8 times
(b) Decreases by 2 times
(c) Increases by 2 times
(d) Increases by 8 times

111111@mM014RbbiM\iiih*)Hyderabad l Delhi l Bhopa! I Pune 1 Bhubancswar l Lucknow l Pama l Bengaluru l Chennai l Vtjayawada j Vwg I T1n1pati I Knkatpallyl Kolbta I
SOLUTIONS
02. Ans: (b)
Sol: F = K(8 )
One Mark Solutions 1
2x 9.81 = K (L-0.2) ............ (1)
0 1 . Ans: (d)
22 x 9.81 = K (L - 0.1) ........ (2)
Sol:
Solving equations (1) & (2)
We get,
Initial length, L = 0.210m = 210 mm
K = 1962 N I m.

03. Ans: (d)


_l => K2 = d/ = (20 ) 3
Sol: K oc =8
d3 K1 d/ (10) 3

(2 K2 ) (K 1 ) 2 K1 K 2 04. Ans: 6
K = =
e 2 K 2 + K1 K 1 + 2 K2 Sol: Shear stress in helical spring is
8PD
't max = x kw
02. Ans: (b) 1td 3
4C - l 0.615
kw = Wahl's factor = -- +--
4C-4 C
't = 24 MPa
Two Marks Solutions rnax

For same load, D and d are doubled


. . D 2D
01. Ans: (d) So there 1s no change m C = = -
-d 2d
Sol: When a spring is made into two equal pieces
.·. Wahl's factor is same
stiffness doubles. The total effective
8P(2D)
stiffness when the two parts kept parallel to 't 'max = k w x
1t(2d)3
each other is '4K' 8PD
't' max = k w X X I = 'tmax
:.The deflection becomes 7td3 S 4
=- 1 =-
24 =
6 MPa
(10) = 2.5mm 't max
1

4 4

lltl••@h@iijj4@ni@mii!+1ydcrabad I Delhi I Bhopal I Pune I Bhubaneswar I l.ncknowj Patna I Bengaluru I Chennai IVtiayawada I Vizag I Tuupati I Kukatpally I Kolkata I
Ce Torsion
04. Two solid circular shafts of R1 and R2 are
One Mark Questions subjected to same torque. The maximum
shear stresses developed in the two shafts
0 1 . Maximum shear stress developed on the R1
are and If = 2 , then � is
surface of a solid circular shaft under pure R2
T1 T2 .
TI

torsion is 240 MPa. If the shaft diameter is (GATE-14-SET-3)


doubled then the maximum shear stress
developed corresponding to the same torque 05. Consider a stepped shaft subjected to a
will be (GATE-ME-03) twisting moment applied at B as shown in
(a) 1 20 MPa (b) 60 MPa the figure. Assume shear modulus, G = 77
(c) 30 MPa (d) 1 5 MPa GPa. The angle of twist at C (in degrees) is

02. For a circular shaft of diameter of 'd'


subjected to torque T, the maximum value of
the shear stress is (GATE-ME-06) 1 0 mm
_J-..l'----�...- B C
64 T 32 T
(a) (b)
•14

,r d3
----+I
3 500 mm
1rd
16T
(c)
,r d3
(GATE-15-Set 1)
03. A solid circular shaft of diameter d is
subjected to a combined bending moment, 06. A rope brake dynamometer attached to the
M and torque, T. The material property to be crank shaft of an I.C. engine measures a
used for designing the shaft using the brake power of 1 0 kW when the speed of
16 rotation of the shaft is 400 rad/s The shaft
relation 3 �M + T is (GATE-ME-09)
2 2
,rd torque (in N-m) sensed by the dynamometer
(a) Ultimate tensile strength (Su) is ----
(b) Tensile yield strength (Sy) (GATE-15 -Set 2)
(c) Torsional yield strength (Ssy)
(d) Endurance strength (Se)

!M1•ih§jjji41hijRflni1\hijj� yderabad I Dellii I Bhopal I Pune I Bhubaneswar I Lucknow I Patna I Bengaluru I Chennai I Vijayawada ! Vmg I Tuupari I Kukatpally l Kolkata
: 87 : Torsion

07. The cross sections of two hollow bars made


:-
�--- 3d -----
of the same material are concentric circles as F
'' ''
shown in the figure. It is given that r3 > r 1 I i
and r4 > r2, and that the areas of the cross­
I I I
: -------- d - -- : -<(+-- -- -
sections are the same. ' I I
I
I

J 1 and Ji are the torsional rigidities of the


,.,,...c....,'-------1.---...,...• ,'
I

bars on the left and right, respectively. The


ratio Ji/J1 is 02. The compound shaft shown is built-in at the
two ends. It is subjected to a twisting
momentT at the middle. What is the ratio of
the reaction torquesT 1 andT2 at the ends?
(GATE-ME-93)

T1
(GATE-16-SET-1) � tT
(
(a) >1 (b) < 0.5 d 2d
(c)= 1 (d) between 0.5 and 1
t t
08. A motor driving a solid circular steel shaft �l z---1
transmits 40 kW of power at 500 rpm. If the
1 1
diameter of the shaft lS 40 mm, the (a) - (b) ! (c) - (d) !
16 8 4 2
maximum shear stress in the shaft is
----- MPa. 03 . Two shafts A and B are made of the same
(GATE-17-SET-1) material. The diameter of shaft B is twice
that of shaft A. the ratio of power which can
Two Marks Questions be transmitted by shaft A to that of shaft B is
(GATE-ME-94)
1 1
01. A circular rod of diameter d and length 3d is (a) - (c) ! (d) -
2 8 16
subjected to a compressive force F acting at
the top point as shown below. Calculate the 04. A torque of 10 N-m is transmitted through a
stress at the bottom-most support point A. stepped shaft as shown in figure. The
(GATE-ME-93) torsional stiffnesses of individual sections of

jlHIM!@Oii4,jji4Rflni1jjjijj+Jiyderabad I Delhi I Bhopal I Pune I Bhubaneswarl Lucknow I Patna I Bengaluru I Oiennai I Vgayawadal Vwg I Tirupali I Kukatpally I Kolkata I
"'-�CE . .
. . S trength of Materials
.. .. ===============================
'!.
"
.•.��Pnbliratioos : 88 :

lengths MN, NO and OP are 20 N-m/rad, 30 (a) Tc = TA (b) Tc = STA


N-m/rad and 60 N-m/rad respectively. The (c) Tc = 16TA (d) TA = 16Tc
angular deflection between the ends M and
P of the shaft is (GATE-ME-04) 07. A solid shaft of diameter, d and length, L is
fixed at both the ends. A torque, TO is

I I I
0 p
T= l ONm M N applied at a distance, L/4 from the left end
GI
T
� as shown in the figure given below.
(GATE-M E-09)
(a) 0.5 rad (b) 1.0 rad
(c) 5.0 rad (d) 10.0 rad

05. A solid circular shaft of 60 mm diameter 3L/4

transmits a torque of 1600 N-m. The value The maximum shear stress in the shaft is
of maximum shear developed is 16T,, 12T,, 8T,, 4T,,
(a) (b) (c) (d)
(GATE-ME-04) tr d3 tr d3 tr d3 tr d3
(a) 37.72 MPa (b) 47.72 MPa
(c) 57.72 MPa (d) 67.72 MPa Statement for Linked Answer Q.08 & Q.09
A machine frame shown in the figure below is
06. The two shafts AB and BC, of equal length subjected to a horizontal force of 600 N parallel
and diameters d and 2d, are made of the same to z-direction. (GATE-ME-07)
material. They are joined at B through a shaft
coupling, while the ends A and C are built-in
(cantilevered). A twisting moment T is
applied to the coupling. If TA and Tc
I
represent the twisting moments at the ends A
and C, respectively, then (GATE-M E-05)
600N
coupling
� � �

§ d
1'
D 2
d
1' �
1.-
L .., I.. L
•I
A B C

lltliii@h/Pi@i@b!Mh!h� yderabad I Delhi I Bhopal I Pune I Bh,;baneswar I Lucknow I Patna I Benga]uru I Chennai I Vijayawada j Vmg I Tirupati I Kukatpally I Kolkara
: 89: Torsion

08. The normal and shear stresses in MPa at minimum allowable design diameter in mm
point P are respectively lS (GATE-ME-12)
(a) 67.9 and 56.6 (b) 56.6 and 67.9 (a) 8 (b) 16 (c) 24 (d) 32
(c) 67.9 and 0.0 (d) 0.0 and 56.6
12. A hollow shaft of 1 m length is designed to
09. The maximum principal stress in MPa and transmit a power of 30 kW at 700 rpm. The
the orientation of the corresponding maximum permissible angle of twist in the
principal plane in degrees are respectively shaft is 1°. The inner diameter of the shaft is
(a)-32.0 and-29.52 (b) 100.0 and 60.48 0.7 times the outer diameter. The modulus
(c) -32.0 and 60.48 (d) 100.0 and -29.52 of rigidity is 80 GPa. The outside diameter
(in mm) of the shaft is __
10. A torque T is applied at the free end of a (GATE -15 -Set 2)
stepped rod of circular cross-sections as
shown in the figure. The shear modulus of 13. A rigid horizontal rod of length 2L is fixed
the material of the rod is G. The expression to a circular cylinder of radius R as shown in
for diameter to produce an angular twist 0 at the figure. Vertical forces of magnitude P
the free end is (GATE-ME-11) are applied at the two ends as shown in the
figure. The shear modulus for the cylinder is
�---- L ------- /2
G and the Young's modulus is E.
2d _ _ _q

32TL 4 18TL 4
(a) ( ) (b) ( )
n0G n0G

16TL 4 2TL 4
(c) ( ) (d) ( )
n0G 1t0G
The vertical deflection at point A is
11. A solid circular shaft needs to be designed to (GATE - 16 - SET - 2)
transmit a torque of SON.m. If the allowable (a) PL3 !(nR4G) (b) PL3 !(nR4 E)
shear stress of the material is 140MPa,
(c) 2PL3 !(nR4E) (d) 4PL3 !(nR4G)
assuming a factor of safety of 2, the

j1Jlli!i§jjji§jjjj$Rbftiilhii!� yderabadlDelhilBhopall Punc !Bhubaneswarl Lucknowl Pa1nalBcngalurul Chcnnai !Vuayawadal V17.3g IT=pari I Kukalpa))yl Kolkala
: 90 : Strength of Materials

14. Two circular shafts made of same material, (a) 0s = 0H and 'ts = 'tH
one solid (S) and one hollow (H), have the (b) 0s > 0H and 'ts > 'tH
same length and polar moment of inertia. (c) 0s < 0H and 'ts < 'tH
Both are subjected to same torque. Here. 0 s (d) 0 s = 0H and 'ts < 'tH
is the twist and 'ts is the maximum shear
stress in the solid shaft, whereas eH is the
twist and 'tH is the maximum shear stress in
the hollow shaft. Which one of the following
TRUE?
(GATE-16-SET - 3)

!1Jlli@hi44ih1Rflbi¥\hih+iyderabad I Delhi ! Bhopal IPune I Bhubancswar l LucknowI Patna I Bengaluru J Chennai J VtjayawadaJ Vu.ag ITirupati I Kukatpally l Kolkata I
SOLUTIONS

One Mark Solutions

01. Ans: (c) 05. Ans: 0.22 to 0.25


1 1 Sol: G = 77 GPa
Sol:
d3
't OC - OC -
zp 0 =?
dI3 e _TL = 10x l 03 x 500x32
=
d23 B - GJ 77x l 03 x nx 20 4

< 2 = , . ( :: J' = 240x ( �) ' = 30 MPa


2
0 8 = 4.13 X 10 -3 radians
0 8 = 0.236 °
As, 0s = 0c
02. Ans: (c)
=- T
T -,-- 16T 06. Ans: 25
Sol: = -,-
Zp 1td 3 Sol: One revolution = 21t radians
tmu

d3 ) ( i:
21tNT
Brake power = -- = roT
60
03. Ans: (c) 10x10 = 400 xT
3

Sol: Material subjected to torsion fails by => T = 25 N-m


torsional shear stress (Ssy)
07. Ans: (a)
04. Ans: 8 Sol: Bar 2 c/s
Sol: Torsional shear stress in a shaft is given by
Bar 1 c/s
16T
t = --
1td 3
1 1
:. t oc d3 or
R3
Where d = diameter = 2R Given r3 > r,,
R = radius of shaft A 1 = Area of cross section of bar-1
!1i11ii@jjjl4.jdAflGi@!IMjHyderabad I Delhi I Bhopal I Pune I Bhubaneswar I Lucknow I Patna I Bengaluru I Chennai I Vuayawada l Vu.ag f Tirupati I Kukatpa]ly I Kolkata I
: 92 : Strength of Materials

A 2 = Area of cross section of bar - 2


A 1 = A2 Two Marks Solutions
=> n{r; - r12 ] = 1r [rJ - r32 ]
01.
Sol: The circular rod is subjected to axial
compression and bending, (sign convention:
compression +, tensile(-))
p M
Resultant cr ---
A z
=

r42 +r:32 R

=
r22 + r.I2 F F x (d / 2)
� (d 3 )
32
4F 16F
1td 2 1td 2
08. Ans: 60. 47 12F
= (-) (Tensile)
Sol: Power transmitted, P = 40kW 1td 2
Rotations, N = 500rpm
Diameter of shaft, d = 40mm 02. Ans: (a)
Power transmission Sol: el =
e2
21tNT
P= TI Li T2 L2
60
G J1 G2 J2
2n x 500 x T
40 = TI L T2 L
60 =
� �
T = 0.76 kN-m G x d4 G x (2d)
4
32 32
Maximum shear stress developed,
T1 = 1 = 1
16T
't max = -- T2 2 4 16
1td 3

03 . Ans: (c)
Sol: Power transmitted, P = 2 nNT
P oc T. -----( 1)
. . T 't
F rom torsion equation - = -
J R

IM•lii@hiii•fhYPfldi@jj.jj*)ttydmbad I Delhi I Bhopal I Pune I Bhubaneswar I Lucknow I Patna I Beagaluru I Chennai I Vuayawada I Vu.ag I T=pari I Kukatpally I Kolkata I
: 93 : Torsion

07. Ans: (b)


T 3L/4) 3T0
T = 't X � d 3 · S0l •• Tmax = )
16 L 4
_ T
'tmax - 3T _ 12To
:.T oc d 3 --- (2) - = --o----,- - --
ZP 4 ( d 3 ) 1td 3
From (1) and (2) , p OC d3 1:

08. Ans: (a)


Sol: M p = 600 x 0.3 N.m
Tp = 600 x 0.5 N.m
04. Ans: (b)
T T Mp 600x 0.3 =
Sol: KT = - �0=-
0 KT
cr bending =
z = -x 0.03
7t
67.9 MPa
3
32
e = OMN +O NO + Oop Tp = -600x 0.5 =
't p = - --- 56.6 MPa
Z p � x 0.03 3
16

09. Ans: (b)


05. Ans: (a)
Sol: M = 600x300 = 1800000 N- mm
T-=- 't
Sol: Maximum bending stress at base
J R
32 M = 32x180000 = _
cr Y = 67 9 MPa
= 1600x10 = 37.72 MPa 1t d 3 1tx30 3
3

� (60) 3 Torque T ' ' = 600 x 500 = 300000 Nm


16
Maximum torsional shear stress
16T = 16x300000 = _
06. Ans: (c) 't xY = 56 6 MPa

r
1t d 3 1tx303
Sol: For Parallel system , 0 AB = 0 8c
Maximum principal stresses

. = "; + ( "; + ·�
"

= 6:. 9 + ( 6:. 9 r + (56.6)2

= l OO MPa

lMliii@hiiiih!MAdbiMIIIMffydexabadJ Delhil BhopalJPunel Bhubaneswarl Lucknow l Patna l Bengaiuru ! Chennai J VuayawadaJ Vizag J Tirupati I Kukalpa])yl Kolk.ala I
: 94 : Strength of Materials

Principal planes, C = 80x103 N/mm2 , do = ?


crx = 0 MPa, cry = 67.9 MPa P x 60
T = --
2rxy 2nN
tan (2a) = = 2 x 56.6
67.9 = (30 x 10 )x 60 = 409.2x 103 N-mm
3

2 x 7t X 700
2a = -59.04° & 180 - 59.04 = 120.96°
TL
a = -29.52° & 60.48° 0=
CJ
Maximum principal stress will occur at
TL
a = 60.48° J=
c0

10. Ans : (b) 1td� TL


(l-k 4 ) =
Sol: 0 = 01 + 02 32 c0

T (½) _ 409.2 x l 03 x1000 x 32


TL
= -- d 40
-- + -�c......- - n x 80 x l 0 x 0.0174 x 1-0.7
3 4
4
G x _2:_ (2d) G x _2:_ d 4
32 32 do = 44.55 mm
� 32 32
0= [ + ]
nGd 4 16 2 13. Ans: (d)
I Sol: T = P(2L) = 2PL
18TL = 18TL 4
d=v ( ) 32 x 2PL2
TL = (2PL XL ) = --,---,-
1t0G n0G 0=
G J (G ) -2:_ (2R )4 nGR4 (16)
32
11. Ans: (b)
4PL2
0 = -­4
Sol: T = -"
X r permissible X d
3
nGR
16
140 3 4PL3 y�
50 X 103 = � X Xd
y L0
nGR 4
= =

16 2
d = 16mm
14. Ans: (d)
12. Ans: 43 to 45 TL
Sol: 0 5 = (solid)
Sol: / = l m = 1000 mm , P = 30x103 W GJ
TL
0 per = f,° X � = 0.0174 radians 0H = (hollow)
180 GJ

!IJll4jj§jjji4i04Rflbfflj/jjj� yderabad I Delhi I Bhopal I Pune I BhubaneswarJ Lucknow! Patna I Bengaluru I Chcnnai I Vtjayawada I Vizag I Trrupati I Kukalpally I Kolkata
.,..:.�..,,.., . A. CE . .
:�PnNirafioos
..,
� : 95: Torsion

Since material length, polar moment of


inertia and applied torque all are same.
T
ts = - x rs (or ) ts oc rs s
J Since <l < 1
dH
T
'tH =- X rH (or ) 'ttt oc rH :. Ds < Dtt :::::> rtt > rs :::::> ttt > ts
J
:. 1tt = Js
�(D 4 -d 4 )=�D 4
32 H H 32 S
Dtt4 -ds4 = D/

!ltllii@hiiiih4@bi@jjih+1yderabadl Delhi I Bhopal I Pune I Bhubancswarl wcknowl Patna! Bengaluru I Chcnnai!Vliayawada!Vwg JTirupari I Kukatpally! Kolkata I
Slopes and Deflections
03. A cantilever beam having square cross­
One Mark Questions section of side a is subjected to an end load.
If a is increased by 19%, the tip deflection
01. A cantilever type gate hinged at Q is shown decreases approximately by
in the figure. P and R are the centers of (GATE -16-SET-1)
gravity of the cantilever part and the (a) 19% (b) 29%
counterweight respectively. The mass of the (c) 41% (d) 50%
cantilever part is 75 kg. The mass of the
counterweight, for static balance, is
Two Marks Questions
(GATE-ME-08)

01. A simply supported laterally loaded beam

R
was found to deflect more than a specified
value. Which of the following measures will
reduce deflection? (GATE-ME-03)
0.5m 2m (a) Increase the area moment of inertia
(b) Increase the span of the beam
(a) 75 kg (b) 150 kg (c) Select a different material having lesser
(c) 225 kg (d) 300 kg modulus of elasticity
(d) Magnitude of the load to be increased
02. A cantilever beam of length L is subjected to
a moment M at the free end. The moment of
Common Data for Questions 02 & 03:
inertia of the beam cross section about the
neutral axis is I and the young's modulus is A steel beam of breadth 120 mm and height 750
E. The magnitude of the maximum mm is loaded as shown in figure. Assume
deflection is modulus of elasticity as 200 GPa.

prrrrrrEJ
(GATE-ME-12)
ML2 ML2
(GATE-ME-04)
(a) (b) /120 kN/m
2EI EI
2ML2 4ML2
(c) (d) ,.
EI EI 15m

\( I I 11�111t't 1J11� P11lil1t.1l1tnh �ydcrabad I Delhi I Bhopal I Pune I Bhubancswar I Lucknow I Patna I Bengalwu I Cheru,ai I Vuayawada I Vi.zag I T,rupati I Kukatpally I Kolkata I
ACE
':,� ..i-u�• ; nn;;:11�-·........,.;;
= F:;;-; -;:- -.;"';,============================
., ..r �,.
• • : 97 : Slopes and Deflections
...... ,.,, PtJ'LI:.......!-

02. The beam is subjected to a maximum concentrated load P is applied at the free end of
bending moment of the beam. (GATE-ME-11)
(a) 3375 kN-m (b) 4750 kN-m p
(c) 6750 kN-m (d) 8750 kN-m t

----L-----..
03. The value of maximum deflection of the
beam is
(a) 93.75 mm (b) 83.75 mm
(c) 73.75 mm (d) 63.75 mm

04. A frame of two arms of equal length L is


shown in the adjacent figure. The flexural 05. The area moment of inertia of inertia about
rigidity of each arm of the frame is EI. The the neutral axis of a cross-section at a
vertical deflection at the point of application distance x measured from the free end is
ofload P is (GATE-ME-09)
bxt 3 bxt 3
(a) (b)
6L 12L
bxt 3 3
(c)
24L 12
( d) �
L

lp 06. The maximum deflection ofthe beam is


24PL3 12PL3
I· (a) (b)
L �1 Ebt 3 Ebt 3
8PL3 6PL3
PE 3 (c) (d)
(a) (b)
2PL Ebt 3 Ebt 3
3EI 3EJ
PL3 4PL3 07. The flexural rigidity (El) of a cantilever
(c) (d)
El 3EI beam is assumed to be constant over the
length ofthe beam as shown in the figure. If
Statement for Linked Answer QOS & Q06 a load P and bending moment PL/2 are
A triangular-shaped cantilever beam of uniform­ applied at the free end of the beam then the
thickness is shown in the figure. The young's value ofthe slope at the free end is
modulus of the material of the beam is E. A (GATE-14-SET-2)

!Iii• IUHIIJIIIPII 0jnftij ½yderabad I Delhi I Bhopal I Pune I Bhubaneswar I lllcknow I Patna I Bengalwu I Chennai JVtjayawada JVizag I Tll\lpati I Kukatpally I Kolkala I
" "' . AJCE
. . .
'!.�-�:F�PubJi.P..n : 98: Strength of Materials
� 4==================================

10. A frame is subjected to a load Pas shown in


p! the figure. The frame has a constant flexural
l)ru2 rigidity EI. The effect of axial load is
neglected. The deflection at point A due to
��
� L ..1
the applied load Pis
_!_ PL2 PL2
(b)

(a)
2 EI EI
PL2 � PL2
(c)I (d) L
2 EI 2 EI

08. A cantilever beam of length L, with uniform


l..._____,A L --
cross-section and flexural rigidity EI, is p

loaded uniformly by a vertical load w per (GATE-14-SET-4)


unit length. The maximum vertical _!_ PL3 PL 3
(a) (b) I
deflection of the beam is given by 3 EI EI
3
(GATE-14-SET-2) PL3 4 PL3
(c) (d)
wL4 wL4 EI 3 EI
(a) (b)
8EI 16EI
wL4 wL4 11. A cantilever beam with flexural rigidity of
(c) (d)
4EI 24EI 200 N.m2 is loaded as shown in the figure.
The deflection (in mm) at the tip of the
09. A force P is applied at a distance x from the beam is___ (GATE-15-Set 1)
end of the beam as shown in the figure. 500N
What would be the value of x so that the
displacement at 'A' is equal to zero? �
· 50mm
(GATE-14-SET-3)
100mm

(a) 0.5 L 0
I· L
·I
A
(b) 0.25L 12. A cantilever beam with square cross section
(c) 0.33L of 6 mm side is subjected to a load of 2kN
�p B
.,
(d) 0.66L normal to the top surface as shown in figure.
I• L The Young's modulus of elasticity of the

jltlIQ@OiiiihiQPflGi@!llh+1yderabad I Delhi I Bhopal I Pune I Bhubaneswar I Lucknow I Patna I Bengaluru I Chcnnai I VuayawadaI V17.ag I Trrupati I Kukatpally I Kolkata I
:.t
..,,.
� w •
ACE
.,F�PnNir:nml
• • • : 99: Slopes and Deflections

material of the beam 1s 2 10 GPa. The


magnitude of slope (in radian) at Q (20 mm
from the fixed end) is__
t. r r r r r r r r r r r
(GATE-15-Set 2)
2kN ! wL 3wL
(a) (b)
2 8
Q

� wL wL
I• 20mm •I (c) (d)
I• •I
4 8
100mm

13. A simply supported beam of length 2L is 15. The simply-supported beam shown in the
subjected to a moment M at the mid-point Figure is loaded symmetrically using two
x =0 as shown in the figure. The deflection equal point loads P. The radius ofcurvature
in the domain O � x � L is given by of the deflection-curve is 15 m for the
-Mx portion ofthe beam that is subjected to pure
W= ( L-xXx+ c)
12EIL bending. The vertical deflection (in mm) at
Where E is the Young's modulus, I is the point M, equidistant from both the supports

r
area moment of inertia and c is a constant is __ (up to two decimal places).
(to be determined).

pl )} M

I· +
0.25
m
0.75m ·I 0.75m lo.2sl
m
The slope at the center x = 0 is
(GATE- 16- SET-2)
(GATE-PI-17)
(a) ML/(2EI) (b) ML/(3EI)
(c) ML/(6EI) (d) ML/( 12EI)

14. A beam of length L is carrying a uniformly


distributed load w per unit length. The
flexural rigidity of the beam is El. The
reaction at the simple support at the right
end is (GATE-16- SET-3)
!lillii@Oliiihi4Jmft1Mih•i!� yderabad I Delhi I Bhopal I Pune I Bhubaneswarl Lucknow I Patna I Bengaiuru I Chennai IVijayawada jV11.ag IT=pari I Kukatpally I Kolkata
SOLUTIONS

One Mark Solutions Two Marks Solutions

01 . Ans: (d) 01 . Ans: (a)


Sol: "'i,MQ = 0 1
Sol: 8oc-
75(2)-M(0.5)=0
To reduce deflection Moment of Inertia
M = 300Kg
should be increased

02 . Ans: (a)
02 . Ans: (a)
Sol: Cantilever beam subjected to moment
w 2
. . ML 2 Sol: Maximum bending moment= f,
deflectlon 1s -- 8
2EI
1 2 0( 1 5)
2
= =3375kN -m
8
03. Ans: (d)
Sol:
03. Ans: (a)
a

�f---1 ____,! Sol: 8=


5 w/ 4 __5 �1 2 _0 x_ l_ 0_ �
384EI
=
3
�00_ 0_ )� -........
15

1 2 0 X 7503
4

384x 2 00x l 03 x �--�


12
Wf,3
- =93.75mm
8=
1 3EI
1
82
o, -(
_ (a 4 112)
(!.�:·)' 1.1 9
( ) =0.5
4
04. Ans: (b)
Sol:
J
62 = 0.5(61)
82 reduced by 50%

\( l I 111.!,lllt� 11111.!, P11lilH,I\J()Jl", I I I


�yderabadl Dellii Bhopal Pune Bhubaneswarl Lucknow! Patna! Bengaluruj CliennaijVtjayawadajVizag jTtrupari I Kukatpallyj Kolkata I
: 101: Slopes and Deflections

8 1 = deflection at free end due to bending of 2M = xbt 3

__ x X and use I = --
vertical member 2P 1 2/
:.

82 = deflection at free end due to deflection Px .x


I

of horizontal bar as a cantilever 8=!2_


E O bxt3
fdx

=-
PL3 I
8 = 1 2P
2 3EI fxdx
Ebt3 0
ML = (PL)(L) PL2
S= =
3EI 3EI 3EI = 1 2P/ � ' 1 2P .e 3 6P/3
[ ] = =
PL3 Ebt3 2 0 2Eb t Ebt 3 3

8 1 L.0
= = -
3EI
PL3 PL3 2PL3 07 . Ans: (b)
8= 8 1 +8 2 = -+-= --
3EI 3EI 3EI Sol: Slope at free end

= PL2 (PL/2)L
0 +
05. Ans: (b) 2EI EI
Sol: PL2 PL2 PL2
=-+-=-
2EI 2EI EI
b
08. Ans: (a).
Sol: w
x.b
Wl"dth = b =-
L
x
�N\N\N\/\
L -I
- b xt- _ -
3
xbt- 3
I -- - Maximum deflection of the beam at free end
12 1 2L
. wL4
1s = --
8EI
06. Ans: (d)

Sol: U= J M! dx 09. Ans: (c)


0 2EI X
Sol: Free body diagram up to 'A '
1 2Mx p
8=- fM dx
�-L�GA
EI x 2P
Use M x = Px
M= P(L-x)

l11•1i!@jjj4Uljjgpjjbi@jjfjj+yderabad I Delhi I Bhopal I Pune I Bhubancswarl Lucknow I PalDal Bengaluru I Chennai IVtiayawadaJ V17.3g I Tuupari I Kukatpally I Kolkata I
: 102: Strength of Materials

Displacement at A = 0
PL3 =---
M (L) 2
3EI 2EI
1 L
PL =-----
P(L-x) (L)
3 2 =-
2EI 0
-f
(PL) 2 dx
3EI 2EI
p2L2 p 2L 3
L (L-x) dx =
L
=
3 2 2EI I 2EI

2L = 3L-3x U =U1+U2
3x = L p 2L3 p 2L3
:. U = --+--
1 6EI 2EI
x = -L = 0.33L.

Deflection at 'A'
10. Ans: (d) 0 = au= 2PL3 + 2PL3
Sol: :0 C A 8P 6EI 2EI
PL3 PL3
= -+-
L 3EI EI

B ---�� A
L
X
=
PL3
EI 3
[_!_
+
1J=i PL3
3 EI
p

Strain energy of AB 11. Ans: 0.24 to 0.28


U1 = -I- Mx2dx Sol:
L

2EI f

l
0

Since (Mx = P.x)


1 L
50
U1 = -f(Px) 2 dx 100mm
2EI 0

· - :�1 [ x:I <i


25xlo'N�

Strain energy of BC
A•
I somm

B

100mm
�,C

l M 2 dx
L
U1 = --
f
Fig: "BMD" Diagram
2EI 0
Since (M = PL)
IM•iih§hillihAAflGiM\dih� yderabad I Delhi I Bhopal I Pune I Bhubaneswar I Lucknow I Patna I Bengaluru I Chermai IVtjayawada jVmg ITirupari I Kukatpally I Kolkata
: 103 : Slopes and Deflections

Equivalent Conjugate beam


E{:)
� [- 2xI

3
x 80
2
+ � x IO' x JOO']

dy = 3600
= 0.158 Radians
dx EI
C

13. Ans: (c)


Sol: M
Deflection of at tip of actual beam
= Bending moment at 'C' of the

conjugate beam
1 25x10 3 s
2EI p
o = x ox(so+ I ( so ))
c 2 EI 3
25xl 0 3
= _!_x x50x83.33
2 200xl0 6 _ 1 First moment of area
dBA- -- ( J
=0.26 mm EI APB about B
2
12. Ans: 0.15 to 0.16
=-
�i(�x�xLx �) =
:;
Sol: Bending moment (M) =-P.x
d ML
d2 y Slope at 'A' = � = -
M = EI 2 L 6EI
dx
(·: at 'A' x=O, W = 0, so slope = � )
d
d y 2
L
� f-Px = EI dx 2 f
(OR)
-Px i + C = EI dy
2 dx
dy =
At x = R 0
' dx
p£ 2
C=-
M M
- a 6
R1 = R2 = -
4 4
2L 2L
I- - = - = 108 mm4
12 12 On right side from centre
At x = 80 mm M x=-R 1 (L-x)

\CI l 11g111tt1111� P11!ilu.1t1(111-.. yderabadl Delhil Bhopal I Punel Bhubaneswarl Lucknow I Patna! Beoga)uru I Chennai IVtjayawadalViz.ag ITuupari I Kiikatpallyl Kolkala
�-t"� ACE Strength of Materials
.. ..... PuhlinP:n : 104:
w • • • •

Where, 'x' is measured from centre 15. Ans: 18.75


Sol:
p p

From deflection equation, M


EIW"=M X
•C

EIW" =- M (L-x) :o.25 m 0.75 m 0.75 m


2L I
I

M x2
EIW'=-- (Lx--) +c 1
2L 2

M x2 x3 )
EIW=-- ( L--- +c I x+c 2 ----(1)
2L 2 6

:0.25 P
From given deflection in domain O � x � L
W = O@x = O
W = O@x = L Ye-Ya = �/AX)

1 0 75
On substituting in eq. (1) = - (o.75 x 0.25P x · )
EI 2
C2 = 0
0.25 P = M (moment)
L 3
0.75
c 1 L=�( ) = _l (o.75x x M)
2L 3 EI 2
ML M 0 75
C1
=
-
6 Yc -Y8= (0.75x · )
EI 2
1 M
R EI
ML ML R = 15 m (given)
W'Ix=o=-�(o)+ =
2LEI 6EI 6EI - M
1 --
15 EI
14. Ans: (b) O.?S)
Yc -YB=_!_(0. 75 X
wL4 RL3 3wL 15 2
Sol: = � R=
8EI 3EI 8 Ya = 0 [at support, no deflection]
Ye= 18.75 mm
Ye = deflection at c.

!lii•i!@hiiUI04Afi61Mi!ifjj� yderabad I Delhi I Bhopal I Pune I Bhubaneswar I Lucknow I Patna I Bengaluru I Oiennai I Vgayawada jVizag I Tuupati I Kukatpally I Kolkata
Cto Thin Cylinders
04. A gas is stored in a cylindrical tank of inner
One Mark Questions radius 7 m and wall thickness 50 mm. The
gage pressure of the gas is 2 MPa. The
01. A thin walled spherical shell is subjected to maximum shear stress (in MPa) in the wall
an internal pressure. If the radius of the shell 1s (GATE-15-Set 2)
is increased by 1% and the thickness is (a) 35 (b) 70
reduced by 1%, with the internal pressure (c) 140 (d) 280
remaining the same, the percentage change
in the circumferential (hoop) stress is 05. A thin cylindrical pressure vessel with
(GATE-ME-12) closed-ends is subjected to internal pressure.
(a) 0 (b) 1 The ratio of circumferential (hoop) stress to
(c) 1.08 (d) 2.02 the longitudinal stress is
(GATE- 16- SET-2)
02. A long thin walled cylindrical shell, closed (a) 0.25 (b) 0.50
at both the ends, is subjected to an internal (c) 1.0 (d) 2.0
pressure. The ratio of the hoop stress
(circumferential stress) to longitudinal stress 06. The inside and outside radii of a thick­
developed in the shell is walled cylindrical pressure vessel are
(GATE-ME-13) denoted by a and b, respectively. If the
(a) 0.5 (b) 1.0 vessel is subjected to an internal pressure P,
(c) 2.0 (d) 4.0 then the magnitude of the radial stress crrr is
(GATE-PI-17)
03. A cylindrical tank with closed ends is filled (a) zero at r = a and maximum at r = b
with compressed air at a pressure of 500kPa, (b) maximum at r = a and zero at r = b
the inner radius of the tank is 2m, and it has (c) constant over the entire thickness
wall thickness of 10 mm. The magnitude of (d) zero at both r = a and r = b
maximum in-plane shear stress (in MPa)
1s (GATE-15-Set 3)

lrrt•lihYiiii;.m;@niM\iiih� yderabad I Delhi I Bhopal I Pune I Bhubaneswar I Lucknow I Patna I Benga)wu I Chennai I Vijayawada I Viz.ag I Tlfllpati I Kukatpally I Kolkata
: 1 06 : Strength of Materials

(a) ( 1 0, 1 0) MPa (b) (5, 1 0) MPa


Two Marks Questions (c) ( 1 0, 5) MPa (d) (5, 5) MPa

01 . A thin cylinder of 1 00mm internal diameter 03. If the Young's modulus and Poisson 's ratio
and 5mm thickness is subjected to an of the container material are 1 OOGPa and
internal pressure of 1 OMPa and a torque of 0.3, respectively. The axial strain in the
2 000 Nm. Calculate the magnitudes of the cylinder wall at mid height is
principal stresses. (GATE-ME-96) (a) 2 X 10-5 (b) 6 X 10-5
(c) 7 x 1 0-5 (d) 1.2 X 10-5
Statement for Linked Answer Q02 & Q03
04. A thin gas cylinder with an internal radius of
A cylindrical container of radius R=1 m, wall 1 00 mm is subject to an internal pressure of
thickness 1mm is filled with water upto a depth of 1 0 MPa. The maximum permissible working
2m and suspended along with its upper rim. The stress is restricted to 1 00 MPa. The
density of water is 1 000kg/m3 and acceleration minimum cylinder wall thickness (in mm)
due to gravity is 1 0 m/s2 • The self weight of the for safe design must be __
cylinder is negligible. The formula for hoop stress (GATE-14-SET-4)
in a thin walled cylinder can be used at all points
along the height of the cylindrical container. 05. A thin-walled cylindrical pressure vessel of
internal diameter 2 m is designed to
withstand an internal pressure of 500 kPa
(gauge). If the allowable normal stress at
any point within the cylindrical portion of
the vessel is 1 00 MPa, the minimum
thickness of the plate of the vessel (in mm)
(GATE-ME-08) lS---- (GATE - PI-16)

02. The axial and circumferential stress (cra, crc)


experienced by the cylinder wall at mid­
depth ( 1 m as shown) are

\( I l 11g1 1 1,·,·1111g 1'11hlu .111om �yderabad ! Dellii ! Bhopa) i Pune ! Bhubaneswarl Lucknow l Patna l Bengaluru i Chennai l Vuayawadal V17.3g jTuupari I Kukatpal)y i Kolkata I
SOLUTIONS
- cr 2 =
't max = cr 1 25 MN/ m 2 (or) MPa
One Mark Solutions 2

01. Ans: (d) 04. Ans: (c)


Sol: D1 =1.01 D Pr 2 x 7000 =
Sol: cr H = cr I = = 280 MN / m 2
t, =0.99 t t 50
PD _ _ Pr _ 2 x 7000 _
- 140 MN / ill
2
ah = - O'L - 0'2 - -
2t 2t 2 X 50
PD cr3 = 0 MN/m2
ahl = --I
2t cr 1 - cr 3 = 1
l max = 40 MN / m 2 (or) MPa
't
2
P(l.01D )
cr hi = = l .0202( PD )
2 X 0.99t 2t
05. Ans: (d)

�J:�t) 2
% Change in Hoop stress = 2.02%

02. Ans: (c) Sol:


Sol: For a cylindrical shell,
PD
O' (

( :!
Hoop stress (ah ) =
2t
06. Ans: (b)
PD
Longitudinal stress (a1 ) = Sol:
4t

hoop stress--
----- =2
Longitudinal stress

03. Ans: 25
Pr 0.5 x 2000
Sol: cr H = cr 1 = - = ---- = 100 MN/ m 2 maximum at a
t 10 cr rr {
zeroatb
_ _ Pr _
O'L - 0' 2 - -
0.5 x 2000 _
- 50 MN/ ill
2
2t 2x10

!IJll4j@m4lijj/4Aflfi1Mjj@� yderabad I Delhi I Bhopal I Pune I Bhubaneswarj LucknowJ Patna I Benga)wu I Chennai I Vijayawada I Vmg I TIIUpati I Kukatpal)y I Kolkata
: 108 : Strength of Materials

Axia l stress ( cra)


pgh X 1tr 2 pgh r
Two Marks Solutions cra = =
2nrt 2t
1000 x 10 x 2 x l
= = 10 x 106 Pa = l OMPa
0 1. 2 X 0.00 1
PD l Ox l O
Sol: crh = = = l OMPa
2t 2x5 03. Ans: (a)
PD
a1 = -= 50 MPa Sol: s 1 = -1 -
cr cr h
µ. -
4t E E
nd 3 t nd 2 t 1
For ring ofthickness t, J= and Z= = - [a1 - µah ]
4 2 E
2
Torsiona l shear stress, 1= � = -;- [5 - 0.3 10 ] = 2 10-5
10
X X
1td t
= 2 x 2000 x l 03
= 25.46 MPa 04. Ans: 10 mm
7t X 100 X 5
2

Sol: Interna l diameter, D = 2R = 200mm


Interna l pressure, P = 10 MPa
Hoop stress, crh = 100 MPa
- 10o; so ± coo; s o )' + 25.46 PD
' crh = -
2t
(10)(200)
= 75 ± .J25 2 + 25.46 2 = 75 ± 35.68 l =
00
2xt
cr 1= 1 10.68 MPa , cr 2= 39.32 MPa Thickness, t = 10mm

02. Ans: (a) 05. Ans: (5)


Sol: F luid pressure at a depth of l m from free Sol: P = 500 kPa = 0.5 MPa
surface (P) = y h D = 2m, crh = 100 MPa,
P = pgh = 1000 x 10 x 1 = 1000N/m2 PD
crh =-
= 0.01 MPa 2t
PD 0.5 X 2000
cr c (or) cr h = - l O=
2t 2xt
(0.0 1) ( 2000) Thickness, t = 5mm
= = l M
O Pa
2x l
!lfl1ijj§jjji4.jj@QnbiM\hfh� yderabad j Delhi j Bhopal j Pune j Bhubaneswar! Lucknow j Patna j Bengaluru j Chennai !Vuayawada ! Vu.ag ! Tuupati I Kukatpal)y l Kolkata
Lapter 11 Columns and Struts
;r 2 E/
(b) per =
El
(a) Per = 22
One Mark Questions ;r L 3L2
;rEJ ;r 2EI
(c) Per = -2- (d) per =
0 1. If the length of a column is doubled, the L L2
critical load becomes: (GATE-ME-88)
04. For a long slender column of uniform cross
(a) ..!_ ofthe original value
2 section, the ratio ofcritical buckling load for

(b) ..!_ ofthe original value the case with both ends clamped to the case
4 with both ends hinged is
(c) _!_ ofthe original value (GATE-ME-12)
8 (a) 1 (b) 2 (c) 4 (d) 8
(d) _!_ ofthe original value
16
05. Consider a steel (Young 's modulus E = 200
GPa) column hinged on both sides. Its
02. For the case ofa slender column of length /,
height is 1.0 m and cross-section is
and flexural rigidity EI built-in at its base
10mm x20mm. The lowest Euler critical
and free at the top, the Euler's critical
bucking load (in N) is ___
buckling load is (GATE-ME-94)
(GATE-15-Set 1)
41t:EI 2 EI
(a) (b) �:
I Two Marks Questions I
1t EI
2
1t EI2
(c) (d) 4 2
/
0 1. The rod PQ of length L and with flexural
/2

rigidity EI is hinged at both ends. For what


03. A pin-ended column oflength L, modulus of minimum force F is expected to buckle?
elasticity E and second moment ofthe cross­ (GATE-ME-08)
sectional area I is loaded centrally by a
compressive load P. The critical bucking
load (Per) is given by (GATE-ME-06)

!lfl•l!@jjjl4omQQnnlM\jijjj+iyderabad I Delhi I Bhopal I Pune I BhubancswarJ Lucknow! PatnaJ Bengaluru I Chennai JVtjayawadaJVmg I T,rupati I Kukatpally I Kolkara I
: 1 10 : Strength of Materials

EI h1r 2 EI Young's modulus E and coefficient of


(a) (b)
2
,r

L2 L2 thermal expansion a
EI EI
(c) (d)
2 2
,r ,r

J2L2 2L2

02. A column has a rectangular cross-section of


10 mm x 20 mm and a length of l m. The If the beam is slowly and uniformly heated,
slenderness ratio of the column is close to the temperature rise required to cause the
(GATE-ME-11) beam to buckle is proportional to
(a) 200 (b) 346 (c) 477 (d) 1000 (GATE- 17- SET- l)
(a) d (b) d2 (c) d3 (d) d4
03. An initially stress-free massless elastic beam
of length L and circular cross-section with
diameter d (d<<L) is held fixed between two
walls as shown. The beam material has

l1t11M@hl44ih401Miiiiili� yderabad I Delhi I Bhopal I Pune I Bhubaneswarl Lucknow I Patna I Bengalwu I Chennai I Vuayawada I Vmg I Tirupari I K�y I Kolkata
SOLUTIONS
05. Ans: 3289 (range 3285 to 3295)

One Mark Solutions


7r 2
Sol: />,;r = x EImin
7
0 1. Ans: (b) 20 x 10 3 .
I xx = ---= l .66 x l 0 3 . ... mm MI
1t 2 EI 12
Sol: P= -­
L2 yy 10 X 20 3
I = = 6.66 X 10 3
As L 1 = 2L 12

1t 2 EI _!_ 1t 2 EI rr 2 X 200 X 10 3 X 1.66 X 10 3


p1 = = ( J= _!_ [P] = = 3289N
(2L )2 4 L2 4 ( 1000)2

02. Ans: (d) Two Marks Solutions


Sol: Effective length, le = 2 /.
1t 2 1t 2 0 1. Ans: (c)
Pe = EI= EI
(2 / ) 2 41 2 Sol:

03. Ans: (d)


Sol: For pin-ends l= L

T = tension in rod PQ
04. Ans: (c)
Form equilibrium of the rod at Q
= _L_ = 4
,e, 2
Sol: --- T cos 45 ° + F = 0
2
-=
pFix-Fix hinge - hinge

phinge-hinge £ ' so-,;, '


( �) T sin45 °+ Ro = 0
:. RQ = F
T = hF

iltlli@OiiU.jjijmftjM\@j� yderabad I Delhi I Bhopal I Pune I Bhubaneswari Lucknow I Pama I Benga)wu I Chennai I Vijayawada I Vizag I Tirupati I Kukatpally I Kolkata
..,
. ..��
�.• ,
� . A. CE . .
Pnblbmna : 112: Strength of Materials

Euler's theory is applicable for long and


axially loaded columns 03 . Ans: (b)
7t 2 Sol: As per Euler's theory
J2. F = - EI
z2 7t 2
Pe = - EI . . . . . . :. .. (1)
1t EI2
.e, 2
There fore, F =
v 2 l2

but !_ = er = (at XE)
A
P = atEA. . . . . . (2)
02. Ans: (b)
Equating (1) and (2)
Sol: Slenderness ratio, A = _I_ 1[ 2
rmin - EI = atE(A)
. =�
.e, 2
I min
rmm
A
20 X 103
Where Imm = 1666.67 Inm 4

12
A = 20 x 10 = 200 mm2 :. t OC d2
/ = 1000 mm
:. A = 346

!IJIIQl@QRflftiMihiM* Hyderabadl Dclhil Bhopal i Pune l Bhubaneswarl Bengaluru JLucknowl Patna JChennai JVuayawada JV,zag IT,rupati I Gunturl Kukatpally(Hyd) I
I 1 2 Propped and Fixed Beams
Chapter

One Mark Questions Two Marks Questions

01. Two identical cantilever beams are O1. A uniformly loaded propped cantilever
supported as shown, with their free ends in beam and its free body diagram are shown
contact through a rigid roller. After the load below. The reactions are
P is applied, the free ends will have
(GATE-ME-OS)

Jlll!l!!ll!l!I I I I !
(
(a) Equal deflections but not equal slopes
rR1 RJ
(b) Equal slopes but not equal deflections
(c) Equal slopes as well as equal (GATE-ME-07)
deflections
(d) Neither equal slopes nor equal 5 3
(a) RI = qL R2 = qL M = qL
2

deflections 8 , 8 , 8

(b) R = qL , _.R = qL , M = q
3 5 L2
I 8 � 8 8
3
(c) RI = SqL , R2 = qL , M = O
8 8
3 R 35qL M = O
(d) RI = qL
8 2
= 8
, ,

\( I l 1 1g1111 t 1 1 11g Piildu ,111 1 1 1 1 -.. �yderabad I Delhi I Bhopal I Punc I Bhubancswar I Lucknow I � J Bcngaluru I Oiennai I Vgayawada j V17.3g I Tirupari I Kukalpally I K.olkata I
SOLUTIONS
5q
Rl = L
One Mark Solutions 8

0 1. Ans: (a)
Sol: The free ends of cantilevers are resting one R 2 ( L)+ M-q ( L {� )= 0

over other, therefore deflections must be the


qL
2
same. The slopes may be in opposite M =-
directions (i.e. one clockwise and another
anticlockwise).

Two Marks Questions

0 1. Ans: (a)
Sol:

At Prop deflection is zero

_ 3q L
R2 -
8

LFy = 0

IM•Ni@jjjqg.111Qjmftj@jiim.� yderabad I Delhi I Bhopal I Pune I Bhubaneswar l Lucknow I Patna I Bengaluru I Chennai IVliayawada I V17.3g I Tirupati I K.ukalpally I Kolkata
C13 Strain Energy

One Mark Questions Two Marks Questions

0 1. Two threaded bolts A and B of same 0 1. A stepped steel shaft shown below is
material and length are subjected to identical subjected to 10 N-m torque. If the modulus
tensile load. If the elastic strain energy of rigidity is 80 GPa, the strain energy in the

I--d- - - -1 -J-t-
stored in bolt A is 4 times that of bolt B and shaft in N-mm is (GATE-ME)
the mean diameter of bolt A is 12 mm, the
mean diameter ofbolt B in mm is
Ta I ONm
(GATE-ME-13) -CT
(a) 16 (b) 24 1 00mm 1 00mm
(c) 36 (d) 48
(a) 4. 12 (b) 3.46
02. For a ductile material, toughness ts a (c) 1.73 (d) 0.86
measure of (GATE-ME-13)
(a) resistance to scratching 02. The strain energy stored in the beam with
(b) ability to absorb energy up to fracture flexural rigidity EI and loaded as shown in
(c) ability to absorb energy till elastic limit the fi gure is (GATE-ME-08)
(d) resistance to indentation p p

l
03. A cantilever beam of length L and flexural
modulus EI is subjected to a point load P at
L I•
L
•I•
2L
•I•
L�
the free end. The elastic strain energy stored
in the beam due to bending (neglecting P 2 L3 2P 2 L3
(a) (b)
transverse shear) is (GATE- 17- SET 2) 3EI 3EI
P 2 L3 P 2 L3 PL3 PL3 4P 2 L3 8P 2 L3
(a) (b) (c) (d) (c) (d)
6EI 3EI 3EI 6EI 3EI 3EI

!lfl1Qjj§jjji4.jjji@biM\@U�J:IYderabad I Delhi I Bhopal I Punc I Bhubancswar I Lucknow! Patna I Bcngaluru I Oiennai I VyayawadalVu.ag I Tll\lpati I Kukatpally I Kolkata I
SOLUTIONS
One Mark Solutions Two Marks Solutions

01. Ans: (b) 0 1. Ans: (c)

(u) =--
P2 L T2L T2L
Sol: Strain energy Sol:
2AE 2GJ 2GJ 2
U = U ' + U 2 = -- + --
I
UA = 4Ua
J =!!_ (50)4 ,• J2 =!!_ (25)4
1
U cc - 1 I 32 32
cc -2
A d L= l OOmm
1 1
d A2 d 2B
-=4x- G = 80 x 10 3 N/mm 2
On substitution,U = 1.73 N-mm
1 1
122 d 2B
-= 4 x -
02. Ans: (c)
d a = 4x 12
2 2
Sol:
:. d 8 2x 12 = 24 mm
=

02. Ans: (b)


Sol: Toughness is the material property. It says
about energy absorbed by the material
Maximum bending moment occurs under
against failure. (i.e. the area under the stress
load 'P' from any support
versus strain curve up to failure).
L M 2 dx 2L
(PL) dx
2
U-
03. Ans: (a)
J0 2EI + J
0 2EI
L
Sol: _ (Px) dx + (PL) 2L
2 2

I.
-
X J" I 2EI

4 p 2 L3
3EI

= M dx ' (Px) dx P x
t = --
3EI
2 2 2 3
U =J = [
2EI O 2EI 2EI 3 J 0

p 2 .e 3
U = --
6EI
!1111ih4hiiiihi4AflGiMhiii+1ydcrabad I Delhi I Bhopal I Punc I Bhubancswarl Lucknow I Patna.I Bcagaluru I Chcnnai IVuayawada I Vmg I TllUpari I Kukatpallyl Kolkata !
Theory of Mach ines
(Questions)
Page No. 118

CONTENTS
-
- - �,-- --- - - -� - ----- - ,- - - -- -----,

Chapter Questions Solutions


N ame o f the Ch ap ter
N o . __ � - -- _ __ _____ __ 1 P age N o . P age N o .
01 Analysis of Planar Mechanism 1 19 - 130 1 3 1 - 140
Dynamic Analysis of Single-Crank
02 141 - 142 143 - 144
Mechanism
03 Gear & Gear Trains 145 - 150 151 - 155

04 Fly Wheels 1 5 6 - 158 1 59 - 161

05 Mechanical Vibrations 162 - 1 74 1 7 5 - 189

06 Gyroscope 190 - 190 191 - 191

07 Balancing 192 - 192 193 - 193

08 Governors & Cams 194 - 194 195 - 196

... .r ' . .... ... .·.. .


• ..,. ...... ....
,......
. '¥ �
I
Chapter
1 Analysis of Planar Mechanism

03. The lengths of the links of a 4-bar linkage


One Mark Questions with revolute pairs only are p, q, r and s
units. Given that p < q < r < s. Which of
01. For the planar mechanism shown m Fig. these links should be the fixed one, for
select the most appropriate choice for the obtaining a "double crank" mechanism?
motion of link 2 when link 4 is moved (GATE-ME-03)
upwards. (GATE-ME-99) (a) link of length p (b) link of length q
(c) link of length r (d) link of length s

04. For a mechanism shown below, the


3

mechanical advantage for the given


configuration is (GATE-ME-04)

(a) Link 2 rotates clockwise (a) 0 (b) 0.5 (c) 10 (d) 00


(b) Link 2 rotates counter - clockwise
(c) Link 2 does not move 05. The number of degrees of freedom of a
(d) Link 2 motion cannot be determined planar linkage with 8 links and 9 simple
revolute joints is (GATE-ME-05)
02. The mechanism used in a shaping machine WI �2 �3 �4
is (GATE-ME-03)
(a) a closed 4-bar chain having 4 revolute 06. For a four bar linkage in Toggle position,
pa1rs the value of mechanical advantage is
(b) a closed 6-bar chain having 6 revolute (GATE-ME-06)
pa1rs (a) 0.0 (b) 0.5 (c) 1.0 (d) oo
(c) a closed 4-bar chain having 2 revolute
07. The number of inversions for a slider crank
and 2 sliding pairs
mechanism is (GATE-ME-06)
(d) an inversion of the single slider-crank
(a) 6 (b) 5 (c) 4 (d) 3
chain
\( 'l I 1u..:. 111t t irng P11hla .111011, Fydcrabad ! Delhi l Bhopal I Pune l Bhubaneswarl Lucknow I Patna I Bengaluru I Chennai I VuayawadajVIZag ITirupati I Kukatpallyl Kolkata I
ACE : 120 : Theory of Machines
�-���PuNimioos
" 'Ill • • • •

08. A planar mechanism has 8 links and 10 (c) should always be perpendicular to PQ
rotary joints. The number of degrees of (d) should be along QP when the body
freedom of the mechanism, using Gruebler's undergoes pure translation
criterion, is (GATE-ME-08)
(a) O (b) l (c) 2 (d) 3 1 2. Which of the following statements 1s
INCORRECT (GATE-ME-10)
09. A simple quick return mechanism is shown (a) Grashof s rule states that for a planar
in the figure. The forward to return ratio of crank-rocker four bar mechanism, the
the quick return mechanism is 2 : 1. If the sum of the shortest and longest link
radius of the crank 0 1 P is 1 25mm, then the lengths cannot be less than the sum of
distance 'd' (in mm) between the crank the remaining two link lengths.
centre to lever pivot centre point should be (b) Inversions of a mechanism are created
by fixing different links one at a time
(c) Geneva mechanism is an intermittent
motion device
(d) Gruebler's criterion assumes mobility
of a planar mechanism to be one.

(GATE-ME-09) 1 3. A double - parallelogram mechanism 1s


(a) 144.3 (b) 2 1 6.5 shown in the figure. Note that PQ is a single
(c) 240.0 (d) 250.0 link. The mobility of the mechanism is
(GATE-ME-11)
10. Mobility of a statically indeterminate
structure is (GATE-ME-10)
(a) � -1 (b) 0
(c) 1 (d) � 2

1 1. There are two points P and Q on a planar


rigid body. The relative velocity between (a) - 1 (b) 0
the two points (GATE-ME-10) (c) 1 (d) 2
(a) should always be along PQ
(b) can be oriented along any direction

\( } I 11g11H I I 1 1 11,!, P11hli< ,IIJOIJ', yderabad I Delhi I Bhopal I Pune I Bhubaneswar I Lucknow I Patna I Bengaluru I Chennai I Vijayawada I Vi7.ag I Tirupati I Kukatpally I Kolkata
" ACE
.. 1i'....:.......:.. p.,blicdiooa : 121 : Analysis of Planar Mechanism
:.-:. �;;;;�
� ��.....-- ��=============;;.;;,,,;�======�����������
14. A solid disc ofradius r rolls without slipping
on a horizontal floor with angular velocity co
and angular acceleration a. The magnitude
ofthe acceleration ofthe point ofcontact on
the disc is (GATE-ME-12)
(a) zero (b) ra
(a) 3 (b) 4 (c) 5 (d) 6
(c) �(ra ) + (rco) (d) rco2
2 2

17. A planar closed kinematic chain is formed


15. In the mechanism given below, if the with rigid links PQ=2.0 m, QR = 3.0 m,
angular velocity of the eccentric circular RS = 2.5 m and SP = 2.7 m with all revolute
disc is 1 rad/s, the angular velocity (rad/s) of joints. The link to be fixed to obtain a
the follower link for the instant shown in the double rocker (rocker-rocker) mechanism is
figure is (GATE-ME-12) (GATE-ME-13)
(a) PQ (b) QR (c) RS (d) SP

18. A circular object of radius r rolls without V


slipping on a horizontal level floor with the
center having velocity V. The velocity at the

Note : All dimensions are in mm point of contact between the object and the
floor is (GATE-ME-14)
(a) 0.05 (b) 0. 1 (c) 5.0 (d) 10.0 (a) zero
(b) V in the direction ofmotion
16. A link OB is rotating with a constant angular (c) V opposite to the direction ofmotion
velocity of 2 rad/s in counter clockwise (d) V vertically upward from the floor
direction and a block is sliding radially
outward on it with an uniform velocity of 19 A rigid link PQ is 2 m long and oriented at
0.75 mis with respect to the rod, as shown in 20 ° to the horizontal as shown in the figure.
the figure below. If OA= l m, the magnitude The magnitude and direction of velocity VQ,
of the absolute acceleration of the block at and the direction of velocity Vp are given.
location A in mls2 is The magnitude of Vp(in mis) at this instant
(GATE-ME-13) is (GATE -ME-14)

\( 'l l 11�111l l 1 111� P11hlu .111011, Fydcrabad l Delhi l BhopaI I Pune l Bhubancswarl Lucknowl Pa1na l Bengaluru l Chennai lVuayaw.idalVmg ITirupari I Kukatpally l Kolkata I
t
:.:�PnJmn.q
':i
. A:CE . . : : 122 Theory of Machines
� � ==========================================
(a) Both I and II are false
V0= lm/s:
(b) I is true and II is false
(c) I is false and II is true
(d) Both I and II are true

22. A wheel of radius r rolls without slipping on


(a) 2. 14 (b) 1.89 (c) 1.2 1 (d) 0.96 a horizontal surface shown below. If The
velocity ofpoint P is 1Om/s in the horizontal
20. A 4-bar mechanism with all revolute pairs direction, the magnitude of velocity of point
has link lengths Ir = 20 mm, /in = 40 mm, Q (in mis) is __ (GATE-15-Set 1)
lco =50mm and lout = 60 mm. The suffixes Q
'f , 'in', 'co' and 'out' denote the fixed link,
the input link, the coupler and output link
respectively. Which one of the following
statements is true about the input and output
links ? (GATE -ME-14)
(a) Both links can execute full circular 23. The number of degrees of freedom of the
motion planetary gear train shown in the fi gure is
(b) Both links cannot execute full circular (GATE -15-Set 2)
motion 20 teeth gear
(c) Only the output link cannot execute
full circular motion
50 teeth gear
(d) Only the input link cannot execute full
circular motion

2 1. For the given statements :


I. Mating spur gear teeth is an example of (a) 0 (b) 1 (c) 2 (d) 3
higher pair
II. A revolute joint is an example oflower 24. In the fi gure, link 2 rotates with constant
patr. angular velocity CO2. A slider link 3 moves
Indicate the correct answer. outwards with a constant relative velocity
(GATE -ME-14) VQ/P, where Q is a point on slider 3 and p is a

jlJ•lii@jjji4.jjjiRbftiM\jjfjj+yderabad I Dcllii I Bhopal I Pune I Bhubaneswarl Lucknow I Patna I Bengaluru I Chennai I Vyayawada jVizag I Tirupati I Kukatpally I Kolkata I
: 123 : Analysis of Planar Mechanism

point on link 2. The magnitude and direction


of Coriolis component of acceleration is
given by

Q on 3

p (GATE-16- SET-3)
Which one ofthe following is TRUE?
(a) VQP has components along and
(GATE-15-Set 3) perpendicular PQ
(a) 2ro 2 VQ/P; direction of VQIP rotated by (b) VQP has only one component directed
90° in the direction of OJi. from P to Q
(b) ro 2 VQ/P; direction of VQ/P rotated by (c) VQP has only one component directed
90° in the direction of m2 from Q to P

(c) 2ro 2 VQIP; direction of VQ/P rotated by (d) VQP has only one component
perpendicular to PQ
90° Opposite to the direction of (1)2
(d) ro 2 VQ/P; direction of VQ/P rotated by
27. The number of degrees of freedom in a
90° Opposite to the direction of (1)2
planner mechanism having n links and j
25. A single degree of freedom mass-spring - simple hinge joints is
viscous damper system with mass m, spring (GATE-16- SET-3)
constant k and viscous damping coefficient (a) 3(n -3)-2j (b) 3(n- 1)-2j
(c) 3n-2j (d) 2j-3n+ 4
q is critically damped. The correct relation
among m, k and q is
(GATE-16- SET-2) 28. In a slider-crank mechanism, the lengths of

(b) q= 2� the crank and the connecting rod are 100mm


(a) q = .J2km
and 160mm, respectively. The crank 1s
(c) q {2k (d) q = 2 -/f rotating with an angular velocity of 10
= f;;
radian/s counter-clockwise. The magnitude
26. A rigid link PQ is undergoing plane motion of linear velocity (in mis) ofthe piston at the
as shown in the fi gure (Vp and VQ are non­ instant corresponding to the confi guration
zero). VQP is the relative velocity ofpoints Q
shown in the figure is__
with respect to point P.
!IJ11i@jjji4.jjiRflbi@ufjj� yderabad! Delhi ! Bhopal ! Pune ! Bhubaneswarl 1ncknow ! Patna ! Bengaluru! Chellllail Vliayawada j V17.3g ITirupari I Kukalpallyl Kolkata
"�.t-. . .
. . ........" ACE . .
. :�PnNiranooa : 124: Theory of Machines

03. Fig. shows a quick return mechanism. The


cranks OA rotates clockwise uniformly.
OA= 2 cm, 00 1 = 4 cm. The ratio of time
for forward motion to that for return motion
is (GATE -ME-95)

(GATE - 17 - SET-2) (a) 0.5

(b) 2.0

Two Marks Questions (c) ../2

(d) 1
O1. Instantaneous center of a body rolling with
sliding on a stationary curved surface lies 04. A rod of length l m is sliding in a comer as
(GATE -ME-92)
shown in Fig. 2.7. At an instant when the
(a) at the point of contact rod makes an angle of 60 degrees with the
(b) on the common normal at the point of
horizontal plane, the velocity of point A on
contact the rod is l m/s. The angular velocity of the
(c) on the common tangent at the point of (GATE -ME-96)
rod at this instant is
contact
(d) at the center of curvature of the
stationary surface

02. The number of degrees of freedom of a five


link plane mechanism with five revolute
pairs as shown in the figure is: (b) 1.5 rad/s
(a) 2 rad/s
(GATE -ME-93)
(c) 0.5 rad/s (d) 0.75 rad/s

05. For the audio Casette mechanism shown in


Figure given below where 1s the
instantaneous centre of rotation (point) of
the two spools? (GATE -ME-99)
(a) 3 (b) 4 (c) 2 (d) 1

l11i1ij@jjji4.jjiNfl6i@!iii1+1yderabad I Delhi I Bhopal I Pune I Bhubaneswarl Lucknow! Patna! Bengaluru I O,ennai IVtjayawada IVl2.3& ITtrupati I Kukatpally I Kolkata I
: 125 : Analysis of Planar Mechanism

06. The velocity of point B with respect to point


A is a vector of magnitude
p
(a) 0
(b) ro(ra - TA) and direction opposite to the
direction of motion of point B
C + + F
D E (c) ro(ra-rA) and direction same as the direction
of motion of point B
(a) Point P lies to the left of both the spools (d) ro(ra -rA) and direction being from O to Z
but at infinity along the line joining A
&H 07. The acceleration of point B with respect to

(b) Point P lies in between the two spools point A is a vector of magnitude
on the line joining A & H, such that (a) 0
PH = 2AP (b) ro(ra2 - rA2) and direction same as the
(c) Point P lies to the right of both the direction of motion of point B
spools on the line joining A & H, such (c) ro2(ra -rA) and direction opposite to the
that AH = HP direction of motion of point B
(d) Point P lies at the intersection of the (d) ro2cra -rA) and direction being from Z
line joining B & C and the line joining to o
G&F
08. In the figure shown, the relative velocity of
link 1 with respect of link 2 is 1 2 m/sec.
Common Data for Questions 06 & 07
Link 2 rotates at a constant speed of 1 2 0
The circular disc shown in its plan view in the
rpm. The magnitude of Coriolis component
figure rotates in a plane parallel to the horizontal
of acceleration of link 1 is
plane about the point O at a uniform angular
(GATE -ME-04)
velocity ro. Two other points A and B are located
on the line OZ at distances rA and r8 from O
respectively. (GATE -ME-03)
,Z

A 0/
I

I
I

!Jfll4jj§jj/i4.jjiJmnj1ijj.jj*)ttyderabad I Delhi I Bhopal I Pune I Bhubaneswar I Lucknow I Patna I Benga)uru I Chennai I Vtjayawada I Vtzag ITIIUpati I Kukatpally I KolkataI
ACE
-:;rm�111�Pu The:
l.�
;
,Ii
�· i;
..
Fii
=� :
11p1
�· ��
�-��========�:::�:==========�� :�::���
Nnric«w
�· 1 26 · �e:,.
ory of Machin s

(a) 302 m/s2 (b) 604 m/s2 1 1 . Match the following with respect to spatial
(c) 906m/s 2
(d) 1 2 08 m/s 2
mechanisms. (GATE -ME-04)
Type of Joint Motion constrained
09. The figure below shows a planar mechanism P. Revolute 1 . Three
with single degree of freedom. The instant Q. Cylindrical 2. Five
center lz4 for the given configuration in R. Spherical 3. Four
located at a position . (GATE -ME-04) 4. Two
2 5. Zero
L - - - - - - - - - - �' N

p Q R p Q R
(a) 1 3 1 (b) 5 4 3
I
(c) 2 3 1 (d) 4 5 3
1�90° M
- · · T · · -\· · - · · - · · - · · - · · - · · -,__
· · -.......
-
I , ....,r

I Common Data for Questions 12, 13 & 14.


1

(a) L (b) M (c) N (d) 00 An instantaneous configuration of a four-bar


mechanism, whose plane is horizontal, is shown
1 0. Match the following: (GATE -ME-04) in the figure below. At this instant, the angular
Type of Mechanism velocity and angular acceleration of link Q2 A are
P. Scott -Russel mechanism co = 8 rad/s and a = 0, respectively, and the
Q . Geneva mechanism driving torque (T) is zero. The link Q2 A is
R. Off-set slider- crank mechanism balanced so that its centre of mass falls at 02 •
S. Scotch Yoke mechanism (GATE -ME-05)
Motion achieved B
1 . Intermittent motion
2. Quick return motion
3 . Simple harmonic motion
160 mm
(()
4. Straight line motion a,
60 mm
90°
P Q R S
(a) 2 3 1 4
(b) 3 2 4 1
(c) 4 1 2 3
(d) 4 3 1 2

!IHl4jj§jjjl41hlNflfl1Mijjfjj� ydcrabad I Delhi I Bhopal I Pune I Bhubaneswarl Lucknow I Patna I Bengaluru I Chennai I Vtjayawada I Vmg I Tuupari I Kukatpally I Kolkata
: 127 : Analysis of Planar Mechanism

12. Which kind of 4-bar mechanism 1s p Q R s (GATE -ME-06)


02AB04? (a) 2 6 4 3
(a) Double crank mechanism (b) 6 2 4 1
(b) Crank-rocker mechanism (c) 6 2 5 3
(c) Double rocker mechanism (d) 2 6 5 1
(d) Parallelogram mechanism
16. In a four-bar linkage, S denotes the shortest
13. At the instant considered, what is the link length, L is the longest link length, P
magnitude of the angular velocity of 04B? and Q are the lengths of other two links. At
(a) l rad/s (b) 3 rad/s least one of the three moving links will
(c) 8 rad/s (d) 64/3 rad/s rotate by 360 ° if (GATE -ME-06)
(a) S + L � P + Q (b) S + L > P + Q
14. At the same instant, if the component of the
force in joint A and AB is 30N, then the (c) S + P � L + Q (d) S + P > L + Q
magnitude of the joint reaction at 02 .
(a) is Zero (b) is 30N 1 7. The input link 02P of a four bar linkage is
(c) is 78 N rotated at 2 rad/s counter clockwise
(d) cannot be determined from the given direction as shown below. The angular
data velocity of the coupler PQ in rad/s, at an
instant when L0402P= 180 °, is
15. Match the items in columns I and II (GATE -ME-07)
List I Q
P. Higher kinematic pair
Q. Lower kinematic pair
R. Quick return mechanism
02P=0204=a

S. Mobility of a linkage PQ=04Q= .fi. a

List II
1. Grubler's equation 2 rad /sec

2. Line contact
3. Euler's equation (a) 4 (b) 2-fi.
4. Planer 1
(c) 1 (d) -
5. Shaper ,fi.
6. Surface contact

!ltl•i!@jjl44ijjgRfl§iM!dih�yderabad I Delhi I Bhopal I Pune I Bhubaneswarl Lucknow! Patna I Bengaluru I Chennai I Vuayawada j Vmg I T=pati I Kukatpally I Kolkata l
t ACE
�·::.r.
: : :=a;;;;
�e:o:ry�o;;;;f�M
�Th :·:e�
� :l'!nll
�·-:·�rc�Pu �11:,========�;,;:�:.=:=========
::b�•�rati�:«.J 1 28 chin s
,I. I,. =
Fii�p

Statement for Linked Answer Q. 18 & 19 Approach


1. D' Alembert's principle
A quick return mechanism is shown below.
2. Grubler's criterion
The crank OS is drive at 2 rev/s in counter
3. Grashoff's law
clockwise direction. (GATE -ME-07)
R
4. Kennedy's theorem

p Q R s
Q (a) 1 2 3 4
(b) 3 4 2 1
0 (c) 2 3 4 1
sool (d) 4 2 1 3

21. For the configuration shown, the angular


velocity of link AB 1s 10 rad/s
18. If the quick return ratio is 1 : 2, then the counterclockwise. The magnitude of the
length of the crank in mm is relative sliding velocity (in ms- 1 ) of slider B
(a) 250 (b) 250 ,.fj with respect to rigid link CD is
(c) 500 (d) 500 ,.fj (GATE -ME-10)

AB = 250
19. The angular speed of PQ in rev/s when the BC = 250.fi
block R attains maximum speed during AC = 500
forward stroke (stroke with slower speed) is C
(a) 1/3 (b) 2/3 (c) 2 (d) 3

20. Match the approaches given below to (a) 0 (b) 0.86 (c) 1.25 (d) 2.50
perform stated kinematics/dynamics analysis
of machine (GATE -ME-09) 22. For the four - bar linkage shown in the
Analysis figure, the angular velocity of link AB is 1
P. Continuous relative rotation rad/s. The length of link CD is 1.5 times the
Q. Velocity and acceleration length of link AB. In the configuration
R. Mobility shown, the angular velocity of link CD in
S. Dynamic - static analysis rad I s is (GATE -ME-11)

\( I I 1 1 .:1 1 1 1 , r 1 1 1:.!, P11hlu .11101 1.., ydcrabadl Delhi I Bhopal I Punc I Bhubancswarl Lucknow I Patna l Bcngaluruj Chennai I Vuayawadal Vtzag I Tirupari I Kukatpallyj Kolkala
: 129 : Analysis of Planar Mechanism

25. In a certain slider crank mechanism, lengths


C
B of crank and connecting rod are equal. If the
crank rotates with a uniform angular speed
of 14 rad/s and the crank length is 300mm,
the maximum acceleration of the slider (in
mls2) is ___ (GATE-15 -Set 2)

(a) 3 Cb) I2 (c) 1 (d) 3.


3 26. The number of degree of freedom of the
linkage shown in figure is
23. A slider-crank mechanism with crank radius
60mm and connecting rod length 240mm is
shown in figure. The crank is rotating with a
uniform angular speed of 10 rad Is, counter
clockwise. For the given configuration, the
speed (in mis) of the slider is ____ (GATE -15-Set 3)
(a) -3 (b) 0 (c) 1 (d) 2

27. Figure shows a wheel rotating about 02 .


Two points A and B located along the radius
(GATE -ME-14) of wheel have speeds of 80mls and 140 mis
respectively. The distance between the
24. An offset slider - crank mechanism is points A and B is 300 mm. The diameter of
shown in the figure at an instant. the wheel (in mm) is _____
Conventionally, the Quick Return Ratio
(QRR) is considered to be greater than one.
The value of QRR is ----

(GATE -15 -Set 3)

(GATE -ME-14)

!ltl1ih§jjji4.jji@ftiM\h1h*)ttyderabad I Delhi I Bhopal I Pune I Bhubaneswar I Lucknow I Patna I Bengaluru I Chennai I Vtjayawada l VIZ.ag I Tirupati I Kukatpally I Kolkata I
: 130 : Theory of Machines

28 . In an off-set slider crank mechanism, shown


in figure, the crank is rotated at a constant 30. For an inline slider-crank mechanism, the
speed of 150 rpm. The value of the angle 0 lengths of the crank and connecting rod are
shown in the figure is 20 °. What is the ratio 3 m and 4 m respectively. At the instant
of forward to return stroke time? Can this when the connecting rod is perpendicular to
mechanism be used in an application the crank, if the velocity of the slider is 1
involving quick return ? mis, the magnitude of angular velocity (upto
... . - · - . 3 decimal points accuracy) of the crank is
'
I ---- radian/s.
(GATE- 17 - SET-1)

500 mm 31. Block 2 slides outward on link 1 at a


_ _ _ _ _ _ _ _ ::j��J _ _ uniform velocity of 6mls as shown in the
figure. Link 1 is rotating at a constant
I I

Forward angular velocity of 20 radian/s


(GATE - PI-15) counterclockwise. The magnitude of the
(a) 3.33, No (b) 0.73, yes total acceleration (in mls2) of point P of the
(c) 1.25, yes (d) 0.73, No block with respect to fixed point O is---

29. The rod AB, of length 1 m, shown in the


figure is connected to two sliders at each end
through pins. The sliders can slide along QP
and QR. If the velocity VA of the slider at A
is 2 mis, the velocity of the midpoint of the
--- mis.
0
(GATE- 16- SET-2)
X

(GATE-17- SET-2)
,
,,
'
I
I

I
I
I

,''"\ 600 600


L,_ _J_ _ _ _ _ _ _ - --- R
Q

!li11Q@jj/i4jjjjQjmnj@jjj.jj� yderabad l Dellii l Bhopa! I Pune ! Bhubaneswar l Lucknowl Patna l Bengaiuru ! Chennai j Vrjayawada j Vizag I Tirupati I Kukatpally l Kolkata
SOLUTIONS
DOF = 3(N-1)--2P1
One Mark Solutions = 3(8-1)--2(9)
= 2 1- 18 = 3
01. Ans: (b)
Sol: Link 2 rotates counter clockwise. Identify 06. Ans: (d)
the h4 . Direction of the velocity of h4 gives Sol: At toggle position velocity ratio is 'zero' so
the direction ofrotation oflink 2. mechanical advantage is 'oo'.

02. Ans: (d)


07. Ans: (c)
Sol: It is 2nd or 3 rd Inversion of single slider -
Sol: .·. Number ofInversions = number oflinks.
crank chain which gives quick return motion
Inversion mechanism obtained by fixing
different links at a time.
03. Ans: (a)
Sol: P = Shortest link
08. Ans: (b)
S = Longest link
Sol: Gruebler's Criterion
If shortest link P is fixed then we get double
DOF = 3(N-1)--2P1
crank mechanism.
N = Number oflinks,
04. Ans: (d) P 1 = Number ofrotary joints
Sol: Mechanical advantage is the reciprocal of Given
velocity ratio. Velocity ratio at this :. DOF = 3(8-1)- (2 x 10) = 2 1-20 = 1
configuration is zero being the dead center
position. Hence mechanical advantage m 09. Ans: (d)
this configuration is infinite 'oo" Sol: The mechanism 1s shown m the extreme
position
05. Ans: (c) 180+ 2a
Sol: Grubler's equation QRR= => a= 30 0
180-2a
DOF = 3(N - 1)-2P1
-- = sm a= - => 0 1 0 2 = d = 250mm
01P . 1
N = Number .oflinks 0,0 2 2
P = Number .ofrevolute pairs

!M11M@jjj44ijjjQAflb1Mih.jj� yderabad l Delhi l BhopaI I PuneJ Bhubaneswar J LucknowJ Patna l Bengalwu l Chennai J VrjayawadaJ Vizag ITuupati I Kukatpallyl Kolkata
: 132 : Theory of Machines

10. Ans: (a) \

Sol: Indeterminate means super structure Locus of 123 ¥/\,


.

:. Mobility �-1
\

11. Ans: (c)


A

12. Ans: (a)


Locus of l23

13. Ans: (c) So the I - Center h3 will be on the line


Sol: It is the failure of Gruebler's equation of perpendicular to the link - 2.
DOF because it does not consider the shape So the point C is the intersection of these
and dimensions ofthe mechanism. two loci which is the center ofthe disc.
So m2 (I, 2 , I 2 = ro3 (I 1 3 , I 23 )
J
14. Ans: (d) � ffi2 X 50 = 1 X 5
Sol:
� m 2 = 0. 1 rad / sec

aTo = ra
16. Ans: (c)

0 Sol: Given ro= 2 rad / sec, a= 0


V= 0.75 m / sec
Acceleration at point 'O'
OA = r = 1 m

a� and a;._ are linear accelerations


with same magnitude and opposite m
direction.

Centrifugal acceleration,
f = r ro 2 = 4 m / sec 2
15. Ans: (b)
[As a = 0, tan gential acceleration, f1= 0 ]
Sol: hJ should be in the line joining I 1 2 and I 1 3•
for = 2 Vro = 3 rn/sec2
Similarly the link 3 is rolling on link 2.
Resultant acceleration,
tR = �4 2 + 3 2=5 m / sec 2
!IJ11ijj§jjj4i.jjiRflnjffijuo\j� I I I I I I I I I I
yderabad Dellii Bhopal Pune Bhubaneswarl Lucknow! Patna! Bengaluru Otennai Vtjayawada Vizag TlrUpati Kukatpally Kolkata
: 133 : Analysis of Planar Mechanism

17. Ans: (c) 'I' is the instantaneous centre.


Sol: From sine rule
s PQ IQ IP
-- = - - = --
3 sin 45 sin 70 sin 65
2.7
IP = --sin 65 °
-
IQ sin 70°
p 2 Q
V
The given dimensions of the linkage VQ = I Q x ro = l => ro = __g_
IQ
satisfies Grashofs condition to get double
IP sin 65 °
rocker. We need to fix the link opposite to VP = IP x ro = - x VQ = ---° x l
IQ sin 70
the shortest link. So by fixing link 'RS' we
= 0.9645
get double rocker.

18. Ans: (a) Alternate solution:


Sol: The point of contact is the instantaneous For a rigid link, AB = constant
centre whose velocity is zero VPQ =0

19. Ans: (d) :. Vp cos20 = Vo cos(45-20)


Sol: Refer the figure shown below, By knowing cos 25
=> VP = = 0.96m / s
the velocity directions instantaneous centre cos 20
can be located as shown. By knowing
velocity (magnitude) of Q we can get the 20. Ans: (a)
angular velocity of the link, from this we Sol: Given links satisfy Grashofs criterions
can get the velocity of 'P using sine rule. (/ + s <P + q) The shortest link is fixed so
the resulting mechanism is a double crank
mechanism where l= longest link, s =
shortest link.

vQ= lm/sec 2 1 . Ans: (d)


Sol: Mating of spur gear teeth forms higher pair
and Revolute joint is a lower pair.

!lflliUYOiiiih!MRdbiM\Ulh+iydcrabad I Delhi I Bhopal I Pune I Bhubaneswarl Lucknow I Patna I Bcngaluru I Chcnnai IVuayawadal Vmg I T1111pati I Kukatpally I KolkalaI
: 134 : Theory of Machines

22. Ans: 20 28. Ans: 1


Sol: Velocity of P= rro= 10 m / sec Sol: B

VP 10
OJ = - =- 1 0 rad/s
R R
- - - - - - - - - - - - - - - mm
Velocity of Q= 2Rro
A

10
I

= 2R x = 20 m l sec
I

R
b /_lAB
/JG - - - - ----- a,g
,' C
I
I
I

23. Ans: (c)


(Velocity Diagram)
l_BC

Sol: No .of Links, L = 4


No. ofclass 1 pairs J 1 =3 From the velocity diagram when crank is
No. ofclass 2 pairs Jz = 1 (Between gears) perpendicular to the line of stroke, the
No. ofdof= 3(L-1)-2J 1-Jz = 2 velocity of slider =
velocity of crank and
angular velocity ofconnecting rod is zero.
24. Ans: (a) ro 2 = 10 rad/s
Sol: The direction of coriolis component is At this position, 1.e. crank angle 0 =
90 °
obtained by rotating the linear velocity Vslider = rro 2 = 10 x l OO mm/s
vector in the direction ofangular velocity by :. Slider velocity =
1 mis
90 °. Its magnitude = 2 VQ/p ro 2

Two Marks Solutions


25. Ans: (b)
Sol: Critical Damping Constant = 2 �
01. Ans: (b)
q= 2 � Sol: In a pure rolling the I - centre lies at the
point ofcontact at the given instant.
26. Ans: (d) In sliding motion, the I - centre lies at
Sol: infinity in a direction perpendicular to the
path ofmotion ofslider. As for three centers
in line theorem, in both rolling and sliding
motion the I -center lies in between point of
27. Ans: (b) contact and in the direction of the centre of
Sol: "Grubler" s equation sliding i.e. perpendicular to the sliding
direction.
!M11i@jjji4ijjjiAfl§5ijjfjjlj yderabad ! Delhi I Bhopal I Pune I Bhubaneswar l Lucknow I Patna I Benga]uru I Chennai I Vtjayawada l V17.3g ITuupari I Kukarpally l Kolka!a
: 135 : Analysis of Planar Mechanism

02. Ans (c) 04. Ans: (a)


Sol: Gruebler's equation
F = 3 (N - 1)- 2P1 - P2
F = degrees of freedom
N = total number of links in a mechanism
Sol:

- - - - ,nu,r \ _ T _ _ _ _
A
I
I
I

P1 number of pairs having one degree of


=
lm :

freedom 60° �

P2 = number of pairs having two degree of 0


freedom (Higher pairs)
Above mechanism have 5 joint pairs which Va = 1 mis
have one degree of freedom = (P 1 ) = 5 Va = Velocity along vertical direction
Vb = Velocity along horizontal direction
There is No class II pairs (P2) = 0, So instantaneous center of link AB will be
Number of links N = 5 perpendicular to A and B respectively i.e at I
:. F = 3 (5 - 1) - 2 x 5 = 2
IA = OB = cos e = 1 X cos 60 ° = .!. m
2
03. Ans: (b)
Sol:
IB = OA = sin 0 = 1xsin 60 ° = fj m
2
Va = roxlA
V l
=> ro = a = __ = 2 rad / sec
IA V2

05. Ans: (c)


Sol: Consider the three bodies the bigger spool
. OA 2 1
sm a = -- = - = - (Radius 20), smaller spool (Radius 10) and
0 01 4 2
the frame. They together have three I
=> a =30 ° centers, I centre of big spool with respect to
R 180 + 2a 180 + 60 the frame is at its centre A and that of the
Q R= =
180 - 2a 180- 60 small spool with respect to the frame is at its
=> QRR = 2 centre H. The I centre for the two spools P is
to be located.

l1i•1i!i§jjjl41h4Aftft!Midfjj� yderabad ! Dellii ! Bhopal ! Pwie ! Bhubaneswari l.ucknow ! Patna ! Bengaiuru ! Chennai l Vtjayawada ! Vizag !TJrupati I Kukatpallyi Kolkata
. 1\,CE ., . Theory of Machines
.-:,
1 36
�... . l'!fn�1p
�m:nn
:111Po
��bn=ati
��IID
===========�:�,;;,:,..;,:=============�====
07. Ans: (d)
Sol: fBA = fB - fA = rb ro 2 - ra ro 2 = (rb - rJro 2
Being centripetal, the direction is towards
p

the centre of rotation


i.e., from Z to 0
C + + F
D E
08. Ans: (a)
As for the three centers in line theorem all Sol: The block has linear velocity along the axis
the three centers should lie on a straight of the link which is in angular motion hence
line implies on the line joining of A and H it has coriolis acceleration give by 2 Vco
more over as both the spools are rotating in
Given: N =120 R.P.M
the same direction, P should lie on the same
side of A and H. Also it should be close to 2,rN 2 x 1r x 120
co = = = 12.566 rad/sec
60 60
the spool running at higher angular velocity.
V =12 mis
Implies close to H and it is to be on the right
of H. Whether P belongs to bigger spool or acor =2Vro = 2 x 12.566 x 12 = 301.5 m / sec 2
smaller spool its velocity must be same. As Magnitude of Coriolis acceleration
for the radii of the spools and noting that the = 302 m/sec2
velocity of the tape is same on both the
spools 09. Ans: (d)
ffiH =2COA Sol: 2
L- - - - - - - - - - - ¥'i -
I

:.AP. ro A = HP ro H and
AP = AH+ HP => HP = AH I
°
i\0 M
06. Ans: (c)
_ _ _ _l _ _ _ _ _ _ _ _ _ _ _ _ _ _ _ _ _ _

Sol: VBA = VB - VA = rB ro - rA ro = co (rB - rA )


By considering the links 1 , 2 and 4 as for
Both are in the direction of motion three centers in line theorem, 11 2 , 114 and h4
The relative velocity is in the direction of lies on a straight line 1 1 2 is at infinity along
motion of B. the horizontal direction while 114 is at
infinity along vertical direction hence h4
must be at infinity

!iti•M!@jjji4im44flfijffijjjjjj� yderabad I Delhi I Bhopal I Pune I Bhubaneswar I Lucknow I Patna I Bengaluru I Chennai j Vtjayawada I V12ag I Tuupari I Kukatpally I Kolkata
ACE Analysis of Planar Mechanism
-:.:··E�PnNirariooa : 137 :
. � ========================================
10. Ans: (c) 13. Ans: (b)
Sol: (P) Scott-Russel mechanism-straight line Sol: 02A I 04B
motion Then linear velocity is same at A and B.
(Q) Geneva mechanism - intermittent :. CO2 X 02A = C0 4 X 0 4 B
motion .". 8 X 60= CO4 X 160
(R) Off-set slider crank mechanism-Quick
=:> C04 = 3 rad/sec
return motion
(S) Scotch yoke mechanism- simple
14. Ans: (b)
Harmonic motion
Sol: For the given data the only force acting on
the link is 30N at A along AB hence the
11. Ans: (c)
reaction at joint 02 is 30N
Sol: For revolute joint, degree of freedom is 1
and constrained DOF = 5
15. Ans: (d)
For cylindrical joint, degree of freedom is 2
and constrained DOF = 4
16. Ans:(a)
For spherical joint, degree of freedom is 3
Sol: Crank is possible only in case of Grashofs
and constrained DOF = 3
chain (/ + s < p + q) and special Grashof s
[Degree of constraints = 6 - Degree of
chain (/+ s = p+ q).
freedom]
Hence /+ s � p+ q

12. Ans: (b)


17. Ans: (c)
Sol: Calculate AB that will be equal to 260 mm
Sol: L0 40 2P = I 80 ° sketch the position diagram
L = 260 mm, P = 160 mm
for the given input angle and identify the
S = 60 mm Q = 240 mm
Instantaneous Centers.
L+ S = 320
P+ Q = 400
Q

L+ S < P+ Q
02P=0204=a
It is a Grashoff s chain
PQ=04Q= Ji a
L ink adjacent to the shortest link is fixed
.·. Crank- Rocker Mechanism.

!Ml•lh§jjjiiijjjMRflnj@jjjjj� yderabad I Delhi I Bhopal I Pune I Bhubaneswarj Lucknow I Patna I Benga)wu I Chennai I Vyayawada I Vu.ag I T1rt1pari I Kukatpallyl Kolkata
. :.t\CE . .

�-�.. =
� :� �-
L�p m Puhlicaliooa: ���
:������==========:::::::: Theory of Machines
: ==========���.;;,;;,,;���= 1 38

19. Ans: (b)


Sol: Maximum speed during forward stroke
occurs when PQ is perpendicular to the line
of stroke of the tool i. e. PQ, OS & OQ are
in straight line
� V = 250 x 2 = 750 x ro PQ

1 1 3 is obtained by joining 112 h3 and l14 l3

=-
1 1 2 1 23 a
-
ffi3
= --
0) 2 1 13 1 23 2a 20. Ans: (b)
ro3 = _!_ � ro
3 =1 rad /sec
2 2 21. Ans: (d)
Alternate Method: Sol: As for the given dimensions the mechanism
The position diagram is isosceles right angle is in a right angle triangle configuration and
triangle and the velocity triangle is similar to the crank AB is perpendicular to the lever
the position diagram CD. The velocity of B is along CD only
which is purely sliding component
18. Ans : (a) :. Velocity of the slider
Sol: =AB x ro AB = I 0 x 250 = 2.5m / sec

22. Ans: (d)


Sol: As both the links AB and CD are
perpendicular to the fixed link AD.
So the velocity of AB and CD are equal.
p � VBA =Yeo
� roAB x AB = ffico x CD
180 + 2a
QRR = I � a = 30
=> ffico = --- = - rad / sec
180 - 2a 1
0
l x AB 2
OS OP 1 .5 x AB 3
sin a= � OS = = 250mm
OP 2

jlJ11ij@jjj04.jji4RflijiM\jj.jj..)liyderabad I Delhi I Bhopal I Pune I Bhubaneswar I Lucknow I PatnaI Bengaluru I Chennai I Vuayawada j Vizag I T=pati I Kukatpally I Kolkataj
: 139 : Analysis of Planar Mechanism

23. Ans:0.618
Sol:
02 = sin -1 (�) = sin -1
.f - r 20
= 30° (.!Q)
B b a. = 02 -01 = 20.41 °
Quick return ratio,
180+ a.
(QRR) = = 1.2558
180 - a.
V
(Velocity diagram)
25. Ans: 115 to 120
Refer the configuration diagram and Sol: Acceleration of the slider
velocity diagram

tan 0 = { =
240
=> 0 = 76°
fmax = rco 2 (1+ !) = 2rco 2

r 60 = 2 x 0.3 x l 4 2 = 117.6 m / sec 2


rco2 vs
-
sin 0 sin 90°
26. Ans: (c)
rco
=> Vs = --2- = 618 mm/ sec Sol: Number of links, L = 6
sin 76
= 0.618 mlsec Number of joints, J = 7 (class - I)
Degrees of freedom, dof = 3(L -1) - 2J
=3 x 5-2x 7=1
24. Ans:1.2558
Sol: Two extreme positions are as shown m
figure below. 27. Ans: 1400
Let r = radius of crank = 20 mm Sol: VA = 80 mis , Vs = 140 mis,
I = length of connecting rod = 40 mm AB = 300 mm = 0.3 m
h = 10 mm VsA = Vs - VA = (02Bxco2) - (02Axco2)
=> 60 = (02B - 02 A) X CO2
,
----- => 60 = 0.3 X CO2
,,
,
=> CO2 = 200 rad/sec
140
Vs = OB x CO2 => OB = - = 0.7 m
\
'' ,' 200
,,
'
.... _ _ _ _ _ ... Diameter = 2x02B

01 = sin -1 (�) = sin - 1


.e + r
(.!Q)
60
= 9.55°
= 0.7x2 = 1.4 m = 1400 mm

!1tii•@jjji4iOQRbniMih•h+Yderabadj Delhi j Bhopal j Pune j Bhubaneswarl Lucknowj Patna j Bengalwu i Chennai l VuayawadalViz.ag jTuupati I Kukatpallyl Kolkatal
ACE
��
· �t�iblicaoooa :;;,:·��
�:;��
�·�·=============�:�;�:==========���:cy
hines
�- &�1p1
.:, "EE= :
w:m
�·:
Theo of M

1 40

28. Ans: (c) L


tan 0= = _± = 1.33 = n
180+ 0 180+ 20 r 3
Sol: QRR = = = 1.25
180-0 180-20 0 = 53. 13 °
rro2 rro2
Vslider - -�-= 1.25rro2
29. Ans: 1 (range 0.95 to 1.05) sin 0 sin 53. 13
Sol: Locate the I-centre for the link AB as shown 1 = 1.25 X 3 X 0)2

in fig. M is the mid point of AB 1


ro2= ---= 0.267rad / sec
Given, VA = 2m/sec 3 X 1.25

31. Ans: 243. 3

y
Sol:
p

VA r = 100 mm = 0. 1 m
VA= IA.ro => ro=
IA Uniform angular velocity = ro = 20 rad/s
VA IM Angular acceleration, a = 0
VM= IM.ro= IM - = -.VA
IA IA Uniform sliding velocity = Vs = 6 mis
= Sin 30 °.VA = ..!.. .2= 1m / sec :. Sliding acceleration f' = 0
2
Coriolis acceleration, f = 2Vro = 2x 6 x 20
= 240 m/sec 2 .1. Ir to OP
30. Ans: 0.267
Centripetal acceleration = rro2 = 0. 1 x202
Sol: Vstider = 1 m/sec, r = 3 m, L =4 m
= 40 m/sec2 towards the
B b center ofrotation

Resultant acceleration = �(r e + (r cor J J


= .J402 + 240 2 = 243 .3 m / s
2

Vs
(Velocity diagram)

Refer the confi guration diagram and


velocity diagram

!M•lii@hiiiihiiRflftiM\j@+yderabad I Delhi I Bhopal I Punc I Bhubancswar I Lucknow I Patna I Bcngaluru I Chennai I Vyayawada IVmg ITrrupati I Kukatpal)y I Kolkata I
Dynamic Analysis of single
slider - Crank Mechanism
03. A slider crank mechanism has slider mass of
Two Marks Questions 10 kg, stroke of 0.2 m and rotates with a
uniform angular velocity of 10 rad/s. The
01. The cross head velocity in the slider crank primary inertia forces of the slider are
mechanism, for the position shown in partially balanced by a revolving mass of 6
Figure. (GATE-ME-97) kg at the crank, placed at a distance equal to
crank radius. Neglect the mass of connecting
rod and crank. When the crank angle (with
respect to slider axis) is 30 ° , the unbalanced
force (in Newton) normal to the slider axis is
(GATE-ME-14)
(a) Vc cos �O- a + p )cos p
04. A slider crank mechanism with crank radius
(b) VC cos �o - a + p )sec p
200 mm and connecting rod length 800 mm
VC cos �o - a - p )cos p
is shown. The crank is rotating at 600 rpm in
(C)

(d) VC cos �o - a - p )sec p the counterclockwise direction. In the


Where Vc is the linear velocity of the crank configuration shown, the crank makes an
pm. angle of 90 ° with the sliding direction of the
slider, and a force of 5 kN is acting on the
02. Consider the triangular formed by the slider. Neglecting the inertia forces, the
connecting rod and the crank of an IC turning moment on the crank (in kN-m) is
engine as the two sides of the triangle. I f the
maximum area of this triangle occurs when
the crank is 75 °, the ratio of connecting rod SkN
length to crank radius is (GATE-ME-98)

(a) 5 (b) 4 (c) 3 .73 (d) 3


(GATE - 16 - SET - 1)

!M11i@jjji4.jjijRflft1Mij@� yderabad I Delhi I Bhopal I Pune I Bhubaneswarl Lucknow I Patna I Bengaluru I Chennai I Vijayawada I V123g I TlfUpari I Kukatpally I Kolkata
�:�.
... -....., ACE
�E�Pobtiariooa : 142 : Theory of Machines
=============================================
02. In the mechanism shown a force F2 is applied on

Five Marks Questions link 2 to overcome a torque T4 of 7500 Nm


acting on link 4. Find the required value of F2
01. A reciprocating engine slider crank mechanism (Neglect friction, gravity and inertia forces)
has a crank of 1 00 mm length and a connecting
rod of 450 mm length. Line of reciprocation of
the slider passes through the centre of rotation of
the crank shaft. If the total axial force on the
piston is 1 kN, determine the torque produced on
the crank shaft when the crank is 60° away from
the inner dead centre position. Crank shaft is
rotating at 1 800 RPM. Neglect frictional losses.
(GATE-ME-89)

(GATE-ME-90)

!Mtlii@hi4Uihi4Aflbli\iiil!i�-Iyderabad l Delhi I Bhopal I PuneJ Bhubaneswarl Lucknow I Patna I Benga)uru I Chennai JVtiayawada l Vizag I Tuupati I KukatpallyJ Kolkata I
SOLUTIONS
03. Ans: 30 N
Two Marks Solutions Sol: r ''
''
'

01. Ans: (b)


Sol: Velocity of the cross head is the velocity of
mb = 6 kg
the I centre, h4 = Oh4 x cooc
Crank radius
Oh4 = OC.cos(90-a+ 13 )secl3 = stroke/2 = 0. 1 m,
But OCx cooc = Ve co = 10 rad/sec
Unbalanced force along perpendicular to the
Vs = Vc cos (90-a+ 13 ).secl3
line of stroke = mbrco2 sin 30 °
= 6 x (0. 1) x ( 1 0)2 sin 30 ° = 30 N
02. Ans: (c)
Sol: 04. Ans: 1 (range 0.9 to 1.1)
Sol:

0.2m
Sk:N
�= _!_ (AB)(AO)sin A
L-------'--'--1-"- --
0

2
Given Fp = 5kN
Area will be maximum when A = 90°
F
Where LOAB is a right angled triangle. =-P- , Ft =Frod COS <p
cos <p
Frod

Ratio of connecting rod length to crank


:. Ft = 5kN
radius,
Turning moment = F1.r = 5x0.2 = l kN-m
l
- = tan 75 ° =3.732;
r

llff•i!@jjji4@IRflftiM\ilijj� yderabad I Delhi I Bhopal I Pune I Bhubaneswar I Lucknow I Patna I Bengaluru I Chermai I Vijayawada j Vmg I Tuupati I Kukatpally I Kolkata
: 144 : Theory of Machines

Five Marks Solutions G (due to ground)

,,
,,

,' 3 0°
,
01.
Sol: Torque (T) =Tangential fore
x Radius of crank
R
F
= _P_ x sin(0 + <j>)x r
cos <j>
Fp =1 kN.
Reos 30° =F2
. 1 sin 0 . _1 sin 60°
Where , <j> = sm - ( = sm [ ] R= F2
-;-) 450 cos 30 °
100 T4 = R x r
<I> =11.096° F2 h
T4 = -� x -- = 7500
l cos 30 cos 30
T= x sin(60 ° + 11.096 )x 0.1
cos l 1.096 °
7500 x cos 2 30
= 0.0964 kN-m= 96.4 N-m F2 = = 3 700 _ 66 N
1.52

02.
Sol:

llfllih4.$Qffj\j§\j..jj.�> Hyderabad ! Dellii l BhopaJ ! Punc l Bhubancswarl Bcngalwu i wcknowl Patnal Chcnnai l Vtjayawadal V,zag jTll'llpari I Gnnturl Kukatpally(Hyd)
Gear and Gear Trains
Gear II: Pitch circle diameter in the plane
One Mark Questions of rotation 120mm and helix angle
22.5°
01. In spur gears having involute teeth, the If the input speed is 1440 rpm. The output
product of circular pitch and diametral pitch speed in rpm is (GATE-ME-12)
1s (GATE-ME-94) (a) 1200 (b) 900 (c) 875 (d) 720

02. A 1.5 kW motor is running at 1440 rev/min. 05. Which one of the following is used to
Its is to be connected to a stirrer running at convert a rotational motion into a
36 rev/min. The gearing arrangement translational motion? (GATE-ME-14)
suitable for this application is (a) Bevel gears
(GATE-ME-00) (b) Double helical gears
(a) differential gear (b) helical gear (c) Worm gears
(c) spur gear (d) worm gear (d) Rack and pinion gears

03. Tooth interference in an external involute 06. A gear train is made up of five spur gears as
spur gear pair can be reduced by shown in the figure. Gear 2 is driver and
(GATE-ME-10) gear 6 is driven member. N2, N3, N4, Ns and -
(a) decreasing center distance between N6 represent number of teeth on gears
gear pair 2,3,4,5, and 6 respectively, The gear(s)
(b) decreasing module which act(s) as idler(s) is/ are
(c) decreasing pressure angle
(d) increasing number of gear teeth

04. The following are the data for two crossed


helical gears used for speed reduction:
Gear I: Pitch circle diameter in the plane (GATE-15-Set 3)
of rotation 80mm and helix angle (a) Only 3 (b) Only 4
30° (c) Only 5 (d) both 3 and 5
�ydcrabadlDelhilBhopailPunelBhubaneswarl LuclmowjPatnajBengaJurulChenna\jVtjayawadalViz.ag ITU'Upati I Kukatpallyl Kolkata I
ACE : 146 : Theory of Machines
-:,�
t-.'!. :Euginecriug Pi,bliatinns
4==========================================

Two Marks Questions

01. List I (Gear types)


(a) Worm gears (a) 0 rpm (b) 20 rpm
(b) Cross helical gears (c) 33.33 rpm (d) 66.67 rpm
(c) Bevel gears
04. Match the items in columns I and II
(d) Spur gears
(GATE-ME-06)
List II (Applications)
Column I
1. Parallel shafts
P. Addendum
2. Non-parallel, intersecting shafts
Q. Instantaneous center of velocity
3. Non-parallel, non intersecting shafts
R. Section modulus
4. Large speed ratios
S. Prime circle
(GATE-ME-95)
Column II
02. The arm OA of an epicylic gear train shown
1. C arn 2. Beam
in Fig. revolves counter clockwise about 0
3. Linkage 4. Gear
with an angular velocity of 4 rad/s. Both
p Q R p Q R
gears are of same size. The angular velocity
s s
(a) 4 2 3 1 (b) 4 3 2 1
of gear C, if the sun gear B is fixed, is
(c) 3 2 1 4 (d) 3 4 1 2
(GATE-ME-95)
Data for Q. 05 & 06 are given below.
(a) 4 rad/s The overall gear ratio in a 2 stage speed reduction
(b) 8 rad/s gear box (with all spur gears) is 12. The input
(c) 10 rad/s and output shafts of the gear box are collinear.
(d) 12 rad/s The countershafts which is parallel to the input
and output shafts has a gear (Z2 teeth) and pinion
B

03. The sun gear in the figure is driven (Z3 = 15 teeth) to mesh with pinion (Z 1 = 16
clockwise at 100 rpm. The ring gear is held teeth) on the input shaft and gear (Z4 teeth) on the
stationary. For the number of teeth shown output shaft respectively. It was decided to use a
on the gears, the arm rotations at gear ratio of 4 with 3 module in the first stage and
(GATE-ME-01) 4 module in the second stage. (GATE-ME-03)

IM•IP@iii§§,jjjjQffftjMJ!lijjj.� yderabad I Delhi I Bhopal I Pune I Bhubaneswarl Lucknow! Patna I Bengalurul Chennai IVijayawadalViz.ag ITirupari I Kukatpal)yl Kolkata
: 147: Gear & Gear Trains

05. Z2 and Z4 are


(a) 64 and 45 (b) 45 and 64
(c) 48 and 60 (d) 60 and 48

06. The centre distance in the second stage is


(a) 90 mm (b) 120 mm
08. For ro1 = 60 rpm clockwise (CW) when
(c) 160 mm (d) 240 mm
looked from the left, what is the angular
velocity of the carrier and its direction so
Common Data for Questions 07 & 08
that Gear 4 rotates in couriter clockwise
A planetary gear train has four gears and one
(CCW) direction at twice the angular
carrier. Angular velocities of the gears are ro1, ro2, velocity of Gear 1 when looked from the
ro3, and ro4 respectively. The carrier rotates with left? (�ATE-ME-06)
angular velocity ros. (GATE-ME-06) (a) 130 rpm, CW (b) 223 rpm, CCW
Gear2 (c) 256 rpm, CW (d) 156 rpm, CCW
45T
09. An epicyclic gear train 1s shown
Gear3

schematically in the figure. The sun gear 2


20T

on the input shaft is a 20 teeth external gear.


The planet gear 3 is a 40 teeth external gear.
The ring gear 5 is a 100 teeth internal gear.
Gear 1 The gear 5 is fixed and the gear 2 is rotating
at 60 rpm CCW (CCW = couriter- clockwise
Carrier 5
15T

and CW = clockwise).
Gear4
40T
The arm 4 attached to the output shaft will
rotate at (GATE-ME-09)
07. What is the relation between the angular
velocities of Gear 1 and Gear 4? (a) 10 rpm CCW
(b) 10 rpm CCW
(c) 12 rpm CCW
(d) 12 rpm CCW
(b) 0)
4
-0)
6 =6
0)1 - 0)5

!11iiQj@UiQY.fi1QRflniM\hijj*)HyderabadJDelhi JBhopal JPune I Bhubaneswar J L.icknowJPatna I Bengalwu I Chennai I VijayawadaJVmg JT,rupati J Kukatpa]]yJ Kolkata I
: 148: Theory of Machines

10. For the epicyclic gear arrangement shown in 12. Gear 2 rotates at 1200 rpm in counter
the figure, co2 = 100 rad/s clockwise ( C W) clockwise direction and engages with Gear
and = 80 rad/s counter clockwise 3. Gear 3 and Gear 4 are mounted on the
( C C W). The angular velocity co5 (in rad/s) same shaft. Gear 5 engages with Gear 4. The
COarm

1s (GATE-ME-10) numbers of teeth on Gears 2, 3, 4 and 5 are


20, 40, 15 and 30, respectively. The angular
N 1 = Number of
teeth for gear i
speed of Gear 5 is (GATE-ME-14)
3 4 N2 = 20
N3=24

lJ
5 N4=32
Ns = 80

(a) 300 rpm counter clockwise


arm Shaft axis

(a) 0 (b) 70 cw (b) 300 rpm clockwise


(c) 140 C C W (d) 140 cw ( c) 4 800 rpm counter clockwise
( d) 4 800 rpm clockwise
11. A compound gear train with gears P,Q,R 13. It is desired to avoid interference in a pair of
and S has number of teeth 20, 40,15 and 20,
spur gears having a 20 ° pressure angle. With
respectively. Gears Q and R are mounted on
increase in pinion to gear speed ratio, the
the same shaft as shown in the figure below.
minimum number of teeth on the pinion
The diameter of the gear Q is twice that of
(a) increases
the gear R. If the module of the gear R is 2
(b) decreases
mm, the center distance in mm between
( c) first increases and then decreases
gears P and S is (GATE-ME-13)
( d) remains unchanged (GATE-ME- 14)
Q
14. A pinion with radius r 1 , and inertia 1 1 is
driving a gear with radius r2 and inertia h
Torque 't 1 is applied on pinion. The
following are free body diagrams of pinion
and gear showing important forces (F I and
(a) 40 (b) 80 (c) 120 (d) 160 F2) of interaction . Which of the following
relations hold true? (GATE -15 -Set 1)

!Mlli@iii44•h/i@biMihfh� yderabadl Delhi I Bhopal I Punel Bhubaneswarl Lucknow! Patna I BengaluruJ ChennaiJVrjayawadaJVizag I Tirupati I KukaJpallyl Kolkala
: 149: Gear & Gear Trains

16. In an epicyclic gear train, shown in the


figure, the outer ring gear is fixed, while the
0 1 , 02 = Angular sun gear rotates counterclockwise at 100
rpm. Let the number of teeth on the sun,
Displacements

Inertia = 11 Inertia = 12 planet and outer gears to be 50, 25 and 100


respectively. The ratio of magnitudes of
angular velocity of the planet gear to the
angular velocity of the carrier arm is __.

Sun gear

(GATE-17-SET-1)
15. In the gear train shown, gear 3 is carried on
arm 5. Gear 3 meshes with gear 2 and gear 17. A gear train shown in the figure consists of
4. The number of teeth on gear 2, 3, and 4 gears P, Q, R and S. Gear Q and gear R are
are 60, 20, and 100, respectively. If gear 2 is mounted on the same shaft. All the gears are
fixed and gear 4 rotates with an angular mounted on parallel shafts and the number
velocity of 100 rpm in the counterclockwise of teeth of P, Q, R and S are 24, 45, 30 and
direction, the angular speed of arm 5 (in 80, respectively. Gear P is rotating at 400
rpm) is (GATE-16-SET-1) rpm. The speed (in rpm) of the gear S is

(a) 166.7 counterclockwise


(b) 166.7 clockwise 80
(c) 62.5 counterclockwise
(GATE-17-SET-2)
(d) 62.5 clockwise
I �
iJIWlll!lll11111tmHDEnrn1:,1f yderabad I Dellii I Bhopal I Pune I Bhubaneswar I Lucknow I Patna I Bengaluru I Chennai I Vtjayawada I Vu.ag I TlfUpati I Kukatpally I Kolkata
: 150: Theory of Machines

02. A planetary gear train is shown in Figure.


Internal gear (1) has 104 teeth and is held
fixed and planet gear (2) has 96 teeth. How
Five Marks Questions

much does the planet gear rotates for sixty


01. Below Fig show a planetary gear train. revolutions of the planet carrier (3) in clock
Gears 2, 4 and 5 have 24, 40 and 144 teeth wise direction ? (GATE-ME-99)
respectively. Gear 5 is fixed. Gear 2 is
rotating clockwise at 700 rpm. What will be
the rpm of the arm and gear 4 ?
(GATE-ME-96)
I
Vi-

!l1•1ii@mi4.jj4Rflnli\ii!jj� yderabad I Dclhi I Bhopal I Pune I Bhubaneswar I Lucknow I Patna I Bengaluru I Chennai I Vijayawada I Vizag I Ttrupati I Kukatpally I Kolkata
SOLUTIONS
03. Ans: (d)
One Mark Solutions
04. Ans: (b)
01. Sol: Gear 1: q,80, a 1 = 30
Sol: Circular pitch (Pc):- It is the distance
Gear 2: q,120, a2 = 22.5
measured along the circumference of the
pitch circle from a point on one tooth to the
ct; = d l cosa 1
corresponding point on the adjacent tooth = 80 x cos30 ° = 40 Jj mm
p = 1td
T
= d2cosa2
C

= 120x cos22.5° = 110.87 mm


Pit��,cir- e---- - - -
NI d'2
Nz d'I
,'

1440 110.87
Diametral Pitch (Pd):- It is the number of N2 40,fj
teeth per unit length of the pitch circle
:. N2 = 900 rpm
diameter in inches.
T
pd= - 05. Ans: (d)
d
Sol: Rack and pinion converts Rotary
:. Product of Pc ( circular pitch)
Motion to translating motion
x Pd (diametral pitch)

06. Ans: (c)


Nz N 3N sN 6 N3N 6
Sol: =
N6 N 2 N 4N s N 2N4
=
02. Ans: (d)
1440 Wheel 5 is the only Idler gear as the number
Sol: Velocity ratio : = 40
36 of teeth on wheel '5' does not appear in the
For 35 to 45 velocity ratios, suitable gear velocity ratio.
train is "worm gear".
\( l l 11t.•;i11tt 1rn� P,1hlll,1t1(11i... Fyderabad I Delhi I Bhopal I Pune I Bhubaneswar I lncknow I Patna I Bengaluru I Chennai I Vrjayawada I Vizag I Trrupati I KukatpaJly I Kolkata I
: 152: Theory of Machines

(Q) Instantaneous centre of velocity -


Two Marks Solutions Linkage
(R) Section modulus -Beam
01. Ans: a- 4, b-3, c-2, d-1 (S) Prime circle- Cam.

�-- - -I
02. Ans: (b) 05. Ans: (a)
Sol:
Sol: {J)c - {J)a = - TB
{J)B {J)a Tc

Given Ts = Tc, IDs = 0,


-

0-4
{J)
c
IDa = 4 rad/sec
-4 = -1 � roe = 8 rad/sec w
Z1 = 16 , Z3 = 15 , Z2 = ? , Z4 = ?
03. Ans: (b) First stage gear ratio, G1 = 4 ,
Sol: Second stage gear ratio, G2 = 3 ,
m12 = 3, m34 = 4
Z2 = 16 x 4 = 64
Ann Z4 = 15 X 3 = 45

06. Ans: (b)


Ts = 20 , Tp = 30 , TR = 80, Sol: Centre distance
Ns = 100 rpm CW, NR = O = m12 x (z + z2 ) = m 34 x (z + z )
Na = ?. 2 2
l 3 4

N s -N a -TR 4
= -x(15+45)=120mm
--
N R -N a Ts 2
=

100-N a --
- -80 07. Ans: (a)
0-N a 20
Sol: By Analytical Approach
� Na = 20 rpm CW
ro, -ro 5 -T2 -T4 =-X-
45 40
Tl T3 15 20
---=--X--
04. Ans: (b)
©4 - ID5

Sol: (P) Addendum - Gear

�ydcrabad I Delhi I Bhopal I Punc I Bhubaneswarl Lucknow I Patna I Bcngaluru I Chcnnai IVuayawada jVu.ag I Tirupati I Kukalpallyl Kolkala I
��-��F.ngineeriogPnbliatious
ACE : : 153 Gear & Gear Trains
: �================================= =
08. Ans: (d)
#

60 - N4
= 100
Sol: Data given: O - N4 20
---

ro1= 60 rpm ( C W) � N4 = IO C C W
©4 = - 120 rpm
[2 times speed of gear - 1] 10. Ans: (c)
We have Sol: ro2 = 100 rad/sec,
:. (J)l -(J)5=6 ©arm= 80 rad/s ( C C W)
©4 - © 5 T4
-Tz -
-
{J)5 -
-=--X
60 - © . l"fy"
- ma

�----5 = 6 , s1mp 1 mg
- 120 - © 5
0)2 - {J)a T3 Ts
© 5 - (-80) -
- 20 32 1
----= -X -=- -
60 - ros = -720 -6ros 100- (-80) 24 80 3
ros = - 156 rpm � (J)5 = - 140 ccw
ro5 =156 rpm. C C W
11. Ans: (b)
09. Ans: (a) Sol: Given Tp = 20, T0 = 40, TR = 15, Ts = 20
Sol: An epicyclic gear train 1s shown Dia of Q = 2 x Dia of R
schematically the gear 5 is fixed and gear 2 m0.T0 = 2mR.TR
is rotating 60 rpm ( C C W) Then the arm 4 Given, module of R = mR = 2mm
attached to the output shaft will rotate at? T 15
� IDQ = 2 mR; ___B...=2 x 2 x-=I.5 mm
TQ 40
T5X-
Nz - N4 - T3
---= . mp = mo=2mm; ms = mR = 1.5 mm
N5 - N4 T3 Tz
No.of teeth
Nz - N4 T5 Rad.ms = modu1e x----
--- =
N5 - N4 Tz
C entre distance between P and S is given by
T2 = 20
R p +R Q +R R +R T
T3=40
T T T T
T s = 100 =m ____f_+m Q _g_+m R �+m 8 s
2 2 2
p
N2 = 60 C C W (+ro)
2
Ns = O 40 20 15 20
=1.5[ ; ]+2[ � ]
N4 = ?
=45 + 35= 80 mm
Nz - N4 - T5
-- -=

!M•IMO§hiiiihliRftaiM\hlh*)ayderabadi Delhi I Bhopal! Punc I Bhubaneswarl Lucknow! Patna! Bengalwui Chennai !VuayawadalVizag !TllUpari I Kukatpallyj Kolkata I
�-t . A;CE .. : 154: Theory of Machines
� �============================================
':i .:EngineenngPw,liramns

12. Ans: (a) 't1 = 11 81 + F1 r1


N 5 -T2 -T4 = 20 15 = 1 Put F1 value in above equation
Sol: - = - -x -- -x - -
N2 T3 T5 40 30 4
_ N _ 1200 _
Ns - 2 - - 300 rpm m the same
4 4
direction as that of gear 2 i.e, CCW

13. Ans: (a)


15. Ans: (c)
14. Ans: (b) Sol: Given T2= 60
Sol: Velocity at point of contact T4= 100 N4= 1OOrpm (ccw +ve )
= co1r1= co2r2 .... ...(1) Relative velocity equation
Considering pinion and gear as a system net N 4 -N a T2 100-N a -60
N 2 -N a
---= =>--�=
force is zero on system so, from Newton's T4 0-Na 100
third law, F 1= F2 1. 6 Na= 100
Rewrite equation (1) 100
N a = -= 62.5 rpm (ccw)
1.6
rJ ) 1 = r/} 2 ........(2)

Differentiating above equation with respect


16. Ans: 3
to Time 't'.
Sol: NR= 0
rl e l = r2 0 2 .....(3) Ns= 100 rpm (ccw)
Consider gear as a system
NP=?.
F2 x r2 = I202 Na
r18 .. N S -N a __TR
F2 -
-� X
-
- I 2 �0
r2 r2 r22
1
N R -N a Ts
Consider Pinion as a system 100-N a = 100
-
Net torque on pinion ('text) = F 1 x r1 - 't1 0-N a 50
100
N a = +-rpm(ccw)
From Newton's second law 3

(take a anticlockwise as positive)

!lfllQj/§jjj§4@4Rft§iM\Uijj+yderabadl Delhi I Bhopal I Pune I Bhubaneswarl Lucknow! Patna I Benga]uru I ChennailVijayawadalVw.ag ITirupati I Kukat:pallyl Kolkata I
100-N a = _2 5 N2
Ns = 0 , N a = - = l OO rpm(CW)
N P -N a 50 7
Np = 3 Na - 200 From eq.(2)
700 -100 -40
:.
100 --- =
3 x --200 =-l OO rpm
3 N 4 -100 24
=

= 100 rpm (cw) => N4 = 460 rpm (CW)


Np =
3
Na 02.
Sol: T1 = 104, N1 = 0
17. Ans: 120 T2 = 96 Na = 60 N2 = ?
Sol: Np = 400 rpm , Ns= ?.
N P = Ts
N S Tp
400 80
NS 24
:. Ns = 120 rpm

N1 -N a T2 96
Five Marks Solutions

N 2 -60 _ 104
01. 0-60 96
Sol: Given T2 = 24, T4 = 40 & Ts = 144
104 -60 x 8
Ns = 0, N2 700rpm, N4 ? N2 = 60 [1- ] = -5 rpm
96 96
=
= =

N 4 -N a - T 144
----'--------"- = 5 = - . . . . . . . . . . . . . (1) 5 rpm in CCW
N 5 -N a T4 40
=

N 2 - N a _ _ T4 __ 40
-- - - ............ (2)
N 4 -N a T2 24
Solve the Equation ( 1) and (2) for N4 & Na
N 2 -Na =-144 =
-6
N 5 -N a 24

!IJ11i@Oii4.jjQjjflfiiMiiifo+yderabad I Delhi I Bhopal I Pune I Bhubaneswar I Lucknow I Patna I Bengaluru I Chennai jVtjayawada I Vmg jTllUpati I Kukatpally I Kolkala I
C4 ============================::::!J
Flywheels
(a) 395 (b) 790
One Mark Questions (c) 1580 (d) 3 160

0 1. Which of the following statement is correct? 03. C onsider a slider crank mechanism with
(GATE-ME-01) nonzero masses and inertia. A constant

(a) Flywheel reduces speed fluctuations torque 't is applied on the crank as shown in

during a cycle for a constant load, but the figure. Which of the following plots best
flywheel does not control the mean resembles variation of crank angle, 0 versus
speed of the engine if the load changes. time
(b) Flywheel does not reduce speed
fluctuations during a cycle for a
constant load, but flywheel does control
the mean speed of the engine if the load
changes
(c) Governor control a speed fluctuations (GATE-15 -Set 1)
during a cycle for a constant load, but (b)
(a)
governor does not control the mean
speed of the engine if the load changes. e e
(d) Governor controls speed fluctuations
during a cycle for a constant load, and time
governor also controls the mean speed
of the engine if the load changes.

(c) (d)
02. A circular solid disc of uniform thickness
20mm, radius 200mm and mass 20kg, is e e
used as a fly wheel. If it rotates at 600 rpm,
time
the kinetic energy of the flywheel, in joules
1s (GATE-ME-12)

liti1ijj§jjji4.jjiAflD1Mi@j.� yderabad j Delhi J Bhopa!J Punc I BhubaneswarJ Lucknow! Patna! Bengaluru I Chennai jVtjayawada jVizag IT=pari I Kukatpallyj Kolkat,
: 157 : Fly Wheels

in kinetic energy is found to be 400 Nm.


Two Marks Questions The inertia of the flywheel in kgm2 is
(GATE-ME-07)
01. A flywheel of moment of inertia 9.8 kg m 2
(a) 0. 1 0 (b) 0.20 (c) 0.30 (d) 0.40
fluctuates by 30 rpm for a fluctuation in
energy of 1 936 Jouls. The mean speed of 05. A flywheel connected to a punching
the flywheel is (in rpm) (GATE-ME-98) machine has to supply energy of 400 Nm
(a) 600 (b) 900 (c) 968 (d) 29470 while running at a mean angular speed of 20
rad/s. If the total fluctuation of speed is not
02. For a certain engine having an average to exceed ±2%, the mass moment of inertia
speed of 1 200 rpm, a flywheel approximated of the flywheel in kg-m2 is
as a solid disc, is required for keeping the (GATE-ME-13)
fluctuation of speed within 2% about the (a) 25 (b) 50 (c) 1 00 (d) 1 25
average speed. The fluctuation of kinetic
energy per cycle is found to be 2kJ. What is 06. Consider a flywheel whose mass M is
the least possible mass of the flywheel if its distributed almost equally between a heavy,
diameter is not to exceed l m? ring-like rim of radius R and a concentric
(GATE-ME-03) disk-like feature of radius R/2. Other parts
(a) 40 kg (b) 51 kg of the flywheel, such as spokes, etc, have
(c) 62 kg (d) 73 kg negligible mass. The best approximation for
a, if the moment of inertia of the fly wheel
03. If Cf is the coefficient of speed fluctuation of about its axis of rotation is expressed as
a flywheel then the ratio of ffimax/ffimin will be (GATE-ME-14)
(GATE-ME-06)
1-2Cr 2-Cr 07. Maximum fluctuation of kinetic energy in an
(a) (b)
1 + 2Cr 2 + Cr engine has been calculated to be 2600 J.
1-2Cr 2 + Cr Assuming that the engine runs at an average
(c) (d)
1- 2Cr 2-Cr speed of 200 rpm, the polar mass moment of
inertia (in kg.m2) of a flywheel to keep the
04. The speed of an engine varies from 2 10
(GATE-ME-14)
speed fluctuation within ±0.5% of the
rad/s to 1 90 rad/s. During cycle the change average speed is __

ACE Engineering Publications �Hyderabad I Delhi I Bhopal I Pune I Bhubaneswar I Bengaluru I Chennai I Vijayawada I Visakhapatnam I Tirupati I
: 158 : Theory of Machines

08. Torque and angular speed data over one 09. The torque (in N-m) exerted on the crank
cycle for a shaft carrying a flywheel are shaft of a two stroke engine can be describeq
shown in the figures. The moment of inertia as T=l OOOO+ 1 000sin20-1 200 cos20, where
(in kg.m2) of the flywheel is __ 0 is the crank angle as measured from inner
(GATE-ME-14) dead center position. Assuming the resisting
torque to be constant, the power (in kW)
Tor ue

3000 N-m .......... . developed by the engine at 1 00 rpm is


(GATE -15 _;Set 3)
3n/2 2n
..,,__......,_--+---�--e
7t
0 n/2
-1 500 N-
_____.
....................... ....__ 10. An engme, connected with a flywheel, is
designed to operate at an average angular
speed of 800 rpm. During operation of the
Angular speed

engine, the maximum change in kinetic


energy in a cycle is found to be 6240 J. In
1 0 rad/s order to keep the fluctuation of the angular
speed within ± 1 % of its average value, the
moment of inertia (in kg-m2) of the flywheel
should be _____. (GATE - PI-16)

!lfllih§OiiilhiMRflbjfW111h+yderabad I Delhi I Bhopal I Pune I BhubaneswarJ LJ.Jcknowl Patna I Bengaluru I Chennai I Vrjayawada IVu.ag I Tuupati I Kukatpally I Kolkata I
SOLUTIONS

One Mark Solutions 0 1 Parabola

01. Ans: (a)


because of inertia

Sol: Flywheel reduces speed fluctuations during


a cycle for constant load, but flywheel does
not control the mean speed of the engine if
t

the load changes.


Two Marks Solutions
02. Ans: (b)
01. Ans: (a)
Sol: E � ½ /w 2 ½ ( m;')w)'
Sol: Max fluctuation of K.E

� �G x 20 x 0.2 ' )(2 x


:�
600 '
)
�E = max K.E-min K.E

= 790 N-m

03. Ans: (d)


d20
Sol: I- = T + f(sin 0, cos0)
dt 2
(ro is mean angular velocity)
Where 'T' is applied torque, f is inertia torque
(ro 1 -ro 2 )
which is function of sin0 & cos0 = l ro 2 = l ro 2 Cs
dS T
= t + f'(sin 0,cos 0)+ c 1
0)

Cs = coefficient of fluctuation of speed


dt I
�E = I.ro . (ro, - ro2)
T 2
0= t + c 1 t + f"(sin 0, cos0)
21tN 1 21tN 2 )
I = I 21tN (
_

0 is fluctuating on parabola 60 60 60
and @ t = 0 , 0 = 0 , =-- x l x N x (N 1 - N i }
2
41t

e(slope) = 0 (because it starts from rest) 3600

!ltl1ih§jjji4.jjiRftbiM\d.jj� yderabad I Delhi I Bhopal I Pune I Bhubaneswar I Lucknow! Patna I Bengaluru I Chennai I Vuayawada IVizag I T,rupari I Kukatpally I Kolkata
� :».aw,qNiliratvn
,l.
.,
, . �....\it •
ACE • • •
: 160 : Theory of Machines

Given, �E = 1936 N - m 05. Ans: (a)


(N1 - N2) = 30 rpm Sol: Given �E = 400 N - m
1 = 9.8 Kg - m2 ro = 20 rad/sec
4x n 2 Cs = 0.04
:.1936 = -- x 9.8 x N x 30
3600 We know �E = l ro C s 2

:. N ::: 600 rpm


�E
� l = -- = 25kg - m 2
ro C s
2

02. Ans: (b)


06.
Sol: Cs = 0.02
m
�E = IciCs Sol: The flywheel is considered as two parts
2
21t x1200 m
ro = --- = 401t = 125.6 rad / sec as rim type with Radius R and as disk
60 2
l = _!_ mr = _!_ x m x 0.5 = 0.125 m R
type with Radius
2 2

2 2 2
� 2000 = 0.125m (125.6)2 0.02 X X

� m = 51 kg.
mR mR
= _2_ mR
2

0.5625 mR2
2

03. Ans: (d) I= +


2

16
=
2 16
SoI: Cr = -----
(romax - O)min ) O)max + O)min
'· ro = ---- : . a = 0.5625
ro 2

Cr _
2(
- O)max - O)min ) 07. Ans: 592.73 kgm2
romaX + (Omin
Sol: �E = 2600 J, N = 200 rpm
400 x 1t
� ro = --- rad / sec
60
Cs = 0.01 ( ± 0.5% = 1%)
04. Ans: (a) �E = lro 2 C s
1 1
Sol: �E = - Joi - - Joi 2600 X 60 2
2 I 2 2 �I=� =
ro 2 C s (400 X 1t)2 X 0.01
�E = _!_ I(rof - ro� ) 592.73 kgm2
2
=

�E = 400 Nm, ro 1 = 210 rad/sec,


ro 2 = 190 rad/sec � I = 0.1 kgm2
\( I l 1 1�111l l 1 111� P11l ilh .1!11111-. �yderabad! Delhil Bhopal i Pune l Bhubaneswari Lucknowl Patna l Bengaiwu ! Chennai l Vuayawada l V17.3g ITirupati I Kukatpally i Kolkata I
t.t
""· .," �ACE . . . . : 161 : Fly Wheels
., PIJblii:amna

08. Ans: 31.42 21tNT


Power = --
Sol: From the T - 0 diagram energy is 60
7t 2 x 1t x l OO x l OOOO =
Stored into flywheel during to 7t = 104719.75 W
2 60
7t
P = 104.719 kW
�E = - x 3000 N - m
2
10. Ans: (44.45)
[ ·. · mean torque is zero]
Sol: Given mean speed N = 800 rpm
From ro-0 diagram romax = 20 rad/sec
21tN
romin = 10 rad/sec ro = -- rad / sec
60
1 2 2 2�E
�E = - I((t)max - (t)min ) � I = Fluctuation of energy �E = 6240 J
2 2 2
Fluctuation of speed = ± 1%
(t)max - (t)min

7t
2 x - x 3000 Moment of inertia I = ?
I= � = 31.42 kgm 2 We know
20 -10 2

�E = I ro2Cs
09. Ans: 104.32
:J
�E 6240
I= = = 44.45 kg-m2
Sol: N = 100 rpm ro'C, 2
( x 0.02

7t 0
Tmean = -
1 1" Td0
r"
= .!_ o (10000 + 1000 sin 20 -1200 cos 20)de
7t J

= .!. [100000 - 500cos 20 - 600sin 20 ]�


7t

= l OOOO Nm

\( I I 11g11H l l l l l!.!, P11hl u .il11111, �yderabad I Delhi I Bhopal I Pune I Bhubaneswar I wcknow I Patna I Bengaluru I Chennai I Vijayawada I Vizag I Tirupati I Kukatpally I Kolkata I
cs Mechanical Vibrations
03. There are four samples P, Q, R and S with
One Mark Questions natural frequencies 64, 96, 128 and 256 Hz,
respectively. They are mounted on test
01. In the figure shown, the spring deflects by 8 setups for conducting vibration experiments.
to position A (the equilibrium position) If a loud pure note of frequency 144Hz is
when a mass m is kept on it. During free produced by some instrument, which of the
vibration, the mass is at position B at some samples will show the most perceptible
instant. The change in potential energy of induced vibration? (GATE-ME-OS)
the spring mass system from position A to (a) P (b) Q (c) R (d) S
position B is (GATE-ME-01)
04. The differential equation governing the
vibrating system is (GATE-ME-06)

m
C

(a) _.!. kx2 (b) - kx2 - mgx


1
2 2 (a) mx + cx + k (x - y)= o

(c) _.!. k(x+8)2 (d) _.!. kx2 + mgx (b) m(x - y) + c(x - y) + kx=O
2 2
(c) mx + c(x - y) + kx= o
(d) m(x - y) + c(x - y)k(x - y)=O
02. A vibrating machine is isolated from the
floor using springs. If the ratio of excitation
05. For an under damped harmonic oscillator,
frequency of vibration of machine to the
resonance (GATE-ME-07)
(a) occurs when excitation frequency is
equal to 0.5, the transmissibility ratio of
natural frequency of the isolation system is

greater than undamped natural frequency.


isolation is (GATE-ME-04)
(b) occurs when excitation frequency is less
1 4 than undamped natural frequency
(a) - Cb) I4 (c) - (d) 2
2 3

IM••i!@jjj44ijjijAflnlM\@j+yderabadj Delhij Bhopal j Punc l Bhubaneswar l LucknowjPatnaj Bcnga)uruj Chcnnai lVtjayawadajVmg !Tuupati I Kukatpal)yj Kolkata I
: 163 : Mechanical Vibrations

(c) occurs when excitation frequency is equal (b) smallest amount of damping for which
to undamped natural frequency no oscillation occurs in free vibration
(d) never occurs (c) largest amount of damping for which
the motion is simple harmonic in free
06. The rotor shaft of a large electric motor vibration
supported between short bearings at both the (d) smallest amount of damping for which
ends shows a deflection of 1.8 mm in the the motion is simple harmonic in free
middle of the rotor. Assuming the rotor to vibration
be perfectly balanced and supported at knife
edges at both the ends, the likely critical 10. In vibration isolation, which one of the
speed (in rpm) of the shaft is following statements 1s NOT correct
(GATE-ME-09) regarding Transmissibility (T)?
(GATE-ME-14)
(a) 350 (b) 705 (c) 2810 (d) 4430
(a) T is nearly unity at small excitation
frequencies
07. The natural frequency of a spring mass
(b) T can be always reduced by using higher
system on earth is co0 • The natural frequency
damping at any excitation frequency
of this system on the moon (gmoon = gearth/6)
(c) T is unity at the frequency ratio of Ji
(GATE-ME-10)
(d) T is infinity at resonance for undamped
IS

(a) COn (b) 0.408co0


systems
(c) 0.204co0 (d) 0. 167co0
1 1. Consider a single degree-of-freedom system
08. If two nodes are observed at a frequency of with viscous damping excited by a harmonic
1800 rpm during whirling of a simply force. At resonance, the phase angle (in
supported long slender rotating shaft, the degree) of the displacement with respect to
first critical speed of the shaft in rpm is the exciting force is (GATE-ME-14)
(GATE-ME-13)
(a) 0 (b) 45 (c) 90 (d) 135
(a) 200 (b) 450
(c) 600 (d) 900 12. A point mass is executing simple harmonic
motion with an amplitude of 10 mm and
09. Critical damping is the (GATE-ME-14) frequency of 4 Hz. The maximum
(a) largest amount of damping for which acceleration (m/s2) of the mass is ____
no oscillation occurs in free vibration (GATE-ME-14)

!li11Qjj§jjj4i.jji4jmnlM\jjjjj� yderabad I Delhi I Bhopal I Pune I Bhubaneswar I Lucknow I Patna I Bengaluru I Chennai I Vijayawada I Vuag J TIIUpati I Kuhtpally I Kolkata
Pubticdioos
. . . Theory of Machines
� �:�
�.�-�
"" .,, . �CE : 164:
================================== =
13. In a spring-mass system the mass is m and 15. A single degree of freedom spring mass
the spring constant is k. The critical system with viscous damping has a spring
damping coefficient of the system is 0. 1 constant of 10 kN/m. The system is excited
kg/s. In another spring mass system, the by a sinusoidal force of amplitude 100 N. If
mass is 2m and the spring constant is 8k. the damping factor (ratio) is 0.25, the
The critical damping coefficient (in kg/s) of amplitude of steady state oscillation at
this system is ___ (GATE -15 -Set 2) resonance 1s ____mm
(GATE - 16 - SET - 1)
14 . Which of the following statements are
TRUE for damped vibrations? 16. The static deflection of a spring under
P. For a system having critical damping, gravity, when a mass of 1kg is suspended
from it, is 1 mm. Assume the acceleration
the value of damping ratio is unity and
due to gravity g = 10 m/s2 • The natural
system does not undergo a vibratory
frequency of this spring-mass system (in
motion.
rad/s) is----
Q. Logarithmic decrement method is used
(GATE - 16 - SET - 3)
to determine the amount of damping in a
physical system. 1 7. The damping ratio for a viscously damped
R. In case of damping due to dry friction spnng mass system, governed by the
between moving surfaces resisting force d2x dx
relationship m- 2 + c- + kx=F(t) ,
of constant magnitude acts opposite to dt dt
IS

the relative motion. given by


(GATE - 17 - SET - 1)
S. For the case of viscous damping, drag
force is directly proportional to the (a) (b)
ff 2�
square of relative velocity.
(GATE -15 -Set 3) (c) - (d) �
2mk
C C

(a) P and Q only


(b) P and S only


18. A mass m is attached to two identical
(c) P,Q and R only springs having spring constant k as shown in
( d) Q and Sonly figure. The natural frequency ro of this single
degree of freedom system is

jlJllih§Oiiiiihi@biM\Uijj� yderabad I Delhi I Bhopal I Pune I Bhubaneswar I Lucknow J Patna I Bengaluru I Chennai IVtjayawada I Vizag I TIIUpati I Kukatpally I Kolkata
: 165 : Mechanical Vibrations

02. Consider the system of two wagons shown


in Figure. The natural frequencies of this
system are (GATE-ME-99)

(GATE - 17 - SET- 2)
(a) O, f! (b)
JE, f!
(a) f;f (b)
ff (c)
vfk,;; v/k
� (d) 0,
v/k

(c) nz­
V� (d) l 03. As shown in Figure, a mass of 100 kg is
held between two springs. The natural
frequency of vibration of the system in
cyclesls, is
Two Marks Questions

01. A mass of 1 kg is suspended by means of 3


springs as shown in Figure. The spring
constants K 1 , K2 and K3, are respectively
l kN I m, 3kN I m and 2 kNlm. The natural
(GATE-ME-2000)
1
frequency of the system is approximately)
(a) -
10 20
(GATE-ME-96) 2n
(b) �
7t
(c)
7t
(d)
7t

(a) 46.90 rad/sec 04. The assembly shown in the figure is


(b) 52.44 rad/sec composed of two massless rods of length /
(c) 60.55 rad/sec with two particles, each of mass m. The
(d) 77.46 rad/sec m= lkg natural frequency of this assembly for small
oscillations is (GATE-ME-01)

jlHIMI/Yhi4UI04jjnni@jjih� yderabad I Delhi I Bhopal I Pune I Bhubaneswar I wcknow! Patna I Bengaluru I Chennai I Vliayawada I Vizag I T1111pati I Kukatpally I Kolkata
: 166 : Theory of Machines

Common Data for Questions 06 & 07

A uniform rigid slender bar of mass 10 kg.


hinged at the left end is suspended with the help
IS

of spring and damper arrangement as shown in


the figure where K = 2kN/m, C = 500Ns/m and
the stiffness of the torsional spring Ke is
l kN/m/rad. Ignore the hinge dimensions.
(a) �g / f (b) �2g /(f cos a) (GATE-ME-03)

(c) �g /(R cos a) (d) �(g cos a) / f

05. A flexible rotor-shaft system comprises of a


10kg rotor disc placed in the middle of a Ko @'1------- --�--....
Bar

massless shaft of diameter 30 mm and 400mm

length 500mm between bearings (shaft is 500mm

being taken mass-less as the equivalent mass


06. The un-damped natural frequency of
of the shaft is included in the rotor mass)
oscillations of the bar about the hinge point
mounted at the ends. The bearings are
assumed to simulate simply supported
(a) 42.43 rad/s (b) 30 rad/s
IS

boundary conditions. The shaft is made of


(c) 17.32 rad/s (d) 14.14 rad/s
steel for which the value of E is 2.1 x 10 11

Pa. What is the critical speed of rotation of 07. The damping coefficient in the vibration
the shaft? (GATE-ME-03) equation is given by

(a) 60 Hz (b) 90Hz (a) 500 Nms/rad (b) 500 N/(m/s)


(c) 135 Hz (d) 180Hz (c) 80 Nms/rad (d) 80N/(m/s)

08. A uniform stiff rod of length 300 mm and


having a weight of 300 N is pivoted at one
end and connected to a spring at the other
end. For keeping the rod vertical in a stable
!l!lliii4hiiiiii4RflbiMihill;i yderabad I Delhi I Bhopal I Pune I Bhubaneswar I Lucknow I Patna I Bengalwu I Chennai I Vliayawada IVizag I Tuupati I Kukatpally I Kolkata
: 167 : Mechanical Vibrations

the minimum value of spring constant K (a) 1.2 N.slm (b) 3.4 N.slm
needed is (GATE-ME-04) (c) 8.7 N.slm (d) 12.0N.slm

11. A machine of 250 kg mass is supported on


1 50 mm springs of total stiffness 100 kNlm. Machine
w has an unbalanced rotating force of 350 N at
l-
1 50 mm
speed of 3600 rpm. Assuming a damping
factor of O.15, the value of transmissibility
i
ratio is (GATE-ME-06)
(a) 300Nlm (b) 400 Nim (a) 0.0531 (b) 0.9922
(c) 500 Nim (d) 1000 Nim (c) 0.0162 (d) 0.0028

09. A mass M, of 20 Kg is attached to the free Statement for Linked Answer Q 12 & 13
end of a steel cantilever beam of length 1000 A vibratory system consists of a mass 12.5 kg, a
mm having a cross-section of 25x 25 mm. spring of stiffness 1000 Nim, and a dashpot with
Assume the mass of the cantilever to be damping coefficient of 15 Nslm
negligible and Esteel = 200 GPa. If the lateral
vibration of this system is critically damped 12. The value of critical damping of the system
using a viscous damper, the damping 1s: (GATE-ME-06)
constant of the damper is (a) 0.223 Nslm (b) 17.88 Nslm
(GATE-ME-04) (c) 71.4 Nslm (d) 223.6Nslm

13. The value of logarithmic decrement is


(GATE-ME-06)
(a) 1250 Nslm (b) 625 Nslm (a) 1.35 (b) 0.42
(c) 312.50 Nslm (d) 156.25 Nslm (c) 0.68 (d) 0.66

10. In a spring-mass system, the mass is 0.1kg 14. The equation of motion of a harmonic
and the stiffness of the spring is l kNlm, By oscillator is given by
introducing a damper, the frequency of d2x dx. + co 2 x = O and the initial
+ 2,;con -
oscillation is found to be 90% of the original dt dt
-2 n

value. What is the damping coefficient of


(GATE-ME-OS)
conditions at t = 0 are x(O) = X, dx, (0) = 0.
the damper? dt
!Mllii@hiiiih!QRflbiM\h!h*)Hyderabad I Dellti I Bhopal I Pune I Bhubaneswar I Lucknow I Patna I Bengalwu I Chennai IVuayawadal V11.3g ITirupati I Kuk.atpally I Kolkata I
..,·•·" . ACE
. . . Theory of Machines
..
� �� �
: 168 :

The amplitude of x(t) after n complete spnng constant k = 300N/m. The natural
cycles is (GATE-ME-07) frequency ron in rad/s is (GATE-ME-08)
(a) 10 (b) 20 (c) 30 (d) 40
(a) Xe-,.,[<,',, ] (b) x/{r� J
li;) 18. An automotive engine weighting 240 kg is
(c) Xe (d) X supported on fo ur spnngs with linear
-2nn(

characteristics. Each of the front two


15. The natural frequency of the system shown springs have a stiffness of 16 MN/m while
below is (GATE-ME-07) the stiffness of each rear spring is 32 MN/m.
The engme speed (in rpm), at which
resonance is likely to occur, is
k/2

(GATE-ME-09)
k

k/2 (a) 6040 (b) 3020


(c) 1424 (d) 955

(a) � v2m (b) ff 19. A vehicle suspension system consists of a


spring and a damper. The stiffness of the
(c) f§
(d) f§- spring is 3.6 kN/m and the damping constant
of the damper is 400 Ns/m. If the mass is 50
16. The natural frequency of the spring mass kg, then the damping factor (�) and damped
system shown in the figure is closest to natural frequency (fn) respectively are
(GATE-ME-08) (GATE-ME-09)
m= l .4kg (a) 0.47 and 1. 19 Hz
(b) 0.47 1 and 7.48 Hz
(c) 0.666 and 1.35 Hz
(d) 0.666 and 8.50 Hz
��= K2 1 600 N/m

(a) 8Hz (b) l OHz


(c) 12Hz (d) 14Hz 20. A mass m attached to a spring is subjected
to a harmonic fo rce as shown in figure. The
17. A unifo rm rigid rod of mass m = 1kg and amplitude of the forced motion is observed
length L = 1 m is hinged at its centre & to be 50 mm. The value of m (in kg) is
laterally supported at one end by a spring of (GATE-ME-10)

\( I I 11g11 tt < 1 111g P11hlu .1t101h yderahad I Delhi I Bhopal I Pune I Bhubaneswar I Lucknow I Patna I Bengaluru I Chennai I Vijayawada I Vi.zag I Tirupari I Kukatpally I Kolkata
: 1 69 : Mechanical Vibrations

23. A concentrated mass m is attached at the


centre of a rod of length 2L as shown in the
F(t) =100 figure. The rod is kept in a horizontal
equilibrium position by a spring of stiffness
cos( l OOt)N

k. For very small amplitude of vibration,


(a) 0. 1 (b) 1 .0 (c) 0.3 (d) 0.5 neglecting the weights of the rod and spring,
the un damped natural frequency of the
2 1 . A disc of mass m is attached to a spring of system is (GATE-ME-12)
stiffness k as shown in the figure. The disc
rolls without slipping on a horizontal
surface. The natural frequency of vibration
of the system is (GATE-ME-11)

(a) (b) �
frn

l
(a) - - (b)
_1 �2 K
(c)
v�
/K (d)
f!
2 1t frn
m 2 7t m
24. A single degree of freedom system having
1 1 K mass 1 kg and stiffness 10 kN/m initially at
(c) _ {2K (d) __ r
2 n �� 2 n 2m rest is subjected to an impulse force of
22. A mass of 1 kg is attached to two identical magnitude 5 kN for 1 0-4 seconds. The
springs each with stiffness k=20kN/m as amplitude in mm of the resulting free
shown in the figure. Under frictionless vibration is (GATE-ME-13)
condition, the natural frequency of the (a) 0.5 (b) 1 .0 (c) 5.0 (d) 1 0.0
system in Hz is close to (GATE-ME-11)
25. Consider a cantilever beam, having
K negligible mass and uniform flexural
rigidity, with length O.O l m. The frequency
1 kg
of vibration of the beam, with a 0.5 kg mass
K attached at the free tip, is 1 OOHz. The
flexural rigidity (in N.m2) of the beam is
(a) 32 (b) 23 (c) 16 (d) 1 (GATE-ME-14)
!IJ11ijj§jjj44ijjj4Rflbftijj.jj� yderabad I Delhi I Bhopal I Pune I Bhubancswar l Lucknow I Patna I Bcngaluru I Chennai ! Vuayawada I Viz.ag I Tirupati I Kukatpally I Kolkata
�.:-�.4:FJJgiocelingPuNirafims
ACE : 1 70:
-:.=====================================
Theory of Machines
====

26. A rigid uniform rod AB of length L and K + K2 K + K2


(a) � 1 (b) � I
mass m is hinged at C such that AC = L/3, 2m 4m
C B = 2L/3. Ends A and B are supported by
springs of spring constant k. The natural (d) �K1 : 2
K

frequency of the system is given by


(GATE-ME-14) 28. The damping ratio of a single degree of
freedom spring-mass-damper system with
mass of 1 kg, stiffness 1OON/m and viscous
damping coefficient of 25 N.s/m is ___
I\. B (GATE-ME-14)
l+- L/3 2L/3 •I
29. A mass-spring-dashpot system with mass
m= 10 kg, spring constant K = 6250 Nim is
(a) ff; 2m (b) l
excited by a harmonic excitation of
10cos(25t) N. At the steady state, the
(c) � (d) �
vibration amplitude of the mass is 40mm.
The damping coefficient ( C , in N.s/m) of the
27. What is the natural frequency of the spring dashpot is ___ (GATE-ME-14)
mass system shown below? The contact
between the block and the inclined plane is
F= I Ocos (25t)

frictionless. The mass of the block is


m

denoted by m and the spring constants are


denoted by k1 and k2 as shown below.
(GATE-ME-14)

30. A single degree of freedom system has a


mass of 2kg, stiffness 8 N/m and viscous
damping ratio 0.02. The dynamic
magnification factor at an excitation
frequency of 1.5 rad/s is __
(GATE-ME-14)

!1H1ih§jjji4.jjijRflfiiM\djjj+yderabad I Delhi I Bhopal I Pune I BhubaneswarI Lucknow I Patna ! Beng;iluru I Chennai I Vgayawadal Vu.ag I T'1111pati I Kukatpa))y I Kolkala I
: 171 : Mechanical Vibrations

31. Considering massless rigid rod and small mobile will get just lifted off the ground at
oscillations, the natural frequency (in rad/s) the end Q is approximately
of vibration of the system shown in the (GATE -15 -Set 1)
figure is (a) 3000 (b) 3500
(c) 4000 (d) 4500
k = 400 N/m
33. A precision instrument package (m= l kg)
m = 1 kg

needs to be mounted on a surface vibrating


at 60 Hz. It is desired that only 5% of the
2r

(GATE -15-Set 1) base surface vibration amplitude be


4 0 4 0 transmitted to the instrument .Assume that
(a) J � (b) J �
the isolation is designed with its natural
frequency significantly lesser than 60Hz, so
4 0 4 0
(c} J � (d) J � that the effect of damping may be ignored.
The stiffness (in N/m) of the required
32. A mobile phone has a small motor with an mounting pad is ____
eccentric mass used for vibrator mode. The (GATE -15 -Set 1)
location of the eccentric mass on motor with
respect to center of gravity (C G) of the 34. A single degree freedom spring mass is
mobile and the rest of the dimensions of the subjected to a sinusoidal force of 10 N
mobile phone are shown. The mobile is kept amplitude and frequency ro along the axis of
on a flat horizontal surface. the spring. The stiffness of the spring is
150N/m, damping factor is 0.2 and
undamped natural frequency 1s 10 ro. At
steady state, the amplitude of vibration
1 0c m

(in m) is approximately
(GATE -15 -Set 2)
6cm + 3cm •I
(a) 0.05 (b) 0.07
Given in addition that the eccentric mass = 2 (c) 0.70 (d) 0.90
grams, eccentricity = 2. 19 mm, mass of the
mobile = 90grams , g = 9.81 m/s2 .Uniform
speed of the motor in RPM for which the

!l1•1i@jjji4ijjQRftliM\jjfjj.yderabad I Delhi I Bhopal I Pune I Bhubaneswar I Luclmow I PatnaI Bengaluru I Chennai I VtjayawadaI Vizag I Tuupati I Kukatpally I Kolkata I
: 172 : Theory of Machines

35. Figure shows a single degree of freedom 37. A single degree of freedom spring-mass
system. The system consists of a mass less system is subjected to a harmonic force of
rigid bar OP hinged at O and a mass m at constant amplitude. For an excitation
end P. The natural frequency of vibration of
frequency of �' the ratio of the amplitude
the system is
of steady state response to the static
deflection of the spring is ____

m
�I
(GATE-15-Set 3)
a

(GATE - 16 - SET - 3)
a

1
(a) In = - - (b) 1n = �/f
21e 4mff 2 38. A solid disc with radius a is connected to a
1 spring at a point d above center of the disc.
(c) In = - - (d) In = - {2k
ll
21e m 21e f;; The other end of the spring is fixed to the
vertical wall. The disc is free to roll without
36. The system shown in the figure consists of slipping on the ground. The mass of the disc
block A of mass 5 kg connected to a spring is M and the spring constant is K. The polar
through a massless rope passing over pulley moment of inertia for the disc about its
B of radius r and mass 20 kg. The spring . Ma 2
centere 1s J = --
constant k is 1500 Nim. If there is no
slipping of the rope over the pulley, the
natural frequency of the system is _ rad/s.

The natural frequency of this system in rad/s


is given by (GATE - 16 - SET - 1)

2K(a + d) 2K
(a) (b) �
2

3Ma 2 3M
(GATE - 16 - SET - 2)
2K(a + d) K(a + d)2
2
(c) (d)
Ma 2 Ma 2
!11•1ii@OiiiiOIQ@biM\!iii!"' yderabad I Delhi I Bhopal I Pune I Bhubaneswar I Lucknow I Patna I Bengaluru I Chennai I Vtjayawada I Vizag I Tuupati I Kukatpally I Kolkata
� �li'....:.......:..PoNicmooa Mechanical Vibrations
ACE
�-�
� �=� ==�
: 173 :
=================================
39. A thin uniform rigid bar of length L and
mass M is hinged at point 0, located at a Five Marks Questions
L
distance of from one of its ends. The bar
01. A cylinder of mass 1 kg and radius 1 m is
is further supported using springs, each of connected by two identical springs at a
stiffness k, located at the two ends. A height of 0.5m above the center as shown in
M the Fig. The cylinder rolls without slipping.
particle of mass m = is fixed at one end
If the spring constants is 30 kN/m for each
of the bar, as shown in the figure. For small
4
spring, find the natural frequency of the
rotations of the bar about 0, the natural system for small oscillations.
frequency of the system is
L
L/3

(GATE - 17 - SET - 1)
(GATE-ME-96)
5k
(a) � (b) �
2M 02. The suspension system of a two-wheeler can
3k be equated to a single spring-mass system
(c) � (d) �
2M with a viscous damper connected in series.
Sketch the free body diagram and give the
40. The radius of gyration of a compound equations of motion. For a mass m = 50 kg
pendulum about the point of suspension is and a spring with a stiffness of 35 kN/m,
100 mm. The distance between the point of determine what should be the damping
suspension and the centre of mass is 250 coefficient (damping constant) for critical
mm. C onsidering the acceleration due to damping. What can be the damping force
gravity as 9.8 1 m/s2 , the natural frequency for a plunger velocity of0.05 mis?
(in radian/s) of the compound pendulum is (GATE-ME-97)
(GATE - 17 - SET - 2)

lltlli@jjji4.jji40jnjjf1n1jj� yderahad I Delhi I Bhopal I Pune I Bhubaneswar! Lucknow I Patna I Bengaluru I Chennai jVijayawada jVizag I Tirupari I Kukatpal]yj Kolkata
: 174 : Theory of Machines

03. A spring mass - dashpot system is shown in X'j"O

the figure. The spring stiffness is K, mass is :- X

m, and the viscous damping coefficient is c.


� F:Cos(rot)
The system is subjected to a force F0coscot
as shown. Write the equations of motion
which are needed to determine x. (No need (GATE-ME-01)
to determine x.)

!M••ib§jjji4.jji4iditijMii&jj+yderabad I Delhi I Bhopal I Pune I Bhubaneswar I Lucknow I Patna I Bengalwu I Chennai IVrjayawada jVmg I Tirupati I Kukatpally I Kolkata I
SOLUTIONS
03. Ans: (c)
One Mark Solutions Sol: Because f = 144 Hz execution frequency.
(Natural frequency) is 128.
01. Ans : (a) � _!__ 144= 1.12 5
!Rn

Sol: Take the reference position as 'B ' = =


(!)Rn fRn 128
(PE)A = _!_ k{8 2 + mgx) It is close to 1, which ever sample for which
2
� close to 1 will have more response, so
(PE)B = _!_ k(8 + x)
2
2
sample R will show most perceptible to
(!) n

Change in potential energy: vibration

1 1 04. Ans: (c)


=- k (8 + x ) - - x k x 8 2 - mgx
2

Sol: The differential equation govemmg the


2 2
1
=- k X X 2 + k 8 X -mgx vibrating system
2
mx + c( x - y) + kx= o
X

1
=- X k X X 2 + m g X mgx
Free body diagram:
-
2
[·: k8=mg]
1
=- x k x 2

05. Ans: (c)


02. Ans: (c) Sol: Resonance occurs when forcing frequency is
Sol: �= 0.5 equal to natural frequency,
1.e., (!) = ©n
For undamped isolation system the
(!)
n

transmissibility ratio 06. Ans: (b)

TR - 1-(:.( 3
l
1 4 Sol: Critical or whirling speed

r oe = ©n = = Ji rad / sec

!M•lii@m4iljjiAflftlM\jjijj� ydcrabad I Delhi I Bhopal I Pune I Bhubaneswarl Lucknow I Patna I Bcngalwu I Chennai I Vijayawada IViz.ag I Tirupari I Kukatpally I Kolkata
ACE :.

�-� F'ii
E·.l!:� �·
ll1Jl ��00:11�========:��1 :.;,76�
: ===========Th
�:eo�ry �·�e
�ac�hin
�o�f�M
.. " = :·�1g�P1�1�Ni�:nm
letJ'lll
:

If Ne is the critical or whirling speed in rpm The whirling frequency of shaft,


then 7t gEI
f= x n2

2nNc 2 WL4
/g
60
=
V8 7t gEI
For 1st mode frequency, f1 = x � 4
2nNc 9.8lm / s 2 2 WL
=> -- =
60 1.8 X 1 0-3 m fn = n2 f1
=> Ne = 705.32 rpm � 705 rpm As there are two nodes present in 3rd mode,
f3 = 32 f1 = 1 800 rpm
07. Ans: (a) 1800
=-- = 200 rpm
Sol: Note: co n depends on mass of the system not 9
:. f1

on gravity . ·. The first critical speed of the shaft = 200 rpm

:. (!)n oc rm1 09. Ans: (b)


Sol: In damped free vibrations the oscillatory
lf co n = mg
/g 8= motion becomes non-oscillatory at critical
V8 K
damping.
· co - � /K Hence critical damping is the smallest
.. , - v(i) -v;;; damping at which no oscillation occurs in
free vibration
OJ n is constant every where.

10. Ans: (b)


08. Ans: (a)
Sol: Transmissibility (T) reduces with increase in
Sol: Number of nodes observed at a frequency of
damping up to the frequency ratio of ../2 .
1 800 rpm is 2
Beyond ../2 , T increases with increase m
damping

1 1 . Ans: (c)
Sol: At resonance for all the damping values the
phase is 90°.
k=>c>q n=3

n-mode number

!ltl1i!i§jjji4jjjjiRfffi5ijj.jj+yderabad l Delhi l Bhopa) I Pune l Bhubancswarl Lucknow l Patna l Bengaluru i Chennai l Vliayawada!Vmg ITirupali I Kukatpal)y i Kolkata I
: 1 77 : Mechanical Vibrations

12. Ans: 6.32 m/sec2 16. Ans: 100 (range 9� to 101)


Sol: In simple harmonic motion, x = X sin rot
Sol: ro0 = {g = � = 100 rad / sec
sin rot
x = -Xro 2
VB \/wo-3
Given X = 10 mm = 0.01 m
f = 4 Hz 17. Ans: (b)
:. ro = 21tf = 8 7t rad/sec Sol: mx + ex + kx = F(t)
Magnitude of acceleration or maximum 2c
We have - = ro0 = 2- ro0
C C
acceleration = Xro2 m 2� km cc
O.Ol x (8 1t)2 = 6.32 m/sec2 . .
Dampmg ratio = - = �
=
C C
cc 2-vkm
13. Ans: 0.38 to 0.42
Sol: Given C c = 2� = 0.1 kg / sec 18. Ans: (a)
m 1 = 2m k 1 = 8k Sol: Springs are in parallel connections
keq = k +k = 2k
C c 1 = 2�k 1 m 1 = 2 x .J8k x 2m
= 4 x 2� = 4.C c = 0.4 kg / sec

14. Ans: (c)


Two Marks Solutions
15. Ans: 20 (range 19.9 to 20.1)
Sol: k = l OkN / m 01. Ans : (b)
Fo = 100 N Sol:
s = 0.25

K3 2 kN/m
K12 =

3 kN/m K2
- = 1 at resonance
0)

O)n

100
X = _§_ = = 20 mm
2k� 2 x10 x 0.25 x l 03 m= l kg

\( } l 11gll ll t [ Il l:.! P11!,lt< ,tl H >ll', ydcrahad I Delhi I Bhopal I Punc I Bhubaneswarl Lucknowl Pa!nal Bcnga)uru I ChcnnailVuayawadalV1.7.3g I Tirupari I Kukatpallyj Kolkata
: 178 : Theory of Machine

K1 & K2 springs are in series Hence the frequencies for the given system
Resultant spring stiffness (K 12)
are O and
��
(2K
-1 1 1 1 1
=- + -= - + -= 1.33
K 12 K 1 K 2 1 3
K12 = 0.75 kN/m

K12 & K3 are in parallel


: . Resultant K = K12 + K3
= 0.75 + 2 = 2.75 kN/m
Alternative Method:
:. Natural frequency For the above diagram the equation can
be written as
=
15 =�
2.75
:
1000
m 1 x 1 + K(x 1 - x 2 ) = 0

= 52. 44 rad/sec = 8.35 Hz m 2 x + K(x 2 - x 1 ) = 0


Assuming the solution of the form.
02. Ans: (a) x 1 = A 1 sinrot
Sol: The given system is a 2 D.O.F one without x2 = A2 sinrot
constraints and exhibits a rigid body motion � -m 1 ro2 A 1 + K(A 1 - A 2 ) = 0
for which the frequency is zero. The node
� -m 2ro2 A 2 + K(A 2 - A 1 ) = 0
shape corresponding to the non zero
Amplitude ratio
frequency is as shown in figure. As the
A 1 ----K K - m 2 ro2
masses are equal in both sides, the node will
A 2 K - m 1 ro2 K
=
be at the middle. By fixing the spring at the
node we can separate into two single D. 0 .F K K - m 2 ro2
� ---- =------"--
systems and both will have same natural K - m 1 ro2 K
frequency. As the node falls in the middle of � -K(m 2 ro + m 1 ro )+ m 1 m 2ro
2 2 4

the spring, the spring is divided into two


equal halves and each will have stiffness of � m , m , {oo' -Koo' (�, + �,)}-o
2K. So the frequency for each system is
...... (m1 = m2=m)
equal to (2K . Solving this equation we get ro1 = 0 and
��
ro2 = ff

!ffl•li@Oii§.jji4Rflnj@j/m� yderabad I Delhi I Bhopal I Pune I Bhubaneswar I Lucknow I Patna I Bengaluru I Chennai I VijayawadaJVizag I Tirupati I Kukatpally I Kolkata
: 179 : Mechanical Vibrations

03. Ans: (c) OS. Ans: (b)


Sol: Springs are in parallel, smce deflection is Sol: Here, m= 10 kg = Mass of Rotor
same d = Diameter of shaft
1 �K 1 +K z = 0.03 m
f =-
21t m I = Length of shaft
=500x 1 o-3m = 0.5 m
n

= 1 40 x 1000 20 10
Hz
= =
21r 100 21r 1r E for steel = 2. l x 10 1 1 N/m2

04. Ans: (d) The Given shaft can be considered as a


Sol: simply supported beam for which the
. 48EI
stiffness K=-3-
.e
Natural frequency

co =
{K=�4 8EI
n f� m.e 3

mg mg f, = �; = 90 Hz (·: I = 7 • )
4
Let the system is displaced by 0 from the
equilibrium position. It's position will be as 06. Ans: (a)
shown in figure. Sol: KL0
By considering moment equilibrium about
the axis of rotation (Hinge)
..
I 0 +m g .e sin (a+0) - m g .e sin(a - 0)=0
I mR 2 +m.e 2=2mR 2
= a ••
---- 1 0
After simplification L
..
2mR 2 0+2mg.e cos a sin 0=0
For small oscillations ( 0 is small) sin 0 = 0 By moment equilibrium about the hinge I
.. .. .
:. 2 m .e 0 + 2 m g .e cos a . 0 = 0
2 0+C a 2 0+KL2 0+K 8 0=0

2 m g .e cos a g cos a mL2 •• • \.,


co n= =� -- 0+Ca 2 0+ (KL2 +K 8 p=0
2 m .e 2
.e 3

!M•1iii§jjji4.jji4Rffn1il1H¥+iyderabad j Delhi j Bhopal j Pune j Bhubaneswarj Lucknowj Patna j Bengaluru j Chennai j VtjayawadajVmg IT=pati I Kukatpallyj Kollwal
... ,.......
... ACE
�..a: "E·=

��""
�·-
::m�w���1�
��n11
��:oo:•=========�:���: ===========Th
eocy of M hine
����:�:;,:
l 80
- ��:·�

09. Ans: (a)


Sol: Given length of cantilever beam,
/ = l OOO mm = l m, m = 20 kg

�-t=:=========1r:i
� I Im LJ
07. Ans: (c)
=

Sol: Refer to the above equilibrium equation


C ross section of beam = square
C eq = C a 2

N- m - sec
= 500 x 0.4 2=80
rad
� C = 80 N m.s/rad

08. Ans: (c)


Moment of inertia of the shaft,
Sol:
, -------

Kx -�l+--X----t 25 x (25) 3
I = _!_ bd 3= = 3.25 x10-8 m4
12 12
\

Esteel = 200 X 10 9 Pa
\
\
\ 1 50mm

Mass, M = 20kg
3 1
1 50mm
Stiffness K= �
f,
Q C ritical damping coefficient,
sec
Let the weight be displaced by x from C c= 2.JKm=1 250 N.
m
equilibrium position. Then

tanS = �=i_ 10. Ans: (c)


300 150
Sol: Given m = 0. 1 kg , /<l = 0.9 In
.. y = - = 0.9 COn
2
K = 1 kN/m
� C0d

cod=co n �
Taking moment about hinged point

Kx.(0.3) = W.y = W. -
X

2
� 0.9co n= (O n �1 - � 2
300
K= = 500 N/m � � = 0. 4 35
2 x 0.3

!Mlli@jjj§lfoijQnftjM\,@+yderabad I Delhi I Bhopal I Pune I Bhubaneswar l Lucknow I Patna I Bengaluru I Chennai I Vuayawada JVizag I Tirupati I Kukatpal)y I Kolkata I
" . ACE
. . Mechanical Vibrations
..�� PuNiratiooa
':,�-�
,.., . : 181 :

C 14. Ans: (a)


1; =
2-JKm Sol: X 1 = xe-�@.Td
=> C = l; x 2.JKm = 8.7 N.sec/m

11. Ans: (c)


Sol: Given m = 250 kg , K = 100, 000 N/m
N = 3600 rpm
1; = 0.15

ro0 = ! = 20 rad /sec


15. Ans: (a)
2n x N
ro -- = 377 rad/sec Sol: (1) when spring are in parallel
60
=

K K
K = K1 +K2 => - + - = K
2 2
(2) Spring are in series then
1 1 1
- = -+-
K K1 K2
1 1 1 1 2
- = - + - => - = -
12. Ans: (d) Ke K K Ke K
Sol: Given K
2
=> K = -
m = 12.5 kg , K = l OOO N/m , C = 15 Ns/m e

Cc = critical damping coefficient


We know that
Cc = 2.JKrn 2 x ,J12soo = 223.6 Ns/m
16. Ans: (b)
=

Sol: Springs are parallel


13. Ans: (b)
fn = -
1
Sol: 1; = _£ =
I5Ns / m /IC
= 0.0670 21t f-;;;
Cc 223.6Ns / m
Ke = K1 +K2 = 4000+1600 = 5600 Nim
o= 2
ns
1 5600 = l O Hz
fn = __ �
= 0.4222 ,
�1- 1; 2
2,r 1.4
Near value is selected

l111•i!@jjjiiijjj4Aflaji¥\jjfij.yderabad I Delhi I Bhopal I Pune I Bhubaneswar I Lucknow I Patna! Benga)uru I Chennai j Vrjayawada I Vu.ag I Tirupati I Kukatpally I Kolkata I
: 182 : Theory of Machine

17. Ans: (c) m = 240 Kg


Sol: 96 X 10 6
= ---= 632.455 rad/sec
240
COn

x 60
N= = 6040 rpm
2,r
I
(On

19. Ans: (a)


By energy method
Sol: Damping factor(�)=�
1 . 1 cc
E = - 10 2 + - Kx 2 = constant
Actual damping coefficient
2 2
1 1 (f, )2
E = - 18 2 + - K x - 0 =cons tan t
critical damping coefficient
2 2 2
C c = 2-JKm= 2 X .J3.6 X 10 3 X 50
Differentiating w.r.t 't'
=84 8.528 N-s/m
dE •• K £ 2
-= 1 0+- x - x 20=0 . C 400
dt 2 4 Dampmg factor � = -=--=0. 47
c c 84 8.52
m£ 2
l= -
12
m£ .. K£ .
2 2
co n= i = 8. 4 8 rad/ sec

- 0+- 0=0
12 4 cod = (�1- � 2 � n=7 .4833 rad/sec
·· 3K · co
� 0+ - 0=0 f0= d=1. 19 100 Hz
21t

� co n=
v --;-
{3K=30 rad / sec fn=1. 19 1 Hz

20. Ans:(a & d)


18. Ans: (a)
Sol: m x +Kx=F cos cot
Sol: K1 , K2 = 16 MN/m
K3, � = 32 MN/m m= ? K = 3000 Nim,
K.,q= K, + K2 + K3 + �
F= 100 N, co = l OOrad / sec
X = 50 mm = 0.05 m

m = 240 kg

(On = li X= F 2
K - m co
°Keq= ((16 X 2)+ (32 X 2)) x 10 6= 96 X 1Q 6
!MIIM@h/ilih4Pffbi@dfjj� yderabad ! Delhi ! Bhopal ! Pune l Bhubaneswarl Lucknow ! Pa,tna ! Bengaluru ! Chennai l VuayawadajVizag !Tuupati, I Kukaq>ally! Kolkata
"...
" ' �..," . . .
. ACE : 1 83: Mechanical Vibrations
� . ��PnNir:,t',oos

K F 23. Ans: (d)


=> m = - 2 -
-- = 0. l kg
ro Xro2 Sol:
F
or X = => m = 0.5 kg
moo -K
2

21. Ans: (c)


Sol: Applying Energy method

� (KE + PE) = O Equation of motion = mL2 0 +k(2L)28 = 0


dt K4L2
0+ 8=0
=> � ( .!. 10 2 + .!. Kx 2 ) = 0 ( ': x = rS ) mL2
dt 2 2 ·· 4K
8 + ( )8 = 0
m
=> � (.!. (I mr 2 )0 2 + _!_ K(rS )2 J = o
dt 2 2 2
:. ©n = � rad/sec
: . (I o = Ia + mr = ½ mr + mr = �
2 2 2 mr 2 )

24. Ans: (c)


=> � [I mr 2 0 2 + .!. K r 2 0 2 ] = o
dt 4 2 Sol: Let,
·· 2K V0 is the initial velocity, 'm' is the mass
=> 8 + - 8 = 0
3m Equating Impulse = momentum
2K 1 mV0 = 5kN x 10 -4 sec
=> 002 =
0
3m
=> f = _
21t
O
v�
{2K
=5 x 103 x 10-4 = 0.5 sec
0.5
22. Ans: (a)
:. V0 = - = 0.5 m I sec
m

Sol: "'· � ��
{K 10000
000 = v -; = �-- = I OO rad / sec
1
K eq = K 1 + K 2 � for parallel When the free vibrations are initiate with
=> K eq = 20 + 20 = 40 kN I m
initial velocity,
The amplitude
000 = �{40x1ooo = 200 rad / sec . x = Vo (Initial displacement is 0)

(l)n
1 100
=> f = - x 200 = - = 32 Hz :. X = V0 = 0.5 x l0 5 mm
3

n 21t 7t 1 00
(l)n

!Jfll4jj§jjji11hiJNlnj@j@� yderabad I Delhi I Bhopal I Pune I Bhubaneswar I l.ncknowl Patna! Bengaluru I Chennai I Vrjayawadaj Vizag I Tuupati I Kukatpally I Kolkata
�-ti
W
,. . .,V F.ngioeering
• •
ACE
Publiratioos
• • : 184 : Theory of Machine

25. Ans: 0.0658 N.m2


3 1
Sol: For a cantilever beam stiffness, K = �
R

Natural frequency ro 0 = � = � ��!


Given f0 100 Hz =

=> ©n =
21tf0 200 7t
=

3EI 27. Ans: (d)


200n = �
mR 3 Sol: The springs are in parallel
Flexural Rigidity K eq = K 1 + K 2
3
(2oo x 7t J mR
EI = = 0.0658 N.m2 : K,
3
ov �� = r•
26. Ans: (d)
28. Ans: (1.25)
Sol:
Sol: Given m = 1 kg , K =
100 Nim
N-sec
L C = 25
K 0
3
Critical damping
- -------
r N-sec
Cc = 2../i(;; = 20
I0
25
Moment equilibrium about the pivot gives Damping Ratio = S_ = = 1.25
Cc 20
·· 2L 2L L L
I0 + K x -x -0 + K x - x - x 0 = 0
29. Ans: 10 N.seclm
3 3 3 3
.. 4KL2 KL2 Sol: Given systems represented by
10 + (-- + --J 0 = 0
9 9
mx + ex + kx = F cos rot
mL2 F
Mass moment of Inertia I = -- For which X =
9 2
�(K - mro2 ) + (c ro)2
mL2 0 5KL2 =
:. + 0 0 Given K = 6250 Nim, m = 10 kg,
9 9
F IO N =

ro 25 rad/sec, X 40x 10-3


= =

\('I I 11g1 t 1 l ( 1 111g P11hl1<. ,1t1111h �yderabad I Delhi I Bhopal I Pune I Bhubaneswar I Lucknow! Patna I Bengalwu I Chennai ! Vuayawada j Vu.ag I Tirupati I Kukatpally I Kolkata I
Moment equilibrium about hinge
con = != 25 rad / sec
m2r0.2r + kr0.r = 0
cot = 25t => co = 25 rad/sec 4mr 0 + kr 0 =0
2 2

co = con or K = mco!
co = W= fk
=
:. X=_!__ => C= _!__
n v� v� (4oo
V4
C eo Xco
10 N - sec 32. Ans: (b)
= =10
4 0 x l 0- x 25
3
m Sol: When lifted from ground at Q, reaction = 0.
Taking the moments about ' P ' and equating
30. Ans: (2.28) to zero.
Sol: Given, m = 2 kg, K = 8 N/m, � = 0.02 90x 10-3 x9.8 Ix6x10-2 = meco2 x 9 x 10 -2
co = 1.5 rad/sec 90 10-3 X 9.8 1 6 10-2=2 x lQ-3 X 2. 19
10-2
X X X

10-3 9
(O n=!=2
X
2
X CO X X

=> co = 366.58 rad/sec


co 1.5 3 21tN
- =- = -
(O n 2 4 60
C0 = - -

Magnification factor (M.F) 60 x 366.58


=> N= = 3500 rpm
1 21t
=

33. Ans: 6750 to 7150


Sol: Given f = 60 Hz, m = 1 kg
= -----1---- - = 2.28 co=21tf=120 mad/ sec
Transmissibility ratio, TR = 0.05
Damping is negligible, C = 0 , K =?
{, - (¾) ' } ' + (2 x o o2 x ¾) '
K
We know TR = when C = 0
K - mco2
31. Ans: (d)
Sol: Free body diagram As TR is less than 1 => co/ co n >> Ji
K
TR is negative => - 0.05 =
K - mro2
kr0 m2r 0
e_ _ _ _ _ _ J Solving we get, K = 6767.7 Nim

\( 'I J llgllH'\ 1 1 1 1 !.!, PtililH .1[11111-.. Fydcrabad l Delhi I Bhopal I Pune I Bhubaneswar l I..ucknow! Patna l Bcngalwu i Chcnnai j Vtjayawada l VJZag l1irupati I Kukatpallyl Kolkala I
" " . � . .
':.... :E�Pn'filiratiooa : 186 : Theory of Machine
A ,CE
.. ... =====================================
34. Ans: (b) ·: con = 21tf
Sol: Given Problem of the type
mx + ex + k:x = F cos cot
for which, X = (
k - mco2)' + (cco)2 36. Ans: 10 (range 9.9 to 10.1)
F/K
or X = Sol: KE = _!_ mx 2 + _!_ 10 2
(1-(:, )}(2�:) 2

m = 5kg,
2
0-�

Given F = 10, con = l Oco 20 x r 2 = I 2


1= 0r
k = l50 N i m or � = _!_ = 0. 1
2
10 2
(O n
2 2 r 2
.*
KE = _!._ 5x 2 + _!_ 10r 2 2 = _!_ (15)x 2
l; = 0.2
10 / 1 50 :. m eq = 15
X=
�(1- 0. 1)2 + (2 x 0.2 x 0. 1)2
PE = _!_ kx 2
2
= 0.0669 � 0.07
:. k eq = k = 1500N / m
35. Ans: (a) Natural frequency
Sol: � I OO
ro = = � S = IOrad / sec
m2a 0 n �� 15
0

37. Ans: 0.5 (range 0.49 to 0.51)


By taking the moment about 'O', �lllo = 0
Sol: � =
.J3k / m = ..[3

l
(m2a0 x 2a) + (ka0 x a) = 0
(On
=> 4a m 0 +ka 0 = 0
2 2

Where, Illeq = 4a2m, keq = ka2


k
Natural frequency, ro , = � ..,
m eq

1
= - = 0.5
2

IM11i@jjgg.jj§Aflnjjjii,jjf}yderabad ! Delhi l Bbopal l Pune ! Bhubaneswarl Lucknow ! Patna ! Bcngaiuru! Chennai ! Vuayawada ! Vizag I TllUpati I Kukalpally l Kolkala I
: 187 : Mechanical Vibrations

38. Ans: (a) 40. Ans: 15.66


Sol: Moment equilibrium above instantaneous Sol:
centre (contact point)
- k(a + d)0.(a + d) = I)�

mg

Radius of gyration, r = 1 00 mm = 0. 1 m,
L = 250 mm = 0.25 m,
g = 9.8 1 m/sec2
k(a + d)2
C
3
I c = - Ma 2 , coa = =?
2 i Ma 2
COn

2 The equation of motion is


2k(a + d) 2 I 0 0 + mgLsin 0 = 0
co 0 =
3Ma 2 For small value of 0, sin0 :::: 0

39 Ans: (b) :. w" = �mgL


Sol: Take the moment about the hinge 'O' Io
The equation of motion is "
· I0-
- mr
2

= �mgL = �L
co n
2
mr r2
9·8 1 x 2
m= - = �- \ad / s = 1 5.66 rad / s
0. 1

ML2 4ML2 5kL2


(-- + -- 0 + - - 0 = 0
9 9x4 9
.J
SkL2
(2ML2 ) + 0=0
9
Natural frequency,
5kL
2

( J
9
= [5k
2ML2 � 2M
9
\Cl l.11g111< <1 111g 1'11lil1< .111<111, yderabad I Delhi I Bhopal I Pune I Bhubaneswar I Lucknow I Patna I Bengaluru I Chennai I Vgayawada I V=g I 'rrrupati I KukatpalJy I Kolkata
•,"
.,
.,
\II •
:FipeerqNn,mro
ACE • • •
: 188 : Theory of Machine

Or
Five Marks Solutions -,---....._
I
I)i

01.
Sol:
-· ·­ I
I

'
I

A ,I

! / Taking the moment about the instantaneous


0 centre 'A'.
· - · - · - · ..... · "t-
' ·-·-·

r
1

/: :� 0
I I

IA 0 + 2K (r+a) 0 (r+a) = 0
I I

mr 2 3
I A = -- + mr2 = - mr 2
2 2
2
Imr 2 0 + 2k(r + a ) e = O
2
KE = .!. mr 2 8 2 + .!_ 10 2 2
2 2 ro , = � k ., = 2k(r + a)' = 4k(r + 2a)
3 2 3mr
= .!. mr 2 0 2 + .!.. mr 2 0 2 = I mr 2 0 2 - mr
m

2
eq

2 4 4

PE = _!_ Kx 2 + .!. Kx 2 = Kx 2
2 2 02.
x = (r + a)0 Sol:
=> PE = K{(r + a)0} 2 X mx
..
d d
- KE + - PE = O
dt dt
X

m
Substituting in the above equation m

I m r 2 0 + 2K(r + a)2 9 = 0
2 C
K

Natural frequency Cx Kx

f =- 1 4K(r + a)2 Free body diagram


21t 3mr 2
n

So fn = 47.74 Hz.
\('l l 1 1gllh l l l l lg P11hltl ,IIJt)[h �yderabadj Delhi I Bhopal j Pune l Bhu�cswarl lncknowl Patna! Bengalwuj Chennai I Vuayawadaj Vizag jTrrupari I Kukatpallyj Kolkata I
{.E,;;;;&i;:1111'1
�·_ ;;;;;rm;;;;::�;,;;J!t
;;,;AJ
.., :.Oll•=========:==
;.;;;;;;;;ili;;;;;®
;;;;;;;,;;; =e=
1 8=9=:==========M ch=·==
=amca1 Vi b=·=
=1=ratI ns
o=

Considering the system in Figure for free Cc = 2m COn (or) 2 .JK.m


vibrations of the system, we get = 2 X 50 X 26.45
m x + C x +Kx = O . . . . . . (A) (or) 2.J35000 x 50 = 2645 N-s/m
where x = est
satisfies the above equation, giving 03.
ms2 + Cs + Kx = 0 Sol:
[2nd order differential Equation]
The two roots are,
) -K C
2
s ,, 2 �!:_±�(
2m m 2m
Let the displacement of the mass from its
static equilibrium position be x and the
Where A and B are system constants of displacement of the free end of the damper
integration that depend on the initial where the force is applied be y the free body
conditions with which the motion of mass, diagram will be as follows
m is started. It depends on the two roots The equations of motion are
s 1 and s2 which may be real or complex
depending on the quantity under the square
mx + c( x-y) + Kx = O
root is +ve or -ve. For critical damping.
c( x-y) + F cos cot = O
0

Where co0 = natural circular frequency,


Cc = critical damping coefficient.
Cc = 2mcon = 2 .JK.m

The critical damping is a system property


that depends on the mass and stiffness
{K = �35000 = Jmo
000 =
v� 50
=
26.45 rad/s

\( I I ri..!,1 1 1( , 1 1 1 1.., P1ild1, .111, 1 1 1 " �yderabad j Delhi j Bhopal j Pune j Bhubaneswar l Lucknowl Patna l BengaJuru l Chermai j Vliayawada j V,zag I T1n1pali I Kukalpallyj Kolkala I
Gyroscope

One Mark Questions

01. A car is moving on a curved horizontal road


of radius 100 m with a speed of 20 mis. The
rotating masses of the engine have an
angular speed of 100 rad/s in clockwise
direction when viewed from the front of the
car. The combined moment of inertia of the
rotating masses is 10 kg-m2 • The magnitude
of the gyroscopic moment (in N-m) is__
(GATE - 16 - SET- 1)

\ ( l l 1 1l_111, , 1 u 1..., P1 1hl1, .it1,111.., �yderabad J Dellii J Bhopa) J Pune J Bhubaneswar J wcknow l Patna l Bengaluru J Chennai J Vgayawada J Vmg J Tirupari. J �yj Kolkala I
SOLUTIONS

One Mark Solutions

01. Ans: 200 (range 199 to 201)


Sol: R= 1 OOm,
v = 20m/sec,

ro = V = 0.2 rad
P R sec
ro5 = l OOrad / sec
I = 1 0kg-m2

Gyroscopic moment
= Iros ro p = 1 0x0.2x 1 00N-m

= 200N-m

\l I I 1 1 ..,\ , lt t 1 11 1 ..:, Pi d d u ,t!JcJtl" �ydcrabadl Delhi f BhopaI I Pune l Bhubaneswarl tllCknow l Pama l Bcngaluru lCl\cnnai lVuayawadaj Vmg ITtrupali I Kukalpally� Kolka!:, I
C7 Balancing
03. Three masses are connected to a rotating
Two Marks Questions shaft supported on bearings A and B as
shown in the figure. The system is in a
0 1 . A rotating disc of 1 m diameter has two space where the gravitational effect is
eccentric masses of 0.5 kg each at radii of 50 absent. Neglect the mass of shaft and rods
mm and 60 mm at angular position of 0° and connecting the masses. For m1 = 1 0 kg, m2=
1 50°, respectively. A balancing mass of 0. 1 5 kg and m3 = 2.5 kg and for a shaft angular
kg to be used to balance the rotor. What is speed of 1 000 radian/s, the magnitude of the
the radial position of the balancing mass ? bearing reaction (in N) at location B is
(GATE - OS)
(a) 50 mm (b) 1 20 mm
(c) 150 mm (d) 280 mm

02. Two masses m are attached to opposite


sides of a rigid rotating shaft in the vertical
plane. Another pair of equal masses m1 is
X

attached to the opposite sides of the shaft in


the vertical plane as shown in figure.
Consider m = 1 kg, e = 50 mm. e1 = 20 mm,
b = 3 m, a = 2 m and a1 = 2.5 m. For the O. l m O. l m
system to be dynamically balanced, m1
should be __ kg.
(GATE - 16 - SET - 3)

(GATE- 1 7 - SET - 2)

a1

�Hyderabad I Delhi I Bhopal I Punc I Bhubancswar I Bcngaluru I l.ucknow I Patna I Chcnnai I Vuayawada j V17.3g I Tirupati I Guntur I Kukatpally(Hyd) I
SOLUTIONS

Two Marks Solutions 02. Ans: 2 (range 1.9 to 2.1)


Sol: By symmetric two system is in dynamic
01. Ans: (c) balance when
Sol:
e a 50 2
m 1 = m - .- = l x - - = 2kg
e1 a 1 20 2.5

03. Ans: 0
Sol: y
Given, m1 = 0.5 kg , m2 =
r1 = 50mm ' r2 = 60 mm ' e = 150°,
mb = 0.1 kg m, rb = ?

= 0.5 x 50 + 0.5 x 60 cos150


= -0.9807 kg-mm F 1 = m1r1ro2 = 10 x 0.1 x ro2 = ro2
FY . F2 = m2 r2 ro2 = 5 x 0.2 x ro2 = ro2
-
2
= m 2 r2 sm 9
F2 = m2r2 ro2 = 2.5 x 0.4ro2 x ro2
0)

= 0.5x60 sin150 15 kg-mm


IFx = ro2 [1+ cos120 ° + cos120°] = 0
=

m b rb = �Fx + FY LFy = F2cos30° -F3cos30° = 0


2 2

:. Net force = 0
�Fx 2 + F/
=> rb = :. Bearing reactions = 0 N
mb

�-
- ( 0.9807) + (15} = 1 50.32 mm
2 2

0.1
F 15
9b = tan -1 _y_ = tan -1 (---) 93.74°
Fx - 0.9807
=

\( I l 111411H l I 1 1 1g P1 1lil11 ,lll(llJ', yderabad I Delhi I Bhopal I Pune I Bhubaneswar I Lucknow I Pama I Bengaluru I Chennai I Vtjayawada I V,zag I Tuupati I Kuk.atpallyl -Kolkata
ca Governors & Cams

One Mark Questions Two Marks Questions

01. In a cam - follower mechanism, the


01. Consider a circular cam with a flat face
follower needs to rise through 20mm during
follower as shown in the figure below. The
60° of cam rotation, the first 30° with a
cam is rotated in the plane of the paper
constant acceleration and then with a
about point P lying 5 mm away from its
deceleration of the same magnitude. The
center. The radius of the cam is 20 mm. The
initial and final speeds of the follower are
distance (in mm) between the highest and
zero. The cam rotates at uniform speed of
the lowest positions of the flat face follower
300 rpm. The maximum speed of the
is
follower is (GATE-ME-05)
(a) 0.60 mis (b) 1 .20 mis
(c) 1 .68 mis (d) 2.40 mis

02. In a cam design, the rise motion is given by


a simple Harmonic Motion (SHM)

(GATE - PI-16) s = � (I - cos ;a) where h is total rise, 0 is


(a) 5 (b) 1 0 (c) 40 (d) 45
camshaft angle, J3 is the total angle of the
rise interval. The jerk is given by
(GATE-ME-08)

(b) --sm -
(a)
h (1-cos B
1t0) 1t h . (7t0J
2 J3 2 J3

(c) -
1t
- cos -
2
h (7t0) 7t3 h . (7t0J
(d) -3 -sm -
J3 2 J3 2
2
J3 J3

l•••Ml@Ui44ihi4WGiMOii!&ij>ttyderabad l Dclhi l Bhopal j Punc l Bhubaneswar l lncknow! Patna ! Bcngaluru l Chennai lVuayawada j Va.ag I TIJ'Upali_ l Kukatpally l Kolkala I
SOLUTIONS
[ ·: u = O ]
One Mark Solutions
1
s 1 = lOmm =-a t 21
01. Ans: (b) 2
Sol: For the highest position the distance
between the cam center and follower
= (r+5) mm
For the lowest position it is (r -5) mm 20x3600
a= = 72m/s 2
So the distance between the two positions 1000
= (r+5) -(r-5) = 10 mm
(;)
2

v = at 1 = 72x
101t
Two Marks Solutions

72 6
01. Ans: (b) = - =-= 1.2 m / s
60 5
Sol: h = 20mm, 0 = 60°,

81 = 30 = rad , 82 = 30°
7t
02. Ans: (a)
°
6
2x1tx300 d3s
co=----= 101t rad/ s Sol: Jerk = -3
60 dt
1
s 2 =-a t 22 : = � x; sm(
�J
e
2
92
d 2 s h £ (1t8J
92 =1 = x cos
2
�=.s_=_I
S2 t 2
2 2 dt 2 2 p 2 p
d 3s = - h 1t8
x �sin( J
dt 3
2 p 3
p

\( I I rn.:.111tl 1111,..: P11ld1t,1111111-.. FydcrabadlDclhilBhopaIIPunclBhubancswarl I..ncknowjPatnajBcngalurujChennailVvayawadajVizag jTirupari I Kukatpallyj Kolbta I


: 196: Theory of Machine

\( I I 11.,111t « r 111 ...., P1 ii )It{ 111, ,r 1�


Machine Design
(Questions)
PageNo.198

CONTENTS
-

Chapter Questions Solutions


Name of the Chapter
No. --
P_t!ge �o. Page No.
-

01 Static Loading 199- 202 203- 206

02 Fatigue 207- 210 211- 214


Bolted, Riveted and Welded
03 215 - 218 219- 222
Joints
04 Gears 223- 227 228 - 230

OS Rolling Contact Bearings 231- 232 233- 233

06 Sliding Contact Bearings 234- 234 235 - 235

07 Brakes 236 - 238 239- 240

08 Clutches 241- 241 242- 242


Ct Static Loading
(a) 100 MPa (b) 285 MPa
One Mark Questions (c) 190 MPa (d) indeterminate

01. In the design of shafts made of ductile 04. The outside diameter of a hollow shaft is
materials subjected to twisting moment and twice its inside diameter .the ratio of its
bending moment, the recommended theory torque carrying capacity to that of a solid
of failure is (GATE-ME-88) shaft of the same material and the same out
(a) maximum principal stress theory side diameter is (GATE-ME-93)
(b) maximum principal strain theory (a)15/16 (b)3/4 (c)l/2 (d)l/16
(c) maximum shear stress theory
(d) maximum strain - energy theory 05. Two shafts A and B are made of the same
material. The diameter of shaft B is twice
02. Strength to weight ratio for a circular shaft that of Shaft A. The ratio of power which
transmitting power is directly proportional can be transmitted by shaft A to that of shaft
to the (GATE-ME-91) B is (GATE-ME-94)
(a) square root of the diameter (a) 1/2 (b) 1/4 (c) 1/8 (d) 1/16
(b) diameter
(c) square of the diameter 06. A solid shaft can resist a bending moment of
(d) cube of the diameter 3.0 kNm and a twisting moment of 4.0kNm
together, then the maximum torque that can
03. A large uniform plate containing a rivet hole be applied is (GATE-ME-96)
subjected to uniform uniaxial tension of 95 (a) 7.0 kNm (b) 3.5 kNm
MPa. The maximum stress in the plate is (c) 4.5 kNm (d) 5.0 kNm
(GATE-ME-92)
07. Which theory of failure will you use for
aluminium components under steady loading
95 MPa lS (GATE-ME-98)
(a) principal stress theory
(b) principal strain theory

\(} l.1112;111lt till� P11lilH,1l1011, �yderabad I Delhi I Bhopal I Pune I Bhubaneswar I Lucknow I Patna I Bengaluru I Chennai jVtjayawadajVu.ag I Tirupati I Kukatpally J Kolkata I
"
ACE
"Fqpn,emg PnJtliranml : 200: Machine Design
�-�
" " • • I.. •

(c) strain energy theory (c) As per the maximum strain energy
(d) maximum shear stress theory theory, failure occurs when the strain
energy per unit volume exceeds a
08. Match the following criteria of material critical value
failure, under biaxial stresses cr1 and cr2 and (d) As· per the maximum distortion energy
yield stress cry with their corresponding theory, failure occurs when the
graphic representations: (GATE-ME-11) distortion energy per unit volume
P. Maximum normal stress criterion exceeds a critical value
Q. Maximum distortion energy criterion
R. Maximum shear stress criterion 10. The uniaxial yield stress of a material is 300
MPa. According to Von mises criterion, the
L) M) cry
shear yield stress (in MPa) of the material is
(GATE-15-Set 2)
-cry cr
-cry
11. A machine element XY, fixed at end X, is
N) subjected to an axial load P, transverse load
F, and a twisting moment T at its free end Y.
The most critical point from the strength
point of view is
(a) P-M, Q-L, R-N
(b) P-N, Q-M, R-L
(c) P-M, Q-N, R-L p
(d) P-N, Q-L, R-M y

09. Which one of following is NOT correct?


(GATE-ME-2014-SET-3) (GATE-16- SET-2)
(a) Intermediate principal stress is ignored
(a) a point on the circumference at location Y
when applying the maximum principal
(b) a point at the center at location Y
stress theory
(c) a point on the circumference at location X
(b) The maximum shear stress theory gives
(d) a point at the center at location X
the most accurate results amongst all
the failure theories

\( I I lll...,IIH l 1111� P11hl1l ,tlJolh �yderabad I Dellii I Bhopal I Pune I Bhubaneswar I Lucknow I Patna I Bengaluru I Chennai IVtiayawada I Vmg I Tirupati I Kukatpally I Kolkata I
: 201: Sta.tic Loading

12. Consider the schematic of a riveted lap joint 02. The homogeneous state of stress for a metal

J
subjected to tensile load F, as shown below. part undergoing plastic deformation is
Let d be the diameter of the rivets, and S1 be l

the maximum permissible tensile stress in T = [ i ;O �


the plates. What should be the minimum 0 0 -10 ·
value for the thickness of the plates to guard Where the stress component values are in
against tensile failure of the plates? Assume MPa. Using von Mises yield criterion, the
the plates to be identical. value of estimated shear yield stress, in MPa
F (GATE-ME-12)

t�
lS

(a) 9.50 (b) 16.07


(c) 28.52 (d) 49.41
w 2
d
03. A solid circular shaft needs to be designed to
transmit a torque of 50N.m. If the allowable
L shear stress of the material is 140MPa,
(GATE-17-SET- l) assuming a factor of safety of 2, the
F minimum allowable design diameter in mm
(a) (b)_!_
sf(w - 2d) sr w lS (GATE-ME-12)
F (a) 8 (b) 16 (c) 24 (d) 32
(d) �
sr w
04. The state of stress at a point is given by
Two Marks Questions ax = -6 MPa, cry = 4 MPa, and 'txy = -8 MPa.
The maximum tensile stress (in MPa) at the
point is__ (GATE-ME-14-SET-1)
O1. A small element at the critical section of a
component in bi - axial state of stress with
05. Consider the two states of stress as shown in
the two principal stresses being 360 MPa confi gurations I and II in the figure below.
and 140 MPa. The maximum working stress From the standpoint of distortion energy
according to distortion energy theory is (Von-Mises) criterion, which one of the
(GATE-ME-97) following statements is true ?
(a) 220 MPa (b)l lO MPa
(GATE-ME-14-SET-2)
(c) 314 MPa (d) 330 MPa
jltllii@hiiiihiMAflfiiMjjih+yderabadlDelhilBhopallPunelBhubaneswarlLucknowlPatnalBengaiuru!ChennailVliayawadalV17.ag
ITuupati I Kukalpallyl Kolka!a I
: 202: Machine Design

the material is 200 MPa. The factor of safety


cr
calculated using Tresca (maximum shear

(a) I yields after II


-'t
I stress) theory is nT and the factor of safety
calculated using Von Mises (maximum
distortional energy) theory is nv. Which one
I II
(b) II yields after I of the following relations is TRUE?
(c) Both yield simultaneously (GATE-16-SET-1)
(d) Nothing can be said about their relative
yielding
(a) nr-(�}. (b) nT = ( ../3) n v

(c) nT = nv (d) n v = (../3� T


06. A shaft is subjected to pure torsional
moment. The maximum shear stress
09. The principal stresses at a point in a critical
developed in the shaft is 100 MPa. The yield
section of a machine component are cr 1 = 60
and ultimate strengths of the shaft material
MPa, cr2 = 5 MPa and cr3 = -40 Mpa. For
in tension are 300 MPa and 450 MPa,
the material of the component, the tensile
respectively. The factor of safety using
yield strength is = 200 MPa. According
maximum distortion energy (Von-Mises)
cry

theory is ___ (GATE-ME-14-SET-4) to the maximum shear stress theory, the


factor of safety is (GATE-17-SET-2)
07. A machine element is subjected to the (a) 1.67 (b) 2.00
following bi-axial state of stress: crx = (c) 3.60 (d) 4.00
80MPa; cry = 20MPa; 'txy = 40MPa. If the
10. A solid circular shaft is subjected to a
shear strength of the material is 100 MPa,
bending moment M and torque T
the factor of safety as per Tresca's
simultaneously. Neglecting the effects of
maximum shear stress theory is
stress concentration, the equivalent bending
(GATE-15-Set 1)
moment is expressed as (GATE-PI-17)
(a) 1.0 (b) 2.0 (c) 2.5 (d) 3.3
(a)� (M+�M 2 +T 2 ) (b) (�+�M 2 +T 2 )
08. The principal stresses at a point inside a
solid object are = 100 MPa, = 100 (c) � (M+�M 2 +4T 2 ) (d{�+�M 2 +4T 2 )
cr1 cr2

MPa and cr3 = 0 MPa. The yield strength of

j1t11Mii40i§4ihi4RbbftM\hMij.HyderabadlDelhl1BhopaljPunelBhubaneswarl LucknowjPatnajBengaluruJChennailV\iayawadalV,zag rr.rupati I Kukalpallyj Kolkata I


SOLUTIONS
06. Ans: (d)
One Mark Solutions Sol: Te= -JM 2 +T 2 = -J32 + 4 2 = 5

01. Ans: (c)


07. Ans: (d)

02. Ans: (b)


08. Ans: (c)
-'t d3
1t
T __1_6 __ _ = _d = d
3

Sol: = 09. Ans: (b)


W 1t 2 d2
4
px-d x.f xg Sol: Distortion energy theory gives more
accurate value.
03. Ans: (a)
Load 95x100xt 10. Ans: 171 to 175
Sol: cr = = = l OOMPa.
Sol: According Von-mises theory of failure
max

Amin (100-5)x t
Ssy = 0.577Syt
04. Ans: (a) 't = 0.577 x 300 = 173.1 MPa

�'tD 3 (l - K 4 )
TH 16 11. Ans: (c)
Sol: =-----
-'tD
1t 3 Sol:
16

=l-K 4 =1
- GJ = : :

05. Ans: (c)


\, 't

y
Sol: p = 21tNT => PocT. X
60
't"

-'tD A 1t Element Y(crb = 0)


TA
3

= 16 =1
TB � (2D ) 3 8 At centre crb = 0 and torsional shear stress is
't
16 A
zero.
\( ( J IIL,111( I 1111 '. P11lil11 ,111111', �ydcrabadlDclhilBhopaI!Pune!Bhubaneswarl l.ucknowlPatnalBengalwu!ChennailV\iayawada!Vu.ag l'Hrupati I Kukalpallyl Kolkala I
.r.�
':.
J_ �

12. Ans: (a)


ACE
:�Pnhliramna : 204: Machine Design
==================================
Similarly, Minimum principal stress, cr 2
,

Sol: sr = permissible tensile stress in plates 2


crx +cry crx - cry
d = diameter of rivet cr2 = - ( ) +'t2xy
2 2
F = tensile load
t= ? cr2 =15 - 5.Ji = 7.93MPa
For teasing failure of plates,
s
F =(W -2d)tx1 =(W -2d)xtxSr According to Von-Mises yield criterion,
FS permissible value of tensile yield strength,
F
. t=
: (w -2d)xsr

22. 07 2 +7.932 +(-10) 2 -(22. 07x7.93)


Two Marks Solutions +(7.93 X -10)+(-10x 22.07)
= 27.838 MPa
01. Ans: (c) .·. Estimate value of shear stress,
Sol: According to Distortion Energy Theory
Ssy Sy
= 0.577( t)
S yt =�CJ�+ cr; -(JI (j2 FS FS

s yt = �3602 + 140 2 - (360x140) = 0.577x 27.838= 16. 07MPa


=> s yt = 314.32MPa
03. Ans: (b)

02. Ans: (b) Sol: T = �-rd 3 (Torsion of Shaft Equation)


16
Sol: Given, crx =1OMPa, crY = 20MPa,
y S
-r xy = 5MPa and crz =-lOMPa T = �( s )d 3
16 FS
Maximum principal stress, cr1 ,
(T is in N-mm, d in mm and -r in N/mm2)
2
crx +cry crx - cry 2 40
-
cr1 --- -+ ( --- ) +'t xy 50 x 103 = �x I xd 3
2 2 16 2
d = 16mm
10 20 o 20 '
= : + c ; r +5

=15+5,fj_ = 22.07MPa ,
!l1••ijj§jjji4.jl4Aflij5in.jj+yderabad I Delhi I Bhopal I Pune I Bhubancswarl Lucknow! Patna I Bengaluru I Chcnnai I Vuayawada I Vizag I Tllllpari I Kukalpally I Kolkata I

: 205: Static Loading

04. Ans: 8.434 MPa. 06. Ans : 1.7 to 1.8


.Sol: crx= -6 MPa, cry = 4 MPa, Sol: For pure shear condition
'txy = -8 MPa.
+ cr y Maximum Distortion energy theory
0"1 = --- +
O"x
2

� 't 2 + (- 't) 2 - ( 'tx- 't) = s yt


F.S
s � F.S = 300
.fix 't = yt =1.132
05. Ans: (c) F.S ../3 x lOO
Sol:
07. Ans: (b)
s yt = ½[(crx -cry Y + {cry -crz Y + (crz -cr J ] Sol: crx = 80 MPa, cry = 20 MPa,
2

FS
+3 ('t;y + 't� + 't�) 'txy = 40 MPa, Ssy = 100 MPa

Given,
For first stress element,
crx = 0 , cry = cr , crz = 0
'txy = 0 , 'tyz = 't , 'txz = 0
Syt According to maximum shear stress theory,
So = �cr2 +3 't 2
' FS
'tmax = --
ssy

For second stress element F.S

crx= 0 , cry = cr , 100


50=
F.S
'txy = 0 , 'ty z = 0 , 'txz 't F.S = 2
=

08. Ans: (c)


Sol: According to maximum shear stress theory
As
s yt same for both yield
lS 200
FS Z max = � � n = =2
2nT T 100
simultaneously.
According to Distortion Energy Theory:
s
+ai -0'10'2 = yt
2
�a-1
nv
\( I l lll.!,llll l ! l'J.., P11h!:t ,11]1)[1',. ydcrabadlDelhilBhopaIIPunelBhubancswarl LicknowlPatnalBengaiurulOiennai!VuayawadalVuag ITuupati I Kukatpallyj Kolkata
: 206: Machine Design

But cr 1 = cr2, let it is cr 1 10. Ans: (a)

�a} +a} - a} = syt


nv

llT = lly

09. Ans: (b)


Sol: cr1 = 60MPa, cr2 = 5MPa, CJ3 = -40MPa,
Syt = 200MPa

According to MSST,
s
max = yt
't 2x FS

s yt
=
(JI -(J3 FS
200
60-(-40)=
FS
200
100 = => FS = 2
FS

!IJll4j@ml4jjjjQRd§jjjj.j¥,t))HydcrabadjDclhijBhopaljPunelBhubaneswariLucknowjPatnajBengalurujChennaijVuayawadajVu.ag jTirupali I Kukalpallyj Kolkata I


Chapter
2 Fatigue
04. Fatigue strength of rod subjected to cyclic
One Mark Questions axial force is less than that of a rotating
beam of the same dimensions subjected to
O I. Stress concentration m a machine steady lateral force because (GATE-ME-92)
component of a ductile material is not so (a) axial stiffness is less than bending
harmful as it is in a brittle material because stiffness
(GATE-ME-89) (b) of absence of centrifugal effects in the
(a) in ductile material local yielding may rod
distribute stress concentration (c) the number of discontinuities
(b) ductile material has larger Young's vulnerable to fatigue are more in the
materials. rod
(c) Poisson's ratio is larger in ductile (d) at a particular time the rod has only one
materials. type of stress whereas the beam has
(d) Modulus of rigidity is larger in ductile both the tensile and compressive
materials. stresses

02. The process of shot peening increases the


05. The yield strength of a steel shaft is twice its
fatigue life of steel springs mainly because it
endurance limit. Which of the following
results in
torque fluctuations represent the most
(GATE-ME-90)
(a) surface hardening
critical situation according to Soderberg
(b) increased stiffness of the material
criterion? (GATE-ME-93)
(c) structural changes in the material
(d) residual compression at the surface (a)-T to+T (b) -T/2 to+T
(c) 0 to +T (d) +T/2 to +T
03. In a shaft with a transverse hole , as the hole
to the shaft diameter ratio --- increases 06. A static load is mounted at the centre of a
/decreases), the torsional stress shaft rotating at uniform angular velocity.
concentration factor ___ _ This shaft will be designed for
(increases/ decreases) (GATE-ME-91) (GATE-ME-02)

\( I I r1!.!_11H 1 1 111!.!; Ptil1l!{ .1tHnh �yderabad I Delhi I Bhopal I Pune I Bhubaneswari Lucknow I Patna I Bengaluru I Chennai IV\iayawada 1Vl7.3& I T,rupati I Kukatpally I Kolkata I
: 208 :
.. ==================================
...
... ,�..,_:&iginraiogPnbtiatima
�..
ACE Machine Design

(a) the maximum compressive stress (static) 1 1 . Which one of the following is the most
(b) the maximum tensile stress (static) conservative fatigue failure criterion?
( c) the maximum bending moment (static) (GATE-15 -Set 1)
( d) fatigue loading (a) Soderberg
(b) Modified Goodman
07. In terms of theoretical stress concentration ( c) ASME Elliptic
factor (kt) and fatigue stress concentration (d) Gerber
factor (kr),the notch sensitivity 'q ' is
expressed as (GATE-ME-03) 12. For the given fluctuating fatigue load, the
(a) (kr-1)/(kt-1) (b) (kr -1)/(kt+l) value of stress amp litude and stress ratio are
(c) (kt- 1 )/(kr-1) (d) (kr+ 1)/(kr+ 1) respectively.

08. The S-N curve for steel becomes asymptotic


nearly at (GATE-ME-03)
(a) 10 3cycles (b) 10 4 cycles
! 1 50 1---�---------
CIJ 1 001-------'-------+----,f-
(c) 10 6cycles (d) 10 9cycles 5 0 _____.._'--____..........__

09. In a structure subjected to fatigue loading,


Time

the minimum and maximum stresses


(GATE-15-Set 3)
developed in a cycle are 200MPa and
(a) 100 MPa and 5 (b) 250 MPa and 5
400MPa respectively. The value of stress
(c) lOO MPa and 0 .20 (d) 250 MPa and 0 .20
amplitude (in MPa) is __
(GATE -14-SET-2)
13. A machine component made of a ductile
10 . A rotating steel shaft is supported at the material is subjected to a variable loading
ends. It is subjected to a point load at the with O'min= -50MPa and O'max = 50MPa. If
center. The maximum bending stress the corrected endurance limit and the yield
developed is 1 OO MPa. If the yield, ultimate strength for the material are cr : = 1 OO MP
and corrected endurance strength of the shaft and cry = 300MPa, respectively, the factor of
material are 300 MPa, 500MPa and 200 safety is ___
MPa, respectively, then the factor of safety (GATE - 17-SET-2)
for the shaft is __. (GATE-14-SET-3)

IM•IMO§jjj§p.jjjjQnflffi\j/jjj+yderabadJ Delhi J Bhopal I Pune I BhubaneswarJ LucknowI Patna J Benga)wu I Chennai J VtjayawadaJ Vmg JTirupati I Kukatpallyl Kolkata I
: 209 : Fatigue

04. A forged steel link with uniform diameter of


Two Marks Questions 30 mm at the centre is subjected to an axial
force that verses from 40 kN in compression
to 160 kN in tension. The tensile (Su), yield
01. A ductile material having an endurance limit
(Sy) and corrected endurance (Se) strengths
of 196 N/mm2 and the yield point at 294
of steep material are 600 MPa, 420MPa and
N/mm2 is stressed under variable load. The
240 MPa respectively. The factor of safety
maximum and minimum stresses are
against fatigue endurance as per Soderberg's
147N/mm2 and 49N/mm2 • The fatigue stress
criterion is {GATE-ME-09)
concentration factor is 1.32. The available
(a) 1.26 (b) 1.37 (c) 1.45 (d) 2.00
factor of safety for this loading is
(GATE-ME-87)
05. A bar is subjected to fluctuating tensile load
(a) 3.0 (b) 1.5 (c) 1.33 (d) 4.0 from 20 kN to 100 kN. The material has
yield strength of 240 MPa and endurance
02. A cylindrical shaft is subjected to an limit in reversed bending is 160 MPa.
alternating stress of 1OOMPa. Fatigue According to the Soderberg's principle, the
strength to sustain 1000 cycles is 490MPa.If area of cross-section in mm2 of the bar for a
the corrected endurance strength is 70MPa, factor of safety of 2 is {GATE-ME-13)
estimated shaft life will be (a) 400 (b) 600
{GATE-ME-06) (c) 750 (d) 1000
(a) 1071 cycles (b) 15000 cycles
(c) 281914 cycles (d) 928643 cycles 06. In a structural member under fatigue
loading, the minimum and maximum
03. A thin spherical pressure vessel of 200mm stresses developed at the critical point are 50
diameter and Imm thickness is subjected to MPa and 150 MPa, respectively. The
an internal pressure varying from 4 to 8MPa. endurance, yield, and the ultimate strengths
Assume that the yield, ultimate and of the material are 200 MPa, 300 MPa, and
endurance strength of material are 600, 800, 400 MPa, respectively. The factor of safety
400MPa respectively. The factor of safety as using modified Goodman criterion is
per Goodman's relation is (GATE-ME-07) (GATE-16-SET-2)
3
(a) 2.0 (b) 1.6 (c) 1.4 (d) 1.2 (a) - (b) � (d) 2
2 5
\( I l 11:...,11 a , 1 1 1 1::, P11hlll ,111, 11h �ydcrabad l Dclhi l Bhopai l Punc j Bhubancswarl Luclmow l Patna l Bcnpiuru j Chennai lVtjayawadajV17.ag I Tirupati I Kukatpallyl Kolkata I
: 210 : Machine Design

07. A machine element has an ultimate strength


(cru) of 600 N/mm 2 , and endurance limit A 0.8cru
(O'en) of 250 N/mm 2
• The fatigue curve for
the element on a log-log plot is shown
B
below. If the element is to be designed for a
finite life of 10000 cycles, the maximum
amplitude of a completely reversed
operating stress is ____ N/mm 2 . 106 No. ofcycles

(GATE-17 - SET- l)

iM1•i!@mi41jjjiUflnlM\j@+yderabad ! Dellii l Bhopa] I Pune ! Bhubaneswar! Lucknowl Patnal Bcngaluru ! Chennai j Vrjayawada ! Vizag ITuupari I Kukatpallyi Kolkatal
SOLUTIONS
The stress for option (a) is more and hence
One Mark Solutions critical.

0 1 . Ans: (a) 02. Ans: (d) 06. Ans: (d)


03. Ans: Increases , Decreases 07. Ans: (a)
04. Ans: (c)
08. Ans: (c)
05. Ans: (a)
Sol: Soderberg's equation 09. Ans: 100 MPa
1 Sol: O'min = 200 MPa
-
'ta
+-
't m
=-
s se S sy Fs. ' O'max = 400 MPa
1 Stress amplitude 'cra '
-
'ta
+ --
't m
=
S se 2x S se FS 400-200
= = lOOMPa
s se 2
+-
't m
=- .
FS
't a
2
10. Ans. 2
Sol: O'max = 100 MPa

I6Tm Bending stress is a completely reversible


't m = 3 � 'tm OC
Tm. stress and for bending stress mean stress,
1t d

Tm O'rn = 0
Option Ta Tm Ta + -
2 � Stress amplitude 'cra ' = 100 MPa
a T 0 T S yt = 300 MPa, Sut = 500 MPa
3T T S e = 200 MPa, F.S = ?
b - 2T
4 4 8
From Soderberg's equation
T T
IT cr a cr m 1
2 2 4 - +-=-
S e S yt FS
3T
d
T - �T 100 = _
� F.S = 2
1
-
4 4 8
200 FS

\ ( [ l 11...,ll ll t l lll..., P11hlH ,tll<I]\', ydcrabad I Delhi I Bhopal I Punc l Bhubancswari l.ncknow! Patnal Bcngalwul Chcnnai !Vuayawada! Vu.ag ! Tuupati I Kukatpallyi Kolkata
.,4i.,....,,��Pnhliratv:ms
':.
"': \f.
ACE • • • • . -
: 212 : Machine Design

11. Ans: (a) (J' a + - 1


(J' m = -
-
Sol: Soderberg line: The most conservative, it S e S yt F.S
connects Se on aa axis with Sy on am axis. 49 98 1
=> --- + - = -
1 48.48 294 FS
12. Ans: (c) => FS = l . 50
Sol: crmin =
50 MPa
CJ'max = 2 50 MPa 02. Ans: (c)
(J'max - (J'min 2 50 - 50 Sol: Basquin's equation , A Sr LB.
100 MPa
=

CJ'a
2 2
= = =

A = 490 x 10 38 ----( 1 )
Stress ratio, A = 1 0 x 10 68 ----(2)
crm = 50 Solving 1 & 2, A = 3397, B = 0 .281
R= in � _!_ = 0.20
3397 = 100 x Lo.2si => L = 28 1026 cycles
(J'max 250 5

13. Ans: 2
03. Ans: (b)
Sol: crmin - 50MPa, CJ'max 50MPa,
Sol: D = 200mm, t 1 mm, Pmin 4 MPa
= =

Syt = 300MPa
= =

Se l OOMPa,
=

Pmax 8 MPa.
The stress is completely reversed stress and
=

Syt 600 MPa , Sut = 800 MPa ,


=

mean stress, crm = 0 and stress amplitude.


Se 400 MPa.
=

cra 50MPa.
= (YI max = - = --- = 400.MPa,
PD 8 x 200
=

(J'
se
=-
(Yh
4t 4x 1
a FS
· =
- 1 = --- = 200.MPa.
PD 4 x 200
50 = l => F.S = 2
OO (YI mm
4 4x 1
FS
cr 1 = cr 2 max = 400 MPa,
cr2 min = 200MPa.
Two Marks Solutions
(J' l m =
CJ' I max + CJ' I min
01. Ans: (b) 2
147 42 400 + 200
Sol: cr m = + = 98 MPa = = 300MPa
2 2
147 - 49
aa 49 MPa
2
= =

1 96 400-200
S = = 1 48. 48 MPa = = 1 OOMPa.
e 1 .32 2
!li11ijj§jjji4.jjjQRflnjM\jjfjj� yderabadl Delhi I Bhopal I Punc I Bhubaneswarl Lucknow! Patna! Bengaluru I OiennailVijayawadalVmg ITirupati I K!i,katpallyl Kolkat,
.!.
" , , ,.,,,, . ACE
':,
. . .
�,� PubJicap.q
: 213 : Fatigue

CF2 m = 300.MPa, CF2 0 = 100.MPa 1 00 X 1 0 3 N


(jmax
According to distortion energy theory, A mm 2
20 x 1 0 3 N
(jmin
A mm 2
= 300MPa
(jmax - (j min 80 x l0
3
40 x l 0 3
=
(j a ---=:;;.;;__2---==- N / mm 2
2A A
Goodman's equation 60 x 10 3
(jmax + (jmin
(j m = N / mm 2
(j aes -
(jmes 1 2 A
- + -=-
Se S u1 FS 1
Soderberg's equation � + crm =-
Se S y1 FS
1 00 300 _1
� + = � F.S = 1 .6.
400 800 FS

04. Ans: (a)


160 - 40 p
Sol: pm = = 60kN � cr = m
2 m A
60 x 1 0 3 N
= = 84.88N / mm2
7t (30) 2 mm2
4
06. Ans: (d)
Pa = 1 60 - (- 40) = l OO kN. Sol: crmax = 150 MPa, crmin = 50 MPa
2
� cra = 1 4 1 .47N / mm2 S e = 200 MPa, Syt = 300 MPa,
Sut = 400 MPa
Soderberg's Equation: - 1 50 - 50
cr - (jmax (jmin - - 50 MPa
2 2
3 -

(j a -
- 1
(jm = -
+ cr cr 1 50 + 50
S e S yt FS cr m = max + min = = l OO MPa
2 2
1 4 1 .47 84.88 -
1

240
+ = � F.S = 1 _26_
420 FS Goodman's equation :
(Fa (Fm 1
- + -= -
05. Ans: (d) S e S ut FS
Sol: Pmax = 1 00 kN, Pmin = 20 kN, 50 100 _1_
+ = � FS = 2
Syt = 240 MPa, Se = 1 60 MPa, 200 400 FS
A =?, F.S = 2
\( l I 1H.!, l!H t I t11:... P11hlu ,1111111 .... �yderabadj Dclhi j Bhopal j Punc! Bhubancswarl htcknowj Patnaj Bengaiuru ! Chennai jVijayawadal Vmg IT=pati I Kukatpallyj Kolkata I
...
""
, ,�
':.
...
��N,li,w.111
. A.CE . . : 214 : Machine Design

07. Ans: (385.42) 1 = 0.520x1038


Sol: Sut = 600MPa, Se = 250MPa, B = 0.094
L = 10000 cycles , sr = ? :. A = 250x106x0.094 = 916.09
0 094
Basquin's equation, : .916.09 = srL ·
A = srL8 916.09 = srxl 04x0.094
A = (0.8x600)x1038 . . . . . . . . . ( 1 ) sr = 385.42 MPa
A = 250x1068 . . • . . • • • (2)
Dividing equation (2) with equation (1)

\l I l 1 1 ... , l l l t l [ I l l � P11hl1< ,lll< ll ] , �ydcrabad I Delhi I Bhopal I Pune I Bhu�I I..ucknow I Patna I Bengaluru I Chennai I Vuayawada I Vizag I TuupatiJji�11iatpally I Kolkata I
Bolted, Riveted and
Welded joints
(a) Pre- tensioning helps to seal the
One Mark Questions pressure vessel
(b) Pre- tensioning increase the fatigue life
0 1 . Weldments in fabricated steel beams are of the bolts
designed for (GATE-ME-87) (c) ·Pre-· tensioning reduces the maximum
(a) bending stresses at the flange tensile stress in the bolts
(b) shear stresses in transverse plane (d) Pre- tensioning helps to reduce the
(c) combination of bending and shear effect of pressure pulsations in the
(d) none of these because in fabricated pressure vessels
beams welds do not get stressed .
Two Marks Questions
02. If the ratio of the diameter of rivet hole to
the pitch of rivets is 0.25, then the tearing 0 1 . A 60mm long and 6mm thick fillet weld
Efficiency of the joint is (GATE-ME-96) carries a steady load of 1 5kN along the
(a) 0.50 (b) 0.75 (c) 0.25 (d) 0.87 weld. The shear strength of the weld
material is equal to 200MPa. The factor of
03. The bolts in a rigid flanged coupling safety is (GATE-ME-06)
connecting two shafts transmitting power (a) 2.4 (b) 3.4 (c) 4.8 (d) 6.8
are subjected to (GATE-ME-96)
02. A bolted joint in shown below. The
(a) shear force and bending moment
maximum shear stress, in MPa, in bolts A
(b) axial force
and B respectively are (GA'f.E-ME-07)
(c) torsion
(d) torsion and bending moment 3 holes of M 1 0 x 1 .75 mm bolts
20] --¢ c
04. Bolts in the flanged end of pressure vessel
are usually pre- tensioned. Indicate which of
40 _!!

40_
----¢
-B) B
the following statements in not true 20- - 1 A
(GATE-ME-98)
I

111111
150

!1H•M@j(ii4.jjj!!'biMJd.jj+yderabad I Delhi I Bhopal I Pune I Bhubaneswar I Lucknow! Patna I Bengaluru I Oiennai I Vijayawada I Vmg I T1IUpati I Kukatpallyl_ Kolkata I
: 216 : Machine Design

(a) 242.6, 42.5 (b) 42.5, 242.6


(c) 42.5, 42.5 (d) 242.6, 242.6
l+--32 _......
+---32 ----+j

Statement for Linked Answer Q03 & Q04 \\


c::::====::::::t =
I OOON \
A steel bar of 10 mm x 50 mm is cantilevered 100 == ::::

-e-r ------0
== 7

with two M 12 bolts (P and Q) to support a static


load of 4 kN as shown in figure aside.
-0 -75- - - - - - - 0
(GATE-ME-08) 25

40
4 kN

-®-------EB- 1 -----------------
p Q
Direct shear stress in MPa m the most
heavily loaded rivet is
1 .8m
(a) 4.4 (b) 8.8
(c) 17.6 (d) 35.2
03. The primary and secondary shear loads on
rivet p, respectively are 06. A fillet welded joint is subjected to
(a) 2 kN, 20 kN (b) 20 kN, 2kN transverse loading F as shown in the figure.
(c) 20 kN, 0 kN (d) 0 kN, 20 kN Both legs of the fillets are of 10mm size and
the weld length is 30mm. If the allowable
04. The resultant shear stress on rivet p IS shear stress of the weld is 94MPa,
closest to considering the minimum throat area of the
(a) 132 MPa (b) 159 MPa weld, the maximum allowable transverse
(c) 178 MPa (d) 195 MPa load in kN is (GATE-ME-12)

05. A bracket shown in fig. is rigidly mounted Fee::


on wall using four rivets. Each rivet is 6 mm '
in diameter and has an effective length of
12 mm. (GATE-ME-10) (a) 14.44 (b) 17.92
(c) 19.93 (d) 22. 16

\( I l 11.
.
.,1 1 il , 1 1 1 1..: P1 1 l il11 .111 1 ,r i -. �yderabadlDelhilBhopa] IPunelBhubaneswarl LucknowlPatnalBengaJurulChennai j Vtjayawada l Vizag I Tirupati I �YI Kolkata I
. i\CE . .
:.t�&,p,c,erqPitNiramoa
'!. : 217: Bolted, Riveted & WeldedJoints
� � ==========================================
Common Data for Question 07 & 08 09. For the three bolt system shown in the
A single riveted lap joint of two similar plates as figure, the bolt material has shear yield
shown in the figure below has the following strength of 200 MPa. For a factor of safety
geometrical and material details. of 2, the minimum metric specification
required for the bolt is (GATE-14-SET2)
0
w
0
0 (a) M8 (b) M IO (c) M12 (d) M16

10. A bolt of major diameter 12mm is required


to clamp two steel plates. Cross sectional
area of the threaded portion of the bolt is
Width of the plate w = 200 mm, thickness of 84.3 mm2 • The length of the threaded
the plate t = 5 mm, number of rivets n = 3, portion in grip is 30 mm, while the length of
diameter of the rivet dr = 10 mm, diameter the unthreaded portion in grip is 8mm.
of the rivet hole dh = 11 mm, allowable Young's modulus of material is 200 GPa.
tensile stress of the platecrp = 200 MPa, The effective stiffness (in MN/m) of the bolt
in the clamped zone is ___
allowable shear crs = 100 MPa and allowable
(GATE-14-SET-4)
bearing stress of the rivet crc = 150 MPa.

11. A horizontal plate has been joined to a


07. If the rivets are to be designed to avoid
vertical post using four rivets arranged as
crushing failure, the maximum permissible
shown in the figure. The magnitude of the
load P in kN is (GATE-ME-13)
load on the worst loaded rivet (in N)
(a) 7.50 (b) 15.00
1s____ (GATE-15-Set 1)
(c) 22.50 (d) 30.00

08. If the plates are to be designed to avoid


tearing failure, the maximum permissible 0 500mm

load P in kN is (GATE-ME-13) ---o 0


(a) 83 (b) 1 25 (c) 167 (d) 50
40mm
400N

!ltllMU44AflOM!iii!a);Hyderabad l Dclhi l Bhopal i Punc i Bhubancswar i Bcngaluru l I..ucknowl Patnal Chcnnai l Vuayawada l Vizag I Tirupali I Guntur l Kukatpa))y(Hyd) I
: 218: Machine Design

1 2. A cantilever bracket is bolted to a column 1 4. A steel plate, connected to a fixed channel


using three M 1 2x l .75 bolts P, Q and R. The using three identical bolts A, B and C,
value of maximum shear stress developed in carries a load of 6 kN as shown in the figure.
the bolt P( in MPa) is ___ Considering the effect of direct load and
moment, the magnitude of resultant shear
9 kN force (in kN) on bolt C is
l Channel

3or - -c/>- :
:
4of p I

![ ::�g::---------------0--- - · - · A· - · - · B· - · - · -C- · �:
' -·-·-·-·-·-·-·-·-·-·-·- -·-
lO mm
I
I

I
thickness
3 50 50 O' 1 70

250
(GATE-15-Set 3) All dimensions are in mm

1 3. A bolted joint has four bolts arranged as (GATE-17-SET-2)


shown in figure. The cross sectional area of
each bolt is 25 mm2 • A torque T = 200 N-m (a) 1 3 (b) 1 5 (c) 1 7 (d) 30
is acting on the joint. Neglecting friction due
to clamping force, maximum shear stress in
a bolt is _ __ MPa.

q>lOOmm

(GATE - 16 - SET - 3)

111111;;;;;;;.;;;Rdfi!Mhiii+yderabad jDelhilBhopaljPune lBhubaneswarl Lucknow l Pama l Bengaluru l Cbennai l Vuayawadaj V,zag jTirupali I �I Kolkala I
SOLUTIONS
FsA = l Q x 1 50
I
x 40 = 1 8.75 kN.
One Mark Solutions 40 + O 2 + 40 2
2

F�B = 0 ( ·: rB = 0}
0 1 . Ans: (c) (Centroid of bolt group lies at B)

02. Ans: (b) Resultant force on A,


p-d d
Sol: 11 - -- = 1-- = 1 - 0 . 2 5 = 0. 7 5 F = .J3. 33 2 + 1 8.75 2 = 1 9.04 kN
p p
F
Stress on A, er A = ---
0 3 . Ans: (c) 1t
X 10
2

04. Ans: (c) 1 9.04 x 10 3 N


= = 242_5N / mm 2
7t X 10 2
4
Two Marks Solutions Stress on B
3.3 3 x 10 3 N
cr B = = 42. 4N / mm2
0 1 . Ans: (b) 7t X 10 2

Sol: P - 0.707 8 I ( !� J
0 3 . Ans: (a)
(200)
1 5 x l0 3 N = 0.707 x 6 x 60 x -­ Sol:
F.S 4KN
Q
F.S = 3 . 39

02. Ans: (a)


Sol: Primary shear force, 0.2 0.2 1 .8
10
Fs = - = 3 . 33 kN = FsB
3
Primary shear force
Secondary shear force,
F SP = F SQ = - = 2 kN
4
2

\( I l 1 1� 1 1 1 \ l 1 1 1 1..., P1 d 1 l u .1t1111i-,. Fydcrabad J Dclhi J Bhopa! J Pune J BhubaneswarJ Lucknow J Patna J Benga!uru J Chennai l Vliayawada l Vizag JTirupa!i I KublpallyJ Kolkala I
.,• . .
- .." ACE
'!. :r.,.,......Nir-n
. . : 220 : Machine Design

Secondary shear force 07. Ans: (c)


Pxe Sol: p = d r X t X crC X n
Fs1p = x r1
= 10 X 5 X 150 X 3
2 2
r1 + r2
4X 2 = 22500 N = 22.5 kN
= X 0.2 = 20 kN
0.2 2 + 0.2 2
08. Ans: (c)
04. Ans: (b) Sol: P= (w -3d h )txcr p
Sol: Fs = F�P - Fsp 20 - 2 = 18kN
(200 -3x l l) x 5 x 200
=
=

18x 1 0 = 167x5 x 200 = 1 67 kN


cr = 1 59MPa
3
=

7t X 12 2
4
09. Ans: (b)
Sol: Ssy = 200 MPa, F.S = 2
05. Ans: (b)
Sol: Direct shear stress in all the results is same Load 19 x 1 03
no.of bolts x area
't = ------ = ---
3 x tr d2
Fs = l OO = 250N 4
4
200 19 X 10
250
3

cr = = 8. 84N / mm 2· 2
=> - = ---
7t x d2
7t (6) 2 3x-
4
4
d = 8.97 �9
06. Ans: (c) :. d = M l O
Sol: S = 1 0mm,
/ = 30mm, 10. Ans: 468.827 MN/m
Ssy Sol: d = 12 mm E = 2x105 N/mm2
= 94MPa
we
't Id -FS 7t x l2 2 mm 2 ,K = ?.
- 84 . 3 mm2 ' A 2
=
A1 - =
Maximum allowable transverse load, ( in 4
'1 = 30 mm, I = 30 + 8 =38mm
kN), F = 0.707S X .f X 't weld
/2 = 8 mm
S sy
= 0.707S x .f x -
FS K = W = W = AE
= 0.707 X 10 X 30 X 94 8 (:) .e
= 1 9.93kN
W.f 1 Wx30
o.f 1 mm
84.3 X 2 X 1 05
= =
A1E1

yderahad I Delhi I Bhopal I Pune I Bhubancawarl Lucknow! Patna! Bengaluru I Chcnnai I Vgayawadal Vizag ITirupari I Kukatpally I Kolkata
., "
-:.�-·
. . . . ..
�,..,,.,,..�
AJCE : 221 : Bolted, Riveted & WeldedJoints
� � ==================================
r1 = r2 = r3 = r4 = ..J202 + 202
= ./2.
X 20= 28.28 mm

Wx8 Primary shear load,


= ---- mm
113x2x105 400
ps i = - = 100 N = Ps2 = Ps3 = Ps4
w 4
K = --- Secondary shear load,
8£ 1 + 8£ 2
Pxl
= -------- PS1, =-----
J:2 +f.2 +f.2 +r 2 xr1
( 2 �05 )[ 8��3+1:3]
I 2 3 4

400x500
= x 28.28 = 1768 N
4x 28.282
2 X 105 2 X 105
= = = 468.827
0.3558+ 0. 07079 0.4265
= 468.827 MN/m
= .J100 2 + 1 768 2 + 2 x l00 x l768 x cos 45
11. Ans: 1835 to 1845 = 1840
Sol: Worst loaded rivet is 1 or 4
12. Ans: 332 to 494

Sol: Primary load on 'P'

PS1 = 2. = 3kN
3
Secondary load on 'P'
500
)

, Pxl
400 N
PS1 = J:2 f.2 f.2 xr1
I + 2+ 3

9x250
= x30
30 +0+3022

9x250
= x30 = 37.5 kN
2x302
Resultant load,
Ps = P82I +P:I2 = ..J32 +37. 52 = 37.61 kN
P's3 't
=
l = 3 7 ·6 1 x 103 = 33 2. 63 MPa
7t d 2 7t xl 22
4 4

\(} I 11�l [ J t ! 1111..., P1,ld1! tlH•ll ... FydcrabadlDclhilBhopallPunelBhubaneswarl wcknowlPa!nalBengalurulChennailVuayawadajVizag ITirupali I Kukalpally1 Kolkala I
".,,4·
':i
. ACE
. ..
· ' ,l,..,
:&p,riqPnhlirJbl : 222: Machine Design

13. Ans: 40 (range 39.9 to 40.1)


D
Sol: T = Fx-
2

200 = Fx.2:_!
2
F = 4000 N
. 4000
=:> Shear force m each bolt = -- = 1000
4
OOO
= I = 40MPa
't

25

14. Ans: (c)


Sol:

A B C

200
50 50
6kN

Primary shear, load, Ps I = � = 2kN


3
Secondary shear, load, P:, = kr1
= r. Pxe xr1
r.
2
I + +3
r. 2
2
2

r1 = r3 = 50 & r2 = 0 & e = 250mm


6x 250
:. P's, = x 50
50 + 0 +50
2 2 2

6 X 250
= X 56 = 1 5kN
2x502
:.Resultant load on c,
Ps = Ps, + P's, = 2+15 = 17kN

FyderahadlDelhilBhopaJ I Pune JBhubancswarl ux:knowJPatnalBengaluruJ ChennailVuayawadalV:izag ITirupati I �I Kolkata I


C4 Gears
intersect common tangent to the base circles
One Mark Questions with in the points of tangency.
(a) True (b) False (GATE-ME-95)
01. For full depth of involute spur gears,
minimum number of teeth of pinion to avoid 05. Match the following: (GATE-ME-95)
interference depends upon List-I (Gear types)
(GATE-ME-88) (A) Worm gears
(a) pressure angle (b) speed ratio (B) Cross helical gears
(c) circular pitch (d) pitch diameter (C) Bevel gears
(D) Spur gears
02. Modem gear tooth profile is given involute List-II (Applications)
shape because (GATE-ME-89) 1. Parallel shafts
(a) this is a very easy . curve for 2. Non-parallel, intersecting shafts
manufacturing 3. Non-parallel, non-intersecting shafts
(b) sliding does not takes place anywhere 4. Large speed ratios
on meshing teeth
(c) involute is the only profile that gives 06. In spur gears, the circle on which the
conjugate action involute is generated is called the
(d) change in the centre distance does not (GATE-ME-96)
change gear ratio (a) pitch circle (b) clearance angle
(c) base circle (d) addendum circle
03. F o! a pinion of 15 teeth , under cutting -----­
---------( increases / decreases ) with --------­ 07. To make a worm drive reversible, it 1s
(increase I decrease ) of pressure angle. necessary to increase (GATE-ME-97)

(GATE-ME-91) (a) centre distance


(b) worm diameter factor
04. Interference in a pair of gears is avoided, if (c) number of starts
the addendum circles of both the gears (d) reduction ratio

\( I I 11�1111 ( 1111..., P11lili( ,111, 11i-. �ydcrahad l Delhi lBhopai lPunc lBhubancswar l Lucknowl l,'atna. l BcngaiuruJ Chcnnai l VuayawadaJVu.ag ITirupati I Kukatpally l Kolkata I
.., . A. CE . . Machine Design
�"••:r..,,,.,,.
, PnNirJt..111 : 224 :

08. A 1.5 kW motor is running at 1440 rev/min. (a) decreasing center distance between gears
It is to be connected to a stirrer running at 36 (b) decreasing module
rev/min. the gearing arrangement suitable (c) decreasing pressure angle
for this application is (GATE-ME-00) (d) increasing number of teeth
(a) differential gear (b) helical gear
(c) spur gear (d) worm gear Two Marks Questions

09. Large speed reductions (greater than 20) in 01. In order to test the efficiency of reducer gear
one stage of a gear train are possible through train l kW input was given at the input end
(GATE-ME-02) at a speed of 1440 rpm and at the output end
(a) Spur gearing the measured torque was 56.36 N.m. If the
(b) Worm gearing ratio of speed reduction in this unit is 10:1,
(c) Bevel gearing the efficiency is about (GATE-ME-89)
(d) Helical gearing (a) 78 % (b) 85%
(c) 63% (d) 96%
10. The minimum number of teeth on the pinion
to operate without interference in standard Common Data Questions Q. 02 and Q. 03.
full height involute teeth gear mechanism
+
with 20 ° pressure angle is (GATE-ME-02) IOOON
�---..

(a) 14 (b) 12 (c) 18 (d) 32 /T=80 RACK T=80 �

11. Two mating spur gears have 40 and 120


teeth respectively. The pm1on rotates at
1200 rpm and transmits a torque of 20N.m.
The torque transmitted by gear is
MOTOR
(GATE-ME-04)
(a) 6.6Nm (b) 20Nm A compacting machine shown in the figure below
(c) 40Nm (d) 60Nm is used to create a desired thrust force by using a
rack and pinion arrangement. The input gear in
12. Tooth interference in an external involute mounted on the motor shaft. The gears have
spur gear pair can be reduced by involute teeth of 2mm module.
(GATE-ME-10) (GATE-ME-04) (2M)
\( I l 1 1 .., 1 r 1t l 1 1 1 1 .., P1d,li, .1111,11, �ydcrabad j Delhi j Bhopal j Pune j Bhubancswarl Lucknow j Patna j Bengaluru l Chennai l VliayawadajVi.zag I T=pati I Kukatpallyj Kolka!a I
.,
��-�
" . A. CE .
. �fqprwt"ql'uNirahms
. : 225 : Gears

02. If the drive efficiency is 80% , the torque 06. The contact ratio of the contacting tooth is
required on the input shaft to create 1 OOON (a) 1 .2 1 (b) 1 .2 5
output thrust is (c) 1 .29 (d) 1 . 33
(a) 20 Nm (b) 2 5 Nm
(c) 32 Nm (d) 50 Nm 07. The resultant force on the contacting gear
tooth is N is
03. If the pressure angle of the rack is 20 ° , the (a) 77.23 (b) 212.20
force acting along the line of action between (c) 2258.1 (d) 289.43
the rack and the gear teeth is
(a) 2 50 N (b) 342 N 08. A spur gate has a module of 3mm, number
(c) 532 N (d) 600 N of teeth 16, a face width of 36mm and a
pressure angle of 20 ° .It is transmitting a
04. Twenty degree full depth involute profiled power of 3KW at 20rev/s.Taking a velocity
1 9-tooth pinion and 37-tooth gear are in factor of 1 . 5 and a form factor of 0 .3, the
mesh. If the module is 5mm, the center stress in the gear tooth is about
distance between the gear pair will be (GATE-ME-08)
(GATE-ME-06) (a) 32MPa (b) 46MPa
(a) 140mm (b) 1 50mm (c) 58MPa (d) 70MPa
(c) 280mm (d) 300mm
09. Match the type of gears with their most
Statement for Linked Answer QOS to Q07 appropriate description (GATE-ME-08)
A gear set has a pinion with 20 teeth and a gear Type of gear
with 40 teeth. The pinion runs at 30rev/s and (P) Helical
transmits a power of 20KW.The teeth are on the (Q) Spiral Bevel
20 ° full-depth system and have a module of (R) Hypoid
5mm. The length of the line of action is 19mm. (S) Rack and pinion
(GATE-ME-07) Description
1 . Axes non parallel and non intersecting
0 5. The center distance for the above gear set in 2. Axes parallel and teeth are inclined to
mm is the axis
(a) 140 (b) 1 50 (c) 1 60 (d) 1 70 3 . Axes parallel and teeth are parallel to
the axis

!li11ii@jjji4ijjQRbbiMihMij>HyderabadJ Delhi J Bhopa! J Punc I Bhubaneswarl Lucknow I Patna I Bcngaluru I Chennai JVgayawadaJ Vmg JTirupati I Kukaq,allyJ Kolkala I
4. Axes are perpendicular and intersecting 11. The tangential force transmitted (in N) is
and teeth are inclined to the axis (a) 3552 (b) 2611
5. Axes are perpendicular and used for (c) 1776 (d) 1305
large speed reduction
6. Axes parallel and one of the gears has 12. Given that the tooth geometry factor is 0.32
infinite radius and the combined effect of dynamic load
and allied factors intensifying the stress is
(a) P-2,Q-4,R-1,S-6 _ (b) P-l ,Q-4,R-5,S-6 1.5; the minimum allowable stress (in MPa)
(c) P-2,Q-6,R-4,S-2 (d) P-6,Q-3,R-1,S-5 for the gear material is
(a) 242. 0 (b) 166.5
10. One tooth of gear having 4 module and 32 (c) 121.0 (d) 74. 0
teeth is shown in the figure. Assume that the
gear tooth and the corresponding tooth space 13. A pair of spur gears with module 5 mm and
make equal intercepts on the pitch a center distance of 450mm is used for a
circumference. (GATE-ME-08) speed reduction of 5: 1. The number of teeth
on pinion is ___ (GATE-ME-14)
Pitch circle
14. A spur pinion of pitch diameter 50 mm
rotates at 200 rad/s and transmits 3 kW
power. The pressure angle of the tooth of the
pinion is 20°. Assuming that only one pair
The dimensions 'a' and 'b' respectively, are
closest to of the teeth is in contact, the total force (in
(a) 6.08 mm, 4 mm Newton) exerted by a tooth of the pinion on
(b) 6.48 mm, 4.2 mm the tooth on a mating gear is __
(GATE-ME-14-SET-2)
(c) 6.28 mm, 4.3 mm
(d) 6.28 mm, 4.1 mm 15. A helical gear with involute tooth profile has
been machined with a disc-type form gear
Statement for Linked Answer Q.11 & Q.12
milling cutter. The helical gear has 30 teeth
A 20° full depth involute spur pinion of 4mm and a helix angle of 30° . The module of the
module and 21 teeth is to transmit 15 kW at gear milling cutter is 2. The pitch circle
960rpm.Its face width is 25mm. diameter (in mm) of the helical gear
(GATE-ME-09) 1s___ (GATE - PI-16)

jltl1i!@jjji4#jjjiAffftiMjjih+yderabadlDelhi J Bhopal i Punc l BhubancswarJ Lucknow!PatnalBcngaJurul Oicnnai J VtjayawadaJVmg IT'trupati I �I Kolkata I


: 227 : Gears

1 6. A pair of spur gears with 20° full-depth


involute teeth is used to transmit 3.5 kW of
power. The pinion rotates at 700 rpm and
has pitch circle diameter of 100 mm.
Assuming a single pair of teeth in contact,
the total force acting on a gear tooth (in kN)
1s (GATE - Pl - 17)
(a) 0.347 (b) 0.954
(c) 1 . 01 6 (d) 1.302

\( ) I ! l., l l l C c 1 1 1 1 P , i l d H 111• 1 ! 1 FyderabadjDclhijBhopalj PuncjBhubaneswarl LucknowjPatnajBcnplurujOicnnaijVtjayawadajVizag j Tirupati I Kukalpally j Kolkata I


SOLUTIONS

One Mark Solutions


Two Marks Solutions
01 . Ans: (a) 02. Ans: (d)
01 . Ans: (b)
03. Ans: Decreases, Increases
1
Sol: Out-put shaft speed = 1 440x- = 144rpm
04. Ans: True 40
. output power
05. Ans: A -4, B -3, C -2 , D - 1 Effi1c1ency, 11 = -.- -- ­
mput power
06. Ans: (c) 07. Ans: (c) 21tx1 44
5636x
To ro o _
08. Ans: (d) = =_ __6_ 0_ = 84.9%
l kW lx1 0 Watts
3
1 440
Sol: Velocity ratio = = 40
36
02. Ans: (b)
09. Ans: (b) Sol:
500 N
10. Ans: (c)

Sol: -- = -- -= 1 8
2 2
sin cl> sin 2 20
Tm1n
=
2

500 N
1 1 . Ans: (d)
Sol: Tp = 40 , To = 1 20, Np = 1 200 rpm
Torque by pinion = 20 N- m.
T Thrust shared by each gear = 500 N
Torque by Gear = 20 x _Q_
Tp
Torque required
1 20 500xm z 2 500x2x80
20x- = 60N - m T2 = = = 1 00 N-m
40
=
Tl 1000 X 0. 8
1 2. Ans: (d)

\( I I 1 .., . 1 1 t ( 1 1 1..., P,ilil1t . 1 1 1 1 1 1 1 -, �ydcrabad I Delhi I Bhopal I Pune I BhubancswarJ Lucknow! Patna! Bengalwu I Oicnnai I Vtjayawada JV,zag I T'1n1pali I KnkalpaUy I KolkataI
.,...
'!,
, �,."
. . .
ACE
:r,.,.,q Nifiratirn
. . : 229: Gears

20 08. Ans: (b)


T1 = T2 x -
CO2 =
T2 x -
Z1
=100x-
co1 Z2 80 Sol: Ft SwmyCv
=

T1 Tinput 25 N -m 3x l 03 Watt 1
= = S x 36 x 3x 0.3x --
= =

X 3 X 1 6 X 20 1 .5
7t
03. Ans: (c) 1000
Sol: Fcos cj> = 500 S 46. 04MPa
=

500
F= =532 N 09. Ans: (a)
coscj>

04. Ans: (a) 10. Ans: (d)


Sol:
m{TP + T0 ) 5(19+37)
--"----= --'-----'-=140mm
a
Sol: C

D , , Pitch circle
=

2 2

05. Ans: (b)


m(TP + Ta ) = s J2o + 40 ) = l 5
Sol: C =
0mm
2 2

06. Ans: (c)


0

Sol: Contact ratio ,


Angle made by 32 teeth+ 32 tooth space
Length of action L ab
111c = ------ = --- 360° .
Pb P c cos cj>
=

360
20 = = 5.625
19mm 64
= =1.287
n x 5 x cos 20
0 = 2. 8125.
mT 4 x 32
07. Ans: (c) R = = = 64mm
2 2
Power 20 x 103
Sol: F1 = = 20.123kN a R sin0 x 2
V n x 5 x 20x 30
= =

1000 =
64 x sin(2. 81) x 2 =
6.28

F= __s__ =
2.123
cos cj> cos 20
= 2.259 kN
OE Rcos0
=
64 xcos(2. 8125) 63.9 mm
= =

b = addendum+ CE = module +(OC - OE)


°

=
4+ (64- 63.9) 4.1
=
: 230: Machine Design

11. Ans: (a)


P 15xl 03 W 15. Ans: (69.28)
= =
Sol: F1 = nx4x21 960 3552N
V --- Sol: mo = normal module = 2mm
1000 60
x-
Pitch circle diameter,
Z mn 30x2
12. Ans: (b) d= = 69.28mm
cos 'I' cos 30
=

SwmyxC v
Sol: F1 _
- Stress int ensity factor
16. Ans: (c)
S X 25 X 4 X 0.32 X 1
3552 = Sol:
1.5
S = 166.5

13. Ans: 30
Sol: m = 5 mm, C = 450 mm
Gear

T0 N p
- =- = 5 , => Ta _
-5Tp
Tp N G
cj> = 20° '
m(TP + T0 } 5(TP + 5Tp )
C = ���� => 450 = - --'----- Power = 3.5 kW
2 2
Npinion = 700 rpm
:. Tp = 30
rpinion = 50 mm = 0.05 m
14. Ans: 638.5 N 21tx N P x Ft x rP
Power =
Sol: d = 50 mm, 60
r =25 mm= 25xl 0-3m ' 21tx700xF x 0.05
3500= t
60
co = 200 rad/s.
F1 = 954.92 N
Power, P = 3 kW
Resultant force,
cj> = 20°, V = rco
954 92
3 FN = � = · = 1016.21 N
F =�= = 0.6 kN cos cj> cos 20
t V 25xl 0-3 x200
F FN = 1.016 kN
0.6 kN
Total force, F = _t_ = --°
cos¢ cos20
= 0.6385 kN = 638.5 N

\C I I 1 1 :..:, 1 1 1 1 1 r 1 1 1...:, P1 i l 1 l u ,1111 , 1 1 , �yderabad l DclhilBbopallPunclBhubaneswar l Lucknow lPatnal BcngalurulOicnnailVuayawada lVizag ITuupati I Kukatpallyl KolkataI
cs Rolling Contact Bearings

(r) Heavy radial load with impact


One Mark Questions (s) Light radial load with space limitation
(t) Heavy radial and axial load
01. The expected life of a ball bearing subjected (u) Fatigue load
to a load of 9800N and working at
1000RPM is 3000 hours. What is the 04. Spherical roller bearings are normally used
expected life of the same bearing for a (GATE-ME-92)
similar load of 4900 N and a speed of (a) for increased radial load
2000rpm (GATE-ME-87) (b) for increased thrust load
(a) Unchanged (b) 12,000 hours (c) when there is less radial load
(c) 1,500 hours (d) 6,000 hours (d) to compensate for angular misalignment

02. If the load on a ball bearing is reduced to 05. The basic load rating of a ball bearing is
half, the life of the ball bearing will (GATE-ME-98)
. (GATE-ME-88) (a) the maximum static radial load that can
(a) increase 8 times (b) increase 4 times be applied without causing any plastic
(c) increase 2 times (d) not change deformation of bearing components
(b) the radial load at which 90% of the
03. Match the rolling element bearings with the group of apparently identical bearings
most appropriate loading condition run for one million revolutions before
(GATE-ME-91) the first evidence of failure
Bearing - type (c) the maximum radial load that can be
(a) Ball bearing applied during operation without any
(b) Roller bearing plastic deformation of bearing
(c) Needle bearing components
(d) Taper roller bearing (d) a combination of radial and axial loads
Loading condition that can be applied without any plastic
(p) Tangential load deformation
(q) Radial load
\( I } 11..,![lt t !l[ll.;, P1 1l il1t .1111 )Ii', Fyderabad l Delhi l BhopaI I Pune l Bhubaneswarl Ludnowl Patna l BengaJuru l Cbennai l Vuayawadaj Vu.ag IT�pati I K�I Kolkata I
•' .,"
..,
..
. ACE
. . .
:&gmc,r,q PuNaacn : 232: Machine Design

06. Ball bearings are rated by a manufacturer for 2. Angular contact bearing
a life of 1 06 revolutions. The catalogue 3 . Taper roller bearing
rating of a particular bearing is 1 6 kN. If the 4. Hydrodynamic journal bearing
design load is 2 kN, the life of the bearing 5. Sintered metal bearing
will be P x 1 0 6 revolutions, where P is equal 6. Teflon/Nylon bush
to _____ (GATE-ME-14-SET-4)
03. The life of a ball bearing at a load of 1 0 kN
07. Which of the bearings given below is 8000 hours. Its life in hours, if the load is
SHOULD NOT be subjected to a thrust increased to 20 kN, keeping all other
load? (GATE - 16 - SET - 3) conditions same, is (GATE-ME-00)
(a) Deep groove ball bearing (a) 4000 (b) 2000 (c) 1000 (d) 500
(b) Angular contact ball bearing
(c) Cylindrical (straight) roller bearing 04. A ball bearing operating at a load F has
(d) Single row tapered roller bearing 8000hours of life. The life of the bearing, in

I Two Marks Questions I hours, when the load is doubled to 2F is


(GATE-ME-07)
(a) 8000 (b) 6000 (c) 4000 (d) 1 000
0 1 . The dynamic load capacity of 6306 bearing
is 22KN. The maximum radial load it can 05. Two identical ball bearings P and Q are
sustain to operate at 600 rev/min, for 2000 operating at loads 30kN and 45kN
hours is (GATE-ME-97) respectively. The ratio of the life of bearing
(a) 4. 16 kN (b) 3 .60 kN P to the life of bearing Q is
(c) 6.25 kN (d) 5 .29 kN (GATE-ME-11)
(a) 8 1/16 (b) 27/8 (c) 9/4 (d) 3/2
02. Match the following (GATE-ME-97)
List - I 06. For ball bearings, the fatigue life L measured
(a) Automobile wheel mounting on axle in number of revolutions and the radial load
(b) High speed grinding spindle F are related by FL 113 = K, where K is a
(c) IC-Engine connecting rod constant. It withstands a radial load of 2 kN
(d) Leaf spring eye mounting for a life of 540 million revolutions. The
load (in kN) for a life of one million
List - II
revolutions is ___ (GATE -15 -Set 3)
1 . Magneto bearing
\( I I 11�111( t 1 1 1 1� P11hlu . 1 1 1 0 1 1 , yderabad I Delhi I Bhopal I Pune I Bhubaneswar I Lucknow I Patna I Bengaluru I Chcnnai I Vtjayawada I Vu.ag ·1 Tirupati I KukatpaIIy I Kolkata
SOLUTIONS

One Mark Solutions Two Marks Solutions

01. Ans: (b) 01 . Ans: (d)


P So I•. L -- 60nL6 8 _ 72 M"ll"
1 10n rev.
Sol : � = ( 2 J
3 _ 60 x 600 x 2000 -
-
10 6
L2 P1
10

60 X 1000 X 3000 L =
(c)p => 72 = p3
3
22
=> P = 5.29kN.
3

106 = ( 4900)
3
=>
60x 2000xL h 9800
02. Ans: a-3, b-2, c-4, d-6,
106
=> Lb = 1500 x 8 = 12,000 hrs. 03 . Ans: (c)

.!:L = ( P2 ) => 8000 hrs = 20 ) => L = 1 000 hrs


3 3
Sol:
02. Ans: (a) L hrs ( I O
2
L2 P1 2
Sol: For ball bearing, life

oc ( Equivalen:radial load J' 04. Ans: (d)


L p2
Sol: -1 = ( -
8000 2
3

J => -- = (-) => L2 = 1 000 hrs


3

03. Ans: a - u , b -q , c -r , d -t. L2 P1 L2 1


04. Ans: (d) 05. Ans: (b)
05. Ans: (b)
06. Ans: 512 N
Sol: C = 16 kN , P = 2 kN Sol: �: =(;: J' =(;�)' =(¾)' = 2
:
n
L= (�) , n = 3 for ball bearing
06. Ans: 15 to 17

L= (1;)' = 512 million revolutions Sol: FL 113 = k => F1 L31 -


- F2 L32
1 1

2x(540) 1 '3 = Fx(1) 1 '3


512xl 0 revolutions = Px10
6 6
F = 1 6.286 kN
=

:. P = 512 N

07. Ans: (c)

!11••i@i!iii@4Qnbi¥11@ffyderabad l Delhi I Bhopal I Pune I Bhubancswarl Lucknow I Patna I Benga)uru l Chcnnai JVtjayawadaJVizag I Tuupati I Kukatpally l Kolkala I
Sl{ding Contact Bearings
(c) Specific dynamic capacity
One Mark Questions (d) Rotation factor

O 1 . Starting friction is low in (GATE-ME-92) I Two Marks Questions


(a) hydrostatic lubrication
(b) hydrodynamic lubrication O I . A natural feed journal bearing of diameter
(c) mixed (or semi-fluid) lubrication 50mm and length 50mm operating at 20
( d) boundary lubrication revolutions/sec carries a load of 2kN. The
lubricant used has a viscosity of 20mPa-s.
02. In thick film hydrodynamic journal bearings,
The radial clearance is 50 µm.The
the coefficient of friction (GATE-ME-96)
Sommerfeld number for the bearings is
(a) increases with increase in load
(b) is independent of load
(GATE-ME-07)
(a) 0 .062 (b) 0 . 12 5 (c) 0 .2 50 (d) 0 .785
(c) decreases with increase in load
(d) may increase or decrease with increase 02. A journal bearing has a shaft diameter of
in load 40mm and a length of 40mm. The shaft is
rotating at 20 rad/s and viscosity of the
03. To restore stable operating condition in a
lubricant is 20mPa.s.The clearance is
hydrodynamic journal bearing, when it
0 .020mm.The loss of the torque due to
encounters higher magnitude loads.
viscosity of the lubricant is approximately
(GATE-ME-97)
(GATE-ME-08)
(a) oil viscosity is to be decreased
(a) 0 .040 Nm (b) 0 .252 Nm
(b) oil viscosity is to be increased
(c) 0 .400 Nm (d) 0 .6 52 Nm
(c) oil viscosity index is to be increased
( d) oil viscosity index is to be decreased 03. A hydrodynamic journal bearing is subject
to 2000 N load at a rotational speed of 2000
04. Which one of the following is criterion in
rpm. Both bearing bore diameter and length
the design of hydrodynamic journal
are 40mm. If radial clearance is 20µm and
bearings? (GATE-ME-OS)
bearing is lubricated with an oil having
(a) Sommerfeld number
viscosity 0 .0 3 Pa.s, the Sommerfeld number
(b) Rating life
of the bearing is ___ (GATE-14-SET-l)

!ih1w,,w11111 WD kyderabad J Delhi J Bhopa! J Pune J BhubancswarJ Lucknow J Pama J Benga!uru J Chennai J Vuayawada J Vizag JTirupali I KukatpallyJ Kolkata I
SOLUTIONS
µV 2mN
= - x 2m£ x r = µx • x2m£xr
One Mark Solutions C C
= µx-rco
x2m£xr
01. Ans: (a) C
0.02 X 20 X 21t
20 X 10_3
0.020xl 0-3
= X
02. Ans: (c)
N fd X 21t X 0.02 X 0.04 X 0.02
Sol: f oc µ: oc µ ;
(·: p = :) = 0.040 N - m

03. Ans: (b) 03. Ans: 0.8


Sol: W = 2000 N, N = 2000 rpm.
04. Ans: (a)
l = d = 40 mm
C = 20x10-6 m' µ = 0.03 Pa-s
' ' Radialload(W)
Two Marks Solutions Pressure, P
projected area
W 2000
01. Ans: (b) = = = 125 x l 0 Pa
f X d 0.04 X 0.04
4

Sol:
s - (�' X�)' Sommerfeld number,

20xl 0-3 x20 25 '


= ( 3)
2 X 103
50xl 0-
0.050 X 0.050 2000
0.03 X (
60 ) 20 X 10-3 =
2

1 = ( J O8
= -= 0.125 125 X 104 20 X 10-6

02. Ans: (a)


Sol: Torque = F x r = 'txA x r

\( ( I 11�1111 l I I J H..:, P11!1ltt .1111111, FyderabadlDelhilBhopa) I Punc J Bhubancswarl Lucknow l Patnal BengaJuru J Otennai J VijayawadaJ V12.3g I T,rupati I Kukatpallyl Kolkala I
C7 Brakes
(a) Self energizing for clockwise rotation of
One Mark Questions the drum
(b) Self energ1zmg for anti-clockwise
0 1 . The force F 1 and F2 in a brake band and the rotation of the drum
direction of rotation of the drum are as (c) Self energizing for rotation m either
· shown in the figure. The coefficient of direction of the drum
friction is 0.25. The angle of wrap is 3n/2 (d) Not of the self energizing type
radians. It is given that R = 1 m and
F2 = l N. The torque (in N-m) exerted on the
Two Marks Questions
drum is ---

0 1 . In a band brake the ratio of tight side band


tension to the tension to the tension on the
slack side is 3 . If the angle of overlap of
band on the drum is 1 80° the coefficient of
(GATE - 16-SET-2) friction required between drum and the band
1s (GATE-ME-03)
02. For the brake shown in the figure, which one (a) 0.20 (b) 0.25 (c) 0.30 (d) 0.35
of the following is TRUE?
Brake shoe Statement for Linked Answer Q02 & Q03

A band brake consists of a lever attached to one


end of the band. The other end of band is fixed to
��- · - · - · - · the ground. The wheel has a radius of 200 mm
and the wrap angle of band is 270° . The braking
force applied to the lever is limited to 1 00 N and
the coefficient of friction between the band and
(GATE-16-SET - 2) the wheel is 0.5. No other information is given.

\( } I 1 1�1111 t 1111:.: P11lilll ,ll)O J I ', �yderabad j Delhij Bhopal j Pune j Bhubaneswarl Lucknowj Patnaj Bengaluru l Chennail VliayawadajVmg j1irupati I Kulwpally l .Kolkata I
: 237 : Brak.es

0 5. A band brake having band width of 80 mm,


drum diameter of 2 50 mm, coefficient of
friction 0 .2 5 and angle of· wrap of 270 .
degrees is required to exert a friction torque
1 00 mm of 1000 N-m. The maximum tension (in kN)
developed in the band is (GATE-ME-10)

--1�_1m__., I� 1m ... 1 (a) 1.88 (b) 3 . 56 (c) 6. 12 (d) 1 1 . 56

(GATE-ME-OS)
06. A force of 400N is applied to the brake
02. The maximum tension that can be generated drum of 0 . 5m diameter in a band - brake
in the band during braking is system as shown in the figure, where the
(a) 1200 N (b) 2 1 10 N wrapping angle is 1 80 ° . If the coefficient of
(c) 3224 N (d) 4420 N
friction between the drum and the band is
0 .25, the braking torque applied, in N.m is.
03. The maximum wheel torque that can be
completely braked is
(a) 200 N-m (b) 382 N-m
(c) 604 N-m (d) 844 N-m

04. A block brake shows below has a face width


(GATE-ME-12)
of 300 mm and a mean coefficient of friction
of 0 .25. For an activating force of 400 N, the (a) 100 .6 (b) 54.4
braking torque in N-m is (GATE-ME-07) (c) 22. 1 (d) 1 5.7

200mm 400 mm P 07. A drum brake is shown in the figure. The


·�
I Ill ------ 400 N
drum is rotating in anticlockwise direction.
The coefficient of friction between drum and
shoe is 0 .2. The dimensions shown in the
figure are in mm. The braking torque
(in N.m) for the brake shoe is __
(a) 30 (b) 40 (c) 4 5 (d) 60

!1J•li!@jjji4.jji@bftiiUijj� yderabad j Dellii j Bhopal j Pune j Bhubaneswarj Lucknow j Patna j Bengaluru j Chennai j Vgayawada j Vizag j Tirupati I Kukatpally j Kolkata
: 238 : Machine Design

1 000N applying the brake, the mass is brought to


rest in a distance of 0.5 m. The energy
800
480
absorbed by the brake (in kJ) is ___

(GATE-14-SET-3)

08. A mass of 2000 kg is currently being 2000 kg

lowered at a velocity of 2 mis from the drum


as shown in the figure. The mass moment of (GATE-16-SET-2)
inertia of the drum is 150 kg-m2 • On

!IHliiMhiiiihAPdtidilliii+1yderabad l Delhi l BhopallPunc lBhubaneswar l Lucknow l Pama l Bcngaluru l Chcnoai l Vuayawada l Vlllll lTuupali I Kukalpallyl Kolkata I
-SOLUTIONS

One Mark Solutions


Two Marks Solutions

01. Ans: 2.248 (range 2.2 to 2.3) 0 1 . Ans: (d)

Sol: � = 3 ' 0 = 1 80 °
Sol: -1 = eµ8 = e
R o.2sx 31t
2
= 3.248 T2
F2
1 80xrr
T
:. F1 = 3 .248N = e µ8 µ = 0 . 35
flX-
1 80 �
_J � 3=e
T2
Torque, T = (F 1 -F2)xr
= (3.248- l ) x l = 2.248 N-m 02 . Ans: (b)
Sol:
l l OO N
T
02. Ans: (a)
Sol:
I
--------� F �---'---------

lm lm

Pivot 0 = 270 ° ' T = 1 00N, µ = 0.5.


LMo = 0.

For clockwise rotation I OO x 2 = 7 x 1 � T = 200N.

IMrIVoT = 0 T T1 0.5x270x
-1 = e µ8 = -
_.2:_

=e 1 80
T 200
F x/ - RN x x + µRN x a = 0 2

(Taking T = T2 ) .
When applied moment and frictional
moment are in same direction is called self � T1 = 2 1 1 0N

energizing brake.
03. Ans: (b)
Sol: Torque, T = (T 1 - T2) r
= (2 1 1 0-200) 0.2 = 382 N-m

\( I I 1 1 . 1 1 1 1 r 1 1 1 ... 1 '1 d il i t t ! l l , i i " �yderabad I Delhi I Bhopal I Punc I Bhubancswar I Lucknow I Patna! Bcngalwu I Chennai I Vtjayawada I Vu.ag I Tuupati I Kukatpal)y I Kolkata I
.,.
,,, ......, .A!CE
V V
: 240: Machine Design
.. ' =====================================
• ' • •
� �&ipw.rqPoNiaaool

04. Ans: (c) 800


I �
1 000:t-
�1
Sol: M pivot
=0.
P x 600 = R N x 200
r ___J!:
t �
=;=;
===:::::
1 oo
4 ::::==::::!l,
8 o

l H,
=> 400x 600 = R N x 200 Fe
Ve Vs
=> RN = 1200N
Friction force, µR N = 0.25x 1200N =300N. FBD ofDrum Brake
Torque, T = 300x 0. 150 =45N.m.

05. Ans: (d)


IMa = O
Sol: Torque, T = (T1-T2) r
Vex 480 + Fe x 100-1000 x 800 = 0
lOOON-m = (T 1- T2 ) x 0. 125
=> T1 - T2 = 8000N. ---( 1) Fe = µVe = 0 .2 Ve
480Ve + 0.2Ve x lOO = 800000
T T
=e µe => _1 =e 500Ve = 800000
0.25x270x....':....
_1 1so
= 3.25 --(2)
T2 T2
Ve = 1600 N
8000 Fe = 0.2 Ve = 0.2x 1600 = 320 N
From ( 1) & (2), T2 = -- = 3555 N.
2.25
M = 0.2xFe = 0.2x320 = 64 N-m
T1 = 1 1555N.

08. Ans: 14.11 (range 14.1 to 14.3)


06. Ans: (b)
1
Sol: K.ET = - m (vi2 -v22 )
o .2sx
..!!_ = e µe => 400 = e 2
x 1 so
7t

Sol: 1 so =2 . 19
Tz T2 = 1-x 2ooox (2 2 - o) = 4000 1
T2 = 182.37N 2
:. Braking torque applied, in N.m, 1 / 2
K.ER = - l\© 1 - (02
2)
2
d
TB = (TI - T2) X -
2 _!_x 150x (2 2 - o) = 300 J
=
2
0.5
= (400 - 1 82.37) x = 54.4 N-m P.E = mgh = 2000x9.8 1x0.5 = 98 10 J
2
Total, E = 4000 + 300 + 98 10
07. Ans: 64 N-m = 14 1 10 J = 14. 1 1 kJ

Sol: FBD ofshoe bar:

!IJllibil.QRflft!Mihijj� Hyderabad j Delhi j Bhopal j Punc j Bhubancswarj Benga)wu j Lucknowl Patnal Chennai l VtjayawadajV,zag I T=pari I Gunturl Kuk.atpally(Hyd) 1
Ce One Mark Questions
Clutches
03. A disc clutch with a single friction surface
has coefficient of friction equal to 0.3. The
maximum pressure which can be imposed
01. Axial operation claw clutches having self- on the friction material is 1.5MPa. The outer
locking tooth profile (GATE-ME-87) diameter of the clutch plate is 200mm and
(a) can be disengaged at any speed its internal diameter is 100mm. Assuming
(b) can be disengaged only when unloaded uniform wear theory for the clutch plate, the
(c) can be engaged only when unloaded maximum torque (in N.m) that can be
(d) can work only with load transmitted is
(GATE--14-SET-2)
Two Marks Questions
04. A single-plate clutch has a friction disc with
01. A disk clutch is required to transmit 5 kW at inner and outer radii of 20 mm and 40 mm,
2000 rpm the disk has a friction lining with respectively. The friction lining in the disc
Coefficient of friction equal to 0.25. Bore is made in such a way that the coefficient of
radius of friction lining is equal to 25mm. friction µ varies radially as µ = 0. 01 r, where
Assume uniform contact pressure of r is in mm. The clutch needs to transmit a
1MPa. The value of outside radius of the friction torque of 18. 85 kN.mm. As per
friction lining is (GATE-ME-06) uniform pressure theory, the pressure (in
(a) 39. 4 mm (b) 49.5 mm MPa) on the disc is ----
(c) 97.9 mm (d) 142.9 mm (GATE-17-SET-2)

02. A clutch has outer and inner diameters


100mm and 40mm respectively. Assuming a
uniform pressure of 2MPa and coefficient of
friction of inner material 0.4, the torque
carrying capacity of the clutch is
(GATE-ME-08)
(a) 148 Nm (b) 196 Nm
(c) 372 Nm (d) 490 Nm

\( l l 11�\IH l I I l l� P11Ji!1t ,llJc )JI", �yderahad ! DelhilBhopallPunclBhubaneswarl LucknowlPatnalBengaiuru! ChcnnailV,jayawadalV17.ag ITlfllpati I Kukatpally i Kolkata I
SOLUTIONS
03. Ans: 530.142 N-m
One Mark Solutions Sol: µ = 0.3, Pmax = 1.5 MPa,
200 100
r = = 100mm ' r2 = - = 50mm
01. Ans: (a) I 2 2
W = 2nc (r1-r2)
Two Marks Solutions = 21txPmax xr2 (r1 -r2)
= 21t xl .5x50 (100-50) = 2nxl .5 x50 x50
01. Ans: (a) = 23561.9 N
(r +r )
60p 60 X 5 X 103 Tw = µWx 1 2
Sol·. T = = = 23.87N -m. 2
2nN 2nx2000
(lOO + 50)
W = Pxn(r/ -r; ) = 0.3 X 23561.9 X
2
r/ -r; = 530142.75 N-mm = 530. 142 N- m
T = I µw [ 2 2 ]
3 r:I -r2
04. Ans: 0.5
= f µPn(r/ - r; ) Sol: r1 = 40mm, r2 = 20mm, µ = O.O l r,
2 T = 18.85 kN-mm ,
23.87 = 0.25 X 1 X 7t(r/ = 253 ) p=c
3
-X

:. r1 = 39.41 mm Torque, 8T = µPx2nrdrxr


r,
T = P x 2nf µr 2 dr
02. Ans : (b) '2

r/ -r; r,
Sol: T = I µw ( 2 2 J
= Px2nx0.0lxf r 3 dr
3 rI r
- 2
'2

r -r
= I x 0.4 x 2 x n(rI2 -r22 ) ( ( /2 ;2)J = p X 27t X 0. 01 X [
rI4 � r24 ]
3 rt - r2

= Ix 0.4x 2xnx(503 203 ) 404 -204 ]


18.85xl 0 = Px2nx0.0lx[
3
3
4
-

= 196 N-m
� P = 0.5N/mm2
!ltllib§ifti4@iRflbiM\@1iffyderabad l Delhi lBhopal l Pune lBhubaneswarl Lucknowl Patna l BengaJuruJ CbennailVvayawadaJVmg IT1111pali I Knkatpallyl Kolkala I
Fluid Mechanics & Turbo Machinery
(Questions)
J
PageNo. 244

CONTENTS
r--- -_ -�- - -- -- � - - -- - =- - =-- - --o,__ =--=-=====
r

Chapter Questions Solutions


Name of the Chapter
..'c
No.
---- - -- --- - - - - -
Page No. Page No .
---- -- -- - -- - -

01 Property of Fluids 245 - 247 248 - 25 1

02 Fluid Statics 252 - 257 258- 263

03 Fluid Kinematics 264 - 270 271- 281

04 Fluid Dynamics 282 - 290 291 - 302

05 Laminar Flow 303- 306 307 - 312

06 Turbulent Flow 313- 316 317 - 321

07 Boundary Layer 322- 325 326 - 330

08 Dimensional analysis 331- 331 332 - 332

09 Turbo Machinery 333- 337 338 - 344


Ct Properties of Fluids
05. If 'P' is the gauge pressure within a spherical
One Mark Questions droplet, then gauge pressure within a bubble
of the same fluid and of same size will be:
01. A fluid is said to be Newtonian fluid when (GATE-PI-99)
the shear stress is (GATE-ME-95) p
(a) - (b) p (c) p (d) 2P
(a) directly proportional to the velocity 4 2
gradient 06. Kinematic viscosity of air at 20° C is given to
(b) inversely proportional to the velocity be 1.6 x 10-5 m2/s. Its kinematic viscosity at
gradient 70° C will be varying approximately
(c) independent of the velocity gradient (GATE-ME-99)
(d) none of the above (a) 2.2 x 10- m /s 5 2
(b) 1.6 x 10-5 m2/s
(c) 1.2 x 10-5 m2/s (d) 3.2 x 10-5 m2/s
02. A fluid is one which can be defined as a
substance that: (GATE-ME-96) 07. The SI unit of kinematic viscosity ( u) is:
(a) has that same shear stress at all points (GATE-ME-01)
(b) can deform indefinitely under the action (a) m /sec
2
(b) kg/m-sec
of the smallest shear force (c) m/sec2 (d) m3/sec2
(c) has the small shear stress in all directions
(d) is practically incompressible 08. An incompressible fluid (kinematic viscosity
= 7.4x10-7 m2/s, specific gravity, 0.88) is
03. The dimension of surface tension is: held between two parallel plates. If the top
(GATE-ME-96) plate is moved with a velocity of 0.5 mis
(a) ML- 1
(b) L T 1
2
while the bottom one is held stationary, the
(c) ML- 1 T 1 (d) MT2 fluid attains a linear velocity profile in the
gap of 0.5 mm between these plates; the
04. The dimension of surface tension is: shear stress in Pascal on the surface of
(GATE-Pl-97) bottom plate is: (GATE-ME-04)
(a) N/m2 (b) J/m (c) J/m2 (d) W/m (a) 65.1 (b) 0.651 (c) 6.51 (d) 651

\( I l [l�l!H ( tl[l� Ptil)lJt ,\\]l)lh �ydcrahadJ Dellii I Bhopal I Puncl Bhubaneswarl Lucknow! Patna I Bcnga!uruJChennai IVuayawadaJV17.3g JTirupati I KukatpallyJ Kolbla I
: 246: Fluid Mechanics & Turbo Machinery

09. For a Newtonian fluid: (GATE-ME-06) 02. A cubic block of side 'L' and mass 'M' is
(a) shear stress lS proportional to shear dragged over an oil film across table by a
strain string connects to a hanging block of mass
(b) rate of shear stress is proportional to 'm' as shown is fig. The Newtonian oil film
shear strain of thickness 'h' has dynamic viscosity 'µ'
(c) shear stress is proportional to rate of and the flow condition is laminar. The
shear strain acceleration due to gravity is 'g'. The steady
(d) rate of shear stress is proportional to rate state velocity 'V' of block is:
of shear strain
V

10. The difference in pressure (in N/m2) across


an air bubble of diameter 0.001 m immersed
in water (surface tension = 0.072 Nim) is
(GATE-ME-14) (S-2)

11. Assuming constant temperature condition


and air to be an ideal gas, the variation in
atmospheric pressure with height calculated
from fluid statics is (GATE- 16 - SET- 2)
Mg h Mg h
(a) linear (b) exponential (a) (b)
µL µ
2
(c) quadratic (d) cubic
mgh mgh
(c) (d)
µL µ
2

Two Marks Questions


03. Oil in a hydraulic cylinder is compressed
01. Newton's law of viscosity states that the from an initial volume 2m3 to 1.96 m3 . If the
shear stress in a fluid is proportional to: pressure of oil in the cylinder changes from
(a) The velocity of the fluid. 40 MPa to 80 MPa during compression, the
(b) The time rate of change of velocity of bulk modulus of elasticity of oil is .....
the fluid. (GATE-PI-07)
(c) The rate of change of velocity of the (a) lOOO MPa (b) 2000 MPa
fluid with the height of fluid film. (c) 4000 MPa (d) 8000MPa
(d) The square of the velocity of the fluid.

!M•l4j@ml4@MRflnj@jj1h+yderabadlDclhilBhopa)IPunclBhubancswarlLucknow!Patna!BcngaiurulChcnnailVtjayawadalVu.ag ITirupari i l{ukaq,allyl KoUwal


: 247: Properties of Fluids

04. A journal bearing has a shaft diameter of 40 07. Which of the following statement are TRUE,
mm and a length of 40mm. The shaft is when the cavitation parameter cr = O?
rotating at 20 rad/s and the viscosity of the (i) The local pressure is reduced to vapor
lubricant is 20 mPa-s. The clearance is pressure.
0.020 mm. The loss of torque due to the (ii) Cavitation starts
viscosity of the lubricant is approximately: (iii) Boiling of liquid starts
(GATE-ME-OS) (iv) Cavitations stops
(a) 0.040 Nm (b) 0.252 Nm (GATE-15-Set 3)
(c) 0.400 Nm (d) 0.652 Nm (a) (i),(ii) and (iv)
(b) only (ii) and (iii)
05. A lightly loaded full journal bearing has (c) only (i) and (iii)
journal diameter of 50 mm, bush bore of (d) (i), (ii) and (iii)
50.50 mm and bush length of 20 mm. If
rotational speed of journal is 1200 rpm and 08. Consider fluid flow between two infinite
average viscosity of liquid lubricant is 0.3 horizontal plates which are parallel (the gap
Pa-sec, the power loss (in Watt) will be : between them being 50 mm). The top plate is
(GATE-ME-10) sliding parallel to the stationary bottom plate
(a) 37 (b) 74 (c) 118 (d) 237 at a speed of 3 mis. The flow between the
plates is solely due to the motion of the top
06. In a simple concentric shaft-bearing plate. The force per unit area (magnitude)
arrangement, the lubricant flows in the 2 mm required to maintain the bottom plate
gap between the shaft and the bearing. The stationary is ____ N/m2 .
flow may be assumed to be a plane Couette Viscosity of the fluid µ = 0.44 kg/m-s and
flow with zero pressure gradient. The density p = 888 kg/m3 .
diameter of the shaft is 100 mm and its (GATE-16 - SET- 2)
tangential speed is lOm/s. The dynamic
viscosity of the lubricant is O.lkg/m.s. The
frictional resisting force (in Newton) per
100mm length of the bearing is ----
(GATE-ME-14) (S-1)

Fyderabadl Delhi I Bhopal I Pune I Bhubaneswarl Lucknow[ Patna! Bengalurul Oiennai IVtjayawadalVizag IT,rupati I Kukatpal)yl Kolkata I
SOLUTIONS

One Mark Solutions

01. Ans : (a)


dv
ffint: -r=µ.­
Newtonian fluid
dv
'tOC-
00Bubble Droplet

dy dy 06. Ans: (a)


Sol: p = po (1 - J3T)
Where J3 =1/ 273
dv
dy 20
1-
= 2 = 1,246
P20
/J
02. Ans: (b) P10 1--
273
For gauge,
03. Ans: (d)
Sol: Surface tension (cr) µ = µo + at - J3t2
= Force per unit length (N/m) Where, µo = 1.7 x 10-5 poise
kg m Cl= 5.6 X 10-S
= sec = � = M1LoT -2 J3 = 0.1189 10-9
2

sec 2
X
m
µ 20 = l.7x10- + 5.6x10- x20 - 0.1189x10- x20
5 8 9 2

l.75x10-5 + 5.6x10-8 x70 - 0.1189x10-9 x70 2


04. Ans :(c)
µ70

-5
Sol: Surface tension is also defined as the surface = l .807244x 10 = 0.88863 12
energy per unit area 2.033739 X 10-5
. _ Energy
1.e. cr - --- (J/ m2)
Area

= l
xl.246xl.6xl0-5
05. Ans : (d) 0.88863 12
Sol: A bubble will have two surface layers where = 2.24x10-5
as a spherical droplet will have only one.
Hence, Pbubble =2 x Pdroplet

l Delhi I Bhopal l·Pune I Bh�I Lucknow IPatna I Bengaluru I Chennai IVliayawadalVu.ag IT1111pari I Kukatpallyl K.olkala I
Fyderabad
ACE
�� �===================
EF..ugiomiog : 249: Properties of Fluids
�-� Pnbmnn
===============
07. Ans: (a) 09. Ans: (c)
du Sol: According to Newton's law of viscosity, a
Sol: Shear stress, t = µ-
dy Newtonian fluid is one type of fluid, m
F du F dy which shear stress is proportional to rate of
-=µ-�µ=--
A dy A du shear strain.(i.e, velocity of gradient
. N m
µ-Umts = -x-­
m ml 2
/sec
2

kg.
% sec-x m 10. Ans: 288 N/m2
=---'---':....::....::.. --
m 4cr 4x 0.072
2

/sec Sol: �p= = = 288N / m 2


ml
D 0.001
2

= kg/ m -sec
. . . . dynamic vis cos ity
Kmematlc v1scos1ty ( v ) = 11. Ans: (b)
mass density
µ kg/ m-sec
v= (
p kg/ m 3 J
Two Marks Solutions

:. v-units = m2/sec
01. Ans :(c) [ t oc :; ]
08. Ans: (b)
Sol: v = 7.4 x 10-7 m2/sec,
02. Ans : (c)
s = 0.88� p = 0.88x1000 = 880 kg/m3
Sol: Newton'sLaw of Viscosity
V = 0.5 mis,
y = 0.5 mm = 0.5 x 10-3 m. Area(A) =L2

t=? -v
According to Newton'sLaw of Viscosity -�1- M
---------------
T
_ ·-----
du
= µ. - = (v xp) (V/y) ---------------·µ -----
dy �T
t
/71///&//777///////)7
0·5
= (7.4 X 10-7 x880) ( F=µ. A.
0. 5 X 10-3 J dy
du

V
= 0.651 N/m2 (Pascal) T = µ .Lz . h

!Mllih§hi4iihii01biMIIMl4 yderabad I Delhi I Bhopal I Pune I Bhubaneswarl 1.ncknow.1 Patna I Bengaluru I Chennai I Vuayawada I Vmg I TllUpati I Kukaq,ally I Kolkata
"�.... . .
�-......"' ACE..
�F�P.ffraiml : 250: F1uid Mechanics & Turbo Machinery

Clearance (y) = 0.020 mm = 0.02 x 10-3 m


:. mg =µ.L2.-
V

h Loss ofTorque (T) in N - m = ?


mgh Torque = Force x Radius
...
µL2
V=
d V d
T= F X -= (µ.A-)-
Free Body Diagrams 2 y 2
T
- µ( 1tdl i('; } �
X 20

J
0.04
0.04
j V (Steady
m
Velocity) 2
=(2ox10- )(1t x0.04 x0.04) [
3
-3 x-
0.02 xlO 2

mg = 0.04 (N.m)

Where T = Tension in string = mg


05. Ans: (a)
21tNT
Sol: Power loss = ---
60
2 1t N
= (F r )
60
SO -40 2 1t N
= (µ A Jr
V

2-�.96
K= 2000 M Pa 60 y
( )
=

1td N
( )
2 1t N ( 60 d
04. Ans: (a) µ 1td£)
60 2
=
(D�d)
Sol: Given Data:
Diameter ofshaft (d) = 40 mm 2x 7tX 1200
= 40 x 10-3 m = 0.04 m
= ---- X 0.3 X ( 7t X 0.05 x0.02 )
60
Length ofshaft (l) = 40 mm = 0.04 m 7tX 0.05 X 1200
Shaft angular speed (ro) = 20 rad/ s 60 0.05
X x--
Viscosity oflubricant (µ) = 20 m Pa-S 0.050 -0.05
( � )
= 20 milli Pascal - second
= 20 X 10-3 Pa-sec = 37 Watts

= 20 x 10-3 �2
- sec
m

!1D1ii@jjj4i@iRflGiMh/fjj+yderabadlDelhilBhopallPunclBhubanesw.irl LucknowlPatnalBcngalurulCbcnnailVuayawadajVizag ITuupati I �YI Kolkala I


06. Ans: 15.7 N Where, cr = 0 , P = Pv
Sol: By Newton's law of viscosity At P = Pv cavitation
V
= µ.- (formation of vapour bubbles or boiling) will
start.
J
h
't

� F = µ.A.: [·: -r = :
08. Ans: 26.4 (26.3 to 26.5)
V µV 0. 44 x3
F = µ (1tDL) Sol: !_ = -r= = = 26. 4 N/m2
h A y 50xl 0-3
(IO)
F = (O. l) x ( nxO. l xO. l)
(0. 002)
= 15.7 N

07. Ans: (d)


Sol: Cavitation parameter or cavitation number is
given by
= P-PV
1
-p y2
Q"

!ltl1ih§jjj4i@Mllflbi@jjil!*J:Iyderahad I Delhi I Bhopal I Pune I Bhubaneswar I Lucknow I Patna I Bengaluru I Chennai I VijayawadajVmg I Ttrupati I Kukatpally I Kolkata I
Fluid Statics
1! __:;,.---------------------------·
_:_:_:_:_:-:_:-:_:_:_:-: _:_:_:_: _:-:_:_:_:_: _:_:_:_:_
One Mark Questions �
ell
:-:--:--:--:--:-:--:--:--:--:-:-:-:- --:-:- --:-:-:
._- - - - -_- - - - -_-_- - - - - - - - - - - - - - - -.
-:-:-:-:-:-:-:-:-:-
- - - - - - - - - - -

. ,I
01. Shown below are three cylindrical gates pipe H2 0
which restrain water in a 2-D channel. Which
gate experiences the maximum vertical
component, the minimum vertical component
Hg

and the maximum horizontal component of (a) 1236 Pa (b) 1333 Pa


the hydrostatic force? (GATE-ME-93) (c) Zero (d) 98 Pa

04. A static fluid can have (GATE-ME-2001)


(a) Non-zero normal and shear stress
(b) Negative normal stress and zero shear
stress
(c) Positive normal stress and zero shear
stress
(d) Zero normal stress and non-zero shear
stress
(B) (C)
02. Bodies m floatation to be in stable 05. A cylindrical body of cross - sectional area
equilibrium the necessary and sufficient A, height H and density P s , is immersed to a
condition is that the centre of gravity is depth h in a liquid of density p, and tied to
located below the _ _ (GATE-ME-94) the bottom with a string. The tension in the
_
string is (GATE-ME-03)
03. A mercury manometer is used to measure the
static pressure at a point in a water pipe as
shown in Fig. The level difference of
h
mercury in the two limbs is 10 mm. The
gauge pressure at that point is (GATE-ME-96) ::::::::::::::::: :strin
:--------- g::::::::
---------
-------- ---------
\( I l 11J!,111l l 1111J!, P11lili( .111011-. yderabadl Delhi I Bhopalj Punel Bhubancswarj Lucknow! Patna! Bengaluru I Chennai IVuayawadalVuag ITrrupati I Kukatpallyl Kolkata
"':.t" . .
ACE
.... ,..,�Pmw:alioos
. . : 253: Fluid Statics

(a) pghA (b) (Ps - p)ghA


(c) (p- Ps)ghA (d) (ph - PsH)gA Two Marks Questions

06. For the stability of a floating body, under the 01. A circular plate 1 m in diameter is
influence of gravity alone, which of the submerged vertically in water such that its
following is TRUE? (GATE-ME-10) upper edge is 8m below the free surface of
(a) Metacentre should be below centre of water. The total hydrostatic pressure force on
gravity. one side ofplate is: (GATE-ME-88)
(b) Metacentre should be above centre of (a) 6.7 kN (b) 65.4 kN
gravity. (c) 45.0 kN (d) 77.0 kN
(c) Metacentre and centre ofgravity must lie
on the same horizontal line. 02. The cross-sectional area of one limb.of a U­
(d) Metacentre and centre ofgravity must lie tube manometer [figure shown below] is
on the same vertical line. made 500 time larger than the other, so that
the pressure difference between the two
07. For a completely submerged body with limbs can be determined by measuring 'h' on
centre of gravity 'G' and centre of buoyancy one limb of the manometer. The percentage
'B', the condition ofstability will be error involved is: (GATE-ME-90)
(GATE-ME-14)(S-1)
(a) G is located below B
(b) G is located above B h _.
..,. . j_ . :
(c) G and B are coincident Initial.....,_-=----= .=,;:;.,=:..-=-
I :�:
=

level =:=:=:=:=:=:=
(d) Independent ofthe locations ofG and B
----------
----------·
----------
08. For a floating body, buoyant force acts at the
(GATE-16-SET-1)
(a) centroid ofthe floating body (a) 1.0 (b) 0.5
(b) center ofgravity ofthe body (c) 0.2 (d) 0.05
(c) centroid of the fluid vertically below the
body 03. The force F needed to support the liquid of
(d) centroid ofthe displaced fluid density d and the vessel on top (fig) is
(GATE-ME-95)

!IJ11ii@bi44ijjiRftn1Mijj.jj+yderabadlDelhilBbopaljPunel Bhubaneswarl I..ucknowlPatnalBengaiurujChcnnaijVgayawadalV,zag jTirupati I Kukalpallyj Kolkata I


"'.
,��
�.,
.." . ACE
.. ..��PnbJi,,ihooa
. . . : 254: Fluid Mechanics & Turbo Machinery

05. In given figure, if the pressure of gas in bulb


Tube of
area =a A is 50 cm Hg vacuum and Pann = 76 cm Hg,

then height ofcolumn H is equal to
(GATE-ME-00)
_ Vessel h H
Pann -
�"77'7771'777tJ j_ !
---------- = JH
- - --------
::::::::::::::::::::::::::: Hg :::::::::::
-----------------------
Frictionless piston -----------------------
ofarea = A
(a) 26 cm (b) 50 cm
(a) gd [ ha - (H - h) A] (c) 76 cm (d) 126 cm
(b) gdHA
(c) gdHa 06. The horizontal and vertical hydrostatic forces
(d) gd ( H - h)A F x and Fy on the semi-circular gate, having a
width 'w' into the plane offigure, are
04. Refer to figure, the absolute pressure of gas
A in the bulb is (GATE-ME-97)
---1·
(GATE-ME-01)

===========wF:- y :========: - -
-----
------ --
----- ------·
.----- r__ J.
h

:=:=:=:=:=:-:-:-:-:-:Fx : :
------ -----
--- ----

- . 10 cm

B -.: --· A
:: 5cm
(a) Fx = pghrw and Fy = 0
D (b) Fx = 2 pghrw and Fy = 0
(c) Fx = 2 pghrw and Fy = pgwr2/ 2
p = 13.6 g/ ml
(d) Fx = 2 pghrw and F = 7t pgwr2/ 2 y
(a) 771.2 mm Hg
(b) 752.65 mm Hg 07. The pressure gauges G 1 and G2 installed on
(c) 767.35 mm Hg the system shown pressures ofPG! = 5.00 bar
(d) 748.8 mm Hg' and P02 = 1.00 bar. The value of unknown
pressure P is (GATE-ME-04)

!ltili!i§hiiiihiRflijiM\j@*)HyderabadlDclhilBhopaliPunejBhubaneswarlLucknowlPatnalBengalurujChennailVuaya�jVu.ag jT1IUpari I Kukatpallyj Kolkata I


ACE : 255: Fluid Statics
�-�
• • • •
,; ....�PoNir:Pxw
w �

(a) 5000 (b) 6600


(c) 7546 (d) 9623

p 10. A spherical balloon with a diameter of 10m,


shown in the figure below is used for
Atmospheric Pressure is 1.01 bar
advertisements. The balloon is filled with
(a) 1.01 bar (b) 2.01 bar
helium (RHe=2.08 kJ/kg.K) at ambient
(c) 5.00 bar (d) 7.01 bar
conditions of 15° C and 100 kPa. Assuming
08 A two dimensional fluid element rotates like no disturbances due to wind, the maximum
a rigid body. At a point with in the element, allowable weight (in Newton) of balloon
the pressure is 1 unit. Radius of the Mohr's material and rope required to avoid the fall of
circle, charactering the state of stress at the the balloon (Rair = 0.289kJ/kg.K) is ---
point, is (GATE-ME-08) (GATE-ME-14)
(a) 0.5 unit (b) O unit
(c) 1 unit (d) 2 units

09. A hinged gate of length 5 m, inclined at 30°


with the horizontal and with water mass on
its left, is shown in the figure below. Density
of water is 1000 kg/m3 • The minimum mass 11. An inverted U-tube manometer is used to
of the gate in kg per unit width measure the pressure difference between two
(perpendicular to the plane of paper), pipes A and B, as shown in the figure. Pipe A
required to keep it closed is (GATE-ME-13) is carrying oil (Specific gravity = 0.8) and
Pipe B is carrying water. The densities of air
and water are1.16 kg/m3 and 1000 kg/m3 ,
respectively. The pressure difference
between pipes A and B is ___kPa.
Acceleration due to gravity g = 10 m!s2

!1J1ii!@jjji4@iRflnfMidii!lt ydcrabad I Delhi I Bhopal I Punc I Bhubaneswarl Lucknow! Patna! Bcngaiuru I Chcnnai I Vuayawada I Vi.zag I TllUpati I K.ukatpalJy I Kolkata
density p up to the tip of the plug. The
Air gravitational acceleration is g. Neglect the
effect of the atmospheric pressure.
A +-:

L.i�----_- -_-_-_-d b-_"'l::: l -� "l


+ './"<::'
Water
F
-:::--'-:-�=1:,===.-
�:-:-:- � :-:-:-:� °/� - -.
:tF ��
Sectional view A-A A�
oil
(GATE-16-SET-l)
12. The large vessel shown in the figure contains The force F required to hold the plug in its
oil and water. A body is submerged at the position is (GATE - 16-SET-2)
interface of oil and water such that 45
(b) 2pR 2gL(l + :)
percent of its volume is in oil while the rest is
in water. The density of the body is ___
2
(d) pR 2 gL
1t
kg/m3 •
The specific gravity of oil is 0.7 and density
of water is 1 000 kg/m3 • 14. A 2m x 2m square opening in a vertical wall
Acceleration due to gravity g = 1 0 m/s2 .
is covered with a metallic plate of the same
dimensions as shown in the figure below.
Consider the acceleration due to gravity to be
1 0.0 m/s2 • The force (in kN) exerted by water
on the plate is ____
(GATE-16-SET-2) Pa1rn

:-:-:-:-:-:-= 2m -:-:-:-
1 3 . Consider a frictionless, massless and leak­ ------- ----- vertical wall

proof blocking a rectangular hole of 2m x 2m


dimensions 2R xL at the bottom of an open ------- ----- square plate

tank as shown in the figure. The head of the


plug has the shape of a semi-cylinder of ------------- vertical wall

radius R. The tank is filled with a liquid of (GATE - PI-16)

\( l l 1 1l.!, 1 1 1 t ( 1 1 1 1 l.!. P11lil1t ,ll1oih �yderabad I Delhi I Bhopal I Pune I Bhubaneswar I Lucknow I Patna I Benga)uru I Chennai I Vtjayawada I Vu.ag I Tirupati I Kukatpally I Kolk.ata I
: 257 : Fluid Sta.tics

1 5. For the stability of a floating body the


(GATE- 17 - SET- 2)
(a) centre of buoyancy must coincide with Five Marks Questions
the centre of gravity
(b) centre of buoyancy must be above the 0 1 . A cylinder of mass 1 0 kg and area of cross­
centre of gravity section 0. 1 m2 is tied down with string in a
(c) centre of gravity must be above the vessel containing two liquids as shown in
centre of buoyancy Figure. Calculate gauge pressure on the
(d) metacentre must be above the centre of cylinder bottom and the tension in the string.
gravity Density of water = 1 000 kg/m3 • Specific
gravity of A = 0.8. Specific gravity of B
1 6. A manometer is used for the pressure (water) = 1 .0. (GATE-ME-98)
measurement in a closed tank. The three
fluids fl , f2 and f3 have specific weights y, 10.!kg
I

2y and 0.5y, respectively. The schematic I

Fm
arrangement with manometric readings and O.lm A

other dimensions are shown in the Figure. Water


In order to ensure zero gauge pressure in the B
- string
tank at the mid-height level (h/2), the height
of the tank h (in m) is ___

h/21
Fluid fl
·=-=-:Fluid f3 -=-:
h/2I -----------

0.6m -·

0 2"'.t(c

Fluid t2
(GATE- PI- 17)

!ltlli!l§Oi@iijjjj@fl!Mihfjj&yderabad I Delhi I Bhopal I Pune I Bhubaneswar I wcknowl Patna I Bengaluru I Chennai I Vtjayawada J Vmg I 'firupati I Kukatpal)y I Kolkata I
SOLUTIONS
-------- - - - - - - - - - - - - - - - - - - - -
One Mark Solutions water --+- --------- --- - -- - - - - - - - - - - -
-_-_ -------- ---------------
01. Ans :
Sol: Maximum vertical component
D
lS

experienced by (A)
Minimum vertical component is experienced E
by (C)
Hg
Maximum horizontal component 1s
experienced by (C)
hw = Gauge pressure of water in pipe
02. Ans: Meta Centre.

03. Ans: (a)


Sol: Given data: hw = 0.01 (1�· 6 - 1)
Fluid in pipe - water
hw = 0.126 m of water
Fluid in manometer - Mercury
Gauge pressure (in pa) = Pw .g.hw
= 1000 X 9. 81 X 0. 126
Let Pw = mass density of water
= 1000 kg / m3
= 1236 Pa
:. Sw = I (specific gravity of water)
Note :
Pttg = Mass density of mercury The height 'AB' is not given. neglecting this
kg the answer is (a).
= 13600 J

If water column of total ABC is neglected


m
= 1 3 .6 ( specific gravity of mercury)
then answer is (b).
:. SHg

x = Difference in mercury level in limb of


However the more appropriate answer is (a).
manometer = 10 mm
10
x = -- m=O.Ol m
1000

!IJl•i!@jjjiUijjjijmbiffih@+1ydcrabadjDclhi I Bhopal I Punel Bhubaneswarj Lucknow I Patna! Bcngaluru I Chcnnai j Vtjayawada j V17.ag j Tirupari I Kukatpally j Kolkata. I
: 259 : Fluid Statics

04. A ns : (b) 07. A ns: (a)


Sol: In static fluid shear stress zero because Sol: Condition of equilibrium of a "sub -m erg ed
relative motion is not present. Pressure is b ody" is that 'B' is above 'G'. (i.e., 'G' is
present which is normal to the surface and located below 'B')
always compressive in nature. Compressive
normal stress is considered negative.

05. A ns : (d)
Sol: Let T = Tension in string (N)
W = mg = Ps ·AH.g
w
08. A ns: (d)
Jc"A
Sol: For floating body Buoyancy force acts
through the centre ofbuoyancy which is C.G

�:;J;L - ---- ====== for displaced volume.

Two Marks Solutions

F8 = Buoyancy force (up thrust)


= Weight ofliquid displaced 01. A ns : (b)
FB Buoyancy force = p.Ah. g Sol:
V Free water level
=

FB = W+T
FB = mg + T
p. A.h.g = Ps -AH.g + T y
T = p A h g - Ps . AH. h
T =(p h - p s H) A g

F
06 . A ns : (b)
Sol: For a stable floating body the Metacentric
D= 1 m ; y = 8 m;
height is positive, hence Metacentre should
be above centre of gravity of the floating h =vertical depth of C.G. of plate from free

body. water surface

!l1•1i@jjji4jjjjjRflbiMihii1+1yderabad l Delhil Bhopall Punel Bhubaneswarl Luckno� I Patna l Bengalwui Chennail Vtjayawad
a j Vizag I Tuupari I Kukatpallyl Kolkata I
PAABs + 1000 x 9. 81 x 0.17
F = p. g. h . A = p.g (y +�) . : D 2
13600 X 9.8 } 0.02 + 1000 X 9.8 } X 0.05
+ 13600 X 9. 81 X 0.76
=

1000 X 9. 81 (8 + �) X : (1) 2 = 65.4 kN


X

PAABs 102. 88 X 103 N/m2


=
=

pA ABs SpHg gLHg


02. Ans : (c)
=

102. 88 X 103 = 13600 X 9. 8 } hH g


Sol: Given Data
Area of left limb = 500 x Area of small limb h HgAbs = 0.771 m of Hg
X

A = 500 a = 771 mm of Hg (abs)


Volume fall in larger limb = volume rise in
small limb [byLaw of conservation of mass] 05. Ans: (b)
A.Afi = a.h [mass remains constant] Sol: PA = Gauge pressure in bulb
�h !!_ = _ a_ _I_ = A = 50cm Hg vacuum
= = 0.002
gauge

h A 500 a 500
Afi
-= 0. 002= 0.002 X 100%= 0. 2%
h

03. Ans: (b)


Sol: Force = Pressure x Area
---------------------
----------------------
---------- - - -------- -
----------------------
----------- ----------
= pg h xA = d.g. H. A= gdHA
p a1m =Atmoshpheric pressure= 76cm of Hg
04. Ans: (a) :. pressure at surface=P a1m
Sol: According to principle of manometry :
Using principle of manometry
palm = H + PA
O = H + (-50)
P =760 mm of Hg atm
i
.·. H= 50cm of Hg

06. Ans : (d)


Water

Sol: Fx = Horizontal hydrostatic force


_Datum line
= Pressure x projected area of contact
y = 1 3600 kg/m3 = p gh x (2r x w)
\( l I 11�111l l 1 1 1 1;., 1'11lil1{ ,t1H111" �ydcrabad j Delhi ! Bhopal I Ptmc I Bhubancswarl Lucknow! Patna I Bcngaluru l Oiennai IVliayawadajV11.3g ITrrupari I Kukatpally l Kolkata I
: 261 : Fluid Statics

Fx= 2 p g h r w Stress Element at point is


Fy= Vertical hydrostatic force
Fy = Weight of fluid above the semi-circular
gate
= mg = ( p V) x g = p g x V

Where, V= Volume of fluid above plate


= p g x (els area of semi- circular Radius of Mohr circle

plate x Width)

Fy = n pg wr 2 / 2
09. Ans: (d)
Sol:

07. Ans : (d)


Sol: Given data:-
Gauge pressure reading, P = 5bar
Gauge pressure reading, P0 2 = 1.00 bar
01

Atmospheric pressure, Patm = 1.01 bar


Unknown gauge pressure, P= Pa, + P0 2
= 5 + 1 = 6 bar
h = centre of pressure = h + _c=."---
• - I G.sin 2 0
Unknown absolute pressure
= Gauge pressure + Patm
Ah

=6+ 1.01= 7.01 bar = 1.25 + ----"-'


12cc__�-�
3
lx5 . 2 300

(1 X 5Xl.25)
-- X Slil

08. Ans :(b) = l .25+0.417 = 1.667 m


Sol: In rigid body motion, velocity uniform. As h. 1.667
C= = = 3.334m
velocity is uniform velocity gradient is zero sin 30 sin 30 °
Moment of forces about hinge 'O' for
°

. dv dv
1.e. - = 0, as - =0 , shear stress 't =O.

If 't=O, pressure is uniform in all directions


equilibrium'
F x L° =Mg x x
dy dy

at a point. [where F = p.g.h.A (Per unit length)]


\( I I ll�llll t 1 111:.:, P11lilH .il1u11-. yderabad I Delhi I Bhopal I Pune I Bhubaneswar I Lucknow I Pama I Bengalwu I Oiennai IV\iayawada ! Vu.ag I Tirupati I Kukatpal)y I Kolkat,
.,
�-�
..
"' . .
. ACE
:&ip,mqPublic,arims
. : 262 : Fluid Mechanics & Turbo Machinecy

F = I000x9.81xl .25x(1x5) = 61312.5 N/width 11. Ans: 2.2 (range 2.19 to 2.21)
X= 2.5 X COS 30 ° = 2.165 m Sol: PA - (Poil X g X 0.2) - (Pair X g X 0.08) + (prog
61312.5 x 3.334 = M x 9.81 x 2.165 x 0.38) - Pa = 0
=> M = 9624.7 kg/unit width Pa - PA = 2.2 kPa
Nearly 9623 kg/unit width
12. Ans: 865 (860 t o 870)
10. Ans : 5306 N Sol: pbxg.V = Poil xgx 0.45 V + Pwaterxgx0.55 V
Sol: => Pb = 865 kg/m
3

Wsallon + WHe
13. Ans: (a)
Sol:
Rope
C D
Fe AIR

T = Tension in rope

PV = mRT
A E

P = pRT F

P 100000 Force required to balance hydrostatic force


P tte = =
R HeT 2080 x (273 + 15) Fv = [(Volume)Rectangle - (Volume)semicircle)]xpg
PHe = 0.167 kg/m3 R2
Fv = [ 2R x R x L -2:.__L ]xpg
P 100000
2
P Air = =
R Air T (0.289 x l 0 3 )(273 + 15 = 2pR2gL [1- ;J
= 1.2 kg/m3
Under limiting equilibrium (TRope=O)
14. Ans: (120)
Waalloon + Rope + WHe + TRope = Fa Air
Sol: F = pgAh = I 000x10x 2x 2x (2 + 1)
Waalloon+Rope + PHe VBallon• g PAir- Vaallon g
120 kN
=
=

W Ballon+Rope = (p Air - p He )vBalloong


15. Ans: (d)
= (1.2 - 0.167) x-rc(5) x9.81
3 4
3 Sol: For stability of floating bodies GM > 0 or M
= 5306 N must be above G.

!MINM@hillih§jmni&ijjfjj+yderabad I Delhi I Bhopal I Pune I Bhubaneswar I Lucknow I Patna I Bengaluru I Chennai I Vuayawada I Vizag I TU11pati I Kukalpally I Kolkata I
.,:.t
..
" ACE . .
_, &ip,,,erqPoolir,n,n
. . : 263 : F1uid Statics

16. Ans: 2
Sol: Let P be the point at the mid- height level of
the tank.

Fluid A
Water
(B)
- string

Density of liquid, A = 0.8 1 000 x

= 800 kg/m3
Y2 =2y Density of liquid, B = 1 1 000 = 1 000 kg/m3
x

Mass of liquid 'A' displaced by body


The pressure at point P can be written as, mA = p A VA
x

pp = palm + y 1.2 - 2V 0.2 - 0.5,{0.6 + �) = P A A h A = 800 O.l x 0. 1


x x x = 8kg
Mass of liquid 'B' displace by body
X X

or (PP ) - Pa1m = y[l.2 - 0.4 - 0.3 - 0.25h] m 8 = p 8 V8 x = lOOO O.l 0. 1 25 = 1 2.5 kg


x x

(Pp \a ge
u
= y[0.5 - 0.25h] Total mass of liquid displaced
= 8 + 1 2.5 = 20.5kg
According to problem, (Pp)gauge = 0
or 0.5 - 0.25 h = 0 FB = Upward thrust = Weight of liquids
or h=2m displaced = 20.5 9.8 1 = 201.1 (N)
x

Weight of cylinder
W = 1 0 9.8 1 = 98. 1 (N)
x

:. Net upward in thrust = 201 . 1 - 98.1


Five Marks Solutions
T = 1 03(N)
01. i.e. Tension in string (T) = 1 03 (N)
Sol: Given data Gauge Pressure on cylinder at bottom,
Thrust
Mass of cylinder (m) = 1 0kg
Area of cylinder
C/s area of cylinder (A) = O. l m2
20 l.l kN
Density of water (pw ) 1 000 kg I m 3 201 1 N/m 2 2.0 1 1
0.1 m2
= = =
=

\( 'I I 11L',1 1 1 t t 11r1g P11hlu ,11111n.., �ydcrabad l Delhi l BhopaI I Punc 1 Bhubancswar l Lucknow l Palna l BcngaJurul Chennai j Vtjayawada j Vizag ITuupali I Kukalpally l Kolkatl I
Fluid Kinematics
(a) Fluid is in continuum
(b) Fluid is irrotational
(c) Fluid is ideal
One Mark Questions

0 1 Streamlines, path lines and streak lines are (d) Fluid is compressible
virtually identical for (GATE-ME-94)
(a) Uniform flow 05. Circulation is defined as line integral of
(b) flow of ideal fluids tangential component of velocity about a
(c) Steady flow ___ (fill in the blank)
(d) Non uniform flow (GATE- ME-94)

02. In a flow field the stream lines and 06. A fluid flow is represented by the velocity
equipotential lines
field V = ax i + ay j , where a constant. The
(GATE-ME-94)
(a) Are parallel
-t -t -t

equation of stream line passing through a


(b) Cut at any angle
point ( 1 , 2) is
(c) Are orthogonal every where in the field
(GATE-ME-04)

(d) Cut orthogonal except at the stagnation (a) x - 2y = O (b) 2x + y = 0


points (c) 2x-y = 0 (d) X + 2y = 0

03. For a fluid element in a two dimensional 07. The velocity components in the x and y
flow field ( x - y plane), it will undergo directions of a two dimensional potential

flow are u and v, respectively. Then au 1s


(GATE- ME-94)
(a) Translation only oy
(b) Translation and rotation equal to (GATE-ME-OS)
(c) Translation and deformation av av
(d) Deformation only ox ox
(a) (b) - -

(d) - ­
av av
04. Existence of velocity potential implies that
(c)
fJy ay
(GATE- ME-94)

\( I ( 11!.!,!IH t I lllg P11i>lh ,lllt>JI', yderabad I Delhi I Bhopal I Pune I Bhubaneswar I Lucknow I Patna I Bengalwu I Chennai I Vyayawada I Vizag I Tuupati I Kukatpally I Kolkata
: 265 : Fluid Kinematics

08. In a two-dimensional velocity field with 11. For a continuity equation given V.V = 0 to be
velocities u and v along x and y directions
valid, V where is the velocity vector, which
respectively, the convective acceleration
one of the following is a necessary
along the x-direction is given by
condition? (GATE- ME-08)
(GATE- ME-06)
(a) Steady flow
(a) u
ou + v ou , (b) Irrotational flow
ox oy
(c) Inviscid flow
ov + v ou
(c) u ox oy (d) Incompressible flow

12. A streamline and an equipotential line in a


09. A two-dimensional flow filed has velocities
flow field (GATE� ME-11)
along the x and y directions given by
(a) are parallel to each other
u = x 2 t and v = - 2xyt respectively, where t (b) are perpendicular to each other
is time. The equation of streamline is (c) intersect at an acute angle
(GATE- ME-06) (d) are identical
(a) x2y = constant
(b) x y2 = constant
13. For an incompressible flow field, V , which
(c) x y = constant
one of the following conditions must be
(d) not possible to determine
satisfied ? (GATE- ME-14) (S-2)
10. In a steady flow through a nozzle, the flow (a) V.V = 0
velocity on the nozzle axis is given by
u =u 0 (1 + 3x I L) , where x is the distance (b) V x V = O

along the axis of the nozzle from its inlet (c{v.v )v = O (d)
av (___.. )___..
+ V .V V = O
plane and L is the length of the nozzle. The at

time required for a fluid particle on the axis


to travel from the inlet to the exit plane of the 14. A flow field which has only convective
nozzle is (GATE-ME-07) acceleration is (GATE- ME-14) (S-4)
(a) a steady uniform flow
L
(a) - (b) � In 4 (b) an unsteady uniform flow
3u 0
(c) a steady non-uniform flow
Uo

L L
(c) - (d) -- (d) an unsteady non-uniform flow
4u 0 2.5u 0

l@•lii@hiUl•hiMdbi@!iih+iyderabad j Dellii j Bhopa) j Punc! Bhubancswar l Lucknow j Pama j Bcngaluru j Chennai jVgayawada j V,zag I Tirupati I Kukatpal]y j Kolkatal
1 5. If the fluid velocity for a potential flow is 1 8. For a certain two-dimensional
given by V(x,y) = u(x,y)i + v(x,y)j with incompressible flow, velocity field is given
usual notations, then the slope of potential by 2xy i - y 2 J The streamlines for this flow
line at (x,y) is (GATE -1 5 -Set 2) are given by the family of curves
u v u
(a) - (b) -- (c) - (d)
u v
(GATE-16-SET-3)
u2
2

(a) x y constant (b) xy2 = constant


V
2 2=

(c) 2xy - y2 = constant (d) xy = constant


16. The volumetric flow rate (per unit depth)
between two streamlines having stream 1 9. Consider the two-dimensional velocity field
functions 'l'I & '1'2 is given by
(GATE-16-SET-2) V = (5 + a,x + b1 y)i + (4 + a 2 x + b2 y)J,
ca) I"', + 'l' 2 I (b) \jl I \jf2 where a1, b1, a2 and b2 are constants. Which
(d) 1'1' , -'1' 2 1 one of the following conditions needs to be
satisfied for the flow to be incompressible?
(C) \jl 1/\jf2

1 7. A channel of width 450 mm branches into


(GATE-17 - SET-1)
(a) a 1 + b1 = 0 (b) a1 + b2 = 0
sub-channels having width 300 mm and 200
(c) a2 + b2 = 0 (d) a2 + b 1 = 0
mm as shown in figure. If the volumetric
flow rate (taking unit depth) of an
incompressible flow through the main
channel is 0.9 m3/s and the velocity in the
sub-channel of width 200 mm is 3 mis, the
Two Marks Questions

velocity in the sub-channel of width 300 mm


0 1 . A Newtonian fluid has the following velocity
1s ----. Assume both inlet and outlet
field:
to be at the same elevation.
V= x 2 y i + 2 xy 2 z j - yz3 k
- I\ I\ I\

The rate of shear deformation at the point


x=- 2, y=- land z=2 for the given flow is:
E yz
Width = 450mm
Flow rate = 0.9m3/s

(GATE-ME-88)
Width
= 200mm
(a) -6 (b) -2 (c) - 1 2 (d) 4
Velocity = 3m/s

(GATE - 16-SET - 3)
!11•110Yhiiiii!AINIMM\hih� yderabad! Delhil Bhopal ! Punc l Bhubaneswarl l.ucknowl Pa!nal Bcngaluru l Chcnnai l Vuayawadal Vu.ag ITirupati I Kukaq,allyl Ko1ka1a
"...
'!.
. .
.., .....,....:" E.,.,,.,,.
ACE
PnNirarima . . : 267 : Fluid Kinematics

02. The stream function in a two dimensional 06. For a fluid flow through a divergent pipe of
flow field is given by \jl=x 2 -y 2 length L having inlet and outlet radii of
The magnitude of the velocity at point ( 1 , 1 ) R 1 and R 2 respectively and a constant flow
1s (GATE-ME-89) rate of Q, assuming the velocity to be axial
and uniform at any cross- section, the
(a) 2 (b) 2-,J2 (c) 4 (d) 8
acceleration at the exit is
03. A velocity field is given as (GATE-ME-93) (GATE- ME-04)
-V = 3 x y i - 6 xyzk
2
/\ /\

(a)
2Q (R 1 -�2 )
(b)
2Q 2 (R I - R 2 )
1tLR 2 1tLR 2
3

Where x, y, z are in m and V mis. Determine


2Q 2 (R 1 -R 2 ) 2Q 2 (R 2 -R I )
if (c) (d)
7t 2 LR 2 s 1t2 LR/
(i) It represents an incompressible flow
(ii) The flow is irrotational
(iii) The flow is steady. 07 . A closed cylinder having a radius R and
(a) (i)and (iii) (b) (i) and (ii) height H is filled with oil of density p. If the
(c) (ii) and (iii) (d) (i) only cylinder is rotated about its axis at an angular
velocity ofro, then thrust at the bottom ofthe
04. The velocity components in the x and y cylinder is (GATE-ME-04)
directions are given by u =Axy - x y ,
3 2
(a) 1tR 2 pgH

v = xy 2 _ y 4 . The value of A for a possible


i p ro_
2
R2
4 (b) 1tR 2 + .:._ _
4
flow field involving an incompressible fluid
(c) 1tR2 ( pro2 R2 + pgH)
(GATE- ME-95)
p ro R2
lS
2
3 (d) 1tR [ + pgH
(b) - -
4 4
2
(a) - - (c) (d) 3 4 )
4 3 3
05. The 2 D flow with, velocity 08. A leaf is caught in a whirlpool. At a given
; = (x +2y+2)i + (4-y)J lS instant, the leaf is at a distance of 1 20 m
(GATE- ME-01) from the centre of the whirlpool. The
(a) Compressible and irrotational whirlpool can be described by the following
(b) Compressible and not irrotational velocity distribution;
(c) Incompressible and irrotational
(d) Incompressible and not irrotational
\C l l 11�1 1 1t , 1 1 1 1 � P11l1l1< .1111111, Fydcrabad I Delhi! Bhopal I Pune J BhubancswarJ Lu�w J Patna J Bcngaluru I Oiennai J Vtiayawada J Vu.ag JTuupati I KuliaipallyJ Kolkala I
: 268 : Fluid Mechanics & Turbo Machinery

flows out radially. The fluid is assumed to be


and v, - (3 �� ' }vs.
0 0
incompressible and inviscid. (GATE- ME-08)

Where r (in meters) is the distance from the :--R Moving


centre of the whirlpool. What will be the
� Circular plate
W/f////$
�1' I
t
distance of the leaf from the centre when it T
h V
I

:
has moved through halfa revolution? j_
I

(GATE- ME-OS)
Y/
.//////,Q/
' ///M- stationary
Surface
(a) 48 m (b) 64 m
(c) 120 m (d) 142 m 10. The radial velocity V r , at any radius r, when
the gap width is h, is
09. Which combination of the following Vr
(a) V
statements about steady incompressible 2h
=
r

forced vortex flow is correct? Vh


(d) V , = -
(GATE-ME-07)
P : Shear stress is zero at all points in the
flow. 11. The radial component of the fluid
Q : Vorticity is zero at all points in the flow. acceleration at r = R is
R : Velocity is directly proportional to the 3V 2 R V2R
(a) (b)
radius from the centre ofthe vortex. 4h 2 4h 2
S : Total mechanical energy per unit mass is V2R V 2h
(c) (d)
constant in the entire flow field. 2h 2 4R 2

(a) P and Q (b) R and S


12. You are asked to evaluate assorted fluid
(c) P and R (d) P and S
flows for their suitability in a given
laboratory application. The following three
Stateme nt for Li nked Answer QlO & Q l l
flow choices. Expressed in terms of the two­
The gap between a moving circular plate and a
dimensional velocity fields in the x- y plane,
stationary surface 1s being continuously
reduced, as the circular plate comes down at a
are made available.
P: u = 2y, v = -3 x
uniform speed V towards the stationary bottom
Q: u = 3xy, v=O
surface, as shown in the figure. In the process,
the fluid contained between the two plates R: u = - 2x, v = 2y

!M1•i@jjjl4.jj4Nflldih.jj..-Iyderabad I Delhi I Bhopal I Pune I Bhubaneswar I wcknow I Patna I Bengaluru I Chennai IVtjayawada JV,zag J Tuupati I KukatpalJy I Kolkata I
: 269 : Fluid Kinematics

Which flows should be recommended when 14. Velocity vector of a flow fields is given as
the application requires the flow to be
V = 2xy i - x 2 z j . The vorticity vector at
-t A /\

incompressible and irrotational?


(1, 1, 1) is (GATE- ME-10)
(GATE- ME-09)
(a) P and R (b) Q only (a) 4 i - j (b) 4 i - k
A A A A

(c) Q and R (d) R only


(c) i - 4 j (d) i - 4k
A A A A

13. Consider steady flow of water in a situation


15. Consider the following statements regarding
where two pipe lines (pipe 1 and pipe 2)
streamline(s)
combine into a single pipe line (pipe-3) as
(i) It is a continuous line such that the
shown in the figure. The cross-sectional areas
tangent at any point on it shows the
of all three pipelines are constant. The
velocity vector at that point
following data is given (GATE- PI-09)
(ii) There is no flow across streamlines
..' dx dy dz . .
Pip e Ar ea (mz) V elocity 1 1erentla1 equation
(m) -= - = - 1s the d·f-c:
numb er (m/s)
U V W

of a streamline, where u, v and w are


1 1 1
velocities in directions x, y and z,
2 2 2
respectively
3 2.5 ?
(iv) In an unsteady flow, the path of a
particle is a streamline
PIPE 1
Which one of the following combinations of
7 PIPE 3 the statements is true?
(GATE- ME-14) (S-4)
�-----

(a) (i), (ii), (iv) (b) (ii), (iii), (iv)


PIPE 2

(C) (i), (iii), (iv) (d) (i), (ii), (iii)


Assuming the water properties and the
velocities to be uniform across the cross
section of the inlets and the outlet, the exit 16. Consider a velocity field V = K&t + xk),
velocity (in mis) in pipe 3 is where K is a constant. The vorticity , nz , is
(a) 1 (b) 1 .5 (c) 2 (d) 2.5 (GATE- ME-14) (S-4)
(a) -K (b) K (c) -K /2 (d) K /2

i
!IJl•i!@jjji4jjji4AflfiiM\111jj+yderabadj Delhij BhopaljPunejBhubancswar LucknowjPatnajBcngalurujOiennaijVtjayawadajVmg jTirupati I Kukatpallyj Kolkata I
: 270: Fluid Mechanics & Turbo Machinery

1 7. Match the following pairs: (3) y-component ofacceleration,


Pr oduct Pr oc ess ay -y
I Incompressible (x + y 2 J
p V x V =O
- 2

Continuity equation (4) x-component ofacceleration,


Q V.V =O II Steady flow (x + y)
aX = -2
-

R III Irrotational flow ( x + y 2 )2


DV
-
=O
Dt (GATE - 16 - SET - 3)
IV Zero acceleration of (a) (2) and (3) (b) ( 1) and (3)
-
s av =O
fluid particle (c) ( 1 ) and (2) (d) (3) and (4)

(a) P-IV, Q-I, R-II, S-III


20. For a steady flow, the velocity field is
(b) P-IV, Q-III, R-I, S-II
(c) P-III, Q-I, R-IV, S-II
V = (- x 2 + 3y ft + (2xy)] . The magnitude of

(d) P-III, Q-I, R-II, S-IV the acceleration ofa particle at ( 1 , -1) is
(GATE-17-SET-l)
1 8. The velocity field on an incompressible flow (a) 2 (b) 1 (c) 2,Js (d) 0
is given by
V = (a1x+a2y+a3z)i + (b1x+b2y+b3z)j
+ (c1 x+c2y+c3z)k,
where a 1 = 2 and c3 = --4. The value of b2
IS---
-
(GATE-15 -Set 1)

1 9. For a two-dimensional flow, the velocity


y �
field is U= + J, where
- X ,:-- -=-

+y + y2
1
2 2 1 2
X X

and J are the basis vectors m the x-y


Cartesian coordinate system.
Identify the CORREC T statements from
below.
( 1 ) The flow is incompressible
(2) The flow is unsteady

!IH1ii@jjji4.jjQRflntM\u.jj+yderabad I Delhi I Bhopal I Pune I Bhubaneswar I LucknowI Patna I Bengaluru I Chcnnai IV\iayawada I Vmg I TlfUpati I Kukatpally I Kolkata I
SOLUTIONS
Where u = ax
One Mark Solutions v = ay
Substitute u and v in above Eqn
dx = dy
01. Ans: (c) ax ay
Integrating both sides
02. Ans: (d) dx = dy
Sol: The product of slopes of tangents at the point J X J y
of inter section m1m2 = -1 log e x = log e y + C (constant)
Where C is integration constant
y log e X = log e y + log e C
x i y =c
Using coordinates (1, 2)
C=- 1
=-
y 2
X

03. Ans: (c)


Sol: For general 2 -D flow fluid element can have 2x - y = 0
translation, deformation (linear as well as
shear) and rotation also. However if the flow 07 . Ans : (a)
is potential flow then angular velocity is Sol: Potential flow means irrotational flow. For
zero. i.e., no rotation. an irrotational flow cross velocity gradients
. au av
are equa1 1.e. - = -
04. Ans : (b) 8y ox

05. Ans : Closed contour 08. Ans : (a)


Sol: In a three dimensional velocity flow field,
06. Ans: (c) the convective acceleration is defined as the
dx = dy rate of change of velocity due to change of
Sol: Equation of stream line is
U V position of fluid particles in a fluid flow. The
\( } } lll.!;IIIC l IIll� P11lil11 .111011, I I I I I I I I I I I
�yderabad Delhi Bhopal Pune Bhubaneswar Lucknow! Patna Bengaluru Chennai Vuayawada j Vizag Ttrupati Kukatpal]y Kolkata I
.., . . .
" . ACE
-�:�N,liranme
'!.,.
: 272 : Fluid Mechanics & Turbo Machinery

acceleration (convective) along x, y and z 10. Ans: (b)


direction are as follows. Sol:
au au au
a x =u - + v - + w -
Bx By 8z
av av av Exit
a = u - + v- + w - - _..,. .
Y ax By az inlet
aw aw aw
az = u - + v - + w -
ax By 8z
au
For 2 - D w - = 0
' az
I· L ·I
3
au + V -
au U = u 0 (l + �)
.". a x = U. -
ax ay dx x
= u O (l + 3 )
u=
dt L
09. Ans : (a) Separating the variables and integrating both
Sol: Given data. sides
u=x t2
& v = -2 xyt
dx dy
- =-
U V

dx dy dx dy
= => - = --
x t - 2xyt
2
X - 2y

By integrating both sides and obtain


2 In x ==. - In y + const.
In x2y = In c
3x
L
� [log e (l + ) ] = T
:. x2y = constant. 3u0 L 0
Here the slope of stream line function is not
� [log e (1 + 3) - log e (1 + 0)] = T
a function of time ( ! =-
2
;) . So
3u o

_!'.._ [log e (4) - log e (1) ] =T


streamline does not vary w.r.to time. Thus it 3u o
is a steady flow. So it is possible to J::_log e ( i) =T
3u o 1
determine streamline equation.
L
T = - log e (4)
3u o
IM•liiiYhiiUlbiRdbiilhih� yderabad I Delhi I Bhopal I Pune I Bhubancswar I Incknow I Patna I Bcngaluru I Chennai ! Vuayawada I Vizag I Tuupati I Kukalpal)y I Kolkata
".,.
.. ,�"" ACE . . . .
: 273: Fluid Kinematics
11. Ans: (d)
':, ��Poblicatir.m
16. Ans: (d)
Sol: V. v = 0 means divergence of velocity vector

1s zero. 1. e. - + -+ - = 0
. . ou av ow 17. Ans: 1 (ra nge 0.99 t o 1.01)
ox &y oz Sol: A1V1 = A 2V2 + A3V3
This equation is valid for incompressible Q , = Q2 + Q3
flow only valid for both steady and unsteady 0. 9 = 3 x 0.2 x 1 + V x 0.3 x 1
flow. V = 1 m/sec

12. Ans: (b) 18. Ans: (b)


dx dy
=
13. Ans: (a)
Sol:

dx - dy-
U V

14. Ans: (c) 2xy -y2


-- =
Sol: Total Acceleration (a)
R,n-.Jx = -Rny + c
=Local acceleration + Convective
acceleration Rn(Fx y)= c
-.Jx y= c
a = - + u-
ou ou
8t ox Squaring on both sides
For steady non-uniform flow, local :=> xy2 = c
acceleration is zero but velocity varies form
point to point. Hence convective acceleration
19. Ans: (b)
exist.
Sol: for incompressibility
ou + ov = O
. ox &y
15. Ans: (b)
Sol: Equation strea!Illine, _E_ (5 + a 1 x + b1 y ) + _E_(4 + a 2 x + b 2 y ) = 0
dx dy
ox &y
-
U
=-
V
or

dy
( ) = -1
dx potential line

\( l I 11g11 1t t 1 1 11� P11l d11 , 1 1 1 t , 1 1 , �yderabadl Delhi l BbopaljPunel Bhubancswarl �!>wl Patnal Benga1urul Oienna.ilVuayawadalV17.ag ITuupati I Kukatpally l Kol.kata I
...
... ,......��PnNiafirm
'!.
.,. .
..." . ACE
. . . : 274 : Fluid Mechanics & Turbo Machine:ry

03. Ans: (a)


Sol: Given data:
Two Marks Solutions
V =(3x 2 y )i -(6xyz)k
01. Ans:(c) (i) The flow is incompressible, if it satisfies
Sol: Given data continuity equation
Velocity field ofa Newtonian fluid au av ow
- + - + - =0
V = x 2 y i + 2xy 2 z i - yz 3 k
� " " "
ax ay az
The rate of shear deformation Evz at :. � (3 x 2 y ) + �(o) + � (- 6xyz)
(- 2, -1, 2) =?
ax ay az
=> 6xy + O+ (-6xy) => 6xy -6xy = 0
, ,, - ,,, - {;; + :) - µs,, The flow velocity components satisfied law
of conservation of mass i.e., continuity
Hence, shear strain rate ,
equation hence the given flow field
OW av
= - + - = -z + 2xy 3 2
represents an "incompressible flow".
ay az
t yz

At x = -2 , y = -1 , z = 2 (ii) Ch eck for irr otati o nal fl ow:


= -(2) + 2 (- 2) (- 1) 2
k
J
3
t yz
1

= - 8 - 4 = -12 a a a
ax ay az
u V w
02. Ans : (b)
Sol: Given data
Stream function in a 2-D flow field ,
'V = x 2 - y 2

The magnitude ofthe velocity at point (1, 1)= ?


1
=- [0 + 6yz]
We know 8\j/ = -u · 8\j/ = v
WY
2
ay , ax
=> ; (x 2 - y 2 ) = -u ; => ! (x 2 - y 2 ) = v
W Y :;it Q

Hence flow is "rotational"


:. U = 2y = 2, V= 2x = 2 =
Short Cut : lf
az ow
au

ax
JvJ = .J2 2 + 2 2 = 2J2
The flow is irrotational.

jltl1i@jjj04jjj/NRftGIM\j/.jj.yderabad l Delhi l Bhopa!IPunelBhubaneswar l Lucknow l Patna l Bengaiuru l Chennai l Vuayawada l Vmg J Tirupati I �I Kolbta. J
: 275: Fluid Kinematics
(iii) The given flow is steady since the velocity
components of each does not contain time't'
(i.e., "t" appears explicitly in case of un­ Hence flow is "not irrotational" 1.e.,
steady flow). Hence given flow field is rotational.
"steady flow"
06. Ans : (c)
Sol: Given data:
04. Ans : (d)
Let Q = Discharge (Volume flow rate) in m3/sec
Sol: For steady, incompressible flow, the
R 1 = Inlet Radius (m)
au av
continuity equation - + - =0 must be R 2 = Outlet Radius (m)
ax ay
satisfied.
y
au av
=').., y - 2xy ,
3 = 2x y - 3 y 3
� •
ax ay X

:. ').., y3 -Zxy +Zxy - 3y3 = O


� y3 ( ').., - 3) = 0
Continuity equation: Q = A1V1 = A2V2
:. ').., = 3
y2 - Q - Q_
A2
__
nR;
__

05. Ans: (d)


Sol: Given Data
u =x + 2y + 2
v =4 - y

:. au = 1 + o + o =1
ax
av
... = 0 - 1 =-1
ay
:. The Equation ofcontinuity is
au + av = 0 satisfied
ax ay
Then fluid is "incompressible"

ro , = ½ (: - : J
!lfl1ij@jjjl4.jj4UfiniM!jjjjj� ydcrabad I Delhi I Bhopal I Pune I Bhubaneswar l Lucknow! Patna I Bcnga)uru I Chennai I Vgayawada I Vizag I Tnupati I Kukatpally I Kolkara
: 276 : Fluid Mechanics & Turbo Machinery

FRotation =Pressure intensity x Area of ring


Where Pressure "P" change w.r.t "r"
dp p V 2 p (rro) 2
dr r r
= = = p r ro 2

At x= L (exit) f dp = p ro fr dr2

- 32Q 2 (D2 - DJ p ro 2 r 2
n2 L(D2 )s P = ---

2 Q 2 (R 1 - R 2 ) p ro 2 r 2
a 2 = ----- = p X dA = 2m dr
1t 2 R 2s L
d FRotation
2
X

= p ro 2 7tf r 3 dr
07. Ans: (d)
Sol: Given data R 4 p 2 . 1tR 4 00
= p ro 2 . 1t- = ----
Thrust on the bottom of the cylinder due to 4 4
rotation is equal to sum the weight of oil on Thrust on cylinder = W + FRotation
the bottom cylinder and total force due to n R4
4
rotation p g H. 1t R 2 + p ro 2 --
T = W + F Rotation 00 2 R 2 ]
Where W = mg = p.v.g
= [
p 1t R 2 gH +
4
= p.A.H.gV
pro 2 R 2
V = (Volume = Area x Height) = 1tR 2 [ + pgH ]
4
= p.1t R 2 . H.g = p g H. 1t R 2

08. Ans: (b)


R
---- -- -- Sol: Given data:
---------
---- ---- Radial distance (r) = 120 m
---------
Radial velocity distribution of leaf,

0)
v, = { 6 ;� ' ) m i s
0 0

Angular velocity distribution of leaf,

---- --- - v, =( 30�� o ' )m/ s

!lfl1ijj@il4.jjjiRbGiM!!iii!i>8yderabad I Delhi I Bhopal I Pune I Bhubaneswar I Lucknow I Patna I Benga)uru I Chennai I Vuayawada ! V17.3g ITirupati I Kuk.atpally I Kolkata I
: 277 : Fluid Kinematics
Let 0 = Angle turned by a leaf 09. Ans : (b)
Sol: F or st eady, i nc ompr essibl e forc ed v ort ex
For e = _!_ revolution (i.e 180 ° = 1t radians) ,
2 fl ow:
the radial distance of leaf from centre ? Forced vortex is defined as that type of
dr vortex flow, in which external torque is
We know, Vr = = Rate of change of radial
dt required to rotate the fluid mass. The fluid
distance of leaf
de mass rotates at constant velocity (angular co).
V8 = r.co = r - = Rate of change of angular 1. Velocity is directly proportional to the
dt
displacement of leaf radius from the centre of the vortex

Given Vr = -(
60 x 10 3
2m
dr
J = - (1)
dt -
IV rI oc

i.e., Statement R is correct


Ve = (
300 x 10 3
2m
Dividing Eq (1) and Eq (2)
= r.
de
J
dt ___
(2)
2. Total energy per unit mass is constant for
the given control volume,
dr i.e., Statement S is correct

J
dt =-( 60 X 10 + ( 300 X 10
J 3. Forced vortex for real fluids. Hence
3 3

r. - Vorticity exist at all points in flow field


de 2m 2m
dt
i.e., Statement Q is wrong
-dr- -
- 60 -
-1
= -= 4. Shear stress is always exist due to torque
r.de 300 5
acting on mass of fluid
dr -1
= de i.e, Statement P is wrong
r 5
Integrating both sides, with m boundary
10. Ans : (a) & 11. Ans: (a)
limits
Sol: Given data :
j drr = 5-=-!
o
I de Let V = Downward velocity of circular
120
plate (in m/sec)
[log (r) ]R -1
120 = - [0lo R = Radius of circular plate (m)
5
1t

h = Gap (width) between the moving


( R ) -1
log - = - (1t -O)
R
[ :. - = e s ]
-lt

plate and stationary plate.


1 20 5 120
Yr = Radial velocity at radius 'r' .
e

R = 120 x e-rs
R = 120 x 0.533 = 64 m

�ydcrabad j Dclhi j Bhopall Puncl Bhubaneswarl LucknowlPatna! BcngaiurulChennai j V\iayawada j Vizag I T"11Upati I Kukalpallyl Kolkata I
.,,,f.,.... ,,
...
. .
ACE
� . �Eogineerq Pi,lmmons
. . : 278 : Fluid Mechanics & Turbo Machinery

At, r = R
3V 2 R
--
aR -
- 4h2

Method-II
The acceleration 'a/ is obtained by taking
total time derivative ofradial component of
As plate is moving down, the volume swept
velocity Yr.
by the plate must be equal to the volume of
water going out radially away due to i.e. a r = �(vr )
dt
continuity considerations.
1.e V x m 2 =(Vr )x 2mx h
V =
r
Vr
2h _ V
[h -r dh l
dr
dt dt
-2 h
2

Note that above flow field is unsteady as h is


function oftime (h decreases with respect to
=
V h X vr - r(- V
[
)J
time) 2 h2
The radial acceleration (ar) can be found by
any oftwo methods. h x ( �) + rV
V
Method - I 2 h2

3Vr

a r = Vr _
av av [·: V. = 0 V = O]
arr + _
atr e ' z
12. Ans : (d)
Sol: Velocity components in x - y planes :
p : U = 2y ; V = - 3x
r
= �: X : + � ( - h\ . :) Q : U = 3xy ; V= 0

R : u = - 2x ; v = 2y
r Continuity equation for incompressible fluid
= �� 2 - :� x (-v) [-: : =-VJ
flow
au
+ = O ___ ( 1 )
av
ax ay
l11114@iil441d4RflbiMihiih\�)Ryderabad l Delhi I Bhopal I Punc I Bhubancswarl ln�ow l Patna! Bcngaluruj Chcnnai I Vtjayawada j Vit.ag !Tirupati I Kukalpal)yf Kolkata I
. A:CE . . Fluid Kinematics
:.� : 279 :
Irrotational flow condition is 13. Ans: (c)
� �F,wnr,:nncl'nNnmas

av = au O Sol: Explanati on:


ax a y = --- (2)

For Choice P : u = 2y , v = - 3x

� -+-
au a v
ax a y
= - (2y) + - (-3x)
a a
ax ay By continuity equation,
=O+O=O Q i + Q2 = Q3
Satisfied the Equation of Continuity A1V1 +A2V2 = A3V3
i.e., fluid flow is "Incompressible". I x 1 + 2x 2 = 2.5 x V3
au = 2 5
V3 =-=2 m/ s
ay 2.5
av = - 3
ax 14. Ans : (d)
:. Fluid is rotational Sol: Given data,
Velocity Vector of a flow field
For Choice Q : u = 3xy, v = 0
not satisfied V=2xy i-x 2 z j
� I\ I\

For Choice R : u = - 2x ,v = 2y Vorticity ( � ) is defined as twice of the

= - (-2x) + - (2y)
a a rotation and hence it is given as = 2ro

� = 2ro = curl V
ax ay
= - 2 + 2 = 0 ( Incompressible flow )
j k
av = 0, au =0 ("Irrotational flow")
1
a a
ax ay �=
ax
The flow application requires is to be
ay az
u w
"Incompressible and Irrotational" then
V

k
choice 'R' only satisfies these two
J
a a
1

correlations. �= az
: . Hence correct choice is "R".
ay
2xy - x2z 0

Fyderabad I Delhi I Bhopal I Pune I Bhubaneswarl Luda\owl Patna!Bengalwul Chennai I VliayawadalVizag jTirupati I Kukalpallyl Kolkala I
: 280: Fluid Mechanics & Turbo Machinery

16. Ans: (a)


Sol: u = Ky ; v = O
w = Kx
k
Q= 2co=k
J
; = x 2 i + Oj + k(- 2xz-2x)
1
a a a
ax ay az
; = x 2 i -(2xz + 2x)k , y 0 X

At x = 1, y = 1, and z = 1 2co= (-l)k(� (x)-_q_ (y ))


;(1,1,1) = i - 4k
ax az
=-K

15. Ans: (d) 17. A ns: (c)


Sol: Note: Option (iv) is wrong
18. A ns: 2 (1.9 t o 2.1)
An unsteady vector field
u = a 1x+a2y + a3z
Example:
expressed as a system of differential
Sol:
V = b IX + b2y + b3Z
dx W = C1X + C2Y + C3Z
equation = V(x, t) with initial condition
dt Given a 1 = 2, C3 = -4
x(to) = xo, its solution is integral curve, Velocity field should satisfy the following
trajectory, or orbit
equation - +- + -=0
. au av aw
ax ay az
a1 + b2 + C3 = 0
2 + b2 - 4 = 0
=> b2 = 2

Hence path of a particle is not stream line. In


unsteady fluid flow, the path line of
successive particle will be different and
stream line pattern of the flow will be
changing at every instant
_ x (x + y -2x )-2xy = -x -xy
-
(x 2 + y 2 )(x 2 + y 2
2

(x 2 + y 2
2 2 2 3 2

J J
\( I I 11,..,, 1111 ( 1 111� Pulda .1t ic 111... �ydcrabad l Dclhi l Bhopal l Punc l Bhubaueswar l Lucknowl Patna l Bcngaiuru l Chennail Vuayawadal Vizag I Tirupati I Kukatpallyl Kolkata I
'!.�-·
...
:
AJCE
y • •
: 281 : •
Fluid Kinematics

� � ===========================================
�Nmfirm
a x =-
20. Ans: (c)
(x 2 + y 2 )2 Sol: u = -x2 + 3y =-12 + 3(-1) = -4
X
:.

= 2xy = 2 (1)(-1) = - 2
a = u- + v-
av av V

- = -2x =-2,
ax ou ou = 3
'
Y O'f
ax oy
x
r -y
r 2 +
y
(
r
{x + y }- y x 2 y l
2 2

-
av = 2y = 2(-1) = -2
= (x 2 + y2 (x 2 + y 2 X X (x 2 + y2) X (x 2 + y 2
- 2x y + yx - y ax
(x 2 + y 2 J
2 2 3

-
av = 2x = 2(1) = 2
oy

ax = U
ou + v ou = (- 4X- 2) + (- 2) x 3 = 2
The velocity components are not functions
ax oy
of time, so flow is steady according to av + v av = (- 4X- 2) + (- 2X2) = 4
aY = u &
continuity equation,
oy
8u + av = - (x 2 - y 2 (x 2 - y 2 ) o L a = fa 2 + a = .J22 + 42 = .fio = 2./5
=
2

ox oy ( ( x 2 + y2 )2 x 2 + y2 )2
'\/ X y

Since it satisfies the above continuity


equation for 2D and incompressible flow.
:. The flow is incompressible.

l1t11i!i§iiiiiih4RflGiMi!U*fyderabad lDcllii 1 Bhopa)I PunelBhubancswar l Lucknowl Pa!nalBengalwu i ChennailVuayawadal V17.ag ITirupati I Kublpallyj Kolkala I
C4 Fluid Dynamics
force on the pipe section due to the flow of
One Mark Questions water. (GATE- ME-93)

0 1 . In a hand operated liquid sprayer (figure


shown below) the liquid from the container
rises to the top of the tube because of:
(GATE- ME-90) Pipe 1 Pipe 2 Pipe 3

piston State which of the options indicate the


tube correct direction of force for pipe 1 , pipe 2
and pipe 3 .
(a) 45 ° to both +ve X and +ve Y axes
(b) 45 ° to both -ve X and +ve Y axes
(c) 45 ° to both +ve X and -ve Y axes
--- -----
·------­
·-------------­- (d) 45 ° to both -ve X and -ve Y axes
--------

03. Bernoulli's equation can be applied between


(a) Capillary effect any two points on a stream line for a
(b) Suction produced by the air jet at the top rotational flow field.
end of tube State: True/Fal se. (GATE- ME-94)
(c) Suction produced by the piston during
the backward stroke 04. In a venturimeter, the angle of the diverging
(d) Pumping of the air into the container section is more than that of converging
section.
02. Shown below are three pipe sections through State: True !False (GATE- ME-94)
which water flows as shown. Option (a) to
(d) below pertain to the direction of the net
ubaneswarl Lucknowl Patna lBengaJwu l Chennai lVuayawadalVizag
!IHIM!@iH4i.ji44fifi5ijj1h +Yderabad l Delhi l Bhopai l Pune l Bh ITuupati I Kukatpallyl Kolkatal
: 283 : Fluid Dynamics

05. Navier Stoke's equation represents the 02. Water flows through a pipe of diameter 0.30
conservation of (GATE-ME-00) m. What would be the velocity V for the
(a) Energy (b) Mass conditions shown in the figure below?
(c) Pressure (d) Momentum (GATE-ME-88)
S.G = 0.8
06. A vertical cylindrical tank of 1 m diameter is
filled with water up to a height of 5 m from
its bottom. Top surface of water is exposed
to atmosphere. A hole of 5 mm2 area forms at
the bottom of the tank. Considering the 0.3 m Dia
water _____,
coefficient of discharge of the hole to be
unity and the acceleration due to gravity to
be 10 mls2 , the rate of leakage of water (in 03. A jet of water issues from a Nozzle with a
litre/min) through the hole from the tank to velocity 20 mis and it impinges normally on
the atmosphere, under the given conditions, a flat plate moving away from it at 10 mis.
lS --- (GATE - PI-16) The cross-sectional area of the jet is 0.01 m2 ,
and the density of water =1000 kg/m3 • The
force developed on the plate is:
Two Marks Questions
(GATE-ME-90)
01. A mercury manometer is attached to a section
of the pipe shown in the figure. Mercury
levels are indicated when there is no water u = IO m / s
flowing through the pipe. When water starts
flowing through the pipe continuously at
(a) IOOO N (b) IOO N
constant rate in the direction of the arrow, the
(c) 10 N (d) 2000 N
level of mercury at B:
(GATE- ME-87)
04. A fan in the duct shown below sucks air from
the ambient and expels it as a jet at 1 mis to
the ambient. Determine the gauge pressure at
the point marked as A. Take the density of
air as 1 kg/m3 • (GATE- ME-93)

\( l l 11�111t t 1 11 1g P11lilu ,111011:-i �yderabad J Delhi J BhopaJ J Pune J Bhubaneswarl LucknowlPatnal Bengaluru j Chennai l Vtjayawada lVmg J T1n1pati I Kukatpallyl Kolkala I
".
" �.a:��
.,
-:.
,." . ACE
. . . : 284 : Fluid Mechanics & Turbo Machinery

is 'm'. The force registered by the weighing


balance at this instant of time is
1 m/sec
(GATE- ME-03)
(a) mg + p QU (b) mg + 2 p QU
(c) mg + p QU2/2 (d) p QU2/2

07. Air flows through a venture and into


05. Water flows through a vertical contraction atmosphere. Air density is p; atmospheric
from a pipe of diameter d to another of pressure is 'Pa ' ; throat diameter is 'Dt' ; exit
diameter d/2 (see Figure). The flow velocity diameter is D and exit velocity is 'U'. The
at the inlet to the contraction is 2m/s and throat is connected to a cylinder containing a
pressure 200 kN/m2 • If the height of the frictionless piston attached to a spring. The
contraction measures 2m, then pressure at the spring constant is 'k'. The bottom surface of
exit of the contraction will be very nearly the piston is exposed to atmosphere. Due to
(GATE- ME-99) the flow, the piston moves by distance 'x'.
Assuming incompressible frictionless flow,
d<I>

then 'x' is (GATE-ME-03)



2m

(a) 168 kN/m2 (b) 192 kN/m2


(c) 150 kN/m2 (d) 174 kN/m2 (a) (pU2/2k) 1t D/

06. A water container is kept on a weighing (b) �u' / 8 k{ i ', - 1 } 0 .2


,

into the container with a volume flow rate of m{ i:, - I)no, '
balance. Water from a tap is falling vertically

(c)
' Q '; the velocity of the water when it hits the
�u '
water surface is 'U ' . At a particular instant of
time the total mass of the container and water ( d) �U 2 /8k{i,: - 1} 0 , '

\( I I 11�\lll l I l!I� P 1 1 l il1t ,IIJC )Jl',


Fydcrabad I Delhi I Bhopal I Punc I Bhubaneswar I Lucknow I Patna I Bcngaluru I Chcnnai I Vtjayawada I Vu.ag I T=pari I Kukatpally I Kolkata I
: 285 : Fluid Dynamics

08. A venturimeter of 20 mm throat diameter is 10. Consider steady incompressible and


used to measure the velocity of water in a irrotational flow through a reducer m a
horizontal pipe of 40 mm diameter. If the horizontal pipe where the diameter is reduced
pressure difference between the pipe and from 20 cm to 10 cm. The pressure in the 20
throat sections is found to be 30 kPa then, cm pipe just upstream of the reducer is 150
neglecting frictional losses, the flow velocity kPa. The fluid has a vapour pressure of 50
1s (GATE-ME-05) kPa and a specific weight of 5 kN/m3 .
Neglecting frictional effects, the maximum
(a) 0.2 mis (b) 1 mis discharge (in m3/s) that can pass through the
(c) 1.4 mis (d) 2.0 mis reducer without causing cavitation is

09. A U-tube manometer with a small quantity of


(GATE- ME-09)
(a) 0.05 (b) 0.16 (c) 0.27 (d) 0.38
mercury is used to measure the static
pressure difference between two locations A 11. A smooth pipe of diameter 200 mm carries
and B in a conical section through which an water. The pressure in the pipe at section
incompressible fluid flows. At a particular S 1 (elevation: 10 m) is 50 kPa. At section
flow rate, the mercury column appears as S2 (elevation : 12 m) the pressure is 20 kPa
shown in the figure. The density of mercury and velocity is 2 mis. Density of water is
is 13600 kg/m3 and g = 9.81 mls2 . Which of 1000 kg/m3 and acceleration due to gravity is
the following is correct? (GATE-ME-05) 9.8 ms-2 • Which of the following is TRUE?
(GATE- ME-10)
(a) flow is from S 1 to S2 and head loss is 0.53 m
(b) flow is from S2 to S 1 and head loss is 0.53 m
T
(c) flow is from S 1 to S2 and head loss is 1.06 m
.!. (d) flow is from S2 to S 1 and head loss is 0.53 m
1 50mm

12. Figure shows the schematic for the


(a) Flow direction is A to B & PA-PB = 20 kPa measurement of velocity of air (density
= 1.2 kg/m3) through a constant -area duct
(b) Flow direction is B to A & PA-PB = 1.4 kPa
(c) Flow direction is A to B & PB-PA = 20 kPa using a Pitot tube and a water-tube
(d) Flow direction is B to A & PB-PA = 1.4 kPa manometer. The differential head of water
(density = 1000 kg/m3) in the two columns of
the manometer is 10 mm. Take acceleration
\( I I rn!,11ll l 1 111.., P11ld1t .111,111 ... ydcrabad J Delhi J BhopaJJ Punc J BhubaneswarJ LucknowJ Palna J BengaluruJ Chcnnai J Vliayawada J V17.i1g JT1IUpati J KukalpallyJ Kolkala
: 286 : Fluid Mechanics & Turbo Machinery

due to gravity as 9.8 mls2 . The velocity of air 14. Water is coming out from a tap and falls
in mis is (GATE- ME-11) vertically downwards. At the tap opening, the
stream diameter is 20 mm with uniform
velocity of 2 mis. Acceleration due to gravity
is 9.81 mls2 . Assuming steady, inviscid flow,
constant atmospheric pressure everywhere
and neglecting curvature and surface tension
effects, the diameter in mm of the stream
0.5 m below the tap is approximately
(a) 6.4 (b) 9.0 (c) 12.8 (d) 25.6
(GATE-ME-13)
(a) 10 (b) 1 5 (c) 20 (d) 25
13. A large tank with a nozzle attached contains
three immiscible, inviscid fluids as shown.
15. An ideal water jet with volume flow rate of
Assuming that the changes in h 1 , h2 and h3
0.05m3Is strikes a flat plate placed normal to
are negligible, the instantaneous discharge
its path and exerts a force of 1000N.
velocity is
Considering the density of water as 1000
(GATE-ME-12)

kg/m3, the diameter (in mm) of the water jet


h1 P1
,�
1s ___ (GATE-ME-14) (S-1)
P2
16. A siphon is used to drain water from a large
h2

p3 tank as shown in the figure below. Assume


that the level of water is maintained constant.
h3 '-
)( _ _ _ _ _ _ _ _ _ _ _ _ _ _ _ _ _ _ _

Ignore frictional effect due to viscosity and


losses at entry and exit. At the exit of the
(a) siphon, the velocity of water is

(b) �2g(h 1 + h 2 + hJ

h + h + p3 h 3
p

(c) 2g( P1 1 P 2 2
P1 + P 2 + P 3 J
0

z
(d)

Fydcrabad l DelhiJBhopa!JPuncJBhubancswarJ LucknowJPalna l BengaiuruJChennaiJVuayawadaJVizag JTIIUpati I Kukalpally l Kolkala I


:.t
�� ..�·.r;
. .t\CE . . = 87 Flmd D narrucs
:•�========.,..:.;2:��:�==========���·���y��·��
=Fii�p�-
:�nn�111�Pu
�,�·�ra�.,..

(GATE-ME-14) (S-3)
(a) J2g(Z Q -Z R )

(b) �2g{Z p - Z R )
(c) �2g{Z0 - ZR )

(d) J2gZ Q

17. Water (p = 1 000kg/m3 ) flows through a


venturimeter with inlet diameter 80mm and
throat diameter 40mm. The inlet and throat l]Force

gauge pressures are measured to be 400 kPa (a) (small and large), (large and zero) and
and 1 30 kPa respectively . Assuming the (zero and zero)
venturimeter to be horizontal and neglecting (b) (small and zero), (large and large) and
friction, the inlet velocity (in mis) is___ (small and zero)
(GATE -15-Set 1) (c) (large and zero), (zero and large) and
(large and zero)
1 8. A Prandtl tube (Pitot-static tube with C= 1 ) is
(d) (large and small), (small and zero) and
used to measure the velocity of water. The
( small and large)
differential manometer reading is 1 0 mm of
liquid column with a relative density of 1 0. 20. The water jet exiting from a stationary tank
Assuming g=9.8 mls , the velocity of water
2
through a circular opening of diameter 300
(in mis) is ___ (GATE-15 -Set 3) mm impinges on a rigid wall as shown in the
figure. Neglect all minor losses and assume
19. In a vertical piston-cylinder arrangement the
the water level in the tank to remain constant.
force applied to the piston, pushes water
The net horizontal force experienced by the
through a nozzle as shown in the figure. The
wall is --- kN.
water flows out from the nozzle, and reaches
the top of its traj ectory. The kinetic and
Density of water is 1 000 kg/m3 .
Acceleration due to gravity g = 1 0 mls2 .
pressure energies at points ( 1 ), (2) and (3),
respectively, are (GATE - PI-15)

ii1•1ii@jjji4.jjQAfltiitwl&il+1yderabad I Delhi I Bhopal I PuneI BhubancswarI 1.ucknowl PatnaI Benga)uru I ChennaiIV\iayawadaiVmg I Tirupati I Kukatpal)yI Kolkata r
: 288 : Fluid Mechanics & Turbo Machinery

are as indicated in the figure, the magnitude


Stationary rigid wall
/ of horizontal force (in N) required to hold
the plate is

1
6.2
Jet
Plate

D 1 = 6cm D2 = 4cm
Circular opening of 20 m/s
diameter 300mm

21. The arrangement shown in the figure


(GATE- 16 - SET- 3)
j (GATE- 17 - SET- 2)
measures the velocity V of a gas of density
l kg/m3 flowing through a pipe. The 23. A pipeline with variable cross-section
acceleration due to gravity is 9.8lm/s2 . If the contains water with specific weight 104
manometric fluid 1s water (density N/m3 • The flow conditions at two points 1
1000kg/m3 ) and the velocity V is 20m/s, the and 2 on the axis of the pipe are:
differential head h (in mm) between the two P1 = 3 bar,
V1 = 10 m/s
arms of the manometer is P2 =
l bar,V2 20 m/s =

Consider frictional losses to be negligible.


v -======::::::;--,
For no-flow condition between points 1 and
2 (as shown in Figure), if the height z 1 from
water the datum is 1 m, then the height z2 (in m) is
(g = 9.81 m/s2)

(GATE- 17 - SET- 2)

22. A 60mm - diameter water jet strikes a plate


containing a hole of 40mm diameter as
shown in the figure. Part of the jet passes Datum
through the hole horizontally, and the
remaining is deflected vertically. The (GATE-Pl - 17)
density of water is 1000kg/m3 • If velocities
!M11ii@jjji4.jjjiAflftiM\hlh+1ydcrabad I Delhi I Bhopal I Punc I Bhubaneswar I Lucknow I Pama I Bcngalwu I Chcrmai I Vliayawada j Vizag I Tirupati I Kukatpa]Jy I Kolkata I
., .., . . .
. ACE Fluid Dynamics
��-·l�Publicatiooa : 289 :
.

-= -==- =- =- =-0�-=-= -=
A

Three Marks Questions


-:-:-:-:-:-:-:-:-::a�-:-:-:-:-
_ _ _ _ _ _ _ _ __.Q. _ _ _ _ _

01. When a Pitot-static tube is immersed in a


:::::::::: B : ::::: :
�--
-- ----
--------
- --
----
water stream (density Pw), the differential :: C :
head measured by a mercury (density Pm)
manometer is hm. What is the velocity of the
J_
water stream? (GATE- ME-87) Saline i

Five Marks Questions (GATE- ME-87)

03. In a syringe as shown in the figure, a piston


01. A jet of water (area Aj, velocity Vj ,
of 1 cm2 cross section is pushed at a constant
density p ) impinges horizontally on a curved
speed of 10 emfs to eject water through an
vane which deflects the jet through 60 °
outlet of 1 mm2 . Determine the force
upwards. If the vane travels horizontally at a required to move the piston. Neglecting
speed 'u' find (GATE-ME-87) losses. (GATE- ME-89)
(a) the force experienced by the vane, and I cm2

(b) the power developed by the vane. I mm2


F =:+:I O emfs :-:-_

02. Consider the saline drip bottle shown. If p is


04. A jet of water with a velocity V 1 (Figure
the density of saline, find
shown below) and area of cross-section A 1
(a) Pressure at A, enters a stream of slow moving water in a
(b) The velocity of flow of saline through pipe of area A2 and velocity V2. The two
the tube. (Neglect viscous losses m streams enter with the same pressure P1.
tube). Atmospheric pressure = Patm· After thoroughly mixing in the pipe . the
stream emerges as a single · stream with
velocity V3 and pressure p2. If there are no

llti1i@jjji41h@RflliM\hii!+yderabad I Delhi I Bhopal I Punc I Bhubancswar I Lucknow I Patna I Benga)uru I Chcruiai I Vuayawada l Vizag I T=pati I Kukalpal)y I Kolkata I
losses in the flow, determine (p2 - p1) for
V 1 = 20 mis, V2 = 10 mis, A1= 0.01 m2 , A2 = 06. Water ( p = 1000 kg/m3) flows horizontally
0.02 m2, density ofwater p =1000 kg / m 3 • through a nozzle into the atmosphere under
the conditions given below. (Assuming
steady state flow)
(GATE- ME-01)

(GATE- ME-90)

05. A Venturimeter (throat diameter = 10.5 cm)


is fitted to a water pipe line (internal At i nl et: At outl et:
diameter = 21.0 cm) in order to monitor flow A 1 = 10-3 m2 ; A2 = 10-4 m2 ;
rate. To improve accuracy of measurement, V 1 = 2 mlsec; P2 = Patm
pressure difference across the venturimeter is P 1 = 3 x 10 Pa (gauge)
5

measured with the help of an inclined tube


manometer, the angle of inclination being Determine the external horizontal force
30 (see figure below). For manometer
°
needed to keep the nozzle
reading of 9.5 cm of mercury, find the flow
rate. Discharge coefficient of venture is
0.984. (GATE- ME-92)

From
Venturi

----------
--------- ---------
-------·

!ltl•i!@jjjl41h4Pflni@jjih+yderabad I Delhi I Bhopal I Punc I Bhubancswarl Lucknow I Patna I Bcnga)uru I Chcnoai I Vuayawadaj VJZag I Tirupati I Kukatpally I Kolkata I
SOLUTIONS
i.e. in 'x' direction
Fx = rilV - m(O) = rilV
Similarly in 'y' direction
One Mark Solutions

FY = m(o)- m(v)
FY =-rilV
Sol: Moving fluid (i.e. Air) produce suction (i.e.
0 1 . Ans: (b)

negative) pressure at the top of tube, due to


which fluid in the container may rise in tube. - 'r::::==========3:
Fx ( --------------- ­
V

y
I

02.
I

Sol: We will use linear momentum equation to


analyze direction of force exerted by water
on the bend. l�
I
Fext
(Pipe-I) Pipe-I

Therefore, external force must be in fourth


Assumptions:

quadrant and force exerted by water must be


(i) Pipe area is constant

opposite to it i.e. in second quadrant


(ii) Frictional losses are negligible.
(iii) Elevation change is negligible.
Above assumptions together imply that Pipe-2
velocity and pressure is same at inlet and
exit.
y

Let Fx and Fy be the 'x' and 'y' components


of 'external force' acting on the bend.
(The external force is opposite to the force
exerted by water on the bend). '- - - - - - - - - - - - - - - Fx Fwater

Fy
Pipe-2

Applying linear momentum equation for the


Pipe-2

Following the same methodology


control volume we get Fx = m(O)- m(V)= - mV
(F) = (mv)out - (mvt + �{mvk v FY = m(v)- m(o) = mv
ot
\( I I 11 ..., 111t t r 1r 1t., P11li\1c ,lllt)JI" Fyderabad l Dclhil Bhopal l Pune l Bhubaneswarl Lucknow l Patna l Bengalwu l Oiennai l Vliayawada lVu.ag ITuupari I Kukatpal)yl Kolkala I
: 292: Fluid Mechanics & Turbo Machinery

Reason:
If angle of divergence is more then flow
Pipe - 3

y separation occurs resulting in eddies and


vortices. It causes loss of energy.
Hence the angle of divergence IS less and
Fext

divergent cone length is more.


The same concept is adopted in the design of
draft tube of reaction turbines.
The angle of divergence of a .draft tube is
Pipe-3 Pipe-3

Similarly approximately 8°.


Fx = m(V) - m(O) = mV
05. Ans : (d)
FY = m(o ) - m(- v) = mv
Sol:Navier stokes equation Is obtained when
Newton's second law IS applied to
03. Ans: True. differential fluid element by considering
Sol: Bernoulli's equation can be applied even if pressure, body and viscous forces. As
the flow is rotational along a stream line if Newtons second law deals with rate of
the flow is steady. Hence the statement is change of momentum.
true.
If the fluid is irrotational, it can be applied Note:
between any two point even though they are Euler's equation of Motion represent two
different streamlines. forces only.
When the flow is steady but may not be F = Fp + Fg
irrotational i.e., rotational flow: It doesn't contain viscous forces.
In this case, Bernoulli's equation is Hence Euler's equation represent Ideal
applicable only to particular stream line that fluids.
is the value of constant is different for Note:
different stream line. Reynold's Equation of Motion represents
four forces.
04. Ans: False F = Fp + Fg + Fv + Ft
Sol: In a Venturimeter, the angle of convergent Fi = Turbulent force
section is about 15° to 20° where as angle of Hence Reynold's equation represents
divergence is about 5° to 8°. Turbulent flows.
\( I l.w.;111t l 1 111� P11lilh .111011, Fydcrabad! Dclhi ! Bhopal ! Pune! Bhubancswar l Lucknow l Patna ! Bengaluru l Oiennai lVliayawadalVw,g rnrupati I Kukatpally! K.olkala I
: 293 : Fluid Dynamics

06. Ans: (3) hm = Differential height of manometric fluid


Sol: V = �2x g x h = �2x10x5 = 10m l s Sm = Specific gravity of manometric fluid.
Sr = Specific gravity of flowing fluid.
Q = AV = 5xl f -<>x10
f

= 5x10-5 m3/sec = 0.05 liter/sec 03. Ans : (a)


= 0.05 x 60 = 3 lit/min Sol: Jet velocity (V) = 20 mis
Flat plate velocity (u) = 10 mis
C/S area of Jet, A = 0.01 m2
Density of water, p = 1000 kg/m3
Two Marks Solutions

01. If the Jet and plate moving in the same


Sol: At section 'B' area of cross section is less. direction, force developed by Jet of water on
As area is less, velocity is more compared to plate (F) = pA (V - u)2
= lOOO xO.Ol x(20 - 10)2 = 1000 (N)
section 'A' (for subsonic flows). According
to Energy equation, as kinetic energy
04.
(velocity) increases, pressure decreases (at
Sol: Apply Bernoulli's equation between point A
'B'). Hence the level of mercury at 'B' rises
and outlet
where as it falls at 'A'.
� + O + O = O +- + O
y2
02. pg 2g
p

pV-2 1 X (1) 2 N2
PA - =
= - 0.5 -
Sol :
2 2 m

05. Ans: (c)


Sol:
V
0.3 m Dia
water -------'

V =C v �2g h

V -C, 2.g. h. (1-t)


V = I .Ox 2x9.81x o.3(1- o�
g
)
V = l .085m/ s
\( l I 11�111t l 1 111!!. Plllil1t .111,,11.., Fyderabad I Delhi I Bhopal I Pwie I Bhubancswar I hicknow I Patna I Bengaluru I Chennai I Vrjayawada I Vizag I Tirupati I Kukatpally I Kolkata I
.,.
V
'!.
,�.. ACE
:� Pi�
V • • • •
: 294: Fluid Mechanics & Turbo Machinery

By c ontinuity equati on:­ 07. Ans: (d)


Q = A 1 V1 = A 2 V2 Sol: Given data:
Type of fluid flow through Venturimeter = AIR
d
7t d .vI = 7t ( ) .v2 p = Density of air (kg/m3 )
2

4 4 2
2

Pa = Atmospheric pressure (N/m2)


v2 = 4VI = 4x 2 = 8m/ s
D1 = Throat diameter (m)
D = Exit diameter (m)
U = Exit velocity (mis)
By Bern oulli's equati on:­
v v K = Spring constant (Nim)
_!i_ + z 1 + / = .!i_ + z 2 + ;
pg 2g pg 2g
x = Distance moved by piston(m)
-200
-x-- l 03
+ 0 + ---
22 Ds = Spring piston diameter (m)
1000 x 9.81 2 x 9.81 Using continuity & Bernoulli's Eqn s
p 82 pressure on spring piston can be arrived.Then
= + 2 + ---
1000 X 9.81 2 X 9.81 force on spring is equal to product of
2

On Simplification, P2 = 150 kN/m 2


pressure and area of spring piston. [Force on
spnng = Spring stiffness constant x
06. Ans: (a) deflection of spring].
Sol:

' ·Jtt· Q' 1. Eqn. of c ontinuity between thr oat and exit :
A . V = A.V
Throat Throat exit exit

rr D� . vt = rr D 2 . u
_ _ .11_ _

m 4 4

Ci:) Weighi g machine V, =(;)'.u ___ (1)


Force registered by weighing machine
= Weight of water with container +
2. Applying Bern oulli's Eq0 • between thr oat

Force exerted by striking water


and exit:
pThroat + Viiiroat pExit z . V!it
[Rate of change of momentum of water] = g + Exit + g
p g Z Throat + 2g p 2
= mg + pAu2
= m g + p. Au.u [·: ZThroat = ZExit ]
= mg + pQu pThroat + Viiiroat = P Exit + Vixit
pg 2g pg 2g
\( } ( ll:�11\t t I Ill� P1 1 l il11 ,11](111'- �ydcrabad I Delhi I Bhopal I Pune I Bhubaneswarl Lucknow! Patna! Bengaluru j Chennaij Vgayawadal Vu.ag I T=pati I Kukalpallyl Kolkata I
:295: Fluid Dynamics

pThroat - pExit=-----
Vixit V-iirroat Continuity E qn :
pg 2g 2g As water is in compressible (p1 = p2)
A1 V1 =A2 V2
pThroat - pExit = � [vixit - V-iirroat ]
: (40) 2 V, =: (20) 2 V2

=� {u' -[(;)' u n � V2 =4V, ........... (1)


Applying Bernoulli's equation
[Neglecting losses]
PTh=• -P&, =� [ U' -(; J.u'] y2 yz
RL+z 1 +-' = h +z 2 +-2
pg 2g pg 2g
_ 2
(Ps., -P_,)= � U' [(iJ-1] V22 y
P1 -p 2 =---
pg 2g
I

(z, = z2 as pipe is horizontal)

P, -p 16V 2 -V 2
3. Spring stiffness (k)
Force on spring(N) 2 = -�--=-
p 2
�-=- I I
=-------"-----"--'----
Deflection of spring (compression)
30000 15VI2
pThroat) X Area Of Spring piston
1000 2
k = (p Exit - --=
X

Solving V1 = 2 m / s
� u'[(�J-i]x: D!
k=-�--�-- 09. Ans: (a)
Sol: Given data:
x= P �
8
2
[(�J-i]x�o: p 8g = 13600kg/ m
g =9.81 m/s2
h 8g = 150mm = 0.15m
08. Ans: (d)
Sol: Given data:
d2 = 20 mm
d1 = 40 mm T

P1 -P 2 = �p = 30kPa ..l
150mm

Flow velocity in pipe, V =? .


I

\( l l 11!.!,\lltt 1111!.!, P11lil1t,11Jt)I]', �yderabad I Delhi I Bhopal I Pune I Bhubaneswarl wcknowl P-.itnal Bengalurui ChennailVtjayawadalVmg IT=pati I Kukalpa)Jyi Kolkala I
: 296: Fluid Mechanics & Turbo Machinery

Using continuity equation

= 13600 X 9.81 X 0.15 A1 V1 = A2V2


= 20.0124 x 103 N/m2 7t 27t
-d 1 V1 =-d 2 V2
2

PA -Pa = 20.0124 kPa


4 4

As pressure is decreasing from A to B, hence 1t (0.2)2 V1 = 1t (0.1)2 V2


flow direction is A to B.
4 4
V1 = 0.25 V2
Note: If fluid flows from B to A and flow is Using Bernoulli's equation at (1) and (2)
sections of reducer pipe
attached then still pressure at B is more than
that at A. (because of less velocity)
p V,2
p V, 2

pg 2g pg 2g
2
_I +Z +-' =-2 +Z2 +__
However for sharp diverging section as
Since Z1 = Z2, since pipe is horizontal
shown in the diagram flow tends to separate
hence pressure will not increase much.
150kPa

10. Ans: (b)


50kPa

Sol: d, = 20 cm = 0.2 m,
d2 = 10 cm = 0.1 m

Pressure gauges
Reducer Pipe

150 (0.25V2 ) 50 V/
-+--'-----=-.:.- =-+- -=----
2

5 2x9.81 5 2x9.81
30 + 0.0032 v/ = 10 + 0.051 v/

Reducer pipe

P1 = 150 kPa 7t 2
Q = A2V2 = (0.1) x20.46
P2 50 kPa (Permissible pressure in
4

= 0.16 m3 /sec
=

pipe i.e vapour pressure)


y =pg =5 kN/m2•

�yd�JDclhiJBhopalJPunclBhubancswarl LucknowlPainalBcngaJuruJOtennailVoayawadaJVizag JTirupari I Kukatpallyj Kolkata I


1 1. Ans: (c) 12. Ans: (c)
Sol: Sol: Given data
Density of air, p Air = 1.2kg / m
3

Density of water, Pwater = 1000kg / m 3

I Acceleration due to gravity (g) =9.8 m/s2


P

Manometer reading, h water = 10 mm = 0.01 m


2

Z = 12m
1'

l
Velocity of air = C y = �2.g.h Air
2

Where, C y = l

!IA
t !IA
Datum line

!IA h Air = h Water (P


- water
-- -1) ,
PAir
Pwater = 1 000kg/ m
3
l OO
g = 9.8m /s 2 = h Water ( Q -1)
1.2

E1 Total energy at S 1 = -+Z 1 +- 1000 -


= 0.01( 1) = 8.323meterS'of air
y2
1 p1
p g 2g 1.2
=

_ 5_0_x_ l 0_ 3_ (
_ 2)_ 2_ VAir = lx.J2x9.81x8.323
E1 = +l0+-
1000 X 9.8 2 X 9.8 = 12.78m/s = 12.8m/s
=>E1 =15.306 m ... .... ...(1)
13 Ans: (a)
E2 = Total energy at S2 = - +Z 2 +--
y2
2 2 p
pg 2g Sol:

_ 20xl0 3 (2)2
E 2 - ----+12+�-=>
1000 X 9.8 2 X 9.8
E 2 = 14.244 m................(2)
:. Head loss, h L = E 1 -E 2 = 15.306-14.244 h3 p 3
---------------------PAtm
Head loss = 1.062 m P 1 (1) (2)
It is observed that total energy at S1 > S2
(15.306m > 14.244m) => (E1 > E2) P1 = Patm + p1 gh1 + p2gh2 + p3gh3
Hence the flow direction from S 1 to S2 and Bernoulli's equation between (1) and (2)
p _1 y2 p y2
head loss is 1.06 m. _ +_ 1
+z I =-2-+-2 +z 2
p 3 .g 2g p 3 .g g
\( J I ll�l!ll l I 111:.:, P11lil1< ,tilt 111, �ydcrabadlDclhilBhopal!PunclBhubaneswarl l.ucknow[Pa!nalBcngaJurulCbermailVuayawadalV,zag ITirupati I Kukatpallyl Kolkala I
{Where P2 = Pann) By continuity equation for incompressible fluid
Substitute P1 in above equation A I VI = A 2 V2
Patm +P1 gh 2 +P 2 gh 2 +p 3 gh 3 0 2 Patm-+-
V;
---------- -+- = - (0.02)2 X 2 = (d i }2 X 3.716
p3g 2g p 3 g 2g
7t 7t
4 4
d2 = 0.0147 m
Patm p,gh , +P2 gh 2 +p 3 gh 3 _ Patm V;
p3g p3g p 3 g 2g = 14.7 mm = 15 mm
--+--------- --+-

2gh 3 [1+12__�+
p3 h 3 p 3 h 3
£1._.�J 15. Ans: Range 56 to 57
pQ 2 ·: [Q = AV]
Sol: F = pAV 2 = pQV =
A [V =Q/A]
· 14. Ans: (b)
Sol: By Bemouli's equation lQOO =
1000 X (0.05) 2

y d2
P, p V;
7t

-+z l +- = -2 +Z 2 +- [·: P, = P2 = PAim ]


2
1
pg 2g pg 2g :. d =0.564 m= 56.4mm

16. Ans: (b)


Sol: Apply Bernoulli's equation between 'O' and
'R' with respect to datum
So, V; = V/ +2gH

V2 = .J2 +2x9.8lx0.5
2 (p )
_Q_
pg gauge O
y
2g
2
+ Z + -0- = i (p )
pg gauge
y
2
+ Z +---.!..
R 2g

V2 = 3.716 mis

-d 1=20mm--

H=0.5 m

0
0
0

IKi!il¥h4hiMhi4Rfil!MINU+ydcrabadlDelhilBhopalJPunelBhubancswarl LucknowlPatnalBengaluruJChennaiJVuayawadaJVmg JTirupali I Kukatpal)yJ Kolkala I


: 299: Fluid Dynamics

17. Ans: 6 20. Ans: 8.765 (range 8.76 to 8.78)


Sol: d1 = 80 mm = 0.08 m , Sol: Fx = pAV2
d2 = 40 mm = 0.04 m, p = 1000 kg/m3 , = 103 x 1t x0.32 x2x10x6.2 8.765 kN
4
=
P1 = 400 kPa, P2 = 130 kPa
Z1 = Z 2 21. Ans: 20.4
p, y2 ' z p2 y
_ +- + , = -+- +z2
2

Sol: V = ,/2gh = 2g,{ P; -1J


2
pg 2g pg 2g
400xl0 3 V- 130x103 V2 y 2
--- +-' = ---+-2-
2
20
lOOOxlO 2x10 lOOOxlO 2x10 ... X -
2
- -------
p l O
- ---� -

y y 2g ( ; -l J 2 x9 81x ( � -1)
40+- = 13+-
2 2
1
20 20
2

20.4 mm
V; -V/ = 540 ................... (1)
=

22. Ans: 628.3


From continuity equation
Sol: F = p(A1 -A2) V2
A1V1 = A2 V2
d�V, = d;V2 = 1000 X 1t X (o.062 -0.042 )x 202 628.3N
4
=

0.082 X V1 = 0.042 V2
V2 = 4V1 ................... (2) 23. Ans: 5.5 to 6.0
(2) Sub in (1) Sol: Applying Bernoulli equation t>etween (1)
(4v1 )2 -v/ = 540 and (2)
_p, y2 p y
(15v/ ) = 540 + -' +z I = -2 +- +z 2
2
2
pg 2g pg 2g
v/ = 36
pg = 104 N/m3 (given)
V, = 6 m / sec
3xl0 5 10 2 lxl0 5
- 20
-
--+- -
- +1 = -+- -+z
2

18. Ans: 1.328 10 4 2x9.81 10 4 2x9.81 2


30 + 5.096 + 1 = 10 + 20.387 + Z2
Sol: h = x(�� -1)= 0.01 (10- I) = 0.09 m Z2 = 5.709 m
Note: Conflict situation is given statement,
Velocity, V = �2gh = .J2x9.8x 0.09
velocities are given even though no flow
V = 1.328 m/sec condition is mentioned.
19. Ans: (a)

�ydcrabadlDclhilBhopaJIPunclBhubancswarl LucknowlPalnalBengalwulCbcnnailVuayawadalVmg ITuupati I Kukatpallyl Kolkata I


Force experienced by the Vane (Fx)
In the direction of vane motion ,
Three Marks Solutions Fx = m 11V
Fx = pQ((v-u)cos60 ° + (V-u))
01. = pQ (v-uXI+cos 60° )
Sol: Velocity of water stream in pipe
Fx = p.A(V-u)2 (1+cos60 ° Xnewton)
Vw = �2ghw
= 1.5 pA(V -u)2 (newton)
Where, h. = h. [�: - I] meters of water Power developed by vane Px = Fx. u
= p.A(V-u)2 (1+cos60 ° )x u
=h.[�:-1] = 1.5 pA(V-u)2 u Watt

02.
Sol: Given data :
A
-:-:-:-:-:-:-=-��-:-:-:-
Five Marks Solutions

01.
-:-:-:-:. B :
t
- - - - - - - - -o-- - -
--------:{)-----

Sol: Given data :


:-:-:-:-:-:-:-: ;o -:-:-:-:

Area ofjet of water = A


Velocity ofjet of water = V
Density of water =p J_
Deflection angle ofjet at out put, 0 =60 °
Vane velocity (horizontally) = u
Salinei
VoFx

:<
(a) Pressure at B = Atmospheric pressure, Pann
! A1
Pressure at A = Pressure at B - pgh 2

:· -:-:-·.·.+.·.·.·.f·.·.·.·.·.·.·.·.·1�
v,
g•: ·:<: : :-j@··
.. . ... . ... . . .. ... .......
......................... = Patm- pgh 2
(b) To determine the velocity of flow of a line,
use Bernoulli's equation between points A
and D. Neglecting losses and taking datum
line at D.

.\CI: b1g111L·t1111i, Pui>\1,.,tJ, ,11' FyderabadlDelliilBhopallPuneJBhubaneswarl LucknowlPatnalBengaluruJChcnnaiJVtjayawadaJVu.ag JTirupati J KukatpallyJ Kolkata I


"
��•.
..
., ACE . .
:�P.Jbliraba
. . : 301: Fluid Dynamics

OOO ( 2 - o.i 2 ) �
P1 =l 10 50 kPa
2
F = P 1 A 1 = 50 X 103 104 = 5 N
Patm -pgh 2 +h , +0 -0 = palm + VD2
X

pg pg 2g
04.
Patm _ pgh 2 +h , = Patm + VD2
pg pg pg 2g Sol:

03.
Sol: Given data: V1 = 20 m/sec, V2 = 10 m/sec
A 1 = 0.01 m2 , A2 = 0.02 m2
.

p =1000 kg / m3
----- (2)
V3 = ? ,
By continuity equation
Here linear momentum equation alone
A1 V1 + A3V2 = A2 V3
cannot give the value of force 'F' because
A 1 V1 + (A2 -A1)V2 = A2 V3
reaction at the support is unknown.
However the force 'F' can be easily found 0.01 x 20 + (0.02 -0.01) x 10 =0.02 V3
out by applying Bernoulli's equation. 0.2 + 0.1 = 0.02 V3

+- +Z 1 = -1..+_2 +
Z2 V3 = __22_=15 m / sec
0.02
p y2 p y2
1
pg 2g pg 2g
_I

Applying momentum theorem to the control


P1 = P(vf-v/) ................ (1) volume
2
By continuity equation (P 1 -P2)A2 =pA2V3 (V3 )- pA1V 1 (V 1 )
A1V1 = A2V2 -p(A2 -A1) V2 (V2)

V2
_ 10--4 X 0.1 -
-
10-6
- lO m / s (P1 -P,)= p[ v,' -�; w-(1-�:)vi]
From equation ( 1)
(P, -Pi) = {(1-�: Jv; +(�:)w-v; J

jltll4jj§jj/i4.jjjiRflblM\don+yderabadlDelhilBhopallPwielBhubancswarl LucknowlPatnalBcngalwu!ChcnnailVliayawadalVu.ag IT=pari I Kukatpallyl Kolkata I


: 302: Fluid Mechanics & Turbo Machinery

(<: -o.sJ ... Q =


0.984 X 0.0346 X 0.00866

�(0.0346) -(0.00866)
2 2

= 1oool(1 - o.5X10)2 +(o.5X 20)2 -(15)2 ] X .J2 X 9.81 X 0.5985


= 25000 N/m2 = 25 kPa = 0.03016 m3/sec = 30.16 lts/s

05. 06.
Sol: Given data: Sol: Consider the control volume which encloses
Venturimeter throat diameter (d2) = 10.5 cm the nozzle. Let F be the backward force
Inner diameter of pipe line (d1 ) = 21 cm required to hold the nozzle. This force is
Coefficient of discharge (Co) = 0.984 provided by the pipe connected to the
Inclined Manometer reading (L) = 9.5 cm nozzle.
Angle of manometer tube (0) = 30°
Let Q = volume flow rate (m3/sec)
Fluid in pipe is water Density
p = 1000 kg/m3
Applying linear momentum balance m x
:. Q,-, - Cn ��,.}zii, direction we get,
J
Where h = pressure head difference between
main pipe line and throat of venturimeter, in­
LFX =(mv)out -(mv)D + (mv1.v !
=> -F+P1 A 1 = m(V2 ) -m(V1) = 0
terms of water (in meters)
1.e F = P1 A 1 +m(V1 -V2 )
h = x (SHg -1J
SW

h = 9.5 x sin 30° (1�·6 -1)(cm)


- P, A 1 + pA 1 v,( vi-;._�,
J
(by continuity equation)
h = 59.85cm of water = 0.5985 m of water
= 3x105 xl0-3 +lOOOxl0- 3 x2x 2 -
2 X 10-

J
3
(
10_4
A 1 = 1t (0.21)2 = 0.0346 m 2
4 = 300-36 = 264 N
A2 = 1t (0. 105)2 = 0.00866 m2
4

\( l l 11�11.t t Ill!� P11hl11 .t!JPII"- �ydcrabadjDclhilBhopaljPunclBhubancswarl l.nclmowlPatnalBcngalwulChcnnailVuayawadalVmg ITuupatl I Kukatpa)lyj Kolkala I


cs Laminar Flow
The average velocity of fluid in the pipe is
R 2 dp - R 2 dp J
(a) -- (-) (b) (
One Mark Questions
8µ dx 4µ dx,
01. For a fully developed laminar flow through a R 2 dp R 2 dp
(c) -- (-) (d) - ( )
pipe, the ratio of the maximum velocity to 2µ dx µ dx
the average velocity 1s ___ (fill in the
blank) (GATE-ME-94) 04. Maximum velocity of a one-dimensional
incompressible fully developed viscous flow,
02. In fully developed laminar flow in the between two fixed parallel plates, is 6 ms- 1•
circular pipe, the head loss due to friction is The mean velocity (in ms-1) of the flow is
directly proportional to ........ . (Mean (GATE-ME-10)
velocity/square of the mean velocity) (a) 2 (b) 3 (c) 4 (d) 5
(GATE-ME-95)
05. Consider fully developed flow in a circular
pipe with negligible entrance length effects.
03. The velocity profile of a fully developed
Assuming the mass flow rate, density and
laminar flow in a straight circular pipe, as
friction factor to be constant, if the length of
shown in the figure, is given by the
the pipe is doubled and the diameter is
expression.
halved, the head loss due to friction will
- R 2 dp r2
u�)=- (-)( 1--2 J increase by a factor of (GATE -15 -Set 1)
4µ dx R
(a) 4 (b) 16 (c) 32 (d) 64
dp
Where is a constant.
dx
06. Three parallel pipes connected at the two
ends have flow-rates Q1, and Q3
R
respectively, and the corresponding frictional
Q2,

head losses are hu, hL2,and hLJ respectively.


The correct expression for total flow rate (Q)
and frictional head loss across the two ends
(GATE-ME-09)
(hI) are. (GATE -15 -Set 3)
!lfl•i@i!iiiiOQRflnjffijdijj� yderabad I Dellii I Bhopal I Pune I Bhubaneswar I Lucknow! Patna I Bengaluru I Chennai I Vijayawada jVtzag I Tirupati I Kukatpa)ly I Kolkata
"':,
�- :
"'

. &CE. PuNirarima
. . : 304 : Fluid Mechanics & Turbo Machinery
" =============================================
(a) Q = Qi + Qz + Q3 ; hL= hu + hLZ + hu 09. Consider a laminar flow at zero over a flat
(b) Q = Q i + Qz + Q3 ; hL= hu = hLZ = hu plate. The shear stress at the wall is denoted
(C) Q = Q i = Qz = Q3 ; hL= hu+ hLZ + hu by 'tw. The axial positions X 1 and Xz on the
(d) Q = Q1 = Q2 = Q3 ; hL= hu = hL2 = hu plate are measured from the leading edge in
the direction of flow. If xz>x 1 , then
07. Couette flow is characterized by (GATE - 17 - SET - 2)
(GATE-15 -Set 3)
(a) 't w J = 't w J = 0 (b) 't w J = 't w J :;e O
X1 X2 X1 X2
(a) Steady, incompressible, laminar flow
through a straight circular pipe
(b) Fully developed turbulent flow through a
straight circular pipe
(c) Steady, incompressible, laminar flow
between two fixed parallel plates Two Marks Questions
(d) Steady, incompressible, laminar flow
between one fixed plate and the other 0 1 . The discharge in m3/s for laminar flow
moving with a constant velocity through a pipe of diameter 0.04 m having a
centre line velocity of 1 .5 mis is:
08. For steady flow of a viscous incompressible (GATE- ME-88)
fluid through a circular pipe of constant (a) 31t/ 50 (b) 31t/ 2500
diameter, the average velocity in the fully
(c) 31t/ 5000 (d) 31t / 1 0000
developed region is constant. Which one of
the following statements about the average
02. For laminar flow through a long pipe, the
velocity in the developing region is TRUE?
pressure drop per unit length increases.
(GATE - 17 - SET - 1)
(GATE- ME-96)
(a) It increases until the flow is fully
(a) in linear proportion to the cross-sectional
developed.
area
(b) It is constant and is equal to the average
(b) in proportion to the diameter of the pipe
velocity in the fully developed region.
(c) in inverse proportion to the cross-
(c) It decreases until the flow is fully
sectional area
developed.
(d) in inverse proportion to the square of
(d) It is constant but is always lower than
cross sectional area.
the average velocity in the fully
developed region.
!i1••id§jjji4ijji@lfi!Miili0� yderabad I Delhi I Bhopal I Punc I Bhubaneswar I Lucknow I Patna I Bengaiwu I Chennai I Vijayawada I Vu.ag ! Tirupari I Kukatpally I Kolkata
: 305 : Laminar Flow

03. The velocity profile m fully developed keeping the volume flow rate constant, then
laminar flow in a pipe of diameter D is given the pressure drop in the pipe due to friction
by u =u 0 (1-4r 2 / D 2 ) , where r is the radial will decrease by (GATE- PI-1 1)
distance from the center. If the viscosity of (a) 33% (b) 50% (c) 70% (d) 80%
the fluid is µ, the pressure drop across a
length L of the pipe is (GATE-ME-06)
07. Water flows through a pipe having an inner
radius of 10 mm at the rate of 36 kg/hr at
µ u 0L 4µ u 0 L
(a) (b) 25° C. The viscosity of water at 25 ° C is 0.001
kg/m.s. The Reynolds number of the flow is
D2 D2

(GATE- 14) (S-1)

04. Two pipes of uniform section but different 08. For a fully developed flow of water in a pipe
diameters carry water at the same volumetric having diameter 10cm, velocity 0.1 mis and
flow rate. Water properties are the same in kinematic viscosity 10-5 m2/s, the value of
the two pipes. The Reynolds number, based Darcy friction factor is
on the pipe diameter, (GATE- PI-08) (GATE- 14) (S-1)
(a) is the same in the both pipes
(b) is large in the narrow pipe 09. Water flows through a 10mm diameter and
(c) is smaller in the narrower pipe 250 m long smooth pipe at an average
(d) depends on the pipe material velocity of O. lm/s. The density and the
viscosity of water are 997kg/m3 and 855x10-
05. The pressure drop for laminar flow of a 6N.s/m2, respectively. Assuming fully­
liquid m a smooth pipe at normal developed flow, the pressure drop (in Pa) in
temperature and pressure is (GATE- PI-09) the pipe is __ (GATE- 14) (S-2)
(a) directly proportional to density
(b) inversely proportional to density 10. Consider laminar flow of water over a flat
(c) independent of density plate of length lm. If the boundary layer
(d) proportional to (density)°-75 thickness at a distance of 0.25 m from the
leading edge of the plate is 8mm, the
06. Water is flowing through a horizontal pipe of boundary layer thickness (in mm), at a
\

1 distance of 0.75m, is
constant diameter and the flow is laminar. �f
the diameter of the pipe is increased by 50% (GATE- 14) (S-2)

!M11ih§jjji4iOMRftb1Mih1i!ii,ttyderabad ! Delhi I Bhopal I Pune I Bhubaneswarl Lucknow! Patna I Bengaluru I Chennail Vtjayawada ! Vizag I Trrupati I Kukatpallyl Kolkata I
ACE
.:-\.J=
�� · ==·=
=== cnng =·=·==========:=
Publicatioos 6=
0=
3= =rn
: =====H d=
=·= ==
M h=
ec= =
am s=
·c= =ur=
&=T =M
oo = =·=e=cy
�=hin =

1 1. A fluid of dynamic viscosity 2x 10-5 kg/m.s 15. Consider steady flow of an incompressible
and density l kg/m flows with an average
3
fluid through two long and straight pipes of
velocity of l m/s through a long duct of diameters d1 and d2 arranged in series. Both
rectangular (25 mm x 15 mm) cross-section. pipes are of equal length and the flow is
Assuming laminar flow, the pressure drop turbulent in both pipes. The friction factor
(in Pa) in the fully developed region per for turbulent flow though pipes is of the
meter length of the duct is ____ form, f = K(Reln, where K and n are known
(GATE-ME-14) (S-3) positive constants and Re is the Reynolds
number. Neglecting minor losses, the ratio
12. For a fully developed laminar flow of water of the frictional pressure drop in pipe 1 to
(dynamic viscosity 0.00 1 Pa-s) through a
pipe of radius 5 cm, the axial pressure
that in pipe 2, ( !: ) , is given by

gradient is - 1 0 Palm The magnitude of axial (GATE - 17 - SET - 1)


velocity (in mis) at a radial location of 0.2
(a) � (b) (
cm 1s (GATE -15 -Set 2)
!:
( )(5-n}

di J

13. The head loss for a laminar incompressible d2


(c) � (d)
( )(3-n ) ( ) +n )

di di
{5

flow through a horizontal circular pipe is h 1


Pipe length and fluid remaining the same, if
the average flow velocity doubles and the
pipe diameter reduces to half its previous
value, the head loss is h2. The ratio h2/h 1 is
(GATE -15 -Set 2)
Five Marks Questions
(a) 1 (b) 4 (c) 8 (d) 1 6
0 1. A 0.20 m diameter pipe 20 km long
14. Consider a fully developed steady laminar transports oil at a flow rate of O.Ol m3/s.
flow of an incompressible fluid with Calculate the power required to maintain the
viscosity µ through a circular pipe of radius flow if the dynamic viscosity and density of
R. Given that the velocity at a radial location oil are 0.08 Pa-sec and 900 Kg/m3 •
of R/2 from the centerline of the pipe is U 1 , (GATE-ME-88)
the shear stress at the wall is KµU 1 /R, where
K is ___ (GATE - 16 - SET - 3)

l11•1i@jjj§§jjj\4Rflnijjjlj.m.� yderabad l Dellii I Bhopal I Pune j Bhubaneswarj Lucknow I Patna! Bengaluru I Chennai j Vrjayawada l Vu.ag I T�pati I Kukatpallyl Kolkata
SOLUTIONS
. =-
v
max
. . vmean -
One Mark Solutions 1 .5
6
: VMean = -= 4m/s
0 1 . Ans : Twice . 1 .5

02. 05. Ans: (d)


_ 32µVR
Sol: Mean velocity, h f Sol: h f
4fLV 2
-- (·: Q = AV)
pgd 2
=

2gD
:. h f oc V 2
4fL-- 4fLQ 2
Q = -----'--
-
h - -
' 2gDA '
2g x ( :) ' D'
03. Ans : (a)
Sol: Average velocity lies at a distance of L
}i
h f oc -5 (If f, Q are cons tan t)
r= from centre of pipe. D

:. Vavg
2
= -R ( )
dp
4 µ dx
[i-�J 2R 2
=
- R 2 dp
( )
8 µ dx
Note: Quantity outside bracket represent
hr 1
maximum velocity. Average velocity _1 = -
hf2 64
is half of maximum velocity.
h r2 = 64 h rI
04. Ans: (c)
Sol: Given data : 06. Ans: (b)
Maximum velocity, VMax = 6m/s Sol:
Mean velocity, VMean -?
- •

Between Fixed parallel plates of laminar


vmax = 1 .5
flow, -- Q = Q1 + Q2 + Q3
vmean
h f I = h r2 = h f) = h L

Fydcrabad j Delhi j BhopaljPuncJ Bhubaneswarl wcknow j Patna j Bcngaluru j Chcnnai J Vtjayawada j Vu.ag I Tuupari I Kukatpallyj Kolkala I
07. Ans: (d)
Sol: Couette flow is the laminar flow of a
Two Marks Solutions
viscous fluid m the space between two
parallel plates, one of which is moving
01. Ans (d)
relative to the other. The flow is driven by
Sol: Given data :
virtue of viscous drag force acting on the
Diameter of pipe (d) = 0.04m
fluid and the applied pressure gradient
Centre line velocity (Vmax) = 1.5 mis
parallel to the plates. Discharge (Q) = ?
Vmax
08. Ans: (b) For pipe flow, =2
vavg
Sol: Average velocity is defined as ratio of

2
discharge and area. In fully developed :. Vavg = � = 0.75 m / s

region average velocity is constant hence


discharge is constant. As long as discharge
Discharge (Q) = A.Vavg = 7t
4
(0.04 ) x 0.75
2

= 7t (
2
and area is constant average velocity in 4 75 n x 4 x 75
) =
developing region as well has to be constant. 4 100 X 100 1000000

n x 300 3n 3 (m
10000 00 10000 Sec J
= =
09. Ans: (c)
(X ) 1 1 0.332
Sol: = -Crx P Uoo = - x
2
x pU<X)2
2 2 pU00 x
'tw
02. Ans: (c)
µ
t'¥ Sol: In Laminar flow through a long pipe, the
1 32 µ. V. L
pressure drop (hr) =
pg . D 2
oc -
,J-;.
3 2 µ.�
Pressure drop per unit length =
pg .D
hr cx: - 1
L D
- 2

Pressure drop per unit length is inversely


proportional to the cross - sectional area.

!lflNM@Oiiiih4AflniM\jjih+yderabad l Delhi l Bbopal l Pwic l Bhubancswarl Lucknow l Patnal Bcnga)urui Chcnnai l VliayawadaJ Vmg ITlfllpati I Kukatpallyi Kolkata I
" " ACE
�•-�PuNiramw
. . . . : 309: Laminar Flow

03 . Ans: (d) (2) = (1)


Sol: For fully developed laminar flow, the d u - ap r
µ . - =- . -
velocity distribution is given by dr ax 2
U = Uo (1-�J
J] =! .
D2

Where, D =Diameter of pipe 4r 2


� [u O 8p . r
r = Radial distance from the centre line of pipe dr D2 2µ
(1- =
ox

Circular pipe
u0 [o -�] = -
D2
I 8p . r
2µ ox
8r 2µ 16µu 0
Ap
=U o ( 2 ) ( )=

..u
D r D2
L L

Shear
04. Ans : (b)
Velocity stres

* d2
profile
Sol: d1

R e, = v1 .D 1 ,
V1

I· L
·I
Shear stress (t) across a section varies with Similarly

'r' as 1
ap
R e oc (Keeping volume flow rate and
D
ax
ap r
t =- .- -- - (1) fluid properties same)
ax 2
Newton's law of viscosity across section, 05. Ans: (c)
du Sol: Pressure drop for laminar flow of fluid in
-r =µ . [where y =R -r :. dy =0 -dr]
dy
� 32µUL
[Negative sign is used, as r is measured from pipe P=
centre line of profile]
D2
Where,
du µ = Dynamic viscosity of liquid (Pa-se�)
t =-µ . - ___ (2)
dr
\( } l 11i.:,111t \ ! I l l.., Pllhl1t ,1111)[1-. Fyderabad l Dclhi l Bhopal l Pune l Bhubaneswar l Lucknow l Patna l Bengaluru l Chennai l Vliayawada l Vizag l 'Hrupali I Kukatpally l Kolkata I
U = Average velocity (mis) Af> oc ­1
L = Length of pipe (m) D4
Pressure drop in the pipe decrease by
D = Diameter of pipe (m)
(1 .SD ) -(D )
4 4
Pressure drop is independent of density, = = 0.8 = 80%
(1.5D )
4

06. Ans : (d)


07. Ans: 635 to 638
Sol: Given data:
Sol: m = p Q = pAV = p 1t R2V

re--
:WATER
R =
� = l OOO x n(0.01) 2 x V
3600
:. V = 0.0318 mlsec
pVD

-:,,:.x--'"*'"�,• ---- •--- ·-!--¥--,cc. µ


,I Laminar flow
I000 x 0.03 1 8 x (2 x 0.01)
Re = = 636.62
I

---- L ---�
0.001

Let D 1 = D = Diameter of pipe (m)


08 . Ans: 0.06 to 0.07
D2 = l .5D1 = 1.5 D
VD V .D
Q = volume flow rate (m3/sec) (remains constant) Sol: R e = p =
µ V

Pressure drop in the pipe O . l x O. l


= = 1000 < 2000
32 µ 10-s
= �p = � (For laminar flow)
D : . For laminar flow,
Where, µ = Dynamic viscosity of fluid (Pa-sec) 64
f= = � = 0.064
L = Length of pipe (m) Re 1000

U = Average (or) mean velocity of fluid (mis)


09 . Ans: 6840
For given condition, U =
Q
= _g_ --0
Sol: �p =
3 2 µV£ 3 2 x 855 x 1 0 x 0. 1 x 250
A 1t D 2
=
d2 (0.01) 2
1 28 µ�L = 6840 N/m2 or Pa
:. Af> =
nD

!IJllii@jjjd@iAflbiM\hii!+Yderabad I Delhi I Bhopal I Punc I Bhubancswarl Lucknow I Pama I Bcngaluru I Chennai !Voayawada IVtzag I Tirupati I KukalpaJJy I Kolkata I
: 31 1 : Laminar Flow

10. Ans: 13.5. to 14.2 :. Darcy's friction factor,


Sol: In laminar boundary layer region 64
f= = � = 0.0683
R e 937.5
f..f..V 2 p.f.R.V 2
= pg h f =p.g =
2gD m
An
l..1r

lx0.0683xlx(1) 2
= = l.8N / m 2 = l.8Pa
2 X 0.01875
x 1=0.25m
I
4 �,

j.--x2 =0.75m----+I

12. Ans: 6.2 to 6.3


14 L= l m---

Sol: µ = 0.001 Pa-s , R = 5 cm, - = l OPa / s


dP
dL

� �0.25 dP 32µVavg
= =
82 0.75 dL d2
:. 82 = 13.86 mm 32 X 0.001 X Vavg
=> lQ =
(10x10-2 J
11. Ans : 1.7 to 2.0
=> Yavg = 3.125 mis
Sol:
V or laminar flow
b� Ymax = 2 X Yavg = 2x3.125 = 6.25 mis

)'J-

0
V, - 6.2{1- ( / 6.24 m /sec
2ab
Hydraulic mean diameter (Dm) =
a+b
25x15 13. Ans: (c)
Dm = 2 ( )
25 + 15 Sol: For laminar flow
Dm = 18.75mm = 0.01875m 32µVL
hr =
pgd 2
p.V.D m 1 X 1 X 0.01875
V2 = 2v 1
Re = = d
2
µ 2 x 10 � , d 1 _ _2
Re = 937.5 < 2000 (Laminar flow)
-

.\( l. l .11�l ! ll t I I l l!.!, Ptildh ,ll!Olh �yderahad j Delhi j Bhopalj PuneJ Bhubaneswarl Lucknow j Palna j Bengaluru j Chennai l VtiayawadaJVu.ag jTirupali I KukatpallyJ Kolkala I
':,
"•
"�Publirabl
' ;a,." ACE . . . . : 312 : Fluid Mechanics & Turbo Machinery

r 14. Ans: 2.667 (range 2.6 to 2.7)


_ 8P �
( )
ax
2
(
't
Sol: w =

n 5-n
8P -n d;- d
u = -� ( )(1-__c_2 J = 2 -n =
J
2
� d 5- n
•.. �pl Q
dI
2
4µ dx R
X
2 I
Q

R 2 8P - R 2 R 2 8P l
u· 1 = - ( ) (l = - ( )x
4µ c1x 4 x R2 J 4µ 4 ax Five Marks Solutions
8P l6µu 1
(- )= 01.
ax 3R 2 Sol: Diameter of pipe (d) = 0.2m
8P R K x µ x u Length of pipe (l) = 20 km = 20,000 m
= -- x - = ---1
2 axR
't
Oil flow rate (Q) = 0.01 m 3 / Sec
w

16 x µ x--C..
- u R K x µxu Dynamic viscosity of oil ( µ )=0.08 pa-sec
-2 1 x - = __ 1
3R 2 R Density of oil (p) = 900kg/m 3
___c_

: . K = l6µ 2u 1 R _!_ �
= = 2.66
Loss of head due to viscosity (hL) of oil,
x

3R 2 µu 1 3 32µVf
x x

hL=
pgd 2
Discharge (Q) = Area x velocity (Average)
15. Ans: (a)
. fLy2 ptl.,y2 = d2 X V
Sol: �P = pghr = pg x -- = -- 4
7t

2gd 2d
0.01 =
For both the pipes p & L are same 4( 0.2)2 x v
7t

fV 2 :. V = 0.3183 m/Sec
:.� oc -
. h _ 32 X 0.08 X 0.3183 X 20000
900x9.8Ix(0.2) 2
Now V= _Q_2
.. L -
Q
A d hL= 46.146 meters of oil
oc

Power required to maintain the flow against


-n

viscosity of oil
p = pgQh L
= 900 x 9.8 1 X 0.01 X 46. 146
= 4074.23 Watt
= 4.074 kW

\( l I r1;..,111t t 1 11 1� PulilH ,1[101 1.., �ydcrahadl Delhi l Bhopal l PuncJBhu�I Luclutow l Patna l Bcnga)uru l Chcnnai j VtjayawadaJVw.ag JTirupari I KukatpallyJ Kolkata I
Turbulent Flow
04. Oil flows through a 200mm diameter
One Mark Questions horizontal cast iron pipe (friction factor,
f = 0.0225) of length 500m. The volumetric
01 . The parameters which determines the friction
flow rate is 0.2m3Is. The head loss (in m) due
factor for turbulent flow in a rough pipe are:
(GATE-ME-88)
to friction is (assume g = 9 .81 m/s2)
(a) 116.18 (b) 0.116
(a) Froude number and relative roughness
(c) 18.22 (d) 232.36
(b) Froude number and Mach number
(c) Reynolds number and relative roughness.
05. For steady, fully developed flow inside a
(d) Mach number and relative roughness
straight pipe of diameter D, neglecting
02. Prandtl's mixing length in turbulent flow gravity effects, the pressure drop �p over a
signifies (GATE-ME-94) length L and the wall shear stress 'tw are
(a) the average distance perpendicular to the related by (GATE-ME-13)
mean flow covered by the mixing pD pD 2
(a) t w = � (b) t w = �
particles. 4L 4L2
(b) the ratio of mean free path to pD
(c) t w = �
characteristic length of the flow field. 2L
(c) the wavelength corresponding to the
lowest frequency present in the flow field. 06. Consider the turbulent flow of a fluid
(d) the magnitude of turbulent kinetic energy. through a circular pipe of diameter, D.
Identify the correct pair of statements.
03. Consider steady laminar incompressible axi­ I. The fluid is well-mixed
symmetric fully developed viscous flow II. The fluid is unmixed
through a straight circular pipe of constant III. Ren < 2300
cross-sectional area at a Reynolds number of IV. Ren > 2300 (GATE-ME-14) (S-3)
5. The ratio of inertia force to viscous force (a) I, III (b) II, IV
on a fluid particle is (GATE-ME-07) (c) II, III (d) I, IV

(a) 5 (b) !5 (c) 0 (d) CX)

!ltllih§Oillih4RdliiM!llh+1Yderabadl Delhi !Bhopal IPune IBhubaneswarl Lucknow! Palnal Bengaluru I ChennailVuayawadalV123g I Ttrupati I Kukatpal)yl Kolkaaa !
,r ' .
· · · .·.·. ·.·.·.·. .· · · ·
07. The instantaneous stream-wise velocity of a
. .. ...
turbulent flow is given as follows: � �
I I

u(x,y,z,t) = u (x,y,z) +µ' (x,y, z,t) � LO


.. : * :
•:•:•:•f:•:•:• • : • : • : • : •! • : • : • : • : •

The time -average of the fluctuating velocity I

u' (x,y, z, t)
L
(GATE - 16 - SET - 1) TANK. I
t !
(a) u'/2 (c) zero (d)
TANK. 2
u
(b) -- u
2 2
03. Fluid is flowing with an average velocity of
V through a pipe of diameter D. Over a
Two Marks Questions length of L, the "head" loss is given by
fLV 2
The friction factor, f, for laminar
01. In the case of turbulent flow of a fluid through 2gD

a circular tube (as compared to the case of flow in terms of Reynolds number (Re) is
laminar flow at the same flow rate) the ___ (fill the blank) (GATE-ME-94)
maximum velocity is .................., shear
stress at the wall ts 04. The discharge velocity at the pipe exit in
. . . . . . . . . . . . . . . . . . . . . . . . . . . . . . . , and the pressure figure is (GATE- ME-98)
drop across a given length is .............. The
correct words for the blanks are, respectively: h
(GATE- ME-87)
(a) higher, higher, higher
(b) higher, lower, lower
(c) lower, higher, higher
(d) lower, higher, lower (a) figli (b) �2gh
(c) �g(H+h) (d) 0
02. Shown below are three tanks, tank 1 without
an orifice tube and tanks 2 and 3 with orifice
tubes as shown. Neglecting losses and Statement for Linked Answer QOS & Q06
assuming the diameter of orifice to be much A syringe with a frictionless plunger contains
less than that of the tank, write expressions water and has at its end a 100 mm long needle of
for the exit velocity in each of the three 1 mm diameter. The internal diameter of the
tanks. (GATE-ME-93) syringe is 10 mm. Water density is 1000 kg/m3 •

FyderabadJ Delhi J Bhopa) J PuncJ Bhubaneswarl I..ucknow J Patna l Bcngaluru l Chennai l Vuayawada l Vizag I TIIUpati I Kukatpally j Kolkala I
: 315 : Turbulent Flow

The plunger is pushed in at 10 mm/s and the (b) �


(GATE-ME-03)
(a) figh;
water comes out as a jet
(c) �2g (h 2 - h 1 ) (d) �2g (h 2 + h,)
1 0 mm/s 10 mm � 1 00 mm �
I
--+(Jl======ll I : :
�1 mm - -Water
i Nee01e =---=-.:-:. : -
jet
08. Oil is being pumped through a straight pipe,
the pipe length, diameter and volumetric flow
Syringe rate are all doubled in a new arrangement.
05. Assuming ideal flow, the force F in newtons The pipe friction factor, however, remains
required on the plunger to push out the water constant. The ratio of pipe frictional losses in
the new arrangement to that in the original
(GATE-PI-08)
lS

(a) 0 (b) 0.04 (c) 0.13 (d) 1.15 configuration would be


1
(a) - (b) _!_ (c) 2 (d) 4
06. Neglect losses in the cylinder and assume 4 2
fully developed laminar viscous flow
throughout the needle; the Darcy friction 09 . Water at 25 °C is following through a 1 .0 km
factor is 64t'Re, Where Re is the Reynolds long G.I. pipe of 200 mm diameter at the rate
number. Given that the viscosity of water is of 0.07 m3/s. If value of Darcy friction factor
1.0x l0-3 kg/m s, the force F in Newton for this pipe is 0.02 and density of water is
required on the plunger is 1000 kglm3 , the pumping power (in kW)
(a) 0.13 (b) 0.16 (c) 0.3 (d) 4.4 required to maintain the flow is
(GATE-ME-09)
07. A siphon draws water from a reservoir and (a) 1.8 (b) 17.4 (c) 20.5 (d) 41.0
discharges it out at atmospheric pressure.
Assuming ideal fluid and the reservoir is 10. In a fully developed turbulent flow through a
large, the velocity at point P in the siphon circular pipe, a head loss of h1 is observed.
(GATE-ME-06)

-
tube is The diameter of the pipe is increased by 10%
p for the same flow rate and a head loss of h2 is
noted. Assume friction factor for both the
cases of pipe flow is the same. The ratio of

� is closest to (GATE - PI-16)


.. .. . .
... .
... · · .. .. .. .
..
h,
(a) 0.34 (b) 0.62
. . . . .. .

: : ..) /<:.. . :::. :


. .
. . .

(c) 0.87 (d) 1.00


... ... .. .. ... .. ... .. ... .. . . ... ... .. .. ...

\( l } [ IL,ll ll l [ ll!L, P11lilH <1IJl )[I', �ydcrabadj Delhi jBhopa) I Punc 1Bhubancswarl Lucknow I Pama I Bcngalwu j Chcnnai j VgayawadajVmg l'!"irupati I Kukatpallyj Kolkata I
"
:.,,..,...�
..-�"' . .
ACE . : 316 : Fluid Mechanics & Turbo Machinery

Five Marks Questions

01. A 400m long horizontal pipe is to deliver 900


kg of oil (S=0.9, u =0.0002m 2 / s ) per
minute. If the head loss is not to exceed 8 m
of oil, find the pipe diameter.
(Friction factor in laminar flow: f = 64 / R J
(GATE-ME-89)



I Delhi I Bhopal I Pune I Bhubaneawarl 1.ucknowl Patna.I Benpluru I Chcnnai IVuayawada I Vmg I Tuupati I Kukaq,ally I Kolkata I
SOLUTIONS
03. Ans : (a)
One Mark Solutions Sol: Reynold's Number =5
. Inertia force
1.e ----- = 5
01. Ans:(c) Viscous force
Sol:
04. Ans: (a)
Q
2
{ -)
f..f.v f..e A
Sol: hr = =
2

2gd 2gd
f.£.Q 2

12.lxd
VD
Reynolds number, Re = -
V
= ---5

0.0225x500x(0.2) 2
From Moody's diagram it can be concluded = 116_18 m
12.1X (0.2) 5
_

that for laminar region friction factor


depends only on Reynolds number and for
05. Ans: (a)
fully developed turbulent flow it depends
Sol: Shear stress at wall of pipe,
only on surface roughness. It transition zone
- .-dp R
it depends on both Re and relative 't w = --
dx 2
roughness. However of the available
options, (c) is correct.
LiP (�) LiP.D
L 2 4L
't w = - -- =
02. No Answer , Nearest Answer is (a)
Sol: Mixing length is the distance one must 06. Ans: (d)
move transversely to the direction of flow Sol: In turbulent flow, the fluid is well mixed
such that u=u 1 • and Reynolds number is more than 2300.
u =Velocity of flow in X-direction
u 1 = fluctuation component of velocity in 07. Ans: (c)
the direction of 'u' Sol: Time average of fluctuating velocity is zero.

\( 'I I 1 1.., 1 1 t l l r , 1 1 .., P11!1l1< .1111 > 1 \ , ydcrabad I Delhi I Bhopal I Pune I Bhubaneswar l Lucknow! Patna I Bcnga1uru I Chennai ! Vuayawadal Vizat I �pati I � I �
..
"
:
V ACE
�•· r..,,,,,.Pnlimnllfl
• • • • : 318 : Fluid Mechanics & Turbo Machinery

05. Ans: (b)


Sol: Given data
Two Marks Solutions
D =lOmm

L-----"--� d =lmm

-�-:-�1
-�1 : -_ :. : : :

�q\:_- -
needle Vz, .<;;
01. Ans : (c)
-f�.-
10 mm/s
F

Sol: By comparing velocity profile between


£;100-�� - -

laminar and turbulent flow it is clear that


syiinge

average velocity of turbulent flow is less as D = lO mm = 0.01 m


V 1 = 10 mm/sec = 0.01 m/sec
velocity profile is flat. Wall shear stress is
d = 1 mm = 0.001 m
more in case turbulent flow because
velocity gradient more near to wall. As wall
1. Eqn of continuity between {1) & {2):
shear is stress more pressure drop is also
A1V1 = A2V2
more in case turbulent flow.
1t
4
D 2 VI = 1t
4
d 2 V2
02. 0.01
D
V2 = ( ) · V = (
2 2

Sol: For tank 1: ) x0.01


d I 0.001
Exit velocity V= �2gH V2 = 1 m/sec
For tank 2:
Exit velocity V= �2g(H+L) 2. Applying Bernoulli's Eqn. between (1) &
{2):-
For tank 3:
E.!_+z, +-' = P 2 +z+-2 +h L
y2 y2
Exit velocity V=�2g(H + L )
pg 2g p g 2 2g
[ Z1 =Z2), hL =0
64 P2 = 0 (Gauge pressure = 0)
03. Ans : -
Re
E.!_ v/ v]
+ =o+
p g 2g 2g
04. Ans: (b)
PI V22 - vz
Sol: Discharge velocity at exit of pipe = �2gh =
pg 2g
I

(velocity depends upon net head over exit)


PI =P (v22 - vI2 )
2

Fydcrabad l Delhi l BhopaJ I Puncl Bhubaneswarl Lucknowl Patna l BengaJurul Chennai l Vliayawada l Vu.ag I Tuupati I Kukatpallyl Kolkata I
: 319 : Turbulent Flow

1000 2
P1 = (1 -O.O I 2 )
2
P1 = 499.95 N / m 2 P1 v; - v/
= ---+ h f [where y = pg]
Fore on Plunger _ F_ pg 2g
p1 = =
Area of plunger 1t 0 2 p
4 => P1 = (V; -V/ ) +pgh r
F
2
499.95 = lOOO
p1 = ((1) 2 -(0.01) 2 )+ lOOO x9.81x 0.3262
1t (0.01) 2
2
P1 = 499.95+3200 � 3 700 N/m 2
4
:. F = 0.0393 say 0.04(N)
Force on Plunger
P = Pr essure on Plunger = -----­
Area of Plunger
06. Ans (c)
1

F
V.D 3 700 = => F � 0.3N
Sol: Reynolds number of water in needle = 1t (0.01) 2

Re = p.V.D = lOOOxlx0.001
V
4
= lOOO 07. Ans:(c)
µ 1.0xl0-3
Since, Re is less than 2000, the flow 1s Sol: Applying continuity equation and law of
Laminar. conservation of energy;
The velocity at P and at end of pipe (open to
f = Darcy friction factor
atmosphere end) is same since diameter of
64
= = � = 0.064 pipe is same i.e.,Vp = Vend
Re 1000
Kinetic energy of water = potential energy
. f.R.V 2
Head 1oss m needle , h r =
2gd
2
0.064 X 0. 1 X (1)
2 X 9.8 1 X 0.00 1

h r = 0.3262m of Water
Applying Bernoulli's Equation at point 1
and point 2
p1 y p y
+Z +- ' = -2 + Z 2 + - 2 + hr
2 2

Y 2g Y 2g
_
I

·: Z 1 = Z 2 , P2cgauge> = O(Atmosphere)
\( I I 11�1 1 1t t 1 1 1 1 .., P11hlh .111, , 1 1 , �ydcrabadlDclhilBbopallPunclBhubancswarl Lucknow 1 Patna 1 BengaJurulCbennai l V1iayawadajVizag IT1n1pati I Kukatpally!Kolkata I
: 320 : Fluid Mechanics & Turbo Machinery

08. Ans :(a)


Sol: L2 = 2L1 Five Marks Solutions
d2 2d2 ;
= fl = f2
Q2 =2Q1 , 01.
_ 8fLQ 2 Sol: Length of horizontal pipe (L)= 400 m
h f --- 5 (hr interms of discharge)
1t gd
Mass flow rate ( m) 900 kg/min
2

Specific Gravity of oil (s) = 0.9


Kinematic viscosity of oil ( v )
= (2) (2)2 ( ;s ) =�: = :
= 0.0002 m 2 / S
hL=Loss of head =8 m of oil
09. Ans : (b)
64
Sol: Length of Pipe (L) = 1 km = 1000 m Friction factor for laminar flow f =
Re
Diameter of pipe (d) = 200 mm = 0.2 m Mass density of oil
Rate of flow (Q) of water = 0.07 m3/s
kg
p =S X Pwater =0. 9 X 1000 =900 -
Density of water ( p ) = 1000 kg / m3
m3
Darcy's friction factor , f = 0.02
Diameter of pipe, d = ?
8fLQ 2
Head loss hr = = 25.3m
1t gd
2 5 By continuity equation: m = pQ
Pumping Power p. g. Q.hr (Watt) 900
(kg/Sec)= 900 x Q
=

= 1000 X 9.81 X 0.07 X 25.3 60


;e; l 7.4 kW :. Q=0.0167 m3 / Sec
Q = AV
10. Ans: (b)
fLQ2 0.0167 = (d 2 )x V
1t

For turbulent flow h =


12. ld 5
Sol: 4

d 2 V = 0.02122 _(l )
h OC -5 � -2 = =0.62
5
1 h I d
d h i (l . l d 1 J Reynolds Number,
V.d V.d
Re = = = 5000V.d
V 0.0002

Fyderabad j Delhi j Bhopal j Pune J BbubancawarJ urlnowJ Patna J Benga!uru J Chennai J Vuayawada J Vu.ag J T1n1pati I Kukatpal)yj Kolkat, I
: 321 : Turbulent Flow

Friction factor
64 64 0.0128
(f ) = = =
V.d
Re 5000 V.d
Darcy's friction head loss in pipe flow
f.L.V 2
hc -
2gd
0.0128
x400xV 2
V.d
8 = ----- = 0.0128x 400V
2x9.8I xd 2x9.81 d 2
0.261V V
8 = -- 2 => -2 = 30 .656 ------ (2)
d d

Solving Eq (1) and (2), we get


d = 0.162 m say 162 mm.

\( I I 11�1 ! \ ( t J l l l!.!, P11h]H ,IIJ1)J[" �yderabad I DelltiJBhopal I Pune I BhubaneswarJ LJcknowJPatna I Bengaluru I ChennaiJVtjayawadaJVu.ag JTuupali I Kukatpally I Kolkala I
C7 Boundary Layer
05. Flow separation in flow past a solid object is
One Mark Questions caused by (GATE-ME-02)
(a) A reduction of pressure to vapour
01. A streamlined body is defined as a body pressure
about which (GATE-ME-87) (b) A negative pressure gradient
(a) The flow is laminar (c) A positive pressure gradient
(b) The flow is along the streamlines (d) The boundary layer thickness reducing
(c) The flow separation is suppressed to zero
(d) The drag is zero
06. If 'x' is the distance measured from the
02. The predominant forces acting on an element leading edge of a flat plate, then laminar
of fluid in the boundary layer over a flat plate boundary layer thickness varies as
in a uniform parallel stream are: (GATE-ME-02)
(GATE-ME-90) (a) 1/x (b) X 4 5
I (C) x 2
(d) X 112
(a) Viscous and pressure forces
(b) Viscous and inertia force 07. Consider an incompressible laminar
(c) Viscous and body forces boundary layer flow over a flat plate of
(d) Inertia and pressure forces length L, aligned with the direction of an
incoming uniform free stream. If F is the
03. The necessary and sufficient condition which ratio of the drag force on the front half of the
brings about separation of boundary layer is plate to the drag force on the rear half, then
dP (GATE-ME-07)
- > 0 (True /False) (GATE - ME-94)
dx (a) F < ½ (b) F = ½
(c) F = 1 (d) F > 1
04. As the transition from laminar to turbulent
flow is induced in a cross flow past a circular 08. Within a boundary layer for a steady
cylinder the value of the drag coefficient incompressible flow, the Bernoulli equation
drops. State: True /False. (GATE -15 -Set 2)
(GATE-ME-94)
!IHIQ@jjji4ijjjjAfllftih•Mfjiydcrabad l DelhilBhopal l Pune lBhubancswarl LuclmowlPamalBengaiurul ChennailVliayawadalVm,g IT1111pati I Kukatpallyl Kolkata I
: 323 : Boundary Layer

(a) holds because the flow is steady


(b)
(c)
holds because the flow is incompressible
holds because the flow is transitional
-
- · q r U

-
u I

(d) does not hold because the flow lS


I

frictional p

Two Marks Questions 02. The mass flow rate (in kg/s) across the
section q-r is
01. For air flow over a flat plate, velocity (U) (a) zero (b) 0.05 (c) 0.10 (d) 0.15
and boundary layer thickness (8) can be
expressed respectively, as 03. The integrated drag force (in N) on the plate,
between p-s, is
::. = ¾� - �(�)' ; 0 = � (a) 0.67 (b) 0.33 (c) 0.17 (d) zero

If the free stream velocity is 2 mis, and air


Statement for Linked Answer Q04 & Q05
has Kinematic viscosity of 1.5x10-5m2/s and
Consider a steady incompressible flow through a
density of 1.23 kg/m3 , then wall shear stress
channel as shown below.
at x = 1 m, is (GATE-ME-04)

(a) 2.36 x 102 N/m2 (b) 43.6 x 10-3 N/m2


(c) 4.36 x 10-3 N/m2 (d) 2.18 x 10-3 N/m2
H

Statement for Linked Answer Q02 & Q03

A smooth flat plate with a sharp leading edge is A B X

placed along a gas stream flowing at U = 10 mis. The velocity profile is uniform with a value
The thickness of the boundary layer at section r-s of Uo at the inlet section A. The velocity
is 10 mm, the breadth of the plate is 1 m (into the profile at section B down stream is
paper) and the density of the gas, p = 1.0 kg/m3 . y
vm 8'
Assume that the boundary layer is thin, two -
o ::; y ::; 8

dimensional, and follows a linear velocity


U= vm , 8 ::; y ::; H -8

distribution, u = U (y/8), at the section r-s, where


H - y H - 8 ::; y ::; H
Vm -8- ,
y is the height from plate. (GATE-ME-07)
\( I } 1 11!,l llt t l l l l l!, P11h]J< ,t!JOII', �yderabad I Delhi lBhopal I PunelBhubaneswarl Lucknow I Patnal Bengaluruj Chcnnai I Vuayawadaj Vizag ITirupari I Kukatpallyl Kolkala I
: 324 : Fluid Mechanics & Turbo Machinery

V coefficient is Cr = , the total drag force


04. The ratio m is

Uo Re x
1 (in N) per unit width of the plate is _ _
(a) (b) 1
1-2(8 / H) (GATE -15 -Set 1)
1
(c)
1-(o / H) (d) 1 + / 08. A steady laminar boundary layer is formed
over a flat plate as shown in the figure. The
free stream velocity of the fluid is U0• The
p -P
05. The ratio t : (where P and P 8 are the velocity profile at the inlet a-b is uniform,
2PU o
A
while that at a downstream location c-d is
pressures at section A and B respectively
given by u =u. [ 2(n-(�)']
and p is the density of the fluid) is
1
(a) -
(1-2(o / H)) 2 I
(b)
(1-(8\ H))2 u
(c)
1
-
(1-(2o / H)) 2 I
(d)
(1+( o\ H)) 2
u. �---------r�"'.'- ----�: . Io
.
a�-Cy'

The ratio of the mass flow rate, rilbd leaving


06. An incompressible fluid flows over a flat through the horizontal section b-d to that
plate with zero pressure gradient. The entering through the vertical section a-b is
boundary layer thickness is 1mm at a (GATE -16 -SET -1)
location where the Reynolds number is 1000.
If the velocity of the fluid alone is increased 09. The velocity profile inside the boundary
by a factor of 4, then the boundary layer layer for flow over a flat plate is given as
thickness sat the same location, in mm will 7t y
� = sin ( ) , where Uoo is the free stream
be (GATE-ME-12) uoo 28

(a) 4 (b) 2 (c) 0.5 (d) 0.25 velocity and '8' is the local boundary layer
thickness. If o * is the local displacement
07. Air ( p =1.2 kg/m and kinematic viscosity,
3
8*
v = 2x10-5m2/s) with a velocity of 2m/s 8
thickness, the value of is

flows over the top surface of a flat plate of (GATE - 17 -SET -l)
2
length 2.5m. If the average value of friction (a) - (b) 1 -� (C) 1+� (d) 0
7t 7t 7t

Fyderabad I Delhi I Bhopal I Pune I Bhubaneswar I Lucknow I Patna I Bengaluru I O.Cnnai I Vuayawada I V17.3g I Tirupali I Kukatpally I Kolkala I
" . ACE . . : 325 :
�"··�:,,.........PuHir,qg Boundaiy Layer

(a) Use an appropriate control volume to


determine the rate of mass influx into the
Five Marks Questions
boundary layer up to x.
(b) Obtain the momentum thickness into the
01. The velocity profile across a boundary layer
boundary layer up to x.
on a flat plate may be approximated as linear
(c) In which direction (up or down) does the
Vo Y
V (x , y) -
- shear stress act on the face AB of the
8(x)
fluid element shown near the plate?
X

Where Vo is the velocity far away and 8(x) is


(GATE-ME-93)
the boundary layer thickness at a distance x
from the leading edge, as shown below.

Vo
YL Yo

,,.,•'

I
I
�yderahadl DelhilBhopall PunelBhubancswarl Lucknow l l'alnalBcngalwu l ChennailVuayawadalVizag ITU'Upali I Kukalpal)yl Kolkala I
SOLUTIONS
05. Ans : (c)
One Mark Solutions
06. Ans : (d)
0 1 . Ans : (c) eg : Aero foil Sol: For Laminar boundary payer 8 oc x 1 1 2
For Turbulent boundary layer 8oc x 4 1 5
02. Ans : (b)

03. Ans: True 07. Ans : (d)


Sol: For separation to occur, two conditions are Sol:
to be satisfied.
(i) There should be adverse pressure
. . dP >
grad1ent 1.e., - O
dx A = Leading Edge
(ii) Mechanisms such as friction to reduce B = Trailing Edge
the K.E of the fluid stream should exist Shear stress decreases from leading edge to
in the flow. trailing edge.
In case of flat plate for which flow is parallel
04. Ans: True drag force is predominantly due to viscous
Sol: Turbulent boundary layer separates late force.
because of its comparatively high Hence drag force on front half is more than
momentum flux near wall. Therefore wake rear half.
of turbulent flow over cylinder is narrower � = Drag force an front half >
... F = 1
as compared laminar flow. This reduces FR Drag force an rearhalf
pressure drag hence overall drag is reduced.
08. Ans: (d)

�-.-9 �
Laminar Turbulent
0 = 82° 0 = 120°

\( I l 1 1141 11t ( 1 111l4 P11hlu ,1IH•11..., yderabad I DeUtil Bhopal I Punc I Bhubancswarl Lucknow I Patna I Bcngaluru I Chennai I Vuayawadal Vizag I TlfUpati I Kukatpa))y l Kolkata
: 327: Boundary Layer

Differentiate 'u' w.r.t 'y '

Two Marks Solutions

01. Ans : (c)


Sol: Given data:
U 00 = Free stream velocity of air = 2 m/sec 3U 00
- --
u = Kinematics viscosity of air 28
= 1.5 x 10 -5 m2/sec.
3 U 00
't wall = µ. U
k
p = Density of air = 1.23 �
m
3.U
= u. p. - -00
us: = Boundary 1ayer th'1ckness =
4.64 X
v�
R e,
2.8

Where x = Distance from leading edge (m) = l m = (1.5 X 10-5 X 1.23) x __3_ 2_
2 X 0.0127
x_

u= Velocity of stream at given section.


We know that Reynolds number of stream of
air flow at section x is
02. Ans: (b)
2Xl
= = 1.33 X 10 5 < 5 X 10 5
U. X

V 1.5 X 10-5
Sol: m = pQ

(Critical Reynold's Number for laminar =p dA.U
flow)

O=
4.64 x 4.64xl = p. B.dy. U y = p . B. -.
u Jo d
JR::
y y
.J1,33 X 10 6
= 8 8

�.[�I -
0

=0.0127 (m) =12.7 mm - p.B. p.B. � . �

Shear stress ( -r wan ) = µ ( : J


u
y y=O 1
= -.p.B.U.8
2
Where � = [I. Y _.!. Y
( )]
3
= .!_ X 1 X 1 X 10 X (lQ X 1 0-J )
u oo
2 o 2 o 2
=0.05 kg/sec
u [I.
3

u= Y _.!.(Y ) ]
2 8 2 8
00

�yderabad iDclhilBhopallPunelBhubaneswarl LucknowlPa!nalBengaiuru l OiennailVuayawadalVizag I TlfUpati I Kukatpal)yl Kolkata I


: 328 : Fluid Mechanics & Turbo Machinery

03. Ans: (c) . . dm


. Steady, m in -m out =-
S.mce flOW IS
dt
Sol: Integrated drag fore (Fm) = -r. Bdx f dm
=0
dt
0

't 8
Where -2 = 0 = -
pu 6 :. mentering = m leaving

(for linear velocity distribution )


= P U o H x1 [Width = 1]
0 = momentum thickness
mAF

Proof :
. . pVm 8
m Bc = m oE = --x 1
0 = J � (1-� ) dy 2
ffi co = pVm {H - 28)xl
OU U

0 = J Y (1- Y ) dy V 8 pV 8
0 8 8 :. pu 0 Hxl = P
-m-+pVm {H -28)+-m-
6
0= [r -L]
28 382
2
PUoH =pVm8 + p Vm (H -28)
2

Incompressible (p = c)
0

8
0 =- UoH =Vm8 +VmH - Vm28
uoH =VmH -Vm8
UoH =Vm (H - 8)
't =p.u2.0
Vm --H
! O xt'
- I x ( 1 0) x ( J U o H -8
2
- =

1
vm = --
=0.167 N say 0.17 N
Uo 1 - l
-

H
04. Ans : (c)
Sol: 05. Ans : (b)
Sol: Applying Bernoulli's Equation at (1) and (2)
outside Boundary layer
2 2

llo
-+ -
PAVA = PB + VB
- -
pg 2g pg 2g

A B X

�ydcrabad j Dclhi j Bhopal j Punc j Bhubancswar j Lucknow j Pama j Bcnga)uru j Chennai j Vgayawada j V,zag jT1111pati I Kukatpally j Kolkata I
-:,".•" · �CE
.. .. =Fii�1p1�-
:·.!
·
::rm�11[!Pu�li�
· ·
��-=·�=======....:,.;
ilir-i = 329 =========== Boundary Layer
�� ����:����

08. Ans: 0.33 (range 0.32 to 0.34)


Sol: Mass entering = mass leaving
6
rilba ril bd
= + J pudy . . . . . . . (1)
0

! u dy u,! 2
= (
r)- (r)'
6
= u0 [i__LJ
8 38 2
= u0 [ 8 -§. J
3
06. Ans: (c)
6
0

2
Sol: J pudy = - u 0 p8 . . . . . . . . . .. (2)
0
3
Substitute (2) in 1
8/ = � · . . 2 s:
m(ba) = m(bd) + 3 u ou
1 2p u 0 8
82 = 1 X - = 0.5 tnm
2 ril 3
1 = _M_ + --=---
rilba P 8
Uo
07. Ans: 0.0158 to 0.0162
Sol: p = 1.2 kg/m3
v = 2 x 10-5 m2/sec
V = 2 m/sec 09. Ans: (b)

![1 -sm(; r)}y


L = 2.5 m
_ 1.328
Sol: &* = [ (, - i.}y =

nI
Cr -
�Rex

pV 2
F0 = C r x -- x A
2
++: cot

2 2
= [& + : cot) - o - : cos(o)]
1.328 l .2 x 2 2
x x (2.5 x l )
2 x 2.5 2
=
- 28
2 x l0-5 =8
7t
= 0.0 1 59 N o· 2
:. - = 1 - -
()
7t

\( I I 11c;111t t 1 1 1 1� P11Llu .111011" FydcrabadlDelhilBhopaJ I Punc lBhubancswarl Lucknow I Patna I BengalurulOiennai ! Vuayawada jVizag ITuupati I Kukatpallyl Kolkata I
.,
��-�:
" . ACE
. . .
�Niliralml : 330: Fluid Mechanics & Turbo Machinery
� � =========================================
Any particle which crossing line AC has
Five Marks Solutions velocity in x direction as free stream
velocity Vo.
Momentum influx m x direction can be
01. calculated as
Sol: Consider the control volume ABC as shown (momentum in flux)x =( rhin ) x Vx
in the diagram.
pV0 xv = p Vg
=(
2 ) ° 2
8 8

_ _ _ _ _ __ __ _c
As the layer above any fluid element in the
I

:---,r
1 1---+--..r

I
boundary layer moves faster, the shear stress
on top face of the element is towards the
I

direction of flow
The continuity consideration shows that the
mass coming into the control volume by ---+'tyx
crossing line AC must be same as the mass
going out of control volume by crossing line
BC because there is no mass flow across line
AB
l DL
By the complementary action of shear
stresses 'txy ='tyx and the direction of stresses
must be such that there is balance of all
=rn.BC =f pudy
forces as well as moments.
rn.AC
0

Therefore, stress on face AB must be in


upward direction.
1>
= PVo [_i]
8 2 0
. . pV0
1.e. m in = m AC =--
2
8

\( l l 1 1gl ! H l / l !IL, P11hlH ,111, 111, �yderabad J Delhi I Bhopal I Pune J Bhubancswarl Lucknow I Pama! Bengaluru I Chennai I Vuayaw.ida J Vmg J Tuupali I Kukatpa)ly I Kolkata I
C8 Dimensional Analysis
02. For the laminar flow of water over a sphere,
the drag coefficient CF is defined as CF =
F/( pU2D2), where F is the drag force, p is the
Two Marks Questions

01. Oil (kinematic viscosity, = 1.0 x 10-5 fluid density, U is the fluid velocity and D is
m2/s) flows through a pipe of 0.5 m the diameter of the sphere. The density of
Voil

diameter with velocity of 10 mis. Water water is 10000 kg/m3 . When the diameter of
(Kinematic viscosity, = 0.89 x 10� m2/s) the sphere is 100 mm and the fluid velocity
is flowing through a model pipe of diameter is 2 mis, the drag coefficient is 0.5. If water
Vw

20 mm. For satisfying the dynamic now flows over another sphere of diameter
similarity, the velocity of water (in mis) is 200 mm under dynamically similar
conditions, the drag force (in N) on this
(GATE - 16 - SET - 1) sphere is _____
(GATE - 17 - SET - 2)

\( I I 1 1 � 1 1 1 t t 1 1 1 1� Ptilil1< ,1 t 1 e 1 1 1 ..., yderabad I Delhi I Bhopal I Pune I Bhubaneswarj Lucknow! Patna I Bengaluru I Chcnnai I Vrjayawadaj Vizag I T1ntpati I Kukatpally I Kolkata
SOLUTIONS
02. Ans: 20
Two Marks Solutions Sol: For dynamic similarity
(Re)1 = (Re)i
01. Ans: 22.25 (range 22.0 to 22.5)
Sol: Oil Water
U1D1 = U2D2 . . . . . . . . . . . . (1)
d = 0.5 m d = 0.02 m In general CF = f (Re) hence in dynamically
v = l O m/sec v=? similar conditions CF1 = CF2 . . . . . . . . . . (2)
Now,
F2 = CF2 pU;D;

--- V x0.02
l O x 0.5 ---- = cF1 pu�o�
=
l .O x l0-
5 6
0.8 9 x l0-
( ": CF1 = CF2 & U 1D1 = U2D2)
� V = 22.25 mis = 0.5 X 1000 X 2 X 0. 1 2
= 20 N

\( I l.11g11a t 1111g P11hlh .111011.., �yderabad ! Delhi l Bhopal ! Pune ! Bhubaneswarl Lucknow I Patna I Benga)uru l Chcnnai l Vuayawada ! Vizag IT,rupali I Kublpally l Kolkata I
Cg Turbo Machinery
(b) 15.0 kW
(a) 7.5 kW
One Mark Questions (c) 22.5 kW (d) 37.5 kW

01. In terms of speed of rotation of the impeller 05. A phenomenon 1s modeled usmg n
N , discharge Q and a change in total head H , dimensional variable with k pnmary
through the machine, the Specific Speed for a dimensions. The number of non-dimensional
pump 1s ____ (fill the blank) variables is (GATE-ME-10)
(GATE-ME-94) (a) k (b) n (c) n - k (d) n + k

02. Kaplan turbine is (GATE-ME-97) 06. A hydraulic turbine develops 1000 kW


(a) A high head mixed flow turbine power for a head of 40 m. If the head is
(b) A low head axial flow turbine reduced to 20 m, the power developed (in
(c) An outward flow reaction turbine kW) is (GATE-ME-10)
(d) An impulse inward flow turbine (a) 177 (b) 354
(c) 500 (d) 707
03. If there are 'm' physical quantities and 'n'
fundamental dimensions in a particular 07. In order to have maximum power from a
process, the number of non-dimensional Pelton turbine, the bucket speed must be
parameters is (GATE-ME-02) (GATE-ME- 13)
(a) m + n (b) m x n (a) equal to the jet speed
(c) m - n (d) m / n (b) equal to half of the jet speed
(c) equal to twice the jet speed
04. In a Pelton wheel, the bucket peripheral (d) independent of the jet speed
speed is 10 mis, the water jet velocity is 25
mis and volumetric flow rate of the jet is 08. Kaplan water turbine is commonly used
when the flow through its runner is
O. l m3/s. If the jet deflection angle is 120°C
and the flow is ideal, the power developed is (GATE-14) (S-4)
(GATE-ME-06) (a) axial and the head available is more than
100m
\( I I 11glllll 1 111g 1'11li\1, .1t1t>11, �yderabad I Dellii I Bhopal I Pune I Bhubaneswar I Lucknow I Patna I Bengaluru I Oiennai I Vtiayawada I Vttag I T1111pati I Kukatpally I Kolkata I
(b) axial and the head available is less than 11. Which one of the following is NOT a
10m rotating machine? (GATE - 17 - SET - l)
(c) radial and the head available 1s more (a) Centrifugal pump (b) Gear pump
than 100m (c) Jet pump (d) Vane pump
(d) mixed and the head available is about
50m 12. Which one of the following statements is
TRUE? (GATE - 17 - SET - 2)
09. Consider two hydraulic turbines identical (a) Both Pelton and Francis turbines are
specific speed and effective head at the inlet. impulse turbines.
(b) Francis turbine is a reaction turbine but
If the speed ratio ( :: ) of the two turbines
Kaplan turbine is an impulse turbine.
(c) Francis turbine is an axial-flow reaction
is 2, then the respective power ratio ( :: ) is turbine.
(d) Kaplan turbine is an axial-flow reaction
(GATE - 16 - SET - 1)
turbine.

10. The blade and fluid velocities for an axial


turbine are as shown in the figure.
Blade speed Two Marks Questions
1 50 mis

�mis 01. Cavitation in a hydraulic reaction turbine is


most likely to occur at the turbine
300 ��5 0

(GATE-ME-93)
_
Exit
Entry

(a) Entry (b) Exit


The magnitude of absolute velocity at entry
(c) Stator exit (d) Rotor exit
is 300 mis at an angle of 65 ° to the axial
direction, while the magnitude of the 02. Specific speed of a Kaplan turbine ranges
absolute velocity at exit is 150 mis. The exit between (GATE-ME-93)
velocity vector has a component in the (a) 30 to 60 (b) 60 to 300
downward direction. Given that the axial (c) 300 to 600 (d) 600 to 1000
(horizontal) velocity is the same at entry and
exit, the specific work (in kJ/kg) is _ _ _
(GATE - 16 - SET - 3)
\( I I 11�\lll l llll� P11h\1t ,tlHtll" yderabad i DelhilBhopa) iPunelBhubancswari LucknowlPatnalBcngalurul ChennailVuayawadalVizag ITirupati I Kukatpallyi Kolkaaa
"... . .
-:,,. �FnpwllJIW PnNiratioos
,.,,...,,"' ACE . . : 335 : Turbo Machinery

03. At a Hydro-electric power plant site, this point equals 200 kPa gauge and velocity
available head and flow rate are 24.5 m and 1s 3 mis. Steam tables show saturation
3
1 0. l m /s respectively. If the turbine to be pressure at 65 °C is 25 kPa, and specific
installed is required to run at 4.0 revolution volume of the saturated liquid is 0.00 1 02
m3/kg. The pump Net Positive Suction Head
(GATE-ME-06)
per second (rps) with an overall efficiency of

(GATE-ME-04)
90%, then suitable type of turbine for this (NPSH) in meter is
site is ----------------------
(a) Francis (b) Kaplan ---------------
--------
(a) 24
(c) Pelton (d) Propeller
(b) 26

04. A centrifugal pump is required to pump (c) 28


(d) 30
water to an open water tank situated 4 km -------
away from the location of the pump through
a pipe of diameter 0.2m having Darcy's 07. The inlet angle of runner blades of a Francis
friction factor for 0.0 1 . The average speed of turbine is 90° . The blades are so shaped that
water in the pipe is 2 mis. If it is to maintain the tangential component of velocity at blade
a constant head of 5 m in the tank, neglecting outlet is zero. The flow velocity remains
other minor losses, the absolute discharge constant throughout the blade passage and is
pressure at the pump exist is (GATE-ME-04) equal to half of the blade velocity at runner

(GATE-ME-07)
(a) 0.449 bar (b) 5.503 bar inlet. The blade efficiency of the runner is
(c) 44.9 1 1 bar (d) 55.203 bar
(a) 25% (b) 50 %
05. A large hydraulic turbine is to generate 300 (c) 80% (d) 89%
kW at 1 00 rpm under a head of 40m. For
initial testing, a 1 :4 scale model of the 08. A model of a hydraulic turbine is tested at a

turbine operates under a head of 1 0 m. The head of 114th of that under which the full
scale turbine works. The diameter of the

(GATE-ME-06)
power generated by the model (in kW) will
be model is half of that of the full scale turbine
If N is the RPM of the full scale turbine, then
(a) 2.34 (b) 4.68 (c) 9.38 (d) 1 8.75

(GATE-ME-07)
the RPM of the model will be
06. A horizontal - shaft centrifugal pump lifts
water at 65 °C. The suction nozzle is one (a) N/4 (b) N/2
meter below pump centerline. The pressure at (c) N (d) 2N

!lfll4j@jjj4iljjQAflijiM\jjih�.J:ydcrabad I Delhi I Bhopal I Pune I Bhubaneswarl Lucknow I Patna I Bengaluru I Chennai I Vijayawada I Vizag I T1111pati J Kukatpally I Kolkata I
09 . Match the items in columns I and II. (a) 0 (N-m) / (kg/s)
(GATE-ME-07) (b) 1 .25 (N-m) / (kg/s)
Column I Column II (c) 2.5 (N-m) / (kg/s)
P: Centrifugal compressor 1 : Axial flow (d) 3.75 (N-m) / (kg/s)
Q: Centrifugal pump 2: Surging
R: Pelton wheel 3 : Priming
Five Marks Questions
S: Kaplan turbine 4: Pure impulse
(a) P-2, Q-3, R-4, S-1
(b) P-2, Q-3, R-1 , S-4 01. Identify the type of turbo machinery for
(c) P-3, Q-4, R- 1 , S-2 the following case. Specify the reasons.
(d) P-1, Q-2, R-3, S-4 Power developed = 430 kW;
Operating head = 300 m;
10. Water having a density of 1000 kg/m3 , issues Speed = 600 rpm. (GATE- ME-90)
from a Nozzle with a velocity of 10 mis and
the jet strikes a bucket mounted on a Pelton 02. A 7MW hydro turbine working under a
wheel. The wheel rotates at 10 rad/s. The head of 10 m at a rotational speed of 125
.mean diameter of the wheel is 1 m. The jet is rpm is to be designed and developed by
split into two equal streams by the bucket; conducting model tests in a laboratory.
such each stream is deflected by 120° , as Maximum possible discharge to the model is
shown in the figure. Friction in the bucket 600 liters per second at a constant head of
may be neglected. Magnitude of the torque 5m. Determine the minimum scale of the
exerted by the water on the wheel, per unit model and its speed if the expected
mass flow rate of the incoming jet, is efficiency of the model is 85 %.
(GATE-ME-OS) (GATE- ME-91)
Deflected Jet

03. A Hydro turbine is required to give 25 MW


at 50 m head and 90 rpm runner speed. The
Incoming Jet laboratory facilities available, permit testing
of 20 kW models at 5 m head. What should
be the model runner speed and model to
prototype scale ratio? (GATE-ME-92)
Deflected Jet

!IJ11ijj§jjji4ijjji@ftiMii1MjHyderabadl DelhiIBhopal I Punc IBhubancswarl LucknowIPatna! BengalurulChennaiIVtjayawadalVmg ITuupati I Kukatpa))yl Kolkata I


: 337: Turbo Machinery

04. In a hydroelectric station, water is available 05. A water turbine delivering 10 MW power is
at the rate of l 75m3/s under a head of 18m. to be tested with the help of a geometrically
The turbines run at a speed of 150 rpm with similar 1 :8 model, which runs at th� same
overall efficiency of 82%. Find the number speed as the proto type.
of turbines required if they have the (a) Find the power developed by the model
maximum specific speed of 460. assuming the efficiencies of the model
(GATE-ME-96) and the proto-type is equal.
(b) Find the ratio of the heads and the ratio
of mass flow rates between the proto-
type and the model. (GATE-ME-97)

\( I 1 11!.!,l ! H ( T I ii� P1i\d]( .tlit )lh �yderabadl Delhi I Bhopal I Pune I Bhubaneswarl Lucknow I Pain.al Benp)uru I Otennai IVuayawada j Vizag I T1n.tpati I Kukatpallyl Kolkata I
SOLUTIONS
05. Ans: (c)

06. Ans : (b)


One Mark Solutions

N .jQ Sol: Given Data P = 1000 kW


01. Ans .. Ns -
-
(H) 31 4 H1 = 40 m, H2 = 20 m, P2 = ?

Unit power,
02. Ans: (b)
p
Pu = (for hydraulic turbines)
03. Ans: (c) (Ht 2
Sol: Number of Non-dimensional Parameters P, P2
=m -n (as per Buckingham's 7t -Theorem) (H , yi (H 2 )3 1 2
--- 2
= ---

Where, m = No. of Physical quantities 1000


n =Number of fundamental dimensions
P2
(400 t (20)3 1 2
--- 2 =

in a process selected like mass (M)


=> P2 =353.6 kW =354 kW
, Length (L) and Time (T) etc.

04. Ans : (c) 07. Ans: (b)


Sol: V= Jet velocity = 25 mis Sol: For an Impulse turbine (i.e. Pelton wheel), to
u = bucket velocity = 10 mis deliver maximum power, the tangential
Blade angle ( P ) = 180 -120 = 60° bucket velocity must be equal to half of the
Q =0.1 m3/s jet velocity of water.
p =1000kg/ m 3 vjet
U =-
K = bucket friction factor
= 1 for ideal fluid
08. Ans: (b)
Sol: Kaplan water turbine is axial flow type and
For Pelton wheel
Power, P = pQ[V -u][l+K cos p]u
low head turbine and also high discharge and
=> P =lOOOx0.1 [25 -10] [1+cos 60 ° ](10)
high range of specific speed.
= 22500 W = 22.5 kW

\( } l .llglllt l ! IIIC, P1il1lil,ltlOlh �yderabad j Delhi j Bhopal j Pune j Bhubaneswarj Lucknow j Patna j Bengaluruj Chennai jVtjayawada j Vmg j Tirupari I Kukatpal)yj Kolkata I
: 339: Turbo machinery

09. Ans: 0.25 (range 0.24 to 0.26) Specific work = [Vw l + Vw2] . u
N-# = [271.8 9 + 80.76] 150
Sol: N5 = -5-
X

= 52807.5 J/kg = 52.81 kJ/kg


H4

11. Ans: (c)


N1
=2
N 2 12. Ans: (d)

p
� p: = N�
[N ]2 (1 )2 = = 0.25 Two Marks Solutions
2
01. Ans: (d) 02. Ans: (c)
10. Ans: 52.807 (range 50 to 54)
Sol: Given: V1 = 300 mlsec
03. Ans: (a)
u = 150 mlsec
Sol: Given data:
Vn = Vn (a = 25 °)
H = Head available = 24.5 m
Q = Flow rate = 10.1 m3/sec
N = Speed of turbine = 4 rev/sec
= 4 X 60 = 240 rpm
r1 o = Overall efficiency = 0. 9 .
Power developed by turbine,
p = p g QH x 11 0
1000 x 9 .8 l x l0. l x 24.5 x 0. 9
p= ;;; 2185 kW
Specific work = [Vw l + Vw2] . u 1000
Vw 1 = V1 cos 25 Specific speed of turbine, Ns:-
Vw 1 = 300 cos 25 = 271.8 9 mis _ N fp _ 240
N - -
.J2i8s
Vn = Vn = V 1 sin 25 s (H) ¾ (24.5) ¾
Vn = 300 sin 25 = 126. 78 mis Ns = 205.816
Yw2 = )v; -Vf� The value of Ns in the range of 50 - 250.

= .J1502 -126.782 = .J6426.83


Hence the suitable turbine for the given site
conditions is "FRANCIS TURBINE".
Vw2 = 80.16 mis
\( } J J l .., l l lt l [ l l lL, P11lil1( ,l!U)I[', ydcrabad I Dclhil Bhopal IPunc I Bhubancswarl Lucknow I Patna I Bcngaluru I Chcnnai IVuayawadalV17.ag I Ttrupari I Kukatpallyl Kolkata
., V • �CE
• •
: 340:

Fluid Mechanics & Turbo Machinery
'!.••
J . ��r..,,,,,.Pohlir,pg
========================================
04. Ans: (b) :. P oc D 2 H 3 1 2
Sol: Given data: p
L = 4000m
=> ( 2 31 2 ) - constant
D H
D = 0.2 m
=(

r
f = 0.01
( : ) � )
02 m . D' ,12
,
V =2m / s
H = 5m
P2 = ?
:. Pm = P, ( �: J (::
Frictional head loss in pipe - 300(-1 ) (-
2
-
1 0 ) .5
1

4 40
f.LV 2 O.O lx4000x2 2
hr = =
2gD 2x9.81x0.2 = 2.34 kW
40.77 meters of water
06. Ans: (a)
=

Pressure at exit (P2 ) =pg (H +h r )


Sol: Given data
= 1000x9.81(5+40.77) Guage reading,
= 449.05xl 0 3 N / m 2 P1 =200x10 3 N / m 2 [ P1 =pg H 1 ]
449.05 kPa (gauge)
200x10 3
=

(449.05 + 101.3) (Absolute) -


H 1 = --- = 20.387 (gauge)
lOOOx9.81
=

= 550.35 kPa (Absolute) Velocity, V1 =3m / s


= 5.503 bar (Absolute).
Velocity head at inlet,

05. Ans: (a)


---'-'---- =0.46m
= (3)2
Sol: Given data: PP =300 kW 2g 2x9.8
v?

Np = 1000 rpm, Hp = 40 m, Hm = 10 m Psat =25kPa =25x10 3 N / m 2


D m _!_
= =0.00102 m 3 / kg
DP 4
V8

l
In model analysis approach Ps 980.4 kg / m 3
0.00102
We have, P ocQH
:. H v = 25x10 2.6m
3

oc (AV) H 980.4x9.81
oc D 2 .JH. H NPSH = ?

11,11M@hiiiiiilidiGl¥1NiifjYderabad l Dclhil Bhopal l Puncl Bbubaneswarl LucknowlPatnalBcnga1uru l CbennailVuayawadalV17.3g I Trrupati I Kukatpallyl Kolkala I
"·•"
'!.
. .ACE
.. ,,.,,.,,..Ni:atm
. : 341 : Turbo machinery

NPSH = pressure at centre line (guage reading) + y2


:. H = He + -2
2g
static head (Hs) + velocity head ( �� ) + H v
= u 1 Vw 1 + V;
g 2g
= 20.3 87 + 1 + 0.46 + 2.6
= 24 m = u l (u i ) (u l / 2 ) [·: V = & V =
2
+ w U1 2 U1 1 2]
g 2g
07. Ans: (c) 9 u�
H=
Sol: 8g

= u2 = 8
. ', llb I ) - � 8 9%
9 u 21 9
g(-
8g

08. Ans: (c)


Sol: Given H m = H P / 4
Where, m = model ,P = proto type (actual)
Dm = DJ>"2
Let Nm = Speed of model = ?

The blade efficiency or hydraulic efficiency Np = N = Speed of prototype (or) Full

is given by scale (or) actual turbine

= He = U 1 Vw1 = u �
llb H g.H g.H
The energy balance gives
(Energy supplied to the turbine) = (Turbine
output) + (Losses at exit)
= /f x 2 x N = � x 2 x N
Assuming no losses with turbine and no
draft tube is used. The energy loss will be : . Nm = N
unconverted kinetic energy at exit i.e., Speed of model turbine is equal to speed
(Note: draft tube converts this kinetic energy of prototype (actual) turbine.
at exit partially into useful work)

FydcrabadjDclhijBhopal jPuncjBhubancswarl utcknowjPalnajBengaluru j ChennaijVgayawadajVizag j Tirupari I Kukatpallyj Kolkala I


09 . Ans: (a)

10. Ans: (d)


..--------�·:
Five Marks Solutions

Sol: Given data: 01.


V 1 Absolute velocity of water issued
=
Sol: Given data :
from Nozzle 10 mis =
Power (P) 430 kW , Speed (N)
= =
600 rpm
co = Angular speed of Pelton wheel Head (H) 300 m =

=
10 rad/sec. N# 600J4'3o
Specific speed (Ns) = =

D Mean diameter of Pelton wheel = 1 m


( H)¾4 ( 300)¾
=

Deflection angle =
120°
9 . 965 =

Blade angle (J3 ) = 180 - 120= 60 °


Specific speed is very low. Hence Pelton
wheel is suitable.
Friction between water and buckets

Neglected i.e Vr1 =


Vr2 [K = Vr 2
= 1] 02.
Sol: Given data :
vrl

Let T Torque exerted by water on


=
Power of prototype turbine (Pp)
wheel/unit mass flow rate [N-mkg/sec]. 7MW 7000 KW = =

D Head of water of prototype turbine (Hp)=1 Om


=Fx-
2
Speed of prototype turbine (Np) 125 rpm =

Let u =
u1 u2 mean tangential velocity of
= =

Discharge of model turbine (Qm)


Pelton wheel bucket r co =

= 600 lit/sec 0.6 m3/sec =

D 1
=
- . co = - x l O = 5 m / s Head of model turbine (Hm) = 5 m
2 2
Nm Speed of model turbine
=

Force exerted by water jet on bucket (F)


Power developed by model turbine
Per unit mass flow rate =
(v1 - u )(1 + cos �)
= flo X pgQ m H n
= (10 - 5)(1 + cos 60) =
7.5
0.85 x1000x 9.8I x0.6 x 5
D
=
kW = 2 5 kW
Torque exerted (T) Fx- 1000
2
=

For geometrically similar turbines specific


N m
= 7.5 x _!_ = 3.75 ( - J speeds must be equal
2 kg / sec
N = --,--
NP .jP; Nm .JP:,
s (Hp )514 ( Hm )514

!IJl@ih§hillihAAflftiMihii!+-1yderahad j Delhi j Bhopal jPuncj Bhubaneswarj Lucknow j l'alnajBcngalurulCbcnnailV\iayawadajViz.ag j Tirupati I Kukalpallyj Kolkata I
: 343 : Turbo machinery

125 Jmoo Nm Jii For geometric turbines, specific speeds are


( 10)514 ( 5)514 equal.

Ans: :. Nm � 880 rpm

Scale Ratio: 90.J25000 ---


= m
(50) 5 1 4 (5)
N .fio
----
Pp 5/4

( Hp )½ ( DP )
pm
( Hm )½ ( Dm ) :. N m = 178.94 rpm
2 2

Dm Pm H p 312
7000 2_ ½ Scale ratio ( ) )
( ) Dr Hm
=
Pp
X(

25 10
=

Dp 9.95
= say(lO:l)
(5 3 1 2
Dm 1 25000 5
= 20 0)
X

Dm 1
Scale ratio = 1 : 10 -- 0.159
DP 6.287
= = =

04.
03.
Sol: Discharge of water (Q) = 175 (m3 I sec)
Sol: Given data:
Head of water (H) = 18 (m)
Prototype turbine power (Pp) = 25 MW
Speed of turbine (N) = 150 (rpm)
25000 (kW)
Overall efficiency (11 0) = 82% = 0.82
=

Prototype turbine head(Hp) = 50 m


Sp . Speed of turbine = 460
Prototype turbine speed (Np) 90m =
Let n = Number of turbines required
Model turbine power (Pm) = 20kW We have
Model turbine head (Hm) = 5 m PO .;:;._
110 = - = power
__ output of turbine
Pw water power
....:;c_____

Let Nm be the speed of runner turbine in


0.82 = -­
r.p.m pgQH
po

D 0.82 = P0 / 1000 x 9.81 x 175 x 18


scaleratio = m
DP : . P = 25.83 x 106 Watt = 25830 kW
\( I } 1 1.., llH t l !II-, P1 1hl11 ,tlJ(l!I .... �ydcrabad I Delhi I Bhopal I Punc I Bhubaneswar I l.ucknowl Pa!nal Bcngaluru I Oiennai IVuayawada I Vizag I Tirupati I Kukatpal)y I Kolkala I
: 344 : Fluid Mechanics & Turbo Machinery

N (b) (i) we know,


- )P:
Sp. speed (Ns) - Specific speed of model
(H) ¾4 = specific speed of prototype

460 =
rp;:
150 -y ro
(18) ¾
:. Po = 12927.5 kW
:. Number of turbines required
25830
s14 s14
Hm
p

= - --
H

1.998 say 2 turbmes. Given Nm = Np


12927.5
=

05.
Sol: Given data :
Power developed by proto-type turbine,
Pp = lO MW
Dm
Geometric scale ratio = =1:8
DP

Speed of proto-type turbine (Np)


(ii) Q =ND3

(a) ( ) =
N;D, . ( /o' ),

= 3.05x10 -4 MW
= 0.305 kW

\( I I I Delhi I Bhopal I Pune I Bhubaneswar I wcknowl Patna! Benga)uru I Chennai I Vgayawada I Vizag ITirupari I Kukatpally I Kolkala I
Fyderabad
1 1 . I I H ( ! I ll� P1 1 l d 1t .1!11 1 1 1 ...
Heat (Questions)
Transfer
PageNo.346

CONTENTS
-- --- ---=- ---------===---
---= - -- -- - --- -

Chapter Questions Solutions


Name of the Chapter
No. Page No. Page No.
- - -� - - - � - - � -- --

01 Conduction 347-357 358 - 371

02 FINS & THC 372-376 377 - 384

03 Convection 385-390 391-395

04 Radiation 396-401 402-409

05 Heat Exchangers 410-415 416-428


Ct Conduction
(a) Copper (b) Steel
One Mark Questions (c) Glass wool (d) Refractory brick

01. Thermal conductivity is lower for 04. In descending order of magnitude , the
thermal conductivity of a) Pure iron, b) liquid
(a) wood (b) air water, c) Saturated water vapour, d) Pure
(GATE-ME-90)

(c) water at 100 °C (d) steam at 1 bar aluminum can be arranged as

(a) a b c d (b) b c a d
(GATE-ME-01)
02. Two insulating materials of thermal
conductivity K and 2K are available for (c) d a b c (d) d Cb a
lagging a pipe carrying a hot fluid. If the
05. One dimensional unsteady state heat transfer
radial thickness of each material is same
equation for a sphere with heat generation at
the rate ' qg ', can be written as
(a) Material with higher thermal conductivity
(GATE-ME-94)

should be used for inner layer and one


(GATE-ME-04)

with lower thermal conductivity for the 8T q _!_ 8 T


(a) J+ =
r 8r 8r k a 8t
!�(r
outer.
(b) Material with lower thermal conductivity
should be used for inner layer and one
with higher thermal conductivity for the
outer.
(c) It is immaterial in which sequence the
82 q 1 8T
insulating material are used (d) -(r T)+-=--
8r k a 8t
(d) It is not possible to judge unless numerical
2

values of dimensions are given 06. In case of one dimensional heat conduction
in a medium with constant properties, T is

thickness, the temp drop across the material


the temperature at position x, at time t. Then
8T
03. For a given heat flow and for the same

is propor tional to (GATE-ME-OS)


will be maximum for (GATE-ME-96) 8t
!lflli!Mhliiih4Pflbiihiih� yderabad I Dellti I Bhopal I Pune I Bhubaneswar I Lucknow I Patna I Benga)uru I Chennai IVuayawada IVizag ITlnipati I Kukatpal]y I Kolkata
: 348: Heat Transfer

T (a)Th> Ti> Tc andq1<q2


(a) - (b) 8T
ax (b) Th< Ti< Tc andq1 =q2
8 2T 8 2T (c) Th = (Ti+ Tc ) /2 andq1>q2
(c) (d)
8x&t ax2 (d)Ti< (Th+ Tc ) /2 andq1>q2

07. A well machined steel plate of thickness L is 08. In a composite slab, the temperature at the
kept such that the wall temperatures are Th interface (Tinter) between two materials is
and Tc as shown in the figure below. A equal to average of the temperatures at the
smooth copper plate of the same thickness L two ends. Assuming steady one dimensional
is now attached to the steel plate without any heat conduction, which of the following
gap as indicated in the figure below. The statements is true about the respective
temperature at the interface is Ti. The thermal conductivities. (GATE-ME-06)
temperatures of the outer walls are still the
same at Th and Tc · The heat transfer rates are
q1 and q2 per unit area in the two cases
respectively in the direction shown. Which
of the following statements is correct ?
(GATE-PI-05) 2b b
L
Tc (a)2K1 = K2 (b)K1 = K2
(c)2K1 = 3K2 (d) K1 =2K2

STEEL
09. A pipe of 25 mm outer diameter carries
steam. The heat transfer coefficient between
the cylinder and surrounding is 5 W/m2K. It
is proposed to reduce the heat loss from the
,.._L ...... L ... pipe by adding insulation having a thermal
conductivity of 0. 0 5 W/mK. Which one of
q2 T;
q2 the following statements is TRUE?
(GATE-ME-11)
Steel Copper

Th Tc (a) The outer radius of the pipe is equal to


the critical radius.

\( I } 11!..,lllll[II! .., P11hlH,IIJJJJl', �y derabad I Delhi I Bhopal I Punc I Bhubaneswar I Licknow I Patna I Bcnga)uru I Chcnnai I Vrjayawada jVu.ag I Tirupari I Kukatpally I Kolkat,. I
· .A!
�.. ..t'"=.!Fn�,gm
" � · CE
:� ��l,li:·.r,ti
:-�·0111
==========::::.,·· ��
349 · · =============�����
Conduction
...
f.t�,,

ic,:rq Pu

(b) The outer radius of the pipe is less than (a) The direction of heat transfer will be
the critical radius. from the surface at 100 ° C to the surface
(c) Adding the insulation will reduce the at 0 ° C
heat loss. (b) The maximum temperature inside the
(d) Adding the insulation will increase the wall must be greater than 100 ° C.
heat loss. (c) The temperature distribution is linear
within the wall
1 0. Consider one-dimensional steady state heat (d) The temperature distribution 1s
conduction, without heat generation, in a symmetric about the mid-plane of the
plane wall; with boundary conditions as wall.
shown in the figure below. The conductivity
of the wall is given by k=ko+bT; where ko 12. Consider a long cylindrical tube of inner and
and b are positive constants, and T is outer radii, ri and r0, respectively, length, L
temperature. (GATE-ME-13) and thermal conductivity, k. Its inner and
outer surfaces are maintained at Ti and To,
respectively (Ti > T0). Assuming one­
dimensional steady state heat conduction in
the radial direction, the thermal resistance in
the wall of the tube is
As x increases, the state temperature gradient (GATE-ME-14-SET-3)
(dT/dx) will
(a) remain constant (b) be zero
(c) increase (d) decrease

11 . Consider one-dimensional steady state heat


1 £ r
2 1tkL r;
conduction along x-axis (O�x�L), through a (c) _ n( J 0

plane wall with the boundary surfaces (x=O


and x=L) maintained at temperatures of 0 °C 1 r
4 1tkL r;
(d)- -Rn( oJ
and 100 °C. Heat is generated uniformly
throughout the wall. Choose the CORRECT
statement. (GATE-ME-13)

!lflli!i§hidihi4AflftiMi!iih+1ydcrabadlDelhi I Bhopal I Punel Bhubancswari Lucknow! Patna! Bcngaluru I Chennai IVtjayawadalVlZ31 ITuupari I KukalpallyJ Kolkata I
ACE
t.•·=
,:·���·-:
·� �m���N��Y�®�::oo:•=========�:��:�:============��:���:::
35 0 Heat Transfier

1 3. As the temperature increases, the thermal 16. A hollow cylinder has length L, inner radius
Ji

conductivity of a gas r 1, outer radius r2 , and thermal conductivity


k. The thermal resistance of the cylinder for
(a) increases radial conduction is
(GATE-ME-14- SET-4)

(b) decreases
(c) remains constant ln(r2 / r1) ln(r1 / r2)
(GATE-16- SET-2)

(a) (b)
(d) increases upto a certain temperature and 2 1tkL 2 1tkL
then decreases 2 1tkL 2 1tkL
(c) (d)
ln(r2 /r1 ) ln(r1 !ri}
14. If a foam insulation is added to a 4cm outer
diameter pipe as shown as figure, the critical 17. Steady one-dimensional heat conduction
radius of insulation ( in cm) is __ takes place across the faces 1 and 3 of a
composite slab consisting of slabs A and B in
perfect as shown in the figure, where kA , k8
(GATE-15-Set 2)

denote the respective thermal conductivities.


Using the data as given in the figure, the
interface temperature T2 (in °C) 1s ___
Foam

A B
Pipe
kp;pe= 15 W/m-K
2 3

15. A plastic sleeve of outer radius r0 = 1 mm


1

covers a wire (radius r = 0.5 mm) carrying


k8 1 OOW/rnK
=
T3 = 30°C
T 1 = 130°C

electric current. Thermal conductivity of the


kA=20W/rnK
14O.lm+ ·I
plastic is 0.15 W/m-K. The heat transfer
0.3m

coefficient on the outer surface of the sleeve (GATE-16- SET- 3)


exposed to air is 25 W/m2-K. Due to the 18. A metallic cylindrical casing of an exhaust
addition of the plastic cover, the heat transfer pipe has inner radius 50 mm and wall
from the wire to the ambient will thickness 7 mm. If the thermal conductivity
of the material of the casing is 50 W/m-K,
(a) increase (b) remain the same
(GATE-16- SET-1)

then the thermal resistance of the casing in


(c) decrease (d) be zero K/kW is ____ (up to three decimal
places). (GATE-Pl-17)

\( l I 11�111tt1111� Puhlu.111t111, yderabad I Delhi I Bhopal I Pune I Bhubancswar I Lucknow! Patna I Bcngaluru I Chennai I Vuayawada IVizag I Tirupati I Kukatpally I Kolkata
: 351: Conduction

04. The temp variation under steady heat


conduction across a composite slab of two
materials with thermal conductivies K1 and
Two Marks Questions

K2 is shown in fig. then , which one of the


01. Match the property with their units following statements holds?
(GATE-ME-91) (GATE-ME-98)
Property Units
A. Bulle modulus 1. W/s
B. Thermal conductivity 2. N/m2
C. Heat transfer coefficient 3. N/m3
D. Heat flow rate 4. W
5. W/mK
6. W/m2K (a)K1 > K2 (b)K1 = K2
(c)K 1 = 0 (d)K1 <K2
02. For a current carrymg wrre of 20 mm
diameter and (k = 0.5W/mK) exposed to air 05. It is proposed to coat a 1mm diameter wrre
(h = 20 W/m2K), maximum heat dissipation with enamel paint (k = O.l W/m-K) to
occurs when thickness of insulation is increase heat transfer with air. If the air side
(GATE-ME-93) heat transfer co-efficient is 100 W/m2K , the
(a) 30 mm (b) 25 mm optimum thickness of enamel paint should be
(c) 20 mm (d) 15 mm (GATE-ME-99)
(a)0.25mm (b) 0.5 mm
03. For a current carrying wire of 20 mm (c) 1 mm (d)2 mm
diameter and having k = 0.5W/mK is
exposed to air (h = 20W/m2K), maximum Common Data for Question 06 & 07
heat dissipation occurs when thickness of Heat is being transferred by convection from
insulation is (GATE-ME-96) water at 48°C to a glass plate whose surface that
(a) 30 mm (b) 25 mm is exposed to the water is at 40°C. the thermal
(c) 20 mm (d) 15 mm conductivity of water is 0.6 W/mK and the
thermal conductivity of glass is 1.2 W/mK. The
spatial gradient of temp in the water at the water­
glass interface is dT/dy = 1 x104 Kim..

!M•ll@jj/i4ijjikflbi1jjj.jj+yderabad I Delhi I Bhopal I Pune I Bhubaneswar I Lucknow! Patna I Benga)uru I Chennai I Vuayawada I Vizag I Tuupari I Kukatpally I K.olkata I
: 352: Heat Transfer

09. The maximum temp within the plate in


48 C degree C is
y
°

(a) 160 (b) 165 (c) 175 (d) 250

Water 10. A stainless steel tube ( Ks= 19 W/mK) of a


Glass
2cm ID and 5 cm OD is insulated with 3 cm
thick asbestos (Ka = 0.2 W/mK). If the
06. The value of the temperature gradient in the temperature difference between the
glass at the water-glass interface in Kim is innermost and outermost surface is 600C, the
(GATE-ME-03) heat transfer rate per unit length is
(a)-2 xl0 (b) 0.00 (GATE-ME-04)
(c) 0.5x 10 (a) 0.94 W/m (b) 9.44 W/m
4

(d) 2xl0 4
(c) 944.72 W/m (d) 9447.21 W/m
4

07. The heat transfer coefficient h in W/m2 K is


(GATE-ME-03) 11. Heat flows through a composite slab, as
(a) 0.0 (b) 4.8 (c) 6 (d) 750 shown below. The depth of the slab is 1 m.
The k values are in W/m.K. The overall
Statement for Linked Answer Q08 & Q09 thermal resistance in K/W is
Consider steady one-dimensional heat flow in a (GATE-ME-2005)
plate of 20mm thickness with a uniform heat
gener�tion of 80MW/m3 . The left and right faces
are kept at constant temperatures of 160 °C and k= 0.02
k = 0.10 0.5m
n
120 °C respectively. The plate has a constant qc=> j_ lm

thermal conductivity of 200W/mK.


k=0.04

(GATE-ME-07)
0.5m 0.25m

08. The location of maximum temp within the (a) 17.2 (b) 21.9 (c) 28.6 (d) 39.2
plate from left face is
(a) 15mm (b) 10mm
2.5x3x3m size thermally insulated room
12. A 100 W electric bulb was switched on in a

(c) 5mm (d) 0mm


having temperature of 20 °C. Room temp at
the end of 24 hours will be
(GATE-ME-06)
\l I I 11�
..
,llHt l[ll� P1d,lit,tl1t1I]', Fydcrabad I Delhi I Bhopal IPwie I Bhubaneswar I Lucknow! Patna I Bengaluru I Chennai IVuayawada IV17.38 ITirupati I Kukatpally I Kolkala I
" : 353:
��-·:�. PnNicmoos
" ACE .. . Conduction

(a) 321° C (b) 341°C (a) 0.471 (b) 0.407


(c) 450°C (d) 470° c (c) 0.321 (d) 0.125

13. With an mcrease in the thickness of 16. For the three dimensional object shown in
insulation around a circular pipe, heat loss to the fig below. Five faces are insulated. The
surroundings due to (GATE-ME-06) sixth face (PQRS) , which is not insulated,
(a) Convection increases, while that due to interacts thermally with the ambient, with a
conduction decreases convective heat transfer coefficient of
(b) Convection decreases, while that due to 10W/m2K. the ambient temperature is 30°C.
conduction increases heat is uniformly generated inside the object
(c) Convection and conduction decreases at the rate of 100W/m3 • assuming the face
(d) Convection and conduction increases PQRS to be at uniform temperature, its
steady state temp is (GATE-ME-08)
14. A long glass cylinder of inner diameter =

0.03m and outer diameter = 0.05m carries


hot fluid inside. If the thermal conductivity
of glass = 1.05 W/mK, the thermal resistance
(K/W ) per unit length of the cylinder is
(GATE-PI-07)
(a) 0.031 (b) 0.077
(a) l OC (b) 20C (c) 30C (d) 40C
(c) 0.17 (d) 0.34
17. Steady two dimensional heat conduction
°
15. Building has to be maintained at 21 C (dry takes place in the body shown in the fig
°
bulb) and 14.5 C (wet bulb). The dew point below. The normal temperature gradients
°
temp under these conditions is 10. l 7 C. the over surface P and Q can be considered to be
°
outside temp is-23 C (dry bulb) and internal uniform. The temperature gradient of/& = at
and external surface heat transfer coefficients surface Q is equal to 10 Kim. surfaces P and
are 8 W/m2K and 23 W/m2K respectively. If Q are maintained at constant temperatures as
the building wall has a thermal conductivity shown in the fig. While the remaining part of
of 1.2 W/mK, the minimum thickness (m) of the boundary is insulated. The body has a
wall required to prevent condensation is constant thermal conductivity of 0.1 W/mK,
(GATE-ME-07) the value of of/& and of/oy at surface Pare
\( I I 111.�1111t 1 111� P11lilu,1111,11..., �ydcrabadlDclhilBhopallPuncJBhubancswarJ wcknowlPatnaJBcnga!uruJChcnnaiJVuayawadaJVizag JTirupati I KukalpallyJ �1
" " . �CE . .
. Pnblir*u : 354: Heat Transfer

.� �==========================================
-:.�
:�

Assuming negligible contact resistance


y
between the wall surfaces , the interface temp
T(C) of the two walls will be
(GATE-ME-09)
(a)- 0.50 (b)2.75
(c) 3.75 (d) 4.5
'\_surface P, 100°C
�-------x
1 9. Heat is being transferred convectively from a
(GATE-ME-08)
(a) of/ox =20K/m, ofloy = 0 Kim
cylindrical nuclear reactor fuel rod of 50mm

(b) of/ox = OK/m, of/oy = IO Kim


diameter to water at 75°C, under steady state

(c) of/ox = IOK/m, of/oy = IO Kim


condition, the rate of heat generation within

(d) of/ox = OK/m, of/oy =20 Kim


the fuel element is 50x10 6 W/m3 and the
convective heat transfer coefficient is 1
1 8. Consider the steady state heat conduction kW/m2K. the outer surface temperature of the
across the thickness in a plane composite fuel element would be (GATE-PI-09)
wall as shown in fig exposed to convection (a) 700 °C (b) 625 °C
condition on both sides (c) 550 °C (d) 400 °C

20. A material P of thickness 1 mm is


ho, Too,o sandwiched between two steel slabs, as

ii ii shown in the figure below. A heat flux IO


kW/m2 is supplied to one of the steel slabs as
shown. The boundary temperatures of the
slabs are indicated in the figure. Assume

Given thermal conductivity of this steel is


h·' =20 W/m2K-' ho 50 W/m2K;
= 1 OW/m.K. Considering one-dimensional
Tc.o,i = 20 c;
°
Too,o = -2°C ; steady state heat conduction for the
K1 =20 W/mK; K2 = 50 W/mK; configuration, the thermal conductivity (k, in
L1 = 0.3 m; L2 = 0.1 5 m; W/m.K) of material P is
(GATE-ME-14- SET-2)
\( ·1 I .11g11H t 11111.!, P11hlu .111011.., �yderabad I Delhi I Bhopal I Pune I Bhubaneswar I Lucknow I Patna I Benga)uru I Oiennai I Vuayawada I Vu.ag I Tirupali I KukatpaI)y I Kolkata I
: 355: Conduction

design thickness (in meter) of the wall to


/k =? maintain a steady heat flux of 2500 W/m2
should be ___ (GATE-ME-14- SET-4)
q"=l OkW/m2
CQ
� 23. A 10 mm diameter electrical conductor is
� �
en en
� � covered by an insulation of 2 mm thickness.
� �
SOO K � � The conductivity of the insulation is 0.08
E--< E--<
en en
W/m-K and the convection coefficient at the
insulation surface is 10 W/m2-K. Addition of
further insulation of the same material will
20 Il I
20 (GATE-15-Set 1)
All dimensions in mm
(a) increase heat loss continuously
(b) decrease heat loss continuously
21. Heat transfer through a composite wall is (c) increase heat loss to a maximum and
shown in figure. Both the sections of the wall then decrease heat loss
have equal thickness (l). The conductivity of (d) decrease heat loss to a mm1mum and
one section is k and that of the other is 2k. then increase heat loss
The left face of the wall is at 600 K and the
right face is at 300 K. 24. A cylindrical uranium fuel rod of radius 5
600K Ti 300K mm in a nuclear reactor is generating heat at
the rate of 4x107 W/m3. The rod is cooled by
at flow
k 2k a liquid ( convective heat transfer coefficient
1000W/m2-K) at 25° C. at steady state, the
surface temperature (in K) of the rod is
(GATE-15-Set 2)
The interface temperature Ti (in K) of the (a) 308 (b) 398 (c) 418 (d) 448
composite wall is __
(GATE-ME-14- SET-3)
25. A brick wall (k = 0.9�) of thickness
m.K
22. A plane wall has a thermal conductivity of 0.18m separates the warm air in a room the
°
1.15 W/m.K. If the inner surface is at 1100 C cold ambient air. On a particular winter day,
and the outer surface is at 350°C, then the the outside air temperature is - 5° C and the

l1ti•i!@jjj4ii0i4@ft1Mjj@"' yderabad I Delhi I Bhopal I Pune I Bhubaneswar I Lucknow I Patna I Bengaluru I Oiennai jVtjayawada jVmg I T,rupari I Kukatpally I Kolkata
: 356 : Heat Transfer

room needs to be maintained at27° C . The


heat transfer coefficient associated with
Five Marks Questions
outside air is 20 W/m2K. Neglecting the
convective resistance of the air inside the
01 . Steam is flowing through an insulated steel
room, the heat loss in (; ) is. pipe shown in the fig, is losing heat to the
surroundings. The details are as follows
(GATE -15-Set 3)
(a) 88 (b) l l O (c) 128 (d) 1 6 0
(GATE-ME-1988) (SM)
Inner radius of steel pipe = 50mm,
Outer radius of the steel pipe = 57mm,
26 . In a 1 m thick wall, the temperature
Outer radius of insulation = 1 57 mm,
distribution at a given instant 1s
Thermal conductivity of steel = 43 W/mK
T(x) = c0+c1x+c2x2 where T is in °C and x is
Thermal conductivity of insulating material
in m. The constants are: c0 = 800 ° C,
= 0 .1 W/mK
c1= -250 °C/m and = -40 ° C/m2 . The
Heat transfer coefficient on steam side
C2

thermal conductivity of the wall is 50 W/mK


570W/m2K =
and wall area is 5 m2 • If there is a heat source
Heat transfer coefficient on air side
generating uniform volumetric heating at the = 12W/m2K,
rate of 500 W/m3 inside the wall, then the
Temperature of steam = 500 °C
rate of change of energy storage in the wall,
Temperature of surroundings = 30 °C
in kW, is -----
(GATE - PI-15)

27. Heat is generated uniformly in a long solid I - Insulation, K = 0. 1 W/ m K

cylindrical rod (diameter =10 mm) at the


2 - Steel pipe, K = 43 W/ m K
3 - Steam, 500°C, h = 570 W/m2K
rate of 4 x 10 7 W/m3 . The thermal 4 - Air, 30° C, h = 1 2 W/ m2 K

conductivity of the rod material is 25


W/m.K. Under steady state conditions, the
temperature difference between the centre
and the surface of the rod is --- ° C.
Calculate the heat loss per meter length of
pipe and temp of the outer surface of the
(GATE - 1 7- SET - l)

insulation.

!lti•i!@jjj4i.jl4Aflnj@jj.jjii I I I I I I I I I I
yderabad Delhi Bhopal Punc Bhubaneswar Lucknow! Patna Bcngaluru Olennai Vuayawada Vmg T'lfUpari I Kukatpally I Kolkata
"" . ACE
. .. : 357: Conduction
:.�.,"F�PoNir:mma
02. An electric hot plate 1s maintained at a
temperature of 350°C, and is used to keep a
Insulated

solution boiling at 95 °C. the solution is


contained in cast iron vessel of wall
T
thickness 25mm, which is enameled inside to
a thickness of 0.8mm. the heat transfer
coefficient for the boiling solution is 5.5
kW/m2K and the thermal conductivities of
cast iron and enamel are 50 and 1.05W/mK,
respectively. Calculate the OHTC and the Thermal conductivity of the materials are
Insulated

rate of heat transfer per unit area KA = Ko = 50 W/mK,


Ka = 10 W/mK , Kc = 1 W/mK
The fluid temps and HTC (see fig) are
(GATE-ME-1993)

03. A gas filled tube has 2mm inside diameter T1 = 200°C, h 1 = 50 W/m2K,
and 25cm length. The gas is heated by an T2 = 25 °C, h2 = 10 W/m2 K,
electrical wire of diameter 50microns located Assuming one dimensional heat transfer
along the axis of the tube. Current and condition. Determine the rate of heat transfer
voltage drop across the heating element are through the wall. (GATE-ME-2001)(5M )
0.5A and 4 volts, respectively. If the
05. A copper tube of 20mm outer diameter' 1 mm
measured wire and inside tube wall temps are
thickness and 20m long (thermal
175 °C and 150°C respectively, find the
conductivity 400 W/mK) is carrying
thermal conductivity of the gas filling the
saturated steam at 150°C (Convective HTC
=

tube.
=150 W/m2K). The tube is exposed to
(GATE-ME98) (SM)

ambient air temperature of 27 °C. the


04. A composite wall , having unit length normal
convective HTC of air is 5 W/m2K. Glass
to the plane of paper, is insulated at the top
wool is used for insulation (Thermal
and bottom as shown in fig. it is comprised
conductivity = 0.075 W/mK). If the thickness
of four different materials A, B, C and D .
of the insulation used is 5mm higher than the
Dimensions are
critical thickness of insulation, calculate the
HA = Ho = 3cm,
rate of heat lost by the steam and the rate of
Hs = Hc = 1.5cm,
steam condensation in kg/hr (The enthalpy of
condensation of steam = 22 30 kJ/kg)
L1 = L3 = 0.05m,

(GATE-ME-2002)(5M)

!ltilQl@jjji4iOARfldiM\!&jj.jHyderabad lDelhilBhopallPunc lBhubaneswarl Lucknow l Patna ! BcngaJuru l Oicnnai l Vuayawada l V17.3g ITuupati I Kukatpallyl Kolkatal
SOLUTIONS
03. A ns: (c)
One Mark Solutions Sol: . Q = same , �x = same
. �T
0 1 . A ns: (b) Q=-
R th
Sol: Generally fluids will have lower K than
:. �T oc Rth
solids and within fluids, gases will have low
� 1
"K" and out of steam and air the density and �T oc -· � �T oc -
K
viscosity of steam is higher than air hence
Ko1asswool< K refractory brick < Ksteel < Kcopper
air has low "K"
Hence (�T) for glasswool is maximum
02. A ns: (b)
Sol: Case (i) Case (ii)
04. A ns: (c)
Sol: Out of the given substances pure Aluminum
has high K and steam has low K.

05. A ns: (b)


Sol: 1 -D general heat conduction equation for
sphere
) + q = _!_
BT BT
Let radii be r, 2r,3r
r2
r or or k O't
J_�(
2
U

ln(2) ' ln(2)


21r(2K)L
R1 = R1 =
21tKL 06 . A ns: (d)
R2 =
ln(I .5) ' _ ln(l .5) Sol: For one dimensional unsteady state heat
2 - 2m<.L
R
21t(2K)L conduction without heat generation, the heat
Q _ �T(21tKL) Q ' _ �T(21tKL) conduction equation is
ln{2)
8 2 T 1 of
ln{l .5)
ln(2) + + ln(l .5)
i - i -

ax a. at
2 2 --2 = --

BT 8 2 T
Q I < Q; - oc --2
Hence case(ii) is correct at ax

!IJl@M!@jjji4.jjQRflniN\u1h..)ttyderabad I Delhi I Bhopal I Punc I Bhubaneswar i Lucknow ! Patna I Bcngalwu I Chennai I Vijayawada I V,zag I Tirupali I Kukatpally I Kolkata I
: 359 : Conduction

07. Ans: (d) ( 1 )and (2)will become equal if and only


- Tc if R1 = R2
qi -
Th

Rs 2b - b
S ol:

K I A K 2A
-- = -
q2 - - Tc
Th

Rs + R c 2 1
=> - = -
Because Rs + Re > Rs Kl K2
:. q 1 > q2 => K1 = 2 K2
1
·: Q oc ----
Resistance 09. Ans: (c)
Resistance is inversely proportional to S ol: r0 = 0.01 25 m = 1 2.5 mm ,
thermal conductivity. Hence , it is fact that ho = 5 W/m2 K, Ki = 0.05
conductivity of steel is less than conductivity K i 0.05 �
r = = = = -1- = O.O l m
of copper. h 5 1 00 1 00
: . Rs Re
C

= 1 0 mm
0

(Th - TJ (T - TJ
>

i re < r0 addition of insulation will reduce


heat loss
-'---''-------'-'- > �-- �

Q Q
- Ti > Ti - Tc
Th

Th +Tc > 2Ti Q


+T
TI < _'I=h__�c
t
2

08. Ans: (d)

re = 1 0 r0 12.5 �r
S ol:
=

1 0. Ans: (d)
S ol: Given , K = Ko + bT
T - T1 - Ti
Q= i =
! ( K :) = 0 - ( 1 )
T2

RI R2

=> T = R 1 T2 + R 2 T1 ------- (1) Integrate equation(!)


RI + R2
dT
I

= Tl + T2 -------- (2)
(K 0 + bT)- = C 1
Tavg
dx
2

\( J ( 11!.!,l[I( t I ll lL, P11lil1t.1!11 111, �yderabad ! Dclhi l BhopaJ I Pune l Bhubaneswar l Lucknow l Patna ! BengaJuru ! Chennai J Vtjayawada l V,zag I Tirupati I Kukatpally l Kolkata I

"" ..�
-:, , , �FtipiC'ftqNJiracv.D

dT
. . ACE

C1
. . : 360 :

From the above figure.


Heat Transfer

dx K 0 + bT 03 < 02 < 0,
� - = ----'--

(Ko + bT)dT = C, . dx - (2) :. slope is decreasing in nature.

Now integrate equation (2)


11. Ans: (b)
bT 2
� KoT + - = C 1 x + C 2 - (3) Sol: Heat generation
2
At x = O , T = T1
bT12
From equation (3) K T + - - = C2
1 00°c
0 1
2
At x = L, T = T2

From equation (3)


bT22 = bT 2
-
K o T2 + -- C 1 L + K o T1 + - 1
2 2
Due to heat generation maximum
K 0 (T2 - Ti )+ � (T; - Ti2}
� C1 = 2 temperature inside the wall is greater than
L 100°C.
K 0 (T2 - Ti )+ � (T; -Tn
dT
12. Ans: (c)
L(K 0 + bT)
= ---------
dx
Sol: Q = Ti - T0 = 21tkL( Ti- T0 )
From above as 'L' increase

Note:
dT
dx
decreasing.

Q always occurs in the direction of


R
,�, ln( :: J
decreasing temperature gradient.
.
ln( ;, J
Hence (:) decreases. 2 1tkL
• • R ,01a1
= ---

13. Ans: (a)


(OR)
Sol: k 8as oc &
With increase in temperature, Kgas increases

�ydcrahad l Dclhi J Bhopa) I Punc J Bhubaneswar J Lucknow l Patna J Bcngaluru J Chcnnai J Vtjayawada J Vmg JTirupali I KukatpallyJ Kolkata I
':,��-�
. �CE . . : 361 : Conduction
�:&ia,n,,erqPoNicaarm
=====================================
14 . A ns: 4 .9 to 5.1
S ol: Critical Radius = klho
l30 - T
Q= = 1 2500
0. 1

= 0.1/2 = 0.05 m = 5 cm
:. T = 67.5°C
20

15 . A ns: (a)
18 . A ns: 0 .4 17
S ol:
S ol:

ro
ro = 1mm , k = 0.15 W/m-K
h = 25W/m2-K 50 mm

re = !._ for cylindrical shape


57 mm
ho
K = 50 W/m-K
0.15
= x1000 = 0.15 x 40 = 6mm Assume length of pipe = 1 m
25
: . re > ro ::::::> The heat transfer from the wire
to the ambient will increase. Thermal resistance =
fn

( r, )
21tKL

16 . A ns: (a)
T - Tz L'.lT
S ol: Q = 1 =
.en(�)
2n x 50 x l
R"
= 4.17075 X 1 0-4 K/W
1 03 K
fn(})
21t kL = 4. 1 7075 x10- 4 X
103 W
RTo = 0.417 K/kW
Rth = _.en_(���)
21tkL

17 . A ns: 67 .5 (range 67 to 68)


130 - 30
= _ = 12500 W / m 2
O.
Q -l 0 3
+-
Sol:

20 100

�yderabadl Delhi! Bhopal I Pune I Bhubaneswarl Lucknow! Patna! Bengaluru I Chennai I VtjayawadalVu.ag I Tirupati I Kqkatpally l Kolkata I
�.••
If'

: �
AJCE
V

P.iira«vn
• •
: 362 :• •
Heat Transfer
. � ==========================================

�T , Tanp = -
Tana = - 1 L 2 L
�T-
Two Marks Solutions
K 1 x A(�Ti ) K 2 x A(�T2 )
i 2

Q= =
01. Ans: A-2, B-5, C-6 , D-4 LI L2
K1 K 2- K 1 Tana
02. Ans: (d) => - = --
Tana Tanp K2 Tanp
:. -- =-
20
Sol: r2 = - = 10 mm , Ki = 0.5 As a > P
Tan a > Tan P => K1 > K2
2
ho = 20 W/m2K
0· 5
rC = K; 0.025 = 25 mm Note:
h0 20
= =

Alternate solution: In steady state heat


tc = re - r2 = 25 - 10 = 15 mm
transfer as Q is same �T oc Rth
critical thickness of insulation
For slab ' 1 ' �T is less than compared to
=

03. Ans: (d), slab'2'


Sol: r2 = 10 mm , ho = 20 W/m2K , Hence �T1 < �T2
Ki = 0.5 W/mK => Rthl < Rth2
K;
re = -
0.5
- = 25 mm => K1 > K2 (provided � is same)
h0 20 A
=

tc = re - r2 = 25 - 10 = 15 mm
05. Ans: (b),
Sol: r0 = 0.5mm
04. Ans: (a)
ki = O. l W/mK
Sol:
hi = l OOW/mK
O. l =
r = .!s_ = 0.001m = Imm
C h i 100

Critical thickness = re - r0 = I - 0.5 = 0.5mm


Note:

Wire should be operated in the region where


ro< re ; max Q occurs at re , hence Q
optimum corresponds to re - r0 = 0.5 mm.

\( I l 11t !1ll t JIii� P11li!1t ,i!J\11)', �)'dcrabad j Delhi l Bhopal j Punc l Bhubaneswarl Lucknowl Patna l Bengaiuru l Oiennai lVuayawadajVizag ITU11pati I Kukatpallyl Kolkata I
: 363 : Conduction

06 . A ns: (c) 08. A ns: (c)


S ol:
+Y Sol: q = 80 x l0 6 W / m3 , k =200 W/mK
L 20 mm 0.02 m
j. :-:-:-�- -_-_- - -__;_ _ _ �����'.
_ _ _ _--------_-:-:-:-:-:-:+:-:-: �
= =

:-:-:-:-:-:-:� fluid 1 -D steady state equation for plate


- -_--_--_--_,_-_-
No shp.---------48°C----_- _-_-_-_-_- - - - -.
8 2T
_ _ _ _ _ _ _ _ _ _ _ _ .,. - - - - - - - - - - ·
zone :=:=:=:=:=:=:=:=:=:=:=:=:�:=--_::T. = 4ooc::::
r-- - - - - - --------- _
- -
ax. 2 k - 0
+ q
8 2T
ax. 2 = - k
I

glass : K� = 1.2W/mK
I
q
I
solid
After integration w.r.t. x
8T
k
= - q X+ Cl ------ ( 1 )
Tw 48 C, Tg 40 C, 8x
=
° =
°

Kw 0.6 W/mK,
=
Kg 1 2
. W/mK
= Again integration w.r.t. x
x2
( :a = l x ! O ' Kim T = - q + c 1 x +c 2 ----- (2)
k2
(Q cond)glass = (Q cond)water Boundary condition at x 0, T 1 6 0 °C
= =

dT dT 1 6 0 0 +0 + C2 => C2 1 6 0 °C
- K g .A( J = - K w .A( J
= =

dy g dy w At X 0.02 m, T
= =
120 °c
80 x 10 6 0.022
dT K dT 0.6 120 = +C l X 0 •02+1 6 0
=> ( J = w ( J = X l X l0 200 X 2
4

dy g K g dy 1 2
.
=2000 °C/m

� (:;J,
w
C1

Kim equation (2) became


= 0 - 5 x ! O'
x2
T = - q +2000x +1 6 0 --- (3)
k2
07. A ns: (d) . dT
At maximum temperature , - = O

(:a
dx
S ol: (Qconv) water (Qcond} water
Differentiating equation (3) w.r.t. x
=

hHT = -Kw - A dT q x+2000 = 0


- = --
dx k
0.6 X 1 X 10 4 X = 2000
--- =
Xk 2000 X200
=> h = q 80 x l0 6
48 - 40
0. 6 x 10 4 = 7 5 => x = 5x 10- 3 m = 5 mm
= 0 W/m2 K The location of minimum temperature within
8
the plate is 5 mm from left side.
yderabad l Delhil BhopaI I Pune l Bhubaneswarl Lucknowl Patna l Bengalurui Chcnnail VtjayawadalV17.3g I TlfUpati I Ku4tPaDYI Kolkata
· ACE
��•":E
= ·-

&�,am �·�w���diliratiooa
:P'ffll ��,,::::,
�·�·�========�:��:�:===========���
364 He Transrer

09. Ans: (b)


Ji

6 00
Sol:
= -------
0.025 0.055
ln In
0.01 + 0.025
21t x1 9 21t x 0.2
I 60°C
= --- 0__ = 944.23 W/m
6 0_
0.0076 79+0.627
°
120 c

., 11. Ans: (c)

ITmm 20 mm
Sol:

Im
From the equation (3)
At x = 5 mm = 0.005 m
.
q 0 ·005 2
T = Tmax = - +2000 x0.005+1 6 0
k 2
0.5m
Tmax = 1 6 5 ° C

10. Ans: (c),


Sol: r 1 = 1 cm = 0.01 m,
r2 =2.5 cm = 0.025 m
A = l X l =l
r3 =2.5 + 3 = 5.5 cm = 0.055 m
0.5 0.5
R1 = = =25
0.02 x A 0.02 x l
Ks = 1 9 W/mK
0.25 0.25 x2
R2 = = =5
Ka = 0.2 W/mK A 0.1
2
O. l x
T1 - T3 = 6 00 °C
0.25 0.25 x2 =12.5
R3 = =
A 0.04
2
0.04 x

R.,q = R1 + R23
R 2 R 3 =25 + 3.57
= R1 +
R 2 +R 3
=28.57 k/W � 28.6 k/W

!lti1ijj§jjj4ijjj/QRftn1Mfolh4 yderabad I Dcllii I Bhopal I Pune I Bhubaneswarl Lucknow I Patna I Bengalwu I Chennai I Vijayawada I V17.3g I T1111pati I Kukatpally I K.olkata
�·• · ACE
:�PuNic:atiooa : 365: Conduction
� � ====================================
12. A ns: (d) 13. A ns: (a)
S ol: r ---- Electrical work (8W) S ol: As the insulation thickness increases, the
I
I
surface area exposed for H.T by convection
I
I
I increases and wall thickness also increases
I
: Boundary hence the convection H.T increases and
I
I
conduction H.T reduces. This is because as
I
I
the radius increases effective material
I
·---------------� I
increases � Resistance increases
Assuming, air inside the room 1s at
atmospheric condition 14. A ns: (b)
Patrn = 100 kPa 0.03 m =
S ol: ri = 0.015 m ,
Cv = 718 J/kgK, 2
R = 287 J/kgK 0.05
r0 = = 0.025 m
2
PV = mRT
Kg = 1.05 W/mK
100 x 3x 3x2.5 = m x 0.287 x 293
� m = 26.75 kg
(8W)electrical = -100 J/s X 24 hr
(work done on the system)
0.025
= -100 J/s X 24 X 3600 S In
= O.Ol 5 = 0.0774 K / W
=-100 X 24 X 3600 J 2n x l .05 x l
For non-flow process
8 Q = dU + 8W 15. A ns: (b)
0 = dU + (8W)e1ectrical [8Q = 0 insulated] S ol:

0 = dU- 100 x 24 x 3600


dU = 100 x 24 x 3600
m Cv dT = 100 x 24 x 3600
26.75 x 718 x dT = 100 x 24 x 3600
� dT = 449.73
(T2- 20) = 449.73 Note:
T2 = 449.73 + 20 = 469.73 WBT is not thermodynamic state of
T2 :::: 470°C temperature, it gives indirect measurement of
humidity.
!Mi1i@jjj4Uijjj4jmnlM\jjjjj� ydcrabad I Delhi I Bhopal I Punc I Bhubaneswarl Lucknow I Patna I Bcngaluru I Chennai I Vtjayawadal V17.3g I T11Upari I Kukatpa)Jy I Kolkata
ACE
�••"�·=&

Ji
�·-
.l!i�lffl:!!1111
�·�,g�Pl�di�ili,:rafi�ni �ea;:t�T�ran
:•:_========�:�3:;6:6�:===========�H =s;:fi;;,,
er

If temperature of wall falls below Tdp (dew 1 7. A ns: (d)


point), then condensation occurs on wall. Sol: At Q

Hence 'Twau' is limited to Tdp . of = 1 0'


ox
of
=O
8y m
K
L

-+ due to 'x' direction at P


8xA 1 .2 x A 23 x A

K
2 1 0c -23 ° C

of = 0' of = 1 0 x 2 = 20
1 0. 1 10c
(DBT)

m
(Tdp)
ox oy
As Q is same -+ due to y- direction.
2 1 - 1 0. 1 7 1 0. 1 7 - (- 23)
= => L = 0. 407 m
1 L 1
-- + - - 1 8. A ns: (c)
8A 1 .2A 23A
S ol:
1 6. A ns: (d)
Sol: Q=O 20°c - 2° C
0.3 0. 1 5
20A 20A 50A 50A

Q
Q=O
Q=O
Q
conduction

Q=O s
20 - T T+2
As all other '5' surfaces are well insulated, 0.3 1 0. 1 5 1
--
- +- +-
heat transfer occurs in 1 dimension that is 20 20 50 50
froI? surface PQRS to outside.

= Q convection 1 5.38(20 - T) = 43.48(T + 2)


Therefore ,
307.6 - 1 5.38T = 43.48T + 86.96
Q
conduction

(Q g x volume) = h.A 5 .(Tw - T, J


307.6 - 86.96 = 43.48T + 1 5.38 T
1 00 x (l x 2 x l .5) = 1 0 x (l .5 x 2) x (Tw - 30)
220.64 = 58.86T
300 = 30 x (Tw - 30 )
=> T = 3.75 °c
=> Tw = 40° C

!liiiQl@jjj4U4jj4RflfiiMihiiilij#ydcrabad I Delhi I Bhopal I Pune I Bhubaneswar I Lucknow I Patna I Benga)uru I Chcnnai I Vuayawada IV� I Tirupati I Kuk.atpally I Kolkata I
: 367 : Conduction

21. Ans: 400 K


Sol:
19. Ans: (a)
Sol: 50mm

:-:·L· <· ....... .. .........

600 - Ti Ti - 300
f. / kA f. / 2 kA
= --'----
. ,,, .... - -- ... ... . . . . . . . . . . . . . . . . . .
(600 - Ti )k = 2k(Ti - 300)
. . . .

600 - T = 2T - 600
Assuming water is in contact with the lateral
1200 = 3 Ti
I I

surface, then Q g X volume = Q


Ti = 400 K
nvection
co

22. Ans: 0.345 m


� Ts = Ta +
Q xd
--"-g - Q
4h Sol: q = = 2500W / m 2 ; k = 1.15 W/mK
A
50 x 10 6 x 50 x 10-3 . k(T1 - T2 )
T = 75 + 1.15 x 750
4 x 10 3 q= �L =
L 2500
s

Ts = 75 + 625 = 700°C L = 0.345 m

20. Ans: (O.lW/mK) 23. Ans: (c)


Q
Sol: q ,, = - = (T1 - T2 ) k 0.08
Sol: ro = 0.005m , re = - = -- = 0.008 m
A R 101a1 h 10
Q (500 - 360) 140 As ro < re , heat transfer increases upto re
A 0.02 0.001 0.02 0.04 0.001 beyond which it decreases.
= --'----'-- = ----
- - + -- + -- -- + --
10 Kp 10 10 Kp
140 24. Ans: (b)
� 10 x l0 3 =
1 X 10- Sol: Total Heat generated = Heat convected
4 X 10- +
KP qg x volume = hA !)..T
3
10 1td 2
:. 40 + - = 140 q g x 4 L = h(1tdL)!)..T
Kp
10 1 0 X 10-3
� - = 100 � K p = 0.1 W / mK 4 X 10 7 X = l OOO(T - T )
KP
s <X>
4
\( l I 1 1�11 1 t , 1 11il, P11hl1t 1t1(ll1" yderabad l Delhi l Bhopal l Punc l Bhubancswarl l.ucknow l Patnal Bcngalwuj Chcnnai lVliayawadal Vu.ag IT1111pati I Kukalpallyj Kolkata
: 368 : Heat Transfer

Ts- Too = 100 Multiplying by volume, V in above equation


Ts = 100 + 25 = 125 °C 398 K d 2 T Q g v V pC P 8T
( Vx + x ]=
=

dx 2 k k at
X

25. Ans: (c)


d 2T 8T
T T Vk 2 + Qg
xv = pVC P X
Sol: Q / m 2 = � = � dx at
L 1
R Total - +-
K h
= Rate of Heat storage
27 - (-5) => 5x50x(-80)+500x5 = Rate of heat storage
= 128 W/m2
0. 18 1 :. Rate of heat storage = - 17500 Watt
=

0.9 20 =- 17.5 kW
-- +-

26. Ans: -17.5 kW 27. Ans: 10


Sol: A = 5m2, k = 50 W/mK, Sol: Given data:
L = lm , Q g =500-3 ' R = 5x10- m
. w
m
3

qg = 4x10 7 W/m3
Co = 800 C, CJ
° = -250 C /m, C2 = -40 C/m
° ° 2
k = 25W/mK
T(x) = Co + C1X + C2X qs R 2
Tmax - Ts = 4K
2

( !: J = C1 + 2C 2
4xl0 7 x(5xl0 -3 ) 2
Tmax - Ts = -------
X

d2 = 4 X 25
': 2c 2 = 2 x(- 40) = -80 °C / m 2
dx Tmax-Ts = 10 °C
dT
( J = c 1 =-250 C
dx x=O
°

dT Five Marks Solutions


( d J =-250 + 2x(- 40)=- 3 30
x x=I
1 -D, unsteady conduction equation with heat
generation is
Sol: r1 = 0.05 m, Kp = 4 3 W/mK
01.

d 2 T Q 8 1 of
-2 +- = -- r2 = 0.057 m, Ki = O. l W/mK
dx k a at
ro = 0. 157 m, hi = 570 W/m2K
k
Where, a = - ho = 12 W/m2K, Ti = 500 C
p cP
L= l m To = 30 ° C

�ydcrabad l Dclhi l Bhopal ! Punc l Bhubancswarl LucknowlPa!nalBcngaluru l Chcnnai l VvayawadalVizac ITirupati I Kukalpallyl Kolk.ala I
: 369 : Conduction

The resistance diagram is


R. cond
3so0c 9s 0c

1 1
Ri conv = o 0.025
h; x A1 570 x 1t x 2 x 0.05x l (R:ond)CI = -- = -- = 0.0005
=

Kc1 A 50 x l
5.58 X 10-3
0.0008
=

(R:ond)E = -- 0.000762
o
1n r2 1n
57 K E A l .05 x l
Rs Cond -
- r, 5o - 4.852 x 10-4
2:n:K p L l l
21t x 43 x l (R:onv) - 0.000182
=

h s x A 5500 x l
ln ro 1n
157
Rtotal = 1.444 X 10- 3
Ri Colltl - r2 - - 57 = 1.613
2:n:K . L 21Z' X 0.1 X 1 1
(i) UA = --
1 = 1 R totat
I

Ro conv =
h Ao 12 x 2.1r x 0.157 x l 1
:. U = = 692.5 W/m2 K
0.0845 A (R totat )
0

Req = Ri Conv + Rs Cond + Ri Cond + Ro Conv �T = l


(ii) Q = �T
= 1.70358 A(R total ) A(R total )J
(
� T => = 500 - 30 = 692.65x(350 - 95) = 176.62 kW/m2
Q= Q 275.89 W
R eq 1.70358
=

- o
Q = Ts T 03.
Ro conv Sol: r, = 1 mm , L = 0.25 m ,
=> Ts = To + Q x Ro conv. = 53.31°C r2 = 25 microns = 0.025 mm
I = 0.5 A , V = 4 volts.
02. T1 = 175 C ,
°
T2 = 150°C
Sol:
: ho = 5.5kW/m2K
9s 0c
j_
}mm
KE = l .05 Enamel 0.8mm

C.I
Kc1 =50
3so0c

\( l I 1 1:...,l l li. l l l l l. J '1 d d 1 , l ! l ! J I I-, �yder.ibad I Deihl I Bhopal I PuncI Bhubaneawar I UICknowlPatna I Bcngaluru I Chcnnai JVtjayawada I Vizag I Tiru� I KukalP-allY I Kolkara I
_
" � . . . . : 370 : Heat Transfer
'!.,."F....,,..Pwiliranrm
ACE

Heat generated Qgen = VI = 0.5 x 4 = 2W O. l


Rb = � = = 0.67
Heat transfer through gas by conduction K 8 .A I 0 x 0.015 x1
T2 - T1 O. l
= R =�= = 6 67
In (r2 /r1 ) Kc .A 1 x O.015 x 1
21t K L
c

Under equilibrium conditions


Heat generated = Heat Transferred by
conduction 0.05
Rd = � = = 0.0335
21t x K x 0.25(175 -150) K .A 50 x 0.03 x l
2=
1
ln -- 1 1
0

0.025 R 2 = -- = = 3.33
h 2 A I 0 x 0.03 x1
1
2 x ln --
0.025 = 0.1878 W/mK Reg = R1 + Ra + Rbc + � + R2
=> K =
21t X 0.25(25) 0.67+0.0335+0.6088 + 0.0335 + 3.33
4.67
=

04. T1 - T2 200 - 25
Sol: B = 1 m Q= = = 37.4 W
R eq 4.67
Ka = Ki = 50 ,
Ha = Hd = 3 cm , Hb = He = 1.5 cm
05.
L1 = L3 = 0.5 m , L2 = 0.1 m.
do = 20 mm , di = 18 mm ,
The resistance network for the given
Sol:

problem L = 20 m , Kc = 400 W/mK


Ts = 150°C , hs = 150 W/m2K ,
T0=27°C , ha = 5 W/m2 K
Ki = 0.075 W/mK, Di = 5 mm

0.075
R1 = Conv. Resistance of fluids r = .!s_ = x l OOO = 15 mm
ha 5
1 1
= - = --- = 0.67
C

h 1 A 50x 0.03 x l <lc = 30 mm


dins = de + 5 x 2 = 30 +10 = 40 mm ,
0· 05
Ra = � = = 0.0335 hrg = 2230 kJ/kg
K A .A 50x 0.03 x l

\('l I 1 1 � J l l l l I ll IL', P 1 1 l i l H , l l J 1 ) ! ]'\ Fyderabad l Delhi l BhopaJ I Pune J Bhubaneswarl l.ncknow l Patna l BengaJuru J Chennai l VuayawadalVtzag I Tarupati I Kukatpallyj Kolkam I
'!.�-�
., \I •
ACE
..��PnNirma
• • •
: 371 : Conduction

ln � 40
1n
r2 =
Ri cootl - 20
21t K . L 21t x 0.075 x 20
20 mm Insulation
-

= 0.0735
I

l���I:((� Pipe
Ra conv =
h. x A 3
1
=
5 x 2n x 0.04 x 20
1

= 0.0398
1---------\ \/

Req = 0.119
�T 150 - 27 =
Q= 1033 W
R. CODY R eq 0.119
=
Rp cond R; coDd Ra coDY

1 Rate of condensation of steam


Rs conv = --- l 033
h xA
, X 3600 = 1.667 kg/hr
2230 X 1000
l
s

= = 0.00589
150 x n x 0.018 x 20
r
1n 2 ln
20
- r , = 18
Rp cootl
- 21t K c L 21tK400 x 20
= 0.0000021

�yd I I I I I
erabad Delhi Bhopal Punc Bhubaneswar Lucknow I Patna! Bcngalwu I Chcnnai !Vuayawada I Vmg I TllUpati I I
Kuk.alpaily Kolkatt I
Fins and THC
(GATE-ME-1994)
One Mark Questions
04. Lumped heat transfer analysis of a solid
01 . The heat transfer process between a body object suddenly exposed to a fluid medium
and its ambient is governed by an internal at a different temp is valid when.
conductive resistance (ICR) and an external (GATE-ME-01)
convective resistance (ECR). The body can (a) Biot number < 0.1
be considered to be a lumped heat capacity (b) Biot number> 0.1
system if (GATE-ME-89) (c) Fourier number < 0.1
(a) ICR> ECR (d) Fourier number >O.1
(b) ICR is marginally smaller than ECR
(c) ICR = ECR 05. The value of Biot number is very small (less
(d) ICR is negligible. than 0.1 ) , when (GATE-ME-02)
(a) The convective resistance of fluid is
02. Biot number signifies (GATE-ME-91) negligible
(a) The ratio of heat conducted to heat (b) The conductive resistance of fluid 1s
convected negligible
(b) The ratio of heat convected to heat (c) The conductive resistance of solid 1s
conducted negligible
(c) The ratio of external convective (d) None of the above
resistance to internal conductive
resistance 06 . Which one of the following configurations
(d) The ratio of internal conductive has the highest fin effectiveness?
resistance to external convective (GATE-ME & Pl-12 )
resistance (a) Thin, closely spaced fins
(b) Thin, widely spaced fins
03. When the fluid velocity is doubled the
(c) Thick, widely spaced fins
thermal time constant of a thermometer
(d) Thick, closely spaced fins
used for measuring the fluid temperature
reduces by a factor of 2 (TIF )
lli11ih§jjji4@4RflftiM\j/fjj.yderabad I Delhi I Bhopal I Pune I Bhubancswar l Lucknow I Patna! Bengaluru I Chennai I Vijayawada IVmg ITlfUpari I Kukatpallyj Kolkata I
.!'.�
".. .,,,," �PnNnmna
. ACE
. . .
07. Biot number signifies the ratio of
: 373 :

02. A spherical thermocouple junction of


FINS & THC

(GATE-ME-14- SET-2) diameter 0.706mm is to be used for the


(a) convective resistance in the fluid to measurement of temperature of a gas
conductive resistance in the solid stream. The convective heat transfer
(b) conductive resistance in the solid to coefficient on the bead surface is
convective resistance in the fluid 400W/m2K. Thermo-physical properties of
(c) inertia force to viscous force in the thermocouple material are k =20W/mK,
fluid C = 4001/kgK and p = 8500 kg/m3 . If the
(d) buoyancy force to viscous force in the thermocouple initially at 30°C is placed in a
fluid hot stream of 300°C , the time taken by the
bead to reach 298°C is (GATE-ME-04)
08. The heat loss from a fin is 6W. The
(a)2.35s (b)4.9s
effectiveness of the fin are 3 and 0.75 ,
(c)14.7s (d)29.4s
respectively. The heat loss (in W) from the
fin , keeping the entire fin surface at base
03. A small copper ball of 5 mm diameter at
temperature , is----
500 K is dropped into an oil bath whose
temperature is 300 K. The thermal
(GATE-1 7- SET- 2)

conductivity of copper is 400 W/m . K , its


density 9000 kg/m3 and its specific heat 385
Two Marks Questions

J/kg. K. If the heat transfer coefficient is


01. Two rods , one of length L and the other of
250 W/m2.K and lumped analysis is
length 2L are made of the same material
assumed to be valid , the rate of fall of the
and have the same diameter. The two ends
temperature of the ball at the beginning of
of the longer rod are maintained at 100°C.
cooling will be , in K/s.
One end of the shorter rod is maintained at
100°C while the other end is insulated. Both
(GATE-ME-05)
(a)8.7 (b)13.9 (c)17.3 (d)27.7
the rods are exposed to the same
environment at 40°C. The temp at the
04. A Fin has 5mm diameter and 100mm long.
insulated end of the shorter rod is measured
The thermal conductivity of fin material is
to be 55°C. The temp at the mid point of the
400W/mK. One end of the fin is maintained
longer rod would
at 130°C and its remaining surface is
(GA TE-ME-92)
(a)40 c (b)50°c
exposed to ambient air at 30C. If the
°

(c)55°C (d)100°c
\( l l 1 1 �!llt ( !Ill� P11hlH,l[JIIII" Fyderabad I Delhi I Bhopal I Pune I Bhubaneswar I Lucknow! Patna I Bengaluru I Chennai I Vuayawada I Vu.ag I Tlnlpati I Kukatpally I Kolkata I
: 374 : Heat Transfer

convective heat transfer coefficient 1s c = 600 J/kgK. The time required in seconds
40W/m2K, the heat loss (in W ) from the fin to cool the steel ball in air from 10 30 ° C to
1s 4 30 °C is
(a) 0.08 (b) 5.0 (b) 9 31
(GATE-ME- IO)
(a) 519
(GATE-ME-13)

(c) 7.0 (d) 7.8 (c) 1195 (d) 2144

05. A spherical steel ball of 12mm diameter is 07 . A steel ball of 10 mm diameter at 1000 K is
initially at 1 OOK. It is slowly cooled in a required to be cooled to 350 K by
surrounding of 300K. The heat transfer immersing it in a water environment at 300
coefficient between the steel ball and the K. The convective heat transfer coefficient
surrounding is 5W/m2K. The thermal is 1000 W/m2-K. Thermal conductivity of
conductivity of steel is 20 W/mK. The steel is 40 W/m-K. The time constant for
temperature difference between the centre the cooling process is 16s. The time
and the surface of the steel ball is required (in s) to reach the final temperature
't

1s___
(a) large because conduction resistance is
(GATE-ME-11) (GATE-16- SET-1)

far high than the convective resistance. 08. Two cylindrical shafts A and B at the same
(b) large because conduction resistance is initial temperature are simultaneously
far less than the convective resistance. placed in a furnace. The surfaces of the
(c) small because conduction resistance is shafts remain at the furnace gas temperature
far high than the convective resistance. at all times after they are introduced into the
(d) small because conduction resistance is furnace. The temperature variation in the
far less than the convective resistance. axial direction of the shafts can be assumed
to be negligible. The data related to shafts A
06. A steel ball of diameter 60 mm is initially in and B is given in the following Table.
thermal equilibrium at 10 30 °C in a furnace.
It is suddenly removed from the furnace and
Diameter (m) 0.4 0.1
Quantity Shaft A Shaft B

cooled in ambient air at 30 ° C , with


Thermal conductivity
convective heat transfer coefficient h = 20 40 20
(W/m-K)
W/m2K. The thermo-physical properties of
Volumetric heat capacity
steel are: density p = 7800 kg/m3 , 2 10 6 2 x10 7
(J/m3 -K)
conductivity k = 40 W/mK and specific heat
X

!ffllii@hiiiihQNflnfiijj.jj+yderabad IDelhi !_ Bhopal I Punc I Bhu�cswar I Lucknow! Patna I Benga)uru I Chennai IVtjayawada lViz.agjTlfU_l'31i I Kuk�/:pall}d Kolkata I
: 375 : F1NS & THC

The temperature at the centerline of the


shaft A reaches 400 C after two hours. The
( A)
°

time required (in hours) for the centerline of i ti,

the shaft B to attain the temperature of 6j


(GATE - 16 - SET - 2) -QJ
t..
- L
0 X

09. A cylindrical steel rod , 0.01 m in diameter


(B )

and 0.2 m in length is first heated to 750°C


Ir-
QJ
ti,

and then immersed in a water both at


100°C. The heat transfer coefficient is 250
W/m2 -K. The density , specific heat and
QJ

0 - X L

thermal conductivity of steel are p = 7801


(C)

kg/m3 , c = 473 J/kg-K , and k = 43 W/m-K , i ti,

respectively. The time required for the rod [j


to reach 300°C is ---- seconds.
(GATE - 16 - SET - 3) (D)
t..
0 - X L

10. A long slender metallic rod of length L is


used as a fin. As shown in the figure below ,
the left end of the fin is kept at a constant
temperature tb. The fin loses heat by -x

convection to the atmosphere which is at a 11. A metal ball of diameter 60 mm is initially at


temperature ta (ta< th). Four options of 220°C. The ball is suddenly cooled by an air
temperature profiles are shown. Identify the jet of 20°C. The heat transfer coefficient is
correct option. 200 W/m2 .K. The specific heat , thermal
conductivity and density of the metal ball are
_./
convection

I 400 J/kg.K , 400 W/m.K and 9000 kg/m3 ,


respectively. The ball temperature (in ° C)
-x---------'x=L

after 90 seconds will be approximately


atmosphere at t..

(GATE - 17 - SET - 2)
(GATE - PI-16) (a)141 (b)163 (c)189 (d)210

\ ( ' I I 111.,'lTH't 11111.!, P11l,l1< .11Jtm, �yderabad j DelhijBhopal j PunejBhubaneswarl LucknowjPatnaj Benpluru j <liainai jVuayawadalVwg ITirupati I Knkalpally j Kolkatl I
w
,l
..
...
,,.t w
I,
• •
F.ugioeaq • •
ACE
PuNiravn : 376 : Heat Transfer

Statement for Linked A nswer Q03 & Q04


A wall is heated uniformly at a volumetric heat
generation rate of 1 kW/m3 . the temperature
Five Marks Questions
distribution across the 1m thick wall at a certain
01 . The total efficiency , Tlt for a finned surface instant of time is given by
may be defined as the ratio of the total heat T ( X)= A + BX + CX2
transferred by the combined area of the Where A = 900°C,
fins, Ar, and the exposed surface to that by B = - 200C/m and
the total area, A, if it were maintained at the C = - 50C/m2
base temp, T0. Assuming uniform heat The wall has an area of 1Om2 (as shown in the
transfer coefficient over the entire surface, fig)and a thermal conductivity of 40W/mK.
derive an expression for the relationship
between efficiency and effectiveness of the
fin .
Im

.,,,,??
(GA TE-ME-1989) (SM)
Area= I 0m2
02. An iron rod (K= 41.5W/mK) of 15mm
diameter and 160mm long extends out of a
hot surface of temp 150°C into environment
at 36°C. the free end of the rod is insulated.
x= l

If the film heat transfer coefficient is


x=O

25W/m2K, calculate the rate of heat flowing (GA TE-Pl)(4M)


out of the hot surface through the rod and
the temp at the insulated end of the rod. 03. The rate of heat transfer (in kW) into the
wall ( at x = 0 )
(a)900 (b)450 (c)120
(GA TE-ME-1990)(5M)
(d)80

04. The rate of change of energy storage (in


kW)in the wall is
(a)130 (b)120 (c)- 10 (d)30

\( I l 1 1 -. 1 1 1 ( t r 1 1 1 .., P 1 i l , l 1 1 .ilii 111.., �yderabadl Dclhi I Bhopal I Pune I Bhubaneswarl Lucknow I Patna! Benga)uru I Chcnnai IVuayawadalVizag J Ttrupari I Kukalpallyl Kolkata I
SOLUTIONS
1
't'* OC -
One Mark Solutions
oc ! For laminar
v ·5
't'* �

01. Ans: (d) 1


oc 08 � For turbulent
Sol: Lumped capacity analysis is also called as v·
't'*

negligible internal resistance v = velocity


i.e., ICR <<< ECR if velocity is doubled .

02. Ans: (d) (i) In case of laminar flow ,


. R . ICR- • = 0.5 = ( v )0 5 =
Sol: B1ot number = con<luctton
R convection
=-
ECR
:� ( :: J
2v
. }i = 0 .707
ICR = Internal Conductive Resistance
ECR = External Convective Resistance (ii) In case of turbulent flow

; = �J
.8 = (_y_)0.8 = ( )0 8
_!_ . = 0 .574
't'•
0
0 3. Ans: False 2v 2
(
Sol: From Transient heat conduction equation
't' 1 V2

In both the cases, thermal time constant does


-,
_T- Too_ not reduced by factor of 2.
_ = e ··
To - Too
Where Thermal time constant 04. Ans: (a)
pVc
* -- -- Sol: Lumped heat capacity analysis is used when
hA Bi < 0.1
't'

Here p density , V= volume , c = specific


. heat , h = convection heat transfer
=

05. Ans: (c)


coefficient, A = exposed Area
Sol: B; = Rcond , when Rcond is negligible
h oc v For laminar flow [Nu oc Re Rconv

h oc v0·8 � For turbulent flow [Nu oc Re0·8]


05

Bi becomes less than 0. 1


05
� ]

Fyderabad l Delhi l Bhopal ! Pune l Bhubancswar l Lucknowl Patna f Bengalwu l Chcnnai l Vtjayawada l Vmg I Tuupati I Kukatpallyl Kolkala l
: 378 : Heat Transfer

06. Ans: (a)

Sol: s fin = _Q with fin


= � Two Marks Solutions
Q without fin V �
1 1 01. Ans: (c)
s oc --- => s oc ----
�A c I p thickness
Sol:
(i) Hence thin fins are preferred
X =L Too = 40°C

(ii) YI of multiple fins is more than a single


I
I
I

1 00 °C 1 00° c

fin.
I
I

Hence closed spaced fins


I

2L

Note: If the spacing is too close then 01 = T1 - Too = 100 - 40 = 60° C


convective environment will cease and 02 = T2 - Too = 100 - 40 = 60° C
arr acts as conductive insulating e = 8 1 sin h[m(2L - x)] + 02 sin h(mx)
medium. Which will decrease Q sin h(2mL)
60sin h(mL) + 60sin h(mL)
T - 40 =
07. Ans: (b) sin h(2mL)
. R conduct1on 120sin h(mL)
R T = 40 + · · · · · · · · · · · · · · · · · · ·· ( l )
Biot Number = conduction = sin h (2mL)
· m · sord

R convection R convection in fluid


I

Too = 40°C

08. Ans: 8
°
1 00 °c

Sol: Q t = 6W
_____ 55 C
_

Effectiveness ( E)= 3 Tb = 100° C , Too = 40° C , T1 = 55°C


ac

Efficiency (YI)= 0.75


Entire fin is maintained at base temperature
means Qmax
(:.L =
cos h{mL)
15 1
YI = 60 cos h(mL)
=
Q max
Q act

=> cos h (mL) = 4


0.75 = -- => mL cos h - 1 (4)= 2.063 . . . . .. . . . . . . . . .. (2)
Q max
6 =

As fins are made of same material , kept in


6
Q max = same environment and are of same diameter
0.75
'mL' is same
Qmax = 8W
.\t I I [l�l l l l t l lll� P11hlH ,ilJ1 111 .... FyderabadJ Dclhi J Bhopa) J Pune l Bhubancswarl Lucknow ! Patna J Bengaruru l Chennai J VtjayawadaJ Vizag j T1IUpati I KukatpallyJ Kolkata I
: 379 : FINS & THC

Hence substitute (2)in (1) 02. Ans: (b)


We have Sol: d = 0.706 mm = 0.706 x 10-3 ,
120sin h(mL) h = 400 W/m2K , K = 20 W/mK
T = 40 +
2 sin h(mL )cos h(mL)
C = Cp = 400 J/Kg K , p = 8500 kg/m3 '
60 60
= 40 + = 40 + - = 550C T = 30°C , Too = 300° C , T = 298 °C
cosh (mL) 4

. hf
B =
k k k
(OR)
Shorter rod I

dT 400x0.706xl0-3
100°C ___ --Insulated
=0 0.00235
.,.__

20 x 6
L
°
100 c dx
T
Hence Bi <0.1 � The problem can be
ss 0c analyzed by using Lumped heat parameter

[100°c [,___
• �j_
L :
_____,<I
x longer rod

L
10o•c I
analysis

-
T ---
T00 = -B x F
Tb - Too
e I 0

298 - 300
--- = e -B x F
T

30 -300
°
100 c - -------·------- - I 0

-2
-- = e -B, x F
ss 0c - - - - - -
- 270
0

270
= e ( P�p t)
X -

In shorter rod ,
dT
at x = L , = 0 and (T)x=L = 55° C - = - (for sphere)
V d
dx A 6
Similarly , 270 = hA 't
fn( )
dT 2 p v cP
For longer rod , at x = L, - = 0 ,
400 x 6 x -r
fn(I35) =
dx
The temperature of longer rod will be 55 C °
8500 X 0.000706 X 400
(T)x=L = 55° C [for longer rod also] -r = 4.906 sec

!ffllii@h/lUIOAPflflilmlh.+)ttyderabad I Delhi I Bhopal I Pune I Bhubaneswar l Luckn�wl Patna I Bengaluru I Chennai I Vtjayawada I Vuag I TllUpati I Kukatpally I Kolkata I
03. Ans: (c)
m= {4h = 8.944
S ol: Given, TO = 500 K, Too = 300 K \l kd
d = 5 x10-3 m , K = 400 W/mK Le = 100 +1 .25 = 101 2
. 5mm
p = 9000 kg/m3 Cp = 385 J/kg K
h =250 W/m2 K Q = �hP k A c x S b x Tan h(mLJ
Heat loss by cooper ball = heat gained by fluid
= �40 x n x 5 x l0 -3 x 400 x : x25 x l0 -6
p at
- m e of = h A (T.O - T00 )
x (1 30 - 30) x Tan h(8.944 x101 2
. 5 x10 -3 )
of
at
- p vcP = hA (500 - 300) = 0.0702 x 100 x0.71 9 = 5.05W

- p(:) cP ( :) =250 x200 05. Ans: (d)


S ol: Given d = 12 mm, T0 = 100 K, T00 = 300 K
- p(!} P (:) = 50000 [: = ! for sphere] h = 5 W/m2K , K =20 W/m K

0.005 d 12x l0 -3
of = 50000 5 x - 5 x ---
6 at
- 9000 X X 385 h 6 = --- 6 - = 0.0005
Bi = f = -
k K 20
of Since Bi < 0.1 , lumped head analysis
at
=> - = -1 7.31 K/s IS

valid.
Hence Rconctuction <<<<< Rconvection
(-ve sign indicates cooling of copper ball)

Because Rconctuction I S very small =>


04. Ans: (b)

t------'
temperature drop from centre to surface is
S ol: Too = 30°C
almost tends to zero
1 30°C ct5mm
10mm I
06 . Ans: (d)
h = 40W/m2K ' k = 400 W/mK
Sol : Given ' d = 6 0 X 10-3 m ' To = 1030 °C '
using concept of correction length 1.e =20
Too = 30 ° C, h W/m2K ,
L c = L +(:) , it can be converted as short p= 7800 kg/m3 , k = 40 W/mK,
Cp = 6 00 J/kg K , T = 430 °C
fin with insulated end tip.
h.L c h.R
(a) Bi = = = 5 x10 -4
k 3k
Bi < 0.1 Lumped analysis can be applied .
Fydcrabad l Delhi I Bhopal I Pune l Bhubaneswarl Lucknow I Patna. I Bengaluru l Chcnmi IVuayawada j Vizag I Tirupari I Kukatpally l Kolkala I
: 381 : FINS & THC

430 - 30
20x6xt

1 0 30 - 3 0
3
= e 7800x600x60xW-

400
-- = e -4 2735x!0- xt
4

1000 .
- 0.9162 = - 4.2735 X 10--4 X't Temperature difference = 0, but still heat is
't = 2144.122 secs transferring, this will happen only when heat
transfer coefficient will be infinite.
07. Ans: 42.22 sec (range 42.0 to 42.5) If h � oo,
hLc Then Bi � oo
Sol: Biot Number =
Dimensionless from the functional
Volume d dependence may be expressed as
For sphere Le = =
surface area 6 0* = f (r*, Fo, Bi)
hd l OOO x 0. 01 0 0 As Bi � oo
:. Bi = = = . 416 < 0 .1
6k 6 x 40 And at center line, r* = 0
Hence lumped heat analysis is used. The only variable is Fourier number
T-T :. (Fo)A = (Fo)B
-hA,t -t

-=e = e(
pVCp
00

Ti - Too
=(
(;:t
--

Thermal time constant, )


;: B
pVC
t· = --P = 16sec
hAS ( p c: ,, tJ A � ( p c: ,, tl
350 - 3oo
= e 16 =>t = 42.2249 s
( 2o x1i�x 0 .42 ) x 2 = ( 2 x10� x o .12 ) x 't a
-t

1000 - 300
0

=> 'tB = 2.5 hrs


08. Ans: 2.5 (range 2.4 to 2.6)
Sol: Ts = surface temperature of shaft 09. Ans: 43.49 (range 42 to 45)
Tg = gas temperature of furnace Sol: d = 0 . 01 m, L = 0 .2m , To = 750°C ,
Tg = Ts (always, as given in question)
Too = 100°C , h = 250 W/m2K
p = 7801 kg/m3 , C = 473 J/kgK

\( [ [ 1 1gll l l t J Iil� J 11!d1l,llltllJ-,


>
�ydcrahadl Delhi l Bhopal l Punc l Bhubanc.swarl L.icknow,I Patnal BcngaJuru l Cbcnnai l Vuayawadal Vizag ITuupati I Kukalpa)lyf Kolkata I
ACE : 382 : Heat Transfer
··�:� PiiNianra
" V • • • •
�..

T- Tco Bi<0 . 1 hence lumped analysis is valid


=e
hAt

T0- T00
pvc

1t 2
V --4-=-D---
L DL
= = -__
1t
A 2x-D2 + 1tDL 2D + 4L T = Tco + (TO- Tco ) e
-ht
pcL,

T = 141 °C
= 0.0024 39
4

300- 100
=e
750 - 100
t
780 1x4���� 002439

250x 't
Five Marks Solutions
- 1.178 =
7801 X 473 X 0.002439
't = 42.4 3 sec 01.

Sol: ll r.n =
(Q
�rx
Sol: For long fin
10 . A ns: (b) 8=8b

Q max
T - Tco
--'-'-- = e-mx

According to the above equation temperature


Ti - T00

profile is exponential and temperature


t = . Q fin

decreases with x.
Q without fin

Q without fin =
h . A (Tb- Too)

If the total area is extended as shown in figure (2)


11 . A ns: (a)

R = 0.0 3m , To = 220 C
Sol: Given data
°

Tao = 20 °C , h = 200W/m2K -------


c = 400J/kgK , k = 400W /mK -------
p = 9000kg/m T=? , t = 90 sec
[)-------,--
-------

The hatched area indicates the cross


3
,

V 3 1tR R sectional area of single fin which was earlier


L = - = -=-------2 =
- = 0.01
4
3

c As 4 1tR 3
the surface area.
. . hL 200x0 .01
B 10t number 8 1 = --c = ---- Q without fin = h. Ac (Tb- Too )
k 400
= 5x10-3
\( 1 I 11g11u t1111c, P11lil1t.1IH)lh yderabad I Dcllti I Bhopal I Punc I Bhubaneswar I LucknowI Patna I Bengaluru I Cheruiai I Vtjayawada I Vizag I Tuupati I Kukatpal)y I Kolk.ala
.,....
� ,�""' ACE : 383: FINS & THC
':, :�PnlilirPD
. . . .
.

=> E = ll fin X h X A s X eb 03. Ans: (d)


hxA C xe b Sol: Q g = lkW / m 3 = 1000 W / m 3
T = A + Bx + Cx2 , A = 900 °C
B =- 200 °C/m, C =- 50 °C/m2
A = 10m2 ' k = 40W/mK
02.
(Q t=o =
Sol: Given K = 41.5 W/mK, h = 25 W/m2K,
-k.A(:)
I = 160 mm, d = 15 mm,
To = 150 C, T00 = 36 ° C,
ofox = B + 2Cx

p = 1td = 7t X 0 .015 =0 .0471 (of) = B = (- 200)


Heat transfer in fin with insulated at the tip
OX x=O

of the end Q = (�h p k A c ) 8 0 tanh(mL) (Q }x=O = -40x10x(- 200) = 80000 = 80 kW

A c = 7t (0.015)2 = l .744x10 -4 m 2 04. Ans: (d)


4
Sol:
hp 25x0 .047
m =� = = l l. S l
KA c 41.5xl .744x10 -4
00 = T0 - T00 =150- 36 = 114

Q =�2 5 x0 .0471x41.5xl .744x10 --4


900°c

xl 14x tan h(12 .75x0 .16)


= 10 .1746 W
Temperature distribution profile for fin tip is
insulated is
Im

T = A +Bx + Cx2
T - T 00 cosh(m(L - x)) At x = 0 , T = A = 900 ° C
T - T 00 cosh(mL)
0 At x = 1 , T = 900 + -
( 200)-50
_
-

At x = L , = 900- 250 = 650 °C


T - 36 _ cos h(O) _ 1
- -
150 - 36 cos h(mL) cos h (12 .75x0 .16) 1-D heat conduction equation with heat
=> T =65.18 ° �65 ° C generation

FydcrabadlDclhilBhopallPunclBhubancswarl LucknowlPatnalBcngaJurulChennailVuayawadalVIZag ITuupati I Kukalpallyj Kolkala I


" " . . . . ACE Heat Transfer
..-�:r.,..,.,,.PoNir,n,n : 384 :

82 T . = a T
- k- + q p c-
ax.
in
82 (A + Bx + C x 2 ) 8T
2

k + q - pc
m
• _
-
ax. 2

8T
- k(0.8x0 +2c)+ q = p c
in
8T
40(0 + O + 2x(- 50))+1000 = p c
in
aT
- 4000 + 1000 = p c-
m
aT
- 3000 = p c-
m
aT
p c- = rate of change of energy storage per
in
unit volume.
Total rate of change of energy storage
aT
= p c-xvol
in
=- 3000 x (A x L)
= -3000 ( l Q X } ) =- 30000 W
=- 30 kW
X

= 30 kW -
( ve sign indicates that decrease
in energy storage).

\( I } [l�lllt t I Ill� P11lilll .ti!( Ill', Fyderabad l Delhi I Bhopal I Punc I Bhubaneswar l Lucknow ! Patna! Bcnga)urul Chcnnai I Vuayawada j Vizag j Tirupali I Kukatpallyj Kolkata I
Convection
04. Heat transfer coefficients for free convection
One Mark Questions in gasses , forced convection in gases and
vapors , and for boiling water lie ,
01. For the fluid flowing over a flat plate with respectively , in the range of
Prandtle number greater than unity , the (GATE-ME-98)
thermal boundary layer for laminar forced (a)5-15 , 20-200 and 3000 - 50000 W/m2K
convection (GATE-ME-88) (b) 20-50 , 200-500 , and
50000-l OOOOOW/ m2K
(a) is thinner than the hydrodynamic (c) 50 -100 , 500 -1000 and
boundary layer 100000 - 1000000 W/ m2K
(b)Has thickness equal to zero (d) 20 - 100 , 200 - 1000 and
(c) is of same thickness as hydrodynamic a constant 1000000 WI m2K
boundary layer
(d) is thicker than the hydrodynamic 05. For flow of fluid over a heated plate , the
boundary layer following fluid properties are known:
Viscosity = O.OOl Pa.s; Specific heat at
02. In pool boiling the highest HTC occurs in constant pressure = 1 kJ/kgK; Thermal
(GATE-ME-90) conductivity = 1 W/mK , The hydrodynamic
(a)Sub-cooled boiling zone boundary layer thickness at a specified
(b)Nucleate boiling zone location on the plate 1s 1mm , thermal
(c)Partial film boiling zone boundary layer thickness at the same
(d)Film boiling zone location is (GATE-ME-08)
(a)0.001mm (b)0.01mm
03. For air near atmospheric condition flowing (c)1mm (d)1000mm
over a flat plate the laminar thermal
boundary layer is thicker than hydrodynamic 06. A coolant fluid at 30°C flows over a heated
flat plate maintained at a constant
temperature of 100°C. The boundary layer
boundary layer (T/F) (GATE-ME-94)

temp distribution at a given location on the


�yderabad I Delhi I Bhopal I Pwic I Bhubancswarl wcknowl Patna I Bcngaluru I Chennai I Vuayawada i Vizag I Tuupati I � I Kol.kalaI
ACE
•."E·=1Fii�1p1 :·:,========..,;:�3�8�6:,,::============��:.,;�=:�
Heat Trans£er
� ·-
!:�- 11�Pu
�·� �+�•�rw�:m
plate may be approximated as T = 30 + 09 . Match Group A with Group B
Ji

70exp(-y) , where y (in m) is the distance


normal to the plate and T is in °C . If thermal P: Biot number
Group A

conductivity of the fluid is l .OW/mk, the Q: Grashof number


local convective heat transfer (in W/m2K) at R: Prandtl number
that location will be S: Reynolds number
(a) 0.2 (b) 1 (c) 5 (d) 10
(GATE-ME-0 9)

1 : Ratio of buoyancy to viscous force


Group B

07 . Consider a two-dimensional laminar flow 2 : Ratio of inertia force to viscous force


over a long cylinder as shown in the figure 3 : Ratio of momentum to thermal
below. diffusivities
4 : Ratio of internal thermal resistance to
- 2 3
boundary layer thermal resistance
-
u 1 Q4
(a) P-4, Q-1, R- 3, S-2
00
(GATE-ME- 14- SET-4)
Too -

(b) P-4 , Q-3, R-1, S-2


- ,

(c) P-3, Q-2, R-1 , S-4


Ts

The free stream velocity is Uoo and the free


stream temperature Too is lower than the (d) P-2, Q-1, R-3, S-4
cylinder surface temperature Ts. The local
heat transfer coefficient is minimum at point 10 . The Blasius equation related to boundary
layer theory is a
�2 (a) third-order linear partial differential
(GATE-15-Set 1)
Wl �3 00 4
(GATE-ME- 14- SET-2)

equation
08. For laminar forced convection over a flat (b) third-order nonlinear partial differential
plate, if the free stream velocity increases by equation
a factor of 2, the average heat transfer (c) second --0rder nonlinear ordinary
coefficient differential equation
(a) remains same (d) third-order nonlinear ordinary
(GATE-ME- 14- SET-2)

(b) decreased by a factor of ..f2 differential equation.

(c) rises by a factor of ..f2


(d) rises by a factor of 4

\( I l 111-;111t t l lllg P11hl ]( ,IIJ1 ) ( ] ', �yderabad I Delhi I Bhopal I Punc I Bhubanesw.ir j Lucknow! Patna I Bcngaluru I Chennai I Vliayawada I Vizag jT,rupali I Kublpally I Kolkat, I
: 387: Convection

11. For flow of viscous fluid over a flat plate, if


the fluid temperature is the same as the plate Two Marks Questions

(GATE -15 -Set 1)


temperature, the thermal boundary layer is
01. A fluid flowing over a flat plate has the
(a) thinner than the velocity boundary layer. following properties: dynamic viscocity
6
(b) thicker than the velocity boundary layer = 25 x 10- kg/ms, specific heat = 2.0
(c) of the same thickness as the velocity kJ/kgK, thermal conductivity 0.05W/mK.
boundary layer The hydrodynamic boundary layer
(d) not formed at all thickness is measured to be 0.5 mm. The

(GATE-ME-92)
thickness of the thermal boundary layer
12. In the laminar flow of air (Pr = 0.7) over a would be
heated plate, if 8 and 8r denote, respectively (a) 0.1mm (b) 0.5mm
the hydrodynamic and thermal boundary (c) 1.0mm (d) none

(GATE -15 -Set 2)


layer thickness, then
02. Match List- I with List- II and select the

(GATE-ME-96)
(b) 8 >81 correct answer using the code given below
(d) 8 = 0 but 81 0 * the Lists:

List - I List - II
13. The ratio of momentum diffusivity (v) to A. Grashof number 1. Mass diffusion

(GATE -15 -Set 3)


thermal diffusivity (a), is called B. Schmid number 2. Transient heat
conduction
(a) Prandtl number (b) Nusselt number C. Weber number 3.Free convection
(c) Biot number (d) Lewis number D. Fourier number 4.Forced convection
5. Surface tension

(GATE - 16 - SET - 3)
14. Grashof number signifies the ratio of 6. Radiation

(a) inertia force to viscous force 03. Water (Prandtl number = 6) flows over a flat
(b) buoyancy force to viscous force plate which is heated over the entire length.
(c) buoyancy force to inertia force Which one of the following relationship
(d) inertial force to surface tension force between the hydrodynamic boundary layer

(GATE-ME-01)
thickness(c5) and thermal boundary layer
thickness (81) is true

!M•j4i@jjji4.jjQ@niN\Uijj..;yderabadl Delhi I Bhopal I Puncl Bhubaneswarl Lucknow ! Patna! Bcngalwul Chennai IV,jayawada!Vizag l 'firupali I Kukatpallyl Kolkata I
.. .,., ».,,w:rmcPnNnnra
ACE : 388 : Heat Transfer
�-� " • • • •
w

(b)8t <8 for constant wall temperature conditions and, for


(d)can not be predicted turbulent flow, Nu = 0.023 Re0· 8 Pr033 .
(GATE-ME-OS)
04. The properties of mercury at 300K are 06. The Reynolds number for the flow is
density=l 3529kg/m3 , Cp = 0.1393 kJ/kgK, (a)444 (b)890
dynamic viscosity= 0.1523 x 10 -2 N-s/m2 (c)4.44x105 (d)5.33x 105
and thermal conductivity = 8.540 W/m-K.
The Prandtl number of the mercury at 300K 07. The heat transfer per meter length of the
duct, in watts, is
(a)3.8 (b)5.3
ts (GATE-ME-0 2)
(a)0.0248 (b)2.48
(c)24.8 (d)248 (c)89 (d)769

08. The temp distribution within the Laminar


05. Consider a laminar boundary layer over a
thermal boundary layer over a heated
heated flat plate. The free stream velocity is
isothermal flat plate is given by (T -Tw) /
Uoo, At some distance x from the leading
(Too-Tw) = (3/2) (y/8t) - (1/2) (yl8t )3 , where
edge the velocity boundary layer thickness is
Tw and Toc are the temp of plate and free
8v and the thermal boundary layer is Or, if
stream respectively, and 'y' is the normal
the Prandtl number is greater than1, then
distance measured from the plate. The ratio
of Average to the local Nussult number
(GATE-ME-03)
(b)8r Ov
based on the thermal boundary layer
(d)8v = 8r - X -l/2
>

thickness Ot is given by (GATE-ME-07)


(a)1.33 (b)1.5
(c)2.0 (d)4.64
Statement for Linked A nswer Q06 & Q07
An un-insulated air conditioning duct of
rectangular cross section 1 m x 0.5 m, carrying
09. The average heat transfer coefficient on a
air at 20°C with a velocity of 10 mis , is exposed
thin hot vertical plate suspended in still air
to an ambient of 30 °C. Neglect the effect of duct
can be determined from observations of the
construction material. For air in the range of change in plate temperature with time as it
20-30° C, data are as follows: thermal cools. Assume the plate temp to be uniform
conductivity = 0.025 W/m. K; viscosity = 18 at any instant of time and radiation heat
µPa.s; Prandtl number = 0.73; density = 1.2 exchange with the surroundings 1s
kg/m3 • The laminar flow Nusselt number is 3.4
\( I I 1 1 � 1 1 1 t ( 1 1 1 1:.:, P,il,111 . t 1 1 ( 1 1 1 " yderabadJDelliiJBhopa)JPuneJBhubancswarJ LucknowJPatnaJBengaluruJChennaiJVtjayawadaJVizag JT1JUpati J Kukatpa)lyJ Kolkata
: 389 : Convection

negligible. The ambient temperature is 25 °C, 11. The ratios of the laminar hydrodynamic
the plate has a total surface area of O. l m2 boundary layer thickness to thermal
and a mass of 4 kg. boundary layer thickness of flows of two
The specific heat of the plate material is
fluids P and Q on a flat plate are _!_ and 2
2.5KJ/KgK. The convective heat transfer 2
coefficient in W/m2K, at instant when the respectively. The Reynolds number based on
plate temp is 225 °C and the change in plate the plate length for both the flows is 10 4 • The
temp with time d T/dt = -0.02K/s, is
Prandtl and Nusselt numbers for P are _!_ and
8
(a) 200 (b)20 (c) 15 (d) 10 35 respectively. The Prandtl and Nusselt
(GATE-ME-07)

number for Q are respectively


10. Match the following
(a) 8 and 140 (b) 8 and 70
(GA TE-ME-10) (GA TE-ME-11)

P. Compressible flow (c) 4 and 70 (d) 4 and 35


List- I

Q. Free surface flow


R. Boundary layer flow 12. Consider one dimensional steady state heat
S. Pipe flow conduction across a wall (as shown in figure
T. Heat convection below) of thickness 30 mm and thermal
conductivity 15 W/m.K. At x = 0 , a constant
heat flux, q" = 1x10 5 W/m2 is applied. On
U. Renolds number the other side of the wall, heat is removed
List- II

V. Nussult number from the wall by convection with a fluid at


W. Weber number 25 °C and heat transfer coefficient of
X. Froude number 250W/m2.K. The temperature (in°C ), at
Y. Mach number = 0 is
Z. Skin friction coefficient
X

(a) P-U' Q-X' R-V' S-Z ' T-W


(b) P-W Q-X, R-Z, S-U, T-V
(c) P-Y ' Q-W' R-Z ' S-U' T-X
(d) P-Y Q-W, R-2, S-U, T-V Too = 25 °C
I-+ X
x =O
(GATE-ME-14-SET-1)
\( I l 11�111c t 1 1 1 1� P11li!1t .1t101J, ydcrabadl Delhi I Bhopal I Punel Bbubancswarl Lucknow I Pamal Bengaluru I Cbennai l Vliayawadal VIZ3g ITuupali I Kukatpallyl Kolkala
: 390 : Heat Transfer

1 3. The non-dimensional fluid temperature 15. For flow through a pipe of radius R, the
profile near the surface of a convectively velocity and temperature distribution are as
cooled flat plate is given by follows :
Tw - T = a + b 1. + c(.r.J where y is 1-(;)'l
2

U(r, x) -c, and T(r, x) - C, [

measured perpendicular to the plate, L is the


Tw ..:. Too L L '

where C 1 and C 2 are constants. The bulk


length, and a, b and c are arbitrary constants.
mean temperature is given by
Tw_ and Too are wall and ambient
2
temperatures, respectively. If the thermal Tm = f u(r, x)T(r, x)rdr,
UmR
R

conductivity of the fluid is k and the wall


2

with Um being the mean velocity of flow.


o

heat flux is q", the Nusselt number


q" L The value of Tm is
N = --- is equal to
0.5C 2
(GATE-15-Set 1)

(a) (b) 0.5C 2


u Tw - Too k
um
(a) a (b) b (c) 2c (d) (b + 2c)
(GATE-ME- 14- SET- l)

(c) 0.6C 2

14. Water flows through a tube of diameter


25mm at an average velocity of l .Om/s. The 16. A fluid (Prandtl number, Pr = 1 ) at 500 K
properties of water are p= 1000 kg/m3, flows over a flat plate of 1 .5 m length,
µ = 7.25x10 --4 N.s/m2, K = 0.625W/m.K, Pr maintained at 300 K. The velocity of the
= 4.85. Using Nu = 0.02 3 Pr ·4, the Re · fluid is 10 mis. Assuming kinematic
convective heat transfer coefficient (in viscosity, v = 30 x 10-6 m2 /s, the thermal
08 0

W/m2 .K) is ___ boundary layer thickness (in mm) at 0.5 m


(GATE-ME- 14- SET-2) from the leading edge is __
(GATE- 16- SET- 1)

!Ml•R!@hiiiih4@b¥hiM'*Iyderabad j Delhij Bhopalj Punc l Bhubaneswarl LucknowlPatna j Bengalurui Ch.cnnailVtjayawadalVizag j T1IUpati I Kukatpallyj Kolkatal
SOLUTIONS
agitation induces considerable fluid m1xmg
One Mark Solutions and that promotes substantial increase in the
heat flux and boiling heat transfer coefficient.
01 . Ans: (a)
Sol: Pr> l 03. Ans: False.
=> momentum diffusion > thermal diffusion Sol: For air near the atmosphere condition,
oh Prandtle number, Pr = 1 => Ob = Ot
= (Pr)½ => oh > ot
ot
04. Ans: (a)
02. Ans: (b) Sol: h = 5 to25 � free convection in gasses
Sol: =25 to250 � force convection in
Interface
-=-==---- Bubbles --o...,____ Film
evaporation
gasses and vapours
1 --- II IV v-----'VI-- = 5000 to 10 5 � Boiling and
condensation
· · · hgases < hvapour < hliquid
Pure convection:
heat transferred
by super heated

05. Ans: (c)


liquid rising to
the liquid-vapour
interface where

Sol: o =1mm,
evaporation talces
place

Partial, nucleate
c O.OO l x l OOO
Pr = µ P =
boiling & unstable
interface nucleate film =l
K 1
Because Pr = 1
0.1 1 .0 10 100 1 000 1 0,000

oh
Temperature excess, (ts - tsa1)C
=> - =1 => oh = ot = 1
ot
(Boiling curve for water : Surface heat flux as a

06 . Ans: (b)
function of excess temperature)

The nucleation boiling is characterized by the Sol: Tp = 100 °C K = l .O W/mK


formation of bubbles at the nucleation sites Local h.t.c. on the surface where y = 0
and the resulting liquid agitation. The bubble Hence when y = 0

\( I l 11!!,JTH l lllll:, P1 1lilu ,111011, yderabad l Delhi l Bhopal/ Pune / Bhubaneswar / Lucknow/ Patna / Bcnga]uru/ Chcnnai /Vuayawada !Vu.ag / Ttrupali I Kukatpally/ Kolkata
: 392 : Heat Transfer

os
� = ( 2v ) .
h1 V
- - -)
- - -)

09. Ans: (a)


region

S ol: P - 4 ; Q - 1 ; R - 3 ; S -2
In the No slip region
Heat transfer by conduction = Heat transfer
10. Ans: (d)
by convection in y direction
S ol: The differential equation is 2f "' + ff ' = 0
- KA( ! J = hA(Tp - TJ
T

y =O
y 11. Ans: (d)
- K(: (30 + 70e-y )J = h(Tp - T"' ) S ol: No thermal boundary layer smce heat
y ) y=O transfer is zero
- (1)(0 + 70 X (-1)e- y t=O = h(100 - 30)
12. Ans: (c)
- (- 70e -y t=O = h X 70
S ol: Hydrodynamic thickness/ thermal thickness
70 x l
70e --0 = h x 70 =:> h = = 1 W/m2K
70 � Pr
81

113

If Pr > 1 � 8 > 81
07. Ans: (b)
If Pr < 1 � 8 <81
Sol: For laminar flow � flow separation occurs
at 82 ° i.e. , approx at point 2. so that local
13. Ans: (a)
heat transfer coefficient will be minimum at
C
point 2. S ol: Prandtl number = µ P = �
K a

08. Ans: (c)


14. Ans: (b)
S ol: For laminar forced convection over a flat
Buoyancy force
plate SoI : Grashof No. = ----­
Viscous force
Nu = 0.332(Re )°"5 (Pr )°"33
h oc (v )05

\( } ( 1 1�111{ t I l[ ll.!, P11hl1t ,t!JOII'- �ydcrabad I Delhi I Bhopal I Punc I Bhubaneswar I Lucknow I Patna I Bcngaluru I Chcnnai I Vtjayawada I Vizag I Tirupati I Kukatpally I Kolkata I
: 393 : Convection

05. Ans: (a)


8
Sol: v = Pr
Or
Two Marks Solutions ½

01. Ans: (b), Ov > Or ( ·: Pr >1 )


Sol: µ = 25 x 10--6 kg/ms,
Where 8v = hydrodynamic layer thickness
Cp = 2000 J/ kg.K
Or = thermal boundary layer thickness.
K = 0.05 W/mk, 8h= 0.5
µ cP 25 x10 -6 x 2 x10 3
Pr =l 06. Ans: (c)
k 0.05
=

pV Dh
·: Pr = 1 => Sol: Reynolds No (Re)=
� (Pr)3 µ
I

81
_ h
()

If Pr = 1 Because of rectangular duct, it has to be


8h = 8t = 0.5 mm converted into equivalent circular duct.
4Ac 4 x 0 .5 x l _
Dh = 0 67 m
02. Ans: A-3, B-1, C-5, D-2 P 2(1 + 0.5)
l .2 x10 x 0.67
03. Ans: (b) : . Re = 4.46xl0 5
18 X 10-6
Sol: Given Pr = 6 => � = (Pr)½ = 6 ½ =1.82
81
07. Ans: (d)
8h = 1.82 8t => 8h > 8t
Sol: Because Re > 2000, the flow is turbulent
Nu = 0.023 Re0· 8 Pr0. 3 3
04. Ans: (a)
= 0.023 (4.4 6 X 105)°'8 (0.73)0.3 3
Sol: T = 300k, p = 13529kg/m3 ,
Cp = 139.3 W/kgK h De = 685.6
µ 0.1523 x 10- N-s/m
= 2 2

k = 8.54 W/mK 685.6x 0.025


,

=> h = = 25 _ 58 W / m 2 K
µc 0.67
Pr = _P = 0.0248 Heat transfer Q = hA 11 T = h( Pl )11T
2(1.5) X 1 (30-20)
k
Note : = 25.58 X

For liquid metals Pr <0.1 = 769 W/m


Liquid mercury is molten metal hence it
should have Pr <0.1 .
Fyderabadl Delhi I Bhopal I Puncl Bhubancswari Lucknow I Patna I Bcngaluru I Chennai I Vtiayawadal V1.7.3g ! Tuupari I Kukatpallyi Kolkata I
!.,"EJ� JNn4irm : 394 : Heat Transfer
========================================
08. Ans: (c), dT
=> - me p - = hA(Tp - T )
Sol: In laminar flow over flat plate dt
=> - 4 x 2500 x (0.02)= h x 0.1 x (225 - 25)
CX)

Nux = 0.332 (Re)� x (Pr }i


_!_ I

=> h = 10 W/m2K
Local heat transfer coefficient ,

, 0 . 3 3:xk ( p�x r (Pr)i 10. Ans: (d)


h - x
I

11. Ans: (a)


hx oc (x)2
-I

Sol: � = _!_ for fluid P and � = 2 for fluid Q


=> h x = K(x}2 8, 2 8,
-I

Average heat transfer coefficient is given 1


(Pr}i = - and (Pr}i =2
I I

2
by , -h = -
1 L
h .dx
L Io x Pr = .!. for fluid P
h = 2 X hL
and Pr = 8 for fluid Q
�=2
hL For laminar flow over flat plate
:. In laminar flow h = 2h L Nu oc (Re)°. s . (Pr)°-3333

Where , Nu = Average Nussult number Nu Q Re Q l o.5 f PrQ lo.3333


=>
Nu r r Re p 1 Prp
=
Nx = local Nussult number
8
= (1)0.5. - . =4
0

09. Ans: (d) 11/ 8 1 33333


Sol: Given ,
NUQ = 4 Nup = 4 x 35 = 140
Teo =
25 C , A 0.1 m
° = 2

m = 4kg , C P = 2.5 kJ / kg K , h=?


12. Ans: 625°C
TP = 225° C , dT = -0.02 K / s T -T
Sol: q" = { t
dt
Because the temperature of the plate 1s -+­
uniform , lumped analysis is applicable.
k h
T1 - 25
Heat balance equation:
l x l 05 = 0.03 => T = 625° C
1
-- + -
Rate of decrease of Internal Energy 15 250
I

= Heat Transfer by convection

�ydcrahad. l Delhi l BhopaJ I Pune j Bhubaneowar l Lucknowl Palna i Bengaluru j Chennai j Vuayawadal V123g I Tuupati I Kukatpallyj Kolkata I
.,
�..
.•,
. ACE
. . .
:&,pwtqPi,Ni,,di,;n
" : 395: Convection

13. Ans: (b) 2 R


Tm = f U(r,x)T(r,x) rdr
Um R 2 o
Sol:-k.A(ofJ =h. A s (Tw -Too)
O'j y=O

-k(ofJ = h(Tw -T J
O'j y=O 2C 1 C 2 R ( �
r- J dr
U m R J0
=

{{�;.J
2 3
R

- h .L

a(r)
=> Nu = - = ----,-----,--
K
2'.=o
L

{•+b(t)+{t)'J
a(r)
=

=
(f) o
=i._ c 1 c 2
10 u m
= [b + c2(L)] =b + 2c(0) = b
L ( )=o = 0.6 C2
f
16. Ans: 6 (range 5.90 to 6.25)
14. Ans: 4613.66
Sol: V = 10 mis, x = 0.5m,
pVD 10 3 x1 x 25 x10-3
Sol: R = = v = 30x10 -6 m 2 /s
µ 7.25 X 10-4
e
V xx lOx0.5
Re = =
=�xl0 4 =34482.75
X
U 30 x10-6
7.25 = 166666.66 = 1.667xl0 5 < 5x10 5
Nu = 0.023(34482.75) °- 8 x (4.85) 0.4
:. Flow islaminar,
Nu =184.54 = hD/k
0
2. =(Pr)3 =1
I

Nu xk 184.5488x0.625 8,
h= = =4613_66
D 25xl0-3
:. 8h = 8t
5x0.5
0 -�-
15. Ans: (c)
Sol: U (r,x) = C1 h - Fe: - .Jl.667 X 10 5

T(r, x )-c,[1-(�)'] = 6.123x10-3 m = 6.12 mm

\( l I 11g111t t 1111g Plil,lu.1111111, �yderabadlDelhi!BhopaliPunelBhubancswari l.ucknowl�IBcngaiurulChennailV\iayawadalVu.ag l'Tirupari I Kukatpallyl Kolkala I


Radiation
The temperature and emissivity values for
One Mark Questions the surfaces of the room are 300 K and 0.3
respectively. Boltzmannn constant cr = 5.67
01. For a glass plate transitivity and reflectivity x 10- W/m K . The total heat loss from the
8 2 4

are specified as 0.86 and 0.08 respectively, two surfaces of the plate is
the absorptivity of the glass plate is (GATE-ME-03)
(ME-GATE-88) (a) 13.66 W (b) 27.32 W
(a) 0.86 (b) 0.08 (c) 27.87 W (d) 13.66 MW
(c) 1.00 (d) 0.06
05. The following figure was generated from
experimental data relating spectral black
02. A diffuse radiation surface has
body emissive power to wave length at three
(ME-GATE-91)
temperatures Ti, T2 and T3 (T, > T 2 > T3 ).
(a) Radiation intensity independent of angle
(b) Emissive power independent of angle
(c) Emissive power independent of wave
length
(d) Radiation intensity independent of both
angle and wavelength

A.(µm)
03. The shape factors with themselves of two
infinitely long black body concentric The conclusion is that the measurements are
cylinders with a dia ratio of 3 are ...... For (ME-GATE-OS)
the inner and ...... for the outer (a) correct because the maxima in Eb;i.. show
(ME-GATE-94) the correct trend
(b) correct because Planck's law is satisfied
04. A plate having 10 cm 2 area each side is (c) wrong because the Stephen Boltzmannn
hanging in the middle of a room of 100 m 2 law is not satisfied
total surface area. The plate temperature and (d) wrong because Wien's displacement
emissivity are respectively 800 K and 0.6. law is not satisfied

!MN1i@jjji4ijjg@ijj@jjjjj.�Iyderabad I Delhi I Bhopal I Pune I Bhubaneswar I Lucknow I Patna I Bengaltuu I Chcnnai IVuayawada I Vmg !Tuupati I Kukatpally I Kolkala I
" .� •·CE . .
'!..•:F�Pm,li,:ati,u
" : 397: Radiation
� �==========================================
06. For an opaque surface , the absorptivity (a),
transmissivity (t) and reflectivity (p) are Two Marks Questions
related by the equation:
(ME & PI-GATE-12) 01. The radiative heat transfer rate per unit area
(a) a+p = t (b) p+a+t=O (W/m2) between two plane parallel gray
(c) a+p =1 (d) a+p=O surfaces (emissivity = 0.9) maintained at
400K and 300K is (crb= Stephen Boltzmann
07. Consider the radiation heat exchange inside constant 5.67 x 10-8 W/m2K4)
an annulus between two very long concentric (ME-GATE-93)
cylinders. The radius of the outer cylinder is (a) 992 (b) 812
Ro and that of the inner cylinder is Ri. The (c) 464 (d) 567
radiation view factor of the outer cylinder
onto itself is (GATE - 16 - SET- 2) 02. F or the circular tube of equal length and
R diameter shown in fig below , the view

(
(a) 1-ffe, (b) �I- ;
Ro factor F 13 is 0.17. The view factor F 12 in this
(ME-GATE-01)

J
l/3 case will be.
(c) 1- ::
(a) 0.17

08. The emissive power of a blackbody is P. If


(b) 0.21
T
its absolute temperature 1s doubled, the L=D

emissive power becomes


(c) 0.79
(GATE-17-SET-2)
(a) 2P (b) 4P (c) 8P (d) 16P
(d) 0.83

03. What is the value of the view factor for two


inclined flat plates having common edge of
equal width, and with an angle of 20
degrees? (ME-GATE-02)
(a) 0.83 (b) 1.17
(c) 0.66 (d) 1.34

\( I I 111..'.;llll l 111!� P11\1l1t ,Ille 111-. �yderabad!Delhi!Bhopal!Pune1Bhubancswarl LucknowlPatnalBenp)wulChennailVtiayawadalViz.ag !Tirupari I Kukatpallyl Kolkata I
ACE
:'EFn�.,,.�· -:: :�:m:·:,========:=:=�3:9�8,:,:===========�H
��- �eat�T�ran
�s:fe�r
� "= �·�Po�b�ili-,_
rin

04. A solid cylinder (surface 2) is located at the form a two surface enclosure and steady state
centre of a hollow sphere (surface!). The condition to exist. Stephen Boltzmannn constant
diameter of the sphere is 1 m, while the as 5.67 x 10-8 W/m2K4
cylinder has a diameter and length of 0.5 m
each. The radiation configuration factor F 11 06. The irradiation (in kW/m2) for the upper
IS. (ME-GATE-OS) plate is (ME-GATE-09)
(a) 0.375 (b) 0.625 (a) 2.5 (b) 3.6
(c) 0.75 (d) 1 (c) 17.0 (d) 19.5

05. A hollow enclosure is formed between two 07. If plate is also a diffuse gray surface with an
infinitely concentric cylinders of radii l m emisivity value of 0.8, the net radiant heat
and 2m respectively. Radiative heat exchange (in kW/m2) between plate 1 and
exchange takes place between the inner plate 2 (ME-GATE-09)
surface of the larger cylinder (surface -2) (a) 17.0 (b) 19.5
and the outer surface of the smaller cylinder (c) 23.0 (d) 31.7
(surface-I) the radiating surfaces are diffuse
and the medium in the enclosure IS non­ 08. Consider two infinitely long thin concentric
participating. The fraction of the thermal tubes of circular cross section as shown in
radiation leaving the larger surface and the figure. If D 1 and D2 are the diameters of
striking itself is (ME-GATE-OS) the inner and outer tubes respectively, then
(a) 0.25 (b) 0.5 the view factor F22 is given by
(c) 0.75 (d) 1 (ME&PI-GATE-12)

Common Data for Question 06 & 07


Radiative heat transfer is intended between the
inner surfaces of two very large isothermal
parallel metal plates. While the upper plate
(b) Zero
(designated as plate 1) is a black surface and is
the warmer one being maintained at 727°C, the
lower plate (plate2) is a diffuse and gray surface (d)l-(�:J
with an emisivity of 0.7 and is kept at 227°C.
Assume that the surfaces are sufficiently large to

\Cl } 11g111t { I Ill� P11lil1t ,1111 lib �yderabad I Delhi I Bhopal I Ptmc I Bhubancswar I wcknow I Patna I Bcngaluru I Chcnnai I Vtjayawada I Vmg I TirUpati I Kukatpally I Kolkata I
: 399: Radiation

09. Two large diffuse gray parallel plates, 12. Two infinite parallel plates are placed at a
separated by a small distance, have surface certain distance apart. An infinite radiation
temperatures of 400 K and 300 K. If the shield is inserted between the plates without
emissivities of the surfaces are 0.8 and the touching any of them to reduce heat
Stefan-Boltzmann constant is 5.67 x10-8 exchange between the plates. Assume that
W/m2K4, the net radiation heat exchange rate the emissivities of plates and radiation shield
in kW/m2 between the two plates is are equal. The ratio of the net heat exchange
(GATE-ME-13) between the plates with and without the
(a) 0.66 (b) 0.79 shield is (GATE-ME-14-SET-4)
(c) 0.99 (d) 3.96 (a) 1/2 (b) 1/3 (c) 1/4 (d) 1/8

10. A hemispherical furnace of 1 m radius has 13. The total emissive power of a surface is 500
the inner surface (emissivity, E =1) of its W/m2 at a temperature T 1 and 1200 W/m2 at
roof maintained at 800K, while its floor a temperature T2, where the temperature are
(E=0.5) is kept at 600K. Stefan-Boltzmannn in Kelvin. Assuming the emissivity of the
constant is5.668xl 0- W/m .K
8 2 4
. The net surface to be constant, the ratio of the
radiative heat transfer (in kW) from the roof
temperatures -
7; 1s (GATE-15-Set 2)
to the floor is_ (GATE-ME-14-SET-2) T2
(a) 0.308 (b) 0.416
11. A solid sphere of radius r 1 = 20 mm is placed (c) 0.803 (d) 0.874
concentrically inside a hollow sphere of
radius r2 = 30 mm as shown in the figure. 14. A solid sphere 1 of radius 'r' is placed inside
a hollow, closed hemispherical surface 2 of
radius '4r'. The shape factor F2_ 1 is ...
(GATE-15-Set 3)

The view factor F 2 1 for radiation heat


transfer is
(a) 2/3
(GATE-ME-14-SET-3)
(b) 4/9
G2)
(c) 8/27 (d) 9/4
8r

(a) 1/12 (b) 1/2 (c) 2 (d) 12

�ydcrabadi Delhi !Bhopal I Punc I Bhubancswari Lucknow I Patna I Bcngalw,i I ChennailVtjayawadalVizag ITIIUpati I K.ukatpallyi Kolkala I
"" . ACE
. .. :400:
"

��-� ..
:� Publiacima Heat Transfer

15. An infinitely long furnace of 0.5m x 0.4m F12 = 0.5, T1= 800 K, T2 = 600 K, Tsurrounding
cross - section is shown in the figure below. = 300 K and Stefan Boltzmann constant,
Consider all surfaces of the furnace to be cr = 5.67 x 10-8 W/(m2K4), the heat transfer
black. The top and bottom walls are rate from surface 2 to the surrounding
maintained at temperature T1 = T 3 = 927 °C
environment is ____ kW.
while the side walls are at temperature T2 =
A
T4 = 527°C. The view factor, F1-2 is 0.26.
The net radiation heat loss or gain on side 1
1s W/m. Stefan-Boltzman constant =
5.67 x10- W/m2-K4
8 Surrounding

Side 3, T3

Surface 1
Side 4, T4 Side 2, T2 0.4 m
(GATE- 17- SET-1)
Side l. T,
1� 0.5m �1
(GATE- 16- SET- 1)
Five Marks Questions
16. Two large parallel plates having a gap of 10
mm in between them are maintained at
O1. Two black plates, each one meter square, are
temperatures T1 = 1000 K and T 2 = 400 K.
placed parallel to each other in such a way
Given emissivity values, E1 = 0.5, E2 = 0.25 that the radiation shape factor for the system
and Stefan-Boltzmann constant cr = 5.67 x is 0.4. If the plates are maintained at 800°C
10-8 W/m2-K, the heat transfer between the and 400°C respectively, determine the net
plates (in kW/m2) is ____ radiant heat transfer between the plates. Also
(GATE- 16- SET- 3) calculate the net heat exchange if the plates
were infinite in size. Stephen Boltzmann
17. Two black surfaces, AB and BC, of lengths constant= 5.67 x10-8 W/m2K4
5 m and 6 m, respectively, are oriented as (ME-GATE-89)
shown. Both surfaces extend infinitely into
the third dimension. Given that view factor
!M11ii/Yjjji4.jjQ0iblii\Uih� yderabad I Dcllii I Bhopal I Pune I Bhubaneswar I Lucknow I Patna I Bcngaluru I Chcnnai I Vrjayawada I V1Zag I Tirupati I Kukatpally I Kolkata
'!.�-·
" "
1-��
. P11hlic:aliooa
. ACE . . : 401: Radiation

02. An object has the shape of cubical box of absorptivity of 0.9 at solar wavelength and
side 10cm, with no top cover. The box is an emissivity of 0.1 at the long wavelength.
placed inside a room whose dimensions are Neglecting any heat loss from the lower
much larger than those of the box. All the surface, determine the incident solar
five surfaces of box are at a temp of 500 C °
radiation intensity in kW/m , if the measured2

and have an emissivity of 0.6. The walls of equilibrium temperature of the plate is 50 °C.
the room are at 25°C and have an emissivity Stephen Boltzmannn constant is
of 0.4. All these surfaces can be assumed to 5.67x10- W/m K
8 2 4
(ME-GATE-00)
be diffuse-gray. Find the net radiative heat
loss from the inner surface of the box to the
walls of the room. Stephen Boltzmann
constant 5.67 x 10-8 W/m2K4. View factor
between two parallel square plates placed
directly opposite to each other is 0.2
(ME-GATE-91)

03. Consider two large parallel plates, one at


T1=727 °C with emissivity s1 = 0.8 and the
other at T2 = 227 °C with emissivity £2 = 0.4.
An aluminum radiation shield with an
emissivity, s s=0.05 on both sides is placed
between the plates. Calculate the percentage
reduction in heat transfer rate between the
two plates as a result of the shield.
Use cr = 5.67 x 10-8 W/(m2K4)
(ME-GATE-95)

04. A thin metal plate is exposed to solar


radiation. The air and the surroundings are at
30 °C. The heat transfer coefficient by free
convection from the upper surface of the
plate is 17.4 W/m2K. The plate has an

!fi1i4jj§jjj4iiOIRdft•@hiiUffydcrabad I Delhi I Bhopal I Punc I Bhubancswarl l.AJcknowl Patnaj Bcngaluru I Chcnnai I Vtjayawadaj Vizag I Tirupati I Kukatpally I Kol.kata I
SOLUTIONS
F11 = 0 ( ·: surface 1 is convex )
One Mark Solutions :. F12 = 1
From Reciprocity theorem
01. Ans: (d)
A1F'i 2 = A 2F2 1
Sol: Reflectivity p 0.08
F21 _ A1F'i 2 _ mi1L X l-�-_!_
= =

Transitivity = t = 0.86 -
A2
-
ml 2L - d 2 - 3
Absorptivity = a = ?
a + t + p=l
a = 1-(t + p)
= 1 - ( 0.08 + 0.86) = 0. 06

02. Ans: (a)


Sol: F 21 + F 22 = 1
1 2
= 1- F21 = 1-- = -
3 3
� F22

2
:. F11 = 0 & F22 =-
3
Diffuse body
04. Ans: (b)
Sol: Total Area of the plate
Ap = 2 x 1 0cm 2
= 2 0 X 1 0---4 m 2

Specular body Area of the room AR = 1 00 m 2


Plate temperature TP = 800 K ,
03.
Room temperature TR = 3 00 K
Sol: Given �= 3 (·: d 2 > d 1 ) S p = 0.6 , SR = 0.3
di
F pp = 0,

!Ml•MHYhiiiih4RbbiMhiii+iyderabad I Delhi I Bhopal I Pune I Bhubancswar I Lucknow I Patna I Bengaluru I Chennai I Vuayawada I Vu:ag ITirupati I Kukarpa))y I Kolkata I
: 403: Radiation

07. Ans: (d)


Sol:

Q = --5- .6_ 74 _ _ x l_ 0-
_ _ (�80_0 _ - _ _0�)__
_ 30
8 4 4

1- 0.6
----- 1 - 1 -0.3
+-----+ --
0.6 X 20 X 10- 20 X 10- X 1 0.3 X 100
4 4

Q = 27.32 W F1 1 = O,
F12 =1
05. Ans: (d) A1 2nRiL - Ri
F21 -
- X F12 -
- X1-
Sol: According to Wein's displacement law A2 21tR 0 L R0
Amax T = 2898
1
=> Toc--
A max
From the above as the temperature is 08. Ans: (d)
increasing, A should reduce i.e the max peak
Sol: Eoc T4
has to shift towards left but it is not
T 2 = 2T1
happening in the given figure, hence it is
E1 P
violating Wein's displacement law .
=

EI TI4
E2 T24

Wave length (nm) � E 2 = l6E1


Fig: Wein's displacement curve E2 = 16P

06. Ans: (c)


Sol: For opaque surface, Transitivity 't' = 0
:. a + p = 1

jM11ih§jjj4ii04AflGiii!lf¥ij>Hydcrabad Delhi
I I Bhopal I Punc I Bhubaneswarl Lucknow I Patna! Benga)uru I Chennai I VuayawadaJVu.ag I Tuupati I Kukatpally I Kolkata I
W 2 = 2W 2 - 2W 2 cos0
Two Marks Solutions W3 = 2W sin(0 / 2)
F32 = F31
F32 + F31 = 1
01. Ans: (b)
F32 = F31 = 0.5
Sol: A = 1.0 m 2,
F 13+ F 12 = l
t 1 = t 2 = 0.9 A3F32
+ rD12 -
-1
Tl = 400K, T 2 = 300K, (jb = 5.67 X 10-8 A1
Q_ cr(Tt-T;) A3 F3 2
==> R1 2= 1 -
A _!__ _ 1 -l AI
+
t1 t 2
=1-sin� l-sinl0=0.83
1 X 5.67 X 10-8 (400 4 - 300 4 ) 2
=

Q 12net =
1 1
- + --1 04 . Ans: (b)
0.9 0.9
Sol: D = lm, d = / = 0.5 F 22 = 0
= 812 W/m 2
( ·.· Inner cylinder has convex and flat surface)
02. Ans: (d) :. F 21 + F 22 = 1 ==> F 21 = 1
Sol: Given, F., 3= 0.17
Now F 11 + F 12 = 1
F.12 -?
- •

F., 1 = F33 = 0 ( ·: surfaces 1 & 3 are Flat )

F12 = 1-(0.17)= 0.83


.2

t}
03. Ans: (a), Front view
D

Sol: Given 0 = 20 ° ==> 0/2 = 10


side view

A 1 = 41tR 2 = 41t x 0.52 = 3.14 m 2


A2 = mlL + 2m2
A2 = 1t x 0.5 x 0.5 + 2 x 7t x 0.25 2 = 1.178
From reciprocity theorem A 1 F 12 = A2 F 21
A2 l .l 78
Fi 2 = x F 21 = = 0.375
A1 3.14
F 11 = 1 - F1 2 = 1 - 0.375 = 0.625

!Mllih9iiiiiiil4idttifMUll!i>8YdCrabadjDclhijBhopaljPunelBhubancswarl LucknowjPmiajBcngalurujO.cnnailVuayawadajVizag IT'=pati I Kukatpallyj Kolkata I


: 405 : Radiation

05. Ans: (b) a + p + 1: = l


Sol: r 1 = l m, r2 = 2m a + p = 1 (assuming 1: = 0)
F22 = ?
According to kirchoff' s law
F11 + F12 = 1 2

F11 = O
F12 = 1 s+p= l

A1 F12 = A2 F21 p= (1 - s)
A dl Equation (1) we get
F2 1 = 1 = n i = � = !!._ = _!_ = 0.5
A 2 1t d i £ d 2 r2 2 = (1 - E2) CJ TI 4 + E2 CJ T2
4

F21 + F22 = 1 = (1 - 0.7) X 5.67 X 10-8 X 10004 + 0.7 X


F22 = 1 - 0.5 = 0.5
5.67 X 10-8 X 5004
= 19.49 kW/m2 ::::! 19.5 kW/m2
06. Ans: (d)
Sol: E 1 = 1,
07. Ans: (d)
E2 = 0.7, T2 = 227°C = 500 K
Sol: If E 1 = 0.8
cr = 5.67 x 10-s W / m 2 K
Q net cr (T14 T; )
Irradiation for Body 1 : = � ; = 31.7 �
- + - -1
t1 t2
Reflected energy of plate
( 1 ) by plate (2)
08. Ans: (d)
Sol: F1 1 + F12 = 1
Energy emitted F 1 1 = 0 => F 12 = 1
by plate (2)
A1F12 = A2F21
(2) =
Al DI
F21 - x F1 2 = -
A2 D2
Irradiation for upper plate = incident
F21 + F22 = 1
radiation on upper plate
= Reflected energy of plate (1) by plate
(2) + energy emitted by plate (2)
= 2 (j T 1 4 + E2 (j T/ ---------- ( 1 )
P

\( I l IH�llll l 11111.!, P11hlh ,\[J(Jf\', yderabad I Delhi I Bhopal I Punc I Bhubaneswar I Lucknow I Patna! Bcngaluru I Oiennai I Vtjayawada I Vi.zag I Ttrupati I KukatpaJJy I Kolkata
" . ACE
"':. . . . :406 : Heat Transfer
,.,��PnblirJnms
09. Ans : (a) 11. Ans: (b)
Sol: Given , T 1 = 400 K , T 2 = 300 K Sol: r 1 = 20mm, r2 = 30 mm
E 1 = f:2 = 0.8 F1 1 + F12 =1 R
2
� 1
cr b (T14 - Ti ) F 1 1 = O ; F 12 = 1
QIA = 1 0 1
-1 1
+ -- 1 A1F 12 = A2 F21
E1 E2
2 4/9 5/9
A1
5.67 x l 0-8 ((400)4 - (3oo)4 ) => F21 = - x F1 2
= A2
-1 1
+ -- 1
0.8 0.8
= 0.66 15 kW/m2
4
F21 = -
10. Ans: 25.0156 kW 9
Sol:
12. Ans: (a)

Sol: (o) with out shield


1
x -- = Q shield
n+1

1 1
Q shield
-----'-'=- = -- - -
n +l 2
Q with out shield
2 0
A1 = 1t r , F11 =

A2 = 21t r2 , F 12 = 1 13. Ans: (c)


E 1 = 0.5, Sol: E b1 = 500 W I m 2 at T1

E b2 = 1200 W / m 2 at T2
Q= cr[T - T; ]
4

-
1__ E 1 - 1- 1-- - E2 => IL = � 500
1

= = 0.803
Ehl Tl:
+ + T2 T2 1200
e 1 A 1 A 1 F1 2 E 2 A 2 E b2

Q = 5.688 X 10- [800 4 - 600 4 ] 15926.4 X 7t


8
=
X
0.5
- - 1 2 14. Ans: (a)
- - + -+ O
0.5 x 7t 7t X 1 Sol: F11 = 0
= 250 15.6 W F12 = 1
= 25.0 156 kW F21 + F22 = 1
A1 F1 2 = A 2F21
\( l I 11�111t t 1 111..!. P11 l i l u .1t11 )1 i-. �yder.wad j Dclhi j Bhopal j Punc ! BhubanCSW3:I l..ucknowl Patna l BcngaJuru l Chennai l Vliayawada lVIZ-'!l! �T��- 1 �I Kolkata I
: 407 : Radiation

_A _ 4m 2 17. Ans: 13.797


F2 1 - 1 -
A 2 2n(4r) 2 + n(4r) 2 Sol: Technically question is not correct because
4m 2 length given in third dimension is infinite
= -----
32m + 16m
2 2
but we have to assume unit width into the
1 plane of the paper, then only we can get
F2 1 = -
12 heat transfer rate. They should ask heat flux
between surface 2 and surrounding.
15. Ans: 24530.688 (range 24528 to 24532)
A2 = 5 x 1 = 5 m 2
Sol: ,-----------,
Side 3, T3
.--
\
\

\
Surrounding
Side 4, T4
\
\
Side 2, T2 0.4 m \
\

B Surface 1 C
Side 1. T 1 '
0.5 m A1F1 2 = A2F21

T 1 = 927 ° C = 1200K, A
F21 = -1 x F12
A2
T2 = 527 ° C = 800K
F 1 2 = F 1 4 = 0.26 6
F2 1 = - x 0.5
F 1 1 + F 1 2 + F13 + F 1 4 = 1 5
F13 = 0.48 F21 = 0.6
Q = Q12 + Q13 + Q1 4 F21+F22+F23 = 1 [·: F22 = 0]
Q13 = 0 since the temperatures are same F23 = l-F21
Q = Q1 2 + Q14 = 2x crb xA x F 12( T14 - T: ) F23 = 1-0.6=0.4
Q = 2 x5.67 x I0- x (0.5 x l ) x 0.26 x(1200 -800 )
8 4 4
Q23 = A2 F 23 cr (T/ - T /)
= 24530.688 W
=5 X 0.4 X Cf (6004-3004)
= 13.797 kW
16. Ans: 11.05 (range 10.9 to 11.2)
cr(T14 - T:) 5 .67 X1o-s (1000 4 - 400 4 )
Sol: Q = =
1 1 1 1
- + --1 - + ---1
E1 E2 0.5 0.25
= 11.05 kW/m2

\( } l.11�J 1 l t t 1111g P1dd1, ,IIJll!l .... Fydmbadj Dclhi l Bhopal j Punc ! Bhubaneswarl Lucknow l Patna ! Bcnga)uru j Oiennai l VuayawadalVtzag ITuupati I Kukatpallyj Kolkala I
: 408 : Heat Transfer

Five Marks Solutions Top surface


Radiation
from top open
01.
3 4 1
surface to
Sol: A1 = A2 =lm 2, c 1 = c 2 = 1 �-----
room
,
I

(black plates) ,, 2
F 1 2 = F 2 1 = 0.4
T1 = 800+273 = 1073 K
T 2 = 400 +273 = 673 K cr b = 5.67 xl 0-8 W / m 2 K 4
F1-1 = O
F1-3 = 0.2, F 2-R = 0.2
F1-1 + F1- 2 + F1_3 + F1-4 + F1-s + F 1-R= 1
5.67 xl0-8 (10734 - 6734 ) F1 - 2 = f1 -4= F 1-s
=
1 4F1-R = 1-0.2
l x0.4
08
= 25410 W = 25.4 kW/m 2 F1-R = · = 0.2
4
If the plates are infinite in size F12= F21=1
Similarly, F 2-R = FJ-R = F4-R= Fs-R = 0.2
A crb (T/-T/ )
Q 12 net = ,
1 1
- + - -1 1- E
c, C2 __I

l x5.67 x10-8 (10734-6734 ) e1 A 1


=
1 Q1-R = Q 2-R = QJ-R = Q4-R = Qs-R (R = Room)
= 63527 = 63.53 kW cr(T,4 - T:)
Q l-R=
-c-' -1- -
1- 1-c R
+ + -
02. c1A1 Ai f, -R c R AR
Sol: Inner surface of cubical box = Surface 1, 2, cr A, (Tt -T: )
Q ,_ R =
3, 4, 5 1 1 A, 1-C R
+ + ( )
A1 = (0. l x 0.1) = 0.0 l m 2 0.6 0.2 A R ¾
AR = very large when compared to Ap AR >>> A 1
C p= 0.6, Tp=500 + 273= 773K A
:. - ' �o
C R =0.4, TR = 25 + 273= 298 K AR

\('I l 11:.!,I!!l t J 111� P11lilu.11u,1i-. �ydcrabad l Delhi l BhopaJ I Pune l Bhubaneswarl I..ncknow l Patna l Bcngaluru l Cbennai l Vliay.,.wadal V17.ag ITirupari I Kukalpallyl Kolkat, I
: 409 : Radiation

5.67 X 10-s X A , X (7734 - 2984) s = 0.1 at long wave length


Q1 -R=
-1 1 (at room temperature)
-1+ - + 0
0.6 0 .2
= 34.936 W
Total heat transfer = 5 x Qi -R = 174.68 W

03.
Sol: Given ,
T1 = 727 °C = 1000 K , CJ = 0.8 , (1) = plate , (2) = surrounding
T2 = 227 ° C = 500 K ' S2 = 0.4, F1 -2 = 1
ts = 0.05 TP = 50 ° C = 50 + 273 = 323K
Q-Q cr b = 5.67 xl0-8 W / m 2 K 4 ,
% Reduction in Heat Transfer = '
Q
Qa = 0.9Q
cr (T, ; T2 ) 4 4
= HT by convection+ HT by radiation
Q = H.T. without shields = ;
- + --1 0.9Q = h 6T + \
4
c/T - T24 )
s, t 2 1-s' + -- 1 + -- 1-s2
I

--
s, A, Aif,_ 2 s2 A2
crA, (T,4 - T;)
0.9Q = l 7.4x(50-30)+
_!_ _ 1 + ! + _& (_!_-1)
1 1 s, 1 A2 s 2
1 1 1 1 2
- + - -1 - + - + --2
A2 >>> A1
% Reduction
1
:. A , � O
1 -
1 +-
- 1 A2
s, t 2
cr , (T,4-T;
= 0.3636 -0.0239 = 93.4% 0 .9 Q = 17.4 x 20 + t )
0.3636 --1 + 1 + 0
s,
04.
0.9 Q = 17.4 x 20 + CJ cr A1 (3244- 3034)
Sol: T00 = 30 ° C = 30 + 273 = 303K
= 17.4 x 20 + 0.1 x 5.67x 10-8x 1 x (3244 - 3034)
h v = 1 7.4 W / m 2 K Q = 0.402 kW/m 2
a = 0.9 at solar wave length

�yderabad l Delhi j Bhopail Pune l Bhubaneswar l Lucknowl Patna l BengaJuru l Cbennai l VliayawadalV,zag ITuupali I Kublpal)yl Kolkata I
cs Heat Exchangers
(a) Greater for parallel flow heat exchanger
One Mark Questions than the counter flow heat exchanger
(b) Greater for counter flow heat exchanger
0 1. In shell and tube heat exchanger , baffies are than the parallel flow heat exchanger
mainly used to (ME-GATE-91) (c) Same for both parallel and counter flow
(a) Increase the mixing of fluid heat exchangers.
(b) Increase the heat transfer area (d) Depending on the properties of fluid .
(c) deflect the flow in desired direction
(d) Reduce fouling ofthe tube surface. 04. In a condenser of a power plant , the steam
condenses at a temperature of 60 ° C. The
02. The practice to use steam on the shell side cooling water enters at 30 °C and leaves at
and water on the tube side in condensers of 45°C. The logarithmic mean temperature
steam power plant is because difference (LMTD) of the condenser is
(ME-GATE-94) (ME-GATE-1 1)
(a) to increase overall HT coefficient, water (a) 16.2 C
°
(b) 21.6 ° C
side velocity can be increased ifwater is (c) 30 °C (d) 37.5°C
on the tube side
(b) condenser can act as a storage unit for 05. In a heat exchanger , it is observed that
condensed steam LiT 1 = LiT2 , where LiT 1 is the temperature
(c) rate of condensation of steam is
difference between the two single phase
invariably smaller than the mass flow
fluid streams at one end and LiT2 is the
rate of cooling water
temperature difference at the other end. This
(d) it is easier to maintain vacuum on the
heat exchanger is (GATE-ME-14-SET-2)
shell side than on the tube side
(a) a condenser
(b) an evaporator
03. For the same inlet and exit temps of the hot
(c) a counter flow heat exchanger
and cold fluids, the Log mean temperature
(d) a parallel flow heat exchanger
difference (LMTD) is
(ME-GATE-02)
\( l l 11g111t l I lll� P11li!H ,l!JuJJ" �yderabad l Delhi l BhopaI IPunel Bhubaneswar l I..ucknow l Pama l Bengaiuru l Chennai l Vuayawada l Vu.ag ITrrupati I �I Kolkalal
"...
.r ....,,"'
� :�Nwatvn
. .
ACE . . : 41 1 : Heat Exchangers

06. Saturated vapor is condensed to saturated


liquid in a condenser. The heat capac ity ratio
Two Marks Questions
c .
is cr = = .The effectiveness (s) of the
cmax

condenser is. (GATE-15 -Set 3) 01. In certain HE, both the fluids have identical
mass flow rate-specific heat product. The hot
1 -exp [- NTU(I + Cr )] fluid enters at 76°C and leaves at 47°C, and
(a)
l + Cr the cold fluid entering at 28°C leave at 55°C.
1-exp[- NTU(I - Cr )] the effectiveness ofthe HE is
(b)
1- cr exp[- NTU(I - Cr )] (ME-GATE-97)
NTU (a) 0.16 (b) 0.58
(c) (c) 0.72 (d) 1.0
I+NTU
(d) 1-exp(- NTU)
02. Air enters a counter flow HE at 70 °C and
leaves at 40°C. Water enters at 30 °C and
07. For a heat exchanger, �T max is the maximum
leaves at 50°C. the LMTD in deg C is
temperature difference and �Tmin is the
(ME-GATE-00)
minimum temperature difference between
(a) 5.65 (b) 14.43
the two fluids. LMTD is the log mean
(c) 19.52 (d) 20.17
temperature difference. C min and C max are the
minimum and the maximum heat capacity
03. In a counter flow heat exchanger, for the hot
rates. The maximum possible heat transfer
fluid the heat capacity=2 kJ/kg K, mass flow
(Qmax) between the two fluids is
rate=5kg/s, inlet temperature=l 50° C, outlet
(GATE- 16- SET- 3)
temperature = 100 ° C. For the cold fluid,
(a) C minLMTD (b) C min�Tmax
heat capacity = 4 kJ/kg K, mass flow rate=10
(c) C max�Tmax (d) C max�Tmin
Kg I s, inlet temperature = 20 ° C. neglecting
heat transfer to surroundings, the outlet
08. Saturated steam at 100 °C condenses on the temperature of the cold fluid in °c is
outside of a tube. Cold fluid enters the tube (ME-GATE-03)
at 20 ° C and exits at 50 °C. The value of the (a) 7.5 (b) 32.5 (c) 45.5 (d) 70.0
Log Mean Temperature Difference (LMTD)
lS --- °C . (GATE- 17- SET- l)

\( I l JH!,llll l l ll l � P1il1lH ,lllO]l"> �yderabad I Delhi I Bhopal I Pune I BhubaneswarI Lucknow I Patna I Benga)uru I Chennai I Vuayawada I Viz.ag I Tuupati I K�y I Kolkata I
: 412 : Heat Transfer

04. In a cond enser, water enters at 30°C and specific heat at constant pressure of the fluid
flows at the rate l 500Kg/hr. The condensing is twic e that of the cold fluid . The exit
steam is at a temperature of 120°C and temperature of the cold fluid is
cooling water lea ves the cond enser at 80°C . (ME-GATE-08)
Specific heat of water is 4.187KJ/KgK. If (a) 40 C
°
(b) 60 C
°

the o verall heat transfer coefficient 1s (c) 80 C


°
(d) can not be determined
2000W/m2K, the heat transfer area is
(ME-GATE-04) 08.In a parallel flow heat exchanger operating
(a) 0.707m 2
(b) 7.07m2 under steady state, the heat capacity rates of
(c) 70.7m2 (d) 141.4m2 the hot and cold fluids are equal. The hot
fluid flowing at l Kg/sec with sp. heat 4
05. Hot oil is cooled from 80° to 50 °C in an oil kJ/kgK, enters the heat exchanger at 102°C
cooler which uses air as the coolant. The air while the cold fluid has an inlet temp of
temperature rises from 30 ° to 40 ° C . The l 5°C . the OHTC of the heat exchanger is
d esigner uses a LMTD value of 26 °C . The estimated to be 1 kW/m2K and the
type of heat exchanger is (ME-GATE-05) corresponding heat transfer surfac e area is
(a) parallel flow (b) double pipe 5m2 • neglecting heat transfer between the
(c) counter flow (d) cross flow heat exchanger and the ambient.
The heat exchanger is characterized by the
06. In a counter flow heat exchanger , hot fluid following relation 2e = 1 - Exp ( - 2NTU )
enters at 65°C and cold fluid lea ves at 30 °C . The exit temp ( in °C ) for the cold fluid is
mass flow rate o f the hot fluid is 1 kg/s and (ME-GATE-09)
that of cold fluid is2 kg/s. specific heat of the (a) 45 (b) 55 (c) 65 (d) 75
hot fluid is 10 kJ/kgK and that of cold fluid
is 5 kJ/kgK.The LMTD for the heat 09. An industrial gas (Cp=lkJ/kgK) enters a
exchanger is (ME-GATE-07) parallel flow heat exchanger at 250°C with a
(a) 15 (b) 30 (c) 35 (d) 45 flow rate of 2 kg/s to heat a water stream.
The water stream (Cp=4kJ/kgK) enters the
07. The LMTD of a counter flow heat exchanger heat exchanger at 50 °C with a flow rate of 1
is 20°C . the cold fluid enters at 20°C and the kg/s. the heat exchanger has an effectiveness
hot fluid enters at 100°C . mass flow rate of of 0.75. the gas stream exit temperature will
the cold fluid is twice that of the hot fluid . be (PI-GATGE-10)

�ydcrabad!Delhi!Bhopal!Pune!Bhubaneswarl LucknowlPalna!BengaJurulCbennaiJVtjayawadaJVu.ag J T1111pali I �J Kolkata I


: 413 : Heat Exchangers

(a) 75°C (b) 100°c 12. If qw = 2500x, where x is m and in the


(c) 125°C (d) 150°C direction of flow (x=O at the inlet), the bulk
mean temperature of the water leaving the
10. Cold water flowing at 0.1 kg/s is heated pipe in °C is (GATE-ME-2013)
from 20°C to 70°C in a counter-flow type (a) 42 (b) 62 (c) 74 (d) 104
heat exchanger by a hot water stream
13. If qw = 5000 and the convection heat transfer
flowing at 0.1 kg/s and entering ijt 90 C. The
°
coefficient at the pipe outlet is 1000 W/m2K,
specific heat of water is 4200 J/(kg K) and
the temperature in °C at the inner surface of
density is 1000 kg/m3 • If the overall heat
the pipe at the outlet is (GATE-ME2013)
transfer coefficient U for the heat exchanger
(a) 71 (b) 76 (c) 79 (d) 81
is 2000 W/ (m2 K), the required heat
exchange area (in m2) is 14. In a concentric counter flow heat exchanger,
(PI-GATE-1 1) water flows through the inner tube at 25 °C
(a) 0.052 (b) 0.525 and leaves at 42°C. The engine oil enters at
(c) 0.151 (d) 0.202
100 °C and flows in the annular flow
passage. The exit temperature of the engine
1 1. Water (Cp = 4.18 kJ/kg. K) at 80° C enters a
oil is 50 °C. Mass flow rate of water and the
counter flow heat exchanger with a mass
engine oil are l .5kg/s and 1 kg/s,
flow rate of 0.5kg/s. Air (Cp = l kJ/kg. K)
respectively. The specific heat of water and
enters at 30 °C with a mass flow rate 2.09
oil are 4178 J/kg.K and 2130 J/kg.K,
kg/s. If the effectiveness of the heat
respectively. The effectiveness of this heat
exchanger is 0.8, the LMTD (in °C) is
exchanger is ___
(ME&PI-GATE-12)
(GATE-ME- 14-SET-2)
(a) 40 (b) 20 (c) 10 (d) 5
15. A double-pipe counter-flow heat exchanger
Common Data for Question 12 & 13 transfers heat between two water streams.
Water (specific heat, Cp = 4.18 kJ/kgK) enters a Tube side water at 19 liter/s is heated from
pipe at a rate of 0.01 kg/s and temperature of 10°C to 38°C. Shell side water at 25 liter/s is
20°C. The pipe, of diameter 50 mm and length 3 entering at 46°C. Assume constant properties
m, is subjected to a wall heat flux qw in W/m2 : of water; density is 1000 kg/m3 and specific
heat is 4186 J/kg.K. The LMTD (in °C) is
(GATE-ME-14-SET-3)

er ....,,"
...., �� �
ACE . . . . : 414 : Heat Transfer

1 6. A balanced counter-flow heat exchanger has


a surface area of 20 m2 and overall heat Five Marks Questions
transfer coefficient of 20 W/m2 -K Air.
(Cp = 1000 J/kg-K) entering at 0.4 kg/s and 0 1 . A shell and tube heat exchanger is to be
280 K is to preheated by the air leaving the designed for heating pressurized water by
system at 0.4 kg/s and 300 K. the outlet means of hot gasses which get cooled . the
temperature (in K) of the preheated air is data are as follows (ME-GATE-88)
(GATE -15-Set 2) Temp of water at the inlet = 80°C,
(a) 290 (b) 300 (c) 320 (d) 350 Temp of the water at the outlet = 1 40°C
Temp of hot gasses at the inlet = 340°C
1 7. Consider a parallel-flow heat exchanger with Temp of hot gasses at the outlet = 1 80°C
area Ap and a counter-flow heat exchanger Mass flow rate of water = 1 2 kg/s, Specific
with area Ac. In both the heat exchangers, heat of water = 4.2 kJ/kgK
the hot stream flowing at 1 kg/s cools from OHTC = 30 W/m2K,
80°C to 50°C. For the cold stream in both the Correction factor for LMTD based on
heat exchangers, the flow rate and the inlet counter flow conditions = 0.9
temperature are 2 kg/s and 1 0°C, Calculate the tube surface area required in
respectively. The hot and cold streams in the heat exchanger and the effectiveness of
both the heat exchangers are of the same the heat exchanger.
fluid. Also, both the heat exchangers have
02. A double pipe counter flow heat exchanger
the same overall heat transfer coefficient.
is to be designed to cool 1 2000 kg/hr of an
The ratio Ac/Ap is ____
oil of specific heat 1 .95 kJ/kgK from 85 °C to
(GATE- 16- SET- 2)
55 °C by water entering the heat exchanger at
30°C and leaving at 45 °C. If the OHTC of
1 8. In a counter-flow heat exchanger, water is
heat exchanger is 400 W/m2K. Calculate the
heated at the rate of 1 .5kg/s from 40°C to
LMTD and the surface area of the heat
80°C by an oil entering at 1 20°C and leaving
exchanger (ME-GATE-90)
at 60°C. The specific heats of water and oil
are 4.2kJ/kgK and 2kJ/kgK, respectively. 03 . A one shell pass, one tube-pass heat
The overall heat transfer coefficient is exchanger, has counter flow configuration
400W/m2K. The required heat transfer between the shell side and tube side fluids.
surface are (in m2) is The total number of tubes within the heat
(GATE- 17- SET- 2) exchanger is 1 0 and the tube dimensions are
(a) 0. 104 (b) 0.022 ID = 1 0 mm, OD = 1 2 mm and length = 1 m.
(c) 10.4 (d) 2 1 .84 saturated dry steam enters the shell side at a
!l\l@Ml@jj/i4.jjiNflliM!hi11+1yderahadl Delhi I Bhopal I Pune I Bhubancswarl l.ncknowl Patna! Bengaluru I Chennai I V\iayawada j V,z.ag I TlfUpati I Kukatpa))yj Kolkata I
: 415 : Heat Exchangers

flow rate of 2 kg/s and the temp of 100°C . in remains constant at 2270 W /m 2K. Calculate
the tube side, cold water enters at a flow rate the heat transfer area required, assuming the
of 10 kg/s with an inlet temp of 25°C. the two streams are in parallel flow, and for
OHTC based on the outer surface area of the both the streams C p= 4.2 kJ/kg K
tubes is 50 W/m2K . the specific heat of (ME-GATE-97)
water is 4. 1 8 kJ/kg-K and the latent heat of 07. A hot fluid at 200 ° C enters a heat exchanger
steam is 2500 kJ/kg. What is the condition of at a mass flow rate of 104 kg/hr. Its specific
the steam at the exit. (ME-GATE-91) heat is 2000 J/kg-K. It is to be cooled by
another fluid entering at 25 ° C with a mass
04. Two streams of fluids of unit constant
flow rate 2500 kg/hr and specific heat 400
specific heats and unit mass flow rate
J/kg-K. the overall heat transfer coefficient
exchange thermal energy in an adiabatic heat
based on outside area of 20 m 2 is 250 W /m 2
exchanger. The inlet temps of hot and cold
K. Find the exit temperature of the hot fluid
streams are 300°C and 30 °C respectively.
when the fluids are in parallel flow.
Calculate the LMTD and effectiveness ofthe
(ME-GATE-98)
heat exchanger if the hot fluid is cooled to
zero entropy condition. (ME-GATE-94) 08. Hot water flows with a velocity of 0.1 mis in
a 100mm long, 0.1 m diameter pipe. Heat lost
05. A counter flow heat exchanger is to heat air from the pipe outer wall is uniform and
entering at 400 °C with a flow rate of 6 kg/s equal to 420 W/m2 . If the inlet water
by the exhaust gas entering at 800 ° C with a temperature is 80°C, calculate the water
flow rate of 4 kg/s. the overall heat transfer temp at the exit . Neglect effect of pipe wall
coefficient is 100 W/(m 2K) and the outlet thickness. Cp (water) = 4.2 kJ/kg-K and
temperature of the air is 55 l .5 ° C. Specific density ofwater = 1000 kg/m 3 .
heat at constant pressure for both air and the (ME-GATE-98)
exhaust gas can be taken as 1 100 J/(kgK). 09. Two fluids, A and B exchange heat in a
Calculate the heat transfer area needed and counter-current heat exchanger. Fluid A
the number oftransfer units.
enters at 420 ° C and has a mass flow rate of
(ME-GATE-95)
1 kg/s. Fluid B enters at 20 ° C and also has a
06. In a certain double pipe heat exchanger hot
mass flow rate of 1 kg/s, Effectiveness of
water flows at a rate of 50,000 kg/h and gets
heat exchanger is 75%. Determine the heat
cooled from 95 ° to 65 °C. At the same time
transfer rate and exit temperature of fluid B.
50,000 kg/h of cooling water at 30 °C enters
(Specific heat of fluid A is 1 kJ/kg K and
the heat exchanger . The flow conditions are that offluid B is 4 kJ/kgK). (ME-GATE-99)
such that the overall heat transfer coefficient
FyderabadlDelhilBhopa) I Punc lBhubancswarl LucknowlPatnalBcngaiwu l Cbcnnai lVuayawadalVizag I T"IIUpati I K.ukaq,allyi Kolkata I
SOLUTIONS
For counterflow :
One Mark Solutions Hot fluid 200 7 100
Cold fluid 80 7 20
01. Ans: (a)
01 = 180, 02 = 80
02 Ans: (d) 120 - 80
Li0 me= 98.65° C
Sol: If steam is supplied on tube side, one single 120
=

Rn ( -)
vacuum pump is not sufficient to maintain 80
vacuum but if the steam is supplied on shell
side single vacuum pump is sufficient 04. Ans: (b)
to maintain the vacuum. Sol: Hot fluid is steam & Cold fluid is cooling
water
03. Ans: (b)
Thi = Tho = 60 °C
Sol: For same inlet and exit temp of hot and cold
fluids in parallel and Counter flow heat Tei = 30°C ,
exchanges are Tc0 = 45°C
= 0counter
0parallel Thi = 60°C
(UALi0 Jp = (UALi0Jc T
01 = Th,Tci
(ALi0JP = (ALi0Jc
A Li0mc
-P = -- To; = JO'C 1
Ac Li0mp
Means that, if Li0 mc > Li0 mp �Ap > Ac 01 = 60 -30 = 30 ,
Eg: Letfor parallelflow: 02 = 60-45 =15
Hot fluid 200 7 100 01-02 30-15
LMTD= = 21.6 o C
Cold fluid 20 7 80 ln(01 / 02 ) 3 0
=
ln
01 = 180, 02 = 20 15

Li0 mp = 180 - 20 =72.82 o C


180
.en( )
' 20

Fyderabad I Delhi I Bhopal I Punc I Bhubancsw.ir I Lucknow I Patna I Bcnpluru I Chcnnai I Vijayawada IV17.ag I Tuupati I KukatpalJy I Kolkata I
: 417 : Heat Exchangers

05.Ans : (c)

I
Sol: If �T 1 �T 2 then temperature distribution
=
Thi

can be as follows: AT1

Tei
______
.__

�T 1 = T h1-T 1 =
c 100-20 =80 °C
�T2 = Thi-Tc2 = 100-50 = 50°C

(Counter flow)
�T1 = �T2 = LMTD
The heat exchanger is only counter flow
Heat exchanger only when C min = C max Two Marks Solutions

06. Ans: (d) 01. Ans : (b)


Sol: For condenser ' C =
0 ' �= -1!!!!!...=0
C . C . Sol: Given, T hi = 76 °C, T ho = 47°C
C max oo Tei = 28°C, Teo= 55°C
::::> Effectiveness = 1 - e-Nru C min= C h = C c

07. Ans: (b)


Sol: The temperature difference is not for a given
fluid but across the fluids and maximum
heat transfer occurs for C min and the
temperature difference 1s equal to
(Th ;-TC; ).

C h (Th ' -Th o ) 76-47


08. Ans: (63.82) E= 0.604
c min (Th ; -TC ; ) 76-28
Sol: Hot fluid:
C (Teo -TC; ) 55-28
T h1 =Th2 = 100°C E= c = =0.5625
c min (T h ; -TC; ) 76-28
Cold fluid
T c1 = 20 °C Chose the answer in between the above
T c2 = 50°C values i.e, 0.58 •
\{ } I 111.!,lllt t I I l l � P11lil1t ,Ill< 111, �ydcrahad l Delhi Bhopal Pune I Bhubancswar l Lucknow Patna Bcngaluru Chcnnai j V,jayawadaj Vmg I Tuupali
I I I I I I Kukalpallyl.Kolkala I
ACE
� : &,
i. 1111
� · �
.,): � ·= :nn�w�Pl�.,�-:'*'�:c.i;:•::========:=::::��:0::===========��:��,::;:;;:_
··� = 41 8 Heat Transfer
:
-
02. Ans: (b) 10 4 (150 -100)= 4 X 10 4 (Tco - 20)
Sol: T hi 70 °C, T ho 40°C
50
Tc = 20 + = 32.5° C
= =

Tei 30 °C, Te0 50 °C


= =
0
4
Th; = 70°C
01 = Th;-TC0 I
04. Ans: (a)
Sol: T ei 30 ° C , Tc0 80 °C
°
00 TC0 = 50 C = =

1500
m e= --=0.417 kg / sec
3600
T hi = 120 °C, CPc 4187 J/kg K
8 1 70 - 50 20
=

U = 2000 W /m 2K
= =

8 2 = 40 -30 = 10 Because the hot fluid is steam and whose


temperature remains constant, the flow is
immaterial

03. Ans: (b)


Sol: Given
Ci>h 2000 J/kgK,
=

Tc; = 30°C
rilh=5kg/s
C pe 4000 JI kgK,
= 8 1 = 120 - 30 = 90, 8 2 = 120 - 80 = 40
rile=10kg/s 90 - 40
�0 = = 61.66 o C
ch rilh X C Ph = 5x2000 = 10 J/K
=
4
mp
Rn( :� )
Cc rile x C Pc =10x4000 = 4x104 J/K
Heat transfer = ril c C P c (Teo - TC i ) = UM0 m
=

T hi= 150 C,
0.417 x4187(80-30)
°

T h0 100 °C , =:> A= 0.707 m 2


2000 x61.66
=
=

T ei 20 °C,
=

05. Ans: (d)


Energy balance, Sol: Given T hi = 80 °C , T h0 50°C
Heat lost by hot fluid = Heat gain by cold
=

Tei 30 °C, T e0 = 40 °C
fluid
=

If the given heat exchanger is assumed as


counter flow
\( 1 . 1 .l l�ll l(C.:l lllg P\Jlil H ,ll]IJI\'\ �ydcrabad I Delhi I Bhopal I Punc I Bhubaneswarl Lucknow I Patna I Benga)uru I Chennai I Vtjayawada IVazag !Tirupati I Kukalpally I Kolkata I
: 419 : Heat Exchangers

06. Ans: (c)

01 =
Thi = 80°C
Thi-Tc0
Tc0 40°C
I Sol: T hi = 65 °C
ril h = 1 kg/s ,
Tc 0 = 30 °C
ril e =2 kg/s
Cph = 104 J/kgK, Cpc = 5000 J/kgK
=

01 = 80-40 = 40 ° c ,
02 = 50 - 30 = 20 ° c
0 1-0 2 40 - 20
( LMTD)parallel 28.85 C
0
J 40
0

(Counter flow)
= = =

ln (_1 ln -
02 20
If the given H.E is parallel fluid C h = 1X10 4 = 10 4
Ch Cc
}

Cc = 2x5000 = 10 4
=

65 � Th2
30 +- Tei
When C h = Cc, the temp distribution lines are
straight lines and in case of counter flow H.E
01 =0
2
= �0 m = LMTD
01 = 80-30 = 50 ° C , :. LMTD = 65 - 30 = 35 °C
0 2 = 50 - 40 = 10 ° C
0 1 -0 2 50 -10 07. Ans: (c)
( LMTD)counter = 24.5 C

J 50 Sol: LMTD = 20 � Counter flow


0
= =

ln (� ln -
02 10 Tei = 20 ° C, T hi = 100 ° C
But designer chosen LMTD is 26 which is Cp h = 2 Cp c
greater than parallel flow LMTD and less
than counter flow LMTD; Hence it is taken
as cross flow H.E :. Temperature distribution is straight line
01 = 0 2 = LMTD=20 (given)

!lfllin§jjj4iiOQRflliMlb1hdj yderabad I Delhi I Bhopal I Pune I Bhubaneswar l Lucknow! Patna I Bengalwu I Cbennai jVtjayawada j Vu.ag I Tuupati I Kukalpally I Kolkala
: 420 : Heat Transfer

ch = thh x C P h = 2 xlOOO = 2000 = C min


Thi = 100°c
81 = Th,TeoI Cc = th e X Cp c = 1 x4000 = 4000 = C max
Teo = ?°C Too =? °C
82 = Tho-Tei
Tei = 20°c

:. 0 1 = 100-Tc = 20
0

:=>Teo = 80 ° C
And also, :. 0 2 = Th -20 = 20
0

:=>Tho = 40°C

08. Ans: (b)


Sol: ch = C c = 1 x 4000 = 4000,
Thi = 102°C , Te i = 15°C,
U = 1000 W /m 2K , 10. Ans: (b)
A = 5m 2 , parallel flow heat exchanger, Sol: th e = 0. 1 kg/s , Tei = 20°C , Te0 = 70°C
2s = 1- e<-2NTU)
th e=0 . 1 kg/s , Thi = 90°C,
UA 1000 x5 =
NTU = = 1.25 U = 2000 W/m 2K
c min 4000
( 1 C p., = Cpc = 4200 J/kgK, p = 1000 kg/m 3
1-e - 2x . 2s) C c {Tc.-Tei )
s= ---- - ch = C c = 0. 1 x 4200 = 420
2 - C min {Thi -Tei )
4000 1Tc -15) Th; =90°C
0.4589 = \
4000(102-15) 81 = Th,TeoI
0

=> Teo= 40+ 15 = 55°C Teo = 70°C

09. Ans: (b)


Sol: Given parallel flow heat exchanger
C ))h = IOOOJ / kg K, th h=2 kg/s
Energy balance :
Tei = 50° C
Heat lost by hot fluid =heat gained by cold fluid
C pc = 4000 J/kg K, th e = 1 kg/s , s = 0.75 C h (Th,Th.) = C c(T c. -Tc,)

yderabad I Delhi I Bhopal I Pune l Bhubaneswarl w:knowl Pama ! Bengalwu l Oicnnai I Vliayawada!Vizag I Tirupati I Kukalpallyl Ko1kata
: 421 : Heat Exchangers

90- T1i. = 70 -20 D = diameter of pipe = 50 mm,


=> T1i. = 90 - 50 = 40°C Tb exit = 62.27 °C
01 = 90- 70 = 20 ° C , 02 = 40 - 20 = 20 ° C Tb exit = 62 °C
·: 01 = 02
LMTD = 01 = 02 13. And: (d)
Total heat transfer =Ch(Th; -T1i.) = UA �0m Sol: q w x 1t DL = m C p (T2 -T1 )
420(90- 40) 420 x 50
=> A = = = 0.52 5 m2 => 5000x 1t x 50 x 1 0-3 x 3
2000 X 20 2000 X 20
= 0.01 x 4. 1 8 x l 03 (T2 - 20)
1 1 . Ans: (c) T2 = 76° C
Sol: ch = 4. 1 8 x 0.5 = 2.09 , qw = h(J: - TJ
Cc = 2.09 x 1 = 2.09, hence, ch = Cc
=> 5000 = l OOO (Ts - 76)
. C h (Th ' - Th • )
Effectiveness =
C c (Th ' - Tc' ) => Ts = 8 1 ° C

14. Ans: 0.667


Sol:
------��
� ne oil

Wat.;,\�
Te o - 30
3=-
r
0.8 = => Tc ° =70
80- 30
0 1 = 1 0, 0 2 = 1 0
rilw C Pw = 1 .5 x 4 1 78
LMTD = 0 1 = 0 2 = 1 0
m01.• c P.;1 = l x 2 1 30
12. Ans : (b)

0.01 x 4. 1 8 x 1 03 (Tb exit - 20) = J: 2500x x d(1tDX)


1 00 - 50 = -
50 -
0.0 1 4. 1 8 x l 0 3 (Tbexit - 20) => =2
X
1 00- 25 75 3
= J: (n x D x 2500 x)dx e = 0.667

\( J l 1l�l]l( t ! Ill� Ptild11 .1t1,lll" �ydcrabad j Delhi I Bhopal I Pune I Bhubaneswar j, ·Lucknow l Patna! Bengaluru I Otennai j Vgayawada j Vizag ITirupali I Kublji,i'ly I Kolbta I
ACE Heat Transfer
':,
�•"fnpneerql'oNiramna : 422 :
" V • • • •

15. Ans: 11.02 17. Ans: 0.9278 (range 0.91 to 0.95)


Sol: Sol: m h = 1kg / s, me = 2kg / s
46 T ,
h = 80 ° C, T 2
h = 50 ° C

Tc = l 0 ° C, TC2
38 I
-?
- •

Heat lost by hot fluid = Heat gained by cold


fluid
mh cp X (Th , - Th 2 ) = me X cp (TC 2 - Tc , )
Energy_ balance: � 1 X (80 - 50) = 2 X (TC 2 - 1 0)
Heat lost by hot fluid = Heat gain by cold
fluid
m 1 Cp 1 (46 - T) = m2 Cp 2 (38- 1 0)
25 x 10-3 x 10 3 x 41 86(46 - T) = 1 9 x 10-3 x 10 3 x 4 1 86(28)

T = 24.72 ° C
� T 1 = 46 - 38 = 8
Parallel flow Heat exchanger
� T2 = 24.72 - 1 0 = 1 4.72
0° c
0 1 =55 °C {
2s 0c so0c }
02=40°C

16. Ans: (a) 10°c

UAs 20 x 20 = Counter flow Heat exchanger


Sol: NTU = 1
�!(OJ;���-)
=
400
(80 25)
(LMTD),, F = :(1 � , =
cmin T

NTU
C = 1 , effectiveness = ( = 0.5 J
l + NTU ) �T2 50- 25
. (t- 280) 43.7054
Effectiveness = = 0.5 =

(300 - 280)
(LMTD)c. F =
�TI 2 = (80 - 2 - ( - 1 0)
�: Jo ��
T

�t = 290 K -
ln(-J1 In ( --- )
�T2 50- 1 0
= 47. 1026
\C I l 11g11tl l t 111g P1illli( ,1t1cH1..., �yderabad j Delhi j Bhopalj Punel BhubanCllwarl LucknowlPatnal BengalwulOicnnailVuayawadalV17.3g ITirupati I Kukarpallyl Kolkata I
: 423 : Heat Exchangers

(Q) C. F = (Q) P.F


U(A)c. F x(LMTD)c.F= U x (A) P.F x(LMTD) P.F
Five Marks Solutions
(A)c . F
= 43.7054 / 47. 1026 = 0.9278
(A) P.F
0 1.
Sol: Twi= 80°C Two = 1 40°C
18. Ans: (d)
Tg; = 340°C Tg. = 1 80°C
Sol:
fil W = 12 kg/s C Pw = 4.2 kg /kgK
OHTC = 30 W /m 2k; F = 0.9
Assume counter flow heat exchanger
Tw; =340°C
01 = Tw,Tg.,
Tg.,= 140 C°
I
Cold fluid
Tc t = 40°C Tc2 = 80°C
ril e=l .5kg / s Cc = 4.2kJ/kgK
01 = 340 - 140 = 200 °c
Hot fluid
0 2 = 1 80 - 80 = 100 °c
Th i = 1 20°C
200-100 100
LMTD = =1443 C
200 ln 2
0

In
Counter flow
8T1 = Thi - Tc 2 100
8T1 = 120 - 80 = 40 °C (LMTD) act = 0.9(LMTD) = 129.84°C
8T2 = Tb 2 - Tc1 C w = fil w X C Pw
8T2 = 60 - 40 = 20 = 12 x 4.2 x 10 3 = 50.4 kW/K

-::ti) - ;:i)
Heat transfer from cold water
LMTD
( Qw= C w X (Tw0 - Tw;}
= 50.4( 140-80)=3024 kW

LMTD = 28.854°C But Qw = UA 8 0m


Q = Ille C c (Tc - Tc )= UA(LMTD) Q w 3024000
A= 77632m 2
U0 m 30 x1 30
2 1

l .5x4.2 x 1 0 x (80 -40) = 400 xAx 28.85


3

Energy balance :
A = 21.84 m 2
Heat lost by gas = heat gained by water
\( l I 11g1rll t 1 1 11� P1 1 l d 1 1 . l l ! (111, �ydcrabad lDelhilBhopa) I PunclBhubancswarl LucknowlPatnal Benga)urulChennailVuayawadalVizag ITnupati I Kukatpallyl Kolkala I
I<.,..,,:r...q
ti,
'!.
V ACE• •
l'oolir*n
• •
: 424 : Heat Transfer

ffiW cpw (140 -80)= ffi h c ph (340-180) C h = ril h x CPh= 6500


C w x 60 = ch x 1 60 01 = 85 - 45 = 40°c ,
0 2 = 55 - 30 = 25°c
C min = Cb
-0 2 40 - 25
C max = Cw d0m = e l = 31.9 °C
40
C 60 In � In
C= min = =0.375 02 25
cmax 160
Heat transfer ,
4200 x 12 x 60 =
C mm. = 1 8900 Q = C h(Tb;- T11.) = U A d 0m
160
6500 (85- 55) = 400 x A x 3 1.9
UA 30 x 776.32
NTU= = 6500(30)
c min 1 8900 �A= =l5.28m 2
400 x 31.9
NTU = 1.2322
1- e-(1-C}NTU
Effectiveness, (s)= 03.

1-C x e--(i-e)N
Sol: Counter flow,
s- 1 _ e-(1-o.m}<1 .2322 -
- ----��- No. of tubes = 10,
1 _ 0.375 X e--{J-0.375}xl.2322
di = ID = 10 mm,
= 0.6498 = 64.98 %
<lo = OD = l 2 mm, L = l m
E

ril h = 2 kg/s,
02.
Tb; = T11. = 100 °C ,
Sol: C ounter flow H E
ffi c = 1 0 kg/S,
ril h = 1 2000 = 333 kg/s Tc; = 25°C ,
3600
U = 50 W/m 2K
C ph = 1.95 kJ/kgK,
C pc = 4. 1 8 kJ/kgK,
Tb; = 85°C , T11. = 55°C ,
hrg = 2500 kJ/ kg
Tc; = 30 °C , Tc. = 45°C
U = 400 W /m 2K
Tho = I 00°C t-----------1Th; = I 00°C
Th; =85°C
01 = Th,Tc0 I
Tc0 = 45°C

Fydcrabad J DclhiJBhopal J Pune lBhubaneswarl LucknowJ Patna J Bengaluru J ChcnnailVuayawadaJV,zag J T11Upali I Kukaq:i;i)ly J Kolkata I
"•
"
, ...,,," ACE . .
:� PmlirJcm
. . : 425 : Heat Exchangers

A = 1tdo Lxn 2 ( 1-x) hrg = 141.3684


= 1tX0.0 12
X 1 X 10 = 0.03768 m 2 [hrg = latent heat of condensation
C min = C c = 10 X 4 1 80 = 4 1 800 W/K. = 2500 kJ/kg = 2500000 J/kg]

UA 50x 0 .03768 2 ( 1 - X) X 2500000 = 1 4 1.3684 '


NTU = = = 4_5 x 10_5
c min 4 1 800 � x = 99.997 % (final condition of seam)
Very little amount ofsteam condensed .

If C =0 ,the effectiveness is given by 04.


E = 1 -e- NTU =l-e -4.Sxl O-s=4 . 49x10-S
Sol: Given, c h= cc= 1, Th ;= 300, TC ;= 30,
Zero Entropy conditio n means change m
But entropy is minimum.
Minimum entropy occurs m counter flow
� Tc =25.003382° C heat exchanger.
Heat transfer from cold fluid,
0

C = C min/C max = 1
Qc = m c C Pc(Tco - TCi) = 1 4 1 W

x = 99.9972%

s Energy balance,
( 1) = initial condition of steam Heat lost by hot fluid = heat gain by cold
(2) = final condition of steam fluid
For minimum entropy case,
Heat loss by steam = heat received by cold 0 1 = 02 = LMTD
fluid T ee = T he = T
ms (h i-h 2) =m W cpw (Teo-Tei) C h (Thi-The) = Cc (T ce - Tci)
ms (h g - (h r + x(h g-h f )))=lOx41 8(\25.003382-25) 300 - T = T - 30
� T = 1 65°C
2(h 8 - hr - x(h 8 - hc )) = 10 x 4 1 80 (0.003382) Effectiveness,
C h (Thi -The) 300 - 1 65
2 (hrg - x hrg) = 4 1 80 x 0.003382 E= = =0_5
C miJTm - TJ 300-30
\( I J Il�lllt l ! Ill;., P11ld11 ,tlli HI'- �yderabad I DelliiJBhopal I Punc I BhubaneswarJ Luckno..,,JPatnaJBcngaluru I Chennai J VuayawadaJ Vtzag JTlfUpati t·�yJ Kolkata I
Heat Transfer
,, "'..t

{-�� � : 426 :
� �=================================
05. 4400(800 - 572.75)
47_99m 2
Sol: C ounter flow heat exchanger, 100x208.33
Tc; = 400 ° C, m e= 6kg/s, UA 100x47.99=
NTU = 1.09
Th; = 800°C, m h = 4 kg/s, c min 4400
U = I OO W/ m2K
Tc. = 551 .5 °C, 06.
C ph = C p. =l 100 J/kg K 50000
Sol: m h = 1 3.88 kg I s
ch = 1 100 x 4 = 4400 = C min, 3600
C c = 1 100 X 6 = 6600 = C max Th; = 95°C, Th. = 65°C
. 50000
Th; =800 me= 1 3.88kg /s
l 3600
Tc; = 30 °C
Tc0 = 55 I .5°C
81 = Th,Tco U = 2270 W /m 2K,

Th; = 95°C

Energy balance,
Heat lost by hot fluid = heat gain by cold
fluid
C h(Th;-Th.) = C c (Tc.-Tc;)
4400 (800-Th.) = 6600 (55 1 .5 - 400)
T ho = 572.75 Both are water,
0 1 = Th; - Th. = 800 - 551 .5 = 248.5 °C C ph = C p. = 4.2 kJ/ kg-K
02= Tb. - Tc; = 572.75 - 400 = 172.75°C 50000
Ch = C = x4200 = 58.33 x 10 3
3600
L\9m = 0 1-0 2 75.75= 208.33
C

Parallel flow heat exchanger


In � 6.3635
02 Energy balance,
Heat transfer Heat lost by hot fluid = heat gain by cold
Qc = Ch (Th-; Th.) = UA 60m fluid

C h (Th -Th ) C h(Th; - Tb.) = C c(Tc0 Tc;) -

=> A =
Uxl\0 m Tc. = Tc; + (Th; - Th.)
I •

= 30 + (95 - 65) = 60°C

\( I I 111..',1 1 1 t < 1111L: P11lil1l .111t1ri-, �ydcrabad l Delhi l Bhopal i Punc J Bhubaneswar l Incknow l Patna l Bcngaluru l Chcnnai l Vuay.,.wada l Vizag l'I'irupati I Kukatpally J Kollwa I
: 427 : Heat Exchangers

e, = 95 - 3 0 = 65 25 00
C c = -- x4 00 = 277.8 = C mm.
82 = 65 - 6 0 = 5 36 00
1 04
�em = B, -:2 =23.39 Ch = X 2 000 = 5555.6 = C max

ln -1 36 00
02 c.
C= � = 0. 05
Heat transfer from hot fluid is given by cmax
C h (Th - Th ) = UM0 m UA 25 0x2 0 =
NTU = 17.99 � 18
cmin 277.8
1 0
=
5 0000
X 42 00(95 - 65)
=> A = 36 00 =32.96m 2 Effectiveness for parallel flow in heat
227 0x23.39 exchanger is given by
1-exp (- NTU(l + C)) = cc (Tc. -TCJ
07. cp =
1+ C cmin (Th ; -TC; )
Sol: Th ; = 2 00° C
1-eHs c1+ 0.osJJ 277 .8(Tc. - 25 )
-
ril h = 1 04 kg/hr = � = 2.77kg /s 1 + 0. 05 277.8(2 00- 25)
36 00
(T -25)
C P h = 2 000 J / kgK 0952= --"-__
175
co

Tc ;= 25 ° c Tc 0
= 191.67 ° C

25 00 = Energy balance ,
rilC = 0.694kg / s
36 00 Heat lost by hot fluid = heat gain by cold
C Pc = 4 00J / KgK fluid
U = 25 0 W /m 2K, Ch(Tb; - Th.) = C c(T c. - Tc;)
A = 2 0m 2 554 0(2 00 - Th.) = 277.6(191.67 - 25)
Parallel flow heat exchanger Tb. = 191.67°C

08.
Sol: V = O.l mis , d = lOO mm = 0.1 m
Thi = 200°C

::::.,·-•Tho
= Tco L = O.l m q = 42 0W/m 2
= 1 9 1 .67 °C
Ti = 8 0° C ,
Tc; = 25 °C C p = 42 00kJ / kgK
p = 1 000kg/m 3

�ydcrabad I Delhi I Bhopal I Pwic I Bhubancswar I Luclmow I Patna I Bengalwu I Chcnnai I Vliayawada jVizag I TJrupati I Kuka,lpa)ly I Kolkata I
ACE
�".�
= 428 Heat Transfur
� "=
·�·
:� �
· im
= m�·�Pub
��-:
·��:
•===.========��:��:===========::..:�::,:��,:::�

Mass flow rate of water


7t d 2
= -
Ch = l x lOOO = 1000 = Cmin
xVxp= m
4 Cc = 1 X 4000 = 4000 = C max
= 0. 12 X 0.1 X 1000 C (T - T )
E = 0.75 =
7t
4
b h; h.

(Th ; - TC; )
c min
=
m 0. 785 kg/sec
1000(420-Th )
Total Heat transfer from pipe to water 0.75 = 0

= q xA
1 000(420- 20 )
°
= 420 X 1tdL � Tb = 420-300 = 1 20 C
0

= 420 X 1t X 0.1 X 0.1 Heat loss by heat fluid = heat gain by cold fluid
= 1 3. 1 88 W mh cp h (Thi - Tho ) = me c pc (Teo - Tei )
Neglecting other losses 1 X 1 X {420 - 120) = 1 X 4 X {Teo - 20)
Heat transfer from pipe = Heat rejected by Teo = 95°C
water
1 3. 1 88 = mCp (Ti - T0 )

= 1 X 1000 X (420-120)
.9
� T0 = Ti - 0.004 = 79.996 C
= 300 kW

09.
Sol: Counter flow heat transfer exchanger
Fluid A � hot fluid
Th; = 420°C , m h = 1 kg/s
Te; = 20°C , m e = 1 kg/s , s = 0.75 ,
Cph = 1 000 J/kgK CPc = 4000 J/kg

Th; =420°C

\( l I ! J .,, l l l l ( l l l l� Ptildtl ,l\ll>II'- �yderabad I Delhi I Bhopal I Punel Bhubaneswarl Lucknowl l'alna l Bengaluru I Olennai I Vuayawada !Vizag ITirupati I Kukatpallyl Kolkata I
Thermodynamics
(Questions)
Page No. 430

CONTENTS
:' Chapter Questions Solutions
, N ame o f th e Chap te r
No. Page No. Page No.
- - - - - - - - I - - - --

01 Zeroth Law & Basic Concepts 43 1 - 433 434 - 43 6

02 Work & Heat 437 - 439 440 - 442


03 First Law of Thermodynamics 443 - 447 448 - 45 3
04 Second Law of Thermodynamics 454 - 45 7 458 - 461
05 Entropy 462 - 467 468 - 472
06 Property of Pure Substances 473 - 476 477 - 479
07 Availability 480 - 481 482 - 483
08 Air Cycles ., 484 - 49 1 492 - 5 0 1
09 Psychrometry . 502 - 508 509 - 5 1 5
10 Rankine Cycle 516 - 527 528 - 537
11 Gas Turbines 538 - 542 543 - 550
12 Refrigeration 5 5 1 - 558 5 5 9 - 567

13 Thermodynamic Relations & 568 - 568 569 - 569


Reciprocating Compressors
14 Internal Combustion Engines 570 - 5 74 575 - 580
Ct Zeroth Law & Basic Concepts

(b) 1/273 .15th the normal freezing point of


One Mark Questions water
(c) 100 times the difference between the
01. A body of weight 100 N falls freely a triple point of water and the normal
vertical distance of 50m The atmospheric freezing point of water
drag force is 0.5N. For the body, the work (d) l /273.16th of the triple point of water
interaction is (GATE-ME-93)
(a) + 5000 J (b) - 5000 J 04. The specific heats of an ideal gas depend on
(C) - 25 J (d) + 25 J its (GATE-ME-96)
(a) Temperature
02. An insulated rigid vessel contains a mixture (b) Pressure
of fuel and air. The mixture is ignited by a (c) Volume
minute spark. The contents of the vessel (d) Molecular weight and structure
experience. (GATE-ME-93)
(a) Increase in temperature, pressure and 05. The Vander-waals equation of state 1s
energy
( p + :2 ) <v-b) = RT, where p is pressure, v
(b) Decrease in temperature, pressure and
en�rgy is specific volume, T is temperature and R is
(c) Increase in temperature and pressure but characteristic gas constant. The SI unit of a
no change in energy is (GATE -15-Set 2)
(d) Increase in temperature and pressure but (a) J/kg-K (b) m3 /kg
decrease in energy (c) m5 /kg-s2 (d) Pa/kg

03. The definition of lK as per the 06. The internal energy of an ideal gas 1s a
internationally accepted temperature scale is function of (GATE - 16- SET - 2)
(GATE-ME-94) (a) temperature and pressure
th
(a) 11100 the difference between normal (b) volume and pressure
boiling point and normal freezing point (c) entropy and pressure
of water (d) temperature only

\l I I r1 ...,111t t r 111L, P1ilil1l ,111, ,ii.,, Fydcrabad l Delhil Bhopal I Pune I Bhubaneswarl Lucknow I Patna I Bengaluru I Chennai l VuayawadajVizag ITIIUpati I Kukatpallyj Kolkara I

��-�;' .t;Fn�p:;:AJCE
�- ���========�:432
:�•�Pl�:tblicdima
- The�rm
��:===========�� odyn�anu
�� ��c:.s
"=
" V 1 • • 1

.,a;,

07. The molar specific heat at constant volume (a) No heat is transferred
of an ideal gas is equal to 2.5 times the (b) No work is done
universal gas constant (8.314 J/mol.K). (c) No mass flows across the boundary of
When the temperature increases by 100 K, the system
the change in molar specific enthalpy is (d) No chemical reaction takes place
---- J/mol. within the system
(GATE - 17- SET- l)
03. Match items from groups , I , II , III , IV and
V (GATE-ME-06)

Two Marks Questions Group I Group II


When added to the system , is
E Heat G Positive
01. Match List I with List - II F Work H Negative
(GATE-ME-96) Group III Group IV Group V
List- I Differential Function Phenomenon
A. Cetane number I Exact K path M Transient
J Inexact L Point N Boundary
B. Approach and range
T
C. ( 8 ) # 0
8P h (a) F-G-J-K-M (b) E-G-I-K-M
D. dh = Cp dT, even when pressure varies E-G-1-K-N F -H-I-K-N

List- II
(c) F-H-J-L-N (d) E-G-J-K-N
1. Ideal gas
E-H-I-L-M F-H-J-K-N
2. Vander Waals gas
3. S. I. engine
Statement for Linked Answer Q04 & Q05
4. C. I. engine
A football was inflated to a gauge pressure of 1
5. Cooling towers
bar when the ambient temperature was 1 S °C.
6. Heat exchangers
When the game started next day , the air
temperature at the stadium was S °C.
02. An isolated thermodynamic system executes
Assume that the volume of the football remains
a process. Choose the correct statement (s)
constant at 2S00cm3
from the following (GATE-ME-99)

IM•li@hiiih!4RfltilM\iijjj+1ydcrabad I Delhi I Bhopal I Punc I Bhubancswarl LucknowI Patna I Bcngalwu I Chcnnai I Vtjayawada I Vi.zag I Tuupati I Kukatpally I Kolkata I
: 433 : Therm odynamics

04. The amount of heat lost by the air in the 07. Temperature of nitrogen in a vessel of
football and the gauge pressure of air in the volume 2m 3 is 288 K. A U-tube manometer
football at the stadium respectively equal connected to the vessel shows a reading of
(GATE-ME-06) 70cm of mercury (level higher in the end
(a) 30 .6 J, 1.94 bar (b) 21.8 J, 0.93 bar open to atmosphere). The universal gas
(c) 61.1 J, 1.94 bar (d) 43.7 J, 0.93 bar constant is 8314 J/kmol -K, atmospheric
pressure is 1.01325 bar, acceleration due to
05. Gauge pressure of air to which the ball must gravity is 9.81 m /s 2 and density of mercury
have been originally inflated so that it would is 13600 kg/m 3 • The mass of nitrogen (in kg)
equal 1 bar gauge at the stadium is in the vessel is ---
(GATE-ME-06) (GATE-15-Set 1)
(a) 2.23 bar (b) 1.94 bar
(c) 1.07 bar (d) 1.00 bar 08. The volume and temperature of air (assumed
to be an ideal gas) in a closed vessel is 2.87
06. A certain amount of an ideal gas is initially m 3 and 300 K, respectively. The gauge
at a pressure p 1 and temperature T 1. First, it pressure indicated by manometer fitted to
undergoes a constant pressure process 1-2 the wall of the vessel is 0.5 bar . If the gas
such that T 2 =3T 1/4. Then, it undergoes a constant of air is R = 287J/kgK and the
constant volume process 2-3 such that atmospheric pressure is 1 bar, the mass of air
T3 =T 1/2. The ratio of the final volume to (in kg) in the vessel is
the initial volume of the ideal gas is (GATE- 17 - SET - 2)
(GATE-ME-14-SET-3) (a) 1.67 (b) 3.33
(a) 0.2� (b) 0.75 (c) 5.00 (d) 6.66
(c) 1.0 (d) 1.5

�yderabad J Delhi J Bhopal J Pune J BhubaneswarJ LucknowJ PatnaJ Benga!uru J Chennai J VuayawadaJ Vizag J'firupati J Kukatpally J Kolkata I
SOLUTIONS
N m 6 Nm 4
a=- X - = --
One Mark Solutions m 2 kg 2 kg 2
kgm - m4 -m5
01. Ans: (d) = -- x = -
s 2 kg 2 kgs 2
Sol: W = 0.5 x 50 = 25 J

06. Ans: (d)


02. Ans: (c)
Sol: Energy can neither be created nor destroyed.
07. Ans: 2909.9
Here chemical energy is converted to
Sol: Cv = 2.5R
Internal energy. There by increasing
pressure and temperature but energy remains R = 8.3 l 4J/mol.K
constant. dT = l OOK
Cp = Cv+R = 2.5R+R = 3.5R
03. Ans: (d) dh = CpdT = 3.5x8.314x l 00
= 2909.91/mol
04. Ans: (d)

Sol: Cp = yR
M(y -1) Two Marks Solutions
M = Molecular weight
y depends on Atomicity of gases. 01.
Sol: A - 4 B -5 C -2 D- 1
05. Ans: (c)

Sol: P = � 02. Ans: (a), (b), (c)


v2

m' (;;)'
N a
= ---
03 . Ans: (d)

04. Ans: (d)

!11i•i!@hiiii04Rftffii!jjih*)ttyderabad I Delhi I Bhopal I PuneI Bhubaneswarl wcknow I Patna I Bengaluru I Chennai I VliayawadalViz.ag IT11Upari I Kukalpa)lyl Kolkata I
" . ACE. . .
.,':. :EPpw.nngPnNicdions : 435 : Zeroth Law & Basic Concepts
�-�
05. Ans: (c) 06. Ans: (b)
Sol: P1 100 + 101.325 = 201.325 kPa
= Sol:
p
T1 = 288 K , V1 = 2500 C.C
-6 2----
pl vi --------
-- 201.325 x l0 3 x 2500 x 10- P=c
m= =
R Tl 287 x 288
V=c
= 6.089 X 10 -3 kg
As volume is constant, dW = 0 3
:. dQ = dU = m Cv dT
= 6.089 x 10-3 X 0.718 X 10 3
V
(15 -5) Process 1-2 (P = c)
= 43.7 J
v1 = -
- v2
Tl Tz

T2
P 2 = Pressure on day of play = P1 x -
T1
= 201.325x
278
288
= 194.33 kPa (absolute)

Gauge pressure on day of play


07. Ans: 4.4 to 4.6
= 194.33 - 101.325

= 93 kPa = 0.93 bar


Sol: V= 2m 3
T= 288K
P 1 = Pressure to which it must be inflated
P1 = -
P2 � 8314
R= =
MN 2 28
-
Tl Tz
pl - 201.325 1
= -- = 296.928--= 0.2969 __!!_
288 278 kgK kgK
288
P1 = 201.325 x -= 208.56kPa.
278 70
P= Patm + Pgauge=101.325 + - xlOl .325
Gauge pressure to which it must be inflated 76
on previous day P= 194.65kPa
= 208.56 - 10 1.3 PV 194.65 x 2
m= = 4_5528 kg
= 107.24 kPa = 1.07 bar RT 0.2969 x 288
=

lli11ii@Oii4.j!iAflbiMihfjj� yderabad j DelhijBhopal j Pune jBhubaneswar i Lucknow j Patnaj Bengaluru i Cheruiaij Vijayawada j V,z.ag ITirupat( j Kukatpallyj Kolkata
08. Ans: (c)
Sol: V 1 = 2.87 m3
T1 = 300 K
+
Pabs = Pa1m Pgauge

Pabs = P1 = l+o.5 = 1 .5 bar = 150 kPa

R = 0.287 kJ/kgK
m = P1 V1 = 150 x 2. 87 = 5kg
RTI 0.287 x 300

!iill&A AM flydmbad I Delhi I Bhopal I l'uncl Bhubwawarl·Lucknowl hlnal I


Benpluru Oiamai I Vuayawadal Vaz.a« ITirupati I Kublpallyl Kolkala I
Work and Heat
04. The contents of a well-insulated tank are
One Mark Questions heated by a resistor of 230 in which l OA
current is flowing. Consider the tank along
01. For reversible adiabatic compression m a with its contents as a thermodynamic
steady flow process, the work transfer per system. The work done by the system and
unit mass is (GATE-ME-96) the heat transfer to the system are positive.
(a) JPdv (b) J vdP The rates of heat (Q), work (W) and
change in internal energy (AU) during the
(c) J Tds (d) JsdT
process in kW are (GATE-ME-1 1)
(a) Q=O, W =- 2.3, AU =+2.3
02. A compressor undergoes a reversible, steady
(b) Q =+2.3, W = O, AU =+2.3
flow process. The gas at inlet and outlet of
(c) Q =- 2.3, W = O, AU =- 2.3
the compressor is designated as state l and
state 2 respectively. Potential and kinetic (d) Q =O, W =+2.3, AU= -2.3
energy changes are to be ignored . The
05. A pump handling a liquid raises its pressure
following notations are used : v = specific
from 1 bar to 3 0 bar. Take the density of the
volume and P = pressure of the gas. The
liquid as 99 0 kg/m 3 • The isentropic specific
specific work required to be supplied to the
work done by the pump in kJ/kg is
compressor for this gas compression process
(GATE-ME-1 1)
(GATE-ME-09)
(a) 0.1 0 (b) 0.3 0 (c) 2.5 0 (d) 2.93
lS

(a) rPdv (b) fvdP


I
06. A cylinder contains 5 m 3 of an ideal gas at a
(c) v 1(P2-P 1) (d) -P2(v 1-v2)
pressure of 1 bar. This gas is compressed in
a reversible isothermal process till its
03. Heat and work are (GATE-ME-11)
pressure increases to 5 bar. The work in kJ
(a) intensive properties
required for this process is
(b) extensive properties
(GATE-ME-13)
(a) 804.7 (b) 953.2
(c) point functions
(d) path functions
(c) 981.7 (d) 1 012.2
" " . ACE
. Pnblir»
. . ... Thermodynamics
,...��
'!. � : 438 :

resistor for 90 secs, the piston sweeps a


07. Which of the following statements are volume of 0.01 m 3 . Assume (i) piston and
TRUE with respect to heat and work? the cylinder are insulated and (ii) air
(i) They are boundary phenomena behaves as a ideal gas with C v = 700 J/kg K.
(ii) They are exact differentials Find the rise in temperature of air.
(iii) They are path functions (GATE-ME-93)
(GATE- 16-SET-1)
(a) Both (i) and (ii)
(b) Both (i) and (iii)
(c) Both (ii) and (iii) m=O. l kg

(d) Only (iii)

06. A mass m of a perfect gas at pressure p 1 and 02. Nitrogen at an initial state of 10 bar , lm 3
volume VI undergoes an isothermal pro cess. and 300 K is expanded isothermally to a
The final pressure is p2 and volume is V2. final volume of 2m 3 • The p-v-T relation is
The work done on the system is considered
positive. If R is the gas constant and T is the
(p + ;2 ) v= RT, where a >O. The final

temperature, then the work done in the pressure (GATE-ME-OS)


process 1s (GATE- 17- SET- 2) (a) will be slightly less than 5 bar
(b) will be slightly more than 5 bar
(b) -p I Vi Rn .£.!_
P2 ( c) will be exactly 5 bar
V (d) cannot be ascertained in the absence
(c) RTRn -2
v1 of the value of a.

03. A 100 W electric bulb was switched on in a


Two Marks Questions 2.5 m x3mx3 m size thermally insulated
room having a temperature of 20 °c. The
01. A vertical cylinder with a freely floating room temperature at the end of 24 hours
piston contains 0.1 kg air at 1.2 bar and a will be (GATE-ME-06)
small electrical resistor. The resistor is wired (a) 321 C
°
(b) 341°C
to an external 12 Volt battery. When a (c) 450 ° C (d) 470 ° C
current of 1.5 amps is passed through the
\( l l 1 1�1r1i l 1 111� P11hlh .1t11 111, �yderabadlDdhilBhopaI I PunelBhubaneswarl LucknowlPatna ! BengaiurulChennailVuayawadalVmg I T1n.tpati I �I Kolkata I
: 439 : Work and Heat

04. In a steady state steady flow process taking


place in a device with a single inlet and a 07. 1 .5 kg of water is in saturated liquid state at
single outlet, the work done per unit mass 2 bar (vr = 0. 001 06 1 m 3 /kg, ur = 5 04. 0kJ/kg,
flow rate is given by W =-J
outlet
vdp , where hr = 5 05 kJ/kg). Heat is added in a constant
inlet
pressure process till the temperature ofwater
v is the specific volume and P is the reaches 4 00° C {v = l .5493m 3 /kg,u=2967 . 0
pressure. The expression for 'W ' given
kJ/kg, h = 3277. 0 kJ/kg). The heat added (in
above (GATE-ME-08)
kJ) in the process is _
(a) is valid only if the process is both (GATE-ME-14-SET-l)
reversible and adiabatic
(b) is valid only if the process 1s both 08. A mixture of ideal gases has the following
reversible and isothermal composition by mass:
(c) is valid for any reversible N2 02 C O2
(d) is incorrect, it must be w = J. pdv 6 0% 3 0% 1 0%
outlet

mlet

If the universal gas constant is 8314 J/kmol­


05. A frictionless piston-cylinder device
K, the characteristic gas constant of the
contains a gas initially at 0.8 MPa and
mixture (in J/kg-K) is____
0. 015m 3 • It expands quasistatically at
(GATE -15-Set 3)
constant temperature to a final volume of
0. 03 0 m 3 • The work output (in kJ) during 09. An ideal gas undergoes a reversible process
this process will be (GATE-ME-09) in which the pressure varies linearly with
(a) 8.32 (b) 12. 00 volume. The conditions at the start
(c) 554.67 (d) 832 0. 00 (subscript 1) and at the end (subscript 2) of
the process with usual notation are: P 1 = 1 00
06. A mono-atomic ideal gas (y = 1 .67,
kPa, V 1 = 0.2 m 3 and P2 = 2 00 kPa, V2 = 0. 1
molecular weight=4 0) 1s compressed
m 3 and the gas constant, R = 0.275 kJ/kgK.
adiabatically from 0. 1 MPa, 3 00K to 0.2
The magnitude of the work required for the
MPa . The universal gas constant is 8.314 kJ
process (in kJ) is___
kmor 1 K· 1 . The work of compression of the
gas(in kJ kg.1 ) is
(GATE- 16- SET- 1)
(GATE-ME-10)
(a) 29.7 (b) 1 9.9 (c) 13.3 (d) O

Fydcrabad l Ddhi l Bhopal ! Pune l Bhubaneswarl Lucknow l Patna ! Bcngaluru ! Olennai l Vuayawadal Vu.ag ITirupati J �I Kolkm I
.
SOLUTIONS
05. Ans: (d)
One Mark Solutions Sol: W = vdP

= -dP = - x 30 - 1 x l O 2
1 1 ( )
0 1. Ans: (b) 02. Ans: (b) p 990
kJ
= 2.93 -
03. Ans: (d) kg

04. Ans: (a) 06. Ans: (a)


Sol: Power R = 10 x23 =I 2 2
Sol: P1 = 1 bar = 100 kPa
= 2.3 kW V1 = 5 m3
� - bW = dU P 2 = 5 bar
� = 0 (insulated)
W1_, - P1 v1 1n( :: J
bW= negative sign
+
(External Agent) = 100 X 5 X fn (-fJ = 804.7 kJ
0 - (-P) = dU
0 - (-2.3) = dU
07. Ans: (b)
dU = 2.3 kW; 8Q = 0
8W = - 2.3 kW 08. Ans: (b)
In insulated tank there is no energy transfer Sol: P1 V 1 = P 2V 2 (isothermal)
across system boundary. Hence, Heat P V2
transfer ;,, 0.
-1 = -
P 2 V1
To operate the resistor an external agent has V
Isothermal work = -P1 V1 fn -2 =
to do work hence, work transfer is negative. v1

-PI VI fn .!i_
Whatever work is done is converted to
internal energy which will raise the p
2
temperature of the system.

\( I I ll..,111( I 111 .., PtildH ,l[I( 111 ... �ydcrahad I Delhi I Bhopal I Pune I Bb�I Lucknowl Patmj Bengahuu I Cbennai I Vgayawada I Vizag jTuupari I Kukatpallyj Kolkati. I
"
��••:E
��•
"
:.,,...
�- :!��
N�
· ·
ACE ·= 441
�=.,========� ��= ==========��
·
Wo:rk
�and He:
��: at
Ji "= Niarioos

03. Ans: (d)


Two Marks Solutions Sol: P air=1.2 kg / m 3 , ( cJair =0.718kJ / kg.K
m C v dT =Pt
01.
pV C v dT =Pt / 1000
Sol: dV = 0.01 m 3 , cv = 700 J/kg.K = 0.7 kJ/kg.K
l .2x(2.5x 3x3) (0.718) (T-20)
P = 1.2 bar
24
Q = 0 (Insulated) =100x3600x--
1000
Ws = Work done by system
19.386(T - 20) = 8640
= PdV = 120 x 0.01 = 1.2 kJ
T = 465.6°C ::::: 470 °C
WR = VI t ; V = 12 V
I = 1.5 A, t = 90 sec.
04. Ans: (c)
= -12 X 1.5 X 90 = -1.62 kJ
Negative sign indicates external agent is
05. Ans: (a)
doing work
cQ - bW = dU Sot: , w, = P, v, 1n(�: )
cQ - [ bWs - bWR ] = dU
0 030
0 - [ 1.2 - 1.62 ] = m C v dT = 800 x0.015 ln ( · ) = 8.317 kJ
0.015
0.42 = 0.1 x 0.7 dT
06. Ans: (a)
0.42
dT = =6 Sol: T1 = 300 K ,
0.07
P1 = O.l MPa ,
dT = 6° C
P2 = 0.2 MPa
02. Ans: (b)
y -1

Sol: (P, + :}, = (P, + :}, �: =( :: r


02
)1.67
0.67

T2= 3oo( ·
(P, + T) x1 = (P2 + �) x 2 0.1
= 396.18 K

8Q -f>W = dU
=> (10 + a)=2P2 + � 8Q = O
2
- 8W = dU
P2 =5 + -
a
4 R
-W = C V (T 2 - T 1) = -- (T2-TI )
As 'a ' is positive P 2 is > 5 bar M(y-1)
\ ( ' l l 111:,111t 1 1111� Pulil u ,1111,11, �yderabad l Dclhi l Bhopa! I Pune j Bhubancswarl Lucknowj Patna j Benga)wu j Chennai j Vqayawada j Vizag I Tuupati I Kukalpal)yj Kolkala I
: 442 : Thermodynamics

8·314 09. Ans: 15 (range 14.75 to 15.25)


= (396.18-300)
40 (1.67-1) Sol:
W =- 29.83 kJ/kg
p

200
07. Ans: 4158 kJ I
Sol: Constant pressure process (P = C)
I
1 00 I
I
P1 = P2 = P , I

Heat added ( 1Q2)


= m [(u2- u 1 ) + P(v2- v 1 ) ]
= m [(u2- u r) + P (v2 - vr) ] 1
1 w2 = - (P1 + P2 Xv1 - vJ
= m [(u2 + P2v2}-(ur + Pr v r) ] 2
= m [h2 - hr ] = ! (100+200)(0.2 - 0.1) = 15 kJ
= 1.5 X [3277 - 505 ] = 4158kJ 2

08. Ans: 274 to 276


8314 1
Sol: R N 2 = � = = 296.928 --
M N2 28 kg K

8314 1
R 02 = � = = 259.8125 --
M 02 32 kg K

8314 1
R co 2 = __!_ = = 188.954 --
M CO2 44 kg K

0.6 X 296.928+0.3 X 259.8125+0 . 1 X 188.955


= ---------------
0.6+0.3+0.1
= 178.1568+77.9437+18.8956
= 274.996 J/kg K

\( l l 1ti,:,l!H l 1111� P11lilu .111, 11h �yderabad l Delhi j Bhopal j Pune j Bhubaneswarl I.ncknowl Patna ! Bengaluru j Chennai l Vuayawada j Vu.ag I Tuupati I Kukalpallyj Kolkatal
I
Chapter
3 First Law of Thermodynamics

negligible. The heat transfer from the


One Mark Questions turbine casing to the surroundings is equal to
(GATE-ME-00)
01. The first law of thermodynamics takes the (a) 0 kJ (b) 50 kJ
form W = - � H when applied to: (c) 150 kJ (d) 250 kJ
(GATE-ME-93)
04. A 2 kW, 40 litre water heater is switched
(a) A closed system undergoing a
on for 20 minutes. The heat capacity Cp
reversible adiabatic process.
for water is 4.2 kJ/kg K. Assuming all the
(b) An open system undergoing an adiabatic
electrical energy has gone into heating the
process with negligible changes in kinetic
water, increase of the water temperature in
and potential energies.
degree centigrade is (GATE-ME-03)
(c) A closed system undergoing a
reversible constant volume process. (a) 2.7 (b) 4.0 (c) 14.3 (d) 25.25
(d) A closed system undergoing a
reversible constant pressure process. 05. A gas contained in a cylinder is compressed,
the work required for compression being
02. A steel ball of mass 1 kg and specific heat 5000 kJ. During the process, heat interaction
0.4 kJ/ kg is at a temperature of 60 °C. It is of 2000 kJ causes the surroundings to the
heated. The change in internal energy of the
dropped into 1 kg water at 20 °C. The final
gas during the process is (GATE-ME-04)
steady state temperature of water is
(a) - 7000 kJ (b) - 3000 kJ
(GATE-ME-99)
(c) + 3000 kJ (d) +7000 kJ
(a) 23.5 °C (b) 30 ° c
(c) 35 °C (d) 40 °c 06. Water (density = 1000 kg/m3) at ambient
temperature flows through a horizontal pipe
03. A steam turbine receives steam steadily at of uniform cross section at the rate of 1 kg/s.
10 bar with an enthalpy of 3000 kJ/kg and If the pressure drop across the pipe is 100
discharges at 1 bar with an enthalpy of 2700 kPa, the minimum power required to pump
kJ/kg. The work output is 250 kJ/kg. The the water across the pipe, in Watts, is
changes in kinetic and potential energies are (GATE- 17- SET- l)
!ltllii@jjjdilbMJMftiMmm+Yderabad I Delhi I Bhopal I Pune IBhubaneswar I Lucknow I Patna I Bengaluru I Chennai IV\iayawadalVJZag I Thupari I Kuk.atpally I Kolk.ala I
"
"
"
., F:npw,:qP.�
:.� ,.
. . .
. ACE : 444 : Thermodynamics

04. What is the speed of sound in Neon gas at a


Two Marks Questions temperature of 500 K (Gas constant of Neon
is 0.4210 kJ/kg.K) ? (GATE-ME-02)
(a) 492 mis (b) 460 mis
0 I . A rigid insulated cylinder has two
(c) 592 mis (d) 543 mis
compartments separated by a thin
membrane. While one compartment contains
05. A balloon containing an ideal gas is initially
one kmol nitrogen at a certain pressure and
kept in an evacuated and insulated room .
temperature, the other contains one kmol
The balloon ruptures and the gas fills up the
carbon dioxide at the same pressure and
entire room . Which one of the following
temperature. The membrane is ruptured and
statements is TRUE at the end of above
the two gases are allowed to mix.
process? (GATE-ME-08)
Assume that the gases behave as ideal gases.
C alculate the increase in entropy of the (a) The internal energy of the gas
contents of the cylinder. Universal gas decreases from its initial value, but the
constant equal to 8314.3 J/kmol K. enthalpy remains constant
(GATE-ME-93) (b) The internal energy of the gas increases
from its initial value, but the enthalpy
02. When an ideal gas with constant specific remains constant
heats is throttled adiabatically, with (c) Both internal energy and enthalpy of
negligible changes in kinetic and potential the gas remain constant.
energies (GATE-ME-00) (d) Both internal energy and enthalpy of
(a) � h = O, � T = 0 (b) �h > O, � T = O the gas increase.

(c) � > 0,�s > 0 (d) � = 0, �s > O


06. A rigid, insulated tank is initially evacuated .
The tank is connected with a supply line
03. A small steam whistle (perfectly insulated
through which air ( assumed to be ideal gas
and doing no shaft work) causes a drop of
with constant specific heats) passes at I
0.8 kJ/ kg the kinetic energy of the steam at
MPa, 350 °C . A valve connected with the
supply line is opened and the tank is charged
entry is negligible, the velocity of the steam
at exit is (GATE-ME-01)
with air until the final pressure inside the
(a) 4 mis (b) 40 mis
tank reaches 1 MPa. The final temperature
(c) 80 mis (d) 120 mis
inside the tank. (GATE-ME-08)
\( l l !tl.;llll l !Ill.., }'1Jl,lu ,\lltilh yderabad J Delhi I Bhopal I Pune l Bhubaneswarl Lucknow! Patna I Benga)uru I Chennai J Vuayawada j V11.3g I TllUpati I Kukatpally j Kolkata
"'..
:.t . .
ACE
.." �PubliraicD . . : 445: First Law of Thermodynamics

07. The density of air in kg/m3 at the nozzle exit


Air Supply
IS

(a) 0.560 (b) 0.600


line

Valve (c) 0.727 (d) 0.800

08. The mass flow rate of air through the nozzle


Tanlc in kg/s is
(a) 1.30 (b) 1.77 (c) 1.85 (d) 2.06
(a) is greater than 350°C
09. Steam enters an adiabatic turbine operating
(b) is less than 350°C
at steady state with an enthalpy of
(c) is equal to 350°C
3251.0kJ/kg and leaves as a saturated
(d) may be greater than less than, or equal
mixture at 15 kPa with quality (dryness
to 350° C, depending on the volume of
fraction) 0.9. The enthalpies of the saturated
the tank.
liquid and vapor at 15 kPa are
hr = 225.94kJ/kg and hg = 2598.3kJ/kg
Statement for Linked Answer Q.07 & Q.08
respectively. The mass flow rate of steam is
The temperature and pressure of air in a large
1Okg/s. Kinetic and potential energy changes
reservoir are 400 K and 3 bar respectively. A
are negligible. The power output of the
converging diverging nozzle of exit area 0.005m2
turbine in MW is (GATE-ME-12)
is fitted to the wall of the reservoir as shown in
the figure. The static pressure of air at the exit (a) 6.5 (b) 8.9 (c) 9.1 (d) 27.0
section for isentropic flow through the nozzle is
Statement for Linked Answer QlO & Qll
50 kPa. The characteristic gas constant and the
Air enters an adiabatic nozzle at 300kPa, 500K
ratio of specific heats of air are 0.287 kJ/kgK and
with a velocity of 10 mis. It leaves the nozzle at
1.4 respectively. (GATE-ME-11)
1OOkPa with a velocity of 180m/s. The inlet area
is 80cm2 • The specific heat of air Cp is 1008

Flow from the -- J/kg.K

10. The exit temperature of the air is


reservoir

(GATE-ME-12)
Nozzle exit

(a) 516 K (b) 532 K


(c) 484 K (d) 468 K
FydcrabadlDelhilBbopa)IPuncJBhubancawarl l..uclmowlPaJnalBcngaluruJChcnnailVuayawadaJVizag IT1n1pati I Kukalpa)lyj Kolkala I
: 446: Thermodynamics

11. The exit area of the nozzle in cm2 is 14. Work is done on a adiabatic system due to
(GATE-ME-12) which its velocity changes from 10 mis to 20
(a) 90.1 (b) 56.3 mis elevation increase by 20 m and
(c) 4.4 (d) 12.9 temperature increases by 1K. The mass of
the system is 10 kg, Cv = 100 J(kg.K) and
12. Specific enthalpy and velocity of steam at gravitational acceleration is 10 mls2 . If there
inlet and exit of a steam turbine, running is no change in any other component of the
under steady state, are as given below: energy of the system, the magnitude of total
work (in kJ) on the system is ___
(GATE-15-Set 2)
Specific Velocity
enthalpy (mis)
(kJ/kg) 15. A piston-cylinder device initially contains
Inlet steam condition 3250 180 0.4 m3 of air (to be treated as an ideal gas) at
100 kPa and 80° C. The air is now
Exit steam condition 2360 5

The rate of heat loss from the turbine per kg isothermally compressed to O.l m 3 . The work
of steam flow rate is 5 kW. Neglecting done during this process is ____ kJ.
changes in potential energy of steam, the (Take the sign convention such that work
power developed in kW by the steam turbine done on the system is negative)
per kg of steam flow rate, is (GATE-16- SET-2)
(GATE-ME-13) 16. Steam at an initial enthalpy of 100 kJ/kg and
(a) 901.2 (b) 911.2 inlet velocity of 100 mis, enters and
(c) 17072.5 (d) 17082.5 insulated horizontal nozzle. It leaves the
nozzle at 200 mis. The exit enthalpy (in
13. A well insulated rigid container of volume kJ/kg) is__ (GATE-16- SET-3)
l m 3 contains 1.0kg of an ideal gas [Cp =
1000 J/(kg.K) and Cv = 800J/(kg.K) at a 17. One kg of an ideal gas (gas constant, R =
pressure of 105 Pa. A stirrer is rotated at 400J/kg.K; specific heat at constant volume,
constant rpm in the container for 1000 Cv = 1000 J/kg.K) at 1 bar, and 300 K is
rotations and the applied torque is 100 N-m. contained in a sealed rigid cylinder. During
The final temperature of the gas (in K) is an adiabatic process, 100 kJ of work is done
(GATE-15-Set 1) on the system by a stirrer. The increase in
entropy of the system is ___ J/K.
(GATE-17- SET-1)
lajjl4jj§jjji4.jjlAfl§5ijijjj+yderabad IDelhi I Bhopal IPuncl Bhubaneswar I Lucknow IPatna I Bcngaluru IChcnnai I Vgayawada jVizag IT111lpari I K�y I Kolkata I
: 447: First Law of Thermodynamics

18. A calorically perfect gas (specific heat at H1 =220 kJ, H2=247 kJ, and S1 = 0.23 kJ/K
constant pressure 1OOOJ/kgK) enters and and S2 = 0.3 kJ/K where T, U, H and S
leaves a gas turbine with the same velocity. represent temperature, internal energy,
The temperatures of the gas at turbine entry enthalpy and entropy respectively.
and exit are 1100K and 400K, respectively. (GATE-ME-00)
The power produced is 4.6MW and heat
escapes at the rate of 300kJ/s through the
turbine casing. The mass flow rate of the gas
(in kg/s) through the turbine is
(GATE-17- SET-2)
(a) 6.14 (b) 7.00
(c) 7.50 (d) 8.00

Five Marks Questions

01. Air enters a frictionless adiabatic converging


nozzle at 10 bar, 500K with negligible
velocity. The nozzle discharges to a region
at 2 bar. If the exit area of the nozzle is 2.5
cm2 find the flow rate of air through the
nozzle. Assume for air Cp = 1005 J/kg K and
Cv = 718 J/kg K
(GATE-ME-97)

02. A certain mass of a pure substance


undergoes an irreversible process from state
1 to state 2, the path of the process being a
straight line on the T-s diagram. Calculate
heat transfer & work done. T1 = 330 K,
T2 = 440 K; U1 =170 kJ, U2 = 190 kJ;

�yderabadj Dclhil Bhopal I Pune I BhubaneswarJ Lucknow! Palnal Bcnpluruj Oiennai IVgayawadaJVmg IT,rupati I Kukalpillyj Kolkata I
SOLUTIONS
02. Ans: (a)
One Mark Solutions Sol: msX c p, (Tl -Tr) = mwX CPw (Tr-T2 )
=> 1 x 0.4 x [60-Tc] = 1 x 4.2 (Tr-20)
01. Ans: (b).
=> 24-0.4 Tr = 4.2 Tr-84
Sol: For flow Process W =-JVdP
=> 4.6 Tr = 24 + 84 = 108
Jv dP = Area ( OA1C) +Area(C12B)-Area(DOB2) 108
:. Tr = = 23.48 ° C
= P1 V1 + JPd V-P2V2 4.6

p 03. Ans: (b)


Sol: h1 + Q = h2 + W

D - -r- - 2
3000 + Q = 2700 + 250
:. Q =-50 kJ

0 C BV 04. Ans: (c)


Adiabatic process:( Q = 0 ) Sol: Pw = 1000 kg/m3 ,
mw CPwX (�T)w = PX t
J VdP = P1 V1 + PdV-P2V2 J2

=> Vw Pw CPw(�T)w = PX t
I
=> 40 x 10-3 xlOOO x4.2x(�T)w= 2x20x60
:. (�T)w = 14.285°C = 14.3°C

=-Y- ( P1 V1 -P2V2) 05. Ans: (c)


y-1
Sol: 8Q-8W = dU
R(T1 -T2) -2000-(-5000) = dU
y-1
= ym

:. dU = + 3000 kJ
JvdP=m CP(Tl -T 2) = H1 -H2
=-[Hz-Hi] =-�H 06. Ans: 100
i.e. An adiabatic flow work is equal to Sol: Power = ril.vdP
change in the enthalpy of the system. p = 1000kg/m 3
\( I } ,l ,1111 t 1 ill. P1ilil11 .111, HI" �ydcrabadl Delhi I Bhopal I Pune I Bhubancswarl Lucknow I Patnal Bcngaluru I Chcnnai IVuayawadalVizag IT1n1pati I Kukatpallyl Kolkala I
: 449: First Law of Thermodynamics

1 1 oQ oW
V = - = -- m / kg = 0 (adiabatic ), =0
3
p 1000 dm dm
dP = 1OOkPa = 105Pa h1 = h2 => Cp (T1 -T2) = 0
ril = l kg / sec :. T1= T2
1
Power = l x -- x l 0 5 = 100W 03. Ans: (b)
1000
Sol: By applying steady flow energy equation to
nozzle:

+Q = h 2 + +W
y2
h1 +
y2

2x1000 2x1000
2
I
Two Marks Solutions
V1 = O, Q = O, W = O
01. :.V2 = 44.72./M = 44.72.Jo.s
Sol: Rigid, W = 0 ; = 40 ml sec
Insulated, Q = 0
:. dU = O; Cv dT = O 04. Ans: (c)
T=C Sol: Neon is Mono atomic gas, y = 1.67
dT = O .. .
P = C} Ysonic = �yRT = �1.67x421x 500

= 592 m/se
(dS)co 2
= O; (dS) N = 0
2

(dS)mix = -n R :r Xi /n Xi
05. Ans: (c)
= -2 x 8.31 4 [ X /nX + X
N N c /n Xc 02 02]
Sol: oQ = 0 (Insulated)
/n(�) + (�) /n(½}
2 2

= -2 x 8.314 [ ½ oW = 0 (free expansion)


oQ-oW = dU::::> dU = 0
= -2 X 8.314 /n (�) Cv dT = 0 => T = Constant.

= 11.525 kJ I K 06. Ans: (a)


Sol: oQ= 0 Insulated and vessel is empty
02. Ans: (a) & (d) It is charging process as it
Sol: By applying steady flow energy equation to is filling of gas.
throttle valve: lllo = 0 For charging process,
Tr = y T = 1.4(350 + 273) > 350°C

\(} I 11�lllt( IIIIL, P1ddH,llltlll', Fydcrabad!Dcllri!BhopallPunclBhubancswarl LucknowlPatnalBcngaiwu!ChcnnailVtiayawadalVizag ITllUpati J Kukaq,allyl Kolkata I


: 450: Thermodynamics

07. Ans: (c) = 225.94 + 0.9 (2598.3 -225.94)


= 2361.064 kJ/kg
08. Ans: (d) WT = ms [h1 - h2]
Sol: In let Exit = 10 (3251-2361.06) = 8.9 MW
P1 = 300 kPa Ae = 0.005 m 2
T1 = 400K Pe = 50 kPa 10. Ans: (c)
y-1 1.4-1

= = 11. Ans: (d)


�: (::r uo:r Sol: 1 2
� Te= 239.73K
Ve=�2CP (T1 -Te)

= �2 X 1005(400-239.73)
=567.57 ml sec T1 = 500K, P2 = 100 kPa
Pe 50 P1 = 300 kPa, V2 = 180 m/sec
p --- -
e - RTe 0.287 X 239.73 V1 = lOm/sec, T2 = ?
k A, = 80 cm 2 , A2 = ?
=0.7267 �
m Cp = 1008 J/kgK
2 2
v + v2 +
h 1 +-1 Q=h 2 +-
rh= Pe Ae Ve
W
= 0.727 0.005 X 567.57
X 2 2
= 2.06 kg/sec ril = P1 A1 V1 = p 2 A2 V2
Q = O, W = O
09. Ans: (b)
:. V2 =�Vl2 +2C p (TI -T2 )
2 g
Sol: m s [h I +l+ 1 J+Q � 180=�10 2 +2x1008(500-T2 )
2000 1000

=m
[ h2 +-2-+-
g] +W
v 2 z 2- �32400 = 100 + 2016 (500-T2)
:. T2 = 484K
s
2000 1000 T

Z, =Z2 A2 = E!_ X vi xA. =_!j_ X RT2 X vi xAI


p 2 V2 RT1 P2 V2
V1 = V2
kJ 300 484 _!_Q_
h, = 3251- = x x x 80 (·: P=pRT)
kg 100 500 180
A2 = 12.906 cm 2
h2 = [hr+ x ( h8-hr)li5kPa
: 4.51: First Law of Thermodynamics


12. Ans: (a) 14. Ans: 4.5

Sol: + �� + �� =
2 0 J m( h +
, 2 �io J +
6
:
Sol:U 1 +-
V12
2
8Q V22
+z 1 g +-=U2 +-
dm 2
8W
+z2 g +- •

Q
dm
8W _ . . (v/ - vf 8Q ·: adiab atic process, f:J =O
- - m (h - h2 )+m ---'----=-J +-
dt I 2000 dt dm
8W V2 -V22
180 - 5 2 2 =( u 1 - u2 )+( i J+(z 1 -z2 )g
=1[ 3250 - 2360]+1( J-5 dm 2
2000
= 90 1.2 kW

13. Ans: 1285 K (1283.4 to 1287.4)


y 2 _ y2
=C)T1 -Ti ) + ( 1
2
2
+(z 1 - z2 )g
J
102 - 202
=100(-1) +( ] +(-20)x10
Sol: P = 100 kPa 2
m = 1kg 3 0
=-100 -( � )-2 00
V= 1m 3

R=C p -C v = -450] / kg=-0.45kJ / kg

=1000 -800 = 0.2 kJ/kg K W =m( !: ) =10(-0.45) =-4.5kJ


PV I 00xl
T= = =500 K
mR lx0.2
15. Ans:-55.45 (range -55.4 to 55.6)
Work=TS=T x21tN
Sol: , w, = P, V, en(�'. J
=628kJ
21txlOOOxlOO
=
1000
628kJ 0.1
kJ 100x0.4x.en( -) = -55.45 kJ
Work(kJ/kg)= =628
=

1kg kg 0.4

8Q-8W=dU; 8Q=O
16. Ans: 85 (range 84 to 86)
Sol:
0 - (-W) =C v dT
2
W = 628
dT = =785
CV 0.8 h 1 = lOOkJ/kg

T2 -T1 =785 v 1 = lOOm/sec v2 = 200m/sec


T2 =T1 +785 = 500+785 =1285 K

\( ·1 l 11�11u ( 1111g P11hlu .1l1t>ll.., �yderabad I Delhi I Bhopal I Pune I Bhubancswar I Lucknow I Patna I Bengaluru I Chennai I Vtjayawada I VIZllg I Tiru� -L Kubtpal)y I Kolkata I
: 452: T hermodynamics

v ? oQ
h +-1-+- = h
v
+--+-
2 ow Cp = 1000 J/kgK = 1 kJ/kgK
2000 dm 2000 dm V 1= V2
2
2
I .

y12 - V; T 1 = 1100 K, T2 = 400 K


h2 = hI +
2000 Power = 4600 kW
100 2 -200 2 Heat loss from turbine casing = Q =-300 kW
100 + ( )
2000 Mass flow rate = ri1 (kg/sec)
=

= 100 -15 = 85 kJ / kg Apply SFEE


ril h 1 + Q = ril h 2 + Vf
17. Ans: 287.68
Sol: m = 1 kg
m(h 1 - h 2 ) = W - Q � rilcP (T1 - T2 ) = W - Q
W-Q
R = 400 J/kgK ri1 = = 4600+300 7 kg/sec
Cv = 1000 J/kgK c p (T1 -T2 ) 1(1100-400)
=

P = 100 kPa
T 1= 300 K
Wstirrer = Ws = -lOOx lOOOJ
Adiabatic process dQ = 0
dQ -dW = dU = mCv(T2-T 1) Five Marks Solutions
lx lOOO(T2-300)
0-(-105) =
T2 = 100+300 = 400K
T 400 01.
ds = mC .en 2 = 1x1000.en
v Tl 300 Sol:
= 287.681/K

18. Ans: (b)

( 1)
Sol:

dW A2 = 2.5 cm2 = 2.5 x 10- 4 m2


P 1 = 10 bar, P2 = 2 bar
T 1= 500K, T2 = ?
Cp= 1005 J/kg.K, Cv = 718 J/kg.K
(2)
c 1005
y= p = =1.4
CV 718

�ydcrabad I Delhi I Bhopal I Puncj Bhubancswarl Lucknow! Patna! Bengaluru I Chennai IVtiayawada IVmg I Tirupali I Knkatpally I Kolkata I
1
\(} l 11�l!il l Ill,� l 1dd11 ,t\]llll"
" " ACE . .
':.··�:&,n,,,qPrffmiml
. . : 453: First Law of Thermodynamics

y;I 02.
P 2 �: :
T2 =T1 ( 2 J =500x ( ) =315.7K Sol: Heat transfer =Area under T-S diag ram
Pi 10
1
y2 y2 1 Q 2 =-x{ T1 +T2 )x{S 2 -S i )
h i +-I + Q= h 2 +-2 + w 2
2 2g
= !(33o+44oXo.3-o.23)=26.95k1
Q=O, W=O, V1 =O 2
:. V2 = � 2CP (T1 -T2) I W2 =I Q2 -dU =26.95-(190-170)
=6.95kJ
= �2x1005{500-315.7)
=608.64 m I sec
RT
T2= 440K 2
Specific volum e, v2 = 2 T
P2
287x 315.7 m3
= =0.453
2x10 5 kg
T 1 = 330K
1
A 2V2
Mass flow rate, ril=
V2
St = 0.23 kJ/ K S2=0.3 kJ / K
2.5 x 10-4 x 608.64
= 03358kg / sec
0.453
----+S

\( I I 11�1r11 t 1111� P1il>IH.tl1111b Fyderabadj Delhij BhopaljPuncj Bhubaneswarl LucknowlPatnalBengalurulOtennailVuayawadajVu.ag jTirupati I ·Kukatpallyl Kolbla I
C4 Second Law of
Thermodynamics
04. Consider a refrigerator and a heat pump
One Mark Questions working on the reversed Carnot cycle
between the same temperature limits. Which
01. A condenser of a refrigeration system rejects of the following is correct? (GATE-ME-95)
heat at a rate of 120 kW, while its (a) COP of refrigerator = COP of heat
compressor consumes a power of 30 kW. pump
The coefficient of performance of the (b) COP of refrigerator = COP of heat
system would be (GATE-ME-92) pump+ 1
(a) 1/4 (b) 4 (c) 1/3 (d) 3 (c) COP of refrigerator COP of heat
pump - 1
02. A reversible heat transfer demands: (d) COP of refrigerator = mverse of the
(GATE-ME-93) COP of heat pump
(a) The temperature difference causmg
heat transfer tends to zero 05. In the case of a refrigeration system
(b) The system receiving heat must be at a 8
undergoing an irreversible cycle, f ;
constant temperature.
(c) The system transferring out heat must is___(<0/ = 0/>0) (GATE-ME-95)
be at a constant temperature.
(d) Both interacting systems must be at 06. An industrial heat pump operates between
constant temperatures the temperatures of 27°C and -13°C. The
rates of heat addition and heat rejection are
03. Any thermodynamic cycle operating 750 W and 1000W, respectively. The COP
between two temperature limits is reversible for the heat pump is (GATE-ME-03)
if the product of the efficiency when (a) 7.5 (b) 6.5
operating as a heat engine and the COP (c) 4.0 (d) 3.0
when operating as a refrigerator is equal to
1. (GATE-ME-94) 07. The following four figures have been drawn
to represent a fictitious thermodynamic
cycle, on the P-v and T-S planes.
li\lli!i§jjj4i.jj4Rflnj@n1n�)Hyderabad I Delhi I Bhopal I Punc I Bhubaneswar I Luclmowj Patna I Bcngaluru I Chennai I Vrjayawada I Vuag I 1irupati I Kukatpally I Kolkata I
: 455 : Second Law of Thermodynamics

(GATE-ME-OS) reservoirs B and C, where Tc = 300 K. In


each cycle of CE-1 and CE-2, all the heat

• •
p T
rejected by CE-1 to reservoir B is used by
CE-2. For one cycle operation, if the net Q
absorbed by CE-1 from reservoir A is 150
MJ, the net heat rejected to reservoir C by
CE-2 (in MJ) is ____
V s
Figure 1 Figure 2

(GATE -15 -Set 1)

• •
p T

10. The COP of a Carnot heat pump operating


between 6°C and 3 7°C is__
(GATE -15-Set 2)
V s
Figure 3 Figure 4
11. The heat removal rate from a refrigerated
According to the first law of space and the power input to the compressor
thermodynamics, equal areas are enclosed are 7.2 kW and 1.8 kW, respectively. The
by coefficient of performance (COP) of the
(a) figures 1 and 2 (b) figures 1 and 3 refrigerator is _ _
(c) figures 1 and 4 (d) figures 2 and 3 (GATE - 16- SET- 2)

08. A reversed Carnot cycle refrigerator 12. A heat pump absorbs 10 kW of heat from
maintains a temperature of -5 °c. The outside environment at 250 K while
ambient air temperature is 35 °c. The heat absorbing 15 kW of work. It delivers the
gained by the refrigerator at a continuous heat to a room that must be kept warm at
rate is 2.5 kJ/s. The power (in watt) required 300 K. The Coefficient of performance
to pump this heat out continuously is (COP) of the heat pump is ____
(GATE - 17- SET- l)
(GATE-ME-14-SET-4)

09. A Carnot engine (CE- 1 ) works between two

TA = 900K and TB = SOOK. A second Carnot


temperature reservoirs A and B, where

engine (CE-2) works between temperature


Fyderabadl Delhi I Bhopal I Pune I Bhubancswarl Luclmowl Patnal Benga)urul Chcnnai IVtjayawadalVizag ITirupati I Kukalpallyl Kolkata I
: 456: Thermodynamics

05. A Carnot cycle is having an efficiency of


0.75. If the temperature of the high
temperature reservoir is 727°C, what is the
Two Marks Questions

01. Round the clock cooling of an apartment temperature of low temperature reservoir?
having a load of 300 MJ/day requires and (GATE-ME -02)
air-conditioning plant of capacity about (a) 23° C (b) -23° C
(GATE-ME-93) (c) 0° C (d) 250° C
(a) 1 ton (b) 5 tons
(c) 10 tons (d) 100 tons 06. A heat engine having an efficiency of 70%
is used to drive a refrigerator having a
02. A solar energy based heat engine which
coefficient of performance of 5.The energy
receives 80 kJ of heat at 100° C and rejects absorbed from low temperature reservoir by
70 kJ of heat to the ambient at 30° C is to be the refrigerator for each kJ of energy
designed. The thermal efficiency of the heat absorbed from high temperature source by
engme 1s (GATE-ME-96) the engine is (GATE-ME -04)
(a) 70% (b) 18.8% (a) 0.14 kJ (b) 0.71 kJ
(c) 12.5% (d) Indeterminate (c) 3.5 kJ (d) 7.1 kJ

03. For two cycles coupled in series, the topping


07. A solar collector receiving solar radiation at
cycle has an efficiency of 30% and the
the rate of 0.6kW/m2 transforms it to the
bottoming cycle has an efficiency of 20%.
internal energy of a fluid at an overall
The overall combined cycle efficiency is
efficiency of 50%. The fluid heated to 350 K
(GATE-ME-96)
is used to run a heat engine which rejects
(a) 50% (b) 44% (c) 38% (d) 55%
heat at 313 K. If the heat engine is to deliver
04. A cyclic heat engine does 50 kJ of work per 2.5 kW power, then minimum area of the
cycle. If the efficiency of the heat engine is solar collector required would be
75%, then heat rejected per cycle is (GATE-ME -04)
(GATE-ME-01) (a) 8.33 m 2
(b) 16.66 m2
2 (c) 39.68 m2 (d) 79.36 m2
(a) 16 - kJ (b) 33_!_ kJ
3 3
1
(c) 37 - kJ
2

\( I I 1 ...,1111 1 ! 111:..:, P1il.!11 1IH ,n, �yderabad l Delhi l BbopaI I Pune l Bhubancswar l Lucknowl Palna ! Bcngaiwu l Cbcnnai l Vtjayawada ! Vu.ag ! Tirupati I Kukatpallyl Kolkata j
: 457 : Second Law of Thermodynamics

08. A heat transformer is a device that transfers the heat pump delivers heat to its high
a part of the heat, supplied to it at an temperature sink is (GATE-ME-09)
intermediate temperature, to a high (a) 50 (b) 250
temperature reservoir while rejecting the (c) 300 (d) 360
remaining part to a low temperature heat
sink. In such a heat transformer, l OOkJ of 1 0. A reversible heat engine receives 2 kJ of
heat is supplied at 350 K. The maximum heat from a reservoir at 1000 K and a certain
amount of heat in kJ that can be transferred amount of heat from a reservoir at 800 K. It
to 400K, when the rest is rejected to a heat rejects 1 kJ of heat to a reservoir at 400 K.
sink at 300K is (GATE-ME-07) The net work output (in kJ) of the cycle is
(a) 12.50 (b) 14.29 (GATE- ME-14- SET-l)
(c) 33.33 (d) 57.14. (a) 0.8 (b) l .O (c) l.4 (d) 2.0

09. An irreversible heat engine extracts heat 1 1 . A reversible cycle receives 40 kJ of heat
from a high temperature source at a rate of from one heat source at a temperature of
1 OOkW and rejects heat to a sink at a rate of 127°C and 37 kJ from another heat source at
50kW. The entire work output of the heat 97°C. The heat rejected (in kJ) to the heat
engine is used to drive a reversible heat sink at 47°C is
pump operating between a set of (GATE-16- SET-2)
independent isothermal heat reservoirs at
I 7°C and 75 °C. The rate (in kW) at which

\( f I I H!,ll H l l l lll!, 1'11lil1, 1111111, Fydcrabad l Dclhil Bhopal l Pune l Bhubaneswarl LucknowlPatnal Bengaiuru l Oiennai lVuayawada jVmg ITll"Upali I Kukatpallyl Kolkala I
SOLUTIONS
06. Ans: (c)
One Mark Solutions Sol: 300 K

Q1 1000 W
01. Ans: (d)
=

...
Sol: =
250 W

W=30 kW 260 K

Q 1000
(COP)HP = I = =4
W 250

07. Ans: (a)


Q2 = Qi - W = 120 -30 = 90 kW
Sol: Clockwise devices are heat engines and anti
COP = Q 2 = go = 3 clockwise devices are heat pumps or
W 30
refrigerators.
02. Ans: (a)
08. Ans: 373.1 Watt
Sol: T 1 = 308K

T2 = 268K
04. Ans: (c)

05. Q T2
(COP) R = 2 =
W T1 -T2
Sol: f Bi < 0 for irreversible process. 268
2.5 ---
w 308-268
=
f Bi = 0 for reversible process. (w) Ref = i6� = 62.·75 = 0.3731kW = 373. lW
f Bi > 0 for impossible process. 40
: 459 : Second Law of Thermodynamics

09. Ans: 50
QA Qc
Sol: =
TA Tc
Two Marks Solutions

1 50 -
Q 0 1 . Ans: (a)
900
= C
300 300 x 1 0 3 kJ
-
Sol: = 3 .472 kW = 1 Ton
Q C = 50MJ 24 x 3600

10. Ans: 9.8 to 10.2 02. Ans: (c)


_ T1 _ 273 + 37 Sol:
Sol·• (COP)HP
- Tl - T2 - (273 + 37)- (273 + 6)
310 W 10
= = 10 ll E = - = -
31 Q5 80 - W=Q1-Q2= 1 0kJ
= 0. 125 = 1 2.5 %,---'-----..
Q2=7 0kJ

11. Ans: 4 (range 3.9 to 4.1)


T2=3 03 K

Sol: NRE = 7.2 kW


W = 1 .8 kW 373 - 303
= = 0. 1 87 or 1 8.7 %
NRE 7.2 = 373
COP = = 4
W 1 .8 ll E < ll c hence it is possible.

12. Ans: 1.67 03 . Ans: (b)

Sol: Sol: 11 0 = 11 1 + 112 - 111 . 112 (Overall efficiency of


coupled cycle)
T 1 = 300K

Q1 Q 1 = Q2+W = 0.3 + 0.2 - 0.3 0.2


= 0.44 or 44 %
X
= 1 0+ 1 5=25kW
W = 1 5kW

04. Ans: (a)


W 50
Sol: 11 = - => 0.75 = -
Qs Qs
T2 = 250K

Qs = 66.67 kJ
W = Q s - QR
� 50 = 66.67- QR
:. QR = 1 6.67 kJ

lit1•4i@jjji4ii!ARftni@h61..)1Iyderabad l Delhi ! Bhopal ! Punc I Bhubaneswar l LucknowI Patna! Bcngaiwu l ChennaiI Vtjayawada l Vmg I TIIU� l Kuµtpallyl KolkataI
t . : 460 :
.t\CE . .
�.r�.�:�Publicabrm Thermodynamics
================================
05. Ans: (b) T, - T2 350-3 1 3
2_ 5 = ( ) x Q, = ( x Q1
T2 � 350 )
Sol : 11E - -- � 0.75 = 1 - - ;
- T1 - T2
T, Ti = 0. 1 057 1 Q i

0.25 = � · Q i = 23 .65 kW
1 000 Q1 23 · 65
Area = = = 78.83 m 2
T2 = 250 K = - 23 C 11 c 0.6 X 0.5
°
Cd X

08. Ans: (d)


Sol: Sol: T 1 = 400 K
Q+W

T2 = 350 K

100 - Q
Q2
w
(COP)R = 5 = = -2L_
W 0.7 Q
Q2 = 3.5 Q = 3.5 X 1 = 3.5 kJ
( 100 - Q -W)
T2=300 K

07. Ans: (d)


Sol: W 350-300
Carnot, 11 E = = ---
T 1 = 350K 1 00-Q 350
1 00-Q
:. W =

Q+W 400 T,
W = 2.5kW

( COP ) = = =
HP w 400-350 T,-T2

_g_ = 7 · Q = 7W
w
cd = Collector density = 0.6 kW/m2 7(1 0 - Q)
Q= � ; Q = 50 kJ
11c = Collector efficiency = 0.5 Q 50
W= = = 7. 1 43 kJ
W = 11 Q 1
Heat Rejected = Q + W = 57 . 1 43 kJ
7 7

!lfll4j@jjji4i!i4Rfl§1Mijjih+ydcrabadlDelhi l Bhopal I Punc I Bhubancswarl Lucknow l Patna l Bcngahuu IChcnnaijVtjayawadaj Vmg ITuupati I Kukatpallyl Kolkala I
09. Ans: (c)
Sol: High T3 = 273+75
temperature =348 K
source

--2 Q2 --1
+- =0
Irreversible
1000 800 400
Q2 = - 1 2 5 -4
- --- = --
800 400 1000 2000
800
:. Q z= -- = 0.4kJ
2000
Low T4 = 273+ 1 7
temperature = 290 K
sink Wnet=Q i+ Q 2 -Q3

= 2 + 0.4 -1 = l .4kJ
Q i = 100 kW ; Q2 = 50 kW ;
Work 'W' = 100 - 50 = 50 kW 1 1 . Ans: 64 (range 63 to 65)
T3 Sol:
Q
(COP) H.P = 3 = T 1 = 400 K
w T3 -T4
Q 3 = 348
=6
50 348-290
Q3 =300 kW

10. Ans: (c)


Sol:
Applying Classius inequality
T 1 = 1000K T2=800K o
f i =O
2L + Q 2 - Q 3 = 0
R t----� T1 T2 T3
Q3= lkJ 40 37 Q 3
+ - =0
400 370 320
T3=400K 0.1 + 0.1 - Q 3 = 0
Engine is reversible hence
Q3 =0.2X 320= 64 kJ
320

!ltlll@jjji4.jjQjmft5i!lijj+yderabadj Delhi ! Bhopal I Pune I Bhubaneswarj LucknowI Patna I Benga]uru I Chennai jVtjayawadaj V17.3g j Tirupati I Kukalpallyj Kolkata I
cs Entropy
m entropy for the heater is ___ J/K and
One Mark Questions for water ___ J/K.
(GATE-ME -94)
O 1 . Which among the following relations is/are
valid ONLY for reversible process 05. For an ideal gas the expression
undergone by a pure substance?
(GATE-ME -93)
[r( ;; ), -T( ;;)J is always equal to:

(a) 8Q = dU + 8W (GATE-ME-97)
(b) T dS = dU + 8W (a) Zero (b) Cp I Cv (c) R (d) RT
(c) T dS = dU + p dv
(d) oQ = p dV + dU 06. A system undergoes a state change from 1 to
2. According to the second law of
02. When a system executes an irreversible thermodynamics, for the process to be
cycle: feasible, the entropy change, S2 - S1 of the

(a) f 8,i < 0 (b) ds > O


f system
(a) is positive or zero
(GATE-ME -97)

(c) f ds = o (d) f oi > O (b) is negative or zero


(c) is zero
(d) can be positive, negative or zero
03 . The slopes of constant volume and constant
pressure lines in the T-s diagram are __ 07. Which of the following relationships is valid
and __ respectively. (GATE-ME -94) only for reversible process undergone by a
closed system of simple compressible
04. A 1 500 W electrical heater is used to heat 20 substance (neglect changes in kinetic and
kg of water (Cp = 4 1 86 J/kg K) in an
insulated bucket, from a temperature of 30°
potential energy? (GATE-ME - 07)
(a) oQ = dU + 8W (b) T dS = dU + p dV
C to 80 C. If the heater temperature is only
(c) T dS =dU + ow (d) oQ = dU + p dV
°

infinitesimally larger than the water


temperature during the process, the change

lmlQj/§jjjl41hi4kfifi!Mi!iih+yderabad I Delhi I Bhopal I Punc I Bhubaneswar I Lucknow I Patna I Bcngaluru I Chcnnai I Vuayawada I Vmg I T1JUpari I Kukatpally I Kolkata I
".!'....,..,t"' �
...
ACE . .
Pl� . . : 463 : Entropy

08. 2 m oles of oxyg en are m ixed adiabatically 0


(a) f oQ > Oand f ; < 0
with another 2 m oles of oxyg en in m ixing
c ham ber, so that t he final total pressure and 0
(b) f oQ < Oand f ; < 0
tem perature of the m ixture becom e sam e as
those of the individual constituents at their 0
(c) f oQ > Oand f ; > 0
initial states. The universal g as constant is
g iven as R. The chang e in entropy due to 0
(d) f oQ < Oand f ; > 0
m ixing , per m ole of oxyg en, is g iven by
(GATE-ME -08)
(a) -R /n2 (b) 0
(c) R /n 2 (d) R ln4
Two Marks Questions

09. If a closed system is underg oing an 01. Fig ure below s hows a reversible heat eng ine
irreversible process, the entropy of the ER having heat interactions with three
system (GATE-ME -09) constant tem perature system s. Calculate the
(a) m ust increase therm al efficiency of the heat engine.
(b) always rem ains constant
(c) m ust decrease
(d) can increase, decrease or rem ain constant

10. One kilog ram of water at room tem perature


is broug ht into contact with a hig h
tem perature therm al reservoir. The entropy (GATE-ME-93)

c hang e of the universe is (GATE-ME -10)


(a) equal to entropy c hang e of the reservoir 02. One kilom ole of an ideal g as is throttled
(b) equal to entropy chang e of water from an initial pressure of 0.5 MPa to 0.1
(c) equal to zero MPa. T he initial tem perature is 300 K. The
(d) always positive entropy c hang e of the universe is
(GATE-ME-95)

11. Whic h one of the following pairs of (a) 13.38 kJ/K (b)40 1 4.3 kJ/K
(c) 0.0446kJ/K (d) -0.0446 kJ/K
equations describes an irreversible heat
eng ine? (GATE-ME-14- SET-3)

\l I I 11gu1t , 1111� P1ilil1l 111( n i , yderabad l Delhi l BbopaI I Pune l Bhubaneswarl Lucknow l Patna l Bengaiwu l Cbennai l Vtjayawadal V17.3g j 'firupali I Kukatpally l Kolkatl
�•-!'FJllll'N'ff'W Publiarima
" �CE
111 •
: 464 :
• •
Thermodynamics

� � ====================================
03 . Considering the relationship 06. A vapour absorption refrigeration system is
TdS = dU + PdV between the Entropy (S), a heat pump with three thermal reservoirs as
Internal energy (U), Pressure (P), shown in the figure. A refrigeration effect of
temperature (T) and Volume (V), 1 00 W is required at 250 K when the heat
Which of the following statements 1s source available is at 400 K. Heat rejection
correct? (GATE-ME-03) occurs at 300 K. The minimum value of heat
(a) It is applicable only for a reversible required (in W) is (GATE-ME-05)
process 1400K I
(b) For an irreversible process,
TdS > dU + PdV
(c) It is valid only for an ideal gas
( d) It is equivalent to first law, for a
reversible process 250K
(a) 1 67 (b) 1 00 (c) 80 (d) 20
Common Data for Question 04 & 05
Nitrogen gas (molecular weight 28) is enclosed in 07. A cyclic device operates between three
a cylinder by a piston, at the initial condition of 2 thermal reservoirs, as shown in the figure.
bar, 298 K and 1 m3 • In a particular process, the Heat is transferred to/from the cyclic device.
gas slowly expands under isothermal condition, It is assumed that heat transfer between each
until the volume becomes 2m3 . Heat exchange thermal reservoir and the cyclic device takes
occurs with the atmosphere at 298 K during this place across negligible temperature
process. difference. Interactions between the cyclic
device and the respective thermal reservoirs
04. The work interaction for the Nitrogen gas is that are shown in the figure are all in the
(GATE-ME-03) form of heat transfer.
(a) 200 kJ (b) 1 38.6 kJ (GATE-ME-08)
(c) 2 kJ (d) - 200 kJ SOO K
I OOO K 300 K

05 . The entropy change for the system during 50 kJ


the process in kJ/K is (GATE-ME -03)
(a) 0.4652 (b) 0.0067
100 kJ 60 kJ
(c) 0 (d)-0.67 1 1

jM11M!i§hiiiihi@nii\jjMij.Hyderabad I Delhi I Bhopal I Pune I Bhubaneswar I I..ucknowl PatnaI Bengaluru I Chennai !Vliayawada I Vizag I Ttrupali I Kukatpally I Kolkata I
., " . �CE
. . . : 465 : Entropy
�-�
�. � ==================================
��Nwa,ims

The cyclic device can be Specific heat at constant pressure,


(a) a reversible heat eng ine Cp = 1.005 kJ/kg K;
(b) a reversible heat pum p or a reversible Specific heat at constant volum e,
refrig eration Cv= 0.718 kJ/kg K;
(c) an irreversible heat eng ine C haracteristic g as constant, R = 0.287 kJ/kg K.
(d) an irreversible heat pum p or an Enthalpy h = CpT,
irreversible refrig erator Internal energ y, u = Cv T.

08. Consider the following two processes; 09. If the air has to flow from station P to station
(GATE-ME -10) Q, the m axim um possible value of pressure
(I) A heat source at 1200 K loses 2500 kJ in kPa at station Q is close to
of heat to a sink at 800 K (GATE-ME -11)
(II) A heat source at 800 K loses 2000 kJ (a) 50 (b) 87 (c) 128 (d) 150
of heat to a sink at 500 K
10. If the pressure at station Q is 50 kPa, the
Whic h of the following statem ents is true? c hang e in entropy (SQ - Sp) in kJ/kg K is
(a) Process I 1s m ore irreversible t han (GATE-ME -11)
process II (a) - 0.155 (b) 0
(b) Process II 1s m ore irreversible than (c) 0.160 (d) 0.355
process I
(c) Irreversibility associated in both the 11. An ideal g as of m ass m and tem perature T 1
processes are equal underg oes a reversible isotherm al process
(d) Bot h t he processes are reversible from an initial pressure P1 to final pressure
P2. T he heat loss during the process is Q.
Common Data for Question 09 & 10 The entropy chang e dS of the g as is
In an experim ental set-up, air flows between two (GATE-ME -12)
stations P and Q adiabatically. The direction of
flow depends on the pressure and tem perature (a) mR � :: ) (b) mR ln ( :: )
conditions m aintained at P and Q. The conditions
at station P are 150 kPa and 350 K. The (d) zero
tem perature at station Q is 300 K. The following
are the properties and relations pertaining to air:

\C l I m!,111t 1 1 1 11..,. P1d d u .111, 11i-. �ydcrahad Delhi Bhopal Pune Bhubancswar Lucknow Patna Bengaluru Chennai Vuayawada Vizag Tirupati
I I I I I I I I I I I I Kukatpal]y I Kolkata I
: 466: Thermodynamics

12. An amount of 100kW of heat is transferred 16. An ideal g as of mass m is contained in a


throug h a wall in steady state. · One side of rig id tank of volume V at a pressure P.
the wall is maintained at 127°C and the During a reversible process its pressure
other side at 27°C. The entropy g enerated reduces to P1 . Following statements are
(in W/K) due to the heat transfer throug h the made reg arding the process.
wall is ___ (GATE-ME -14- SET-3) (P) Heat is transferred from the g as.
(Q) Work done by the g as is zero.
13. A closed system contains 10 kg of saturated (R) Entropy of the g as remains constant.
liquid ammoma at 10°C. Heat addition (S) Entropy of the g as decreases.
required to convert the entire liquid into
Among the above statements, the correct
saturated vapour at a constant pressure is
ones are (GATE - PI-16)
16.2 MJ . If the entropy of the saturated
(a) P and R only
liquid 1s 0.88 kJ/kg .K, the entropy (in
(b) P, Q and R only
kJ/kg .K) of saturated vapour is ___
(c) Q and R only
(GATE- ME-14- SET-4)
(d) P, Q and S only

14. Two identical metal blocks L and M


17. Onekg of an ideal g as (g as constant R = 287
(specific heat = 0.4 kJ/kg .K), eac h having a
J/kg K
. ) underg oes an irreversible process
mass of 5 kg , are initially at 313 K. A
from state-I (1 bar, 300 K) to state-2 (2 bar,
reversible refrig erator extracts heat from
300 K). The chang e in specific entropy (s2-
block L and rejects heat to block M until the
s1) of the g as (in J/kg K
. ) in the process is
temperature of block L reac hes 293 K. The
final temperature (inK) of block M is __
(GATE-17- SET-2)
(GATE-ME-14- SET-4)
18. A reversible heat eng ine (E) operating in a
15. One kg of air (R= 287 J/kg -K) under g oes cycle interacts with three reservoirs 1, 2 and
irreversible process between equilibrium 3 maintained at temperature T 1 = 500 K,
state 1 (20°C, 0.9 m3) and equilibrium state T2 = 400 K and T3 = 300 K, respectively.
2 (20°C, 0.6 m3 ). The c hang e in entropy The eng ine receives 10 kJ of heat from
s2-s1 (in J/kg -K) is ___ reservoir 1 and rejects 3 kJ to reservoir 3.
(GATE-15-Set 2) The net work output, Wnet (in kJ ) from the
eng me 1s

li1•1ih§hii41hiAflfii@dfjj+ydcrabadJ Delhi I Bhopa! J Punc I BhubancswarJ Lucknow I Patna! Bcngaluru I Chennai J VtjayawadaJ Vizag JTuupati I Kukatpa!IyJ Kolkata I
"
��-�
" ACE . .
..�� PuNicatioos . . : 467 : Entropy

(1 ) (2) Five Marks Questions


T 1 = 500 K T2 = 400 K

01. An iron cube at a temperature of 400 deg C


is dropped into an insulated bath containing
1 0 kg water at 25 °C. The water finally
reaches a temperature of 50°C at steady
T3 = 300K state. Given that the specific heat of water is

(GATE-PI-17) equal to 4186 J/kgK, find the entropy


(3)

changes for the iron cube and the water. Is


the process irreversible? If so, why?
(GATE-ME -96)

!1ti•Qj@hii4ijjjQRflfti@bijj+yderabad l Dellii l Bhopal j Pune l Bhubaneswarl Lucknowj Patna j Bengaluru j Oiennai l Vuayawadaj V11.3g ITirupati I Kukatpallyl Kolkata I
SOLUTIONS
06. Ans: (d) 07. Ans: (d)
One Mark Solutions
08. Ans: (b)
01. Ans: (d) Sol: When similar gases are mixed
Sol: dQ = dU + Pd V for reversible process (dS)mixing = 0

02. Ans: (a) 09. Ans: (a)


03. Ans: higher , lower Sol: For irreversible process dS > 0
dT dT __
Sol: ( ) )
'( 10. Ans: (d)
T T

ds cv ds cp
Sol: (M tiv > 0 , Irreversible process
v=c P=C

11. Ans: (a)


04. Ans: O, 12.787 kJ/K Sol: Standard definition of irreversible Engine.
Sol: (dS)Heater = 0 (with work transfer
entropy change is zero) Two Marks Solutions

(dS)w,,., - m Cp In ( � J 01.
:
8
353 Sol: f ; = 0 For reversible engine
= 20 x 4.186 /n (-)
303
= 12.787 kJ/K � + Q 2 -Q 3 = 0
T1 T2 T3

05. Ans: (c) 100 50 _ Q 3 =

(:a -;,
+ O
1000 500 300
Sol:
(:t = ;v Q 3 = 0.2 . 3 = 60 kJ
300 ' Q
Cp = T (:l Cv = T (:t 150 -60
11 = 150 = 0.6 Or 60 %
:, Cp -Cv = R

!Mlli04iiiiiih401blM\hih*l1yderabad I Delhi I Bhopal I Punc I Bhubaneswar I Lucknow I Patna I Bengaluru I Chcnnai I Vtjayawada Iv� I T1111pati I K.ukatpal)y r Kolkata I
: 469 : Entropy

02. Ans: (a) Consider a heat engine is operating between

Sol: �S - n R In ( !:J T I and T2 and work output of the heat


engine is used to operate a heat pump
between T3 and T2.
05
1 X 8.314 /n ( · ) 13.38 kJ/K
0.1 f o; = O
= =

03. Ans: (d) 04. Ans: (b) Q, + Q3 _ Q2 = 0


T.. T3 T2
05. Ans: (a)
Sol: P1 = 2 bar = 200 kPa , ii_ + 100 - Q, +100 = 0
( )
400 250 300
T 1 = 298 K, V 1 = l m3
200xl Q, --
- Q, =-- 2 1
2.26 kg +-
8314 400 300 5 3
x298
28 3 Q, -4 Q, = -6+5 =-!_
=
1200 15 15
For Isothermal process
1200
, Q, - , W, - mRT, tn ( �: J Q, = 15 = sow

= 2.26 x (
8.314 2
) x298xln( ) = 138.61 kJ 07. Ans: (a)
28 1
Sol: f o Q = 0 for Reversible cycle
(·: R-�J T
Q, + Q2 - Q3 = 0
I Q 2 - 138.61 kJ T.. T2 T3
�S-S2 -S1 -- - - 0.4652
298
-

100 50 _ 60 =
+ O
T1 K
1000 500 300
06. Ans: (c) Cycle is Reversible; As it is a forward
Sol: T1 =
400K
device it is a heat engine.
Q2 = Q3 +W + Q1 - W
08. Ans: (b)
= Q3 + Q1

2500 _ 2500
Sol: ((�S) . ) I = _g_ _ _g_ =
uruv T2 TI 800 1200
= 3.125 - 2.083 = 1.042 kJ/K
T3 = 250K

\( f l 11�l'lt { l l !ll-, P1il>IH ,t\]Cl[ j', Fyderabad j Delhij BhopaI I Punc j Bhubaneswarl I.ucknow l Patna l Bengaluru j Chennai j VgayawadajV17.3g I Tirupati I Kukatpallyl Kolkata I
: 470: Thermodynamics

2000 _ 2000 So it is an irreversible process and flow is


((ilS) um.v ) IJ = _g_ _ _g_ =
T2 TI 500 800 from P to Q.
= 4 - 2.5 = 1.5 kJ/K
.". ((ilS) univ )II > ((ilS) univ )I 11. Ans: (b)

09. Ans: (b)


Sol: ( dS);roiliorm>l = mR 1n( �: = mR 1n(
J ::J
10. Ans: (c) 12. Ans: 83.3 WIK
Sol: ///////////)'"� Sol:
Pe Q= l OOkW
T1=400K
77777777777777
Q= l OOkW
P1 = 150 kPa P2 = ? T2=300K
T 1 = 350 K T2 = 300K
r;'
J
T2 (P2 sgen = -
Q --
Q
= T2 TI
T1 P,
100 - -
100

:: =(�: t = G��y--,
y

300 400
1.4 =-

= 0.3333 -0.25 = 0.0833 kW/K


= 83.3 WIK

13. Ans: 6.6044kJ/kg


300
150(-) = 87.45 � 87kPa
35

350 Sol: s g -S r = __g__


=

mT
P1 = 150 kPa, P2 = 50 kPa
Q
s g = S r +­
T1 = 350 K T2 = 300 K mT
T P
S Q -S P = C P 1n( 2 J - R 1n( 2 J = 0.88+
16.2x l0 3
TI pI lOx 283
300 50 = 0.88 + 5.7244 = 6.6044 kJ/kg.K
1.005 1n() -o.2811n( )
350 150
=

= -0.1549+0.3153 = 0.16>0
\( l I 11g111l 1 1111� P1 1J.ia .111t,1 1-. ydcrabadl Delhi I Bhopal ! Punc I Bhubaneswarl Lucknow I Patna! Bcnga)uru I Chennai I Vtjayawadal V.a.agj'[uupati I Kukatpallil Kolk.ata
: 471 : Entropy

14. Ans: 334.3K T2 V


=C In +R ln 2
Sol: T1 v1
T
V

Given , T = C
V
dS = S 2 - S 1 = R ln-2
v1

Tr
= 287 .en( 06
· = -116.368kJ / kg
0.9 )

Ti = 313K ; Tr = 293K 16. Ans: (d)


For minimum work done condition Sol: At constant volume to decrease pressure,
{dS)universe = 0 temperature has to be decreased. So heat has
(dS)1 = m C ln(� ) to be transferred from the gas.
As the change in volume is zero, work done
by the gas is zero. Entropy of the gas
decrease as system is losing heat.
(ds), = mC ln( � )
,

(ds)�,- = m C lt}} o 17. Ans: -198.93


Sol: m = 1 kg
Tr T R = 287 J /kgK
Rn( J = 0 = fol
Ti 2 P1 = 1 bar,
Tr T .2
T- P2 = 2 bar,
-- = 1 �T = -1
Ti 2 Tr T1 = 300 K,
T2 = 300K,
313 2 =
�T = 334.3K T 1 = T2 isothermal process
293
T2
S2 - SI = m(c p.en R(n p 2)
T1 P1
-

15. Ans: -117 to -115


Sol: dQ = dU + Pdv = mRRn-
�1

Tds = Cv dT + Pdv P2
dT 1
ds = C + p dv = 1 X 287.en - =-198.93 J /K
V T T
dT R
= C - +-dv
V T V
F� I Delhi I Bhopal I Pune I Bhubaneswarl Lucknow I Patna I Benga)uru I °'=1ai I Vgayawada I Vu.ag I ThuJ!ali I Kukatpal)yj Kollwa I
: 472 : T hermodynamics

18. Ans: 3 kJ
Sol:
=> IDi x C P; x(673 -323) = 1Ox4. l 86 x(323 -298)
(1)
kJ
mI x CP; = 2.99 -
T1 = 500 K

K
Cpw = 4. 1 86kJ/kg K

mw = 1 0 kg

T3 = 300K T; = 673K
(3 )
Reversible Engine

f ; =o
8

Tr = 323K

-9.!_ + Q 2 -Q 3 =0
T
T1 T2 T3 (8 S)·I mI x C P; /n ( r J
T
=

3
_!_Q_+ Q 2 _ __ = 0
I

500 400 300 323 kJ


2.99 /n (-) =-2.1949 -
673 K
=

0.01+ Q2 = 0
400 T
=> (8 S)water _
- IDw X C Pw X /n ( r J
Q2 = -4 kJ = 4 kJ (heat rejected) Tw
Wne1 = Q1 -Q2 -Q3
323 kJ
= 10 -4 -3 = 3 kJ = 10 X 4.186 /n = 3.372
298 K
kJ
(8S)univ = (8S) iron + (8 S) HzO = 1.1773 K
Five Marks Solutions
(8S)un iv > 0 so it is an irreversible process.
01.
Sol:
Heat lost by Iron = Heat gained by water

\( I I 11�1ru t 1 111.,, P1 dd1< ,1\11 111" Fyderabadl Delhi l Bbopai l Puncl Bhubancswarl Lucknowl PatnalBcnga)wu l Cbcnnai l VvayawadaJVma ITIIUpati I Kukatpallyl Kolkata I
I
Chapter
6 Properties of Pure substances

05. When wet steam flows through a throttle


valve and remains wet at exit
(GATE-ME -96)
One Mark Questions

(a) Its temperature and quality increase


0 1 . The relationship (of/8p)h = 0 holds good for (b) Its temperature decreases but quality
(GATE-ME-93) mcreases
(a) An ideal gas at any state (c) Its temperature increases but quality
(b) A real gas at any state decreases
(c) Any gas at its critical state (d) Its temperature and quality decrease
(d) Any gas at its inversion point
06. Water has a critical specific volume of
02. During the phase change of a pure 0.003 1 55 m3/kg. A closed and rigid steel
substance: (GATE-ME -93) tank of volume 0.025m3 contains a mixture
(a) dG = O (b) dP = O of water and steam at O. l MPa. The mass of
(c) dH = O (d) dU = 0 the mixture is 1 0 kg. The tank is now slowly
heated. The liquid level inside the tank.
03 . At the triple point of a pure substance, the (GATE-ME -07)
number of degrees of freedom is (a) will rise
(GATE-ME -93) (b) will fall
(a) O (b) l (c) 2 (d) 3 (c) will remain same.
(d) may rise or fall depending on the
04. Constant pressure lines in the superheated amount of heat transferred
region of the Mollier diagram will have
(GATE-ME -95) 07. A pure substance at 8 MPa and 400° C is
(a) A positive slope having a specific internal energy of 2864
(b) A negative slope kJ/kg and a specific volume of
(c) Zero slope 0.03432m /kg. Its specific enthalpy (in
3

(d) Both positive and negative slope kJ/kg) is __ (GATE-ME -14-SET-2)

FydcrabadlDclhil Bhopal l Punc l Bhubaneswarl LucknowlPatnal Bcnga)uru l ChcnnailVuayawadalVmg I Tirupati I Kukalpallyl Kolkala I
: 474: Thermodynamics

08. For an ideal g as with constant values of


specific heats, for calculation of the specific
Two Marks Questions
enthalply, (GATE -15 -Set 1)
(a) it is sufficient to know only the
01. A vessel of volum e 1.0 m 3 contains a
tem perature
m ixture of liquid water and steam in
(b) both tem perature and pressure are
equilibrium at 1.0 bar. Given that 90% of the
required to be known
volum e is occupied by the steam , find the
(c) both tem perature and volum e are
dryness fraction of the m ixture. Assum e, at
required to be known
1.0 bar, vr = 0.001 m 3/kg and vg = 1.7 m 3/kg .
(d) both tem perature and m ass are required
(GATE-ME -93)
to be known

09. A rig id container of volum e 0.5 m 3 contains 02. In the vicinity of the triple point, the vapour
1.0 kg of water at 120°C (Vr = 0.00106 pressures of liquid and solid am m onia are
m 3/kg , vg=0.8908 m 3/kg ). The state of water respectively g iven by
IS (GATE -15 -Set 3) 1n P = 15.16 -3063/T and
(a) Com pressed liquid 1n P = 18.70 - 3754/T
(b) saturated liquid Where p is in atm osp heres and T is in
(c) A m ixture of saturated liquid and Kelvin. Wh at is the tem perature at the triple
saturated vapor point. (GATE-ME -93)
(d) Super heated vapor
Statement for Linked Answer Q03 & Q04
10. The INCORRECT statem ent about the
c haracteristics of critical point of a pure
The following table of properties was printed out
substance is that
for saturated liquid and saturated vapour of
(GATE - 16 - SET - 3)
am m onia. T he titles for only the first two
(a) there IS no constant tem perature
colum ns are available. All that we know is that
vaporization process
the other colum ns (colum ns 3 to 8) contain data
(b) it has point inflection with zero slope
on specific properties, nam ely, internal energ y
(c) the ice directly converts from solid
(kJ/kg ), enthalpy (kJ/kg ) and entropy (kJ/kg .K).
p hase to vapor p hase
(d) saturated liquid and saturated vapor
states are identical
\( L l rn:,111u 1 J t 1� PulilH ,1111 111, Fyderabad l Delhi l Bhopal l Pune l Bhubancswarl Lucknowl Patna l Bengaluru l Chennai ! Vliayawada l Vizag ITuupari I Kukatpallyl KolkalaI
: 475 : Properties of Pure Substances

T(oC) P(kPa) containing 1kg of liquid and 0.03kg of vapour at


-20 1 90.2 88.76 0.3657 89.05 5 6155
.
1 299.5 1 4 1 8. 0 a pressure of lOOkPa. Initially, the piston rests on
0 429.6 179 69
. 0. 7 1 1 4 1 80.36 5 33 09
.
1 3 18.0 1 442.2
a set of stops, as shown in the fig ure. A pressure
of 200kPa is required to exactly balance the
20 857 5 272.89 1 .0408 274 .30 5. 0860 1332.2 1 460.2
.

weig ht of the piston and the outside atm ospheric


40 1554.9 272.89 1 .3574 371.43 4 .8662 134 1 0 1 470 .2
.

pressure. Heat transfer takes place into the system


03. The specific enthalpy data are in colum ns
until its volum e increases by 50%.Heat transfer to
(GATE-ME -OS)
the system occurs in such a m anner that the
(a) 3 and 7 (b) 3 and 8
piston, when allowed to m ove, does so in a very
(c) 5 and 7 (d) 5 and 8
slow(quasi-static / quasi-equilibrium ) process.
The therm al reservoir from which heat is
04. When saturated liquid at 40 ° C is throttled to
transferred to the system as a tem perature of
-20 ° C, the quality at exit will be 400°c. Averag e tem perature of the system
(GATE-ME -OS) boundary can be taken as 175°C. Heat transfer to
(a) 0.189 (b) 0.212 the system is lkJ , during which its entropy
(c) 0.231 (d) 0.788 increases by 10 J/K.

05. Given below is an extract from steam tables.


Atmospheric
pressure

Piston
Specific Volume
3
Enthalpy (kJ/kg)
Temp Psat (m /kg)
(OC) (bar) Saturated Saturated Saturated Saturated
liquid vapour liquid vapour
45 0.9593 0.001 0 1 0 1 5.26 1 88.45 2397.8
342.24 150 0.00 1 658 0.0 1 0337 1 6 1 0.5 26 1 .5 Stop System

Specific enthalpy of water in kJ/kg at 150


bar and 45 ° C is (GATE-ME -06)
Specific volum e of liquid (vr) and vapour (vg)
(a) 203.60 (b) 200.53
phases, as well as values of saturation
(c) 196.38 (d) 188.45
tem peratures, are g iven in the table below.
Common Data for Question 06 & 08
Saturation
Pressure
temperature, Vr Vg

In the fig ure shown, the system is a pure (kPa) ° (m 3


/kg) (m 3
/kg)
T ( C)
sa1

substance kept in a piston- cylinder arrang em ent. 100 100 0.001 0. 1


The system is initially a two- phase m ixture 200 200 0.0015 0.002

I
_9llW111,111111111,__
Qnjj'JIPl(IH�:·
_
.yderabad! Dellii ! Bhopal l Pune J Bhubaneswarl Lucknow! Patna! Bengaiwu J Chennai J VgayawadaJ V17.ag !Tuupati I Kukaq)a))yl Kolkata I
: 476 : Thermodynamics

06. At the end of the process, which one of the 08. The net entropy generation (considering the
following situations will be true? system and the thermal reservoir together)
(GATE-ME -08) during the process is closest to
(a) superheated vapor will be left in the (GATE-ME -08)
system (a) 7.5 J/K. (b) 7.7 J/K.
(b) no vapour will be left in the system (c) 8.5 J/K. (d) 10 J/K.
(c) a liquid + vapour mixture will be left in
the system 09. Steam enters a turbine at 30bar, 300° C
(d) the mixture will exist at a dry saturate (u = 2750 kJ/kg, h=2993 kJ/kg) and exits the
vapour state. turbine as saturated liquid at 15 kPa (u= 225
kJ/kg, h=226 kJ/kg). Heat loss to the
07. The work done by the system during the surrounding is 50 kJ/kg of steam flowing
process 1s (GATE-ME -08) through the turbine. Neglecting changes in
(a) 0.1 kJ (b) 0.2 kJ kinetic energy and potential energy, the
(c) 0.3 kJ (d) 0.4 kJ work output of the turbine (in kJ/kg of
steam) is ----
(GATE -15 -Set 3)

\( I I 11..,tl H t l !I!� P11ld1, , 1\Jt1!b �ydcrabad ! Dclhi ! Bhopal ! Pune ! Bhubaneswar l Lucknow ! Patna ! Bcngaluru l Chermai l Vuayawada l Vmg I T=pati I Kukalpal)y l Kolkat,I
SOLUTIONS
06. Ans: (a)

Sol: v, =spec1·ri1c volum e = -- = 0 .0025 -


One Mark Solutions 0·025 m3
10 kg .
01. Ans: (d) v 1 < v c (critical volum e ) after heating at
constant volum e it g oes into liquid reg ion
02. Ans: (a) & (b)
hence level of liquid in the vessel rises.
Sol: During p hase chang e dP is zero, dT is also
zero. 07. Ans: 3137.6 kJ/kg
dT=0, dP =0 Sol: h=u + Pv
dG =-SdT + VdP = 2864 + 8000 X 0.03432 =3138.56 kJ/kg
:. dG=O
08. Ans: (a)
03. Ans: (a) Sol: Cr = (dh)
Sol: F+P =C+2 ( Gibb's p hase rule) dt P=C
At triple point, P =3, C =1 =C p (dT)
:. F =1+2-3 =0
dh
Specific enthalpy 1s a function of
F =Deg rees of freedom , tem perature only. Hence only tem perature is
P =No. of p hases. enoug h.
C =No. of constituents.
09. Ans: (c)
04. Ans: (a) Sol: m =1 kg , V=0.5 m 3
Hint: dh T = V 0.5 m3
Spec1·ri1c vo1um e =- = - = 0.5 - =v
ds m 1 kg

05. Ans: (b)


Yr < v < vg

Sol: Hence t he steam is in vapour dom e.


h
Steam is partly in liquid and partly in

vapour.

s 10. Ans: (c)


.\('l. I 11g1J ll l l lll� P11h!H.tllll[l" I I
yderabad Delhi Bhopal I Pune I Bhubaneswarl Lucknow! Pama! Bengalwu I Qicnnai I VuayawadajVm,g !TU11pati I Kukalpallyl Kolkala
: 478 : Thermodynamics

h 1 =h2
Two Marks Solutions 371.43 = h12 + x2 (hg2 - h12 )
= 89.05+ X2(1418 - 89.05)
01. X2=0.212
Sol: Mass of liquid (m1) =
Ve O. l x1
= = I 00 kg
0.001
05. Ans: (a)
Yr

V 0.9xl Sol: Specific volume in compressed liquid region


Mass of steam (ms) = _s= -- = 0.5294 kg
vg 1.7 does not change with pressure as water is
0.5294 incompressible.
x = -m
----'---
s = --- -
100+ 0.5294 (u)4s0 c = (h r t c - P (v r ts c
0 0

= 5.266 X 10-3 = 188.45 - 0.9593 X 0.001010


= 188. 44 kJ/kg.

02. 1 sobar (h )4s0 c = (U)4s0 c + P (v r ts0 c


3063 3754
Sol: 15.16 - = 18.7 - = 188.44+ 150 X 100 X 0.00101
T T = 203.6 kJ/kg.
3754 - 3063 = 18.7 - 15.16
T
Common Solutions for 06 to 08:
3754 3063 691
T= - = = I 95_2 K. 06. Ans: (a) 07. Ans: (d)
18.7 - 15.16 3.54

03. Ans: (d) 08. Ans: (c)


Sol: Enthalpy values are greater than internal Sol: Mass of liquid = m1 = I kg
energy values. Mass of vapor = mv=0.03 kg
Q = Heat supplied = 1 kJ
04. Ans: (b) m
Dryness fraction, x = v
Sol: m e+ m v
0·03
= =0.029
1+ 0.03
V2= 1.5 V1
At state 1, P 1 = 100 kPa
v, =vn + x, (vg 1 - vn)
s Vt = 0.001+ 0.029 X (0.1 - 0.001)

!MIIQ/j4Qifflflii\!lijj� HyderabadJ DelhiJ Bhopal J Pune J BhubaneswarJ Bengaluru JLucknow J Patna ! Chcrmai J Vtjayawada J Vmg J T=pati J GuntnrJ JMatpally(Hyd)
: 479 : Properties of Pure Substances

V1 =3.871 X 10-3 m 3/kg S2- S1 = lO J /K.


V d
V = -I sgen = (82 - 81 ) - J ;
I

m = mv + m, = 1.03 kg 1000
= 10- = 8.5 J /K.
V1 = v1 x m = 3.871 x 10-3 x 1.03 673
= 3.987 X 10-3 m3
V2 = 1.5 X 3.987 X 10-3 09. Ans: 2717

Sol: h i + - = h 2 + -
8Q 8W
V2 = 5.98 x10-3 m3
dm dm
Vz = 0.00598
Vz = = 0.0058 m3/kg 8W h _ h 8Q
m 1.03 =( I 2)
+
dm dm
At 200 kPa, vg2 = 0.002 m3/kg
= (2993-226) - 50 (-: Q = -50)
As v2 > Vg2 => state (2) is in superheated
= 2993-276 = 2717 kJ / kg

W =P (V2-V1)
= 200 X (0.00598 -0.00398) = 0.4 kJ

Fyderabad l Delhi l BhopaJ I Pune l Bhubaneswar l Lucknow l Pa!na l Bengaluru J Oicnnai l Vuayawada J Vizag I TIIUpati I Knkatpallyl Kolkatl I
C7 Availability

One Mark Questions Two Marks Questions

01. Availability of a system at any given state is 01. A heat reservoir at 900 K is brought into
(GATE-ME -00) contact with the ambient at 300 K for a short
(a) a property of the system time. During this period 9000 kJ of heat is
(b) the maximum work obtainable as the lost by the heat reservoir. The total loss in
system goes to dead state availability due to this process is
(c) the total energy of the system (GATE-ME -95)
(d) the maximum useful work obtainable as (a) 18000 kJ (b) 9000 kJ
the system goes to dead state (c) 6000 kJ (d) None of above

02. The maximum theoretical work obtainable, 02. A steel billet of 2000 kg mass is to be cooled
when a system interacts to equilibrium with from 1250 K to 450 K. The heat released
a reference environment, is called during this process is to be used as a source
(GATE-ME-14- SET-1) of energy. The ambient temperature is 303 K
(a) Entropy and specific heat of steel is 0.5 kJ/kg K. The
(b) Enthalpy available energy of this billet is
(c) Exergy (GATE-ME -04)
(d) Rothalpy (a) 490.44 MJ (b) 30.95 MJ
(c) 10.35 MJ (d) 0.10 MJ
03. A source at a temperature of 500K provides
1000 kJ of heat. The temperature of 03. A gas expands in a frictionless piston­
environment is 27° C. The maximum useful cylinder arrangement. The expansion
work (in kJ) that can be obtained from the process is very slow, and is resisted by an
heat source is ambient pressure of 1OOkPa. During the
(GATE-ME -14- SET-3) expansion process, the pressure of the
system (gas) remains constant at 300 kPa.
The change in volume of the gas is O.O lm3 •
\( l } 1 1...., 1 1 1 ( < l l l l _ P1d1l1( .1!1111 1-. �ydcrabad J Dclhi J BhopaI J Pune J Bhubancswar J Lucknow J Patna J Bengaluru J Chennai J Vgayawada J Vizag J luupati J Kukalpally J Kolkata I
.,· ·" ACE . .
�•:�Publbriooa
. . : 481 : Thermodynamics

The maximum amount of work that could be


utilized from the above process is
(GATE-ME -08) Five Marks Questions
(a) 0 kJ (b) 1 kJ
(c) 2 kJ (d) 3 kJ 01. At a place where the surroundings are at 1
bar, 27°C, a closed rigid thermally insulated
04. The pressure, temperature and velocity of air tank contains 2 kg air at 2 bar, 27° C. This
flowing in a pipe are 5 bar, 500 K and 50 air is then churned for a while, by a paddle
mis, respectively. The specific heats of air at wheel connected to an external motor. If it is
constant pressure and at constant volume are
given that the irreversibility of the process is
l.005kJ/kgK and 0.718 kJ/kgK,
100 kJ, find the final temperature and the
respectively. Neglect potential energy. If the
increase in availability of air. Assume for air
pressure and temperature of the
surroundings are 1 bar and 300 K, Cv = 0.718 kJ/kg K (GATE-ME -97)
respectively, the available energy in kJ/kg of
the air stream is (GATE-ME -13)
(a) 170 (b) 187
(c) 191 (d) 213

05. One side of a wall is maintained at 400K


and the other at 300K. The rate of heat
transfer through the wall is 1000W and the
surrounding temperature 1s 25° C .
Assuming no generation of heat within the
wall, the irreversibility (in W) due to heat
transfer through the wall is __
(GATE-15-Set 3)

\('I· I· IH;lllt'<'I !Iii; 1'11hh, ,111< lll, �Hyderabad I New Delhi I Bengaluru I Bhubaneswar I Vtjayawada I Visakhapatnam I Tirupati I Pune J Chermai
SOLUTIONS

One Mark Solutions


Sol: AE - m C , (T, - T, ) - m c , T, In G:J
= 2000 0.5(1250 - 450)
0 1 . Ans: (b)
X

- 2000x 0.5 X 303 X ln --


1 250
450
02. Ans: (c) = 490.44 MJ
Sol: Available Energy is called Exergy
03 . Ans: (c)
03. Ans: 400 kJ Sol: Wu = (P -Po)(V2 -V 1 )
Sol: Wusefat = Q(Ti - T2 ) = Wcamo1 = (300 - 1 00)(0.0 1 ) = 2kJ
T.I
1 000 (500 - 300)
= = 400kJ
500

Two Marks Solutions

0 1 . Ans: (c)
Sol:
Total loss in available energy = Irreversibility 300 1
= 1 .005 ln - - 0.287 1n -
= To X �s

(_g_ - _g_J
500 5
( s2 - S1 ) = -0.5 1 34+0.46 1 9 = - 0.05 1 5
= T0
T2 T1 ( s1 - S2) = + 0.05 1 5

9000 _ 9000 50 2
= 300( ) = 6000 kJ AE = [0. 05(500 - 300 ) + ]
300 900 2000
- 300x(0.05 1 5)
02. Ans: (a) = 20 1 + 1 .25- 1 5 .45
= 20 1 .25- 1 5.45 = 1 87 kJ/kg

Fyderabadl Delhil Bhopall Punel Bhubancswarl Lucknowl ParnalBengaluru l CbennailVliayawadal V,zag I Tlrupari I Kukalpal)yi KolblaI
: 483 : Availability

05. Ans: 247 to 249


= 300 x 2 x 0. 7 1 8 In ( T
Sol: Irreversibility = I = T0 S gen 300
2)

= T0 (_g_ _ _g_J
T2 T1
In (
T
300
2 ) = 0.2321

= 298[ 1 000 _ 1 000 ] T2


= eo.232 1
300 400 300
0·2321 =
= 248.33 W T2 = 300 x e 3 78.3 7 K
T2 = 1 05.3 7 C
°

Increase in Available Energy


Five Marks Solutions

01.
Sol: I = To . (�S)univ
= m [ c. (T, -Ti ) - C, T0 ( �'. £n J]
3
Sgen = TO (�Ssys + �S surr) = 2x [0.7 ! !{37&37-300)-0.7 1 8x 300x l{ :�� )]
7

(�S)surr = 0 (Insulated)
= 1 2.56 kJ
I = To X (�S)sys .

I 00 = T0 x m x C, In ( �: J

\( I } lll.!,lllt t 1111g P1il 11It ,tl!OII'- �ydcrabad l Delhi l Bhopalj Puncj Bhubancswarl LucknowlPatnalBcngalurul Chennai j VliayawadajVizag �Tirupali f �J Kolkata I
I8
Chapter
Air cycles
03. Which one of the following is NOT a
necessary assumption for the air-standard
(GATE-ME -08)
One Mark Questions
Otto cycle?
01. A cycle consisting of two reversible (a) All processes are both internally as well
isothermal processes and two reversible as externally reversible.
isobaric processes is known as (b) Intake and exhaust processes are
(GATE-ME -96) constant volume heat addition process.
(a) Atkinson cycle (c) The combustion process is constant
(b) Stirling cycle volume heat addition process.
(c) Brayton cycle (d) The working fluid is an ideal gas with
(d) Ericsson cycle constant specific heats.

02. A P-v diagram has been obtained from a test 04. Air enters a diesel engine with a density of
on a reciprocating compressor. Which of the 1.0 kg/m3.The compression ratio is 21. At
following represents that diagram? steady state, the air intake is 30x 10-3 kg/s
(GATE-ME -05) and the net work output is 15 kW. The mean
(a) effective pressure (in kPa) is ___
P (b)
Pout
p (GATE-15-Set 1)

---u_
Pout -

05. An air-standard Diesel cycle consists of the


following process:
1-2: Air is compressed isentropically.
V V0 V

2-3: Heat is added at constant pressure.


(c)
Pout
p (d)
p
--n 3-4: Air expands isentropically to the
original volume.

-1�
Pout

-- 1� 4-5: Heat is rejected at constant volume.

If r and T denote the specific heat ratio and


Pm - -
V
I

V
temperature, respectively, the efficiency of
the cycle is (GATE-15-Set 3)
yderabad I Delhi I Bhopal I Pune I Bhubaneswar I Lucknow I Patna I Benga)uru I Oicnnai I Vliayawada I V.u.ag I Tirupali I Kukatpally I Kolkata
: 485 : Air Cycles

T4 -Ti
(a) 1 -
T3 -T2
T4 -T1
Two Marks Questions
(b) 1-
y (T3 -T2 )
(T4 -Ti ) 01. In air - standard Otto cycle the terminal
(c) 1 y pressures at the end of compression, heat
T3 -T2
release and expansion are respectively P2,
T4 -T1
(d) 1- P3 and P4. If the corresponding values are
(1-1XT3 -T2 )
P2', P3', and P4', taking into account the
effect of variable specific heat and
06. Propane (C3Hs) is burned in an oxygen
dissociation of the working fluid, then.
atmosphere with 10% deficit oxygen with
(GATE-ME -89)
respect to the stoichiometric requirement.
Assuming no hydrocarbons in the products, (a) P2 < P2' and P3 > P3',
the volume percentage of CO in the products (b) P3 < P3' and P4 > P4',
1s ___ (GATE - 16 - SET - l) (c) P2 > P2', P3 > P)' and P4 < Pi
(d) P2 >P2', P3 > P3'
07. The figure below shows an air standard
Diesel cycle in p-V diagram. The cut-off 02. An air -standard Diesel cycle consists of:
ratio is given by: (GATE - PI-16) (GATE-ME -90)
(a) Two adiabatic and two constant
volume processes

i (b) Two constant volume and two


isothermal processes.
(c) One constant pressure, one constant
p

volume and two adiabatic processes.


4

V - (d) One constant pressure, one constant


volume and two isothermal processes.

(a) 2 (b) �
V1 V1 03. The Figure below shows a thermodynamic
cycle undergone by a certain system. Find
(c) V 3 (d) 2 the mean effective pressure in N/m
V2 V3
(GATE-ME -93)
\{ I } ! l;...,lll( l / lll� P11lil1l,llit>!I" yderabad I Delhi I Bhopal I Pune I Bhubaneswar I Lucknow I Patna I Bengaluru I Chcnnai I Vliayawada I Vizag I TU"Upati I Kµkatpally I Kolkala
: 486: T hermodynamics

06. In order to bum 1 kilogram of CRi


p completely, the minimum number of
kN/m2
kilograms of oxygen needed is (take atomic
I
I
500
-----� l weights of H, C and O as 1, 12 and 16
200 - - - - - � respectively) (GATE-ME -95)
(a) 3 (b) 4 (c) 5 (d) 6
0.01 0.03 V(m ) 3

07. Consider a two stage reciprocating atr


04. Isentropic compression of saturated vapour compressor with a perfect intercooler
of all fluids leads to superheated vapour. operating at the best intermediate pressure.
(True/False) (GATE-ME -94) Air enters the low pressure cylinder at 1 bar,
27°C and leaves the high pressure cylinder
05. List-I (Heat Engines) (GATE-ME -95) at 9 bar. Assume the index of compression
A. Gas Turbine and expansion in each stage is 1 .4, and that
B. Petrol Engine for air R = 286.7 J/kgK. The work done per
C. Stirling Engine kg air in the high pressure cylinder is
D. Diesel Engine (GATE-ME -97)
(a) 1 1 1 kJ (b) 222 kJ
List - II (Cycles) (c) 37 kJ (d) 74 kJ
1. Constant volume heat addition and 08. Air (Cp= 1 kJ I kg, r = 1.4) enters a
constant volume heat rejection
compressor at a temperature of 27 ° C. The
2. Constant pressure heat addition and
compressor pressure ratio is 4. Assuming an
constant volume heat rejection
efficiency of 80%, the compressor work
3. Constant pressure heat addition and
required in kJ/kg is (GATE-ME-98)
constant pressure heat rejection
(a) 160 (b) 1 72 (c) 182 (d) 225
4. Heat addition at constant volume
followed by heat addition at constant 09. A single-acting two-stage compressor with
temperature. heat rejection at constant complete inter cooling delivers air at 1 6 bar.
volume followed by heat rejection at Assuming an intake state of 1 bar at l 5 °C,
constant temperature the pressure ratio per state is
(GATE-ME -01)
(a) 1 6 (b) 8 (c) 4 (d) 2
\( I I llJ:,lllt l 1 111::, P11lili{ ,llH)II', Fyderabad I Dellii I Bhopal I Pune I Bhubaneswar I Lucknow I Patna I Bengalwu I Otennai I Vtjayawada I Vizag I Tuupati I Kukatpal)y I Kolkata I
., . .
" ACE . .
':,.....��Pulilirama
. : 487 : Air Cycles

10. In a spark ignition engine working on the (a) 879.1 kJ (b) 890.2 kJ
ideal Otto cycle, the compression ratio is (c) 895.3 kJ (d) 973.5 kJ
5.5. The work output per cycle (i.e., area of
the P- V diagram) is equal to 23.625xI05xVc 14. A reversible thermodynamic cycle
J, where Ve is the clearance volume in m3 • containing only three processes and
The indicated mean effective pressure is producing work is to be constructed. The
(GATE-ME -01) constraints are (GATE-ME -05)
(a) 4.295 bar (b) 5.250 bar (i) there must be one isothermal process,
(c) 86.870 bar (d) 106.300 bar (ii) there must be one isentropic process,
(iii) the maximum and minimum cycle
11. An ideal air standard Otto cycle has a pressures and the clearance volume are
compression ratio of 8.5 If the ratio of the fixed,
specific heats of air (y) is 1.4, what is the (iv) polytropic processes are not allowed.
thermal efficiency in percentage of the Otto Then the number of possible cycles are
cycle? (GATE-ME -02) (a) 1 (b) 2 (c) 3 (d) 4
(a) 57.5 (b) 45.7 (c) 52.5 (d) 95
Common Data for Question 15 & 16
12. For an engine operating on air standard Otto In two air standard cycles - one operating on the
cycle, the clearance volume is 10% of the Otto and the other on the Brayton cycle - air is
swept volume. The specific heat ratio of air isentropically compressed from 300 to 450 K.
is 1.4. The air standard cycle efficiency is Heat is added to raise the temperature to 600K in
(GATE-ME -03) the Otto cycle and to 550 K in the Brayton cycle.
(a) 38.3% (b) 39.8 %
(c) 60.2% (d) 61.7% 15. If TJo and TJs are the efficiencies of the Otto
and Brayton cycles, the (GATE-ME -05)
13. An engine working on air standard Otto (a) TJo =0.25, TJs =0.18
cycle has a cylinder diameter of 10 cm and (b) TJo=TJ B=0.33
stroke length of 15 cm. The ratio of specific
(c) TJo=0.5, TJs =0.45
heats for air is 1.4. If the clearance volume
(d) It is not possible to calculate the
is 196.3 cc and the heat supplied per kg of
efficiencies unless the temperature after
air per cycle is 1800 kJ/kg, then work output
the expansion is given
per cycle per kg of air is (GATE-ME -04)

�ydcrabad I Delhi I Bhopal I Punc I Bhubancswar I Lucknow I Patna I Bcnga)uru I Chcnnai IVuayawada I Vmg I 'Hrupari I Kukatpally I Kolkala I
� l�PnNirJMDI
ACE Thermodynamics
4
"

.�
V • • • •
: 488 :

16. If W0 and W8 are work outputs per units 18. The Stroke and bore of a four stroke spark
mass, then (GATE-ME -05) ignition engine are 250mm and 200mm
(a) Wo > Ws respectively. The clearance volume is
(b) Wo < Ws 0.001m3 • If the specific heat ratio y = 1.4,
(c) Wo = Ws the air-standard cycle efficiency of the
(d) It is not possible to calculate the work engine is (GATE-ME-07)
outputs unless the temperature after (a) 46.40% (b) 56.10%
. . .
expansion is given (c) 58.20% (d) 62.80%.

17. Group I shows different heat addition Common Data for Question 19 & 20
processes in power cycles. Likewise, Group A thermodynamic cycle with an ideal gas
II show different heat removal process. working fluid is shown below.
Group III lists power cycles. Match items
from Groups I, II and III (GATE-ME -06)
p
3
400kPa

Group I Group II Group III


P. Pressure S. Pressure 1 . Rankine cycle
constant constant
Q. Volume T. Volume 2. Otto cycle lOOkPa - - - -i,
I
,1
I
constant constant I I

3. Carnot Cycle lm
R. Temperature U. Temperature
4. Diesel cycle
constant constant
5. Brayton cycle
19. The above cycle is represented on T-S plane
by (GATE-ME -07)

(a) P-S-5 (b) P-S-1 (a) (b)


R-U-3
i
T
R-U-3
T�
r
P-S-1 P-S-4
Q-T-2 P-T-2 �
s

(c) R-T-1 (b) P-T-4


P-S-1 R-S-3 (c) T � (d) 3
P-T-4 P-S- 1
T

Q-S-5 P-S-5 d1

\( I I 11�1111 t 1 111� P11lili1 .111011, �yderabad I Delhi I Bhopal I Punc I BhubancswarI Lucknow I Patna I Bcngaluru I Chcnnai I Vijayawada I V123g I TIIUpati I Kukatpally I Kolkata I
: 489 : Air Cycles

20. If the specific heats of the working fluid are 24. A diesel engine has a compression ratio of
constant and the value of specific heat ratio 17 and cut-off takes place at 10% of the
y is 1.4, the thermal efficiency (%) of the stroke. Assuming ratio of specific heats (y)
cycle is (GATE-ME -07) as 1.4, the air-standard efficiency (in
(a) 21 (b) 40.9 (c) 42.6 (d) 59.7 percent) is----
(GATE-ME -14- SET-3)
21. In an air-standard Otto cycle, the 25. In a compression ignition engine, the inlet
compression ratio is 10. The condition at the air pressure is 1 bar and the pressure at the
beginning of the compression process is end of isentropic compression is 32.42 bar.
1 OOkPa and 27°C. Heat added at constant The expansion ratio is 8. Assuming ratio of
volume is 1500 kJ/kg, while 700 kJ/kg of specific heats (y) as 1.4, the air standard
heat is rejected during the other constant efficiency (in percent) is ---
volume process in the cycle. Specific gas (GATE-ME -14- SET-4)
constant for air = 0.287 kJ/kgK. The mean
effective pressure (in kPa) of the cycle is 26. In an air-standard Otto cycle, air is supplied
(GATE-ME -09) at O. lMPa and 308 K. The ratio of the
(a) 103 (b) 310 (c) 515 (d) 1032 specific heats (y) and the specific gas
constant (R) of air are 1.4 and 288.8 J/kg.K,
22. A turbo-charged four-stroke direct injection respectively. If the compression ratio is 8
diesel engine has a displacement volume of and the maximum temperature in the cycle
0.0259m 3 (25.9 liters). The engine has an is 2660 K, the heat (in kJ/kg) supplied to the
output of 950 kW at 2200 rpm. The mean engme 1s ___ (GATE-ME -14- SET-l)
effective pressure (in MPa) is closest to
(GATE-ME -10) 27. For the same values of peak pressure, peak
(a) 2 (b) 1 (c) 0.2 (d) 0.1 temperature and heat rejection, the correct
order of efficiencies for Otto, dual and diesel
23. The crank radius of a single-cylinder I.C. cycles is (GATE-15-Set 2)
engine is 60mm and the diameter of the (a) 11otto >
11dual> 11Diesel
cylinder is 80 mm. The swept volume of the (b)
(GATE-ME -12)
11diesel >
11Dual >
11otto
cylinder in cm3 Is
(C) 11Dual 11Diesel 11otto
(a) 48 (b) 96 (c) 302 (d) 603
> >

(d) 11 diesel >


11 otto >
11 dual

\( l l 11L!,11H t 1 1111.!. P11lil1t .1111111, ydcrabad I Delhi I Bhopal I Pune I Bhubaneswarl LucknowI Patna I Bengaluru I Chennai J Vuayawada J Vizag J 'firupati I Kukatpal)yl Kolkata
�-t . .t\CE . .
� ..,:'�:&i�..,
!:- �,1,�ili�d
::•�t1�Pu
490 Thermod anncs
�-�cci:•�=======�:��:.;.:==============,,::;;:,;�:yn
�:;:'�
04. An air standard Otto cycle has a volumetric
compression ratio of 6, the lowest cycle
Five Marks Questions pressure of 0.1 MPa and operates between
temperature limits of 27° C and 1569° C.
01. The exhaust gas composition obtained from (GATE- ME-94)
an Orsat apparatus in a test on a spark (a) Calculate the temperature and pressure
ignition engine is as follows: CO2 = 11 after the isentropic expansion (Ratio of
percent , 02 = 0.4 percent and CO = 9 specific heats = 1.4).
percent. If the fuel is !so-Octane (Cs H is). (b) Since it is observed that values in are
Determine the Air/Fuel ratio of the mixture well above the lowest cycle operating
supplied to the engine. Assume molecular conditions, the expansion process was
weight of Air = 29, H2 = 2.0 and C = 12.0 allowed to continue down to a pressure
Molar ratio of Nitrogen to oxygen in air is of 0.1 MPa. Which process is required
3.76 (GATE-ME -89) to complete the cycle? Name the cycle
so obtained.
02. A four cylinder, four stroke compression (c) Determine by what percentage the
ignition engine develops an indicated power cycle efficiency has been improved.
output of 150 kW at 150 rpm. The stroke is
1.5 times the bore and the indicated mean 05. Propane (C3 Hs) is used as a fuel in an
effective pressure (imep) is 6.0 bar. engine with 30% excess air. Assuming
Determine the dimensions of the cylinder. complete combustion determine the
(GATE-ME -90) composition of exhaust gases on mass basis.
Atomic weights are C = 12, 0 = 16, N = 14,
03. A double acting single stage compressor H = 1. Molar ratio of nitrogen to oxygen is
running at 200 rpm has an average piston 3.76. (GATE -ME-94)
speed of 3m/s. Its indicated horse power is
50 hp while compressing air from 1 bar to 6 06. The minimum pressure and temperature in
bar with a compression index of 1.2. If the an Otto cycle are 100 kPa and 27°C. The
temperature of air at inlet is 20°c, determine amount of heat added to the cycle is 1500
the dimensions of the cylinder neglecting kJ/kg. Determine the pressures and
clearance. (GATE-ME - 91) temperatures at all points of the air standard
Otto cycle. Also calculate the specific work

Fyderabad ! Delhi l BhopaI ! Pune l Bhubancswarl wcknow l Patna ! BengaJuru l Chennai l Vtjayawada l Vmg I T=pati I Kukatpally l Kolkata I
: 491 : Air Cycles

and the therm al efficiency of the cycle for a m 3/s m easured at suction conditions. Given

J.
com pression ratio of 8 : 1 [Take Cv(air) as that the percentage clearance is 4 and that
the index of compression and expansion is
0.72kJ/kgK and (�r =1.4 ].
1.2, calculate (a) the ideal volum etric
V air

(GATE-ME-98)
efficiency and (b) the cylinder bore and
stroke, assum ing the bore/stroke ratio is 0.8.
(GATE-ME-00)
07. A large diesel engine runs on four stroke
cycle at 2000 rpm . The engine has a
10. A reciprocating com pressor 1s to be
displacem ent of 25 liter and a brake m ean
designed to com press 4.5kg/m in of air from
effective pressure of 0.6 MN/m 2 • It
consum es 0.018 kg/s of fuel (calorific value 100 kPa and 27°C through an overall
= 42000 kJ/kg). Determ ine the brake power pressure ratio of 9. The law of com pression
and the brake therm al efficiency. is PV 1 3 = constant. Calculate the savings in
(GATE-ME -99) power consum ption and gain in isotherm al
efficiency, w hen a two-stage com pressor
08. An isentropic air turbine is used to supply with com plete inter-cooling is used in place
0.1kg/s of air at 0.1 MN/m 2 and at 285 K to of a single stage com pressor. Assum e equal
a cabin. The pressure at inlet to the pressure ratio in both the stages of the two
turbine is 0.4 MN/m 2 . Determ ine the stage com pressor.
tem perature at turbine inlet and the power (Gas Constant = 0.287kJ/kg-K).
developed by the turbine. Assum e Cp = 1.0 (GATE-ME-02)
kJ/kgK. (GATE-ME -99)

09. A single acting single cylinder reciprocating


air com pressor running at 7 .5rev/s, takes in
air at 100 kPa, 27°C. The com pressor
delivers air at 600kPa at a flow rate of O.12

liii•Qj@jjl41!!4@ijiM\jijjj� ydcrabad I Delhi I Bhopal I Punc I Bhubancswarl Lucknow I Patna I Bcngaluru I Chcnnai j Vrjayawada I Vizag I T,rupari j Kukatpally I Kolkara
SOLUTIONS
05. Ans: (b)
One Mark Solutions Sol: Heat supply = Qs = C P (T3 -T2 )
Heat rejection = Q R = CJT -T1 )
01. Ans: (d) 4

T
02. Ans: (d) 2 Qs 3

Sol: Valve Bounce 1s represented by irregular s=c


area.
2
.,
QR

03. Ans: (b) Volume -> Entropy --> s


Sol: Intake and exhaust take place at constant

6:
pressure.
11 diesel
=1 -
04. Ans: 525
Sol: rk =21 = l - C v (T4 -T1 )
C p (T3 - T2 )
1 = m3
v1 = - l-
P1 kg
06. Ans: 14.29% (range 13.7 to 14.9)
1 m3
V = - = -=0.04762 - Sol: C3Hs+ x02 � aC02+ bH20
V1
2 21 21 kg
Carbon balance :
P(kW) a=3
m(kg / sec)
hydrogen balance:
m3
(v 1 - v ) 2b = 8 � b = 4
2 kg
Oxygen balance:
2x = 2a+b
-
_ 30 x l 0-3
b 4
(1- 0.04762) � x = a+ - = 3+ - = 5
15
2 2
= =525 kPa For chemically correct or stoichiometric
30 X 10-3 X 0.95238
burning, no. of moles of 02 required are = 5.

!ltl•i!@jjji4ijji4Rffn5\,@*)Hyderabad I Delhi I Bhopal I Pune I Bhubaneswarl Lucknow! Patna I Benga)uru IChennai IVtjayawada !Vu.ag ITirupa!i I Kukalpally I Kolkatl I
ACE · cycles
�.�•..,:·=
l!i&�p
�·- :•:,.========....,;,.= :;:49:3�:
::rm�i�Pu�li�ifi.�'311�:c.i Air
=============::���:;;;;:
As it is burnt with 10% deficient oxyg en, it
will g enerate CO. p 3
The new equation is
C3Hs + 0.9 x 502 � aC02 + bCO + cH20
Carbon balance: 4'
a+b=3
Hydrogen balance:
2c = 8 � c = 4
V
Oxygen balance:
2a + b + C =0.9 X 5 X 2 =9
2a + b + c = 9 02. Ans: (c)
=> 2a + b + 4 =9 => 2a + b = 5 ... (1)
a + b =3 (2) 03.
By solving (1) & (2) 1
Sol: W = - x (500-200) x lOO x 0.02
2
a=2&b= l
in the ex haust products the no. of m oles of _!_ x 300 x0.02 = 3 kJ
=
2
CO are 1.% by volum e ofCO in exhaust.
W = 3 kJ
b Mep =
x lOO V8 (0.03-0.01)
a +b+c
1 3
=---x lOO = - X 100 = 150 kPa.
2 +1 + 4
1
= - x lOO = 14.29% 04. Ans : True

07. Ans: (c) 05. Ans : A-3, B-1, C-4, D-2

06. Ans: (b)


Sol: Clit + 202 � CO2 + 2H20
Two Marks Solutions

16 kg s + 64 kg s � 44 + 36
0 1 . Ans: (d) 1 kg CH4 requires 4 kgs of 02
Sol: Dotted line with variation of specific heats,
s olid line without variation of specific heat

!lillM@jjji§ijjiRftftlM\iifjj.yderabad I Delhi I Bhopal I Pune I Bhubancswar I Lucknow I Patna! Benga)uru I Chcnnai I V\jayawada I Vi7.ag I Tirupari I Kukatpally I Kolkab. I
. .t\CE . �.-:oo:,•========�:494 ·
��-� Thermod anncs
"'t'=
.l!lfti�,am�IH'l ::! • �Pn�Ni�d
:;1'!'111 =
.a; :;:�==========����yn�=�:_
07. Ans: (a) 11. Ans: (a)

Sol: 11 = 1 -
( 1 J y -l
rk
= 1-
( 1 )0.4

8.5
= 0.5751 or 57.51%
T1 = 300 K ; Pd = 9 bar ; Ps = 1 bar.

1.4 9 2.8 12. Ans: (d)


- - X 0.2867 X 300 (-)
0.4

-1
1.4-1 1
=

r ]
Sol:
111.13 kJ / kg
vs +O. l VS = 11
=

=
O.l VS
08. Ans: (c)
1 Jy-l ( 1 ) 0.4
11 =1-( = 1- = 0.6167 or 61.7 %
1 .
-
Sol: T2 = T1 x r/ 300 (4) 445.8 K
01 44 =
rk 11
y = .

T2 -T1 445.8-300
T� = Ti = 300 +
'Il e 0.8 13. Ans: (d)
+

= 482.25 K Sol: Vs = 1t D 2 L = 1t xl 0 2 x l 5 = 1178.1 CC.


Wc = Cp ( T21 -I'i ) = 1 (482.25 - 300) 4 4
Ve = 196.3 C.C.
= 182.25 kJ / kg
rk = Vs + Ve = 1374.4 = 7
Ve 196.3
09. Ans: (c)

Sol: p2 =M=4
Pi

10. Ans: (b) +-(ff} 1800 - 973.51 kJ

Sol: Mep = - = --w- w


Vs V1 -Ve 14. Ans: (d)
Sol: p p
_ 23.625 x 10 x Ve
5

5 .5 Ve -Ve 3 1

- 23.625 10 ve 5.25 X 105 Pa


s

4.5 Ve
X
=
�2

V V

\( } l 11g111t t 1 11 11.!; P11lil1t ,IIH11l'- �yderabad l Delhil Bhopall Puncl Bhubaneswarl Lucknow l Patna l Bengaluru l Chcnnai J Vtjayawada !Vmg IJ"'uupati I Kukatpallyj Kolkalal
: 495: Air Cycles

1-2 -Reversible adiabatic process WB = 11B Cp(T3 -T2)


2-3- Isothermal process = 0.33 X 1.005(550-450) = 33.16
3-1- Constant pressure process Wo>WB
2' -1- Constant volume process
17. Ans: (a)
15. Ans: (b)
18. Ans: (c)
16. Ans: (a) Sol: Clearance volume = Ve= 0.001 m3
Sol: OTTO
Stroke volume = Vs = 1t D2 L
T 4
T 1 = 300K T3
T2= 450K T2 s=c = 7tx(0.2)2 (0.25)= 7.85xl0-3
T3= 600K 4
T4
s=c
C
= 0.00785 m3
T1 Vs + Ye 0.00785 + 0.001
s rk = = = 8.85
Ye 0.001
BRAYTON
1 Jy-l - (- 1 )0.4
T 11 Thermal-1- (-k -1-
T3 - r 8.85
T 1 = 300K
s=c
T2= 450K T2 = 58.2% = 0.582
T3= 550K s=c T4

T1 19. Ans: (d)


s

20. Ans: (a)


rk = compression ratio, Sol: This is a Lenoir cycle and its efficiency is
rp = pressure ratio given by
1 T, = 300
n·,ouo =l-- = l-- l--=0.33
rt' T2 450
1 T 300
11Brayton =1- =1-T1 = l-
fr-I =0.33
(rp r
2 450
1.4 X [(4)/4 -1]
Hence :.11 0=11 8 =0.33 = 1 ----=--� = 0.2104 = 21.4%
4-1
Wo = 110Cv(T3 -T2)
= 0.33 X 0.718(600-450) = 35.54
I I I I I I I
�yperahad Delhi Bhopal Pune Bhubaneswar Lucknow Patna Bengaluru I Chennai I Vli:,yawada I Vizag I Tuupali I Kukatpally I Kolkala I
A,CE · · .
=�==========��:��ynanu
" " · �
��:
= Thermod cs
-:,·•· ��.,..=·�=======�: 496
�ili�.,..
i'.1!.&�..,.�-=:""!ll!N
.. "=
21. Ans: (d) Compression ratio,
RT, = 0.287 x300 = m3 rk =-=17
v,
Sol: v, = 0.861
P1 100 kg V2
V1 17V2
= .2 = 0.861 = 0.0861 m
3 =

10 10 kg V3 -V2 0.1 (V1-V2)


V2 =

0.1 (17V2-V2) l.6V2


-
mep = = QS QR
= =

Wnet
v8 v1 - v 2 V3 =
2.6V2
V3
1500 -700 Cut offratio (re) = 2.6
1032.39 kPa. V2
=

0.861- 0.0861
= =

22. Ans: (a) 11th =[1-� (�JJxlOO


Yr r r -1

( 2;�:�� 1
Power(kW)
Sol: mep=
k C

Actualdiplacementvol( in m 3 /sec)
-[ 1 - \ ,, J]xtOO
950 1.4x 7
=-----
2200
0.0259x _ 1 3.8103-1
120 - [1 - ( )] X 100
1.4x3.1058 2.6 -1
= 2000 kPa � 2 MPa
=
59.6 %.
23. Ans: (d)
Sol: L = 2r 2 x 60= 25. Ans: 59.6%
=
120 mm = 0.12 m = 12cm Sol:
p
D 0.08 m 8 cm
2
= =

Vs = 1t 1t 2
4 D L = 4 x8 x12
2
4

= 602.88 � 603 cm3

24. Ans: 59.6 % V

p2
Jr
Sol: I

Compression ratio(rk) = �= (
V2 pt
4
32.42 1.4
= (--) =12
1 1
V
i.-- V 1 -V2--+I
\( I l 11g1r1tl 1111g P11hlu.111c)il" �ydcrabad I Delhi I Bhopal I Puncl Bhubaneswarl Lucknow I Patna! Bcngaluru I Chcnnai jVgayawadajVizag jTirupali I Kukatpallyl Ko1kata I
.,•�
':,, . ACE
. Ptiir,a,n
:,......
.
. . : 497: Air Cycles

v4 27. Ans: (b)


Expansion ratio (rE) = =8
V3 Sol:
. rk 12
Cut-off ratto(rc) = -= -=1.5
T
p
2' 3
rE 8 pie process

_ 1 [ r[ -lJ
11th- 1-� --
4
re -1
4
yrk . I I v=c
Isentrop1c process
_ 1 (1.5)1-4 -1
- 1- [ ]
(1.4x12r4 1.5-1
V s

0 7641 1-2' -3-4 � Diesel Cycle


=1- X r4 [ · ]=0.596(or)59.6%
(1.4 12 0.5 1-2-3-4 � Otto Cycle

26. Ans: 1409.63 kJ/kg Area of Diesel is more than Area of Otto
Sol: (Work)oiesel > (Work)otto
Hence (11)niese1 > (11) > (rt) otto
For any comparison dual cycle is a1Wf1YS in
P1 = 100 kPa
between Otto and Diesel cycles.
T 1 = 308 K
T3 = 2660 K V

y= 1.4
Five Marks Solutions

R = 288.8 J/kg.K
01.
rk = compression ratio -= 8 Sol: xCsH1s + a(02 + 3.76N2)
. • = V1

� I !CO2 + 9CO + 0.402 + 79.6N2 + d H20


V2

288·8
Cv = � = = 7221 /kgK C -balance fu- balance
y- 1 1.4-1
8x = 11+9 = 20 2d = 18x
T2 = T1.(rk)Y-1 = 308 (8)°"4 = 707.6K
Heat supplied (Q) = Cv (T3- T2) X = 2.5 d = 9x = 9 X 2.5 = 22.5
= 0.722(2660 - 707.6) Qi -balance
= 1409.63 kJ/kg 2a = 22+ 9+ 0.8 + 22.5 = 54.3

a = = 27.15
543

Fyderahad j Delhi j Bhopal j J'unej Bhubaneswarl Lucknow! Patna j Benpiuruj Chcnnai IVuayawadal Vizag IT1111pati I .Kukatpallyj Kolkata I
: 498: Thermodynamics

. . mass of air 50 X 0.746 = 12 X 100 X 0.785


A1r Fue1 Ratlo = AFR = ----
mass of fuel X0 x 0.45 X 3.3333 X 0.348
2

27.15(2x18 + 3.76x 28) = 491.723 D 2


2.5(12 X 8 +1 X 18)
=

02 = 50x0.746
3835.72 491.723
13.458
285 D = 0.2754 metres 27.54 cm
= =
=

02. 04.
= pmi LANn
Sol: IP (kW)
Sol:
120 p

600xl.5Dx ,r D2 x150x4
150 = 4
120
3 150x120 = 0.042463
D =
600 X 1.5 X 1[ X 150
D = 34.88 cm V

L = 1.5 D = 1.5 x 34.88 = 52.32 cm


P 1 = 100 kPa;
03. T3 = 1569 + 273 = 1842 K;
2LN = · L 90 = 90 = 5 · T1 = 300 K
3 = 0.4 m V, V4
60 ' N 200
Sol:
rk= 6 = -=-
Double Acting V2 V3

f-,J
P 1 = 1 bar ; P 2 = 6 bar T 1842
(a) T4 = rr-13 = = 899.55 K

1 [(;,
k 6 o.4
2
Power (kW)- � . P1 V T4
P 4 �P4= P 1 X -
n 1 -
Pi -
T4
=
1'i 1'i
= 100 X 899·55 = 299.85 kPa
,r D2 LN [( P2 J lj
1

= � . P1 n�
- 300
n -1 60
x

J r;'
4 Pi
(b) = (p3
2 X 1.2
T3
50 X 0.746 = 100 T4 P4
1.2-1
X

X 1t D2
0.45 X 200
[(6)�:� -l] P = (T r�'
3 3
60 J
x
4 P4 T4
\( l l 11..,1111 t 1111;,, Ptild1t.il1i1t1, �yderabadjDelhiI Bhopal IPune IBhubancswarl LucknowIPatna.IBengalurul Chennai jVuayawadalVmg IT1n1pati I Kukalpal)yj Kolkata I
: 499 : Air Cycles

05.
p3 = p4 (
T3 r-
y
1
= 299_85
(
1842 )o.4
1.4

Sol: C3 Hs + x (02 + 3.76 N2) �


T4 J 899.95
aC02 + bH20 + cN2
= 3678.35 kPa
a = 3; b = 4;
2x = 2a+ b =(2x 3) + 4 = 10,
x=5
2c = 7.52x
C= 3.76x = 3.76 X 5 = 18.8

The process required to complete the cycle C3 Hs + 1.3 x 5 (02 + 3.76 N2 )


is constant pressure and cycle is Atkinson. � aC02 + bH20 + c02 + dN2
(re -rk)
11 - 1 y
A- - {r:-r�)
Composition of exhaust
= 1 - 1.4
(13.13-6)
a = 3 � CO2
(13.131 .4 -6 1 .4)
b = 4 � H2 0
(13.13-6)
1 - 1.4 2a + b + 2c = 1.3 x 5 x 2
(36.776-12.286)
=

2 X 3 + 4 + 2c = 13
(7.13)
1 - 1.4 0.592 or 59.2 % 2c = 13 - 10 = 3 , C = 1.5 � 02
24.49

r
= =

2d = 1.3 x 5 x 3.76 x 2 = 24.44 � N2

(c) l]o - I - (:. Mass Analysis


CO2 = 3x44 = 132
= 1-
( 1 )0.4
= 0.5116 or 51.16 % H 2 0 = 4x18 = 72
6
- 0 2 = l.5x32 = 48
Increase in 11 = llA 110 x100
110 N 2 = 24.44x 28 = 684.32
59.2-51.16 Total = 936.32 kgs
xlOO
51.16
=

= 15.71 %

\( l } 11..!,l]H( !Ill.!, P11hlh,\1i(ll]'> �yd


erabad I Delhi I Bhopal I Pune I Bhubancswarl luclcnowl Patnal Bcngaluru I Chennai IVuayawadalVizag ITirupari t �, Kolkata I
" "
'!.�-�
� �
AJCE . .
Pobi:ancn : 500: . .
� �================================
Thermodynamics

06. W = Qs-QR = 1500-652.8


Sol: = 847.2 kJ/kg.

Qs = 1500 kJ I kg 07.
fk = 8
Sol: BP (kW) =
b xLANn
_m_ep___

T2 = T, .(rk y-i 2x60


600x25x10-3 x2000
300 (8) " = 250 k W
°4 s
120
=

689.21 K
Brake Thermal efficiency
=

brake power(k W)
(ri) =
mr (kg/ sec) xCV(k J / kg)

1 .4
250
----- = 0.3307 or 33.07 %
p2
T
pI ( 2
r- y
1
l 00 (
689.21 o.4
)
=
0.018 X 42000
TI J 300
= =

= 1838 kPa 08.


=
(T3 - T2) Sol: mr = 0.1 kg I sec ;
T2 = 285 K;
Qs Cv

1500 = 0.718 (T3 - 689.21)


1500 P 2 = 0.1 MN/ m2
T3 = 689.21 + -- 2778 K
0.718
=
P1 = 0.4 MN I m2
P3 P2 P 1832 p_2 - 1
- = -=>-3 - = -- rp-
__
T3 T2 2778 689.21 - -
Pi 4
P3 = 7385.16 kPa T
2 = (rp f; = (0.25)°·2ss11 = 0.673
1
T 2778 T1
T4 = 3 = = 1209.2K
285
Iy-1
k 80.4

TI = � = = 423.5 K
0.673 0.673
11,.- t-( :.f-1-( ½f-o.S647
WT = Cp (T1 -T2) = 1 (423.5 -285)
Qs = Cv (T3 - T2) = 138.5 kJ/kg
= 0.718 (2778 - 689.21) = 1500 kJ / kg
WT (kW) = 0.1 x 138.5 = 13.85 kW
QR = Cv (T4 -T1)
= 0.718 (1209.2 - 300)

= 652.8 kJ/ kg

111111;;;;;;;;.;;;w&!Miiiih*fyderabadlDclhilBhopallPunelBhubanesw.u-l LucknowlPaJnalBcngaiurujCbennailVuayawadajV123g ITirupati I Kukatpallyj Kolkata I



l••1lif
" • ACE
�&pi,,iqPnlimnn
• • •
: 50 1 : Air Cycle s

09. 45
=
(_!2_) X · X 0.287 X 300[9�:! - 1 ]
Sol: Single Acting; Single Cylinder, D = 0.8 1.3- 1 60
L
= 18.48 kW
= 1 + C- C - mRT1 /n -
(Wc)Isothermal -
Pi
11Vol
(p J¼

Pi
600 ½. 45
1 + 0.04 - 0.04 (-) = 0.862 = · x0.287x300x£n _!_
2

60 9
=
1 00
= 14. 1 88 kW

(We) Polytropic - 2 stages (kW)

V1 =V2 (
p ½ 6 ½.
0. 1 2 ( ) = 0.431 5
J
4


=
I
N
;r D L-xn 45 2xl.3
Volume flow rate = - x17.,"0 · = · x 0.287 x300[9�:! - 1 ]
2

4 60
1
60 0.3
0.43 1 5 = ;r x x D x7.5xlx0.862 1 6. 1 49 kW
4 D 0.8
2
=

Savings in power (kW)


D3 0.067984
= 18.48- 1 6. 149 = 2.83 1 kW.
=

D = 0.4082 meters = 40.82 cm


(Work)
L E_ = 40.82 = 5 1 cm 11 isothermal = (Work)isothermal
0.8 0.8
=
polytropic

14·188
Single stage = xlOO= 76.77%
18.48
11isothermal -
1 0.
Sol: m = 4.5 kg/min P1 = 1 00 kPa 14· 1 88
2 stage = xlOO = 87.85%
T1 = 300K;
llisothermal -
1 6. 1 49
% gain in Isothermal efficiency
Pi
= 9 = pressure Ratio ·
p 87.85- 76.77
xlOO 14.43%
76.77
= =

py l .3 = C
I

\(} l 11gl[Ht lllli.,: P11hl1t.11I1111-. �yderabad I Delhi I Bhopal I Pune I Bhubancswar I Lucknow I Patna I Bengalwu I Chennai I Vuayawada I Vu.ag I Tuupari I Knkalpally I Kolkata I
Psychrometry
(a) 0.02 (b) 0.027
One Mark Questions (c) 0.25 (d) 0.75

01. For air with a relative humidity of 80% 04. Water at 42° C is sprayed into a stream of
(GATE-ME-03) air at atmospheric pressure, dry bulb
(a) the dry bulb temperature is less than temperature of 40 ° C and a wet bulb
the wet bulb temperature temperature of 20° C. The air leaving the
(b) the dew point-temperature is less than spray humidifier is not saturated. Which of
wet bulb temperature the following statements is true?
(c) the dew point and wet bulb (GATE-ME--05)
temperatures are equal (a) Air gets cooled and humidified
(d) the dry bulb and dew point (b) Air gets heated and humidified
temperatures are equal (c) Air gets heated and dehumidified
(d) Air gets cooled and dehumidified
02. During chemical dehumidification process
of air (GATE-ME-04) 05. Dew point temperature is the temperature at
(a) dry bulb temperature and specific which condensation begins when the air is
humidity decrease cooled at constant (GATE-ME -06)
(b) dry bulb temperature increases and (a) Volume (b) Entropy
specific humidity decreases (c) Pressure (d) Enthalpy
(c) dry bulb temperature decreases and
specific humidity increases 06. If a mass of moist air in an airtight vessel is
(d) dry bulb temperature and specific heated to a higher temperature, then
humidity increase (GATE-ME-11)
(a) Specific humidity of the air increases
03. For a typical sample of ambient air (at 35°C, (b) Specific humidity of the air decreases
75% relative humidity and standard (c) Relative humidity of the air increases
atmospheric pressure), the amount of (d) Relative humidity of the air decreases
moisture in kg per kg of dry air will be
approximately (GATE-ME -OS)

j1Dli@jjj4§iii4RflbiM\!ijjj+yderabadlDclhilBhopallPunelBhubancswarl LucknowlPatnalBengalurulChennailVtjayawadajV� JT1rUpati I Kukatpal)yl Kolkata I


: 503 : Psychrometry

07. · The pressure, dry bulb temperature and with no condensation, the humidity ratio of
relative humidty of air in a room are 1 bar, kg
the final stream (in J is ---
30° C and 70 %, respectively. If the saturated kg dry air
steam pressure at 30°C is 4.25 kPa, the (GATE -15 -Set 1)
specific humidity of the room air in kg water
vapour/kg dry air is (GATE-ME -13) 11. If a mass of moist air contained in a closed
(a) 0.0083 (b) 0.0101 metallic vessel is heated, then its
(c) 0.0191 (d) 0.0232 (GATE - 17-SET -2)
(A) relative humidity decreases
08. Moist air at 35° C and 100% relative (B) relative humidity increases
humidity is entering a psychrometric device (C) specific humidity increases
and leaving at 25°C and 100% relative (D) specific humidity decreases
humidity. The name of the device is
(GATE-ME -14-SET-4)
(a) Humidifier (b) Dehumidifier Two Marks Questions
(c) Sensible heater (d) Sensible cooler
01. Wet bulb depression, under saturated
09. A sample of moist air at a total pressure of ambient air conditions: (GATE-ME-89)
85 kPa has a dry bulb temperature of 30° C (a) is always positive
(Saturation vapour pressure of water = 4.24 (b) is always negative
kPa). If the air sample has a relative (c) is always zero
humidity of 65 %, the absolute humidity (in (d) may have a value depending upon the
gram) of water vapour per kg of dry air is dew point temperature
(GATE-ME-14-SET-3)
02. Atmospheric air from 40° C and 60 percent
10. A stream of moist air (mass flow rate = 10.1 relative humidity can be brought to 20°c
kg/s) with humidity ratio of 0.01 and 60 percent relative humidity by
kg (GATE-ME-90)
mixes with a second stream of
kg dry air (a) Cooling and dehumidification process
superheated water vapour flowing at 0.1 (b) Cooling and humidification process
kg/s. Assuming proper and uniform mixing (c) Adiabatic saturation process
(d) Sensible cooling process
llfl1Qj@jjj4ii04Aflnj@jj.jj4,ayderabad I Delhi I Bhopal I Pune I Bhubaneswar I Lucknow! Patna I Bengaluru I Oiennai jVtjayawada IV,zag ITIIU� I Kukatpally I Kolkata I
:504: Thermodynamics

03. If moist air is cooled by sensible heat (a) the wet bulb temperature and specific
removal, which of the following is true? enthalpy increase
(GATE-ME-91) (b) the wet bulb temperature and specific
(a) Neither relative humidity nor specific enthalpy decrease
humidity changes ( c) the wet bulb temperature increases and
(b) Specific humidity changes but not specific enthalpy decreases
relative humidity ( d) the wet bulb temperature decreases and
( c) Both relative humidity and specific specific enthalpy increases
humidity change
( d) None of the above 06. Dew point temperature of air at one
atmospheric pressure ( l .013bar) is 18°C.
04. Select statements from List II matching the The air dry bulb temperature is 30°C. The
processes in List I. Entry your answer as D, saturation pressure of water at l 8°C and.
C if the correct choice for ( 1 ) is (D) and that 30°C are 0.02062 bar and 0.04241 bar
for (2) is (C) (GATE-ME -99) respectively. The specific heat of air and
List -I water vapour respectively are 1 .005 and 1 .88
( 1 ) Cooling and dehumidification kJ/kg K and the latent heat of vaporization
(2) Chemical dehumidification of water at 0°C is 2500 kJ/kg. The specific
List II humidity (kJ/kg of dry air) and enthalpy
(a) Dry bulb temperature increases, but (kJ/kg of dry air) of this moist air
dew - point temperature decreases. respectively, are (GATE-ME -04)
(b) Dew - point temperature increases and
(a) 0.0 1 05 1 , 52.64 (b) 0.0 1 291, 63. 1 5
Dry bulb temperature remains
(c) 0.0 1 481, 78.60 (d) 0.1 532, 8 1.40
unchanged
( c) Dry-bulb and wet - bulb temperatures
07. Various psychrometric processes are shown
decrease
in the figure below. (GATE-ME -05)
( d) Dry - bulb temperature decreases, but
dew - point temperature increases

05. For air at a given temperature, as the relative


5
co(kg/kg.da)
humidity is increased isothermally,
(GATE-ME-01) 3�
t(OC)

\( I l J1glllll llll..; Pi:lilH,llltlll'I �yderabad I Delhi I Bhopal I Punc I Bhubancswar I Lucknow I Pama I Bcngaluru I Chennai I Vtjayawada I Vtzag I Tirupati I �I Kolkata I
: 505 : Psychrometcy

Process in Figure 09. A thin layer of water in a field is formed


P. 0 - 1 Q. 0 - 2 after a farmer has watered it. The ambient
R. 0 - 3 S. 0 - 4 T. 0 - 5 air conditions are: temperature 20° C and
Name of the process relative humidity 5%.
An extract of steam tables is given below.
1. Chemical dehumidification
2. Sensible heating Temp
-15 -10 -5 0,01 5 10 15 20
3. Cooling and dehumidification
(OC)
Saturation

4. Humidification with steam injection Pressure


0. 10 0.26 0.40 0.61 0.87 1 .23 1 .71 2.34
5. Humidification with water injection
(kPa)

Neglecting the heat transfer between the


The matching pairs are
water and the ground, the water temperature
Codes:
in the field after phase equilibrium is
(a) P - 1, Q - 2, R- 3, S - 4, T - 5
reached equals. (GATE-ME -06)
(b) P - 2, Q- 1, R- 3, S - 5, T - 4
(a) 10.3 C
°
(b) -10.3°C
(c) P - 2 ' Q - 1' R - 3' S - 4' T - 5
(c) -14.5° C (d) 14.5° C
(d) P - 3' Q - 4' R-5' S - 1' T - 2

08. The statements concern psychrometric chart. 10. Atmospheric air at a flow rate of 3 kg/s (on
(GATE-ME -06) dry basis) enters cooling and dehumidifying
1. Constant relative humidity lines are coil with an enthalpy of 85 kJ/kg of dry air
uphill straight lines to the right and a humidity ratio of 19 grams/kg of dry
2. Constant wet bulb temperature lines are air. The air leaves the coil with an enthalpy
downhill straight lines to the right of 43 kJ/kg of dry air and a humidity ratio of
3. Constant specific volume lines are
8 grams/kg of dry air. If the condensate
downhill straight lines to the right
water leaves the coil with an enthalpy of
4. Constant enthalpy lines are coincident
with constant wet bulb temperature lines. 67kJ/kg, the required cooling capacity of the
Which of the statements are correct? coil in kW is (GATE-ME -07)
(a) 2 and 3 (b) 1 and 2 (a) 75.0 (b) 123.8
(c) 1 and 3 (d) 2 and 4 (c) 128.2 (d) 159.0

\( I I 11i..;111t 1. 1 1 11� P11hl1< .it11i11, Fyderabad l Dellii l Bhopal l Pune l Bhubaneswarl Lucknow l Patna l Benga)uru i Chennai l Vuayawada l Vizag I Tuupati I Kukalpallyi Kolkml
��-�
ACE : 506 : Thermodynamics
� � ==================================
��PnNir#ms

11. Moist air at a pressure of 1OOkPa is is 4.24 kPa and the total pressure is 90 kPa,
compressed to 500kPa and then cooled to then the relative humidity (in %) of air
35°C in an after cooler. The air at the entry sample is (GATE-ME -10)
(a) 50.5 (b) 38.5
to the after cooler is unsaturated and become
(c) 56.5 (d) 68.5
just saturated at the exit of the after cooler.
The saturation pressure of water at 35° C is 14. A room contains 35kg of dry air and 0.5kg
5.628 kPa. The partial pressure of water of water vapor. The total pressure and
vapour (in kPa) in the moist air entering the temperature of air in the room are 1OOkPa
compressor is closest to (GATE-ME-08) and 25°C respectively. Given that the
(a) 0.57 (b) 1.13 saturation pressure for water at 25° C is
(c) 2.26 (d) 4.52 3. l 7kPa, the relative humidity of the air in
the room is (GATE-ME-12)
12. Air (at atmospheric pressure) at a dry bulb (a) 67% (b) 55%
temperature of 40° C and wet bulb (c) 83% (d) 71%
temperature of 20° C is humidified in an air
washer operating with continuous water 15. Air in a room is at 35° C and 60% relative
recirculation. The wet bulb depression (i.e. humidity (RH). The pressure in the room is
the difference between the dry and wet bulb 0.1 MPa. The saturation pressure of water at
temperatures) at the exit is 25% of that at 35° C is 5.63 kPa. The humidity ratio of the
the inlet. The dry bulb temperature at the air (in gram/kg of dry air) is___
exit of the air washer is closest to (GATE -15 -Set 3)
(GATE-ME-OS)
(a) 10 C
°
(b) 20° c 16. The partial pressure of water vapour in a
(c) 25 C
°
(d) 30 c °
moist air sample of relative humidity 70% is
1.6 kPa, the total pressure being 101.325
kPa. Moist air may be treated as an ideal gas
13. A moist air sample has dry bulb temperature mixture of water vapour and dry air. The
of 30°C and specific humidity of 11.5 g relation between saturation temperature (Ts
water vapour per kg dry air. Assume in K) and saturation pressure cPs in kPa) for
molecular weight of air as 28.93. If the water is given by
saturation vapour pressure of water at 30°C /n(PJP0) = 14.317 - 5304/Ts ,
Fydcrabad J Dclhi l Bhopa) J Pune l Bhubaneswar l Lucknow l Patna l Bengaluru l Chennai l Vuayawada J V,zag ITllUpali I �I Kolkatl I
: 507 : Psychrometry

where P 0 = 1 0 1 .325 kPa. The dry bulb


temperature of the moist air sample (in °C)
Five Marks Questions
is ___ (GATE - 16 - SET - 2)
O1 In an air conditioning system, air is to be
1 7. In a mixture of dry air and water vapor at a
cooled and dehumidified by means of a
total pressure of 750 mm of Hg, the partial cooling coil. The data are as follows:
pressure of water vapor is 20 mm of Hg. The
(GATE-ME-88)
humidity ratio of the air in grams of water
Initial condition of the air at the inlet to the
vapor per kg of dry air (gwlkgda) is
cooling
(GATE - 16 - SET - 3)
Dry bulb temperature = 25 ° C.
Partial pressure of water vapour = 0.01 9 bar
1 8. Moist air is treated as an ideal gas mixture
Absolute total pressure = 1 .02 bar ( 1 02 kPa)
of water vapour and dry air (molecular
Final condition of the air at the exit of the
weight of air = 28.84 and molecular weight
cooling coil:
of water = 1 8). At a location, the total
Dry bulb temperature = 1 5 ° C.
°
pressure is 1 00 kPa, the temperature is 30 C
Relative humidity = 90%
and the relative humidity is 55%. Given that
Absolute total pressure = 1 .02 bar ( 1 02 kPa).
the saturation pressure of water at 30° C is
Other data are us follows:
4246 Pa, the mass of water vapour per kg of Specific gas constant for air = 287 J/kg K
dry air is _____ grams. Specific gas constant for water vapour
(GATE - 17 - SET - l ) = 46 1 .5 J/kg K
Saturation pressure for water at 45 ° C
= 0.0 1 7 bar ( 1 .7 kPa)
Enthalpy of dry air = 1 .005 t kJ/kg.
Enthalpy of water vapour
h = (2500+ 1 .88t) kJ/kg.
where t is temperature in °C.
Determine
(a) Moisture removed from air per kg of
dry air,
(b) Heat removed by the cooling coil per
kg of dry air.
l b
_lilBl1,:1u11111mNDEl!IPHl!l�:. _Yderabad I Delhi I Bhopal I Pune I Bhubaneswar I Lucknow!
--
Patna I Bengalwu I Chennai j Vgayawada I Vmg I T'rrupati I Kukatpally I Kolkata I
. ACE
. . .
�-�.."Fqprw:migPuNiratiin
".,,,. : 508: Thermodynamics

02. 20 kg/s of air at 30° C and a humidity ratio saturation, enthalpy and dew point. Assume
of 0.01 kg water vapour/kg air are mixed that air and water vapour may be treated as
with 1 2 kg/s of air at 38° C and a humidity perfect gases. The following data may be
ratio of 0.02 kg water vapour/kg air. If there used:
is no external addition or removal of 29 ; Mwater = 1 8 ;
Mair =

moisture and heat, determine the Universal Gas constant= 8.3 1 4 kJ/kg moleK
temperature and humidity ratio of the CPa = 1 kJ/kgK, CPw = 4. 1 86 kJ/kgK
resultant air steam from first principles. Vapour pressure of water follows the
Indicate the process on a psychrometric equation
chart. 5325
(GATE-ME-89) In P = 1 9.0 1 3 - , P in kPa & T inK
T
03. In an air-conditioning system 200 m3/min (GATE-ME-94)
air is cooled and dehumidified from 32°C
and specific humidity 0.01 4 kg/kg of dry air 05. On the psychometric chart consider a point

to 16°C and specific humidity 0.01 1 kg/kg at a given dry and wet bulb temperature. At
this point show directions of
of dry air. Draw the psychrometric process
and calculate the sensible heat factor.
(GATE-ME -95)
(a) Sensible cooling
Average property values may be taken as:
(b) Sensible heating
Density of air, p = 1.2 kg/kg of dry air
( c) Adiabatic saturation
Specific heat of air, Cp = 1.0216 kJ/kgK
( d) Isothermal humidification
Latent heat of water, h1 = 2500 kJ/kg
( e) Cooling and dehumidification
(GATE-ME -90)
( f) Chemical dehumidification

04. At a location where the atmospheric


pressure is 960 milli bar, the temperature is
30° C and the relative humidity 60%,
calculate the humidity ratio, degree of

\( l l 1 1g1, 1i t l lllg PnlilH,1lloih �yderabad J Delhi J Bhopa) J Pune ! Bhubaneswar l Lucknow l Patna l BengaJurul Chennai l Vtiayawada J Vizag J Tirupati I KukatpallyJ Kolkata I
SOLUTIONS
06. Ans: (d)
One Mark Solutions Sol: Sensible Heating process

01. Ans: (b) 07. Ans: (c)


Sol: Sol: P 1 = 1 bar T 1 = 30° C
cj, = 70% , P sat = 4.25 kPa
80% RH

Py
(j> =
psat

0.7 = -Y- =>P v = 2.975 kPa


p
02. Ans: (b) 4.25
Py
Sol:
ro = 0.622-- -
patm -Py

2 975
= 0.622 X ·
1 00- 2.975
03. Ans: (b)
kg of vapour
Sol: From steam table at 35°C, Psat = 5.628 kPa = 0_0191
kg of dry air
(j> = pv =:> 0.75= �
Psat 5.628
08. Ans: (b)
P v = 4.221 kPa Sol: Dehumidifier
ro = 0.622 pv
patm - Pv

0.622 x 4.221 kg of vap


= = 0_027
1 0 1.325 - 4.221 kg of dry air

04. Ans: (b)


Sol: ( Temp) > ( DBT ) . 09. Ans: 20.84
H2 0 air

Sol: Tsat =30°C � Psat = 4.24.kPa


05. Ans: (c) pv = - pv
(j> = 0.65 = _
psat 4.24
!lillil@llliiifiARdftliiliiil-1 yderabad l Delhi I Bhopal I Pune I Bhubaneswarl LucknowI Patna I Bengaluru I Chennai IVuayawadajVtzag ITtrupati I Knkatpally l KolkaJa
: 5�1�0�:�==========�Th rmodynann·cs
ACE
:.� :·�Po
= · :enns
� ,.,:·F.iwnrai;��
-:, :•:::========�
�b�ifi.�ra11�:mi �e������

Pv = 2.756 kPa
pv 2· 756
ro= 0 .622 = 0.622 X Two Marks Solutions
patrn - pv 85- 2.756
= 0 . 0 20 84 kg ofVapour /kg of dry air 0 1. Ans: (c)
= 20 .84 gm ofVapour /kg of dry air Sol: At RH = 1 00% DBT = WBT = DPT.

10. Ans: 0.02 02. Ans: (a)


Sol: m = m a + m vI Sol:

m 1 0. 1
- = l + rol � m a = -= 1 0 kg / s
ma 1 .0 1 20°C 40°c
m v l = 0) 1 X m a =O O. lx1 0 =O . lkg / sec DBT

mv2 = 0 . 1 kg/sec 0 3. Ans: (d)


Total mass of vap or after mixing
0 4. Ans: ( 1 )- (c) ; (2) - (a)
= 0 . 1 + 0 . 1 = 0.2 kg/sec Sol:
Specific humidity of mixture
0.2kg / sec
(J) RI O)
m
ffi = -v = ----
m a lO kg / sec
= 0 .02 kgvap / kgd.a
- DBT

_ DBT

05. Ans: (a)


11. Ans: (a)
Sol: Sensible heating process, RH decreases
Sol:

2V i
(J)

t - DBT
0)
06. Ans: (b)
0.622 pv
Sol : ro =
DBT - patrn - pv
kg of vap
ro = 0 .622x 2. 062 = 0 _ 0 1 292
1 0 1 .3- 2.062 kg of dry air

!M11ii@i/i4lfh/MNflnj@jj.jjl4 yderabad I Delhi I Bhopal I Punc I Bh�baneswar l Lucknow I Patna I Bengaluru I Chcnnai I Vtjayawada I Vmg !Tuupati I Kubtpally I Kolkata
�•,I

,I
:E
ACE
' =L�·�·..,,
:��·!
•�
N���-
�-�·:=:=======�

h = c P. (T-O) + ro{2500+1.88(T-O)}
: 5���:============P
11
����:::�
y try s chfome

= 1.005(30 - 0) + 0.01292(250Q+l.88 X 30)


= 63.17 kJ/kg of dry air.

1 kg vap
07. Ans: (b) 08. Ans: (a) kgd.a
co1 = 0.0 1 9 -"---C-

09. Ans: (c)


= 0.008
kg vap
Sol: .....___________, ro2
T 1 kgd.a
2 is phase
equilibrium point
2 11. Ans: (b)
s Sol: P1 = 100 kPa
P2 = 500 kPa
T = 20°C
Psat = 2.337 kPa DBT = 35° C
P vs = 5.628 kPa
¢, = Pv
cos = 0.622 x Pys - 0.622 x 5 ·628
�al
P2 - Pys 500 - 5.628

0.05 = � Py = 0.11685 kPa = 7.08x10-3 kg vapour/kg dry air


2.337 '
CO2 = ©83 = 7.08xl0-3
Corresponding to P v whatever is the
( ·: pressure remains constant in after cooler)
saturation temperature is called phase
Moisture is same throughout the process i.e.,
equilibrium temperature.
in compressor no moisture is removed or
By interpolation, T = -14.5° C.
added and in after cooler, the pressure is
constant so the moisture content will not
10. Ans: (b)
change.
Sol: h1 = 85 k:J/kg, h2 = 43 k:J/kg,
Py
h f 2 = 67 kJ / kg C0 1 = 0.622 X ---
P1 -Py
m a .h i -Q = tha .h 2 + th a .(ro, - © 2 )h r 2
7.08 x l0-3 _ Py
Q = th a .( h 1 - h 2 )- m a .(ro, - © 2 )h f 2 0.622 - (1 00 - Py )
= 3(85 - 43) - 3(0.019 - 0.008) 67 :. Pv = 1.125 kPa
= 123.79 kW
"'•,."
..
. . .
. ACE
:�P.tiirJn.m : 512 : Thermodynamics

12. Ans: (c) l.02296P v = 100 X 0.02296


Sol: 100 X 0.02296
pV = = 2244kP a
[( DBTti, - WBT] = 0.25 [( DBT )ENT -WBT] 0.02296
Pv 2·244
(DBTtxit - 20= 0.25 (40 - 20) RH = = = 0.708 (or) 71%
psat 3.17
(DBT) exit = 25° C
(at 25 c P sat = 3.l7kP a)
°

13. Ans: (b) 15. Ans: 21.745


Sol: Sol: Tsai = 35 °C , Psat = 5.63 kP a
Patm = lOO kP a ,
Py
11.5 X 10-3 = 0.622 cl>=Relative humidity = 60%
90 - Py

Py cl>=
0.01847 psat
90 - Py
=

1.01849 P v = 90 x 0.01849 0.6 = �; Pv = 3.378kP a


5.63
py =
90x0.01849 _
l.6338 kP a. 3378
1.01849 =0.622x
100 - 3.378
P 1.6338
cl>= y = = 0.3853 (or) 38.53% 3378
Psat 4.24 =0.622x =0.021745 gv p
k a
96.622 kg d.a

gmvap
14. Ans: (d) = 21.745
kg d.a

Sol: Specific humidity = ro = mv


ma
16. Ans: 19.89 °C (range 19.5 to 19.9)
0,5 kg
0.01428 Sol: cl> = 0.7 , Po = 101.325 kP a
35kgd.a
= =

q> = pv => psat = pv


0.01428 = 0.622 pv psat q>
patm -p v
1.6
pv Psat = -=2.28571 kP a
= 0.622 0.7
100 - Pv
Corresponding to saturation pressure what
Pv 0.01428 ever is the temperature is the DBT
0.02296
100 - Pv 0.622
=

�yderabad j Delhi j Bhopal j Pune j Bhubaneswarl Lucknow l Patna l BengaJuru l Cbcnnai l Vuayawadal V17.ag I Tuupati I K�I Kolkala I
: 513 : Psychrometry

.en (Ps J = 14.317 -5304


Po Tsat Five Marks Solutions

.en( 2.28571) =14317 _ 5304


101.325 Tsat 01.
5304 Sol: DBT = 25°C Inlet condition
- 3.7916 = 14.317-
Tsat P v = 1.9 kPa ; P ann = 102 kPa
5304 = 0.622 pv
T.Sat = = 292.898 K = 19.89°C
18.1086 Patm - Py
CO t

19 kg of vap
= 0.622x · O.Ol18
17. Ans: 17.0410 (range 16.9 to 17.1) 102 -1.9 kg of dryair
P Enthalpy at inlet condition
Sol: co= 0.622 y
Patm - Pv h1 = 1.005t + C01 (2500 + 1.88t)
20 kg vapour = 1.005 X 25 + 0.0118 (250Q+ 1 .88 X 25)
= 0.622x = 0.01704
750 - 20 kg dry air
= 25.125 + 30.055
= 17.0410 gm vap/kg da = 55.18 kJ/kg d.a.

DBT = l 5°C • Outlet condition


18. Ans: 14.87 Psat = 1.7 kPa
Sol: P ann = l OOkPa, Tsat = 30°C, Psat = 4.246kPa
cj> = 0.55 Relative humidity = cj> = pv
psat
cl> = � pY
�0.9 = _
psat 1.7
PY = cJ>P sat = 0.55x4.246 = 2.3353kPa �P y = 1.53 kPa
Py 23353
ro= 0.622x = 0.622x = 0.622 pv = 0.622x 1 .
Patm - Pv 100- 2.3353
53
(Patm -Py ) 102 -1.53
CO2

= 0.622 X 23353 = 0.009472


kg of vap
97.6647 kg of dryair
kg vap
= 0.01487 Enthalpy of exit air
kg da
h2 = 1.005t + co1 (2500 + 1.88t)
= 1.005 X 15 + 0.009472(2500 + 1 .88 X 1 5)
= 1 4.87gm vap/kg.da

= 15.075 + 23.95 = 39.025 kJ/kg dry air

\( I I 1 1�11•t t 1 1 11g P1 1 l d 1, . 1 1 1 1 1 1 1.., �yderabad I Delhi I Bhopal I Pune I Bhubaneswar I Lucknow I Patna I Bengaluru I Chennai I Vtjayawada I Vm,g I T11Upati I �� I KolkataI
: 514 : Thermodynamics

Moisture removed = 0>1- 0>2 . kg


m 3 - 32 -
= 0.0118 - 0.009472 sec
kg of vap Water Balance:
= 0.002328
kg of d.a
Cooling load on coil
20x 0.01 + 12 x0.02 = 32 003
h1 - Q = h2 + (m1 - m2 ) hI
0.2 + 0.24 kg of vap
003 = = 0.01375
h r = saturated liquid enthalpy of water 32 kg of d.a
kJ T; + m 2 T2 20x30 +12x38 _ 0
at 15 °C = 62.98- (from steam table)
kg T3 = m1 = 33 C
m3 32
55.18 - Q = 39.025 +0.002328 X 62.98 kg vap
Q = 16 kJ/kg of d.a. O) J = 0 • 01 -- ,
kg d.a
kg vap
02. ro2 = 0.02 --,
kg d.a
Sol:
kg vap
0>3= 0.01375 , T 3 = 33°C
kg d.a

03.
Sol:

- DBT
Section - I Section - II
. kg kg
m 1 = 20 -, m2 = 12
sec sec
�ro = ro1- ro2 = 0.014 - 0.011
kg vap kg of vap
ro1 = O.01 --, ro2 = 0.02 = 0.003 kg vap / kg of dry air.
kg da kg of d.a
�T = T 1 - T2 = 32- 16 = 16° C
T1 = 30°C,
SHF = 1

Mass Balance: ( 245 1 :; + 1)

= 0.6851
l
0 003
=
20+12 = ffi3 1 + 2451x ·
16
" " . ACE
. .
-:.�-�� FflllDN',IDII Pnbtiaooo•
. : 515 : Psychrometry

04. =
5325
DPT = 294.44 K
Sol: P atm = 960 milli bar = 96 kPa 1 8 .0852
T = 303 K , $ = 60 % = 294.44 - 273 . 1 5 = 2 1 .29 OC
° =
From the steam table at 30 c, Enthalpy ha + rohv
Psat = 4.2 1 5 kPa = ha + ro [25 1 4 + 1 .88 (T - O)]
5325
/n(Psat ) = 1 9. 0 1 3 - -- = Cpa (T - 0) + ro [25 14 + 1 .88 (T - 0)]
T = 1 .005 (30 - 0) + 0.01 683
5325 [25 14 + 1 . 88 X (30)]
= 1 9.0 1 3 -
303
kJ
P sat = 4.2 1 5 kPa = 30. 1 5 + 43.26 = 73.4 1 - .
kg
$ = Pv
Psa1
05 .
0.6 = ____!}_ Pv = 2.529 kPa Sol:
4.2 1 5 ' d
C

Humidity Ratio, ro = 0.622 Pv


Pa1m - Pv b

X
2· 529 f
= 0.622
96 - 2.529
= 0.0 1 683 kg of vap / kg of dry air.

X
PSat 42 1 5 o-a : Sensible cooling
rosat = 0.622 =
96 - 4.2 1 5
0.622
Patm - Psat
o-b : Sensible heating
= 0.028563 kg vap / kg d.a o-c : Adiabatic saturation
. ro 0.0 1 683
Degree of saturation = - = = O .5 89
0.028563
--- o-d : Isothermal humidification
(!)sat
o-e : Cooling & dehumidification
5325
ln (Pv) = 1 9.0 1 3 - -- o-f : chemical dehumidification
DPT
5325
ln (2.529) = 1 9.0 1 3 -
DPT
5325
0.9278 = 1 9.0 1 3 -
DPT

!1i11i@jjji41hiRftbiMhih+yderabad I Delhi I Bhopal I Punc I Bhubaneswar I Lucknow! Patna I Bcngaluru I Chennai I Vliayawada I Vu.ag I Tirupati I Kukalpally I Kolkata I
CtO Rankine Cycle
05. Which among the following is the boiler
One Mark Questions mounting? (GATE-ME-97)
(a) Blow- of cock (b) Feed pump
01. For a given set of operating pressure limits
(c) Economizer (d) Super-heater
of a Rankine cycle the highest efficiency
occurs for
06. If V N and a are the nozzle exit velocity
(GATE-ME-94)
(a) Saturated cycle
(b) Superheated cycle and nozzle angle in an impulse turbine, the
(c) Reheat cycle optimum blade velocity is given by
(d) Regenerative cycle (GATE-ME-98)
(a) V N cos 2a (b) V N sin 2a
02. For a single stage impulse turbine with rotor
V N cos a V N sin a
diameter of 2 m and a speed of 3000 rpm (c) (d)
2 2
when the nozzle angle is 20°, the optimum
velocity of steam in mis is
07. A Curtis stage, Rateau stage and a 50%
reaction stage in a steam turbine are
(GATE-ME-94)
(a) 334 (b) 356
examples of
(c) 668 (d) 711
(GATE-ME-98)
(a) different types of impulse stages
03. In adiabatic flow with friction, the (b) different types of reaction stages
stagnation temperature along a streamline (c) a simple impulse stage, a velocity
(increases / remains constant) compounded impulse stage and reaction
(GATE-ME-95) stage
(d) a velocity compounded impulse stage, a
04. Consider a Rankine cycle with superheat. If simple impulse stage and a reaction stage
the maximum pressure in the cycle is
increased without changing the maximum 08. Which of the following is a pressure
temperature and the minimum pressure, the compounded turbine? (GATE-ME-00)
dryness fraction of steam after the isentropic (a) Parsons (b) Curtis
expansion will increase / decrease (c) Rateau (d) all the three
(GATE-ME-95)

!lfl•i@hliiiiiiiPflDM!iih+1yderabad l DelhilBbopall Punc1 Bhubancsw.irl Lucknow l Patna l Bcnphnu l Chennai l VuayawadalV,zag ITuupati I Kukatpallyl Kolkata I
: 517 : Rankine Cycle

09. The Rateau turbine belongs to the category water remains the same, the specific work
of (GATE-ME 01) (in kJ/kg) supplied to the pump is
(a) pressure compounded turbine (GATE-ME-14-Set-l)
(b) reaction turbine (a) 0.34 (b) 2.48 (c) 2.92 (d) 3.43
(c) velocity compounded turbine
(d) radial flow turbine 1 3 . Which of the following statements regarding
a Rankine cycle with reheating are TRUE ?
1 O. In a Rankine cycle, regeneration results in
(i) Increase in average temperature of heat
higher efficiency because (GATE-ME-03)
addition
(a) pressure inside the boiler increase
(ii) Reduction in thermal efficiency
(b) heat is added before steam enters the
(iii) Drier steam at the turbine exit.
low pressure turbine
(GATE -15 -Set 2)
(c) average temperature of heat addition (a) only (i) and (ii) are correct
in the boiler increases
(b) only (ii) and (iii) are correct
(d) total work delivered by the turbine (c) only (i) and (iii) are correct
increases
(d) (i) , (ii) and (iii) are correct

1 1 . Considering the variation of static pressure


1 4. The INCORRECT statement about
and absolute velocity in an impulse steam
regeneration in vapor power cycle is that
turbine, across one row of moving blades
(GATE - 16 - SET - 1)
(GATE-ME-03)
(a) It increases the irreversibility by
(a) both pressure and velocity decrease
adding the liquid with higher energy
(b) pressure decreases but velocity increases content to the steam generator
(c) pressure remains constant, while
(b) Heat 1s exchanged between the
velocity increases expanding fluid in the turbine and the
(d) pressure remains constant, while
compressed fluid before heat addition
velocity decreases.
(c) The principle is similar to the principle
of Stirling gas cycle
12. In a power plant, water (density = 1 000
(d) It is practically implemented by
kg/m3) is pumped from 80 kPa to 3 MPa. providing feed water heaters
The pump has an isentropic efficiency of
0.85. Assuming that the temperature of the

\( I I rig11h t 1 11 n!, P11l,IH 11111th �yderabaclj Dellti j Bhopal j Pune j Bhubaneswarl Lucknowl Patnal Bcngaluru j Chcnnail Vuayawadaj Vizag IT=pati I Kukalpallyl Kolka.ta I
: 518: Thermodynamics

05. In a Rankine cycle heat is added


Two Marks Questions (GATE-ME-91)
(a) Reversibly at constant volume
0 1 . An economizer m a steam generator (b) Reversibly at constant temperature
performs the function of: (c) Reversibly at constant pressure and
(GATE-ME-89) temperature
(a) Preheating the combustion air (d) Reversibly at constant pressure
(b) Preheating the feed water
(c) Preheating the input fuel 06. Velocity compounded steam turbine known
(d) Raising the temperature of steam as ____ (Curtis / Rateau) turbine uses
the principle of converting entire___
02. The fundamental objective of employing the (pressure I velocity) energy before entering
condenser in a steam power plant is to the first stage runner itself.
(reduce /increase) the ---- of steam (GATE-ME-91)
(GATE-ME-89)
07. Subsonic and supersonic diffusers have the
03. In the Rankine cycle when superheated following geometry: (GATE-ME-92)
steam is used: (GATE-ME-90) (a) Divergent and convergent respectively
(a) Thermal efficiency increases (b) Both divergent
(b) Steam consumption decreases (c) Both convergent
(c) Steam dryness after expansion mcreases (d) Convergent and divergent respectively
(d) All of the above
08. Boiler rating us usually defined in terms of:
04. When initially dry and saturated steam flows (GATE-ME 92)
through a nozzle, the ratio of actual (a) Maximum temperature of steam m
discharge to calculated discharge is Kelvin
(GATE-ME-90) (b) Heat transfer rate in kJ/hr
(a) Equal to 1 .0 (b) Greater than 1 . 0 (c) Heat transfer area in metre2
(c) Less than 1 .0 (d) Steam output in kg/hr
(d) Independent of inlet conditions

!l"S•IMh§hlUUlhiiRflfliM\i/ijj+yderabad I Delhi I Bhopal I Pune I Bhubaneswar I wcknow I Patna I Bengaluru I Chenmi I Vtjayawada I Vizag I Trrupari I Kukatpally I Kolkata I
: 519 : Rankine Cycle

09. In steam and other vapor cycles, the process 13. The following data pertain to a single stage
ofremoving non-condensable is called: impulse steam turbine: (GATE-ME-97)
(GATE-ME-92) Nozzle angle = 20°;
(a) Scavenging process Blade velocity = 200 mis;
(b) Deaeration process Relative steam velocity at entry = 350 mis;
( c) Exhaust process Blade inlet angle = 30°;
( d) Condensation process Blade exit angle = 25°.
If blade friction is neglected the work done
10. Asteam turbine operating with less moisture per kg steam is
1s __ (more/less) efficient and__ (a) 124 kJ (b) 164 kJ
(less/more) prone to blade damage. (c) 169 kJ (d) 174 kJ
(GATE-ME 92)
14. Consider an actual regenerative Rankine
11. The equivalent evaporation (kg/hr) of a cycle with one open feed water heater. For
boiler producing 2000 kg/hr of steam with each kg steam entering the turbine, if m kg
enthalpy content of 2426 kJ/kg from feed steam with a specific enthalpy of h1 is bled
water at temperature 40° C (liquid enthalpy from the turbine, and the specific enthalpy
= 168 kJ/kg) is (Enthalpy of vaporization of of liquid water entering the heater is h2, then
water at 100° C = 2258 kJ/kg). h3 the specific enthalpy of saturated liquid
(GATE-ME 93) leaving the heater is equal to
(a) 2000 (b) 2149 (GATE-ME-97)
(c) 186 (d) 1649 (a) mh1- (h2 - h1)
(b) h1 - m(h2 - h1)
12. A steam power plant has the boiler (c) h2 - m (h2 - h1)
efficiency of 92%, turbine efficiency (d) mh2 - (h2 - h1)
(mechanical) of 94%, generator efficiency of
95% and cycle efficiency of 44%. If 6% of 15. Match the following
the generated power is used to run the List -I
auxiliaries, the overall plant efficiency is A. Steam nozzle
(GATE-ME-96) B. Compressible
(a) 34% (b) 39% C. Surface tension
(c) 45% (d) 30% D. Heat conduction

\( I I 111.!,1t11 l 1 1 1 1 g P1ild1c ,111(111' �ydcrabad l Delhi I Bhopal I Punc I Bhubancswar l Lucknow I Patna l Bcngaluru I Chcnnai I Vuayawada l Vu.ag ITirupati I Kukalpallyl Kolkata I
: 520: Thermodynamics

List - II 18. The efficiency of superheat Rankine cycle is


1. Mach Number higher than that of simple Rankine cycle
2. Reaction Turbine because (GATE-ME-02)
3. Biot Number (a) the enthalpy of main steam is higher
4. Nusselt Number for superheat cycle
5. Super saturation (b) the mean temperature of heat addition
6. Weber Number is higher for superheat cycle
(c) the temperature of steam in the
16. The isentropic heat drop in the nozzle of an condenser is high
impulse steam turbine with a nozzle (d) the quality of steam in the condenser is
efficiency 0.9, blade velocity ratio 0.5, and low
mean blade velocity 150 mis in kJ/ kg is
(GATE-ME-98) 19. Match the following (GATE-ME-03)
(a) 50 (b) 40 List - I
(c) 60 (d) 75 P. Curtis
Q. Rateau
17. For a compressible fluid, sonic velocity is R. Kaplan
(GATE-ME-00) S. Francis
(a) a property of the fluid
List - II
(b) always given by (yRT )½ where y , R 1. Reaction steam turbine
and T are respectively the ratio of 2. Gas turbine
specific heats, gas constant and 3. V elocity compounding
temperature in K 4. Pressure compounding
(c) always given by ( 8p/ 8p ½ . Where p, t 5. Impulse water turbine
6. Axial turbine
p and s are respectively pressure,
7. Mixed flow turbine
density and entropy
8. Centrifugal pump
(d) always greater than the velocity of
fluid at any location. (a) P - 2 , Q - 1 , R- 7 , S - 6
(b) P - 6 , Q - 3 , R-4, S - 4
(c) P - 4 , Q - 1 , R-6, S - 2
(d) P - 3 , Q - 4 , R - 6 , S - 7

\( I I 1 ) .,1 1 1 , t 1 1 1 1 ..., P1il)h .111, 1 1 1 " Fyderabad J Dellii l Bhopal J Punc J Bhubancswarl Lucknow J Patna l BcngaJuru J Oicnnai J Vuayawada. l Vizag J Tirupati I Kukatpallyl Kolkata I
"•
..
.
' ,,"' . ACE
:�PubtirJCitw
. . : 5.21: Rankine Cycle

Common Data for Question 20 & 21


Consider a steam power plant using a reheat cycle
as shown. Steam leaves the boiler and enters the
turbine at 4 MPa, 350°C (h3 = 3095 kJ/kg). After
expansion in the turbine to 400 kPa (14 = 2609
kJ/kg), the steam is reheated to 350°C (hs = 3 1 70 (a) an impulse turbine
kJ/kg), and then expanded in a low pressure (b) a reaction turbine
turbine to 1 0 kPa (h6 = 2165 kJ/kg). ( c) a centrifugal compressor
( d) an axial flow compressor
h = 3095kJ/k
3 23. Determine the correctness or otherwise of
Boiler the following Assertion (a) and the
Reason(r).
Assertion(A) : In a power plant working on
l¼ = 21 65kJ/kg 6 a Rankine cycle, the regenerative feed water
heating improves the efficiency of the steam
2 Pump
h1 - 29,3 kJ/kg on enser

turbine.
Reason(R): The regenerative feed water
20. The thermal efficiency of the plant heating raises the average temperature of
neglecting pump work is (GATE-ME-04) heat addition in the Rankine cycle.
(a) 15.8% (b) 4 1 . 1 % (GATE-ME-06)
(c) 48.5% (d) 58.6% (a) Both A and R are true and R is the
correct Reason for A
21. The enthalpy at the pump discharge (h2) is (b) Both A and R are true but R is NOT the
(GATE-ME-04) correct reason for A
(a) 0.33 kJ/kg (b) 3.33 kJ/kg ( c) Both A and R are false
(c) 4.0 kJ/kg (d) 33.3 kJ/kg ( d) A is false but R is true

22. In the velocity diagram shown below, u = 24. Assertion (A): Condenser 1s an essential
blade velocity, C = absolute fluid velocity equipment in a steam power plant.
and W = relative velocity of fluid and the Reason(R) : For the same mass flow rate
subscripts 1 and 2 refer to inlet and outlet. and the same pressure rise, a water pump

(GATE-ME-05) requires substantially less power than a


steam compressor. (GATE-ME-06)
ydcrabad l Dclhi l Bhopal l Punc l Bhubaneswarl LucknowJ Patna l Bcngaiuru l Chcnnai JVtjayawada l Vizali I Tirupati I Kukatpallyl Kolkata
�......�,,..,,..Niiam
ACE : 522: Thermodynamics

25. Which combination of the following Common Data for Question 27 & 28
statements is correct? (GATE-ME-07) The inlet and the outlet conditions of steam for an
The incorporation of re-heater in a steam adiabatic steam turbine are as indicated in the
power plant. figure. The notations are as usually followed.
P: always increases the thermal efficiency h 1 =3200kJ/kg
of the plant. V 1 = 160m/s
Z 1 = 10m
Q: always increases the dryness fraction of P1 = 3MPa h2=2600kJ/kg
steam at condenser inlet. V2 = lOOm/s
R: always increases the mean temperature Z2 = 6m

of heat addition
P2 = 70kPa

S: always increases the specific work


output. 27. If mass flow rate of steam through the
turbine is 20kg/s, the power output of the
(a) P and S (b) Q and S
turbine (in MW) is
(c) P, R and S (d) P, Q, R and S
(GATE-ME-09)

(a) 12.157 (b) 12.941


26. A thermal power plant operates on a (c) 168.001 (d) 168.785
regenerative cycle with a single open feed
water heater, as shown in the figure. For the 28. Assume the above turbine to be part of a
state points shown, the · specific enthalpies simple Rankine cycle. The density of water
are: h 1 = 2800 kJ/kg and h2 = 200 kJ/kg. The at the inlet to the pump is 1000 kg/m3 •
bleed to the feed-water heater is 20% of the Ignoring kinetic and potential energy effects,
boiler steam generation rate. The specific the specific work (in kJ/kg) supplied to the
enthalpy at state 3 is (GATE-ME-08) pump 1s (GATE-ME-09)
(a) 0.293 (b) 0.351
(c) 2.930 (d) 3.510

Common Data for Question 29 & 30


In a steam power plant operating on the Rankine
Open feed
water heater extraction pump
cycle, steam enters the turbine at 4 MPa, 350°C
and exits at a pressure of 15 kPa. Then it enters
(a) 720 kJ/kg (b) 2280 kJ/kg the condenser and exits as saturated water. Next,
(c) 1500 kJ/kg ( d) 3000 kJ/kg a pump feeds back the water to the boiler. The
Fyderabad l Delhi l Bhopal l Punc l Bhubancswarl Ludmow l Patna l Bcngaluru l Chcnnai l Vuayawada j Vizag ITirupa!i I Kukatpallyl Kolkata I
: 523: Rankine Cycle

adiabatic efficiency of the turbine is 90%. The 32. An ideal reheat Rankine Cycle operates
thermodynamic states of water and steam are between the pressure limits of 1 OkPa and 8
given in table. MPa, with reheat being done at 4MPa. The
temperature of steam at the inlets of both
State h (k:J/kg) s(k:J/kg.K) v(m3/kg)
turbines is 500 °C and the enthalpy of steam
is 3185 kJ/kg at the exit of the high pressure
Steam

turbine and 2247 kJ/kg at the exit of low


4MPa, 3092.5 6.582 1 0.06645
350 C
Water hr hg Sf Sg Vr Vg pressure turbine. The enthalpy of water at
the exit from the pump is 19 IkJ/kg. Use the
15
225.94 2599. 1 0.7549 8.0085 0.00 1 0 1 4 1 0.02
following table for relevant data.
kPa

h is specific enthalpy, s is specific entropy


and v the specific volume; subscripts f and g Superheated Pressure V h s
(MPa)
denote saturated liquid state and saturated
steam 3
(m /kg) (kJ/kg) (k:J/kgK)
temperature
vapour state. (OC)
500 4 0.08644 3446 7.0922

29. The net work output (kJ/kg) of the cycle is 500 8 0.04 1 77 3 399 6.7266

(GATE-ME-10) Disregarding the pump work, the cycle


(a) 498 (b) 775 efficiency (in percentage) is ___
(c) 860 (d) 957 (GATE-ME-14-SET-1)

30. Heat supplied (kJ/kg) to the cycle is


33. Steam at a velocity of I Om/s enters the
impulse turbine stage with symmetrical
(GATE-ME-10)
(a) 2372 (b) 2576
blading having blade angle 30°. The
(c) 2863 (d) 3092
enthalpy drop in the stage is 1OOkJ. The
31. The values of enthalpy of steam at the inlet nozzle angle is 20° . The maximum blade
and outlet of a steam turbine in a Rankine efficiency (in percent) is __
cycle are 2800 kJ/kg and 1800 kJ/kg (GATE-ME-14-SET-2)
respectively. Neglecting pump work, the
specific steam consumption in kg/kW-hour 34. At the inlet of an axial impulse turbine rotor,
lS (GATE-ME-1 1) the blade linear speed is 25 mis, the
(a) 3.60 (b) 0.36 magnitude of absolute velocity is 100 mis
(c) 0.06 (d) 0.01 and the angle between them is 25° . The

!11i•i!@jjl41h401b!M\Uijj+1yderabad J Delhi I Bhopal J Pune J Bhubaneswar J Lucknow I Patna I Bengaluru I Chcnnai I Vyayawada I Vi.zag I Tirupati I Kukatpally J Kolkata I
�-t . A:CE . . : 524 : Thermodynamics
� �==========================================
� :F�PnNicmooa

relative velocity and the axial component of Intermediate stage: h = 2776 kJ/kg
velocity remain the same between the inlet Exit of turbine : P = 9kPa, hr = 1 74 kJ/kg,
and outlet of the blades. The blade inlet and hg = 2574 kJ/kg, Sf = 0.6 kJ/(kg.K) ;
outlet velocity triangles are shown in the Sg = 8. 1 kJ/(kg.K)
figure. Assuming no losses, the specific If the flow rate of steam entering the turbine
work (in J/kg) is ___ is 1 00 kg/s , then the work output (in MW)
1s ___ (GATE -15 -Set l)

3 7. In a Rankine cycle, the enthalpies at turbine


entry and outlet are 3 1 59 kJ/kg and 2 1 87
kJ/kg respectively. If the specific pump
25m/s
work is 2 kJ/kg, the specific steam
(GATE-ME-14-SET-3)
consumption (in kg/kW-h) of the cycle
based on net output is ____
35. Steam with specific enthalpy (h) 3214 kJ/kg
(GATE -15 -Set 2)
enters an ·adiabatic turbine operating at
steady state with a flow rate 1 Okg/s. As it 38. In a steam power plant operating on an ideal
expands, at a point where h is 2920 kJ/kg, Rankine cycle, superheated steam enters the
1 .5 kg/s is extracted for heating purposes. turbine at 3 MPa and 350°C. The condenser
The remaining 8.5kg/s further expands to pressure is 75 kPa. The thermal efficiency of
the turbine exit, where h = 2374 kJ/kg. the cycle is --�percent.
Neglecting changes in kinetic and potential Given data:
energies, the net power output (in kW) of For saturated liquid, at P = 75 kPa,
the turbine is ---- hr = 384.39 kJ/kg,
(GATE-ME-14-SET-4) vr = 0.00 1 037 m3/kg,
sr = 1 .2 1 3 kJ/kg-K
36. Steam enters a well insulated turbine and At 75 kPa, hrg = 2278.6 kJ/kg,
expands isentropically throughout. At an Sfg = 6.2434 kJ/kg-K
intermediate pressure, 20 percent of the At P = 3 MPa and
mass is extracted for process heating and the T = 350° C (Superheated steam),
remaining steam expands isentropically to 9 h = 3 1 1 5.3 kJ/kg,
kPa. Inlet to turbine P = 14 MPa, T = 560°C, s = 6.7428 kJ/kg-K
h = 3486 kJ/kg, s = 6.6 kJ/(kg.K) (GATE - 16 - SET - 1)

!ltl•i!i§jjji4jjjjiRflnftiO,¥�Iydcrabad l Delhi I Bhopal I Pune I Bhubaneswar l Lucknow I Patna! Bcngaluru I Chcnnai IVuayawada ! Vizag IT=pati I Kukatpally I Kolkata I
.t . �CE . . = 525 Rankine C cle
w ;!
&�ip
�-::
m
�•t� ���========::..:;�:�:============�:��-���y�:
�,�Niratio,,a

39. In the Rankine cycle for a steam power plant 02. A power station produces 500 MW of
the turbine entry and exit enthalpies are 2803 power. Assuming a plant thermal efficiency
kJ/kg and 1800 kJ/kg, respectively. The of 33.3 per cent, calculate the rate of cooling
enthalpies of water at pump entry and exit water flow required in tones per hour, if the
are 121 kJ/kg and 124 kJ/kg, respectively. rise of cooling water temperature is to be
The specific steam consumption (in restricted to 5° C. The specific heat of the
kg/k.W.h) of the cycle is ___ water used is 4.2 kJ/kg-K.
(GATE - 17 - SET - 2) (GATE-ME-90)

03. In a single heater regenerative cycle, steam


enters the turbine at 30 bar, 300°C and
Five Marks Questions
condenser pressure is 0.096 bar. The feed
water heater is a direct contact type which
01. The layout of a steam power plant working
operates at 5 bar. Draw flow and h-s
on the Rankine Cycle with single stage
diagrams considering the condition of steam
regeneration is shown in figure. Assuming
at entry to the heater in mixture region.
the exit stream from the regenerator to be
Calculate steam extracted from heater to
saturated draw the corresponding ideal cycle
turbine in kg per kg of steam flow the cycle
on a Mollier Chart and derive an expression
and cycle efficiency. The enthalpies of
for the thermal efficiency of the cycle.
steam at different locations of the cycle are
B given in a table below. Pump work may be
neglected. (GATE-ME-91)

Exit Exit
Entry to Entry to Entry to
m kgs Condenser
Location from from
Turbine Heater Condenser
heater
7 2 h
2993.50 2622.22 2056.39 1 88.45 640.23
(kJ/kg)

04. In a steam power plant operating with


simple Rankine cycle, the turbine is fed with
steam at 100 bar and 673 K. Determine the
Boiler- B actual enthalpy and quality of the steam
Feed water heater- R extracted at 14 bar if the turbine stage
Pump - P (GATE-ME-89) efficiency is 90 per cent. (GATE-ME-92)

!1i11ii@jjj44ijjjQRflnlM\ijijj+yderabad I Delhi I Bhopal I Pune I Bhubaneswarl I.ncknowl Patna I Benga)uru I Chennail Vtjayawada I Vmg I T1I11pati I Kukatpally I Kolkata I
" "' · � A,CE · · .
526 ThermodynaII11cs
.. "=
��- tL ��'IPl
� :l'!'lll Nb cm,M
iei:1
:• ! ���•���!
� =========�:
� �� : ============�������:�

05. In a single stage single row impulse turbine, p To Specific Specific enthalpy Specific entropy

the steam is entering at a velocity of 1 200


ba C volume kJ/kg kJ/kg.K

mis with a nozzle angle of 20° and leaving


r m3/kg
Vf Vg hr hrg hg Sr Srg Sg
the blade in the axial direction. The ratio of 0.1 45. 0.0010 14.6 191 2392 2584 0.64 7.50 8.14

blade velocity to the tangential (whirl)


84 103 8 .9 .3 .2 88 06 94
80 295 0.0013 0.02 131 1440 2757 3.20 2.53 5.74
velocity of steam is 0.6. Sketch the velocity .1 85 35 7 .5 .5 73 51 24

diagram and calculate the work output.


(GATE-ME-93) Superheat table V 0.486
80 bar , -600° C h 3642
06. Dry saturated steam enters a frictionless s 7.0206
adiabatic nozzle with negligible velocity at a
temperature of 300° C. It is expanded to a 08. An adiabatic steam turbine receives dry
pressure of 5000 kPa. The mass flow rate is saturated steam at 1 .0 MN/m 2 and
1 kg/s. Calculate the exit velocity of steam. discharges it 0. 1 MN/m2 • The steam flow
(GATE-ME-95) rate is 3 kg/s and the moisture at exit m
Properties of Steam negligible. If the ambient temperature 1s
Specific 300K, determine the rate of entropy
production and the lost power.
Enthalpy Entropy
Volume
Sat Sat. (kJ/kg) (kJ/kgo C)
(m3/kg)
Temp. Press
(OC) (kPa) Sat Sat Sat Sat Sat Sat (GATE-ME-99)
liq. Yap. liq. Yap. liq. Yap.
Steam properties:
300 8593 1345 2751 3.2552 5.7081 0.0014 0.021 6
hr hg Sr Sg
Tsai
26.91 5000 1 154.5 2794.2 2.9206 5.9735 0.0012 0.0394
p
MN/m2 oc
kJ/kg kJ/kg kJ/kgK kJ/kgK
10 1 79.9 762.8 2778. 1 2 . 1 39 6.586
07 . A Rankine cycle operates between pressures
of 80 bar and 0.1 bar. The maximum cycle 0. 1 99.6 17.5 2675.5 1 .303 7.359
4
temperature is 600 ° C. If the steam turbine
and condensate pump efficiencies are 0.9 09. A simple impulse turbine expands steam
and 0.8, respectively, calculate the specific frictionlessly from 1 2 bar, 250 ° C with an
work and thermal efficiency. Relevant steam enthalpy of 293 5 k J/kg to an enthalpy of
table extract is given below 2584 k J/kg at 0. 1 bar. Assuming that the
(GATE-ME-98) nozzle makes an angle of 20 ° with the blade
motion, and that the blades are symmetrical,

!1J•1i@jjjl4.jjgRdnfMljj.jj� ydcrabad I Delhi I Bhopal I Pune I Bhubaneswar I Lucknow I Patna I Bcngaluru I Chennai I Vijayawada I Vu.ag I Tirupati I Kubtpa))y I, Kolkata
· · ACE
�:s•"= �·-
�· i;L�p :- �:.:..:·�========.,;,=,,::5�2:7.,;,:============�Rankin
:·�11�Pl�ii�ili�.,,. ��·:e�Cy
�c�le

find the blade velocity that produces


maximum efficiency for a turbine speed of
3600 rev/min. Assume that the steam enters
the nozzle with negligible velocity.
(GATE-ME-00)

1 0. Steam at 300 kPa and 500°C (h = 3486.0


kJ/kg) enters a steam turbine and exits at
atmospheric pressure and 350°C (h = 3 1 75.8
kJ/kg). Heat losses in the turbine are 50 kW
and the mass flow rate is 0.25 kg/s.
D etermine the power output of the turbine if
kinetic energy losses are negligible .
(GATE-ME-01)

Fydcrabadl Delhi I Bhopal I Punc l Bhubancswar l Lucknow I Patna I Bcngaluru I Chcnnai I Vliayawada l V17.3g I Tirupati I Kukatpally l Kolkata I
SOLUTIONS
07 . Ans: (d)
One Mark Solutions
08 . Ans: (c)
0 1 . Ans: (d)
09. Ans: (a)
02. Ans: (c)
cos a
Sol: - - --
u 1 0. Ans: (c)
2 Sol : 11 = f(Tm)us
2u
V 1 = -- 21tDN Thermal efficiency of a Rankine cycle
= ---
Cosa 60Cosa depends on mean temperature of heat supply
2 X 7t X 2 X 3000 = --
----- 628
1 1 . Ans: (d)
=
60 x cos 20 cos 20
= 628 I 0 . 9397 = 668 . 64 m/sec Sol: Pressure is constant and velocity decreases
in moving blades
03 .
Sol: Increases 1 2 . Ans: (d)
w S =c
Sol: 1l p t p = --
04 . w actual
Sol: decreases vdp
= -- = --
W =
S C
Wactual
1l p t p 1l p t p
T
dp
=
P 1l p t p

= (3000 - 80) =
3.435 kJ/kg
lOOO x 0.85

s
13. Ans: (c)
05. Ans: (a)
14. Ans: (a)
06. Ans: (c)

\( I } l l !!,I I H t I I l l� P11lil1t ,1111 I i i '> �ydcrabad l Delhi l Bhopal l Punc l Bhubancswar l 1.ucknow l Patna l Bcngaluru j Chennai l Vuayawada l Vizag j'l'irupati I Kukatpally j Kolkata I
':.·•..,..,,...PuNn,i.
ti ..,ACE• •
.. • •
: 529 : Rankine Cycle

12. Ans: (a)


Two Marks Solutions Sol: llo = llb X llcycle X llmech(T) X ll gen X llAux
= 0.92 X 0.44 X 0.94 X 0.95 X 0.94
01. Ans: (b) = 0.34 or 34 %

02. 13. Ans: (a)


Sol: Reduce, dryness fraction Sol: a. = 20°
u = 200 m / sec
03. Ans: (d) vrl = 350 m / sec

= 30° ; 25° ; k = 1
04. Ans: (b)
�I �2 =

Sol: Super saturated flow condition.

05. Ans: (d)

06.
Sol: Curtis; Pressure

07. Ans: (a)

08. Ans: (d) u+ vrl cos30 = 200 + 350 cos30


= 503.1
09. Ans: (b)

= 350 cos20 - 200 = 328.89 - 200


10.
= 128.89
Sol: less; less; condition for Corrosion & Erosion
and quality is more turbine work is less. u(vw + vw )
Work done /kg =
1000
1 2

Hence less efficient.


200(503.1 + 128.89)
1000
=

1 1 . Ans: (a)
= 126.39 kJ / kg
(2426 -168)2000
Sol: We = = 2000 kg / hr.
2258

l@••AIHd•IIA4144Qjif}tyderabad l Delhi l Bhopal I Punc I Bhubaneswar l "r.ucknowl Pa!na l Bcngaluru j Otennai j Vtjayawada j Vu,ag I Trrupati I Kukalpally j Kolkata I
: 530 : Thermodynamics

14. Ans: (c) 20. Ans: (b)


Sol: m h 1 + (1 - m) h2 = h3
h3 = h2 - m (h2 - h1) 21. Ans: (d)
Common Solution for Q20 and Q21
(h - h 4 )+(h 5 - h 6 )
Sol: 11 th = ( 3
h 3 - h 2 ) + (h 5 - h 4 )
(1-m) _ (3095- 2609)+(3170-2165)
-
FWH (3095 -33.3)+(3170- 2609)
h2
486 +1005 1491
= ----
3061.7 + 561 3622.7
15. Ans: A - 5, B - 1, C - 6, D - 3 = 0.41 l(or) 41.1%
h2 > h 1 , Enthalpy at the discharge from the
16. Ans: (a) pump > inlet Enthalpy
v2 kJ
Sol: 'lln = � , - = 0.5
u
:. Answer = 33.3-
ve i Ve kg

l 50
Ve = �= = 300 m / s
22. Ans: (a) 23. Ans: (a) 24. Ans: (b)
0.5 0.5
V
Ve = vlC-
ln ,
25. Ans: (b)
Sol: Reheat Cycle
ei

Ve 300
Ve =
' -= � = 316.23 m / sec
v'll n -v 0.9
26. Ans: (a)
I

ve i = 44.72� Sol: m kg

316.23 = 44.72 .JM h1 (0


316.23
.,{iiz = = 7.07
44.72
2
(Afl)s=c = 50 kJ/kg 1 kg
(1-m) kg
h2
17. Ans: (b), (c)
mh1 + (1-m) h2 = 1 h3.
18. Ans: (b) 19. Ans: (d) 0.2 X 2800 + 0.8 X 200 = h3
=> h3 = 720 kJ/kg
\( I l .rn.;111n Jill� P11l1lu ,111oih Fyderabad l Delhi l BhopaI I Pwic ! Bhubaneswar l UJcknow l Patna ! Bengaiuru l Chennai l Vliayawadal Vizag I Tuupali I Kukalpal)yl Kolkatl I
�-t &i1181..t\CE . .
-:.. . l���w.
:-
53 1 Rankine C cle
���=========;,,::��:===========:::::.=:·�;�r�:
•!Pi�:tbtiatiooa
:!

27. Ans: (a) = 225.94 + 0.803(2599.1 - 225.94)


= 2131.59 kJ/kg.
28. Ans: (c)
1
Sol: p = 1000 kg/m3 , vr = -- m 3 / kg
1000 Wr = 0.9(3092.5 -2131.59) = 864.82 kJ/kg.
Pump work,
Wp = vr dP = 0.001 x (3x103 -70) T
= 2.93 kJ/kg
By applying steady flow energy equation to
turbine
y12 _z
ril[h I + - + 1 _ +Q
- g] .
l.
2000 1000

y2 z2g
= ril[ h 2 + -2- + -
s
- +W
2000 1000
T
Wp = V 3(PBoil - P con)
(Q = 0) = 0.001014(4000 - 15)
2 2 114 - h3 = 4.04 kJ/kg.
114 = h3 + 4.04 = 225.94 + 4.04
1 60 - 100 9.81(10 - 6)
WT = 20 (32 00 - 2 600) + ---- + ---]

= 229 . 98 kJ/kg
2 000 1000
[

WT = 12156.78 kW = 12.157 MW Wnet = Wr -Wp = 864.82- 4.04


= 860.7 kJ/kg
29. Ans: (c) & 30. Ans: (c) Qs = Heat supplied
Common Solution for 29 & 30 = h1 - 114 = 3092.5 - 229.98
Sol: h 1 = 3092.5 kJ / kg ; = 2862.52 kJ/kg.
= 0.001014 m3/kg
31. Ans: (a)
V3
SJ = S2 = 6.5821 kJ/kg.K
Sol:
Sr2 + x 2 (sg2 -srJ = 6.5821 T
0.7549 + X2(8.0085 - 0.7549) = 6.5821
6.5821-0.7549 5.8272
X2 = = = 0. & 03
8.0085-0.7549 7.2536
h 2 = h f2 + X 2 (h g2 -h f2 ) s

!li11Qj@f;j4i10i@biM\hfjj� yderabad I Delhi I Bhopal I Pune I Bhubaneswar I Lucknow I Patna I Bengaluru I Chennai I Vliayawada I V17.3g I Tuupati J Knkatpally I Kolkata
Wnet = WT-Wp= (h 1-h2)- Wp 33. Ans: 88.3
= (2800-1800)- 0= 1000-
kJ Sol: ll rnax = cos2 a = cos2 20
kg = 0.883x100 = 88.3
3600 __!L 3600 __!L
SSC = = 3.6
wnet kWhr 1000 kWhr
=
34. Ans: 3280.65 J/kg

32. Ans: 43.38%


Sol:

Sol: T
-------
1 3-

Vw 1 = V 1 cos a, = 100 cos 25


= 90.63 m/sec

Vn = V 1 sin a 1 =100 sin 25


s
h 1 = inlet of high pressure turbine = 42.26 m/sec = Vf2
h, = 3446 kJ/kg
h2 = Exit of high pressure turbine
v;2 = v; - vf2 = 58.62 - 42.262
h2 = 3185 kJ/kg = 3433.96 -1785 . 91 = 1648.05
h3 = inlet of low pressure turbine Vw2 = .JI 648.05 = 40.596 m/sec
h3 = 3399 kJ/kg Blade speed (u) = 25 m/sec
hi = Exit of low pressure turbine Work (J/kg) = U (Vw l + V w2)
hi = 2247 kJ/kg = 25 (90.63 + 40.596)

hs = h6 = 191 kJ/kg = 3280.65 J/kg

Turbine work (wT) = (h1-h2) + (h3 -hi)


= (3446 -3185) + (3399-2247) 35. Ans: 7581 kW
= 261 + 1152 = 1413 kJ/kg Sol:

As pump work is neglected, Wp = 0


Heat supplied (Qs) = (h 1 -h6)+(hrh2)
= (3446- 191) + (3399- 3185)

= 3469 kJ/kg
WT -W P = 1413-0 ril 2 = 8.5kg / s
11 th = xl OO = 40.73 %
2

Q. 3469 m1 = 1.5kg I s 3

!ltl1ih§jjji4.jj§Rfln,@jj.jj �)ttyderabad j Delhij Bhopal j Pune j Bhubaneswar l Lucknow ! Patna ! Bengaluru j Chennai j Vijayawada j Vmg ! Tirupati I Kukatpallyj Kolkala I
'!.�.)•'":t·=
J!l .t\CE . .
.
l'n�,p1�IC'J'!l
:- :•!;Pu
�� �=.,========:::.=533 :===========Rankin
bticmoos · :e;,.:Cy
�cl;,:;;e
�;:;;� ;,:::,::
w(kw)=m(h 1 - h 2 ) + mi {h 2 -h 3 ) 37. Ans: 3.6 to 3.8

=10(3214-2920 ) + 8.5(2920-2374) kJ
Sol: WT = h 1 -h 2 = 3159- 2187 = 972 -
kg
= 2940 + 4641 = 7581 kW
kJ
Wp = 2 -
kg
36. Ans: 123.56 to 127.57
= 972-2 = 970 -
kJ
Sol: wnet = WT -WP
T kg
3600 = 3600 =
Steam rate = 3.711__!L
wnet 970 kWhr

38. Ans: 26 (range 25.8 to 26.1)


s
Sol:
kJ kJ
T
h i = 3486- ·, h 2 = 2776-
kg kg

6.6 = 0.6 + X 3 (8.1-0.6)


6
X3 =-= 0.8
7.5 s

h1 = 3115.3 kJ/kg
= 174 + 0.8(2574-174) = 2094kJ / kg S 1 = 6.7428 kJ/kg-K
SI = S2 = Sf X Sfg
WT (kJ I kg)= l(h l - h i ) + 0.8(h 2 - h 3 )
6.7428 = 1.213 + X X 6.2434
=1[3486-2776] + 0.8[2776-2094] 6.7428-1.213 = 5.5298
X= = 0_8857
kJ 6.2434 6.2434
= 710 + 545.6 = 1255.6
kg h2 = hr + x hrg
= 384.39 + 0.8857 X 2278.6
Power(kW) = �. (kg / sec)W, (�J
= 384.39 + 2018.16 = 2402.55 kJ/kg
=100xl255.6 =125560 Pump work, Wp = vr(P4 - P 3 ) = 3.033 kJ/kg
14 = h3 + Vf X (P4 - P3)
125560 =
Power(MW) = 125.56 MW = 384.34 + 0.001037 (3000 - 75)
1000
= 384.34 + 3.033 = 387.37 kJ/kg
l1ti1i!i§jjj4i@MAflnj@jj/fo+yderabad I Delhi I Bhopal I Pune I Bhubaneswar I Lucknow I Patna I Bengalwu I Chennai I V\iayawada I Vu.ag I Tuupari I Kukatpally I Kolkata I
: 534 : Thermodynamics

Wnet = WT- WP (h1 - h2)- Wp


= Pump work = Wp
= (31 1 5.3 - 2402.55)- 3.033 = ( 1-m) (hs-14 ) + 1( h1-h6)

= 709.72 kJ/kg Heat supplied (Qs) = (h1- h1)


Qs = HEAT SUPPLIED = h1 - 14 WT - W
- P
11th -- --
= 31 1 5.3 - 387 .37 = 2727.93 kJ/kg
Qs
Wnet 709 .72
11 th = = xlOO = 0.26 or 26%
Qs 2727 .93

1- m
39. Ans: 3.6
1
R hs
Sol: WT = h1-h2 = 2803 - 1800 = 1 003 kJ/kg
Wp = l4- h3 = 1 24 - 1 2 1 = 3 kJ/kg
WNET = WT - Wp = 1 003 - 3 = 1 000 kJ/kg Energy balance:
Specific steam consumption mh2 + ( 1- m)hs = l h6
3600 m(h2-hs ) = h6-hs
kg /kWhr
WT - Wp h6 - h5
m = ---'---'-
3600 = h2 - h5
= 3.6 kg/kWhr
1 000
02.
Five Marks Solutions Sol: Power = 500 MW
11th = 0.3333
Power W 500
01. Qs = = = = 1 500 MW
11 th 11 th 0.333
Sol: h
Heat Rejected ,Qs - W = 1 500- 500
= lOOO MW
m W (kg/sec) Cpw (kJ/kgK) (�T)w = lOOO x 1 03

. 1 000 X 1 0 3
m w (kg / sec) =
s
4.2 x 5
= 47 .62 x 1 03 kg /sec
Turbine work = WT
= 47.62 Tons/sec
= 1( h 1 --'-h2 ) + ( 1-m ) (hi-h )
3

\( l l .11g111t ( 1 111g Puhl1t .1llo1h �yderabad I Delhi I Bhopal I Punc I Bhubaneswar I lll<:kliowl Patna I Bcnga)uru I Chennai ! Vtjayawada j V,zag I T,rupati I �_y I Kolkata I
: 535: Rankine Cycle

03. 451.79
= = 0.1856 kgs.
Sol: 2433.78
1 04.
T
Sol: Superheated steam properties
B
h = 3097 kJ/kg
At 100 bar and 673K {
4 Sg = 6.218 kJ/kgK

C - Saturated steam properties


R 6 .__...--, ( t -m)kg
hr = 830 kJ/kg, sr = 2.284 kJ/kgK
At 14 bar {
hg = 2790 kJ/kg, Sg= 6.469 kJ/kgK
B-Boiter
R-Feed water heater
kJ kJ
C-Condenser
ht = 3097 - · S t = 6.218 -­
T-Turbine
P-Pump
kg ' kgK
3 kg
S l = S2 = Sn + X2 ( Sg2 - sn)
h
2622.22
6.218 = 2.284 + X2 (6.469- 2.284)
6.218 - 2.284 = 3.934
h
X2 = = 0.94
1 kg ( 1 -m) 6.469 - 2.284 4.185
h1 h2 = hn + x2 (hg2 -hn)
FWH
hs = 11(;
640.23. = 830 + 0.94 (2790 - 830)
= 2672.4 kJ / kg

s
T
h2 2993.5 kJ/kg ;
=

h3 = 2622.22 kJ/kg
hi = 2056.39 kJ/kg
hs = � = 188.45 kJ/kg
Neglect pump work
h1 = 640.23 kJ/kg s
m h3 + (1 - m) hs= lh1
h7 - h5 640.23 -188.44
m = =
3097 - h 2' .
h3 - h5 2622.22 -188.44 0.9 =
3097 - 2672.4
�yderabad l Delhi I Bhopal I Punc I Bhubaneswar l UJCknowl Patna l Bengaluru I Oicnnai I Vuayawadal V,zag I Ttrupali I Kukalpally l Kolkala I
: 536 : Thermodynamics

kJ h2 = h / +
h2 = 2714.86 -
(h h/
kg
Xz g2 - 2 )
2

= 1154.5 + 0.913 (2794.2 - 1154.5)


1

h2• = ht2 + X21 (hg2 - ht2) = 2651.65 kJ/kg


h 2, -h r2 2714.86 - 830 V = 44.72 �(h1 - h2 )
Xz• = = = 0.9616
h g2 - h r2 2790-830
= 44.72 �(2751- 2651.65) = 445.73 m/sec

05.
07.
Sol: Axial exit V w 2 = 0
Sol:
V = V 1 cosa 1 = 1200cos20
Wt

= 1127.63 m/sec.
u
- = 0.6; U = 1127.63 X 0.6
vw I
Blade velocity = 676.57 m/sec
s
Vw I UX
= 676.57 X 1127.63
W= h 1 = 3642 kJ/ kg
1000 1000
= 762.92 kJ/kg s1 = 7 .0206 kJ / kg.K
s, = S2 = 7.0206 kJ/kg.K
06. sli + x2 (sg2 - s1J = 7.0206
Sol: h 1 = 2751 kJ/kg ; s 1 = 5.7081 kJ/kgK
0.6488 + X2 7.5006 = 7.0206
s 1 = s2 = 5.7081 < 5.9735 (sg at discharge)
X

7.0206-0.6488
Wet state Xz = = 0.85
7.5006 .
s f + x2 (s - s f ) = 5.7081
2 K2 2 h2 = h r2 + x 2 (hrg ) 2
2.9206 + X2 (5.9735 - 2.9206) = 5.7081
=191.9+0.85x2392.3=2225.36 kJ/kg
5.7081-2.9206 2.7875
2 = = = 0.9 l 3 h3 = 191.9 kJ / kg
X

5.9735 -2.9206 3.0529


hi -h3 = V r3 (PBoil - Peon )
= 0.0010103 (8000- 10)
h
= 8.073 kJ / kg
hi = 191.9 + 8.073 = 199.97 kJ/ kg

s
\( l l llg111t , 1 1 11!.., P1dd1t.il1 1 , 11, �yderabad l Delhi I Bhopal I Pune l Bh�I I.ucknowl Patna! Benga)uruj Chcnnai j Vtjayawada j Vmg I Tirupati I �I Kolkata I
"T.1 •
ACE
• pg1..r • : 537 : Rankine Cycle
�-�
tit

� �,ngme,,Jq A1Rl'#iOOI
� 4 ===============================

- h4 -h3 08.
h 4 ' - h 3 = wa
ll p
-

Sol: = Sprodn = m (s2 - S 1 ) - d Q


8 073
Sgen
T
= · = 1 0.09 kJ/ kg
0.8 Adiabatic turbine, dQ = 0
Sprodn = m (s2 - s1) = 3 [7.359 - 6.586]
h 4, = h 3 + 1 0.09 = 1 9 1 .9 + 1 0.09
= 2.3 1 9 kW/K
= 201 .99 kJ I kg
To x Sprodn = 300 x 2.31 9
h 1 - h 2.
I=

llT = 0.9 = = 695.7 kW


hI -h2

3642- h 2 . 09.
=> 0.9 =
3642 - 2225.36 Sol: V = 44. 72 �( Mi )
h 2. = 2367.02kJ / kg
= 44.72 �(2935- 2584) = 837.83 m / sec
Wr = h1 h 2 . = 3642- 2367.02
u = --
cos a
-

= 1 274.98 kJ/kg V 2
-

Wp = h 4. - h 3 = 201 .99 - 1 9 1 .9 U =V
--cos a
2
= 1 0.09 kJ/ kg
837.38cos 20
= = 393.44 m/sec
Qs = h1- h 4, = 3642- 201 .99 2
= 3440 kJ/ kg
1 0.
11th =
WT - WP = 1 274.98- 1 0.09
Q8 3440 Sol: m h1 + Q = m h2 + W
(0.25x3486)-50 = (0.25 X 31 75.8) + W
= 0.3677 or 36.77 %
W = 0.25x (3486 - 31 75.8) - 50
= 27.55 kW

\( } l 111!,ll\t ( [ !II� P11hl1t ,ll1t >ll', �yderabad l Delhi l Bhopal l Pune l Bhubancswarl Licknow ! Patna ! Bengalwu ! Chennai l Vuayawada l Vizag I Tuupati I Kukalpal)y l Kolbta. I
Chapter 11 Gas Turbine
(a) P, Q (b) R, S
One Mark Questions (c) Q, R (d) P, S

0 1 . A gas turbine cycle with infinitely large 04. Consider a simple gas turbine (Brayton)
number of stages during compression and cycle and a gas turbine cycle with perfect
expansion leads to (GATE-ME-94) regeneration. In both the cycles, the pressure
(a) Stirling cycle (b) Atkinson cycle ratio is 6 and the ratio of the specific heats
(c) Ericsson cycle (d) Brayton cycle of the working medium is 1 .4. The ratio of
minimum to maximum temperatures is 0.3
02. The thermal efficiency of an air-standard (with temperature expressed in K) in the
Brayton cycle in terms of pressure ratio rp regenerative cycle. The ratio of the thermal
and y (= Cp/Cv) is given by efficiency of the simple cycle to that of the
(GATE-ME-14-SET-2) regenerative cycle is __
1 1 (GATE - 16 - SET - 2)
(a) l - -
r-1
(b) 1 - -
rP rYp

1 1 Two Marks Questions


(c) l - -
r11r
(d) 1 - (r-1 )
p
rp r 0 1 . The current level of the maximum
temperature at steam turbine inlet is much
03. For a gas turbine power plant, identify the lower than that at Gas turbine inlet because :
correct pair of statements: (GATE-ME-89)
P: Smaller in size compared to steam (a) The fuel combustion temperature in a
power plant for same power output. steam generator is lower than that in a
Q: Starts quickly compared to steam gas turbine engine.
power plant. (b) Of the corrosive nature of high
R: Works on the principle of Rankine temperature steam on super heater tubes
cycle. (c) The materials used for the gas turbine
S: Good compatibility with solid fuel. blades are not suitable for the steam
(GATE-ME-14-SET-3) turbine blades.
.\( l. l.1 1;.;,11 1el·1111g P1ti)IH.•ttHm.., Fyderabad l Delhi J Bhopal l Pune l Bhubaneswarl LucknowlPatnal Bengaiuru l Chennai l Vuayawada lVu.ag ITirupati I Kukatpally l Kolkala I
.,v.�""
-:, .
A,CE

��PnNirafiooa
. . : 539 : Gas Turbines
� � ===================================
( d) Unlike the gas turbine blades the steam 06. Select statements from List II matching the
turbine blades cannot be cooled. processes in List I. Enter your answer as
A,B if the correct choice for ( 1 ) is (A) and
02. In a gas turbine power plant intercoolers are that for (2) is (B) (GATE-ME-99)
used to cool the --------- (hot gases /
compressed air) in order to decrease the ----­
List I
( 1 ) Intercooling
-- (expansion work / compression work)
(2) Isothermal compression
(GATE-ME-89)
List II
(A) No heat transfer during compression
03. Air expands steadily through a turbine from
(B) Reduces low pressure compressor work
6 bar, 800 K to 1 bar, 520K. During the
(C) Heat rejection during compression
expansion, heat transfer from air to the
(D) Reduces high pressure compressor work
surroundings at 300 K is 1 0 kJ/kg air.
Neglect the changes in kinetic and potential 07. In a gas turbine, hot combustion products
energies and evaluate the irreversibility per with the specific heats Cp = 0.98 kJ/kg K,
kg air. Assume air to behave as an ideal gas and Cv = 0.7538 kJ/kgKenter the turbine at
with Cp= l .OkJ/kgKand R=0.3 kJ/kgK. 20 bar, 1 500 K and exit at 1 bar. The
(GATE-ME-93) isoentropic efficiency of the turbine is 0.94.
The work developed by the turbine per kg of
04. In problem 03 find the actual work and gas flow is (GATE-ME-03)
maximum work per kg air. (GATE-ME-93) (a) 689.64 kJ/kg (b) 794.66 kJ/kg
( C) 1 009. 72 kJ/kg (d) 1 3 1 2.00 kJ/kg
05. A gas turbine cycle with heat exchange and
reheating improves (GATE-ME-93) 08. The pressure ratio of a gas power plant cycle
(a) Only the thermal efficiency corresponding to maximum work output for
(b) Only the specific power output the given temperature limits of Tmin and Tmax
( c) Both thermal efficiency and specific will be (GATE-ME-04)
power output
T ) 2< : -1 J T . ) 2< : -1 J
( d) Neither thermal efficiency nor specific (a) ( � (b) (___!!!!!!_
power output
Tmin Tmax

!IS11ih§jjji4ijjjiQnGiMmih� yderabad I Delhi I Bhopal I Pune I Bhubaneswar I Luclmow l Patna I Bengaluru I Chennai I Vijayawada I Vuag I Tirupati I Kukatpallyl Kolkata
".
.... '<>,,ACE : 540 : Thermodynamics

"��Pnlmtiom
===================================
09. An ideal Brayton cycle, operating between 1 2. In an ideal Brayton cycle, atmospheric air
the pressure limits of 1 bar and 6 bar, has (ratio of specific heats, Cp I Cv = 1 .4, specific
minimum and maximum temperatures of heat at constant pressure = l .005kJ/kg.K) at
300 K and 1 500 K. The ratio of specific 1 bar and 300 K is compressed to 8 bar. The
heats of the working fluid is 1 .4. The maximum temperature in the cycle is limited
approximate final temperatures in Kelvin at to 1280K. If the heat is supplied at the rate
the end of the compression and expansion of 80MW, the mass flow rate (in kg/s) of air
processes are respectively. required in the cycle is ____
(GATE-ME-11) (GATE-ME-14-set-2)
(a) 500 and 900 (b) 900 and 500
(c) 500 and 500 (d) 900 and 900 1 3 . The pressure ratio across a gas turbine (for
air, specific heat at constant pressure, Cp =
Statement for Linked Answer QlO & Qll 1 040 J/kg.K and ratio of specific heats, y =
In a simple Brayton cycle, the pressure ratio is 8 1 .4) is 1 0. If the inlet temperature to the
and temperatures at the entrance of compressor turbine is 1 200 K and the isentropic
and turbine are 300 K and 1400 K, respectively. efficiency is 0.9, the gas temperature at
Both compressor and gas turbine have isentropic turbine exit is --- K.
efficiencies equal to 0.8. For the gas, assume a (GATE - 17 - SET - l)
constant value of Cp (specific heat at constant
pressure) equal to 1 kJ/kgK and ratio of specific Five Marks Questions
heats as 1 .4. Neglect changes in kinetic and
0 1 . A gas turbine is supplied with gas at 5 bar
potential energies.
and 1 000 K and expands it adiabatically to 1
bar. The mean specific heats at constant
1 0. The power required by the compressor in
pressure and constant volume are
kW/kg of gas flow rate is (GATE-ME-13)
l .0425kJ/kgK and 0.7662 kJ/kg K
(a) 1 94.7 (b) 243 .4
respectively.
(c) 304.3 (d) 378.5
(a) Draw the temperature-entropy diagram
to represent the processes of the simple
1 1 . The thermal efficiency of the cycle in
gas turbine system.
percentage (%) is (GATE-ME-13)
(b) Calculate the power developed in kW
(a) 24.8 (b) 38.6
per kg of gas per second and the
(c) 44.8 (d) 53 . l
exhaust gas temperature.
jl\llM@OidijjjQRfiftiMijjijj+yderabad l Delhi l Bhopali Pune l Bhubaneswari LucknowlPatnal Bengaiuru ! Chennai l Vuayawada l Vmg IT=pari I Kukatpally i Kolkata I
: 541 : Gas Turbines

(GATE-ME-92) that the efficiency of the regenerator and the


adiabatic efficiency of the turbine are each
02. A closed cycle ideal gas turbine plant 80%. Assume for air the ratio of specific
operates between temperature limits of 800° heats is 1.4. Also, show the cycle on a T-s
C and 30 C and produces a power of 100
°
diagram. (GATE-ME-97)
kW. The plant is designed such that there is
no need for a regenerator. A fuel of calorific 05. Find the required air- fuel ratio in a gas
value 45000 kJ/kg is used. Calculate the turbine whose turbine and compressor
mass flow rate of air through the plant and efficiencies are 85% and 80% respectively.
the rate of fuel consumption. (Assume Cp =
Maximum cycle temperature is 875 ° C. The
1 kJ/kg K and the ratio of specific heats working fluid can be taken as air (Cp 1.00 =

= 1.4). (GATE-ME-94) kJ/kg K, y= 1.4) which enters the


compressor at 1 atm and 27 C. The pressure °

03. In a gas turbine the compressor is driven by ratio is 4. The fuel used has calorific value
the high pressure turbine. The exhaust from of 42000 kJ/kg. There is a loss of 10% of
the high pressure turbine goes to a free low calorific value in the combustion chamber.
pressure turbine which runs the load. The air (GATE-ME-98)
flow rate is 20 kg/s and the minimum and
maximum temperatures are respectively 300 06. In an ideal air - standard Gas turbine cycle,
K and 1OOOK.The compressor pressure ratio the minimum and maximum temperatures
is 4.Calculate the pressure ratio of the low are respectively 310 K and 1100 K. Draw
pressure turbine and the temperature of the the cycle on a T-S diagram and calculate the
exhaust gases from the unit. The compressor optimal pressure ratio of the cycle for
and turbine are isentropic. Cp of air and maximum work output. Assume for air
exhaust gases 1.005 kJ/kgKand y
= =
1.4. (y - 1)1 r is the ratio of specific heats.
(GATE-ME-95) (GATE-ME-00)

04. In an air - standard regenerative gas turbine 07. A Brayton cycle ( air standard) has a
cycle the pressure ratio is 5. Air enters the pressure ratio of 4 and inlet conditions of
compressor at 1 bar, 300Kand leaves at 490
K. The maximum temperature in the cycle is
one standard atmospheric pressure and
27 °C. Find the air flow are required for 100
1000K. Calculate the cycle efficiency given kW power output if the maximum
!lflli@jjji4@$jnnj@jn.jj�)llyderabad ! Delhi l Bhopa) I Pune l Bhubancswar l Lucknow ! Patna ! BengaJuru ! Chennai l Vliayawada l Vu.ag ITuupati I Kukatpallyl Kolkata I
: 542: Thermodynamics

temperature in the cycle is 1 000°C. Assume 09. An ideal, air standard regenerative Brayton
y = 1 .4 and Cp = 1 .0 kJ/ kg-K. cycle is working between minimum and
(GATE-ME-01) maximum temperatures of 300 K and 1 200
Krespectively. (GATE-ME-02)
08. Air at 327 ° C and 400 kPa with a volume (a) Find out the value of critical pressure
flow rate of 5 m3/s flows through a turbine rat io where the degree of regeneration
and exits at 1 00 kPa and 182° C. If the becomes zero.
expansion process is polytropic, calculate Calculate the efficiency of the cycle when
power output , rate of heat transfer and rate the operating pressure ratio is 60% of the
of change in entropy(specific heat at critical pressure ratio.
constant pressure of air = 1.0035 kJ/kg-K,
and Gas constant of air = 0.287 kJ/kg-K).
(GATE-ME-02)

!ltl1ih§jjji4jjjjiRflbiM111ih� yderabad l Delhi I Bhopal I Punei Bhubaneswarl Lucknow! Patna I Bengaluru I Chennai I Vijayawada! Vizag I Tirupari I K�y l Kolkata
SOLUTIONS

One Mark Solutions

01. Ans: (c) {11th }Idealregen - 1 - -'!!.!!L


T . • (r Fr
-1
Tmax
- Y
p

02. Ans: (d) = 1-0.3(6)1.4 = 0.49816


1 .4 -1

(11th )B = � = 0.803
I

Sol: 11 th = - (
1 1 J y; 1 (1
(11th )IR 0.498
y -1

rP
= - -;:-
J

Where, rp = Pressure Ratio


rk = Compression Ratio Two Marks Solutions
Pressure Ratio (rp) = p2 = p3
P1 P4
01. Ans: (d)
Compression Ratio (rk) = 2
V2 02.
Sol: Compressed air , Compression work

03.
Sol:
T

20
· '\
3
V
4'
f 4
03. Ans: (a) 1

s
04. Ans: 0.803 (range 0.79 to 0.81)
P3= 6 bar, r = - p3 = 6
Sol: _!!!!!.!.. = 0.3 p4
T .
p

T3= 800K
Tmax

Pressure ratio = rp = 6
P 1 = P4· = 1 bar
y= 1.4
Fyderabad l Delhi I Bhopal I Punc I Bhubaneswar l 1..ucknowl Patnal Bengaluru I Olcnnai I Vuayawada j Vu.ag I TIJUpari I Kukalpallyj Kolkata I
: 544 : Thermodynamics

T4· = 520 K 07. Ans: (a)


To = 300 K Sol: 3
T
T3 = 800 = 2
T = 479.47K
(r , r 4
0.4
\I:! (6)u
p
4

c1

I = Irreversibility = T0 [( ds tys + (ds) surr ] s

T3 = 1500 K
0.98
y- - =1.3
cp
--
C V 0.7538
-
520
30o {[ un -0.3 ln _!._J+ � }
800 6 300 T3 1500
=

T4 = =
751.36K
= 300 [-0.4307 + 0.5375 + 0.0333] ( rp )( y; ) (201 ii
0. 3 )
l

= 42.04 kJ/kg. Tl _ wactual


T -
wisen

04. Wact = llT Cp(T3 - T4)


Sol: Wmax = Cp (T3 - T4) = 0.94 X 0.98 (1500 - 751.36)

= 1 (800 - 479.47) = 320.53 kJ/kg = 689.64 kJ/kg


(Work)actual = Cp (T3 - T/)
= 1 (800 - 520) = 280 kJ/kg 08. Ans: (a)

09. Ans: (a)


05. Ans: (c)
Sol : T

06. Ans: (1) - (d); (2) -(c)


Sol:

3� 2
t1 4

s
T1 300 K ,
=
T 3 1500 K
=

P P
4 1 rP = -2 = -3 =
pressure ratio
P1 P4

6
V
r =-=6
p 1
Fydcrabad j Dcllii j Bhopal j Punc ! Bhubwswai:I Lucknowj Pamaj Bcngaluru j Chcnnai l V\iayawada l Vizag !T1111pari I Kukatpallyj Kolkatal
:545: Gas Turbine

Qs =C P (T3 -T;) = 1(1400-604.29) = 795.71


11 = wnet = 196.71 = 0.247
T3 1500 Qs 795.71
T4 = ( \12 = 0.4 = 900K
rp , y 61.4
12. Ans: 108.072

10. Ans: (c) Sol: y= 1.4


Cp =1. 005 kJ/kg.K
11. Ans: (a) Tmax = 1280K
Sol: Qs= 80MW

0
T
ril=?
rp = 8 T1 = 300K
'Ile = 11T = 0.8 T3 = 1280K
T1 = 300K 1 rp = Pressure ratio = 8
T3 = 1400K s T

T2 = T1 (rP Fr = 300x8 ii = 543.43K


0.4
y-1
2

1- T2 -Ti
T2- T1+�- 1
1le
s
544-300
= 300+ = 604.29K
0.8
T3 1400
T4 = = = 772.86K
(r r )r-i
(8)°'41 1 .4
P

T� = T3 -ri T (T3 -T4 ) T,- T,(�: f-300 (8):: -543.43K


1400 -0.8(1400 -772.86)
Qs = ril Cp(T3 -T2)
=

= 898.29K

80 X 103 = ril X l.005(1280-543.43)


We =C P (T;-T1 )
• = 80 X 10 3
= 1.00 (605-300) = 304.29 kW/ kg = 108-072 kg/sec
m l.005(1280-543.43)
WT =C p (T3 -T�) = 1(1400-898.29) = 501.71
wnet = WT - We = 501.71-305 = 196.71
\( I I IH.!,lllt t 1111� P1il,lu .1111 >11, �yderabadlDelhilBhopalJPuneJBhubancswarl LucknowlPatnaJBcngaiuruJOiennailVuayawadaJVizag ITuupati I Kukatpal)yj KQlkala I
: 546: Thermodynamics

13. Ans: 680


Sol:
Five Marks Solutions
T
3

01.
Sol:
3
T
s 2
4

Cp = 1040J/kgK
C
r=-P =1.4 s
Cy
P2 = P3 = 5 bar
p2 =10
r =- T3 = 1000 K
p pI
Cp = 1.0425 kJ/kgK
Tinlet = T1 = 1200K
Cv = 0.7662 kJ/kgK
Cp 1.0425
= = =1.36
y Cv 0.7662
Temperature at exit is isentropic P1 = 1 bar
T2 = 621.53K P2 _ P3 _ 5 _
------ 5
P1 P4 1
llT = 0,9 =
T-T
I 3
T1 -T2

T3 = TI - llT (TI-T2)
T
T4 = _ �\_-1 =
10
= 653.098K ?.�
(rp ) Y (5) u6
= 1200-0.9(1200-621.53)
WT = ma Cp (T3-T4)
= 679.38K = 680K
= 1 X 1.0425 [ 1000 -652.098]
= 361.645 kW

�ydcrabad I Dellii I Bhopal I Punc I Bhubancswarl Lucknow I Palnal Bcngaluru I Cheruiai I Vuayawada I Vizag I Tllllpati I Kukatpa))y I Kolkata I
: 547: Gas Turbine

02. AFR = 89.051


Sol: ma = 89.051
T mr
0.423 kg/sec = 89_051
mr
3
mr= 4.75 x 10- kg/ sec = 17.103 kg/ hr
s
03.
Sol:
T1 = 303 K; T3 = 1073 K
T

= .J303 x 1073 = 570.19K


No need for regenerator condition.
It is operating at critical pressure ratio. s
p p2
W = ma C Pa [Jr; -jr;f rp= 4= _ =-3
P1 Ps
100 = ma X 1.005 [.J1073 - .J3mr T1 = 300 K
= ma x 1.005 [32.756-17.406]
2
T3 = 1000 K
ma 0.423 kg/ sec
y-1 0.4

300 (4) 14
=
T2 = T1 (rp) r = = 445.8 K
ma C pa T2 + mr(CV) YJc = ma C Pa T3
WHPT = We
Cp (T3-T4) = Cp (T2-T,)
1000-T4 = 445.8-300
(AFR) C Pa T2 + (CV) YJc = (AFR) C Pa T3 T4 = 854.2 K

(AFR) 1.005 x 570.19 + 45000 x 1 T3 r-1


= (r Jr
T4
(AFR) 1.005 x 1073
P HPT
=

T r�1 1000 �:: =


(rp )HPT = ( 3 J = ( ) 1.736
T4 854.2

(rp )comp (rp tPT (rp )LPT


=
X

4 = 1.736 X (rP)iPT

(rp )LPT = 2.304


4
--
=
1.736
!M11iii§jjji4.jjj$Rflb1Mihih•� yderabad I Delhi I Bhopal I Pune I Bhubaneswar I Lucknowl-Fatna I Bengalwu I Chennai I Vtjayawada I Vmg I Tuupati I K�y I Kolkalll.
. .t\CE . . 548 : Th erm od aIIll · C S
':,.,,�. ��'.r.,11F �tp1�1M
. "= ���=======,,;,= ��
=ffll�·!Pn�bticfuw ==========�� � ��yn� � �

Wr = Cr(T3- T4,)
( )-
r-1
T
--1__ r
rP LPT
T. = 1.005 (1000- 705.1)
0
5

T4 854.2 = 296.37 kJ/kg


Ts = =
0 4 = 672.96K.
Wc = Cr( T2 -Ti)
t
(r LPT \I='-
p / Y (2.304 )ii
= 1.005 (490 - 300) = 190.95 kJ/kg
04. Qs = Cr(T3-T3,)
Sol: = 1.005 (1000 -662) = 339.7 kJ/kg
T _ WT -We 296.37-190.95
11r- --- =
Q5 339.7
= 0.3103 or 31.03 %.

s 05.
Sol:
P 1 = 1 bar; T1 = 300K
p
T
rp = 5 = .....1.
pI
T� = 490K; T3 = 1000K
T3, - T2
= effectiveness ofregenerator =
T4,-T2
E
s
T1 = 300K
T3 -T4,
llT = 0.8 = T3 = 875 + 273 = 1148K
T3 -T4
rP = 4 ; llc = 0.8 ; llr = 0.85
- T3= 100_0 04 = 631.38K
T 4-
= 300 (4) 1.4 = 445.8K
=T1 . rr r
y-1

T2
y-1
(rp )r (5)ii
0.4

1000-T4,
O8=
flT . (1000- 631.38)
=

T4, = 705.1K

T3 -490
0.8 = . => T3. = 662K
705.1-490

!M11ijj§jjji4.jjiRflftjM\11.jj..)ltyderabad I Delhi I Bhopal I Pime I Bhubaneswar I Lucknow! Patna I Bengaluru I Chennai I Vuayawada I Vizag I Tirupati I Kukatpally I Kolkata I
Pnb6rarioos
ACE
� EF.ngintaing : 549 : Gas Turbine
�-�
. �=====================================
T� = T3 -1h(T3 -T4 ) T1 = 300 K , T3 = 1273 K ; rp = 4
=1148-0.85(1148-772.5) =829.5K y-1 0.4
T2 = T1 (rP )r = 300 (4)ii = 445.8 K
T 1273
T4 = 3r-1 0.4 = 856.7 K
-
(rp )r 4 1 .4
WT = Cp (T3-T4 )
= 1(1273 - 856.7) = 416.3 kJ / kg

Cv;ll c Wc = Cp (T2-T1)
m. = 1(445.8 - 300) = 145.8 kJ/kg
Cpa;
,__--+--� T3
= 416.3 - 145.8 = 270.5 kJ/kg
Power = ma X Wnet
CV· llc 100 = ma x 270.5;
(AFR) =
er. (T3 - T2I )
ma = 0.3696 kg I sec.
42000x0.9
= =56_76
1(1148-482) 08.
Sol: T 1 = 600 K ; P 1 = 400 kPa ;
06. V = 5 m3 /sec ; P2 = 100 kPa ;
Sol: 3 T2 = 273 + 182= 455 K
T

T 2(:- 1 )
(rP )optimum = ( max ]
Tmm.
---------
1 .4
1100 2(0.4) 455
=(--) =9.715 1n( )
300 600
n
s
= ---
l
ln( OO)
400
07.
Sol:
0·2766
T
= = 0.199 n = 1.25
1.3863
P1 V1 _ 400x5
m= ---
RT! 0.287x 600
= 11.616 kg/sec
s

!IJ11ij@jjjl4@iRflbjM\jijjj*JlYderabad I Delhi I Bhopal I Pune I Bhubaneswar I Lucknow! Patna I Benga)uru I ChelUlai IVuayawada I Vizag I Ttrupati I Kukatpa))y I Kolkata I
: 550 : Thermodynamics

Turbine work = _n_ m R(T 1 - T2)


n-1 09.
= 1.25 Sol:
( )x 1l.616x 0.287(600-455)
1.25-1
= 2417 kW '
?
,,,
ideal regenerative

fl.Brayton

--
' /
'
'

Heat Transfer = y -n (m)R(T, -T )


(y-l)(n -1) 2
r C

__l _.4 _ -_l_.2_5__


X 11.616
(1.25 -1)(1.4 -1)
0.287(600 -455)
X
= 724.72 kW 1.4

1200 = -75
2x0.4

= { } (4) 1 = 11.313
Rate of change of entropy 300

mR l n ( 2)
n-y T r p = 0.6x rp c = 0.6 x 11.313 = 6.78
-
- (y -l)(n-1) � = 1 _ Tmin (rp f�')
1.25 -l.4
fl.
Tmax
X 11.616
= ( Brayton )
ideal regenrative
(1.4 -1)(1.25-1)
300
455
= 1- (6.78) �::
X 0.287 X ln (-) 1200
600 = 0.25(6.78)° -2857
= 1.3832 kW/K = 0.568 or 56.8%.

!Mlli@jjj44ijj/4jmnlM\hfjj� yderabadlDelhijBhopal!PunelBhubaneswarl LucknowjPatnalBengalurujChennailVijayawada!Vmg !Trrupari I Kukatpallyj Kolkata


c12 Refrigeration
04. A po s1·t·1ve value of Joule-Thomson
One Mark Questions coefficient of a fluid mean s
(GATE-ME-02)
O1. In a vapour compression refrigeration (a) Temperature drops during throttling
system, liquid to suction heat exchanger is
(b) Temperature remains constant during
u sed to (GATE-ME-00)
throttling
(a) Keep the COP constant
(c) Temperature rise s during throttling
(b) Prevent the liquid refrigerant from
entering the compressor (d) None of these
(c) Sub-cool the liquid refrigerant leaving
05. In the window air conditioner, the expansion
the conden ser
(d) Sub-cool the vapour refrigerant from device u sed is (GATE-ME-04)
the evaporator (a) capillary tube
(b) thermostatic expansion valve
02. Global warming is caused by (c) automatic expansion valve
(GATE-ME-00)
(d) float valve
(a) ozone
(b) carbon dioxide 06. Environment friendly refrigerant Rl 34a i s
(c) nitrogen u sed in the new generation domestic
(d) carbon monoxide refrigerators. It chemical formula i s
(GATE-ME-04)
03. A ga s having a negative Joule-Thompson
(a) CH ClF2
coefficient (µ<O), when throttled, will
(b) C2 Ch F3
(GATE-ME-01)
(c) C2 Ch F4
(a) become cooler
(d) C2 H2 F4
(b) become warmer
(c) remain at the same temperature
07. In an ideal vapour compression refrigeration
cycle, the specific enthalpy of refrigerant (in
(d) either be cooler or warmer depending on
the type of ga s
kJ/kg) at the following states is given a s:
\('I, I 11g1Tll'l 11ng Puhln ,1t1oth yderabad I Delhi I Bhopal I Pune I Bhubaneswar I Lucknow I Patna I Bengaluru I Chennai I Vtjayawada I Vu.ag I Tuupati I Kukaipally I Kol.k.ata
:552: Thermodynamics

02. The use of Refrigerant-22(R-22) for


T temperatures below -30°C 1s not
recommended due to its (GATE-ME-93)
(a) good miscibility with lubricating oil
(b) poor miscibility with lubricating oil
�I (c) low evaporating temperature
s
(d) high compressor discharge temperature
Inlet of condenser: 283
Exit of condenser: 116 03. Clearance volume of a reciprocating
Exit of evaporator: 232 compressor is 100 ml, and the volume of the
(GATE-ME-09) cylinder at bottom dead centre is 1.0 liter.
The COP of this cycle is The clearance ratio of the compressor is
(a) 2.27 (b) 2.75 (GATE-ME-97)
(c) 3.27 (d) 3.75 (a) 1/11 (b) 1/10 (c) 1/9 (d) 1/1.1

08. Which one of the following 1s a CFC 04. List- I


refrigerant ? (GATE-ME-14-SET-1) A. Liquid to suction heat exchanger
(a) R744 (b) R290 B. Constant volume heat addition
(c) R502 (d) R718 C. Normal shock
D. Ammonia -water
List-II
1. Vapour absorption refrigeration
Two Marks Questions
2. Vapour compression refrigeration
3. Diesel cycle
01. A refrigeration compressor, designed to 4. Otto cycle
operate with R22 __ (can/cannot) be 5. Converging nozzle
operated with R12 because the condensing 6. Converging -diverging nozzle
pressure of R22 at any given temperature is
(higher/lower) than that ofRI2. Common Data for Question 05 & 06
(GATE-ME-92) A refrigerator based on ideal vapour
compression cycle operates between the
temperature limits of-20°C and 40°C. The

I I I I I I I I I I
�ydcrabad Delhi Bhopal Punc Bhubaneswar Lucknow Patna Benga)wu Chennai Vtjayawada Vizag rnrupati I Kukatpally I Kolkata I
ACE �·��========�: :55�3�: ============�R:e:frig
�·:era
:�ti:o:n
�-�

� ..E'.!=Fn�p�· -:�rin�·�Puhtiamooa��·
refrigerant enters the condenser as saturated
vapour and leaves as saturated liquid. The
enthalpy and entropy values for saturated
liquid and vapour at these temperatures are
given in the table below.

Temp hr hg Sf
Sg
(kJ/kg
65 207
O
( C) (kJ/kg) (kJ/kg) (kJ/kg K)
K)
176
h kl/kg
-20 20 180 0.07 0.7366
40 80 200 0.3 0.67

(a) 6.35xl 0-3 m 3 Is (b) 63.5xl 0-3 m 3 Is


05. If refrigerant circulation rate is 0.025 kg/ s, (c) 635xl 0-3 m 3 Is (d) 4.88x10-3 m 3 Is
the refrigeration effect is equal to
(GATE-ME-03)
08. The vapour compression refrigeration cycle
(a) 2.1 kW (b) 2.5 kW
is represented as shown in the figure below,
(c) 3.0 kW (d) 4.0 kW
with state 1 being the exit of the evaporator.
The coordinate system used in this figure is
06. The COP of the refrigerator is
(GATE-ME-05)
(GATE-ME-03)
(a) 2.0 (b) 2.33 2

(c) 5.0 (d) 6.0

07. A R-12 refrigerant reciprocating compressor


operates between the condensing
temperature of 30°C and evaporator
(a) p-h (b) T-s
temperature of-20°C. The clearance volume
(c) p-s (d) T - h
ratio of the compressor is 0.03. Specific heat
ratio of the vapour is 1.15 and the specific
09. Which combination of the following
volume at the suction is 0.1089 m3 /kg. Other
statements is correct? (GATE-ME-07)
properties at various states are given in the
P: A gas cools upon expansion only when
figure. To realize 2 Tons of refrigeration, the
its Joule-Thomson co-efficient is
actual volume displacement rate considering
positive in the temperature range of
the effect of clearance is (GATE-ME-04)
expans10n.
\{'I l 111..;111tt llll� P11lilh,IIJ1)[I', I I I I I I I I I I I
�ydcrabad Delhi Bhopal Punc Bhubancswar Lucknow Patna Bcnga)uru Chcnnai Vgayawada Vizag T1111pati I Kukatpally I Kolkara I
Po-'-�d
" • ACE
• · 554 �·
· ===========� ermodynann·cs
Th�����;
�-:oo:•=========�· ·�
v,.,____
�:�v�1111�-:: •�w����•�
�......"=

Q: For a system undergoing a process, its 11. The power required for the compressor in
entropy remains constant only when kW is
the process is reversible. (a) 5.94 (b) 1.83 (c) 7.9 (d) 39.5
R: The work done by a closed system in
12. A heat pump with refrigerant R22 is used for
an adiabatic process is a point function.
space heating between temperature limits of
S: A liquid expand upon freezing when
-20° C and 25° C. The heat required is
the slope of its fusion curve on
200MJ/h. Assume specific heat of vapour at
Pressure Temperature diagram 1s
the time of discharge as 0.98kJ/kg.K. Other
negative.
. (b) P andQ
relevant properties are given below. The
(a) R and S
enthalpy (in kJ/kg) of the refrigerant at
(c)Q, R and S (d) P,Q and R
isentropic compressor discharge is ___
(GATE-ME-14-SET-2)
Common Data for Question 10 & 11
Saturation Pressure Specific enthalpy Specific entropy
A refrigerator operates between 120kPa and
p hr hg Sg
temperature

800kPa in an ideal vapor compression cycle with Sr


R -I34a as the refrigerant. The refrigerant enters
[:] [:]
(MPa)

the compressor as saturated vapor and leaves the


[k:KJ [k;K]
-20 0.2448 177.21 397.53 0.9 139 1.784 1
condenser as saturated liquid. The mass flow rate 25 1.048 230.07 413.02 1.1047 1.7183
of the refrigerant is 0.2 kg/ s. Properties for
R-I34a are as follows. ( GATE-ME-12) 13. The thermodynamic cycle shown in figure
(T-s diagram) indicates.
Saturated R - 134a
p T(OC) hr
kPa kJ/k
120 -22.32 22.5

Su erheated R-134a
800 31.31 95.5

p T(OC) h(kJ/kg) s(kJ/kg.K)


kPa
800 40 276.45 0.95

(GATE-15-Set 3)
10. The rate at which heat is extracted, in kJ/ s
(a) Reversed Carnot cycle
from the refrigerated space is
(a) 28.3
(b) Reversed Brayton cycle
(b) 42.9
(c) 34.4 (d) 14.6
(c) Vapor compression cycle
(d) Vapor absorption cycle
yderahadl Delhi I Bhopal I Punel Bhubaneswarl Lucknow I Patna! Bengaluru I Chennai IVuayawada JVu.ag ITU11pali I Kukatpallyj Kolkala
: 555 : Refrigeration

14. Refrigerant vapor enter s into the compressor evaporation for the refrigerant at 310 K is
of a standard vapor compression cycle at 1 0 54 kJ/kg.
-10 ° C (h = 402kJ/kg) and leaves the
compressor at 50 ° C (h 432 kJ/ kg). It
T
=

leave s the condenser at 30 ° C ( h=237


kJ/kg). The COP of the cycle 1s
(GATE-15 -Set 3)

1 5. A refrigerator use s R-134a a s its refrigerant


s

The difference between the enthalpies at


and operates on an ideal vapour ­
state points 1 and O (in kJ/kg) is ----
compression refrigeration cycle between
(GATE - 16- SET - 3)
0.14 MPa and 0.8 MPa. If the mass flow rate
of the refrigerant i s 0.05 kg/s, the rate of
heat rejection to the environment is kW.
Given data : Five Marks Questions
At P = 0.14 MPa, h = 236.04 kJ/kg ,
01. A reciprocating compressor produces 1 Oton s
s = 0.9322 kJ/kg-K
of refrigeration at an evaporator temperature
At P = 0.8 MPa,
h = 272 .05 kJ/kg (superheated vapour) of 5 ° C and a condenser temperature of 3 5 °c.
At P = 0.8 MPa , E stimate how much cooling effect it can
h = 93 .42 kJ ( saturated liquid) produce at an evaporator temperature of-
(GATE - 16 - SET - 2) 30 °C, the condenser temperature remaining
un-altered.
16 . In the vapour compression cycle shown in Given : Refrigerant : R- 12 , clearance = 5% ,
the figure the evaporating and conden sing index of compression(in both case s) i s 1. 1 5.
temperatures are 260 K and 31 O K , Extract from refrigerant property tables:
respectively. The compressor takes in liquid­ (GATE-ME-87)
vapour mixture ( state 1) and isentropically Specific volume

compresse s it to a dry saturated vapour


Enthalpy(k.J/kg)
Pressure (m3/l,a\
Temp
(bar) Liquid
Vapour Liquid Vapour

condition (state 2). The specific heat of the


X l o'
-30 c
° 1 .005 0.672 0. 1 600 8.88 1 74.96

liquid refrigerant is 4.8 kJ/kg-K and may be


+5°c 3.63 1 0.725 0.0478 40.8 1 1 90.88
+35 °c 8.50 1 0.787 0.0208 69.74 202.96
treated as con stant. The enthalpy of
lli•IMhYh!iiih!QRflnj@\j/jjj� yderabad I Delhi I Bhopal I Pune I Bhubaneswarl Lucknow I Patna I Bengaluru I Chennai I Vijayawada I Viz.ag I Tirupati I Kukatpallyj Kolkata
ACE
�-
�� ..:�=&�1g111
�·:m ·i
:
:m w ��
mlicatirm
Thermod armcs
�·�·============::���:===========��;��yn
55 6
��·�

02. A Freon - 12 vapour compression system is (i) The refrigerant mass flow rate , and
operating at a condenser pre ssure of 9.6 bar (ii) The compressor intake volume flow
(960 kPa) and an evaporator pressure of 2 .19 rate if the compressor volumetric
bar (219 kPa). Its refrigerating capacity is 15 efficiency is 0.72 .
tons. The value s of enthalpy at the inlet and Use the refrigerant property data given m
the outlet of the evaporator are 64.6 kJ/kg Table below . (GATE-ME-93)
and 195 .7 kJ/kg . The specific volume at the Enthalpy Volume

inlet to reciprocating compressor is 0.082


Temp Pressure Sat. Sat. Sat. Sat.
oc bar Liquid Vapour Liquid Vapour

m 3/kg . The index of compression (or


kJ/k2 kJ/k2 m3/k1? m3/k2

polytropic exponent) for the compressor i s


5 5.836 205.9 407. 1 0.791 0.0404
50 1 9.423 263 .3 4 1 7.7 0922 0.0 1 1 7

1.13. (GATE-ME-88)
Determine 05 . Sketch the standard vapour compression
(a) The power input in kW refrigeration cycle on the enthalpy (y-axi s) -
(b) The coefficient of performance entropy (x-axis) plane , indicate the
directions and processe s clearly.
03 . A reciprocating compressor in a vapour (GATE-ME-94)
compression refrigeration system has a
clearance factor of 4 percent and a swept 06 . A refrigeration compressor of 50 mm bore
volume of 26 liter s/second . If the ma ss flow and 40 mm stro ke operates at a speed of
of refrigerant is 0.46 kg/s and the specific 1460 rpm between condensing and
volume of vapour at compressor suction and evaporating pressure limits of 1 .219 MPa
discharge are 37 liters/kg and 12 liter s/kg and 0.151 MPa . The clearance ratio is 5%,
respectively, calculate the clearance ratio of specific heat s of refrigerant is 1 .18 ,
volumetric efficiency and the actual specific volume of refrigerant at suction is
volumetric efficiency of the compre ssor . 0.11 m 3/kg and the enthalpy change of
(GATE-ME-89) refrigerant in the evaporator i s 93 .7 kJ/kg .
Calculate the refrigeration load the
04. A refrigeration system of 15 ton s capacity
compre ssor can serve .
operates on standard simple vapour
(GATE-ME-94)
compression cycle using Refrigerant-22 at
an evaporator temperature 5 °C and
condensing temperature of 50° C. Draw the
p-h diagram for the cycle . Calculate .
!li11Qjj§jjj44ijji4Rflfl!M\iiih� yderabad I Delhi I Bhopal I Pune I Bhubaneswar I Lucknow I Patna I Bengaluru I Chennai I Vliayawada j Vizag I Tuupati I Kukatpal)y I Kolkata
: 557: Refrigeration

07. A refrigeration cycle uses Freon-12 as condenser temperature of 40°C, the enthalpy
working fluid. The temperature of the of the refrigerant, Freon-12, at the end of
refrigerant in the evaporator is - 1 o° C. The compression is 220 kJ/kg. Show the �ycle
condensing temperature is 40° C. the cooling diagram on T-s plane. Calculate (a) the COP
load is 150 W and the volumetric efficiency of the cycle (b) the refrigerating capacity
of the compressor is 80%. The speed of the and the compressor power assuming a
compressor is 720 rpm. Calculating the mass refrigerant flow rate of 1 kg/min. You may
flow rate of the refrigerant and the extract of the Freon - 12 property table
displacement volume of the compressor. given below. (GATE-ME-97)
(GATE-ME-95)
Properties of Freon- 12
TCUC) P(MPa) hr (kJ/kg) hg(kJ/kg)
-10 0.2 1 9 1 26.85 1 83 . 1
Enthalpy (kJ/kg) Specific
Satu- 40 0.9607 74.53 203 . 1
Saturated volume
-ration
Temp (m3 /kg)
pressure Liqui
(OC) Saturated

10. An R-717 (ammonia) system operates on the


Vapour
(MPa)
vaoour
-IO

basic vapour compression refrigeration


0.22 26.8 1 83.0 0.08
40 0.96 74.5 203 . 1 0.02

cycle. The evaporator and the condenser


08. In a simple vapour compression cycle, pressures are 0.119 MPa and 1.389 MPa
following are the properties of the respectively. The mass flow rate of
refrigerant R-12 at various points: refrigerant 1s 0.1 kg/s. If the volumetric
(GATE-ME-96) efficiency of the compressor is 84%.
Compressor inlet: h 1 = 1 83.2 kJ/kg ; v 1 = 0.0767 m3 /kg
Determine the compressor displacement
Compressor discharge: h2 = 222.6 kJ/kg ; v2 = 0.01 64 m3 /kg
Condenser exit: h3 = 84.9 kJ/kg ; v3 = 0.00083 m /kg rate. If the COP of the cycle is 2, determine
the power input to the compressor.
3

The piston displacement volume for the


compressor is 1.5 litres per stroke and its Saturation properties of R-717 (ammonia)
volumetric efficiency is 80%. The speed of
the compressor is 1600 rpm.
Specific Enthalpy kJ/kg
Temp. Pressure

Find (a) power rating of the compressor


Volume Liquid Vapour
•c MPa
m3/kg

(kW), (b) refrigerating effect (kW).


-30 0. 1 1 9 0.9638 63.9 1 423.6
36 1 .389 0.0930 37 1 .4 1 488.6

09. In a standard vapour compression


refrigeration cycle, operating between an (GATE-ME-99)
evaporator temperature of 10°C and the

!li•ii@jjjqg.mjRflnlM\dijj.yderabad I Delhi I Bhopal I Pune I Bhubaneswar! Lucknow ! Patna I Bengaluru I Chennai I Vrjayawada IVizag I Tuupati I Kukatpally I Kolkata I
,:.:.�
. .t\CE . . ���========:::::,,���:===========�����yn
Thermod anncs
��·�
.. ..t'=.l!:Fii�,gm
�W'l:!'ffll:•!:Pl
��,Nic:atiooa
= 55 8

1 1 . In a 5 kW cooling capacity refrigeration


system operating on a simple vapour Saturation
compression cycle, the refrigerant enters the
Temp Pressure
Enthalpy (kJ/kg)
evaporator with an enthalpy of 75 kJ/kg and
(kPa)
Liquid Vapour
(OC)

leaves with an enthalpy of 1 83 kJ/kg. The


enthalpy of the refrigerant after compression
-25 1 23.7 1 3.3 1 76.5
35 850.0 69.6 201 .5
is 2 1 0 kJ/kg. show the cycle on T-S or P-h Enthalpy of superheated refrigerant

diagram. Calculate the following: at 850 kPa and 65 ° C = 225.5 kJ/kg.

(a) COP
(b) Power input to compressor and (a) Calculate the Coefficient of performance
(c) Rate of heat transfer at the condenser. (COP) of this system
(GATE-ME-00) (b) If the capacity of the plant is 5 kW, calculate
mass flow rate of refrigerant
12. An ice making plant using refrigerant R-1 2
1s having an evaporator saturation
temperature of -25 ° C and the condenser
saturation temperature of 35 ° C. The vapour
is leaving the compressor at 65 °C. The
following table shows the properties of the
refrigerant. (GATE-ME-02)

llJ•li@jjj44ijjj$RftijiM\jj.jj� yderabad I Delhi I Bhopal I Pune I Bhubaneswar I Lucknow I Patna I Benga]wu I Chennai I Vijayawada J Vmg I Tlfllpati I Kukatpally I Kolkatl
SOLUTIONS
h 3 =h 4 =116kJ / kg
One Mark Solutions h -h4
(COP) = I

01. Ans: (c) 02. Ans: (b)


h2 - hi

232 -116 116


= = = 2.27
283-232 51
03. Ans: (b)
Sol: A throttling process dP (-ve) hence dT 08. Ans: (c)
(+ve) Sol: R- 744 � CO2
R - 718 � H20
04. Ans: (a) 05. Ans: (a)
R290 = �m-l)(n+l) p
m -1= 2, m= 3
06. Ans: (d)

�LL
n+1 = 9, n = 8 H2
Sol: R 1 3 4a
P = 0 flourine
p = 4 � Fluorine n + p + q = 2m + 2
n + 1 = 3; n = 2 � Hydrogen 8 + 0 + q = 2x3 + 2
m-1= 1; m = 2 � Carbon q = O � Ch
Formula is Cm Hn F p C/q Formula CmHnF pClq � C3Hs
C2 H2 F4
R502 � Azeotrope which is a mixture of two

07. Ans: (a)


CFC refrigerants in a definite proportion and

Sol:
exhibits entirely different properties from
2 that of the parent.
T
Two Marks Solutions

s
01.
Sol: Cannot; higher
kJ kJ

02. Ans: (d)


h i = 232 - ; h 2 = 283-
kg kg

\( } I tl;..";l!H t l ! I H.: P1ililh ,i\JUll', �yderabad l Delhi J Bhopal j Pune l Bhubaneswar j Lucknow j Patna j BengaluruJ Oiennai l Vuayawada J Vu.ag j TlfUpati I Kukatpally l Kolkata I
ACE
":�
= GJ:•�=======�:�::�:==============��:�yn
Thermod � =·c:s
��- :'� 5 60 arm
.Iii.
� · :enng
F.nginra: �®
N�� =

03. Ans: (c) Net Refrigerating effect


Sol: p kJ
= m r ( kg/sec) x(h 1 - 14) -
kg

= 0 .025 x (164-80) = 2.1 kW


164-80
2.33
200-164

07. Ans: (a)


V
V1 - V3

V3 100 = NRE ( kw)


- 1 Sol: m r ( kg / sec Xh , -h 2 )
kJ
C= = kg
V, - V3 1000-100 9
-

m r (kg / sec X176 - 65)


kJ
= 2 x3 .517( kW)
kg
04. A-2, B-4, C-6, D-1
m r = 0.0634 kJ/kg
Sol: Shoc k occurs only in diverging portion of
(Volume )act = 0.0634 x 0.1 089
nozzle

= 6.904 x l 0 -3 �
3

05. Ans: (a) sec

08 . Ans: (d)
06. Ans: (b)
Sol: 09. Ans: (a)
T
Sol: For irrever sib le process if frictional heat
gain equals to heat loss from turbine casing
40°c

-20° C then entropy remain constant. Ex: Turbine


expansion.
s

s2 = s , = sn + x(sg 1- sn) 10. Ans: (a)


0.67 = 0.07 + x(0.7366- 0.07) 11. Ans: (c)
0.67-0.07 0 .6 Sol:
X = ----- = 0.9
T
=
0 .7366-0.07 0.6666
2

h 1 = hn + x(h g 1-hn)
= 20 + 0.9(180-20)
= 20 + 144 = 164 kJ/kg
h3 = 14 = 80 kJ/kg s

!M11i@jjj40ijjjjRfln!Mjj.jj..)Hydcrabadl Delhi I Bhopal I Puncl Bhubancswarl U1Cknowl Patna! Bcngaluru j Chennai I Vtjayawada l V17.3g I Tuupati I K.ukatpallyl KolkataI
AJCE
'!.�-�
:&pw,qJ'ubliomN :561 : Refrigeration
J . � ================================
" \I • • • •

ht = 237 kJ/kg ' mr = 0.2 kg/ sec 13. Ans: (b)


St = 0.95 kJ/kgK Sol: Two constant pressures and two reversible
h3 = 114 = 95.5 kJ/kgK adiabatic in reverse direction
St = S2 = 0.95 kJ/kgK , h2 = 276.45 kJ/kg
NRE (kW) = mr (kg/ sec) (ht - 114) kJ/kg 14. Ans: 5.5
= 0.2 (237 - 95.5) = 28.3 kW Sol: T
Wc (kW) = mr (kg/ sec) (h2 -ht) kJ/kg
= 0.2 (276.45 - 237) = 7.89 kW

12. Ans: 433.3 kJ/kg


Sol: T
h , - h 4 402-237 165 _
2s 0c COP = = = =5 5
h 2 -h, 432-402 30

15. Ans: 8.9315 (range 8.90 to 8.95)


Sol: h 2 = 272.05kJ / kg
Cpv = 0.98 kJ/kg K h 3 = h 4 = 93.42kJ / kg
St = 1.7841 kJ/kg K
h, = 236.04kJ / kg
h2, = 413.02 kJ/kg K
mr = 0.05 kg/ sec
S2• = 1.7183 kJ/kg K
T2 T
s, = S 2 = 1.7841 = S 2, + c pv .en [ ]
T2,

1.7841 = 1.7183 + 0.98 f n[ T2 ]


298
T2 1.7841-1.7183 =
Rn [ ] 0_06714 s
298 0.98
T2 = 298 e0·067 14 = 1.06945 x 298 = 318.7 K kg kJ
Heat Rejected = m, ( )(h 2 -h 3 )
h2 = h2•+ Cpv (T2 -Tsat) sec kg
= 413.02 + 0.98 (318.7 - 298) = 0.05 (272.05 - 93.42)
h2 = 433.3 kJ/kg = 8.93 1 5 kW

\( I I 11�11h l 1111:.., P11hlll ,1t1,111... yderabad I Delhi IBhopal IPunc IBhubancswar I Lucknow I Patna IBcngaluru I Chennai IVtjayawada IVizag IT1n1pati I Kukatpally I Kolkata
: 562 : Thermodynamics

16. Ans: 1103.44 (range 1095 to 1130) Alternate solution:


Sol:
T T

s
s

h -h 3 1054
h 2-h3 = 1054 s2 - s3 = 2 = = 3 .4 kJ / kgK
Tsat 310
h 2 - Cpl (T3-To) = 1054
S2-S3 = SJ- S3•
h 2-4.8 (310-273) = 1054
Take reference point a s 'O'
h 2 = 1231.6 kJ/kg
T3 310
Reference temperature = 273 = T s 3-s 0 = c P .en ( J = 4.8 .en ( ) = 0.8442 kJ / kgK
T0 260
S3 - So = S3•- So
At reference temperature entropy is zero. s1- s0= 3.4 + 0.8442 = 4.2442 kJ/kgK
s 3 =c ,, {i) =4.8£n G��)=0.61kJ / kgK h1-ho = T sat(S 1 - So)
= 260 x4.2442 = 1103.492 kJ/kg
T2( s2- S3) = h 2-h3
310 (s2 - 0.61) = 1054
Five Marks Solutions
1054
S2 = 0.61 + -- = 4.01 kJ/kgK
310
01.
S2 = SJ = 4.01 kJ/kgK
Sol:
s, =c,, e{;) T

3s0c
260
= 4.8.en ( -) =-0.23 kJ I kgK
273 --1 '
h1 -ho = To (s 1- so)
4'

= 260 (4.01- (-0.23)) = 1103.44 kJ/kg s

Net refrigerating effect ( NRE)


= 10 x 3.517 = 35.17 kW

!ltilQjj§jjji41hQRftni@mi0+1Ydcrabad j Delhi I Bhopal I Punc l Bhubancswarl Lucknow I Patna! Bcngaluru l Chcnnai I V\iayawada l V17.ag j Tuupati I Kukatpa))yj Kolkata I
: 563 : Refrigeratio n

h : = 174.96 kJ/kg ; kg
m, ( ) (195 .7 - 64.6) = 52.755
sec
h � = h3 = 14 = 69.74 kJ/kg ;
52.755
h 1 = 190.88 kJ/kg m, = -- = 0.4024 kg / sec
131.1
Work of compression
Between 35 °C and 5 °C

mr (kg/sec) (h 1 - 14) kJ/kg = NRE (kW)

m r x(190.88 - 69.74) = 35.17


35 17 1. l 3 96
· = 0.2903 kg/sec
t t

121.14 x 219 x 0.082 [( · ) /.�; -11


1.13-1 2.19
=

Between 35 °C and -30° C


kJ 28.93 kJ/kg.
NRE (kW) = m r ( kg
sec
) - X (hi h� )
kg
=

Work of compression (kW)


0.2903 (174.96 - 69.74)
= m , ( kg ) we ( kJ J
=

= 30.54 kW sec kg
0.4024 x28.93 = 11.64 kW
02.
=

NRE(kW) = 52.755 = _
Sol: COP = 4 53
Wc (kW) 11.64
T

03.
Sol: Clearance ratio(C) = 0.04
Swept volume (Vswept) = 26 litres / sec
Mass flow rate of refrigerant
s

mr = 0 .46 kg/sec
P 1 = P 4 = 2 .19 bar P 2 = P 3 = 9. 6 bar
Specific volume at suction (v1)
h 3 = h 4 = 64.6 kJ h t 195.7 kJ/kg.K
= = 3 7 litres / kg,
v t = 0.082 m 3 /kg n = 1.13 Specific volume at discharge (v2)
Net refrigerating effect = NRE = 12 litres / kg
= 1 5 x 3 . 5 1 7 = 52.755 kW Volume flow rate at suction
= 0.46 kg/sec x 37 litres/kg
kg kJ
m, ( ) (h t - h 4 ) = NRE (kW) 17.02 litres/sec
sec kg
=

!ltllib§ihiiih/iQnijiMihfjj� yderabad j Delhi jBhopal jPune jBhubaneswar j Lucknow jPatna j BengalurujChennai jVijayawada JVizag j Tirupati J �I Kolkata
Volumetric efficiency = 1lvolumetric m r x(407 .1 - 2 63.3) = 52 .7550
_ Volume flow rate at suction 52.755 = kg
mr = 0367
vswept 143.8 sec
17 02 Volume flow rate (m 3 / sec)
= · = 0.6546
26 kg m3
= mr ( ) v1 ( )
Clearanc e volumetric efficiency sec kg
N
tr D 2 Ln x -
=- x ny
4 60 '
I o
1


0.367 X 0.0404 = X X 0.72

Theoretical strok e volume flow rate


0.367 X 0.0404
= (x) =
0.72
= 0.0205 m /sec
3

:: = ( ::Y = :�

= 1+O.04-0. 04 ( ��) 05.


Sol:
(TJvol)clearance

= 0.9167 or 91.67 %

04 .
Sol:
�2
-s
/
4Lf Fig : h- s diagram
h
06.
11Vol = 0.72 Sol:
N RE � 15 x 3.517 = 52 .755 kW
T
h 1 = 407 .1 kJ/kg
h 3 = h i = 263.3 kJ/kg
V1 = 0.0404 m 3/kg
m r (kg / sec) (h1 -h i) kJ/kg
= Net Refrigerating Effect (kW) s

lli11ijj§jjj40@MRflnj@@jj.jHyderabad I Delhi I Bhopal I Pune J Bhubaneswarl Lucknow I Patna! Bengaluru I Chennai J Vijayawada I Vmg I Tirupati I Kukatpally J Kolkata I
'!.�-���PuNicmona : 565 : Refrigeration
" A ,CE · ·
"' · �
� � =================================
D = 0.05 m ; L = 0.04 m ; mr (kg/sec)x (h 1 h.i) kJ/kg = CL (kW)
kJ mr ( 1 83 - 74.5) = 0. 1 5
-

v, O . l l m / kg ; h1 - h.i = 93.7 -
kg
3

O. l 5
=

kg
mr = = 1 .3824 1 0-3
N = 1 460 rpm ; C = 0.05 ; 1 08.5 sec

r
X

P1 = 0. 1 5 1 MPa ; P2 = 1 .2 1 9 M Pa ; Volume flow rate (m3/sec)


n = 1 . 18
m, (kg / sec) x v 1 ( :; J

TJv,1 - I + C - C (�
n N
= - D i Ln x - x n' I Vol
4 60
1 2 1 9 ½. is
= 1 + 0.05- o . o 5 ( ) 720
0. 1 5 1 1 .3824 1 0-3 0.08 = X X X 0.8
60
X X

= 0.7564
Displacement Volume(x) = 1 . 1 52 x 1 0-5 m3 .
kg N
mr (-) v 1 ( -J = - D 2 Ln x - x n· i voi
m 1t
3

sec kg 4 60 08.
1 460
mr x 0. 1 1 = tr (0.05)2 (0.04) x 1 x x 0.7564 Sol:
4 60 T
mr = 0.0 1 3 1 4 kg I sec
Refrigerating Load
= mr (kg I sec) x (h1 - h.i) kJ/kg
= 0.0 1 3 1 4 x 93.7 = 1 .23 1 kW s
= 0.00083 m 3
kg
07. h3 = 84.9 kJ/kg ; h2 = 222.6 kJ/kg ;
V3 /

Sol: h1 = 1 83.2 kJ/kg


T v2 = 0.0 1 64 m3/kg ; v1 = 0.0767 m3/kg

D 2 Ln = 1 .5 x 1 0-
m3
4 Stroke
3
° 7t
40 c

-10 C °
llVol = 0.8 ; N = 1 600 rpm

s Volume flow rate (m3 / sec)


CL = 0. 1 5 kW ; h 1 = 183 kJ/kg mr (kg/sec) v1(m3/kg)
h3 = 74.5 kJ/kg N
tr D z Ln x -
4 60
x 'I
n Vol
= 0.8 ; N = 720 rpm
=-
llVol

!M•1i!@jjji4ibi4RflniM\jijjj. I I I I I I I I I
yderabad Delhi Bhopal Pune Bhubaneswar Lucknow Patna Bengaluru Cheruiai Vtjayawada !Vizag I Tirupati I Kukatpally! Kolkata
mr (kg I sec) x 0.0767 m3 I kg 108.57
= = 2.942
1600 . 36.9
= 1.5 10-3 X X 0.8
60 Capacity of plant
X

mr = 0.4172 kg I sec = mr (kg I sec) x(h1 - hi) kJ/kg


1
= - x 108.57 = 1.8095 kW
Power Rating of Compressor 60
= mr (kg I sec) (h2 - h 1 ) kJ/kg Compressor power
= 0.4172 (222.6- 183.2)
- m, (kg I sec) xWc ( �J
= 16.44 kW
X

Net Refrigerating effect 1


= - x 36.9 = 0.615 kW
= mr (kg I sec) x(h 1 - hi) kJ/kg 60
= 0.4172 (183.2 - 84.9)
= 41.01 kW 10.
Sol: 2
09. T
Sol:
2
T

mr = 0.1 kg I sec ; = 0.84 ;


m3
T)Vol
s

h 1 = 183.1 kJ/kg ; h2 = 220 kJ/kg ; kg


VJ = 0.9638 - ;
h3 = hi = 74. 53 kJ/kg h1 = 1423.6 kJ/kg ;
Net refrigerating effect ( NRE) h3 = hi = 371.4 kJ/kg
= (h1 - hi) kJ/kg Volumetric flow rate ( m 3 /sec )
= (183.1 - 74.53) = 108.57 kJ/kg
kg m3
WC = (h2 - h1) kJ/kg mr ( ) x v1 ( J
sec kg
= (220- 183.1) kJ/kg = 36.9 kJ/kg
1
COP =
NRE(kl I kg) =-
7t D 2 LNn x - x TJ
4 60
Wc (kl I kg)
Vol

0.1 X 0.9638 = X X 0.84

!MIIQ@jjjH@MQnbiM\Uijj.jHyderabad I Delhi I Bhopal I Punc I Bhubaneswarl Lucknow I Patna I Bengaluru I Chermai I Vijayawada I Vmg I Tuupati I Kukatpa1Jy I Kolkata I
: 567: Refrigeration

O. l x 0.9638 = m 12.
x= 0 . 1 1474
3

0 .84 sec Sol:


T
NRE (kW) = m r (h 1 - 14) 2

= 0 . 1 ( 1423 .6 - 37 1.4) = 105 .22


NRE(k w) -25° C
We (kW) =
2
= 1 05 ·22 = 52.6 1 kW s
2
h1 = 176 .5 kJ/kg ; h2 = 225.5 kJ/kg;

1 1. h3 = 14 = 69.6 kJ/kg ;
Sol: N RE h1 - h4
COP = =
WC h2 - hi
2
176 .5-69.6 106 .9
T = = = 2. 18 1
225 .5- 176 .5 49
�2
NRE(kW) = m r(kg/sec)(h 1 -14) kJ/kg
/44
5 = ffir X ( 176.5 - 6 9.6)
ffir = 0.04677 kg/sec
s h

Power (kW) = mr(kg/sec)(h2-h1) kJ/kg


NRE = 5 kW
= 0 .04677 x 49 = 2.291 kW
m(kg / sec) (h 1 - 14) kJ / kg = NRE
m( 183 - 75) = 5

m = � = 0 .0463 kg / sec
1 08
We ( kW) = m (h2 - h1)
= 0.0463 ( 2 1 0 - 183) = 1 .25 kW

COP = NRE = -5- = 4


We 1 .25
Heat Rejected to Conden ser
= NRE + We = 5 + 1 .25 = 6.25 kW

\( l l 1 1l!,l [lt t l l l 1 ..., P1ild11 ,!IJ1 1 J I ', yderabad I Delhi I Bhopal I Pune I Bhubaneswarl Lucknow I Patna I Benga)uru I Chennai I Vgayawada I Vizag I Tirupati I KubtpaUy I Kolkata
C13
Thermodynamic relations &
Reciprocating compressors
========================:::::::!J
02. In a 3-stage air compressor , the inlet
pressure is P 1 , discharge i s P4 and the
Two Marks Questions
intermediate pre ssure are P2 and P3 and P3
(P2 < P3 ). The total pressure of the
01. For water at 25 ° C, dPsfdTs = 0.189 k Pa/K compressor is 10 and the pressure ratio s of
(P s is the saturation pre ssure in kPa and Ts the stages are equal. If P 1 = 100 kPa , the
is the saturation temperature in K) and the value of the pre ssure P3 (in kPa) is __
specific volume of dry saturated vapour i s (GATE- 16 - SET - 3)
43.38 m 3/k:g. Assume that the specific
volume of liquid i s negligible in comparison
with that of vapour. Using the Clau siu s­
Clapeyron equation , an e stimate of the
enthalpy of evaporation of water at 25 ° C (in
kJ/k:g) is _
(GATE- 16- SET- 1)

�yderabad J Delhi JBhopa! JPuneJBhubane-.arJ LucknowJ Patna J Bcnga!uru J ChennaiJVgayawadaJVmg JTirupati I KukalpallyJ Kolkala I
SOLUTIONS
02. Ans: 464.151 (range 460 to 470)
Two Marks Solutions
Sol: ( r, lon = (� )' = (10)¼ = 2. 1544
01. Ans: 2443.25 kJ/kg (range 2400 to 2500)
dPs kPa
Sol: = 0.189
dTS K
P2 = 2.1544 x 100 = 215.44
Tsat = 273 + 25 = 298 K
= 43.38 m3/kg p3 = 2.1544
P2
Vg

vr = 0
P3 = 2.1544 x 215.44 = 464.15 kPa
Vfg = Vg - Vf
m3
= 43.38 - 0 = 43.38-
kg
dP
dT Tsai X v fg
h fg h fg
0.189 = =- ----''---
Tsai x vfg 298 x 43.38
hrg = 2443.25 kJ/kg

!1J•l4jj§jjjl4.jjj4Aflfti&ij@j.ydcrabad l Delhi l Bhopal l Punc


j Bhubaneswarl Lucknow l Patna l Bcngaluru j Chcnnai l V\iayawada j Vazag ITuupari I Kukatpal)yl Kolkala I
Ct4 IC Engines
(b) the air standard efficiency of diesel
One Mark Questions cycle is higher than the Otto cycle , at a
fixed compression ratio
01. With increasing temperature of intake air , IC (c) the compression ratio of a diesel engine
engine efficiency (GATE-ME-98) is higher than that of an SI engine
(a) decreases (d) self ignition temperature of diesel 1s
(b) increases higher than that of gasoline
(c) remains same
(d) depends on other factors 04. At the time of starting , idling andlow speed
operation , the carburetor supplies a mixture
02. For a spark ignition engine , the equivalence which can be termed as (GATE-ME-04)
ratio(<I>) of mixture entering the combustion (a)lean
chamber has values. (GATE-ME-03) (b) slightlyleaner than stoichiometric
(a) < 1 for idling and
<I> <I> > 1 for peak (c) stoichiometric
power conditions. (d) rich
(b) > 1 for both idling and peak power
<I>

conditions.
Two Marks Questions
(c) <I> > 1 for idling and <I> < 1 for peak
power conditions.
(d) <I> < 1 for both idling and peak power 01. The power output from a spark ignition
conditions. engine is varied by : (GATE-ME-90)
(a) Changing the ignition timing.
03. A diesel engine is usually more efficient (b) Regulating the amount of air-fuel
than a spark ignition engine because inducted.
(GATE-ME-03) (c) Regulating the amount of air-fuel
(a) diesel being a heavier hydrocarbon , mixture.
releases more heat per kg than gasoline (d) Regulating the amount of fuel.

!ltl•i!i§jji§4ijj,i@fliM\jjii!� yderabad I Delhi I Bhopal I Pune I Bhubaneswar I Lucknow! Patna I Bengaluru I Chennai I Vijayawada I V17.ag ! Tirupati I � I Kolkala
�-t Thermodynamics
ACE : 571 :
':. ��� Pnblic:atiooa

02. For determining the ignition quality of 06. Brake thermal efficiency of the three basic
compression ignition engine fuels, the types of reciprocating engines commonly
reference fuels used are (GATE-ME-91) used in road vehicles are given in the
(a) !so-octane and n- heptane increasing order as: (GATE-ME-92)
(b) Cetane and a-methylnapthalene (a) 2 Stroke SI engine, 4 stroke SI engine,
(c) Hexadecane and n-heptane 4 stroke CI engine
(d) cetane and iso-octane (b) 2 Stroke SI engine, 4 stroke CI engine,
4 stroke SI engine
03. If air fuel ratio of the mixture m petrol (c) 4 Stroke SI engine, 2 stroke SI engine,
engine is more than 1 5 : 1 (GATE-ME-91) 4 stroke CI engine
(a) NOx is reduced (b) CO2 is reduced (d) 4 Stroke CI engine, 4 stroke SI engine,
(c) HC is reduced (d) CO reduced 2 stroke SI engine

04. BHP of a diesel engine can be increased by 07. Knocking tendency in a SI engine reduces
(GATE-ME-91) with increasing (GATE-ME-93)
(a) Increasing the pressure of intake air (a) Compression ratio
(b) Increasing the temperature of intake air (b) Wall temperature
(c) Increasing the density of intake air (c) Supercharging (d) Engine speed
(d) Decreasing the density of intake air
08. An IC engine has a bore and stroke of 2
05 . Alcohols are unsuitable at diesel engine units each. The area to calculate heat loss
fuels because: (GATE-ME-92) can be taken as (GATE-ME-98)

(a) The cetane number of alcohol fuels is (a) 4n (b) 5n (c) 6n (d) 4

very low which prevents their ignition


by compression 09. An a1r breathing aircraft is flying at an

(b) The cetane number of alcohol fuels is altitude where the air density is half the

very high which prevents their ignition value at. ground level. With reference to the
by compression ground level, the a1r - fuel ratio at this
(c) The octane number of alcohol fuels is altitude will be (GATE-ME-98)
very low which prevents their ignition
by Compression
( a) 1/2 ( )
b ff
(d) None of the above. (c) 2 (d) 4

!M11Mh§ml4@4RflbfM\hi\i� yderabad I Delhi I Bhopal I Pune I Bhubaneswar I Lucknow! Patna I Bengalwu I Chennai I Vijayawada I Vmg 1-Tirupatt I Kukatpally I Kolkata
Ni�a�ti.:011:18�========�:5�7�2�: Thermo dynaIIll·cs
'i·=li'.!n�..,,�·-:·�w�Pu
ACE
��-� :l'!l'lll �� ===========�;,::;:;�:�:;;��
10. The silencer of an internal combustion 13. Air contains 79% N2 and 21% 02 on a molar
.,j

engme (GATE-ME-99) basis. Methane (CH4) is burned with 50%


(a) reduces noise excess air than required stoichiometrically.
(b) decreases Brake Specific Fuel Assuming complete combustion of methane,
C onsumption (BSFC) the molar percentage of N2 in the products is
(c) increases BSFC
(d) has no effect on its efficiency (GATE - 17- SET - 1)

11. An automo bile engine operates at a fuel air


ratio of 0.05, volumetric efficiency of 90% Five Marks Questions
and indicated thermal efficiency of 30%.
Given that the calorific value of the fuel is 01. In a test on a four cylinder spark ignition
45 MJ/kg and the density of air at intake is 1 engine the following power and fuel
kg/m3 , the indicated mean effective pressure consumption measurements were made
for the engine is (GATE-ME-03) (GATE-ME-87)
(a) 6.075 bar (b) 6.75 bar with all cylinders firing 30.00 kW
(c) 67.5 bar (d) 243 bar with cylinder 1 only not firing 20.75 kW
with cylinder 2 only not firing 20.50 kW
12. During a Morse test on a 4 cylinder engine, with cylinder 3 only not firing 20.50 kW
the following measurements of brake p ower with cylinder 4 only not firing 20.75 kW
were taken at constant speed
The time taken to consume 250ml of fuel is
(GATE-ME-04)
85seconds.Specific gravity of fuel = 0.85.
All cylinders firing 3037 kW
Calorific value of fuel 1s 40
Number 1 cylinder not firing 2102 kW Ml/kg.Determine the mechanical efficiency
Number 2 cylinder not firing 2102 kW and brake thermal efficiency of the engine
Number 3 cylinder not firing 2100 kW
Number 4 cylinder not firing 2098 kW 02. An engine is used on a job requiring a shaft

efficiency o f the engine is 80% and it uses


output of 1 00 kW. The mechanical

(a) 91.53% (b) 85.07%


The mechanical efficiency of the engine is

30 kg of fuel per hour under conditions of


(c) 81.07% (d) 61.22%
operation. Calculate the indicated thermal
efficiency. If an improvement m engme
. .

!ID1i@jjji4.jji4Aflni@iu1jj� yderabad I Delhi I Bhopal I Pune I Bhubaneswar I Lucknow! Patna I Bengaluru I Chennai I Vijayawada j Vmg I Tirupati I Kukat:pally I Kolkata
: 573 : Thermodyn ami cs

design reduces the frictional losse s by 5 kW, 05. A four cylinder, four stroke, spark ignition
calculate the amount of fuel saved per hour. engine develop s a maximum brake torque of
Assume that the indicated thermal efficiency 160 N.m at 3000 rpm. Calculate the engine
remains the same. Calorific value of fuel i s displacement, bore and stroke. The brake
42000 kJ/kg. (GATE-ME-90) mean effective pressure at the maximum
engine torque point is 960 k Pa. Assume bore
03. A four stro ke, four cylinder spark ignition i s equal to stroke. (GATE-ME-'92)
engine having bore 7 cm and stroke 9 cm
develops 20 kW at 3000 rpm. If the 06. A six cylinder four stroke CI engine
clearance volume in each cylinder is 50 cm 3 , developing a power output of 270 kW at
the brake thermal efficiency i s 50 % of air 1000 rpm has a fuel consumption of 0.25
standard efficiency and the calorific value of kg/kWh. The injection takes place over 20 °
the fuel is 43 MJ/kg, find out torque, brake cran k angle with pressure across the injector
thermal efficiency and fuel con sumption. orifice of 100 MPa. Fin d.
(GATE-ME-91) (i) The rate of fuel injection in mg/s
through each hole of a four hole
04. A single cylinder four stroke Diesel engine injector fitted in the engine cylinders,
having bore 18 cm and stroke 32 cm and
develops torque 390 Nm and indicated mean (ii) Thermal efficiency of the engine.
effective pressure (imep) 700 kPa at 280 (GATE-ME-93)
rp�. The following observations were made
during experiment; fuel consumption 3 07. A single cylinder, 4 -stroke diesel engine
kg/hr, cooling water flow 4 kg/min, increase running at 1800 rpm ha s a bore of 85 mm
in cooling water temperature 35 ° C, air fuel and a stroke of 110 mm. It takes 0.56 kg of
ratio 22, room temperature 20 ° C and air per minute and develops a brake power
barometric pressure 1 bar. If the calorific output of 6 kW while the air fuel ratio i s
value of the fuel i s 42 MJ/kg, find out IHP, 2 0 :1. The calorific value of the fuel u sed i s
indicated thermal efficiency ( llth), 42550 kJ/kg and the ambient air density is
mechanical efficiency ( llm), volumetric 1.18 kg/m 3 • Calculate
efficiency ( ll voI) and heat lost in cooling (a) The volumetric efficiency, and
water. (GATE-ME-91) (b) Bra ke specific fuel consumption.
(GATE-ME-95)

!M•lid§i!iiiih4Aflb1Mi!iMiiiyderahad I Delhi I Bhopal I Pune I Bhubaneswar I Lucknow I Patna IBenga)uru IChennai IVtjayawada IVizag ITuupari I KukatpallyI Kolkata I
08. The power output of an LC. Engine 1s are 120 mm in diameter. The mechanic
measured by a rope brake dynamometer. bores the cylinder and replaces the piston
The diameter of the brake pulley is 700 mm with new pistons that are 2 mm larger in
and the rope diameter is 25 mm. The load on diameter than the originals.
the tight side of the rope is 50 kg mass and (a) Keeping all other factors same, what
the spring balance reads 50 N. The engine will be the percentage change in power
running at 900 rev/min consumes the fuel, of output?
calorific value 44000 kJ/kg, at a rate of 4 (b) By what percentage will the engine
kg/h. Assume g = 9.8 m/s2 . Calculate efficiency change?
(a) Brake specific fuel consumption. (GATE-ME-01)
(b) Brake thermal efficiency.
(GATE-ME-97) 11. The Willan's line measured for a four­
stroke, four-cylinder is expressed as:
09. A Diesel engine develop a Brake power of FC = 0.15 + 0.03 x B.P., where FC is the
4.5 kW. Its indicated thermal efficiency is rate of fuel consumption in gm/s and B.P. is
30% and the, mechanical efficiency is 85%. the brake power in kW. The bore of each
Take the calorific value of the fuel as 40000 cylinder is 75 mm and stroke is 90 mm and
kJ/kg and calculate the speed is 3000 rpm.
(a) The fuel consumption in kg/h and Calculate indicated power, mechanical
(b) The indicated specific fuel consumption. efficiency and indicated mean effective
(GATE-ME-00) pressure, when the engine is developing a
brake power of 20 kW.
10. A mechanic has an engine from a 1970 (GATE-ME-02)
model car which works on the basis of Otto
cycle. The engine displaces 1.8 liters, has a
compression ration of 10.2: 1 and has six
cylinders. The pistons in the original engine

!lfll4jj§jjji4m/4AflijiMfoijj� yderabad j Delhij Bhopal j Pune j Bhuban.!:swar i lncknow l Patna l Bengaluru j Chennai j Vuayawada j V� j TllUpati I K�l .Kolkata
SOLUTIONS
08. Ans: (c)

Sol: A = 7t D L + 2 x tr D
One Mark Solutions 2

4
01. Ans: (a) tr X 2 2 = 47t + 2 7t = 61t
= 7t X 2 X 2 + 2 X -
4
02. Ans: (b)
Sol: 09. Ans: (b)

�> l ( AFR ti
S�: -- Ai -
� - /E J½
(AFR ) sL - PsL - 2
Idling Cruising Peak
Power (AF s,,
(AFR),1 = !J = O.?O?( AFR) s,,

03. Ans:(c) 10. Ans: : (a)


Sol: rk > 12 for Diesel engines
rk � 10 for SI engines 11. Ans: (a)
lkg fuel
Sol: FAR = 0.05 =
04. Ans: (d) 20kg air

Two Marks Solutions


20 ·
0.9 = , = 22.22 kgs
m th
ffith

01. Ans: (c) 02. Ans: (b)


Pa = 1 kg / m
3

Stroke Volume
03. Ans: (a) 04. Ans: (a) and (c)
m 22.22
= th = = 22.22 m3.
1
05. Ans: (d) 06. Ans: (a) Pa
W = 1J x mr (CV)
07. Ans: (d) = 0.3 X 1 X 45000 = 13,500 kJ

\( 1 l 111!111t t t111J.!. Pt1hl1< ,11J1111, �yderabad I Delhi I Bhopal I Punc I Bbubancswar I Lucknow I Patna I Bcngaluru I Chennai I Vtjayaw.ula I Vmg I T1IUpari I Kukatpally I Kolkata I
: 576 : Thermodynamics

Work 13500 0: 2a + b + 2c = 6
Mep = =
Stroke volume 22.22 2xl + 2 + 2c = 6
= 607.56 kPa = 6.075 bar 2c = 2
c =l
12. Ans: (c) N: 2d = 6x3.76
Sol: I = 4B - (B1 + B2 + B3 + B4) d = 3x3.76 = 11.28
(I = Indicated power) CO2 = lmole
= 4x3037-(2102+2102+2100+2098) H2 = 2moles
( B = Brake power) 02 = lmole
= 3746 kW N2 = l1.28moles
(B1 , B2 , B3 , B4 , Brake power with respective
Percentage nitrogen in products
cylinders cutoff).
Numberof molesof N2
= x lOO
B 3037 Totalnumber of moles
ll m = - = -- =0.8107 (or) 81.07%
I 3746
11.28
=( )XlQO
1 + 2 + 1 + 11.28
13. Ans: 73.82 %
Sol: Cfit+a(02+3. 76N2)�bC02+CH20+dN2 = ll.28 X 100 = 73.82%
15.28
C: b = 1
H: 2C = 4
C=2 Five Marks Solutions
0: 2a = 2b+c
01.
a = b+c/2
Sol: Brake power with all cylinders firing
=1 + �=2 = B = 30 kW
Brake power with first cylinder cut off (B1 )
N2 = 7.52a = 2d
= 20.75 kW
d = 3.76a = 3.76x2 = 7.52
Brake power with second cylinder cut off
50% excess air is used
(Bn) = 20.50 kW
Clit+2x 1.5(02+3.76N2)�aC02+bH20
Brake power with third cylinder cut off (Brn)
+C02+dN2
= 20.50 kW
C: a = 1
Brake power with fourth cylinder cut off
H: 2b = 4
(BIV) = 20.75 kW
b=2
\( I l r 1 1.!, 111t l r 1111.!, P11hlh .u1, m" yderabad I Delhi I Bhopal I Pune I Bhubaneswar I Lucknow I Patna I Bengaluru I Chcnnai I Vtjayawada l V,zag I Trrupati I Kukatpa)Iy I Kolkata
: 577 : Internal Combustion Engin e

Indicated power ( I) 5 kW reduced (Friction Power)


= 4 B- (B1 + Bn + Bm + B1v) 120 x3600
mf (kg / hr) -
= 120 - (20.75 +20.5 +20.5 + 20.75) 0.3571 X 42000
= 37.5 kW = 28.8 kg / hr
Mecha nica l e fficiency = Tt mech
Fuel saved = 30 - 28 . 8 = 1.2 kg / hr
B = 30 kW
= = 0 .8 (or) 80% 03.
I 37.S kW
Brake thermal e fficiency = Tl Br. Th
Sol: 4 stroke, 4 Cylinder, SI engine

Bra kepower (kW) D = 0.07 m ; L = 0.09 m


N = 3000 rpm; B P = 20 kW
= _____..;;;,____;____,;____
m r ( kg I sec) x Calorific value ( kJ / kg)
VC = 50 C.C.; Tl Br. Th = 0 .5 Tla
BP( kW)
= -----'----'-- c.v = 43000 kJ/kg
.v
p x cv( kJ J
t kg
Vs = 7t D2 L = Jr X 7 2 X 9 = 346.360 C.C.
p = 850 kg I m t = 85 sec
3 4 4
Vs Ve
= 346.185 + 50
,

rk = = 7 _93
+
and v = 250 x10 ---6 m 3
VC 50
30
= x l OO 4
850 x 250 x l 0 -3 x l 0 -3 x 40000 = 1-
1 y-l 1 ) o.
Tl a (-J = 1 - (- = 0.5632
85 rk 7.93
= 30% = 0.5 Tla = 0.5 X 0.5632
llar.Th

= 0.2816 = 28.16 %
02.
2:rNT
Sol: BP = 100 kW ; = 0. 8 ; BP (kW) = -- (T � Torque(Nm))
60,000
Tt m

th r = 30 kg I hr , CV= 42000 kJ/kg


BP( kW) ::;:: l OO 20 =
2 x 3 . 14 x 3000 x T
IP ( kW) = = 125 kW
0. 8
60000
T'l mech
T = 63.7 Nm

Tl Br.Th -
_ IP(kW) x 3600
mr (kg / hr) x CV (kl I kg)
T'l i th - _ BP(kW) x 3600
mr (kg / hr) x CV(kJ / kg)

= 125 x 3600 = 0.3571 or 35.719 % m =


20 x3600 = 5.9482 kg / hr
30 x 42000 r 0.2815 x 43000
IP1 = 120 kW ;

!lfl1ijj§jjji4.jj/Q@ft1Mijjjjj+yderabad I Delhi I Bhopal I Pune I Bhubaneswarl Lucknow ! Patna I Benga!W11 I Chennai I Vgayawada j Vizag I Tl11lpati I Kukalpally I Kolkata I
...,.., ........ ACE :. ��e����ann �·c;s
�..a . '= E·l!l&i�•�·.,,..
:- :·!Pn�ilw�.._
•� · =======�:�5�78�:========== Th rmodyn
�:,...�
0 4. m.
AFR =
Sol: Single Cylinder ; 4- Stroke rilr
d = 18 cm ; L= 32 ems m.
22 =
T = 390 Nm ; N = 280 rpm 3 '
Pmi = 700 kPa ; ril r = 3 kg/ hr m. = 66 kg/hr
m . = 4 kg / min ; ( L\T)w = 35 ° C 100 = 1.189 kg Im
p
Pa = =
3

AFR = 22 ; TR = 20 ° C ; RT 0.287 x 293


_ m ( kg / sec)
llvo1 - .
Pbar = 100 k Pa N
P vs x ­
120
CV = 42,000 kJ / kg

66
2 1tNT 3600
BP(kW) = =
60000 280
l.189 x " x (0.18}2 (0.32) x
2 x3.1 4 x 280 x390 4 120
= = l l. 43 kW
60000 0.01833333
= ----
0.0225799
P . x 7t D LNn 2

= 0.8119 or 81.19 %
IP(kW) = , ---'4--
_m_
120

700 x 7t x (0.18} (0.32 )280 x1


2 05.
4 2 _,.,T 2 - x 3000 x 160
Sol: BP = ,1.1.v ,.
=
120 =
60,000 6 0,000
= 13.29 kW
= 50.2 4 kW
BP( kW) 11. 43
Ti m
= = = 0.86 L= D
IP( kW) 13.29
BP(kW )
_ IP(kW) x 3600 Bmep =
flITh . l] - ------­ N
ffir (kg / hr) x CV(kJ / kg) V8 x - x n
120
13.29 X 3600 =
= 37.97% 50.2 4
3 x 42000 = ------
N
-1t x D 2 x L x - xn
Heat lost to cooling water 4 120
= m (kg I m in) X Cpw X ( L\T)w
50.2 4
960 = -----
W

x 4.187 x 35 1t D 2 L x -N
= 4
- x4
4 120
= 586.18 kJ I min

\( ' l , l 11g-111t ( ?1111-!, P11hlH ,IIJ<)I]" Fydcrabad l Dclhi l Bhopal l Pune l Bhubaneswar l Lucknow l Patna l Bcngalwu l Cbennai l Vuayawada l V� I Tuupati I Kukatpal)yj Kolkata I
: 579 : Internal Combustion Engine

50.24 X 1 20 X 4 0.56
6.66666 1 0-4 84.47 %
7Z" X 3000 X 960 X 4 0.66289
X =

D = 8.736 ems ; L = 8.736 cm


mr (kg / hr)
Vs = n x 7t D 2 L = 4 x n- x 8.7362 x 8.736 b) Bsfc =
4 4 BP(kW)
= 2093.47 C.C. 1 ·68
0.28 kg / kW hr
6
= =

06.
. . Bsfc x (BP) x 1 0 6 08.
Sol: Fuel mJected per hole = ------
3600 x 4 Sol: D = 700 mm , d = 25 mm
_ 0.25 x 270 x 1 0 6 mg mg W = mg = 2 x 25 x 9.8 = 2 x 245 N
4687_5
3600 x 4 sec sec 490 N
=

3600
=

S = 50 N ; N = 900 rpm
Bsfc X CV
mf = 4 kg I hr ; CV = 44000 kJ/kg
=
llBr th

(D + 2d)
Torque =
(W - S) ---
2
0.36 or 36 % (490- 50) (700 + 2 X 25)
=
=

1 000 2 x 1 000
07. 0. 1 65 kN-m
Sol: ma 0.56 kg / min
=

m a 0 56 = kg 21tNT = 2n x 900 x 0. 1 65
mr = = · 0.028 kg I min = 1 .68 BP (kW) =
20 20 hr 60 60
1 5.56 kW
a) D = 0.085 m ; L = 0. 1 1 0 m ;
=

N = 1 800 rpm. k
4 stroke Single Cylinder 4 = kg
bsfc = m, ( ¼,) = - - 0.257
1 5.56 kW hr
ma ma
TJVol -
- - -
BP(kW)
m th N 3600 3600
p x - D 2L x -
=
- -----'----
1t

bsfc x CV 0.257 x 44000


=
ll Br.Th
4 2
0.56kg / min = 0.3 1 83 or 3 1 .83 %
1800
l . 1 8 x 7t (0.085) (0. 1 1 o )x
4 2
2

Imlih4hl41ihiNRflnj@mjjj"' yderabad I Delhi I Bhopal I Pune I Bhubaneswar I wcknow I Patna I Bengalwu I Chennai I Vtjayawada I Vizag I Tirupaji I Kukatpally I Kolkata
: 580 : Thermodynamics

09.
l 1 ) .4
Sol: BP = 4.5 kW 11 1 1- ( k J -1 1- (
r 0

r, 10.511
= =

BP 45 = .
IP = = · 5 294 kW
0.85 = 0.6098
11 m
IP(kW) x 3600 % Increase in efficiency
11 1 th =
mr (kg / hr) x CV(kJ / kg) 0.6098 - 0.605
X l OO = 0 _ 793 %
0.605
5.294 x 3600
0_ 3 =
=

m1 x 40000
1.5882 kg / hr 11.

mr 1 ·5882 Sol: FC = 0.15 0.03 x BP


ffif =

ISFC = = = 0.3 kg/kW.hr


+

IP 5.294 When FC = 0 indicates friction power which


is the negative intercept on Willans grap
10.
Sol: Volume per cylinder = 300 c.c. FC = 0 => BP = FP (Friction Power)
rk = V 1 Nc = 10.2 0 = 0.15 + 0.03 x FP
V1 - Vc = Vs O. l 5
FP = = 5 kW
10.2 Ve- Ve = 300 0.03

300 IP (kW) = BP (kW) + FP (kW)


Ve = - = 32.6 c.c.
9.2 = 20 + 5 =
25 kW ;

122 2 -120 2 ) . BP(kW) 20


% change in Power = ( 100 Mechamcal (11) = --- - 0.8
120 2 IP(kW) 25
= =

= 0.03361 or 3.361 % Pm; ·LANn


IP (kW)
120
103361
=

Vs = 300 x 310.083
100 P . x 0.09 x 1t x (0.075) 2 x 3000 x 4
=> 2 5 =
=

4 x120
V Ve = 310.083 + 32.6
rki = s
_
m1__________

= 10_ 511
+

Ve 32.6 :. Pmi =
628.76 kPa

�-1-(,'.f- 1-C�J· =
0.605

IIJllj@jjj4U.jj/4RflfiiMihih.ij>ttyderabad l Delhi I Bhopal I Pune I Bhubaneswar l wcknowl Patna ! Bengaluru I Chennai lVuayawadal V17.3g I Tlfllpati I Kukatpally J Kolkata I
Production
(Questions)
Page No. 582

CONTENTS
-Cha pte r ' -N �11t� 0; t�� Ch a;te r - -TQi.;stions 1Sol�ti ons--
1,
J
No . _ _ L __ _ -- ___ ___ _ � _____ _Page No .__�Pag� No._ ;
01 Casting 583 - 602 603 - 623

02 Welding 624 - 641 642 - 659

03 Metal Cutting 660 - 687 688 - 7 1 7

04 Machining 7 1 8 - 737 738 - 755

05 Metal Forming 756 - 770 771 - 785

06 Sheet Metal operations 786 - 797 798 - 808

07 Metrology 809 - 826 827 - 839

08 Advanced Machine Methods 840 - 853 854 - 862


Non-traditional machining
09 863 - 879 880 - 888
Methods
Ct Casting
(b) permeability to go through a maxima
(c) Compressive strength to go through a
maxima
One Mark Questions

01. Riser is designed so as to (d) strength to go through a maxima


(GATE-ME-87)
(a) Freeze after the casting freezes 05 . The pressure at the in-gate will be maximum
(b) Freezing before the casting freezes with the gating system
(c) Freeze at the same as the casting (GATE-ME-90)
(d) Minimize the time of pouring (a) 4:8:3 (b) 1 :3:3
(c) 1 :2:4 (d) 1:2:1
02. The contraction allowance provided on the
pattern and core boxes compensates for the 06. When there is no room temp change, the total
following type of contraction. shrinkage allowance on a pattern 1s
(GATE-ME-88) INDEPENDENT OF
(a) Liquid contraction (GATE-ME-91 )
(b) Solidification contraction (a) Pouring temp of the liquid metal
(c) Solid contraction (b) Freezing temp of the liquid metal
(d) All the above three types of contractions. (c) The component size
(d) Coefficient of thermal contraction of
03. Chills are used in molds to solidified metal
(GATE-ME-89)
(a) achieve directional solidification 07. Converging passage is used for feeding the
(b) reduce possibility of blow holes liquid molten metal into the mould to
(c) reduce the freezing time (GATE-ME-91)
(d) smoothen the metal for reducing spatter. (a) Increase the rate of feeding
(b) Quickly break off the protruding portion
04. Increase in water content in molding sand of the casting
causes (GATE-ME-89) (c) Decrease wastage of cast metal
(a) Flowability to go through a maxima (d) Avoid aspiration of air

\( l I 11�111t t 1111g Pul,lu ,11i1 )1i'i �yderabad j Delhi j Bhopa) I Pune l Bhubancswar l l.ucknowj Patna j Bengalwu j Chennai j Vijayawada j Vu.ag ITuupati I Kukatpally l Kolkatal
: 584 : Production

08. In a green sand moulding process, uniform 1 2. Disposable patterns are made of
ramming leads to (GATE-ME-2000)
(GATE-ME-92) (a) wood (b) rubber
(a) Less chance of gas porosity (c) metal (d) Polystyrene
(b) Uniform flow of molten metal into the
1 3 . Shrinkage allowance on pattern is provided to
mold cavity
compensate for shrinkage when
(GATE-ME-01)
(c) Greater dimensional stability of the
casting
(a) the temp of liquid metal drops from
(d) Less sand expansion type of casting
pouring to freezing temp
defect
(b) the metal changes from liquid to solid
state at freezing temp
09. Centrifugally casted products have
(GATE-ME-93)
(c) the temp of solid phase drops from
freezing to room temp
(a) Large grain structure with high porosity
(d) the temp of metal drops from pouring to
(b) Fine grain structure with high density
room temp
(c) Fine grain structure with low porosity
(d) Segregation of slug towards the outer 14. In centrifugal casting, the lighter impurities
skin of the casting are (GATE-ME-02)
(a) uniformly distributed
1 0. Light impurities in the molten . metal are (b) forced towards the outer surface
prevented from reaching the mold cavity by (c) trapped near the mean radius of the
providing (GATE-ME-96) casting
(a) Strainer (d) collected at the center of the casting
(b) Button well
1 5 . The primary purpose of sprue in casting mold
(GATE-ME-02)
(c) Skim bob
is to
(d) All of the above
(a) Feed the casting at rate consistent with
1 1 . Which of the following materials requires the the rate of solidification
largest shrinkage allowance, while making a (b) Act as a reservoir for molten metal
pattern for casting (GATE-ME-99) (c) Feed molten metal from the pouring
(a) Aluminum (b) Brass basin to the gate.
(c) Cast iron (d) Plain carbon steel (d) Help feed the casting until all
solidification takes place
\( I I 1 1 g 1 1 1 t t 1 111:..; P11l,!u .111111b I I I I I I I I
�ydcrabad Delhi Bhopal Punc Bhubancswar wcknow l Patna Bcngahrru Chennai J Vuayawada J Vu.ag Tirupati I I
Kukatpally Kolkata I
: 585 : Casting

(GATE-ME-03)
1 6. Hardness of green sand mould increase with 20. Two streams of liquid metal which are not

casting defect known as (GATE-ME-09)


hot enough to fuse properly result into a
(a) mcrease in moisture content beyond 6
percent (a) cold shut (b) Swell
(b) increase in permeability (c) blow hole (d) Scar
(c) decrease in permeability

(GATE-ME- 11)
(d) increase in both moisture content and 2 1 . Green sand mould indicates that
permeability
(a) Polymeric mould has been cured

(GATE-ME-04)
1 7. Misrun is a casting defect which occurs due (b) mould has been totally dried
to (c) mould is green in colour
(a) very high pourmg temperature of the (d) mould contains moisture
metal

(GATE-PI-89)
(b) insufficient fluidity of the molten metal 22. Negative allowance is provided on the pattern
(c) absorption of gases by the liquid metal to take care of
(d) improper alignment of the mould flasks (a) The distortion allowance
(b) The draft allowance

(GATE-ME-06)
1 8. An expandable pattern is used in (c) The machining allowance
(d) The shake allowance
(a) slush casting

(GATE-PI-89)
(b) squeeze casting 23. The process by which shell like castings (like
(c) centrifugal casting toys) are made is
(d) investment casting (a) Shell molding
(b) centrifugal casting
19. Which of the following engineering materials (c) slush casting
(d) die casting
(GATE-ME-07)
1s the most suitable candidate for hot
chamber die casting
(a) Low carbon steel 24. Poor collapsibility of the mold leads to
(b) titanium . . . . . . . . . . . . type of casting defect in aluminum
(c) Copper castings. (GATE-PI-89)
(d) tin

!Mlli!@jjji4ijjjQ@niN\jj¥�?1yderabad I Delhi I Bhopal I Pune I Bhubancswar I Lucknow I Patna I Bcngalwu I Chennai I V\iayawada I V,zag I T11Upati I Kukarpally I KolkalaI
.. � "E·=.!&i�1p1�·1CC1 �·�·::�========::::;���:=============�.:;��:�:;:�
ACE
��-�
= 586 Pro ductio n
::nn�w�Po
��Niraticus
25. For casting of turbine blades made of high 30. The optimum pouring time for a casting
temp and high strength alloys, the mo st depends on several factors. One important
suitable process is (GATE-PI -90) factor among them is (GATE-PI -91)
(a) Die casting (b) Investment casting (a) location of riser
(c) Centrifugal casting (d) Slush casting (b) porosity of sand mold
(c) fluidity of casting metal
26. For the same material, powder metallurgy (d) area o f the pouring basin
process is superior to casting for
(GATE-PI -89) 3 1. Ceramic slurry is used in making the molds
(a) making large products for (GATE-PI -91)
(b) better control over the density of product (a) investment casting
(c) better strength of the finished product (b) die casting
(d) making parts with wide variations of (c) shell mold casting
thickness at different sections (d) centrifugal casting

27. Green strength in powder metallurgy refers to 32. Low permeability molds and low pouring
the strength of (GATE-PI -89) temperature of the molten metal causes
(a) original material in the bulk form ...................... defect in castings.
(b) the powder before compaction (GATE-PI -91)
(c) The powder after compaction
33. Pressure die casting deals with casting metals
(d) the powder after sintering and cooling
of relatively . . . . . .. . . . .. A......... melting
point and provides ..... . B .. . ... production
28. The problem of shrinkage cavity is more
rate.
likely to occur while
(GATE-PI -91)
A. higher / lower B. higher / lower
casting ....... A................ B........ a chill
block (GATE-PI -90)
34. Increase in carbon content in plain carbon
A. metal / plastic B. with / without
steels raise its (GATE-PI -92)
29. Assertion: Co nverging passage is used for (a) Ductility and UTS
feeding liquid metal into a mould (b) Tensile strength and malleability

Reason: Inhalation of air promotes blow (c) Tensile strength and hardness

holes in casting (GATE-PI -90) (d) Ductility and melting temperature

!1i11ijj§jjjl4...4Rflnftijj.jj4' yderabad I Dellii I Bhopal I Pune I Bhubaneswar I Lucknow I Patna I Benga)lUll I Chcnnai I Vuayawadal Vizag I T111.1pati I Kukatpally I Kolkata
ACE : 587 : Casting
�-�
It V • • • •

� .,: � P.d,lir;ariooa

35. Investment casting uses A . . . . . . . . . . . . as 40. Shell molding 1s an investment casting


pattern material and is preferred for small process (T/F) (GATE-PI-95)
parts with . . . . . . . . . B . . . . . . designs.
(GATE-PI-92) 41. Which one of the following is NOT a
A. wax I resin coated sand property of a sand mold? (GATE-PI-03)
B. simple I intricate (a) permeability (b) Collapsibility
36. The primary function of a riser is to (c) Strength (d) Fluidity
(GATE-PI-92)
(a) feed molten metal to casting as it 42. Wood flour is added to core sand to improve:
solidifies (GATE-PI-04)
(b) prevent atmospheric air from (a) Collapsibility of core
contaminating the metal in the mold (b) Dry strength of core
(c) allow gases to easily escape from mold (c) Shear strength of core
cavity (d) Tolerance on casting
(d) allow molten metal to rise above the
mold cavity 43. An expandable pattern is used in
. (GATE-PI-OS)
3 7. Chaplets are placed between mold in order to (a) slush casting
(GATE-PI-92) (b) squeeze casting
(a)promote directional solidification (c) centrifugal casting
(b)help alloying the metal (d) investment casting
(c) facilitate easy removal of core from
casting 44. Cold shut is a defect in casting due to
(d)prevent core movement due to buoyancy (GATE-PI-06)
(a) sand sliding from the cope surface
38. Investment casting uses . . . . . . . . (wax/resin (b) internal voids or surface depression due
coated sand) as pattern material and is to excessive gas trapped
preferred for small parts with . . . . . . . . . . . . (c) discontinuity resulting from hindered
(simple/intricate) designs (GATE-PI-92) contraction
(d) two streams of material that are too cold
39. Bentonite 1s commonly used bonding
to fuse properly
material for dry sand cores (TIF)
(GATE-PI- 94)
IM•l�!i§hii11hQk11niMlh•h� yderabad J Delhi J BhopaJ IPune J Bhubaneswar J LucknowJ Patnal Bengalwu J Chennai J Vtjayawada J Vmg J Tirupati I Kukalpally l Kolkata
: 588 : Production

45. In hollow cylindrical parts made by 49. Which of the following casting processes
centrifugal casting, the density of the part is uses expandable pattern and expandable mold
(GATE-PI-08) (GATE-PI-11)
(a) Maximum at the outer region (a) Shell mod casting
(b) maximum at the inner region (b) Investment casting
(c) Maximum at the mid point between (c) Pressure die casting
outer and inner surfaces (d) Centrifugal casting
(d) Uniform throughout
50. A cube shaped casting solidifies in 5 mm.
46. Hot chamber die casting is NOT suited for The solidification time in min for a cube of
(GATE-PI-09) the same material, which is 8 times heavier
(a) Lead and its alloys than the original casting, will be
(b) zinc and its alloys (GATE-ME & PI- 13)
(c) Tin and its alloys (a) 1 0 (b) 20 (c) 24 (d) 40
(d) Aluminum and its alloys
51. Match the casting defects (Group A) with the
47. Solidification time of a metallic alloy casting probable causes (Group B):
(GATE-PI-10)
Group A Group B
(a) Directly proportional to its surface area
lS

(b) Inversely proportional to the specific P: Hot tears 1 : Improper fusion of tw0
heat of the cast material streams of liquid metal
(c) Inversely proportional to the thermal Q: Shrinkage 2: Low permeability of the
diffusivity of the mould material sand mould
(d) Inversely proportional to the pouring R: Blow 3 : Volumetric contraction
temp holes both in liquid and soli d
stage
48. In sand casting fluidity of the molten metal S: Cold shut 4: Differential cooling rate
increases with (GATE-PI-11)
(GATE-ME-14-SET-3)
(a) Increase in degree of superheat
(b) Decrease in pouring rate
(a) P- 1 , Q-3, R-2, S-4
(b) P-4, Q-3, R-2, S-1
(c) Increase in thermal conductivity of the
(c) P-3, Q-4, R-2, S-1
mold
(d) P- 1, Q-2, R-4, S-3
(d) Increase in sand grain size.

lit11i@jjj§4ijj/jRflGIM\11m�/ .yderabad J De1hi J Bhopal l Pune l Bhubaneswarl wcknowl PatnaJ Bengaluru J Chennai l Vijayawada ! Vu.ag J Tlfllpari I Kukatpally J Kolkata I
: 589 : Casting

52. The hot tearing in a metal casting is due to (a) P-l Q-2
, , -3 ,S-4
R (b) P-2 Q-4
, R, -1 , S-3
(GATE-ME-14-SET-3) (c) P-4Q-2, R, -1 ,S-3 (d) P-2Q-4
, , -3 ,S-l
R
(a) high fluidity
(b) high melt temperature 56. The part of a gating system which regulates
( c) wide range of solidification temperature the rate of pouring of molten metal is
( d) low coefficient of thermal expansion (GATE - 16- SET- 1)
(a) pouring basin (b) runner
53. Chills are used in casting moulds to
(GATE-PI-14)
(c) choke (d) ingate

(a) achieve directional solidification


57. The welding process which uses a blanket of
(b) reduce the roughness of top surface of the
fusible granular flux is
(GATE - 16 - SET - 2)
cast product
( c) increase the solidification time
(a) tungsten inert gas welding
( d) reserve excess molten metal
(b) submerged arc welding
54. In full mould (cavity-less) casting process , ( c) electroslag welding •
Pattern is made of (GATE -15 -Set 3) ( d) thermit welding
(a) Expanded polystyrene
(b) Wax 58. Equal amounts of a liquid metal at the same
(c) Epoxy temperature are poured into three moulds
(d) Plaster of Paris made of steel , copper and aluminum. The
shape of the cavity is a cylinder with 15 mm
55. Match the items in the first column to their diameter. The size of the moulds are such .
functions in the second column. that the outside temperature of the moulds do
P. Sprue 1. Regulates flow of molten not increase appreciably . beyond the
metal into mould cavity atmospheric temperature during
Q. Riser 2. Feeds molten metal from solidification. The sequence of solidification
pouring basin to gate in the mould from the fastest to slowest is
R. Gate 3. Acts as a reservoir for molten (Thermal conductivities of steel , copper and
metal aluminum are 60.5 , 401 and 237 W/m-K ,
s. Pouring 4. Supplies molten metal to
respectively. Specific heats of steel , copper
and aluminum are 434 , 385 and 903 J/kg-K ,
basin compensate for liquid

(GATE - PI -15)
shrinkage
respectively.

jltlli@jjji4.jjjQRflnfMjd.jj�)"yderabad I Delhi I Bhopal I Pune I Bhubaneswari Lucknow I Patna I Bengaluru I Chennai I Vuayawadaj Vu.ag I Tuupati I Kukatpally I Kolkata I
ACE 0: Prod tio=
�... .. = Fn
:.� E... !!l'lll
�·=·�Pu
;;F
-
�·� :,;;;:;':·:;:::�=========:=5=
Nicdioos 9==================u=c==n

Densities of steel, copper and aluminum are D. Centrifugal casting 4. Hollow castings
7854 , 8 933 and 2700 kg/m 3 , respectively.) like lamp shades
(GATE - 16 - SET- 3) 5. Rain water pipes

(a) Copper - Steel - Aluminum 6. Cast iron shoe


brake
(b) Aluminum- S teel - Copper
(c) Copper - Aluminum - Steel
(d) Steel - Copper- Aluminum 03. Only 4 pair s can be matched
(GATE-ME-94)

59. In carbon dioxide molding process, the Casting Process

(GATE - P I- 17)
binder used i s A. Investment ca sting
(a) Sodium bentonite B. Die casting
(b) Calcium bentonite C. Centri fugal casting
(c) Sodium silicate D. Drop forging
(d) Phenol formaldehyde E . Extrusion
F. Shell molding

Product
Two Marks Questions
1. Turbine rotor s
2. Turbine blades
0 1. Two cubical casting s of the same metal and 3. Connecting rod s
sizes of 2cm side and 4cm side are moulded 4. Galvanized iron pipes
in green sand. If the smaller ca sting solidifie s 5. Cast iron pipe s
in 2min s, the expected time of solidifications 6. Carburetor body
of larger casting will be
(GATE-ME-89) 04. Match the following (GATE-ME-98)
(a) 16 min (b) 2 .Js min Group - I

(c) 8min (d) 4min A. Sand casting


B. Plaster mold casting interior
02. Match the following : (ME-GATE-92) C. Shell mold casting
D. Investment casting
Casting Process Product Group- II
A. Slush casting 1 . Turbine blade
1. Symmetrical and circular shapes only
B. Shell molding 2. Machine tool bed
2. Par ts have hardened skin s and soft interior
C. Dry sand molding 3. Cylindrical block

jltl1ii@jjji4@ijmftiM\iiijj� yderabad ! Delhi! Bhopal! Pune J Bhubaneswar! Lucknow ! Patna! BengaJurul Chennai J Vrjayawada ! Vmg j Tirupari I Kuk.atpalfy! Kolkata
.., " · AJCE · · · C
tiiratioos ========� 59 1

•• •� = : ��:�=============�:
tmg
'
� : L � �- m
� �
E p:: ·����=== ���

3. Minimum po st casting process (a) A-3 ' B-2 ' C-4 ' D-5
4. Part s have soft skin and hard interior (b) A-1 ' B-3 ' C-4 ' D-5
5. Parts a have tendency to warp (c) A-1 , B-2 , C-3, D-4
6. Suitable only for non ferrous metal s. (d) A-3 ' B-2 ' C-5 ' D-4

05. With a solidification factor of 0.97 x I 06s/m 2 , 08. A mould has a down sprue whose length is
the solidification time (in seconds) for a 20 cm and the cross sectional area at the base
spherical casting of 200mm diameter is of the down sprue is 1 cm 2 • The down sprue
(GATE-ME-03) feeds a horizontal runner leading into the
(a) 539 (b) 1078
mould cavity of volume 1000 cm 3 • The time
(c) 4311 (d) None
required to fill the mould cavity will be
(GATE-ME-05)
06. Gray cast iron bloc ks 200 x 100 x 10 mm are
(a) 4.05 s (b) 5.05 s
to be cast in sand moulds. Shrinkage
(c) 6.05 s (d) 7.25 s
allowance for pattern making is 1 %. The ratio
of the volume of pattern to that of the casting
09. In a sand casting operation , the total liquid
will be (GATE-ME-04)
head is maintained constant such that it is
(a) 0.97 (b) 0.99 (c) 1.01 (d) 1.03
equal to the mould height. The time taken to
fill the mold with a top gate is tA, If the same
07. Match List - I (products) with List - II
mold is filed with a bottom gate , then the
(casting process) and select the correct
answer u sing the codes given below the list s time taken is ta. ignore the time required to
(GATE-ME-05) fill runner and frictional effects. Assume
atmospheric pressure at the top molten metal
List - I List - II
surfaces. The relation between the t A and ts i s
A. Hollow statues I .Centrifugal casting
(GATE-ME-06)
B. Dentures 2.Investment casting
(b) ts = 2 ,tA
C. Aluminum
alloy pi ston s 3.Slu sh casting (d) ts = 2 ..J2 ,tA
D. Rocker ar m 4.Shell moulding
5.Gravity die casting

!1i11ih4jjji4ijjj4@biM\dih� yderabad I Delhi I Bhopal I Pune I Bhubaneswarl Lucknow I Patna I Bengalwu I Chennai I Vuayawadal Vizag ITuupati I Kukatpallyl Kolkala
: 592 : Production

10. If a particular Fe-C alloy contains less than


0.83% carbon, it is called
Pouring
Basin

(GATE-ME-07)
(a) High speed steel 200 mm
(b) Hypo-eutectoid steel
(c) Hyper eutectoid steel
(d) Cast irons
(a) 650 (b) 350 (c) 290.7 (d) 1 90.0

11. Volume of a cube of a side "I" and volume of


13. While cooling, a cubical casting of side
sphere of radius 'r' are equal. Both the cube
40mm undergoes 3% , 4% and 5% volume
and the sphere are solid and same material. shrinkage during the liquid state, phase
They are being cast. The ratio of the transition and solid state respectively. The
solidification time of the cube to the same of volume of metal compensated from the riser
the sphere is (GATE-ME-07) (GATE-ME-OS)
3 (a) 2% (b) 7% (c) 8% (d) 9%
(a) (41t/6) . (rll)6
lS

(b) (41t/6).(rll/
2 3
(c) (41t/6) . (rll) (d) (41tl6)2 . (r/[)4
14. Match the following (GATE-ME-09)

12. A 200 mm long down sprue has an area of Part used in casting Purpose

cross section of 650 mm2 where the pouring


P. Metallic chills 1. Support for the core

basin meets the down sprue (i.e. at the


Q. Metallic 2. Reservoir of the
chaplets molten metal
beginning of the down sprue). A constant R. Riser 3. Control cooling of
head of molten metal is maintaining by the critical sections
pouring basin. The molten metal flow rate is s. Exothermic 4. Progressive

6.5 x l 05 mm3/s. considering the end of down


padding solidification

sprue to be open to atmosphere and an (a) P-1, Q-3, R-2, S-4


acceleration due to gravity of 10 mm/s , the 4 2
(b) P- 1 , Q-4, R-2, S-3
area of the down sprue in mm2 at its end (c) P-3, Q-4, R-2, S-1
(avoiding aspiration effect) should be (d) P-4, Q-1, R-2, S-3
(GATE-ME-07)

IM•IM@hia§jjji4Rflft1Mihih� yderabad I Delhi I Bhopal I Pune I Bhubaneswar I Lucknow I Pama I Bengaluru I Chennai I Vijayawada I Vizag I Tuupati I Kukatpal]y I Kolkata
"�. " A!CE
: ==============::::;:as
".

il!i
·
F.ngioeer
,, �.a,l

� :�
· · ·
:r.zmg Pl�Jh:ili:: :·� �=========,:_= 593
oo: �� C= '�
tmg
.. '= IICati

15. In a gating system, the ratio 1 :2 :4 represents 18. Shrinkage allowance on pattern is provided to
(GATE-ME-10) compensate for shrin kage when
(a) Sprue base area : Runner area : in-gate (GATE-PI-89)
area (a) the temperature of liquid metal drop s
(b) Pouring basin area in-gate area from pouring to freezing temperature
Runner area (b) the metal changes from liquid to solid
(c) Sprue base area : in-gate area casting state at freezing temperature
area : (c) the temperature of solid pha se drops from
(d) Runner area : in-gate area : Casting area freezing to room temperature
(d) the temperature of metal drops from
16. For the case of molding , it is decided to pouring to room temperature
replace a spherical ri ser of diameter 100 mm
19. What is the velocity of the steel at the bottom
by a cylindrical riser. Determine size of the
of the sprue if the sprue height i s 30 cm?
cylindrical riser that will have the identical
a ssume the frictional and other losses to be
solidification time. Assume blind risering & 20%. (GATE-PI-89)
ratio of height to radius of cylinder i s 2.
(GATE-ME-10) 20. When there i s no room temp change , the total
(a) 10cm (b) 100cm shrin kage allowance on a pattern 1s
(c) 50cm (d)5cm INDEPENDENT OF (GATE-PI-90)
(a) pouring temperature of the liquid metal
17. A cubic casting of 50 mm side undergoes (b) freezing temperature of the liquid metal
volumetric solidification shrinkage and (c) the component size
(d) coefficient of thermal contraction of
volumetric solid contraction of 4% and 6%
solidified metal
respectively. No riser is u sed. Assume
uniform cooling in all direction s. The side of 21. Match the application with type of pattern
cube a fter solidification and contraction is Application
(GATE-ME-1 1) A. Undercut in components
(a) 48.32 mm (b) 49.90 mm B. Large bells
(c) 49.94 mm (d) 49.96 mm C. Mass production of casting by Machine
molding
D. Components with irregular parting lines

l
!1D1ih§jjj4U@MAflbi@hiiR�)Hyderabad j Delhi j Bhopal j Pune Bhubaneswarl
wcknow j Patna j BengalW'U j Chennai l Vtjayawada J Vizag jT1IUpati I Kukatpallyl Kolkata I
Type of pattern 4. Loosely rammed sand mold
1. Cope and drag 5. Gas entrapment (GATE-PI-95)
2. Follow board
3. Gated 25. The height of the down sprue is 175 mm and
4. Loose piece its CS area at the base is 200 mm2 • The CS
5. Sweep (GATE-PI-92) area of the horizontal runner is also 200 mm2 •
Assuming no losses , indicate the correct
22. Manufacturing process Property choice for the time (seconds) required to fill a
(GATE-PI-93) mold cavity of volume 106 mm3 .( use g =10
A. Hot tears 1. Mould restraint m!s2) (GATE-PI-02)
B. Porosity 2. Inadequate risering (a) 2.67 (b) 8.45 (c) 26.72 (d) 84.50
C. Sand inclusions 3. High pouring temp
D. Shrinkage cavity 4. Loosely rammed 26. The permeability of molding sand was
sand mold determined using a standard AFS sample by
5. Gas entrapment passing 2000cc of air at a gage pressure of 10
g/ cm2 . If the time taken for the air to escape
23. According to Chvorinove ' s rule , the was 1 min , the permeability number is:
solidification time of a casting is proportional (GATE-PI-02)
to [Volume/ Surface area ] n where n =... (a) 112.4 (b) 100.2 (c) 75.3 (d) 50.1
(GATE-PI-94)
(a) 0.5 (b) 1 (c) 2 (d) 4 27. Proper gating design in metal casting
(GATE-PI-02)
24. Match the cast defect with their cause P: Influences the freezing range of the melt
Casting Defect Q: Compensates the loss of fluidity of the
A. Hot tears melt
B. Porosity R: Facilitates top feeding of the melt
C. Sand inclusions S: Avoids misrun
D. Shrinkage cavity (a) P, R (b)Q , S (c) R , S (d) P ,S
Most Appropriate Cause
1. Mold restraint 28. A 10 mm thick steel bar is to be horizontally
2. Inadequate risering cast with two correctly spaced top risers of
3. High pouring temp adequate feeding capacity. Assuming end

111111;.;;;;gg.o;waiMihlO.� yderabad I Delhi I Bhopal I Pune I Bhubaneswar l Lucknow! Patna I Bengaluru I Chennai I Vliayawadaj Vizag I T1111pati I Kukatpally I Kolkata
� ACE
::ll'....:..-:....Publicatiooa : 595: Casting
��•.�=........- ====�==================================
effect without chill, what should be the (a) 14.38 (b) 20.34
theoretical length of the bar ? (GATE-PI-02) (c) 28.76 (d) 57.52
(a) 96 mm (b) 132 mm
(c) 192 mm (d) 156 mm 32. Chaplets are placed between mould and core
surfaces in order to: (GATE-PI-03)
29. Process - Mold making technique (a) Reduce directional solidification
(GATE-PI-02) (b) Help local alloying of molten metal
P: Green sand molding 1. Pouring (c) Help easy removal of core from casting
Q: Shell molding 2. Dipping (d) Prevent core movement due to buoyancy
R: Investment molding 3. Compaction
S: Ceramic molding 4. Resin bonding 33. Match the items of List I (Equipment) with
p Q R s p Q R s the items of List II (Process) and select the
(a) 2 1 3 4 (b) 1 3 4 2 correct answer using the given codes
(c) 4 2 2 3 (d) 3 4 1 1 (GATE-PI-04)
List-I(Equipment) List-II (Process)
30. Gating ratio of 1: 2: 4 is used to design the
P : Hot Chamber Machine 1 - Cleaning
gating system for magnesium alloy casting.
Q : Muller 2 - Core making
This gating ratio refers to the cross section
R : Dielectric Baker 3 - Die Casting
areas of the various gating elements as given
S : Sand Blaster 4 - Annealing
below (GATE-PI-03)
2. Runner 3. Jngates
5 - Sand mixing
1. Down sprue
p Q R s p Q R s
The sequence of the above elements in the
(a) 2 1 4 5 (b) 4 2 3 5
ratio 1: 2; 4 is
(c) 4 5 1 2 (d) 3 5 2 1
(a) 1 , 2& 3 (b) 1, 3& 2
(c) 2, 3 & 1 (d) 3, 1 & 2
34. The shape factor for a casting in the form of
an annular cylinder of outside diameter 30
31. A casting of size 100 mm x100 mmx50 mm
cm, inside diameter 20 cm and height 30 cm
is required. Assume volume shrinkage of (correction factor k = 1.0) will be
casting as 2.6%. If the height of the riser is 80
(GATE-PI-05)
mm and riser volume desired is 4 times the (a) 21.77 (b) 6.28
shrinkage in casting, what is the appropriate
(c) 9.42 (d) 12.28
riser diameter in mm? (GATE-PI-03)

l@•1ih§jjji4jjj/QRfl§iMjO,jj.yderabad I Delhi I Bhopal I Pune I Bhubaneswar I Lucknow I Patna I Bengaluru I Chennai jVgayawada jVmg I Tuupati I Kukatpally I Kolkata I
ACE 5 96 Producnon
:.
� E'E&�rgm�·-::m�w���b�£�""1��:•:.::
.. "= ========�:�;�:============��:��:· :�

35. A cast steel slab of dimension 30x2ox5 cm is 39. The mold filling time in seconds is
poured horizontally using a side riser. The (GATE-PI-07)
riser is cylindrical in shape with diameter and (a) 2.8 (b) 5.78 (c) 7. 54 (d) 8. 41
height, both equal to D. The freezing ratio of
the mould is (GATE-PI-05) 40. In sand casting of hallow part of lead , a
(a) 8D / 75 (b) 4D / 75 cylindrical core of diameter 120 mm and
(c) 75 I 8D (d)75 / 4D height 180 mm is placed inside the mould
cavity. The densities of core material and
36. A casting of size 400mmx200mx140mm
lead are 1600kg/m3 and 11300 kg/ m3
solidifies in 20min. the solidification timefor
respectively. The net force ( in N ) that tends
a casting 400mmx200mmx35mm under
to lift the core during pouring of molten
similar conditions is (GATE-PI-06)
metal will be (GATE-PI-08)
(a) 2min (b) 3.3min
(a) 19.7 (b) 64. 5 (c) 193.7 (d) 257.6
(c) 4min (d) 8. l min

37. Match the following. (GATE-PI-07) 41. A solid cylinder of diameter D and height
Group-1 Group-II equal to D, and a solid cube of side L are
P. Sand casting 1. Turbine blades being sand cast by using the same material.
Q. Centrifugal casting 2. I.C engines Assuming there is no superheat in both the
R. Investment casting 3. Large bells cases, the ratio of solidification times of the
S. Die casting 4. Pulleys cylinder to the solidification time of the cube
p Q R s p Q R s lS (GATE-PI-09)
(a) 4 1 3 2 (b) 2 4 3 1 (a) ( L ID) 2
(b) ( 2L /D)2
(c) 3 4 1 2 (d) 3 2 1 4 (c) ( 2D/L)2 (d) ( DIL)2

Statement for Linked Answer Q38 & Q39 42. During the filling process of a given sand
In a sand casting process, a sprue of 10mm base mould cavity by molten metal through a
diameter and 250mm height leads to a runner horizontal runner of circular C.S, the
which fills a cubical mold cavity of 100mm size. frictional head loss of the molten metal in the
(GATE-Pl-07) runner will increase with the (GATE-PI-10)
38. The volume flow rate (in mm /s ) is
3
(a) Increase in runner diameter
(a) 0. 8 X 105 (b) 1.1 X 105 (b) Decrease internal surface roughness of
(c) 1. 7 X 105 (d) 2.3 X 105 the runner

!ltili@jjji4@$AflGi@ijj.jj+yderabadlDelhilBhopaIIPunelBhubaneswarl wcknow1Patna1BengaJurulChennailVuayawadajVizag ITirupati I Kukatpallyl Kolkata I


�-twli'.....:.....:...
ACE : :
Pub1icationa 597 Casting
� �=�==...
..
=================================
(c) Decrease in length of runner 46. A cylindrical blind riser with diameter d and
(d) Increase in average velocity of molten height h, is placed on the top of the mold
metal cavity of a closed type sand mold as shown in
the figure. If the riser is of constant volume,
43. In a sand casting process, a sphere and a then the rate of solidification in the riser is
cylinder of equal volumes are separately cast the least when the ratio h:d is
from the same molten metal under identical (GATE-ME-14-SET-3)
conditions. The height and diameter of the
cylinder are equal. The ratio of the
solidification time of the sphere to that of the
cylinder is (GATE-PI-11)
h
I
(a) 1.14 (b) 0.87 (c) 1.31 (d) 0.76 Mold cavity

44. A mold having dimensions 10ox9ox20 (all (a) 1:2 (b) 2:1 (c) 1:4 (d) 4:1
in mm) is filled with molten metal through a
gate with height 'h' and C. S area A, the 47. A cylindrical riser of 6 cm diameter and 6 cm
mould filling time is t1. The height is now height has to be designed for a sand casting
quadrupled and the cross sectional area is mould for producing a steel rectangular plate
halved. The corresponding filling time is t2. casting of 7cmx10cmx2cm dimensions
The ratio t2/t 1 is (GATE-PI-12) having the total solidification time of 1.36

(a) }i (b) 1 (c) ../2 (d) 2


minute. The total solidification time (in
minute) of the riser is ___
(GATE-ME- 14-SET-4)
45. An aluminium alloy (density 2600 kg/m ) 3

casting is to be produced. A cylindrical hole 48. For a given volume of a riser, if the
of 100 mm diameter and 100 mm length is solidification time of the molten metal in
made in the casting using sand core (density riser needs to be quadrupled, the surface area
1600 kg/m3 ). The net buoyancy force (in of the riser should be made
Newton) acting on the core is ___ (GATE-PI- 14)
(GATE-ME-14-SET-l) (a) one-fourth (b) half
(c) double (d) four times

!11•1i@jjji4@iRffbiM\hijj� yderabad I Delhi I Bhopal I Pune I Bhubaneswar I wcknowl Patna I Bengalwu I Chermai I Vijayawada I Vmg I Tuupati I Kukatpal)y I Kolkata
: 598: Production

49. The solidification time of a casting 1s 52. Ratio of solidification time of a cylindrical
casting (height = radius) to that of a cubic
proportional to (�)' ,Where Vis the volume
casting of side two times the height of
of the casting and A is the total casting cylindrical casting is ____
surface area losing heat. Two cubes of same (GATE-15-Set 3)
material and size are cast using sand casting
process. The top face of one of the cubes is 53. In a metal casting process, molten copper
completely insulated. The ratio of the alloy is poured into a sand mould. The level
solidification time for the cube with top face of molten metal in the pouring basin is at a
insulated to that of the other cube is height of 300 mm from the runner having
(GATE-15-Set 1) diameter of 10 mm. if the density and melting
25 36 temperature of molten copper alloy are 9000
(a)
36
(b)
25
(c)l (d) i5 kg/m3 and 1000°C. respectively then the rate
of flow of molten metal into the mould
50. A cube and a sphere made of cast iron (each neglecting friction and other losses, in cm3 Is
volume 1000 cm3 ) were cast under identical lS

conditions. The time taken for solidifying the (GATE - PI-15)


cube was 4 s. The solidification time ( in s)
for the sphere is ____ 54. Heat is removed from a molten metal of mass
(GATE-15-Set 2) 2 kg at a constant rate of 10 kW till it is
completely solidified. The cooling curve is
51. The dimensions of a cylindrical side riser shown in the figure.
(height = diameter) for a 25cm x 15cm x5cm
1100 -
steel casting are to be determined. For the s, 2023K)
l OOO _�
tabulated shape factor values given below, ,..._

the diameter of the riser (in cm) is___


� 900 -I "' (20s, 873K)

800 - (I Os, 873K)


I
0..
2 4 6 8
-
10 12
E 100
Shape factor
600 -
f-<
Riser volume/ 1. 0 0.70 0.55 0.50 0. 40 0. 35 (30s, 600K)
500
Casting
-+-----., --�, --�1 ---1
volume 10 20 30 40
Time (s)
0
(GATE-15-Set 3)

!1ti•Qb§jjji4.jjjQRflniM\jjf¥j.Hyderabad I Delhi I Bhopal I Pune I Bhubaneswarl Lucknow I Patna I Benga]uru I Chcnnai jVrjayawada jVizag ITirupati I Kuk.atpally I Kolkata I
�-t ACE :599: Casting
-:,� �====================================
:�Pnliliratioos

Assuming uniform temperature throughout of the ratio of its volume to its surface area.
the volume of the metal during solidification, The ratio of solidification time of the block P
the latent heat of fusion of the metal (in to that of the block Q is ___
kJ/kg) is __ (GATE - PI-16)
(GATE-16-SET-1)
58. A 300 mm long copper wire of uniform
55. A cylindrical job with diameter of 200 mm cross-section is pulled in tension so that a
and height of 100 mm is to be cast using maximum tensile stress of 270 MPa is
modulus method of riser design. Assume that developed within the wire. The entire
the bottom surface of cylindrical riser does
deformation of the wire remains linearly
not contribute as cooling surface. If the
elastic. The elastic modulus of copper is 100
diameter of the riser is equal to its height,
then the height of the riser (in mm) is GPa. The resultant elongation (in mm) is
(GATE-16-SET-1) (GATE - PI-16)
(a) 150 (b) 200 (c) 100 (d) 125
59. A sprue in a sand mould has a top diameter
56. Gray cast iron blocks of size 100 mm x 50 of 20 mm and height of 200 mm. The
mm x 10 mm with a central spherical cavity velocity of the molten metal at the entry of
of diameter 4 mm are sand cast. The the sprue is 0.5 mis. Assume acceleration
shrinkage allowance for the pattern is 3%. due to gravity as 9.8 m/s2 and neglect all
The ratio of the volume of the pattern to losses. If the mould is well ventilated, the
volume of the casting is ___
velocity (upto 3 decimal points accuracy) of
(GATE-16- SET-2)
the molten metal at the bottom of the sprue
IS ___ m/s.
57. Two cast iron blocks P and Q, each of 500
mm length, have the same cross-sectional (GATE-17-SET-l)
area. Block P has rectangular cross-section
of 100 mm x 200 mm. Block Q is of square 60. Schematic diagram of pouring basin and
cross-section. Both P and Q were cast under sprue of a gating system is shown in the
the same conditions with all their surfaces Figure. Depth of molten metal in the
enclosed within the mould. The solidification pouring basin is 100 mm and the height of
time of a casting is proportional to the square the sprue is 1,500 mm.

!1i11ij@jjj44ijji4RflGi@jijjj+ydcrabad I Delhi I Bhopal I Pune I Bhubaneswar I Lucknow! Patna I Bcngaiuru I Chennai IVuayawada I V=g I Tirupali I Kukatpa!Jy I Kolkata I
ACE: 600: Production
�-\
�... �� Pnbli,7',(1111
"====================================
of cast iron is 78 kN/m3 and that of the mold
sand 16 kN/m3 , find lifting force on the cope
created by buoyant force, weight of sand in
the cope, the net force at the liquid-sand
interface and additional weight to be kept on
1500 mm the cope.
(GATE-ME-91) (6M)

03. Calculate the ratio of the solidification times


H
d,
Considering the cross-section of the sprue is of two steel cylindrical risers of sizes 30cm
circular, the ratio d 1 :d2 to avoid aspiration is diameter by 60 cm in height and 60cm
(GATE- PI- 17) diameter by 30 cm in height subjected to
(a) 3:2 (b) 5:6 identical conditions of cooling
(c) 15:16 (d) 1:2 (GATE-ME-92) (4M)

04. An aluminum cube of 10 cm side has to be


cast along with a cylindrical riser of height
Five Marks Questions
equal to its diameter. The riser is not
insulated on any surface. If the volume
01. A down sprue of 180mm length has diameter shrinkage of aluminium during solidification
of 20mm at its top end. The liquid metal in is 6%, calculate shrinkage volume of cube on
the pouring cup is maintained upto 60 mm solidification and minimum size of the riser
height. What should be the theoretical so that it can provide the shrinkage volume.
diameter of down sprue at its lower end to (GATE-ME-93) (SM)
ensure that there is no aspiration effect.
(GATE-ME-87) (4M) 05. The pouring basin in a sand mold is 200 mm
diameter 100 mm height. The mold cavity is
02. A flak consisting of a cope and drag has the a cube of 125 mm side and must be filled in
following dimensions: length 300mm, width 25 seconds. The maximum velocity must
200mm and total depth 200mm (cope height ensure laminar flow into the mold cavity and
50mm) . A 150mm x 100mm x 50mm cast in the gates of circular cross section. Assume
iron block is to be cast. If the specific weight (a) constant temperature, (b) no loss in the

\( I l 11�][\( t llll� P11l1!11 ,tl]t)Jl", �ydcrabadlDclhilBhopalll'wlelBhubancswarl wcknowlPatnalBengaiurulChcnnailVijayawada!Vmg ITirupati I Kukatpallyl Kolkata I


ACE
�•
-:i
J
"'.l!iF'n�"'°�· -:P!ffll
"= :·�11�Po�bticmooa �·�·�========�= �60�1�: ==============�C�as:tmg
�· �
velocity head and (c) diameter of gate =8 09. Molten aluminum was poured in a sand mold
times diam�ter at the sprue base. Properties and the thickness of solid skin formed after
of the molten metal are y =0.9 mm2 per 20 seconds and 50 seconds were fond to be
second, p =700 kg/m3 . & Cp=33.6 J/mol-K. 3mm and 4.5mm respectively. What would
The diameter of sprue to avoid aspiration be the thickness of the solid skin at the end of
effect. (GATE-ME-96) (SM) 100 seconds after pouring? (GATE-PI-89)
(a) 5.5mm (b) 6.19mm
06. Two castings of the same metal have the (c)7.1mm (d) none
same surface area . one casting is in the form
of a sphere and the other is a cube. What is 10. A steel plate 40 cm x 30cm x 0. 3cm is to be
the ratio of the solidification time for the cast. The volume shrinkage of steel during
sphere to that of cube. solidification is 3%. A cylindrical side riser
(GATE-ME-98) (4M) with diameter 4cm and height 4cm is used.
The riser volume should at least be 3 times
07. In a particular mold design, the down sprue
that the dictated by shrinkage consideration.
has an area of Cross section of 6.45 cm2
Is the riser volume sufficient if not what is
where the pouring basin leads into the sprue .
the riser size? (GATE-PI-90) (6M)
The sprue is 20 cm long. The required metal
flow rate at the top section of the sprue is 820
11. A rectangular block of steel C.S
cm3/s. Determine the pounng height
100 x 150mm and 250mm height is to be
necessary above the sprue top. Also
determine the area ofC.S of the sprue at its cast with out any riser. The block is moulded

bottom to avoid the aspiration of liquid entirely in the drag of a green sand flask & is

molten metal. top gated. The cope of the flask is 200mm

(GATE-ME-99) (SM) height & the height of the metal during


pouring is 100mm above the cope level. A
08. Two castings, a cube and a slab of the same tapered sprue is employed & the gating ratio
material solidify under identical conditions. is 1:4:2. assume no energy losses in the
The volumes of the castings are equal but the system, The time taken (in seconds) to fill the
slab dimensions are in the ratio of 1 :2:4. find casting cavity, if the tapered sprues with 500
the ratio of the solidification times of the mm2 exit area is used. (GATE-PI-91) (6M)
cube to that of the slab.
(GATE-ME-00) (4M)

\('I l 11g1lltl 1111� P11hlu.1\J(111..., �yderabadlDelhilBhopaliPunclBhubancswarl LucknowlPatnalBcngaJurulChcnnailV\iayawadalV,z.ag ITirupati I Kukatpa))yl Kolkata I


��-� ========�:���:============�����·o�:..:
:i;&�rp1�·-:en111·� �pg�1,�.s.:r;m�ai:•:.,
ACE 602 Product:l n
.. "=
12. If the freezing ratio equal to (casting area/
casting volume)/(riser area/riser volume) & 15. Three castings are of the same material,
the volumetric ratio equal to riser volume/ volume and cast under similar conditions.
casting volume, for a disc casting of 20cm One is a sphere, another is a cube and the
diameter and 10 cm thick are 1.4 and 0.8 third is cylinder with its length being equal to
respectively. The corresponding values for a its diameter. (GATE-PI-9S) (6M)
square disc of 20cm side and 10cm thick if i) The ratio of solidification times of
the same cylindrical riser with a height to sphere to the cube is
diameter ratio of unity is used in both cases. (a) 1.54 (b) 0.65 (c) 0.848 (d) 1.3
(GATE-PI-92) (SM)
(a) 1.4 & 0.68 (b) 1.4 & 0.8 ii) The solidification time of the sphere to
(c) 1.2 & 0.8 (d) 1.2 & 0.68 the cylinder .
(a) 0.767 (b) 1.3
13. A casting of 200 mm x 100 mm x 70 mm (c) 1.18 (d) none
size solidifies in ten minutes. Estimate
solidification time for a 200 mm x 100 mm x 16. For true centrifugal casting process with the
10 mm casting under similar conditions? axis of rotation in horizontal direction, an
(GATE-PI-93) (SM) acceleration of 75g, where g is the
acceleration due to gravity, is required for
14. A cylinder of 150mm diameter & 200mm sound casting. the relation between diameter
height is to be cast with out any riser. The .'D' (in mm) and the rotational speed N (
cylinder is moulded entirely in the drag of a l rev/s) for this casting is N2 D = constant,
green sand flask & is top gated. The cope of determine the constant. Also calculate the
the flask is 200mm height & the height of the mold speed (in rev/se(c) for casting CI pipes
metal during pouring is 50mm above the of 5m long with outside and inside diameters
cope. A tapered sprue is employed & the of 0.52m and 0.5m respectively
gating ratio is 1:1.5:2. The time taken (in (GATE-PI-Ol)(SM)
seconds) to fill the casting cavity neglecting
energy losses, if the in-gate area is
400sq.mm. (GATE-PI-94)(SM)

.\t l. l.11�111ll1111� P,1hl1<.1t11111 .... FyderahadlDelhil BhopaIIPunelBhubaneswarl Lucknow1Patna1BengaiurulChenoailVuayawadalVizag ITtrupati I Kukatpal)yl Kolkatal
SOLUTIONS
05. Ans: (c)
Sol: Maximum pressure occurs m the gating
One Mark Solutions
system where the ratio of cross sectional
01. Ans: (a & c) area of in gate to sprue is maximum.
Sol: The riser should be designed in such way For option(a) the ratio is 3/4
that it should supply molten metal to For option(b) the ratio is 3/1
compensate liquid shrinkage. This can be For option(c) the ratio is 4/1
achieved only when the molten metal in the For option(d) the ratio is 1/1
riser solidified after or at the same time of Among above all options option (c) has max
the casting. ratio.

02. Ans: (c) 06. Ans: (a)


Sol: The shrinkage allowance provided on the Sol: Shrinkage allowance on the pattern
pattern is to compensate the solid shrinkage = La(�T)
taking place during cooling of material from = La(Tr-Tr)
freezing temp to room temp as a solid Where Tf = freezing temperature
means that it is for compensating solid Tr = room temperature
shrinkage. From the above equation, shrinkage
allowance depends on dimension of the
03. Ans: (a)
part, coefficient of thermal expansion,
Sol: By providing chills (high thermal
freezing temperature and then room
conductivity metal pieces) it is possible to
temperature. Hence the only parameter
direct heat to transfer in a required direction
which is not influencing is pouring
called as directional solidification.
temperature.

04. Ans: (a)


07. Ans: (d)
Sol: By adding more water into the molding
Sol: By using converging passage for the sprue
sand, the flow ability of moulding sand
with straight tapered or parabolic tapered
increases so that it is possible to make
will eliminate the aspiration effect.
complex cavity shapes.
!li11i@jjj4iijjjjQAflnj@11.jj� yderabad I Dellii I Bhopal I Pune I Bhubaneswar I Llcknowl Patna! Bengaluru I Chennai I Vijayawada I Vm,g I Tirupati I Kukatpa)ly I Kolkata
: 604 : Production

08. Ans: (c) polystyrene will be removed from cavity


Sol: With uniform ramming action, the mould through porosity property of mold or using
will not get distorted and hence the casting through riser.
is dimensionally stable.
13. Ans: (c)
Sol: In casting process the solid shrinkage is
09. Ans: (b & c)
compensated by providing shrinkage
Sol: Both are the answers, due to centrifugal
allowance on the pattern. The solid
force the compaction of the molten metal
shrinkage is the shrinkage taking place
particles will take place which avoids the
during cooling of material from freezing
porosity, density becomes high and due to
temperature to the room temperature in
fast cooling fine grain structure will take
solid state.
place.

1 4. Ans: (d)
10. Ans: (c)
Sol: In centrifugal casting the centrifugal force
Sol: The bottom part of the skim bob 1s
acting the particles depends on the density
sometimes called as bottom well as it
of the particles, the lighter impurities have
prevents only heavy impurities, some
lower density and so they are collected at
extent strainer may removes lighter
the center.
impurities but skim-bob is only separating
the lighter impurities.
15. Ans: (c)
Sol: Sprue is the connecting passage between the
11. Ans: (b)
pouring basin and runner for supplying
Sol: Out of all the materials brass has the highest
molten metal to the casting cavity. It is
coefficient of thermal expansion hence it
always vertical with straight tapered circular
requires highest shrinkage allowance to be
cross section.
provided on the pattern.
16. Ans: (c)
12. Ans: (d) Sol: To increase the hardness of green sand mold
Sol: In case of polystyrene patterns, the pattern it is necessary to apply large amount of
will not be removed from the mold, but the ramming force so that the porosity property
hot molten metal is directly poured so that of the mold will comes down which reduces
the molten metal will melt and evaporate the permeability of the mold.
the polystyrene pattern. In this way
NIIQl@lliiiillliRbnl.dn,ft�
I. �derabad l Delhi l BhopaJ ! Pune I Bhubaneswarl Lucknowl Patna l Bengaluru ! Chennai l Vijayawada j Vu.ag I Tirupati I Kuk;,.tpally l Kolkata
-·-·····�
: 605 : Casting

17. Ans: (b) 22. Ans: (d)


Sol: Misrun is the defect produced due to non­ Sol: Shake allowance 1s the only allowance
availability of molten metal for filling of which decreases the dimensions of the
farthest point from pounng point, this pattern hence called as negative allowance.
happened due to the solidification of molten
metal is started before complete filling of 23. Ans: (c)
the molten metal into casting cavity . This Sol: Slush casting is the open casting process in
can be eliminated by either increasing the which the liquid molten metal is poured
pouring temperature of molten metal or by into the cavity, allow it to solidify for some
reducing pouring time. The pouring time is time, invert the mold so that the un­
reduced by increasing the fluidity of molten solidified molten metal will fall down and
metal. solidified metal retains in the mold. Now by
breaking the mold the casting can be taken
1 8. Ans: (d) out. This is generally used for producing
Sol: The expandable pattern like wax pattern is decorative parts like lamp shades.
used in the investment casting
24.
1 9. Ans: (d) Sol: Distortion or bending type of defect 1s
Sol: In die casting only low melting point produced in the casting
materials like lead, tin, zinc, cadmium upto
aluminum can be casted , out of the given 25. Ans: (b)
materials only tin has less melting point Sol: Investment casting is used for producing
than aluminum complex shapes of the components like
turbine blades.
20. Ans: (a)
Sol: Cold shut or lap is a defect produced due to 26. Ans: (b)
the discontinuity produced due to hindered Sol: By varying the force applied during
contraction that is when two liquid streams compaction stage of powder metallurgy
are not mixing which is due to the low process, the density of the product can be
temperature of the molten metal. varied.

2 1 . Ans: (d) 27. Ans: (c)


Sol: The molding sand which contains moisture Sol: The strength of the part obtained after the
is called green sand mold. compacting stage is called green strength.
!lffil1i!i§jjji4fjjjijmnjM\!iii!\4 yderabad I Delhi I Bhopal I Pune I Bhubaneswar I I.J.Jcknowl Patna! Bengaluru I Chennai I Vtiayawada I Vizag I Tuupati I Kukatpally I Kolkata
"'
"... ACE
�� ' -..."...E� Pnblirarioos : 606 :
=========================
Production
============
28. 36. Ans: (a)
Sol: Metals, without chill block. Sol: Riser is acting as a reservoir for supplying
molten metal to casting cavity for
29. Ans: (a) compensating liquid shrinkages during
Sol: Because of usage of converging passage solidification
only the aspiration effect will be eliminated
so that the defect produced due to aspiration 37. Ans: (d)
effect like blow holes can be eliminated. Sol: Chaplets are used as additional supports for
supporting the core to prevent movement of
30. Ans: (c) core due to buoyancy.
Sol: Both A and R true and R explains A.
As the fluidity of casting metal is more 38.
means viscosity is less and hence flow rate Sol: Wax, Intricate
increases which reduces the pouring time.
39. Ans: True
31. Ans: (c)
Sol: In general, m shell molding the phenolic 40. Ans: False
resm is used as mold material, but some
41. Ans: (d)
time ceramic slurry will be used as mold
Sol: Fluidity is the flow property of liquids but
material called as ceramic molding
for defining the flow property of solids we
use the word called as "flow ability", hence
32. Blow holes,
solid fluidity is not the property of molding
Sol: Because due to low permeability of molds
sand.
the air cannot escape from the mold and
hence blow holes will be form. 42. Ans: (a)
Sol: Generally wood flour is added to molding
33. Ans : A. Lower, B. Higher
sand as an additive to improve collapsibility
34. Ans: (c) and porosity property of molding sand
Sol: With increase m carbon content strength
43. Ans: (d)
and hardness of the material will increase.
Sol: The expandable pattern like wax pattern is
35. Ans: A. Wax pattern, B. intricate shapes used in the investment casting

!IJ11ijj§jj/§4ijjjjRflftiMi,@� yderabad I Delhi I Bhopal I Pune I Bhubaneswar I Lucknow I Patna I Bengalwu I Chennai I Vijayaw.ula I Vmg I Tirupati I Kukatpally I Kolkata
: 607 : Casting
ACE
:.�
...
"
., ,,.,.,...,,. PoNmn
ti, • • • •

44. Ans: (c) "t1 = solidification time for the 1st casting = 5 min
Sol: Cold shut is the discontinuity produced due Given V2 = 8V1 = a� . . ... say
a 3 = 8a 3 => a2 -
hindered contraction. This is due to
- 2a1
unmixing of two streams of molten metal
2 1

coming from two sides of cavity.

45. Ans: (a)


Sol: Due to centrifugal force density of part is
"t2 = 4 "t 1 = 4x5 = 20 min
highest at the outer surface.
5 1. Ans: (b)
46. Ans: (d) Sol: Hot tears - differential cooling
Sol: Hot chamber die casting used for producing (Different parts of a component is cooled at
the castings which has very low melting different rates)
point like lead, tin and zinc Shrinkage- reduction m volume or
dimensions
47. Ans: (c) Blow hole - due to low porosity property of
Sol: As the thermal diffusivity increases means molding sand
that thermal conductivity increases, so that Cold shut - due to improper mixing of two
rate of heat transfer increases and hence streams of molten metal
solidification time reduces
52. Ans: (c)
Sol: Hot tears - differential cooling rate
48. Ans: (a)
(different parts of a component are cooled at
Sol: In case of liquids with increase of temp the
different rates). Also if the range of
viscosity reduces means fluidity increases
solidification temperature is high or wide,
solidified metal will allow the conduction
49. Ans: (b)
heat transfer and liquid is allowing the
Sol: The expandable pattern like wax pattern is
convection, hence the cooling rates will
used in the investment casting
become different.
50. Ans: (b)
Sol: Let a 1 = side of cube
53. Ans :(a)
Sol: The basic purpose of using the chills is to
V 1 = Volume of 1st casting = a i produce directional solidification
�yderabad I Delhi I Bhopal I Punc I Bhubaneswar I Lucknow I Patna I Bcnga)uru I Chcnnai I Vijayawada I Vmg I Tirupati I Kukatpa)ly I Kolkata I
: 608 : Production

54. Ans: (a) 55. Ans: (b)


Two Marks Solutions
56. Ans: (c)
01. Ans: (c)
Sol: Rate of pouring of molten metal depends on
the flow rate of molten metal. This depends Sol: Solidification time = T oc M2

on the choke area and it is the minimum volume


Where M = modulus =
surf:acearea
area out of the cross sectional areas of
sprue, runner and ingate. a3 a
For cube M =- 2 =-
6a 6
57. Ans: (b) a = side of the cube.
Sol: In submerged arc welding, the arc 1s
completely submerged inside the granular
flux powder and forming as blanket.
:: =(::J =(:: =(¾)' =: J
58. Ans: (c)
02. Ans: A-4, B-3 , C-6, D-5
Sol: Solidification time is inversely proportional
Sol: Slush casting is mainly used for producing
to diffusivity and based on the values
decorative parts like lamp shades, X-mas
diffusivity is highest for copper, next is
trees etc, shell molding is used for
aluminium and then steel. Hence the
producing circular and symmetrical jobs
solidification time is lowest for copper, next
only(cylinder block), Dry sand molding is
aluminium and then steel.
for CI brake shoe and centrifugal casting is
used for producing large size hallow
59. Ans: (c)
cylindrical parts without using core.
Sol: When carbon dioxide is supplied to the dry
sand it is chemically reacting with sodium
03. Ans: A-2, B-6, C-5, D-3
silicate and produces silica gel called as
Sol: In the given question only 4 of the casting
paste like material and on drying it gives
processes will have match with the
high strength to the mold.
products. Always connecting rods are made
by forging operation, turbine blades are
made by investment casting, cast iron pipes
by centrifugal casting and carburetor body
is made by pressure die casting

\( I l 11!.!,IIH t 1 1 11!.!, P1d1lu tlll llh yderabad I Delhi I Bhopal I Pune I Bhubaneswar I Lucknow I Patna I Bengaluru I Chcnnai J Vtjayawada I Vu.ag
i I Tirupan. I Kukatpally I Kolkata
: 609: Casting

04. Ans: A-5, B-3, C- 1 , D-2 07. Ans : (d)


Sol: In sand casting because of chances of Sol: In casting always Gravity die casting is used
compression of molding sand parts may for producing simple parts like IC engine
warp, in plaster mold casting due to good piston made by aluminum alloy and
surface finish no post casting process like pressure die casting is used for producing
machining are required, shell molding is complex shapes like carburetor body made
used for circular and symmetrical shapes by aluminum alloy.
only and due to high thermal conductivity
of the mold cooling rate is high and hence 08. Ans: (b)
surface hardening treatment will takes place Sol: Length of sprue = 20 cm
which gives hard surface and soft interior. C.S area of Base of sprue = 1 cm2 = A3=Ac
Volume of casting = 1 000 CC
05. Ans: (b) . . Volume
T1me require
. d fior fill
1 mg =
Sol: Solidification time = r =K M 2
Ac x V
Where K = solidification factor 1 000 1 000
= -�=
M = modulus 1 X �2gh 1 x .J2 X 981 X 20
_± 1tR 3 = 5.05 sec
Volume 3
For sphere M = = --
surface area 41tR 2
=R D 09. Ans: (b)
M =
3 6 Volume
Sol: t A = ----
A c X vmax
Volume
r - 0.97 x l 06 x (� )'
t a = ----
A c x Vavg
(0 2)
= 0.97 x l 0 x -t = 1 078 sec.
6
2

06. Ans: (d)


Volume of pattern
Sol:
Volume of casting
(1 .01 x 200X1 .01 x 1 00X1 .01 x 1 0)
= = 1 .o3
200 x l OO x l O
Note: Solid shrinkage 1s dimensional
shrinkage where as liquid shrinkage is Top G.S(A) Bottom G.S(B)
volumetric shrinkage.
jltlliii§jjjqg.jj/Q0WM\n.jj� yderabad J Delhi J Bhopal J Pune J BhubaneswarJ LucknowJ Patna J Bengaluru J Chennai J Vijayawada J Vizag IT=pati J Kukatpally J Kolkata
�..... : 610 : Production
" ,�.... ACE
�E�PuNiravn
===================================
V
t A -- (Vmax + vmin ) / 2 6.5 X 10 5 =
- = avg = ------ V2 --- 1 000 mm 2 / Sec
t a Vmax Vmax 650
=

Vmax/ 2 1 �2gh = .J2 x l 04 x h


= --- =-
=

Vmax 2 h= 50 mm = height of molten metal


t B = 2 tA in the pouring basin
ht = total height of molten metal above the
10. Ans: (b) bottom of the sprue
Sol: Steels with carbon percentage less than = 200 + 50mm

0.83% are called hypoeutectoid steels and Q = A 2 V2 = A 3 V3


above 0.8% is called hyper eutectoid steels.
6.5 x 105 = A 3 .J2 x l 04 x 250

1 1. Ans: (d) � A3 = 290. 7 mm2

/A (A 13. Ans: (b)


= ( (v . tube J = ( . )Sp J
2

Sol:
2

(V / A . ).P Sol: Riser is compensating the liquid shrinkages


't c ube

(A . tube
taking place during solidification like liquid
tsp

- ( 4;,:')' -( 4: )t)' phase and liquid to solid phase


i.e, 3% + 4% =7%

12. Ans: (c) 14. Ans: (d)


Sol: h = height of sprue = 200mm Sol: Chills are used for getting directional
A2 = 650 mm2 solidification, chaplets are used for
Q = flow rate = 6.5 x 10 5 mm 3 I s supporting the core to avoid the movement
g = 104 mm/sec2 of core due to buoyancy, riser is acting as a
reservoir to supply molten metal to cavity
for compensating liquid shrinkages taking
place during solidification and pads are used
for avoiding sand erosion and for
controlling cooling of critical sections

15. Ans: (a)


Sol: The given ratio is the gating ratio and it
means that the ratio of C.S areas of bottom
of sprue, runner and ingate
\( I, l.rn,11rn c11111-:" P11!1ht ,If]()!}',. Fyderabad l Delhi I Bhopal I Pune IBhubaneswarl LucknowI Patna! Benga)urul Oiennai IVuayawadajVizag ITtrupati I Kukatpally I K.olkata I
: 61 1 : Casting

16. Ans: (a) shrinkage is the shrinkage taking place


h
= 2 � h = 2r during cooling of material from freezing
Sol: -
r
temp to the room temp in solid state.
·: h = d
consider it as a taken as side riser 19.
From the given condition Sol: hact = 30 cm
2 2
h with loss=0. 8hact = 0.8x30 cm
= 't cyl � ( ; ) = ( ; )
V= �2gh with loss = .J2 X 98 X 30 X 0. 8
'tsp
S sp S ey/

217 cm/sec

�(�I -(�I
=

20. Ans: (a)


� D sp = Dey! = 100 mm
Sol: Shrinkage allowance on the pattern
=L . a. �T
17. Ans: (a)
=L . a.(Tr - Tr ),
Sol: Side of cubical casting = 50mm
Vc = volume of cavity Where Tr = freezing temperature
= 50 50 50 = 125000 Tr = room temperature
From the above equation,shrinkage
X X

Volume of casting after solidification


shrinkage = 0. 96 x 125000 allowance depends on dimension of the
= 120000 mm3 part, coefficient of thermal expansion,

Volume of casting after solid shrinkage freezing temp and then room temp. hence
= 0. 94 x120000 the only parameter which is not influencing
=112800 mm3 = a3 is pouring temp

� a = (112800)°.3 33
21. Ans: A-5, B-3,C-2, D-4,
= 48. 32mm
Side of the cube after solidification and
22. Ans: A-3, B-5,C- 4, D-2,
shrinkage and contraction = 48. 32mm

23. Ans: (c)


18. Ans: (c)
Sol: Solidification time = 't oc (V/As )2
Sol: In casting process the solid shrinkage is
compensated by providing shrinkage
24. Ans: A-3, B-5, C-4, D-2,
allowance on the pattern. The solid

iltllijj§jjj4i@Q@Gt@O,jj� yderabad j Delhi j Bhopa! I Pune j Bhubaneswarj Lucknow l Patna j Bengaluru j Chennail Vijayawada l Vmg j Tirupati I Kukatpally j Kolkata
: 612 : Production

25 Ans: (a) 3 1 . Ans: (d)


Sol: Ve = 1 00 x l OOx 50 = 5 1 05 mm3
Sol: P ourmg time = ----
. . Volume x

A c x Vmax 26
Vsc = · x Ve = 1 3000mm 3
106 1 00
= ------;:::===== 2.67 mm3/sec
200 x .J2 x 98 1 0 x 1 75 Riser height = 80mm
Vr = 4 Vsc
26. Ans: (d)
2
7t D H = 4 X 1 3000
Sol: Permeab·1·
501 .28 4
1 1ty number = ---
PT
4 X 1 3000 X 4
501 .28 � D= = 28_ 7? mm
= --- = 50. 128 1t x H
lOx 1
According to sufficient condition
27. Ans: (b)
Sol: Gating system will be designed such that
the time taken for pouring the molten metal
into the cavity must be as minimum as
possible, it should avoid aspiration effect, it
should not allow the impurities present in :. Based on this Dr > 28.77
the molten metal etc.
Hence Dr = 57.52 cm
28. Ans: (b)
Sol: Length of casting plate with two risers with 32. Ans: (d)
end wall effect and without chills is Sol: Chaplets are used as additional supports for
L = 1 3 x t = 1 3 x 1 0 = 1 30mm supporting the core to prevent movement of

29. Ans: (c)


core due to buoyancy.

Sol: The required strength of green sand mould


33. Ans: (d)
is obtained by compaction, in shell
Sol: Hot chamber machine is used for die
moulding the phenolic resin is used as mold
material. casting, muller is used for mixing sand, die
electric baker is used for making the cores
30. Ans: (a) with bentonite as bonding material and sand
Sol: Gating ratio is the ratio of cross sectional blaster is used for cleaning.
area of bottom of sprue, runner and in gate.
!ltlli@hii4i0i4Rflftftifoijj� yderabad j Delhi j Bhopal j Pune j Bhubaneswar j Lucknow j Patna j Bengaluru j Chennai j Vtjayawada j Vu.ag j Tuupati I Kukatpally j Kolkata
"' .
��-���Pubticatiom
" . ACE . . : 613 : Casting
� � ==================================
34. Ans: (a) As2 =2(400x200+200x35+35x400)
= 202000 mm2
Sol: In Castings shape factor =
L + W - S.F
t 1 12 x 10 s
MI 34. 15 ( ·: M = V )
L = height = 30 cm 32800 A
= =

30 + 20 28 x 10 s
W =nx D = nx M2 13.86
2 =
202000
=

= n x 25 = 78.5
30 - 20
t= = 5 ,, =•,(�:]'
30 + 78.5 13·86
S.F = 2 1.7 = 20( ) 3.29min .
2

5 34.15
= =

37. Ans: (c)


35. Ans: (a)
Sol: Sand casting is used for producing large
Sol: Casting size = 30 x 20 x 5 cm
bells, centrifugal casting is used for
Riser, D = H
producing pulleys, investment casting is
V 30 x 20 x 5
( J used for turbine blades and die casting for
A s C = 2(30 X 20 + 20 X 5 + 5 X 30 )
producing IC engine pistons and carburetor
3000 =
1.76
1700 38. Ans: (c)
=

Sol: <ls = 10 mm, h = 250mm,


(;,J. =
� Casting size = 100 x l OOx 100 mm
Vmax = �2gh �2 x 98 10 x 250
(v / A s )R = ( %
=

Freezing ratio = J 2215mm / sec


(V / A s )c 3
l Yi7
=

Ac =- <l s - 10 2 = 78.54 mm 2
= ( D x 17 ) 8D
7t 2 = 7t

= 0.095 D :::::- 4 4
30 x 6 75 Flow rate = 173966 mm3 /sec = 1.7 x 105

39. Ans: (b)


36. Ans: (b)
Sol: V , = 400x200x 140=1 12x 105 mm3
. . Volume(in mm 3 )
Sol: Pourmg time = --------
flow rate (in mm 3 / sec)
V2 = 400x200x35 =28x 105mm3
As , =2 (400x200+200x 140+ 140 x400) 100 3 =
5.75 Sec.
= 328000 mm 2 1.7 10
---
5
X

!M11i@jjji41jjj4Rflftftijjjjjjli yderabad I Delhi I Bhopal I Pune I Bhubaneswar I Lucknow I Patna I Bengalwu \ Chennai I Vtiayawada I Vmg I Tuupati I Kukatpally I Kolkata

ACE : 614 : Production
�-�Ii'....:.....:.... Publicatiom
�.=;;;;;;;;;:;,.============================
�=�
� �=�
40. Ans: (c)
Sol: Net B. F = V.g ( p - d )

= n(0. 12)2 x 0. 18 x 9. 81x


(I l 300 - l 600)
4
= 193.6N
44. Ans: (b)
41. Ans: (d) Volume Volume
Sol: t 1 = --- , ti = ------
Sol: Size of cylinder = D = H A x �2gh A
x �2g x (4h)
Side of cube = L 2
Volume Volume
M=
Area A
x �2g x (4h)
2 1
(:: )
=
Volume
A x �2gh

45.Ans: 7.7

� (�] 2 =�2
Sol: Net Buoyancy force = V.g ( PAI - Pcore )

= 76
( ) = (; D L ) x g(p AI
2
- P cor J
't cube

= 7t 1 00 2 X 100 X 10-9 X 9.81(2600 - 1 600)


42. Ans: (d)
4
= 7. 7 N
Sol: Loss of head due to friction depends on
velocity which is based on the Darcy­ 46. Ans: (a)
weisbach equation in fluid mechanics. Sol: The given casting is a top riser, hence for
top riser the optimum relationship between
43. Ans: (c) diameter and height of the riser is H = D/2
D C = _i nR S , where De = diameter of or D = 2H.
3 3
Sol: 7t
4 3
cylinder and Rs = radius of sphere V= 1t D 2 H
4

De = R{Il = 1. 75 Rs H=
4V
-----------(1)
nD 2

l@•li!@jjj44ijj/QRffnftMjjjjjj� yderabad I Dellii I Bhopal I Pune I Bhubaneswar I wcknow I Patna I Benga)wu I Chennai I Vtjayawada I Vizag I Tnupati I Kukatpally I Kolkata
t . A:CE . .
���•:Lpw,qPnblicatims : 615 : Casting
' ===================================
Surface area of riser exposed for heat 48. Ans :(b)
Sol: Volume of riser 1 = Volume of riser 2 ,
transfer As= 1tDH + - D 2
i.e. V1 = V2
1t
4
4V 1t t2 = 4t1 => M ; = 4M �
A 8 = nD x --2 + - D 2
1tD 4

= 4V + 1t D 2
D 4
For minimum As

d(A s )
=0
dD
- 4V -
=> -- 2 + x 2D = 0 '·
D 4
1t

49. Ans: (b)

Sol: , oc (�) '


4 1t
=> - D = -2 x - x D 2 H
2 D 4 Casting 1 = 6 faces exposed to H.T
1t

D Casting 2 = 5 faces exposed to HT & 1 face


=> - = H
is insulated
D = 2H V , = V2
A 1 = 6x2
47. Ans: 3 A2 = 5x2
V D 6
Sol: M =-=-=-= 1 (·: D = H )
' As 6 6

Mc =
7 x 10 x 2 = 140 _ 6)
- (- _ 36
2

2(7 X l Q + l Q X 2 + 2 X 7 ) 208
( 5 25
- -

t, M, 1
�= M
J =( ) 50. Ans: 6.16
2 2

c 140 / 208 Sol: Vs = Ve


208 4 3
t , = 1.36( -) = 3.00 min 1tR = X
2

140 3
3

x = R vF = l .6 12 R

llfl•Qj@jjji48ii1Rflfij@hifjj+yderabad I Delhi I Bhopal I Pune I Bhubaneswar l LucknowI Patna I Bengaluru I Oiennai I Vtjayawada I Vmg j 'firupati I Kukatpally I Kolkata I
"·•·" ACE . .
� :F,wner,mcNi&rmcn
. . : 616 : Production

Acube = 6x2 = 6 x(2R)2 = 6x4R 24R2

J J
.'.S_ = ( (V / A) s A
2 2 2=

( c
(V / A) c As
'tc
(�J 't cyl = Vcyl A cube
2
=

[ ]
J 't cube Vcube A cyl
X

6(1.6 12) 2 x R 2
2 2

(
4nR 2 4nR 2
(
nR 3 --
24R 2
J
= =

= --x 2
8R 3 4nR 2

- (H - 05625
'ts = 1.54 x 4 = 6. 16 sec

Sl. Ans: 10.S to 10.7 S3. Ans: 18S to 19S


Sol: Side riser = D = H
Sol: Total height = ht = 300 mm , dr = 10mm
Casting size 25 x 15 x5
Flow rate = velocity of molten metal x cross
=

25 + 15 = 40
L + W = --
Shape factor = -- sectional area
t 5 5
From table corresponding to S.F = 8
vr = 0.5
Ve
Vr = 0.5 xVc = 0.5 x 1875 = 937.5 CC
n 2 =n 3 = 190.45 eels
DH D 937 .5
=
=
4 4
D = H = 10.6 cm 54. Ans: 50 (range 49.9 to 50.1)
Sol: m = 2 kg, Q = l O kW
52. Ans: 0.5625 Time taken for removing latent heat
't cy1indnca1 = ? = 20 - 10 = 10 sec
't cube
Sol:
. Latent heat
T1me = ----
Given that
H = R � Cylinder Latent heat = time x Q
x= side of cube = 2x H = 2R = D = 10 X 10 = 100 kJ
Vcy1 = nR2H = nR3
100
Vcube= x3 = D3 = 8R3 Latent heat/kg = - = 50 kJ/kg
2
Acyl = 2nRH +2nR2 = 2nR2 +2nR2
= 4nR2
\( I l 1 1g11H t l l l lC, P11!1l1t .il11 11 1 -. ydcrabad I Delhi I Bhopal I Punc I Bhubancswar I Lucknow I Pama I Bcngaluru I Chennai I Vijayawada I Vu.ag I Tirupati I Kukatpally I Kolkata
: 617: Casting

55. Ans: (a) = ---,---------------,-


100 X 200 X 500
Sol: According to modulus method 2(100 X 200 + 200 X 500 + 500 X 100)
MR = 1.2 Mc = 29.411

=
(:!_) 1 2(:!_)
As R • As C

Diameter of riser Height of riser for top (141.42) X 500


= = 30.974
2
=

riser, D = H 4 x 500 x 141.42 + 2 x (141.4)


2

� D = 6 Mc � Mr
2

=( J
x ( 200} x lOO
2 'tQ MQ
4
7t
D = 6 x -- ______
...,:__
29.411
2
2 x 7t x 2002 + 1t x 200 x100 =( ) = 0.9016
4 30.974
6 x 200 x 100
D=H= = 150mm
400 + 400 58. Ans: (0.81 mm)
Sol: Given, / = 300 mm
56. Ans: 1.0927 (range 1.08 to 1.10) O'max= 270 MPa
Sol: Ratio of volume of pattern to casting E = 100 GPa
1.033 x100 x 50 x 10
= = 1.0927 /if,
= E£
l OO x 5 0 x 10
(J' max

Volume of cavity is assumed to be small and


270 = l OO x 103 x /if,
negligible. 300
M = 810x10-3 m = 0.81 mm
57. Ans: (0.9016)
Sol: Lp = LQ= 500, 59. Ans: 2.042
Block "P" is rectangular = 100x200 Sol: ·_ ________________(J) ___________________
- -- - - - - - - - - - - - - - - - - - - - -
----------------------------------------------------------------
Block "Q" is square
(Ac)p = (Ac)Q
------·=:=:=:��== �- - - ---
---------
100 X 200 = X 2

X = .J100 X 200 = 141.42mm h2 = 200 mm


M = :i_
r As
.-....(3)-
1111•i@h@Uijjji RflftiM\Uih*;Hydcrabad I Delhi I Bhopal I Punc I Bhubaneswar I Lucknow I Patna I Bcngaluru I Chcnnai I Vijayawada I Vizag I Tirupati I Kukatpally I Kolkata I
., ,, . ACE
. . . : 618 : Production
.,.
� .. .....�.."�Ptffrai.q
d2 = 20 mm,
h2 = 200 mm,
V2 = 0.5 mis
g = 9.8 m/s2
v2 = �2gh 1
52
h 1 = v; = (o . ) x 1000
2g 2 x 9.8
= 12.755 mm
Velocity of molten metal at top of sprue
ht = h2 + h1 = 200 + 12.755 = 212.755
V2 = �2gh = 1085 mm/sec.
V3 = �2gh 1 = .J2 x 9.81x 1000 x 212.755
= 2042 mm/sec = 2.042 m/sec
To avoid aspiration effect
A 3 �h , - h 2 240 - 180 =
60. Ans: (d) R= = = 0.5
Sol: Given that ht = 1500 + 100 = 1600mm, A2 ht 240
h2 = 1500mm
- d32 d2
7t

To avoid aspiration effect 4- = -t = 0.5


R =-
A1 / A2 = [(ht- h2 )/ ht ] "5 d � d2
° 7t

= [(1600- 1500) I 1600] 05 = (d /d )2


4
1 2

(d1/d2) 1/2
= d 3 = �0.5 x 20 2 = 14.14 mm.

02.
Five Marks Solutions Sol: Weight of cope = volume of cope x density
= 03x0.2x0.05 x 16

01. = 48 N
Sol: Height of sprue = 180 mm = hs Buoyancy force = weight of liquid displaced
Diameter of sprue on top = 20 mm = d2 = Vol. of casting x Density
Height of molten metal in the pouring basin
= 60 mm = hb
= 0.15 xO. l x0.05x78
.·. Total height of molten metal
= 58.5
= 180 + 60 = 240mm = ht Net B.F. = (58.5 - 48) = 10.5 N
Additional Weight = 10.5 N

yderabad l Delhil Bhopal l Pune l Bhubaneswari Lucknowl Patna 1 Bengaiuru l Chennai l V1iayawada l V=g ITuupali I Kukatpal)yl Kolkala
: 619: Casting

03. According to necessary condition


Sol: For Riser 1, Vr � 3Vsc
Vr = 3 x Vsc
M , = Modulus = ( :',]
1t D 2 H = 180 CC
4
7t 30 X 60
2

= - 4 -----
- ---'- � 7t D 3 = 180 � D = 6.2cm
4
-
7t 30 X 2 + X 30 X 60
2
7t
As per sufficient condition
13500
= =6
2250 'tr � 't c

For Riser 2, Mr � Mc
D a
-�-�D�a
M 2 = Modulus = ( :: ) 6 6
� 6. 2 � I O. It is not satisfied.
60 2
30
27000
7t
4
X
= = = 7.5 Hence by using sufficient condition
7t 60 2 x 2 + 1t x 30 x 60 3600
4 rr = rc � Mr = Mc � D = a = l O cm
Ratio ofSolidification times

� = (�) = �) = 0. 64 05.
2 2

(
t2 M2 7.5 Sol: The dimension of pouring basin will not
affect the pouring time
04. L et V= maximum velocity of molten metal
Sol: Casting size = 10 x 10 x 10 cm cube in the gating system,
For Riser D = H d = dmin = dia. Sprue bottom
Volumetric % shrinkage = 6% volume.of casting
Pourmg time = -------=-
. .
A c X vmax

Shrinkage volume of casting 1253


25 = ---
7t d x V
= __i_ x Vol.of casting
2

100 4
1253
= __i_x l O x l O x l O � V= --- = 99472/ d 2 • • • • • • • • • • • • • • • (1)
1 00 7t d X 25
2

= 60 cc = Vsc
4

!IJIIMl@hiiiihlMfmniM\jjjjj�)Hyderabad I Delhi I Bhopal I Pune I Bhubaneswar I Lucknow I Patna I Bengaluru I Chennai I Vijayawada I Vu.ag I TllUpati I Kuk.atpally I Kolkala. I
: 620 : Production

To ensure the laminar flow in the gating hb = Height of molten metal in the pouring
system Re s 2000 basin from top of sprue
At limiting case Re = 2000 ht = h2+hb = 8.24+20=28.24 cm
A3 � 8.24
V�
pVd R= = =� = 0.54
R = 2000 =
A2 28.29
2000 X 0.9 1 800
� V = --- = -........................ (2)
d d
ct[ = 0.54
d2
from (1) & (2)
d 3 = d 2 .J0.54 = d 2 X 0.734
99472 1 800 99472
2
= �d= = 55_3 mm = 2.87 x 0.734 = 2 . 1 1 cm
d d 1 800

08.
06. Sol: Volume of cube = Volume of slab
Sol: Asp = Acube
a3 = l x 2 x 4 = 8
1/8 = 2 = side of cube
4nR2 = 6a 2

a=
�R = a (6/4n)°.s = 0.69.a

( )
a3 a 2 1

= = =
,:: (:ru: r [n � '
Mcube = -2 = - = - = -
6a 6 6 3
V l x 2x4 8 2

( 2 ( 2 x o.:9 x a
Mstab == - = ------- = - = -
A s 2 (1 x 2 + 2 x 4 + 4 x l) 28 7

= ;) = J
2

2
= (2 X 0.69 ) = 1 .9 1

07.

Sol: Larger area of sprue = 6.45 cm 2 = 7t d;


4
09.
Sol: The skin thickness equation in casting 1s
d2 = 2.87, h2 = 20 cm ;
Qat top of sprue = 820 cm3/sec
t= c1 J; + C2
= A top x V2
from the given data
820
V2 = -- = 1 27. 1 3 cm / sec = v�
2gh b 3 = C1 .Jw + c2
6.45
127. 1 3 )2 4.5 = C1 .Jso + c2
hb = ( = 8.24 cm
2 x 98 1 1 .5 = C, x 2.599
!lflliiiYhiiiiihiRflbiiihiii� yderabad I Delhi I Bhopal I Pune I Bhubaneswar I Lucknow I Patna I Bengaluru I Chennai I Vuayawacla I Vizag I Tirupati I Kukatpally I Kolkata
�-� ACE
E�PiHcmoos : 621 : Casting
- � ==================================
c, = 0.58 = 1 00mm
C2 = 3 - 0.58+.../20 = 0.406 Gating ratio = 1 :4:2
If r3 = 1 00 sec As = 500 mm2 = min.C.S area =Ac
h1 = 200 + 1 00 = 300 mm
f3 = 0.58 X MQ + 0.406
= �2gh t = �2x98 1 0 x 300
= 6.206 mm
= 2426 mm 2 I s
1 0. Vol 3750000
ourmg time = --- = ----
P . .
Sol: Ve = 40 x 30 x 0.3 = 360cc A c x V 500 x 2426
Vsc = shrinkage volume = 3.09 sec
= � x 360 = 1 0.8 cc
1 00 1 2. Ans: (a)
Yr = Jr 4 2 x 4 = 50. 24 cc Sol: Circular disc casting Squared disc casting
4
Yr 2: 3 Ysc � vr � 3 x 1 0.8 = 32.4cc d = 20cm a = 20cm
Yr 2: 3 Ysc - necessary condition satisfied t = l Ocm t = l Ocm
360
Mc= .:!.__ -
As 2(40 x 30 + 30 x 0.3 + 0.3x 40)
= 0. 1 47
Freezing ratio (F.R) = X 1 = 1�:, = I .4
D
Mr = - = 0.67

Jtl
6

A
� ( s)
- sufficient condition satisfied V R 1 .4
Hence diameter of riser = 4 cm
L
X , = (t = ( tL = l .4
11.
Sol: Volume of casting = Vc
= 1 00 X 1 50 X250
(� l (tl,
1 .4
= 3750000

Cavity is kept in drag and is top gated


Height of cope = 200 mm
Height of molten metal in pouring basin

jai4l4jj§jj@4ihi4jmftlM\hih+1yderabad I Delhi I Bhopal I Pune I Bhubaneswarl Lucknow I Patna I Bcngalwu I Chennai I Vijayawada I Vu.ag I Trrupati I Kukatpally I Kolkata I
: 622 : Production

VR Ac = Amin = sprue base area


Volumetric ratio,(V.R) = Y 1 = = 0.8
Ve 400
=
= 200

Gating ratio = 1 :1. 5:2


Vol
Pouring time = ----
A c x Vmax

o.8( 1 x 20 2 x 10 ) = -----;======
353429 1
2oo x .J2 x9810x 250
20 x 20 x 1 0
1767 1
= 0.628 = = 8 sec
.J2 x 9810 x 250

1 3.
1 5.
Sol: Volume ofCasting l =V1 = 200 x l OOx70
Sol: i) A & ii) B,
14x 105 mm3
3 castings of spherical, cylindrical and
=

Volume ofCasting 2 = V2 = 200 x l OO x10


cubical
= 2x 105 mm3
Vsp = Vcube
As t = 2(200 x100 + 1 OOx70 + 70x200)
= 82 000 mm2 _± JZ"R 3 = a 3
3
As2 = 2(200 x100 + l OOx 1 0 + 1 0x200)
=
46000 mm2 a =
R � = 1 . 61 R
1 4 x 1 05
Module, M 1 = - = --- = 17. 07
vl
Vcyl = Vsp
A81 82000
D2H = ffR3
ff 4
V2 2x 10 s
M2 = = = 4. 348
4 3

= 4 ffR3
A82 46000 ff
D3
2
't 2 = 't 1 ( M 2 J = 1 0( 4. 348 ) = 0.65min .
2 4 3

M1 17.07 D = v16 R 3 = R/f = 1. 75R


3

14.
Sol: Volume of casting

= x150 x 200 = 3534291


2
:: {�:J-[�J- (�)'
2R
7t 2 2
4 = ( ) = (� ) = 1 . 54
ht =
200+ 50 = 250 mm a 1 . 6 1R

jltlli@hiiiihiRbftftWjj.jj� yderabad I Delhi I Bhopal I Pune IBhubaneswarl Lucknow I Patna I Bengaluru I Chennai IVuayawada I Vizag I Tuupati I Kukatpally I Kolkata
: 623 : Casting

D
75 g = - (21t N)2
2
41t 2
75 x98 10 = N2 D x -
2
N 2 D = 75 x 982 10 = 37273
2 1t
= 1.306.
N 2 D = 37273
0 5 + 0 · 52
16. D = · = 0.5 1 m = 5 10 mm
2
Sol: In centrifugal casting
Centrifugal force = Fe = ma = m r ro2 N = �37273 = �37273 = 8 _55 rp 8
D 5 10
a = rro2

Fyderabad Delhi Bhopal Punc Bhubaneswar i Lucknow Palm Bcngaluru Chennai Vuayawada ! V,zag j Tirupati
I I I I I I I I I Kukatpal)y I KolkataI
c2 Welding
04. For resistance spot welding of 1.5mm thick
One Mark Questions steel sheets, the current required is of the
order (GATE-ME-92)
01. Penetration is increased by (a) l OAmp (b) l OOAmp
(GATE-ME-90) (c) l OOOAmp (d) 10,000Amp
(a) Increasing welding current and welding
speed 05. In an explosive welding process, the . . . . . . . . . .
(b) Increasing welding current and (maximum/minimum) velocity of impact is
decreasing welding speed fixed by the velocity of sound in the . . . . . . . . .
(c) Decreasing welding current and welding (flyer/ target) plate material. (GATE-ME-92)
speed
06. In DC welding, the straight polarity
(d) Decreasing welding current and
(electrode negative) results in
increasing welding speed
(GATE-ME-93)
02. At small variations of arc length at operating (a) Lower penetration
conditions, the manual metal arc welding (b) Lower deposition rate
transformer provides nearly (c) Less heating of work piece
(GATE-ME-90) (d) Smaller weld pool
(a) Constant power
07. The electrodes used m arc welding are
(b) Constant power factor
coated. This is not expected to
(c) Constant voltage
(GATE-ME-94)
(d) constant current
(a) Provide protective atmosphere to weld
03. For gas welding a particular job using a (b) Stabilize the arc
neutral oxy-acetylene flame the acetylene (c) Add alloying elements
consumption was 1 0 ltrs. The oxygen (d) Prevent electrode from contamination
consumption from the cylinder in liters will
08. The ratio of acetylene to oxygen 1s
be (GATE-ME-91)
approximately . . . . . . . . . for neutral flame used
(a) 5 (b) 1 0 (c) 1 5 (d) 20
in gas welding (GATE-ME-94)

!IDlii@ih§§lh401etMiiHi� yderabad l Delhi I Bhopal I Punc I Bhubaneswarl Lucknow I Patna I Bengaluru I Chennai !Vuayawada !Vi.zag ITirupati I Kukatpally l Kolkata
ACE
: : Welding 625
��-�Z'li'....:..-:.... PnNicatiom
� �:�
���
�============���==============���
09. Generally cylindrical parts produced by 1 3. Which of the following arc welding
powder metallurgy should not have non­ processes does not use consumable electrode
uniform cross section and length to diameter (GATE-ME-02)
ratio exceeding . . . . . . . . . . . (GATE-ME-94) (a) GMAW (b) GTAW
(c) SAW (d) none
1 0. Preheating before welding is done to
(GATE-ME-96) 1 4. In oxyacetylene gas welding, temperature at
(a) Make the steel softer the inner cone of the flame is around
(b) Bum away oil, grease etc from the plate (GATE-ME-03)
surfaces (a) 3500 C
°
(b) 3200 C
°

(c) Prevent cold cracks (c) 2900 C


°
(d) 2550°C
(d) Prevent plate distortion
1 5. The strength of a brazed joint
1 1 . Two plates of the same metal having equal
(GATE-ME- OS)
thickness are to be butt welded with electric
(a) Decreases with increase in gap between
arc. When the plate thickness changes,
the two joining surfaces
welding is achieved by
(b) Increases with increase in gap between
(GATE-ME-01)
the two joining surfaces
(a) Adjusting the current
(c) Decreases up to certain gap between the
(b) Adjusting the duration of the current
two joining surfaces beyond which it
(c) Changing the electrode size
increases
(d) Changing the electrode coating
(d) Increases up to certain gap between the
two joining surfaces beyond which it
1 2. The temp of a carburizing flame m gas
decreases
welding . . . . . . . . . . . . . . . . . . that of a neutral or
an oxidizing flame
1 6. Which of the following 1s a solid state
(GATE-ME-02)
welding process. (GATE-ME-07)
(a) Lower than
(a) GTAW
(b) Higher than
(b) Resistance spot welding
(c) Equal to
(c) Friction welding
(d) Unrelated to
(d) SAW

!IJ11M@jj/4ijjjQAflfifMlhijj+yderabad I Delhi I Bhopal I Pune I Bhubancswar I LucknowI Patna I BengaluruI Chennai I Vtjayawada IV17.3g I Tirupari I Kukatpa]]y I Kolkata j
: 626: Production

1 7. Which one among the following welding 2 1 . In welding brass with oxy-acetylene flame ,
processes uses non-consumable electrode? the type of flame used is (GATE-PI-89)
(GATE-ME-11)
(a) Gas metal arc welding 22. The mode of metal transfer in CO2-MIG
(b) Submerged arc welding welding at low temperature is
(c) Gas tungsten arc welding (GATE-PI-90)
(a) Spray transfer
(d) Flux coated arc welding
(b) Short circuiting type of transfer
1 8. The major difficulty during welding of (c) Globular transfer
aluminium is due to its (d) Droplet transfer
(GATE-ME-14-SET-l)
(a) high tendency of oxidation 23. With increasing joint thickness, the tensile
(b) high thermal conductivity strength of a brazed joint (GATE-PI-90)
(c) low melting point (a) Continuously decreases
(d) low density (b) First decreases and then increases
(c) Continuously increases
1 9. In solid-state welding, the contamination (d) First increases and then decreases
layers between the surfaces to be welded are
removed by (GATE-ME-14-SET-l) 24. Welding Process
(a) alcohol (b) plastic deformation A. Thermit welding
(c) water jet (d) sand blasting B. Projection welding
C. MIG welding
20. Within the Heat Affected Zone (HAZ) in a D. Friction welding
fusion welding process, the work material Heat Source
undergoes (GATE-ME- 14-SET-4) 1 . Electric arc
(a) microstructural changes but does not 2. Mechanical work
melt 3. Exothermic chemical reaction
(b) neither melting nor microstructural 4. Ohmic resistance (GATE-PI -90)
changes
( c) both melting and microstructural 25. In TIG welding a . . . . A. . . . and . . . .B . . . .
changes after solidification electrode is used . (GATE-PI- 90)
(d) melting and retains the original A. non-cons umable I cons umable
microstructure after solidification B. coated I bare

!li11ib§h@4ijj4Afini4mil!+Yderabad I Delhi I Bhopal I Pune I Bhubaneswar I Luclmow I Patna IBenga)uruI Chennai jVtjayawadaJ Vizag I Tirupati I Kukatpa)Jy I Kolkata I
"•t.. �ACE Welding
�� �==================================
Pub1icationa : 627 :

26. High alloy steel components are preheated (a) cellulose (b) acidic
before welding for reducing (c) rutile (d) oxide
(GATE-PI-91)
(a) Heat affected zone 32. Filler material 1s . . . . A. . . in resistance
(b) Total energy consumption welding and the heat generated in the process
(c) Total time of welding is directly proportional to . . . . . . B . . . . . . .
(d) Welding stresses (GATE-PI-92)
a. used / not used
27. In spot welding of aluminum sheets, the b. square of the current / cube of the current
welding current should be . . . . . . A . . . . and weld
time should be kept . . . . . . . . . B . . . . . . than for 33. The composition of an oxy-acetylene flame
MS sheet of same thickness. for cutting of steel contain more oxygen.
(GATE-PI-91) (GATE-PI-94)
a. higher I lower b. higher I lower
34. The consumption of an oxy-acetylene flame
28. Continuous rails for Indian railways are for cutting of steel contains more oxygen
welded by . . . . . .. Welding process. (T/F) (GATE-PI-95)
(GATE-PI-91)
35. Which of the following powders should be
29. . . . . . . . . . . . . . . . . . . is the non-fusion method of fed for effective oxy-fuel cutting of stainless
joining dissimilar metals with metals whose steel (GATE-PI-01)
melting temp is greater than 400 C. °
(a)Steel (b) Aluminum
(GATE-PI-91) (c)Copper (d)Ceramic

30. The joint configuration best suited for 36. High speed electron beam welding is focused
adhesive bonding is on the weld spot using
(GATE-PI-92) (GATE-Pl-03)
(a) butt (b) fillet (c) lap (d) spot (a) vacuum lens
(b) inert gas lens
31. The type of coated electrode most widely (c) optical lens
used for welding low carbon steel is (d) magnetic lens
(GATE-PI-92)

!IJIIMO§hlii@i@Gi@u.jj+yderabadl Delhi I Bhopal I Punc I Bhubaneswarl Lucknow! Patna I Bengalwu I Chennai I Vliayawadal Vmg I Tirupari I �YI Kolkara I
:.t ACE :.
i· .r;&gmea
· �:011:•�========;.,;:6�2:8�:=============Pr �·�
�o�d�u�ct:I �
� . " =��
=ring
�Pu ��bli�d on
-:,

37. In resistance welding, heat is generated due 42. Brazing and soldering are (GATE-PI-14)
to the resistance between (GATE-PI-03) (a) plastic joining methods
(a) electrode and work piece (b) homogeneous joining methods
(b) asperities between touching plates (c) autogenous joining methods
(c) two dissimilar metals being in contact (d) heterogeneous joining methods
(d) inter-atomic force s
43. In a linear arc welding process, the heat input
38. The current in Amperes used in resistance per unit length is inversely proportional to
spot welding of plain carbon steel sheets (1 (GATE - 15-Set 1)
to 3 mm thick) lies within the range: (a) welding current
(GATE-PI-04) (b) welding voltage
(a) 10-50 (b) 50-500 (c) welding speed
(c) 500-5000 (d) 5000-50000 (d) duty cycle of the power source

44. Which two of the following joining processes


39. The metal powder used in Thermit welding
are autogeneous?
of steel is: (GATE-PI-04)
(i) Diffusion welding
(a) Al (b)Cu (c) Pb (d) W
(ii) Electro slag welding
40. Which of the following welding process (iii) Tungsten inert gas welding
results in the smallest heat affected zone (iv) Friction welding
(GATE-Pl-11) (GATE-15 - Set 3)
(a)Shielded metal arc welding (a) (i) and (iv) (b) (ii) and (iii)
(b) Gas welding (c) (ii) and (iv) (d) (i) and (iii)
(c) Laser beam welding
45. Under optimal conditons of the process the
(d) Thermit welding
temperatures experienced by a copper work
41. In resistance seam welding, the electrode is piece in fusion welding, brazing and
in the form of a (GATE-Pl-11) soldering are such that
(a) cylinder (GATE- 16- SET- l)
(b) flat plate
(a) Twelding > Tsoldering > Tbrazing

(c) coil of wire


(b) Tsoldering > Twelding > Tbrazing

(d) circular disc


( C) Tbrazing >
Twelding> Tsoldering

(d) Twelding > Tbrazing > Tsoldering

!ltlli@hi§§ih4RflbiM\hih� yderabad I Delhi I Bhopal I Pune I Bhubaneswarl Lucknow I Patna I Benga)uru l Chennai I Vijayawada ! Vmg ITirupati I Kukatpallyl Kolkata
: 629 : Welding

46. In an arc welding process, welding speed is 02. List - I


doubled. Assuming all other process A. Welding of aluminum alloy
parameters to be constant, the cross B. Ship building
sectional area of the weld bead will C. Joining of HSS drill bit to shank
(GATE- 17- SET- 1) D. Deep penetration precision welds
(a) increase by 20 % (b) increase by 50 %
(c) reduce by 25 % (d) reduce by 50 % List - II
1. Submerged arc welding
47. In gas tungsten arc welding process, the 2. Electron beam welding
material coated on pure tungsten electrode 3. TIG welding
to enhance its current carrying capacity is 4. Friction welding
(GATE- PI- 17) 5. Gas welding (GATE-ME -96)
(a) Titanium (b) Manganese
(c) Radium (d) Thorium 03. Resistance spot welding is performed on two
plates of 1. 5 mm thickness with 6 mm
diameter electrode, using 15000A current for
Two Marks Questions a time duration of 0. 25sec. assuming the
interface resistance to be O. OOO l Q, the heat
01. Welding process generated to form the weld is
A. Themit welding (GATE-ME-01)
B. Seam welding (a) 5625 W-sec
C. MIG welding (b) 8437 W-sec
D. Friction welding (c) 22500 W-sec
(d)33750 W-sec
Heat source
1. Chemical reaction 04. Match the following (GATE-ME-03)
2. Exothermic chemical reaction Work material
3. Electric spark P. Aluminum
4. Ohmic resistance Q. Die steel
5. Mechanical work R. Copper wire
6. Electric arc (GATE-ME-91) S. Titanium sheet

l11i•i@jj@41jjjj@niffijn.jj� yderabad Delhi Bhopal Pune Bhubaneswarj Lucknow Patna! Bengalwu Chcnnai j Vtjayawada V,zag j Tuupati
I I I I I I I I Kukatpally Kolkata
I
Type of joining 07. In arc welding process, the voltage and
1. Submerged Arc Welding current are 25V and 300A respectively. The
2. Soldering arc heat transfer efficiency is 0.85 and
3. Thermit Welding welding speed is 8 mm/sec. The net heat
4. Atomic Hydrogen Welding input (in J/mm) is (GATE-ME-06)
5. Gas Tungsten Arc welding (a) 64 (b) 797 (c) 1103 (d) 79700
6. Laser Beam Welding
P Q R S p Q R S 08. A DC welding machine with a linear power
(a) 2 5 1 3 (b) 6 3 4 4 source characteristic provides open circuit
(C) 4 1 6 2 (d) 5 4 2 6 voltage of 80V and short circuit current of
800A. during welding with the machine, the
05. Two 1 mm thick steel sheets are to be spot
measured arc current is 500A corresponding
welded at a current of 5000 A. Assuming
to an arc length of 5.0 mm and the measured
effective resistance to be 200 micro-ohms
arc current 460A corresponding to an arc
and current flow time of 0.2 second, heat
length of 7.0 mm. the linear voltage (E) &
generated during the process will be
arc length (L) characteristic of the welding
(GATE-ME -04)
arc can be given as (where E is in Volts and
(a) 0.2 Joule (b) 1 Joule
L is in mm) (GATE-ME-07)
(c) 5 Joule (d) 1000 Joules
(a) E = 20 + 2L (b) E = 20 + 8L
06. Spot welding of two 1 mm thick sheets of (c) E = 80 + 2L (d) E = 80 + 8L
steel (density=8000kg/m3) is carried out
successfully by passing a certain amount of 09. Two metallic sheets , each of 2.0 mm
current for 0.1 second through the electrodes. thickness are welded m lap joint
The resultant weld nugget formed is 5 mm in configuration by resistance spot welding at a
diameter and 1.5 mm thick. If the latent heat welding current of 10 KA and welding time
of fusion of steel is 1400 kJ/kg and the of 10 milliseconds. A spherical fusion zone
effective resistance in the welding operation extending up to the full thickness of each
is 200µQ, the current passing through the sheet is formed. The properties of the
electrodes is approximately metallic sheets are given below:
(GATE-ME-OS) Ambient temp = 293 K
(a) 1480 A (b) 3300 A Melting point temp = 1793 K
(c) 4060 A (d) 9400 A Density = 7000 kg/m3

!ltlii@hidiOQRflni@dih� yderabad l Delhil Bhopal ! Punc l Bhubancswar i wcknow! Patna ! Bcngaluru j Chennai lVijayawada ! Vizag ! Tirupati I Kukaiwi)JyL Kolkata
: 631 : Welding

Latent heat of fusion = 300 kJ/kg 314th radius of the rotating bar . The power
Specific heat = 800 J/kgK (in kW ) consumed at the interface of the
Assume: welding is (ME-GATE-10)
(i) Contact resistance along sheet - sheet (a) 12.33 (b) 16.44 (c) 18.50 (d) 24.66
interface is 500 micro-ohm and along
electrode -sheet interface is zero.; 12. In welding a mild steel sheet using an oxy­
(ii) No conductive heat loss through the bulk acetylene flame, the total amount of
sheet material; and acetylene consumed was 8.0 ltrs. the oxygen
(iii) The complete weld fusion zone is at the consumption from the cylinder is
melting temperature, the melting (GATE-PI-89)
efficiency (in %) of the process is..... (a) 8.0 ltrs (b) 16 ltrs
(GATE-ME-07) (c) 20 ltrs (d) 24 ltrs
(a) 50.37 (b) 60.37
13. Match the following (GATE-PI-89)
(c) 70.37 (d) 80.37
List -1 List -2
A. Cellulose coating 1. Alloying agent
10. In arc welding of a butt joint, the welding
B. Rutile coating 2. Absorb gasses
speed is to be selected such that highest
C Iron powder coating 3. forming slag
cooling rate is achieved. Melting efficiency
D Basic coating 4. shielding gasses
and heat transfer efficiency are 0.5 and 0.7,
respectively. The area of the weld cross 14. In welding process, penetration is increased
section is 5 mm2 and the unit energy required by (GATE-PI-91)
to melt the metal is 10 J/mm3 . if the welding (a) increasing current and decreasing speed
power is 2 kW, the welding speed in mm/s is (b) decreasing both current and speed
closet to (GATE-ME-08) (c) increasing both arc voltage and speed
(a) 4 (b) 14 (c) 24 (d) 34 (d) increasing arc voltage and decreasing
current
1 1. Two steel bars each of diameter 10mm are
coaxially friction welded end to end at an 15. Application
axial pressure of 200MPa and rotational A. Highly reactive metals
speed of 400rpm. The coefficient of friction B. long straight butt joint in thick sections
between the mating faces of the rotating bars C. site weld of rails
is 0.50. the torque is assumed to act at the D. MS rods to plate like components

Fyderabad l Dellii l BhopaI I Pune l Bhubancswar l Lucknowl Patna l Bengaiuru l Oiennai l Vuayawadal Vizag I T,rupati I Knkalpally l Kolbtll
�-t ACE
� PoolicalnJa : 632: Production
�� ��==========================================

Welding Process V and welding speed of 300 mm/min. if the


1 . Ultrasonic process efficiency of is 0.8 and surface
2. Thermit resistance is 36 micro-ohm, the heat input
3 . Electron beam will be (GATE-PI-03)
4. Submerged arc (a) 600 J/mm (b) 480 J/min
5. Stud (GATE-PI-92) (c) 1 46 kJ (d) 1 1 6 kJ

1 6. During gas welding of copper sheets a 1 9. The cold cracking susceptibility of the heat
neutral flame was used consuming 8 liters of affected zone in an arc weld is influenced by
acetylene. But the weld was found to be (GATE-PI-03)
defective. So it was re-welded using an P. Entraped hydrogen,
oxidizing flame. The likely consumption of Q. Residual stresses
oxygen in liters will be (GATE-PI-93) R. Martemsitic transformation
(a) 6 (b) 8 (c) 1 0 (d) 1 6 S. Slag inclusion
(a) P, S (b) P, Q, S
1 7 . In spot pulsed laser welding of aluminum (c) P, Q, R (d) R, S
plates (density=2700kg/m3 , specific
heat=896J/kg, melting temp=933K, latent 20. Which of the following process can be used
heat of melting = 398 kJ/kg) at a temp of for welding of Aluminum alloys
30° C, pulse with energy of 0.5J is focused (GATE-PI-03)
onto an area of 0.05mm2 . if the entire energy P. Submerged arc welding
is coupled into the material, what will be the Q. Gas metal arc welding
depth of weld assuming the CS area of the R. Electroslag welding
weld is circular and is uniform throughout its S. Gas tungsten arc welding
depth and only heat conduction in the (a) P,Q (b) Q,S (c) Q,R (d) R,S
direction of penetration (GATE-PI-02)
(a) 5.34mm (b) 2. 1 5mm 2 1 . Group I Group II
(c) 4.23mm (d) 3.85mm P. Arc welding 1 . Diffusion
Q. Friction welding 2. Polarity
1 8. A 900 mm long steel plate is welded by R. Solid state welding 3. Focusing
manual metal arc welding process using S. Laser welding 4. Kinetic energy
welding current of 1 50 A, arc voltage of 20 (GATE-PI-03)

\( I I 1 1 .,1 1 11 l 1 1 1 1 :... P11ldu . 1 I H 1 1 1, Fydcrabad l Delhi l Bhopal j Pune j Bhubaneswar l Lucknowl Pa!na l Bengaluru l Cbennai l Vijayawada j Vizag ITuupati I Kukatpal)yj Kolkata I
. •." ....
�.,.,
ACE
"'"= Engiom
.
� PnNiariooa

p Q R s
: 633 : Welding

(a) 4 3 2 (c)
/
t 40(80)
40( 80)
b 6
6 <

-<
1
(b) 3 2 4 3
(c) 1 2 4 3
(d) �,.............:440
c..::....
0-� 8 0-+-
l'>-�...,...
- 80 [7 6
6
(d) 2 4 1 3

22. Match list-I (welding defects) with List-II 24. Two 8 mm thick steel plates are placed 5 mm
(causes) and select the correct answer using apart and welded by a butt joint. Welding is
the codes given below the lists carried out at 20 V and speed of 5 mm/sec.
(GATE-PI- 03) Heat transfer efficiency is 0.80. If the heat
List- I List- II
required to melt steel is 10 J/mm3 and
A. spatter 1. Damp electrodes
melting efficiency is 0.625. the weld current
B. Distortion 2. Arc blow
(in Amperes) will be (GATE-PI-05)
C. slag inclusion 3. Improper cleaning in
(a) 100 (b) 200 (c) 300 (d) 400
multi pass welding
D. Porosity 4. Poor joint selection
25. Match List - I (Welding problems) with List
(a) A-4, B-2, C-3, D-1 - II (Causes) and select the correct answer
(b) A-4, B-2, C-1, D-3 using the codes given below the lists:
(c) A-2, B-4, C-1, D-3 (GATE-PI- 05)
(d) A-2, B-4, C-3, D-1 List- I (Welding problems)
A. Cracking of weld metal
23. In a fabrication shop, a T-joint is to be made
B. Cracking of base metal
between two plates using a chain
C. Porosity
intermittent, double fillet weld of 6 mm leg
D. Inclusions
length. The intermittent welds are 40 mm
List- II (Causes)
long and spaced 80 mm apart, edge to edge.
1. Excessive stresses
The welding symbol of the joint is
2. High joint rigidity

6 b-,_ 40
(a) �--..:....
6 �V
...,..., (80
---+4o� <8o)--
)
-::-'.'
<
(GATE-PI-OS)
3.

4.
Failure to remove slag from previous
deposit
Oxidation

<
5. Excessive H2 , 02 , in the welding
(b) t6
�6
40- 80
[>40-80 atmosphere
N2 ,

Il!llih4Hiiiiill4 + I I I I I
d rabad Dellii Bhopal Pune Bhubancswar Lucknow I Patna ! Bcngaluru I Chcnnai I Vuayaw.ida j Vi7.ag I Tirupati I Kukatpally I Kolkata I
" .,.,"�
�"�-�
ACE .
. Puhliralooa . . : 634 : Production

Codes: 28. Assertion(A): A sound welded joint should


A B C D not only be strong enough but should also
(a) 2 1 5 3 exhibits a good amount of ductility
(b) 3 4 2 1 Reason(R): Welding process is used for
(c) 2 4 5 3 fabricating mild steel components only.
(d) 3 1 4 2 (GATE-PI-05)

26. Consider the following statements :


29. Assertion(A): In electron beam welding
(GATE-PI-05)
process, vacuum 1s an essential process
The size of the heat affected zone (HAZ) will
parameter.
increase with
Reason (R): Vacuum provides a highly
1 . increased starting temperature
efficient shield on weld zone.
2. increased welding speed
(GATE-PI-05)
3. increased thermal conductivity of the
base metal
30. The DC power source for arc welding has the
4. increase in base metal thickness
characteristic 3 V + I = 240, where V =
Which of the statements given above are
Voltage and I = current m amp. For
correct ?
maximum arc power at the electrode, voltage
(a) 1 , 2 and 3 (b) 1 and 3
should be set at (GATE-PI-07)
(c) 1 and 4 (d) 2 and 3
(a) 20V (b) 40V (c) 60V (d) 80V
27. Consider the following statements :
(GATE-PI-05) 3 1 . Two pipes of inner diameter 1 00mm and
The magnitude of residual stresses in outer diameter 1 10mm each are joined by
welding depends upon flash butt welding using 30V power supply.
1 . Metal melted / deposited At the interface , Imm of material melts from
2. Design of weldment each pipe which has a resistance of 42 Ohms.
3. Support and clamping of components if the unit melt energy is 64.4 MJ/m3 , then
4. Welding process used time required for welding ( in S) is
Which of the statements given above are (GATE-PI-08)
correct ? (a) 1 (b) 5 (c) 10 (d) 20
(a) 1, 2 and 4 (b) 1, 2 and 3
(c) 1 and 3 (d) 2 and 3

!M•i4ji§jjji4ijjjiAflbiiihih+yderabadl Delhi I Bhopal I Pune! Bhubaneswarl Lucknow I Patna! Bengaluru I Chennai j Vtjayawada j Vizag j T11Upati I Kukatpal)yj Kolkata I
: 635 : Welding

32. Aluminum strips of 2mm thickness are (a) 27 (b) 35 (c) 45 (d) 55
joined together by resistance spot welding
process by applying an electric current 36. During a steady gas metal arc welding with
6000A for 0. 1 5sec. the heat required for direct current electrode positive polarity, the
melting aluminum is 2. 9J/mm3 • the diameter welding current , voltage and weld speed are
and thickness of the weld nugget are found to 1 50A, 30V and 6rn/min respectively. A
be 5 mm and 2. 5mm respectively. Assuming metallic wire electrode of diameter 1 . 2mm is
the electrical resistance to be 75 micro-ohms, being fed at a constant rate of 1 2rn/min. the
the percentage of total energy utilized in density , specific heat and melting temp of
forming the weld nugget is (GATE-PI-08) the wire electrode are 7000 kg/m3 , 500 J/kg
(a) 28 (b) 35 (c) 65 (d) 72 and 1530°C respectively. Assume the
ambient temp to be 30 °C and neglect the
33. Which pair among the following solid state
latent heat of melting. Further consider that
welding processes uses heat from an external
2/3rd of the total electrical power is available
source (GATE-PI- 08)
for melting of the wire electrode. The
P- Diffusion welding
melting efficiency (in percentage) of the wire
Q- Friction welding
electrode is (GATE-PI-10)
R- Ultrasonic welding
(a) 39.58 (b) 45.25
S- Forge welding
(c) 49. 38 (d) 54.98
(a) P & R (b) R & S
(c) Q & S (d) p & s
37. In a DC arc welding operating, the length
34. Which of the following powders should be
characteristic was obtained as Varc = 20 +5/
fed for effective oxy-fuel cutting of stainless
where the arc length l was varied between
steel (GATE-PI-08)
5mm and 7mm. Here Varc denotes the arc
(a)Steel (b) Aluminum
voltage in Volts. The arc current was varied
(c)Copper (d)Ceramic
from 400A to 500A. Assuming linear power
35. Autogeneous gas tungsten arc welding of a source characteristic , the open circuit
steel plate is carried out with welding current voltage and the short circuit current for the
of 500A, voltage of 20V, and weld speed of welding operation are
20mrn/rnin.Consider the heat transfer (GATE-ME & PI-12)
efficiency from the arc to the weld pool as (a) 45V, 450A (b) 75V, 750A
90%. The heat input per unit length (in kJ I (c) 95V, 950A (d) 150V, 1 500A
mm ) is (GATE-PI-09)
jJtll@@hid@iRflflffi\ii!jj� yderabad I Delhi I Bhopal I Pune I Bhubaneswar I Lucknow! Patna I Benga)uru I Chennai I Vijayawada I Vizag I Tirupati I Kukaq,ally I Kolkata
: 636: Production

38. For spot welding of two steel sheets (base 41. A DC welding power source has a linear
metal) each of 3 mm thickness, welding voltage-current (V-I) characteristic with open
current of 10000 A is applied for 0. 2 s. The circuit voltage of 80V and a short circuit
heat dissipated to the base metal is 10001. current of 300A. For maximum arc power ,
Assuming that the heat required for melting the current (in Amperes) should be set
1mm volume of steel is 20 J and interfacial
3
as___ (GATE-15 - Set 1)
contact resistance between sheets is 0. 0002
Q, the volume (in mm 3 ) of weld nugget is 42. During a TIG welding process, the arc
(GATE-ME-14-SET-4) current and arc voltage were 50 A and 60 V,
respectively, when the welding speed was
39. A butt weld joint is developed on steel plates 150 mm/min. In another process, the TIG
having yield and ultimate tensile strength of welding is carried out at a welding speed of
500 MPa and 700 MPa, respectively. The 120rnrn/rnin at the same arc welding voltage
thickness of the plates is 8mm and width is and heat input to the material so that weld
20 mm. Improper selection of welding quality remains the same. The welding
parameters caused an undercut of 3mm depth current(in A) for this process is
along the weld. The maximum transverse (GATE-15-Set 2)
tensile load (in kN) carrying capacity of the (a) 40. 00 (b) 44. 7 2
developed weld joint is __ (c) 55. 90 (d) 62. 25
(GATE-ME-14-SET-3)
43. Two aluminum alloy plates each 10 mm
40. In an arc welding operation carried out with a thick and 1 m long are welded without
power source maintained at 40 volts and 400 crowning by multipass tungsten inert gas butt
amperes, the consumable electrode melts and welding. The joint configuration is V-type
just fills the gap between the metal plates to with 60 ° angle and root gap is maintained at
be butt-welded. The heat transfer efficiency
5 mm. if electrode of 5 mm diameter with
for the process is 0.8, melting efficiency is
500 mm length is used for welding, then the
0. 3 and the heat required to melt the
number of electrodes required is
electrode is 20 J/mm3 • If the travel speed of
(GATE - Pl - 15)
the electrode is 4 rnrn/s, the cross-sectional
(a)7 (b) 9 (c) 11 (d) 13
area, in mm2 , of the weld joint is ____
(GATE-PI- 14)

!M11i@jjji4.jji4Rflft1Mihih� yderabad I Delhi I Bhopal I Pune I Bhubaneswar l Lucknow! Patna I Bengalwu I Chennai I Vijayawada I Vizag I Tuupati I Kukatpally I Kolkata
: 637 : Welding

44. The voltage -length characteristic of a direct Two welding cases are given below.
current arc in an arc welding process is V = Case I: V= 15 V, I = 200 A, v = 5 mm/s,
(100 + 40/), where l is the length of the arc in k = 150 W/mK, p = 3000 kg/m
3
,

mm and V is arc voltage in volts. During a C = 900 J/kg-K


welding operation, the arc length varies Case II: V= 15 V, I = 300 A, v = 10 mm/s,
between 1 and 2 mm and the welding current k = 50 W/mK, p = 8000 kg/m3 ,
is in the range 200-250 A. Assuming a linear C = 450 J/kg-K
power source, the short circuit current is
where, V is welding voltage, I is welding
A. (GATE - 16 - SET- 2)
current, v is welding speed, and k, p and C

45. Spot welding of two steel sheets each 2 mm refer to the thermal conductivity, the density

thick carried out . successfully by passing 4 and the specific heat of the plate materials,

kA of current for 0. 2 seconds through the respectively. Consider that electrical energy

electrodes. The resulting weld nugget formed is completely converted to thermal energy.

between the sheets is 5 mm in diameter. All other conditions remain same.

Assuming cylindrical shape for the nugget, The ratio of the peak temperature in Case I to
the thickness of the nugget is ___ mm. that in Case II is (GATE - PI-16)
1 1
L atent heat of fusion for steel 1400 kJ/kg (a) - (b) (c) 1 (d) 2
3 2
Effective resistance of the 200 µ0
weld joint 47. Two flat steel sheets, each of 2.5 mm
Density of steel 8000 kg/m j
thickness, are being resistance spot welded
(GATE- 16- SET- 3) using a current of 6000 A and weld time of
0. 2 s. The contact resistance at the interface
46. In linear gas tungsten arc welding of two
between the two sheets is 200 µO and the
plates of the same material, the peak
specific energy to melt steel is 1Ox 109 J/m3 .
cq
temperature T (in K) is given as T = , , A spherical melt pool of diameter 4 mm is
formed at the interface due to the current
where q is the heat input per unit length (in
flow. Consider that electrical energy is
J/m) of weld, a, is the thermal diffusivity (in
completely converted to thermal energy. The
m2/s) of the plate materials and C 1 is a
ratio of the heat used for melting to the total
constant independent of process parameters
resistive heat generated is ___
and material types.
(GATE - PI-16)
!ltllib§hi4410.YRbbdW!lih� yderahad I Delhi I Bhopal I Pune I Bhubaneswar I Lucknow I Patna! Bengaluru I Chennai I Vuayawada I Vi7.ag I Tirupati I Kukatpally I Kolkata
.,.,, .......... ACE Production
�... . 'E� Pnblirarioos : 638 :
=====================================
linear V-I characteristics DC power source.
The following data is available.
Five Marks Questions
O.C voltage of the power source = 62V,
S.C current for the setting used = 130A,
01. An arc welding DC power source has a linear Arc length = 4mm,
power source characteristic with open circuit Traverse speed of welding = 15crn/min
voltage VO = 80 volts and short circuit Efficiency of heat input = 85%
current Is = 1OOOamps. The voltage-length The relation between the arc length L in mm
characteristic of the arc is given by V= 20 + and arc voltage Vis V= 20 + 1. 5L
4L volts where L is the arc length in mm, Calculate the heat input into the work piece.
calculate the optimum length of arc for (GATE-ME-92)
obtaining maximum arc power at welding. 04. The cross section weld bead is shown in fig
What voltage and current setting should be the profile of the bead and the fusion zone
done on the power source for maximum arc are taken circular for convenience. Bead
power. (GATE-ME-89) width and radii of curvature of circular
profiles are given in fig. The bead height and
02. Manual metal arc welding is to be used for depth of penetration are (GATE-ME-96)
welding two plates each measurmg 1 0mm
800x200x 19mm. Given that
I• •I

L ength of weld : 180mm


r=7mm
L ength of electrode : 450mm
r=20mm

Stub length : 50mm


Diameter of electrode : 3mm
Weld reinforcement area : 10%
Spatter and other losses:15%
Root gap = land = 2mm
Angle of groove : 60° 05. Welding of C40 steel plate of 10mm
Estimate the required number of electrodes thickness requires a current of 160amps,
for the weld. (GATE-ME-90) while it was 360amps when the plate
thickness was increased by 50%. Estimate
03. A low carbon steel is to be welded by the the welding current for 8mm thickness of
manual metal arc welding process using a same material. (GATE-ME-97)

jlflli@idi4@jQnjjjfih@� yderahad I Delhi I Bhopal I Pune I Bhubaneswarj Lucknow I Patna I Bengaluru I Chennai I Vijayawada j V11.3g I Tirupari I Kukatpal]y I Kolkata
: 639 : Welding

06. The voltage-arc length characteristics of a


DC arc is given by V=20+4L. where L is in
Q = 8 K Tc t [0.2 + : J
mm. During a welding operation arc length is Where K = thermal conductivity,
expected to vary between 4 and 6 mm with Tc = temp increases from room temp
the welding current limited between 450 and up to MP,
550 amps. (GATE-ME-98) t = thickness of plate,
(i) If the power source has a linear V = welding speed,
characteristics, the open circuit voltage b = width of weld,
lS
a = thermal diffusivity
Two alloy steel plates as shown in fig are to
(a) 36 V (b) 44 V (c) 72 V (d) 80 V
be welded using a power source rated at 5
kVA having a duty cycle of 75 %. Using the
(ii) The arc power at an arc length of 5mm is
given data, determine the maximum welding
(a) 1 0 kW (b) 1 4 kW
°
speed for the job given in mm/s
(c) 20 kW (d)80 kW Data : K = 45 W/ m° C, Tc = 1 450 C, and
a = 1 .2 x 1 0 - 5 m2 I s.
07. For butt welding 40mm thick steel plates, (a) 14.2 (b) 1 9.0 (c) 1 0.65 (d) none
when the expected quantity of such jobs is
5000per month over a period of 1 0 years, 09. Two different pairs of sheets of same
choose the best suitable welding process out material are welded by resistance-spot
of the following available alternatives. welding. In one pair, the average radius (r)
(GATE-ME- 99) of each spherical bridge is 0.2 mm and the
(a) SAW number of bridges per cm2 (n) is 25. In
(b) Oxy-acetylene gas welding another pair, the number of bridges per cm2
(c) EBM is 50 with the same 'r' of bridge. The contact
(d) MIG resistance R: per unit area is given by R: =
08. In butt welding operation on plates, the heat 0.85 (p / nm) where 'p' is the resistivity of
input necessary is given by metal. If the voltage applied is 5 volts and the
(GATE-ME-99) resistivity of metal is 2x 1 0-5 ohm-cm. The
rate of heat generated per cm2 in each case
(GATE-ME-00)

\( l l 11�1 11l � 11 11i.:, P 1 1hlu .,u,,11 ... �yderabad I Delhi IBhopal IPune IBhubaneswar I Lucknow I Patna I Bengaluru I Chcnnai I Vuayawada I Vi7.ag I TlrUpati I Kukatpally I Kolkala I
.,".

.
, , ��Pnlfumnna
.."" . ACE . . : 640 : Production

10. The arc - voltage characteristics of a DC i) What is the heat generated due to the
power source has a linear power source supply of power if effective resistance
characteristic of V = 20 + 40L , where V is has 200micro ohms,
the arc voltage in volts and L is the arc length ii) The maximum indentation is 1oyo of
in cm. the static volt-ampere characteristic of sheet thickness and density of the spot
the power source is approximated by a weld nugget 8 gm/cc. If 1380 Joules are
straight line with open circuit voltage = 80V required to melt 1gm of steel, determine
and short circuit current is 600Amps, heat required for welding operation if
determine the optimum arc length for D= 6SQRT (t) to determine nugget
maximum power (GATE-ME-03) diameter D and assume the nugget size
to be equal to metal between the two
11. A resistance welder is rated at 50 kVA, at electrodes
50% duty cycle. Secondary voltage is always (GATE-PI-92)
constant at 25 volts. If a job requires
1500Apms current, determine the maximum 14. In MIG welding the power source
welding time in a total cycle time of 30secs. characteristics is Vp=36-I/60 and the arc
(GATE-PI-91) characteristics is Va=2La +27. find the
change in power of the arc if the arc length
12. The voltage-arc length characteristics of a changed from 2mm to 4mm. If the maximum
DC arc is given by V=20+40L. where L = arc current capacity of the power source is 300
length m cm. The power source Amps, the maximum arc length sustainable is
characteristics can be approximated by a (GATE-PI-92)
straight line. Open circuit is 80V and short
circuit current 1OOOAmps. The optimum arc 15. The voltage-arc length characteristics of a
length in mm (GATE-PI-91) power source 1s V=20+40L, where
(a) 0.5 (b) 1 (c) 5 (d) 10 V=operating voltage in volts and L = arc
length in cm. The open circuit voltage and
13. Two sheets of low carbon steel 1. 5mm thick short circuit current for arc lengths ranging
each are spot welded by passing a current from 3 to 5mm and current from 400 to
10000 amps for 5 cycles in 50 Hz main 500amps during welding operation are
supply. (GATE-PI-92) (GATE-PI-93)

!11i1ih§jjji4.jjjiRflbiMJj.jjj� yderabadl Delhi I Bhopal I Pune I Bhubaneswar I Lucknow I Patna I Bengalwu I Chennai I VrjayawadalVizag I Tirupari I Kukatpallyl Kolkata
ACE
�Ii'....;......;... Publicationa : 641 : Welding
��-���

� �;;;;� �;;;;;;;�===========================�
16. Two plates with symmetrical V joint diameter: 1.2 mm; Wire feed rate:4m/min;
penetration of V angle 60deg are to be arc Thermal efficiency of the process : 65%
welded in a single pass. The power P(kW) - (GATE-PI-03)
arc length L (mm) characteristic is given by
19. The heat input per unit length of the weld in
P = 4+0.8L -0. 1L 2 Assuming that the top
kJ/cm is:
surface of the weld is flat & flush with the
(a) 0.18 (b) 0. 28 (c) 10. 83 (d) 16.66
plate top surface, & the energy losses are
20%, The density of filler material is 8gm/cc 20. The area of cross section of weld bead in
and the energy required to melt 1 gm is 1400 mm2 is:
joules. The plate dimensions are 1000mm (a) 16. 3 (b) 25. 1
length & 5mm thickness. The maximum (c) 30.3 (d) 38. 6
welding speed is . . . . . . (GATE-PI-95)
Statement for Linked Answer Q 21 & Q22

Common Data for Q uestion 17 & 18 Resistance spot welding of two steel sheets 1s

A weld is made using MIG welding process with carried out in lap joint configuration by using a

the following welding parameters: Current: 200A; welding current of 3KA and a weld time of 0.2s.

Voltage:25V; Welding speed:18 cm/min; wire a molten weld nugget of volume 20mm3 is

diameter: 1.2 mm; Wire feed rate: 4 m/min; obtained. The effective contact resistance is

Thermal efficiency of the process : 65% 200µ0. The material properties of steel are given
(GATE-PI-01) as (i) latent heat of melting 1400 kJ/kg, (ii)density
8000 kg/m3 , (iii) melting temp 1520 °C, (iv)
17 The heat input per unit length of the weld in
specific heat 0.5kJ/kgC. The ambient temp is
kJ/cm is:
20C. (GATE-PI-09)
(a) 0.18 (b) 0. 28 (c) 10.83 (d) 16.66

21. Heat (in Joules) used for producing weld


18 The area of cross section of weld bead in
nugget will be (assuming 100% heat transfer
mm2 is:
efficiency
(a) 16.3 (b) 25.1 (c) 30.3 (d) 38. 6
(a) 324 (b) 334 (c) 344 (d) 354
Common Data for Q uestion 19 & 20
22. Heat (Joules) dissipated to the base metal
A weld is made using MIG welding process with
will be (neglecting all other heat losses)
the following welding parameters: Current:200A;
(a) 10 (b) 16 (c) 22 (d) 3
Voltage:25V; Welding speed: 18 cm/min; wire

!1Jlli@hii§@$Rflnffinjjj� yderabad I Delhi I Bhopal I Pune I Bhubaneswar I Lucknow I Patna I Bengaluru I Chennai I Vijayawada I Vttag I Tuupari I Kukatpally I Kolkata
SOLUTIONS
06. Ans: (b)
Sol: In DCSP the work is positive and electrode
One Mark Solutions
is negative, due to work is positive the heat
generated at the work is high, depth of
0 1. Ans: (b) penetration is high. Hence high melting
Sol: By increasing welding current the heat point and high thickness plates can be
generated will increase and by reducing easily welded but due to lower heat
welding speed more concentration is taking generation at electrode the melting rate of
place, hence depth of penetration of the electrode is low and so deposition rate is
weld will increase. low.

02. Ans: (c) 07. Ans: (d)


Sol: With small variation m arc length the Sol: Coatings are given on the electrodes to
voltage of power source will be taken as remove oxides, stabilize the arc, absorb
nearly constant. gasses present in the weld pool, producing
shielding gasses, provide alloying elements
03. Ans: (b) etc.
Sol: The amount of oxygen consumed from the
oxygen cylinder in neutral flame is 1 : 1 ratio, 08.
when the acetylene consumption is 1 Oltrs Sol: For the neutral flame the amount of oxygen
then oxygen consumption from the oxygen consumed from the oxygen cylinder is equal
cylinder is also 1 Oltrs to the acetylene consumed hence the ratio of
amount oxygen to amount of acetylene is
04. Ans: (d) 1:1.
Sol: The required current for joining of 1 .5 mm
thick steel plates is 1 0000 Amps. 09.
Sol: 2.5
05. Sol: Maximum & flyer plate

!lflli@hiUUihiMRflfij@jjjfjj+ydcrahad I Delhi I Bhopal I Pune I Bhubaneswar I urlnow I Patna I Bcngaluru I Chcnnai I Vtjayawadal Vizag I 11rupati I Kukatpally I Kolkata I
: 643 : Welding

10. Ans: (b & c) 16. Ans: (c)


Sol: Preheating of weld bead is to evaporate the
17. Ans: (c)
grease oil, water etc present on the weld
Sol: GTAW is also called as TIG welding which
bead surface , to increase welding speed, to
is a non-consumable electrode welding
join high thermal conductivity material and
high melting point materials and also to 1 8. Ans: (a)
prevent cracks during joining of ring like Sol: Aluminum and magnesium are the highly
structures. reactive metals hence it is difficult to weld
metals.
11. Ans: (a)
Sol: When the plate thickness is changed, the 1 9. Ans: (b)
amount of heat generated has to be changed Sol: In solid state welding like cold pressure
for this the current has to be adjusted. welding operation, when large amount of
force is applied on the metals the plastic
12. Ans: (a) deformations of metals will takes place and
Sol: This is due to short supply of oxygen. if the plates are moved slightly, the
contaminated layer will be removed and the
13. Ans: (b) surfaces become cleaned so that it produces
Sol: GTAW is also called as TIG welding which
good bond
is a non-consumable electrode welding
20. Ans: (a)
14. Ans: (b)
Sol: HAZ means, it is the zone where the parent
Sol: At the intersection of inner and outer cone metal will not melt but the temperature of
the maximum temperature induced is the metal is above upper critical temperature.
(i) 3260°C in neutral flame Hence the metal will undergo micro
(ii) 3380°C in oxidizing flame and
structural changes.
(iii)3040 C in carburizing flame.
°

21. Oxidizing flame


15. Ans: (d) Sol: Because during welding with neutral or
Sol: With increase of joint gap the strength of carburizing flame the zinc present in the
the brazed joint will first increases due to brass will get evaporated. But with
increase in wetting area and reduces oxidizing flame, during welding the excess
afterwards due to lower strength of the filler oxygen present in the flame will combine
rod material.
iltilQi/4jjjM04RflGiMiii!\i� yderabad I Delhi I Bhopal I Pune I Bhubaneswar I L.icknow l Patna I Benga)uru I Chennai I Vijayawada I V,z,ag I Tirupati I Kukatpally I Kolkata
: 644 : Production

with the some quantity of zinc and forms materials and to prevent cracks during
zinc-oxide which is floating on the weld . joining of ring like structures.
pool and it is not allowing remaining zinc to
evaporate. 27. Ans: (a)
Sol: Welding current should be higher & Weld

22. Ans: (d) time should be less. Spot welding is a


resistance welding and in resistance welding
23. Ans: (d) because of very high temperature induced if
Sol: With increase of joint gap the strength of time for welding is high, the heat transfer
the brazed joint will first increases due to will takes place to the thickness direction of
mcrease m wetting area and reduces the plates which may promote the melting
afterwards due to lower strength of the filler of total thickness of plates which is not
rod material preferable and for inducing required amount
of heat it is required to supply large quantity
24. Ans: A-3, B-4,C-1, D-2, of current.
Sol: In Thermit welding the heat required for
melting of the plates is obtained by using 28. Thermit welding
exothermic chemical reaction, in projection Sol: In this the heat required for melting of
welding( it is similar to the spot welding plates 1s obtained due to exothermic
used for joining of projected component for chemical reaction and this method is used in
flat component) the heat required 1s remote areas like hill areas and forest areas
generated by using electrical resistance, in for joining of rails.
MIG welding the heat required is generated
due to electric arc and in friction welding 29. Brazing or braze welding
the heat is generated due mechanical work. Sol: Brazing or Braze welding is a operation of
joining two similar or dissimilar metals
25. Ans: A. Non-consumable, B. bare without melting of the parent metal but with
supply of molten metal of filler rod.
26. Ans: (b & c)
Sol: Preheating of weld bead is to evaporate the 30. Ans: (c)
grease oil, water etc present on the weld Sol: For adhesive bonding the area of contact
bead, increase welding speed, easy to join between the two plates must be very high ,
high thermal conductivity and high MP hence lap joint will give the highest joining
area.
!M11i@jjj4ifoi40jijjMb@�)Hyderabad I Delhi I Bhopal I Pune I Bhubaneswarl Lucknow! Patna I Bengaluru I Chermai JVtiayawada I Vi,.ag I TJ.rupati I Kukatpally I Kolkata
:645: Welding

31. Ans: (c) 37. Ans: (b)


Sol: Rutile coatings are given on the electrodes Sol: In resistance welding it is required to have
for producing slag over the weld bead. Low Rt, R2, R3 must be as low as possible and R,i
carbon steels are prone to form oxides, to must be as high as possible, this R,i is
eliminate it requires the slag on the weld coming from the contact resistance of the
bead.
joining surfaces or also called as asperities

Ans: A. Not used , B. Square of the current


between the joining surfaces.
32.

38. Ans: (d)


Sol: heat generated Q = i2 Rt

Sol: The current required for joining of 1.5mm


=>Qoci2

33. True
thick steel plates is lOOOOAmps, for 1mm

Sol: Because steels are having higher MP, if thick steel plates the current is less than
oxidizing flame is used it is possible to melt 1OOOOAmps and for 3mm thick steel plate
and the carbon present in the steel will act the current is more than 10000 amps.
as deoxidizer.
39. Ans: (a)
34. True Sol: In thermit welding Iron oxide + aluminum
Sol: Because steels are having higher MP, if powder will chemically react and produces
oxidizing flame is used it is possible to melt aluminum oxide which is used as slag, iron
and the carbon present in the steel will act is produced which is used as filler material
as deoxidizer. and heat is generated which is used as heat

35. Ans: (a)


for melting of plates.

Sol: By feeding the steel powder, the powder 40. Ans: (c)
will burned for producing heat required for Sol: The size of the LASER is very small , hence
rising the temperature of plates. RAZ also small.

36. Ans: (d)


41. Ans: (d)
Sol: When the electrons are generated at the
Sol: Because the electrode has to continuously
electron gun which is collected and focused
traveled, hence it should be in the disc form.
by using magnetic lens.

jltlli@jjji4ijjji"1bftihifjj� yderabad I Dellii I Bhopal I Pune I Bhubaneswar I wcknow I Patna I Bengaluru I Chennai I Vijayawada I Vuag I Tuupati I Kukatpally I Kolkata
PuNiariooa
ACE : 646 : Production
��-������

....
�����=========��==============.,;,,;.,;,,;,.,;;,,,;,,;
�'Ii'....;,._.;...
;,,,;,;;,;,
42. Ans : (d) 47. Ans: (d)
Sol: Brazing and soldering are the non-fusion Sol: Pure tungsten electrodes are frequently
welding operations i.e. parent metal will coated oxides of Th, Zr, La, and Ce. These
not be melted, hence the joint formation is oxides are expected to perform two
heterogeneous.
important functions (a) increasing arc
stability and (b) increasing the current
43. Ans: (c)
carrying capacity of the electrodes.

44. Ans: (a)


Sol: Autogeneous welding if one in which the
metals are joined without use of filler
material.
Two Marks Solutions

45. Ans: (d)


Sol: In welding (Fusion welding for melting the 01. Ans: A-2, B-4, C-6, D-5,
parent material the temperature should be
greater than the MP of the metal hence it is 02. Ans: A-3, B-1, C-4, D-2,

high. Whereas brazing and soldering are the


03. Ans: (a)
nonfusion welding operations hence the
temperature should be less than the MP of Sol: Heat generated = I 2 R't
the metal. Brazing temp is above 427°C and = 150002 xO.OOO 1 x0.25
soldering is below 427°C = 5625 (in W-s = J)

46. Ans: (d) 04. Ans: (d)


Sol: V2 = 2V1 Sol: Aluminum is welded by TIG or also called

Awl V1 = Aw2 V2 GTAW

_ A 1 xV 1 - A w 1 xV 1 - A w 1
X X

Aw2 - w - 05. Ans: (d)


V2 2VI 2
A w 2 - A w l 0.5-1
- ---
o/cochange- -=---=-0.5
Sol: Heat generated = I2 R't

Aw 1 1 = 50002 x200x 1o-6 x0.2


= 1000 (in J=W-s)
=-50% � reduced by 50%

!1"S•IQ04jji4g;jjij@j,1Miiiih� yderabadJDellrilBhopalJPuneJ Bhubaneswari LucknowlPatnaJBengalurulChermaiJVtjayawadajVizag JTuupati I Kukatpallyj Kolkata


.EJ � : 647: Welding
"=================================
06. Ans: (c) at L=7mm, Is= 800, Vo= 80V, Im = 460A
Sol: Heat generated = Heat utilized 80
2
a + b(7) = 80 --x 460
I Rt = m x hrg 800
2
I Rt = p x volume x hrg a + 7b = 34 ........... ...........(2)
/
2
x200x10 -{; x0.1 solve (1) & (2)
we get a = 20 , b = 2
= 8000x 1t (0.005)2 X 1.5 X 10-3 X 1400x1 3
4 :.V = a + bL = 20 + 2L
=> I= 4061.2 Amp
09. Ans: (c)
Sol: Given I = 10000 A, 't = 10 x10-3 = 0.01 Sec
07. Ans: (b)
hrg= 300 kJ/kg, Cp= 800 J/kgK
Sol: Heat input(J/mm)
Melting point Tm=1793 K
V I (in J / sec = W)
= x� Ambient temperature Ta=293 K
speed of welding(mm I sec)
25 X 300
= X 0.85 = 796.8
8
= 797 J/ mm 2mm
Mass of nugget = volume of nugget x
08. Ans: (a) density
4
Sol: V=a + bL, m = -m 3 xp
3
V
Vc1rop=Vo - o xi m 4
Is m = -n(0.002) 3 x 7000
3
Where V0= open circuit voltage
m = 2.346x10-4 kg
Is= short circuit current
Heat required = m(hrg)+mCp (Tm-Ta)
Im= measured current
=m(hfg+ Cp(Tm-Ta))
V= linear voltage =2.346x10-4 (300+0. 8(1793-293))
At equilibrium V Vdrop = 351.9 kJ
=

V
a + bL=V0 - o xi m Heat generated = I2 R 't
Is
= (10000) 2 x 500 x10 -{; x0.01
at L = 5 mm, Is= 800, Vo = 80 V, Im=500A
=500 kJ
80
a + b(5 80- -x 500
)= 35 .9
800 Efficiency(�)= l = 70.38 %
500
a + 5b = 30 . ... .. . ....... . . ... ....(1)

\( I I .114111t { I111� P11hlH ,tflOlh �yderabad I Delhi I Bhopal I Pune I Bhubaneswar I Lucknow I Patna I Bengalwu I Chennai jVtjayawadal Vizag I Tuupati I Kukatpa))y I Kolk.ala I
: 648: Production

10. Ans: (b) 15. Ans: A-3, B-4, C-2, D-5,


Sol: Net heat input = Velocity x H.R Sol: In EBM because of the complete vacuum
Power x11me1ting x11heat transfer the weld bead is completely protected from
= Area xVelocity x H.R oxidation so highly reactive metal can be
2000 x 0.5 x0.7 = 5 xVelocity xlQ welded, SAW is used for joining of plates
700 = 50xvelocity upto 75mm thickness in single pass
Velocity= 14 mm / min welding, welding of rails in remote areas
can be done by using thermit welding and
11. Ans: (a) rod can be welded to plates by using friction
Sol: Force ( F ) = Pressure x Area x µ welding

= 200 X ,r 10 2 X 0.5 = 7854 N 16. Ans: (c)


Sol: In oxidizing flame the ratio of oxygen to
4

Torque= F x l x Radius acetylene is 1.15 to 1.5


4
Oxygen consumption for 8 liters of
4
Torque = 7854x l x 5 x 10-3 = 29.45 Nm Acetylene is = 1.15 to 1.5 of 8
= 9.2 to 12 liters.
21tNT
Power (P) = --
17. Ans: (d)
60000

Sol: Heat supplied= Heat utilized


21tx 4000x 29.45
=-----= 12.33 kW
60000
0.5 J = m (S.H. + L.H)
12. Ans: (a) 0.5 = (axh) p ( Cp (Tm-Tr)+hrg)
Sol: Even though it was not mentioned about the 0.5 = 0.05x10-6xhx2700(896(630) +398x 103)
type of flame,but from the given choices it � h = 0.00385 m = 3.85 mm

18. Ans: (d)


is possible only the answer as 8ltrs.

13. Ans: A-4,B-3,C-1,D-2, Sol: Welding time =


900
= 3 min =180sec
14. Ans: (a)
300

Sol: By increasing welding current the heat


I = 150 A,V = 20 V ,11 = 0.8
R = 36 x 10-6 n
generated will increase and by reducing
Heat generated = I2 Rt 11
welding speed more concentration is taking = 1502 x36 x 10- 6 x 180x0.8
place hence depth of penetration of the weld
= 116.64 J
will increase
\( I
)
I 11g111,,1111g l 11l1l1, .111,,11.... yderabad I Delhi I Bhopal J Punc J Bhubaneswar I Lucknow I Patna I Bengaluru I Chennai I Vtjayawada I Vizag I T1n1pati I Kukatpally I Kolkata
: 649: Welding

19. Ans: (c) 24. Ans: (b)


Sol: Slag inclusions will never produces the cold Sol: Given t = 8 mm , V = 20 V
cracks, but with entrapped hydrogen Root gap = 5 mm , Speed = 5 mm / sec
material becomes embrittled which 11HT = 0.8, HI = 10 J/mm3
mcreases the chances of cold cracking, 1lmelting = 0.625, I = ?
presence of residual stresses will also Vol. of W.B = t x root gap x L
mcreases the cold cracking and during = 8x5 x L = 40 x L mm3

welding of steel structures the martensite Where L = length of Weld bead


formation will takes place which is highly H.R = 40 L x lO = 400 L (in J) ........... ( 1)
brittle and promotes the cold cracking VIx L 20xix L .
H.I = = (m J) ........(2)
Speed 5
20. Ans: (b) Equating Equations 1 and 2
Sol: In TIG and MIG welding operations the 20 x IL
x 0.8 x 0.625 = 400L
inert gas will be used as shielding gas hence 5
highly reactive metals like aluminium can � I = 200 A
be easily welded. But sometimes even SAW
25. Ans: (a)
also may be used
26. Ans: (b)
2 1. Ans: (d) Sol: With increase of welding speed the rate of
Sol: In arc welding by changing the polarity the heat transfer reduces and with increase of
heat generated at the electrode and work thickness of plates also thermal resistance
piece can be changed. increases , in both of the cases HT will
come down hence the HAZ will be reduced
22. Ans: (d)
Sol: Spatter is caused due to arc blow, distortion 27. Ans: (b)
is due to poor joint selection, slag inclusion Sol: The welding process used will never creates
is due to improper cleaning and porosity is the internal residual stresses hence
due to damp electrode which induces large
28. Ans: (c)
quantity of shielding gasses.
Sol: The weld bead should exhibits ductility to
withstand for impact loads but the welding
23. Ans: (d)
process is not only used for MS and also
can be used for other materials.
!Mfi4jj§ml4ijj/QjmniM\n•n+yderabadJDelliiJBhopalJPunclBhubancswarJ LucknowJPatnaJBcngaluru!ChennailVijayawadaJVmg JT',rupati J KukatpallyJ Kolkatll
� ACE : 650: Production
�-�����
y ������============================
t........;...........;.... Puliicatims

29. Ans: (b) 32. Ans: (b)


SvJ: The basic purpose of providing the vacuum Sol: t= 2 mm, I= 6000 A, t= 0. 15 sec
in EBM is for avoiding the dispersion of Heat rate (H.R) = 2.9 J/mm3
electrons but not for providing efficient D = 5 mm, h= 2.5 mm
shield and it can act as shield r.s additional
R= 75 x 1 o--o Q
functionality.
Total heat required (HR)
30. Ans: (b) = toatl volume xHR/mm3
Svl: Given 3V +I= 240
I= 240-3V
Power (P)= VI = V (240 - 3 V) =-5
1C 2 x2.5 x2.9

= 240 V -3V2
= 142.35 J
For optimum power
Actual heat generated (HG) = I 2Rt
dp = 0 => 240 - 6V = 0 = 60002 x75 xlf 6 x0. 15
f

= 405 J
dV
240
=> V= = 40 V
6 . H.R 142.28
Efficiency 11 = --= --= 35¾o
H.G 405
3 1. Ans: (b)
33. Ans: (d)
Sol: Volume= (1 102 - 1002) x 2
4
1C

= 3298.66 mm3 34. Ans: (a)


Heat required= total volume x heat rate/mm 3
Sol: By feeding the steel powder, the powder
= 3298.66 10- x64.4 X lQ 6
will burned for producing heat required for
2 12.4 Joules
9

rising the temparature of plates to rise to the


X

V v2 kindling temperature
Power (P)= VI = V x-= -
R R
302 35. Ans: (a)
= -= 21.43 J/sec
42 VI
Sol: Power P = --x Tl
Power Speed
Energy(heat)--. -
time 20 x500
= x0.9 = 27 kJ I mm
. 2 12.4 20 / 60
=> Time = -- = 10 sec
2 1.43

l1J11ii@hi§§.lj/40jnftl\lilh+iyderabadlDe!hilBhopallPunelBhubaneswarl LucknowlParnalBengaJwulGiermailVijayawadajV17.3g ITU1.1pali I Kukatpal)yl Kolkala I


: 651: Welding

36. Ans: (a) Is substitute in equation (1)


Sol: I = 150 A, V = 30V, Speed = 6 m/min =>Vo = 95 V
Wire dia = 1.2, f = 12 m / min
H.G = VI = 150 30 = 4500 W 38. Ans: 150
Sol: I = 10000A, 0.2sec,
x

H.R=: X d f X p X C p X (Tm -T.)


't =

heat dissipated = 1OOOJ,


2
X

Heat required = 201/mm3 , interfacial contact


-(1.2) x lO x - x 7000 x 500(1530-30)
resistance = R = 0.00020hms
tr 2 -6 12
=

1187.5 W Heat generated = I2 R 't = 40001


4 60
=

H.R 1 187.5 Heat utilized for welding or melting


Tl =39.58%
H.G 4500 x I = 4000- 1000 = 30001
=

3
=

20 volume of weld nugget


37. Ans: (c) Volume of weld nugget = 3000/20 = 150mm3
=
x

Sol: Va= 20+5L, L = 5 to 7 mm,


I = 400 to 500A 39. Ans: 70
At equilibrium condition Sol: b = 20 mm; t = 8 mm
Va= Vp undercut (d) = 3 mm
V Due to undercut, the load bearing area
20+5L = V --o xI
Is =b
20 x (8 - 3) = 20 x 5 = 100mm2
o X

At L = 5, I = 500
(t-d)

V0 Maximum load carrying capacity of the weld


=

20+5(5) = V0 - x500
Is = CTut x area = 700 x 100 = 70 kN
-

Vo
45- V0 - - x 500 ..................(1) 40. Ans : 48
Is
Sol: Heat generation rate = V.I
At L = 7, I= 400
40 x 400 = 16000W
V0
=

20 + 5(7)= V0 - - X 400 Heat utilized


Is
H.G x melting Efficiency x HT efficiency
V
55- V0--o x 400 ..................(2) = 16000 x 0.8 x 0.3 = 3840W
=

Is
Volume filling rate of weld bead
Solve (1) & (2) for V0 & Is C.S area velocity
We get Is 950 A
=

(A x 4) mm3 /sec
x
=
=

l1tili@hiii•hi4AflbiM\hih� yderabad I Delhi I Bhopal I Pune I Bhubaneswar I Lucknow I Pa1na I Benga)uru I Chennai I Vtiayawada I Viz.ag I Tirupati I Kukatpally I Kolkata
: 652: Production

Rate of heat required


Volume per electrode = (5) 2 x500
20 x 4A = 80AWatt 4
7t
=

= heat utilized = 9812.5 mm3


3840 Number of electrodes required
A= = 48mm 2
80 107735
= 10.97 = 1 1 electrodes
9812
=

4 1. Ans: 149 to 151


44. Ans: 425 (range 423 to 428)
Sol: In general for maximum arc power,
Sol: V = 100 +40 L ,
short circuited current 300
Current = = L = 1 to 2 mm , I = 200 to 250 A
2 2
L = l , 1 = 250
= 150 A
V
V = 100 + 40xl 140 = V0 --0 x250
Is
=

42. Ans: (a)


L = 2, I = 200
Sol: V = 60 V , I = 50A
. V0
Speed = 150 mmlmm � 1st process V = 100 +40 x 2 = 180 = V0 x 200
Is
--

120 mm/min � 2nd process


V
� 40= 50 x-o
=

V1 V2
Is
=

(H.1)1 =(H.1)2

(
V.I
Speed 1
- J( J
V.I
Speed 2
V0 40 _±
= =
I s 50 5

11 V0 = 140 + -x250
12 = xSpeed 2 5
4

Speed,
140 +200 = 340
50
=

V0 4 V x5 _ 340 x 5
- x120
150
40 A = �I _ 0 = 425 A
Is 5 4 4
= =

43. Ans: (c) 45. Ans: 2.91 (range 2.85 to 2.95)


Sol: Area of weld bead Sol: I 2R't = volume x p x H.R /kg
= 5x 10 +2 x 0.5 x 10 tan30 x 10 40002 x 200 x 10-6 x 0.2 = volume x 8000 x
= 107.73 mm 2
1400 103
Volume of weld bead = area x length
n
Volume = 5. 7 x 10-s m3 = x 52 x h x I o-6
X

= 107.73 1000 = 107735mm3 4


�h = 2.91 mm
X

!ltl•iii§jjj4Pmi4RflCIM\iiih� yderabad I Delhi I Bhopal I Pune I Bhubaneswar I Lucknow I Patna I Bengaluru I Chennai I Vtjayawada I V17.3g I TU11pati I Kukatpally I Kolkata
ACE : 653: Welding
��-�EF.agioeeriogPnbtiariooa
� �����;;;;;;:�===========================�
46. Ans: (a)
k
Sol: a=-
pC
Five Marks Solutions

15 0 0 1.
a =--- -=55.5 x10 -6
3 000 x900 Sol: V0 = 8 0V; 1 s = 1 000A
1

50 80
a 2=----= 13.88 x10-6 Va= 2 0 +4L; Vp= 8 0--- xI
8 000 x45 0 1000
VI 15 X 200
q= = =6X 105 At equilibrium condition
i r 5 X 1 0 -3
Va= Vp
15 x300
q 2= = 4_5 xl 05
1 0X 10- 2 0 +4 L = 8 0-� xI
1 000
3

T q a 6 xl 05 13.88 xl 0-6
-1 =-1 X-2 =---5 X---- l OOO
q2 a 1 4.5 x10 55.5 x 10 -6 => I= ( 60- 4L)
80
6X 13.88
= 0333= _!_
T2

= Power (P) = VI
4.5 X 55.55 3
1 0
(20 +4L{ �� (6 0-4L)]
47. Ans: (0.2327)
=

Sol: t = 2.5 mm , I = 6000A, t = 0.2sec lOOO


[1200- 8 0L +240L-16L2 ]
R = 200 µQ 80
=

Heat required, (H.R/m3 )= 1Ox109 J/m3 lOOO


(1200 +16 0L-16L2 )
80
=

nugget diameter = d = 4mm (sphere )


Heat generated = I2 Rt
For optimum power and optimum arc length
= 6 0002 200 x10-6 X 0.2 = 1440 J
dP
- = 0=>160-32L = 0
X

Heat required = volume x heat required/m3


dL
= x n x R 3 x H.R / m 3
.± 160
32
=>L = -=5mm
= X (2X10-3 jX 1 0X 109 :. V = 20 +4 5 = 40 V
.±3 X
7t

335. 1 032 J l 000


x

I= (60-4 x 4)= 55 0 A
=

Heat required 80
=
Heat generated P = VI = 40 x 55 0 = 22 kW
llH.1

335.1032
= = 0.2327
1440

!M•Ni@jjjqg.jjjjUflni@Ujjj+yderabad I Delhi I Bhopal I Pune I Bhubaneswarl Lucknow! Patna I Bengaluru I ChennailVrjayawada!Vmg ITirupati I Kukatpallyl Kolkata I
: 654 : Production

02. 03.
Sol: Total area A = A1 +A2 +A3 Sol: VO = 62, Is = 1 30 , L = 4 mm,
= A1 +2 A2 Welding speed = 15 cm /min
1 TJ 85% , V = 20 + 1 .5 L
=( 1 9 x2)+.,2 ( x 1 7 tan30x 1 7)
=

At equilibrium conditions
2
204.8 mm
Va = Vp
= 2

19 h=1 7

At L = 4 mm,
62
,, 2

20 + l .5(4) = 62 - - x I
\ I

1 30
H
2

Volume of weld bead = 204.8 x 180 36 x 130


=> I = = 75.5 A
62
= 36873 mm3
Power (P) = Heat generation
h = 1 9 -2 = 1 7 mm
VI = 26 x 75.5
Base = b = 1 7 tan 30
=

1 963 Watt
Total Volume of weld bead
=

Actual heat input = 0.85 x 1 963


Volume of actual weld bead
1668.55 Watt.
=

+Reinforcement +spatter loss


=

= 1 .25 x Volume of weld bead


= 46092 mm3 04.
Sol: Given AC = 1 0 mm, 01A = 01 C = 7 mm,
Volume per electrode = - d 2 x Le
4
1t

02A = 02C = 20 mm

=-tr 2
x3 x( 450 - 50)
4
= 2826 mm 3

TotalVolume of weld bead


No of Electrodes = ---------­
Volume/electrode
= 16.3

IIJili@hiiiiihiRftftj@n.jj� ydcrabad I Dellii I Bhopal I Pune I Bhubaneswar I Lucknow I Patna I Bengaluru I Chennai I Vtiayawada I Viz.ag I Tuupati I Kukatpally I Kolkata
: 655:

Height of Bead = BD = 01D - 0 1 B When L = 4, I = 550


= 01D- �01 A 2 - AB 2 V,
20 + 4(4 ) = V0 - _Q_ x 550
ls
= 20 - .J20 2 - 5 2 = 0.64 mm
Depth of Penetration = BE = 0 1 E-0 1 B 36 - V"o x 550 . . . . . . . . . .. . . . . . ( 1
- V0 - - )
ls
2 2
= (DiE) - �( 0 A) - ( AB)
2 When L = 6 , I = 450
= 1 - .J1 2 - 5 2 = 2. 1 0 mm V,
20 + 4(6) = V"o _ _Q_ x 450
ls
05.
Sol: Let t = c 1 Ji + c 2
- --
4- V"o x 450 . . . . . . . . . . . . . . . .. (2
V0 )
. . . . . . . . . . . . . . . . . . ... ( 1 ) ls
Solve ( 1 ) & (2) for Vo & Is
At t l = 10 , I l = 160 � 1 0 = C 1 MO + C 2 · · · ·(2)
VO = 80 V & Is= 1 000 A
At t 2 = 1 5, 1 2 = 360 � 1 5 = C 1 ../360 + c 2 . . . (3)

Solve ( 1)&(2) Now for L = 5 mm

5 Va = 20 + 4( 5) = 40 V
We get C1 = -- = 0.79
6.32
V0 8
C2 = 0 VP = VO - -x I � 40 = 80 - - x I
Is 1 00
t = c 1 Ji + c 2 � t = 0.79./i Current I =500 A
Now for t = 8 mm Power(P) = VI = 40 x 500 = 20 kW

07. Ans: (a)


8 = 0.79./i � I = 1 03 Amps

Sol: Number of jobs per day = 5000/25 = 200


Sol: Va = 20+4L,
06.
jobs per day based on the 25 working days
L = 4 to 6 mm, in a month to weld 200 jobs per day it is
I = 450 to 550 A necessary to perform the welding in one
At equilibrium conditions pass only means that a 40mm thick plates
Va = VP are to be joined in single pass welding, the
20 + 4L = V,0- -
/ xi
Vo process used is SAW.
s

\( I I ngl l ll t r11w, P11hl h ,1t1cl11, �yderabad I Delhi I Bhopal I Pune I Bhubaneswar I Lucknow I Patna I Benga)uru I Chennai I Vijayawada I V17.ag I Trrupari I Kukalpally I Kolkata I
: 656 : Production

08. Ans: (a) 600


=> I = (60- 40L)
Sol: Q = 8 K Tc t [ 0.2 +Vb/4a ] 80

Given, Q =5 kVA = 5 x 103 J 6


P = VI = (20 +40 L) [ 60- 40L) J
°° (
80
K = 45, Tc = 1450 C,
600
°

t = 5mm=5x 10-3 m , a 1.2 x l 0-5 , [1200 -800L +2400L- l 600L2 ]


80
=

B = width of weld
=

600
= [1200 +1600L - 1600L2 ]
B = 2x5 tan 30 = 5.77 mm = 5 . 77x 10- m 3
80
(on substitution of the following For optimum arc length
parameters) dp = 0 => 1600- 3200L = 0
=> V 14.2 mm/s Welding speed dL
1600
= 0_5
= =

=> L =
09. 3200
0 85P 0.85 X 2 X 10-5 At L=0.5 mm
Sol: R e1 = ___:___ = 1.082 10-5
n l nr 25 x ,r x 0.02 V = 20+(40 x 0.5) = 40 V
= X

V2 52 600
H.G 1 VI = -x V = - = ----5 I= (60-40 x .05) 300 A
80
V
R Re, 1.082 x 10-
=
=

P = VI = 40 x300 = 12000 =12 kW


2.3 MW
0.85p = 0.85 x 2 x 10-5
=

Re2 = 1 1.
n2 ,rr 50 X 7r X 0.02
Sol: Rated Power = Yr Ir = 5 x 103
0.54 1 10-S
50 X 10 3
v2 52 => Rated current, 1, = --- = 2000 A
=
X

H.Gz = - = = 4.6 MW 25
Re2 0.54 l x l0-5
Rated duty cycle Dr = 50%,
If desired current Id = 1500 A
10.
1; D, = 2000
Sol: Va 20+40L Dd ( 1500 ) X 50 = 88.8
2

80
= =

Vo = 80 V, 1S = 600A, Vp = 80- - 1
1/

600 Arc on time


D =
Total welding time
at equilibrium condition
Ar . 88.8
d

Va = VP => c on time = - - x30


100
80 = 26.67 sec
20 +40 L = 80- - 1
600
ll!ilihYihiiihiRflblM\iilii� yderabad I Delhi I Bhopal I Pune I Bhubaneswar I Lucknow! Patna I Benga)uru I Chennai I Vuayawada I Vizag I Tirupati I Kukatpally I Kolkata
: 657 : Welding

Ans: (c)
D2h
Sol: At equilibrium conditions Va
12. ,r
Volume of nugget =
= Vp 4

2 0 + 40L = 8 0- (
80 )1 = ,r
2
( 7.35} x 2.7 =
458
mm3
4 4
Heat required = Volumexp x H. Rig
1 00
l OOO
=> I = (60 - 40L )
80 458
= x l 0-3 x 8 x 1 3 8 0
1 0 4
P = VI = (20 + 40L { �� (60 - 40L)]
= 1 264 joule
l OOO [
= 1200 - 800L + 2400L - 1 600L2 ] 14.

Sol: VP = 36 - _!_
80
l OOO
= [12 00 + 1 600L - 1 600L2 ] 60
80

For optimum arc length dp = O


Va = 2L + 27

dL
At equilibrium conditions

I
=> 1 600-32 00L = 0 Va = Vp
=> L = 0 .5 cm = 5 mm 27 + 2 L = 36 - => I = 60 (9 - 2L)
60
13. If current is 3 00 amps

Sol: H.G = I2 Rt: 4


300 = 60 (9 - 2L) => L = - = 2
2
= ( 1 0000) 2 X 200 X 1 0-6 X 2_ lf L = 2 mm,
= 2 000 Joules
50
V = 27 + 2(2) =3 1 V
h = 2t - 2 (0 . 1 t) = 1 .8 t = 1 .8 x l .5 I = 60 (9 - 2 x2) = 3 00 A
= 2.7 mm P = 3 1 x3 00 = 93 00 W
D = 6Ji = 6M = 7.35 mm If L = 4 mm
V = 27 + 2(4) = 35 V

�1�t
I = 60 (9 - 2 x4) = 60 A
p = 35 X 60 = 2 1 00 W
Change in power = 93 00-2 1 00 = 72 00 W
O.lt
If the maximum current capacity is 300A,
the maximum arc length is 2mm

!M11ih§jjj§4ijjj4flftej@jjjjjj+yderabad I Dellii I Bhopal I Pune I Bhubancswar I Lucl\now I Patna I Bengaluru I Chcnnai I Vijayawada j Vizag I Ttrupari I Kukatpally I Ko!kalll. I
: 658: Production

15.
Sol: When L= 3 mm, I = 500 A 5 tan 30

V
V = V0 - -o x I

V = 20 +40 x 0.3 = 32
Is

V
32 = V0 - -o x500 .... ( 1)
Is Vol. of W.B = 14.43 x 1000= 14433 mm3
When L = 5 mm , I = 400A Weight of W.B = 14433 x 10-3 x 8
V = 20 +40 x 0.5 = 40 = 1 15.5 g
Vo H.R for melting of W.B = 1 15.5 x1400
40= V0 x400 . ... (2)
Is 16 1. 66 kJ
-
=

Solve ( 1)& (2) . 16 1.66


Time for weldmg = ---= 36 8ec
0.8 x5.6
1000
Vo 8
vO = 32 + - x 500 = 32 + -x 500 = n v
Welding speed =
Is 1 00

lOO
I s= V.O = 900A
36
x
8 = 27.78 mm /sec

17. Ans: (c)


16. Sol: I = 200, V = 25, speed = 18 cm /min
Sol: Power = P = 4+0.8L- 0. 1L
D = 1.2 mm, f = 4 m /min , 11= 65%,
2

For optimum power


V I_ _ 25 x 200 x 0.65 x60
dp
= 0=> 0.8- 0.2L = 0 H I = __11 = ___________
dL · speed 18

08 10.83K.J I cm
L= · = 4 mm
=

0.2
18. Ans: (b)
P = 4 +0.8L- 0.1 L 2
Sol: Filling rate of weld bead filled rate by
4 +0.8 x4- 0. l x 42 =
=

= 5.6 kW electrode
Energy losses = 20% =>11= 80%
area of W.B x Speed =
4d xf
,r 2

Area of weld bead = 2 x_!_ x base x height


2 7t X 1.2 X 4000
= 5 tan 30 x 5 4
Area of W.B = - '------- = 25. 12 mm2
2

180
= 14.43

jN4l4jj§jj/44ijji4Rflb1Milifh+iyderabad I Delhi I Bhopal I Pune I BhubaneswarJ Lucknow I Patna I Bengaiuru I Chennai I Vijayawada J Vmg I Tuupari I Kukatpally I Kolkata I
t . A:CE . .
��•�Fnpw.:rqPnblicaaiooa : 659 : Welding
� � ==================================
19. Ans: (c) 2 1. Ans: (c)
Sol: I= 200, V = 25, speed= 18 cm /min Sol: I= 3000 A, -c = 0.2, R= 200 µQ
D = 1.2 mm, f = 4 m /min , 11= 65%, Vol. of nugget"= 20 mm3
VI11 25 200 x 0.65 x 60 H.G = 12 R -c = 30002 x200x l 0-6 x0.2
H. I= = x
speed 18 = 360 J
= 10.83 kJ / cm H.R= 20 x 10-9 x 8000 (500 ( 1520-
20)+1400 103 ))
20. Ans: (b) = 344 J
X

Sol: Filling rate of weld bead


= filled rate by electrode 22. Ans: (b)
Sol: Heat dissipated= 360- 344 = 16
Area of W.B x Speed= d2 x f
4
7t

Area of W.B = -'4-----


7t X 1.2 2 X 4000

180
= 25. 12 mm2

!ltllQO§jjj§lh!/MRflftj@ii!jj� I I I I I I I I I I I
yderabad Delhi Bhopal Pune Bhubaneswar wcknow Patna Bengaluru Chennai j Vtjayawada j V12.ag Tuupari Kukatpally Kolkata
Metal Cutting
(c) forms a low shear strength film of work
One Mark Questions material at the tool chip interface
(d) serves as a dielectric , minimizing
01. Cutting tools are provided with large thereby reactions due to EMF at the

(GATE-ME-87)
positive rake angle mainly for interface

(GATE-ME-89)
(a) Increasing the strength of the cutting 04. The size of BUE in metal cutting increases
edge with
(b) Avoiding rubbing action with the (a) very high speed
finished surfaces (b) large uncut chip thickness
(c) Reducing the magnitude of the cutting (c) use of cutting fluid
force (d) increase in positive rake angle
(d) Better heat dissipation
05. Crater wear always starts at some distance

(GATE-ME-89)
02. If in a turning operation both the feed rate from the tool tip because at that point

(GATE-ME-87)
and the nose radius are doubled the surface
finish values will be (a) cutting fluid does not penetrate
(a) Decreases by 50% (b) chip tool interface temp is maximum
(b) Increases by 300% (c) normal stress on rake face is maximum
(c) Increases by 1 00% (d) tool strength is minimum
(d) Remain unaffected

(GATE-ME-90)
06. Most of the metal cutting heat goes into the
03. The ideal cutting fluid for low speed

(GATE-ME-88)
machining of metals should be one which (a) Moving chip (b) cutting tool
(c) Work material (d) machine tool
(a) removes the heat faster from the cutting
zone, 07. The effect of rake angle on the mean friction

(GATE-ME-92)
(b) forms a coating on the cutting tool by angle in machining can be explained by
chemical reaction
!1ti1i!i§jjj4iljj/Q0jnjM\jj.jj+y,1erabad I Delhi I Bhopal I Punc I Bhubancswar I Lucknow I Patna I BengaJuru I Chennai I Vijayawada I Vu.ag I Tirupati I Kukatpally I Kolkata I


.. ACE
� -.) �F..ogineeriogPnblicatioos : 661 : Metal Cutting
� � ����=��=======���=============.,;,,;,,==�
(a) Sliding model of friction 12. A built up edge is formed while machining
(b) Sticking and then sliding friction model (GATE-ME-00)
(c) Sticking friction model (a) ductile materials at high speed
(d) sliding and then sticking friction model (b) ductile materials at low speed
(c) brittle materials at high speed
08. Cutting power consumption in turning can (d) brittle materials at low speed
be significantly reduced by
(GATE-ME-95) 13. Friction at the tool - chip interface can be
(a) Increasing rake angle of the tool reduced by (GATE-ME-09)
(b) Increasing cutting angle of the tool (a) Decreasing the rake angle
(c) widening the nose radius of the tool (b) Increasing the depth of cut
(d) Increasing the clearance angle (c) Decreasing the cutting speed
(d) Increasing the cutting speed
09. Most of the metal cutting heat goes into the
(GATE-ME-91)
14. The minimum shear strain in orthogonal
turning with a cutting tool of zero rake angle
(GATE-ME-09)
(a) Moving chip (b) cutting tool
IS
(c) work material (d) machine tool
(a) 0.00 (b) 0.5 (c) 1.0 (d) 2.0

10. BUE is formed while machining


(GATE-ME-02)
15. The main cutting force acting on a tool
during the turning (orthogonal cutting)
(a) Ductile materials at high speed operation of a metal is 400N. The turning
(b) Ductile materials at low speed was performed using 2mm depth of cut and
(c) Brittle materials at high speed O. l mm/rev feed rate. The specific cutting
(d) Brittle materials at low speed pressure (in N/mm2 ) is
(GATE-ME-14-SET-l)
11. Formation of build-up edge during (a) 1000 (b) 2000 (c) 3000 (d) 4000
machining can be avoided by using
(GATE-ME-03) 16. If the Taylor's tool life exponent n is 0.2,
(a) tool with low positive rake angle and the tool changing time is 1.5 min, then
(b) high feed rate the tool life (in min) for maximum
(c) high cutting speed production rate is ---
(d) large depth of cut (GATE-ME-14-SET-1)

jl!I•pj@jjji4i0i4Rflni@ufjj� yderabad I Delhi I Bhopal I Pune I Bhubaneswar I wcknow I Patna I Bengaluru I Chennai I Vijaya�da I Viz.ag I Tirupati I Kukatpally I Kolkata
: 662 : Production

1 7. A straight turning operation is carried out 20. In HSS, the tungsten can be substituted by
using a single point cutting tool on an AISI (GATE-PI-89)
1 020 steel rod. The feed is 0.2 mm/rev and (a) Chromium (b) Nickel
the depth of cut is 0.5 mm. The tool has a (c) Molybdenum (d) Cobalt
side cutting edge angle of 60°. The uncut
chip thickness (in mm) is_ 21. In comparison to HSS, super HSS contains
(GATE-ME-14-SET-3) higher percentage of (GATE-PI-90)
(a) Tungsten (b) Carbon
18. Cutting tool is much harder than the work (c) Vanadium (d) Cobalt
piece. Yet the tool wears out during the tool-
work interaction, because 22. BUE formation ... A.. the cutting force and
(GATE-ME-14-SET-3) ... B... the surface finish (GATE-PI-90)
(a) Extra hardness is imparted to the work (a) decreases/ increases
piece due to coolant used (b) spoils / improves
(b) Oxide layers on the work piece surface
23. Assertion: a diamond tool is used for USM
impart extra hardness to it
of glass work-piece
(c) Extra hardness is imparted to the work
Reason: diamond is harder than glass
piece due to severe rate of strain
(GATE-PI-90)
(d) Vibration is induced in the machine tool

24. Increase m rake angle causes


1 9. Better surface fmish is obtained with a large
......... A............. in chip thickness and
rake angle because
........B ... in shear angle
(GATE-ME- 14-SET-4)
(a) increase/reduction
(a) the area of shear plane decreases
(b) increase /decrease (GATE-PI-91)
resulting in the decrease in shear force
and cutting force
25. Thrust force in drilling can be reduced by
(b) the tool becomes thinner and the
............ A... ...... the point angle and by
cutting force is reduced
.......B...... the helix angle of the drill
(c) less heat is accumulated in the cutting
(GATE-PI-91)
zone
(a) increasing / reducing
(d) the friction between the chip and the
(b) increasing / reducing
tool is less

jN11ii@jjji4@QRflbi@hM�/fyderabad I Delhi I Bhopal I Pune I Bhuban�I Lucknow I Patna I Bengaluru I Chennai JVtjayawada fVmg I TlfUpati I �y I KolkataI
: 663 : Metal Cutting

26. Amount of energy consumption for unit 31. Hot hardness is an essential property for
volume of metal removal is maximum in (GATE-PI-95)
(GATE-PI-91) (a) Gear materials
(a) turning (b) milling (b) Shaft materials
(c) reaming (d) grinding (c) Welding electronics
(d) Tool materials
27. In metal cutting with a carbide tool, at the
maximum recommended speed, the largest 32. In HSS Tool materials the ele!nent tungsten
% of heat generated goes to the can be completely replaced, without
(GATE-PI-92) changing the material property by
(a) tool (b) chip (GATE-PI-95)
(c) work (d) tool post (a) Molybdenum (b) Carbon
(c) Cobalt (d) Vanadium
2 8. Machinability of steels is improved by the
(GATE-PI-94)
33. Only negative rake angles are used v,rith the
addition of
following tool materials
(GATE-PI-95)
(a) Sulphur (b) Silicon
(c) Phosporous (d) all of the above
(a) HSS (b) Carbon tool steels
(c) Carbides (d) Diamond
29. Only negative rake angles are used with the
following tool material
(GATE-PI-94)
34. In metal cutting BUE fonnation results in
(GATE-Pl-95)
(a) HSS (b) Carbon tool steels (a) Improvement in surface finish
(c) Carbides (d) Diamonds (b) Reduction in cutting force
(c) Deterioration of surface finish
30. In metal cutting BUE formation results in
(GATE-PI-94)
(d) Improvement in tool life

(a) improvement in surface finish 35. When the depth of cut is increased, the
(b) reduction in cutting force specific cutting energy (GATE-PI-95)
(c) deterioration of surface finish (a) Increases
(d) improvement in tool life (b) Decreases
(c) Remains same
(d) Reaches an optimum value

\( I I llL:llll t ,in� Puhlil',tlH l!I:-. yderabad I Delhi I Bhopal I Pune I Bhubaneswar l Lucknow I Patna I Bengaluru I Chennai I Vijayawada I Vizag I Tirupati I &ikatpally I Kolkata I
-:, Production
� ���PnNiraliooa
...
If�.,.,...,. ACE : 664 :
========================================
.., • • • •

36. The effects of setting a boring tool above 39. Diamond cutting tools are not recommended
centre height leads to a/an. for machining of ferrous metals to
(GATE-PI-05) (GATE-PI-08)
(a) increase in the effective rake angle and a (a) High tool hardness
decrease in the effective clearance angle (b) High thermal conductivity of work
(b) increase in both effective rake angle and material
effective clearance angle (c) Poor tool toughness
(c) decrease in effective rake angle and an (d) Chemical affinity of tool material with
increase in the effective clearance angle iron.
(d) decrease in both effective rank angle and
effective clearance angle 40. Diamond cutting tools are not recommended
for machining of ferrous metals due to
37. If each abrasive grain is viewed as a cutting (GATE-PI-09)
tool, then which of the following represents (a) High tool hardness
the cutting parameters in common grinding (b) High thermal conductivity of the work
operations? (GATE-PI-05) material
(a) Large negative rake angle, low shear (c) Poor tool roughness
angle and high cutting speed (d) Chemical affinity of tool material with
(b) Large Positive rake angle, low shear iron
angle and high cutting speed
(c) Large negative rake angle, high shear 41. A spindle speed of 300 rpm and a feed
angle and low cutting speed 0.3mm/revolution are chosen for
(d) Zero negative rake angle, high shear longitudinal turning operation on an engine
angle and high cutting speed lathe. In finishing pass, roughness on the
work surface can be reduced by
38. The cutting tool material normally used for (GATE-PI-14)
turning steel of very high hardness is (a) reducing the spindle speed
(GATE-PI-06) (b) increasing the spindle speed
(a) HSS (b) tungsten carbide (c) reducing the feed of tool
(c) CBN (d) diamond (d) increasing the feed of tool

jl!lli@Oiiiijjji@ftj@\jj.jj.. yderabad I Delhi I Bhopal I Pune I Bhubaneswar I Lucknow I Patna I Bengaluru I Chennai I Vijayawada I Vizag I Tirupati I Kukatpally I Kolkata
: 665 : Metal Cu ",..
".:-.-:.111"....:........:.p11bJic:ations
ACE th
� , �=�
==�
================================ �
= �
42. Under certain cutting conditions, doubling 500 N and 250 N, respectively. The
the cutting speed reduces the tool life to coefficient of friction between the tool and

c�
index
r
of the original. Taylor's tool life

(n) for this tool workplace


the chip is ___
(GATE - 16 - SET - l)

combination will be 47. A single point right handed turning tool is


used for straight turning. The feed is 0.25
mm/rev and the uncut chip thickness is
(GATE -15 -Set 1)

43. The angle of a twist drill that determines its found to be 0.25 mm. The inclination angle
rake angle is of the main cutting edge is 10° . The back
rake angle (in degrees) is ___
(GATE - PI -15)
(a) Lip relief angle (b) Chisel edge angle
(c) Helix angle (d) Point angle (GATE - PI-16)

44. During turning of mild steel work material,


the maximum temperature is observed at
(GATE - PI -15)
(a) Primary deformation zone
(b) Tool and chip interface
(c) Tool-flank and work interface
(d) Machined sub-surface

45. Built-up edge formation decreases under the


conditions listed below EXCEPT
(GATE - PI -15)
(a) at low cutting speeds
(b) using large positive rake angle
(c) with shaper tool
(d) using cutting fluid

46. In an orthogonal cutting process the tool


used has rake angle of zero degree. The
measured cutting force and thrust force are

jltl1ih§jjji4ijjjiRftijiffli!ifjj+yderabad J Delhi J Bhopal J Pune J Bhubaneswar I Lucknow I Patna I Benga)uru I Chennai I Vtjayawada I Vizag I Tirupati I KukatpalJy I Kolkata I
: 666 : Production

04. A cutting tool has a nose radius of 1.8mm.


the feed rate for a theoretical surface
Two Marks Questions
roughness of Rt = 5microns is
(GATE-ME-97)
O1. Pure metal pose machinability problem in (a) 0.268 mm/rev (b) 0.187 mm/rev
turning operations. The reason is the (c) 0.036 mm/rev (d) 0.0187 mm/rev
(GATE-ME-88)
(a) Increased length of contact due to the 05. In orthogonal machining operation, the chip
production of continuous chip thickness and the uncut chip thickness are
(b) Susceptibility to chemical reactions equal to 0.45mm. if the tool rake angle is
(c) Tendency to form intense adhesion joint Odeg. The shear plane angle is
with the tool face (GATE-ME-98)
(d) Absence of inclusions which aids chip (a) 45 (b) 30 (c) 18 (d) 60
formation 06. What is the approximate % change in the
life, t, of the tool with zero rake angle used
02. In turning operation the feed rate could be in orthogonal cutting when its clearance
doubled to increase the metal removal rate. angle, a, is changed from 10 to 7 deg?
To keep the same level of surface finish, the (Hint: flank wear rate is proportional to cot a )
nose radius of the tool has to be (GATE-ME-99)
(GATE-ME-89) (a) 30% increase (b) 3 0% decrease
(a) Doubled (c) 70% increase (d) 70% decrease
(b) Halved
(c) Multiplied by 4 time 07. A conventional lathe and a CNC lathe are
(d) Kept unchanged. under consideration for machining a given
part. The relevant data are shown below
03. In a typical metal cutting operation, using a (GATE-ME-00)
cutting tool of positive rake y = 1Odeg, it Conventional CNC
Lathe Lathe
was observed that the shear angle was
Preparation Cost (Rs) 30 150
20deg. The friction angle is
Production time per 30 15
(GATE-ME-97) part (min)
(a) 45 (b) 30 (c) 60 (d) 40 Machine and operator 75 120
cost (Rs)

jlfllQjj§jjji4fjjjijmftiM\!11h� yderabad I Delhi I Bhopal I Pune I Bhubaneswar I Lucknow! Patna I Benga)uru I Chennai I Vijayawada I Vu.ag I Tuupati I Kukatpally I Kolkata
.. vli'.....:..-:....
ACE Publica6ons : 667 : Metal Cutting
�-�
� - �=.........-
==� =================================
The machine preferred for producing 100 1 1. In a machining operation, doubling the
pieces 1s cutting speed reduces the tool life to 118th of
(a) Conventional lathe (b) CNC lathe the original value. The exponent n in
(c) Any of the above (d) None Taylor's tool life equation VTn = C, is
(GATE-ME-04)
08. During orthogonal cutting of MS with a (a) 1 /8 (b) 1 /4
1Odeg rake angle tool, the chip thickness (c) 1 /3 (d) 1 /2
ratio was obtained as 0.4. The shear angle
(in degrees ) evaluated from this data is 12. The figure below shows a graph which
(GATE-ME-01) qualitatively relates cutting speed and cost
(a) 6.53 (b) 20.22 per piece produced. (GATE-ME-OS)
(c) 22.94 (d) 50.00

09. For turning NiCr alloy steel at cutting


Total Cost

speeds of 64m/min and 1 OOm/min, the


respective tool lives are l 5min and 12 min.
the tool life for a cutting speed of 1 44m/min
(GATE-ME-01)
(a) 8 min (b) 9 min
lS 2

(c) 10 min (d) 1 1 .5min


1
Cutting Speed

10. A batch of 1 0 cutting tools could produce


500 components while working at 50 rpm The three curves 1 , 2 and 3 respectively
with a tool feed of 0.25 mm/rev and depth represent
of cut of 1 mm. A similar batch of 10 tools (a) Machining cost, non-productive cost,
of the same specification could produce 122 tool changing cost
components while working at 80 rpm with a (b) Non-productive cost, machining cost,
feed of 0.25 mm/rev and 1 mm depth of cut. tool changing cost
How many components can be produced (c) Tool changing cost, machining cost,
with one cutting tool at 60 rpm ? non-productive cost
(GATE-ME-03) (d) Tool changing cost, non-productive
(a) 29 (b) 3 1 (c) 37 (d) 42 cost, machining cost

!M1l@@i!i@.jj/4Qflaj@ij.jjj.� yderabad I Delhi I Bhopal I Pune I Bhubaneswar I Lucknow I Patna I Bengaluru I Chennai I Vijayawada j V=g I Tirupati I Kukatpally I Kolkata
·• ACE
�� ..E�=L�'l111�·-�·���J:
;em11 bliadioos 6=
· :·�=========:=6=
� =
8=:==============Pro= =n
=c
du = n
·o=

13. Two tools P and Q have signature 5 ° -5 ° -6 ° 16. The percentage of total energy dissipated
-6 ° -8 ° -30 ° -0 and 5 ° -5 ° -7 ° -7 ° -8 ° - 15 ° -0 due to friction at the tool chip interface is
(both ASA) respectively. They are used to (GATE-ME-06)
tum components under the same machining (a) 30% (b) 42%
conditions. If hp and hQ denote the peak-to­ (c) 58% (d) 70%
valley heights of surfaces produced by the
tools P and Q, the ratio hp / hQ will be Statement for Linked Answer Ql 7 & Q18
(GATE-ME-OS) A Low carbon steel bar of 147mm diameter
with length of 630mm is being turned with
tan8 ° +cot15 ° tan 15 ° +cot8 °
(a) (b) uncoated carbide insert. The observed tool
tan8 ° +cot 30 ° tan30 ° +cot8 °
lives are 24 and 12 for cutting velocities of
tan 15 ° +cot 7 ° tan 7 ° +cot 15 °
(c) (d) 90m/min and 120m/min respectively. The
tan 30 ° +cot 7 ° tan 7 ° +cot 30 °
feed and depth of cut are 0.2mm/rev and
2mm respectively. Use the unmachined
Common Data for Questions 14, 15 & 16: diameter to calculate the cutting velocity.
In an Orthogonal cutting operation
Uncut chip thickness = 0.5 mm 17. When tool life is 20min. the cutting velocity
Cutting speed = 20 m/min in m/min is (GATE-ME-07)
Width of cut = 5 mm (a) 87 (b) 97 (c) 107 (d) 1 14
Chip thickness = 0.7 mm
Thrust force = 200 N 18. Neglect over travel or approach of the tool.
Cutting force = 1200 N When tool life is 20min, the machining time
Assume merchants theory in min for a single pass is (GATE-ME-07)
(a) 5 (b) 10 (c) 15 (d) 20
14. The values of shear angle and shear strain,
respectively are (GATE-ME-06)
19. In orthogonal turning of a low carbon steel
(a) 30.0 and 1.98 (b) 30.3 and 4.23
bar of diameter 150mm with uncoated
(c) 40.2 and 2.97 (d) 40.2 and 1.65
carbide tool, the cutting velocity 1s
15. The coefficient of friction at the tool chip 90m/min. the feed is 0.24mm/rev and the
interface is (GATE-ME-06) depth of cut is 2mm. The chip thickness
(a) 0.23 (b) 0.46 obtained is 0.48mm. if the orthogonal rake
(c) 0.85 (d) 0.95

!IWIM@hi§§jjji4Rflftj@mih� yderabad I Delhi I Bhopal I Pune I Bhubaneswar I Lucknow ! Patna I Bengaluru I Chennai I Vijayawada I Vi7.ag I Tirupari I Kukatpally I Kolkata
-:,�-:�Pnblicdiooa
AJCE : 669: Metal Cutting
� ==================================
" \I • • • •

Statement for Linked Answer Q23 & Q24


J J

angle is zero, and the principle cutting edge


angle is 90, the shear angle in degrees is Orthogonal turning is performed on a cylindrical
(GATE-ME-07) work piece with shear strength of 250Mpa. The
(a) 20.56 (b) 26.56 following conditions are used : cutting velocity is
(c) 30.56 (d) 36.56 180m/min, feed is 0.2mm/rev, depth of cut is
3mm, chip thickness ratio is 0.5. the orthogonal
20. In orthogonal turning of medium carbon rake angle is 7deg. Apply Merchants theory for
steel, the specific machining energy is 2 analysis,
kJ/mm , the cutting velocity, feed and depth
3

of cut are 120m/min, 0.2mm/rev and 2mm 23. The shear plane angle (in degrees) and the
respectively. The main cutting force N is shear force respectively are
(GATE-ME-07) (GATE-ME-08)
(a) 40 (b) 80 (a) 22.65 ; 150N (b) 22.65; 320N
(c) 400 (d) 800 (c) 28; 400N (d) 28; 320N

21. In orthogonal turning of low carbon steel


24. The cutting and frictional forces
(GATE-ME-08)
pipe with principal cutting edge angle of 90,
respectively are
the main cutting force is 1OOON and the feed
force is 800N. the shear angle is 25 and (a) 568N ; 387N (b) 565N ; 381N
orthogonal rake angle is zero. Employing (c) 202N; 120N (d) 202N; 356N

Statement for Linked Answer Q25 & Q26


Merchants theory, the ratio of friction force
to the normal force acting on the cutting tool
lS (GATE-ME-07) In machining experiment, tool life was found to
(a) 1.56 (b) 1.25 vary with the cutting speed in the following
(c) 0. 80 (d) 0.64 manner

Cutting speed (m/min) Tool life (min )


22. In a single point turning tool, the side rake
60 81
angle and orthogonal rake angle are equal. cp
90 36
is the principle cutting edge angle and its
range is O to 90. The chip flows in the
25. The exponent (n) and constant (K) of the
orthogonal plane. The value of cp is closer to
(GATE-ME-08)
(GATE-ME-09)
Taylor's tool life equation are

(a) 0 (b) 45 (c) 60 (d) 90


(a) n = 0.5, and K = 540

!ltl1ii@jjj§4ijjjjRflft1ij..jjjilj yderabad I Delhi I Bhopal I Pune I Bhubaneswar I Lucknow I Patna I Bengahuu I Chennai I Vtjayawada j Vmg I Tuupari I Kukatpally I Kolkata
"...
. ¢ .....a.a" . ACE : 670 : Production
'!. �� Pnblicdiooa
. . .
� "=======================================
(b) n = 1.0, and K = 4860 30. In a cutting test with 0.3mm flank wear as
(c) n = - 1.0 , and K = 0.74 tool failure criterion, a tool life of 1Ornin
(d) n = - 0.5, and K = 1. 155 was obtained at a cutting velocity of
20m/min. taking tool life exponent as 0.25,
26. What is percentage increase in tool life the tool life in minutes at 40m/rnin of
when the cutting speed is halved. cutting velocity will be (GATE-PI-93)
(GATE-ME-09) (a) 0.506 (b) 0.625
(a) 50% (b) 200% (c) 3. 140 (d) 5.002
(c) 300% (d) 400%
3 1. The heat generated in metal cutting 1s
27. For tool A Taylor's tool life exponent (n) is
dissipated in different proportions into
0.45 and constant (K) is 90. similarly for
environment, tool, chip and work-piece. The
tool B, n 0.3 and K= 60. the cutting speed
correct order of this proportion m
=

(in m/min ) above which tool A have a


decreasing magnitude is (no cutting fluid is
higher tool life than tool B is
use (d) (GATE-PI-94)
(GATE-ME-10)
(a) tool, work, chip, environment
(a) 26.7 (b) 42.5
(b) work, tool, chip, environment
(c) 80.7 (d) 142.9
(c) tool, chip , environment, work
28. A single-point cutting tool with 12° rake (d) chip, tool work environment
angle is used to machine a steel work-piece.
The depth of cut, i.e uncut thickness is 0.8 1 32. A 60deg symmetrical V tool is used in
mm. The chip thickness under orthogonal shaping a work piece with a depth of cut of
machining condition is 1.8 mm. The shear 0. 1 mm and feed of O . 1 mm/ stroke. The
angle is approximately (GATE-ME-1 1) theoretical peak to valley height, in mm, of
(a) 22° (b) 26 ° (c) 56 ° (d) 76 ° the surface produced is
(GATE-PI-95)
29. A single point cutting tool with 12 rake (a) 0.043 (b) 0.067
°

angle is used for orthogonal machining of a (c) 0.086 (d) none


ductile material. The shear plane angle for
the theoretically minimum possible shear 33. Two different tools A and B having nose
strain to occur (GATE-PI-90) radius of 0.6mm and 0.33mm respectively
(a) 5 1 (b) 45 (c) 30 (d) none are used to machine C-45 steel employing

!ltllii@jjj§§.jj40fftjM\jjjjj� yderahad l Delhi f Bhopal l Pune l Bhubaneswarl Lucknow l Patna l Bengalwu ! Chennai l Vijayawada ! Vizag I Trrupati I Kukatpa)ly l Kolkata
':i ,... �PnNicdiooa
...
.,,"" .......,
" . ACE
. . . : 671 : Metal Cutting

feed rate of 0.2 mm/rev and O.lmm/rev 36. Orthogonal machining of a steel work-piece
respectively. The tool that gives better finish is done with a HSS tool of zero rake angle.

(GATE-PI-02)
and the value of ideal surface roughness are The ratio of the cutting force and the thrust
force on the tool is 1:0.372. The length of
(a) Tool A and 4.166 µm respectively cut chip is 4.71 mm while the uncut chip
(b) Tool B and 3.78 µm respectively length is 10 mm. What are the shear plane

(GATE-PI-03)
(c) Tool A and 8.333 µm respectively angle � and friction angle B in deg.? Use
(d) Tool B and 8.333 µm respectively Merchant's theory.
(a) 32.49,10.22 (b) 25.22,20.41
34. Tool life equations for two tools under (c) 64.78,20.41 (d) 25.22,23.21

(GATE-PI-03)
consideration are as follows
37. A cylinder of 25 mm diameter and 100 mm
length is turned with a tool, for which the
HSS : VTo.2 = 150
relation VT0·25= 55 is applicable. The
Carbide : VT0.3 = 250
cutting velocity is 22m/min. For a tool feed
Where V is the cutting speed in m/min and
of 0.046 mm/rev, the number of tool
T 1s the tool life m mm. the

(GATE-Pl-03)
regrinds required to produce 425 cylinders
breakeven cutting speed above which the
1s
carbide tool will be beneficial is
(a) 12 (b) 22 (c) 43 (d) 85
(a) 54 m/min (b) 150 m/min
(c) 194 m / min (d) 250 m/min 38. In which of the choices given below, the
cutting tool materials are placed in the
35. A single point cutting tool with a nose
(GATE-PI-04)
ascending order of permissible cutting speed
radius of 0.4 mm was used to turn a for machining of steel ?
component in a lathe employing a feed rate (a) K Group Cemented Carbide-P Group
of 0.3 mm/rev. if the feed-rate is doubled, Cemented Carbide-Ceramic -Cubic
the ideal surface roughness (peak-to-valley Boron Nitride
height ) produced on the component will (b) Cubic Boron Nitride - K Group

(GATE-PI-03)
increase by a factor of Cemented Carbide - P Group Cemented
Carbide - Ceramic
(a) 2 (b ) 4 (c) 8 (d) 16 (c) P Group Cemented Carbide - K Group
Cemented Carbide - Cubic Boron
Nitride - Ceramic
!1S•li@Oii4.jji4Pffbiihiih
+Yderabad I Dellii I Bhopal I Pune I Bhubaneswar I Lucknow I Patna I Benga)uru I Chennai I Voayawada j Vmg I Tuupali I Knkatpally I Kolkata I
ACE Production
�.�-�
E�Pub1iatioos :672:
� � ===========================================
(d) Ceramic - Cubic Boron Nitride - K 41. In a machining operation chip thickness
Group Cemented Carbide - P Group ratio is 0. 3 and the back rake angle of the
Cemented Carbide tool is 10 ° . What is the value of the shear
strain? (GATE-PI-OS)
39. Two identical cylindrical jobs are turned (a) 0.31 (b) 0.13 (c) 3.00 (d) 3. 34
using (a) a round nosed tool of nose radius 2
mm and (b) a sharp comer tool having 42. Consider the following statements
During the third stage of tool-wear, rapid
principal cutting edge angle = 45 ° and
deterioration of tool edge takes place
(GATE-PI-OS)
auxiliary cutting edge angle = 10 ° . If the
because
operation is carried out at a feed of 0.08
1. Flank wear is only marginal.
mm/rev, the heights of micro irregularities
2. Flank wear is large.
on the machined surfaces (in mm) in the two
(GATE-PI-04)
3. Temperature of the tool mcreases
cases will be
gradually.
(a) 0.0001, 0.024 (b) 0.0002, 0.012
4. Temperature of the tool mcreases
(c) 0.0003, 0.024 (d) 0.0004, 0.012
drastically.
Which of the statements given above are
40. Consider the following statements with
correct ?
respect to the relief angle of cutting tool
(GATE-PI-04)
(a) 1 & 3 (b) 2 & 4 (C) 1 & 4 (d) 2 & 3

1. This affects the direction of chip flow.


43. Match List - I with List - II and select the
2. This reduces excessive friction between
correct answer using the codes given below:
(GATE-PI-OS)
the tool and work piece
3. This affects tool life List- I List- II
4. This allows better access of coolant to A. Plan approach angle 1. Tool face
the tool-work piece interface B. Rake angle 2. Tool flank
Which of the statements given above are C. Clearance angle 3. Tool face and
correct? flank
(a) 1 and 2 (b) 2 and 3 D. Wedge angle 4. Cutting edge
(c) 2 and 4 (d) 3 and 4
5. Tool nose

llfl1i@i!ii4ijjj$@ijjM\@j.iHyderabad I Delhi I Bhopal I Pune I Bhubaneswar I Lucknow I Patna I Bengaluru I Chennai I Vrjayawada I Vizag I Tirupati I Kukatpally I Kolkata I
� ACE
�li'....:.....:.... : 673: Metal Cutting
�-�
� �=� ===�==p.,Niariooa
===============================
Codes: 1. As the cutting speed increases, the cost
(a) A-1 ' B-4' C-2 ' D-5 of production initially reduces, then after
(b) A-4' B-1 ' C-3' D-2 an optimum cutting speed it increases.
(c) A-4 ' B-1 ' C-2 ' D-3 2. As the cutting speed increases the cost
(d) A-1' B-4 ' C-3 ' D-5 of production also increases and after a
critical value to it reduces.
44. Match List - I (Cutting tool materials) with 3. Higher feed rate for the same cutting
List-II (Manufacturing methods) and select speed reduces cost of production.
the correct answer using the codes given 4. Higher feed rate for the same cutting
below the lists: (GATE-PI-05) speed increases the cost of production
List- I List- II Which of the statements given above are
A. HSS 1. Casting correct ?
B. Stellite 2. Forging (a) 1 and 3 (b) 2 and 3
(c) 1 and 4 (d) 3 only
C. Cemented carbide 3. Rolling
D. UCON 4. Extrusion 47. Assertion (A): The ratio of uncut chip
5. Powder metallurgy thickness to actual chip thickness is always
Codes: less than one and is termed as cutting ratio
(a) A-3 ' B-1 ' C-5 ' D-2 in orthogonal cutting
(b) A-2, B-5, C-4, D-3 Reason (R): The frictional force is very
(c) A-3, B-5, C-4, D-2 high due to the occurrence of sticking
(d) A-2' B-1' C-5' D-3 friction rather than sliding friction.
(GATE-PI-05)
The rake angle of a cutting tool is 15° , shear
Statement for Linked Answer Q48 & Q49
45.
angle 45° and cutting velocity 35 m/min.
What is the velocity of chip along the tool A batch of 500 jobs of diameter 50 mm and
face? (GATE-PI- 05) length 100 mm is to be turned at 200 rev/min and
(a) 28.5 m/min (b) 27. 3 m/min feed 0.2 mm/rev.
(c) 25. 3 m/min (d) 23.5 m/min
48. Applying Taylor's equation VT0·25 = 160,
the tool life in minutes is (GATE-PI-05)
46 Consider the following statements
(GATE-PI-05)
(a) 20. 36 (b) 22.43
(c) 674 (d) 28.20

ilffilj@jj/§4,ijjjRflni@jn.jj� yderabad I Delhi I Bhopal I Pune I Bhubaneswar l Lucknow! Patna I Bengaluru I Chennai I Vtjayawada ! Vizag J Tuupari I KukatpallyJ Kolkala
ACE
��-� 67 4 Production
.. ..:E=&�'Alll�·-
:eaq
:·���b�•�
rati�a11
::..::=======:::::��;�:=============���:���

49 . The number of components per tool life 53. If the cutting force and the thrust force are
(GATE-PI-05) 900N and 8 1 ON, the mean strength in Mpa
(a) 138 (b) 270 (c) 62 (d) 6 1 (a) 137.94 (b) 477.91
(c) 500.58 (d) 635 . 84
Statement for Linked Answer Q 50 & Q 51
A � 40 mm job is subjected to orthogonal turning Common Data for Question 54 & 55
by a +10 rake angle tool at 500 rev/min. By
°
In an orthogonal cutting test, the following
direct measurement during the cutting operation, observations were made
the shear angle was found equal to 25 . °
Cutting force = 1200N; Thrust force = 500N
Tool rake angle = zero; Cutting speed = 1 mis,
50. The velocity (in m/min) with which the chip
Depth of cut = 0.8mm;
flows on the tool face is (GATE-PI-05) Chip thickness = 1.5 mm
(a) 32 (b) 63 (c) 22 (d) 27.5

54. Friction angle during machining will be


5 1. If the friction angle at the tool-chip interface
(GATE-PI-07)
is 58 ° 10' and the cutting force components
(a) 22.6 (b) 32.6 (c) 57. 1 (d) 67.4
measured by a dynamometer are 600 N and
200 N, the power loss due to friction (in
55. Chip speed along the tool rake face will be
kNm/min) is approximately
(GATE-PI-07)
(GATE-PI-05)
(a) 0. 83m/s (b) 0.53m/s
(a) 20 (b) 18 (c) 16 (d) 350
(c) 1.2m/s (d) l .88m/s

Statement for Linked Answer Q 52 & Q 53


Statement for Linked Answer Q56 & Q57
The following data relate to an orthogonal
In an orthogonal cutting experiment, an HSS tool
turning process:
having the following tool signature in the
Back rake angle = 15deg,
orthogonal reference system (ORS) has been
Width of cut = 2mm ,
used: 0- 10-7-7- 10-75- 1
Chip thickness = 0.4mm,
Given : Width of cut = 3.6mm;
Shear strength of work piece material
(GATE-PI-05)
Feed rate = 0.2mm/rev.
=

460N/mm2 '•
52. The shear angle is Depth of cut = 0.25mm;
(a) 14 ° (b) 25 ° (c) 29° (d) 75 ° Coefficient of friction at chip tool interface = 0.7.

!ltl#Qjj§jjji4.jjjj@fiiiihijj+yderabad I Delhi I Bhopal I Punc I Bhubaneswar I Lucknow I Patna I Bengaluru I Chcnnai I Vliayawada I V1Zag I Tuupati I Kukatpally I K.olkata I
:�.�="t�
" Publications
ACE
====�==================================
1, J.'....:..-.:.... : 675 : Metal Cuttrng
·

56. Shear plane angle (in degrees) for minimum 60. In an orthogonal machining operation, the
cutting force is (GATE-PI-08) tool life obtained is 1Omin at a cutting speed
(a) 20.5 (b) 24.5 (c) 28.6 (d) 32.5 of l OOm/min, while at 75m/min cutting
speed, the tool life is 30min. the value of
57. Minimum Power requirement ( in KW ) at a index (n) in the Taylor's tool life equation is
cutting speed of 150m/min is (GATE-PI-09)
(GATE-PI-08) (a) 0.262 (b) 0.323
(a) 2.6 (b) 3.25 (c) 3.35 (d) 3.45 (c) 0.423 (d) 0.52 1

58. During machining, the wear land (h) has Common Data for Question 61 & 62
been plotted against machining time (T) as An orthogonal operation is carried out at 20m/min
given in the following figure. cutting speed, using a cutting tool of rake angle
(GATE-PI-08) 15deg. The chip thickness is 0.4mm and uncut
chip thickness is 0.2mm.

6 1. The shear angle ( in degrees ) is


(GATE-PI-09)
h(rnm)

(a) 26.8 (b) 27.8 (c) 28.8 (d) 29.8

62. The chip velocity ( in m/min ) is

For a critical wear land of 1.8mm, the (GATE-PI-09)


1 0 T (minute) 60

(a) 8 (b) 10 (c) 12 (d) 14


cutting tool life (in min) is
(a) 52.00 (b) 5 1.67 63. During turning of a low carbon steel bar
(c) 5 1.50 (d) 50.00 with TiN coated carbide insert, one needs to
improve surface finish without sacrificing
59. Brittle materials are machined with tools material removal rate. To achieve improved
having zero or negative rake angles because surface finish one should (GATE-PI-10)
it (GATE-PI-08) (a) Decrease nose radius of the cutting tool
(a) Results in lower cutting force and increase depth of cut
(b) Improves surface finish (b) Increase nose radius of the cutting tool
(c) Provides adequate strength to cutting (c) Increase feed and reduce nose radius of
forces the cutting tool
(d) Results in more accurate dimensions (d) Increase depth of cut and increase feed

iMlli@hi44eh4@Cfti\iiJjj� yderabad I Delhi I Bhopal I Pune I Bhubaneswar I Lucknow I Patna I Bengaluru I Chennai I Vgayawada I Vmg I Tuupati I Kukatpally I Kolkata
: 676 : Production

64. Following data refers to an automat and a 66. Two cutting tools are being compared for a
center lathe, which are being compared to machining operation. The tool life equations
machine a batch of parts in a manufacturing are : (GATE-ME & PI-13)
(GATE-PI- 13) Carbide tool: VT . = 3000
1 6
shop
6
HSS tool : VT0 · =200
Center Where V is the cutting speed in m/min and
Automat
Lathe
T is the tool life in min. The carbide tool
Machine Set-up Time in min 1 20 30
Machine Set-up Cost in Rs/min 800 1 50 will provide higher tool life if the cutting
Machining Time per piece speed in m/min exceeds
2 25 (a) 15.0 (b) 39.4
in min
Machining Cost in Rs/min 500 100 (c) 49. 3 (d) 60.0

Automat will be economical if the batch size Common Data for Question 67 & 68
exceeds In orthogonal turning of a bar of 100 mm
(a) 28 (b) 32 (c) 61 (d) 75 diameter with a feed of 0.25 mm/rev, depth
of cut of 4 mm and cutting velocity of 90
65 Details pertaining to an orthogonal metal m/min, it is observed that the main
cutting process are given below. (tangential) cutting force is perpendicular to
Chip thickness ratio 0.4 the friction force acting at the chip-tool
Under formed thickness 0.6mm interface. The main (tangential) cutting
Rake angle+1o0 force is 1500 N.
Cutting speed 2.5m/s
Mean thickness of pnmary shear zone 67. The orthogonal rake angle of the cutting tool
25microns in degree is (GATE-ME & PI-13)
The shear strain rate m s- 1 during the (a) zero (b) 3.58
process 1s (c) 5 (d) 7.16
(GATE-ME&PI-13)
(a) O . l 781x105 (b) 0.7754 x105 68. The normal force acting at the chip-tool
(c) 1.0104x105 (d) 4.397x 105 interface in N is (GATE-ME & PI-13)
(a) 1000 (b) 1500
(c) 2000 (d) 2500

!ltlli@jjji4ijj§Rflft1Midiii� yderabad I Delhi I Bhopal I Pune I Bhubaneswar I LJ.Jcknow I Palm I Benga)uru I Chennai I Vuayawadal Vu.ag I Tuupati I Kukatpally I Kolkata
: 677 : Metal Cutting

69. During pure orthogonal turning operation of 72. An orthogonal turning operation is carried
a hollow cylindrical pipe, it is found that the out under the following conditions; rake
thickness of the chip produced is 0.5mm. angle = 5 °, spindle rotational speed = 400
The feed given to the zero degree rake angle rpm, axial feed = 0.4 m/min and radial depth
tool is 0.2mm/rev. The shear strain of cut = 5mm. The chip thickness , tc, is
produced during the operation is ___ found to be 3mm. The shear angle (in
(GATE-ME-14-SET-1) degrees) in this turning process is __
(GATE-15-Set 1)
70. Which pair of following statement is correct
for orthogonal cutting using a single-point 73. A single point cutting tool with o0 rake
cutting tool? (GATE-ME-14-SET-3) angle is used in an orthogonal machining
P. Reduction m friction angle mcreases process. At a cutting speed of 180 m/min the
cutting force thrust force is 490N. If the coefficient of
Q. Reduction in friction angle decreases friction between the tool and the chip is 0.7,
cutting force Then the power consumption (in kW) for
R. Reduction in friction angle mcreases the machining operation is _
chip thickness (GATE-15-Set 2)
S. Reduction in friction angle decreases
chip thickness 74. Orthogonal turning of a mild steel tube with
(a) P and R (b) P and S a tool of rake angle 1 o0 is carried out at a
(c) Q and R (d) Q and S feed of 0.14 mm/rev. If the thickness of the
chip produced is 0.28mm, the values of
7 1. A cast iron block of 200 mm length is being shear angle and shear strain will be
shaped in a shaping machine with a depth of respectively. (GATE-15 -Set 3)
cut of 4 mm, feed of 0.25 mm/stroke and the (a) 28°20' and 2.19
tool principal cutting edge angle of 30 °.
(b) 22°20' and 3.53
Number of cutting strokes per minutes is 60.
Using specific energy for cutting as (c) 24°30' and 4.19
l .49J/mm3 , the average power consumption (d) 37°20' and 5. 19
(in Watt) is ---
(GATE-ME- 14-SET-4)

lliili@ih44•hi4Rftafti1U,jj� yderabad I Delhi I Bhopal I Pune I Bhubaneswar I Lucknow I Patna I Bengaluru I Chennai I Vijayawacla I Vizag I Tirupati I Kukatpally I Kolkata
....:..-:-Public:aliooa
��•�E!....� ACE : 678 : Production
��� ��::�=============================
75. In an orthogonal machining experiment (a) 19.24 (b) 29. 70
carried out using a cutting tool with zero (c) 56.00 (d) 68.75
degree rake angle, the measured cutting
79. In a single point turning operation with
force was 1700 N. If the friction angle at the
cemented carbide tool and steel work piece,
rake face-chip interface is 26 ° , then the it is found that the Taylor's exponent is
thrust force value, in N is _____ 0.25. If the cutting speed is reduced by 50%
(GATE - PI-15) then the tool life changes by
times. (GATE- 16- SET- 3)
76. The too life equation for HSS tool is
VTo .1 4f- 7do.4 = Constant. The tool life (T) of 80. A cylindrical bar of 100 mm diameter is
30 min is obtained using the following orthogonally straight turned with cutting
cutting conditions = 45 m/min, f = 0.35 mm, velocity, feed and depth of cut of 120
d = 2.0 mm. If speed (V), feed (f) and depth m/min, 0.25 mm/rev and 4 mm,
of cut (D) are increased individually by respectively. The specific cutting energy of
25%, the tool life (in min) is the work material is 1 x 109 J/m3 . Neglect the
(GATE - 16- SET - 1) contribution of feed force towards cutting
(a) 0.15 (b) 1.06 power. The main or tangential cutting force
(c) 22.50 (d) 30.0 (in N) is ___ (GATE - PI-16)

77. For a certain job, the cost of metal cutting is 8 1. A 60 mm wide block of low carbon steel is
Rs. 18CN and the cost of tooling is Rs. 270 face milled at a cutting speed of 120 m/min,
C/(TV). Where C is a constant, V is the feed of 0. 1 mm/tooth and axial depth of cut
cutting speed in m/min and T is the tool life of 4 mm. A schematic representation of the
in minutes. The Taylor's tool life equation is face milling process is shown below. The
VT0·25 = 150. The cutting speed (in m/min) diameter of the cutter is 120 mm and it has
for the minimum total cost is _ _ 12 cutting edges. The material removal rate
(GATE- 16- SET-2) (in mm3 /s) is _ _ (GATE - PI-16)

78. For an orthogonal cutting operation, tool


q> 1 20 mm

material is HSS, rake angle is 22 ° , chip


thickness is 0.8 mm, speed is 48 m/min and
feed is 0.4 mm/rev. The shear plane angle
work piece

(in degrees) is (GATE - 16- SET -3)


cutter
schematic representation (top view)

!IDIIM@hi§§ihQRflCIM\iijjj� yderabad Dellii Bhopal Pune Bhubaneswar Lucknow Pama Bengaluru Chennai Vtjayawada Vizag TJJUpati
I I I I I I I I I I I I Kukatpally I Kolkata
ACE : 679 : Metal Cutting
�..�-�
... ,,..�&giom'
� Pubtic:atiooa

82. Two cutting tools with tool life equations T = C V -2.S f --0.9 d--0.IS
given below are being compared: where, C is a constant. The suggested
Tool 1: VT0 · 1 =150 values for the cutting parameters are: v=1.5
Tool 2: VT03 = 300 mls, f= 0.25 mm and d = 3 mm for normal
where V is cutting speed in m/minute and T rough turning. If the operation is performed
is tool life in minutes. The breakeven at twice the cutting speed and the other
cutting speed beyond which Tool 2 will have parameters remam unchanged, the
a higher tool life is ___ m/minute. corresponding percentage change in tool life
(GATE- 17- SET- l) IS ------ (GATE-PI- 17)

83. During the turning of a 20 mm-diameter 86. The Merchant circle diagram showing
steel bar at a spindle speed of 400 rpm, a tool various forces associated with a cutting
life of 20 minute is obtained. When the process using a wedge shaped tool is given
same bar is turned at 200 rpm, the tool life in the Figure
becomes 60 minute. Assume that Taylor's
tool life equation is valid. When the bar is
turned at 300 rpm, the tool life (in minute) is -r--�---­
(GATE - 17 - SET - 2)
Uncut chip
thickness
approximately
(a) 25 (b) 32 (c) 40 (d) 50

84. In an orthogonal machining with a tool of 9°


orthogonal rake angle, the uncut chip
thickness is 0.2 mm. The chip thickness
fluctuates between 0.25 mm and 0.4 mm. The coefficient of friction can be estimated
The ratio of the maximum shear angle to the from the ratio (GATE-PI- 17)
f
(b) 3
minimum shear angle during machining is
(GATE- 17 - SET - 2) f4

85. In a machining operation with turning tool,


the tool life (1) is related to cutting speed v
(mis), feed / (mm) and depth of cut d (mm)
as

!lilii!@jjji4jjjjQJMbiM\hih�)Hyderabadl Delhi ! Bhopal I Pune I Bhubaneswar l Lucknow ! Patna! Benga)uru l Oiennai I V\iayawadaJVu.ag I T=pati I Kukatpally J Kolkata I
.!'"...t..w F� ACE : 680: Production
..
• •
PnJiliratiool
• •

04. The following data 1s available for


Five Marks Questions machining a component on a turret or center
lathe: find out the total number of
01. During orthogonal turning of a steel rod by components to be machined to justify the
zero rake tool at feed 0.25mm/rev and depth use of turret lathe (GATE-ME-89)
of cut 2.0mm, the following observations
TURRET CENTER
were obtained. Tangential component of
LATHE LATHE
cutting force =
1000N, Axial component of
Machining time
cutting force = SOON, chip thickness = 4min 14min
per piece
1.0mm. with the help of diagram determine
Operator wages
the yield shear strength of work material 2.0 4.5
(Rs/hr)
under the given cutting conditions.
Machine overheads
(GATE-ME-87) 4.0 2.5
(Rs/hr)
02. In a turning operation the tool life of the Set up cost
5.0 4.5
carbide tool was found to be 20min and (Rs/hr)
1OOmin, at cutting speeds of 120m/rnin and Set up time 3hrs/batch 2hrs / batch
80m/min respectively. What will be the too
life of the tool under same condition but at a 05. Two cutting tools are being tried for an
cutting speed of 1OO m/min. operation. Tailors tool life equation for them
(GATE-ME-87) are as follows (GATE-ME-89)
01 =
HSS tool : VT · 200
03. In an orthogonal cutting test on the Carbide tool : VT0.3 5 = 500
following data were obtained Find out the break even speed above which
Uncut chip thickness = 0.25mm, Cutting the carbide tool will be economical.
speed : 60 m/min, Tool rake angle: 0 Chip
thickness 0.75 mm, Cutting force : 900 N, 06. A tube is orthogonally machined in lathe to
Thrust force : 450 N reduce its length under the following
Calculate the shear angle, total power in conditions. (GATE-ME-90)
making the cut and coefficient of friction Outside diameter of the tube : 100 mm ,
between the chip tool interface. Inside diameter of the tube = 96 mm
(GATE-ME-89) RPM of the work piece = 120,
Longitudinal feed = 0.5mm/rev
!11•1i@hiiiihi4RflbifW!iijj� ydcrabad I Delhi I Bhopal I Punc I Bhubaneswar I Lucknow I Pama I Bcnga)uru I Chennai j Vtjayawada I V17.ag I T1IUpati I Kukatpa]]y I Kolkata
: 681 : Metal Cutting

Cutting ratio=0.3 , 1 0. In an orthogonal cutting operation on a work


Tangential force = 800N, piece of width 2.5mm, the uncut chip
Axial force=600N thickness was 0.25 mm and the tool rake
Calculate the chip velocity in m/rnin and the angle was O (zero) degrees. It was observed
total power consumption in KW that the chip thickness was 1 .25mm. the
cutting force was measured to be 900N and
07. Determine the Merchants constant ' C ' the thrust force was found to be 450N. find
(shear angle relation) for aluminium from the mean shear strength of the work piece
the following orthogonal machining data. material and if the coefficient of friction
rake angle 35° and an uncut chip thickness between the chip and the tool was 0.5, what
0. 1 5mm, the values of F e and F1 are found to is the machining constant.
be 200 and 90 N respectively. The average (GATE-ME-92)
chip thickness is also measured and found to
be 0. 3 mm, width of cut 2.5mm and cutting 1 1 . A single point cutting tool made of HSS has
velocity 30m/min. (GATE-ME-91) the value of constant 'C' 80, and n = 0.2 in
the basic tool life equation. If the tool cost
08. Calculate the MRR and Specific cutting per regrind is Rs 2 and the machine hour
pressure for the following cutting conditions rate is Rs. 30, determine the most
Work material : steel, UTS=980MPA, economical cutting speed( tool cost includes
Tool material : HSS, the cost of time spent on changing.
Depth of cut : 1 .6mm (GATE-ME-94)
Feed : 0. 8mm/rev
Cutting speed =5.5m/min and power 1 2. A mild steel block of width 40mm is being
consumed=0.67 kW (GATE-ME-91) milled using a straight slab cutter 70mm
diameter with 30teeth. If the cutter rotates at
09. Find the percentage change in cutting speed 40rpm and the depth of cut is 2mm,
required to give a 50% reduction in tool life determine the value of maximum uncut chip
(that is required tool life is half of the thickness when the table feed is 20mm/rnin
original tool life) when the value of the tool (GATE-ME-94)
life exponent n=0.1 25 or 1 /8.
(GATE-ME-91)

jlJ1i4j/§jjji4@$PflblM\h1jj.ydcrabad I Delhi I Bhopal I Punc I Bhubancswar l Lucknow ! Patna I Bcngaluru I Chennai I V\iayawada I Vuag I Tirupati I Kukatpally I Kolkata I
ACEPuNic:aaions : 682 : Production
�-�
� �=�
::
==�
=================================
�li'....:..._.:_

1 3. A single point turning tool is designated as 17. In a turning trail using orthogonal tool
10° - 12 ° - 7 ° - 5 ° - 20° - 50° - 0° (0RS) The geometry , a chip length of 84mm was
values of normal rake and normal clearance obtained for an uncut chip length of
of the above mentioned tool.. ... 200mm. the cutting conditions were
(GATE-ME-95) V= 30m/min, t1 = 0.5mm, rake
angle=20deg, cutting tool is HSS.Estimate
14. If under a condition of plain turning the life shear plane angle, chip thickness and the
of the cutting tool decreases by 50% due to shear plane angle for minimum chip strain.
increase in the cutting velocity by 20%, then (GATE-ME-97)
what is the % increase in tool life due to
reduction in the cutting velocity by 20% 18. A Throwaway carbide insert was used to
from its original value. machine a steel work pieces with a cutting
(GATE-ME-95) speed of 60 m/min, tool life of 40minutes
was observed, when the cutting speed was
15. While turning a Cl5 steel of 160mm increased to 100 m/min, the tool life
diameter at 2 15rpm, 2.5mm depth of cut and decreased to 10 minutes. The cutting speed
feed of O. l 6mm/rev by a tool of geometry 0- for maximum productivity, if tool change
10-8-9- 15-75-0mm, the following time is 2 minutes is
observations were made. Tangential (GATE-ME-97)
component of the cutting force = 500N,
axial component of the cutting force = 19. In an orthogonal cutting experiment with a
200N, chip thickness = 0.48mm. Determine tool of rake angle 7deg, the chip thickness
the dynamic shear strength of the work was found to be 2.5mm when the uncut
piece material. (GATE-ME-95) chip thickness was set to 1 mm , find the
shear angle and find the friction angle p
16. Tool life of a 10 hours is obtained when
<I>

assuming that Merchant's formula holds


cutting with a single point tool at 63m/min. good. (GATE-ME-99)
if Taylor's constant C=257.35, tool life on
20. The lives of two tools A & B , governed by
(GATE-ME-96) the equation VT0· 1 25 2.5 and VT0 ·25 7
doubling the velocity will be

(a) 5 hours (b) 25.7 min


= =

respectively in certain machining operation


(c) 38.3min (d) unchanged where V is the cutting velocity in mis and T

!111•i!.Y!ili410!401bi@!IIO*)ttydcrabad I Delhi I Bhopal I Punc I Bhubaneswar I Lucknow I Patna I Bengaluru I Chennai IVuayawada IV17.3g ITll\lpali I Kukatpa))y I Kolk.at, I
: 683 : Metal Cutting

is the tool life in sec. Find out the speed V Length of continuous chip in one revolution
at which both the tools will have the same = 60mm
life. Feed force = 800N. Calculate the chip
Also calculate the corresponding tool life. If thickness, shear plane angle, velocity of
you have to machine at a cutting speed of chip along tool face and coefficient of
l m/s, then which one of these tools will you friction.
choose in order to have less frequent tool
changes? (GATE-ME-99) 24. A cylinder is turned on a lathe with
orthogonal machining principle. Spindle
2 1. Tool life testing on a lathe under dry cutting rotates at 200rpm. The axial feed rate is 0.25
conditions gave 'n' and 'C' of Taylor tool mm per revolution. Depth of cut is 0.4mm.
life equation as 0.12 and 130 m/min, the rake angle is 10°. In the analysis it is
respectively. When a coolant was used, 'C' found that the shear angle is 27.75 ° ,
increased by 10%. The increased tool life (GATE-ME-03)
with the use of coolant at a cutting speed of i) The thickness of the produced chip is
90 m/min is (GATE-ME-01) (a) 0.5 1 1mm (b) 0.528 mm
(a) 47.4min (b) 37.4min (c) 0.8 18 mm (d) 0.846 mm
(c) 27.4min (d) 17.4min
ii) In the above problem, the coefficient of
22. Identical straight turning operation was friction at the chip tool interface
carried out using two tools : 8-8-5-5-5-25-0 obtained using Earnest and Merchant
(ASA) and 8-8-5-5-7-30-0 (ASA). For same theory is
feed the tool which gives better surface (a) 0.18 (b) 0.36
finish is (GATE-ME-01) (c) 0.7 1 (d) 0.98

23. A tube of 32mm outside diameter was 25. In an orthogonal cutting test on mild steel,
turned on a lathe and the following data was the following data were obtained
obtained (GATE-ME-02) (GATE-ME-04)
Rake angle = 35deg, Cutting speed : 40 m/min,
Feed rate O. l mm/rev
=
Depth of cut : 0.3 mm,
Cutting force = 2000N, Tool rake angle: +5 ° ,
Cutting speed = 15m/min Chip thickness : 1.5 mm,

!IJ•li@jjji4ijjjiQJtalM\j/jjj� yderabad I Delhi I Bhopal I Pune I Bhubaneswar I Lucknow I Patna ! Bengalwu I Chennai I Vtjayawada I Vmg I Ttrupati I Kukatpally I Kolkata
: 684 : Production

Cutting force : 900 N, specific cutting pressure = 1300 x t°-4 N/mm2 ,


Thrust force : 450 N (Where t is the uncut chip thickness)
Using Merchant's analysis, the Friction If the tool and work handling time 1s
angle during the machining will be 0.5min, estimate the production rate per
°
(a) 26.5 (b) 3 I .5° hour
(c) 45° (d) 63.40
28. The results of machining steel with 2 grades
26. A standard machine tool and an automatic of tool material are given below. For a 180
machine tool are being compared for the min. Tool life which tool is recommended
production of a component. Following data and why? (GATE-PI-89)
refers to the two machines.
(GATE-ME-04)
A B
Tool Taylor's exponent 0.2 0.25
Cutting Speed for 1 min
Standard Automatic
100 120
tool life
Machine Tool Machine Tool
Set Up time 30 min 2 hours
Machining 22 min 5 min
time per piece If the tool regrinding and changing time is
Machine rate Rs.200/hour Rs. 800/hour 15min, the cutting speed for tool A ahs to be
chosen (40m/min or 50m/min)
The breakeven production batch size above
which the automatic machine tool will be 29. A parting tool has a ground front clearance
economical to use, will be angle of 6° and the back rake angle of 10° .
(a) 4 (b) 5 (c) 24 (d) 225 The tool, by mistake, was set 1.5 mm above
the line of centers while machining a job of
27. Calculate the tonnage capacity of the 50 mm out-side diameter.
broaching machine required to broach on the (GATE-PI-90)
hole under the following conditions.
(GATE-PI-89)
i) The effective rake and clearance angle
are
Diameter of the finished hole= 75mm, rise (a) 2.56 & 1 0 .44
per tooth = 0.03mm (b) 13.44 & 2.56
Tooth pitch = 10mm, (c) 13.44 & 3.56
Length of the broach = 750mm (d) none
Cutting speed = 6m/min,
!1Dllih4h/§41jjj$jmnfMj11.jj.yderabad I Dellii I Bhopal I Pune I Bhubaneswari Lucknow I Patna I Bengalwu I Chennai j Vgayawada I Vi7.ag I Tllllpati I Kukatpally I Kolkata I
....
... ,...... ACE 6 85 Metal Cu

.I "��=
&�p
�· irn
= rq��� · �=======�:����
Jblimiooa
�·� : ===========�����ttmg
�·�

ii) The diameter of the job at which, while 32. A generalized tool life equation for carbide
parting, the tool will start rubbing tool for machining steel is given by
(a) 35mm (b) 28.7mm V T1 Fb Dc = K'
(c) 57.4mm (d) none Where V = Cutting speed, meters/min,
T = Tool life, minutes,
30. The following data apply to machining a F = feed, mm/rev,
part on turret lathe and a general purpose D = depth of cut in mm, Indices have
lath (GATE-PI-90) magnitudes a=0.3, b=0.3, c = 0.15,
(GATE-PI-92)
General Turret
Purpose lathe lathe
i) If the feed is halved and depth of cut
Cycle Time 20 min 5 min doubled, for identical tool life of 60
Labour rate Rs. 1 1/hr Rs. 8/hr minutes, the percentage change in speed
Machine rate Rs. 7/hr Rs. 1 0/hr (a) 5% (b) 11% (c) 25% (d) l 30%

The setup cost and the cost for special ii) The change in productivity for the new
tooling on the turret lathe would be Rs. processing conditions
3000/- . These costs are negligible on the (a) 5% (b) 11% (c) 25% (d)130%
general purpose lathe. Calculate the batch
seize which justifies the use of turret lathe. 33. Parting-off operation is carried out on a
cylindrical work-piece of 100mm diameter.
31. During orthogonal turning a steel rod at feed The groove width is 2 mm and an in feed of
0.25 mm per revolution and depth of cut 0.2 mm per revolution is given at a
°
4.00 mm by a tool of geometry o0 , -10 , 8°, maximum cutting speed of 60 m/mm. The
°
7°, 15°, 60 , 0 (mm) ; the following specific cutting force for the material is
observation were made: Tangential force = 800 N/mm2 • (GATE-PI-94)
1,600N , Axial force in feed direction = 800 i) The tangential force on the tool is
N, Chip thickness = 0.60mm. Find the (a) 160 N (b) 320 N
coefficient of friction between chip tool (c) 480 N (d) 640 N
interface and shear strength of work material ii) The maximum power requirement for
from shear force.
(GATE-PI-91)
the operation is
(a) 80 W (b) 1 00 W
(c) 200 W (d) 320 W

!ltlli@iiimhi401ftiM\iiih+1Yderabad I Delhi I Bhopal I Pune I Bhubaneswarl Lucknow I Patna I Bengaluru I Oiennai I Vijayawada I Vmg I Tuupari I Kukatpally I Kolkata I
ACE : 686 : Production
.:-.�
':. _ :F.Dgineajng Pnbticatiooa

34. A part can be machined in 30 minutes on an (GATE-PI-94)


engine lathe where as it can be machined in i) The average coefficient of friction
6 minutes on a turret lathe. However, it between the chip and the tool is
would cost additionally Rs. 500 to tool up (a) 1/3 (b) 3/5 (c)2/3 (d) none
the turret lathe for the operation. If the
ii) The cutting power in watts is
hourly rate including labour and overhead is
(a) 300 (b) 450
Rs. 80 for the engine lathe and Rs. 160 for
(c) 600 (d) 750
the turret lathe, The minimum number of
parts required to make the turret lathe more iii) The length of the shear plane is
economical to use, for the operation (a) 30mm (b) 15mm
(GATE-PI-94) (c) 10mm (d) 4mm

35. In an orthogonal cutting operation, the 3 7. In an orthogonal machining experiment


cutting velocity is 30 m/min and the chip using a tool having 6 ° . Rake angle, the
velocity 15 m/min. If the rake angle of the following data were collected. Cutting speed
tool is 10 . °
(GATE-Pl-94) 0.5 m/sec, width of cut 3 mm, depth of cut
i) The shear angle is 1mm, chip thickness 1.5mm. Assuming that
(a) 28.33 (b) 30.34 shearing takes place under minimum energy
(c) 45 (d) can not be calculated condition, (GATE-PI-95)

ii) Shear velocity with the help of a i) The coefficient of friction between the
velocity triangle. chip tool interfaces will be
(a) 7.5 m/min (b) 12m/min (a) 0.235 (b) 0.339
(c) 25 m/min (d) 31 m/min (c) 0.467 (d) none

ii) Chip velocity is


36. In an orthogonal cutting operation,
(a) l Om/min (b) 20m/min
The depth of cut= 2 mm,
(c) 30m/min (d) 40m/min
Width of cut = 15 mm,
Cutting speed= 0.5 mis and
The rake angle= 0 deg. (a) 3mm2 (b) 4mm2
iii) Area of shear plane is

The cutting force and the thrust force are (c) 5.2mm2 (d) 7.l mm2
respectively 900 N and 600 N and the shear
angle= 30 deg.
!M•li@hiiQ.jjij@GIM\i/jjj� yderabad I Delhi I Bhopal I Pune I Bhubaneswar I Lucknow I Patna I Bengaluru I Chenna.i I Vijayawada I V,z.ag I Tirupati I Kukatpally I Kolkata
.,... Metal Cutting
� ........."
. ACE
. PnNicwioo•
. .
-:. lFngnicoaq : 687 :

38. In a single pass turning operation the cutting 4 1 . A cutting tool is designated in 'Orthogonal
speed is the only variable Based on the Rake System' as : (GATE-PI-02)
cutting time cost and the cutting edge cost. 0 °- 0° - 6°- 6° - 25° - 75°- 0. 8 mm.
The tool life for minimum cost given that The following data were given
cost of 1 cutting edge is Rs.5, operator S0 = feed = 0. 1 2 mm/rev
wages including the machine tool cost is T = depth of cut = 2.0 mm
Rs.75/hour and tool life equation is VT0· 1 is a2 = chip thickness = 0.22 mm
1 00. (GATE-Pl-95) Vr = chip velocity = 52.6 m/min
(a) 1 0 (b) 1 7 (c) 29 (d) 36 'ts = dynamic yield shear strength = 400 MPa
Pz = main cutting force
39. Tool life in drilling steel using HSS drill is
expressed as T0·2 = 9. 8 D0.4 / V s0· 5 where D
= s
So t 'ts ( secy - tany + 1 )
is the drill diameter (in mm), T is the tool Where s = chip reduction coefficient and
life (in minutes), V is the cutting speed ( in y = orthogonal rake.
m/min) and s is the feed ( mm/rev). the feed The main cutting force (Pz) and cutting
is set at maximum possible value of 0.4 power assuming orthogonal machining are
mm/rev for a given drill diameter of 30mm.
the length of drilling is 50mm. the machine 42. The following data refers to an orthogonal
hour rate Rs 60 and the cost of drill is Rs machining of mild steel with a single point
400. HSS tool. Rake angle of tool = 1 0 °, uncut
i) For the given conditions, the tailor's chip thickness = 0. 3mm, width of cut =
exponent and constant are . . . . . . . . . . . . 2.0mm, single plane shear angle = 36° ,
ii) The optimum cutting speed, Vopt, shear strength of mild steel = 450 MPa,
neglecting the work-piece and tool using Merchants analysis (GATE-PI-03)
changing times is (GATE-PI-01) i) The coefficient of friction between the
chip and tool will be
40. In certain machining operation with a
(a) 0. 1 4 1 (b) 0. 344
cutting speed of 50 m/min, tool life of 45
(c) 0.532 (d) 0.688
minutes was observed, when the cutting
speed was increased to 1 00 m/min, the tool ii) The shear force in cutting will be
life decreased to 1 0 minutes. The cutting (a) 270 N (b) 333.75 N
speed for maximum productivity, if tool (c) 450 N (d) 459. 34 N
change time is 2 minutes is (GATE-PI-01)

IMIIQO§ml41jjj$Qnn,@jufh� yderabad I Delhi I Bhopal I Pune I Bhubaneswar I Lucknow I Patna I Bengaluru I Chennai I Vijayawada I Vizag I Tirupati I Kuk.atpally I Kolkata
SOLUTIONS
05. Ans: (b)
One Mark Solutions Sol: During machining operation, when the force
is applied by the tool in upwards, the chip
01. Ans: (c) produced will directly lifts upwards, but due
Sol: With increase of positive rake angle, the to the self weight of the chip it falls on the
strength of tool reduces, tool life increases, rake face so that when the chip is lifting
forces induced during machining will come upwards and falling back, the chip will not
down etc. have contact with tool at the beginning of
the tool tip, hence no friction at tip and no
02. Ans: (c) wear.
°

Sol: Surface finish is indicated by When it falls back on to the rake face, it is
Rt= maximum peak to valley height trying to penetrate into the rake face so that
= :F/8r, where at some distance away from the tool tip
f= feed and r = nose radius of tool . If feed friction increases, heat generation increases
is doubled the Rt increases by 4 times and if and wear of tool increases and producing
nose radius is doubled the Rt reduces by 2 wear like crater (a shallow spherical
time hence the net value increases by 2times depression) hence named as crater wear.
means surface finish increases by 100%
06. Ans: (a)
03. Ans: (a)
Sol: Out of the total heat generated in the
Sol: The main purpose of using the cutting fluid
machining operation, maximum heat is
in machining is to carry away the heat
transferred into the chip (75 to 80%) , next
generated during machining and also it can
maximum to the work ( 15 to 20%) and least
act as lubricant for reducing friction at the
heat is transferred to tool (5 to 8%)
chip tool interface.

07. Ans: (a)


Sol: The chances of formation and increasing the
04. Ans: (b)
Sol: During machining the chip is always sliding
BUE is due to low rake, low speed of on the rake face of the tool.
machining, large uncut chip thickness and
without use of cutting fluid.
l1Dllii@jjj§§,ijQ41"1Mi@j� yderabad I Delhi I Bhopal I Pune I Bhubaneswar I Lucknow I Patna I Bengaluru I Chennai j Vtjayawada I V17.ag I Tirupari I Kukatpally I Kolkala
.!'.t 1.'....:......:...Publicationa
ACE Metal Cutting : 689:
� �=�
==....
=================================
11
"

08. Ans: (a) materials and hence the wave disappears


Sol: With increase of rake angle, the forces somewhere in the middle , therefore the
induced during machining will be reduces continuity of chip will be maintained and
hence the power consumption reduces. also due to low speed the time available for

Ans: (a)
adhering of the chip will be high hence
09.
Sol: out of the total heat generated in the
BUE forms.

machining operation, maximum heat is 13. Ans: (d)


transferred into the chip (75 to 80%), next Sol: With increase of speed friction reduces

14. Ans: (d)


maximum to the work (15 to 20%) and least

Sol: When the shear is taking in the orthogonal


heat is transferred to tool (5 to 8% )

10. Ans: (b) plane, the shear angle is 45° and due to zero
Sol: During machining of ductile materials rake angles,
because of high toughness, the energy wave The shear strain induced= cot <I> + tan (<I> - a)
produced due to shear is absorbed by the = 1+1 = 2

15. Ans: (b)


materials and hence the wave disappears

Sol: Fe = cutting force = 400N,


somewhere in the middle, therefore the
continuity of chip will be maintained and
also due to low speed the time available for t 1 = uncut chip thickness = feed = 0.1mm
adhering of the chip will be high hence B = width of chip = depth of cut = 2mm
BUE forms. Specific cutting pressure = specific cutting
energy
11. Ans: (c)
Sol: At high cutting speed, the time available for t 1 .B
= Fe = 400/(0.1 x 2 ) = 2000 N/mm 2

adhering of microchips or powdered .


particles will be less and hence chances of 16. Ans: 5.9 to 6.1
BUE formation will be eliminated. Sol: Given data : n = 0.2, Tc = 1.5min

12. Ans: (b)


For maximum production rate

Sol: During machining of ductile materials


Optimum Tool life

because of high toughness, the energy wave T= (1�n x Tc ) = (1�-�·2 x 1 .5) = 6min
produced due to shear is absorbed by the

!l"S•I@h4m§4ijjjj@CiM\h!jj� yderabad I Delhi I Bhopal I Pune I Bhubaneswar I Lucknow I Patna I Beagaluru I Chennai I Vtjayawada J Vmg I Tirupati I Kukatpally I Kolkata
: 690 : Production

17. Ans: 0.08 to 0.12 21. Ans: (c)


Sol: Because nothing is given in the problem, it Sol: Vanadium improves the wear resistance
is assumed that the given process is oblique
machining. 22. Ans: Increases, spoils
Hence in turning, uncut chip thickness
= t1 = f. cos (Cs) 23. Ans: (d)
= 0.2 xcos (60) = 0. 1 mm Sol: The tool used in USM must soft

18. Ans: (c)


24. Ans: Increases, increases
Sol: During machining when the plastic
deformation is taking place, the strain is 25. Ans: Reducing, increasing
also occurring on the work material and due
to this strain some amount strain hardening 26. Ans: (d)
is taking place on the microchips, hence the Sol: Grinding is the one which requires largest
microchips may become harder than the tool specific cutting energy.
and wear of the tool is taking place by
abrasive action. 27. Ans: (b)
Sol: Out of the heat generated in metal cutting,
19. Ans: (a)
by assuming dry cutting conditions, the
Sol: Normally rake angle will not influence on
maximum heat is carried away by the chip,
the surface finish produced on the work
the next is work and least amount of heat is
piece. But in certain cases on microscopic
carried by the tool
observation it is found that with increase of
rake angle, the forces are reducing and shear
28. Ans: (d)
angle will increase. As the shear angle
Sol: By adding small quantities of sulphur,
increases, the shear plane area reduces
phosphorous and silicon will increase the
bt1
because shear plane area= -.-- machinability.
sm ¢
,

Where, �= shear angle.


29. Ans: (c)

20. Ans: (c)


Sol: Carbide cutting tools are weaker in tension
Sol: In HSS for increasing hot hardness hence to avoid this negative rake angles are
temperature 18% tungsten or molybdenum used
is used
jlS11i@jji4q;m;Qflnftii@j� yderabad I Dellii I Bhopal I Pune I Bhubaneswar I wcknow I Patna I Bengaluru I Chennai I Vijayawada I Vm,g I Tirupari I Kukatpally I Kolkata
� Jli'....:......:...Publicaliooa Metal Cutting
,.

: 69 1 :
,..
¢ '->., ACE

� ,. �
�;;g;;;;;;;�
�;,;;;;;;;;;�=============================

30. Ans: (d) 36. Ans: (c)


Sol: With formation BUE, during machining Sol: It is exactly same effect as that of the setting
whatever the wear of the tool to be taken of tool below center line in case of external
place will be wear out as BUE only hence turning operation.
the tool life increases
37. Ans: (d)
31. Ans: (d)
Sol: In grinding wheel most of the abrasive will
Sol: During machining whatever the energy
have zero rake angle, high shear angle and
supplied for machining is simply converted
high cutting speed.
into heat energy only, due to the large
amount of heat, rise in temp will occur to
38. Ans: (c)
the tool tip, to withstand for this high temp
Sol: Very high hard materials can be turned by
without loosing its hardness it requires the
diamond, but diamond cannot be used for
hot hardness.
machining of ferrous components due to
32. Ans: (a) high diffusion wear hence the next material
Sol: In HSS for increasing hot hardness to be used is CBN.
temperature 18% tungsten or molybdenum
is used 39. Ans: (d)
Sol: During machining of ferrous materials using
33. Ans: (c) diamond cutting tool because of diamond is
Sol: Carbide cutting tools are weaker in tension made by pure carbon, the iron has chemical
hence to avoid this negative rake angle is affinity to absorb carbon particles if it is
used available freely and hence diffusion wear
increases.
34. Ans: (d)
Sol: With formation BUE, during machining 40. Ans: (d)
whatever the wear of the tool to be taken Sol: During machining of ferrous materials using
place will be wear out as BUE only hence diamond cutting tool because of diamond is
the tool life increases made by pure carbon, the iron has chemical
35. Ans: (c) affinity to absorb carbon particles if it is
Sol: With increase of depth of cut the cutting available freely and hence diffusion wear
force and MRR both will increase hence the mcreases.
specific cutting energy almost remain same.
!IS11i@jj/§4,jji4RflCiM\iliii� yderabad I Delhi I Bhopal I Pune I Bhubaneswar I Lucknow I Pama I Bengaluru I Chennai I Vijayawada I Vmg I Tirupati I Kukatpally I Kolkata
: 692: Production

41. Ans: (c) 47. Ans: (10°)


Sol: In general the roughness can be reduced by Sol: Turning operation,
reducing feed of the tool. If the feed is kept i = inclination angle = 10 °,
constant, then next it is possible to reduce =
f= 0.25 mm/rev, t1 0.25 = f cos Cs
roughness by increasing speed by keeping
0.25 _ 0.25 _
_ -- _
the tool remains same. cos C s - - -- - l � Cs - 00
f 0.25

42. Ans: 0.25


').,, = 90 - Cs = 90 - 0=90

Sol: V2= 2V1 ,


tan ab=sin ').,, tan i + cos ').,, tan a
= sin90 tan10 + cos 90 tan a
=1 x0.176 + 0 = 0.176
ab = tan- 1 (0.176) =10°

/n (V 2J

n=
in G:J
Yi = /n (2) = 0.25
in (16)
Two Marks Solutions

01. Ans: (a)


43. Ans: (c) Sol: Generally pure metals have high ductility;
hence the continuous chips are produced
44. Ans: (b)
02. Ans: (c)
Sol: Surface finish is indicated by Rt=maximum
45. Ans: (a)
f2
46. Ans: 0.5 (range 0.49 to 0.51)
peak to valley height = -
8r
Sol: Because the rake angle is zero, Where f=feed and r = nose radius of tool.
F= Friction force = Fc = Cutting force = If feed is doubled the Rt increases by 4
500N, N = Normal to friction force = F1 = times, to maintain the same level of surface
Thrust force = 250N, finish the nose radius has to be increased by
Coefficient of friction=FIN 4times.
= 250 I 500 = 0.5

lli•IMOYhiiiihdRbbiM\htii+yderabad l Delhi l Bhopal J Pune l Bhubaneswarl lllcknowl Patnal BengaJurul Chennai l VuayawadaJ Vmg JTirupati I Kukatpa]Jy J Kolkata I
: 693 : Metal Cutting

03. Ans: (c) 07. Ans: (b)


Sol: a = 10 °, <I> = 20 ° Sol: Let Q = no. of parts produced
Here typical cutting operation means that Total cost (T.C) on conventional lathe
Merchants theory is valid 30
= 30 + - x 75 x Q
2<1> + 13 - a = 90 60
f3 =90 + a - 2¢ = 60 = 30 + 37.5Q

04. Ans: (a)


T.C on CNC lathe

Sol: Given ,nose radius (r) = 1. 8 mm. =


15
150+ - x 120 x Q
60
R1 = 5 microns = 0.005mm ,f = ?
= 150 + 30Q
f2
R1 = - If Q = 100 parts
8r
T.C on conventional lathe
f = .Jo.005 x 8 x l .8 =0.268 mm / rev
= 30 + 37.5 X 100 = 3780
05. Ans: (a) T.C on CNC lathe
Sol: t 1 = t 2 = 0.45 => r =1 = 150 + 30 x 100 = 3150
a =O Due to low cost CNC lathe is preferable

08. Ans: (c)


l cos O J
<I> = tan- 1 ( . = tan _1 (1) = 45 °

Sol: a = 10° '


1 - l x smO
r = 0.4
06. Ans: (b)

Sol: Tool life, T oc


1 tan a = (
r cos a
1 - r sin a J
J
Flank. wear
1 0.4 cos10
T oc -- => <I> = tan-1 ( = 22.94 o
cot a l - 0.4sin10

09. Ans: (c)


T oc tan a
Tan a2 Tan 7
Sol: V1
T2
= = = 0_7
I'i Tan a1 Tan 10 = 64 , T1 = 15
V2 =100,T2 =12
T2 =0.7 fi
:. Tool life decreases by 30%

l1t11iOY01i;;w101GiMihih.j>HYderabad I Dellii I Bhopal I Pune I Bhubaneswar l Lucknow I Patna l Bengaluru l Chennai j Vtjayawada j V17.3g I T1111pati I Kukatpally l Kolkata I
..... ;.."=..,... ..,� !,Pl���,bli����ai:•�========�:�:,:;,,;:,.:�==============Pr=o=d=u=ct:I=·o=n
�-�Eli'n�e,m�· :�:Pffll·�
69 4

=> V1N2 = [T2/T 1t 1 1. Ans: (c)


=> taking log on both side �
T2 -
1
-
8

n {�) =
n
1 (W) = o.4463 = 2
0.223 1
n :� ) V2
1n J
� ;:) 1 ( n=
( V1
= ln 2 = 0.333 = .!_
T ) ln 8 3
ln -1
( T2
o
T3 T1 � ; l 5( �) .s = 1 o min
( V3 ) 144
= =
1 2. Ans: (a)
Note: strictly speaking the value of "n < l " Sol: With increase of cutting speed the time
taken for machining will be reduces and so
10. Ans: (a) the machining cost will reduces, the non­
5 00 = productive cost will never depends on the
Sol: Tool life= T1 = 5 0, V1 = 5 0rpm
10
cutting velocity and with increase of cutting
T2 = \ : = 1 2.2, V2 = 80rpm velocity, the tool life will come down hence
2

it is required to change the tool many times


The feed and depth of are same m both
and tool change cost increases
cases

13. Ans: (b)


ln ( 50 ) f
0.47
Sol: h =
80

tan30 + cot 8
1n
n= = = 0.333
(*l )
1.4 1
1n(;: ) 1n(i�o2 )
= P

f
hQ =
tan 15 +cot 8
hp
= tan l5 +cot 8
hQ
50 0.;33 -
tan 3 0 + cot 8

V3 (v
T3 = T1 -J¼ = 50(-)
1

60
_ 29

14. Ans: (c) 15. Ans: (b)

16. Ans: (a)

!ltl1ih§jjji4@iAflbi@..jjj+yderabad I Delhi I Bhopal I Pune I Bhubaneswarl Lucknow I Patna I Bengaluru I Chennai I Vtiaya� I �u.ag ITirupari I �yJ Kolkata I
Sol: Common Solution for 14, 15 & 16 Yr = rVe = 0.714 x20=14.28
t, = 0.5, Ve = 20, Fe
F= · A
smp
w = 5 = b, In
cosw-a )
0.7' FT= 200N Fe = 1200N 1200
xsin 24.59= 506 N
f2 =

Assume Merchants theory cos9.46


=

2<1> + 13-a = 90 Energy lost in friction=F x Vf


05 14·28=
= 506x 120.5
r= .!L = · = 0.714 60
t2 0.7
20
Total energy = Fe xVe = 1200x- = 400W
60
1205 =
% Energy lost in friction = 30%
400

17. Ans: (b) 18. Ans: (c)


Sol: Common Solution for 17 & 18
D=147 mm, L = 630 mm
F T1 =24 when V, =90 m/min
13-a= tan-'( T J = tan-'( 200 ) = 9.46
Fe 1200 T2 =12 when V2 =120 m / min
f=0.2, d=2mm
2<1>+13-a = 90
90-{B-a) = 90-9.46 =
<I> = 40_27
2
1n(�,) 120
1n( )
2
r= st <I> =>
cos{<I> -a)
sin<I> 90 =
0.415
cos <j>-a) r
In(i, ) - :i)
=

n- In(
,
=>("'-a
"' )= cos_ (sin 40.27)
0.714
=><!>-a= 25.13=>a= <j>-25.13 (24)0.4IS

40.27- 25.13 V3 = 90x = 97m / min


=
20
15.13 lOOOxV I000x97=
N3 = = 210
=

Shear strain= cot<I> + tan {<I> -a) 1tD 1tx147


= cot 40.27 + tan 25.13= 1.65 630 =
Time I cut = � = 15 min
f3-a= 9.46=>13= 9.46 + 15.13= 24.59 fN 0.2x 210
µ = tanl3 = 0.478
.\( l. l.nL{1111.·t 1111g P1il,lu ,tll(111, yderabad I Delhi I Bhopal I Pune I Bhubaneswar I Lx:know I Patna I Bengaluru I Chennai I Vrjayawada IVIZ:lg I Tuupati I Kukatpally I Kolk,.;]
:.t ACE
�.. "= :�:=============�����:· :
696 Producnon
:EF.ogiom�·:mw:·�����-�,..�·::,.========�:�

19. Ans: (b)


Sol: Orthogonal machining operation,
D = 150 mm, Vc = 90 m/min,
f = 0.24 mm / rev= t1
d = b = 2 mm, t2 = 0.48,
a= 0, A = 90, Cs= 90 -A = 0
0.24
r =!i_ = =0.5
t2 0.48
F 800
tan(f3-a) =_!_ =-- = 0.8 = tanf3
r cosa Fe 1000
<I> = shearangle = tan-1( )
1-r sma F
. tanf3=- = 0.8
= tan-1 (0.5) = 26.56 N

22. Ans: (d)


20. Ans: (d)
Sol: W hen the chip is flowing in the orthogonal
Sol: Specific cutting energy = 2000 J/mm3
plane, the angle of principle cutting edge
Ve = 120 m / min,
angle is 90° .
f = t1 = 0.2 mm/rev, d= b= 2mm
. F c xV c 23. Ans: (d)
Specific Energy =
t1 xbxVc
=> FcxVc =2x0.2x2x120 24. Ans: (b)
Sol: 't = 250MPa , Vc = 180 m/min,
2x0.2x2x120
FC =
O

120 f = 0.2 mm/rev= t1 , d= 3 mm= b,


= 0.8 x 1000 = 800 N r= 0.5, a= 7
Use merchant theory 2<j>+ f3-a =90
21. Ans: (c) i 0.5cos 7 =27_85o � 28
<I> = tan-( )
Sol: Orthogonal machining operation, 1-0.sin 7
0

"- 90, Cs = 0
= Fs sin<j>
=

Fe = 1000 N , FT = 800 N , Ao
't
u

<I> = 25, a = ab = 0 => F5 =25 0x0.2 x3/ sin 28 =320N


Use 2<j>+ f3-a =90 2<j>+ f3-a =90

IM11ii@jjji41ijjiRfln!M\jjiii+yderabadj Delhi I Bhopal I Pune I Bhubaneswar I Lucknow I Patna I Bengaluru I Chennai I Vijayawada I Vizag I T=pari I Kukatpally I K.olkata I
ACE
��-� �:�6:9�7 �:===========M�e:��C�u�ttmg
� '-:�=&!�:�:ermr:·���,Nicabons�·�·========== :·�
P= 90 + 7- 28X 2= 41 27. Ans: (a)
F Sol: na = 0.45
Fe = s xcos(p-a)
cos( $+P-a) nb = 0.3
320 Tool A VT0 ·45 =90 ,
--xcos 34=565N
cos 62
=

Tool B VT0.3 = 60
F At BEP, TA= TB
F = s sin p
cos($+ p-a)
320
xsin 41 = 447 N
cos 62
=

25. Ans: (a)


603 .-
y3,33-2.22 =yJ.11 = -
33
= 38.26
26. Ans: (c) 902 .22
Sol: Common Solution for 25 & 26 V= 38.26°·9 = 26.57
V1 =60, T1 =81
28. Ans: (b)
V2 = 90, T2 = 36
Sol: a= 12° , t, = 0.81 mm , ti= 1.8 mm
V3 = 30 , T3 = ?
r =..s_ =0 .45
t2
r cosa
$= shearangle= tan-1 ( J
1-r sma
0.45cos12
$= tan-i( J = 25_9
1-0.45 sin 12

V
T3 = T, ( :
n 81( 2)0.s = 324
29. Ans: (a)
Sol: For theoretically minimum possible shear
J
=
v
strain to occur
324-81= 2$-a= 90
% C hange in tool life = 300%
81
90 + ex. = 90 + 12 =
$= 51
2 2

!11•1ih4hiiiii!4@Gi@dfjj+yderabadlDelhilBhopal1PunelBhubaneswarlUJCknowlPatnalBengaJurulChennailVuayawadalVtt.ag !Tirupati I Kukatpallyj Kolkata I


.,·•·"' . ACE
. . . : 698: Production
� ..�&ipw,qNw,m,ma

30. Ans: (b) f O.l2


Rl b = ; = =3.787x l 0- 3 mm
Sol: n = 0.25, T1 = 10 min, 8rb 8X 0.33
V1 = 20 m/min Tool B is giving better surface finish and
I f V2 = 40m / min, T2 -?
- • Rt= 0.003787 = 3.78 µ m

34. Ans: (a)


T2 =Tt:t =!{��)'�' Sol: At BEP , VHSS = VCarbide
THSS = TCarbide
= 1{�J = 0.625 min

31. Ans: (d) 1505


=�
y =V1.61
Sol: The correct sequence is chip, work, tool, 250.3.33 ,
3 33
and environment :. V = 54 m/min

32. Ans: (c) 35. Ans: (b)


Sol:
Sol: Rt = �, as the fe ed rate is increased by 2
Sr
0.1

i.- o.os...i
times the roughness value will increase by 4
1--
0.1
--1
times

36. Ans: (b)


Sol: a= 0,
0.05 => 0.05 FT
tan 30= RI = = 0.372, L 2 = 4.71mm
Rt tan 30 Fe
= 0.086mm=86microns L 1 =10 mm
F
tan{p-a) = tanj3 = T
33. Ans: (b) Fe
Sol: ra = 0.6, rb = 0.33
13= tan-1 (0.372)= 20.71
fa = 0.2, fb = 0.1 4.71
L
r= 2 = = 0.471
f 0.2 LI 10
Rta = a = =8.33X 10-3 mm
2 2

8ra 8X 0.6
\{ l l 11..,,1at Jl[1.., Ptdilu.tllPll-. yderabadjDelhijBhopaljPunejBhubaneswarl LucknowlPatnalBengalurulChennai!VuayawadalV,zag ITirupati I Kukatpal)yj Kolkata

\
� ACE
�li'....:..-:....Publicaliona
··� = =
�' �=.......-.. ...
: 699: Metal Cutting
================================
0.471cos0 40. Ans: (b)
$=tan -1( )= tan-1(0.471)
1-0.471sin 0
41. Ans: (d)
= 25.22
Sol: r = 0.3, a = 10
37. Ans: (d) 0.3cos10
$=tan_1( )=l?.3
1-0.3sin10
Sol: D = 25 mm, L = 100 mm' VT 0·25 =55 '
Vc = 22 m / min , f = 0.046 mm /rev Shear strain = cot$ + tan ( $- a)

1000V = 1000 x 22 = cot 17.3 + tan (7.3)


N= = 280 rpm = 3.34
nD nx 25
. . L
T 1me Ipiece = f
N 42. Ans: (b)
100
= =7.76 min
0.046x280 43. Ans: (c)
Total time = 425 x 7.76 = 3299.7 min
44. Ans: (d)
) °.2s =
I
4

T=(� ( ��) =39


45. Ans: (a)
. 3299.7
No. of too1 regrmds = --= 85 Sol: a = 15, $ = 45°, Ve = 35 m/min
39
sin $ sin 45
r= = =0.816
cos($-a) cos30
38. Ans: (a)
Sol: Out of the given cutting tools, always CBN Ve r => v =r.V =0.816x 35= 28.58
-=
Ve
f c
can be used at high cutting speeds, hence it
should be at last.
46. Ans: (a)
39. Ans: (d)
47. Ans: (c)
0·08 2
Sol: R ti = = .r_ =0.0004 mm Sol: In metal cutting there is no sticking friction
8r 8X2
f
R 12 =-----
tan cs+ cot ce 48. Ans: (c)
0·08
= = 0.012 49. Ans: (b)
tan 45 + cot10

!l!INM@,jj§i@NRflGIM\ufjj� yderabad I Delhi I Bhopal I Pune I Bhubaneswar I Lucknow I Patna I Bengaluru I Chennai I VtjayawadalVuag I Tirupari I K�y I Kolkata
t . A;CE . .
��.,)•
' "'=
:'Elii�p�iea:ez-.��Pl�mticwims�� �
======= ::::���.,::=============,.;;���;;;:�:· �
7 00 Product:J.on

Sol: Common solution for 48 & 49 Loss of energy due to friction


No. ofjobs = 500, D=50, L = 100 mm 764x27.5 =
= F x Vr= 350.16 W
N = 200 rpm, f = 0.2 mm/rev, 60

V = 1tDN/l OOO = 3 l .4m/min


52. Ans: (c)
160 0·25 53. Ans: (b)
VT o.2s =160 � T =( ) = 674
31.4 Sol: Common solution for 52 & 53
L 100 a = 15, b=2,
Time/part = -= ---=2.5 min
fN 0.2x200 t 2 =0.4, t, = 0.2
No. of components produced per tool life
02
674 r=�= · =_!_=0.5
= =269.6 t 2 0.4 2
2.5
' 0.5cos15
50x 200
=31.4 <I>=tan- ( J=29
V= 1-0.5sin l 5
7tX

1000
Fe =900,
50. Ans: (d)
F 810
51. Ans: (d) J3=a+ tan_, (_I_ J=15+ tan_, -
Fe 900
Sol: Common solution for 50 & 51 = 56.98
D 40 mm,a 10,
re
= =

N = 500 rpm, <I> = 25 Fs = xcos(<j>+J3-a)


cos J3-a)
1tDN 1tx40x500
Ve = = =62.8 m /min 900
1000 1000 cos(70.98)
cos 41.98
sin<!> sin 25
r= = =0.437 394.56 N
cos ( <I> -a) cos15
=

Fs
=Shear strength = x sin <I>
r=-�
vr Yr =rVC Ao
,u

Ve
394·56
=0.437x62.8=27.5 m / min = x sin 29 = 478 MPa
2x0.2
J3=58 ° 10 1 , Fe =600N, FT =200N Note: Strictly speaking J3 > 45, hence we are
Fe . supposed to use classical friction theorem.
F= In xsm,-.,A
cosw-a) But if use classical friction theorem and
600 solve the problem, the answer is not
= x sin 58 °10' = 764
cos 48 °10 1 available in the multiple choice.
liti•i@jjj4i@QjlnniM\jjii!� I I I I I I I I I
yderabad Delhi Bhopal Pune Bhubaneswar Lucknow Patna Bengaluru Chennai Vijayawada 1¥17.31! I Tirupati I Kukatpally I Kolkata
..,
':,�-�
"' �CE . .: 701: . .
Metal Cutting
J �=================================
�F.ngioramgPDNiracma

54. Ans: (a) Fs


Fe = "cos(p-a)
55. Ans: (b) cos( <l>+ P-a)
Sol: Common solution for 54 & 55 616
= xcos 25 = 1039N
Fe = 1200N, FT= 500N, a= 0 cos57.5

Ve l m /sec, d = b = 0.8, t2 1.5 mm 150


= = Power, P = Fe xVe = 1039x- = 2597 W
60
=2.6 kW

58 . Ans: (b)
2.0-0·8
Sol: s 1ope = =0. 024
500 60-10
= O+ tan- 1( )= 22.62
°
1200 If wear land=1.8 mm 2
Taking d = t1 = 0.8 (1.8 -0·8)
Tool life = T = 10+
0·8 0.024
r = _S_ = = 0.533
t 2 1.5 =51.67 min
Yr
r = - => Yr = rVe = 0.533 x1
Ve 59. Ans: (c)
= 0.533 m /sec Sol: During machining of brittle materials the
cutting forces induced are higher, to
56. Ans: (d) withstand for this, high strength of tool is
57. Ans: (a) required which is achieved by negative rake
angle.
Sol: Common solution for 56 & 57
w = b = 3.6 mm. 'Cu = 460
60. Ans: (a)
D = 0.2 = t1, µ = 0.7 Sol: T1 = 10 min, Ve1 = l OO m / min
p = tan- µ = tan- 1 1
0.7) = 35
T2 30min, Ve2 l OO m / min
(
= =
a = 10 = orthogonal rake angle in ORS
For minimum force, 2<1> + p -a = 90 1n(�l J l n( 75 )
</J
=
90+ a - /3
2
'Cu xA 0
=
90+ 10-35
2
460x3.6x0.2
=
_
32 5
n=
1{
=
�J =
0.2815 =
wo
1.09
1nG�)
0_262

Fs = = = 616N
sin <I> sin 32.5
!11•1i!i§jjj§§.jj41fflftj@u.jj+yderabad I DelhiJ Bhopal J Pune I BhubancswarJ LucknowJ PatnaJ Bengaluru J Chennai I VtjayawadaJ Vmg I Tl11lpati J Kukatpally I Kolkata I
61. Ans: (c) 0.4 cos10
( )
tan·I l-0.4sinl 0 = 22.94
0
=
62. Ans: (b)
Sol: Common solution for 61 & 62 cos a .Y_
Shear strain rate = ( x J
Ve = 20m/min, a= 15 cos(<I> - a) /1Y
t2 = 0.4mm t1 = 0.2 mm coslO 2500
=( x J
0.2 = = cos(22.94 -10) 0.025
r = ..!.L = _!_ 0.5
t 2 0.4 2 = 101046.8 = 1.0104 x105
' 0.5cos15
<!> = tan- ( J = 29
1-0.5sin15 66. Ans: (b)
Sol: AtBEP, Ve = Vh = V & Te = Th

= 0.5 xV c = 0.5 x 20=10m/min


I
30001116 V 16
--- = I
63. Ans: (b) 2001 / 06
vo.6
Sol: With increase of nose radius, the surface
finish is improved without affecting MRR 2001 / 0.6 = vo.6-�I
___ v1 .0 42
I
=
30001116
64. Ans: (c) = 6839 =vl.042
149
Sol: Let Q = No. of pi eces produced
AtBEP V 1042 = 45.899
(T.C)Automate = (T.C)center lathe V = 39.339 = 39.4
(120x800)+(2xQx500)
= (30 x l 50)+(250xQx100) 67. Ans: (a)
96000 + 1000 Q = 4500 + 2500 Q
1500 Q = 91500 68. Ans: (b)
Q = 61 Sol: D = 100 mm , f = 0.25 mm/sec,
d = 4 mm
65. Ans: (c) V = 90 rn/min
Sol: r = 0.4, t1 = 0.6, a = 10, Ve = 2.5 Fe = 1500N

-i( Fe = N = 1500
<I> = Shear angle = tan rcosa )
. FT = F
1-rsma

!IHNM@hii4@jRflftiMmm� I I I I I I I I I
yderabad Delhi Bhopal Pune Bhubaneswar Lucknow! Patna Bengaluru Chennai Vijayawada jVizag Tirupati I Kuka(pally I Kolkata
" . ACE
"�. . . . Metal Cutting
••�F�Pohlntioos
� : 703:

69. Ans: (2.8 to 3.0) Specific cutting energy


Sol: C hip thickness= t2 = 0.5mm, Fe X Ve - 1.49
Rake angle (a) = 0, t1 xbxVe xlOOO
=

Feed (f) = t1 = 0.2mm/rev. Fe=1.49 X t1 X b X 1000 = 1490N


0.2 Because the power consumption is taking
r=.!!_ = = 0.4
t2 0.5 place only in the cutting stroke ,
r.co� a Velocity of tool in the cutting stroke
¢=tan-'( ) = 21.8
1-r.sm a = length of work or stroke length x
Shear strain= cot <I> + tan (<I> -a) RPM of the crank
= cot (21.8) + tan (21.8) = 2.9 = 200 x60 = l 2m/min
Power required = Fe x cutting velocity
70. Ans: (d) (1490x12)
= = 298W atts
Sol: Reduction in friction angle reduces friction 60
and hence the frictional energy losses are
reducing and therefore cutting forces will 72. Ans: 18.8
reduces. W ith reduction of friction angle, as Sol: a= 5° , N= 400 rpm
the cutting force reduces, the value of f = 0.4 m/min
(J3 -a) reduces, this will increase the shear d= 5 mm= b
angle <j>. ti = 3 mm
{9o - (p - a)} 0.4xlOOO =
2<1> + J3 -a=90 =><I> = t1 = f1 = 1 mm
2 400
t 1
C hip thickness ratio (r) = st¢ )
t2 3
r= _I=-
cos ¢-a
As the <I> increases the chip thickness ratio r cosa 0.33cos 5 =
tan<!> = = 0.342
(r) will increase. 1-r sina 1- 0.33xsin 5
As r= t1 /tz => t2 = t1 /r, the chip thickness t2 Shear angle <I> = 18.8°
will reduce.
73. Ans: 2.1
71. Ans: 298 Sol: a= 0 , Ve= 180 m/min
Sol: The given problem is the oblique machining FT = 490 N, µ = 0 .7
problem. W hen a= O
Hence, t1 .b = f.d = 0.25 x 4 = lmm2 Fe = N = N ormal to frictional force
\(} f !l�llHt 11111-; P11hlil,llltl[I', �ydcrabadl Dcllii IBhopal I PunclBhubaneswarl Lucknow! Patna I Bengaluru I Chcnnai jVtjayawadalV,zag ITirupati I Kukatpallyl Kolkala I
:.t ACE : 704:
':. ��Pubtirationa Production
� �=================================
FT = F = Frictional force � C = V 1 (T1) °- 14 (f1 )° -7(d1)°-4
1
F C = 45(30)°- 4 (0.35)0\2)0.4
µ = tanp = - = 0.7
N C = 45.8425
V2(T2)°- 14 (f2)°-1 (d2)°-4 = 45.8425
N=-=-T = 700 N = Fe
F F
0.7 0.7 (125x45)x(T2 )°-14x(l . 25x0.35)°-7x( l .25x2)°-4
180 = 45.8425
Power = P = Fe x Ve = 700x-
60
= 2100 W = 2.1 kW

77. Ans: 57.91 (range 57.8 to 58.0)


74. Ans: (a)
l C 2
Sol: a = 10 °c Sol: C m = � , C 1 = ;�c, VT 0·5 =150
f = 0.14 mm/rev = t1 TC = k + Cm + Ct
ti 0.28 mm
=
18C 270C
0.14 =k+ +
r=_S_= =0.5 V TV
t 2 0.28 18C 270C
=k + +
r cosa 0.5cos10 V
tane= =
1-r sin a 1-0.5sin10 vx(�i
tan 0 = 0. 539
e = 28.33° = 28 °.20 1 18C 270C V(�-, )
=k+ -+ ----
Shear strain = cot 0 + tan (0-a) v .!_
en
= cot 28.33 + tan (28.33-10)
d(TC)
2.186 For min TC, =0
dV
=

75. Ans: 825 to 835 210c v(�- ) x (!n -1)


2
-18C _0
Sol: Coefficient of friction = tan p = 0.4877 + -
V2 I

= µ = FIN = Fi/Fe= Ft / 1700 en


Thrust force = Ft = 1700 x 0.4877 = 829 N 2 1
270C V( 0-�5- ) x(- --1)
0.25 18C
=
76. Ans: (b)
1
v2
en
Sol: VT 0.1 4 f 0·7 d 0·4 = C 270x3
= 45
xV 2 =�2
� T1 30 min, = V1 m/min, 150 4 V
f1 = 0.35 mm, d1 = 2.0 mm

\( l l 11gJ[lll 1111!.!, P11lil1t,IIH)I\", FyderabadlDelhilBhopaliPwiclBhubaneswarl wcknowlPatnalBengalurulChennailVuayawadajVmg ITirupati I Kukatpallyl Kolkata I


: 705: Metal Cutting

18xl 50 4 Specific cutting energy


=
v4

270x3 _ power (or) Work done


:. V = 57.91 m/min MRR
Fe x Vc J / m3
=
78. Ans: (B) fxdxVc xl0 -6
Sol: a=22 , ti=0.8 , Fe=Sp.C.E x fx d xl0-6
V=48 , t 1 =0.4 =10 9 x0.25x4x10 -6 =1000

r=..!!_=0.5
t2 81. Ans: (1527.8872)
Sol: b=60 mm, V = 120 m/min
-1( r cosa
'I' = tan ) t=0.1 mm/tooth, d = 4 mm
1-r sin a
,1,.

End mill cutter, D=120 mm


0.5cos 22 z=12
=tan-i ( )=29_7 0
1-0.5sin 22 fm=ft X zxN
1000x120x1000
= o.1x12x
79. Ans: 16 (range 14 to 18) nx120
= 381.9718 rpm
v1 v1 1
Sol: V2 = 0.5V, , -= --=-=2
V2 0.5V1 0.5 MRR = bdfm = 60x4x381.9718
V 1 T 1 ° = V2Tz° = 9176.232 mm 3/min

(�: r =(�:J
= 1527 .8872 mm 3/sec

82. Ans: 106.069

l O t 1 0
150 => T = ( � ) o. =( � )
�: =(�} =(�:)'�' =(�:J VT0 · =
_l_ 10

Sol:
1

= (2)4 = 16
3 0 3 0
VTo.3=300 => T=( � ) o.3 =( � )3
I 10

80. Ans: (1000)


Sol: Orthogonal cutting At BEP , T = T and V = V
T=T
D = 100 mm, V e = 120 m/min,
f = 0.25 mm/rev , d = 4mm,
Specific cutting energy (Sp.C.E) = 10 9 J/m3

!MINMiiYhiMhiiRflMW!Hh� yderabad l Delhi l Bhopal ! Pune j Bhubaneswar i Lucknow ! Patna ! Bengaluru i Chennai l Vuayawada l Vu.ag jT1111pati j Kukatpally j Kolkata
: 7 06 : Pr oducti on

yio 10 i 1 __ t _ _ 0.2
- ( ;) - (150) °
-io -V rmin - i - - 0.5
10 t2m ax 0.4
V3 (300)3
V= 106. 069 1 m n co a
cl>min = tan- ( � i � ) = 28.18
1 rmin sma
83. Ans (b) 42 087
Ratio =cl> m ax = · =1. 493=1. 5
Sol: D = 20 mm , N 1 = 400 r pm= V1 , cl>min 28.18
T 1 = 20 min
N2 = 200 rp m= V2 , T2= 60 min , 85. Ans: 82.4 %
N3 = 300 r pm= V3 , T3 = ? Sol: B ec ause ther e is no change in the fe ed and
V1T i ° = V2T2n depth of c ut , the too l life equatio n c an be
wr itten as
T = C. V-2 5 :::::> T = (C / V2·5 )
:::::> y2 . s
T=C
Hence VI 2.5 T I -
- V2 2.5 T2 ,

also given that V2 = 2V1


T2 = T 1 (V1 N2 )2 - 5 = 0.176 T 1
% c hange in to ol life = (T 1 -T2 ) / T 1
= 0.824= 82. 4%

86. Ans: (d)


400 0.631 Sol: µ = co effic ient of fr ic tio n= FIN
= 20(-) 31.557= 32 min
300
=
= f6 / fs (As the diagr am)

84. Ans: 1.5


Sol: a =
9° , t1 = 0. 2 , t2 = 0.25 to 0. 4
t 0. 2
rm ax =_i_ = = 0.8
t2min 0. 25

rm ax co�. a
cl>m ax = tan-'(1-r sm ) = 42.087
m ax a

�ydcrabad ! Dcllii l Bhopal l Punc l Bhubancswar l Lucknow l Patna l BcngaJwu l Oicnnai j Vtjayawada l Vmg I TlfUpali I Kukatpa]]yl Kolkata I
�".
ACE : 707 : Metal Cutting
..- -......,�li'....:..-:....Piffcdiooa
� �=�
===�
=================================
if V3 = 1 00 m /min,
V1 T/1= V2 T;1
Five Marks Solutions

01.
Sol: Orthogonal machining operation,
a = 0, f = 0.25 mm /rev= t 1 ,
d = 2 mm = b
Fe = 1 000 N, t2 = 1 .0, FT = 500
0.25 = 1 20 X 20 0.252
r !.J_= = 0.25 V1 TnI = V3 Tn3 � Tn3
1 00
=
! 1
2
= 2.55 min � T3 = 4 1 min
.h _ r cos a
1(
'I'= tan )
1 - r sin a
03.
0 5
= tan- 1 ( ·� ) = 1 4.03 °
Sol: t 1 = 0.25, Ve = 60 rn/min,
a = 0 , tz = 0. 7 5, Fe = 900 N,
tan (,B - a)= Fr FT = 450 N
Fe
t, 0.25 1
500 r = - = -- = -
fJ= tan -i ( ) = 26.56
°
!2 0.75 3
1 000
<I> = tan-' ( rco� )= 1 8.43
a
Fs = cos (¢ + P - a)
cos ?
1 - rsm a
p - a)
1 000 60
--- cos(1 4 + 26.56) Power = P = Fe x Ve = 900 x -= 900 W
60
=
cos 26.56
F
= 849.36 N tan(p - a)= T
Fe
Fs . ,h'l' 849.36 .
-ru = - x sm = --- x sm 1 4
A0 2 x 0.25
� p - a + tan�' ( � )
= 4 1 0.9 N/mm2 = 41 1 MPa

= 0 + tan- 1 ( 450 )= 26.56


02. 900
Sol: T1 = 20 min V 1 = 1 20 ml min Coefficient of friction = µ = tan p = 0.5
T2 = 1 00 min, V2 = 80 m / min

!ltl1i@jjji4ijjjji QHIM\i/ln� yderabad I Delhi I Bhopal I Pune I Bhubaneswar I Lucknow! Palm I Bengaluru I Chennai I Vijayawada I Vi,.ag I Tuupati'. I Kukatpally I Kolkata
. .
ACE . .
,. , �».n,amgPnblicariooa
.,':.. "
.. � ....., : 7 08 : Pr oducti on

04. I
yo.35 500 0.35
Sol: L et Q = N o. of components produced I =
T.C for produci ngQ components yo. I 200°·1
}
onTurret lathe yI 0 -2.857 = 1.99X 1 015

= 3 X 5+ - x( 4+ 2) X Q
4
60 V= (1. 99xl 01 5
)7.14)

2 ° 39
= 15+ Q = (t . 99xl 01 5 ) " = 133 . 8 m /mi n
5
B reak even cutti ng speed
T.C for produci ng Q components
=

}
on center lathe
06.
14
= 2x 4. 5 + - x(4. 5 + 2.5) xQ Sol: D0 = 100,
60
= 9+ 1. 633Q Di = 96, orthogonal machi ni ng
AtBEP N= 120, f = 0.5 mm/rev
T.C on tu rret L athe= T.C m center lathe r = 0.3 , Fe = 800N , FT = 600N
F 600
15+I Q = 9+ 1. 633 Q -tan-1 -T = tan-1 - = 3 6. 86
B-a -
5 Fe 800

!
15- 9 = (1. 633 - ) Q = 1.233Q
Vr Ve
= --,----,-
si n<I> cos{<I> - a)
6 si n<I>
Q = -- = 4.86 � vf = Ve X
1.233 cos q> -a
( )

If no. of parts produced is less than 5, center


= Ve xr = 11.3 04 m / mi n
lathe i s economi cal and if no. of par ts 1 s
more than 5 turret lathe i s economi cal. 1tD o N
Ve =
1000
05. = 1tx100x120 = 3 7. 68 m / mi n
1 000
Sol: At BEP
37.68
VHSS = VCarbide Power= Fe x Ve = 800x-- = 502.4 W
60
THSS = TCarbide
= 0.5 k W

!Mlli@hiiiih4Rflbii\h•i!�yderabad I Dellii I Bhopal I Pune I Bhubaneswar I Lucknow I Patna I Bengalwu I Chennai I Vliayawada I V17.3g I TU11pari I Kukatpally I Kolkata I
: 7 09 : Metal Cutting

07. 09.
Sol: a=35° , t1 = 0.15 , Fc= 200 , FT =90 Sol: TO = original tool life
T0 1
t2 = 0.3, b=2.5 , Vc = 30m / min T1 = n= 0.125 or
2, 8
0.15=
r= 0.5
0.3

.,, =
,1,
tan
_1 ( rcosa ) V, = V, ( �: r = V, ( 2) 0
"' = 1.09 V0
1-r sina
V1 = l .09 Vo
0.5cos 35 =
tan-i( ) 29_8 6 V1 - V0
= 9%
1-0.5sin 35 % change in sp eed=
=

Vo

/3 -a=tan-1 -
Fr = tan-1 (-
90
10.
200
)
Fe
= 24.22 Sol: w= b= 2.5 mm , t1 = 0.25 , a= 0

Merchants constant , C = 2<j>+ f3 - a t2 = 1.25 , Fe = 900N , FT = 450N


= 2 X 29.8 6+ 24.22 µ = 0.5 ,..• u = ?.
= 8 3.95°
0.25= =
r=_S_ = _!_ 0.2
t2 1.25 5
08 . f3 - a= f3 = tan-l 0.5= 26.56
Sol: cr 0 = 98 0 , b= 1.6=d f, = t1 = 0.8
"'"' r cosa
tan )
Ve = 5.5 m/min , P= 0.67k W 1-r sina
=
_1 (
P= Fc x Vc = tan-I (0.2) =11.31
670
=>Fe = - x 60=7309N
5.5 Fs = xcos(¢ + /J -a)
cos�- a )
MRR = f.dVc = 0.8 x 1.6 x 5.5 x 1000
900 X
= 7040 mm3/ min COS (11.31+ 26.56)
cos 26.56
=

.
Sp ec1' fi1 c cuttmg energy = - p- = 794 N
MRR
670 = F 794
95 MW 'tu = s . sin<!> = x sin l l .31
7040xl 0-9 A0 0.25x 2. 5
=

= Sp ecific cutting p ressure 249MPa =

Machining constant= 2<j> + f3 - a= 49 .18

l111•i@hiii•hi4PfllhW!lih+Yderabad I Delhi I Bhopal I Pune I Bhubaneswar I Lucknow I Patna I Bengaluru I Chennai I Vliayawada I V17.ag j 'firupati I Kukatpally I Kolkata I
: 7 10: Production

11 .
Sol: C = 80, n = 0. 2
30
Cg = 2, L = = 0.5 1
m 60 ln( -)
l .2
= 0. 263
n L
VOP1 =C[- m ]" ln(0.5)
1-n C g
02
02 05
80[ · . · ] = 45. 94 m/min
0. 8 2
= '

1 2.
Sol: w = 40 mm, D = 70 mm, ( 1
38

T0 -) = 2.33T0
·
=
No. of teeth = Z = 30, N =
40 0.8
d = 2, fm = 20 mm/min % chang e in tool life
2 T2 -To
t1 max = fm � = = 1 .33 = 133%
NZ VD To
2 x 20
=
4o x 3o V w
/2
= 0.00563 mm
1 5.
Sol: D = 1 60 mm, N =
215rpm,
1 3. d = 2.5, f = 0. 16 mm/rev t1 ,A0 b x t1 = =

Sol: Norm al rake , an = tan- 1 (tan a x cos i) Tool g eom etry = 0- 10-8-9- 1 5-75-0 mm
Fe = 500, Fr = 200, = 0.48,
tan-1 (tan 120° x cos 10° )
f2
=

Assum e ASA d esignation


= 1 1 . 8°
a= O, 11 = 0. 1 6
tan 0'
Norm al cl earance 0n tan -I ( )

r = !J._ = 0· 1 6 = _!_ = 0.33
= =

COS l
f2 0. 48 3
tan 7°
tan 1 (-- ) 7. 1 </> = Tan - ' (0.33)=18. 43
_
=

-J
Cos l O
=

1 4. /J = Tan_, 200 =Tan-' (-1 = 21 .8


500 2.5
Sol: TO , VO = original tool life and velocity
Fc
If V1 = 1.2V0 T, = 0.5T0 F. = cos(<I> + � -a)
cos(� -a)
V2 = 0.8 V0 , T2 = ?

!IHIIU§jjji4ijjj$Rflftj@jjj.jj� yderabad I Delhi I Bhopal I Pune I Bhubaneswar I Lucknow I Patna I Bengalwu I Chennai I Vijayawada I Vizag I Tuupari I Kukatpally I Kolkata
. A:CE . .
��-
• ":·=,1!;L�1p1�-
:::1111�c�Pu�ili�i-�rJfi�,oo
Metal Cu
�•�========,..:=,::7�1,;,1 ,.:.:============��::::�:ttmg
:·�
500xcos(18. 43+ 21. 8) r co � a
= 411 <j>=tan-1 ( ) =24.74
cos 21. 8
=
1- r sma
0

F For minimum chip strain, shear angle


= s xsin<j> = 325MPa
Ao
't u

relation is
Note: This problem also can be solved by
2<j>-a =90
assuming/machine process. B ut the values
90+ a 90+ 20
obtained are nearly same. <I> = = =550
2 2

16 Ans: (b)
Sol: V1 = 63 m/min, T1 = 10 hrs 18.
C = 257. 35 Sol: V1 = 60 m / min, T1 = 40 min
V2 =lOO m / min, T2 = lO min

ln(�)
V1 A ccording to Tay lor' s tool life equation
Ti° =� =>n =
V1 ln(T1 )
257. 35
1n( )
63
= = 0. 22
ln(600) 1 n(�) ln(1 00 )
, 60 = 0. 368
v2 = 2x 4 =126 m / min n= =

257. 35 0 .22 257. 35 ·


_
I
4 54
m(;:) m(;�)
T2 =( ) =( )
126 126
Tailors constant, C = V1 Ti°
= 25. 6 min
= 60.40°368 = 233. 2
Tool change time= Tc = 2 min
17.
Sol: L2 = 84 mm,
n- --
1
0
L 1 = 200 mm Vopt fo r max. Production = C(- . )
1-n Tc
V= 30 m/min, t1 = 0. 5 mm, 20
0368
a=

0. 368 1
r = L =0. 42 = 233. 2[--- x-]
1-0. 368 2
2

LI

t t 0. 5 148 m/min
r=-i => t2 =-i =- - =1. 19 mm
=

t2 r 0. 42
\(}
>
f 11L:,llll l IIll� J 11lil11 1111111 ...
�ydc
I I I I I I I I I I I
rabad Delhi Bhopal Pune Bhubaneswar lAicknow Pama Bengaluru Chennai V\jayawada Vizag Tirupati I Kukalpally I Kolkata I
: 7 12 : Pr oducti on

19. 21.
Sol: a= 7° , t2 = 2.5 m , t, = 1mm Sol: n= 0.12 , C = 130
t 1 C 1 = 1.1 X 130= 143 ,
r = -i = -= 0.4
t2 2.5 V= V1 = 90 m /min

r cos a 130 o.ii


"'= ta n - i ( ) VT = C � T = ( ) = 21.4 mi n
90
,1,.

1-rs ina
O

0.4cos 7 =
= ta n-1( ) 22. 65o = c =>r = (1 � ) = 47.4 min
Yo. 1 2

9
y 1 r 1"
1-0.4s in 7
i 1

According to mercha nts theory I ncreas ed tool life = 47.4 min


24> + p-a = 90 I ncreas e in tool life = 47.4- 21. 4= 26 min
� P=90 + 7 -2 X 22.65=51.690
22.
f
20. Sol: R, 1 = -----
ta nC s + cotC e
Sol: At brea k even p oint ,
f
Va = Vb a nd Ta = Tb = = 0.084f
ta n 25+ cot 5
f
R1 2 = ---- = 0.114f
ta n 30+ cot 7

\
5 = yH =>y = 2 "
�5)'
( Tool " l" gives low R1 va lue , hence better
s urfa ce finis h is p roduced by tool " 1"
= 0.77 m /s ec
I 23.
( 2·-
T= -
5 )m = 12348 s ec
Sol: D0 = 32 , a= 35° , K 1 = 0.1 mm,
0. 77
If V= 1 m/s ec Fe = 2000 N , Ve = 15 m/mi n ,
8 L 2 = 60 mm , FT = 800N
TA
(2 5)
= -t- = 1526Sec
r= s_
L
=�=�
rtD 32 1t x
= 0.59
1 0

TB = (fr = 2401Sec t t 0.1


r = -i � t2 = -i = - - = 0.169
t2 r 0.59
From the a bove tool B has longer tool life
0.59cos 35
hence a t V= 1 m/s ec , tool B is chos en. q> = ta n-i( J 36_15o
1- 0. 59s in 35
=

\( I I 11g111t • 1 1 111!. P11hlu .111,,11, yderabad I Delhi I Bhopal I Pune I Bhubaneswar I wcknow I Patna I Bengaluru I Chcnnai I Vgayawada I Vizag I Tuupati I Kukatpally I Kolkata
: 713 : Metal Cutting

F 800 Using merchant theory


tan(l3 -a) = T =
Fe 2000 2� +13-a = 9 0
800 9 0 + a -13
13 =a+ tan-1 = 35 + 21.8 = 56 .8 �= = 31.5
2000 2
µ = tanl3 = tan 56. 8 =1. 52
(I n general µ< 1) 26. Ans: (d)
Hence by apply ing classical fr iction Sol: L et Q = N o. of components produced
theorem
1 T.C for producingQ components on s tan<l ard
ln(!) ln(-) machine tool
_r_ 0. 59 0. 5276
µ= = = = 0. 55
I . 04 = 0.5 X 200 +- X 200 X Q
22
-a -35x�
2 2 180 60
7t 7t

= 100+ 20Q
22
6
T.C for producing Q components
=8.85 m/min
on automatic machine
24. (i)Ans: (a), (ii) Ans: (d)
= 2x800+- x800xQ
5
Sol: N =200 rpm, f=t1 =0.25 mm, 60
d = b = 0. 4 mm, � =27. 75 =16 00+ 5/6 0x 800Q
sin� sin 27. 75 AtBE P
r= = = 0.49
cos(� -a) cos 17. 75 T.C on standard machine
t, 0.25 =T.C on automatic machine
- = r � t2 =-- = 0. 511 22
t2 0.49 100+ X 20Q =16 00+66 .6 7Q
6
Using merchant theory
1500
2� +13-a =9 0 Q= = 225
6.6 7
13 =9 0 + 10- 2X 27.75=44. 5
µ = tanl3 = 0.9 8 27.
Sol: D iameter of fi nished hole= 75
25.
Rise per tooth=0. 03,
Sol: a = 5

p = n + tan-'( :: J 450
=5+ tan - =32
9 00
-I
Tooth pitch = 1 0mm
L ength of broach = 750mm
Ve = 6 m/min
!1111i@hiiiih4@fiiMl!ild� yderabad I Delhi I Bhopal I Pune I Bhubaneswarl Lucknow! Patna I Bengaluru I Chennai I Vijayawada j Vizag I T1I11pati I Kukatpally I Kolkata
: 7 14 : Production

Specific cutting pressure If Tc = 1 5 min


= 1 300 x t 0·4 N / mm 2 n 1
VA opt = c[-- x -]"
750 1 - n Tc
No. of teeth = = 75
10 ]
Total material removed = 7 5 x 0.03 =
[o
C --
2 1
· x-
0·2

= 44.09 m / min
0.8 15
= 2.25 mm
= 45 m/min
:. Uncut chip thickness = t = 2.25 mm
Specific cutting pressure = 1 300 x 2.25 °.4
29.
= 1 798 N/mm2
Sol: e e 6° a b = 1 0°
Cutting force,
=

Fe = Sp. Cutting pressure x Area of metal removal Or = 1 .5 mm, R = 25


0 r ) . -1 1 .5
1 798 x tr (75 2 - 70.5 2 ) 924 kN er = sm. -, (- = sm (-)= 3 . 44 0
R 25
= =
4
Capacity of the broaching machine abe = Effective rake angle
= 924 x l OOO I ( l OOO x 9.8 1 )
= a b + er = 1 3 .44
= 94 tonnes
Bee = Effective relief angle
Power required = Fc x Vc
= ee - e f = 2.56
= 924 x l 0 x �= 9 2kW
3
Rubbing of tool on work will takes place
60
750 When
Broaching time = � = = 0. 1 25
w 6000 e ee = 0 � e ee = 0 e - e f � e e = 0r = 6 0
Total time per piece =0. 1 25 +0.5 =0.625 min .
S Ill e e = Sill. e f = Sill. 6 0 = -
or1
R
Production rate = �= 96
0.625
R1= � = 1 4.35mm
sin 6 °
28.
D1 2 x R1 28.7 mm
Sol: If tool life = 1 80 min
= =

l OO �=
VA = 0.2
= 1 800.2 35 4 3 0.
Sol: Let Q = BEQ
T

1 20 1 20 =
VB 32 .76 T.C on G.P lathe = T.C on turret lathe
1 80 0.25
T0.25
= =

Because Tool A has higher cutting speed at 20 5


- x 1 8 x Q = - x 1 8 x Q + 3000
60 60
same tool life. The tool A is the Best
!iJ11ijj§jjj§§.jjjiidjniMJ!i.jj� yderabad I Delhi I Bhopal I Pune IBhubaneswar I Lucknow I Patna I Benga)uru I Chennai IVtjayawada I Vu.ag ITirupati I Kuk.atpal)yl Kolkala
: 715 : Metal Cutting

6Q = 1 .5Q + 3000
f2 = � ' d 2 = 2d
4.5Q = 3000 2
Q = 667

31. b c
Sol: f = 0.25 mm = t 1 , d = 4 mm = b,
Fe = 1 600 N, FT = 800 N, t2 = 0.6,
V2
v1
=
f2 (� J (�J d2

a = 0°
l

(l
O. 5
= 2 °3 -
) =1.11
F
tan (p - a)= T
Fe
800 V2 -V1
tan p= µ= --= 0.5 % change in speed = = 1 1%
1 600 v1
p = tan - I 0.5 = 26.56 ° Productivity is proportional to MRR
r= !i_= 0.25 = 0.42 % change in productivity
12 0.6 _ MRR 2 - MRR I
M RR I
-i( 0.42 cos 0
0= tan J
1 - 0.42 sin 0
= tan - 1 0.42 = 22.62 °

F8 = (c
cos p - a )
x cos(<!> + P - a) 33. (i) Ans : (b), (ii) Ans : (b)
Sol: D = 1 00 mm
1 600
= x cos 49.2= 1 1 69 N Parting off operation 1s orthogonal
cos 26.56
machining
F b = 2 mm, f = 0.2 mm
Shear strength = -r u = s x sin <I>
Ao Ve = 60,
1 1 69 Specific cutting energy = 800 MPa
= --- sm 22.62= 450 MPa
0.25 x 4 Cutting force, Fe = Sp. Energy x A0
= 800 x 0.2 x 2 = 320 N

32. Maximum power required = Fe x Ve


60
a = O, 3 b = O, 3, = 0, 1 5
= 320 x -= 320 W
C 60

!MIIN!Qdiii@QAflbj@j@j.:ij>HYderabad I Delhi I Bhopal I Pune I Bhubaneswar l Lucknow ! Patna I Bengaluru I Chennai I V\iayawada ! Vu.ag I Tuupati I Kukatpallyl Kolkala I
: 716 : Production

34. 37. (i) Ans: (c) , (ii) Ans: (b) , (iii)Ans: (c)
Sol: Let Q = no. of parts produced Sol:
T.C on E.L = T. C on T.L a = 6° , Ve = 0. 05m/s
30 60
- x Q x 80 = 500+- x Q x 1 60 b= w = 3, d = t1 = 1mm
60 60
40Q = 500 + 1 6Q ti = 1 .5 mm

40Q -1 6Q = 24Q= 500 Use the relation 2<j> + j3 -a = 90

500 t, 1 2
Q= = 20.83 = 21 r = - = - = - = 0.67
24 t2 1 . 5 3
35.
1( 0.67 cos6
Sol: Ve = 30m/min ,Vr = 1 5 m/min, a = 1 0° <j> = tan _ ) = 35_ 62
1 -0. 67sin 6
v
r= f = � = 0.5 j3 = 90 + U - 2q> = 90 + 6 -2 X 35.62
Ve 30
= 24. 75
e = tan-I ( 0.5COS 1 0 ) = 283 3 µ = tanj3 = tan 24. 75 = 0.461
1 - 0.5sin 1 0
V5 V V Vr = rVc = 0.67 x 0. 5x 60= 20m/min
-- = -r- = V = -r- x cosa
cosa sin<j> s sin<j> Area of sh ear plane = As = Ls x b
15 t,_
- x_ b= lx3 = _
= --- x coslO = 31 m/min = 5 2mm 2
sin 28.33 sin <j> sin 35.62
36.
Sol: d = 2 mm , w = b = 1 5 mm
38.
Ve = 0. 5m/s, a = 0
Sol: Cg = 5
Fe = 900, FT = 600, <!> = 30
75 =
600 = L = 1 .25
j3 = a+ tan- 33.69
1 m 60
900
VT 0 · 1 = 1 00
2
µ = tanj3 = 33. 69 = -
3 l -n C g
Optimum Tool life = Topi = ( . )
Pow er, P= Fe x Ve = 900x 0.5 = 450W n Lm

Length of shear plane 1 - 0.1 5


=( x -- ) = 36
_ ti 2 0.1 1 . 25
Ls - -- = -- = 4mm
sin<j> sin 30

!li11i@mi4.jji4RflCiffi\@j.ij,Hyderabad J Delhi J Bhopal I Pune I Bhubaneswar I Lucknow I Patna J Bengalwu J Chennai I Vijayawada JV,zag J Tirupati J Kukatpally J Kolkata I
: 7 17 : Me ta l Cutting

39. 41.
Sol: So = 0.12mm= t1 ,
t= 2.0 mm,
9.8x 30°.4 Maj or cutti ng for, b = p2 = Fe
= 60.4
0.40.5
=

=S0 .t.t s (� se cy -tany + 1)


VT0·2 = 60.4, n = 0.2, c = 60.4
So = 0.12, r8 = 400
n
n L
Vopt = c[- . m ] 0.22
1-n C g t=2- 0, � =� =� = =1.83
t1 S0 0.12
]o. y= O
[ °· 6%
2
2 _
= 60.4 ___§Q =13.8lm / mi n
1- 0.2 400 P2 =
0.12x 2.0 x 400 (1.83 se cO- tan O + 1)
=272N

40. Powe r,
Sol: V1 = 50 m/mi n T1 = 45 mi n
52.6
V2 = 100 m/mi n T2 = 10 mi n = p2 x Vr x i:.1.::, = 271x -- x l.83
60
= 436 W

42. (i ) Ans: (c) , (ii ) Ans : (d)


Sol: a= 10° , t1 = 0. 3, b = 2, <I> = 36
t u = 450MPa, 2<!> + B -a =90°
B =90 + a - 2<!> =90 + 10 -36x 2= 28
µ = tanB = tan 28 = 0.532
n 1 t u x A0 450x 2x 0.3
Vopt for max . Producti on= c[__ .--] n Fs =t u x A s = =
1-n TC si n<!> s in 36
0. 61
0.461 _!_ 4 459.35N
289.13[ x ]
=

1-0.461 2
=

= 195.44 m/min

l1i•#i@jjj44ijjj$jmftiM\U,jj� yderabad I Delhi I Bhopal I Pune I Bhubaneswar I Lucknow I Patna I Bengalwu I Chennai I Vijayawada I Vuag I Tirupati I Kukatpally I Kolkata
Machining
(c) Hob and work pi ece bo th are rotating
One Mark Questions (d ) A sp ecial multi- too th cutter (hob) 1s
used
O 1. The stand ard sp ecifica tion of a grinding
w heel is A-46-M- 6-V-21. I t m eans a w heel 05. Teeth of internal spur g ears can be
of (GATE-ME-88) accurately cut in a (GATE-ME-89)

(a) Aluminum oxi d e of m esh siz e 6 (a) Milling m achine


(b) G ear shaping m achine
(b) Boron carbi d e of m esh siz e 4 6
(c) Slotting m achine
(c) Aluminum oxi d e of m esh size 4 6
(d ) Hobbing m achine
(d ) Silicon carbi d e of m esh siz e 6

02. In twist drills . . . .. . . . . (sm all/larg e) point 06. Minimum dim ensional and form a ccuracy
can be obtained in the cylind er bores of
angl e and ..... ... (sm all/larg e) helix angle
automobile engines if the bores are finished
are provi d ed for drilling soft, low streng th
by (GATE-ME -89)
steel. (GATE-ME-88)
(a) L apping
(b) Reaming
03. Cutting sp eed in grinding is set to a hi g h (c) Internal grinding
valu e to (GATE-ME-88) (d ) Honing
(a) Reduce the cu tting tim e
(b) Increase the bond streng th 07. If the longitudinal fe ed in center-less
(c) Improve cooling of job and w heel grinding is expressed by Vr = 7t D. N sin a.,
(d ) Reduce the w heel w ear D stands for (GATE-ME-90)
(a) Di am eter of blank
04 . Gear hobbing pro duces more accurate g ears (b) Diam eter of finished work pi ece
than milling because in hobbing (c) Diam eter of control w heel
(GATE-ME-89) (d ) Di am eter of grinding w heel
(a) There is a continuous ind exing operation
(b) Pressure angle is l arger than in milling

ImI xtllHIIIMIIIII IMCiM\ � yderahad I Delhi I Bhopal I Pune I Bhubaneswar I Lucknow I Patna I Bengaluru I Chennai I Vtjayawada I Vizag I Tirupati I Kukatpally I Kolkata
: 719: Ma chining

08. I n small lot produc tion for mac hining T­ 13. Plain milling of mild steel plates produc es
slots on mac hine tables, it is ex pec ted to use (GATE-ME-95)
(GATE-ME-90) (a) I rregular shaped disc ontinuous c hips
(a) Shaping mac hine (b) Regular shaped disc ontinuous c hips
(b) B roac hing mac hine (c ) C ontinuous c hips without BUE
(c ) V ertic al milling mac hine (d) J ointed c hips.
(d) Horiz ontal milling mac hine
14. D iamond wheels should not be used for
09. W hen 1.0% plain c arbon steel is slowly grinding steel c omponents (T/F)
c ooled from the molten state to 740C , the (GATE-ME-95)
resulting struc ture will c ontain
15. The rake angle in drill
(GATE-ME-90)
(GATE-ME- 96)
(a) Pearlite and C ementite
(a) I nc reases from c enter to periphery
(b) Ferrite and C ementite
(b) D ec reases from c enter to periphery
(c ) Austinite and Ferrite
(c ) Remains c onstant
(d) austenite and C ementite
(d) I s irrelevant to the drilling operation.

10. I n horiz ontal milling proc ess . ... ...... .... ... 16. Hel ix angle of fa st hel ix drill is normally
(up/down) milling provides better surfac e (GATE-ME-97)
finish and .... .. .... . .. ... Up/down) milling (a) 35° (b) 60° (c ) 90° (d) 5°
provide longer tool life . (GATE-ME-92)
17. Abrasive material used in grinding wheel
11. To get good surfac e finish on a turn ed j ob, selec ted for grinding fe rrous alloy s is
one should use a sharp tool with a ... ..Feed (GATE-ME-00)
rate and .. ... Speed of rotation of thej ob. (a) SiC (b) diamond
(GATE-ME- 94) (c ) Ah0 3 (d) boron c arbide

12. Among the c onventional mac hining 18. D eep hole drill ing of small diameter, say 0.2
proc esses, max imum spec ific energy 1 s mm is done with E DM by selec ting the tool
c onsumed in (GATE-ME-95) material as (GATE-ME-00)
(a) Turn ing (b) D rill ing (a) copper wire (b) tungsten wire

(c ) Pl anning (d) G rinding (c ) brass wire (d) tungsten c arbide

jltl1ijj§jjji41hQAflG5ij@� yderabad I Delhi I Bhopal I Pune I Bhubancswar I Lucknow I Patna I Bcngaluru I Chennai I Vrjayawada I Vmg I Tuupari I Kukatpally I Kolkata
w t �"16 ACE
�"•._.,..);'� D..
Cl.w1.� ----:
lulaAluuull
-lUIID : 720 : Production
,.

19. Allow ance in limits and fi ts refers to 23. Quality screw threads are produced by
(GATE-ME-01) (GATE-ME-03)
(a) m aximum clearance betw een shaft and (a) threa d milling
hole (b) thread chasing
(b) minimum clearance betw een shaft and (c) threa d cutting w i th single point tool
hole (d ) threa d casting
(c) difference betw een m aximum and
minimum siz e of hole 24. A goo d cutting flui d should have
(d ) difference betw een m aximum and (GATE-ME- 03)
minimum siz e of shaft (a) low therm al conductivity
(b) hig h sp ecific heat
20. A lea d screw with half nuts in a lathe, free (c) hig h viscosity
to rotate in both directions has (d ) hig h d ensity
(GATE-ME-02)
(a) V- threa ds 25. Internal g ear cutting op eration can be
(b) Wi th-worth threads p erform ed by (GATE-ME-08)
(c) Buttress threa ds (a) Milling
( d) Acm e threads (b) S haping with rack cutter
(c) S haping with pinion cutter
21. The hardness of a grinding w heel 1s (d ) Hobbing
d etermined by the (GATE-ME-02)
(a) hardness of abrasive grains 26. Abrasives are no t used in
(b) abili ty of the bond to retain abrasives (GATE-PI-89)
(c) hardness of the bond (a) Buffing process
(d ) abili ty of the grinding w heel to p enetrate (b) Burnishing process
the work pi ece (c) Polishing process
(d ) Sup er finishing process
22. Trep anning is p erform ed for
(GATE-ME-02) 27. In turning op eration the surface finish can
(a) finishing a drilled hole be improved by d ecreasing
(b) producing a large hole wi thout drilling (GATE-PI- 89)
(c) truing a hole for ali gnm ent (a) cu tting sp eed (b) F eed p er revolution
(d ) enlarging a drilled hole (c) Rake angle (d ) Nose radius

!IJ•lii/�hi§§,ii/Qilftfti@\.iih/.� yderabad I Delhi I Bhopal I Pune I Bhubaneswar l Lucknow! Patna I Bengalwu I Chennai j V\jayawada I Vmg I T111Jpati I K.ukatpally I Kolkata
: 72 1: Machining

28. Reamer is d esigned to have even number of 33. A blind hole 1s better tapp ed with a tap
flutes to: (GATE-PI- 90) having (GATE-PI-92)
(a) Balance the cutting forces (a) rig ht hand flutes (b) left hand flutes
(b) C onform to shop floor stand ard ( c) strai g ht flutes ( d ) no flutes
( c) Enable measurement of the reamer
diameter 34. In W hitw orth quick return mechanism (used
(d) H elp in regrinding of reamer. in shap ers) the velocity of the ram is
maximum at (GATE-PI-01)
29. Assertion: single start threa ds are used for (a) mi d dle of the forw ard stroke
fastening purposes. (b) beginning of return stroke
Reason: single start threads are easi er to ( c) end of return stroke
produce in a l athe. (GATE-PI-90) ( d ) mi d dl e of the return stroke

30. The teeth of both external and internal spur


35. A grinding wheel is sai d to be glaz ed if
g ears can be cut on (GATE-PI-91)
(GATE-PI-95)
(a) milling machine
(a) grains have become blunt, but d o not fall
(b) g ear shaping machine
out
( c) g ear hobbing machine
(b) g ap betw een the grains is filled by sw arf
( d ) g ear shaving machine
( c) it becomes black due to burning
31. Amount of energy consumption p er unit ( d ) part of the w heel is chipp ed off
volume of metal removal is maximum in
(GATE-PI-91) 36. Reamers have usually even number of teeth
(a) Turning (b) Milling so that the cutting forces on teeth are
( c) R eaming ( d ) Grinding prop erly balanced. (T/F) (GATE-PI-95)

32. In a grinding w heel marked with 37. The material most commonly used for
AA -48 - L - 7 - V - 25, L refers to manufacturing of machine tool beds is
(GATE-PI-92) (GATE-PI-95)
(a) abrasive typ e (a) MS (b) Gray CI
(b) w heel structure ( c) W hite CI ( d) Galvanized iron
( c) wheel hardness
( d ) manufacturers cod e

!IS11i@hi§4ijjj4Rflftj@mfij� yderabad I Delhi I Bhopal I Pune I Bhubaneswar I Lucknow I Patna I Bengalwu I Chennai I Vijayawada I Vizag I Tirupati I Kukatpally I Kolkata
...
.A!CE Production
� :Fnpw.rqP11Niratioos
� . ',>,>
.;
.., w • • • •
: 722 :
,> �·,;;;;;;;�;;;�;;;;;;;;�============================
38. W hi ch one of the following is not a (c) friction w el ding
synthetic abrasive m aterial (GATE-PI-03) (d ) laser w elding
(a) silican carbi d e
(b) aluminum oxi d e 42. Tool life in the case of a grinding w heel is
(c) titanium nitri d e the tim e (GATE-PI-05)
(d ) cubic boron nitri d e (a) betw een tw o successive regrinds of the
w heel
39. The drawing shows a m achined shaft. The (b) taken for the w heel to be balanced
purp ose of provi d ing round ed grooves at M (c) taken betw een tw o successive w heel
and N is to (GATE-PI-03) dressings
(d ) taken for a w ear of 1 mm on its diam eter
·1
43. In a shap er m achine, the m echanism for tool
feed is (GATE-PI-05)
(a) G eneva m echanism
(a) Supply lubricating oil for bearing (b) W hitw orth m echanism
m ounting (c) Ratchet and Pawl m echanism
(b) Facilitate facing op eration (d ) W ard -Leonard system
(c) Facilitate grinding of bearing m ounting
surface 44. The final finishing process for the surface
(d ) Reduce stress concentration plate m a d e of cast iron w hi ch is used as a
referenc e surface is (GATE-PI-06)
40. The fl anks of teeth of rack-typ e g ear cutter (a) Buffing (b) Grinding
used for cutting involute g ear profil es are: (c) H and scraping (d ) honing
(GATE-PI-03)
(a) Cycloi d al (b) Circular 45. Reaming is prim arily used for a chi eving
(c) Involute (d ) Straig ht (GATE-PI-07)
(a) Hig her MRR
41 . T he cutting p ortion of a drill is to be w el d ed (b) Improved dim ensional tol erance
to its shank. The process best suited for this (c) Fine surface finish
1s (GATE-Pl-04) (d ) Improved p ositional tol erance.
(a) ultrasonic w elding
(b) electron b eam w elding
IM•lii@iiii4@iRflbii\dih� yderabad I Delhi I Bhopal I Pune I Bhubaneswar I Lucknow I Pama I Bengalwu I Chennai I Vijayawada I V17.3g ! Tlfllpati I Kukalpally I Kolkala
: 723 : Machining

46. W hich of following g ear m anufacturing longitudinal fe ed


(d )
processes is based on g eneration principl e. transverse fe ed
(GATE-PI-07)
(a) Gear hobbing (b) G ear shaping 51 . A steel bar 200 mm in diam eter is turn ed a t
(c) Gear milling (d ) g ear shaving a feed of 0.25 mm/rev wi th a d ep th of cut of
4 mm . The ro tational sp eed of the work
47. In the m anufacture of twist drills, the shank pi ece is 1 60 rpm . The m a terial rem oval ra te
is j oined to the body of the drill using in mm 3/s is (GATE-ME & PI- 13)
(GATE-PI-06) (a) 1 60 (b) 1 67.6
(a) Spo t w el ding (b) TIG w el ding (c) 1 600 (d ) 1 675.5
(c) Proj ection w el ding (d ) Fri ction wel ding
52. Match the Machine Tools (Group A) with
48. W hich of the following pow d er production the probabl e Op erations (Group B)
m ethods produces spongy and p orous
Group A Group B
p articl es? (GATE-PI-10)
P: C entre Lathe 1 : Slotting
(a) Atomization
Q: Milling 2: C ounter-boring
(b) Reduction of m etal oxi d es
R: Grinding 3: Knurling
(c) Electrolyti c d ep osition
S: Drilling 4: Dressing
(d ) Pulveriza tion
(GATE-ME-14-SET-2)
49. The binding m aterial used in cem ented (a) P -1 , Q -2, R - 4, S -3
carbi d e cu tting tools is (GATE-Pl-11) (b) P -2, Q -1 , R - 4, S-3
(a) grap hi te (b) tungsten (c) P - 3, Q -1 , R - 4, S -2
(c) nickel (d ) cobalt (d ) P - 3, Q-4, R - 2, S-1

50. Grinding ratio is d efined as 53. Two separate slab milling op erations, 1 and
(GATE-Pl-11) 2, are p erform ed with i d enti cal milling
Volum e of wheel w ear cutters. The d ep th of cut in op eration 2 is
(a)
Volum e of work m aterial rem oved twice that in op eration 1 . The o ther cutting
Volum e of work m aterial rem oved param eters are i d entical. The ratio of
(b)
Volum e of wheel wear m aximum uncu t chip thickness in op era tions
Cutting sp eed 1 and 2 is ----
(c)
fe ed (GATE-ME-2014-SET-4)

!1"S•l$i@jjj§§jjji4jmn@\liih� yderabad I Delhi I Bhopal I Pune I Bhubaneswar I Lucknow I Patna I Bengaluru I Chennai I Vijayawada I Vuag I Tirupati I Kukatpally I Kolkara
: 724 : Production

54. Match the following: (d ) Cutting spee d in grinding process 1s


Group-I (Mechanism) Group-II (Machines) much lower than that in face milling.
P Quick returns 1 Lathe
Q Apron 2 S haping 58. The following d ata is applicable for a
R Intermittent indexing 3 Gear hobbing turning operation. The length of j ob is 900
S Differential mechanism 4 Milling mm , diameter of j ob is 200 mm , feed rate is
(GATE-PI- 2014) 0.25 mm/rev and optimum cutting spee d is
(a) Pl- Q2-R4-S 3 (b) P2-Ql-R4-S 3 300 m/min. The m a chining time (in min) is
(c) P4- Ql-R2-S3 (d ) P2-Q3-Rl-S4 (GATE -16 -SET -2)

55. Reaming is a process used for 59. Internal gears are m anufactured by
(GATE-PI- 2014) (GATE -16 - SET -3)
(a) creating a circular hole in metals (a) hobbing
(b) cutting a slot on the existing hole surface (b) shaping with pinion cutter
( c) finishing an existing hole surface (c) shaping with rack cutter
(d ) m aking non- circular holes in metals (d ) milling

56. In a m achining operation, if the generatrix 60. Metric threa d of 0. 8 mm pitch is to be cut on
and directrix both are strai g ht lines, the a lathe. Pitch of the lead screw is 1. 5 mm .
surface obtained is (GATE -15 -Set 3) If the spindle rotates at 1500 rpm , the speed
(a) Cylindri cal (b) helical of rotation of the lead screw (rpm ) will be
(c) Plane (d ) Surface of revolution (GATE - 17 -SET -1)

57. W hi ch one of the following statements 61 . Turning , drilling, boring and milling are
related to grinding process is INCORRECT? comm only used m achining operations.
(GATE - PI -15) Am ong these, the operation (s) performed
(a) Grinding w heels m ade of fine abrasive by a single p oint cutting toolis (are)
grains produce better surface finish. (GATE - PI - 1 7)
(b) Abrasive grains tend to fracture (a) turning only
frequently during the grinding process. (b) drilling and milling only
(c) Specific energy is hig her than that in (c) turning and boring only
turning. (d ) boring only

!IH1i@jjjqg.jjj4Rflft@iiiih� yderabad I Delhi I Bhopal I Pune I BhubaneswarJ Lucknow I Patna I Bengaluru I Chennai I Vtjayawada j Vmg I Ttrupati I Kukatpally I Kolkata
.t . A:CE . . : 725 : Machining
� �==================================
� ��Pubtiations

04. Component process (GATE-ME- 92)


Two Marks Questions A. Square hole in a high strength alloy
B. Square hole in a ceramic component
01. For cutting double start screw threads of C. Blind holes in a die
pitch 1.0mm on a lathe, the thread cutting D. Turbine blade profile on high strength
tool should have a feed rate of alloy
(GATE-ME-91) 1. Milling 2. Drilling
(a) 0.5 mm/rev (b) 1.0 mm/rev 3. ECM 4. Jig boring
(c) 2.0 mm/rev (d) 4.0 mm/rev 5. EDM 6. USM

02. A milling cutter having 10 teeth is rotating 05. A milling cutter having 8 teeth is rotating at

at 1OOrpm. The table feed is set at 50mm per 150 rpm. If the feed per tooth is 0.1mm, the

minute. The feed per tooth in mm is speed in mm per minute is

(GATE-ME- 91) (GATE-ME-93)


(a) 5 (b) 0.5 (c) 0.2 (d) 0.05 (a) 120 (b) 187 (c) 125 (d) 70

03. Match the products and their manufacturing 06. A grinding wheel A 27 K7 V is specified for
process (GATE-ME- 91) finish grinding of a HSS cutting tool. What
do you understand about the wheel from the
Products above code. Is this an appropriate choice ?
A. Porous bearings (GATE-ME-94)
B. Fly wheels (a) yes
C. Double end spanners (b) no, because abrasive is not correct
D. Plastic bottles (c) no, grain size is not correct
(d) no, because grade is not correct choice
Manufacturing process
1. Powder metallurgy 07. Machine tool, structures are made . . . . . . for
2. Casting high process capability (tough/ rigid/ strong)
3. Closed die forging
(GATE-ME-95)
4. Blow molding

l1ti•ii@jjjgg.m;QnftlM\11jjj� yderabad I Delhi I Bhopal I Pune I Bhubaneswar I Lucknow I Pa111a I Bengaluru I Chennai I Vijayawada ! Vmg I Ttrupati I Kukatpally I Kolkata
., ... . ACE
. . . : 726 : Production
�-�

� �====================================
:&ipw,qPohliminns

08. 1 2. The tim e taken to drill a hol e throug h a


Manufacturing Conditions 25mm thi ck plate with the drill rotating at
process 300rpm and m oving at a fe ed rate of
A. Finish turning I . backlash eliminator 0.25mm/rev is (GATE-ME-02)
B. Forming 2. Z ero rake (a) 1 0 sec (b) 20sec
C . Thread cutting 3. Nose radius (c) 60sec (d ) 1 00sec
D . d own milling 4 . low sp eed
1 3. Throug h holes of 1 0 mm diam eter are to be
(GATE-ME-95)
drill ed in steel plate of 20 mm thi ckness.
09. D etermine the transformation ratio (TR) of Drill spindle sp eed is 300 rpm , fe ed 0.2
the g ear train connecting the spindle and the mm/rev and drill p oint angl e is 1 20° .
l ead screw of 4TPI required to cut 1 .0mm Assuming drill over travel of 2mm the tim e
pitch screw threads in a center lathe. for producing a hole will be
(GATE-ME-95) (GATE-ME-04)
1 0. The abrasive material used in grinding (b) 25 seconds
(a) 4 seconds
w heel sel ected for grinding of ferrous alloys
(c) 1 00 seconds (d ) 1 1 0 seconds
lS (GATE-ME-00)
(a) Silicon carbi d e (b) Diam ond
14. A 600 mm x 30 mm flat surface of a plate is
(c) Aluminium oxi d e (d ) Boron carbi d e
to be finish machined on a shap er. The plate

1 1 . If each abrasive grain is view ed as a cutting has been fix ed with the 600 mm sid e along
tool, then w hi ch of the following represents the tool travel direction. If the tool over­
the cutting param eters in comm on grinding travel at each end of the plate is 20 mm ,
op eration.? (GATE-ME-06) averag e cutting sp eed is 8 m/min, fe ed rate
(a) Larg e negative rake angle, low shear is 0.3 mm/stroke and the ratio of return tim e
angle and hig h cutting sp eed
to cutting tim e of the tool is 1 : 2, the tim e
(b) Larg e p ositive rake angl e, low shear
required for machining will be
angle and hig h cutting sp eed
(GATE-ME-OS)
(c) Larg e negative rake angl e, hig h shear
(a) 8 minutes (b) 1 2 minutes
angle and low cutting sp eed
(c) 1 6 minutes (d ) 20 minutes
(d ) Z ero rake angl e, hig h shear angle and
hig h cutting sp eed

l1ti1i@jjj4Y@i@blM\iijjj+yderabad J Delhi J Bhopal I Pune I Bhubaneswarl Lucknow J Patna I Bengaluru I Chennai J Vijayawadaj Y� I Tirupati I �1· Kolkata I
. J\,CE . . : 727 : Machining
�-�:Fiwnea"I PuNir,411
� �==================================

1 5 . The figure shows an incomplete schematic 1 7. Match the following (GATE-PI-�9)


of a conventional lathe to be used for cutting List -1 List -2
threads with different pitches. The speed A. Grinding 1 . Compounded oil
gear box Uv is shown and the feed gear box B. Tapping 2. EP additive oil
Us is to be placed. P, Q, R and S denote C. Automatics 3. Chemical fluids
locations and have no other significance. D. Broaching 4. Straight fatty oils
Changes in Uv should NOT affect the pitch
of thread being cut and changes in Us should 1 8. For cutting double start screw threads of
NOT affect the cutting speed. pitch 2.0mm on a lathe, the thread cutting
(GATE-ME-08) tool should have a feed rate of
Spindle (GATE-PI-90)
Motori-+--e---1 (a) 0.5 mm/rev (b) 1 .0 mm/rev
'----' p Q
(c) 2.0 mm/rev (d) 4.0 mm/rev

The correct connections and the correct 1 9. The diameter and rotational speed of a job
placement of Us are given below are 1 00mm and 500 rpm respectively. The
(a) Q & E are connected. Us 1s place high spots (chatter marks) are found at a
between P and Q spacing of 30 deg on the job surface. The
(b) S & E are connected. Us 1s place chatter frequency is (GATE-PI-90)
between R and S (a) 5 Hz (b) 1 2 Hz
(c) Q & E are connected. Us is place (c) 100 Hz (d) 500 Hz
between Q and E
20. A milling cutter having 8 teeth is rotating at
(d) S & E are connected. Us 1s place
1 00 rpm. The work piece feed is set at
between S and E
40mm/min. the feed per tooth is
(GATE-PI-90)
1 6. If the number of double strokes per minute
(a) 5mm (b) 0.05mm
in a shaper is calculated by (0.643 x Cutting (c) 0.4mm (d) 0.2mm
speed in mm/min) / length of the stroke in
m, then the return speed is faster than the 21. The base of a brass bracket has to be rough
cutting speed by (GATE-PI-89) ground to remove the unevenness. The 4
wheels available m the store. The
(a) 40% (b) 60%
appropriate wheel is (GATE-PI-90)
(c) 80% (d) 1 00%
\( 'l l 11g11u t r111g P11hlil .it1011" FyderabadlDelhilBhopal!PunelBhubaneswarl wcknowlPatnalBengaiurulChennailVijayawadalVizag IT=pati I Kukalpallyl Kolkata I
€•�J f� : 728 : Pro duction
� �============::::;::::====================
(a) A 30 K 12 V (b) C 9 0 M 4B strokes/min, the planning tim e for a singl e
(c) C 30 Q 12 V (d ) C 50 G 8V pass will be (GATE-PI-93)
(a) 50 min (b) 100 min
22. A drilling machine has to be d esigned with (c) 200 min (d ) 220min.
8 spindle speeds ranging approximately
betw een 120 to 1200 rpm . T he 5 h spind l e
t 27. A 31.8MM HSS drill is used to drill a hol e
speed is (GATE-PI-91) in cast iron blo ck 100 mm thick at cutting
(a) 445 (b) 480 (c) 620 (d ) 865 sp eed of n20m/min and fe ed 0. 3mm/rev. if
the over travel of drill is 4mm and approach
23. Find the sp eed rang e ratio for the drilling 9mm, the tim e required to drill the hol e is
machine spind l e if the minimum and (GATE-PI-02)
maximum diam eters of drills used are 5 mm (a) l min 40s (b) l min 44s
and 25 mm resp ectively and if the (c) l min 49s (d ) l min 53s
machinability indices for the w ork materials
28. A si d e and face cutter 125mm diam eter has
are 120 ( brass) and 40 (alloy steel)
10 teeth. It op erates at a cutting sp eed of
(GATE-PI-92)
(a) 3/5 (b) 5/3 (c) 5 (d ) 15 14m/min with a tabl e traverse 1OOmm/min.
the fe ed p er tooth of the cutter is
24. If the ind ex crank of a divi ding head 1s (GATE-PI- 02)
turned throug h one compl ete revolution and (a) 10mm (b) 2. 86mm
10 hol es in a 30 hol e circle plate, the w ork (c) 0. 286mm (d ) 0. 8mm
pi ece turns throug h (in d egrees)
29. Match GROUP-I with GROUP-II
(GATE-PI-92)
Group I Group II (GA TE-Pl-02)
(a) 6 (b) 12 (c) 240 (d ) 480
P: Dressing 1. Blunting of grinding w heels
25. Assertion (A): Grinding needs hig her Q: Loading 2. Shaping of grinding w heels
sp ecific cutting energy than milling R: Glazing 3. Sharp ening of grinding w heels
Reason (R): Milling cutter material is S: Truing 4. Clogging of grinding w heels
hard er than abrasive grains. (GATE-PI-93) (d)
(a) (b) (c)
P-2 P-3 P-3 P- 4
26. A w ork-piece of 2000 mm l ength and 300
Q-1 Q-1 Q-4 Q- 3
mm wi dth was machined by a planning
R-4 R-4 R-1 R-1
op eration with the fe ed set at 0.3 mm/stroke.
S-3 S-2 S-2 S-2
If the machine tool executes 10 d ou bl e
\( I l .11J...:,111t. t. 1 1111!. P11hh, .111011 ... Fyderabad I Delhi I Bhopal I Pune I Bhubaneswari Lucknow I Patna I Bengalwu I Chennai ! Vijayawada I V17.a3 I Tirupati I Kukatpally I Kolkata I
: 729 : Machining

30. Match list I (m a chine tool) with list II 33. A job of the shap e shown in the fi gure
(m a chine tool part) (GATE-PI-03) below is to be m achined on a l athe.
List -I List - II T he tool best suited for m achining of this
A. Lathe 1 . Lead screw j ob must have (GATE-PI-04)
B. Milling m achine 2. Rock er arm
C. S hap er 3. Universal ind exing
D. Drilling m achine 4. Flute
A B C D A B C D
(a) 4 2 3 1 (b) 1 3 2 4
(C) 4 3 2 1 (d) 1 2 3 4 (a) p ositive sid e rake angle
(b) p ositive back rake angle
31 . The purp ose of helical grooves in a twist (c) p ositive cutting ed g e inclination angle
drill is to (GATE-PI-03) (d) neg ative cutting ed g e inclination angle
1 . Improve the stiffness
2. Save a tool m aterial 34. A non stand ard thread of pitch 3.17 5 mm is
3. Provi d e spa ce for chip rem oval
to be cut on a l athe having l ea d screw of
4. Provi d e rake angle for the cutting ed g e
pitch 6 mm. A chang e g ear set provi d ed
S el ect the correct answ er suing the cod es
with the l athe has one g ear each with the
given below
(a) 1 & 2 (b) 2 & 3 (c) 3 & 4 (d) l & 4 following num ber of teeth: 20, 30, 40, 50,
60, 70, 80, 90, 1 00, 1 1 0, 1 20, 1 27 . The
32. Match the m echanism with Machine correct pairs of chang e g ears (alb x c/d) for
Mechanism Machine m achining the given thread are (assuming
(P) Pantograp h (1 ) Autom obil e
that the transmission ratio of the rest of the
(Q) Quick return (2) Lathe
kinem ati c train betw een the l athe spindle
(R) Ackerm an (3) Engraver
and l ea d screw is equal to (GATE-PI-04)
(S) Toggle (4) S hap er
(5) Press 1 27 20 1 27 20
(a) X (b) X
30 80 40 1 20
(GATE-PI-03)
(a) P- 3, Q- 1 ,R-2,S-4 1 27 30 1 27 30
(c) (d) X
(b) P- 3, Q-4,R- l ,S-5 20 1 20 50 80
X

(c) P-2, Q- 4,R-5,S-3


(d) P-2, Q-3,R-4,S-5

lM11ih§jjj4ijjjjQRflnjMU@+:iyderabadl Delhi I Bhopal I PuneJ Bhubancswarl Lucknow I Patna! Bengalwu I Chennai l Vuayawadal Vu.ag I Tirupari I Kukalpallyl Kolkala I
: 73 0: Pr oducti on

35. A <I> 20 mm t hro ugh ho le is to be drilled in a It is pro pos ed to mill a s pur gear of 28 t eet h
30 mm t hick plat e us ing a do uble fl ut ed, us ing s imple index ing met ho d. W hich o ne
120° lip angle drill. T he drill t ip is at a of t he following co mbin at io ns of index plat e
dist ance of 3 mm fro m t he plat e s urface and number of revo lut io ns is co rrect?
when cutt ing is st art ed and an o ver t ravel of (a) Plat e 1 : 1 revo lut io n an d 9 ho les in 18
2 mm is reco mmended as a margin to ens ure ho le circles
drilling t hro ugh t he fu ll t hickness of t he (b) Plat e 2 : 1 revo lut io n and 9 ho les in 21
plat e. If t he dr ill rot at es at 500 rev/min and ho le circles
t he fe ed per toot h is 0.01 mm, t he (c) Plat e 2 : 1 revo lut io n and 9 ho les in 33
machining t ime of t he o perat io n (in s ec) will ho le circles
be (GATE-PI-04) (d) Plat e 1 : 1 revo lut io n and 9 ho les in 15
(a) 4. 5 (b) 7.5 (c) 9 (d) 10.5 ho le circles

36. Co ns ider t he follo wing st at ements: 38. Mat ch L ist - I (C utt ing too ls) wit h L ist -II
(GATE-PI-04) (Feat ures) and s elect t he co rrect ans wer
T he helical fl ut e in a twist drill pro vides t he us ing t he co des given belo w t he lists:
necess ary (GATE-PI- 05)
1. C learance angle for the cutt ing edge. List -I List -II
2. Rake angle for t he cutt ing edge. (Cutting tools) (Features)
3. Space for t he chip to co me o ut during A. T urn ing too l 1. C his el edge
drilling. B . Reamer 2. Flut es
4. G uidance for t he drill to ent er into t he C. Milling cutt er 3. Ax ial relief
wo rk piece. 4. Side relief
W hich of t he st at ements given abo ve are (a) A- 1, B- 2, C- 3 (b) A- 4, B- 3, C- 2
cor rect? (c) A- 4, B- 2, C- 3 (d) A- 1, B- 3, C- 2
(a) 1 and 2 (b) 2 and 3
(c) 3 and 4 (d) 1 and 4 39. Mat ch L ist -I (Milling pro blem) with L ist ­
II (Pro bable caus es) and s elect t he cor rect
37 . On e brand of milling machin e has th e answ er using th e cod es giv en b elow th e
follo wing two index plat es s upplied alo ng lists: (GATE-PI-05)
wit h t he index ing head : (GATE-PI-05) List - I (Milling problem)
Plate 1 : 15, 16, 17, 18, 19, 20 ho le circles. A. C hatt er
Plate 2 : 21, 23, 27, 29, 31, 33 ho le circles . B . Poor s urfac e fin is h
\( l l 11i..;11u t J 111� Plil1l1l ,ll1011.., Fyderabad J Delhi JBhopa) J PuncJBhubaneswarJ LucknowJPatna J Bcngaluru J Chennai J Vvayawada J Vizag JT11Upari I KnkatpallyJ Kolkata I
"...(:' ......" . A. CE . .
'!i,. , :�Pulmboos : 731 : Machining

C. Loss of accuracy 42. The quick return mechanism used in shaper


D. Cutter burrs has rocker arm drive of length 200mm. If
List -II (Probable causes) the crank radius is 50mm and the offset
1. Too high feed between crank center and rocker arm pivot
2. Lack of rigidity in machine, fixtures, bar is 20mm, length of the stroke ( in meters ) is
or workpiece (GATE-PI-08)
3. High cutting load (a) 0.5 (b) 1.0 (c) 1.5 (d) 2.0
4. Radial relief too great
5. Not enough lubricant 43. The number of defective produced by a six
sigma process (in Parts per million) is
(a) A-2' B-1 ' C-5 ' D-3
(GATE-PI-1 1)
(b) A-2' B-1 ' C-3 ' D-5
(a) 5.2 (b) 4.2 (c) 3.2 (d) 2.2
(c) A-4 ' B-5 ' C-2 ' D-3
(d) A-4 ' B-2 ' C-3 ' D-5
44. Match the following materials with their
most appropriate application:
Statement for Linked Answer Q40 & Q41
(GATE-PI-1 1)
Blind hole 10mm diameter, 50mm deep are being
Material Application
drilled in steel block, drilling spindle speed is
1. Low carbon steel P. Machine tool base.
600rpm, feed 0.2mm/rev, point angle of drill is
2. Stainless steel Q. Aircraft parts
120.
3. Gray cast iron R. Kitchen utensils
40. Machining time (in min) per hole will be 4. Titanium alloys S. Car body panels
(GATE-PI- 07) (a) 1-P, 2-R, 3-Q, 4-S
(a) 0.08 (b) 0.31 (b) 1 -P, 2-R, 3-S, 4-Q
(c) 0.44 (d) 0.86 (c) 1-S, 2-Q, 3-P, 4-R
(d) 1-S, 2-R, 3-P, 4-Q
41. During the above operation, the drill wears
out after producing 200 holes. Taylor's tool Statement for Linked Answer Q45 & Q 46
life equation is of the form VT03 = C, where During orthogonal machining of a mild steel
V = cutting speed in m/min and T = tool life specimen with a cutting tool of zero rake angle,
in min. Taylor's constant C will be the following data is obtained:
(GATE-PI- 07) Uncut chip thickness = 0.25mm
(a) 15 (b) 72 (c) 93 (d) 490 Chip thickness = D.75 mm

l11•1ib§hi44iib401ei@mm� yderabad I Delhi I Bhopal I Pune I Bhubaneswar I Lucknow I Patna I Bengalwu I Chennai I Vuayawada I Vi?.ag I Tirupati I Kukatpa]ly I Kolkata
: 732 : Production

Wi dth of cut = 2 .5mm Common Data for Question 49 & 50


Norm al force = 950N A disc of 200 mm outer and 80 mm mner
Thrust force = 4 75N diam eter is faced at a fe ed of 0. 1 mm/rev with a
d epth of cut of 1 mm. The facing op eration is
4 5. The shear angle and shear force,
und ertaken at a constant cutting sp eed of 90
resp ectively, are (GATE-Pl-11)
rn/min in a CNC lathe. The m ain (tang ential)
(a) 71 . 565° , 1 50.1 2N
cutting force is 200N
(b) 9.2 1 8° , 861 .64N
(c) 1 8.435° , 751 . 04 N
4 9. Assuming approach and over-travel of the
(d ) 2 3.1 57° , 686.66N
cutting tool to b e z ero, the m achining tim e

4 6. The ultim ate shear stress (in N/mm 2 ) of the m m m 1s (GATE-PI-13)


w ork m aterial is (GATE-PI-11) (a) 2 .93 (b) 5.86
(a) 2 35 (b) 1 39 (c) 564 (d ) 380 (c) 6.66 (d ) 1 3.33

4 7. In a singl e p ass drilling op eration, a throug h 50. Negl ecting the contribution of the fe ed force
hol e of 1 5 mm di am eter is to b e drill ed in a tow ards cutting p ow er, the sp ecific cutting
steel plate of 50mm thickness. Drill spindle energy in J/mm 3 is (PI-GATE-13)
speed is 500rpm , fe ed is 0.2mm/rev and drill (a) 0.2 (b) 2
p oint angle is 1 1 8 . °
Assuming 2mm (c) 200 (d ) 2000
cl earance at approa ch and exit, the total drill
tim e (in seconds) is 51 . A hole of 20 mm diam eter is to b e drilled in
(GATE-ME&PI-12) a steel block of 40 mm thi ckness. The
(a) 35.1 (b) 32 .4 (c) 31 .2 (d ) 30.1
drilling is perform ed at rotational sp eed of
400 rpm and fe ed of 0.1 mm/rev. The
48. In a shaping process, the numb er of d oubl e
required approach and overrun of the drill
strokes p er minute is 30 and the quick return
ratio is 0.6. if the length of the stroke is tog ether is equal to the ra dius of drill. The
2 50mm , the averag e cutting velocity in drilling tim e (in minute) is
rn/min is (GATE-PI-12) (GATE-ME- 14-SET-2)
(a) 3.0 (b) 4. 5 (c) 7.5 (d ) 1 2 . 0 (a) 1 .00 (b) 1 .2 5 (c) 1 .50 (d ) 1 . 75

!Mllih§jjjgq.jjjQRflniffljnjjj� yderabad I Delhi I Bhopal I Pune I Bhubaneswar I Lucknow I Patna I Bengalwu I Chennai I Vtjayawada i Vizag I Tirupati I Kukatpally I Kolkata
: 733 : Machining

52. An HSS drill of 20mm diam eter with 5mm 55. In a slab milling op eration, a cutter of 75
cone height is used to drill a through hole in mm diam eter with sufficient wid th is used to
a steel w ork-pi ece of 50mm thi ckness. rem ove 5 mm thi ck m aterial from a 200 mm
Cutting sp eed of 1 Om/min and fe ed rate of long part in a single pass. The minimum
0.3mm/rev are used . The drilling tim e, in length of travel, in mm , for the cutter to
seconds, neglecting the approach and over engage and completely cut the part surface
travel, is ----- (GATE-PI-14) lS ----- (GATE - Pl -15)

53. A shaft of length 90 mm has a tap ered 56. A block of length 200 mm is m achined by a
portion of length 55 mm. The diam eter of slab milling cu tter 34 mm in diam eter. The
the tap er is 80 mm a t one end and 65 mm at d ep th of cut and table fe ed are set at 2 mm
the other. If the tap er is m a d e by tailstock and 18 mm/minute, resp ectively.
set over m etho d , the tap er angle and the set C onsi d ering the approach and the over travel
over resp ectively are. (GATE -15 -Set 3) of the cutter to be sam e, the minimum
(a) 1 5° 32' and 1 2.1 6 mm estim a ted m achining tim e p er pass is __
minutes. (GATE -17 - SET - l)
(b) 1 8° 32' and 1 5. 66 mm

(c) 1 1 ° 22' and 1 0.26 mm 57. A schem a ti c diagram of p eripheral milling


(d ) 1 0° 32' and 1 4.46 mm is shown in the Figure.

54. In drilling op eration, a twist drill of 30 mm 1------------,--- -


I t
, - - - - - - - - - - - - - - - - - - - - - �- - - -

P\- - - -d/- -2-
I

W ork pi ece i
I I

:
diam eter with p oint angle of 1 1 8 d egrees is
used . If the CNC comm and is used to ---+ Table fe ed :
execu te the drilling op era tion is G90 GO 1 -----------
I la :

Z?? F 20. The d a tum is d efined on the top


If t is the d ep th of cut and d is the di am eter
surface of the w ork m aterial and the
of the milling cutter, then the length of
approach distance is 3 mm . then, to achi eve
approach (/a ) is expressed as
a cylindri cal hole d ep th of 40 mm , the Z
(GATE -PI -17)
coordinate to be provi d ed in the CNC
(a) �d(t - d) (b) �d(d -t)
command, in mm, is ----
(GATE - PI -15) (c) �t(d -t) (d ) �t(t -d)

ll!llii@iii§§mi4Affnffiiiih� yderabad I Delhi I Bhopal I Pune I Bhubaneswar I Lucknow I Patna I Bengalwu I Chennai I Vijayawada I Vizag I Tirupati I Kukatpally I Kolkata
"�.,. .
.... ',.,,�tF'qme,ermg
..� . ACE . .
Pnbood¥>DI : 7 34 : Production

04. The cross fe ed on a shap er consists of a lea d


Five Marks Questions screw having 0.2 thread/mm. A ratchet and
pawl on the end of the lea d screw is driven
01. A. six sp eed g ear box has to be d esigned for from the shap er crank such that pawl indices
the hea d stock of center lathe for turning the ratchet by 1 too th during each return
mild steel and cast iron rods of diameter stroke of the ramp. Ratchet has 20 teeth.
ranging from 1 00 mm to 200mm by carbi d e Find the fe ed in mm and if a plate 100mm
tools. If the maximum and minimum cutting wi d e has to be machine in 10 seconds find
sp eeds p ermissible as 120m/min and the cu tting velocity in m/min. the ratio of
40m/min resp ectively, w hat should be the return to cutting sp eed is 2: 1 and the leng th
valu es of those 6 spindle sp eed? of the strok e is 150mm.
(GATE-ME-87) (GATE-ME-92)

02 . In slab milling op eration the following d a ta 05. Cylindrical bars of 100 mm diameter and
w as observed 576 mm leng th are turned in a single p ass
Diameter of cutter : 90mm, op eration. The spindle sp eed used is 1 44
No. of teeth on cutter : 1 0, rpm and the total sp eed is 0.2 mm/rev.,
Cutting sp eed = 30m/min tailor tool life equa ti on is VT0· 75 = 75, w here
Table fe ed : 1 80mm/min , V = cutting sp eed in m/min and T = tool life
D ep th of cut = 3mm in min. calculate (GATE-ME-93)
C alculate the maximum and averag e chip i) The time for turning one pi ece is ..... .
thickness in milling (GATE-ME-89) ii) The averag e to tal chang e time p er
pi ece given that i t takes 3 minutes to
03. Estima te the shortest machining time chang e the tool each time is. ........... .
required in a shap er to machine a plate of iii) The time required to produce one
2 00 x 90mm und er the following conditions pi ece, given that the handling time is
(GATE-ME-90) 4min.
Cu tting sp eed = 13.3m/min,
Feed = 0 .57mm/double stroke 06. W hen milling a slot 20mm wide 1 0cm long
Number of p asses one Approach =
= in a rectangular plate 1 0cm x 20cm the
overrun = longi tu dinal = 2 0mm & lateral = cutting conditions used w ere
4 mm Cutting sp eed = 60 m/min,
Ratio of cutting sp eed to rapi d return= 0.83 Diameter of the end mill = 20 mm

!lflli!i4jjjqg.jjijjmCfti\l@� yderabad I Delhi I Bhopal I Pune I Bhubaneswar I Lucknow ! Patna I Bengaluru I Chennai I Vijayawada j Vmg I Tirupati I Kukatpally I Kolkata
: 735 : Machining

Number of flutes = 8, Calculate Number of holes produced using


Feed = 0 .01 mm/flute one drill and The Average production time
Depth of cut = 3 mm , per hole. (GATE-ME-00)
Find the cutting time for this operation.
(GATE-ME-94) 10. A drilling machine is provided with
minimum spindle speed; 34 rpm, maximum
07. A drilling machine is t o be designed to have spindle speed 353 rpm, number speeds is 6,
6 spindle speeds ranging from about 110 calculate the 3rd and 4 h spindle speeds that
t

rpm to about 650rpm. Assuming a proper are likely to be available on the machine.
series for the layout of the speeds, determine (GATE-PI-89)
the values of all those 6 spindle speeds.
Modify the computed values so as <ender 11. A surface 80 x 160mm is rough machined
them acceptable as standard using 150mm diameter face milling cutter
(GATE-ME-95) having 10 teeth . The cutter center is offset
by 15mm from the line of symmetry of the
08 . Fifty flat pieces I mm width and initial surface. Estimate the time to rough machine
dimensions 205x30x1mm are to be milled the surface, if a feed per tooth of 0 .25mm
m a single cut to the final dimensions and a cutting speed of 20m/min are
205x25x1mm using end mill cutter . if the employed. (GATE-PI-89)
cutter of diameter 25mm has 10 teeth and
rotates at 100 rpm, find the maximum uncut 12. A hydraulic shaping machine is set for 60
chip thickness if the horizontal feed of the double strokes per minute, while machining
table is 1Omm/min, assuming one teeth in a job of 90mm length. The ratio of the
contact and the material removal rate. cutting and idle speeds 1 :2 . Assume a 5mm
(GATE-ME-96) approach and 5mm overrun. If the lateral
feed per stroke is 0 .6mm and the width of
09. A 15 mm diameter HSS drill is used at a job is 36mm, calculate the machining time.
cutting speed of 20 m/min and a feed rate of (GATE-PI-90)
0 .2 mm/rev. Under these conditions, the
13. A gear having 29 teeth is to be machined on
drill life is 100 min. The drilling length of
a horizontal milling machine using a
each hole is 45 mm and the time taken for
idle motions is 20 s. The tool change time is
standard dividing head with worm gear ratio
40. $design the indexing movement, that is
300 sec.
crank revolution, hole circle selection and
!1J11ijj§Oii4.jjdRfljjj@jjji11.yderabad I Delhi I Bhopal I Pune I Bhubaneswar J wcknowJ Patna I Benga]uru I Chennai I Vgayawada I Vmg I T11Upati I Kukatpal)y I Kolkata I
: 736 : Production

sector p osition. Whol e plates w i th numb er (GATE-PI-92)


of holes along the circl e are (GATE-PI-91) (a) by introducing odd num ber of idle g ears
Plate 1: 24, 25, 28, 3 0, 3 4, 3 7, 3 8, 3 9, 41 , 42, 43 (b) by introducing even num ber of idle g ears
Plate 2: 46, 47, 49, 51 , 53, 54, 57, 58, 59, 62, 66 (c) any num ber of idl e g ears
(d) no idl e g ears
1 4. Diam eter of a MS bar has to be reduced
from 80 mm to 60 mm over a l eng th of 1 1 6 1 7. A rectangular block 200 x 80 x 60 mm is to

mm a t a fe ed rate of 0.2 mm/rev, dep th of be face mill ed on a vertical-milling


cut 2.5 mm and averag e cutting velocity of m achine. A face milling tool of 1 00 mm
1 3 2 m/min. Assum e approach and over diam eter, having 1 2 teeth of carbide inserts
travel distances as 2.5 mm and 1 .5 mm is used for m a chining the 200 x 80 mm face.
resp ectively, tim es for return and for Sugg ested cutting sp eed is 50 m/min and
resetting of tool in betw een the p asses as 1 0 fe ed p er tooth 0.1 mm . (GATE-PI-93)
seconds and 3 0 sec resp ectively. Spindl e (i) T he m achining tim e for one p ass
speed and total m achining tim e is rem oving a layer of 2 mm thickness
(GATE-PI-91) from the surface, with an approach of 5
mm and an over run of 5 mm w i th
1 5. A 20mm diam eter drill w i th p oint angle 1 20
Symm etrical milling is
d egrees is used to drill a throu g h hole in a
(ii) The m a chining tim e for one p ass
plate 3 0mm thi ck, assum e the leng th of
rem oving a layer of 2 mm thickness
approach and over travel as 2 mm and 3 mm
from the surface, with an approach of 5
resp ectively. T he tim e required for drilling
mm and an over run of 5 mm w i th
if the fe ed is 0.1 mm/rev and the m aterial is
asymm etrical milling having offset 5mm
cut with a velo city of 20 m/min ..... ...... .
(GATE-PI-92)
lS

1 6. D ouble start rig ht hand m etric threads w i th 1 8. C alculate the tim e required for compl eting a
0.5 mm pitch are m achined in a g eneral 2mm deep finishing cut on a 1 50mm wide,
purpose lathe having a lead screw of 6 mm 400mm long face of a 1 00mm thick steel
lead. Gears, one number each of 20 teeth in block using a face milling cutter of 250mm
steps of 5 teeth are available. A compound diam eter w i th 8 teeth. T he cutting sp eed 1.2
g ear train for transmi tting the required m/sec and fe et p er tooth is 0.1 mm
m o tion from the spindle to l ead screw is (GATE-PI-94)
used .the hand of the thread can be change
!M11ii@jjji4jjjjiRflbiM\hih� yderabad I Delhi I Bhopal I Pune I Bhubaneswar I Lucknow I Patna I Bengaluru I Chennai I Vijayawada I Vizag I Tirupati I Kukatpally I Kolkala
;.•�.;�Pnbtic:ationa : 737 : Machining
A .CE
� �==================================
19. A planer has a m aximum strok e l ength of 2 21. The following d ata refers to slab milling
m eters. Return stroke occurs at twice the op eration (GATE-PI-02)
sp eed of the forw ard stroke. Six rectangular Diam eter of the cutter : 5 0mm
blokes of 900mm x 300mm are to be Num ber of teeth on cutter: 1 2
planned in one p ass with three tools equi­ Cutter spindle sp eed : 300rpm
spaced arrang ed . On the cross sli d e as D epth of cut : 2mm
shown in fig. (The triangl es in the fig. Length ofj ob : 5 00mm
represents initial position of the tools) Longitu dinal tabl e fe ed : 200mm/min
cutting sp eed is lm/sec and fe ed is i) The fe ed p er tooth during the milling
Imm/stroke. Over travel on either si d e op eration is
l ength wise is 5 0mm and wi dth wise is 5mm (a) 0. 05 6 mm (b) 0. 1 67 mm
on either si d e. The m achining tim e (in sec) (c) 8. 662 ( d ) 1 7.333
p er pi ece is. . . (GATE-PI-95)
ii) If the cutter over travel is 2 mm , the
m achining tim e for the singl e p ass

1°I
50 900 1 00 900 50
1-1----1-- 1----i-1
1
I�
I I IL 3
milling op eration will be
(a) 0.67 min (b) 1 .5 0 min
l"-J ( c) 1 .67 min ( d ) 2.5 6 min
12 1 C=:J i
�I 1�rl:'. Common Data for Question 22 & 23
D ata for plain milling operation are given below
Length of w ork pi ece = 200mm,
(a) 930 (b) 465 (c) 1 55 ( d ) none Cutter diam eter = 1 0mm,
No .of teeth = 4
20. The cutting length of the broa ch for the key Cutter sp eed = l OOrpm,
w ay cutting in a 1 5 0mm long cast iron g ear F eed = 200mm/min
hub. Given that wi dth of the key w ay is D epth of cut = 2mm,
9mm d epth of key w ay is 4.5mm no. of Total cl earance ( entry and exit) = 5mm
finishing teeth = 4 and no. of semi finishing
teeth = 8. (GATE-PI-95) 22. M ean un- d eform ed chip thi ckness (in
Rise per tooth Pitch mi crons) is
Rou g hing teeth O . 1 mm 22 mm (a) 142 (b) 1 00 ( c) 71 (d) 50
Semi finishing teeth 0. 0125 mm 20 mm
Finishing teeth 20 mm 23. Machining tim e for one singl e pass (in sec)
(a) 968 mm (b) 1 60 mm lS

(c) 1 208 mm ( d ) none (a) 60 (b) 66 ( c) 126 (d) 15

Jl,Wl$j@jjjqg;jjjjQfleiffli11jjj� yderabad I Delhi I Bhopal I Pune I Bhubaneswar I Lucknow ! Patna I Bengalwu I Chennai jV,jayawada I Vizag I Tirupati I Kukatpally I Kolkata
SOLUTIONS
06. Ans: (d)
One Mark Solutions Sol: Honing and lapping are same but honing is
used for producing intern al surfaces and
01. Ans: (c) lapping is used for ex tern al surfaces only.
Sol: G rinding wheel has been specified as A­
aluminum ox ide , 46- grain siz e or grit siz e, 07. Ans: (c)
M - grade of G W, 6 - stru cture , V- vitrified Sol: I n centerless grinding operation the ax is of
bonding material and 21- suffix used as regulating or control wheel is tilted by an
secret code of manufa cturer. angle about 1 to 6 deg, this tilt causes the
velocity of regulating wheel will have two
02. components of velocities, one will be used
Sol: S mall point angle and large helix angle. for rotating thej ob and the other will be used
for fe eding the work piece.
03. Ans: (a)
Sol: I n grinding because of very small siz e chips 08. Ans: (c)
the metal removal is low hence time taken is Sol: Vertical mill ing machine with end and side
high . B y. increasing the cutting velocity the milling cutter can be used for producing T­
machining time can be reduced. slots.

04. Ans: (a) 09. Ans: (d)


Sol: I n gear hobbing because of continuous Sol: B ecause 740C is greater than 723C, austinite
index ing, the errors during machining will will still present and some austenite will
come down convert into cementite.

05. Ans: (b) 10. Ans: D own , down


Sol: For making intern al gear teeth the best
method is gear shaping with pinion cutter 11. Ans: S mall fe ed rate and high speed
and nex t best is gear broaching

!IHl•i@jjj4ijjjjiRftbiM\!IMijj{yderabadl Dellii I Bhopal I Pune I Bhubaneswar l l.J.lcknowl Patna ! Bengaluru j Oiennai I Vuayawada l V17.3g l 'firupati I Kukatpal)y l Kolkata I
ACE
�...
.., ......
..
EF.ngioecrmgPnbtiariona : 739 : Machining

12. Ans: (d ) 1 8. Ans: (a)


Sol: In grinding ou t of the abrasive parti cles Sol: Because the copp er is having very hig h
present in the surface of grinding w heel only electrical conductivi ty
about I/3 of abrasives are act in single p oint
rd

cutting tools and rem aining are simply 19. Ans: (b)
rubbing on to the w ork pi ece without d oing Sol: Allow ance 1s the difference betw een the
any m a chining, hence loss of energy due to m aximum m aterial limi ts of shaft and hole,
friction is hig h and so the sp ecific cutting w hich is equal to minimum clearance or
energy is hig h m aximum interference.

1 3. Ans: (c) 20. Ans: (c)


Sol:Because the mild steel is soft m aterial the Sol: G enerally square threads are preferable for
typ e of chips produced is continuous chips. m anufacture of lead screws because i t has
hig hest efficiency of p ow er transmission.
1 4. Ans: True The next preferable is bu ttress threads and
Sol:Because if diam ond w heels are used for the nex t is acm e threads.
m achining of steel or ferrous components,
the diffusion w ear is hig h and tool becom es 21 . Ans: (b)
blunt very fast.
22. Ans: (b)
1 5. Ans: (a) Sol: The m ethod of producing a larg e hole
Sol: In twist drill the helix angle increases from without drilling is called Trep anning, for this
center to outw ards and the rake angle a hallow drill bi t or eccentric single p oint
d ep ends on the helix angle cu tting tool can be used and this m ethod is
used for producing only through holes but
1 6. Ans: (b) not for blind holes.
Sol: (stand ard valu e)
23. Ans: (b & c)
17. Ans: (c) Sol: In g eneral quali ty screw threads are produced
Sol: Because the aluminum oxi d e is softer and by thread rolling, nex t is by thread cutting
toug her than the silicon carbi d e, it can be single point cutting tool or threa d chasing
used for m achining soft and tou g h m a terials.

!1J•li@jjj§4iii/JRftn@j,/jjj� yderabad I Delhi I Bhopal I Pune I Bhubaneswar I Lucknow I Patna I Bengalwu I Chennai I Vijayawada j Vu.ag I Tirupati I Kukatpa!Jy I Kolkata
: 740 : Production

24. Ans: (b) 31 . Ans: ( d )


Sol: As the specific heat increases, the am ount of Sol: In grinding out of the abrasive particl es
energy required for rising 1 °C is larg e and present in the surface of grinding w heel only
hence it can carry larg e am ount of heat. about 113rd of abrasives are acting as singl e
p oint cutting tools and rem aining are simply
25. Ans: ( c) rubbing on to the w ork pi ece without d oing
Sol: T he internal g ears can be best produced by any m a chining, hence loss of energy due to
using g ear shaping with pinion cutter, the friction is hig h and so the sp ecific cutting
next best m ethod is by g ear broaching. energy is hig h

26. Ans: (b) 32. Ans: ( c)


Sol:Burnishing is the op eration of compressing Sol: A refers to prefix, A refers to aluminum
the m aterial to convert jig-j ag ed g es into oxi d e, 48 refers to grain siz e, L refers
straight and sm ooth ed g es hence no abrasives hardness of grinding w heel, 7 refers to
are required. structure , V refers to vitrifi ed and 25 m eans
suffix.
27. Ans: (b)
Sol: Surface roug hness value, m aximum p eak to 33. Ans: (a)
valley heig ht, Rt = f/8.r , m eans that by
reducing the valu e of Rt can be reduced so 34. Ans: ( d )
that surface finish will be better. Sol: In shap er the m aximum velocity of ram
occurs at the mi d dl e of the return stroke, next
28. � ns: (a) m aximum occurs at end of return stroke
Sol: Drill bits and ream ers are provi d ed with even l east occurs at beginning of forw ard stroke.
num ber of flutes for balancing of cutting
forces. 35. Ans: (a)
Sol: This m ainly o ccurs during grinding of hard
29. Ans: (a)
w ork pi eces using hard grinding w heels.
30. Ans: (b) During this tim e due to hig h hardness of the
Sol: In g ear shaping m achine with pinion cutter it w ork, the abrasives becom es blunt very fast
is p ossibl e to cut both external and internal and started rubbing on to the w ork but
threa ds. because of hig h hardness of w heel the

!l!lli@hi44iiii4Rflb1Miiiiii� yderabad I Delhi I Bhopal I Pune I Bhubaneswar I Lucknow! Patna I Bengaluru I Chennai I Vtjayawada I Vmg rnrupati I Kukatpally I Kolkata
ACE 741

��-��!��� : :
... ���================================
� .......--
..........:........: Pnhlicatioos Machining

rubbing forces are not sufficient to pull out 43. Ans: (c)
the blunt abrasives and hence blunt abrasive Sol:Because of the intermittent fe ed used in
will simply rubbing without any m achining shaping op eration, the m echanism used for
called as glazing of grinding w heel. g etting fe ed in the shap er is Ratchet and p awl
m echanism .
36. Ans: True

37. Ans: (b) 44. Ans: (a)


Sol: This is because the gray cast iron has hig h Sol:Buffing op eration will give excell ent surface
rigi dity and also has hi g h d amping qualiti es
finish and good straig htness and fl atness.
and easily castabl e.

38. Ans: (a) 45. Ans: (b)

Sol: Silicon carbi d e 1s the naturally availabl e Sol: Reaming 1s m ainly used for s1z mg and
abrasive m ateri al. finishing the hole.

39. Ans: (a) 46. Ans: (a)


Sol: The curvature given is the concave curvature Sol: G ear hobbing is the only g ear m anufacturing
hence it increases the stress concentration process with g eneration principl e.
factor therefore it is used for supply of
lubricating oil to bearing m ounting
47. Ans: (d )

40. Ans: (c)


48. Ans: (b)
Sol:Because the typ e of profil e is involute and
Sol: M etal oxi d es are p orous in nature and like a
the cutter is the form cutter, the shap e of the
spongy.
cutter is involute.

41. Ans: (c) 49. Ans: (d )


Sol: C obalt is used as binding m aterial during
42. Ans: (c)
m anufacturing of carbi d e cutting tools.
Sol: In grinding, dressing is nothing but re­
sharp ening of grinding w heel, the tim e
betw een the tw o successive re-sharpanings is 50. Ans: (b)
called tool life

li1•1i@jjj44ijj/4RftftlM\ii.jj.� yderabad I Delhi I Bhopal I Pune I Bhubaneswar I Lucknow I Patna I Bengaluru I Chennai I Vuayawada I Vu.ag I Tirupati I Kukatpally I Kolkata
: 742 : Production

51 . Ans: (d ) gear m anufacturing by milling operation the


Sol: D = 200 m m indexing will be given as intermittent
f = 0.25 mm/ indexing w hereas during m anufacturing of
d = 4 mm gears using gear hobbing process differential
N = 160 rpm indexing will be given
1tDN 1t x 200 x 160
V= =
60 60 55. Ans : (c)
= 1674mm/sec Sol: Reaming is similar to drilling operation used
MRR = fd V = 0.25x4x1 674 for rem oving a sm all am ount of m aterial and
= 1 674 mm 3/sec finishing an existing hole.
= 1 675 mm /se c 3

56. Ans: (c)


52 . Ans: (c)
Sol: Knurling is one of the operations performed 57. Ans: (d)
on the lathe m achine. Milling is use d for Sol: Out of all the conventional m achining
carrying out slotting calle d as sl ot milling or methods grinding is the one w hich is carrie d
slab milling . Grinding w heel will be dressed out at hig hest cutting velocity.
to re-sharpen the w heel w hen it has g ot
glaze d . In drilling m achine by m ounting 58. Ans: 7.53 (range 7.4 to 7.6)
single point cutting tool in the drill spindle, Sol: N = rpm ofj ob
the counter boring will be d one 1000V 1 000 x 300
= = 477_ 7 rpm
1tD n x 200
53. Ans: 0.7 to 0.72 . =-L
T.1me .c:
1or m ach'mmg
Sol: Given that, d 2 = 2d 1 , f.N
1'™ � d, 900
= =� = 0. 707 ----- = 7.53 min
0.25x 477.7
t 2 max fa; 2d ,

54. Ans :(b) 59. Ans: (b)


Sol: In shaping to get different speeds in the Sol: Internal gears are m anufacture d by gear
forward and return strokes, quick return Broaching and shaping with pinion cutter
mechanism is use d . Apron is one of the only, w hereas shaping with pinion cutter is
m oving p arts in the lathe m achine. During use d for both external and internal gears.

lffl•IQl@Oi§§.jj/4fjjjniMJ,@j.� yderabad I Delhi I Bhopal I Pune I Bhubaneswar j Lucknow ! Patna I Bengaluru I Chennai I Vijayawada I Vu.ag I Tuupati I Kukatpally j Kolkata
: 743 : Machining

60. Ans: (800) 05. Ans: (a)


Sol: P1ob =0.8 mm, PL s=1. 5 mm, fm =>
Sol: Feed per t oot h, f1 = fm = f1 .N .Z
N1ob =1500 rp m, NL s=? Nz
Nf N 08 =0. 1 x 150 x 8=120 mm/min
G.R = T.V = = LS = pJob = .
Nd N Job PLS 1. 5
0.8 06. Ans: (c)
N L s =- x1500 =800 r pm
1.5 Sol: B ecause t he HSS is not so hard, Aluminum
ox ide abrasives can be used, for fi nish
61. Ans: (c) g rinding fi ne or very fi ne g rain siz e is
Sol: Turn ing and boring are t he single point req uired but g iven is t he medium g rain siz e,
cutt ing t ool operat ions because boring 1 s medium hard so medium g rade can be used,
not hing but int ern alt urn ing operat ion. 7 is t he t ransit ion st ru ctu re which can be
used and V- vit rifi ed bond is a g eneral
purpose bond.

07. Ans: Rigid


Two Marks Solutions Sol: B ecause it has t o wit hst and for t he forces
act ing during machining so t hat close
dimensionalt olerances can be achieved.
01. Ans: (c)
Sol: Feed in t hread cutt ing =lead of t hreads 08. Ans: A- 3, B- 4, C- 2, D- 1,
=number of st art s x pit ch
I

=2 X 1. 0=2 09.
Sol: Transformat ion rat io=t rain value
02. Ans: (d) = Pit ch of j ob t reads / Pit ch of
f 50 lead screw t hreads
Sol: Feed per t oot h=ft =�
NZ I 0x100 =1.0 mm / (1/4 inch)
=0.05mm lx 4 -
- 20
12
7 7
03. Ans: A- 1, B- 2, C- 3, D- 4 (1! )
. (The ter m 12 7/5 is used for co nverti ng i nches
04. Ans: A- 5, B- 6, C- , 2 D- 3, int o mm)

\( I I lll.?;111< l'lIlll\ P11lilu .11101h yderabadj Delhij Bhopal I Punel Bhubaneswarl Lucknow I Patna I Bengaluru I Chennai I Vliayawadaj V,zag jT,rupati I Kukatpallyj Kolkata
\I •
ACE
�.••.:F,wrw:,qPwv:amm
., • • •
: 744: Production

10. Ans: (c) B


Time/cut = �(1+ M)
Sol: Because the aluminum oxide is softer and f V
tougher than the silicon carbide, it can be 30 640
=- -- (1+ 0.5) = 12 mm
used for machining soft and tough 0.3 8000
materials.
15. Ans: (d)
11. Ans: (d) Sol: With this any change in Uv will also
changes the speed of lead screw, the pitch
12. Ans: (b) of the threads produced depends on the
L 25 speed of work and speed of lead screw. Us
Sol: Time per hole = - = ----
fN 0.25 x 300 will not affect the speed of the work
= 1/3 min = 20sec.
Because diameter of drill bit was not given, 16. Ans: (c)
hence AP1 is zero. 1
Sol: No. of D.S/min = ----
Time / D.S

13. Ans: (b) =


V =-(0.643)
V
L( l + M ) L
Sol: N = 300 rpm
f = 0.2 mm /rev
t = 20 mm From the above
AP1 = 0.5D = 0.5 x 10 = 5 1
--=0.643
l+ M
. L 20+ 5
Time / hole = -=---=25 1
fN 0.2 x 300 l+ M=--=1.55
Note: AP1 also can be calculated as 0.643
V
D Hence M = 1.55 -1 0.55 = �
AP1 = -xcot ij3)
=

2 vr
Ve
V= =1.81
14. Ans: (b) r 0.55
Sol: Given, L = 600 mm, AP = OR = 20 mm, Vr - Ve
% increase = =0.81=81 %
V e = 8 m /min, f= 0.3 mm/stroke Ve

M=_!_ · B = 30 mm
2 ' 17. Ans: A-3, B-4, C-1 , D-2
L = / + 20 + 20 = 600+20+20 = 640

liiili@hiiiihiMRflnj@jjfjj4 yderabad I Delhi I Bhopal I Pune I Bhubaneswari Lucknow! Patna I Bengaluru I Chennai I Vtjayawada I V12.ag I Tirupati I Kukatpally I Kolkata
':, �·,!li'n�p. �m c, hining·
:.t CE . . M..,.,���
... '"= :�
-��·�Pu�Nic:alioos : 7,;:45�: =============�
��==========�;,.
18. Ans: (d) 23. Ans: (d)
Sol: Feed rate in thread cutting
Sol: Speed range ratio =
. D xV

= lead of the threads


max max
Dmin xVmin
= no. of starts x pitch 25x120 =
= 15
= 2 x 2 = 4 mm/rev 5x 40

19. Ans: (c) 24. Ans: (b)


Sol: High spots per rewlution 30 x1 + 10 40 = 40
Sol: C.R = 1 .!_.2. = =
= 360/ 30 =12, 30 30 30 N
No. of high spots per second C.R = crank rotation
= high spots/rev x RPS Hence, N = no. of divisions the
= 12 X 500/60 = 100 circumference ofjob is to be divided = 30
Rotation of work piece each time
20. Ans: (b) =
360 = 0
12
Sol: Feed per tooth = ft 30
f 40
= _!!!_ = --
NZ 100x8 25. Ans: (c)
= 0.05mm I tooth Sol: Abrasives used in grinding are harder than
milling cutters.
21. Ans: (a)
Sol: Brass is the soft material so aluminum oxide 26. Ans: (b)
wheels can be used Sol: No. of D.S/ min = 10
B = 300 min, f = 0.3 mm /stroke
22. Ans: (a) B 1
Time / cut = x
f No.of D.S
Sol: A, Nrnin = 120, N =1200
max
300 1 .
N = 8 speeds - =x- =100mm
10
=
0.3
N
f = n-1 �
Nmin
27. Ans: (d)
Sol: t = 100 mm, D = 31.8 mm,
= v 1200 = 1/10 = 1.389
120 Vc = 20 m/min, f = 0.3,
OR = 4 mm, AP = 9 mm

!IJl1i@jjji4jjjj$QflfilM\j/.jj.yderabad I Delhi I Bhopal I Pune I Bhubaneswarl Luclmow I Patna I Bengaluru I Chennai I Vtjayawada IV17.3g I TlfUpati I Kukatpally I Kolkata I
ACE
��-� �·�ig�Pl�Jb�lli.�cm�ioos�
:'.l!;h�p�· -:l'!nll
.. 4;= �o�d:uc�n:o· �
: ========.;.= :,.7;46�: ============�Pr n

l OOOx 20 = 32. Ans: (b)


N= 200
7tX 31.8

1 1 33. Ans: (a)


AP1 = -D=- x31.8=15.9 Sol: Generally this type of shapes can not be
2 2
produced on the lathe machine but some
Time/hole = �
fN extent it may be produced by positive rake
l OO + 4 + 9 angle tool.
=1.88 = l min 53sec
0-3x 200
=

Note: if we consider AP1 and calculate the 34. Ans: (b)


machining time, then there is no answer in Sol: Train value = G.R
the multiple choices pitch of job threads
=---------
pitch of lead screw threads
28. Ans: (c) 3.175x 40 127
=---- =- � Not posst"ble
f 6x40 240
Sol: Feed per tooth (ft ) =
Nz 127 lx20 127 20 .
___!!!_

= -x-- =-x- �Possible


10 40 6x 20 40 120
= = 0.28 mm/tooth
35.67x10
l OOOx14 = 35. Ans: (a)
N= 35_67
7tX 125 Sol: D = 20 mm ; t = 30 mm
Double fluted
29. Ans: (c) AP = 3 mm
OR = 2 mm,
30. Ans: (b)
N = 500 rpm
Sol: In lathe lead screw is used for converting
Feed /tooth =O.Ol mm
rotary motion into linear motion, in milling
L = 30 + 3 + 2 + 0.5 x20 = 45 mm
machine universal indexing is used for
f = 0.01 X 2= 0.02 mm
producing gears of different teeth, in shaper
.. . L
clapper box with rocker arm is used for Machmmg ttme = -
fN
lifting the tool during the return stroke and
45
in drilling flutes are used to remove chips = =4.5min
0.02x500
produced during machining.
36. Ans: (b)
31. Ans: (c)

!lfl•i!i§Oii4ihi4Rflldihii!+1yderabad I Delhi I Bhopal I Pune I Bhubaneswarl U1Cknowl Patna I Bengaruru I Chennai IVtiayawada I V,ug I T=pati I Kukalpally I Kolkata !
t . A:CE ..
�-:,�•�==========================================
�Fnpw:nogPnbtiratiool : 747: Machining

37. Ans: (b) 42. Ans: (b)


Sol: Crank rotation = 40/N = 40/28 Sol:
= 1(12/28) = 1(3/7) = 1(9/21)

38. Ans: (c)

39. Ans: (b)

40. Ans: (c) A


Stroke length (L) = 2 AP(CB/AC)
41. Ans: (b) = 2 200 (50/20)
Sol: Common solution for 40 & 41
X

=lOOOmm= Im
D = 10 mm, t = 50 mm,N = 600,
= 43. Ans: (c)
Sol: According to six sigma limits of
f 0.2 mm /rev
AP1 = 0.3 D .......for blind hole
manufacturing if the number of defective
= 0.3 x}O = 3 mm
per million parts is less than 3.2 the whole
L = t + AP1 = 50 + 3 = 53
lot is acceptable.
L 53
Time/hole = - =--- = 0.442 min
44. Ans: (d)
fN 0.2x 600
No. of holes produced / tool Sol: Low carbon steels are used for making car
= 200 holes body panels, stainless steel is used for
Tool life = 200 x 0.442 = 88.33 min making kitchen utensils, gray cast iron is
used for making machine tool beds and
Given that VT03 = C titanium alloys are used for making aircraft
1tDN nxlOx 600 parts.
V= = =18_84 min
1000 1000
45. Ans: (c)
:. VT03 = C=> 18.84 X 88.33°3 = C
46. Ans: (d)
Sol: Common solution for 45 & 46
=> C= 72.26 = Tailors tool life constant.

a = O, t 1 =0.25mm, t 2 =0.75mm ,
w = b = 2.5mm, Fe = 950N, FT = 475N

IM•1i!i§jjji4ijj/4RftniM\Ufh.j ydcrabadlDelhilBhopallPunclBhubancswarl UJcknowlPamalBcngalurulChennailVtiayawadajVmg ITtrupati I Kukatpallyl Kolkata


...... ...... ACE Production
�....
, .,..�...,....;.._.
� Pnhticwooa .....
_,,.....
: 748:

0.25 _!_ 48. Ans: (d)


r = .!!_ = = = 0.333
t 2 0.75 3 Sol: Double strokes per min = 30,
Quick return ratio = M = 0.6
Stroke length = 250mm
Average cutting velocity= L.N ( 1+ M)
= 30 x250 (1 + 0.6)= 12.0 m/min

49. Ans: (a)

$ = tan -l (r) = tan -l (0.333) =18.43° 50. Ans: (b)

� = tan-'( :J = tan-' 0.5= 26.56


Sol: (Solution for Q49 & QSO)

D 0 =200mm
}D avg =
200 +80
140
D I =80mm 2
Fs = (c xcos($+�-a)
cos �-a) (because constant cutting speed is given)
950 f = 0.1 mm/rev d = 1 mm
= xcos(45)=751
cos 26.56 V = 90 m/min Fe = 200 N
AP = OR = zero
'tu = !s x sin($)
0 1000 x 90
N= =204
751 7t X 140
xsin(18.43)=380N/mm 2
025 X 2.5 D 0 -D; 200-80
Time/cut=�= 2 2
47. Ans: (a) fN O.lxN O.l x204
Sol: t = 50 mm, D = 15 mm 60
= =2.93
N = 500 rpm, f = 0.2 mm/rev O.lx204
2B = 118°, AP = OR = 2 mm Specific cutting energy
D/2 Fe 200
AP 1 = -- = 4.5mm = = =2000] /mm 3
tan 59 A 0 O.lxl
L=50 + 2 + 2 + 4.5 = 58.5
58·5
Time/hole = � = x 60 51. Ans: (b)
f .N 0.2x500
Sol: D= 20mm, t = 40mm, N = 400 rpm,
= 35.1 sec.
f=O.lmm/rev
!M11ih§jj/i4@$0inj@11jjj� yderabad I Delhi I Bhopal I Pune I Bhubaneswar I Lucknow! Patna I Bengaluru I Chennai I Vtjayawada I Vmg I Tirupati I Kuk.atpally I Kolkata

ACE : 749:
�-�
�� .,,v�'Ii'....:.....'
� PuNicwioos ................ Machining

AP + OR = Approach + Overrun 80-65


tan8 = --
= D/2 = 20/2 = 10mm 2x55
= AP 1 = compulsory approach 8 = 7.765°
Note: Because the AP+OR = D/2, it is not Included angle = 15° 321 = 28
necessary to take the AP 1 separately. Offset = LtanS = 90 tan7.765 = 12.27 mm

L = length tool travel = t + AP 1


= 40 + 10= 50mm 54. Ans: (- 50 to - 48)
50 L Sol: D= 30mm, 2B = 118° .
Dn·irmg time
. = = .
=1 .25 mm
(fN) (0.1x 400) Compulsory Approach = AP 1 = (D/2) cot B
= 15 x cot 59 = 9.013mm
52. Ans : 69 Total depth of tool travel = L= AP 1 + t
= - (9.013+ 40)= - 49.013mm
. 1000V
Sol: Speed of dnll (N)= -- =159.2rpm
1tD
Distance traveled by drill (L)= 50 + 5 55. Ans: 218 to 219
= 55mm Sol: D = 75mm, d = 5mm, /= 200mm ,
Feed (f) = 0.3mm/rev Compulsory approach = �d(D -d)

Time per hole = � =1.151min = 69sec =�5(75-5) = 18.708 mm


Minimum length of travel
fN
= compulsory approach + Length of the part
53. Ans: (a)
= 18.708 + 200 = 218.708
Sol: Total length = 90 mm
Taper portion length= 55 mm
56. Ans : 12
Do= 80 mm ; Dr = 65 mm
Sol: l=200 mm, D = 34 mm,
d = 2mm
fm = 18 mm / min ,
80
AP = OR = Compulsory approach

1 L = length of tool travel = / + AP + OR


200 + 8 + 8= 216 mm
55m
=

. L 216 .
T 1meIcut = -=-=12 min
90m
fm 18

!l"S•li@jjj4§..jijQnijftjjmjjj� yderabad I Delhi I Bhopal I Pune I Bhubaneswar I Lucknow I Patna I Bengaluru I Chennai I Vijayawada IVizag I Ttrupati I Kukatpal]y I Kolkata
....... "
.
".. " -� : 750:
"''CE .. Production
-:, �&,pw.rmg
"==========================================
PnNiczioos

57. Ans: (c) 02.


Sol: la = compulsory approach = distance to be 2f {d 2x180 {3
Sol: tl mu = m =
travelled by milling cutter for complete N Z �D 10x106�9()
depth ofmaterial to be removed =0.062 mm
1000x 30
N= = 106
rrx90
1t min =0
t +t
t,avg = l mu i min = 0.031mm
Five Marks Solutions
2

01.
03.
Sol: Dmi n =100mm, Dmax = 200mm Sol: L = 200 + 20+ 20= 240
vmin = 40 m/ min vmax =120m/ min B= 90 + 4 + 4=98
No . ofspeeds= 6= n B
Time/ cut= x�{l+M)
f V
Taking the geometric progression
240
=� x (1+0.83)= 0.57 min
Common ratio, r = n-1 �
� 0.57 13300
'N min
04.
Dmax-
= n-11-- ax = 6_1 200 X 120 = 1.43
Vm-
I

., Dmin Vmin '\ lOOx 40 1


Sol: p =- - = 5mm I thread= pitch
0.2
1000 X Vmin 1000 X 40
NI = = = 200 Pawl is indexing the ratchet by one
Dmax 200
Tooth during every return stroke
N 2 =N I xr = 286� 285 No. ofteeth in the ratchet = 20
N 3 =N I xr 2 = 409� 410 For 1 rev . oflead screw
= 1 rev ofratchet
N 4 =N, xr 3 =585�585
= 20 teeth is to be indexed
N 5 =N, xr 4 =835�835 :. 20teeth �1 rev.of L.S � lpitch � 5mm
lOOxVmax 1000x120 1 tooth is indexed = 2- = _!_= 0. 25mm
N 6 =N mu = = =l200
D mm· 100 20 4
:. feed/ teeth = feed/D.S = 0. 25mm

l1tlli@jjj44.jjQkftajM\jjih�,;:yderabad I Delhi I Bhopal I Pune I Bhubaneswar I Licknow I Patna I Bengaluru I Chennai I Vijayawada I Vizag I Tirupati I Kukatpally I Kolkata I
t . A:CE . . chining·
�-:,•,l'!;L�p�-:!'!l'lll
�:.,11�Pab
��Ji.�r,n
�,n,
;:..:,========.,:,: ,,:,��
75 1
,:.= =============M�..��� "'

D
Time /cut = 10 = B x L (l+M) L =l'+ AP1 =10 + -
f V 2
100 150 ! 20
V= x (1+ ) =10+ --=lO + l=llcm
0.25 10 2 2x10
= 9000 mm/sec = 540 m/min Time /cut = �
f1 ZN
05. 110
= =1.44 min
L 576 O.Olx8x955
Sol: Time / cut = - = ---= 20 min
tN 0.2x144
1tDN 1tx lOOx 144 07.
V= = = 45_2 m/ min
1000 1000 Sol: Nmin =1lOrpm, Nmax = 650 r pm

�J
With G.P
VT'" -75 =>T-(�f' Nmu
Common ratio - r -
)1.333
75
Nmin
= (-- =1.96 min
650
45.2 v =1.43
110
=

No. of tool regrinds = � =11-1=10 N1 = 110 rpm


1.96
(Because 1 tool is already mounted on work N2 = N1 r = llOx1.43=157 =115
piece) N3 =N1 r 2 =llOxl.432 = 225= 225
Total change time / piece = 20+ 10 x 3 N4 = N1 r 3 =llOxl.433 = 321.6= 320
= 50 min
N5 = N1 r 4 =llOxl.434 =460=460
Totaltime required / piece = 50 + 4
= 54min N6 =N1 r 5 =llOxl.43 =650=Nmax
5

06. 08.
Sol: W = 20 mm, / = 10 cm, Sol: Initial dimensions = 205 x 30 x 1
Size = 10 x 20 cm, Vc = 60 m / min, Final dimensions = 205 x 25 x 1
D = 20 mm, No. of flutes = 8, Hence d = 5 mm
Feed = 0.01 mm /flute D = 25 mm,
lOOOxV 1000 x60 Z= 10, N = lOOrpm
N= = =955
1t D 7t X 20 fm = 10 mm / min

\( I J 111.!;llH l 111\g P11hl1< ,111011, yderabad I Dellii I Bhopal I Pune I Bhubaneswar I Lucknow I Patna I Bengaluru I Chennai IVtjayawada I Vizag I T11Upati I Kukatpally I Kolkata
�--:,
'!,� '= . ..t\CE . .
i-fii �-;;;;;;; :,;,;;;;;;;;;;;;;;;;;;;;;,,=========: =75=2=: =================
111I1 ffllll l'l1J,ili.r;mG1III
Production
� �
2xf /d 10.
t1 ax= m
NZ VD Sol: N =34, Nmax = 353 rpm
m
min

2x10 rs No. of speed n = 6


= = 0.009 mm
lOOxlO V25 . ,rm-
Common rat10 = r = n�
34
0.009
= x(50xl)x10x10x100 353
2 = v = 1.597
34
= 2236 mm3 / min
NI =Nmin =34

09. N3 =N1 r 2 =34xl.597 2 =86.7 rpm


Sol: D = 15 mm N4 =N1 r 3 =34xl.5973 =138.5 rpm
Ve= 20 m/min, N = 1000 V ht
D=425rpm 11.
f = 0.2 mm /rev Sol: D = 150 mm, w = 80 mm,
T = 100 min 1=45mm /=160 , Z = 10
Time for idle motions = 20s Offset = Or = 15 mm,
Tool change time = 300 s
Wj = W + 2.0r= 80 + 15 X 2= 110
45 +.!2 ft = 0.25 mm,
. L 2
Time/hole= - = ---=--
fN 0.2x425 Vc = 20 m /min,
=0.617 min = T m l OOOV lOOOx 20
N= = = 42_5
trD ,rxl50
No. of holes produced / drill
APi =½(n-�D 2 -w;2 )
= __!_QQ_ =162
0.617
= .!.(150-.J1so 2 -1102 )=24 mm
2
Avg. production time/hole L = / + APi_ =160 + 24 =184
= Tm + idle time + Tool change time L
Time/cut = --
= 0.617 +
20 300 f 1 ZN
+
60 162x 60
184
0.9812 min = 58.87 = 59 sec = = 1.73 min
0.25x10x42.5
=

lltlli@jjjY4@j0jnjM\hih� yderabad I Delhi I Bhopal I Pune I Bhubaneswarj Lucknow I Patna I Bengaluru I Chennai I Vtjayawada I Vi7.ag I Tirupati I Klikatpally I Kolkata
: 753: Machining

12. 80- 60 20
No . ofcuts = --- = -=4cuts
Sol: No . of D.S/min = 60, / =90, 2x2.5 5
AP = O.R = 5 mm each, IOOO V 1000x132
N2 = = = 560 rpm
F=0.6 mm/ D.S tr D, 1rx 75
1 1000V lOOOx132
W = B = 36mm ' M =- N3 = = = 600 rpm
2 tr D2 1rx 70
Time/cut = Bx!'...(1+ M) 1000V 1000x132
f V N4 = = = 646 rpm
tr D3 1rx65
B
xTime/Double stroke Machining time =I; + T2 + T3 + T4
f
=

B 1 36 1 120 120
-x -= -x -=l mm
f 60 0.6 60
= =---+---
0. 2 X 525 0.2 X 560
120 120
+ ---+ ---
13. 0.2 X 600 0. 2 X 646
40 40 11x 2 22 = 4.14min
Sol: C.R = = =1 =1
N 29 29x 2 58
Total machining time
After every cut the crank has to be rotated
=4.14 + 3 X 40/60
through 1 revolution and 22 holes in 58
6.14 min = 6.2 min
holes circle .
=

14. 15.
Sol: D0 =80, D1 =60 mm Sol: D=20 mm, t =30 mm,
AP = 2mm, OR = 3mm,
.e =116 mm, f =0.2 mm/ rev, d = 2.5mm
f =0.1mm/ rev, V=20 m/min
AP=2.5, OR = 1.5 mm
Time for return and resetting between 1000 V 1000x 20
N= = =318 _5 rpm
passes = 10 sec + 30 sec = 40 sec tr D 1rx20

. 1000V L = t + AP + OR = 30 + 2 + 3 = 35
Spmdle speed = --
tr D 35
Time /hole = ----
1000x132 O.lx 318 .5
_525rpm-
- -N1
,rx80 =1.09 min =1.1min
L= / +AP + OR = 116 + 2.5 + 1.5
= 120 mm

jltl1ih§Oii4.jj4Rflj\jfflmfjj� yderabad I Delhi I Bhopal I Pune I Bhubaneswar I Lucknow I Patna I Bengaluru I Chennai I Vijayawada I Vizag I Tirupati I Kukatpa]]y I Kolkata
: 754: Production

1 6. Ans: (a)
Time/cut= �
Sol: For producing RH threads the direction of f,NZ
rotation ofjob and lead screw must be in the 2 30
= =1.2 min
same direction, for this if the designed gear O.l x159x12
train is simple gear train use 1, 3, 5 odd Ifoffset = 5 mm with asymmetrical milling
number idle gear to get same direction of Wi = w+ 2(0r)= 80 + 2 x 5 = 90
rotation, if the designed gear train is
AP 1 = !(100-�1002-902 ) = 28.2 mm
compound gear train use 0, 2, 4, .. even 2
number of idle gears to get same direction. L = 200 + 28.2 + 5 + 5= 2 38.2
In the given problem the designed gear train L 2 38.2
Time/cut= -- = ---- =1.25 min
is a compound gear train, to change the f1 Nz O.l x12x159
hand of the thread it requires to change the
direction of rotation of job and lead screw 18.
Sol: Given, d = 2 mm, w = 150 mm,
for this use 1, 3, 5 Odd number of idle
t= 400 mm, D= 250 mm, Z= 8
gears.
Ye =1 .2ml sec,
17. l OOOx1.2
N= =1.5 3 RPS
Sol: Part size= 200 x 80 x 60 mm 7tX 250

D = 100 mm, Z=12, ft= 0.1 mm


V= 50 m/min, AP 1 = !( 250-� 2502 -15 02 )=25mm
1000V 1000x50 2
N= = =159 rpm
trD ;r xl OO L= 400 + 25 = 425 mm

f1 =0.1 mm , AP =OR= 5 mm Time/cut= �


f1 ZN
With symmetrical milling
425
= = 347.2sec
APi =!(n - �D -w 2
O.l x8x1.5 3
2
2)

= 5.78 min
= !(100-�1002-802 )=20mm 19.
2
Sol: L=2m
L = l +APi +AP + OR
= 5 0 + 900 + 5 0 + 50 + 900 + 50
200 + 20 + 5+5= 2 30
B = 300 + 5 + 5 = 310
=

f = 1 mm/stroke, Vc = 1 m /sec,

l1Jilih§h/iiihi4Rfl§jjfoijj.\jfyderabad I Delhi I Bhopal I Pune I BhubaneswarI Lucknow I Patna I Bengaluru I Chcnnai I Vijayawada IVizag I T,rupati I Kukatpally I Kolkata I
.,�-t

. ACE
. ..
:&grrw.rmgPnbtiarioos : 755 : Machining

1 OR = 2 mm
M=-
2 J, = -- 200
f, =____!!l_ - =0.056 mm/too th
NZ 300x12
Time per two pieces = B x L (1+M)
f V Ap 1 =�d(D-d) = �2(50 - 2) =9.79
BO 2000
= x (1+ 0.5) = 930 sec L = 500 + 9.79 + 2= 511.8 mm
1 1000
. L 511.8 .
930 T1meIpass = - = -- = 2.56 mm
Time/piece = = 465 sec fm 200
2

22.
20.
Sol: C L = 200mm, Z = 4,
Sol: d 101•1 = 4.5mm
fm = 200 m /min , D= 100mm,
d r =0
N = 1OOrpm, d= 2mm
d 5 = 0.0125x8 = 0.1 AP + OR= 5mm
d , = d total -(d r + d s ) =- x +-
t lma- t l min-
4.5 - 0.1 = 4.4 2
t1 avg
=

4.4 fm
=
/d
200 /2
n =�= = 44 teeth
=

r h , 0.1 NZ � D 4x100 � 100


= 0.0707 mm = 71 microns

23. Ans: (b)

Sol: AP1 = _!_ ( D-�D 2 -w 2 )


Cutting length = effective length 2
= L, + L 5 + L r = (�2(100 - 2 )= 14mm

=44 X 22 + 8 X 20 + 4 20 . Icut =-L


X
Time
= 1208 mm fm
200 + 5 + l 4
= x60 =65.7sec
21. 200
Sol: D = 50 mm, Z = 12,
N = 300 rpm, d = 2 mm
l = 500 fm = 200 mm/min

!ltlli@h/i§.hi$RflblM\hih� yderabadl Delhi I Bhopal I Punc l Bhubaneswar i Lucknow! Patna I Bengaluru I Chennai I Vtjayawada! Vmg I Tirupati I Kukatpally l Kolkata
cs Metal Forming
05. A test specimen is stressed slightly beyond
One Mark Questions the yield point and then unloaded. Its yield
strength (GATE-ME-95)
01. For wire drawing operation, the work
(a) Decreases
material should essentially be
(b) increases
(GATE-ME-8 7)
( c) remains same
(a) ductile (b) tough
( d)become equal to UTS
(c) hard ( d) malleable

06. Hot rolling of mild steel is carried out


02. In forging operation the sticking friction
(GATE-ME-02)
condition occurs near the .... (Center/ ends ) (a) at recrystallisation temp
while sliding friction condition occurs near
(b)between 1OOC to 150C
the . ... (Center/ ends ) (GATE-ME- 87)
( c)below recrystallisation temp
( d) above recrystallisation temp
03. The true strain for a low carbon steel bar
which is doubled in length by forging is
07. Ductility of material with work hardening
(GATE-ME-92)
(GATE-ME-02)
(a) 0.307 (b) 0.5
(a) Increases (b) Decreases
(c) 0. 693 (d) 1.0
( c) Remains same ( d) Unpredictable

04. The process of hot extrusion 1s used to


08. Cold working of steel is defined as working
produce (GATE-ME-94)
(GATE-ME-03)
(a) Curtain rods made of aluminium
(a) at its recrystallisation temperature
(b) Steel pipes of domestic water supply
(b) above its recrystallisation temperature
( c) Stainless steel tubes used in furniture
(d) Large size pipes used in city water (c) below its recrystallisation terpperature
mams. ( d) at two thirds of the melting temperature
of the metal

l1111@114111111!11:-�lJPJJ!i!HlllfjJ yderabacl I Delhi I Bhopal I Pune I Bhubaneswar I Lucknow I Patna I Bengaluru I Chennai j Vtjayawada I V123g I Tuupati I Kukalpa]]y I Kolkata
: 757 : Metal Forming

09. The maximum possible draft in cold rolling 14. While rolling a strip the peripheral velocity
of sheet increases with the of the roll is ....A ... than the entry velocity
(GATE-ME-11) of the strip and is ...B .....the exit velocity of
(a) increase in coefficient of friction the strip. (GATE-PI-9 0)
(b) decrease in coefficient of friction A. greater I less /equal to
( c) decrease in roll radius B. greater than / less than / equal to
( d) increase in roll velocity
15. Seamless long steel tubes are manufactured
10. Collapsible tubes are made by by rolling, drawing and .... (GATE-PI-91)
(GATE-PI-89)
(a) Drawing (b) spinning 16. At 1000° C the crystallographic structure of
( c) Extrusion (d) Rolling iron is .. . .. . . . . . . . . .. (GATE-PI-91)

11. The blank diameter used in thread rolling 17. Thread rolling is restricted to
willbe (GATE-PI-89) (GATE-PI-92)
(a) Equal to the major diameter of the thread (a) ferrous materials (b) ductile materials
(b) Equal to the pitch diameter of the thread ( c) hard materials ( d) none of the above
(c) little higher than the minor diameter of
the thread 18. In order to reduce roll pressure in strip
(d) little higher than the pitch diameter of rolling, back tension can be applied to strip.
the thread (True I False) (GATE-PI-94)

12. At the last hammer stroke the excess material 19. Forces in hot extrusion is a function of the
from the finishing cavity of a forging die is strain hardening component of the billet
pushed into .............. (GATE-PI-89) material. (True I False) (GATE-PI-94)

13. Semi brittle materials can be extruded by 20. A moving mandrel is used in
(GATE-PI-90) (GATE-PI-94)
(a) impact extrusion (a) wire drawing (b) tube drawing
(b) closed cavity extrusion ( c) metal cutting (d) forging
( c) hydrostatic extrusion
( d)backward extrusion

!IJ•li!i§jjj§4jjjj4Rffftttjj@j� yderabad I Delhi I Bhopal I Pune I Bhubaneswar ! Lucknow! Patna I Bengalwu I Chennai I Vijayawada jVizag I Tirupati I Kukatpally I Kolkata
"
��-�
.. :� P.Jln,mma
-' . ACE
. . . : 758 : Production

21. Which one of the following manufacturing 26. Hot die steel , used for large solid dies in
processes requires the provision of 'gutters' drop forging , should necessarily have
(GATE-PI-94) (GATE-PI-10)
(a) closed die forging (a) high strength and high copper content
(b) centrifugal casting (b) High hardness and low hardenability
( c) Investment casting ( c) High toughness and low thermal
( d) impact extrusion conductivity
( d) High hardness and high thermal
22. In order to reduce roll pressure in strip conductivity
rolling, back tension must be applied to
strip.(T/F) (GATE-PI-9S) 27. Cold shut (lap) may occur m products
obtained by: (GATE-PI-10)
23 . Cold shut (lap) may occur m products (a) Casting (b) Forging
obtained by: (GATE-PI-OS) ( c) Machining (d) Welding
(a) Casting (b) Forging
( c) Machining (d) Welding 28. A solid cylinder of diameter 100mm and
height 50mm is forged between two
24. Flash and gutter are provided in drop forging frictionless flat dies to a height of 25mm.
dies at the following stage: The percentage change in diameter is
(GATE-PI-OS) (GATE-ME & PI-12)
(a) Blocking (a) 0 (b) 2.07 (c) 20.7 (d) 41.4
(b) Preforming (Edgering)
(c) Finishing 29. In a rolling process, the state of stress of the
( d) Fullering material undergoing deformation is
(GATE-ME & PI-1 3)
25. Anisotropy in rolled components is caused (a) pure compression
by (GATE-PI-09) (b) pure shear
(a) Change in dimensions ( c) compression and shear
(b) scale formation ( d) tension and shear
( c) closure of defects
( d) Grain orientation

lii11ii@jjji4jjjj4Rfifi1Mijjjjj.ij>HYdcrabad I Delhi I Bhopal I Punc I Bhubancswarl Lucknow I Patna I Bengaluru I Chennai j Vtjayawada l Vizag l Tirupati I � I Kolkata I
ACE : 759 : Metal Forming
�-��� PnNi:M,1111

" V • • • •

30. In a rolling process, the maximum possible (GATE-ME-14-SET-2)


draft, defined as the difference between the (a) P -II, Q -III, R -VI, S -V
initial and the final thickness of the metal (b) P - III, Q -I, R -VI, S -V
sheet, mainly depends on which pair of the
(c) P -Ill, Q -I, R -IV, S -VI
following parameters ?
(d) P - I, Q -II, R -· V, S - VI
P: Strain
Q: Strength of the work material
33. A moving mandrel is used in
R: Roll diameter
S: Roll velocity (a) wire drawing
T: Coefficient of friction between roll and (b) forging
work. (GATE-ME- 14-SET-4) (c) tube drawing
(a) Q, S (b) R,T (c) S,T (d) P,R (d) bending (GATE-PI- 14)

31. The relationship between true strain and


34. In an open die forging, a circular disc is
(ET)

engineering strain (EE) in a uniaxial tension


gradually compressed between two flat
test is given as (GATE-ME- 14-SET-2)
plates. The exponential decay of normal
(a) EE = In ( l + ET) (b) EE = In ( 1- ET)
(c) ET = In ( 1+ EE) (d) ET = In ( 1- EE) stress on the flat face of the disc, from the
center of the disc towards its periphery,
32. With respect to metal working, match Group
indicates that
A with Group B
(GATE-PI- 14)
Group A Group B (a) there is no sticking friction anywhere on
the flat face of the disc.
P: Defect in extrusion
I: Alligatoring

(b) sticking friction and sliding friction and


Q: Defect in rolling II: Scab
R: Product of skew III: Fish tail
rolling sliding friction co-exist on the flat face
S: Product of rolling IV: Seamless tube of the disc
through cluster mill (c) the flat face of the disc is frictionless
V: Thin sheet with
(d) there is only sticking friction on the flat
face of the disc.
tight tolerance
VI: Semi-finished
balls of ball
bearing

�yderabad l Delhi l BhopaI I Pune l Bhubaneswarl Lucknow l Patna l Benga)uru l Chennai l Vuayawada l Vmg ITnupati I Kukatpallyj Kolkala I
: 760 : Production

35. Match the following products with preferred strain hardening exponent of the material is
manufacturing process: (GATE - PI-15)
Product Process
p Rails 1 Blow molding 39. The value of true strain produced in
compressing a cylinder to half its original
(GATE -16 -SET- 2)
Q Engine crankshaft 2 Extrusion
R Aluminium channels 3 Forging length is
s PET water bottles 4 Rolling (a) 0.69 (b) -0.69 (c) 0.5 (d) -0.5
(GATE -15 -Set 1)
40. Engineering strain of a mild steel sample is
(a) P-4, Q-3,R-1,S-2 (b) P-4,Q-3,R-2,S- l
recorded as 0. 100%. The true strain is
(c) P-2,Q-4,R-3,S-1 (d) P-3,Q-4,R-2,S-1
(GATE -16 -SET- 3)
36. The true stress at fracture of a tensile tested (a) 0.010% (b) 0.055%
specimen, having an initial diameter of (c) 0.099% (d) 0.101%
13mm, is 700MPa. If the diameter of 4 1 . In a metal forming operation when the
specimen at fracture is 10 mm, then the material has just started yielding, the
engmeenng stress, in MPa, at fracture principal stresses are cr 1 = +180 MPa,
lS---- (GATE - PI-15) cr2 =- 100 MPa, cr3 � 0. Following Von
Mises ' criterion, the yield stress is ____
37. In rolling of a flat strip, the relative velocity
MPa. (GATE -17 -SET -l)
of strip with respect to the roller is
(GATE - PI -15)
42. It is desired to make a product having T­
(a) positive at entry plane, negative at exit
shaped cross - section from a rectangular
plane
aluminium block. Which one of the
(b) negative at entry plane, positive at exit
following processes is expected to provide
plane
the highest strength of the product?
( c) positive throughout from entry to exit
(GATE -17 -SET -2)
plane
(a) Welding (b) Casting
( d) negative throughout from entry to exit
( c) Metal forming ( d) Machining
plane
43. A steel wire of 2 mm diameter is to be
38. The maximum reduction per pass during wire drawn from a wire of 5 mm diameter. The
drawing of an aluminum alloy ignoring value of true strain developed is ___ (up
friction and redundant work is 77%. The to three decimal places) (GATE -PI- 17)
\( l I 1 1�1/ll l I I l l� P11lil1< .111011-., �ydcrabadl Ddhi l Bhopal l Pwic l Bhubancswari Lucknow i Patna i Bcngalurul Chcnnail Vijayawadal Vu.ag I Tirupati I Kukatpally i Kolkata I
Metal Forming
... ,. ,...... ACE
� EF.nginr.aing PiiNicmoos : 761 :
..
04. A strip with cross sectional area 150mm x
Two Marks Questions 4.5mm is being rolled with 20% reduction of
area using 450mm diameter rollers . The
01. Calculate the bite angle when rolling plates angle subtended by the deformation zone at
12mm thick using work rolls 600mm the roll center is (in radians)
diameter and reducing the thickness by 3mm (GATE-ME-98)
(GATE-ME-94) (a) 0.01 (b) 0.02 (c) 0.03 (d) 0.06

02. A wire of 0.1mm diameter is drawn from a 05. A brass billet is to be excluded from its
rod of 15mm diameter dies giving reductions initial diameter of 100 mm to a final diameter
of 20%, 40% and 80% are available . For of 50 mm . The working temperature is 700°C
minimum error in the final size, the number and the extrusion constant is 250 MPa . The
of stages and reduction at each stage force required for extrusion is
respectively would be (GATE-ME-96) (GATE-ME-03)
(a) 3 stages and 80% reduction for all three (a) 5.44MN (b) 2.72MN
stages (c) 1.36MN (d) 0.36MN
(b) 4 stages and 80% reduction for first
three stages followed by a finishing 06. In a rolling process, sheet of 25 mm
stage of 20% reduction thickness is rolled to 20 mm thickness . Roll
(c) 5 stages and reduction of 80%, 80%, is of diameter 600 mm and it rotates at 100
40%, 40%, 20% in sequence rpm . The roll strip contact length will be
(d) None ofthe above (GATE-ME-04)
(a) 5 mm (b) 39 mm
03. List -I (c) 78 mm (d)120 mm
A . Rivets for aircraft body
B . Carburetor body 0 7. A 4mm thick sheet is rolled with 300 mm
C . Crank shafts D. Nails diameter rolls to reduce thickness without
List - II any change in its width . The friction
1 . Forging 2. Cold heading coefficient at work-roll interface is 0.1.The
3. Aluminum base alloy minimum possible thickness of the sheet that
4. Pressure die casting can be produced in a single pass is
5. Investment casting (GATE-ME-96) (GATE-ME-06)

Luclmowl Patnal Benga)uru l Chennai l Vuayawada l Vizag ITuupati I Kukatpally l Kolkata I


l1t11ih§jjji4jjjiQRflGiMihii!f;iYc1erabad l Delhi l Bhopal l Pune l Bhubaneswar l
: 762 : Production

(a) 1.0mm (b) 1.5mm Processes Associated


(c) 2.5mm (d) 3.7mm state of stress
P. Blanking 1. Tension
08. In a wire drawing operation, diameter of a Q. Stretch forming 2 . Compression
sheet of steel wire is reduced from 10mm to R. Coining 3. Shear
8mm. The mean flow stress of the material is S. Deep drawing 4. Tension &
4 00Mpa. The ideal force required for
compression
drawing ignoring friction and redundant
5. Tension and shear.
work is (GATE-ME-06)
(a) P-2, Q-1, R-3, S-4
(a) 4.48 kN (b) 8.97 kN
(c) 20.11 kN (d) 31 .41 kN (b) P- 3, Q-4, R-1, S-5
(c) P-5, Q-4, R-3, S-1
09. In open die forging, a disc of diameter (d) P-3, Q-1, R-2, S-4
200mm and height 60mm is compressed
without any barreling effect . The final 12. In a single pass rolling operation, a 20mm
diameter of the disc is 4 00mm. The true thick plate with plate width of 100mm, is
strain is (GATE-ME-07) reduced to 18mm thick. The roller radius is
(a) 1.986 250mm and rotational speed is 10 rpm. The
(b) 1. 686
(c) 1.386 (d) 0.602 average flow stress for the plate material is
300 MPa . The power required for the rolling
10. The thickness of metallic sheet is reduced operation in kW is closer to
from an initial value of 1 6mm to a final (GATE-ME-08)
value of 10mm in one single pass rolling (a) 15.2 (b) 18.2
with a pair of cylindrical rollers each of (c) 30.4 (d) 45. 6
diameter of 400mm . The bite angle in
degrees will be (GATE-ME-07) 1 3. If the elongation factor during rolling of an
(a) 5.936 (b) 7.936 ingot is 1 .22. the minimum number of passes
(c) 8.936 (d) 9.936 needed to produce a section 250 x 250 mm
from an ingot of 750 x750 mm are
11. Match the correct combination for following (GATE-PI-92)
metal working processes . (GATE-ME-07) (a) 8 (b) 9 (c) 10 (d) 17

\(_ l l 1 11.:,lm l 1 111� P11hla ,1!1<>11" Fyderabad j Delhi l Bhopal j Pune l Bhubaneswar j l.ucknow j Patna j Bengaiuru i Chennai l Vtjayawada j Vizag jTuupati I Kukatpallyj Kolkata I
: 763: Met.al Forming

14. To stress relieve a cold worked steel part, it 3. Strength and hardness of the metal are
is heated to a temp close to . . . . A... and decreased
grains obtained after cooling are ........ . 4. Surface finish is reduced
B .... (GATE-PI-92) (a) 1 & 2 (b) 1, 2 & 3
A. lower critical temp/ upper critical temp ( C) 3 & 4 ( d) 1 & 4
B. fine / coarser
18. Consider the following steps in forging a
15. Production process Application connecting rod from the bar stock
(GATE-PI-92) (GATE-PI-03)
A. Extrusion 1. blocking 2. trimming
B. Hot forging 3. finishing 4. edging
C. Metal spinning Select the correct sequence of these
D. Explosive welding operations using the codes given below
(a) 1-2-3-4 (b) 2-3-4-1
1. Cladding of noble metal to base metal
( C) 3-4-1-2 (d) 4-1-3-2
2. Long continuous metal tubes
3. Connecting rod of IC engine 19. The extrusion process( es) used for the
4. Long flanged pipes. production of toothpaste tubes is /are
5. House hold utensils. 1. Tube extrusion
2. Forward extrusion
16. An annealed copper wire of 25 mm diameter 3. Impact extrusion (GATE-PI-04)
is drawn into a wire of 5 mm dia. The (a) 1 only (b) 1 & 2
average yield stress in this operation if the (c) 2 & 3 (d) 3 only
flow curve of the material is given
20. A lever having 90° bend is to be produced by
cr =315e 0· 54 MPa (GATE-PI-94)
drop forging using mild steel bar as raw
(a) 592 MPa (b) 458 MPa
material. The various operations to be
(c) 342 MPa (d) None
performed on it during forging are:
P - Cutting;
17. Cold working produces the following effects
Q - Bending;
(GATE-PI-03)
R - Fullering;
1. Stresses are set up in the material
S - Blocking cum finishing;
2. Grain structure get distorted
T - Edgering

!IJll1i@jji44ijji4Rflnjfimfo,t yderabad I Delhi I Bhopal I Pune I Bhubaneswar I Lucknow I Patna I Bengaluru I Chennai I Vijayawada I Viz.ag I Tirupari I Kuka4>a]ly I Kolkata
: 764 : Production

The correct sequence for performing the 24. Assertion (A) : Cold working of metals
operations is: (GATE-PI-OS) results in increase of strength and hardness.
(a) P-Q-R-T-S (b) R-T-Q-S-P Reason (R) : Cold working reduces the total
(c) T-R-S-Q-P (d) R-Q-T-S-P number of dislocations per unit volume of
the material. (GATE-PI-OS)
21. The true stress-true strain curve is given by (a) Both A and R are individually true and R
cr = 1400 c 0 33
· , where the stress cr is in MPa. is the correct explanation of A
The true stress at maximum load (in MPa) is (b) Both A and R are individually true but R
(GATE-PI-OS) is not the correct explanation of A
(a) 971 (b) 750 (c) 698 (d) 350 ( c) A is true but R is false
( d) A is false but R is true
22. A round billet made of brass is to be
25. A solid cylindrical stainless steel work piece
extruded (extrusion constant = 250 MPa) at
of 200mm diameter and 150 mm height. This
700° C. The billet diameter is 100 mm and
component is reduced by 50% in height with
the diameter of the extrusion is 50 mm. The
flat die in open die forging. Assuming the
extrusion force required ( in MN) is
flow stress of the material as 1000 MPa and
(GATE-PI-OS) the coefficient of friction to be 0.2, the
(a) 1.932 (b) 2 .722 estimated forging force at the end of the
(c) 3.423 (d) 4.650 stroke is (GATE-PI-06)
(a) 20.8 kN (b) 31 kN
23. Consider the following statements : (c) 78.6 kN (d) 78.6 MN
In comparison to hot working, m cold
26. A copper strip of 200 mm width and 30 mm
working, (GATE-PI-OS)
thickness is to be rolled to a thickness of 25
1. higher forces are required
2. no heating is required mm. The roll of radius 300mm rotates at 100
rpm. The average shear strength of the work
3. less ductility is required
4. better surface finish is obtained. material is 180 MPa. The roll strip contact
length and the roll force are (GATE-PI-06)
(a) 15.8 mm & 0.569 MN
Which of the statements given above are
correct ?
(a) l , 2 and 3 (b) 1, 2 and 4 (b) 18.97 mm, 0.683 MN
(c) 1 and 3 (d) 2, 3 and 4 (c) 38.73 mm & 1.395 MN
(d) 38.73 mm & 2.09 MN

!MIIMl@OiMihNQnnj@mjjj� yderabad I Delhi I Bhopal I Pune I Bhubaneswar I Lucknow I Patna I Benga]uru I Chennai jVyayawada I Viz.ag I Tuupati I Kukatpally I Kolkata
: 765: Metal Forming

27. In a rolling process, thickness of a strip is 30. The power required for the drawing process
reduced from 4mm to 3mm using 300mm (in kW ) is (GATE-PI-08)
diameter rolls rotating at 1OOrpm. The (a) 8.97 (b) 14.0
velocity of the strip (in m/sec) at the neutral (c) 17.95 (d) 28.0
point is (GATE-PI-08)
31. The maximum possible percentage reduction
(a) 1.57 (b) 3.14
in area per pass during wire drawing of an
(c) 47.10 (d) 94.20
ideal plastic material without friction is of
the order of (GATE-PI-07)
28. By application of tensile force, the cross
(a) 37 (b) 50 (c) 63 (d) 75
sectional area of bar 'P' is first reduced by
30% and then by an additional 20%. Another 32. Using direct extrusion process, a round billet
bar 'Q' of the same material is reduced in of 100mm length and 50mm diameter is
cross sectional area by 50% in a single step extruded. Considering an ideal deformation
by applying tensile force. After deformation, process (no friction and no redundant work),
the true strain in bar 'P' and bar 'Q' will, extrusion ratio 4 and average flow stress of
respectively, be (GATE-PI-08) material 300 MPa, the pressure (MPa) on the
(a) 0.5 and 0.5 ram will be (GATE-PI-09)
(b) 0.58 and 0.69 (a) 416 (b) 624 (c) 700 (d) 832
(c) 0.69 and 0.69
(d) 0.78 and 1.00 33. In a rolling process , the roll separating force
can be decreased by (GATE-PI-10)
Statement for Linked Answer Q29 & Q3 0 (a) Reducing the roller diameter
A 10mm diameter annealed steel wire is (b) Increasing friction between the rolls and
drawn through a die at a speed of 0.5m/sec to the metal
reduce the diameter by 20%. The yield stress (c) Reducing front tension to rolled material
of the material is 800MPa . ( d) Providing back up r_ollers

29. Neglecting friction and strain hardening, the 34. During open die forging process using two

stress required for drawing (in MPa) is flat and parallel dies, a solid steel disc of

(GATE-PI-08) initial radius ( RIN) 200mm and Initial height


(a) 178.6 (b) 357.0 (HIN) 50mm attains a height (HFN) of 30mm

(c) 1287.5 (d) 2575.0 and radius of RFN· Along the die-disc
interfaces (GATE-PI-10)

!IH1i@jjji4.jj/4RftftftWjj.jj� yderabad I Delhi I Bhopal I Pune I Bhubaneswar I Lucknow I Patna I Bengalwu I Chennai I Vijayawada I Viz.ag I T,rupati I Kukatpally I Kolkata
: 766: Production

(i) The coefficient of friction( µ) is : theses drawing operations, the reduction in


µ=0.35 [ 1 + e-RIN/RFN ] cross-sectional area is 35%. The yield

(ii) In the region Rss s r s RFN, sliding strength of the material is 200 MPa. Ignore

friction prevails and strain hardening. (GATE-Pl-11)

p= ,fj .K.e2 µ ( R,N - r ) /HFN and 't=µ.p


36. The total true strain applied and the final
Where p and are the normal and the shear
length (in mm)
, respectively, are
1:

stresses respectively; K is the shear yield


(a) 2.45 and 817 (b) 2.45 and 345
strength of steel and r is the radial distance of
(c) 3.02 and 2043 (d) 3.02 and 33.
any point
(i) In the region O r RIN, sticking
37. Neglecting friction and redundant work, the
s s
condition prevails force (in kN) required for drawing the bar
The value of RSS ( in mm ,
) where sticking through the first die, is
condition changes to sliding friction is
(a) 15 . 71 (b) 10.21
(a) 241.76 (b) 254.55 (c) 6.77 (d) 4.39
(c) 265.45 (d) 278.20

38. In a single pass rolling process using 410mm


35. The thickness of a plate is reduced from 30 diameter steel rollers, a strip of width 140mm
mm to 1O mm by successive cold rolling
and thickness 8mm undergoes 10% reduction
passes using identical rolls of diameter 600
of thickness. The angle of bite in radians is
mm. Assume that there is no change in
(GATE-ME&PI-12)
width. If the coefficient of friction between
(a) 0. 006 (b) 0.031
the rolls and the work piece is O.1, the
(c) 0.062 (d) 0.600
minimum number of passes required is
(GATE-PI-11) 39. A mild steel plate has to be rolled in one pass
(a) 3 (b) 4 (c) 6 (d) 7
such that the final plate thickness is 213rd of
the initial thickness, with the entrance speed
Common Data for Question 36 & 37 of 1Om/min and roll diameter of 500mm. If
In a multi-pass operation, a round bar of 10
the plate widens by 2% during rolling, the
mm diameter and 100 mm length is exit velocity (in m/min) is _ _
reduced in cross-section by drawing it
(GATE-ME-14-SET-2)
successively through a series of seven dies of
decreasing exit diameter. During each ot
Ir+JIBm111111c111111,HJJE1w•m1F
j) yderabad I Delhi I Bhopal I Pune I Bhubaneswar I LucknowI Patna I Bengaluru I Chennai I Vijayawada f V� I Tirupati I Kuloatpally I Kolkaia
: 767 : Metal Forming

40. A metal rod of initial length Lo is subjected in the first stage is 0.4. The fractional
to a drawing process. The length of the rod at reduction in the second stage is 0.3. The
any instant is given by the expression, overall fractional reduction is
L(t) = Lo( l + r), where t is the time in (GATE -15 -Set 2)
minutes. The true strain rate (in min- 1 ) at the (a) 0.24 (b) 0.58 (c) 0.60 (d) 1 .00
end of one minute is
(GATE-ME-14-SET-l) 44. The flowing stress (in MPa) of a material is
given by cr= 500 i? 1 ,
41. A 80 mm thick steel plate with 400 mm
When s is true strain. The Young's modulus
width is rolled to 40mm thickness in 4 passes
of elasticity of the material is 200 GPa. A
with equal reduction in each pass, by using
block of thickness 1 00 mm made of this
rolls of 800mm diameter. Assuming the
material is compressed to 95 mm thickness
plane-strain deformation, what 1s the
and then the load is removed. The final
minimum coefficient of friction required for
dimension of the block (in mm) is __
unaided rolling to be possible?
(GATE -15 -Set 2)
(GATE-PI-14)
(a) 0. 1 1 1 (b) 0. 1 58 45. In a rolling operation using rolls of diameter
(c) 0.223 (d) 0.3 1 6 500mm, if a thick plate cannot be reduced to
less than 20mm in one pass, the coefficient
42. In a slab rolling operation, the maximum of friction between the roll and the plate is
thickness reduction (�h maJ is given by (GATE -15-Set 3)
= 2
� max µ R , where R is the radius of the 46. Open die forging of a cylinder made of a
roll and µ is the coefficient of friction rigid perfectly plastic material with yield
between the roll and the sheet. If µ = 0. 1 , the strength of 200 MPa having a height of 25
maximum angle subtended by the mm and diameter of 25 mm is being carried
deformation zone at the centre of the roll (bit out. The cylinder is subjected to a true
angle in degrees) is____ compressive strain of 3.6 during the process.
(GATE -15 -Set 1) Assuming frictionless and homogeneous
deformation, the energy expended, in kJ is
43. In a two stage wire drawing operation, the (GATE - Pl -15)
fractional reduction (ratio of change in cross
sectional area to initial cross sectional area)

\( I l 111..:,llll ( l l! H.', Pt d)lh ,tllt l ! h Fydcrabadj Delhi j BhopaJ I Punc l Bhubaneswarl I.ucknow l Patna J Bcngaluru J Chennai l VuayawadaJ V123g JTtrupati I KukalpallyJ Kolkata I
., " . ACE
��-���PnNiratiooa
. . . : 768 : Production

47. A 300 mm thick slab is being cold rolled Assuming uniform roll pressure of 200 MPa
using roll of 600 mm diameter. If the in the roll bite and 100% mechanical
coefficient of friction is 0.08, the maximum efficiency, the minimum total power (in kW)
possible reduction (in mm) is _ _
_ required to drive the two rolls is___
(GATE - 1 6 -SET- 1) (GATE - 1 7-SET- 2)

48. In a single-pass rolling operation, a 200 mm 51. A rod of length 20 mm is stretched to make
wide metallic strip is rolled from a thickness a rod of length 40 mm. Subsequently, it is
10 mm to a thickness 6 mm. The roll radius compressed to make a rod of final length 10
is 100 mm and it rotates at 200 rpm. The roll­ mm. Consider the longitudinal tensile strain
strip contact length is a function of roll radius as positive and compressive strain as
and, initial and final thickness of the strip. If negative. The total true longitudinal strain in
the average flow stress in plane strain of the the rod is (GATE - 1 7 - SET -2)
strip material in the roll gap is 500 MPa, the (a)-0.5 (b)-0.69
roll separating force (in kN) is ___ (c)-0.75 (d)-1.0
(GATE - PI-1 6)
52. A metallic strip having a thickness of 12
49. Two solid cylinders of equal diameter have mm is to be rolled using two steel rolls,
different heights. They are compressed each of 800 mm diameter. It is assumed
plastically by a pair of rigid dies to create the that there is no change in width of the strip
same percentage reduction in their respective during rolling. In order to achieve 10%
heights. Consider that the die-workpiece reduction in cross-sectional area of the strip
interface friction is negligible. The ratio of after rolling, the angle subtended (in
the final diameter of the shorter cylinder to degrees) by the deformation zone at the
that of the longer cylinder is _____ center of the roll is (GATE - PI - 1 7)
(GATE - PI-16) (a) 1.84 (b) 3.14
(c) 6.84 (d) 8.23
50. A strip of 120mm width and 8mm thickness
is rolled between two 300 mm-diameter rolls
to get a strip of 120mm width and 7.2mm
thickness. The speed of the strip at the exit is
30m/min. There is no front or back tension.

!li1i4jj§jjji4.jjiRflbiM\hfh� yderabad I Delhi I Bhopal I Pune I Bhubaneswar I Lucknow I Patna I Bengalwu I Chennai I V\iayawada I Vmg I Tirupati I Kukatpa]ly I Kolkata
: 769 : Metal Forming

04. In a wire drawing operation diameter of 6mm


Five Marks Questions is reduced in stages to a diameter of 1.3 4mm .
Assuming ideally rigid plastic material and
01. A strip of thickness T mm is rolled in a 2- ideally lubricant condition,
high single pass rolling mill, having roll i) The minimum number ofpasses required
diameter D mm, to a final thickness of (T - by assuming max reduction per pass is
2�T)mm . If the friction coefficient between obtained with µ= 0.2 and half die angle
the roll and strip is µ, calculate the maximum as 6° . ifthe flow strength ofthe material
reduction 2�T possible in this operation. is 60MPa .
Using this relation, for D = 3 00mm, T = ii) The diameter ofwire after 2nd stage
40mm and friction coefficient is 0.3, (GATE-PI-89)
calculate the outgoing thickness of the strip .
If the inlet velocity is 5m/s, what is the 05. A steel wire of length 100 m and diameter
outgoing strip velocity? . (GATE-ME-91) 12.214 mm is drawn to a final diameter of
10mm . Tensile tests of specimen made
02. A metal strip is to be rolled from an initial before and after the drawing operation gave
wrought thickness of 3. 5 mm to a final rolled yield stress as 200 MPa and 400 MPa
thickness of 2.5 mm in a single pass rolling respectively, (GATE-PI-90)
mill having rolls of 250 mm diameter . The
strip is 450 mm wide . The average i) The length ofdrawn wire is
coefficient of friction in the roll gap is 0.08. (a) 100 m (b) 125 m
Taking plane strain low stress of 140 MPa, (c) 150 m (d)l 7 5 m
for the metal and assuming negligible ii) The yield stress of the steel at a true
spreading, the roll separating force is ........ stain 0.2, by assuming linear strain
(GATE-ME-97) hardening law .
03 . A cylindrical billet of 100 mm diameter is (a) 200MPa (b) 3 00MPa
forged from 50 mm height to 40 mm at 1000° (c) 400MPa (d) 600MPa
C. The material has constant flow stress of
80 MPa . Find the work of deformation . If a 06. Calculate the minimum number ofhot rolling
passes necessary to reduce an ingot of
200mm thickness to 100mm thickness in two
1 0 kN drop hammer is used to complete the
reduction in one blow. What will be the
height of fall. (GATE-ME-00) high reversible rolling mill with roll diameter

Ja1j1iii4jjji4.jj1i@nftihfjj� yderabad I Delhi I Bhopal I Pune I Bhubaneswar I L.icknow I Patna I Bengalwu I Cheruiai I Vijayawada I Vizag I Tirupati I KukatpalJy I Kolkata
flt V • ACE • • •
: 770 : Production
,.�
-:... �f..,,...,..PnNir,tirn

is 500 mm. The coefficient of friction


between the rolls and the hot material is
assumed as 0.20. (GATE-PI-93)

07. In a single pass flat rolling operation, a


400mm wide steel strip having a thickness of
1 0mm is reduced to 8mm by using a roll of
600mm diameter. (GATE-PI-03)

i) The roll-strip contact length is


(a) 24.5 mm (b) 34.6 mm
(c) 1 7.3 mm (d) 49 mm

ii) Considering the situation for a maximum


draft of the process, what is the
coefficient of friction?
(a) 0.082 (b) 0.007
(c) 1 7.3 mm (d) 49 mm

\( f l llJ.!lllt ( l l [IJ.! P11lil1t 1!11 ) 1 1 ', I Delhi I Bhopal I Punc I Bhubancswar I Lucknow I Patna I Bcnga)uru I Chcnnai I Vuayawada I Vizag I TIIUpali I KukatpaIJy I Kolkata I
�yderabad
SOLUTIONS
06. Ans: (d)
One Mark Solutions Sol: The method of deforming a material at a
temp greater than the recrystallization temp
01. Ans: (a) is called hot working.
Sol: Only ductile materials has highest
percentage elongation and hence wire 07. Ans: (b)
drawing can be done easily . Sol: When material is stressed beyond the yield
point under cold condition, the material will
02. Ends, Ce nter. experience the work hardening or strain
hardening which increases the yield strength
03. Ans: (c) and reduces the ductility.
L1 - L 2L - L
Sol: Engg. strain ( e)= ' = -- = 1
L; L 08. Ans: (c)
True strain = ln( l + e)= In (2)= 0. 693 Sol: The method of deforming a material at a
temp less than the recrystallisation temp is
04. Ans: (b) called cold working.
Sol: Curtain rods and SS tubes in furniture are
made by sheet bending and resistance 09. Ans: (a)
welding where as large size pipes used in Sol: �max = µ2 R , means that maximum
city water supply are produced by reduction can be increased by increasing the
centrifugal casting . coefficient of friction.

05. Ans: (b) 10. Ans: (c)


Sol: When material is stressed beyond the yield Sol: Seamless and collapsible tubes are
point under cold condition, the material will manufactured by using impact extrusion
experience the work hardening or strain process.
hardening which increases the yield strength

!Mll$i@jji4g.jj/4Rfln5i11foj.� yderabad I Delhi I Bhopal I Pune I Bhubaneswar I Lucknow ! Pama I Benga)uru I Chennai I Vijayawac:la I V,zag I Tirupari I K�y I l(olkara
�...
ACE Production
�_,....�:f�PoNic,tioos : 772 :
" \I • • • •

11. Ans: (b) 22. Sol: True. With the application of back and
Sol: In thread rolling no material is removed but front tension, the forces induced in rolling
the materials is flown from one place to can be reduced.
other hence the dia. of original rod is equal
to pitch diameter. 23. Ans: (b)
Sol: Cold shut is most commonly produced in
12. Ans: Gutter forging and sometimes it may produces in
the casting operation also.
13. Ans: (c)
Sol: During extrusion of semi-brittle materials, 24. Ans: (c)
due to friction cracks are produced in the Sol: Flash and gutter are related to closed die
surface of the extruded component, hence to forging operation only. And out of the given
avoid the container wall friction the operations finishing is the only closed die
hydrostatic extrusion can be used. forging.

14. Ans: Greater , less than 25. Ans: (d)


15. Ans: Extrusion process Sol: Anisotropy is nothing but the properties are
16. Ans: Austinite or y-iron different in different direction. Even though
the material isotropic at the beginning but
17. Ans: (b)
during rolling the material is converted to
18. Ans: True
anisotropic.
19. Ans: False

26. Ans: (d) 27. Ans: (b)


20. Ans: (b)
Sol: To maintain the internal size of the tube in 28. Ans: (d)
tube drawing the mandrel can be used Sol: Aoho=Athr
d ih 0 = d ih r
21. Ans: (a)
d ih 0 = d ih r
Sol: In closed die forging 10 to 20% of extra
material is provided to ensure that the dies
can be filled completely and to
Dr = do � = 1 00
�� 25 = 141.42
V{so
accommodate this extra material gutter must 141.42-100
- ----- - 41.40/
/o ch ange -
0/
be provided in the dies. 100
/O

!Jfll@04jjj4imijRfln1Mjij.jj� yderabad I Delhi I Bhopal I Pune I Bhubaneswar I Lucknow I Patna I Bengaluru I Chennai I Vijayawada I Vizag I Tirupati I Kukatpally I Kolkata
ACE
: 773 : ·
Metal Forming
� �'Ii'....:......' n.1:1===
�... = •����:;��cnog Cl
..,,....-..
.... - ll'• .!.uul;;iuuu
��� · =============================�

29. Ans: (c) Thin sheet with tight tolerance: During


Sol: (Compression and shear) cluster rolling process because of tight
During rolling process, during travel of a tolerance on the product, thin sections are
material through the rollers a vertical force produced
and a sliding force ( shear force) will act on
the grains. 33. Ans : (c)
Sol: In tube drawing operation, to maintain the
30. Ans: ( b) required inside diameter of the tube the
Sol: Maximum reduction per pass = �Hmax / pass mandrel will be used.
=µ 2
x R, Where µ = coefficient of friction,
R = radius of roller 34. Ans : (d)
Sol: From the figure it is seen that sticking
3 1. Ans: ( c) friction is causing to vary the stress in the
Sol: True strain ( i::) = In ( 1+i::E)
disc from center to the periphery and
Where, EE = engineering strain = In (A0/A 1 )
produces friction hill.

32. Ans: (b)


Sol: Strictly speaking both alligatoring and fish 5.2
tail belong to rolling only.
Alligatoring: In rolling of slabs,
5
p 4
particularly in Al alloys, the work piece 2k 3
may split along the horizontal plane on exit,
with top and bottom parts following the
2

rotation of their respective rolls. This is µ=O


mainly occumng in materials having 0
limited ductility like Al-Mg alloys with
high Mg content and also in some of the
Zinc and Cu alloys.
:- a =-

Fish tail: Unwanted end shapes can be


developed particularly in rolling of slabs
35. Ans: (b)
and blooms and can have important
consequence for production cost.

!lflliii11hiMib4QnflftM\iiih� yderabad I Delhi I Bhopal I Pune I Bhubaneswar I Lucknow! Patna I Bengalwu I Chermai I Vtjayawada I Vizag I T,rupati I Kukatpally I Kolkata
: 774 : Production

36. Ans: 412 to 41 6 41. Ans: 2 45.76


Sol: Initial dia, do= 13mm, Sol: cr 1 = 180MPa, cr2 = -l OOMPa, <J3 = 0
Final dia, d 1 = 10mm Von Mises yield stress,
True stress, cr = 700 MPa
J
r r
True strain =in(�:
=�1802 +1002 -180x(-100)
245.76MPa
en(:: =inc� = 0.524
=

True strain = ln (1 + e) 42. Ans: (c)


=> 0.524= ln( l + e) Sol: Metal forming gives grain orientation and

Engineering strain, e = 0.688 fine grains. Hence strength is high.

True stress, cr = cr(l + e)


43. Ans: 1.832
=> 700= cr(1 + 0.688)
Sol: True strain = ln(A0/A1)
=> cr = 414.69 MPa
=2 ln (do/d1)
2 ln(5/2)= 1.83258
37. Ans: (b)
=

38. Ans: 0.4 to 0.5

39. Ans: (b)


Two Marks Solutions
. change in length
So1 : Engg. stram = ------
original length
01.
( �-L) Sol: Dia of rollers = 600 mm,
= =-0.SL/L=---0.5 R = 300 mm, H0= 12mm, �H= 3mm
L
H 1 = 12- 3 = 9
True strain = ln(l+e) = ln(l-0.5)= -0.69
Bite angle = a = tan -i
40. Ans: (c)
��

Sol: e =In (1+ e) 3 =


= tan -1 � 5.72° (or) 0.1 rad
where, f: = Engineering strain, 300
E = true strain
e = In (1 + 0.001)= 0.099%
!IJll@i@jjj4§foi4Rflftiffi\11jjjj.� yderabad I Delhi I Bhopal I Pune I Bhubaneswar I Lucknow I Patna I Bengaluru I Chennai I Vijayawada IVu.ag I Tlfllpari I Kukalpa!ly I Kolkata
., " . ACE
. . . : 775 : Metal Forming
-:.-�:�Pnhliraoona
02. Ans: (b) 04. Ans: (d)
Sol: do = 15 mm, dr = 0.1 mm Sol: A o = 150 X 4.5mm = 675mm 2
% Reduction
% reduction in area = 20% = A Ai
dia reduced in the draw Ao
0

= --------
-

dia before draw

/st draw
d0 - d1
= � A 1 = Ao - 0.2 Ao = 0.8 Ao
do = 540 mm2 =150x H1
2nd draw
d1 - d2
= � H 1 = 3.6 mm
di
D = 450 mm, R = 225 mm
(a) 3 stages with 80% reduction at each
LiH = 4.5 - 3.6 = 0.9
stage
tan a =
V R = 0.063 rad
- {MI
0.8 = do di => d1 = 0.2 d = 3mm
do
0

d2 = 0.2. d1 = 0.6mm 05. Ans: (b)


d3 =0.2 .d2 = 0.12mm (Error is 2%) Sol: do = 100 mm ,
(b) 4 stages with 80% reduction in dr = 50 mm
1 st 3 stages followed by 20% in K = Extrusion constant = 250 MPa
4th stage
d 1 = 0.2. do = 3 Extrusion force = A, x K 1n( �: J
d2 = 0.2 . d 1 = 0.6
d3 = 0.2 . d2 = 0.12
1n(�J
2
= Jr 1002 X 250 X 2.72 MN
<4 = 0.8 . d3 = 0.096 (Error is 0.4%)
=
4 di
(c) 5 stages, with 80, 80, 40, 40, 20 etc
d, = 0.2. do = 3 06. Ans: (b)
d2 = 0.2. d, = 0.6 Sol: H0 = 25, H 1 = 20, D = 600, N = 100 rpm
d3 = 0.6. d2 = 0.36 LiH = 25 - 20 =5
<4 = 0.6. d3 = 0.0216 R = 300
d5 = 0.8. <4 = 0.1728 (Error is 7.28%) Roll strip contact length
From the given, multiple choice B, the final
dia. of wire close to 0.1 mm. = Ra = R x (,an -• �� } ,;o

03. Ans: A-3, B-4, C-5, D-2. = 38.5mm = 39mm


�ydcrabad l Delhi l Bhopal i Punc J Bhubancswar i l.ucknowJ Patna J Bcngaluru J Chcruiai J Vtjayawada l Vu.ag J Tirupati I Kukatpa)JyJ Kolkata I
.., " . . . . ACE Production
-:.�-:Lpw,qPuNnciooa : 776 :

07. Ans: (c) 11. Ans: (d)


Sol: H0 = 4mm, H1 =?, D = 300mm, 12. Ans: (a)
R=150mm, µ =0.1 Sol: H0=20mm, b = 100mm,
Mlmax µ 2 R = 0.12 X 150 = 1.5mm
µ = " = 0.089
=

Ho - Hi = 1.5
H, = Ho - 1.5= 4 - 1.5 = 2.5mm H1 =18mm,
R = 250mm,
08. Ans: (b) N = 10 rpm, cr y = 300 Mpa
Sol: do = 10, dr= 8 mm, aY = 400 Mpa Ml =20 -18 = 2mm
Under idle conditions L =length of Deform ation Zone =.JRMI
Force Re quired =A rea x Stress x Strain = .J250 x 2 = 22.36 mm
20 +18
H=
2

= tr 8 X 400 X ln(�J = l9
4
2

di

= : 8 X 400 X ln (
2 10
g)
2
=8.97 k:N
Favg = R.S.F = 1 cr y b x L[l + ;� ]

0 089 X 22·36
= � X 300 X 100 X 22.36 [1 + · ]
v3 4 x19
09. Ans: (c)
=795 k:N.
Sol: d i = 200 mm, h i = 60 mm ,
dr = 400 mm
T = Favg x a,
True stram - m(J J - m( � )' Where a = moment arm
= ). L = 0.3 to 0.4 x L
400
ln( 1.386 Favg X 0.4L 795 X 103 X 0.4 X 22.36
2
= ) = = =
200
=7110

10. Ans: (d) 2n NT = 2n x 10 x 7110


pavg =
60000 60000
Sol: H0 = 1 6mm, H1 = 1 0mm, R = 200mm
= 7.44 kW I roller

Angle of Bite =a = tan -i �� Total Power = 7.44 x 2


=14.88 kW
16 -10
= tan -1 � = 9.9
200
li1•i4@jjjl4ijjjQAfjfilM\d.jj� yderabad I Delhi I Bhopal I Pune I Bhubaneswar I wcknowl Patna I Benga)uru I Chennai I Vtjayawada I Vttag I Tirupati I Kukatpally I Kolkata
�..
\...:&,piecnn,Pnbtiariooa
¥ • • •ACE

: 777 : Metal Forming

1 3. Ans: (c) 18. Ans: (d)


A0 Sol: During manufacturing of connecting rod
Sol: Elongation factor = Draft = = 1.22
A1 from bar stock the sequence of performing
operations used are fullering, edging,
A; = 750 x 750 = 562500.
blocking, finishing and trimming.
A1 = 250 x 250. = 62500 mm
2

19. Ans: (d)


A1 = A_ = 461 065 > A1
1.22
20. Ans: (b)
A2 = -- = ..... and so on. Sol: Always perform the open die forgings first,
A1
1.22
A1 0 � 62500, no. of passes = 1 0 next is semi-closed die and last will be the
closed die forging and then trimming or
14. Ans: A. lower critical temp, B. fine cutting

15. Ans: A-2, B-3, C-5, D-1 21. Ans: (a)


Sol: ay =1400 E 0
· 33

1 6. Ans: (a)
At maximum load, true strain = .!.
Sol: do = 25, di = 5mm 3
= 315 E 54
l) 33
cr y =1400( = 971 MPa
0
CY .
0
·
y
3
22. Ans: (b)
cr y = 315 (3. 22) 0.s4 = 592 MPa. Sol: Extrusion constant = K = 250
do = 100mm, dr = 50mm

17. Ans: (a) Extrusion Force - A, K 1{ � J


Sol: During cold working due to work hardening ;

or strain hardening , strength and hardness 100


2
= tr 100 2 X 250 1n( )
increases, ductility reduces, residual stresses 4 50
are generated and grain structure get = 2. 72 MN.
distorted.
23. Ans: (b)

li1•1i@jjji4ijjj$RflGiMiiii1+1yderabad I Delhi I Bhopal I Pune I Bhubaneswar I Lucknow! Patna I Benga)uru I Chennai I Vuayawada I Vizag I TU"Upati I Kukatpal]y I Kolkara I
ACE Production
��-� :F,..,..,..Pobliraioos : 778 :
V V • • • •

..

24. Ans: (c) Roll strip contact length = Arc length of


Sol: Cold working will never reduce the number Deformation Zone = R.a
of dislocation but it may reduce the size of
300x 7.36x- 7t
= 38.72mm
180
=
dislocation
Roll separating force = Favg
25. Ans: (c) µL
Sol: do = 200 mm, ho =150 mm, .fj cr Y bL((l + 4H )
=�

1 0·129 X 38·72
di = d. Jf = 200 :; = 282.8 = � X 150 X 200 X 38.72(1 + )
J3 4x 27.5
h1 =0.5xl 50=75, = 1402 kN = 1.4 MN
CJj = erY = 1000 .MPa, µ = 0.2
27. Ans: (a)
A1 = tr x 282.8 = 62800 mm
2 2 Sol: H0 = 4, H1 = 3mm,
4 R = 150mm, N = l OO rpm.

Forging force = A1 x erY ( I +


2 µ rt
3hf J
2 X 0 2· X l 4l .4
Velocity of strip at neutral point
= Surface Velocity of rollers

=
mJN
= 1r x 300 x100 =1.57 m/sec
= 62800 X 1000(1 + ) 1000 X 60 1000 X 60
3x75
= 78.6 MN. 28. Ans: (b)
Sol: A0p = C. S area of P originally
. 26. Ans: (c)
A1p = C. S area of P after 1st reduction
Sol: b = 200 mm, R = 300 mm,
= 0.7 Aop
L =.JRM! = 38.72 A2p = 0.8 x 0.7 x A0p = 0.56 Aop
H0 30 mm, N = 100 rpm,
A
= True strain in " P' = In( �J
=

µ =tan � = tan a = 0.129 E


P
A2 p
H1 = 25 mm,
180 = ln(Aop J = 0.58
Tu =180 Mpa=0.6eru =;. er,, =-=300 0.56 A0p
0.6
AOQ = C.S area of Q originally
Ml = H O H 1 = 30-25 = 5mm ,
A 1 Q = C. S area of Q after 1st reduction
-

cr y = 0.5cr u = 150 MPa = 0.5 AoQ

a = tan -1 1 � = tan -1 � 5
fi-
!MNIIUYHiiiiMIAbblMihih�
300
= 7.36 °
AIQ J
e Q = ln ( AoQ = ln ( - 1 =
0.5 )
0.693

yderabad l Dclhi l Bhopal l Punc l Bhu�I I..ucknowl �I Bcngalwu l Chennai lVuaya-Ja l Vuag ITuupati I Kukalpally l Kolkala
" "
��-�� �Pnbtiratioo•
. ACE
. . . : 779: Metal Forming

29. Ans: (b) 32. Ans: (a)


30. Ans: (a) Sol: A, L0 = 100 mm, cr Y = 300 MPa
Sol: Common solution for Q29 & Q30 do = 50 mm
do = 10mm, V = 0.5 m/sec A
Extrusion ratio= o = 4, A0 = 1962.5 mm2
d1 =(1-0.2) x d0 =0.8 d0 =8mm Ai
cr y = 800 MPa
7t 50
A0
2

AI = 4- = 490 mm 2
=-
(i) Ideal drawing stress - o , In(: J 4 4
:

Extrusion force - A 0 , In(: )


- soo in ( �: ) ' :
a

=1962.5 X 300 ln(4) = 816.2 kN.

- soo ,n(1i)' Extrusion pressure = Extrusion force/Ao


816.2 x 10 3
357 MPa - (B) = = 416MPa
1962.5
=

Force Required in drawing


= Stress x Ar 33. Ans: (a)
Sol: RSF can be reduced by reducing roll
= 357 X Jr 8 2 = 17.935 kN
4 diameter, reducing friction, reducing yield
(ii) Power required stress of the material, applying back and
= Force x Velocity front tension.
= 17.935 X 10 3 0.5
34. Ans: (b)
X

=
8.967 kW - (A)
35. Ans: (d)
31. Ans: (c) Sol: H 0 = 30 mm (�H)
} = total =
Sol: At maximum possible reduction with µ = 0 Hr = 10 mm 30-10 20

A D = 600 mm , R = 300 mm, µ = 0.1


0'2
=1=.t'n( o J � Ao = e 1 � � = _!_
a-Y A1 A1 A0 e (�H) Max per pass = µ 2 R =0.12 x 300 = 3
A0 - A 1 �H total
Percentage reduction area = No. of passes =-----
Ao �H Max / pass
20
=1--1 =0.63 = 63%. = -= 7 passes.
1
e 3

!Mllih§jjjiUlh/QRfln!M\u,O� yderabad I Delhi I Bhopal I Pune I Bhubaneswar I Lucknow I Patna I Bengalwu I Chennai I Vijayawada I Vizag I Tuupati I Kukatpally I Kolkata
: 780 : Production

36. Ans: (c) 38. Ans: (c)


Sol: D = 410 mm, R = 205 mm
37. Ans: (d)
= 140mm, Ho = 8mm
Sol: Common Solution for Q36 & Q37.
b

d0 = 10mm, L0 = 100mm, H 1 = Ho - 0.1 x Ho = 0.9Ho = 7.2mm


H
- 1 �D. n
A0 = -
1(
x 10 2 = 78.54 mm 2 a= tan R
= 3.57x - = 0.062
4 18

39. Ans: 14.7


� =1-0.35 = 0.65 Sol: H 1 = 2/3 Ho, Vo = l Om/min,
Ao D = 500mm => R = 250mm , b 1 = 1.02 bo
A 1 = 0.65 x 78.54 = 51.05 Volume flow rate of metal before rolling
A2 = 0.65 x 51.05 = 33.2 = Volume flow rate of metal after rolling
A3 = 0.65 x 33.2 = 21.6 Ho.ho.Vo = H 1 . b 1 V1
,¾ = 0.65 X 21.6 = 14.02
V_1
_ H_
_ b_ H ob o
= _o o = =1 47
A5 = 0.65 x 14.02 = 9.11
- H 0 x 1.02b 0
2
A6 = 0.65 x 9.11 = 5.92 3
A1 = 0.65 x 5.92 = 3.85 V 1 = 1.47 Vo = 1.47 x 10 = 14.7 m/min

d
1( 2
=
4 7 40. Ans: 0.693
d1 = 2.21 mm Sol: L 1 = Length of the rod at the end of 1 min
= Lo ( 1 + 12 )= 2Lo
Ao LI
L1 = 2047 Ao. Lo = A 1 L 1 => - =-
A1 Lo

True Strain � 1n[ �: ) � 3.02 True strain � en [:: J� J en[�:


1
cr 2 =cr y .en [ A o J =200x .en(--)=86.156MPa
A1 0.65 � en[ 2� 0
]� In (2) � 0.693
Drawing load = cr2 x A 1 = 86.15x51.05
= 4398 N= 4.398 kN

!IS11i@jjjQ4ijjj$@b1Mmii!� yderabad I Delhi I Bhopal I Pune I Bhubaneswar I wcknowl Patna ! Bengalwu I Chennai I Vijayawada I Viz.ag I TJJUpati I Kukatpally I Kolkata
: 781 : Metal Forming

41. Ans:(b) Overall fractional reduction


Sol: Total reduction in thickness A o- A 2 A
= = 1- 2
= 80- 40 = 40mm Ao Ao
No. of passes =4 0.42A 0 =
=1 1 -0.42 = 0.58
Maximum reduction per pass Ao
40
= - = 10mm 44. Ans: 9 5.14 to 95.2 0
1
Radius of rollers = 400mm Sol: cr = 500 s0·
1

r-
Maximum reduction per pass cr = 500 (ln (1+e))° -
= (Aff)max /pass = µ2R where, e = engineering strain,

� µ = �(Aff)max = � 10 = 0.158 - soo( 1n(1+ 369.66 MPa


R 400 l�O )
e x E = 369.66
42. Ans: 5.6 to 5.8 369.66
e= =1.848 X 10-3
Sol: �Hmax = µ R , 2
µ = 0.1 200x103
At max reduction, a = f3 t
<> =1.848 X 10-3
t
a = f3 = tan- 1 ( µ)= tan- 1 (0.1) = 5.71°
8t = 0.1848 mm

43. Ans: (b) :. tr = 95 + 0.1848 = 95.1848


Ao A ,
Sol: - = 0.4 45. Ans: 1
Ao
Sol: D = 500 mm � R = 250 mm
�Hmax = µ2 Q

� A 1 = 0.6 Ao µ= {Aff /20


� R = f250 = 0.2828
A 1 -A 2
= 0.3
A, 46. Ans: 8.37
A2 Sol: cry = 200 MPa , ho = 25mm , do = 25mm,
� 1-- =0.3
A, true strain = 3.6 = /n(Ar/Ao)
Ar/Ao= e3 · =36. 598 = (dr/do)
2
� A2 = 0.7 A 1
6

(dr/do) = 6.049
A2 = 0.7A1= 0.7 x0.6 xAo= 0.42 Ao
dr = 151.24mm

!IJ•li@OiMjjjjRflftjffim.jj� yderabad I Delhi I Bhopal I Pune I Bhubaneswar I wcknow I Patna I Bengalwu I Chennai I Vijayawada I Vizag I Trrupati I Kukatpal]y I Kolkata
Production
..
ACE
.... . - ..:., hpwmigPnbliclfions
-:,.....
V V
.. • • • •
: 782 :

2 hr = ho- 0.1 ho = 0.9 ho


hr = ( doldr) x ho = 0.683mm
Forging force = cry.Ar
Aoho = Ar hr
200x 7t (151.24)2 do2ho = d/ hr
4
=

=
3592966.884N
Energy required = Force x distance
= 3591145 x (ho -hr) = 87.3 kJ

I l nd cylinder
47. Ans: 1.92 (range 1.90 to 1.94)
Aoho = Athr
Sol: Maximum possible reduction
do2ho = d/ hr
= L1HI pass = µ2 R =
0.082 x 300 =
1.92 mm

48. Ans: (1990 to 2010)


Sol: b = 200 mm, Ho = 10mm, H1 =
6mm,
R = 100 mm, N = 200 rpm
cry = 500 MPa,
L = .JMH = .J100x 4 = 20
For plane strain conditions,
50. Ans: 9.58
Roll separating force = cr0 xbx L Sol: b = 120 mm, H 0 = 8, H1 = 7.2,
= 500 X 200 X 20 = 2000 kN R = 150 , V 1 = 30, p = 200 MPa

49. Ans: 1
Sol: Let, d 1 =
d2 = d
h1 = height of first cylinder = tan-• ( ��;� J =4.186°
h2 = height of second cylinder
assume h, < h2 L e = R a =�MI R 10.951 mm
Let % reduction in height = 10%
=

Area = b x Le = 120 x 10.951 = 1314.1 mm2


1 cylinder
Force = Area x p = 1314.1 x 200
st

h o -h r
=0.1 = 262.82 kN
ho
We know that, power for two rollers
ho-hr = 0.1 ho
P = 2Tro

!Mllii@ih4Y.hi4Rbbiffliii!jj� yderabad I Dellii I Bhopal I Pune I Bhubaneswar I wcknow I Patna I Bengaluru I Chennai I Vijayawada I Vmg I Tirupati I Kukatpally I Kolkata
: 783 : Metal Forming

30
V
ro = = 60 = 3.33 rad/s
R 0.15 Five Marks Solutions
L 10 95
T = F x e = 262.82 x · = 1438.93 Nm 01.
2 2 Sol: Dia. of Rollers = D
Power = 2 x 3.33 x 1438.93=9583.33 W Initial thickness = T = Ho
Power = 9.58 kW Final thickness = T - 2�T = H 1
Coefficient of friction = µ
51. Ans: (b) Max possible reduction = T - (T -2�T)

Sol: L 1 = 20 mm, L2 = 40 mm , L3 = 10 mm,


= 2�T = µ2 R
If D = 300 -R = 150
A1L 1 = A2L 2=A3L3
T = 40, µ = 0.3
A1 L 3
-
150=13.5
=-
A3 LI 2�T = µ 2 R = 0.32 X

Final thickness = T - 2�T = 40- 13.5


True strain - £n( �:) - £n( �: J
· If Vo = inlet velocity = 5 mis
= 26.5

=Rn( ��) = -0.693 V 1 = final velocity = ?

Because the metal forming process 1s the


52. Ans: (b) incompressible process
Sol: �H=10% of 12 = 1.2mm,
Tana = (�HIR)° -5 =(1.2/400)° -5
� a = 3.138

02.
Sol: H0 = 3.5, H 1 = 2.5mm, /)J{ =1
Ha + H 1 = 3mm
D = 250' R =125' H =----"------'-
2
b = 450mm, µ = 0.08
a-Y =140, L = -JR!)J{ = 11.2 mm,

\( I J 111.!,IIH l 1 1111!, P11hlll ,IIIOJI', yderabad I Delhi I Bhopal I Punc I Bhubancswar I Lucknow I Patna I Bcngaluru I Chennai I Vijayawada I Vizag I Tirupari I Kukatpally I Kolkata
"
�,., . . PuNiranooe
ACE . . : 784: Production
., .:"Eo«ior:cnnl
'!.�-

Roll separating force

(or) Favg = 1 cr y (bL{ l + �� ]


:: -1-(1�B l1-(1J ]
Force required for rolling 1 + l .903 �
B

r-
( )[l - ( ) ]
0· 08X l l ,2 ] 1.903 A0
=

� X 1 40X 45 0X 11.2[1 +
v3 4x 3
=

= 8 75.59 kN 1 - ( �: 0. 655 , B - µ cot a - 0.2 cot 6

03. = 1.903
Sol: do = 100mm, h o = 5 0 mm,
hf = 40mm , cr y = 80MPa

df d 100 [so" 111.8mm


v hf
=
1J40
� = =
O

Ao x a -1002 x 80 628 kN d 1 = 0. 755 X 6= 4.5 3 > 1 .3 4


F'; min
,r

4
=

dz = 0. 755 X 4.5 3 = 3 .42 > 1 .3 4


= =

Ff mm. =Af x aY -(111 .8) 2 x 80


,r
d 3 = 0 . 755 X 3 . 42 = 2.582 > 1 .3 4
4
=

(4 = 0. 755 X 2 .5 8 = 1 .95 > 1 .3 4


784.95 kN
ds = 0. 755 X 1 .95 = 1 .472 > 1 .3 4
=

Fi min Ff min c4 = 0. 755 X 1 .472 = 1 .11 < 1 .3 4


Fmin 706.5 kN
+

2
= =
no. ofdraws required
Work done = Fmin x (ho - hf ) = 7065J = 6 draws or passes or stages

= 2xWxH Dia . ofwire after 2 stage = 3 .42mm


nd

7065
H= 0 .3 53 m
2 x10x l03 05.
=

Sol:
04.
Sol: di = 6mm, dr = 1.3 4mm
µ = 0 .2, a = 6° , cr y = 60MPa,
B = µ cot a = 1.903
For maximum reduction
a2
<Y2 = a => _ =1
: 0.2 0.4
ay
Y ---- E

ilflli!@jjji4jjj/iRftftiM\jjjjj� yderabad !Delhi I Bhopal I Pune I Bhubaneswarl u.,cknowl Patna I Bengaluru I Chennai I Vtjayawada j V17.3g I Tirupati I Kukatpally l Kolkata
: 785 : Metal Forming

do = 12 .2 1 4, L0 = 100m
no. of p asses =
Af-[ total

df = 10mm, Lf =? (Aflmax / p ass )


before = 200 MPa, 2 00 - 1 00 100
= = =l O
CY
1

10 10
CY
1
after = 400 MPa,

Ao Lo = Af L r 07.
A Sol: b = 4 00mm, H0 10mm,
Lr = L o =L
2
o
x
Af
(�J
o df H 1 = 8mm,
=

R = 300mm

a = tan -1 �� = tan -1 � � = 4 .67 °


= J Oox(1 � )' =150m
2 14
; 3 0
Roll strip contact length
True strain in the drawing process
= Arc length of Deformation zone .
A d
2

= E = ln o = ln( 0 J = 0.4 = R xa
Ai di
= 300x 4 .67x-
Jr
From the graph 180
CY
1
at E = 0.2 = 24.4 mm
CY = 300 MPa If the condition is assumed as maximum
draft
Y

a = /3 = friction angle
06.
Sol: H0 = 2 00, Hf = lOO, D = 500 mm µ =tan /3 = tan a = tan 4 .67 ° = 0.082
µ = 0.2 , R = 2 50 mm
Aflmax p er p ass = µ 2 R
=0.2 2 x 2 50 =10

lit11iii§jjj4ijjjj$@fti&ijjjjj� ydcrabad I Delhi I Bhopal I Pune I Bhubaneswar I Lucknow I Patna I Bcngaluru I Chennai I Vtjayawada I Vmg I Tirupati I Kukatpally I Kolkata
Sheet Metal Operations
05. In blanking operation the clearance ts
One Mark Questions provided on (GATE-ME-02)
(a) The die
01. For blanking and piercing operations, (b) The punch
clearance is provided on the .. . . .. . . . and the (c) Both die and punch equally
.... ..... ... respectively (GATE-ME-87) (d) Neither the punch nor the die

02 . Wrinkling is a common defect found in 06. In progressive die (for sheet metal), spring
(GATE-ME-91) loaded stripper plate is used to clamps the
(a) Bent components stock until: (GATE-PI-91)
(b) Deep drawn components (a) The punch penetrates twice the stock
(c) Embossed components thickness
(d) Blanked component (b) It removes wrinkles on the product edges
(c) Automatic feeder plate releases it
03. In deep drawing of sheets , the values of (d) Punch is completely withdrawn from the
limiting draw ratio depends on stock
(GATE-ME-94)
(a) Percentage elongation of the sheet 07 . In ......A ..... ....operation the diameter of the
(b) Yield strength of the sheet desired hole will be smaller than the diameter
(c) Type of press used of the .... . . .. .B . .... .... .. .
(d) Thickness of sheet (GATE-PI-91)
A. blanking/ piercing
04. The cutting force in punching & blanking
B. punch/ die
operations mainly depends on
(GATE-ME-01) 08. A cup of 10 cm height and 5 cm diameter is
(a) The modulus of elasticity of the material to be made from a sheet metal of 2mm
(b) The shear strength of the material thickness . The number of deductions
(c) The bulk modulus of the material necessary will be (GATE-PI-91)
(d) The yield strength of the material (a) 1 (b) 2 (c) 3 (d) 4

!1S•lih§Oi44ijjjjRflnjM\11jjj� yderabad I Delhi I Bhopal I Pune I Bhubanesw.ir I Lucknow I Patna I Bengaluru I Chennai I Vijayawada j Vmg I Tuupari I Kukatpa])y I Kolkata
-:,
:.t
:'. .l!E�
. A:CE . .
� IH'flll
1')I! ���========.;.:�;;;,�
�htiariona
��,g�Pu : ========��::,,:;;:;:�::p
����:� 7 87 Sheet Metal O eration s
.. c.=
09. Wall thickness of drawn cup is controlled by 14. For 50% penetration of work material, a
(GATE-PI-92) punch with single shear equal to thickness
(a) Deep drawing (b) Reverse drawing will (GATE-PI-01)
( c) Redrawing (d) Ironing (a) reduce the punch load to half the value
(b) increase the punch load by half the value
10. In a bending operation, if the modulus of
( c) maintain the same punch load
elasticity E is increased keeping all other
( d) reduce the punch load to quarter load
parameters unchanged, the spring back will
(GATE-PI-94) 15. A cup of 10 cm height and 5 cm diameter is
(a) increase to be made from a sheet metal of 2mm
(b) decrease thickness. The number of deductions
( c) remains unchanged necessary will be (GATE-PI-02)
( d)be independent of E (a) one (b) two (c) three (d) four

11. In blanking operation, the best way to


16.
improve the smoothness and squamess of the
Product Manufacturing process
edges is to (GATE-PI-94)
P Food cans 1. Forging
(a) have reduced gap between punch and die
Q Connecting rods 2 . Rolling
(b) increase the ductility of the sheet
R Metal foils 3. Deep drawing
( c) decrease the speed of blanking
4. Extrusion
( d) provide shear on the punch
(GATE-PI-03)
12 . The function of draw bead in the deep A B C D
drawing operation is to (GATE-PI-94) P-3 P-1 P -4 P-2
(a) produce a balance between the amount Q- 1 Q-2 Q-3 Q-4
of stretching and drawing R-2 R-4 R-3 R-1
(b) produce a circular groove on the flange
17. Which one 1s not a method of reducing
( c) reduce the drawing load
cutting forces to prevent the overloading of
( d) reduce the wrinkles on the flange
press (GATE-PI-03)
13. In producing a punched hole of 20mm (a) providing shear on die
diameter, the dimension of the punch is (b) providing shear on punch
. . . . . . .. given the radial die clearance of ( c) increasing the clearance
0.1mm. (GATE-PI-94) ( d) stepping punches

!M•li@hi§§.jjjjjmbftim.jj� _yderabad I Dellii I Bhopal I Pune J Bhubaneswar J Lucknow J PatnaJ Benga)uru J Chennai J Vijayawada J Vizag I Tirupati J Kuk.atpally J Kolkala
: 788 : Production

18. Circular blanks of 10 mm diameter are


punched from an aluminum sheet of 2 mm Two Marks Questions
thickness. The shear strength of aluminum is
80 MPa. The minimum punching force 01. The thickness of the blank needed to
required in kN is (GATE-PI-13) produce, by power spinning a missile cone of
(a) 2.57 (b) 3.29 thickness 1.5mm and half cone angle 30deg
(c) 5.03 (d) 6.33 (GATE-ME-92)
(a) 3.0mm (b) 2.5mm
19. Match the following metal forming processes (c) 2.0mm (d) 1.5mm
with their associated stresses in the Work-
piece (GATE-ME-12 & PI-12) 02. A 50mm diameter disc is to be punched out
from a carbon steel sheet 1.0mm thick. The
Metal forming process dia. of the die should be (Take clearance as
1. Coining 3% of sheet thickness) (GATE-ME-96)
2. Wire Drawing (a) 49.925mm (b) 50.00mm
3. Blanking (c) 50.075mm (d) none
4. Deep Drawing
03. Identify the stress strain in the flange portion
Type of stress of a partially drawn cylindrical cup when
P. Tensile deep drawing without a blank holder
Q. Shear (GATE-ME-99)
R. Tensile and compressive (a) tensile in all three directions
S. Compressive (b) no stress in the flange at all, because
1 2 3 4 1 2 3 4 there is no blank holder
(a) s p Q R (b) p Q s R ( c) tensile stress in one direction and
(c) s p R Q (d) p R Q s compressive stress in other direction
( d) tensile m two directions and
20. The ratio of press force required to punch a compressive in third direction
square hole of 30 mm side in a 1 mm thick
aluminium sheet to that needed to punch a 04. A shell of 100 mm diameter and 100 mm
square hole of 60 mm side in a 2 mm thick height with the comer radius of 0.4 mm is to
aluminium sheet is___(GATE - PI-16) be produced by cup drawing. The required
blank diameter is (GATE-ME-03)

!1D11Q04jjj44ijjdRflbiMmjjj.yderabad I Delhi I Bhopal I Pune I Bhubaneswarl Lucknow! Patna I Benga)uru I Chennai I Vtjayawada IVraag I Tirupati I Kukatpally I Kolkata I
.. ACE
t�• :'.1E�iwn
.. "= ��
Wll�rin�w�Pu
�·� ' �·�========�

hliamma : ========�:;;�:;:����
: :,,�� p����= 7 89 Sheet Metal O erations

(a) 118 mm (b) 161 mm 08. Match the items in column I and II
(c) 224 mm (d) 312 mm (GATE-ME-06)
Column - I Column - II
05. A metal disc of 20 mm diameter is to be P. Wrinkling 1. yield point elongation
punched from a sheet of 2 mm thickness. The Q. Orange peel 2. Anisotropy.
punch and the die clearance is 3%. The R. stretcher strain 3. Large grain size
required punch diameter is S. Earing 4. Insufficient blank
(GATE-ME-03) holding force
(a) 19.88 mm (b) 19.94 mm 5. Fine grain size
(c) 20.06 mm (d) 20.12 mm 6. Excessive blank
holding force
06. 10 mm diameter holes are to be punched in a (a) P-6, Q-3, R-1, S-2 (b) P-4, Q-5, R-6, S-1
steel sheet of 3 mm thickness. Shear strength (c) P-2, Q-5, R-3, S-4 (d) P-4, Q-3, R-1, S-2
of the material is 400 N/mm2 and penetration
is 40%. Shear provided on the punch is 2 09. The force requirement in a blanking
mm. The blanking force during the operation operation of low carbon steel sheet is 5.0 kN.
will be (GATE-ME-04) The thickness of the sheet is't' and diameter
(a) 22.6 kN (b) 37.7 kN of the blanked part is'd'. For the same work
(c) 61.6 kN (d) 94.3 kN material, if the diameter of the blanked part is
increased to 1.5d and thickness is reduced to
07. A 2 mm thick metal sheet is to be bent at an 0.4t, the new blanking force in kN is
angle of one radian with a bend radius of 100 (GATE-ME-07)
mm. If the stretch factor is 0.5, the bend (a) 3.0 (b) 4.5 (c) 5.0 (d) 8.0
allowance is (GATE-ME-05)
10. In the deep drawing of cups, blanks show a
---------+ Ti mm tendency to wrinkle up around the periphery
: 1 radian
(flange). The most likely cause and remedy

' g-
'

·,
'

'
I

I
I
of the phenomenon are respectively,
(GATE-ME-08)
(a) Buckling due to circumferential
' I

'!
'
' I

(a) 99 mm (b) 100 mm compress10n: mcrease blank holder


(c) 101 mm (d) 102 mm pressure

!11•1i@hiMhi4RbblM\hijj� yderabad J Delhi J Bhopal J Pune J Bhubaneswar J Lucknow J Patna J Bengaluru I Cheruiai J Vijayawada I Vizag I Tuupari J KukatpallyJ Kolk ata
: 790: Production

(b) High blank holder pressure and high 13 . The shear strength of a sheet metal 1s
friction: reduce blank holder pressure 3 00MPa . The blanking force required to
and apply lubricant produce a blank of 100 mm diameter from a
(c) High temperature causing increase m 1 .54 mm thick sheet is close to
circumferential length; apply coolant to (GATE-ME-11)
blank (a) 45kN (b) 70kN
(d) Buckling due to circumferential (c) 141kN (d) 3 500kN
compress10n; decrease blank holder
pressure . 14. Calculate the smallest punch diameter that
can be designed for piercing sheet metal strip
Common Data for Question 11 & 12 with the following data. Crushing strength of
In shear cutting operation, a sheet of 5mm the punch material is 1500 MPa . Thickness
thickness is cut along a length of 200mm . the of the sheet is 2 mm, factor of safety is 3, and
cutting blade is 400mm long ( see figure) and shear strength of the sheet material 500 MPa
zero -shear ( S =0 ) is provided on the edge . The 1s (GATE-PI-90)
ultimate shear strength of the sheet is 100 MPa (a) 2mm (b) 6mm
and penetration to thickness ratio is 0.2. Neglect (c) 8mm (d) 24mm
friction (GATE-ME-10)
400 15. A hemispherical cup of radius ' R ' is formed
in a press working operation. The radius of
blank would be (GATE-PI-92)
(a) Ji R (b) Jj R
- - - - - - - - - -- - - - - - - - - - - - - - - - - - - - - - - s _l (c) J½ R (d) Pz R
11. Assuming force Vs displacement curve to be
rectangular, the work done in J is 16. In metal forming operation, the true strain in
(a) 100 (b) 200 (c) 250 (d) 3 00 the X-direction is 0.3 and in the Y-direction
is - 0.1. the true strain in the Z - direction is
12. A shear of 20mm (S 20) is now provides on
the blade . Assuming force Vs displacement
(GATE-PI-94)
(b) - 0.2 (c) 0.2 (d)- 0.03
curve to be trapezoidal, the maximum force
(a) 0

(in kN) exerted is


(a) 5 (b) 1 0 (c) 20 (d) 40

IM•lih§Oi§§@/IAfffliM\iijjj� i
yderabad I Delhi I Bhopal I Pune I Bh�eswar 'Lucknow I Patna I Bengalwu I Chennai I V\iaya"':da Iv'� I Tirupati I K��I. Kolkata
Sheet Metal Operations
ACE
�-�::r..,,,,..� : 791 :
It " • •

'!,
• •

1 7. In sheet _metal working, the spring back 20. It is required to punch a hole of diameter
increase when (GATE-PI-02) 10mm on a sheet of thickness 3mm. The
P: Ratio of bend radius to sheet thickness is shear strength of the work material is
small 420Mpa. The required punch force is
Q: Young's modulus of the sheet is low (GATE-PI-06)
R: Yield strength of the sheet is high (a) 1 9. 78 kN (b) 39. 56 kN
S: Tension is applied during bending (c) 98.9 kN (d) 39 5. 6 kN
(a) P, Q, R (b) Q, R
(c) P, Q, S (d) P, Q, S 2 1 . A cylindrical cup of 48. 5 mm diameter and
52 mm height has a corner radius of 1. 5 mm.
1 8. A hydraulic press is used to produce circular cold rolled steel sheet of thickness 1. 5 mm is
blanks of 10 mm diameter from a sheet of used to produce the cup. Assume trim
2 mm thickness. If the shear strength of the allowance as 2 mm per 2 5 mm of cup
sheet material is 400 N/mm2 , the force diameter. What is the blank size in mm?
required for producing a circular blank is (GATE-PI-04)
(GATE-PI-03) (a) 1 02.0 (b) 1 1 5.4
(a) 8 kN (b) 2 5. 1 3 kN (c) 120. 5 (d) 128. 5
(c) 31.42 kN (d) 1 2 5. 66 kN
22. A hole of 4 0 mm diameter is pierced in a
1 9. Consider the following statements related steel sheet of 4 mm thickness without shear
piercing and blanking (GATE-PI-03) on the tool. Shear strength of steel is 400
1 . Shear on the punch reduces the N/mm 2 and penetration is 2 5%. What is the
maximum cutting force expected percentage load reduction if a shear
2. Shear increases the capacity of the press of 1 mm is provided on the punch?
needed (GATE-PI-04)
3. Shear increases the life of the punch (a) 2 5.00 (b) 33.33
4. Total energy needed to make the cut (c) 50. 00 (d) 66. 67
remains unaltered due to provision of
shear 23. Determine the load required to punch a 20
Which of the above statements are correct mm diameter hole · in 2 mm thick sheet . The
(a) 1 & 2 (b) 1 & 4 properties of the material of the sheet are:
(c) 2 & 3 (d) 3 & 4 (GATE-PI-04)

�ydcrabad i Delhi l Bhopal I Pune I Bhubaneswarl LucknowlPatna l Bcngaluru I Oiennai I Vliayawada j Vm,g ITuupati I Kukatpa)lyj Kolkala I
"•
"
�"
� . ACE
.
�Engiormog
. .
PIJNirJtiooa : 792 : Production

Tensile strength- 580 MPa; (a) 50 min (b) 100 min


Yield strength in tension -410 MPa (c) 166.66 min (d) 333.33 min
Shear strength -350 MPa;
Yield strength in shear -250 MPa 27. A gear with 84 teeth is to be machined using
The load in kN is: a milling process with indexing. The index
(a) 31.40 (b) 43.98 (c) 64.40 (d) 91.10 plate has the following four hole circles: 36,
38, 42, 48. the change gear ratio required is
24. In deep drawing of sheet metal, spring loaded
(GATE-PI-06)
stripper clamps the work until: (GATE-PI-04)
(a) 8/21 (b) 10/21
(a) The punch penetrates twice the stock (c) 21/10 (d) 21/8
thickness
(b) It removes wrinkles on the product edges 28. In a surface broaching process, a broach
( c) Shedder removes the work from the having 5mm pitch and cut per teeth 0.05mm
tools is used for reducing the thickness by 6mm. If
( d) Punch is completely withdrawn from the the length of the work piece is 50mm and the
work material cutting speed is 0.05m/s, the time required I
pass in seconds is (GATE-PI-06)
25. A � 25 mm hole is pierced in a t= 2 .5mm
(a) 11 (b) 12 (c) 13 (d) 1200
thick steel sheet having shear strength ,: =
350 MPa. If the diametral clearance is given
29. Match the items in Group I and II
by the expression c = 0.0064 t /; , the die (GATE-PI-06)
diameter (in mm), punch diameter (in mm) Group - I
and punch force in (k N) respectively are P. Yield point elongation
(GATE-PI-OS) Q. Spring back
(a) 25.0, 25.3, 171.8 (b) 25.0, 24.7, 68.75 R. Wrinkling
(c) 24.7, 25.0, 171.8 (d) 25.3, 25.0, 68.75 S. Residualstresses

26. A work piece of 1000mm length and 300mm


Group - II
width is machined by planning operation
1. Distortion of dimension
with a feed of 0.3mm/stroke. The machine
2. Lauder's bands
tool executes 10 double strokes per minute.
3. Plastic deformation
The planning time for a single pass is
4. Compressive stress
(GATE-PI-06)

l1!11i!i§jjj4ijjjjiAflftj@jjii1� yderahad I Delhi I Bhopal I Punc I Bhubaneswar I Lucknow I Patna I Bcngaluru I Chennai IV\iayawadal Vizag I Tirupati I Kukatpally I Kolkata
J-.t ACE h::t� �O
�e�tal �p� :!·o�n�s=
�.. ,_:�E�· r;dl�:.,.,�11�=======�::,;,��
��rin�ig�Pu�b�ili.�
�·-
793 :
=======�S� eeM eran

(a) P-3, Q-4, R-2, S-1 and die is to be kept as 40microns, the sizes
(b) P-4, Q-4, R-1, S-2 of punch and die should respectively be.
(c) P-2, Q-3, R-4, S-1 (GATE-PI-07)
(d) P-2, Q-1, R-4, S-3 (a) 35 & 35.040 (b) 34.92 & 35
(c) 35 & 35.080 (d) 35.040 & 34.92.
30. It is required to punch a hole of diameter
10mm on a sheet of thickness 3mm. The 34. Match the following (GATE-PI-08)
shear strength of the work material is Group - I Group - II
420MPa. The required punch force is P . Wrinkling 1. Upsetting
(GATE-PI-06) Q. Center burst 2. Deep drawing
(a) 19.78 kN (b) 39.56 kN R. Barrelling 3. Extrusion
(c) 98.9kN (d) 395.6 kN S. Cold shut 4. Closed die forging
(a) P-2, Q- 3 , R-4 , S -1,
Common Data for Question 31 & 32 (b) P - 3, Q - 4 , R -1 , S - 2
A cup is to be drawn to a diameter of 70mm with ( c) P - 2, Q - 3 , R - 1 , S - 4
35mm depth from a 0.5mm thick sheet metal. The (d) P - 2, Q -4 , R- 3 , S -1
cup is drawn in one operation. Assume cru = 430 35. A blank of 50mm diameter is to be sheared
MPa. from a sheet of 2.5mm thickness. The
required radial clearance between the die and
31. The required blank diameter is the punch is 6% of sheet thickness. The
(GATE-PI-06) punch and die diameter (mm) for this
(a) 86.7mm (b) 119.5mm blanking operation respectively are
(c) 121.24mm (d) 169mm (GATE-PI-08)
(a) 50.00 & 50.30 (b) 50.00 & 50.15
32. The maximum drawing force is (c) 49.70 & 50.00 (d) 49.85 & 50.00
(GATE-PI-06)
(a) 0.047MN (b) 0.82MN 36. A disc of 200mm diameter is blanked from a
(c) 0.83MN (d) 9.69MN strip of an aluminum alloy of thickness
3.2mm. The material shear strength to
33. Circular blanks of 35mm diameter are fracture is 150MPa. The blanking force (in
punched from a steel sheet of 2mm thickness. kN ) is (GATE-PI-09)
If the clearance per side between the punch (a) 291 (b) 301 (c) 311 (d) 421

!M11i@jj/4U.jj4jmnftijj.jj�)Hydcrabad I Delhi I Bhopal I Pune I Bhubancswarl Lucknow I Patnal Bengaluru I Chennai I V\iayawada jVizag I Tirupati I Kukatpallyl Kolkata I
.., ,, . AJCE
. .. : 794 : Production
�-�
� �==========================================
'!,�&w,w,q luNirmam

37. Calculate the punch size in mm, for a circular


blanking operation for which details are Five Marks Questions
given below.
Size of the blank 25mm, 01. Determine the maximum possible axial force
Thickness of the sheet 2mm that may be needed to punch a hole of
Radial clearance between punch and die diameter 1 0mm in a metal sheet whose
0.06mm, Die allowance 0.05mm thickness is 2mm and shear strength 200
(GATE-ME&PI-12) MPa. Also determine the diameter of the
(a) 24.83 (b) 24.89 smallest hole that can be safely punched in
(c) 25.01 (d) 25 .1 7 the above steel with a factor of safety of 2.0,
if the compressive strength of the punch
38. A rectangular hole of size 1 00 mm x 50 mm
material is 1 000 MPa (GATE-ME-87)
is to be made on a 5mm thick sheet of steel
having ultimate tensile strength and shear
02. 1 5mm diameter blanks are to be mass
strength of 500 MPa and 300 MPa,
produced from sheet in cold working. The
respectively. The hole is made by punching
thickness of the sheet is 3mm and the
process. Neglecting the effect of clearance,
clearance between the punch and die is
the punching force (in kN) is
0. 3mm. The nominal diameter of the die is
(GATE-ME -14-SET-2)
(GATE-ME-88)
(a) 300 (b) 450 (c) 600 (d) 750

39. In a sheet metal of 2 mm thickness a hole of 03. A cup shown if fig is to be drawn from low
1 0 mm diameter needs to be punched. The carbon steel thickness 0.8mm. The edge
yield strength in tension of the sheet material trimming allowance to be provided is 4 mm
is 1 00 MPa and its ultimate shear strength is on radius. Neglecting comer radius, thinning
80 MPa. The force required to punch the hole & spring back. The diameter of the blank
(in kN) is __ (GATE - 16 - SET - 3) required is ... .... ... (GATE-ME-88)

40. A 1 0 mm deep cylindrical cup with diameter


of 1 5 mm is drawn from a circular blank.
Neglecting the variation in the sheet
thickness, the diameter (upto 2 decimal
points accuracy) of the blank is ___
mm. (GATE - 17 - SET - 1)
\( l l 1 1 1..;II H ( 1 1 [ 1� P1il1iil ,tll1)1J', �ydcrabad I Delhi I Bhopal I Punc I Bhubancswarl Lucknow I Patna I Bcnga)uru I Chcnnai I Vuayawada I Vmg I Tuupati I Kukatpally I Kolkata I
,.,,....,;:· .l!E�Dgll!�· �'"'�ACE
...... 7 95
�· ·�========�: .:,�:;,,: ========
::S:h,::e:e;:;,t ,::M::;e:tal
�O �p;;;
era=ti:o:n:,,
s

"= !'fl[I�· ,g�P1�1�blicanona� =

04. Circular blanks of 50mm in diameter are to operation? The properties of steel are: Yield
be produced from a 3mm thick electrolytic point = 150MPa, UTS = 350MPa, LDR = 1.9
copper sheets for which the resistance to and neglect the entry radius and blank holder
shear is 350Mpa. Calculate the force required force. (GATE-ME-96)
for blanking assuming no shear on the punch.
How will the blanking force change if 08. Estimate the reduction in piercing load for
1

hexagonal blanks of 25mm are produced by a producing circular hole of 50mm diameter in
punch having a shear of 2mm and 50% a 3mm thick steel strip. When the punch was
penetration? (GATE-ME-89) provided with a shear of 1mm. (assuming
30% penetration and shear strength of steel
05. Equal no. of circular blanks of 60 & 100mm as 400 MPa) (GATE-ME-97)
diameter is required for a product line.
Design a strap strip layout which is giving 09. A 5mm thick MS plate is cut in a shearing
highest % utilization of sheet. machine and the length of cut is 500mm. the
(GATE-ME-90) shear strength of the material is 300 MPa.
Find the force required if the cutting blade is
06. Five holes of diameter 10mm each are to be inclined at 1 deg, if the percentage
punched in a sheet 3mm thick at a pitch of penetration is 40%. (GATE-ME-00)
25mm. what should be the minimum
capacity of the press required (in Tonnes) if 10. A contour having a perimeter of 200mm is
the yield point of material is 50 MPa and (a) pierced out from a 3mm sheet having
One hole is punched per stroke, (b) five holes ultimate shear strength of 250MPa. What
are punched in a single stroke (hint no shear will be the amount of shear, if the punch
is provided on the punches) force is to be reduced to 60%. Where it be
(GATE-ME-96) provided? Assume the penetration to be 30%.
(GATE-PI-89)
07. A steel cup of height 30mm and internal
diameter 40mm with a flange of width 10mm 11. A sheet of 1mm thickness and 10mm width
is to be deep drawn from a sheet of 1mm is to be formed by bending as shown in the
thick. Determine the diameter of the blank fig., what should be the length of the blank?
and the drawing force. What is the draw (GATE-PI-90)
ratio? Can the cup be drawn in single

jlJ•lii@ii/§§jjjijjmnjff1nii,j... yderabad I Delhi I Bhopal I Pune I Bhubaneswar I Lucknow I Pama I Bengaluru I Chennai I Vijayawada I Vmg I Tlfllpati I Kukatpal]yj Kolkata
t� ACE : 796: Production
�"....•.,........
....:..-· 'D.�
..,1Wluul;;wuua111A111DWJ

14. The end view of a drawing board clip is


shown in Fig. below. The width of the clip is
20 mm. The length of flat starting strip is
....... .... ... . (GATE-Pl-93)
All Dimensions in mm

RS
R
f
35
:----30----- +
.::C=========i
,
----..iTl
60
lrnm

All dimensions in mm
l_l__--r-_
Tl-s-: 15. A cylindrical cup of diameter 12 mm &
height 16 mm has a comer radius of 0.5 mm,
it is made out of a sheet of 2 mm thick & the
BENT SHAPE

12. A cup of 50mm diameter & 75 mm deep is to


material has a constant flow of 35 MPa.
be drawn from 1.5 mm thick steel sheet of
Neglecting the comer radius,
deep drawing quality of tensile strength 300
(GATE-PI-94)
N/mm2• Neglecting the comer radius '
i) The diameter of the blank would be
i) The diameter of the blank required is .....
ii) Force and energy must be applied for
(a) 20 mm (b) 30.2 mm
first draw with 40 % reduction is .....
(c) 36.1 mm (d) 41.6 mm
(GATE-Pl-91)
ii) The drawing stress based on idle
13. A blank of 200 mm diameter is mass­
deformation (neglecting friction, blank
produced from a sheet of 4 mm thick with
holding force & effect of punch comer
ultimate shear strength of 30 MPa, shear
radii) is ........... .. . ..
provided on one of the tools is 1mm.
(a) 52 MPa (b) 68 MPa
(GATE-PI-92)
(c) 98 MPa (d) 106 MPa
i) The punch force required assuming
iii) The number of draws required to draw
punch penetration as 35 % is
the part if the draw reduction ratio for
(a) 34 kN . (b) 44 kN
first and subsequent draws is 40% and
(c) 54 kN (d) 64 kN
25% is ......
ii) The shear is provided on
(a) 2 (b) 3 (c) 4 (d) 5
(a) Punch (b) die (c) both (d) any

liiiiil@llti§IIIQRflijfM)lfill�
� yderabadlDelhi Bhopal I Pune I Bhubaneswar I Lucknow I Patna I Bengaluru I Chennai I Vijayawada I Vu.ag I Tlfl.lpati I Kukatpally I Kolkata
: 797: Sheet Metal Operations

16. The strain in the cup extrusion for the part 17. The part shown in fig is to be blanked from a
shown in fig is (GATE-PI-95) steel strip of 2 mm thickness. Assuming the
penetration to be 40 % & ultimate shear
strength of the material to 145 MPa,
(GATE-PI-95)
�1
100mm

50mm

45 deg

(a) /n [Db/ Dp] (b) /n [DJ(Db- Dp)]


80mm

i) The force required for blanking


(c) In [Dp/(Db- Dp)] (d) Rn ln� !(D� -D! )]
operation would be .. ... .. . .... ... .
ii) Work done in blanking operation would
be ... . .... ... . .. ..
iii) The shear to be provided to carry out this
operation on a 24 kN press is . . .. . . .

li1•1ii@Oii§.jjQ/jflnj@j11jjj� yderabad I Delhi I Bhopal I Pune I Bhubaneswar I Lucknow! Patna I Bengaluru I Chennai I Vijayawada I Vizag I Tuupati I Kukatpally I Kolkata
SOLUTIONS
punch only, where as in piercing hole size is
One Mark Solutions made equal to punch size and clearance
provided on the die only.
0 1.
Sol: PUNCH and DIE , In blanking the size of 06. Ans: (d)
Blank is equal to size of die and clearance is Sol: Other wise during withdrawal of punch the
provided on the punch only, where as in drawn part may stick to the punch surface
piercing hole size is made equal to punch and it may become difficult to remove the
size and clearance provided on the die only. drawn part.

07. PIERCING, DIE


02. Ans: (b)
Sol: In deep drawn components when the force 08. Ans: (b)
is applied by using a punch, the edges of Sol: h = IO cm, d = 5 cm, t = 2 mm
blank will be lifted up words and produces
D = -Jd 2 + 4dh = -J5 2 + 4 x 5 x 10 = 15 cm
folding on the Blank called as wrinkles.
Because nothing is given in the Problem.
Assume draw ratio method with LDR = 2
03. Ans: (b)
D 15
Sol: In deep drawing forces are applied on the di =-- = -=7.5 >5
L.D.R 2
sheet such that stresses induced in the sheet
75
is greater than yield and less than ultimate d2 = · = 3.75 < 5
2
stress.
No. of draws required = 2

04. Ans: (b)


09. Ans: (d)
Sol: Fmax = p x t x ru
Sol: In deep drawing operation, if the clearance
Where 'ru = Shear strength of material between punch and die is slightly less than
sheet thickness. The wall thickness of cup
05. Ans: (b) can be controlled and it is called as ironing
Sol: In blanking the size of Blank is equal to size effect.
of die and clearance is provided on the
llflll@i@jjj4§Jiii4RffC@mjj,� yderabad I Delhi I Bhopal I Pune I Bhubaneswarf Lucknow! Patna! Bengaluru I Chennai I Vijayawada IVizag ITirupati I Kukatpally I Kolkata
t .
�'!.�•�=
A:CE.. et Mettl Operano ns
:�JE�qm�ftl�rm�'l�Pl�i�6�::aa�-:=":•========�: 7�9�9:::�========�S::;h:e: ��:� ���· �
10. Ans: (b) D 15
di=--=-=7.5>5
Sol: The spring back i n bending operation is L.D.R 2
Inversely proportional to the modulus 75
d2= · =3.75<5
of elasticity. 2
No. of draws required=2
1 1. Ans: (a)
16. Ans: (a)
12. Ans: (d) Sol: Connecting rods are always produced by
Sol: By providing draw bead in he deep t
forging operation Q - 1 --+ 'A'.
drawing operation, the BHF can be reduced,
wrinkling can be eliminated a nd Earing 17. Ans: (c)
defect can be minimized. Sol: By increasing clearance between punch a nd
die the force required cannot be reduced.
13.
Sol: H.S=20 mm=hole size 1 8. Ans: (c)
Clearance=C=0.1 Sol:D = 10,A=2, tu=80MPa
P.S=H.S=20 F= 1t d t tu=1tx 10x2x 80
D.S=P.S+2C=20+2 x 0.1=20.2.
=5024=5.03 kN

14. NO ANSWER,
19. Ans: (a)
Sol: K=50%, I =t
Fmax Kt Fmax x0.5t F
F= = = max 20. Ans: (0.25)
Kt+I 0.5t + t 3
From the given choices, there is no a nswer.

15. Ans: (b) X1t1 30xl


= = =_!_=0.25
X 2 t2 60x2 4
Sol: h = 10 cm, d =5 cm, t=2 mm
D=..Jd 2 +4dh=.J5 2 +4x5x10
= 15 cm
Because nothing is given in the problem.
Assume draw ratio method
with limiting draw ratio (LDR)=2

Fyderabadi Delhi I Bhopal I Pune I Bhubaneswarl Lucknow I l'alna I Bengaluru I Oiennai IVuayawadal Vizag ITU11pari I Kukatpallyj Kolkata I
: 800: Production

05. Ans: (a)


Two Marks Solutions Sol: Disc diameter = Blank size = 20 mm
Blanking
01. Ans: (a) t = 2 mm
Cone Thickness C = radial clearance = 3% t
Sol: Thickness =
cos (2a) 3
=-x2 = 0.06mm
1.5 100
= =3 mm
cos 2x30 In Blanking
D.S B.S = 20
02. Ans: (b)
=

P.S = D.S - 2C = 20 - 2 x 0.06


Sol: A disc can be produced by Blanking = 20 -0.12 = 19.88 mm
operation and in Blanking die size is made
equal to blank size 06. Ans: (b)
Die size = Blank size = 50 mm Sol: d = 10 mm, t = 3 mm, r,, = 400 Mpa,
Punch size = Die size - 2 x C K = 40 %, I = 2mm
3 Fmax = ff d t r u = ff x10 x3x400
= 50 -2x-xl = 49.94 mm
100
= 37.68 kN
03. Ans: (d) Force required with provision of shear
Sol: When the force is applied on the blank Kt 37.68x0.4x3
F= =
without BHF, the edges of blank are lifting Kt +I 0.4 x3 +2
Fmax

upwards and produces wrinkles. So that on = 14.13 kN


outer surface of wrinkle tensile stress, inner Because the force required with provision of
surface of wrinkle compressive stress and shear is not there in the given multiple
along the blank tensile stress will be choices, the answer can be taken without
induced. provision of shear.

04. Ans: (c) 07. Ans: (c)


Sol: d = 100 mm, h = 100 mm Sol: t = 2 mm, 0 = 1 rad, R = 100 mm,
d l 00 Stretch factor = K = 0.5
r = 0.4, = = 250 >20
r 0.4 Bend allowance = B = rn x 0
� Corner radius is neglected
= (r +kt ) X 0 = (100 + 0. 5 X 2) X 1
D =�d 2 +4 dh = 223.6mm = 224mm = 101 mm
\( I I 11g11H( 1111g P11hlu.111c,1h yderabad J Delhi I Bhopal J Pune J Bhubaneswar I Lucknow I Patna J Bengaluru I Chennai J Vtjayawada I Vizag J Tirupati I Kukatpally J Kolkata
: 801: Sheet Metal Operations

08. Ans:(d) Shear provided over a length of


20
09. Ans:(a)
200mm�-x200 =
10 mm
400
Sol: Fmax = 5 kN, t = Thickness of sheet, Fmax Kt =
F(Kt +I)
D =
diameter of Blank =
1td t 'tu 100xl0 3 x0.2x5
F= =9.09=10 kN
If d 1 =
1.5 d 0.2x5+10

13. Ans:(c)
t1 =
0.4t
Fmax = 5 = mi f 'l'u Sol: 'u =300 MPa , D =
100 mm,
5
'l'u T =
1.54 mm
Blanking force =Fmax=mi t ,u
=-
mit
5
Fmax
l
= tr d 1 I
f 'u = 1t X 1 . 5d X O . 4 t X- =tr X 100 X 1.54 X 300= 145 kN.
1tdt
=1.5 X 0.4 X 5=3 kN. 14. Ans:(c)
Sol: acp =1500 MPa, T = 2 mm , FOS=3
10. Ans:(a)
Sol: During deep drawing when the force is allowable =- -=500 MPa
1500
acp
3
applied by using punch on to the sheet, the
'tu =
500 MPa.
4( 'u
edges of blank is lifting upwards, produces
dmm. =
acp allowable
wrinkles and it can be eliminated by
applying BHF or increasing BHF.
4 X 2 X 500
= =8 mm
11. Ans:(a)
500

15. Ans:(a)
12. Ans:(b) Sol: One side surface area of blank= One side
Sol: Common Solution for 11 & 12 surface area of part
t 5 mm, L = 200 mm, 'tu 100 MPa,
= = tr 4 tr R 2
D 2=
K = 0.2 4 2
D2 = 8 R 2
Work done =
Fmax Kt =
Lxt x 'tu K.t
x

=
200 X 5 X 100 X0.2 X 5 D=2.fi R

100x103 Radius of Blank D = Ji R =


= =lOON -m(or)J 2
100

jltll$i@jji§g.jji4j;neftM\,@� yderabad I Delhi I Bhopal I Pune I Bhubaneswar I Lucknow I Patna I Bengaluru I Chennai I Vtjayawada I Vizag I Tirupari I Kukatpally I Kolkata
� :�P.ffcaf.wn
...,.... .... ACE : 802: Production
� �=====================================
16. Ans: (c) d48·5
= =32.33 > 20
Sol: E =E
x y
+ Ez r 1.5
Comer radius is neglected
0.3 =0.1+ E :::>E 2 =0.3-0.1= 0.2
.J
2

D = d 2 + 4dh = 111.53 mm

T.A =- x48.5 =3.88 mm.


17. Ans: (b) 2
25
Sol: Spring back = R r ,
R; D total = D + 2 x Trimming allowance
= 111.53 + 2 X 3.88
Spring back oc R
t =
115.4 mm.
3

22. Ans: (c)


=4(�J -3(.!.)+1
E.K tE
Sol: d = 40 mm, --+ piercing
Spring back oc aY
t = 4 mm, tu= 400 MPa , K 25%,
1
=

oc- I= 1 mm.
E.
Fmax Kt= Fmax x0.25x4
Kt +/
F=
18. Ans: (b) 0.25 x4 + 1
Sol: d = 10 mm = B.S. Fmax
= =50% .
t = 2mm, 'tu = 400 MPa.
F max = 1tdt'tu
1tXI0x2x400 = 25.12 kN. 23. Ans: (b)
Sol: d = 20 mm, t = 2 mm, 'tu = 350 MPa
=

19. Ans: (b) F max = 1tdt'tu


=,rx20x2 x350 =43.96 kN.
20. Ans: (b)
Sol: d = 10 mm, t = 3 mm, 'tu= 420 MPa
24. Ans: (d)
F max = 1tdt'tu= 7tX 10x3x420 = 39.56 kN
Sol: Other wise during withdrawal of punch the

21. Ans: (b)


drawn part may stick to the punch surface

Sol: d = 48.5, h = 52 mm, r = 1.5 mm,


and it may become difficult to remove the

t = 1.5 mm drawn part.


T.A = 2 mm per 25 mm of diameter
\( I I 11..:111t t I !Ill._ P11Ll1( ti!• I]!',
�yd
erabad I Delhi I Bhopal! Pune I Bhubancswarl Lucknow! Patna.I Bcngaluru I OicnnailVoayawadajViug ITirupati I Kukatpal)y I Kolkata I
'!. : 803: Sheet Metal Operations
.:' Eoginecriug Publiariooa
ACE
�-�
� �===================================
25. Ans: (d) Cutting length of Broach = 120 x 5
Sol: d = 25 mm, t = 2.5 mm ---+ piercing = 600 mm.
Tu = 350 MPa L = / +cutting length = 50 + 600 = 650
Diameter clearance = C = 0.0064 Kf; L 650
Time/Cut = - = ---- = 13 sec
V 0.05x1000
= 0.0064 X 2.5.fiso
0.3 mm
29. Ans: (a)
In piercing
=

30. Ans: (b)


P.S = H.S = 25 mm.
Sol: d = 10 mm, t = 3 mm, Tu = 420 MPa
D.S P.S + C 25 + 0.3 = 25.3
Fmax = ndt'tu = TCx10x3x420
= =

Fmax = ndt r u = TC x 25 x 2.5 x350


= 39.56 kN.
= 68.7 kN
31. Ans: (c)
26. Ans: (b) 32. Ans: (a)
B
Sol: Time/Cut = x� x (1 +M) Sol: Common Solution for 31 & 32
f V
d = 70 mm, h = 35 mm, t = 0.5,
B 1
Tu = 430 MPa
j No. of D.S I min
=-X------

cry= 0.5xcru = 215 MPa.


300 1
=-x- = 100 mm.
0.3 10 D = -Jd 2 + 4dh

= -J70 2 +4x70x35 121.2 mm


27. Ans: (b)
=

Drawing force = nDt Tu


Sol: N = 84 = TC X 121.2 X 0.5 X 215
40 10
C.R = - = - , after every cut the crank 0.041 MN.
84 21
=

has to rotate 1Oholes in 21 holes circle.


33. Ans: (b)
Sol: Blank diameter = 35 mm---+ Blanking
28. Ans: (c)
t = 2 mm., C = 0.04 mm, per side
Sol: p = 5 mm, h = 0.05 mm
Because of Blanking operation
Reduction in thickness = 6 mm,
Die size = Blank size = 35 mm
l = 50 mm, V = 0.05 mis.
Punch size = Die size - 2xC = 35 - 2x0.04
6
No. of teeth required = - - = 120 35 -0.08 = 34.92 mm.
0.05
=

iffl•li@jji44ijjjj:maftj1nil4� yderabad I Delhi I Bhopal I Pune I Bhubaneswar I Lucknow I Patna I Bengaluru I Chennai I Vtjayawadal Vmg I TlfUpari I Kukatpalfy j Kolkata
: 804: Production
� Pnblicdima
...... ,...... ACE
.... ,"�Ii'....:..-' ...............

34. Ans: (c) Given is the punching operation


Fmax = force required for punching
35. Ans: (c) = perimeter x t x 'tu
Sol: Blank diameter = 50 mm, ---+ Blanking = 2(100+50)x5x 300 = 450 kN
t = 2.5 mm

C = 6 % t = -x 2.5 = 0. 15mm 39. Ans: 5.024 (range 4.9 to 5.1)


100
6
Sol: Punching force = Fmax = n:dttu
In Blanking
n: x 1Ox 2x 80
D.S = B.S = 50 mm
=

5024 = 5.024 kN
P.S = D.S -2 x C 50 -2 x 0.15
=
=

49.7 mm
40. Ans: 28.7228
=

Sol: h = 10 mm, d = 15 mm,


36. Ans: (b)
D = .Jd 2 + 4dh = .Jl 52 + 4xl 0x15
Sol: Blank diameter = 200 mm ---+ Blanking
t = 3.2 mm, 'u = 150 MPa = .J825 = 28.7228 mm

Blanking force = n:d t t u


= 1C X 200 X 3.2 X 150
= 30 1.44 kN.

37. Ans: (a)


Sol: B.S = 25, t = 2mm, c = 0.06mm
Die size = Blank size -Die allowance
= 25 - 0.05 = 24.95
Punch size = Die size -0.06 x 2 = 24.83

38. Ans: (b)


Sol: Hole = 100 x50mm (Rectangle),
Thickness = t = 5mm, CTut = 500MPa,
'tu = 300MPa

l11••iii§ida§.jj/4jmftj@jj@� yderabad J Delhi J Bhopal J Pune I Bhubaneswar J Lucknow I Patna J Bengaluru J Chennai J Vrjayawada I Vizag I T11Upati I Kukatpally I Kolkata
'!.�-�:�Pnbtiarioos
. A:,CE . . : 805: Sheet Metal Operations
�- �;,,;,;===��===================================
03.
Five Marks Solutions Sol: t = 0.8 ,
Trimming allowance = T.A

01. = 4mm on radius


Sol: Hole size = 10 mm, t = 2mm , d 1 = 40 mm, h = 30
d2 = 40 + 10 + 10 = 60 mm
Tu = 200 MPa, FOS on punch = 2.0
Neglecting Comer radius.
= 1000 MPa.
Blank dia. = D = �d; +4d 1 h
a-ep

1000
allowable = _ P = -- = 500 = .J60 2 +4x40x30
Fos 2
(Ye
a-ep

91.65 mm
Force Required = F,nsx = mi. t Tu
=

Dtotal = D +2 X T.A = 9 1.65 +2 X 4


= Jrxl 0x2x200 12.56 kN = 99.65 mm
=

mi. t ru
= =
Aep 04.
Fmax
a-ep

Sol: d = 50 mm (Blank), t = 3mm ,


JZ' d2

4t T u Tu = 350 MPa,
O'cp = --
d Fmax = mi. t ru = Jr x50 x3x350
4 tTu 4 X 2 X 200 = 164.8 kN
. =
d mm =
cr cp allowable 500
For hexagonal Blank of side 25 mm
= 3.2 mm Fmax = 6x25x3x350 = 157.5 kN
If I = 2 mm, K = 50 %
02.
F x Kt
Sol: Dia. of Blank d 15 mm, t = 3mm , F = ma
Kt -I
= =

C = 0.3 mm on radius. Reduction in blanking force


In Blanking operation die size is made equal 157.5 -67.5
= = 57%
to Blank size 157.5

Die size = B.S = 15 mm


05.
Punch size = D.S - 2 x C Sol: Based on design of different layouts,
= 1 5 - 2 x 0.3 = 1 4.4 mm maximum % utilization with optimum lay
out 73%

!M•li@OiMjjjjjmn!M\@j� yderabad I Delhi I Bhopal I Pune I Bhubaneswar I Lucknow I Patna I Benga)uru I Chennai I Vijayawada I Viz.ag I Tuupati I Kukatpally I Kolkata
�.....
,., ,....:F.ugiomiog
ACE
Puhlic:atiooa : 806: Production
..
'====================================
06. D
LDR = l .9 = -
Sol: 5 holes of dia. = 10 mm each, d,
t = 3mm , Pitch = 25 mm 94 25
dI = · = 49.6 >30
50 1.9
cry = 50 Mpa => cru = - = 100 Mpa
0.5 49 · 6
d2 = = 26.1 < 30
= 0.6x <:Yu 60 MPa 1.9
No. of draws = 2
ru =

(a) If one hole is punched p er stroke


Force Req. = Fmax = nx10x3x60
08.
= 5.652 kN Sol: d = 50 mm, t = 3mm , I = 1 mm,
(b) All five holes are punched together
K = 30%, = 400 MPa
Fmsx = 5 X 5.652 = 28.26 kN
ru

Fmax = mi t r" = n x50 x3x 400


= 188.4 kN.
07.
F(Kt + I ) = F max Kt
Sol: h = 30 mm
188.4 x0.3x3
d1 internal dia + 2t = 40+2xt F= = 89.2 kN.
0.3 X 3 +1
=

42 = outside diameter
Reduction in force = 188.4 - 89.2
=

Flange width = 10 mm
98.9 kN
d2 = d 1 + 2x10 = 42 + 2xlO = 62
=

D = �d; +4d, h = �622 +4 x 42x30


09.
= 94.25 mm Sol:
Neglecting BHF
Drawing force for 1st draw

F, - 1r d, to-, (�-CJ 500 ---


When C = O
F; = n D tCYY = nx94.25xlx150 t = 5 mm,
Length of cut = perimeter = 500 mm,
44.4 kN.
Tu = 300 MPa,
=

. Diameter before the draw


Draw ratio = --------- Shear inclination = 1°, K = 40%
Diameter after the draw
Given that LDR = 1.9 Fmax = p X t X Tu = 500 X 5 X 300 = 750 kN

IM•IQ@idMhAMflfliMJ,jjjj� yderabad I Delhi I Bhopal I Pune I Bhubaneswar I Lucknow I Patna I Bengaluru I Chennai I Vrjayawada I VIZag I Tuupati I Kukatpally I Kolkata
: 807 : Sheet Metal Operations

12.
tan 1°=_!_
500 Sol: d = 50 mm, h = 75 mm, t= 1.5 mm,
I = 500 tan 1° = 8.73 mm UTS = 300 MPa = au
Force required with shear Neglecting corner radius.
Fmax Kt 750 x 0.4 x 5
F= = =l 39_ 8 kN Blank diameter=..Jd 2 +4dh
Kt+I 0.4 x 5+8.73
=..J50 2 +4 x 50 x 75 = 132.3 mm

=> di D - 0.4 D=0.6D


10. D - d1
D
0.4= =
Sol: p = 200 mm, 'tu = 250 MPa
K= 30%
= 79.4 mm
F; =re D t aY
T = 3 mm, F = 0.6 Fmax,
Drawing force for 1st draw
F(Kt+I) = Fmax Kt
=re X 132.3 X 1.5 X 0.5 X au=93.5 kN
Fma Kt
I= x -Kt
D=�d f +4d 1 h 1
Kt
- - Kt=-- - 0.3 x 3
0.3 x 3
= = 0 . 6 mm 4d 1 h 1 =D 2 - df
0.6 0.6
Shear = 0.6 mm provided on the punch D 2 - d f 132.32 - 79.4 2
h 1= = =32. 3 mm
because it is a punching operation. 4d 1 4 x 79.4
Energy required = F 1 xh 1
11. = 93.5 x l Q 3 x 32.3 x l Q-3 = 3 kJ

Sol: t= 1 mm, w = 10 mm
13.
Length of Blank = Lo
= 20+30+5+B 1 +B2
Sol: d = 200 mm, t= 4 mm, 'tu=30 MPa
! = shear = 1 mm K= 35%
re d t
=(r+0.5 x t) x (1 80 - 60 ) x _!!_
(i) Fmax = Tu=re x 200 x 4 x 30
1 80 = 75.36 kN

=(5+0.5 X l ) x 120 X _!!_ Fmax Kt


1 80 F=
Kt+I
=1 1.51 mm
75.36 x 0.35 x 4
= = 43.96 kN
B2 = rn x 0=(5+0.5 x l) x 1 80 x _!!_ 0.35 x 4+1
1 80 (ii) The shear is provided on die because
= 17.27 mm
the process is Blanking operation.
L0 = 55+ 1 1.51 + 17.27 = 83.7 8 mm.

jlfl•lj@jjjl41h4AflbiM\@lfjfyderabad I Delhi I Bhopal I Punc I Bhubaneswarl Lucknow I Patna I Bcngaluru I Chcnnai I Vrjayawada I V17.3g I Tirupati I Kukatpally l Kolkata j
. ,t\CE . .
.:-'!...t &ip,,mngPIJNiramna : 808 : Production
� '======================================
:

14. 16.
Sol: The developed length
Sol: Stram = i:.,n
.
0 ( ------ ]
original C.S. area
= length of original flat starting strip Final C.S. area
= 60 + 35 +38 + 2B 1 + B2 +B3

= 133 + 2B, + B2 + B3

= 133 + 2 x 3.925 + 2.62 + 8.635

= 152.1 mm

B1 01 =(2 + 0.5xl)x90 x� =3.925 mm


180
= r n1

17.
B2 =rn 2 X 02 =(2 + 0.5 X 1)6ox ; 2.62 mm Sol:
1 0
=
1 00

B3 x03 =(5 + 0.5xl)x90 x ; = 8.635 mm


1 0
= rn 3

30
15. 50

Sol: d = 12 neglect corner radius


D =-Jd 2 + 4dh = -Jl 2 2 + 4 x12 x 16 20.fi 20
I

��:
I

= 30.2 mm ,....__ __
80
h = 16 mm, r = 0.5 mm, t = 2 mm

t = 2 mm, K = 40% , 'tu = 145 MPa.


cry = 35 MPa.
F 1 = drawing force for 1st draw = nDtcry
Perimeter(p) = 80 + 50 +100 + 30 + 20.J2
= 1r x30.2 x 2 x35 = 6.64 kN
= 288.28 mm

Fmax =p x A x'u = 288.28 x 2 x 145


cr21 = Drawing stress in zone 2 of 1st draw.
= ------- 83.6 kN
: (d12 - (di -2t)2 )
=

Work done = Energy Required


6.64xl0 3 E = Fmax X Kt = 83.6 X 0.4 X 2
= ------ = 65.6 MPa
Jr (18.12 -14.12 )
2 2 = 66.88 J only
4 Fmax Kt -
D - d1 I= Kt
D
D R RI = = 0.4
83.6 x 0.4 x 2 -
d 1 = 0.6 D = 18.12 > 12 = 0.4 x 2
24
d2 = 0.75 d, = 13.6 > 12
= 1.98 ::::: 2mm
d3 = 0.75 X d2 = 10.2 < 12
No. of draws = 3.
!lfl•Qjj§jjji4ijjjjRbfiiMiiiih� yderabad I Dellii I Bhopal I Pune I Bhubaneswar I Lucknow I Patna I Bengaluru I Chennai I Vtjayawada J Vu.ag I Tirupati I Kukatpa)ly I Kolkata
C7 Metro logy
(GATE-ME-95)
One Mark Questions
06. The fit on a hole-shaft pair system is
specified as H7-s6. the type of fit is
0.020
The hole 40 +o .ooo & shaft , 40 -
+-0.0 1 0

(GATE-ME-96)
01. 0 010
· when
assembled will result in
(GATE-ME-87)
(a) Clearance fit
(b) running fit ( sliding fit )
(a) clearance fit (b) Interference fit
(c) Push fit(transition fit)
(c) transition fit (d) drive fit
(d) force fit ( interference fit )
02. In an engineering drawing one finds the
designation of 20G7f8. the position of 0 8. Allowance in limits and fits refers to
tolerance of the hole is indicated by (GATE:-ME-01)
(GATE-ME-88) (a) Maximum clearance between the shaft
(a) Letter G (b) Letter f and hole
(c) Number 7 (d) Number 8 (b) Minimum clearance between the shaft
and hole
03. Appropriate instrument to check the flatness (c) Difference between the maximum and
of slip gage is (GATE-ME-89) minimum size of hole
(a) dial indicator (d) Difference between the maximum and
(b) Pneumatic comparator minimum size of shaft.
(c) Optical interferometer
(d) Tool makers microscope with projection 09. In order to have interference fit, it is
facility essential that the lower limit of shaft should
be (GATE-ME-OS)
04. Slip gauges are calibrated by outside
(GATE-ME-95)
(a) Greater than the upper limit of the hole
micrometer (T/F) (b) Lesser than the upper limit of the hole
(c) Greater than the lower limit of the hole
05. Checking the diameter of a hole using GO
(d) Lesser than the lower limit of the hole
and NOGO gages is an example of
inspection by . . . . . . . . . (Variable / attributes )

!mli@hiiOiihiRHIMiiiih� yderabad I Delhi I Bhopal I Pune I Bhubaneswarl Lucknow I Patna I Bengaluru I Chennai I V\iayawadalVizag I Tll1lpari I Kukatpally I Kolkata
: 810: Production

(GATE-ME-06) (GATE-PI-91)
1 0. Ring gage is used to measure 1 4. Outside micrometer is calibrated with the
help of
(a) outside diameter but not roundness (a) inside micrometer
(b) roundness but not outside diameter (b) depth micrometer
(c) both outside diameter and roundness (c) ring gages (d) slip gages
(d) only external threads.
1 5 . Abbe's principle of alignment states that the
+0.0 1 5

(GATE-PI-91)
scale axis must coincide with the line of
1 1 . A hole is of dimension <j>9+0 mm. The
corresponding shaft 1s of dimension
+0.0 1 0 1 6. In mass manufacturing, two holes in a plate
<I> 9 +0·001 mm. The resulting assembly has
(GATE-ME-1 1) (GATE-PI-92)
type of component are ideally located with
the help of
(a) loose running fit (b) close running fit (a) one solid pin in the larger hole and one
(c) transition fit (d) interference fit diamond pin in the smaller hole
(b) two solid pins in either holes

(GATE-PI-89)
1 2. Flatness of slip gages is checked with (c) two diamond pins in either holes
(d) one solid pin in the smaller hole and one
(a) Interferometer diamond pin in the larger hole
(b) Optical flat
(c) Electronic comparator 1 7. The skid in a stylus type roughness
(d) Linear measuring machine measuring instrument is used to provide
. . . .. A. . . . . . . . . and define . . . . . . B . . . . . . . .

(GATE-PI-92)
1 3 . For angle measurement in metrology, the geometrical profile of the surface.

(GATE-PI-91)
following pair can be used in conjunction
A. support I datum
with each other
B. micro I macro
(a) sine bar and Vernier calipers
(b) bevel protractor and slip gages 1 8. The diameter of the best wire to measure the
(c) slip gages and sine bar effective diameter of a metric screw threads

(GATE-PI-92)
(d) sine bar and bevel protractor Of 0.5 mm pitch is . . . . . . . . . mm

!lfl1ii@jj44ijjjiRflbiMhiHl*Z1ydcrabad I Delhi I Bhopal I Punc I Bhubancswarj LucknowI Patna I Bcngaluru I Chennai I Vuayawada j Vizag I Tuupali I Kukatpally I Kolkala I
: 81 1 : Metrology

19. In an interference microscope , a groove 24. Powder metallurgical components have


produces a band distortion of 4 band (GATE-PI-92)
spacing. If the wave length of the (a) same density as that of cast products
monochromatic light source is 0.5microns, (b) higher density than cast products
the groove depth is . .. . . .... . . mm (c) lower density than cast products
(GATE-PI-92) (d) higher density than forged products

20. A shaft of diameter 20 -0· 1 5 and a hole of


+0.05

25. The combination of slip gauges to obtain a


diameter 2o +o. i o mm when assembled dimension of 10.35 mm will be
+0.20

would yield. (GATE-PI-92) (GATE-PI-94)


(a) Transition fit (b) Interference fit (a) 10.00 + 0.30 + 0.05
(c) Clearance fit (d) None (b) 8.00 + 1.30 + 1.05
(c) 10.00 + 0.35
21. A straight edge is inspected on a coordinate (d) 5.00 + 4.00 + 1.00 + 0.35
measuring machine. The readings obtained
at intervals of 100mm along the top edge 26. Abbe's principle of alignment 1s used m
are 100.01, 100.05, and 100.04 mm the measurements made in . . . . . ..
straightness error, making the end points as (GATE-PI-94)
reference, is . . . mm (GATE-PI-92)

27. Abbe's principle of alignment is followed


22. The fit on a hole-shaft pair system is
m: (GATE-PI-95)
specified as H7-s6. the type of fit is
(a) Vernier calipers
(GATE-PI-92)
(b) depth vernier
(a) Clearance fit
(c) internal caliper micrometer
(b) running fit ( sliding fit )
(d) height vernier
(c) Push fit(transition fit)
(d) force fit ( interference fit )
28. Instrument Principle of inspection
23. The most widely used reinforcement m P. Dial indicator 1. Non contact
modern day FRP tennis racket is Q. Pneumatic gage 2. Limit of size
(GATE-PI-92) R.GO I NO GO gage
(a) glass (b) carbon
3 . Comparator
(GATE-PI-03)
(c) aluminum (d) magnesium

!IS11i@jjji4.jjQjmftlM\11j\j� yderabad I Delhi I Bhopal I Pune I Bhubaneswari Lucknow I Patna I Bengaluru I Chennail Vtjayawada l Vmg I TU11pati I Kukatpally j Kolkata
-:.:.t
..
ACE
�Eopieamg Pnbtiatinns : 812 : Production

p Q R p Q R 33. A metric thread of pitch 2 and thread


(a) 2 3 1 (b) 3 1 2
mm

angle 60 ° is inspected for its pitch diameter


(c) 1 2 3 (d) 2 1 3 using 3-wire method. The diameter of the
best size wire in mm is (GATE-ME-13)
29. Ring gage is used to measure
(a) 0.866 (b) 1.000
(GATE-PI-OS)
(c) 1.154 (d) 2.000
(a) outside diameter but not roundness
(b) roundness but not outside diameter
34. The flatness of a machine bed can be
(c) both outside diameter and roundness
measured using (GATE-ME- 14-SET-2)
(d) only external threads.
(a) Vernier calipers
(b) Auto collimator
30. The geometric tolerance that does not need a
(c) Height gauge
datum for its specification is
(d) Tool maker's microscope
(GATE-PI-07)
(a) Concentricity (b) Run out
35. Which one of the following instruments is
(c) perpendicularity (d) Flatness
widely used to check and calibrate geometric

3 1. Which one of the Instrument is a comparator features of machine tools during their
(GATE-PI-07) assembly? (GATE-ME- 14-SET-l)
(a) Tool makers microscope (a) Ultrasonic probe
(b) GO/NOGO gage (b) Coordinate Measuring Machine (CMM)
(c) Optical interferometer (c) Laser interferometer
(d) Dial gage (d) Vernier calipers

.
32. In an interchangeable assembly, shafts of 36 . Ho1es of diameter 25 . o +o.
+o.04
o
020 mm are
assembled interchangeably with the pins of
size 25.000 -0·0 1 0 mm mate with holes of size
+0.040

diameter 25 · 0 +0.005 The mm1mum


-o.oos
mm ·

25.0oo clearance in the assembly will be


+0.030
+o ozo
mm. The maximum interference
(in microns) in the assembly is (GATE -15 -Set 1)
(GATE-ME & PI-12) (a) 0.048 mm (b) 0.015 mm
(a) 40 (b) 30 (c) 20 (d) 10 (c) 0.005 mm (d) 0.008 mm

!M•liii§jjjgq.jjijilnnjffi\lj.jj� yderabad J Delhi I Bhopal I Pune J Bhubaneswar I Lucknow I Patna I Bengaluru I Chennai I Vtjayawada I Viz.ag I T'rrupati I Kukatpally I Kolkata
: 813 : Metrology

37. Which one of the following statements 1s 39. The limits of a shaft designated as 100h5 are
TRUE? (GATE -15 -Set 2) 100.000 mm and 100.014 mm. Similarly, the
(a) The ' GO' gage controls the upper limit limits of a shaft designated as 100h8 are
of a hole 100.000 mm and 100.055 mm. If a shaft is
(b) The 'NO GO' gage controls the lower designated as 100h6, the fundamental
limit of a shaft deviation (in µm) for the same is
(c) The 'GO' gage controls the lower limit (GATE - PI-16)
of a hole (a) -22 (b) zero (c) 22 (d) 24
(d) The 'NO GO' gage controls the lower
limit of a hole 40. The roughness profile of a surface is
depicted below.
38. Match the following
List - I
Profile

P. Feeler gauge
height
- - -
Q. Fillet gauge
2 µm

R. Snap gauge
t :
S. Cylindrical plug gauge
c..,.
0
X
o 2 µm :

List -II -L , - -
]
tfi I
'..---
:,
-
I. Radius of an object ----:
, - 5mm
u

II. Diameter within limits by comparison


The surface roughness parameter Ra (in µm)
III. Clearance or gap between components (GATE - PI-16)
IV. Inside diameter of straight hole
lS ---

(GATE -16 -SET - 1) 41. Cylindrical pins of a diameter 15±0 ·020 mm


Codes: are being produced on a machine. Statistical
p Q R s quality control tests show a mean of 14.995
(a) III I II IV mm and standard deviation of 0.004 mm.
(b) III II I IV The process capability index Cp is
(c) IV II I III (GATE -17 -SET - 1)
(d) IV I II III (a) 0.833
(c) 3.333 (d) 3.750
(b) 1 .667

!IJll@@jji44imi@ftt@il@•� yderabad I Delhi I Bhopal I Pune I Bhubaneswar I Lucknow I Patna I Bengaluru I Chennai I Vijayawada I V12ag J Tuupati I Kukatpally J Kolkata
�..
.;•
:F.ugioraiog Pnblicarioos
. . , .... ACE : 814 : Production

42. Accuracy of a measuring instrument 1s 02. Two shafts A and B have their diameters
expressed as (GATE - PI -17) specified as 100±0.1 mm and
(a) true value -measured value 0.1±0.000l mm respectively. which of the
(b) measured value - true value following statements is/are true
true value -measured value (GATE-ME-92)
(c) 1 _
true value (a) Tolerance in the dimension B is greater
in shaft A
true value -measured value
(d) 1 + (b) The relative error in the dimension B is
true value
greater in shaft A
43. A machined surface with standard symbols (c) Tolerance in the dimension A is greater
indicating the surface texture is shown in the in shaft B
Figure. (All dimensions in the Figure are in (d) The relative error m the dimension 1s
micrometer). same for shaft A and shaft B

+0.05
50-50x 1 03

03. A shaft of diameter 20-0· 1 5 mm and a hole of


+0. 20
diameter 2o +o. , o mm when assembled
would yield. (GATE-ME-93)
The waviness height (in micrometer) of the (a) Transition fit (b) Interference fit
surface is (GATE -PI - 17) (c) Clearance fit (d) None of the above
(a) 1 (b) 50 (c) 60 (d) 120
04. 3 blocks of B 1 , B2, B3 are to be inserted in a

Two Marks Questions channel of width 'S' maintaining a


minimum gap of width T=0.125mm, as
01. A bush was turned after mounting the same shown in fig. For P=18.75 ± 0.08, Q = 25.00
on a mandrel. The mandrel diameter in ± 0.12, R=28.125 ± 0.1 and S = 72.35 + X.
+0.00
millimeters is 40 -0·05 and bore diameter of the tolerance 'X' is (GATE-ME-97)
s
bush is 40-0·00 .The maximum eccentricity
+0.06

of the bush , in mm will be


(GATE-ME-91)
(a) 0.01 (b) 0.055 (c) 0.1 (d) 0.11

lffl•li@jji44im§Qnbi&i@j� yderabad j Delhi j Bhopal I Pune I Bhubaneswar l Luclrnow l Patna. I Bengaluru I Chennai I Vtjayawadaj Vi7.ag I TUllpati I Kukatpally l Kolkata
.., " . ACE
'!.�-�:hprmrwPoNiranin
. . . : 815 : Metrology

+
05. The dimensional limits on a shaft of 25h7 0 8. A hole of diameter 25.00 o.o i mm is to be
are (GATE-ME-03) inspected by using GO / NO GO gages. The
(a) 25.000, 25.02 1 mm size of the GO plug gage should be
(GATE-ME-04)
(b) 25.000, 24.979 mm
(c) 25.000, 25.007 mm
(a) 25.00 mm (b) 25.0 1 mm
(d) 25.000, 24.993 mm
(c) 25.02 mm (d) 25.03 mm

06. A part shown in the gig is machined to the


0.050
sizes given below 09. A hole is specified as 40 °·000 mm. The
mating shaft has a clearance fit with
P=35.00 ± 0.0 8 mm
rmmmum clearance of 0.0 1mm. the
Q= 12.00 ± 0.02 mm
tolerance on the shaft is 0.04mm. the
R =13.oo:8:8i mm maximum clearance m mm between the
hole and the shaft is (GATE-ME-07)
With 100 % confidence, the resultant (a) 0.04 (b) 0.05 (c) 0. 10 (d) 0. 1 1
dimension W will have the specification
(GATE-ME-03)
10. A displacement sensor (a dial indicator)
(a) 9.99 ± 0.03 mm (b) 9.99 ± 0. 13 mm
measures the lateral displacement of a
(c) 9.99 ± 0.23 mm (d) 9.99 ± 0.43 mm
mandrel on the taper hole inside a drill

07. A threaded nut of M 16, ISO metric type, spindle. The mandrel axis is an extension of
having 2 mm pitch with a pitch diameter of the drill spindle taper hole axis and the
14.70 1 mm is to be checked for its pitch protruding portion of the mandrel surface is
diameter using two or three numbers of perfectly cylindrical. Measurement are
balls or rollers of the following sizes recorded as Rx = maximum deflection
(GATE-ME-03)
(a) Rollers of 2 mm q>
mmus mm1mum deflection, corresponding

(b) Rollers of 1 . 1 55 mm q>


to sensor position at X, over one rotation ..
(GATE-ME-08)
(c) Balls of 2 mm <I>
(d) Balls of 1 . 155 mm <I>

jlD11ij@jjji1ijji4AflbiMIIIIO+yderabad I Delhi I Bhopal I Pune I Bhubaneswar I LucknowI Pama I Bengaluru I Chennai I Vrjayawada I Vu.ag I Tuupari I Kukatpally I Kolkata I
: 816 : Production

Fundamental tolerance unit , i in microns

drill
spindle
/I
- 1 ¢:::i°'P
= 0.45 D 1 13 + 0.001D, where D is the
respective size in mm. tolerance value for
IT8 =25i, Fundamental deviation for "f'
shaft = -5.5Do.4i (GATE-ME-09)
mandrel 0i �ensor
¢:::i (a) 59.970 & 59.924 (b) 60.00 & 59.954
i \U" Q
(c) 60.016 & 59.970 (d)60.046 & 60.00

drill table 12. A shaft has a dimension 35 -0·025 • The


-0.009
1 <I>

� respective values of fundamental deviation


and tolerance are
If Rp = RQ > 0 , which one of the following (GATE-ME-10)
would be consistent with the observation? (a) -0.025, ±0.008 (b) -0.025, - 0.008
(a) The drill spindle rotational axis is
(c) -0.009, ±0.008 (d) -0.009, 0.016
coincident with the drill spindle taper
hole axis 13. A taper hole is inspected using a CMM,
(b) The drill spindle rotational axis with a probe of 2mm diameter. At a height,
intersects the drill spindle taper hole Z= 10mm from the bottom, 5 points are
axis at point P touched and a diameter of circle (not
(c) The drill spindle rotational axis is compensated for probe size) is obtained as
parallel to the drill spindle taper hole 20mm. similarly , a 40mm diameter at a
axis height Z = 40mm, the smaller diameter (in
(d) The drill spindle rotational axis mm) of the hole at Z = 0 is
intersects the drill spindle taper hole (GATE-ME-t o)
axis at point Q

1 1. What are the upper and lower limits


respectively of the shaft represented by 60fs Z = 20
Use the following data Z =O
Diameter 60 lies in the diameter step of
50-80 mm (a) 13.334 (b) 15.334
(c) 15.442 (d) 15.542

!ffl•li@jjj4i@jRflnftW11m� yderabad J Dellii I Bhopal I Pune I Bhubanesw.ir I Lucknow I Patna I Bengalwu I Chennai ! Vtiayawada j Vu.ag I Tllllpati J Kukatpally I Kolkata
: 817 : Metrology

1 4. Outside diameter of bush is turned using a fundamental deviation. If the tolerance on


mandrel. The mandrel diameter 1s the shaft is 0.08 mm, the maximum size of
the hole is (GATE-PI-92)
maintained as 30- · mm and the bore
(a) 230.1 1 (b) 30.16
+0.000
0 050

diameter is 30- · mm. the maximum (c) 30.19 (d) 30.27


+0.050
0 000

value in mm of the eccentricity on the bush


due to the locating mandrel 18. To obtain dimension of 6 1. 18 mm using slip
(GATE-PI-89) gages, the most appropriate combination is
(a) 0.050 (b) 0.100 (GATE-PI-92)
(c) 0.020 (d) 0.025 (a) 1.18 + 50.00 + 10.00
(b) 0.08 + 1.10 + 60.00
15. Assertion (A):Basic hole system 1s (c) 1.08 + 0. 10 + 50.00 + 10.00
advocated by Indian Standards for fit design (d) 1.08 + 1. 10 + 50.00 + 9.00
Explanation (E): Holes can be produced to
any size. (GATE-PI-90) 19. The maximum interference in mm after
(a) Both A and E are correct & E explains A
assembly between a bush of size 30 +0-03 and
(b) Both A and E are true but E not explains A
+0.06

(c) A is true but E is false a shaft of size 30 -0 02 is


+0.04

(d) A is false but E is true. (GATE-PI-93)


(a) 0.07 (b) 0.05
16. The combination of slip gauges to obtain a (c) 0.02 (d) 0.01
dimension of 10.35mm will be
(GATE-PI-91) 20. Assertion: End standards are used for
(a) 10.00 + 0.30 + 0.05 precision component inspection
(b) 8.00 + 1.30 + 1.05 Reason: m end standards line of
(c) 10.00 + 0.35 measurement coincides with line of
(d) 5.00 + 4.00 + 1.00 + 0.3 5 indication (GATE-PI-93)

2 1. Holes 1,2 , 3 ,4 drilled in sequence, with


17. A 30 mm hole shaft assembly results in
minimum and maximum clearances of angular positions and tolerances are shown
0.03mm and 0. 30 mm respectively. The in Figure. Evaluate closing angle 'X' with
hole has a unilateral tolerance with zero its tolerance band. (GATE-PI-93) .

!li•liii§ihi4iihi@ijj@ujjj� ydcrabad I Delhi I Bhopal I Punc I Bhubancswar I Lucknow I Patna I Bcngaluru I Chcnnai I Vuayawada I Vmg I Tuupari I Kuk.atpally I Kolkata
: 818 : Production

20.00 -0·010 mm diameter form a mating pair.


-0.025

In the worst assembly conditions, the


RMR

clearance between them will be


Tolerance

(GATE-PI-04)
''
' , , in degrees

(a) 40 microns (b) 30 microns


O. I 0 '\ 1 5.00 ± 0. 1

3 HOLE (c) 25 microns (d) 15 microns


I

26. A cylindrical pin of diameter 1.996 -0 ·001 5


'' +o.0015

mm is assembled into a hole of diameter

2.000 -0·00 1 5 mm. The allowance (in mm)


+0.001 5

provided for this assembly is


22. A hole is specified as 0 30 ± 0.04mm with -
(GATE-PI-05)
0.0 1 M. The virtual diameter of the hole
(a) 0.00 1 (b) 0.0 15
(i.e the maximum diameter of the pin that
0

(c) 0.025 (d) 0.035


can enter the hole) is . . . . . . . . . .
(GATE-PI-94)
27. A keyway of 5mm depth is to be milled in a
shaft of diameter 60±0. 1 by positioning the
23. The diameter of a hole is given as
shaft in a V-block of angle 120deg and
50+o. is+o.oo mm. The upper limit on the
setting the tool with reference to the
dimension in mm, of the shaft for achieving
intersection of the face of the V-block. The
maximum interference of 50micron is . . .
error due to shaft tolerance which would be
(GATE-PI-95)
transferred on to the component is
(GATE-PI-06)
24. The least count of a 200 mm long spirit
(a) 0.09mm (b) 0. 1 19mm
level is given as 1Oseconds per division.
(c) 0.22mm (d) 0.3mm
The gradient of the spirit level expressed as
mm over one meter is . . . . . . . . . 28. The surface finish of the component
(GATE-PI-95) produced by a grinding process was
measured. Based on roughness parameters
25. In an interchangeable assembly, a shaft of obtained, which one of the following is true
20.00 - mm diameter and a hole of (GATE-PI-06)
-0.01 5
0 005
·

1 •>i
_1111411,,u,,.,.,,NDQqypurn�:·_Ydcrabad I Delhi I Bhopal I Pune I Bhubaneswar I Lucknow I Patna I Bengaluru I Chennai I Vrjayawada I Vi.zag !Tirupati I Kukalpally I Kolkata I
: 819 : Metrology

(a) Ra < Rs < Rz < Rt +0.07 +0 . 65


(a) 30 +0 03o mm (b) 30 +0 02o
mm
(b) Rs < Ra < Rz < Rt +0 . 08 +0.070
( C) R2 < Ra < Rs < Rt (c) 30 +0 03o mm (d) 30 + 0.040 mm
(d) Ra < Rz < Rs < Rt
32. The following data are given for calculating
29. A tool with side cutting edge angle of 30deg limits of dimensions and tolerances for a
and end cutting edge angle lOdeg is used for hole: Tolerance unit 'i' (microns) = 0.45
fine turning with a feed of l mm/rev . (D) 1 13 + O.OOl D. The unit of D is mm.
neglecting nose radius of the tool, the Diameter step is 18 - 30. if the fundamental
maximum (peak to valley) height of the deviation for hole H is zero and IT8 = 26i,
surface roughness produced will be the maximum and minimum limits of
(GATE-PI-07) dimension for a 25H8 hole (in mm) are
(a) 0.16mm (b) 0.26mm (GATE-PI-08)
(c) 0.32mm (d) 0.48mm (a) 24.984 & 24.967
(b) 25.0 17 & 24.984
30. Tolerance on the dimension 'x' in the two
(c) 25.03 3 & 25.00
component assembly shown below is (all
(d) 25.00, 24..967
dimensions are in mm) (GATE-PI-07)

33. An experimental setup 1s planned to


determine the taper of work piece as shown
in the figure. If the two precision rollers
have radii 8mm and 5mm and the total
thickness of slip gauges inserted between
the rollers is 15.54mm, the taper angle '0'
(a) ± 0.025 (b) ± 0.030 lS (GATE-PI-08)
(c) ± 0.040 (d) ± 0.045
Workpiece
3 1. Diameter of a hole after plating needs to be
+0.05
controlled between 30 +0 0 10 mm. if the
plating thickness varies between 1 0 - 1 5
microns, diameter of the hole before plating
should be (GATE-PI-07) 7

iM•li@iii§§fo401Mimii,..� yderabad I Delhi I Bhopal I Pune I Bhubaneswar I Lucknow I Pama I BengaJwu I Chennai I Vijayawada I Vizag I Tirupati I Kukatpally j Kolkata
: 820: Production

(a) 6 deg (b) 1 0 deg 36. A small bore is designated as 25H7. The
(c) 1 1 degree (d) 1 2 deg lower (minimum) and upper (maximum)
limits of the bore are 25.000mm and 25.021
34. An autocollimator is used to respectively. When the bore is designated as
(GATE-PI-09) 25H8, then the upper limit is 25.033mm.
(a) Measure small angular displacements on when the bore is designated as 25H6, then
flat surfaces the upper limit of the bore in mm is
(b) Compare known and unknown (GATE-PI-10)
dimensions (a) 25.00 1 (b) 25.005
(c) Measure the flatness error (c) 25.009 (d) 25.0 1 3
(d) Measure roundness error between
centers. 37. To measure the effective diameter of an
external metric thread (included angle is
35. Match the following 60°) of 3.5 mm pitch, a cylindrical standard
(GATE-PI-09) of 30.5 mm diameter and two wires of 2
DEVICE FUNCTION mm diameter each are used. The
P. Jig 1 . Helps to place work piece in micrometer reading over the standard and
the same position cycle after over the wires are 1 6.532 mm and 1 5.398
cycle mm respectively. The effective diameter
Q. Fixture 2. Holds the work piece only (in mm) of the thread is (GATE-PI-11)
R. Clamp 3..Holds and position the work (a) 33.366 (b) 30.397
piece (c) 29.366 (d) 26.397
S. Locator 4. Holds and position the work
38. Observation of a slip gauge on a flatness
piece and guides the cutting
interferometer produced fringe counts
tool during a machining
numbering 1 0 and 1 4 for two readings. The
operation
second reading is taken by rotating the set­
up by 1 80° . Assume that both faces of the
slip gauge are flat and the wavelength of the
(a) P-4, Q-3, R- 1 , S-2
(b) P- 1 , Q-2, R-3, S-4
radiation is 0.5086 µm. The parallelism
(c) P- 1 , Q-4, R-3, S-2
(d) P-4, Q-3, R-2, S- 1 error (in µm) between the two faces of the
slip gauge is (GATE-PI-11)

Im•lii@ih§§ih/irmnffihiiih� yderabad I Delhi I Bhopal I Pune I Bhubaneswar I Luclmow l Patna I Bengaluru I Chermai I Vijayawada I Vu.ag I Tirupati I Kukatpally I Kolkata
: 821 : Metrology

(a) 0.2543 (b) 1.172 42. The diameter of a recessed ring was
(c) 0.5086 (d) 0.800 measured by using two spherical balls of
diameter d2 = 60mm and d 1 = 40 mm as
39. A Sine bar has a length of 250mm. each
shown in the figure.
roller has a diameter of 20mm. during taper
angle measurement of a component , the
height from the surface plate to the center of d1
a roller is 100mm. the calculated taper diameter
angle (in degrees) is (GATE-Pl-12)
(a) 21.1 (b) 22.8 (c) 23.6 (d) 68.9

40. Cylindrical pins of 25 mm diameter are


+0.020
+o oio

electroplated in a shop. Thickness of the


plating is 30±0.2micron. Neglecting gage Recessed
Ring
tolerances, the size of the GO gage in mm to
inspect the plated components is
d2 diameter
(GATE-ME & PI-13)
(a) 25.042 (b) 25.052 The distance H2 = 35.55 mm and H 1 = 20.55
(c) 25.074 (d) 25.084 mm. The diameter (D, in mm) of the ring
gauge 1s ____
41. For the given assembly : 25 H7/g8, match
(GATE-ME-14-SET-3}
Group A with Group B
Group A Group B 43. A GO - No GO plug gauge is to be
P. H I. Shaft Type designed for measuring a hole of nominal
Q. IT 8 II. Hole Type diameter 25mm with a hole tolerance of ±
R. IT 7 III. Hole Tolerance Grade 0.015 mm. Considering 10% of work
S. g IV. Shaft Tolerance Grade tolerance to be the gauge tolerance and no
(GATE-ME- 14-SET-l) wear condition, the dimension (in mm) of
(a) P - I, Q - III, R - IV, S - II the GO plug gauge as per the unilateral
(b) P - I, Q - IV, R - III, S - II tolerance system is
(c) P - II, Q - III, R - IV, S - I (GATE-ME-14-SET-4)
(d) P - II, Q - IV, R - III, S - I

ilH1ii@jji4§,jjjQjmftjM\i/jjj� yderabad I Delhi I Bhopal I Pune I Bhubaneswar I Lucknow! Patna I Bengalwu I Chennai I Vijayawada j V1Z3g I Tuupati I Kukatpal]y I Kolkata
: 822 : Production

+0.003 45. In the assembly shown below, the part


(a) 24.985-0 003 (b) 25.015-0·006 dimensions are;
+0.000

+0.03 +0.003
(C) 24.985-0.03 L, = 22.0 ±0·0 ' mm, L2 L 3 = 10.0 ±0 005 mm,
(d) 24.985-0·000
=

Assuming the normal distributions of part


44. The alignment test "Spindle square with dimensions, the dimension L4 in mm for
base plate" is applied to the radial drilling assembly condition would be,

I
machine. A dial indicator is fixed to the
cylindrical spindle and the spindle is rotated
to make the indicator touch the base plate at
different points. The test inspects whether
1
� LJ
the (GATE-PI-14)
(GATE -15 -Set 1)
±0 oos ±0 . 0
Spindle (a) 2.0 (b) 2.0 12
(c) 2.0 ±0.0 1 6 (d) 2.0 ±0 020

46. A surface is prepared specially for an


Dial indicator

application with the profile as shown in the


figure.
Base pla'!e

(a) Spindle vertical feed axis is


perpendicular to the base plate
(b) Axis of symmetry of the cylindrical
spindle is perpendicular to the base
plate.
(c) Axis of symmetry, the rotational axis
and the vertical feed axis of the spindle
are all coincident
(d) Spindle rotational axis is perpendicular The theoretical Ra value for this surface, in
to the base plate. µm, is ___
'(GATE - PI -15)

!l1•l4@jjjl41h/QRflbiM\ii.jj� yderabad J Delhi J BhopalJ Pune J BhubaneswarJ Lucknow J Pama J Benga]uru J Chennai j Vijayawada j Vu.ag I Tiru�•� Kukarpa!ly J Kolkata
" "" . ACE
. . . : 823 : Metrology
�-�
-:... :�PnNnt,rm

47. During the measurement of internal taper of

llllll llllllllllll
Monochromatic light
a part using standard balls of diameter 15
mm and 20 mm, the large ball is found to
protrude by 5 mm (h 1 ) and the top of small
ball is found to be 35 mm (h2) below the top
I
face of the gauge. The taper angle, in degree,
If the wavelength of light used to get a fringe
lS--- (GATE - PI -15)
spacing of 1 mm is 450 nm, the wavelength
of light (in nm) to get a fringe spacing of 1.5
mm 1s ----
(GATE - 16 - SET - 3)

50. A cylindrical pin of 2s:i:i�i mm diameter is


electroplated. Plating thickness lS
48. For the situation shown in the figure below
2.0±0.oos mm. Neglecting the gauge tolerance,
the expression for H in terms of r, R and D is
the diameter (in mm, up to 3 decimal points
accuracy) of the GO ring gauge to inspect
the plated pin is__
(GATE - 17 - SET - 2)

51. Assume that the surface roughness profile is


triangular as shown schematically in the
(GATE - 16 - SET - 2)
figure. If the peak to valley height is 20 µm,
(a) H = D + .Jr2 + R2 the central line average surface roughness Ra
(b) H = (R + r) + (D + r) (in µm) is
(c) H = (R + r) + .JD 2 - R 2

(d) H = (R + r)+ �2D(R + r)- D 2

(GATE - 17 - SET - l)
49. Two optically flat plates of glass are kept at
(a ) 5 (b) 6.67
(d) 20
a small angle e as shown in the figure. (c) 10
Monochromatic light is incident vertically.

�ydcrabadl Delhil BbopaI IPunel Bhubaneswarl I..ucknowl Patna l Bengaluru ! ChennailVuayawadalVizag ITuupati I Kukalpallyl Kolkata I
�..�-�E&lgiommg Pnbtirariooa
ACE : 824 : Production

04. Two slip gauges of 10mm width measuring


Five Marks Questions 1.00mm and 1.002mm are kept side by side
in contact with each other length wise. An
0 1. Calculate the sizes of GO and NOGO (Plug
optical flat is kept resting on the slip gauges
and Ring) gauges to inspect the holes shaft
as shown in the fig. monochromatic light of
pair given below
+0.40 wave length 0.005 892 8mm is used in the
-0.600
Shaft =35 -0 025 , Hole = 35 °·000 .Neglect the inspection. The total number of straight
gauge tolerances and mention the type of fit fringes that can be observed on both slip
and also calculate the maximum clearance / gauges 1s (GATE-ME-03)
interference in the above shaft and hole
(GATE-ME-89)
Optical Flat
assembly.

02. The bore diameter "D" of a plain ring Slip gauges


having a height of 35mm was measured
using two spherical balls, each of diameter 05. A machine operator needs a gauge for
d =25mm. The ring was placed on a surface checking the diameter of holes being
table and then both balls were placed inside +0.06
the ring. In this position, the height "h" of machined to diameter 20-0·04 mm. if wear
the top of the upper ball from the surface allowance and gauge tolerance are equal to
table was found to be 42.00mm. the 10% of work tolerance,
diameter of the ring is . . . . . . (GATE-PI-91)
(GATE-ME-99) (i) the size of GO gauge with unilateral
tolerance is
03. While measuring the effective diameter of -0.02 -0.05
external metric screw thread gauge of (a) 20 -0·03 mm (b) 20 -0·06 mm
+0.05 +0.08
3.5mm pitch, 1 30.5 mm diameter (c) 20 +o.04 mm (d) 20 +o.o7 mm
cylindrical standard and 2mm diameter
wires were used. the micrometer reading (ii) the size of NOGO gauge with unilateral
over the standard and wires was 13.376 8. tolerance is
+0.06 +0.07
the corresponding reading over the thread (a) 20 +0 05 mm (b) 20 +0-06 mm
and wires was 12.242 8 mm. find the thread -0.04 -0.03
gauge effective diameter? (c) 20 -0·05 mm (d) 20 -0·04 mm
(GATE-ME-2002)

!M11i!i§jjjl41hdRftnj@h!@i,Hyderabad J Delhi J Bhopa] J Pune J Bhubaneswarl LucknowJ PatnaJ Bengaiwu J Chcnnai JVtjayawadaJ Vizag IT=pati I Kukatpally l Kolkata I
: 825 : Metrology

06. Mention which angle gauges will be used to 09. With reference to the fig given, write down
make an angle of 7 ° 56'48". The angle , so the equation relating the dimension "h" with
obtained is checked with a sine bar of length the diameter "D" and the angle "a". If a unit
250 mm what slip gauges combination will increase in D has to result in a unit increase
be required? (GATE-PI-91) in "h", what must be the angle "a" ?
(GATE-PI-94)
07. A 'V' block is produced with a 90 angle
a
°

cutter. The height difference measured over


two standard rollers of diameter difference
10mm placed in this 'V' block is 12. 138. D

Derive an expression to find the error in the


angle of the 'V' block. (GATE-PI-92) 10. Straightness measurement is carried out
using a device shown below:

08. A product shown in Figure 3 is turned, step 1 div = 10 µ m ""-

turned and parted off from a rod to get


dimension 'A' in the second setting it is
through drilled and counter bored to get
... 11111 00
1---mm
----·
� •111-1 00
----
dimension 'B'. (a) Calculate the dimension mm

A and B and their tolerances ti A and ti B.


(GATE-ME-92) Initially the device is kept on a reference
surface and the dial is set to zero. On the
-J 20 ± 00• 1--1@ jo . 1• @ !x @ I test surface, the following readings are
obtained ( 1 division = IO µ m)
(GATE-PI-94)
Position: 1 2 3
Reading: +2 0 - 1 +1
(a) Sketch the straightness deviations using
A±M

the above readings


(b) Calculate the maximum straightness
-li o± OO�
I• 30.00± 0. 1 0 •I
error using end-points reference line.

ilffli!@jjjgq.jli4RftnffM\11.jj� yderabad I Dellii I Bhopal I Pune I Bhubaneswar I Lucknow I Patna J Benga)uru I Chennai j Vtjayaw.ula I Vi7.ag I Tirupati I Kukatpally I Kolkata
: 826 : Production

11. A plate with two pins, another plate with two


holes is to be assembled with the pins 12. Statement For Linked Answer Question
entering the holes as shown in fig. In In a three wire inspection of external thread
Assembly the pins touch the outer most of M16 x 2, the measurement over the wires
points of the holes as shown, the limits on was found to be 16.455mm
center distance between the holes is (GATE-PI-06)
100±0. 1
(i) The best wire size is
(a) 1.155 (b) 2mm
(c) 2.309 (d) 4mm

(ii) The effective pitch diameter of the


thread is
(a) 14.723 (b) 16.705
(c) 1 7.122 (d) 17.455
For the above fig, the center distance
between the holes, when the size of the
holes and the center distance between pins
are at their maximum limits, and pins are at
the minimum limit and also specify whether
the assembly is possible or not
(GATE-PI-95)

l11••i@jji44ijjgldnii111.jj+yderabad I Delhi I Bhopal I Pune J Bhubaneswar I Lucknow I Patna I Benga)uru I Chennai j Vtjayawada I Vu.ag I Tllllpati I Kukatpal)y I Kotkat, I
SOLUTIONS
08. Ans: (a)
One Mark Solutions Sol: for interference fit
L - shaft > H -hole
0 1. Ans: (c)
Sol: H - limit of shaft is lying in between H and 09. Ans: (a)
L limits of hole. Sol: Ring gages are used to inspect the outside
dia. of shaft, but it never measures the
02. Ans: (a) roundness.
Sol: Hole 1s indicated by capital letter "G", 10. Ans: (c)
whose lower limit 1s fixed. Hence the Sol: H limit of shaft is in between the H and L
position is decided by letter "G". limits of hole

03. Ans: (c) 1 1. Ans: (a)


Sol: Flatness of measuring equipments 1s
12. Ans: (c)
measured by auto collimator (or) optical
Sol: For measurement (or) setting of an angle the
interferometer.
sine bar in conjunction with slip gages (For
small jobs) and sine bar and height gage
04. FALSE, Accuracies of slip gapes is greater
(For longs jobs) can be used.
than accuracy of outside caliper

13. Ans: (d)


05. ATTRIBUTES.
Sol: Accuracy of slip gages is better than outside
micrometer.
06. Ans: (d)
14. Axis of work piece.
07. Ans: (b)
Sol: Allowance 1s nothing but the difference 15. Ans: (c)
between maximum material limits of hole &
16. Ans: (a)
shaft, which is equal to minimum clearance
(or) maximum interference. 17. Datum, Macro

!IJ•lii@h/44ih/4Rflnjfjjjlr.jj� I I I I I I I I I I I
yderabad Delhi Bhopal Pune Bhubaneswar Lucknow Patna Bengaluru Chennai Vijayawada Vizag Tirupati I I
Kuk.atpally Kolkata
: 828: Production

18. 32. Ans: (c)


Sol: In metric thread angle of thread = 60° Sol: Maximum interference
. . P 0.5 H- shaft -L-hole
The best wire size = - sec a = -x sec a
=

2 2 25.04 -25.02
0.25 sec 30 0.288 mm.
=

= = = 0.02 = 20 microns

19. 33. Ans: (c)


n ').. . . pitch 0
Sol: Groove depth = -x - Sol: The best wire size = --xsec-
2 2 2 2
4 0.5 2 60
= -x- = O .5 m icrons . =-xsec - = sec30 ° = 1.154
2 2 2 2

20. Ans: (c) H -shaft < L - hole. 34. Ans: (b)


Sol: For measurement of flatness errors of
21. surfaces, first it needs to measure the
Sol: Straightness error = 0.25 mm straightness error. Hence the straightness
error can be measured by using spirit level
and autocollimator

35. Ans: (c)


Sol: Ultrasonic probe (UT) is used for NDT and
determination of thickness, CMM is used
for measurement of dimensions and
geometric features of a product, but laser
interferometer is also called as laser tracker
is used for large size components with its
22. Ans: (d)
geometrical features and it is portable.
23. Ans: (b) due to low density
24. Ans: (b) 25. Ans: (b)
36. Ans: (b)
26. Screw threads 27. Ans: (a) Sol: Minimum clearance = difference between
28. Ans: (b) 29. Ans: (a) maximum material limits
30. Ans: (a) 31. Ans: (d) = L-hole -H-shaft
= 25.02 -25.005 = 0.015
!ltlli@jjji4@jQflajMJj/.jj� yderabad I Delhi I Bhopal I Pune I Bhubaneswar I wcknow I Patna I Bengaluru I Chennai I Vijayawada I Vizag I T11Upari I Kukatpa))y I Kolkatl
: 829 : Metrology

-B,------r--1-
CPU = Upper specification calculation
CPL = Lower specification calculation
Minim um _j!J����

µ = 14.995 mm
Clearance

f - ----------------- USL -µ
(CPU ) =
3 cr
15.02 -14.995
= = 2.083
3x0.004
37. Ans: (c) µ -LSL 14.995 - 14.98
(CPL) = = = 1.25
3cr 3 X 0.004
38. Ans: (a) Process capability index = minimum (CPU,
Sol Feeler gauge 1s used for checking the CPL) = 1.25
clearance or gap between the parts, radius is Here, process capability ratio is 1. 667. There is

checked by fillet gauge, limits of diameter of no option matching. Out of given option best

shaft is checked by snap gauge and plug


option (B).

gauge is used for checking the diameter of


IIT key : (C)
hole.
42. Ans: (A)
39. Ans: (b) Sol: Accuracy is the difference between standard
Sol: Fundamental deviation for shaft "h" 1s value and measured value.
always "zero" irrespective of grade of
tolerance. IIT key : (B)
43. Ans: (A)
40. Ans: 2 Sol: It is the surface roughness value in microns.
Lh 16x4 +16x0 64
Sol: R = = = = 2 µm
a n 32 32

4 1 . Ans: (B)
Sol: USL = 15.02 mm , LSL 14.98 mm,
cr = 0.004 mm
USL -LSL
Process capability ratio =
6cr
15.02 -14.98
= = 1.667
6x0.004

l11'1i@himhiiRbbiM\hih� yderabad l Delhi I Bhopal I Pune I Bhubaneswar l Lucknow I Patna! Bengaluru I Chennai I Vijayawada ! Vizag I Ttrupati I Kukatpally I Kolkata
: 830: Production

But for shaft "h",


H - Shaft = 25.000
Two Marks Solutions
L -Shaft = less than 25
And hr-�7 indicates IT 7 not 7 microns.
01. Ans: (b)
06. Ans: (b)
Sol: Hole = 40 -0 00
+0.06

Sol: W max = P max -(Q min + R min)


Shaft = 40 - = 35.08 -(1 1.98 + 12.98) = 10.12 mm
+0.00
0 05

Eccentricity (Max) W min = P min - ( Q max + R max)


= Maximum possible radial clearance. = 34.92 -( 12.02 + 13.04) = 9.86
40.06 -39.95 W = 9.86 to 10.12 = 9.99 ± 0.13 mm
= = 0.055.

07. Ans: (d)


02. Ans: (a & d) Sol: For measurement internal threads balls are
Sol: Tolerance in shaft A = 0.2 mm used.
Tolerance in shaft B = 0.0002 mm
The best wire size = � sec 30 = 1.155.
Relative error in both the cases is same 2

03. Ans: (c) 08. Ans: (a)


Sol: L -hole > H - shaft Sol: Hole size given is 25 .o , mm and lower
+o

limit was not given means that it can be


04. assumed as basic hole, with size equal to
Sol: S max = T + Rmax + Qmax + Pmax basic size.
= 0.125 + 18.83 + 25.12 + 28.225 Hence GO PLUG size = MML of hole
= 72.3 mm = 25.00 mm.
S min = 0.125 + 18.67 + 24.88 + 28.025
= 71.7 mm 09. Ans: (c)
Tolerance in S = 72.3 - 71.7 = 0.6 mm Sol: Max. clearance = H.hole - L.shaft
= 40.05 - 39.95 mm = 0.1 mm
05. Ans: (b) Min clearance = L.hole - H.shaft
Sol: To calculate exactly the data was not given H.Shaft = 40.000 -0.01 = 39.99 mm
in the problem. L.Shaft = 39.99 -0.04 = 39.95 mm.

!ffllii@ii/Mjjijj;nMffj,@j.� yderabad j Delhi I Bhopal ! Pune j Bhubaneswar l Lucknow I Patna I Bengalwu j Chennai j Vijayawacla j Vi.zag jTuupari I Kukatpally j Kolkara
t, V • ACE
• • •
: 831 : Metrology
':i�-��&ypw:rq pqNii,pn

10. Ans: (a)

11. Ans: (a)


Sol: D = �50 x80 = 63.24 mm
i = 1.86 microns = 1.9 microns With probe diameter compensation
IT8 = 25i = 47.5 microns = Tolerance Dactual = 13.334 + 2 xr sec 0
F.D = -5.5 D0.4! = -5.5 X 63.24°·4 1 = 13.334 + 2 x ( l sec x18.435)
= 30 Microns = 15.442 mm.

H. shaft = 60 - F.D = 60 -0.03 = 59.97


L. shaft = H. shaft -Tolerance. 14. Ans: (a)
Sol: Max. eccentnc1ty =-----
= 59.97 - 0.047 = 59.923 mm. . . max clearance
2
12. Ans: (d) = H. hole -L . shaft
------
Sol: F.D of shaft = B.S -H. shaft
= 35 -34.99 = 0.009 30.05-29.95 =
= 0.05.
Tolerance = 34.991 - 34.975 = 0.016. 5

15. Ans: (b)


13. Ans: (c)

Sol: tan 0 = .!Q � 0 = tan- 1 (1/3) � 0 = 18.434 ° 16. Ans: (b)


30 Sol: In other choices non - available slip gages
were used.

17. Ans: (c)


Sol: Min, Clearance = 0.03= L - hole -H - shaft
H - shaft = L - hole -0.03
= 30 -0.03 = 29.97.

1 0 - ( 1 0/3) H - shaft -L - shaft = 0.08


L -shaft = 29.97 -0.08 = 29.89 mm
Distance at Z = 0, Max, Clearance = H. hole -L-shaft
D 0 = 2(I0 -10 tan 30) = 2(10-
1
3
°) H-hole = 0.3 + L-shaft
= 0) + 29.89 = 30.19 mm.

6.67 X 2 = 13.33 mm
,-·
=

!ltl•id§hill@Mjjjjni@dfjj*)1Iyderabad l Delhi l Bhopal i Punc 1 Bhubancswarl Lucknow l Patna l Bcnga)uru l Chcruiai l Vuayawadal Vizag !Tuupati I Kukatpal]yl Kolkata I
: 832 : Production

18. Ans: (a) 24.


Sol: Out of the given choices A and D has the Sol: Gradient of spirit level = Sensitivity spirit
available slip gages but out of which A has level in mm/m
minimum slip gages. 10
= -- X - X 1000 = 0.04845 mm/m.
3600 180
1r

19. Ans: (d)


Sol: Max interference= H.shaft - L.hole 25. Ans: (b)
30.04 - 30.03 = 0.01 mm Sol: Maximum Clearance in the
Worst Condition = H-hole - L- shaft
=

20. Ans: (c) [A -True and R - false].


= 20.025 - 19.995
= 0.03 = 30 microns.
2 1.
Sol: X max = 360 -(80rmn + 11Ornin + 11Ornin) 26. Ans: (a)
= 360 - (79.9 + 109.8 + 109.7) Sol: Allowance = difference between maximum
60.6 mm material Limits
= H-shaft - L-hole
=

Xmin = 360 -(80max + 11Omax + 11Omax)


= 59.4 mm = (1.996 + 0.00 15) - (2 - 0.0015)
= 1.9975 -19985 = 0.001
X ± � = 60 ± 0.6

27. Ans: (c)


22.
Sol: If D = Dmin = 59.9
Sol: Allowable L - hole = 30 - 0.04 - 0.01
X 1 = distance between center of shaft and
30 - 0.05= 29.95 mm.
59.9
=

Allowable H -hole = 30 + 0.04 = 30.04


comer of V - block = -2- = 34.583
Because 0.01 1s considered on the sin 60
maximum material limit only. 60.1
X2 = - 2- = 34.698
sin 60
23. Error in depth = 2(X2 -Xi ) = 0.223 mm.
Sol: Maximum interference = 0.05
= H-shaft - L-hole 28. Ans: (c)
H-shaft = 0.05 + 50 = 50.05 mm. Sol: Always Rz is lowest and R1 is highest

!ltl•ih§Oi4ilmjRflniffi\hijj� yderabadJ Delhi I Bhopal I Pune J Bhubaneswar J wcknow J Patna I Benpluru I Chennai I Vijayawada I Vizag J Ttrupati I Kukatpally J Kolkata
: 833 : Metrolo_gy

29. Ans: (a)


Taper angle = - = 6 °
0
f 2
Sol: R1 = -----
tan Cs +cot C e Included angle = 12°.
1 -
= ---- = 0.16 mm
tan 30 +cot 10 34. Ans: (c) 3 5 . Ans: (d)

30. Ans: (d) 36. Ans: (d)


Sol: Xmax = 5 0.02 - (37.985 +9.99) = 2.045 Sol: 25 H1 ---+- Grade of Tolerance = IT 7
Xmin = 49.98 - (38.01 5 + 10.01) = 1.95 5 = 16i = 21microns
Tolerance on X = Xmax - Xmin = 0.09 2 5 H6 ---+- Grade of Tolerance = IT 6= l Oi
Dimension X = 2 ± 0.045 . = 10 x 1.3125 = 13 microns
H-limit = 25 + 0.013 = 25.0.13
31. Ans: (c)
Sol: L-hole = 30.01 + O.Ol X 2 = 30.03
Difference in Toi Difference in Tolerance
} between 25 H1 & 25H6
Between 25 H8 - 25H7
H-hole = 30.0 5 + 0.01 5 X 2= 30.08.
0.012 > Tolerance

32. Ans: (c)


37. Ans: (d)
Sol: D = .J18x30 = 23.2
60
i = 0.45 3.Jij +0.001 D = 1.3 Sol: a = = 30, p = 3. 5 , S = 30. 5
2
IT8 = 26i = 26 x 1.3 = 33.8 = 34 Wire diameter = d = 2 mm
p
Hole size = 25 H 8 = 25 +o ooo = sec a = 2.02
2
+0.034

� Taken as the best wire


33. Ans: (d)
e 3 M = S + (R2 -R 1 )
Sol: tan - = --
2 28.5 4 = 30.5 + (15 .398 - 16.5 32) = 29.366

2e = tan _, ( 3
)= 6 Effective dia = M -(d +; tan a)
28. 5 4

= 29.366 -( 2 + \ tan 30)


5

= 26.35 5 mm.

!l"S•lii@iii44mYWMiiiiiii'* yderabad I Delhi I Bhopal I Pune I Bhubaneswar I Lucknow I Patna I Bengaluru I Chennai I Vtiayawada j Vtzag I Tl11lpati I Kukatpally I Kolkata
: 834 : Production

38. Ans: (c) 42. Ans: 92 to 94


n2 n1 Sol: Diameter = D = R2 + AB +R 1
Sol: Parallelism error = ( x�
20 +AB +30
; )
=

14 - 10 0.5086 50 +AB
= --- x---
=

2 2 AB = (AC2 -CB2) 1 12
0.5086 mm. = {(R
1 +R2/-[(H2 + R2)-(H 1 +R 1 )] }
2 05

= {502-((35.55+30) -(20.55 +20)] 2 } 0· 5


=

= {502 - [65.55 -40.55 ] 2 } 0·5


39. Ans: (a)
Sol: L = 250mm, d = 20mm, = 93.3mm
h = 100 -d/2 = 100 -20/2 = 90mm
sine = h/L = 90/250 43. Ans: (d)
0 = 2 1.2deg. Sol: Work tolerance = 25.015 -24.985
= 0.03mm
40. Ans: (d) Gauge tolerance = 10% of work tolerance
0.003mm
Sol: Pin = 25
=

Because there is no wear allowance is


+-0.02
+0· 0 1

Plating thickness = 30±2.0 micron given on the GO gauge,


= 28 to 32 micron L-limit of Go gauge = MML of the Hole
= 0.028 to 0.032 mm = L-limit of hole = 24.985mm
After plating thickness, size of pin H-limit of the GO gauge
Maximum size of pin = L-limit of Go + Gauge
H limit+2x max thickness of plating Tolerance
24.985 + 0.003 = 24.988mm
=

25.02+0.032x2 = 25.084 =

Hence Go gauge size


=

Size of go gage = MML of pin = 25.084

= 24.985 - 0·000 or 25 -0·0 1 5


+-0.003 --0.012

41. Ans: (d)


Sol: Given designation is the hole based system 44. Ans: (d)
in which 25 is the basic size. Fundamental Sol: During testing if the dial indicator shows
deviation of hole is "H", FD of shaft is "g", same reading in one complete revolution,
grade of tolerance is IT7 on hole and IT8 indicates that the drill spindle 1s
on shaft. perpendicular to the base plate.

!IJ11ij@jjji4ijjjiRflnj@j/.jj.�!{yderabad l Delhi l Bhopal j Pune l Bhubaneswar l Lucknow l Patna l BengaJuru l Cheruiai l Vuayawada j Vizag ITuupati I Kukalpally l Kolkata I
:g-:.
-:,,. _ ·�
,:,EngioecringPubliarioos
-
ACE : 835 : Metrology
������=============================
45. Ans: (d)

= �(R + r )2 (D - (R +r ))2
= 22.01 -(9.995+9.995) = 2.02
-

= �2D(R +r) -D 2
= 21.99 - (10.005+10.005) = 1.98 H = (R +r ) + �2D(R +r) - D 2
L4 = 2 ± 0.02
49. Ans: 675 (range 674 to 676)
'A
46. Ans: 2.5 Sol: 1mm = n x -
Sol: Average surface roughness
2

5/2 = 2.5 microns. n =-=-


2 2
'A 450
=

'A
47. Ans: 3.7 to 3.9 1.5 = n x -2
2
Sol:
1.5 2 1. x 2
'A, 2 = : = \ = 1.5 X 450 = 675
0 1 02 = (35 + 7.5) - (10 -5) 450
42.5 - 5 = 37.5
A

50. Ans: 29.03 mm


= 37.5

02A = r2 -r1 10 - 7.5 = 2.5


0/2

Sol:
=

. 0 02A 2.5
Sln - =-- =--
2 Op 2 37.5
0
- = Taper angle = 3.82 °
2

48. Ans: (d) .


P1n = shaft = 25+o.oi
+0 02

Sol:
·

Plating thickness = 2± 0.005 mm


Go gauge size ---+ Maximum material limit of
shaft of shaft ---+ H. limit of shaft
H. Limit of shaft after plating
= 25.02 + 2 X 2.005 = 29.03 mm
Go ring size = 29.03 mm

!l"S•lii@hi§§,014iMCtti\nii,,, yderabad I Delhi I Bhopal I Pnne I Bhubaneswar I Lucknow I Pama I Bengaluru I Chennai I Vijayawada I Vizag I Tirupati I Kukatpally1 Kolkata
�:.,)
V •
ACE
:�Publiariooa
• • •
: 836 : Production
,)

51. Ans: (a) 02.


Sol: Sol: d = 25 mm , H = 3 5 mm, h = 42
25 25
0I 02 = + = 25
2 2
25 25
O2 A = h -( + )
R1 = 20 , 2 2
20 42 - 25 = 17
R = - = 5 µm
4
=

Five Marks Solutions


H

0 1.

Sol: Shaft size = 3 5 -o.ozs


--0.6

= 34.975 to 34.4 mm 01 A = �01 0; - 02 A 2 = 18.33


L -Shaft = min. ML = 34.4mm D = r + 0 1 A + r = 25 + 18.33= 43.33 mm.
NOGO Ring gage size
03.
=

H -Shaft = max. ML = 34.97 5


= GO Ring gage size. Sol: S = 30.5 mm, d = 2 mm,
p
Hole = 35 +o oo 3 5 to 35.4 mm Best wire size = sec a
+0.4
=

L - Hole = Max ML = 35 mm 35
= - sec 30 2mm.
GO plug gage size
=
=

H -hole = Min ML = 35 .4 Hence the wire used is the best wire size
= NOGO plug gage size R1 13.3768 mm, R2 = 12.2428 mm
Because L - hole is greater than
=

Micrometer reading ,M = S + (R2 -R 1 )


H -shaft, the type of fit is clearance fit 29.366
Min Clearance L-hole - H-shaft
=

=
=

3 5 -34.975 = 0.025
Effective diameter = M -( d +� tan a)
Max Clearance = H-hole -L-shaft
= 29.366 -( 2 + \ tan 30 )
= 35 .4 -34.4 1 mm.
5
=

= 26.35 5 .

lltlli@hii41jjjjRflnt@n.jj� yderabad I Delhi I Bhopal I Pune I Bhubaneswar I Lucknow I Patna I Bengalwu I Chennai I Vijayawada I Vmg I Tuupari I Kukatpally I Kolkata
: 837 : Metrology

04. 06.
Sol: Difference in height Sol: 0 =7° 56 1 48 1
1

N ')..., = 9° - 1° -3 1 - 18 1 1 + 6 1 1
Of slip gages = x
2 2
Sin 0 = E._
N ')..., L
1.002 -1.00 = 0.002=- X -
2 2 h = L Sin 0 = 250 x Sin 7 ° 56 1 48 1 1
0.002 x 4
N= =13 35.563 mm.
0.00058928
=

= 1.003 + 1.06 + 1.5 +2.0 + 30.

05.
07.
Sol: Work tolerance (W.T) = 20.06 - 19.96
Sol: 0 = 90
0.lmm
0
exp ected

d2 - d 1 10
=

Wear allowance = gauge tolerance


=

h2 -h , = 12.132
= 10%. W.T = 0.0 1 .

��-�-�---- NOGO
= 20.06

F ,NOGO
W.T Hole

A02 = r2 - r 1
O , 02 = (h2 - r2) - (h , -r , )
H.GO L.GO

(h2 - h , ) - (r2 - r , )
L -NOGO = 20.06 - 0.01 = 20.05
=

. A0, r -r
H - NOGO = 20.06 -0 = 20.06 sm a = --
01 02
= -------
(h 2 - h 1 ) - (r2 - r, )
2 2

NOGO size = 2o +o.o s d 2 -d ,


+0 . 06

2(h 2 - h , ) - (d 2 -d , )
L - GO = 19.96 + 0.0 1 = 19.97
= --------

H - GO = 19.96 + 0.02 = 19.98 10


= = 44.465°
2 x 12.138 -10
GO size = 20 -0 03 • Error = 90 - 2 x 44.465 = 1 .069 ° .
-0.02

jlJIIQ@Oii4.jjjQRflftjffijjj.jj.. yderabad I Delhi I Bhopal I Pune I Bhubaneswar I Lucknow I Pama I Bengaluru I Chennai I Vijayawada I Vuag I Tuupati I Kukatpally I Kolkata
:.-t
'!. :F.ngioeaiog PnNicatioos
ACE : 838 : Production
� - ��������============================
08.

Sol: Amax = 15max + 30max


= 15.06 + 30.1 = 45. 16
Amin = 15min + 30min = 4 4.84

A = 45 ± 0.16. = A ± LlA
Bmax = Amax -20min = 45.16 - 19.93
= 25.23
Bmin = Amin -20max 44.84 -20.07 Max straightness deviation occurs at 1
=

= 24.77 = 2 - 0.5 = 1.5 dia.= 15 microns.


B ± fl B = 25 ± 0.23
11.
09. Sol: Let C = center distance between holes
Cmax = max. Outer distance of pins -
a D
Sol: sin = -9-i sum of min rod of holes.
2 h + -9-i 2h + D
= ---
14. 25 9. 25 14 5 9. 5
. a D = ( 10o. 1 + ; + : ) -( : + i )
2 2h + D
Sln - = ---
= 100.075

14.875 9.875 15.05 10.05


Cmm. = (99.9 + + ) -( + )
2 2 2 2
= 99.725

:. C = 100
+0.075

If D = O, h=O
- 0 21s

D = 1, h = 1
For the given conditions
a 1 1
Sin = = 14. 75 9. 7 5 - 1 05 10 05
2 2xl + l 3 C = ( l 00.1+ � + � ) ( 5; + ; )
a
- = 19.47 => a = 38.94 2. = 99.925 mm.
2
Because C is lying in between the limits, the
1 0. assembly is possible.
Sol: Position: 1 3 4
E Reading: +2 +2 +1 +2
2

l1111ii@iii§4,jjjjjmGjjmjju.� yderabad I Dellii I Bhopal I Pune I Bhubaneswar I Lucknow I Pama I Bengaluru I Chennai I Vtjayaw.ida jVizag I Tirupari I Kukatpally I Kolkata
: 839 : Metrology

12.
p
Sol: (i) The best wire size = sec a
2

=- sec 30 = 1.155 ---,. A


2
2

(ii) M = 16.455

Effective diameter = M - ( d + ; tan a)

= 16.455 -( 1.155 + 1 x tan 30)


= 14.723 mm -,. A

lii••i@jjji4.jjjiRflbiM\jjf¥�,Hyderabad I Delhi I Bhopal I Pune I Bhubaneswar I Lucknow I Patna I Bengalwu I Chennai J Vtjayawada J Vizag I Tuupati I Kukalpally I Kolkata I
I
Chapter
8 Advance Machining Methods

05. CNC machines are more economical to use


One Mark Questions even for simple turning jobs. (True / False)
(GATE-ME-94)
0 1. In NC part programming spindle speed of
730rpm will be coded by the magic - 3 rule
06. In computer aided drafting practice, an arc is
as (GATE-ME-89)
defined by (GATE-ME-00)
(a) S673 (b) S730
(c) S630 (d) S037 (a) two end points only
(b) center and radius
02. Numerically controlled machine tools are
(c) radius and one end point
better suited for . . .. . .... (batch/mass)
(d) two end points and center
production, because their setup time is
. . . . . . . . . (larger / smaller) in comparison to
07. Cellular manufacturing is suitable for
special purpose machine tools.
(GATE-ME-00)
(GATE-ME-91)
(a) a single production in large volumes
03. In PTP type of NC system
(b) one-off production of several varieties
(GATE-ME-92)
(c) production with similar features made in
(a) Control of position and velocity of the
batches
tool is essential
(b) Control of only position of the tool is (d) large variety of products m large
sufficient volumes
(c) Control of only velocity of the tool is
sufficient 08. Machining of complex shapes on CNC
(d) Neither position nor velocity need be machines requires (GATE-ME-02)
controlled (a) simultaneous control of x, y, and z axes
(b) simultaneous control of x, and y axes
04. CNC machines are more accurate than
(c) independent control of x and y axes _
conventional machines because they have a
high resolution encoder and digital readouts (d) independent control of x, y and z axes
for positioning (T/F) (GATE-ME-94)
IM•liUYiUiiihiiRbbiMlhih� yderabad I Delhi I Bhopal I Pune I Bhubaneswar I Lucknow I Patna I Bengaluru I Cliennai ! Vuayaw.ida I Vizag I Tirupati I Kukatpally I Kolkata
: 841 : Advanced Machining Methods

09. NC contouring is an example of 13. The most common interpolation methods in


(GATE-ME-06) continuous path NC machining are
(a) continuous path positioning (GATE-PI-90)
(b) point to point positioning (a) linear and circular
(c) absolute positioning (b) linear and parabolic
(d) incremental positioning (c) circular and parabolic
(d) circular and elliptic
10. Which type of motor is used m axial or
spindle drives of CNC m/c tools 14. Use of a robot for spray painting is primarily
(GATE-ME-07) justified for improved. . . . . . A . . . and reduced
(a) Induction motors ...... B . ............. .. (GATE-PI-90)
(b) DC servomotors A. productivity / consistency
(c) Stepper motor B. cost I hazard
(d) Linear servo motor.
15. Feed motion can be provided with stepper
11. For generating coon' s surface we require motors in CNC (GATE-PI-91)
(GATE-ME-08) (a) lathe (b) drilling machine
(a) a set of grid points on the surface (c) milling machine (d) grinding machine
(b) a set of grid control points
(c) four bounding curves defining the 16. Control of group of NC machines by a
surface supervisory computer is known as . . . . . . . . . . .
(d) two bounding curves and a set of grid control. (GATE-PI-92)
control points
17. FMS 1s well suited to mass scale
12. If the z-axis & x-axis of CNC lathe are manufacturing. (True / False).
provided with straight line controls, it is (GATE-PI-94)
possible to carry out (GATE-PI-89)
18. In DNC each machine tool is controlled by a
(a) Turning and facing only
dedicated computer. (True / False).
(b) Turning, facing and taper turning
(GATE-PI-94)
(c) Turning, facing and thread cutting
(d) Turning , facing and drilling 19. F APT is a part programming language.
(True / False). (GATE-PI-94)

!l1'1i@ibi4iidijmft!Mih!jj� yderabad I Delhi I Bhopal I Pune I Bhubaneswar l Lucknow I Patna I Bengaluru I Chennai I V\iayawada I Vizag j 'firupati I Kukatpally I Kolkata
: 842 : Production

20. NC contouring is an example of 24. A robot arm PQ with end coordinates P(O,O)
(GATE-PI-OS) and Q(2,5) rotates counter clockwise about P
(a) continuous path positioning in the XY plane by 90 °. The new coordinate
(b) point to point positioning pair of the end point Q is
(c) absolute positioning (GATE-ME- 14-SET-2)
(d) incremental positioning (a) (-2,5) (b) (-5,2)
(c) (-5,-2) (d) (2,-5)
2 1. The interpolator in a CNC machine controls
(GATE-PI-07) 25. In an open loop, point-to-point controlled CN
(a) Spindle speed (b) Coolant flow drilling machine, a stepper motor, producing
(c) Feed rate (d) Tool change 200 angular steps per revolution drives the
table of a drilling machine by one angular
22. A CNC vertical milling machine has to cut a step per each pulse generated by a pulse
straight slot of 10mm width and 2mm depth generator (shown in figure). Each angular
by a cutter of 10mm diameter between points step moves the table by one Basic Length
(0, 0) and ( 100, 100) on the XY plane Unit (BLU) along X axis with a lead screw
(dimensions in mm). The feed rate used for having a pitch of 4mm. If the frequency of
milling is 50mm/min. Milling time for the pulse generator is doubled, the BLU will
slot (in seconds) is (GATE-ME&PI-12) (GATE -PI-14)
(a) 120 (b) 170 (c) 180 (d) 240
CNC drill table

23. For machining a rectangular island


X axis

represented by coordinates P(O,O), Q(l OO,O),


pulses

R(l OO, 50) and S(0,50) on a casting using


Pulse Driver Stepper
generator motor

CNC milling machine, an end mill with a


diameter of 16mm is used. The trajectory of
the cutter centre to machine the island PQRS
(a) become double of previous value
(GATE-ME-14-SET-1)
(b) become half of previous value
lS

(c) remain the same


(b) (8,8), (94,8), (94,44), (8,44), (8,8)
(a) (-8,-8), ( 1 08,-8), ( 1 05,58), (-8,58), (-8,-8)

(d) became zero


(c) (-8,8), (94,0), (94,44), (8,44), (-8,8)
(d) (0,0), ( 100,0), ( 100,50), (50,0), (0,0)

!ltlli!Mhi44mi@RftbiMiiiih�)ttyderabad I Delhi I Bhopal I Pune I Bhubaneswar I Lucknow I Patna I Bengalwu I Chennai I Vtjayawada ! VJ.Zag I Tuupati I Kukatpally I Kolkata l
�'Ii'....:......:... Publicatiom
ACE : 843 : Advanced Machining Methods
:.-:.
..... ""=
11

� === ... ================================


26. The function of interpolator m a CNC (a) (3, 12) (b) (5, 7)
machine controller is to (c) (7, 12) (d) (4, 7)
(GATE -15 -Set 1)
(a) control spindle speed 29. In Computer Aided Process Planning,
(b) coordinate feed rates of axes. determination of process sequence for
(c) control tool rapid approach speed. manufacture of any part design without
(d) perform Miscellaneous (M) functions predefined standard plans is known as
(tool change , coolant control etc.) (GATE - PI -15)
(a) variant type process planning
27. During the development of a product, an (b) retrieval type process planning
entirely new process plan is made based on (c) generative type process planning
design logic, examination of geometry and (d) group technology based process planning
tolerance information. This type of process
planning is known as 30. Match the following part programming codes
(GATE -15 -Set 2) with their respective functions
(a) Retrieval
(b) Generative Part Programming Functions
(c) Variant Codes
(d) Group technology based P. GO l I. Spindle stop
Q. G03 II. Spindle rotation,

28. A drill is positioned at point P and it has to clockwise

proceed to point Q. The coordinates of point


R. M03 III. Circular interpolation,

Q in the incremental system of defining


anticlockwise

position of a point in CNC part program will


S. MOS IV. Linear interpolation

be. (GATE - 1 6 -SET - 3)


(a) P- II, Q - I, R - IV, S - III
y

-------- i -; ----- t ------- (b) P - IV, Q - II, R - III, S - I


I I

: : Q
(c) P - IV, Q - III, R - II, S - I
T l_ - -----•--------:- -------
12 (d) P - III, Q - IV, R - II, S - I
5
I I
I

X
l
I

•I
I
4
I• 3
(GATE -15 -Set 3)
!IS•li@OIMh/4@ftiM\hijj� yderabad I Delhi I Bhopal I Pune I Bhubaneswar I Lucknow J Pama J Bengaluru J Chennai I Vijayawada J Vizag I TU"Upati I Kukatpal)y I Kolkata
ACE : 844 : Production
��-�
}EogioerJingP.Jblicarioos
3 1. Consider the following statements. lead screw is 3.6mm, what is the expected
(P) Computer aided process planning positioning accuracy? (GATE-ME-97)
(CAPP) takes input from material (a) 1 micron (b) 10 micron
requirement plan (MRP). (c) 50 micron (d) 100 micron
(Q) Production flow analysis helps in work
cell formation. 03. A cylinder of 155mm diameter is to be
(R) Group technology takes input from reduced to 150mm diameter in one turning
choice of machining or cutting cut with a feed of 0.15mm/rev and a cutting
parameters. speed of 150m/min on a NC lathe. What are
Among the above statements, the correct the programmed feed rate and the material
one(s) is (are) (GATE - PI-16) removal rate. (GATE-ME-98)
(a) P only (b) Q and R only
(c) P and R only (d) Q only 04. In finish machining of an island on a casting
with CNC milling machine, an end mill with
10mm diameter is employed. The corner
Two Marks Questions points of the island are represented by (0,0),
(0, 30), (50, 30), and (50, 0). By applying
0 1. With reference to NC machine, which of the cutter radius compensation, the trajectory of
following statement is wrong? the cutter will be (GATE-ME-00)
(GATE-ME-93) (a) (-5,0) , (-5,35),(55,35) ,(55,-5),(-5, -5)
(a) Both closed loop and open loop control (b) (0,-5) ,(55,-5) ,(55,35),(-5, 35) , (-5, -5)
systems are used (C) (5,5),(5,25),(45,25),(45,5),(5,5)
(b) Paper tapes , floppy drives and cassettes (d) (5,5),(45,5),(45,25),(5,25),(5,5 )
are used for data storage
(c) Digitisers may be used as interactive 05. During the execution of a CNC part program
input devices block
(d) Post-processor is an item of hardware N020 G02 X45.0 Y25.0 R5.0. The type of
tool motion will be (GATE-ME-04)
02. In a point to point control NC m/c, the slides (a) circular Interpolation - Clockwise
are positioned by an integrally mounted (b) Circular Interpolation-counterclockwise
stepper motor drive. If the specification of (c) linear Interpolation
the motor is l deg/pulse, and the pitch of the (d) rapid feed

!1Sfl@j@mi4,iji4Rftnifflmjj,� yderabad I Delhi I Bhopal I Pune I Bhubaneswar I Lucknow I Patna I Bengalwu I Chennai I Vijayawada I Vuag I 1Irupati I Kukatpally I Kolkata
ACE
:.�

.. "=
:�.11E�ngmg' :m:Pffll�· ig�Pu��blications�
�·� · = 8 45 : Advanced Machinin
· ·�g,;;M;;;e:th
========�; ;: � ======:::,:;,,:,::,:::�::;;;:,:: = ::;s;,,,
od

06. In a 2-D CAD package, clockwise circular and lead screw-nut mechanism (pitch = 4mm,
arc of radius 5, specified from P 1 (15,10) to number of starts = 1). The gear ratio = Output
P2 (10,15) will have its center at speed / input speed is given by U = 1/4,. The
(GATE-ME-04) stepper motor (driven by voltage pulses from a
(a) (10, 10) (b) (15, 10) pulse generator) executes 1 step / pulse of the
(c) (15, 1 5) (d) (10, 1 5) generator. The frequency of the pulse train from
the pulse generator is f = 10000 pulses per
07. Which among the NC operations give below minute.
are continuous path operations? Arc Welding
Pulse
(A W), Milling ( M), Drilling (D), Punching
Stepper
in Sheet Metal (P), L aser Cutting of Sheet motor

Metal (LC), Spot Welding (SW).


(GATE-ME-OS)
(a) AW, LC andM (b) AW, D, LC and M 09. The basic length unit ( BLU ) , i.e. the table
(c) D, LC, P and SW (d) D, LC, and SW movement corresponding to 1 pulse of the
pulse generator (GATE-ME-08)
08. The tool of an NC machine has to move (a) 0.5 microns (b) 5 Microns
along a circular arc from ( 5, 5) to (10, 10) (c) 50 Microns (d) 500 Microns
while performing an operation. The centre of
the arc is at (10, 5). Which one of the 10. A customer insists on a modification to
following NC tool path commands performs change the BLU of the CNC drive to
the above mentioned operation ? 1OMicrons without changing the table speed.
(GATE-ME-OS) The modification can be accomplished by
(a) NOlO G02 XlO YlO X5 Y5 R5 (GATE-ME-08)
(b) NOlO G03 XlO YlO X5 Y5 R5 (a) Changing U to½ and reducing f to f/2
(c) NOlO GOl X5 Y5 XlO YlO R5 (b) Changing U to 1/8 and increasing f to 2f
(d) NOlO G02 X5 Y5 XlO YlO R5 (c) Changing U to ½ and keeping f
unchanged
Statement for common data Q. 09 & Q. 10: (d) Keeping U unchanged and increasing f
In the feed drive of a point to point open loop to 2f
CNC drive, a stepper motor rotating at
200steps /rev drives a table through a gear box

IM•li@m44im4QnftiM\ilijj� yderabad I Delhi I Bhopal I Pune I Bhubaneswar I Lucknow I Patna I Bengalwu I Chennai I Vyayawada IVizag I TlfUpati I Kukatpally I Kolkata
.!'.t
'!.,.
ACE
lEngineering Pnbticarioos :846: Production

11. Match the following (GATE-ME-09) (c) simultaneous programmable movement


along 3 axes in both directions
NC Code Definition
(d) simultaneous programmable movement
P. M05 1. Absolute coordinate system
along 3 axes but only in one direction
Q. GOl 2. Dwell
R. G04 3. Spindle stop
14. The component development and
S. G90 4. Linear interpolation
manufacturing activities when assisted by a
computer are: Computer Aided Graphics
(a) P-2, Q-3, R-4, S-1
(CAG), CAD, CAM, CAPP and Computer
(b) P-3, Q-4, R-1, S-2
Aided Quality Control (CAQC). The most
(c) P-3, Q-4, R-2, S-1
appropriate sequence of these activities is
(d) P-4, Q-3, R-2, S-1
(GATE-PI-92)
(a) CAD-CAG-CAPP-CAM-CAQC
12. In a CNC program block, N002 G02 G91
(b) CAG-CAD-CAM-CAPP-CAQC
X40 Z40.., G02 & G91 refers to
(c) CAD-CAG-CAPP-CAQC-CAM
(GATE-ME-10)
(d) CAD-CAPP-CAG-CAM-CAQC
(a) Circular interpolation counterclockwise
direction and Incremental dimension
15. A DC servomotor is directly driving an NC
(b) Circular interpolation counterclockwise
table. The pitch of the lead screw of the table
direction and absolute dimension
is 5 mm. The motor rotates at 100 rpm for an
(c) Circular interpolation clockwise
applied voltage of l OV. if the voltage speed
direction and Incremental dimension
characteristic of the motor is linear, the
(d) Circular interpolation clockwise
applied voltage for a table of 3m/min is equal
direction and absolute dimension
to: (GATE-PI-92)
(a) 30 V (b) 60V
13. A 2½ axis control in a typical CNC machine (c) 33 V (d) 50 V

provides (GATE-PI-90) 16. Match the following (GATE-PI-93)


(a) simultaneous tool movements along A. CNC 1. Zero inventory
2 axes B. MRP 2. Rout sheet
(b) programmable movement along 3 axes C. JI T 3. DNC
but allows simultaneous movement D. FMS 4. Cutter path
along 2 axes 5. Lumped demand
!ffl•IQi@jji§§,h/4@nfti\l@�/" yderabad I Delhi I Bhopal I Pune I Bhubaneswarl Lucknow I Palna I Bengalwu I Chennai I Vijayawada Vizag I Tirupati I Kukatpally I Kolkata J
ACE Advanced Machining Methods
�. �==================================
EF� Poblic:ariooa : 847:
�-�
17. Match the following (GATE-PI-93) 21. For machining the circular arc shown in the
A. CNC 1. Zero inventory figure below on a NC machine, the
B. MRP 2. Stepper motor interpolation parameters (I.K) and the
C. TI T 3. Batch production incremental movements in the direction of
D. FM S 4. Post processor the X and Z axes are programmed as
5. Lumped demand (GATE-PI-04)
z
18. Match the following (GATE-PI-94)
Group - I Group - II
P. G09 1. Linear interpolation M - Starting Point
Q. G41 2. Retardation N -End Point

R. GOl 3. Circular interpolation X

S. G03 4. Cutter radius compensation (a) I= 0, K= R, X= -(R-P) , Z= -Q


(a) P-3, Q-1, R-4, S-2 (b) I= R, K= 0, X= -(R-P) , Z = -Q
(b) P-4, Q-1, R-3, S-2 (c) I= P, K= Q, X= R, Z = R-Q
(c) P-3, Q-4, R-1, S-2 (d) I= Q, K= P, X= (R-P) , Z= R
(d) P-2, Q-4, R-1, S-3
22. Automation Strategy Type of equipment
(GATE-PI-04)
19. When 3-2-1 principle is used to support and
P: Standalone programmable I-Transfer
locate a three dimensional work-piece during
line manufacturing automation
machining, the number of degrees of freedom
Q: Programmable automation with 2 - CNC
that are restricted is (GATE-PI-03)
automatic tool changing
(a) 7 (b) 8 (c) 9 (d) 10
R: Dedicated automation with 3 - FM S
automated material transfer
20. Given that the notations of robot joints are
S: Programmable automation with
L-linear, 0-orthogonal , R-rotational,
4-Machining center programmable
T-twisting, and V-revolving, the anatomy of
material handling and transfer
a polar configuration robot is described by
the alphabetical string (GATE-PI-04) (a) P- 2, Q- 3, R-4, S -1
(a) TOO (b) TLL (b) P- 4, Q- 2, R- 3, S- 1
(c) TRL (d) TVO (c) P- 1, Q- 2, R-3, S-4
(d) P - 2, Q - 4, R - 1, S - 3

!lilli@jjji4ijjjiRflbfti\111n� yderabad I Delhi I Bhopal I Pune I Bhubaneswar I wcknow I Patna I Bengalwu I Chennai I Vijayawada I Vizag I Ttrupati I Kukatpally I Kolkata
: 848: Production

23. Consider the following characteristics : 25. Match the following (GATE-PI-06)
(GATE-PI-OS) Group - I Group - II
1. Single machine tool. P. G0 8 1. Linear interpolation
2. Manual materials handling system. Q. G41 2. Acceleration
3. Computer control. 3. Circular interpolation
R. GOl
4. Random sequencing of part to machines.
S. G02 4. Cutter radius compensation
Which of the above characteristics are
associated with flexible manufacturing (a) P-3, Q-1, R-4, S-2
system? (b) P-4, Q-1, R-3, S-2
(a) l , 2 and 3 (b) l and 2 (c) P-3, Q-4, R-1, S-2
(c) 3 and 4 (d) 2, 3 and 4 (d) P-2, Q-4, R-1, S-3

24. Complete the manual part program for 26. In a CAD package, a point P (6 , 3, 1) is
drilling a hole located at point (20, 20) from
projected along a vector v (- 2, 1, - 1). The
the target point. The depth of the hole to be
projection of this point on X-Y plane will be
drilled is 15mm
(GATE-PI-07)
FROM /TARG
(a) (4, 4, 0 ) (b) ( 8, 2, 0 )
GOT0/20, 20, 0 (GATE-PI-06)
(c) (7, 4, 0 ) (d) ( 2, 5 , 0 )

(a) GODLTA /0,0,-15


27. The geometric transformation specified by
GODLTA /0,0,15
(GATE-PI-07)
GOTO/SETP T
5
(b) GODL TA /0,0,15
GODLTA /0,0, -15
[x y'Jj� [x y J°�
11
0�
2
5 �Jin a 2D.
1
GOTO/SETP T
In a 2D CAD system represents
(c) GOTO/SETP T
(a) Scaling and translation
GODLTA /0,0 ,-15
GODLTA /0,0, 15 (b) scaling and rotation

(d) GOTO/SETP T (c) rotation and translation


GODLTA /0,0, 15 (d) rotation
GODLTA /0,0,-15

!ffl•lii@jjji§jjj,j@njjmjjj� yderabad I Delhi I Bhopal I Pune I Bhubaneswar I Lucknow! Patna I Bengaluru I Chennai JVtiayawada JVu.ag ITuupati I Kukatpally I Kolkata
'!.
:. . l\.CE ..
�t Engioet.rq PnNnbooa : 849: Advanced Machining Methods
� '===================================
28. In a CNC feed drive, a stepper motor with
step angle of 1.8deg drives a lead screw with (b)
pitch of 2mm. The basic length unit (BLU)
for this drive is (GATE-PI-08) y
(a) 1Omicrons (b) 20microns
(c) 40microns (d) 100 microns
X

29. A stepper motor has l 50steps. The output


shaft of the motor is directly coupled to a (c )
lead screw of pitch 4mm, which drives a
table. If the frequency of pulse supply to the y

motor is 200Hz, the speed of the table (in


mm /min) is
LNl
(GATE-PI-OS)
(a) 400 (b) 320 (c) 300 (d) 280
X

LNl
30. Suppose point Pl in AP T (Automatically (d)

programmed Tool) programming is coded by


C

statement
y

Pl =POIN T/XSMALL, IN TOF, LNJ, CRJ


CRl
(GATE-PI-08)
The coded geometric situation without
X

causmg error 1s
. .

31. The total angular movement (in degrees) of a


lead screw with a pitch of 5.0mm to drive the
work table by a distance of 200mm in a NC
machine is
(GATE-PI-09)
(a) 14400 (b) 28800
X (c) 57600 (d) 72000

!1111ii@hi§§,h4Rftnj@\@j� yderabadlDelhi! BhopallPunelBhubaneswarl LucknowlPatnalBengaJurulChennaijVtiayawadalVizag ITirupati I Kukatpallyl Kolkata

..
:850:
...
"" '�.i..
:FqmeermgPnlmdions
.,
-:,�
"'. .A!.. CE . . Production
�=======================================
32. A titanium sheet of 5.0mm thickness is cut 35. For the CNC part programmmg , match
by wire cut EDM process using a wire of Group A with Group B:
1.0mm diameter. A uniform spark gap of
0.5mm on both sides of the wire is Group A Group B
maintained during cutting operation. If the P: Circular interpolation, I: G02
feed rate of the wire into the sheet is counter clock wise
20mm /min, the material removal rate (in Q: Dwell II: G03
mm3 /min) will be (GATE-PI-09) R: Circular interpolation, III: G04
(a) 150 (b) 200 (c) 300 (d) 400 clock wise
S: Point to point IV: GOO
33. For a 3-axes CNC table , the slide along the countering
vertical axis of the table driven by a DC (GATE-ME- 14-SET-2)
servo motor via lead screw nut mechanism.
(a) P- II , Q- III, R- I , S - IV
The lead screw has a pitch of 5mm. this lead
(b) P- I, Q - III , R - II, S - IV
screw is fitted with a relative (incremental )
(c) P - I , Q - IV, R - II, S - III
circular encoder. The basic length unit (BLU)
(d) P- II, Q - I, R - III, S - IV
of the slide along the vertical axis of the table
is 0.005mm. When the table moves along the
36. Each axis of NC machine is driven by a
vertical axis by 9mm. the corresponding
stepper motor drive with a lead screw. The
number of pulses generated by the encoder is
pitch of lead screw is p mm. The step angle
(GATE-Pl-10)
of stepper motor per pulse input is a
(a) 1400 (b) 1800 (c) 4200 (d) 9000
degrees /pulse. The ratio of gear drive in
stepper motor drive is g(number of turns of
34. In a CAD package, mirror image of a 2D
the motor for each single tum of the lead
point P(5,10) is to be obtained about a line
screw). The number of pulses required to
which passes through the origin and makes
achieve a linear movement of x mm is
an angle of 45° counterclockwise with the X­
(GATE-PI- 14)
axis. The coordinates of the transformed
360g
point will be (GATE-ME & PI- 13) (a)�x (b)
360p p
X

(a) (7.5 , 5) (b) (10, 5)


g 360g
(c) (7.5 , -5) (d) (10, -5) (c) _ _x (d)
360p pa
X

!lilli@jjj§§ih/iRflbffMid.jj� yderabad I Delhi I Bhopal I Pune I Bhubaneswar I Lucknow I Patna I Bengaluru I Chennai IVrjayawada IVu.ag !Tirupati I Kukatpal)y I Kolkata
.......,." ... ACE : 851 : Advanced Machining Methods
�.. ...�EogineeriogPubticatious
-�������=======================�===
37. A CNC instruction 091001X30Y40F100 1. Stretch in the y-axis
P. [�
commands the movement of tool along the . �]
path at a feed rate of 1OOmrn/min (091- 2. Uniform stretch in x and y-axis
Q. [� �]
incrememtal format and 001-linear
interpolation). The feed rate of the tool (in 3. Projection in x-axis

mm /min) along the X axis will be ___


R. [� �]
S. [� 4. Projection in y-axis
(GATE-PI- 14)
�]
(GATE - PI-15)
38. A triangular facet in a CAD model has
(a) P-1, Q-2, R-3, S-4
vertices: P1(0,0,0); P2(l , l ,O) and P3( l , l , l ).
(b) P-2, Q-3, R-4, S-1
The area of the facet is
(c) P-3, Q-4, R-1, S-2
(GATE-15-Set 1)
(d) P-4, Q-1, R-2, S-3
(a) 0.500 (b) 0.707
(c) 1.414 (d) 1.732
41. In a Flexible manufacturing system, the
Automated Guided Vehicles (AOV) move at
39. In a CNC milling operation, the tool has to
a speed of 50 m /min, cover an average
machine the circular arc from point (20,20)
to (10,10) at sequence number 5 of the CNC distance of 150 m to deliver and 100 m for
part program. If the center of the arc is at return. If the time required for pick-up and
(20,10) and the machine has incremental drop is 30 s each, neglecting idle times, then
mode of defming position coordinates, the the number of AGVs required to meet the
correct tool path command is demand of 50 delivers per hour is
(GATE-15-Set 3) (GATE - PI -15)
(a) N 05 090 001 X-10 Y-10 RIO
(b) N 05 G91 G03 X-10 Y-10 RIO
42. The figure below represents a triangle PQR
(c) N 05 G90 G03 X20 Y20 R 10.
with initial coordinates of the vertices as
(d) N 05 G91 G02 X20 Y20 RIO
P(l ,3), Q (4,5) and R(5,3.5). The triangle is

40. Match the linear transformation matrices rotated in the X-Y plane about the vertex P
by angle 0 in clockwise direction. If sin 0 =
interpretations in the second column. 0.6 and cos0 = 0.8, the new coordinates of
listed m the first column to their

the vertex Q are

!IDJli•@m§4,jjijj/nnjfjhliii� yderabad I Delhi I Bhopal I Pune I Bhubaneswar I Lucknow! Pama I Bengalwu I Chennai jVtjayawada I Vuag I Tirupati I Kukatpally I Kolkata
'!.�-�
ACE
:&gior.ering PtJblicatiooa : 852: Production
� �·�������============================
45. Circular arc on a part profile is being
y 4,5) machined on a vertical CNC milling
j\
machine. CNC part program using metric
units with absolute dimensions is listed
v
� ,3. )
5 5
below:
P(l ,3)
o�------­

(GATE - 16 - SET - 1) N60 GOl X 30 Y 55 Z 5 F50


x

(a) (4.6, 2.8) (b) (3.2, 4.6) N70 G02 X 50 Y 35 R 20


(c) (7.9, 5.5) (d) (5.5, 7.9) N80 GOl Z 5

43. A point P( l , 3, -5) 1s translated by The coordinates of the centre of the circular
2i + 3j-4k and then rotated counter arc are: (GATE-17-SET-1)
(a) (30, 55) (b) (50, 55)
A A A

clockwise by 90° about the z-axis. The new


(c) (50, 35) (d) (30, 35)
position of the point is
(GATE - 16 - SET - 3)
46. The preparatory and miscellaneous codes
(a) (-6, 3, -9)
used in CNC part programming and the
(c) (6, 3, -9) (d) (6, 3, 9)
(b) (-6, -3, -9)

functions are given in the Table

44. A single axis CNC table is driven by a DC Group I Group II


servo motor that is directly coupled to a lead P. GO l 1. Circular interpolation,
screw of 5 mm pitch. The circular encoder counter-clock wise
attached to the lead screw generates 1000 Q. G03 2. End of program
voltage pulses per revolution of the lead R. M06 3. Tool change
screw. The table moves at a constant speed of S. M02 4. Linear interpolation
6 m /min. The corresponding frequency (in
The correct combination of code and the
kHz) of the voltage pulses generated by the
respective function is
circular encoder is----
(GATE-PI-17)
(a) P-4, Q-1, R-3, S-2
(GATE - PI-16)

(b) P-4, Q-1, R-2, S-3


(c) P-1, Q-4, R-3, S-2
(d) P-2, Q-1, R-3, S-4

!ISlli!ii§h/MiOM@nj@m111� yderabad I Delhi I Bhopal I Pune I Bhubaneswari Luclmowl Pama I Bengaluru I Chennai I Vtiayawada I Vtzag jTlfUpari I Kukatpal]y I Kolkata
: 853: Advanced Machining Methods

47. In a numerical control (NC) machine


positioning system, the measures of
precision are expressed by considering a
single axis as shown in the Figure.

Addressable point

Distribution of
mechanical errors

/=n/2+3cr Linear axis

n/2 n/2

If cr is standard deviation of the error


distribution, the l,m and n are
(GATE -PI- 17)
(a) / = Accuracy, m = Repeatability,
n = Control resolution
(b) l= Repeatability, m = Accuracy,
n = Control resolution
(c) / = Control resolution,
m = Repeatability, n = Accuracy
(d) / = Accuracy,
m = Control resolution,
n = Repeatability

!ltllii@hi§§,jjij(nln!@i@jj.� yderabad I Delhi I Bhopal I Pune I Bhubaneswar I Lucknow I Patna I Bengaluru I Chennai I Vijayawacla I Vi7'lg I Tirupari I Kukatpally I Kolkata
SOLUTIONS
06. Ans: (d)
One Mark Solutions Sol: In CAD practice an arc can be defined by
two ends points and center or center, one
01. Ans: (b) end point and angle of arc, or radius, two
Sol: In CNC part program, the number specified end points and angle etc.
after ' S' indicating the spindle speed in rpm
07. Ans: (c)
02. BATCH, LARGER Sol: In group technology lay out if each and
every cell is independently implemented
03. Ans: (b) with DNC +AGV. If flexible manufacturing
Sol: In PTP only end points of movement of tool system to be implemented in each cell
is important but the path followed by the called flexible manufacturing cell or cellular
tool in between the point is not important, manufacturing.
hence the velocity of tool need not be
controlled. 08. Ans: (a)
Sol: For producing complex shapes , it needs
04. simultaneous control of x, y and z axis.
Sol: TRUE, By using encoder positioned on the
axis the actual distance travelled by thee 09. Ans: (a)
axis can be measured accurately and given Sol: Producing a contour is the path function
it as feed back to the MCU so that MCU can
10. Ans: (b)
generate the pulses required for traveling
Sol: D.C servo motors are used as axial or
remaining distance
spindle drives of CNC machine tools.

05. 1 1 . Ans: (b)


Sol: FALSE, CNC machines are used for
producing complicated jobs which can not 12. Ans: (b)
Sol: C & D. The operations like plain turning,
be produced by normal method and for
face turning, taper turning , thread cutting
batch production.
and drilling are the straight operations only.
!11•1ii@h/§§mi4j;nft/M\iiih� yderabad I Delhi I Bhopal I Pune I Bhubaneswar I Lucknow I Patna I Bengalwu I Chennai I Vijayawada !Vmg I 1:irupati I Kukatpally I Kolkata
" " . ACE
. . . : 855 : Advanced Machining Methods
�-��� Pm,li,,nq
'!,

13 . Ans: (a) 22. Ans: (b)


Sol: Linear and circular are the most commonly Sol: P 1 (0, 0) to P2( l OO, 1 00)
used interpolation method in NC machines. L = ( 1 002 + 1 002)°"5 = 1 4 1 .42mm
fm = 50mm/min
14. CONSISTANCY, HAZARDS. . L 1 4 1 .42 .
T1meIslot = - = -- = 2 . 828mtn
Im 50
1 5 . Ans: (b) = 1 69.7sec = 1 70 sec
Sol: In drilling machine all the motions are PTP
only, hence even though the machine 1s 23. Ans : (a)
CNC , stepper motors can work Sol: Because diameter of milling cutter is
1 6mm, the radius is 8mm. the dotted line
1 6. DNC => Direct or distributed numerical indicates cutter center position, which is
control shifted by 8 mm all around the rectangular
slot
1 7. FALSE , FMS is suited for batch production
(-8,58), - - - - - - - - - - - - - - - - - - - - - - R. � 1 08,58)
(0,50) ( 1 00,50
1 8.
Sol: FALSE, In DNC more than one CNC
machine is connected to a host computer 0,0 ( 1 00,0)
which has bulk memory and is controlling (-8,-8) � - - - - - - - - - - - - - - - - _ _ _ _ _QJ(l 08,-8)
all the CNC machines
If the given shape is rectangular hole, then
1 9. the answer is
Sol: FALSE, Part programming does not have (8,8), (92,8), (92,42), (8,42), (8,8)
any language, and F APT is a programming
language 24. Ans: (b)
Sol:
20. Ans: (a)
Sol: Producing a contour is the path function

2 1 . Ans: (c)
P(O, 0)

. \ C l . l.nginn n n � l'uhlH ,tll< >l1' Fyderabad I Delhi I Bhopal I Pune I Bhubaneswarl Luckn,owl Patna I Benga)uru I Chennai I V\iayawada I V17.3g I Tuupati I Kukatpa)]y I Kolkata
: 856 : Production

25. Ans : (c)


Sol: BLU = the distance traveled by the table for
Two Marks Solutions
one pulse of electrical energy input to the
motor. 0 1 . Ans: (d)
Hence 200 pulse = 1 revolution of motor Sol: In NC machine the post processor is not
= 1 revolution of lead screw = 4mm used and hence graphic simulation of
That is 1 pulse = 4/200 = 1/50 = 0.02mm, program is not possible.
hence BLU does not depends on the
frequency of pulse generator. But if the 02. Ans: (b)
speed of the table means it will get doubled. Sol: For 1 rev of motor �360 ° are required
� 360 pulses are required
26. Ans: (b) When motor is rotated by 1 rev
� lead screw will rotate by 1 rev
27. Ans: (b) When Lead screw is rotated by 1 rev
�3.6mm distance is travelled by axis
28. Ans: (d) In total
Sol: In the incremental system every time For 360 pulses
reference point is previous point � 360 deg of motor

29. Ans: (c)


� 1 rev of motor
� 1 rev of lead screw

30. Ans: (c)


� 3.6mm of linear movement of axis

Sol: GO 1 is used for linear interpolation, G03 is


360 pulses = 3 .6mm
used for circular interpolation counter 1 pulse=3.6/360=0.0 l mm= 1 0 microns
clockwise, M03 for spindle rotation
03.
Sol: Programmed feed rate = f xN
clockwise and M05 for spindle top.

31. Ans: (d)


= (0. 1 5 X 1 000x 1 50)/ (1t X 1 55)
= 46.22mm/min

MRR = f d .V
= 0. 1 5 X 2.5 X 1 50 x l Q3
= 56250mm3

IM•li@jjjiiljjj$jjjjnlM\@j.�)Hyderabad I Delhi I Bhopal I Pune I Bhubanesw.u· I Lucknow I Patna I Beagaluru I Cbennai I Vtiayawada j V17.ag I Tuupari I Kukatpa]ly I Kolkata I
: 857 : Advanced Machining Methods

04. Ans: (a) 07. Ans: (a)


Sol: The center point of cutter must be 5mm Sol: Out of the given operations, AW, LC and M
away from the comer of island because the are the continuous path machining D, P &
radius of tool is 5mm SW are the PTP control machining.

08. Ans: (d)


(- 5 , 3 5 ) ( 5 5 ,3 5 )

Sol: Appropriate answer but the correct answer


A ,)I ,)I B
(0 , 3 0) ( 50 , 3 0)
lS

NOS XS Y5
YlO
(0 , 0) ( 5 0, 0)
D \)I N l O G02 X l O RS
C (5 5 ,-5) Because in CNC part program we are not
(-5 , -5)
suppose to indicate information about one
Path trajectory : A - B - C - D axis more than once in one block.

05. Ans: (a) 09. Ans: (b)


Sol: G02 - circular interpolation clockwise,
1 0. Ans: (a)
Sol: Common Solution for 9 & 10:
G03 circular interpolation counter
clockwise
Stepper motor � 200 steps I rev
06. Ans: (c) � 200 pulses /rev
Sol: Because the tool has to travel from P 1 to P2 Pitch = 4mm, no.of starts = 1 ,
in clock wise, the center will be ( 1 5, 1 5). Gear ratio = No/Ni= 1 /4 = U
F = 1 0000 pulses per min
Y P2 = ( 1 0, 1 5 ) C enter C ( 1 5 , 1 5 ) 200 pulses � 1 rev of motor
(8)
� ¼ rev of lead screw
= 1 /4 x 4 x 1 mm linear distance
= 1 mm linear distance

P 1 = ( 1 5 , 1 0) 1 pulse = 1/200 = 0.005mm


= 5 microns = BLU . . . . . . . . B
X
Feed = BLU x pulse /min
= Q.QQ5 X 1 QQQQ = 5Qmm/min

j1D1ih§jjji4.jjj$Rflb1Mih1U+1yderabad I Delhi j Bhopal j Pune I Bhubaneswarj Lucknow! Patna ! Bengalwu I Chennai IVuayawadajVizag jT1n1pati I Kukatpal)y j Kolkata I
: 858: Production

For changing BLU = 1 0 microns 1 5. Ans: (b)


= O.Ol mm Sol: 1 OV => 1 00 rpm => 1 00 x 5 = 500 mm/min
=> gear ratio has to be reduced to ½ That is for 500mm/min => 1 OV
I mm /min => 1 0/500
Feed = BLU x pulse /min 3000mm/min = 1 0 x 3000 / 500 = 60 V
=> Pulses per min = feed / BLU
= 50/0.01 = 5000 . . . A 1 6. Ans: A-4, B-5, C-1 , D-3

1 7. Ans: A-4, B-5, C- 1 , D-3


1 1 . Ans: (c)
Sol: MOS - used for spindle stop, 1 8. Ans: (d)
GO 1 - used for linear interpolation Sol: G09 - Retardation, 1.e the speed of tool
G04 - used for dwell - temporary stoppage increases slowly and reaches to a maximum
of the tool for specified duration G4 1 Cutter radius compensation left and it
G90 - Used for absolute co-ordinate is used in milling
system, that is programming of GO 1 - Linear interpolation
each block is done with reference G03, Circular interpolation CCW
to only one reference point.
19. Ans: (c)
12. Ans: (c) Sol: In 3-2- 1 principle
Sol: G02 - Circular interpolation CW 3 pins - used in one ..L1r plane is arresting 4
G9 1 - Incremental mode of programming in rotation DOF and I linear DOF
which each block is programmed by taking 2 pms - used in second ..L Ir plane is
the present position as reference point. arresting 2 rotation DOF and 1
linear DOF
1 3 . Ans: (b)
3 pins - used in one ..L Ir plane is arresting 1
Sol: Used for producing cylindrical cams.
linear DOF
14. Ans: (a) So that in total 6 rotational DOF and 3
Sol: Whenever a component is to be developed linear DOF will be arrested by using 3-2- 1
first it has to be designed , next graphics to principle.
generate production drawings, next process 20. Ans: (c)
planning for deciding sequence of Sol: It is a standard anatomy of polar
operations, next comes to quality control. configuration
\( l } l l�llll l 1 11 11.,\ P11lil1t ,t(JOII", ydcrabad I Delhi I Bhopal I Pune I Bhubaneswar I Lucknow! Patna I Bcngaluru I Cheruiai I Vuayawada I Vu.ag I Tuupari I Kukatpally j Kolkata
: 859 : Advanced Machining Methods

21. Ans: (b) 22. Ans: (d) 28. Ans: (a)


Sol: 1 pulse = 1.8deg
23. Ans: (c) For 360 deg = 1 x 360 /1 .8 = 200 pulse.
Sol: The FMS is nothing but DNC with AGV, 200 pulse = 1 rev of motor
that is many machines are connected to a 1 pulse = (1 /200 )x 2
host computer and for flexible path of = 2/200 = 0.01mm = 1 0 microns
movement of material AGV can be used.
29. Ans: (b)
24. Ans: (a) Sol: 1 50 pulse � 1 rev of motor
Sol: GODL TA indicates incremental � 1 rev of lead screw � 4mm
movement, once the tool went to the 1 pulse = 4 /1 50 = 0.0267mm
required point, it has to drill the hole to 200 pulses = 0.0267 x 200
depth 1 5 units ( v- e) , bring it back and then = 5.33mm /sec = 320mm /min
send the tool to set point.
30. Ans: (b)
25. Ans: (d) Sol: Tie comes between B & C, but in C the
Sol: G08-Acceleration, 1.e the speed of tool value of x-is large and in B the value of x-is
small.
increases slowly and reaches to a maximum
G41 Cutter radius compensation left and it
31. Ans: (a)
is used in milling
Sol: Pitch of lead screw = 5mm
GO 1 - Linear interpolation
1 rev = 5mm
G02, Circular interpolation CW 1 mm = 1 /5 rev
200mm = 1 /5 x 200 = 40rev
26. Ans: (a) = 40 X 360 = 1 4400 deg.
Sol: x = 6 - 2 = 4
y = 3 + 1= 4
32. Ans: (b)
z= l-1 0
Sol: Cross sectional area of slot = 2x5 = 10 mm2 ,
=

Feed rate = 20mm /min


27. Ans: (c)
MRR = C.S area x feed rate
= 200 mm3 /min

!M11i@ml4ijjijj;fle1Mijj.m.� yderabad I Delhi I Bhopal I Pune I Bhubaneswar I Lucknow I Patna I Bengaluru I Chennai I Vijayawada I Vizag I Tirupati I Kukatpally I Kolkata
"' . ACE
. . . : 860 : Production
.. . l&pccrmg PnNnrioos
-:.�-�
.,

33. Ans: (b) g 360


= - x - pulses
Sol: Pitch of lead screw = 5mm, p a
BLU = 0.005mm � distance travelled /pulse 360
For moving "x" distance = g x pulses
Length of travel = 9mm ap
No. of pulses = L /BLU
= 9 / 0.005 = 1800 pulse 37. Ans: 60
Sol: In the combined movement, the tool 1s
34. Ans: (b) moving for 50mm with a speed of
Sol: 1OOmm /min. whereas in the same time tool
is traveling x-axis by only 30mm. hence
For 50mm �l OOmm /min
l OO
For 30mm � x 30 = 60mm /min
50
35. Ans: (a)
Sol: 002- Circular interpolation clockwise, 38. Ans: (b)
003 - Circular interpolation Counter Sol: The triangle formed will have side = ./2
clockwise and height = 1
004 - Dwell ( Temporary stoppage for
Area = _!_ x base x height
specified duration) 2
GOO - Rapid traverse (For automatic 1
1- x ,Ji x l = -- = 0.707
selection of maximum possible
=
2
speed)
Ji
( or )
P1 P2 = Ji = a
36. Ans:(d)
P2 P3 = 1 = b
Sol: Step angle = a deg /pulse
P3 P1 = FJ = c
1 rev of the lead screw p mm g rev. of
Here a, b, c are the three sides of triangle
= =

the motor
.·. /11eArea = �s(s-a)( s-b)(s-c)
1 rev. of the motor = £. mm
where s = --- = 2.073
g
g
a +b+c
1mm distance = rev of the motor 2
p Area = �2.073(2.073 - 1 .4 1 4)(2. 703 - 1)(2. 703 - 1 .732)
g 0.707
= x 360 degrees
p
=

!M11ih§h14lmi4Rflni1\iijjj� yderabad I Delhi I Bhopal I Pune I Bhubaneswar I Lucknow I Patna I Bengaluru I Chennai I Vijayawada I Vmg I Tirupati I Kukatpally I Kolkata
: 861 : Advanced Machining Methods

39. Ans: (b) PQ = -J22 + 32 3.6055 = PC


Sol: N05 � Block (or) sequence number
=

PD = PC x cos 3.2 = 3.6


G9 l � Incremental mode
x co-ordinate of point C = 1 + 3.6 = 4.6
G03 � Counter clockwise DC = 3.6 sin 3.2 = 0.2
(20,2) y co-ordinate of point C= 3.0- 0.2 = 2.8

43. Ans: (a)


Sol: "P" after translation = ( 1 +2, 3+3, -5 -4)
= (3, 6, -9)
( 1 0, 1 0) C (20, 1 0)

Rotation about z- axis means


40. Ans: (c) x' cos 8 -sin 8 0 0 x
y' sin S cos S O O y
4 1. Ans: 5
z' 0 0 1 0 z
Sol: Speed = 50m/min,
1 0 0 0 1 1
. (1 50 + 1 00)
Time per component = �--� 0 -1 0 0 3
50
250 1 0 0 0 6
= = 5min =
50 0 0 1 0 -9
' /

Total time need for 50 deliveries 0 0 0 1 1


= 50 x 5 = 250 min 0-6 + 0 + 0 -6
Time given = 30 + 30 = 60 sec = l min 3+ 0 + 0+ 0 3
= =
Total time given = 50 x 1 = 50 min 0 + 0-9 + 0 -9
250 O+O+O+l 1
Number of AGV's required = =5
50 Final point = [�, 3, -9]

42. Ans: (a) 44. Ans: 20


Sol: Sol: p = 5 mm
1 000 pulses � 1 rev of motor
y
Q (4,5)

� 1 rev of lead screw


Velocity of table = 6 m /min
= 6000 mm/min
1 00 mm /sec
o �--------- =

!lff lii@OiMhiiRflfliMjiijjj+yderab
ad I Delhi I Bhopal I Pune I Bhubaneswar I Lucknow I Patna I Bengaluru I Chennai I Vgayawada I V,zag I Tirupali I Kukatpally I Kolkata I
: 862 : Production

1000 pulses � 1 rev of lead screw � 5 mm 46. Ans: (a)


5 Sol: GO1 - linear interpolation, G03 - circular
l pulse � -- = 0.005mm
1000 interpolation CCW , M06 - tool change and
BLU= 0.005 mm M02 - end of main program written without
Table speed= BLU x Rate of Pulses use of sub program.
1 00
Rate of pulses = --
0.005 47. Ans: (a)
= 20000 pulses /sec
= 20000 Hz
= 20 kHz

45. Ans: (d)


Sol:

®- - - - - - - - (50, 35)
(30,35)

!lilli!@jji4§jjj/4Rflft/M\11jjj� yderabad I Delhi ! Bhopal J Pune J Bhubaneswari Lucknow I Patna I Benga)uru I Chennai J Vijayawada j Vmg J Tirupati I Kukatpal]y J Kolkata
Non- Traditional
Machining Methods
05. In ultrasonic machining process, the material
One Mark Questions removal rate will be higher for material with
(GATE-ME-93)
0 1 . In ultrasonic machining the tool . . . . . . .. at (a) Higher toughness
very high frequency with the help of . . . . . . . . . . (b) Higher ductility
Transducers. (GATE-ME-87) (c) Lower toughness
(d) Higher fracture strain
02. ECM . . . . . . . (Can/Cannot) be used for all
such materials for which ultrasonic 06. EDM imposes larger forces on tool than
machining is possible, while EDM . . . . . . . . ECM (T/F) (GATE-ME-94)
(Can/Cannot) be used for all such materials
for which ECM is possible. 07. EDM is more efficient process than ECM for
(GATE-ME-87) producing large non-circular holes (T/F)
(GATE-ME-94)
03. In USM the metal removal rate
would. . . . . . . . . . . . with increasing mean grain
08. USM is about the best process for making
diameter of the abrasive material
holes in glass which are comparable in size
(GATE-ME-92)
(a) increase with thickness of the sheet. (True / False).
(b) decrease (GATE-ME-94)
(c) increase and then decrease
(d) decrease and then increase 09. Selection of electrolyte for ECM is as
follows (GATE-ME-97)
04. The two main criteria for selecting the (a) non-passivating electrolyte for stock
electrolyte in ECM is that the electrolyte removal and passivating electrolyte for
should (GATE-ME-92) finish control
(a) chemically stable (b) passivating electrolyte for stock removal
(b) not allow dissolution of cathode material and non-passivating electrolyte for finish
(c) not allow dissolution of anode material control
(d) have high electrical conductivity

!1D1i@jjj4i.jji4RflGiMijj.jj+yderabad I Delhi I Bhopal I Pune I Bhubaneswarl LucknowI Patna l Benga)uru I Chennai IVtjayawadajVuag I Tuupati I Kukatpally ! Kolkata I
Production

(c) selection of electrolyte is dependent on 13. Deep hole drilling of small diameter, say
current density 0.2mm is done with EDM by selecting the
(d) electrolyte selection IS based on tool­ tool material as (GATE-ME-00)
work electrodes. (a) Copper wire
(b) Tungsten wire
1 0. Inter electrode gap in ECG is controlled by (c) Brass wire
(GATE-ME-97) (d) Tungsten carbide
(a) controlling the pressure of electrolyte
flow 14. In ECM the MRR is due to
(b) controlling the applied static load (GATE-ME-01)
(c) controlling the size of diamond particle (a) Corrosion (b) erosion
in the wheel (c) Fusion (d) Ion displacement
(d) controlling the texture of the work
15. As tool and work are not in contact in EDM
1 1 . In EDM, the tool is made of process (GATE-ME-03)
(GATE-ME-99) (a) no relative motion occurs between them
(a) Copper (b) HSS (b) no wear of tool occurs
(c) Cast iron (d) Plain carbon steel (c) no power is consumed during metal
cutting
12. Choose the correct statement (d) no force between tool and work occurs
(GATE-ME-99)
(a) Fixture is to guide the tool as well as to 16. When a cylinder is located in a Vee-block ,
locate and clamp the work piece the number of degrees of freedom which are
(b) Jig is to guide the tool as well as to arrested is (GATE-ME-03)
locate and clamp the work piece (a) 2 (b) 4 (c) 7 (d) 8
(c) Jigs are used on CNC machines to locate
and clamp the work piece and guide the 17. The mechanism of material removal in EDM
tool process 1s
(d) No arrangement to guide the tool Is (a) Melting and Evaporation
(GATE-ME-04)

provided in jig (b) Melting and Corrosion


(c) Erosion and Cavitation
(d) Cavitation and Evaporation

l@•l4j@jjji4@jRflftiM@jj� yderabad I Delhi I Bhopal I Pune I Bhubaneswar l Lucknow I Patna I Beogaluru I Chennai I Vtjayawada l Vmg I Ttrupati I Kukatpally I K.olkata
: 865 : Non Traditional Machining Methods

18. Arrange the processes in the increasing order 23. Fixtures are used in batch production for
of their maximum material removal rate (GATE-PI-90)
(GATE-ME-06) (a) Clamping the job
(a) USM, LBM, EBM, EDM, ECM (b) Supporting and clamping the job
(b) EBM, LBM, USM, ECM, EDM (c) Supporting and clamping the job and
(c) LBM, EBM, USM, ECM, EDM guiding the tool
(d) LBM, EBM, U SM, EDM, ECM (d) Supporting, locating and clamping the
job.
19. In EDM, if the thermal conductivity of tool is
high and the specific heat of work piece is 24. A 10 mm square hole can be cut in a 2mm
low, then the tool wear rate and material thick glass plate by (GATE-PI-91)
removal rate are expected to be respectively. (a) USM (b) EDM (c) LBM (d) ECM
(GATE-ME-07)
(a) high and high (b) low and low 25. The use of fixture reduces
(c) High and low (d) low and high (GATE-PI-91)
(a) Only operation time
20. In ultrasonic machining process, the material (b) Tooling cost
removal rate will be higher for material with (c) Only setting time
(GATE-ME-10) (d) Both setting time and operation
(a) Higher toughness
(b) Higher ductility 26. In PM process of manufacturing maximum
(c) Lower toughness temp is associated with (GATE-PI-91)
(d) Higher fracture strain (a) Briquetting (b) Sintering
2 1 . The operation in which oil is permeated into (c) Pre-sintering (d) Blending
the pores of a powder metallurgy product is
27. The process of shot peening increases fatigue
known as (GATE-ME-11)
life of steel springs mainly because it results
(a) mixing (b) sintering
(GATE-PI-91)
(c) impregnation (d) infiltration
m
(a) Hardening of surface
22. The maximum depth to width ratio in EBM (b) Increased stiffness of material
welding is (GATE-PI-89) (c) Compressive residual stresses.
(a) 1:1 (b) 2:1 (c) 10:1 (d) 25: 1 (d) Microstructural changes in material

!IHIQ@jjjiiijjjQRflijjMbjjjj� yderabad I Delhi I Bhopal I Pune I Bhubaneswar I Lucknow I Patna I Bengalwu I Chennai I Vijayawada I Vizag I Tirupati I Kukatpally I Kolkata
Production

28. Electrochemical grinding removes material 33. Ultrasonic machines used m material
by . . . . . . A . . . . And . . . . . . B . . . processes. removal process, reqmres ultrasonic
(GATE-PI-92) transducers.
A. Electrochemical / Chemical The transducers work on different working
B. Mechanical / Chemical principles. One of the working principles of
such ultrasonic transducers is based on
29. Assertion (A): In electron beam welding
(GATE-PI-10)
process, vacuum 1s an essential process
(a) Eddy current effect
parameter.
(b) Seebeck effect
Reason (R): Vacuum provides a highly
(c) Piezoresistive effect
efficient shield on weld zone.
(d) Piezoelctric effect
(GATE-PI-05)

30. Arrange the processes in the increasing order 34. Keeping all other parameters unchanged, the
of their maximum material removal rate tool wear in electrical discharge machining
(GATE-PI-OS) (EDM) would be less if the tool material has
(a) USM, LBM, EBM, EDM, ECM (GATE-PI-10)
(b) EBM, LBM, USM, ECM, EDM (a) High thermal conductivity and high
(c) LBM, EBM, USM, ECM, EDM specific heat
(d) LBM, EBM, USM, EDM, ECM (b) High thermal conductivity and low
specific heat
3 1 . Which one of the following process (c) Low thermal conductivity and low
conditions leads to higher MRR in ECM? specific heat
(GATE-PI-07) (d) Low thermal conductivity and high
(a) Higher current, large atomic weight specific heat
(b) Higher valency, lower current
( c) Lower atomic weight, lower valency 35. In the 3 -2-1 principle of fixture design, 3
(d) Higher valency, lower atomic weight. refers to the number of (GATE-PI-13)
(a) clamps required
32. In an AJM process, if Q = flow rate abrasives
(b) locators on the primary datum face
and d = the mean diameter of the abrasives,
(c) degrees of freedom of the workplace
then MRR is proportional to
(d) operations carried out on the
(GATE-PI-07)
primary datum face
(a) Q/d 2
(b) Qd (c) Qd 2
(d) Qd3
!ltlli!i§OiiiihiAbbi@jj.jjij.liyderabad I Delhi I Bhopal I Punc I Bhubancswar I Lucknow I Patna I Bcngaluru I Chennai I Vuayawada j Vuag I Tuupati I Kukatpally I KolkataI
: 867 : Non Traditional Machining Methods

36 . In abrasive jet machining, as the distance 39. The principle of material removal in
between the nozzle tip and the work surface Electrochemical machining is
increases, the material removal rate. (GATE-ME- 14-SET-4)

(GATE-ME & PI- 12) (a) Fick's law (b) Faraday's laws
(c) Kirchhoffs laws (d) Ohm's law
(a) increases continuously
(b) decreases continuously
40. Find the correct combination of
(c) decreases, becomes stable and then
manufacturing processes to produce the part,
mcreases. shown in figure, from a blank (holes shown
(d) mcreases, becomes stable and then are with square and circular cross-sections).
deceases

37. The following four unconventional


machining processes are available in a shop
floor. The most appropriate one to drill a (GATE-PI- 14)
Blank Final part

hole of square cross section of 6 mm x 6 mm (a) Drilling and milling on column and knee
type universal milling machine
and 25 mm deep is
(b) Die-sinking and CNC Wire-cut EDM
(GATE-ME- 14-SET-2)
process
(a) Abrasive Jet Machining
(c) Die-sinking and CNC drilling
(b) Plasma Arc Machining
(d) CNC Wire-cut EDM process only
(c) Laser Beam Machining
(d) Electro Discharge Machining
41. The primary mechanism of material removal
38. The process utilizing mainly thermal energy in electrochemical machining (ECM) is
(GATE - 15-Set 2)
for removing material is
(a) chemical corrosion
(GATE-ME- 14-SET-3)
(b) etching
(a) Ultrasonic Machining
(c) ionic dissolution
(b) Electrochemical Machining
(d) spark erosion
(c) Abrasive Jet Machining
(d) Laser Beam Machining

!ISlli@jj/i4.jjjj@Cft46111u� yderabad I Delhi I Bhopal I Pune I Bhubaneswar I Lucknow I Patna I Bengalwu I Chermai I Vijayawada I Vizag I Tirupati I Kukatpally I Kolkata
Production

42. Which of the following DO NOT influence 45. In a wire-cut EDM process the necessary
the material removal rate in Electrical conditions that have to be met for making a
Discharge Machine process ? successful cut are that
(i) Hardness of work piece material (GATE - 16 - SET-3)
(ii) Melting temperature of work piece (a) wire and sample are electrically non-
material
conducting
(iii) Hardness of tool material
(b) wire and sample are electrically
(iv) Discharge current and frequency
conducting
, (GATE - PI -15)
(a) (i) and (ii) (b) (i) and (iii) (c) wire is electrically conducting and
(c) (iii) and (iv) (d) (i),(ii) and (iii) sample electrically non-conducting
(d) sample is electrically conducting and
43. The non-traditional machining process that wire is electrically non-conducting
essentially requires vacuum is
(GATE - 16 - SET- 1) 46. Consider the following statements.
(a) electron beam machining (P) Electrolyte is used in Electro -chemical
(b) electro chemical machining
machining.
(c) electro chemical discharge machining
(Q) Electrolyte is used m Electrical
(d) electro discharge machining
discharge machining.

44. In an ultrasonic machining (USM) process, (R) Abrasive-slurry is used m Ultrasonic


the material removal rate (MRR) is plotted as machining.
a function of the feed force of the USM tool. (S) Abrasive-slurry is used in Abrasive jet
With increasing feed force, the MRR exhibits machining.
the following behaviour: Among the above statements, the correct
(GATE - 16 - SET - 2) ones are (GATE - PI-16)
(a) increases linearly
(a) P and R only
(b) decreases linearly
(b) Q and S only
(c) does not change
(c) Q, R and S only
(d) first increases and then decreases
(d) P and Q only

!ltl1i@jjji4ijjj$RflftjM\,j.jj� yderabad I Dellii I Bhopal I Pune I Bhubaneswar I Lucknow ! Patna I Bengalwu I Cheruiai j Vtjayawada I Vizag I Tuupati J Kukatpally I Kolkata
: 869 : Non Traditional Machining Methods

47. Match

Process
characteristics.
the processes

Characteristics
with their
I Two Marks Questions I
P: Electrical Discharge 1 . No residual stress 01. In ECM , the material removal rate will be
Machining higher for metal with
Q . Ultrasonic 2. Machining of electrically (GATE-ME-89)
(a) large density
Machining conductive materials

(b) larger valency


R. Chemical Machining 3 . Machining of glass

(c) Larger chemical absorption tendency


S. Ion B eam Machining 4 . Nano-machining

(GATE - 17 - SET- 1) (d) large chemical weight


(a) P-2, Q-3, R-1, S-4 (b) P-3, Q-2, R-1, S-4
(c) P-3, Q-2, R-4, S-1 (d) P-2, Q-4, R-3, S-1 02. Match the following lists
(GATE-ME-98)
48. Which one of the following statements is List-I List-II
TRUE for the ultrasonic machining (USM) A. ECM 1. Plastic shear
process? (GATE- 17- SET - 2) B. EDM 2. Errosion/brittle fracture
(a) In USM, the tool vibrates at subsonic C. U SM 3. Corrosive reaction
frequency. D. LBM 4. Melting vaporization
(b) USM does not employ magnetostrictive 5. Ion displacement
transducer. 6. Plastic shear and 10n
(c) USM 1s an excellent process for displacement
machining ductile materials.
03. Match the following lists (GATE-ME-98)
(d) USM often uses a slurry comprising
List-I
abrasive-particles and water.
A. Aluminium brake shoe
49. In chemical machining, the etch factor is B. Plastic water bottles
expressed as (GATE- PI - 17) C. Stainless steel cups
undercut depth of cut D. Soft drink can (Aluminum)
(a) (b)
depth of cut undercut List-II
workpiece wear too� wear 1 . Deep drawing 2. Blow molding
(c) (d) 3. Centrifugal casting 4. Impact extrusion
tool wear workpiece wear
5. Sand casting 6. Upset forging

!lif1ij@jjji4ijjj4RflnlM\n.jj*/:yderabad I Delhi I Bhopal I Pune I Bhubaneswar I Lucknow I Patna I Bengaluru I Chennai IVliayawada!Vizag I Tuupari I Kukatpally I Kolkata I
" ..........:........:.Pubtiaoooa : 870 : Production
:.t ACE
� �-�
----�-----------------------------------------------------------------
04. 3-2-1 method of location m Jig or fixture 3. Electric Discharge Machining
would collectively restrict the work piece in 4. Ultrasonic Machining
'n' degrees of freedom, where the value of 5. Laser beam Machining
'n' is (GATE-ME-01) 6. Electrochemical Grinding
(a) 6 (b) 8 (c) 9 (d) 12 (a) P-1,Q-5,R-3, S-4 (b) P-1,Q-4,R-1, S-2
(c) P-5,Q-1,R-2, S-6 (d) P-2,Q-3,R-5, S-6
05. Estimate the MRR (in cc /hr) of an alloy
containing 18% cobalt, 62% nickel and 20% 07. A zigzag cavity in a block of high strength
chromium during ECM with a current of alloy is to be finish machined.
500Amps. The density of the alloy is This can be carried out by using
8.28g /cc. The following data is available (GATE-ME-05)
(GATE-ME-02)

Gram atomic
Metal Valency
weight
Cobalt 58.93 2
(a) Electric discharge machining
Nickel 58.71 2
(b) Electro-chemical machining
Chromium 51.99 6
(c) Laser beam machining
Assume Faraday's constant as 96500
(d) Abrasive flow machining
coulombs /mole.

08. Match the suitable manufacturing processes


06. Typical machining operations are to be
for the following parts. (GATE-ME-07)
performed on hard-to-machine materials by
using the processes listed below. Choose best Parts Manufacturing
set of operation-Process combination processes
Operation Process (GATE-ME-04) P. Computer chip l . ECM
P. De-burring (internal surface) Q. Metal forming dies
Q. Die sinking and tools 2. USM
R. Fine hole drilling in thin sheets R. Turbine blades 3. EDM
S. Tool sharpening
S. Glass 4. Photochemical
1. Plasma Arc machining machining
2. Abrasive Flow machining
!1Dl@@jjjY4,h§j;flCj$fojj,j.� .yderabad I Delhi I Bhopal I Pune I Bhubaneswar I Lucknow I Patna I Bengaluru I Chennai I Vijayawada I Vizag J Tirupati I Kukatpally I Kalka.ta I
/
ACE
�...

J
F wi
..� ·
P.1'111 Puhtiaoooa
�-;;; ;;;;;;;;
• :'

8_
:W:�;;;;;' �iiiiiiiiiiiiiiiiiiiiiiiiiiiiiiiiiiiiiiiiiiiiiiiiiii:;;;
�·
7 iiiiii N_o_
I : iiiiiiiiiiiiiiiiiiiiiiiiiii n_T iiiiii_·ti_o_
_radi n_
a_
IM a_
chining
iiiiii · iiiiiiiiiiiiiM
_·iiiiii __ etb od s
iiiiii
iiiiii

Machining process Mechanism of


(a) P-4, Q-3, R- 1 , S-2

material removal
(b) P-4, Q-3, R-2, S- 1

P. Chemical machining 1. Erosion


(c) P-3, Q- 1 , R-4, S-2
(d) P- 1 , Q-2, R-4, S-3
Q . Electro-chemical 2. Corrosive
machining reaction
09. A researcher conducts ECM on a binary alloy
R. Electro-discharge 3 . Iron
(density 6000kg/m3 ) of iron (atomic weight
machining displacement
56, valency 2) and metal P (atomic weight
S. Ultrasonic machining 4. Fusion&
24, valency 4). Faradays constant 96500
vaporization
coulombs/mole. Volumetric material removal
rate of the alloy is 50mm3Is at a current of (a) P-2, Q-3, R-4, S- 1
2000A. the percentage of the metal P in the (b) P-2, Q-4, R-3, S- 1
alloy is closest to (GATE-ME-08) (c) P-3, Q-2, R-4, S- 1
(a) 40 (b) 25 (c) 1 5 (d) 79 (d) P-2, Q-3, R- 1 , S-4

1 0.- ' electrochemical machining is performed to 1 2 . Match the following (GATE-PI-89)


remove material from an iron surface of 20 X List -1 List -2
20mm under the following conditions A. ECM 1 . Erosion
Inter electrode gap = 0.2mm, B. USM 2. Fusion and vaporization
Supply voltage ( De ) = 1 2 V C. EDM 3. Vaporization & ablation
Specific resistance of electrolyte = 2ohm-cm D. LBM 4. Ion displacement
Atomic weight of iron = 55.85,
Valency of iron = 2 1 3 . Match the following (GATE-PI-90)
Faradays constant = 96540Couloumbs Process
The material removal rate (in g/s ) is
(GATE-ME-09)
A. LBM B. EDM
C. Chemical milling D. USM

Job
(a) 0.347 1 (b) 3.47 1
(c) 34.7 1 (d) 347. 1
1 . Making a printed circuit board
1 1 . Match the following non-traditional 2. Drilling a hole in glass plate
machining processes with the corresponding 3 . Making a die out of a die-steel block
material removal mechanisms: 4. Cutting a polyester sheet
(GATE-ME-11)
!11i@i!@Oiii1jji4PflniM\d,jj� ydcrabad J Delhi J Bhopal J Punc J Bhubaneswar J Lucknow J Patna J Bcngaluru J Chennai J Vtjayawada J Vmg J T11Upati I KukatpallyJ Kolkata
: 872 : Production

1 4. Match the following (GATE-PI-90) 1 8. Machine components material used


Process Defect (GATE-PI-91)
A. casting 1 . Lack of penetration A. machine tool bed 1 . Forged steel
B. welding 2. Penetration B. machine tool spindle 2. HSS
C. extrusion 3. Flange wrinkles C. cutting tool 3. Bronze
D. deep drawing 4. Center burst D. worm wheel 4. Cast iron

1 5. Match the following (GATE-PI-90) (GATE-PI-9 1)


Products
1 9. Process Media used
A. EDM 1 . Electrolyte
A. Porous bearings B. ECM 2. Dielectric
B. Fly wheels C. USM 3. Abrasive slurry
C. Double end spanners D. EBM 4. Vacuum
D. Plastic bottles
Manufacturing process 20. Match the following (GATE-PI-91)
1 . Powder metallurgy Machining process Associated problem
2. Casting A. machining 1 . Wrinkling
3. Closed die forging B. casting 2. Heat affected zone
4. Blow molding C. welding 3. Hot tear
D. drawing 4. Built up edge
1 6. In ECM, the metal removal rate is directly
(GATE-PI-91)
2 1 . Process Associated mechanism
proportional to

(GATE-PI-91)
(a) density of work material
(b) gram atomic weight of the work materia
A. Explosive welding
(c) concentration of the electrolyte used
(d) thermal conductivity of the work B. brazing
material C. Thermit welding
D. manual metal arc welding
1 7. Process Properties of work material
(GATE-PI-91) 1 . Liquid state
A. Casting 1 . Ductility 2. Solid state
B. Forging 2. Malleability 3. Solid-liquid state
C. Wire drawing 3 . Machinability 4. Liquid state chemical
D. Turning 4. Fluidity

IM•li@Oii41hi4RflfifMn.jj� ydcrabad I Delhi I Bhopal I Punc I Bhubaneswar l Lucknow I Patna I Bcngalwu I Chcnnai I Vliayawada I Vmg I Tuupati I Kukatpally I Kolkata
ACE
.:-.�F.ngioramgPnNiariooa
'!i � : 873 : Non Traditional Machining Methods
� �------iiiiii____________iiiiii__iiiiii__iiiiii__iiiiiiiiiiiiiiiiiiiiiiiiiiiiiiiiiiiiiiiiii--iiiiiiiiiiiiiiiiiiiiiiiiiiiiiiiiiiiiiiiiiiiiiiiiiiiiiiiiiiiiiiii

22. Electrical switches made out of thermostats 26. Match the following: (GATE-PI-93)
materials are produced by (GATE-PI-92) 1 . Masking
(a) compression molding 2. Stamping die
(b) transfer molding 3. Engraving
(c) injection molding 4. Stainless steel profiling
(d) vacuum molding 5. Drilling fine holes
A. Abrasive jet machining
23. Products (GATE-PI-92) B. PAM
A. Porous bearings C. Wire EDM
B. Fly wheels D. Chemical machining
C. Double end spanners
D. Plastic bottles 27. Match the following: (GATE-PI-93)
Manufaduring process A. Stamping die plate 1 . EDM
1 . Powder metallurgy 2. Casting B. Gas turbine blades 2. LBM
3. Closed die forging 4. Blow molding C. Twist drill 3. ECM
D. Carbide tool inserts 4. EBM
24. Manufacturing process Property E. Hole in a ceramic plate 5. ECG
(GATE-PI-93) 6. Friction welding
A. Cold rolling 1 . Machinability 7. Wire EDM
B. Milling 2. Weldability 8. USM
C. Brazing 3 . Ductility 9. Ultrasonic welding
D. Powder metallurgy 4.Tensile strength
5.Compact ability 28. Match the following: (GATE-PI-95)

25. Polymer product (GATE-PI-93)


A. Abrasive jet 1 . Horn
machining
A. Polyester resin B. EDM 2. Wire as cutting tool
B. Methyl methacrylate C. LBM 3 . Light
C. Polyurethane D. USM 4. Vacuum
D. Polyvinyl chloride 5. Nozzle
1 . Molded luggage 6. Cladding
2. Refrigerator insulation
3. FRP 4. Contact lenses 5. Floor tiles

!l1'1iii§jjji4ijjj4Qflnj@\d•Mij.Hyderabad l Dellii l Bhopa! I Punel Bhubaneswar J LuclmowJPatnaJ Benga]wu J Chennai lVrjayawada lVmg ITirupati I Kukatpallyl Kolkata I
29. Match the following: (GATE-PI-95) (a) A-4, B-2, C- 1 , D-3
A. Rail sections 1 . Powder metallurgy (b) A-4, B-2 , C-3 , D- 1
B. Platic sheets 2. Rolling (c) A-2, B-4, C- 1 , D-3
C. Allen bolts 3. Cold forming (d) A-2, B-4, C-3, D- 1
D. Porous bearing 4. Transfer molding
5. Calendering 32. Requirement in the design of jigs and
6. Injection molding fixtures Recommended device
P: Heavy clamping force on the work piece
30. Process Mechanism of material removal Q: Clamping on rough surfaces of work
(GATE-PI-03) piece
P EDM 1 . Erosion R: Four work pieces in a line to be
Q ECM 2. Thermal evaporation machined
R AJM 3. Anodic dissolution S: Drilling and reaming of work piece in
4. Etching one
p Q R p Q R 1: Clamp with a floating pad
(a) 2 3 1 (b) 2 4 1 2: Slip renewable bush
(c) 1 3 4 (d) 4 3 1 3: Indexing bush Machined in one loading
4: Equalizing clamp
3 1 . Match list - I (material) with list - II
5: Strap clamp Loading (GATE-PI-04)
(machining) and select the correct answer
suing the code given below the lists (a) P-5,Q- 1 ,R-4,S-2 (b) P- 1 ,Q-4,R-5,S-3
(GATE-PI-04) (c) P-5,Q- 1 ,R-2,S-3 (d) P-4,Q-5,R-3,S-2
List - I
A. machining of conductive material 33. Mechanism of Material Removal Process
B. ruby rod (GATE-PI-05)
C. electrolyte P: Abrasive grain throwing and hammerin
D. abrasive slurry Q: Erosion due to vaporization
R: Chipping and erosion
List - II
S: Melting and partial vaporization
1 . ECM
T: Electrolysis
2. EDM
3 . USM 1 - ECM 2 - USM 3 - PAM
4. LBM 4 - EDM 5 - AJM
\( I l .1 1i.;11H·,·n11� P11hlu ,tt1011s �ydcrabad j Delhi j Bhopa! I Pune l Bhubaneswar l UICknowj Patna j Bcogaluru l Oiennai l Vtiayawada l Vwg ITtrupati I Kukatpally j Kolkata I
: 875 : Non Traditional Machining Methods

(a) P-3' Q-2' R- 5 ' S-1' T - 4 Group - II


(b) P-4, Q- 3, R- 2, S- 1, T- 5 1.EDM
(c) P-2, Q-4, R - 5, S- 3, T - 1 2. U SM
(d) P-2' Q - 5' R-3' S-1' T-4 3. De-ionized water
4. Water jet machining
34. Match List - I (Machining processes) with (a) P-3, Q-4, R-1, S-2
List - II (Operating media and select the (b) P-2, Q-3, R-4, S-1
correct answer using the codes given below (c) P-4, Q-1, R-2, S-3
the lists: (GATE-PI-OS) (d) P-2, Q-3, R-1, S-4

36 In an electrochemical machining (ECM)


List - I (Machining processes )
operation, a square hole of dimensions 5m X
A. Abrasive jet machining
mm is drilled in a block of copper. The
B. Electron beam machining
current used is 5000A. Atomic weight of
C. Electro-chemical machining
copper is 63 and valence of dissolution is 1.
D. Electro-discharge machining
faradays constant is 96500 coulomb. The
List - II (Operating media material removal rate (g /s) is (GATE-PI-08)
1. Dielectric 2. Electrolyte (a) 0.326 (b) 3.26
3. Abrasive slurry 4.Vacuum (c) 3.15 X 10 3
(d) 3.15 X 105
5. Air
37 Match the following (GATE-PI-09)
Codes:
DEVICE FUNCTION
(a) A-5' B-4' C-2' D-1
P. Jig 1. Helps to place work piece
(b) A-4' B-5 ' C-2' D-1
in the same position cycle
(c) A-4' B-2' C-3' D-5
after cycle
(d) A-2' B-5 ' C-3' D-4
Q. Fixture 2. Holds the work piece only
35. Match the following (GATE-PI-06) R. Clamp 3. Holds and position the
Group-I work piece
P. Permaloy S. Locator 4. Holds and position the
Q. Dielectric work piece and guides the
R. Accumulator cutting tool during a
S. Anodic erosion machining operation

l11•1Qjj§jjj44ijjji@CiM\Uih�/.yderabad I Delhi I Bhopal I Pune I Bhubaneswar I Lucknow I Patna I Bengaluru I Chennai I Vtjayawada I Viz.ag I T=pati I Kukatpally I Kolkata
(a) P-4, Q -3, R-1, S-2 of 2000 A with 90 % current efficiency, the
(b) P-1, Q-2, R-3, S-4 expected material removal rate in gm /s will
(c) P-1, Q-4, R-3, S-2 be (GATE-ME & PI-13)
(d) P-4, Q-3, R-2, S-1 (a) 0.11 (b) 0.23 (c) 0.30 (d) 0.52

38. Match the following: (GATE-PI-11) 41. A hard ceramic marble, having density( p ) of
Type of material Name of material 3000 kg /m3 and diameter(d) of 0.025m, is
1. Thermoplastic P. SiAloN
dropped accidentally from a static weather
2. Thermosets Q. Polyvinylchloride
balloon at a height of 1 km above the roof of
3. Elastomers R. Epoxy
a greenhouse. The flow stress of roof
4. Ceramics S. Latex
(a) 1-Q, 2-R, 3-S, 4-P material ( cr) is 2.5GPa. The marble hits and

(b) 1-R, 2-Q, 3-S,4-P creates an indentation on the roof. Assume


(c) 1-S, 2-R, 3-Q, 4-P that the principle of creation of indentation is
(d) 1-R, 2-Q, 3-P,4-S the same as that in case of abrasive jet
machining (AJM). The acceleration due to
39. While removing material from iron (atomic
gravity (g) is 10m /s2 • If V is the velocity, in
weight = 56, valency = 2 and density = 7.8
mis, of the marble at the time it hits the green
g /c(c) by electrochemical machining, a metal
house, the indentation depth
removal rate of 2 cc /min is desired. The
current (in A) required for achieving this
material removal rate is (GATE-Pl-11)
(8=1000
v�
/p xdxV), in mm, is ---
(a) 896.07 (b) 14.93 (GATE-PI-14)
(c) 448.03 (d) 53764.29
42. A resistance capacitance relaxation circuit is
used in an electrical discharge machining
40. During the electrochemical machining
process. The discharge voltage is 100 V. at a
(ECM) of iron (atomic weight = 56, valency
spark cycle time of 25 µs the average power
= 2) at current of 1 000 A with 90 % current
input required is 1 kW. The capacitance
efficiency, the material removal rate was
(in µF) in the circuit is (GATE -15 -Set 2)
observed to be 0.26 gm /s. If Titanium
(atomic weight = 48, valency = 3) is (a) 2.5 (b) 5.0

machined by the ECM process at the current (c) 7.5 (d) 10.0

!ltl•ii@h/i§jjjijRflftjM\,@� yderabad I Dellii I Bhopal I Pune I Bhubaneswar I lllcknow I Patna I Bengaluru I Chennai I Vtjayawada IVu.ag I T°IJ1lpati I Kukatpally I Kollwa
t.t
� .:'EngioeeriogPnNiratioos : 877 : Non Traditional Machining Methods
ACE
� �--iiiiiiiiiiiiiiiiiiiiiiiiiiiiiiiiiiiiiiiiiiiiiiiiiiiiiiiiiiiiiiiiiiiiiiiiiiiiiiiiiiiiiiiiiiiiiiiiiiiiiiiiiiiiiiiiiiiiiiiiiiiiiiiiiiiiiiiiiiiiiiiiiiiiiiiiiiiiiiiiiiiiiiiiiiiiiiiiiiiiiiiiiiiiiiiiiiiiiiiiiiiii•

43. The surface irregularities of electrodes used 45. A surface of 30 mm 30 mm of an iron


in an electrochemical machining (ECM)
x

block is machined using electrochemical


process are 3 µm and 6 µm as shown in the machining process. The atomic weight and
figure. If the work-piece is of pure iron and valency of iron is 55.85 and 2, respectively.
12 V DC is applied between the electrodes ,
The density of iron is 7,860 kg /m3 . If input
the largest feed rate is ___ mm /min.
current is 1,000 A and Faraday's constant is
Conductivity of the electrolyte 0.02 96,540 Coulombs, then the feed rate (in
ohm mm -I -I
mm /min) is ---- (up to two decimal
Over-potential voltage 1.5 V places). (GATE - PI - 17)
Density of iron 7860 kg /mj
Atomic weight of iron 55.85 gm

Assume the iron to be dissolved as Fe+2 and


the Faraday constant to be 96500 Coulomb.
Five Marks Questions

01. A cylindrical work piece can be clamped by


Tool

usmg (GATE-PI-89)
(a) Fixed rectangular block and movable
V-clamp.
Work-piece (Iron) (b) Fixed V-block and movable rectangular
block
(GATE-16-SET-2)
(c) Fixed V-block and movable V-block for
making axial hole in vertical drilling
44. An electron beam welding process uses 15
mA beam current at an accelerating voltage machine of the diameter of the work
. . .
of 150 kV. The energy released per second piece 1s given as 60 +O·05 _ o.oo mm and
by the beam (in J) is ___ (up to one included angle ofV-block 120degrees.
decimal place). Calculate the positional error of the hole in
(1 Ampere = 6.28 x 10 electrons per 18
each case. Also recommend the choice of
second , 1 eV = 1.6 x 10- 19 J) location for fixture design.
(GATE-PI- 17)

!ltlli@jjj§§ijjjjiRflb!M\U,¥�-Iyderabad I Delhi I Bhopal I Pune I Bhubaneswar I Lucknow I Pama I Bengalwu I Chennai IVijayawada I V1Zag j'Hrupari I Kukatpally I Kolkata I
02. Fig shows a jig plate with two holes. The true (a) calculate the magnitude of the clamping
position coordinate dimensions and force
positional tolerance call outs are indicated in (b) direction of the clamping force with
the fig. determine the mm1mum and respect to vertical axis of symmetry
maximum center distance between the holes. (c) position (A) with respect to the vertical
(GATE-PI-90) axis of symmetry.

04. A jig with 90° V - locater and clamps as


shown in fig is used to drill holes on
+0.03
-<1>1 5 .00 +0.00

cylindrical work piece of 60mm diameter.


I <1>jo.03<14)J

Drill guide bush axis is aligned with respect


+0.02
-<1>1 0.oo +o . oo
to job axis. To drill an eccentric hole block P
60
I <1> 10.02Ǥ

u 0

+-20
of 4mm thickness and block Q 3mm
thickness and placed on the V-faces of the
block and then the job is located and
clamped, what is the eccentricity achieved.
ALL DIMENSIONS IN MM For a same eccentricity of block of identical
thickness used on either face, what should be
03. A square block is located on 3 pins (two on the thickness of block used ? which of the
one side and the one on the second side) as two methods are selected and why.
indicated in fig. If the reactions on the pins (GATE-PI-93)
are to be the same as 1OO Kgs after clamping
from the top side. (GATE-PI-92)

Clamp

1 00

IIS•lijj§jjj§a.jj§RflbiMJhi\i� yderabad I Dellii I Bhopal I Pune I Bhubaneswar l Lucknow I Patna I Bengalwu I Chennai I Vtjayawada l Vizag j Tirupati I Kukatpally l Kolkata

�-t . A:CE . .
.:'fllpll')C'!qPnJ,li,ran,n : 879 : Non Traditional Machining Methods
� �--------------------------------------------------------------;;;;;;;;-------
Statement for Linked Answer Q OS & Q 06 05. The for one cycle, in milliseconds , is
In an EDM process using RC relaxation circuit, a (a) 0.55 (b) 0.32
1 2mm diameter through hole is made in a steel (c) 0.89 (d) 0.24
plate of 50mm thickness using a graphite tool and
kerosene as dielectric . assume discharge time to 06. Average power input (in kW) is
be negligible. Machining is carried out under the (a) 0.373 (b) 0.1 37
following conditions. (c) 0.218 (d) 0.5
Resistance = 40 n,
Capacitance = 20 µF
Supply voltage = 220V,
Discharge voltage = 1 1 OV
(GATE-PI-12)

IM••ijj§jjj4U.jj4Rflnji!!ilh+-1Ydcrabad J Delhi J Bhopal J Pune l Bhubaneswarl Luclmow 1 Patna1 Bengaluru J Chennai lVuayawadalVmg ITirupati I K.ukatpallyl Kolkata I
SOLUTIONS
06.
One Mark Solutions Sol: FAL SE, In EDM the mechanism of material
removal is by melting and vaporization
01. associated with cavitation and ECM it is by
Sol: Tool is vibrate, piezoelectric. ion displacement.

02. CAN NOT, CAN. 07. Ans: True


Sol: The ECM and EDM has a common property
requirement of electrical conductivity of 08. TRUE, USM 1s generally used for
material, but this property does not required machining of brittle materials like glass,
for USM. ceramics etc, it is due to the mechanism of
material removal is by brittle fracturing.
03. Ans: (c)
Sol: As the grain size increases, the size of chip 09. Ans: (b)
removed increases but after certain size of Sol: For Rough machining i.e stock removal the
grains it can try to break the work but not electrolyte should have high electrical
removing material. conductivity, called passivating electrolyte,
where as for finish machining the
04. Ans: (a & d) electrolyte should have low electrical
conductivity called non - passivating
05. Ans: (c) electrolyte will be used.
Sol: In USM brittle fracturing is the mechanism
of material removal. i.e, when the tool is 10. Ans: (c)
vibrating with very high frequency, it
produces impact loads on the abrasive 11. Ans: (a)
particles which inturn produces impact on Sol: The tool m EDM should have high
work, if the work is brittle (low toughness electrical conductivity and high M.P hence
work), the brittle fractures will takes place. suitable materials are Cu, W, Cu -W alloy.

jl!•IP@i049,ii/$Rflniffi\lj.jjj.� yderabad I Delhi I Bhopal I Pune I Bhubaneswar I Lucknow I Patna I Bengaluru I Chennai I Vuayaw.ula j Vizag I Tlf\lpati I Kukatpa))y I Kolkata
: 881 : Non Traditional Machining Methods

1 2. Ans: B & C both are the correct answers. 1 9.Ans: (d)


Sol: The high thermal conductivity of the tool
1 3. Ans: (b), Because 'W' has high M.P.
material will have high electrical
1 4. Ans: (d) conductivity hence the heat generated with
Sol: Ion displacement 1s the mechanism by in the tool is low and what ever heat
which the MR will takes place in ECM. generated it will be distributed easily
therefore tool melting rate reduces and tool
1 5. Ans: (d) wear reduces. Where as due to specific heat
Sol: The forces induced in EDM is zero because of work material , the rise in temp of W.P is
these is no direct / indirect contact is present faster and more amount of MR is possible.
between tool and work.
20. Ans: (c)
1 6. Ans: (c) Sol: In USM brittle fracturing is the mechanism
of material removal. i.e, when the tool 1s
vibrating with very high frequency , it
Sol:

produces impact loads on the abrasive

z
f7, 2
particles which intern produces impact on
work , if the work is brittle (low toughness
work) ,the brittle fractures will takes place.
Four rotary motions and three

2 1. Ans: (d)
linear motions are arrested

Sol: During manufacturing of porous bearings


1 7. Ans: (c) using PM process, the last operation is in
Sol: In EDM the mechanism of MR is due to filtration in which the oil is permeated into
melting and vaporization associated with the pores of bearing so that self lubrication
cavitation and also erosion & cavitation or will takes place.
spark erosion and cavitation
22. Ans: (d)
1 8. Ans: (b)
Sol: Out of all the NTM's EEM will give large 23. Ans: (d)
MRR and LBM will give very small MRR. Sol: Fixtures are the devices used for supporting ,
locating and clamping of the job.

\( l I 111!,lllt ( I Ill!!, P11hlic .ilJt 111' ydcrabad I Delhi I Bhopal I Punc I Bhubaneswar I wcknow I Patna I Bcngaluru I Chennai I Vuayawada I Vizag I Tirupati I Kukatpally I Kolkata
: 882 : Production

24. Ans: (a) 31. Ans: (a)


Sol: Holes in glass plate can be made only by Sol: In ECM
USM. MRR oc gram atomic weight of material
MRR oc Current density
25. Ans: (c) 1
Sol: Fixtures cannot interfere with operation but
MRR oc ------------
dis tan ce between toot and work
by using fixtures setting of work piece MRR oc Thermal conductivity of electrolyte.

�r
becomes easier.
32. Ans: (d)
26. Ans: (b)
Sol: Sintering is the last operation in powder Sol: MRR = K Q. d 3 v ½ (
2
metallurgy in which the compacted part will
Q = Flow rate of abrasives,
be heated to a high temp which is less than
d = mean diameter
the MP of the powders used.
V = Velocity of abrasives,

Ans: (c)
I = density
27.
H = Hardness of work material.

28. Ans:
33. Ans: (d)
Sol: ELECTROCHEMICAL, and
34. Ans: (a)
Sol: As the thermal conductivity of tool material
MECHANICAL

29. Ans: (b)


is high the heat dissipation from the tool is

Sol: In EBM Vacuum is provided to avoid the


taking place and if the specific heat is high,
it needs large amount of heat for rising the
dispersion of electrons after the magnetic
temps of tool material up to MP.
lens, but this vacuum is giving an addition

35. Ans: (b)


function of providing efficient shield to the

36. Ans: (d)


weld bead.

Sol:
30. Ans: (b)
Sol: Out of all the NTM's EDM will give large
MRR and EBM will give very small MRR.
___. Tip distance

!IJllii@OiiU•iiiiRflbiihiii!.\i yderabad I Delhi I Bhopal I Pune I Bhubaneswar l Luclmowl Patna I Benga)uru I Chcnnai I Vijayawada j Viz.ag I Tirupati I Kukatpal)y I Kolkata
: 883 : Non Traditional Machining Methods

37. Ans: (d) 41. Ans: (c)


Sol: In EDM process by using the shape of the
tool as square shape, it is possible to 42. Ans: (b)
produce square holes. But with AJM only Sol: Hardness of work piece and hardness of tool
circular holes and with PAM & LBM it is will not influence the material removal
possible to produce very small size circular EDM.
holes.

43. Ans: (a)


38. Ans: (d)
Sol: Electron beam machining is the only method
39. Ans: (b) carried out under vacuum, to avoid the
Sol: In ECM , when the power supply is given to dispersion of electrons after the magnetic
the workpiece and tool , because of deflector.
Faraday's laws of electrolysis, the ions will
start displacing from workpiece and trying 44. Ans: (d)
to deposit over the tool , but before they are Sol: With increase of feed force, the material
depositing on the tool the electrolyte present removal rate MRR is first increases and then
between the tool and workpiece will decreases.
pumped out so that ions also will be
45. Ans: (b)
pumped out without depositing over the
Sol: In EDM or wirecut EDM , the work and tool
tool.
must be electrically conductive otherwise the
current passage will not takes place.
40. Ans( b)
Sol: The holes looking to very small hence direct
46. Ans: (a)
drilling or milling or CNC drilling cannot Sol: In EDM , dielectric fluid is used. Hence 'Q'
be done. Also only wire cut EDM cannot 1s wrong.
use for producing holes. Therefore die
sinking will be used producing circular 47. Ans: (A)
holes and wire cut EDM will used for Sol: USM is used only for machining of highly
converting circular hole into square hole. brittle materials such as glass.

!ffl•1ii@jji§§jjji4j;jjni@m.jj� yderabad I Delhi I Bhopal I Pune I Bhubaneswar I Lucknow I Patna I Bengalwu I Cbennai I Vijayawada I Vizag I Tirupati I Kukatpally I Kolkata
: 884 : Production

48. Ans: (d) 05.


Sol: In USM vibration is in ultrasonic range, it Sol: A = gram atomic , p = density , I = current,
uses piezoelectric (or) magnetostrictive F = faradays constant , Z = valancy.
transducer for producing high frequency
MRR = �
vibrations and it is used only for brittle pZF
materials.

49. Ans: (a)


Sol: Etch Factor is the ratio of the undercut to 500
= ---------------
the depth of cut. The undercut depends upon 0. 1 8 x 2 + 0.62 x 2 0.2 x 6
8.28 x 965 oo ( + )
the depth of cut, the strength of the etchant 58.93 58.7 1 5 1 .99
solution and work piece material. 500
-
8.28 X 96500 (0.006 1 1 + 0.02 1 1 + 0.023 1)
= 3 . 1 48 x 1 0-s cc / sec = 0. 1 1 33 cc / hr

Ans: (d)
Two Marks Solutions
06.

0 1 . Ans: (a) 07. Ans: (c)


Sol: In ECM Sol: EDM, ECM and AJM are used for
MRR oc gram atomic weight of material producing straight holes only but in LBM
MRR oc Current density by maneuvering or bending laser gun
slightly it is possible perform the Zig - Zag
1
dis tan ce between toot and work
MRR oc ----------- hole.

Ans: (a)
MRR oc Thermal conduction of electrolyte.
08.

02. Ans: A - 5, B - 3, C - 2, D - 4 09. Ans: (b)


Sol: p = 6000 kg/m3 , F = 96500
03 . Ans: A - 6, B - 2, C - 1 , D - 4
A1 = 56,

Ans: (c)
MRR = 50 mm3/sec
04.
A2 = 24, Z2 = 4, I = 2000A

jl"S•lii@jj/qg;jjijRflflj@j...jj+yderabad I Delhi I Bhopal I Pune l Bhubaneswar l Lucknow I Patna I Bengaluru I Chennai l Vijayawada l Viz.ag I Tirupati I Kukatpallyl Kolkata I
�.....��Pnbticdioos
ACE
Non Traditional Machining Methods
" V • • • •
� : 885:

16. Ans: (b)


Sol: In ECM
MRR oc gram atomic weight of material
x1 z 1 + x2 z 2 = 2000 MRR oc Current density
= 0.06908
A1 A2 6 x 96500 x 0.05 1
MRR oc ----------­
dis tan ce between toot and work
xi X 2 X 2 x 4
+ = 0.06908
56 24 MRR oc Thermal conductivity of electrolyte.
By trail and error method
X 1 = 75% , X2 = 25%. 17. Ans: A- 4 , B- 1 , C- 2, D- 3.
18. Ans: A- 4 , B- 1 , C-2, D- 3.
10. Ans: (a) 19. Ans: A-2, B- 1 , C - 3, D- 4.
Sol: A = 55.85 , Z = 2, F = 96540 20. Ans: A - 4 , B - 3 , C - 2, D- 1.
Specific resistance = 2!2-cm
21. Ans: A - 3, B - 2, C - 4, D- 1.
Voltage = 12V
22. Ans: (b)
Inter electrode gap = 0.2 mm
23. Ans: A- 2, B-3, C- 1 , D- 5.
Resistance(R)
24. Ans: A- 3, B- l , C-2, D- 5.
Sp. Resis:ance x Inter electrode gap
Suface area 25. Ans: A- 1 , B-2, C- 3 , D- 4.

2 X 10 X 0.2 26. Ans: A- 5 , B-3, C-2, D- 1.


= = O.O l
20 x 20 27. Ans: A- 7, B- 3, C- 6 , D- 5 , E- 8.
28. Ans: A - 5 , B - 2, C - 3, D- 1.
1 = V = � = 1200A
R 0.0 1 29. Ans: A- 2, B - 5 , C - 3, D- 1.
AI 55.85 x 1200
MRR = =
ZF 2 x 96540 3 0. Ans: (c) 31. Ans: (c)
= 0.347 1 g /sec 32. Ans: (d) 33. Ans: (c)
34. Ans: (a) 35. Ans: (b)
1 1. Ans: (a)
36. Ans: (b)
Sol: I = 5000 A
12. Ans: A - 4, B - 1, C - 2, D - 3 .
13. Ans: A-4 , B - 3, C- 1 , D-2. A = 63, Z = l , F = 96500
14. Ans: A- l , B- 2, C- 4, D- 3. AI 5000 x 63
MRR = = = 3 264 g /sec
15. Ans: A- 1 , B- 2, C- 3, D- 4. ZF l x 96500

!ltl1ii@jjji4ijjji/mfi1Mmjjj� yderabad I Delhi I Bhopal I Pune I Bhubaneswar I Lucknow I Patna I Bengaluru I Chennai I Vijayawada I Vizag I Tirupati I Kukatpally I Kolkata
: 886 : Production

37. Ans: (d) 38. Ans: (a) 42. Ans: (b)


Sol: In EDM process
39. Ans: (c)
0.5CVd2
Sol: A = 56, Z = 2, I = 7.8 s /cc. Power = P =
tc
MRR = 2 CC /min.
P x t c _ l x l 00 0 x 25 x 1 0 -6 --6
- cc /sec C = 5xl0
0.5 X V; 0.5 X (1 00)
Al
- =
MRR = 2
pZF
2 Al 43. Ans: 5 1.542 (range 5 1.0 to 52.0)
60
-=-
1
x 0. 009
IZF
pZF � 7.8 x 2 x 965 00 _ L - 5 0 x 0.009
I=�x = x 448 A Sol: R = -P- = 0.02
=
60 A 60 56 Area Area Area
V (l2-1.5)x Area
I= = = 23.333 x Area
40. Ans: (a) R 5 0 x 0. 009
Sol: For Iron, A = 56 , Z = 2 L = 3 + 6 = 9 µm = 0. 009
I = 0.9x 1000 = 900 A AI 55.85 x 23.333x Area
MRR = =
MRR 0.26 gm /sec p ZF 786 0 x 1 o- x 2 x 965 00
= 6

AI => 56 x 900 = = 0.98 189 x Area


MRR = F= 96923
ZF 2 x 0.26 MRR
-- = 0.8590 mm /sec
Titanium, A = 48, Z = 3 Area
I = 0.9x2000 = 1800 A = 0.8590 x 60mm /min
48 x 18 00 5 1.542 mm /min
MRR = = 0.297 = 0.3
3 X 96923
=

44. Ans: 2250


41. Ans: 1.56 to 1.6 Sol: I = 15 mA, V = 15 0 x 103 V
Sol: Velocity of the ceramic disc (V) = �2gh p = 15 X 1 0-3 X 15 0 X 1 03 = 2250 J

= .J2 x 10 x 1000 = 141.42m /s


45. Ans: 2.453
Indentation depth = 6 = 1 000 x d x V /p Sol: MRR = A.I. I p.Z.F . = Ac . f
Feed = f = MRR /Ac = A.I. / p. Z. F. Ac
��

3000
= 1 000 X 0.025 X 1 4 1 .42 = (55.85 x 1 0 00) I (7860 x 1 0"6 x
2 X 96540 X 30 X 30)
5
6 X 2.5 X 1 0
1.58mm = 0.04089 mm /sec = 2.453mm /min.
=

!lfl1ib§jjjgq;jjj4@nj@ii1jjj*' yderabad I Delhi I Bhopal I Pune I Bhubaneswar I Lucknow! Patna I Bengalwu I Chennai I Vijayawada IV� I Tirupati I Kukaqially I Kolkata
: 887 : Non Traditional Machining Methods

(c). The positional error is mainly depends on


Five Marks Solutions the fixed element. So when fixed V - block
and marble V - block is used, the positional
01. error is remains same as (b)
Sol:
(d). Out of the cases, case (a) is giving lower
(a) Fixed rectangular block and movable V -
3
positional error, hence preferable.
clamp.

0 .
2
Sol: The theoretical center distance
Clamping force .J
Between holes = 40 2 + 0 2 = 50 = C 3

40

Positional error = 3 0.025 - 3 0 0.0 5


I
_________J_____ -
= 2 I
I
I

30
Fixed V - block and movable rectangular
·i2 ·i3
(b)
) = 49.975 mm
block 0 0
cm in = 50 - ( +

cmax = 50 + (
0
·i2 ·i3
+
0
) = 50.0 5 mm2
�Clamping
0 .
3
Sol: Resolving the force "F" into Horizontal
F sina = 1 00 --------------------- ( 1 )
30 F cos a = 1 00 + 1 00 = 00 ------( )
x1 = -- = 3 4.64
Sin 60
2 2
1 00
.Q2 = tan a =
0.0
X2 = 3 2 5 = 34.66 (2) 200
Sin 60
Positional error X2 - X 1 = 0.0 98mm = 2 a = tan - 1 (�) = 26.565
\( l. l 1 1i.; u u:1 1 Jill\ P11lilH ,111, , 1 1 " Fyderabad j Delhi j Bhopal j Pune l Bhubaneswarl Lucknowl Patna l Bengaluru j Chennai l Vuayawada ! Vmg I Tirupati I Kukatpallyj Kolkata I
: 888 : Production
ACE
��-�E�Pnhlirama
� �--iiiiiiiiiiiiiiiiiiiiiiiiiiiiiiiiiiiiiiiiiiiiiiiiiiiiiiiiiiiiiiiiiiiiiiiiiiiiiiiiiiiiiiiiiiiiiiiiiiiiiiiiiiiiiiiiiiiiiiiiiiiiiiiiiiiiiiiiiiiiiiiiiiiiiiiiiiiiiiiiiiiiiiiiiiiiiiiiiiiiiiiiiiiiiiiiiiiiiiiiii-

Common data solution


F= � = 223.6 kg
OO

sm a
05. Ans: (a)
Taking the moments about vertical axis
X.F Cos a + l OO x 30 = 100 x 30 + 100 x 20 06. Ans: (c)
X = 10 mm. Sol: D = 12mm , t = 50mm, R = 40 n,
C = 20 µF, Vs = 220V, Vd = 110V
04.
V
Sol: Cycle time = R.C In [ s ] = tc
vs - vd

220
40 x20x10--6 x In [- ]
110
=

= 554 x 10-6 sec = 0.55 milli sec

Average power input = W


01 02 = 5 = .Jx 2 + X 2
X = 3.5

= 218 W = 0.218 kW

V-block

Block of uniform thickness is preferable


because of balanced condition.

!M1•i!j§jjji4@iRflbiM@i!4 ydcrabadlDclhil Bhopa) I Punc J Bhubaneswarl LucknowlPatnal BcngalurulChcnnaiJ Vijayawada l Vu.ag I Tirupati I Kukatpal)yl Kolkata
Page No. 889

I M (Questions)
& OR
Page No. 890

CONTENTS
-=---------=---=-==----
----=-===-==---=--=--=- --==-=---==-=- -- - - - - - -�- �

Chapter Questions Solutions


Name of the Chapter
No. - ---- - - '
Page !'f_o. Page No.
01 Linear Programming 891 - 895 896 - 904

02 Pert & CPM 905 - 912 9 1 3 - 919

03 Queuing Theory 920 - 923 924 - 926

04 Inventory Control 927 - 932 933 - 939

OS Transportation 940 - 942 943 - 944

06 Production Planning & Control 945 - 946 947 - 948

07 Forecasting 949 - 952 953 - 955

08 Line Balancing 956 - 956 957 - 957

09 Scheduling 958 - 960 961 - 963

10 Sequencing 964 - 965 966 - 967

11 Assignment 968 - 968 969 - 969

12 Material requirement & Planning 970 - 971 972 - 972

13 Miscellaneous 973 - 977 978 - 982


Ct Linear Programming

One Mark Questions Two Marks Questions

01. If at the optimum in a linear programming 01. A manufacturer produces two types of
problem, a dual variable corresponding to a products, 1 and 2, at production levels of x 1
particular primal constraint is zero, then it and x2 respectively. The profit is given is
means that (GATE-ME-96) 2x 1 + 5x 2 • The production constraints are
(a) Right hand side of the primal constraint x 1 + 3x 2 � 40
can be altered without affecting the
3x 1 + x 2 � 24
optimum solution
x 1 + x 2 �10
(b) Changing the right hand side of the
primal constraint will disturb the x , > O, X 2 > 0
optimum Solution The maximum profit which can meet the
(c) The objective function is unbounded constraints is (GATE-ME-03)
(d) The problem is degenerate (a) 29 (b) 38 (c) 44 (d) 75

02. Simplex method of solving linear 02. A company produces two types of toys: P
programming problem uses and Q. Production time of Q is twice that of
(GATE-ME-10) P and the company has a maximum of 2000
(a) all the points in the feasible region time units per day. The supply of raw
(b) only the comer points of the feasible material is just sufficient to produce 1 500
region toys(of any type) per day. Toy type Q
(c) intermediate points within the infeasible requires an electric switch which is available
region @ 600 pieces per day only. The company
(d) only the interior points in the feasible makes a profit of Rs. 3 and Rs. 5 on type P
region. and Q respectively. For maximization of
profits, the daily production quantities of P
and Q toys should respectively be
(GATE-ME-04)

!MlliiiYih44iihi!MmtM\iifo� yderabad I Delhi I Bhopal I Pune I Bhubaneswar I Lucknow I Patna I Bengaluru I Chennai I Vijayawada I Vizag I Tirupati I Kukatpally I Kolkala
" • ACE
. l'"'El!"n�1p1
':.�-� �- :•�11�'11...'Ll!
:- c::u11�

892 ··=============����
..�rafi�•'l0:u:.::========::·�� IM & OR

(a) 100, 500 (b) 500, 1000 05. After introducing slack variables s and t, the
(c) 800, 600 (d) 1000, 1000 initial basic feasible solution is represented
by the table below (basic variables are s = 6
Common Data for Question 03 & 04 and t = 6, and the objective function value is
Consider a linear programming problem with two 0). (GATE-ME-08)
variables and two constraints. The objective
-4 -6 0 0 0
function is: Maximize X I + x2 .The comer points
s 3 2 1 0 6
of the feasible region are (0,0), (0,2), (2,0) and
t 2 3 0 1 6
(4 /3, 4 /3).
X y s t RHS

03. If an additional constraint XI + x2 :S 5 is After some simplex iterations, the following


added, the optimal solution is table is obtained

0 0 0 2 12
(GATE-ME-05)

(a) (%, %) (b) (1, ;) s


y
5 /3 0
2 /3 1
1
0
-2 /3
1 /3
2
2
(c) (d) (5, 0) s t
(%' %) X y RHS

From this, one can conclude that


04. Let Y I and Y2 be the decision variables of the
dual and V I and v2 be the slack variables of (a) The LP has a unique optimal solution
the dual of the given linear programming (b) The LP has an optimal solution that is
problem. The optimum dual variables are not unique
(a) Y I and Y2 (b) Y I and V I (c) The LP is infeasible
(c) Y I and v2 (d) V I and v2 (d) The LP is unbounded

Common Data for Question 05 & 06 06. The dual for the LP is (GATE-ME-08)
Consider the Linear programme (LP) (a) Zmin = 6u + 6v (b) Zmax = 6u + 6v
Max 4x + 6y subject to subject to
Subject to 3u + 2v 2 4 3u + 2v � 4
3x + 2y :::; 6 2u + 3v 2 6 2u + 3v � 6
2x + 3y :::; 6 U,V 20 U,V 20

x, y 2: 0

l1illi!@jjjii.jj4@fliMhiih� yderabad I Delhi I Bhopal I Pune I Bhubaneswarl Lucknow I Patna I Bengalwu I Chennai I Vuayawada I Vu.ag ITirupati I Kukatpa)ly I Kolkata
: 893 : Linear Programming
�Pnblirarinns
ACE
�-��:�

���========���==========��������
� 'Ii'�
���
(c) Zmax = 4u + 6v (d) Zmax = 4u + 6v 09. The manufacturer can make a maximum
subject to subject to profit of Rs.
3u + 2v 2 6 3u + 2v :S 6 (a) 60000 (b) 135000
2u + 3v 26 2u + 3v :S 6 (c) 150000 (d) 200000
u, v 2 0 V20
10. A linear programmmg problem 1s shown
U ,

07. Consider the following Linear Programming below.


problem (LPP) (GATE-ME-09) Maximize 3x + 7y
Subject to 3x + 7y :S 10
Maximize: Z = 3x1 + 2x2
4x + 6y :S 8
Subject to x1 :S 4
X2 :S 6 x, y 2 0

3x1 + 2x2 :S 18 It has ..... (GATE-ME-13)


2 0, 20 (a) an unbounded objective function
(a)The LPP has a unique optimal solution.
XJ X2

(b) exactly one optimal solution


(b) The LPP is infeasible (c) exactly two optimal solutions
(c) The LPP is unbounded (d) infinitely many optimal solutions
(d)The LPP has multiple optimal solutions.
11. Consider an objective function
Common Data for Question 08 & 09 z(x 1 , x 2 ) = 3x 1 + 9x 2 and the constraints
One unit of product P I requires 3 kg of resource
R1 and 1 kg of resource R2. One unit of product
X 1 + 2x 2 :S 4,
P2 requires 2 kg of resource R l and 2 kg of
2 0, x 2 2 0,
resource R2. The profits per unit by selling
X1

product P1 and P2 are Rs. 2000 and Rs. 3000 The maximum value of the objective
respectively. The manufacturer has 90 kg of function is ---- (GATE-ME-14)
resource R1 and 100 kg of resource R2.
(GATE-ME-11) 12. For the linear programming problem:
Maximize Z=3x1+2x2
08. The unit worth of resource R2. i.e. dual price
Subject to -2x1+3x2 :S 9
X1-5X2 2-20
of resource R2 i n Rs. per kg i s
(a) 0 (b) 1350
(c) 1500 (d) 2000 XJ , X2 2 0

!lti1i@jjjl4.jjgRflijffijjj.jj� yderabad I Delhi I Bhopal I Pune I Bhubaneswar I Lucknow ! Patna I Bengaluru I Chennai jVtjayawada IVmg I TlfUpari I Kukatpallyj Kolkat,
ACE : 894 : IM & OR
�-�
�� . �=
�li'.....-
====� ===p.Jblicdiona
....:.......:.. ================================
The above problem has (GATE-15-Set 3) The profit per unit on parts I and II are Rs.40
(a) Unbounded solution and Rs. 100, respectively. The maximum
(b) Infeasible solution profit per week of the firm is Rs _____
(c) Alternative optimum solution
(d) Degenerate solution
Type of Machining Maximum
machine time required machining time
13. The value of (x1, x2) for an optimal solution
for the available per
for
machine part week(minutes)
Minimize Z= 6x1 -8x2
(minutes)
subject to : 5x1+10x2 s 30
I II
4x1+4x2 s 20 Turning 12 6 6000
XJ � 0, X2 � 0 IS
Center
(GATE - PI-15) Milling 4 10 4000
(a) (0,0) (b) (1,6) (c) (0,3) (d) (3,7) Center
Grinding 2 3 1800
14. Maximize Z= 15X1 + 20X2 Machine
Subject to
(GATE- 16 - SET-3)
12X1 + 4X2 � 36
12X1 - 6X2 s 24
16. Two models, P and Q, of a product earn
X1, X2 � 0
profits of Rs. 100 and Rs. 80 per piece,
The above linear programming problem has
respectively. Production time for P and Q
(GATE - 16- SET- 1)
are 5 hours and 3 hours, respectively, while
(a) infeasible solution
the total production time available is 150
(b) unbounded solution
hours. For a total batch size of 40, to
(c) alternative optimum solutions
maximize profit, the number of units of P to
(d) degenerate solution
be produced is ____
(GATE- 17 - SET- 1)
15. A firm uses a turning center, a milling center
and a grinding machine to produce two parts.
17. Maximize Z = 5x 1 + 3x2
The table below provides the machining time
subject to + 2x2 S 10,
required for each part and the maximum
8'
X1

machining time available on each machine.


XJ , X2 2:: 0
X1 - X2 S

!Ml•iii4i@41ii/4Rflij!Mi@i� yderabad I Delhi I Bhopal I Pune I Bhub,ines#.ir I 'Lucknow I Patna I Bengalurn I Chennai I Vijayawad:' I v{.ia�rupati I Kul;aqia]iy j' Kolkata
ACE : 895 : Linear Programming
':.•.� F..,.,.,..PoNn4nw
" V • • • •

In the starting simplex tableau, x 1 and x2 are 02. A furniture manufacturer produces chairs and
non-basic variables and the value of Z is tables. The wood-working department is
zero. The value of Z in the next simplex capable of producing 200 chairs or 1 00 tables
tableau is-----
or any proportionate combinations of these
(GATE - 17 - SET - 2)
per week. The weekly demand for chairs and
tables is limited to 1 50 and 80 units
respectively. The profit from a chair is Rs.
Five Marks Questions
1 00 and that from a table is Rs. 300.
(GATE-ME-02)
01 . Solve the following linear programmmg (a) Set up the problem as a linear program
problem by simplex method (b) Determine the optimum product mix for
(GATE-ME-00) maximizing the profit.
(c) What is the maximum profit?
(d) If the profit of each table drops to Rs.
200 per unit, what is the optimal mix and
(a) What ts the solution to the above profit?
problem?
(b) Add the constraint x2 � 2 to the simplex
table of part (a) and find the solution.

�ydcrabad I Delhi I Bhopal I Punc I Bhulnncswar j Lucknow! Palna I Bcngaluru I Chcnnai j Vgayawadaj Vu.ag I Ttrupali I Kukatpa)ly I Kolkata I
SOLUTIONS
+ X2 � 1 0 ; -
X1
+ 2 � 1 . . . (3)
X
10 10
X1
One Mark Solutions
> 0 ; X2 > 0
X1
0 1 . Ans: (a) Optimal point A
Sol: Intersection of (2) and(3) equations, solving
(i) Shadow Price of Primal is solution of Dual. them x 1 = 7; x2 = 3 .
(ii) Shadow Price zero Signifies un-utilised Zmax = 2 X 7 + 5 X 3 = 29
resource.
02. Ans: (c)
02. Ans: (b) Sol: Let, P type toys produced = x ,
Q type toys produced = y
j Two Marks Solutions
p Q
0 1 . Ans: (a) Time 1 2 2000
Sol: Raw material 1 1 1 500
40 Electric switch - 1 600
30 Profit 3 5

20
X y
10
Zmax = 3x + 5y
x_ + _L_ =s; l
x + 2y =s; 2000 '· _2000 1 000
x Y
x + y � 1 500 . __ + __ � l
' 1 500 1 500
y � 600 . _r___ <
x 1 + 3x2 � 40 ; - + 2 � 1 . . .(1) ' 600 - l
X1 X
40 40
3 x, y � O
+ X2 � 24 ; - Zmax = 3x + 5y
+ -.-2 � 1 . . . . (2)
X1 X
8 24
3X 1
ZA = 3 x 1 500 + 5 x O = 4500
\( I I ri�111t t 1 1 1 1� P11lilu .111011 ... yderabadj Delhi j Bhopalj Pune j Bhubancswarj Lucknow j Patna j Bengaluruj ChennailVuayawadajVu.ag (T,rupati I KukalpaJlyl Kolkata
: 897 : Linear Programming

ZB = 3 x O + 5 x 600 = 3000 8
Zc = -
Zc = 3 x 1 000 + 5 x 500 = 5500 3
Zo = 3 800 + 5 600 5400
Optimal at (c) (; ; ;)
x x =

y
2000 By adding a new constraint solution space
does not change.

Hence Ans: (� ; ;)

Primal variables are(x 1 , x2) and dual


variables are (Y1 , Y2)

05. Ans: (b)


C does not exist in answer. Solution is optimal; Number of zeros are
Hence, Zmax is at D, greater than the number of basic Variables
i.e., Zmax @ D = 5400 in Cj - Zj(net evaluation row) hence
multiple solutions.
03. Ans: (b)

04. Ans: (a)


06 . Ans: (a)
..--------Sol: Zmax = X1 + X2 y
ZA = 2 Sol: Dual X �

ZB = 2 Zmin = 6u + 6v
u 3 2 6
Subject to
(0,5)
3u + 2v z 4 V 2 3 6
2u + 3v z 6
4 6
u, v :2: 0

A (5,0)
(2,0)

�ydcrabad I Delhi I Bhopal I Pune I Bhubaneswar l Lucknow I Patna I Bengaluru I Chcnnai I Vtjayawada I Vizag I Tuupari I K� I Kolkala I
: 898 : IM & OR

07. Ans: (d) Zmax = 2000x+3000y


Sol: Zmax = 3x 1 + 2x2
Subject to
y
3x + 2y � 90 -+-�1
X
10 30 45
8 y
+ 2y � 1 00 -+-�1
X

1 00 50
X
B(0,6) 6 a&-.,....��----
4 x , y 2: 0 x , y 2: 0

2 y

2 4 6 8 10 x,
A(4,0) 50
B(O, 45)
Subjected to
x, � 4
X2 �6 A(30, 0) 1 00
3x,+ 2x2 � 1 8
Zmax = 2000x+3000y
x,2:: O; X2 2:: 0
Comer points satisfying both the constraints
Zmax = 3x, + 2x2
are A(30 , 0) B(O, 45).
Zo = O
Z A =2000 X 30 + 3000 X 0
ZA = 3 x 4 + 2 x O = 1 2
Zs = 3 x O + 2 x 6 = 1 2 60,000/-
=

Za = 2000 x O + 3000 x 45
Zc = 3 6 = 18 } Multiple solutions
x 2+2 x
= 1 ,35,000/-
Z0 = 3 x 4 + 2 x 3 = 1 8
Solution is optimal at B ; x = 0 ; y = 45
08. Ans: (a)
09. Ans: (b)
3x + 2y + S 1 = 90 ; 3 x O + 2 x 45 + S 1 = 90
Sol: => S 1 = 0 i. e., R 1 resource is fully utilized.
x + 2y + S 2 = 100 ; 0 + 2 x 45 + S 2 = 1 00
=> S 2 = 1 0
P1 P2 Availability
R1 3 2 90
i.e., R2 resource is unutilized at optimality.
R2 1 2 1 00
2000 3000 Hence dual price of R2 is zero.
Profit per
unit
y
\{ I I 11L;111l 1 11 1 1� Ptild1t 1 1 1 1 , r i -.
�ydcrabad
I Delhi ! Bhopal I Punc l Bhubancswar l Lucknow ! Patna ! Benpluru I Otennai I Vuayawada j Viz.ag j Tirupali I Kukatpally l Kolkata I
ACE
"•t"'Ii'....:..-:.... P.iNiratiooa : 899 : Linear Programming
�� �:� ��� ���==========��==========��������
10. Ans: (b) Z (x1 ,x2)=3x1+9x2
Sol: Subject to
4 Xt + X2 � 8
Xt + 2X2 � 4
3
Zmax = 3x+7y
Xt � 0, X2 � 0
s.t 2
C (0, 4/3) �+� � 1
3x+7y � 1 0 I 8 8
4x+6y� 8 �+� � 1
4 4 2
x,y � O
Z = 3x 1 +9x2
X y
-+-� 1 Z = (3x0) + (9x0) = 0
10 -
- 10
3 7 ZA = (3x4) + (9x0) = 1 2
X y Zs = (3x0) + (9x2) = 1 8
-+- � 1
2 4/3 X J = 0, X2 = 2 , Zmax = 1 8.

12. Ans: (a)


z max = 3 x + 7y

Sol: Z max=3x1+2x2
ZA = 3x0+7x0 = 0
Zs = 3 x2+7x0 = 6
4 28
Zc = 3 x0+ 7 x - = - = 9.33
3 3
Optimal at 'C' unique solution.

1 1 . Ans: 18
Sol:
X2

6 Unbounded
Solution
4

2 20 10
B(0,2)
:. Unbounded optimal solution to the problem.
(0,0) 2 4 6 8

!l!llib4ii!44•hiiRflCffiii�h� yderabad I Delhi I Bhopal I Pune I Bhubaneswari Lickn�w i�;lll3,1 Bengalwu I Chennai \ Vrjayawada \ Vizag ITtrupati r !(ulGilj,ally l I{olkala
: 900 : IM & OR

13. Ans: (c) Zmin = 6x 1 - 8x2


Sol: Zo = O
Zmin= 6x1 - 8x2 ZA = 6x5 - 8x0 = 30
Subject to Zs = 6x0 -8x3 = -24
5x 1 + l Ox2 s 30 Zc = 6x4 -8x 1 = 16
4x 1 + 4x2 s 20
X1 :?: 0 , X2 :?: 0 Solution is optimal at 'B"

�+� s l X1 = 0 , X2 = 3
6 3 Zmin = 6x1 -8x2
= 6x0 - 8x3= -24
Zmin = -24

"C" is intersection of two lines


14. Ans: (b)
Sol: Max Z = 15X1 + 20X2
X2

5 Subjected to
4 12X1 + 4X2 2: 36
12X1 - 6X2 :S 24
B(0,3)
3

2 X 1, X2 2: 0 ,
0

X, X 2
(0,0) 2 3 4 5A 6 x,

+ =1
3 9
(5,0)

5x1 + 1Ox2 = 30x4


X, X 2
X2
4x1 + 4x2 = 20x5 - =1
2 4

20x 1 + 40x2 = 120 7

20x 1 + 20x2 = 100


6

20x2 = 20
5
4

=> x2 = 1
A (2.6, 1.2 ) ;
3

4x1 + 4x2 = 20
2

B (3, 0)
4x1 + 4x l 20
C (0, 9) ;
=

4x1 16
-1

D (2, 0) ;
= -2

=> XJ = 4
-3
E (0 , -4) -4

\( I . l 1114"111t t i mg P11hlu .111011, �yderabad I Delhi I Bhopal I Pune I Bhubaneswar l Lucknow I Patna I Bengaluru J Chennai I Vtjayawada I Vuag I T,rupari I Kukatpally I Kolkata I
: 901 : Linear Programming

There is a no common reason which satisfy


the constraint. = 6000 =
y 250
:. Unbounded solution. 24
4x + l Oy = 4000
15. Ans: 40,000 (range 39000 to 41000) 4X + 250 l Q = 4000
Sol:
X

4x = 1 500
MIC MIC
= 1 500 = 375
I II X

TC 12 6 6000 Zmax = 40 X + l OOy


MC 4 10 4000
GM
= 40 X 375 + 1 00 X 250
2 3 1 800
Profit/unit
= 1 5000 + 25000
40 1 00
Zo = 40,000/-
X y

Zmax = 40 X + 1 00 y 1 000
Subject to 800
1 2x + 6y � 6000,
600
4 x + 1 Oy � 4000 C(0,400)
2x + 3y � 1 800 400

x, y 2: 0 200

�+- Y- � 1
500 1 000 A(O,O)
200 400
-X
-+-y
�1 8(500,0)
1 000 400
ZA = O
�+l � 1
900 600 Z 8 = 40 x 500 + 1 00 x O = 20,000
1 2x + 6y = 6000 Zc = 40 x O + 1 00 x 400 = 40,000
4x + l Oy = 4000 Optional at (D) & (C)

1 2x + 6y = 6000
1 2x + 30y = 1 2000
- 24y = -6000

!IJ11ijj§jjj4i4jji4@nfMj111jj� yderabad I Delhi I Bhopal I Pune I Bhubaneswarj Lucknow! Patna I Bengaluru I Chennai I Vijayawada I Vizag I Tuupati I Kukatpally I Kolkata
: 902 : IM & OR

16. Ans: 15 + 25 = 40
Sol: = 15
X

Comer points are


X

p Q Availability 0(0,0), A(30,0), B(0,40), C(l 5,25)


Production 5 3 150 Zmax = 100x+80y
Batch 1 1 40 Zo = 100x0 + 80x0 = 0
Profit 100 X 80 y ZA = 100x30 + 80x0 = 3000
Zs = 1 OOxO + 80x40= 3200
Zmax = 1 OOx + 80y Zc = 1 00x l 5 + 80x25= 3500
s.t 5x + 3y :s; 1 50 Maximum at C
+ y = 40 Number units of P = x = 1 5units
x, y > O
X

17. Ans: 40
� + L =1 Sol: Max z = 5x1+3x2
30 50
Subject to x1+2x2 :s; 10 ,
y
+- = l X1 - X2 :s; 8 , XJ, X2 � 0
-
40 40
X

X1 + 2x2 + S1 = 10
X1 - X2 + S2 = 8
y
50
40 cj � 5 3 0 0 Bo Min ratio
30
20
sv-!- X1 X2 S1 S2
10 OS 1 1 2 1 0 10 1 0/ 1
OS2 1 (PE) -1 0 1 8 8/ 1
O(O ,O) 1 0 20 30 4 0 5 0 -+
x
� LV
A(30,0)
zJ 0 0 0 0 0
5x+3y = 150 Ci - Zi 5 3 0 0
+ y = 40 t (EV)
= 40- y
X
OS 1

5 (4 0 - y) + 3 y = 1 5 0
X
5x1 1 -1 0 1 8

200 - 5y + 3y = 150 Z value 40

-2y =-50 No need to calculate other values to save


y = 25 time.
X + y = 40 Zmax = 40

jlJll@@jjji4.jjijjmftjjj111jj� yderabad I Delhi I Bhopal I Pune I Bhubaneswar l Lucknow I Patna I Bengaluru I Chennai I Vtjayawada l Vuag I Tl!Upati I Kukatpally I Kolkata
Zmax = 4 X1 + 6 X2 + X3 + 0 S1 + 0 S2

Five Marks Solutions


Cj � 4 6 1 0 0 Bo mm
s v-!.. X1 X2 X3 S1 S2 Ratio
01. 0 S1 2 -1 3 1 0 5 -5
= 4 X1 + 6 X2 + X3 0 S2 0 (i 0 0 1 2 2 -�
Sol: Zmax LV
z·1 0 0 0 0 0 0
s.t
Cj - Zj 4 EV l 0 0
2 X1 - X2 + 3X3 S 5
@1
2x 1 - X2 + 3x3 + s 1 = 5 Cj � 4 6 1 0 0 Bo min
Zmax = 4x1 + 6x2 + X3 + 0 S 1 s v-1.- X1 X2 X3 s, S2 Ratio
0 s, 2 0 3 1 1 7 7/2 _____.
LV
4 6 1 0 mm 6 X2 0 1 0 0 1 2
Cj �
0 6 0 0 0 12
s v-!.. X1 X2 X3 S1 Bo Ratio Zj
(D 0 1 0 0
0 SJ 2 -1 3 1 5 -5 Cj - Zj

z·1 0 0 0 0 0 t EV
Cj - Zj 4 1 0

Cj � 4 6 1 0 0 Bo mm
s v -!.. x, X2 X3 s, S2 Ratio
4 x1 1 0 3/2 1/2 1/2 7/2
6 X2 0 1 0 0 1 2
Entering vector exists but leaving vector z·1 4 6 6 2 8 26
Cj - Zj 0 0 - 5 -2 -8
doesn't exist as minimum ratio column is
having negative values. It is a case of
= 26;
½; X2 = 2 ; X3 = 0
Zmax
unbounded solution space and unbounded
X1 =
optimal solution to problem.
Zmax = 4 X 1 + 6 X2 + X3 02 .
s.t 2x 1 - x2 + 3x3 s 5 Sol: x + 2y s 200 ; � + L s 1 . . . . ( 1 )
X2 S 2
200 1 00

X1 X2 X3 � 0 X S 1 50 ; . . . . (2)
1 50
�sl
2 X 1 - X2 + 3 X3 + S 1 = 5
X2 + S2 = 2
!11i@pj@jjji4.jjjiAflnj@\jjfo� yderabad I Delhi I Bhopal I Pune I Bhubaneswar I Lucknow I Patna I Bengaluru I Chennai I Vtjayawadal Viz.ag ITuupari I Kukatpally I Kolk.ala
"� " . ACE . . . IM & OR
�-�1.:,..,.,,.. Pntirwn : 904 :

y � 80 ; L �1 . . . (3)
80
Zmax = l OOx + 300y
Comer Point of Solution space are ZA = 1 00 x 1 50 + 300 xO = 1 5,000
0(0, 0) ; A(1 50, 0) ; B(1 50 , 25) ; Zs = 1 00 x 1 50 + 300x 25 = 22500
C(80, 60) ; D(O , 80) Zc = 1 00 x 80 + 300 x 60 = 26000
Zmax = 1 OOx + 300y Zo = 1 00 x O + 300 x 80 = 24000
Zo = l OOxO + 300xO = 0 Zmax = 26000 at x = 80 y = 60 optimal at C
y
1 50 If Zmax = 1 OOx + 200y
Zmax = 1 00 X 80 + 200 X 60
1 00 Zc = 20, 000
D .,____,.�.-------li--
1 50 x 1 00 + 250 x 200=20,000/­
Zs =

50
The problem has multiple solutions

50 1 00 200

�yderabad l Delhi I Bhopal I Pune I Bhubaneswar l LucknowI Palna l Bengaluru I Oicnnai I Vuayawada j Vizag !Tuupati I Kukatpally l Kolkata I
c2 PERT & CPM

One Mark Questions

0 1 . In PERT, the distribution of activity times is


assumed to be (GATE-ME-95)
(a) Normal (b) Gamma 05. A minimal spannmg tree in network flow
(c) Beta (d) Exponential models involves (GATE-ME-14)
(a) all the nodes with cycle/loop allowed
02. A dummy activity is used in PERT network (b) all the nodes with cycle/loop not allowed
to describe (GATE-ME-97) (c) shortest path between start and end nodes
(a) Precedence relationship (d) all the nodes with directed arcs
(b) necessary time delay
(c) Resource restriction 06. In PERT chart, the activity time distribution
(d) resource idleness 1s (GATE - 16 - SET - 3)
(a) Normal (b) Binomial
03 . In PERT analysis a critical activity has (c) Poisson (d) Beta
(GATE-ME-04)
(a) Maximum Float (b) zero Float 07. The standard deviation of linear dimensions
(c) Maximum Cost (d) minimum Cost P and Q are 3µm and 4µm, respectively.
When assembled, the standard deviation (in
04. The expected time (te) of a PERT activity in µm) of the following linear dimension (P+Q)
terms of optimistic time (to), pessimistic(tp) lS ---
and most likely time(tL) is given by (GATE - 17- SET - 2)
(GATE-ME-09)

lli11ii@jjj04ijjjj$(mniNihii!� I I I I I I I I I I I
ydcrabad Delhi Bhopal Punc Bhubanc� Lucknow P�a Bcngalwu Chennai Vijayawada V12ag Tirupati I
Kukatpally Kolkata
"':. � · :� A ,CE
��i-·:rm�,...
· :�========�
: 90 6 �============�:.,;��
IM & OR
.. ":·i;=Fii�1p1�-
".� :n11�1�N ��:
(a) between 1 8, 1 9 day
Two Marks Questions (b) between 20, 22 days
(c) between 24, 26 days
0 I . In the construction of networks, dummy (d) between 60, 70 days
activities are introduced in order to:
(GATE-ME-90) 04. A project has six activities (A to F) with
(a) Compute the slack on all events respective activity durations 7, 5, 6, 6, 8, 4
(b) Transfer resources, if necessary, during days. The network has three paths A-B, C-D
monitoring and E-F. All the activities can be crashed
(c) Clearly designate a precedence with the same crash cost per day. The
relationship number of activities that need to be crashed
(d) Simplify the crashing plan to reduced the project duration by 1 day is
(GATE-ME-OS)
02. A project consists of three parallel paths with (a) l (b) 2 (c) 3 (d) 6
durations and variances of ( 1 0,4), ( 1 2,4)
and(l 2,9) respectively. According to the Statement for Linked Answer QOS & Q06
standard PERT assumptions, the distribution Consider a PERT network for a project involving
of the project duration is (GATE-ME-02) six tasks (a to f)

(a) Beta with mean 1 0 and standard deviation 2. Task Predecessor Expected Variance of
(b) Beta with mean 12 and standard deviation 2. task time the task time
(c) Beta with mean 10 and standard deviation 3. (in days) ( in days2)
(d) Beta with mean 12 and standard deviation 3. a - 30 25
b a 40 64
03. A project consists of activities A to M shown C a 60 81
in the net in the following figure with the d b 25 9
duration of the activities marked in days. e b, c 45 36
The project can be completed f d, e 20 9
(GATE-ME03)
05. The expected completion time of the projects
IS (GATE-ME-06)
(a) 238 days (b) 224 days
(c) 1 7 1 days (d) 1 55 days

�yderahad l Delhi I Bhopal I Pune I Bhubaneswar l lAiclmow l Patna I Beoga)uru I Olcnnai j Vuayawada l V,zag l 1irupati I Kukatpally l Kolkala I
4'.

..
� �.."
:�Pnblicaoooa
ACE : 907 : PERT CPM
4 ==================================

06. The standard deviation of the critical path of The optimistic time, most likely time and
the project is (GATE-ME-06) pessimistic time of all the activities are given in
(a) .Jisi days (b) .Jiss days the table below:

(c) .J200 days (d) m8 days To Tm Tp


T, = cr= J


O' ""
Activity
(�)'
T0 + 4T. +T,
6 6

1 -2 1 2 3 2 1/3 1/9
07. For the network below, the objective is to 1 -3 5 6 7 6 1/3 1/9

find the length of the shortest path from node 1 -4 3 5 7 5 2/3 4/9
2-5 5 7 9 7 2/3 4/9
P to node G. Let dij be the length of directed
3-5 2 4 6 4 2/3 4/9
are from node i to node j . Let Sj be the length 5-6 4 5 6 5 1 /3 1/9
of the shortest path from P to node j . Which 4-7 4 6 8 6 2/3 4/9
6-7 2 3 4 3 1 /3 1/9
of the following equations can be used to
find so? (GATE-ME-OS)
---------:)41o---�e..:Q
08. The critical path duration of the network (in
p days) is (GATE-ME-09)
(a) 1 1 (b) 1 4 (c) 1 7 (d) 1 8
R
09. The standard deviation of the critical path is
(GATE-ME-09)
(a) so = Min{sQ, S R}
(b) so = Min { SQ-doo , SR - dRo}
(a) 0.33 (b) 0.55 (c) 0.77 (d) 1 .66
(c) so = Min{SQ+<lQo , S R + dRo}
(d) So = Min{doo, dRo} 1 0. The project activities, precedence
relationships and durations are described in
Common Data for Question 08 & 09 the table. The critical path of the project is
Consider the following network (GATE-ME-10)

Duration
Activity Precedence
(in days)
p - 3
Q - 4
R p 5
s Q 5
T R,S 7

l11•1ih4hi44ih/QRflMMl!Nh� yderabad I Delhi I Bhopal I Pune I Bhubaneswarl Lucknow4J'atna I Bengalwu I Chennai I Vtjayawada I Vizag I Tuupati r KuEiipany I Kolkala
ACE
.. ":'=�F11�'1)11::M'fl
:nog
� · ==========..:,:;��:�===========IM
· �Pl�dmriooa
�·�

1: 1r I; I
:.-:.
-:,
908 & OR
�:��:

(a) critical path remains the same and the


total duration to complete the project
0 changes to 19 days.
(b) critical path and the total duration to
(a) P-R-T-V (b) Q-S-T-V
complete the project remains the same.
(c) P-R-U-W (d) Q-S-U-W
(c) critical path changes but the total
Statement for Linked Answer Ql l & Q12 duration to complete the project remains
For a particular project, eight activities are to the same.
carried out. Their relationships with other (d) critical path changes and the total
activities and expected durations are mentioned in duration to complete the project changes
the table below. to 17 days.

Activity Predecessors Duration (days)


a - 3 13. A project has four activities P, Q, R and S as

b a 4 shown below.
C a 5 Normal

d a 4
Cost slope
Activity duration Predecessor
(Rs./day)
e b 2
(days)

f d 9
p 3 - 500
7 p
g c,e
Q 1 00
6 R 4 p 400
h f,g 2 s 5 R 200

The normal cost of the project is Rs. 10,000/­


11. The critical path for the project is and the overhead cost is Rs.200 /- per day. If
(GATE-ME-12)
the project duration has to be crashed down
(a) a - b- e - g- h
to 9 days, the total cost (in Rupees) of the
(b) a- c- g - h
project is __ (GATE -ME-14)
(c) a - d - f - h

14. Consider the given project network, where


(d) a - b - C - f - h

12. If the duration of activity "f "alone is numbers along various activities represent
changed from 9 to 10 days, then the the normal time. The free float on activity
(GATE-ME-12) 4-6 and the project duration, respectively, are

!IJ•li@hiii•hiiAflftlM\11jjj� yderabad I Delhi I Bhopal I Pune I Bhubaneswarl Lucknow I Patna I Bengaluru I Chennai I Vtiayawada j V17.3g I Tuupati I Kukatpally I Kolkata
: 909 : PERT CPM

Activity Duration (days)


1 -2 2
2-3 1
4-3 3
1 -4 3
2-5 3
(GATE-ME-14) 3-5 2
(a) 2, 1 3 (b) 0, 1 3 4-5 4
(c) -2, 1 3 (d) 2 , 1 2 The critical path (CP) in the network is
(GATE-15-Set 1)
1 5 . The precedence relation and duration (in
(a) 1 -2-3-5 (b) 1 -4-3-5
days) of activities of a project network are
(c) 1 -2-3-4-5 (d) 1 -4-5
given in the table. The total float (in days) of
activities "e" and "f ", respectively, are 1 7. A project consists of 7 activities. The
Activity Predecessors Duration (days) network along with the time durations (in
a - 2 days) for various activities is shown in figure
b - 4
10 fc\
C a 2 �--0_;
d b 3
e C 2
f C 4
g d, e 5
(GATE-ME-14) The minimum time (in days) for completion
(a) 0 and 4 (b) 1 and 4 of the project is __ (GATE-15 -Set 2)
(c) 2 and 3 (d) 3 and 1
1 8. A project consists of 14 activities, A to N.
1 6. Following data refers to the activities of a The duration of these activities (in days) are
project, where, node l refers to the start and shown in brackets on the network diagram.
node 5 refers to the end of the project. The latest finish time (in days) for node 10
IS----

ilfllQj/§jjj44ijjjiRftliMJhih� yderabad I DeUii I Bhopal I Pune I Bhubaneswar i Lucknow I Patna I Benga)uru I Chennai I Vtjayawada I V12ag I Tuupari I Kukatpally I Kolkata
-:,:.
� ,....,.:FTW,riog PnNimioos
ACE : 910: IM & OR
.. " .. ==========================================
20. A project starts with activity A and ends
with activity F. The precedence relation and
durations of the activities are as per the
following table:
Activity Immediate Duration
predecessor (days)
A - 4
(GATE - 16- SET 2) B A 3
C A 7
19. Consider a network with nodes 1, 2, 3 , 4, 5 D B 14
and 6. The nodes are connected with directed E C 4
arcs as shown in the table below. The F D,E 9
respective costs (in INR) incurred while
traversing the directed arcs are also The minimum project completion time (in
mentioned. days) is ___
Directed arcs Cost (in INR) (GATE - 17 - SET - 2)

21. In a project, tasks A, B , C, D, E, F, G , H, I


1 -2 3
9
and J are to be performed. The precedence
1 -3

relationships and the time required (in days)


2-4 3

to complete the tasks are given in the Table.


2-5 2
3-2 2
3-4 4 Tasks A B C D E F G J
8
H I
3-5 Time 8 10 8 10 16 17 18 14 9 4
4-5 7 (days)
Preceding - - - A A B, C C F ,G E ,
4-6 2
tasks D I,
5-6 2
The second shortest path from node 1 to node
H

6 (i.e. the path that has the second least total The time required (in days) to complete the
cost and does not use any part of the shortest project along the critical path is ___
path) has a total cost (in INR) of (GATE-PI - 17)
(GATE - PI- 16)
(a) 7 (b) 8 (c) 1 5 (d) 1 9

IM•li@jjjl4imQRftbiM\Uljj� yderabad I Delhi I Bhopal I Pune I Bhubaneswar I Lucknow I Patna I Bengalwu I Chennai I Vuayaw.ida I Vizag � Tll';'pati I Kukatpally I Kolkata
: 91 1 : PERT CPM

(c) Determine the peak requirement of


Five Marks Questions money and the day on which it occurs in
above schedule.
0 1 . For a small project with five jobs, the
following data is given (GATE-ME-93) 03. A project plan is given below:
Activity Time duration Predecessors
Job Immediate Duration (days)
in weeks
A 2 None
predecessors mean Std. deviation

2 None
A - IO 2
- B
C 7 A
B 5 1
C A
12 A
16 2
D A 12 2 D
10
E B,C 15 1
E B
(i) Draw the project network in activity on F 3
D, E
arc mode G 4 C, F
(ii) Under PERT assumptions, determine the (a) Construct a PERT network
distribution of project duration. (b) Find the critical path and estimate the
project duration. (GATE-ME-97)
02. A project with the following data is to be
(GATE-ME-95)
04. Given below are the data for a project
(GATE-ME-00)
implemented:
network.
Activity Predecess Duration Cost
Immediately
ors (s) (days) (Rs/day) Activity Duration
preceding
A - 2 50
A -- 3
B - 4 50 ---
B 5
C A 1 40 C A, B 5
D B 2 1 00 D A 4
E A, B 3 1 00 E C, D 4
F E 2 60 F A 4
(a) What is the mm1mum duration of the G A, B 6
project? (a) Draw the network for the above project
(b) Draw a Gantt chart for the early capturing the precedence relationships.
schedule (b) Find the critical path and its duration.
\( J } 11 .,lllt t IIIH.!, P11'1]1l ,llH111, �yderabad I Delhi I Bhopal I Punc I Bhubancswarl Lucknow I Patna I Bcngaluru I Chennai I Vijayawada I Vu.ag I Ttrupati I Kukatpa)]y I Kolkata I
: 912: IM & OR

05. The precedence relations and durations of


jobs in a project are given below:

Job Predecessor(s) Duration


(in days)
A - 2
B - 4
C A 6
D A 8
E B,C 6
F B,C 4
G F 2
H F 8
I D,E,G 6

(a) Draw the activity-on-arc project network


(b) Determine all critical path(s) and their
duration(s).
(c) What is the total float for jobs B and D
(GATE-ME-02)(5M)

!l!lli@MIIIOIQRbbiMhi@+iyderabad I Delhi I Bhopal I Pune I Bhubaneswarl ��, Patml Bengaluru I Chennai I Vuayawadaj vizag mrupali I Kukatpally l Kolkala I
SOLUTIONS
04. Ans: (c)
One Mark Solutions Sol:
Path Duration
0 1 . Ans: (c) 02. Ans: (a) AB 7+5= 1 2
CD 6+6=1 2
03. Ans: (b) 04. Ans: (a) EF 8+4=1 2

05. Ans: (b) Three critical paths, number of activities to


Sol: Definition of minimal spanning tree be Crashed are 3
connects all nodes with no loop and
minimum distance. 05. Ans: (d)
06. Ans: (a)
06. Ans: (d) Sol:
Paths Durations
07. Ans: 5 a b d f 30+40+25+20=1 1 5
a b e f 30+40+45+20=135
Sol: Variance (P+Q) a c e f 30+60+45+20=155
= variance (P) + variance (Q)
Variance (P+Q) = 3 2 + 42 b
cr2 = 25
Standard deviation (P+Q) = ,v25 = 5µm ft\ .
v-;;- 2

Two Marks Solutions

Critical Path Duration days is 155


0 1 . Ans: (c) 02. Ans: (d)
{Var)cP = (Vart + {Var)c + {Vart + {Var)r

03. Ans: (c) = 25 + 8 1 + 36 + 9 = 1 5 1


Sol: Longest path is critical path Standard deviation of critical path
C-F-K-M = 4 + 9 + 3 + 8 = 24 days (<Jcp) = �L, (Var)CP = .Jisi days

erabad I Delhi I Bhopal I Pune I Bhubaneswarl I.J.ick.nowl


�yd �I Bengaluru I Chennai I Vyayawada I Vizag I Tirupati I Kukatpally I Kolkala I
�CE : 914 : IM & OR
'!.�-�
ti, V

� � ======================================
• • • •
: �Pww:aiooa

07. Ans: (c) 11. Ans: (c)


Sol:
08. Ans: (d)
09. Ans: (c)
Common Solution for Q.8 & Q.9

Path Duration
abegh 3 + 4 + 2 + 6 + 2 =17
a cg h 3 + 5 + 6 + 2 =16
adfh 3 + 4 + 9 -l- 2 = 18 � critical path
Path Duration
Longest path = a d f h =18 days
1-2 -5-6-7 2+7+5+3= 17
1-3-5-6-7 6+4+5+3 = 18�Critical Path
12. Ans: (a)
1-4-7 5+6 = 11
Sol: Activity "f' is on critical path and its
duration is changed from 9 days to 10 days
Standard Deviation,
hence the duration of critical path changes
1 4 1 1
- + - + - + - = 0.77 to 19 days.
9 9 9 9
crcp =

10. Ans: (d) 1 3 . Ans: 12,500/­


Sol: Sol:

Paths Duration Paths II


P R TV 3 + 5+ 7 + 2 = 17
I
PRUW 3 + 5+ 5 + 10 = 23 PQ 3+7=10 10 9
Q S TV 4 + 5+ 1 + 2 =A PRS 3+4+5=12 10 9
Q S UW 4 + 5+ 5 + 10 � � Critical path

!l1•l4@jjj4i.jjjQRfl§j@jjj.jj+yderabad I Delhi I Bhopal I Pune I Bhubaneswarl Lucknow I Patna I Bengaluru I Chennai I Vyayawada j V,zag I Tuupati I Kukatpal)y I Kolkat, I
ACE
"�•tv1i'....:......:...Pub1icatioos : 915 : PERT & CPM
� �=� ====� ==================================
Critical path is 12 days 14. Ans: (a)
TC = Total Cost for 12 days Sol:
( TC)1 2 = Normal Cost + Overhead Cost
= 10,000 +(12x200) = 12,400 3 3 3

P = 500 /- per day


R = 400 /- per day
S = 200 /- per day
' S' least cost slope crash by 2 days at a cost of
200 /- per day as after two days of crashing
4

another path is becoming critical.


( TC)1 0 = Previous Cost + Crash cost- Overhead
Cost
4 Frei float 8

12 ,400 + (2x200) - (2x200) = 12,400


Critical path = 13 days
=

Now two paths are critical the possibilities are


Free float = Ej- Tij -Ei 8 - 4- 2 = 2
P = 500 /- per day- common activity
=

15. Ans: (b)


Q and R 500 /- per day- Sol:
=
Q= I OO }
R =400 SOO/-
12

Q and S = 300 /- per day -


Q= l OO } 300/­
S=200

As the minimum cost slope option is 300 /- per


day select Q & S for crashing by 1 day. As lower
limit is not given an activity can be crashed by EST
any number of days.
( TC)9 = ( TC)1 o + Crash cost - Indirect cost
LFT O 4

= 12,400 + (300x l ) - (200x l )


= 1 2,500/- (TF)e = (Lj - Ei) - Tu = 7- 2- 4 = 1
( TF)r = (Lj - Ei)-Tij = 12- 4- 4 = 4

!l"S•lii@jjj§§mSRflniffl@U•� yderabad l Delhi I Bhopal I Pune I Bhubanesw:u'I Lucknow! Patna I Bengaluru I Chennai I Vijayawada I Vizag I Trrupati I �Y I Kolkata
"�,,,
�..
�j"" • ACE
• I." • •
916 · IM & OR
'":·�=&�IDl�-
:crq
:�� ���-���:
,..
·:::=======::..:.·��·=============����

16. Ans: (b)


Sol:
Critical Path is 1-4-3-5 , Duration = 8 days

17. Ans: 39 to 40
Sol:

Paths Duration
1-3-5-6 1 2+ 1 1 + 1 0=33
Paths Duration 1-2-5---0 14+ 1 2+ 1 0=36
1-4-5 3+4=7 1-2-4-5---0 14+7+9+ 1 0=40(CP)
1-4-3-5 3+3+2=8
Critical Duration = 40 days
1-2-3-5 2+1 +2=5
Longest path = minimum time for completion.
1-2-5 2+3 =5

18. Ans: 14
Sol:

(LFT) for node 10 = 14 days

19. Ans: (c) Path Cost


Sol: 1 -3 -4-6 9+4+2 = 1 5
1 - 3 - 2 -4- 6 9+2+3+2 = 1 6
1 -3 -4- 5 -6 9+4+7+2 = 22
1 -3 -2 - 5 - 6 9+2+2+2 = 1 5
1 -3 -2-4-5 -6 9+2+3+7+2 = 23
1 -2-4-6 3+3+2 = 8
1-2-5-6 3+2+2 = 7
1 -2-4-5-6 3+3+7+2 = 1 5
1 - 3 - 5 -6 9+8+2 = 1 9

jlJ11i!i§iU§l.jji4RflijjMJi1jjj� ydcrabad I Delhi I Bhopal I Pune l Bhubaneswarl Lucknow ! Patna I Bcngalwu I Chennai I Vuayawadal V11.3g I Tirupari I Kukatpally l Kolkata
From the given statement, we got shortest
path (least total cost) is 1-2-5-6 and a path Paths Duration
which does not have 1 -2, 2-5, 5-6 activities A -E-J 8+ 1 6+4=28
should be considered. A-D-F-I-J 8+ 1 0+ 1 7+9+4 = 48
The next path which does not have the above B-F-I-J 10+ 17+9+4=40
activities is 1 -3-4-6 = 1 5 and 1 -3-2-4-6 = 1 6. C-G-I-J 8+18+9+4 = 39
.·. In this second least total cost is 15. C-H-J 8+ 14+4 = 26

20. Ans: 30 Critical path is ADFIJ = 48 days


Sol:

F(9)
Five Marks Solutions
r;\
1----0
01.
D

Paths
ABDF 4+3+ 1 4+9 = 30 days
ACEF 4+7+4+9 = 24 days

Longest path is critical path duration = 30


days CP � A C E = 41 days = Te

21. Ans: 48 Paths Duration


Sol: AF 3+4 =7
A DE 3 + 4 + 4 = 11
ACE 3 + 5 + 4 = 12
BCE 5 + 5 + 4 = 14
BG 5+6 =1 1

cr cP = �(Var)A + (Var) c + (Var) E

= ,J2 2 + 22 + 12 = ,Jg = 3
14

!ltlli@jjji4jjjjiPflbi@\!ijjj+yderabad I Delhi I Bhopal I Pune I Bhubaneswar I Lucknow j Patna I Benga)wu I Chennai I Vijayawada IVi7.ag I TlfUpati I KuI<atpally I Kolkata I
:.�
-:, . :A:CE
t'J!;fn�'llll
�- ���Jb�.��. :�=======�:==�:,:::==============IM
crmg Pl Ji.rp"lDl :;;:;,;;�;,;;;:.,. 918 & OR
... .. =
Max CT = Te + 3 = 41 + 3 x 3 = 50 03.
Sol:
O'cp

Min CT = Te - 3 O'CP = 41- 3 x 3 = 32

02.
Sol:
2 C

Paths in Network

0� .
J
A
ACG = 2 + 7 + 4 = 13
EST . )P -..... � .E �

ADFG = 2 + 12 + 3 + 4 = @
/�� 2- \V
B ;l 4 7 9

BEFG = 2 + 10 + 3 + 4 = 19
4
3 J---�,._________,,
LST O / .
4 / D
2 Critical Path - Longest Path - ADFG
Duration - 21 days
4

CP � B E F � 4 + 3 + 2 = 9
04.
Sol:
1 2 5 6 7 8
A
4 9

50 50 40

- _ ._ E

1
B Critical path � B C E � 14 days
50 50 50
05.
100 100 90 100 60 60 Sol:

Peak resource requirement


= 200 in 5th and 6th days.

!Ml•iiMhii411iiiRflniM\Uih� I
yderabad I Delhi I Bhopal I Pune I Bh_ubartesw.ii- Luck.how I Patna I Bengaluru I Chennai I Vijayawada I Viz1g_ 0pati I KukatwilY I Kolkata
(LST)s = (LFT)8-Duration = 8-4 = 4
Paths Duration
�(TF)e = LST-EST = 4-0 = 4
ADI 2+8+6 = 16
ACEI 2+6+6+6 = 20 � CP (LST)o = (LFT)o - Duration = 1 4-8 = 6
ACFGI 2+6+4+2+6 = 2 0 � CP �(TF)o = 6 - 2 = 4
BEi 4+6+6 = 16 = LST - EST = 6 - 2 = 4
BFGI 4+4+6+2 = 16
BFH 4+4+8 = 16

rabad I Delhi I Bhopal ! Pune j Bhubaneswarj Lucknow ! Pama ! Bengaluru j Oiennai I Vtjayawada l V,zag I Tuupati I Kubtpally l Kolkala I
�ydc
Queuing Theory
distributed service time. The average number
One Mark Questions of the customers in the queue will be.
(GATE-ME-06)
01 . The cost of providing service in a queuing (a) 3 (b) 3.2 (c) 4 (d) 4.2
system increases with (GATE-ME-97)
(a) Increased mean time in the queue 04. In an M /M /1 queuing system , the number of
(b) Increased arrival rate arrivals in an interval of length T is a Poisson
(c) Decreased mean time in the queue random variable (i.e. , the probability of there
(d) Decreased arrival rate being n arrivals in an interval of length T is
e-)..T (JTr
---- ). The probability density
02. Consider a single server queuing model with n!
Poisson arrivals (11,= 4 /hour) and exponential function f(t) of the inter-arrival time is given
service (µ = 4 /hour). The number in the (GATE-ME-08)
system is restricted to a maximum of 1 0. The
( e )..2/
(b)
probability that a person who comes in 12 J
leaves without joining the queue is
e A.I
(GATE-ME-OS) (d) _-
-

A
1
(a) - (b)
11 10
_!_
05. Little's law is a relationship between
1
(c) - (d) _!_ (GATE-ME-10)
2
(a) stock level and lead time in an inventory
9

system
03. The number of customers amvmg at a
(b) waiting time and length of the queue in a
railway reservation counter is Poisson
queuing system
distributed with an arrival rate of eight
(c) number of machines and job due dates in
customers per hour. The reservation clerk at
a scheduling problem
this counter takes six minutes per customer
(d) uncertainly in the activity time and
on an average with an exponentially
project completion time

!Mliii@jj6ihi4RflbiM@¥f/iyderabad I Delhi I Bhopal I Pune I Bhubaneswarj Lucknow I Patna I Bengalwu I Chennai I VijayawadaJ Vizag I Tirupati I Kukatpally I Kolkata I
"-:. . Queuing Theory
�-�;FNIIW'J'JIW PuNiraiooa
� . ACE
. . : 921 :

06. Cars arrive at a service station according to (a) Service time distribution and queue
Poisson's distribution with a mean rate of 5 discipline
per hour. The service time per car is (b) Number of servers and size of calling
exponential with a mean of 10 minutes. At source
state, the average waiting time in the queue is (c) Number of servers and queue discipline
(GATE-ME-1 1) (d) Service time distribution and maximum
(a) 10 min (b) 20 min number allowed in system
(c) 25 min (d) 50 min
10. In a single-channel queuing model, the
07. Customers arrive at a ticket counter at a rate customer arrival rate is 12 per hour and the
of 50 per hr and tickets are issued in the servicing rate is 24 per hour. The expected
order of their arrival. The average time taken time that a customer is in queue is ___
minutes. (GATE - 16-SET - 2)
for issuing a ticket is 1 min. Assuming that
customer arrivals form a Poisson process and
11. For a single server with Poisson arrival and
service times are exponentially distributed,
exponential service time, the arrival rate is
the average waiting time in queue in min is
12 per hour. Which one of the following
(GATE-ME-13) rates will provide a steady state finite queue
(a) 3 (b) 4 (c) 5 (d) 6 length? (GATE - 17 - SET - 2)
(a) 6 per hour (b) 10 per hour
08. The jobs arrive at a facility, for service, in a (c) 12 per hour (d) 24 per hour
random manner. The probability distribution
of number of arrivals of jobs in a fixed time
interval is (GATE-ME-14) Two Marks Questions
(a) Normal (b) Poisson
(c) Erlang (d) Beta
01. On the average 100 customers amve at a
09. In the notation (a /b /c) (d /e /f) for place each hour, and on the average the
server can process 120 customers per hour.
summarizing the characteristics of queuing
What is the proportion of time the server is
situation, the letters 'b' and 'd' respectively
idle?
for
(GATE-ME-95)
(GATE -15 -Set 3)

!l!l1ib§jjji4.jji4AfinjMjjQj+iyderabad I Delhi I Bhopal I Pune I Bhubaneswarl Lucknow I Patna! Bengalwu I Chennai I Vuayawada j Vizag I T11Upari I Kukalpal)yl Kolkata I
: 922 : IM & OR

02. At a production machine, parts amve queue discipline. Breakdowns occur on an


according to a Poisson process at the rate of average of 3 per day with a range of zero to
0.35 parts per minute. Processing time for eight. The maintenance crew can service an
parts have exponential distribution with average of 6 machines per day with a range
mean of 2 minutes. What is the probability of zero to seven. The mean waiting time for
that a random part arrival finds that there are an item to be serviced would be (ME-04)
already 8 parts in the system (in machine + in 1 1
(a) - day (b) - day
queue)? (GATE-ME-99) 6 3
(a) 0.0247 (b) 0.0576 (c) 1 day (d) 3 days
(c) 0.01 73 (d) 0.082
06. Jobs arrive at a facility at an average rate of 5
03. In a single serve infinite population queuing in an 8 hour shift. The arrival of the jobs
model, arrivals follow a Poisson distribution follows Poisson distribution. The average
with mean A = 4 per hour. The service times service time of a job on the facility is 40
are exponential with mean service time equal minutes. The service time follows
to 1 2 minutes. The expected length of the exponential distribution. Idle time (in hours)
queue will be (GATE-ME-00) at the facility per shift will be
(a) 4 (b) 3.2 (GATE-ME-14)
(c) 1 .25 (d) 24.3 5
(b)
14
(a) -
7 3
04. Arrivals at a telephone booth are considered 7
(c) - (d) .!Q
to be Poisson, with an average time of 1 0 5 3
minutes between successive arrivals. The
length of a phone call is distributed 07. At a work station, 5 jobs arrive every minute.
exponentially with mean 3 minutes. The The mean time spent on each job in the work
probability that an arrival does not have to station is 1 /8 minute. The mean steady state
wait before service is (GATE-ME-02) number of jobs in the system is ___
(a) 0.3 (b) 0.5 (c) 0.7 (d) 0.9 (GATE-ME-14)

05. A maintenance service facility has Poisson 08. Arrival of machines for repair in a
arrival rates, negative exponential service maintenance shop follows a Poisson
time and operates on 'first come first served ' distribution at a rate of one per 1 8 hours. The

\<. l l 11i.;111t t 111 1i.; P1dil1l ,1tJl J11-. �ydcrabad j Dclhi l Bbopa) I Punc l Bhu�cswarl Lucknowl Patna l BcngaJuru l Chcnnai l Vtjayawada j V17.ag I Tirupati I Kukatpally l KQ1kara. I
: 923 : Queuing Theory

time to repair follows an exponential


distribution with Mean Time To Repair
Five Marks Questions
(MTTR) of 1 4 hours. If the productivity loss
is Rs. 22,500 per hour, then the total 0 1 . People arrive at a hotel in a Poisson
expected loss of productivity due to machine distributed arrival rate of 8 per hour. Service
breakdowns, in Rs., is time distribution is closely approximated by
(GATE - PI -15) the negative exponential. The average service
(a) 78,750 (b) 1 ,0 1 ,250 time is 5 minutes. Calculate (a) the mean
(c) 1 1 ,8 1 ,250 (d) 1 4, 1 7,500 number in the waiting line; (b) the mean
number in the system; (c) the waiting time in
the queue; (d) the mean time in the system
and (e) the utilization factor.
(GATE-ME-97)

�ydcrabad j Delhi l Bhopal j Pune l Bhubancswarl i.;.cknow l Patna l BengaJuru l Chennai l VuayawadajVu.ag ITirupari I Kukatpal)yj Kolkata I
SOLUTIONS
07. Ans: (c)
One Mark Solutions Sol: A = 50 hr- 1 = Arrival Rate
µ = 60 hr- 1 = Departure Rate
0 1 . Ans: (c) A 50
W = =
q µ(µ - 1., ) 60(60 - 50)
02. Ans: (a)
= _!_ hours = 5 minutes
4 12
Sol: A = 4 hour- 1 , µ = 4 hour- 1 , p = "' = = 1 ,
µ 4
Po + P 1 + P2 +....... +Pio = 1 08. Ans: (b)
Po + pPo + p2Po +..... ....= 1 Sol: Random Arrivals is an example of Poisson
Po( 1 + p + p2+ .... + p 1 0) = l distribution.
1
As p = 1 ' Po = -
11 09. Ans: (d)
Sol: (a/b/c : d/e/f)
03. Ans: (b) a : Inter arrival time distribution
Sol: A = 8 hour -I b : service time distribution
1 c : number of servers
µ = l O hour -
L = 2
--,---
"' = 8
2 d : number of buffers (system capacity)
= 3 .2
q µ(µ - 1., )
--,--

1 0(10 - 8 ) e : population size


f : service discipline
04. Ans: (c) 05. Ans: (b)
1 0. Ans: 2.5 (range 2.4 to 2.6)
1
06. Ans: (d) Sol: A = 1 2 hr-
µ = 24 hr-I
12
60
Sol: "- = 5/hr , µ = - / hr
10 "A,
W =
A = 5
q µ(µ - "J..) 24(24 - 1 2 )
W =
q µ(µ - A) 6(6 - 5) 1
= - x 60 = 2.5 mimtes
24
= � hr � � x 60 = 50 min
6 6
FydenbadJ Dclhi J BhopalJ Puncl BhubaneawarJ LucknowJ PatnaJ Bcngaluru J ChcnnaiJ Vuayawada JV123g JTirupati I KukatpallyJ Kolkata I
: 925 : Queuing Theory

11. Ans: (d) 05. Ans: (b)


Sol: Arrival rate = "A = 1 2 hr- 1
Sol: "A =
3 day - 1 , µ = 6 day -
1

For finite Que length departure rate must be 1 1 1


W = -- = -- = - days
greater than arrival rate. (µ > "A) s µ - "A 6 - 3 3
.". µ = 24 hr- I
06. Ans: (b)
Sol: "A = Arrival Rate
Two Marks Solutions
Numberof Arrivals � hr -i
= =
01. Time taken 8
Sol: "A = 1 00 hr -i µ = Service Rate
I
µ = 1 20 hr - Number of Departures _!_ I hr _1
= =
2 2
=
"A 1 00 1 Time taken
Po = 1 - - = 1-- = - 3
µ 1 20 6
PO probability of idleness.
=

02. Ans: (c) (or) probability of no customer in system


Sol: A = 0.35 min- 1 "A 7 ½
(P ) _
- 1 --_ - 1--=-
µ = 0.5 min- 1
0 µ 12 ½
Idle Time (in hours) =
probability of Idleness x Hours per shift
0.35 0.35
8
7 14
= - x 8 = - hours
(1 - )( ) = 0.01 73
0.5 0.5 12 3
=

03. Ans: (b) 07. Ans: 1.67


Sol: "A = 4 hr- ; µ = 5 hr-
1
Sol: A =
1
Numberof arrivals � _
= = 5 min 1
;

4X 4 Time taken 1
Lq = = = � = 3 .2
2

µ(µ - "A) 5(5 - 4) 5 Numberof Departures 1


"'

µ= = __ = 8 min _1
Time taken
04. Ans: (c)
½
Mean number of jobs in system
Sol: J = 0. 1 min· 1 µ =0.33 min- 1
')... 5 5
"A L 8 = -- = -- = - = 1 .67
� p0 = 1 - p 1 - - = 1 - 0.3 = 0.7 µ - "A 8 - 5 3
µ
=

llJl•i!@jjji4iOAAflnji1jjM.-Iyderabad I Dellii I Bhopal I Pune I Bhubaneswar I Lucknow I Patna I Bengaluru I Chennai I Vliayawada j Vttag I Tirupati I KukatpalJy I Kolkala I
: 926 : IM & OR

08. Ans: (d)


1 1
Sol: A = -hr , µ = - hr
18 14 Five Marks Solutions
l
w s = --
µ - ')..
1 1 01.
= --- = ------- 63 hrs
1 1 0.07 1428 - 0.05555 Sol: ').. = 8 hr-I
14 1 8 µ = 12 hr-I
Total loss = Ws x loss/hr /1,2 8X8
a) L = = = ! = 1 .33
= 63 X 22500 = 1 4 1 7500 q µ(µ - ')..) 1 2(1 2 - 8) 6
Time spent in system = loss of production ').. 8
b) Ls = -- = -- = 2
time µ - ').. 1 2 - 8

=
').. 8 1
c) W = = hrs
q -
µ(µ ')..) 1 2(12 - 8) 6
1 1 1
d) Ws = -- = -- = -hrs
µ - ').. 1 2 - 8 4
').. 8
e) p = - = - = 0.67
µ 12

\( I l 11!.!,ttlt t 1 111!.!. Pt1hlh .1t11 1 1 1 ... �yderabadl Delhi I Bhopal I Punej Bhubaneswar j l..ucknowj Patna j Benga)uru l Chennai I Vtjayawada ! Vizag f 't"uupan1 Kukatpallyl �llwaI
C4 Inventory Control
======================="

(d) In PRS, inventory holding costs are


One Mark Questions higher than in Fixed Reorder Quantity
System
01 . If the demand for an item is doubled and the
ordering cost halved , the economic order 04. In inventory planning, extra inventory 1s
quantity (GATE -ME-95) unnecessarily carried to the end of the
(a) remains unchanged planning period when using one of the

(b) increases by factor of Jz following lot size decision polices:


(GATE-ME-98)
(c) is doubled
(a) Lot -for- lot production
(d) is halved
(b) Economic Order Quantity (EOQ) lot size
02. Setup costs do not include (c) Period Order Quantity (POQ) lot size
(GATE-ME-97) (d) Part Period total cost balancing

(a) Labour cost of setting up machines 05. An item can be purchased for Rs. 1 00. The
(b) Ordering cost of raw material ordering cost is Rs.200 and the inventory
(c) Maintenance cost of the machines carrying cost is 1 0% of the item cost per
(d) Cost of processing the work piece annum. If the annual demand is 4000 units'
the economic order quantity (in units) is
03. One of the following statements about PRS (GATE-ME-02)
(Periodic Reordering System) is not true. (a) 50 (b) 100 (c) 200 (d) 400
Identify. (GATE-ME-98)
(a) PRS requires continuous monitoring of 06. The word Kanban is most appropriately
inventory levels associated with (GATE-ME-11)
(b) PRS is useful in control of perishable (a) economic order quantity
items (b) just-in-time production

(c) PRS provides basis for adjustments to (c) capacity planning


account for variations in demand ( d) product design

!ltlli@jj/14ijjjjRflftiMilliii+1Yderabad j Delhi j Bhopal j Pune i Bhubaneswarl Lucknow j Pama j Benga)uru!Chennai j Vuayawada jV,zag jTuupati I Kukatpal)yi Kolk/da , -
: 928 : IM & OR

07. Demand during lead time with associated (a) 500 (b) 2000
probabilities is shown below: ( C) 1 000 /,fj_ ( d) 1 ooo,fi,

Demand 50 70 75 80 85 02. When the annual demand of a product is


Probability O.1 5 0.1 4 0.21 0.20 0.30 24000 units, the EOQ (Economic Order
Expected demand during lead time is ___ Quantity) is 2000 units. If the annual demand
(GATE-ME-14) is 48000 units the most appropriate EOQ will
be (GATE-ME-91)
08. Annual demand of a product is 50,000 units (a) 1 000 units (b) 2000 units
and the ordering cost is Rs.7000 per order. (c) 2800 units (d) 4000 units
Considering the basic economic order
quantity model, the economic order quantity 03. In computing Wilson's economic lot size for
is 1 0000 units. When the annual inventory an item, by mistake the demand rate estimate
cost is minimized, the annual inventory used was 40% higher than the true demand
holding cost (in Rs.) is ____ rate. Due to this error m the lot size
(GATE-15-Set 2) computation, the total cost of setup plus
inventory holding per unit time. Would rise
09. The throughput rate of a production system is above the true optimum by approximately
20 units per hour. The average flow time is (GATE-ME-99)
30 minutes and the cycle time is 3 minutes. (a) 1 .4% (b) 6.3% (c) 1 8.3% (d) 8.7%
The average inventory (in units) in the
system is (GATE - PI-16) 04. Market demand for springs is 8,00,000 per
(a) 1 .5 (b) 9 (c) 1 0 (d) 1 1 .33 annum. A company purchases these springs
in lots and sells them. The cost of making a
purchase order is Rs. 1 ,200. The cost of
I Two Marks Questions I
storage of springs is Rs. 1 20 per stored piece
per annum. The economic order quantity is
0 1 . In an ideal inventory control system, the
(GATE-ME-03)
,, . economic lot size for a part is 1 000. If the (a) 400 (b) 2,828
annual demand for the part is doubled, the
(c) 4,000 (d) 8,000
new economic lot size required will be:
(GATE-ME-89)

,js111n •·hiii,taiMiiiih*·rYderabad I Delhi I Bhopal I Pune I Bhubaneswar I Lucknow I Pama I Bengaluru I Chennai IVuayawada jVu.ag I Tuupati I Kukatpally I Kolkata I
" . ACE
"i:i . . . Inventory Control
�-�;Eupwt1P1 PoNiaams : 929:

05. There are two products P and Q with the 08. Consider the following data for an item.
following characteristics Annual demand: 2500 units per year
(GATE-ME-04) Ordering cost: Rs. I 00 per order, Inventory
Demand Order cost Holding Cost holding rate: 25% of unit price. Price quoted
by a supplier (GATE-ME-06)
Product
(units) (Rs./order) (Rs.Ju nit/year)
100 50 4
Order quantity(units) Unit Price(Rs.)
p
Q 400 50 1
< 500 10
The Economic Order Quantity (EOQ) of
� 500 9
products P and Q will be in the ratio.
(a) 1:1 (b) 1:2 (c) 1:4 (d) 1:8 The optimum order quantity (in units) is
(a) 447 (b) 471 (c) 500 (d) � 600
06. A company has an annual demand of 1000
units, ordering cost of Rs.I 00 per order and 09. A stockiest wishes to optimize the number of
carrying cost of Rs. 100 per unit-year. If the perishable items he needs to stock in any
stock-out costs are estimated to be nearly Rs. month in his store. The demand distribution
400 each time the company runs out-of­ for this perishable item is
stock, then safety stock justified by the (GATE-ME-06)
carrying cost will be (GATE-ME-04)
Demand
(a) 4 (b) 20 (c) 40 (d) 100
(in units)
2 3 4 5

Probability
07. The distribution of lead time demand for an
0. 1 0 0.3 5 0.3 5 0.20

item is as follows: (GATE-ME-05)


The stockiest pays Rs. 70 for each item and
Lead time demand Probability
he sells each at Rs. 90. If the stock is left
80 0.20
unsold in any month, he can sell the item at
100 0.25
Rs. 500 each. There is no penalty for
120 0.30
unfulfilled demand. To maximize the
140 0.25
expected profit, the optimal stock level is
The reorder level is 1.25 times the expected
(a) 5 units (b) 4 units
value of the lead demand. The service level is
(c) 3 units (d) 2 units
(GATE-ME-05)
(a) 25% (b) 50% (c) 75% (d) 100

!M1•i@idi4.jjj4/Ml@jjjjjj� yderabad I Delhi I Bhopal I Pune I Bhubaneswar I wcknow l Patna I Bengaluru I Chennai I Vijayawada! Viz.ag I Trrupati I Kuka
"-:...... . .
.., ,......." . ACE
t�� . : 930 : IM & OR

10. The maximum level of inventory of an item 13. Annual demand for window frames is 10000.
is 100 and it is achieved with infinite Each frame costs Rs.200 and ordering cost is
replenishment rate. The inventory becomes Rs.300 per order. Inventory holding cost is
zero over one and half month due to Rs. 40 per frame per year. The supplier is
consumption at a uniform rate. This cycle willing to offer 2% discount if the order
continues throughout the year. Ordering cost quantity is 1000 or more, and 4% if order
is Rs.I 00 per order and inventory carrying quantity is 2000 or more. If the total cost is
cost is Rs. 10 per item per month. Annual to be minimized , the retailer should
cost (in Rs.) of the plan, neglecting material (GATE-ME-10)
cost, is (GATE-ME-07) (a) order 200 frames every time
(a) 800 (b) 2800 (b) accept 2% discount
(c) 4800 (d) 6800 (c) accept 4% discount
(d) order Economic Order Quantity
11. In a machine shop, pins of 15mm diameter
are produced at a rate of 1000 per month 14. Consider the following data with reference to
and the same is consumed at a rate of 500 per elementary deterministic economic order
month. The production and consumption quantity model
continue simultaneously till the maximum Annual demand of an item 100000
inventory is reached. Then inventory is Unit price of the item (in Rs.) 10
allowed to reduced to zero due to
Inventory carrymg cost per 1.5
consumption. The lot size of production is
unit per year (in Rs.)
1000. If backlog is not allowed, the
Unit order cost (in Rs.) 30
maximum inventory levels is
The total number of economic orders per
(GATE-ME-07)
year to meet the annual demand is ---
(a) 400 (b) 500 (c) 600 (d) 700
(GATE -ME-14)
12. A company uses 2555 units of an item
annually. Delivery lead time is 8 time is 15. A manufacturer can produce 12000 bearings
days. The recorder point (in number of units) per day. The manufacturer received an order
to achieve optimum inventory is of 8000 bearings per day from a customer.
(GATE-ME-09) The cost of holding a bearing in stock is
(c) 56 (d) 60 Rs.0.20 per month. Setup cost per production
run is Rs.500. Assuming 300 working days
(a) 7 (b) 8

!1i•1ii@jjjl4@4Rflbii\hijj� ydcrabad l Dclhi l BhopaI I Pune l Bhubancswar l wcknow l Patna l BcngaJwu ! Chcruiai l Vuayawada l Vizag I. · 1 Kukatpal)y I Kolkata
., " . .
':o�-��F..,,,,,.P®Jiranoos
ACE .. : 931 : Inventory Control

in a year, the frequency of production run charge on an annual basis is 20% of the
should be (GATE -ME-2014)(Set-3) purchase price of the com flour per kg. The
(a) 4.5 days (b) 4.5 months optimal order quantity (in kg) is ___
(c) 6.8 days (d) 6.8 months (GATE - 16 - SET - 2)

1 6. The annual requirement of rivets at a ship 19. An electrical appliances showroom sells
manufacturing company is 2000 kg. The 2,400 ceiling fans in one year (52 weeks).
rivets are supplied in units of 1 kg costing Rs. The holding cost is 1 0% of the cost of the
25 each. If it costs Rs. 1 00 to place an order ceiling fan. The most of one ceiling fan is
and the annual cost of carrying one unit is Rs. 600. The cost incurred for placing an
9% of its purchase cost, the cycle length of order is Rs. 20 1 . There is a lead time of 5
the order (in days) will be ____ weeks. The economic order quantity (EOQ)
(GATE -15 -Set 3) and the reorder level, respectively (rounded
to the next higher integer) are
17. The annual demand for an item is 1 0,000 (GATE - PI - 1 7)
units. The unit cost is Rs. 1 00 and inventory (a) 23 1 , 1 27 (b) 38, 23 1
carrying charges are 14.4% of the unit cost (c) 127, 23 1 (d) 1 27, 1 3
per annum. The cost of one procurement is
Rs.2000. The time between two consecutive 20. The potential production alternatives for
orders to meet the above demand is manufacturing a product along with their
month(s). (GATE - 16 - SET - l) unit cost and monthly capacity are given in
the Table.
1 8. A food processing company uses 25,000 kg
of com flour every year. The quantity­ S.NO Production Unit Capacity I
discount price of com flour is provided in the Alternatives cost month
table below: (Rs.)
Quantity (kg) Unit price (Rs/kg) 1 Regular time 5 300
1 -749 70 production
750- 1499 65 2 Overtime 6 200
1 500 and above 60 production
3 Subcontrating 10 500
The order processing charges are Rs.
500/order. The handling plus carry-over
\( I l 11�111t t 1 111� P1il1\a,1:H111, �yderabad l Delhi ! Bhopal I Punc I Bhubancswar l I.JJcknow l Patna l Bcngaluru j Cheruiai I Vliayawada j V,zag j Tll'Upali I Kukatpallyl Kolkata I
ACE : 932 : IM & OR
':.�-�
t, V ' • • •

_ : &p,rtqPnNira4vn
The inventory at the end of July is 1 00
units. If the demand for the month of 03. A company places orders for supply of two
August is 620, then the minimum total cost items A and B. The order cost for each of the
(in Rs.) to meet the demand is ____ items is Rs.300/order. The inventory carrying
(GATE - PI - 17) cost is 1 8% of the unit price per year per unit.
The unit prices of the items are Rs. 40 and
Rs.50 respectively. The annual demands are
Five Marks Questions 1 0,000 and 20,000 respectively. (a) Find the
economic order quantities and the minimum
0 1 . Consider the following data for a product: total cost? (b) A supplier is willing to give a
Demand = 1 000 units/year 1 % discount on price, if both the items are
Order cost = Rs.40/order ordered from him and if the order quantity
Holding cost = 1 0% of the unit cost / unit-yr for each item is 1 000 units or more. Is if
Unit cost = Rs.500 profitable to avail the discount?
(a) What is the economic order quantity? (GATE-ME-OO)(SM)
(b) Under the EOQ what is the number of
annual orders? 04. A company is offered the following price
( c) With a policy of ordering every month breaks for order quantity

Order quantity Price (Rs)


what would be the total annual cost as a
percentage of the cost at EOQ?
(GATE-ME-95)(5M)
0- 1 00 1 50
1 0 1 and above 1 00

02. Determine the number of production runs Ordering cost is Rs.60 per order while the
and also the total the incremental cost in a holding costs is 1 0% of the purchase price.
factory for the data given below: Determine the economic order quantity
Annual requirement = 1 5,000 units (EOQ) if the annual requirement is 1 000
Preparation cost per order = Rs. 25 Units. (GATE-ME-Ol)(SM)
Inventory holding cost = Rs. 5/unit/year
Production rate = 1 00 units/day
Number of working days = 250/year
(GATE-ME-97)(5M)

\( } I I J�JII: t lllll4 P11li!i(,IIJt)JI", Fyderabad j Dellii j Bhopal j Pune l Bhubaneswar l Lucknowl Patna l Bengaluru j Chennai j Vuayawada j Vmg j Tuupali I Kukalpally j Kolkata I
SOLUTIONS
08. Ans: 34000 to 36000
One Mark Solutions Sol: Given:
Annual demand (D)= 50000 units
01. Ans: (a) Ordering cost per order (Co) = 7000

Sol: EOQ = �
2AS EOQ = 1 0000 units
CI At EOQ
Annual inventory holding cost
= Annual ordering cost

= No. of orders/yearsxCo
50000
= X 7000
:. EOQ unchanged

02. Ans: (b) 03. Ans: (c)


1 0000
= 35000/-

04. Ans: (b) 09. Ans: (c)


Sol: Average inventory
= average flow time x throughput
2 x 4000 x 200 1
1 00 x 0. 1
= 30 x = 30 x .!. = 1 0 units
cycle time 3
= 400 units (or)
max .production + min .production
06. Ans: (b)
A verage =
2
20 + 0 .
07. Ans: 74.55
= -- = 1 0 un1ts

Sol:

Expected demand during lead time = LPi x i


i =l

= (50x0. 1 5)+(70x0. 14)+(75 x0.2 1 )+(80x0.2)


+ (85x0.3)
= 7.5+9.8+ 1 5 .75+1 6+25.5 = 74.55

\( J J ll...!,llll l l l l l ...., P1 1 l il 1 ( .tllll!I" �yderabadl Delhi l Bhopal j Punel Bhubancswarl Lucknow l Patna l Bengalwu j Oiennai l Vuayawadaj V,zag ITirupati I Kukalpally i Kolkata I
: 934: IM & OR

0 5. Ans: (c)
Two Marks Solutions AS
{ �
(EOQ)p - CI } P � CI
Ans: (d)
01 .
{P�8L
Sol:
(EOQ -
)Q
2AS
Sol: EOQ = �
CI
2 x l OO x 5 0
1 000
--- = --===
(EOQ) 1
JW = -'-;:::==== 4
2 x 400 x 5 0
=1:4

� 1

(EOQ) 1 = 1 000 ,fi, 0 6. Ans: (c)


Sol: A = 1 000 units, S = Rs. 1 00/order,
02. Ans: (c) Cc = 1 00/unit/year, Cs = 400/unit/year
Sol: EOQ oc ../A Cs
Qmax = EOQ x
s CC + CS
2 x 1 000 x 1 00 400
= x = 40 units
1 00 5 00
EOQ 2 = .J48000 x 2000
Ans: (c)
,J24000
0 7.
= ,fi, X 2000 = 28 00 Sol: ROL = ALTD + SFcr x er

Ans: (c)
Reorder level (ROL) > Average Lead Time
03 .
Demand (ALTD)
Sol: TVC oc ../A
:. Service Level (SL) = 75%
TVC 1 oc Jl .4 A
(TVC) 1 = (.Ju") TVC
= 1 . 1 83(TVC) or 1 8.3 %

04. Ans: (c) ALTD


2AS 0 8. Ans: (c)
Sol: EOQ = �
2 AS � 2 x 2500 x l 00
Sol: EOQ � �
CI
2 x 8,00,000 x 1 2 00 CI 9 x 0 .25
= = 4000 units.
1 20 = 47 1 .4units

\( l l 111!,111t t 1 1 1 11!. P11lil11 .11t1 > 1 1 ' Fyderabad! Dclliil BhopaI I Punel Bhubanes�I LucknowlPatml Beagalurul ChennailVliayawada l Vi7.ag � ff.: I Kukatpallyi Kolkata I
ACE
::m!�!��iNicdiooa
.. ":E=&�p�·.,,,
�,. �n�tr�o�l
�·�·=========�:;9�35�:==========�ln�v�en�ro�cy�Co
The EOQ does not satisfy the given quantity
range. Hence we need to compare the total
cost and EOQ & discount eligible quantity. Q=l OO
( TAC)EOQ = AC + �2 ASCI

= 2500 x 10 + .J2 X 2500 X 10 X 100 X 0.25


I+- 45 ...
1.. 45 -+I
= 26 ,118 /- Days Days

500 ( TAC)9 /- = AC + � S + i CI A Q
TVC= - S + - CL
Q 2
2500 500 100
= 2500 x 9 + x lOO + x 9 x 0.25 = 8 X 100 + - X 120 = Rs. 6800
500 2 2
= 22 ,500 + 500 + 562.5
= 23,562.5 /- 11. Ans: (b)
( TAC)9 < ( TAC)EOQ Order 500 units Sol:
500

09. Ans: (b) Imax= tp(p - r)


Sol: Cus= SP--CP= 90-70= 20
Cos= CP - Rebate= 70- 50= 20
Service Level (SL)
c 20
= ___u-'-s - = = 0.5 Q 1000
= - (p - r)= -- (1000 - 500) = 500 umts
C 08 + Cus 20 + 20 p 1000

Demand
units Probability Service Level 12. Ans: (c)
0.1 0.1 . 2555
2 Sol: Consumption rate= --= 7
3 0.35 0.45 365
Stock level - @ 0.35 0.8 � 0.5 ROP= AL T x CR= 8 x 7= 56 units
5 0.2 1.0
13. Ans: (c)
2 AS
10. Ans: (d) Sol: EOQ= �
CI
A 12 months _ � _
Sol: (No of orders = -=---- _ 8)
Q 45 days 1.5 2 X 10,000 X 300
= 389 29
40

!M•li@jjji4ijjjj4Rflfij@jjjM�jiyderabadj Delhi j Bhopa) I Pune 1 Bhubaneswar l Lucknow 1 Patna 1 Bengaiurul 0tennai 1 Vliayawadal Vizag ITuupati I Kukatpal)yl Kolkala I
: 936 : IM & OR

( TA C)EOQ = AC + �2ACSI Economic order Quantity = EOQ


2AS
= 10 ,000 X 200 + .J2 X 10000 X 40 X 300 EOQ = �
CI
= Rs 2015491.94
2 X 105 X 30 .
= ---- = 2000 umts
,1 .,'• ( T A C)QI = AC (1- l) + �
100 Q
S 1.5
I
Number of economic orders
+ 2L c1 (1-l) 100000
2 100 -- = 50 orders
2000
=

2%( TAC)1000 = 10000 X 200 (1-2-)


100
15. Ans: (c)
l O ,OOO l OOO Sol: Production rate (P) = 12000day - I
+ x 300 + x 40 (1-2-)
1000 2 100 Annual demand (A)= 8000 x300
= 19,6 0 ,000 + 3000 + 19600 Holding cost (CI) = 0.2 /month = 2.4 /year
= 19,82,600 /- Consumption rate (r) = 8000day- 1

( T A c)2000 = 10 , 000 x 200 [1- i J Set up cost (S) = Rs. 5 00


1 0 Economic Production Quantity = EPQ
4%

(_!__)
10 ,000 2000
+
2000
x 300 +
2
x 40 (1_
100
_±_) EPQ =
2AS
CI P-r
= 19,20 ,000 + 1500 + 38 ,400 = 19,59,900
2 x 8000 x 300 x 500 12000
10•000 = ( )
( T A c) 200 = 10' ooo x 200 + 2.4 12000 -8000
200
200 = 54772.25
x 300 + - x 40 EPQ
2 Frequency of production =
= 20 ,00 , 000 + 15000 + 4000
= 21,90 , 000 /- 54772.25
= 6_84day s
8000
Accept 4% Discount at 2000 order

14. Ans: 50 orders 16. Ans: 76.948


Sol: A = 200 0 kg =Annual Demanq
Sol: Annual Demand (A) = 1,00, 000
Unit price (c) = 10 C = Rs.25 Cost per kg
Cost of Inventory (CI) = 1.5 S = Rs.I 00 Cost per an order
Order cost ( S) = 30 1 = 9%

!IHIM@hiiiihiiRdbiMli&lllj yderabad I Dellii I Bhopal I Pune I Bhubanesw.ir l Lucknow I Patna! Bengaluru I Chennai ! Vuayawada I Vizag I Tinipati I' Kukatpally I Kolkata
�..... Inventory Control
.."�&lgiom· PnNiariooa
...
� -....... ACE : 937 :

2AS 2 x 2000 x I 00 ( TAC)EOQ = AC + �2ACS I


EOQ = � =
CI 25 x 0.09
= 25000 X 6 5 + .J2 X 25000 X 6 5 X 500 X 0.2
421.63 units.
= 1625000 + 18027.75
=

Q
Time between orders = x Time Period 16,43,027.75
A
=

If 1500 kgs are ordered we get it at 60 /-


421.63
= X 365 A Q
2000 ( TAC)Q = AC + S+ CI
= 76.948 days
Q 2

An order is placed every 76.948 days.


25000 1 500
(TAC )1500 = 25000 X 60 + X 500 + X 60 X 0.2

= 15,00,000 + 8333.33 + 9000


1 500 2

17. Ans: 2 (range 1.9 to 2.1) = 15 , 17 , 333.33


Sol: Annual demand (D) = 10000 units ( TAC)1soo < ( TAC)EoQ
Unit cost (Cu) = Rs . 100 Hence order 1500 units
Carrying cost (Cc) = 14.4% of unit cost
Ordering cost (Co) = Rs. 2000 19. Ans: (c)
Cycle time ( T) = ? Sol: Annual Demand = A = 2400
_!_ EOQ No. Of weeks = t = 52
T= =
. A 2400
N D
Consumpt10n rate = r = - = --
2QC t 52
EOQ = � ,
cc = 46.1538 units /week
2 X 10000 X 2000 = . Cost Per unit = C = Rs 600
----- 1666.66 umts
-l OO x 0.144 Inventory Carrying cost, I = 0.1
T= 0.1666 x 12 = 2 months Cost of Ordering = S = Rs. 201
Lead Time = 5 Weeks
18. Ans: 1500 (range 1499 to 1501) 2AS
EOQ = �
Sol: A = 25,000 , S = 500 CI
I = 0.2 , C1 = 60 2 x 2400 x 201
= ----- =126 . 8 � 127 Units
AS 2 X 25000 X 500 600 X 0.1
EOQ = J2 =
C1 I 60 x 0.2 Reorder Level
1443 < 1 500 units
= = Consumption Rate x Lead Time
Hence not the best order = 46.1538 X 5 = 230.77 � 231
EOQ is falling in the 750 - 1499 prime range
!ltlli@jj/l4@4Rflflftiu.jj� yderabad I Delhi I Bhopal I Pune I Bhubaneswar I Lucknow I Patna I Bengalwu I Chennai I Vijayawada I Vizag I Tirupati I Kukatpally I Kolkata
i.t
� :&�!
• �
.:�
:m�CE
·�PoNicatioos
���======�:�9�3:
8�: ===========� �&�OR
��
.. '" =
IM

(TAC)8333 502563.25
x l OO x l OO
20. Ans: 2900
Sol: Demand in August = 620 units (TAC)EOQ 5 02000
=

Inventory at end of July = 100 units = 100.11 %


Demand to be met = 620 - 100 = 520 units
02.
RTP = 3 00 , OTP = 200 , SCP = 20
units
Total Cost = 3 00 x 5 + 200 x6 + 210 x 10 Sol: A = 15000 units , p = 100
day
1500 + 1200 + 200 = 2900 /-
15000 .
=

r = -- = 60 umts I day
250
Five Marks Solutions C I = 5 / unit / year, S = 25

01.

Sol: A = 1000 units , S = 40 /-;


I = 0.1, C = 500
2AS 2 x l 000 x 40
EOQ = � 40 units
CI 500 x 0.l
= =

A 1000
No. of orders = - -- = 25
Q 40
=

2AS P
(TAC)EOQ = .J2ASCI + AC Q=
CI P - r
-- x --

.J2x1000 x 40 x 500 x 0.l + 1000 x 500


2x l 5000 x 25 100
=

= x( ) = 61237
= 5, 02, 000 /- 5 100 - 60
. A 1000 l 5, 000 =
Order Quantity per month = - = -- = 83.33 No. of Pr oduction runs 24.5� 25
12 12 612.3
=

A Q
(TAC)s3 .33 = AC + S + CI P-r
Q 2 Incremental Cost = 2ASCI( p )
1000
= 1000 500 + -- x 40
83.33 l OO - 60
2 X 15000 X 25 X 5 X )
X
(

8333
=

+x 500 x 0. l
1 00
2 = Rs 1224.75
= 5 ,00 , 000 + 480 + 2083.25
= 5 , 02,563.25

jP.J11ijj§jjj§4,jji4pnflttfj,iJjjj.� yderabad I Delhi I Bhopal I Pune I Bhubancswarl Lucknow I Patna I Bengaluru I Chcnnai I Vijayawada I Vizag I T�j' Kukatpally I Kolkatl
: 939 : Inventory Control

0 3. 1 l O, OOO
= 1 0 '000 X 40 ( 1 - -- ) + X 3 00
Sol: First Option: 1 00 1 000
A 1 000
+ -- x 40 x 0 . 1 8( 1 - -)
1
S = 3 00 /- I = 0. 1 8 2 1 00
A 1 = 1 0, 000 = 3,96, 000 + 3, 000 + 3564 = 4, 02,564 /-
2 X 1 0, 000 X 3 00 = (TAC) 1 1 54.7 = A2 C2 ( l - � ) + �: S
913 1%
1 0
40 x 0 . 1 8
Q r
TAC 1 = A 1 C 1 + �2A 1 S C 1 I + -2 C2 I ( 1 - - )
2 1 00
= 1 0,000 X 40 + �2 X 10,000 X 40 X 300 X 0. 1 8 1 20, 000
= 20 '000 X 5 0 ( 1 - -- ) + X 3 00
1 00 1 1 54.7
=4, 00 , 000 + 6572.67
1 1 54 · 7 1
TAC 1 = 4,06,572.67 + x 50 x 0 . 1 8(1 - --)
2 1 00
B = 9,90, 000 + 5 1 96. 1 5 + 5 144. 1 8
S = 3 00 /- , I = 0 . 1 8 = 1 0, 00 ,340 .33
C2 = 5 0 /- , A2 = 20, 000
(TAC) = 4, 02,564 + 1 0, 00 ,340 .33
� 2A2 S � 2 x 20,000 x 300 � I I S4_7 = 14, 02,9 04.33
C2 I
Q, �
50 x 0. 1 8 Second option is better
TAC2 = A2 C2 + �2A 2 SC 2 I
04.
= 20,000 X 5 0 + �2
Sol: A = 1 000 units,
X 20,000 X 50 X 300 X 0. 1 8

1 0,00 ,000 + 1 0 ,392.3


=
I = 0. 1 , S = 60 C = 1 00
TAC2 = 1 0, 1 0,392.3
(TAC) = TACA + TACe 2AS
EOQ = �
2 x 1 000 x 60
=
CI l OO x O . l
= 4,06,572.67 + 1 0, 1 0,392.3
= 14, 1 69,65 /- = 1 09.54 > 1 0 1
Hence best order is EOQ = 1 09.54 units.
Second Option: As EOQ is falling in the range mentioned for
_ r A the least price it is the best order.
1 o/c0 (TAC) 1 000 - A 1 C1 ( 1 - - ) + -1 S
1 00 Q1
Q r
+ -I C 1 I ( l - - )
2 1 00

\( I l 111.!,111t , 1 1 11� P11lilu .1111 1 1 1 " �yderabad I Delhi I Bhopal I Pune I Bhubancswar I Luclmow I Patna I Bengaluru I Chennai I Vuayaw.u:la ! Vizag I Trrupari I Kukatpally I Kolkara l
cs Transportation
02. A company has two factories S 1 , S2 and two
One Marks Questions warehouses D1 , D2 • The supplies from and
S2 are 50 and 40 units respectively.
S1

Warehouse DI requires a minimum of 20


0 1 . If there are m sources and n destinations in a units and a maximum of 40 units. Warehouse
transportation matrix, the total number of D2 requires a minimum of 20 units and, over
basic variables in a basic feasible solution is and above , it can take as much as can be
(GATE -ME-14) supplied. A balanced transportation problem
(a) m + n (b) m + n + 1 is to be formulated for the above situation.
(c) m + n-1 (d) m The number of supply points, the number of
demand points, and the total supply (or total
demand) in the balanced transportation
Two Marks Questions problem respectively are (GATE-ME-05)
(a) 2, 4, 90 (b) 2, 4, 1 1 0
(c) 3, 4, 90 (d) 3, 4, 1 1 0
0 1 . The supply at three sources is 50, 40 and 60
units respectively while the demand at the 03. For the standard transportation linear
four destinations is 20,30 ,1 0 and 50 units. In programme with m sources and n
solving this transportation problem destinations and total supply equaling total
(GATE-ME-02) demand, an optimal solution (lowest cost)
(a) a dummy source of capacity 40 units is with the smallest number of non-zero Xij

needed values (amounts from source i to destination


(b) a dummy destination of capacity 40 units j) is desired. The best upper bound for this
is needed number is (GATE-ME-08)
(c) no solution exists as the problem is (a) mn (b) 2(m + n)
infeasible (c) m + n (d) m + n- 1
(d) none solution exists as the problem is
degenerate

!11•1ijj§jjjqg.jjj$RflGIM\iiih� yderabad I Delhi I Bhopal I Pune I Bhubaneswar I Lucknow I Patna I Bengaluru I Chennai I Vtiayawada IV11.3g I Tirupali I Kukatpally I Kolk.ala
: 94 1 : Transportation

04. Transportation costs (in INR/unit) from destination requires 8 units. If the north-west
factories to respective markets are given in comer method provides the total
the table below. The market requirements transportation cost as X (in Rupees) and the
and factory capacities are also given. Using optimized (the minimum) total
the North-West Comer method, the quantity transportation cost is Y (in rupees), then (X­
(in units) to be transported from factory R to Y), in rupees, is (GATE - 17 - SET - 2)
market II is (a) 0 (b) 1 5 (c) 35 (d) 1 05

Requirements
Factory
(in units) Five Marks Questions
p Q R s
I 3 3 2 1 50 0 1 . A company rents three warehouses A, B and
Market II 4 2 5 9 20 C from which they supply bearings to two
III 1 2 1 4 30 customers P and Q. The profit per piece, the
Factory
annual demands of the customers and the
supplies available from the warehouses are
capacity
20 40 30 10
shown below
(in

p Supply
units)

(GATE - PI-16)
Q
A 3 4 4000
(a) 30 (b) 20 (c) 1 0 (d) 0
B 2 3 4000
C 5 4 3000
05. A product made in two factories, P and Q is
Demand 6000 6000
transported of two destinations, R and S. The
As the company is not able to meet the
per unit costs of transportation (in Rupees)
demands it is proposed to replace warehouse
from factories to destinations are as per the
C with a warehouse rented at D whose
following matrix.
supply capacity would be 5000. The expected
� R
profit would be Rs. 6 and Rs. 5 per piece
s
distributed to P an Q respectively. The rental
P 10 7
charges for warehouse D is Rs. 5000 per
Q 3 4
year. Find the increase in the profit of the
company after replacing warehouse C by D.
Factory P produces 7 units and factory Q
(GATE-ME-97)(5M)
produces 9 units of the product. Each
!IHIM@jjjqg.jjgpyjlfiW!&M�Iyderabad I Delhi I Bho� I Pune I Bhubaneswar l Lucknow I Patna I Bengaluru I Chcnnai I Vijayawada I Vmg I Tuupari I Kukatpally I Kolkata I
: 942 : IM & OR

02. Given below is a basic feasible solution to a (a) Compute the cost corresponding to the
transportation problem with three supply present solution
points (A. B, C) and three demand points (P, (b) It is optimal?
Q , R) that minimizes cost of transportation in ( c) Does an alternate optimum exist ?
the standard tabular format. (GATE-ME-00)(5M)

A B C Demand
4 8 8
p 1 50
50 1 00
12 8 11
Q
1 00 1 00
10 6 9
R 250
50 200
Supply 50 1 50 300

�ydcrabad l Delhi l Bhopal l Punc l Bhubaneswar l Lucknowl Patna l Bcngaim;u l Chcnnai l Vuayawada l Vizag ITtrupali l Kukalpally l KolJwa I
SOLUTIONS

One Mark Solutions 05. Ans: (*)


Sol:
01. Ans: (c) R s Supply

Sol: (m + n -1) is the number of allocations. p [!Q xl2_ 7


7

I Two Marks Solutions Q l ll_ 8 Li_ 9

01. Ans: (b)


Demand 8 8

Sol: Supply = 50 + 40 + 60 = 150 North West comer rule solution


Demand = 20 + 30 + 10+ 50 = 110 TC1 = x = 7x l 0+1x3+8x4 = 105
Supply >Demand by 40 units PS evaluation by stepping stone method is
A dummy destination of capacity 40 units is PS = +7-4+3-10 = -4
needed If some units are transferred to PS cost of
transformation decreases by Rs.4 /- for every
02. Ans: (a) 03. Ans: (d)
unit transferred to PS
04. Ans: (c) Coh�truct a loop and transfer units.
Sol: Supply = 20+40+30+10 = 100 units
Demand 50+20+30 100 units
7
= = .,:;;;::;;.�.... +
4
l - 8
In negative comers least quantity is 7 add it
FACTORY

p Q R s REQ to positive comers and deduct it from


Ii_ ll. l.l negative comers
20 30 X X 50
10 7
7
II X
Li_ ll_ Ll..
10 10 X 20 3
8 l 4

Optimal transport cost = y = 7x7+1x4 + 8x3


III
L!. ll_ lJ... li 30
X X
= 49 + 4 + 24 = 77
20 10

=> X - y = 105 - 77 = 28
20 40 30 10

Factory R to market II is 10 units.


IIJl•i@jjji4ijjjiRflft1Mijjfjj� yderabad I Delhi I Bhopal I Pune I Bhubaneswar I Lucknow I Patna I Benga)uru I Chennai I Vijayawada jVmg I Tirupati I Kukatpal)y I Kolkata
��-�EF.upimmg PnNiratiooa
ACE : 944 : IM & OR
� � ======================================
Optimal Solution = Rs 41,000
Five Marks Solutions S = 4000 + 4000 + 5000 = 13,000
D = 6000 + 6000= 12,000
01. S > D by 1000 units.
Sol: S= 11,000; D= 12,000 ;
D > S by 1000 units PMAX = 4 x 4000 + 1000 x 2 + 3 x 2000 + 0
ui� 1000 + 6 X 5000 = 54,000
Extra rent = 5,000
-3 -4 -1 X

Q Supply
v� p R

Pnet 49,000
0 A
1 CD X X =

Increase in Profit = 49,000-41,000


+1 B 1 (D(l)

= 8,000 /-
-3 1 x x X
0 0 0 X

02.
Sol:
G) 0 0
1 0
-2 0) 0
8 Demand
X 0 8

Pmax = 4 X 4000 + 2 X 2000 + 3


1 50

2000 + 5 3000 + 0 X 1000


11
X X

= 41,000
1 00
9

Solve by VAM after converting


250

MAXIMISA TION to MINIMISA TION


Test for optimality by MODI METHOD.
500

Solution is optimal. One alternate optimum


exists as there is 'O' evaluation for row 2 and
ui� -3 -4
Supply
column 3
v.J.. p Q

Number of allocations = 5
0 4000 1 CD X X

R + C- 1 = 3 + 3- 1 = 5
-3
+1 B 4000 1 1 1 -2
C x x X No degeneracy.
Since, No. of allocations equal to R+C-1
-2 3000 1
+3
D 0 0 0 X

Demand 6000 6000 Total cost = 50 x 4 + 8x 100 + 100x8 + 50x6


+ 9 X 200 = Rs. 3900
(1) 0
1
2
X

l•••ii@hii4iii1i@Miihih� yderabad l Dellii ! Bhopal l Pune l Bhubaneswarl wcknow l Patna l BcngaJuru l Chennai l Vuayawacla !Vizag I T,rupati I Kukalpally l Kolkala
Chapter
6 Production Planning
and control
04. A component can be produced by any of the
four processes I, II, III and IV. The fixed cost
and the variable cost for each of the processes
One Mark Questions

01. The manufacturing area of a plant is divided are listed below. The most economical
into four quadrants. Four machines have to process for producing a batch of 100 pieces is
located one in each quadrant. The total
Process Fixed cost Variable cost per
number of possible layouts is
(in Rs) piece (in Rs)
I 20 3
(GATE-ME-95)
(a) 4 (b) 8
II 50 1
(c) 16 (d) 24
III 40 2
IV 10 4
02. Production flow analysis (PFA) is a method
of identifying part families that uses data (GATE-ME-14)
from (a) I (b) II (c) III (d) IV
(GATE-ME-01)
(a) engineering drawings Two Marks Questions
(b) production schedule
(c) bill of materials 01. List- I (Problem areas)
(d) route sheets A. IlT
B. Computer assisted layout
03. Which one of the following 1s NOT a C. Scheduling
decision taken during the aggregate D. Simulation
production planning stage?
List- II (Techniques)
1. CRAFT
(GATE-ME-12)
(a) Scheduling of machines
2. PERT
(b) Amount of labour to be committed
3. Johnson's rule
(c) Rate at which production should happen
4. Kanbans
(d) Inventory to be carried forward
5. EOQ rule
6. Monte Carl (GATE-ME-95)

!M•li!@jj/i4.jjjiJmGli!iii!� yderabad I Delhi I Bhopal I Pune I Bhubaneswar I Lucknow I Patna I Bengalwu I Chennai I Vijayawada I V17.ag I Tirupati I Kukatpally I Kolkata
.
.,':....�.. ..
"' � . .
... :�Publirarions
A,CE : 946: IM&OR
.. '=====================================
02. Capacities of production of an item over 3
consecutive months in regular time are 100,
100 and 80 and in overtime are 20, 20 and
Five Marks Questions

40. The demands over those 3 months are 90, 01. The forecasts for a product for the next three
130 and 110. The cost of production in months are given as 750, 850 and 1000 units.
regular time and overtime are respectively The number of regular time days and
Rs. 20 per item and Rs. 24 per item. overtime days available are 22, 18 22 and 4,
Inventory carrying cost is Rs. 2 per item per 4, 5 respectively. With the existing number
month. The levels of starting and final of employees, the company can produce 38
inventory are nil. Backorder is not permitted. units per day. To meet the high demand in
For minimum cost of plan, the level of the third month, the company decides to hire
planned production in overtime in the third people to increase the daily production to 45
month is (GATE-ME-07) units. (GATE-ME-OO)(SM)
(a) 40 (b) 30 (c) 20 (d) 0
The following costs are given
03. A manufacturer has the following data Cost of regular time production
regarding a product: = Rs. 20 per unit
Fixed cost per month = Rs 50000 Cost of overtime production
Variable cost per unit = Rs 200 = Rs. 25 Per unit
Selling price per unit = Rs 300 Cost of hiring = 200L2
Production capacity = 1500 units per month where 'L' is the mcrease m daily
If the production is carried out at 80 % of the capacity
rated capacity, then the monthly profit(in Rs)
Inventory = Rs. 10 per unit per month
(based on average inventory)
(GATE-15-S et 2)
lS

Shortage (backordering cost) = Rs. 20 per


unit per month

The beginning inventory is 100 units. The


company decides to produce 800, 700 and
900 units respectively in the three months.
Compute the cost of the production plan.

!l1•1i@jjji4.jjf4jmni&mjjj� yderabad I Delhi I Bhopal I Pune I Bhubaneswar I Lucknow! Patna I Bengalwu I Chennai I Vijayawada jV,zag }Tiru IKukatpal)y I Kolkala
SOLUTIONS
02. Ans: (b)
One Mark Solutions Sol:

01. Ans: (d) Q) 0


RT J-0 100
Sol: 4! = 4 x 3 x 2 x 1 = 24 Q) OT 20 20
RT 100
0 20
02. Ans: (b), (c) OT
RT 80
CD
03. Ans: (a)
OT 10 40
Demand 90 130 110 30 360

04. Ans: (b) 03. Ans: 68000 to 72000


Sol: Total Cost = Fixed cost+(Quantity x Sol: F = 50000=fixed cost
Variable cost)
S = 300 = Selling Price per unit
TC = F+q V
TC1 = 20+ lOOx 3 = 320 V = 200= Variable cost per unit
TC2 = 50+ lOOx 1 = 150 Capacity = 1500 units
TC3 = 40+ lOOx 2 = 240
80% capacity = 0.8x1500 = 1200 units
TC4 = 10+ lOOx 4= 410
Minimum total cost is the best plan. Marginal costing equation is

q(s-v)=F+P

Two Marks Solutions P =q(s-v)-F

= 1200(300-200)-50000
01.
D-6
=120000-50000 = 70000
Sol: A -4 ; B - 1 C -3

Fyderabad I Delhi I Bhopal I Punc I Bhubancswarl LucknowlPatnalBcngaluru I Oicnnai IVliayawadajVizag !Ttrupati I Kukatpally! Kolkata I
ACE
:., .tw1"�1ux--: : 948: IM & OR
.,.>-� "=
����=ILl6�:-�:��:�========���================���
n 91PGIS

Five Marks Solutions


01.
Sol:

P eriod Beginv Production Demand R.P OTP Endinv Shortage


1 100 800 750 800 ---- 150 ----
2 150 700 850 684 16 ---- ----
3 ---- 900 1000 900 ---- ---- 100
2384 16 150 100

End ing inventory = (B eginning Inventory +Production -Demand)


OTP = Over Time Production.
Regular Production Cost= 2384 x 20 = 47,680
Over Time Production Cost = 16 x 25 = 400
Shortage Cost= 100 x 20 = 2,000
Hiring Cost = 200 (45-38)2= 9,800

!1111i@jjjiliii/4RflbiMIU,jj� yderabadlDelhilBhopal!PunejBhubaneswarl LucknowlPatnalBengaJuru!ChennaijVtjayawadajVu.ag l'firupati fi'Kukatpallyl Kolkata


C7 Forecasting
(d) include new demand data in the average
after discarding some of the earlier
demand data
One Mark Questions

01. The most commonly used criteria for


measuring forecast error is 04. A regression model is used to express a
(GATE-ME-97) variable Y as a function of another variable
(a) Mean absolute deviation X. this implies that (GATE-ME-02)
(b) Mean absolute percentage error (a) There is a causal relationship between Y
(c) mean standard error andX
(d) Mean square error (b) A value ofX may be used to estimate a
value ofY
0 2. Which one of the following forecasting (c) Values ofX exactly determine values of
techniques is not suited for making forecasts Y.
for planning production schedules in the (d) There is no causal relationship between
short range ? (GATE-ME-98) YandX
(a) Moving average
(b) Exponential moving average 05. For a product, the forecast and the actual
(c) Regression analysis sales for December 200 2 were 25 and 20
(d) Delphi respectively. If the exponential smoothing
constant (a) is taken as 0. 2, then forecast
03. When using a simple moving average to sales for January 2003 would be
forecast demand, one would (GATE-ME-0 4)
(GATE-ME-01) (a) 21 (b) 23 (c) 24 (d) 27
(a) give equal weight to all demand data
(b) assign more weight to the recent 06. Which of the following forecasting methods
demand data takes a fraction of forecast error into account
(c) include new demand data in the average for the next period forecast?
without discarding the earlier data (GATE-ME-0 9)
(a) simple average method

\( l l 11�1t1ll III!:.!. Pul,!1(,l!Hllh yderabad I Delhi I Bhopal I Pune I Bhubaneswar I Lucknow I Patna I Bengaluru I Chennai I Vgayawada I V1Zag I Tuupari I Kukatpally I Kolkata
: 950: IM&OR

(b) moving average method 10. Sales data of a product 1s given in the
(c) weighted moving average method following table:
(d) exponential smoothing method
Month January February March April May

07. In simple exponential smoothing forecasting,


Number

to give higher weightage to recent demand


of units 10 11 16 19 25

information, the smoothing constant must be


sold

Regarding forecast for the month of June,


close to
which one of the following statements is
(GATE-ME-13)
(a)-1 (b) zero (c) 0.5 (d) 1.0
TRUE? (GATE-15-Set 2)

08 . In exponential smoothening method, which (a) Moving average will forecast a higher
one of the following is true ? value compared to regression
(GATE-ME-14) (b) Higher the value of order N, the greater
(a) 0�a� 1 and high value of a is used for will be the forecast value by moving
stable demand average
(b) 0�a�1 and high value of a is used for (c) Exponential smoothing will forecast a
unstable demand higher value compared to regression
(c) a � 1 and high value of a is used for (d) Regression will forecast a higher value
stable demand compared to moving average

(d) a � 0 and high value of a is used for


unstable demand Two Marks Questions

09. The actual sales of a product in different


months of a particular year are given below: 01. Which of the following is a technique for
forecasting? (GATE-ME-89)
(a) Exponential smoothing
(b) PERT/CPM
Sept Oct Nov Dec Jan Feb
180 280 250 190 240 ?

The forecast of the sales, using the 4-month (d) Control charts
(c) Gantt chart technique

moving average method, for the month of


February is_ _ (GATE -ME-14)

!lfll@h4jj/44ijjjQJmbiM\Ulh+1ydcrabadl DelhiI Bhopal I Punc I Bhubancswarl Lucknow I Patna! Bcnga)uru I Chennai jVuayawadajVmg I-� .1'uka!pallyl Koluta I
: 951: Forecasting

02. In a forecasting mod el, at the end of period 05. The sales of a product during the last four
13, the forecasted value for period 14 is 75. years were 860, 880, 870 and 890 units. The
Actual value in the periods 14 to16 are forecast for the fourth year was 876 units. If
constant at 100. If the assumed simple the forecast for the fifth y ear, using simple
exponential smoothing parameter is 0.5, then exponential smoothing, is equal to the
the MSE at the end of period 16 is forecast using a three period moving average,
(GATE-ME-97) the value of the exponential smoothing
(a) 820.31 (b) 273.44 constant a is
(c) 43.75 (d ) 14.58 (GATE-ME-OS)
1 2 2
(a) - (b) _!_ (c) - (d) -
03. In a time senes forecasting model, the 7 5 7 5
d emand for five time periods was
10,13,15,18 and 22. A Linear regression fit 06. A moving average system 1s used for
resulted in an equation forecasting weekly demand. F1(t) and F2(t)
F= 6.9 + 2.9 t where F is the forecast for are sequences of forecasts with parameters
period .The sum of absolute d eviations for m1 and m2, respectively , where m1 and
the five data is m2(m1>m2) d enote the numbers of weeks
(GATE-ME-00) over which the moving averages are taken.
(a) 2.2 (b) 0.2 The actual d emand shows a step increase
(c) -1.2 (d) 24.3 from d 1 to d 2 at a certain time. Subsequently ,
(GATE-ME-08)
04. The sale of cy cle in a shop m four (a) neither F1(t) nor F2(t) will catch up with
consecutive months are given as 70,68,82,95. the value d2
Exponentially smoothing average method (b) both sequences F1 (t) and F2(t) will reach
with a smoothing factor of 0.4 is used in d 2 in the same period
forecasting. The expected number of sales in (c) F1(t) will attain the value d2
the next month is (d) F2(t) will attain the value d 2
(GATE-ME-03)
(a) 59 (b) 72 (c) 86 (d) 136 07. The d emand and forecast for February are
12000 and 10275, respectively. Using single
exponential smoothening method

!111•iii4iUiii04Rbni;;;;;ij yderabad I Delhi I Bhopal I Pune I Bhubaneswar 1 ,Lucknowj Patna I Bcngaluru I Chennai I Vijayawada IVizag I T1t11pati I' Kukatpa)ly I K.olkatt
" "' · � A.CE · ·
1Nie:aloi& 952 IM&OR
�� '"=
�-· tE��F�-:erq�
: �����========::::,:��:=============�����

(smoothening coefficient = 0.2 5), forecast for 10. In a calendar year, the demand forecast of
the month of March is (GATE-ME- 10) motorbikes for the month of June is 200.
(a)431 (b) 958 7 The actual demand of motorbikes for the
(c) 10 706 (d) 11000 month of June and J uly are 300 and 350,
respectively. If single exponential
08. For a canteen, the actual demand for smoothing method with smoothing constant
disposal cups was 500units in January and 0. 7 is used, then the demand forecast for the
600 units in February. The forecast for the month of August is _____
month of january was 400 units. The (GATE-PI-1 7)
forecast for the month of March considering
smoothing coefficient as 0. 75 is ___
(GATE-15-S et 1)

09. The demand for a two-wheeler was 900


units and 1030 units in April 201 5 and May
201 5, respectively. The forecast for the
month of April 2015 was 8 50 units.
Considering a smoothing constant of0.6, the
forecast for the month of June 201 5 is
(GATE-16-SET-3)
(a) 8 50 units (b) 92 7 units
(c) 96 5 units (d) 9 70 units

!M11ijj§jjjQY.jjj$Aflftifflijjih� yderabadlDelhilBhopal!Pune!Bhubaneswari Lucknow!Patna!Benga)wu!Chennai!VijayawadalViz.ag !Tirupari I Kukatpallyl Kolkata


SOLUTIONS
08. Ans: (b)
One Mark Solutions Sol: Maximum value of a is 1 (one) and is used
for unstable or inflationary demand
01. Ans: (a) 02. Ans: (d) situations .

03. Ans: (a) 04. Ans: (b) 09 . Ans: 240 units


_ Foct +Fnov +Fdec +Fjan
05. Ans: (c) Sol.. F - ---------'--

Ft = Ft-I +a(Dt-1-Ft-1)
4
Feb

280+250+190+240
= 240units
Sol:
= 25 +0.2(20-25) = 24
=
4

06. Ans: (d)


10. Ans: (d)
07. Ans: (d) Sol:

Sol: a = -- where n is number of moving Month Demand


y2
2
n+l y
XY xz
average periods.
X
1 10 1
(Moving average periods are varying from
10 100
2 22 121 4
n=2 to oo and when n=2, a=0.67 and n=1,
11
3 16 48 256 9
a=1 which is an extreme case)
4 19 76 361 16
Next months forecast Previous months
5 25 125 625 25
=

demand
:DC = I:Y= :D(Y= l:Y2= :DC2 =
If ft = dt-1
15 81 281 1463 55
Then a = 1

As per wikipedia : Y= a+bX

St= ax 1 +(1-a)s t-1, t > 0 - I:x 15


X= -- = -= 3
n
Where a is the smoothing factor,
5

and O<a <l . Y= I: y = = 16.2 !!


n 5

\( I 1111..!,llllt I IIIL!, P11hl1<,tll1ll1� FyderabadlDclhilBhopaJIPunelBhubancswarl 1.ncknowlPatna.JBengalurulChcnnaijVuayawadalVmg ITIIUpati I Kukalpallyi Kolkala I


: 954: IM&OR

b=
IxY-nXY L(di- ti = 8 20.31
Ix 2 -nx
-2 '
MSE= :E(di -ti In 8 20.31 / 3= 2 73.44
=

281- 5x3x16. 2
b= 2 = 3.8
55- 5x (3) 03. Ans: (a)
Sol:
a= Y-bX = 16. 2-3.8 x 3= 4.8
Y= a+ b X= 4.8+ 3.8 X Period D1
Fi
= 6.9 + 2.9 t ID1-F1I

When X= 6 ( June) l 10 9.8 0.2


2 13 12.7 0.3
Y = 4.8+3.8 x6 = 2 7.6 3 15 15.6 0.6
4 18 18.5 0.5
By moving average method forecast for 5 22 21.4 0.6
10+ 11+16+ 19+ 2 5 rld;-Fil = 2.2
June month = -------

=!!.=16. 2
5 04. Ans: (b)
2
Regression wil l forecast a higher value Sol: St+I=aS 1+a(1-a); 1_1+a(1-a) S 1 _2
compared to moving average method. + a(l - Cl )
3
s,_3

= (0.4 X 9 5)+(0.4 X 0.6 X 8 2)


+ 0.4 x(0.6)2 X 68+ 0.4(0.6)3 70
Two Marks Solutions
= 73. 52
01. Ans: (a)
0 5. Ans: (c)
0 2. Ans: (b) Sol: Forecast for the fifth year
Sol: ft= ft-1+ a ( dt-1 -ft_1)
= 880+ 8 70+ 890= 880units
f14 = 75; d14 = 100; Cl = 0. 5 3
f1s= f14+ a (d14- f14) Ft = Ft-1+ a(Dt-1 - F t-1)
= 75+ 0. 5 (100- 75)= 8 7. 5 880= 8 76+ a.(890-8 76)
f16 = f1s+ a (dis- f1s) 4= 14 a �a = 2/ 7
= 87.5 + 0.5 (100 - 87.5)
= 8 7. 5+ 6. 2 5= 93. 75 06. Ans: (d)
Period fi d1 (d1-f1i Sol: F2(t) is sensitive(inflationary) data
14 75 100 625
15 8 7. 5 100 1 56. 2 5
16 93.7 5 100 39.06
\( l I I ll.!,llH l I l[J� 1 11 ddtt .111, 111, �yderabadjDeihilBhopaIIPunelBhubancswarl LucknowlPatna!BcngalurulCbennailVuayawadajVmg ITtrupati I Kukalpal)yj.Kolkat, I
t" ACE
",.•.,�Pn}ilntlma

. . . . : 955: Forecasting

07 . Ans: (c) 09. Ans: (d)


Sol: FMarch = FFeb + a (D Feb - FFeb ) Sol: FMAv = FAPRIL + 2 (DAPRIL- FAPRIL)
= 850 + 0.6 (900-850)
= 10275 + 0.25(12000-10275)
= 10706.25 = 850 + 30
= 880 Units
08. Ans: 568 to 570 FJUNE = FMAv + 2(DMAv-FMAY)
Sol: = 880 + 0.60 (1030-880)
Month Actual demand Forecast
= 880 + 90
Jan 500 400
= 970 Units

Feb 600
Mar ? 10. Ans: 326
Sol: FJUNE = 200 units
FJan= 400 units DJUNE = 300 units
D1an = 500 units DJULv= 350 units
a = 0.75 a = 0.7
FJULy = FJUNE + a (DJUNE- FJUNE)
= 400 +0 .75(500-400) = 475
= 200 + 0.7 (300-200) = 270 units
FAUGU ST= FJULy+ a(DruLy- FJULy)
D Feb = 600
= 270 + 0.7 (350-270)
FMar = FFeb +a(D Feb -FFeb) = 270 + 56= 326 units
= 475 +0.75(600-475) = 568.75

\( l } !l�\lllllllll.!, J >11l>l1<,1t11111', Fyderabad I Delhi I Bhopal I Punc I Bhubaneswar I Lucknow I Pafual Bcngaluru I Chennai !Vuayawada Iv� I Tirupari I Kukatpally I Kolkala I
Chapter
8 Line Balancing
For line balancing the number of work
One Mark Questions stations required for the activities M , E and T
would respectively be
01. A line balancing problem is solved m the ( a) 2, 3, 1 ( b) 3, 2, 1
context of (GATE - PI-15) ( c) 2, 4, 2 ( d ) 2, 1 , 3
( a) process lay out
( b) Fixed position lay out 03. The table gives details of an assembly line.
( c) Product layout (GATE-ME-06)
( d ) production sched ule
Work station I II III IV V VI
Total task time at
Two Marks Questions the workstation 7 9 7 10 9 6
(in min)
01. In an assembly line for assembling toys, five
workers are assigned tasks which take times W hat is the line effi ciency of the assembly
of 10, 8, 6, 9 and 10 minutes respectively . The line?
balance d elay for line is (GATE-ME-96) ( a) 70% ( b) 75%
( a) 43.3% ( b) 14. 8% ( c) 80% (d) 85%
( c) 14. 0% ( d) 16. 3%
04. A manufacturing line requires 7.2 minutes to
02. A n electronic equipment manufacturer has make a product. The line has six
d ecided to add a component sub-assembly workstations arranged as per the required
operation that can produce 80 units during a sequence of operations. Total production
regular 8- hour shift. This operation consists required is 300 prod ucts in 7.5 hours. A t
of three activities as below. steady state, the line effi ciency, in % is

Activity StandardTim e (min) (GATE - PI -15)


(GATE-ME-04)

M . M echanical assembly 12
E. Electric wiring 16
T. Test 3

\( I I J t ..., l ! H l 1 1 11g Pt d d u .1111 ,,1..., �ydcrabad J Delhi JBhopal JPunclBhubancswarl LucknowJ Parna J BcngaluruJChennai JVtiayawadaJV� j Tlfllpati I KukatpallyJ Kolkala I
SOLUTIONS
16
= 2 .67 = 3 E
6
=
One Mark Solutions
i = o .5 = 1 T
6
=

01. Ans: (c)


Sol: Product layout 1s for mass production
03. Ans: (c)
operations for which line balancing
technique is used . L ti
Sol: Tl = x 100 = � x 100 = 80%
nc 6x10

Two Marks Solutions 0 4. Ans: 80%


Sol: I\ = 7 .2 minutes, n = 6
. T 7 .5 x 60 .
01. Ans: (c) Cye1e time = C = - = --- = l . 5 mm
N 300
Sol:
Theoretical number of work stations
L t j 7.2 �
x= = 5
C 1.5
Line efficiency
})i = 43; n= 5; C= 10 Time to assemble one limit
= ----------- x JOO
L t. cycle time x Theoritical number
Balance delay = 1- -'
nC of work stations

= 1-� = 0.14 or 14 %
5 x 10
= L ti x 100 = � x 100 = 96%
Cx x 1.5x 5
Lt
Tlactuat = --i X 100 = � = 80°//Co
02 . Ans: (a) nC 6x l .5
480 =
Sol: Cycle Time = 6min .
80
12
No. of work stations = =2 M
6

jlJIl4jj§jjji4ibiRflb!N\III04#yderabad I Delhi I Bhopal I Punc I Bhubancswar I Lucknow I Patna.I Bcngaluru I Chcnnai I Vijayawada I V17.3g I Tuupali I Kukalpally I KolkalaI
Chapter
9 Scheduling
Average flow time (in days) for the above
Two Marks Questions jobs using shortest Processing time ru le is
(a) 20.83 (b) 23.16
01 . A set of 5 jobs is to be processed on a sing le (c) 12 5.00 (d) 139.00
machine. The processing time (in days) is
given in the tab le be low. The ho lding cost for Common Data for Question 03 & 04
each job is Rs . K per day. Four jobs are to be processed on a machine as per
(GATE-ME-08) data listed in the tab le. (GATE-ME-10)

Job Processing
Job Processing Due
p
time
Time (in days) date
6
5
2 1 4
Q
R 3 2 7 9

1
s 2 3 2 19
T 4 8 17

A schedu le that minimi zes the tota l inventory


03. Ifthe Ear liest Due Date ( EDD) ru le is used to
cost is
se quence the jobs , the number of jobs
(a) T -S-Q-R -P (b) P-R -S-Q-T
de layed is
(c) T -R -S-Q-P (d) P-Q-R -S-T
(a) l (b)2 (c) 3 (d)4
02. Six jobs arrived m a se quence as g iven
be low : (GATE-ME-09) 04 . Using the Shortest Processing Time (SPT)
ru le , tota l tardiness is
Job Completion time
(a) 0 (b) 2 (c) 6 (d) 8
I 4
II 9
0 5. Five jobs need to be processed on a sing le
III 5
IV machine. All the jobs are avai lab le for
10
V 6 processing at time t = 0 . Their respective
VI 8 processing times are given be low.

Fyderabadl I>elhi l BhopaJ ! Punc ! Bhubaneswarl wcknow ! Patna ! Bcngaluru ! Chcnnai l Vtjayawada j Vuag !Tuupati I Kukalpallyj KolkataI
ACE : 959 : Scheduling
.:-...�.. �Publntvn
w " • • • •

J obs I II III N V Consid ering today as the Day 1 0 in the


Processing time s 1 3 4 7 14 11 prod uction calendar of the Hobbing S hop,
(in minutes) the se quence of the ord e rs scheduled using
the ' Critical Ratio' rule is
(GATE-PI-1 7)
The average completion time (in minutes) of
j obs as pe r the S hortest Proce ssing Time rule
lS (GATE - PI-16) (a) P - Q -R (b) P -R - Q
(a) 9.8 (b) 24.2 (c) Q - P -R (d ) Q -R - P
(c) 49.0 (d) 1 21 .0

Five Marks Questions


06. F ollowing data refe rs to the j obs (P, Q, R, S )
which have arrived at a machine for
01 . S ervice workshop has fourj obs on hand to be
scheduling. The shortest possible average
processed. The d ate and processing time for
fl ow time is ____ day s
each of thej obs are given be low
J ob Proce ssing Time (days) (GATE-ME-90)
Job Proc essing Du e dat e
P 15
Q No. tim e, davs
R 7
9

6
22 1 3
9
s 12
2
3 5 4
(GATE-1 7-SET- l) 4 9 14
07. In a gear manufacturing company , three
ord e rs P, Q and R are to be proce ssed on a Consid ering mean latene ss and me an fl ow
hobbing machine. The orders were re ce ived time, evaluate the shorte st-time rule and
in the sequence P - Q - R. The Table least- slack rule and re comme nd the d esirable
ind icates the proce ss time re maining and rule.
production calendar d ue date for each other.

Order Proc ess Tim e Du e dat e


02. A j ob shop incurs a cost of Rs. 60/- per d ay

R emaining(day)
for each day a j ob is in the shop. At the
beginning of a month there are five j obs in
4 Day 20 the shop with the follo wing data.
Q 16 Day 30 (GATE-ME-96)
R 6 Day 1 9

11111;;;;;;;;;;;;01aiM\Uiii·/"
·yderabad l Delhi l Bhopa! I Pune l Bhubaneswar l Lucknow l Patna l BengaJuru l Chennai l Vuayawada j Vizag I Tirupati I Kukatpal)yl Kolkala I
: 960 : IM & OR

Job 3
6
1 2 4 5
Processing time 5 3 8 2

9
(days)
Due date (days) 10 12 20 8

Wh ich schedule w ill m in im ize the total cost ?


What is the m in imum total cost ?
Wh ich jobs ( if any) fail to meet the ir due
dates ?

03. A job shop has 6 orders to be completed by a


s ingle turning centre. The process ing t imes
and due dates are as follows :
(GATE-ME-98)
Order 1 2 3 4 5 6
Processing 3 2 9 4 2 4
time
Due date 17 21 5 12 15 24

Assume that all orders are ready for


process ing . Give a product ion schedule that
m in im izes the average flow t ime. Compare
th is schedule w ith one that m in im izes
tard iness (lateness)

�yderabad l DelhiI Bhopal I Punc l Bhubancswarl Licknowl Patna l Bcnga)urul Oicnnai IVuayawadalVllOllf: l'!ruparl I Kukalpallyj Kolkam I
SOLUTIONS
04. Ans: (d)
Sol:
Two Marks Solutions
E DD- Rule
N o. of j obs delivered late are 3
01. Ans: (a)
Job Process C1 D1 C1-D 1
Sol: According to SPT rul e total inventory cost time(days)
. . .
1s mm1mum. 1 4 4 6 -2
2 7 11 9 2
8 19 17 2
02. Ans: (a) 3 2 21 19 2
Sol: SPT rule

Completion Job T 1 DD Slack (DD- T i)


Job Process time (days) 1 3 7 4
time
2 6 9 3
1 4 4 3 5 4 -1
4 9 14 5
3 5 9
5 6 15
6 8 23 SPT- Rule

2 9 32 Process
10
Job C1 D1 C rD1
4 42 time(days)
3 2 2 19 - 17
�C i 125 1 4 6 6 0
2 7 13 9 4
4 8 21 17 4
z:c.
n
1 .
Average F ow Ti me = --'

= 125 = 20. 83 Positive lateness is tardiness.


6 Total tardiness = 8

03. Ans: (c)

ydcrabad I Delhi I Bhopal I Punc I Bhubancswar I Lucknow I Patna I Bcngaluru I Chcnnai I Vuayawada j Vizag I Tirupati I Kukalpally I Kolkata


: 962 : IM & OR

05. Ans: (b)


Sol: SPT RULE ORDER Proc ess tim e DD -TDD
R emaining,
CR =
Job Tim e Compl etion
PTR
PTR (days)
tim e (ci)
p 4 (20 -10) / 4 = 2. 5
II 4 4
Q 16 (30 -10) / 16 =
III 7 11
1. 25
V 11 22
R 6 (19 - 10) / 6 = 1. 5
I 13 35
IV 14 49
CR -+ critical ra tio
L Ci = 121 DD -+ d ue date
TDD -+ today' s date
Mea n completion time PTR -+ process time rema ining
L Ci = 121 = Schedule the j ob with least critical ra tio
= 24.2min
n 5 first, Q-R-P

06. Ans: 31
Sol: SPT rule gives shortest average fl ow time Five Marks Solutions
Job Proc ess tim e Compl etion tim e
(T1) (Ci) 01.
Q 9 9 Sol: SPT R ul e

Job T1 Ci Di C i - D1
s 12 21
p 15 36 1 3 3 7 -4
R 22 58 3 5 8 4 4
2 6 14 9 5
L Ci = 124 4 9 23 14 9
L Ci = 48 L Ci -Di = 14
. L ei 124
Mea n fl ow ti me= -- = - = 31
n 4 Mean F low Ti me= -i
. I:C
- = 12
=
48
n 4
07. Ans: (d) L(C. -D. ) = -
14 =
Sol:
Mea n la teness = 1 1
3. 5
n 4

\( I I 11g1 r H t 1111g P1 1hlH ,tlH111.., Fydcrabadl DelhilBhopall PunclBhubaneswarl LucknowlPatnalBcngaluru i ChcnnailVliayawada!V"17.ag J 1lrupati I Kukatpallyl Kolkata I
: 963: Scheduling

Least Slack Rule : 03.


Sol: SPT - rule mm1m1zes ave rage fl ow time
Job ti Ci Di Ci - Di
3 5 5 4 1 EDD - rule minimiz e s me an tard ine ss

7 7 C1
2 6 11 9 2
1 3 14 Job T1 D1 C1 - Di
4 9 23 14 9 2 2 15 - 13
L Ci = 53 C
r i -Di = 19
2 2 4 21 -17
1 3 7 17 - 10
4 4 11 12 -1
. rc 53
Me an Flow T1me --i = - = 13. 25 =
6 4 15 24 -9
n 4
r(C.' -D. ) = 19 = 3 9 24 5 19
Me an Late ne ss = ' - 4.75
n 4 I Ci = 63 I Ci -Di = 49
The be st ru le is SPT as Me an Flow Time and
Job Ti Ci Di Ci -Di
Me an Late ne ss are minimum.
3 9 9 5 4
4 4 13 12 1
02.
5 2 15 15 --
Sol: SPT rule is use d for minimiz ing me an fl ow
1 3 18 17 1
time
2 2 20 21 -1
EJ - EARLY JOB 6 4 24 24 0
OS - ON SCHEDULE L Ci = 99 I Ci -Di = 6
TJ - TARDY JOB
63
M FT = - = 10. 5; T; = Process Time ;
6 C; = Completion Time
Job ti Ci di Ci -di D; = Due Date ;
4 2 2 9 -7 EJ MT = !2_ = 3. 17 MFT = Mean Flow Time
6 MT = Mean Tardiness
2 3 5 12 -7 EJ
1 5 10 10 OS r ci =99 =
M FT = 16. 5
5 6 16 8 8 TJ n 6
3 8 24 20 4 TJ r c i -Di =� =
MT= 1
n 6

M inimum total cost= 57 x 60 = 3,420


N o. ofj obs which fail to mee t due date are 2

\{ I l 1 11.!,I I H t 1 1 1 1 1.!, P11hh, ,1111,11, . ydcrabad J Delhi J Bhopal J Pune I Bhubaneswar I Lucknow J Patna I Bcngaluru I Chennai I Vtjayawada I Vizag I 'Iirupati I KukatpalJy J Kolkala
Cto Sequencing
03. In a machine shop , four jobs need to be
Two Marks Questions assigned to four di fferent machines. Each
ofthe jobs is to be assigned to one machine
01. A manu facturing shop processes sheet metal only at a time . The time taken to complete
jobs , where in each job must pass through the job in di fferent machines is given in the
t wo machines ( Ml and M2 , in that order). Table.
The processing time (in hours) for these jobs
Machines
1s ( GATE-ME-06)
M1 M2 M3 Mi
Machine Jobs
11 15 13 14 17
pQ R s T u 11
8 2 7 11 16
rn Ji 12 15 13
Ml 1 5 32 ....., 13 13 12 10 11
M2 6 19 13 20 14 7 14 15 17 14 16
The optimal make -span (in hours) of the In order to ensure that the total time
shop is re quired to complete all the jobs in
(a) 120 (b) 11 5 (c) 109 (d) 79 minimum , the optional assignment of the
job is
02. Data on five products to be processed on a (GATE- PI- 17)
single machine is given belo w (a) 1 1 �M i, J i �M2 , 13 �M 3, 14 �M 1
Product Release Processing Due (b) 11 �M2 , J i �M , , l3 �M i, l4 �M 3
time time date (c) 11 �M2 , J i �M 1, l3 �M 3, l4 �M i
p 0 3 10 (d) 11 �M i, J i �M2 , h �M1 , l4 �M 3
Q 2 4 9
R 0 2 15
s 1 5 11
T 1 1 13

For the processing se quence R -P-S-T -Q, total


tardiness is ___ (GATE - PI -15)

yderabadl Delhil Bhopal J Pune J Bhubaneswarl Lucknow l Patnal Bengalurul Chennai J VtjayawadaJVizag jTU'Upati I Kukatpallyl Kolkata
ACE
��-�"=
� E�
&�rgm

·-:�m�·��
��
blicatiooa

· �·�========::::::,����:=============�:�
9 Se uenc
q�:��ing

= 65

Assuming there is only one denter and one


Five Marks Questions painter
(a) What is the sequence in which cars are
0 1 . A job shop has received an order consisting to be processed to minimize the make
of 7 different jobs. These are to be processed span?
through a milling machine and a drilling (b) What is the minimum make span?
machine. Processing time of each job
including set-up time in each machine is
given in the following table. Suggest the
optimum processing sequence. Prepare a
Gantt chart and indicate the total time for
completing the order. If no other order is
processed simultaneously, what are the

(GATE-ME-91)(SM)
percentage utilization of milling and drilling
machines?

Proc essingtim e in min u t es in r esp ectiv e


machin es
Job Nam e
MillingMIC
A B C D E F G

DrillingMIC
12 8 9 14 6 7 10
10 12 6 8 14 9 5

02. There are six cars waiting to be repaired at a


car repair shop. Each car has first to be
dented and then painted. The estimated times

(GATE-ME-9S)(SM)
for each car are given below.

Car 1 2 3 4 s 6
Denting
tim e (hrs)
4 10 2 5 6 1

Painting
tim e (hrs)
3 2 5 4 2 6

\{ } ( Jl�!lll l I lllg P11l,IH ,IIJ()JI- Fyderabad l Delhi lBhopal j Pune!Bhubaneswar l l,.icknow! Patna lBengaluruj Chennai l Vuayawada lV,zag ITirupati I Kuka!pally l Kolkata I
SOLUTIONS
03. Ans: (b)
Two Marks Solutions Sol: Time Matrix --+ Minimi ze
M1 M2 M 3 �
01. Ans: (b) J 1 15 13 14 1 7
Sol: Se quence by J ohnson 's Rule is : J z 11 12 15 13
R, T, S, Q, U, P. J3 13 12 10 11
1 5 1 7 14 16
1 4
M -I M -II ]4
J ob J1 2 () row Transaction
T in T out T in T out
R 0 8 8 21 J2 () 1 4 2
I
T 8 19 21 35
s J3 3 2 0
]4 I
19 46 46 66
46 78 78 97 3 0 2

I i
u 78 94 9 7 104 J1 2 0 3 column transaction
p 94 109 109 11 5
T in = Time In , T out = Time Out
Make Span Time = 115hours
-
Jz 0
J3 3
1 4
2 0
1
0
1 3 0 1
02. Ans: 10
]4

Sol:

Ti = RT+ PT Ci Di Ci- Di
R 2 2 15 - 13
p 3 5 10 -5
s 6 11 11 0
T 2 13 13 0
Q 6 19 9 10
t Tardy Job
Positive lateness is called Tardiness .
Total Tardiness = 10 days

\< I I 11�1111 t 1 1 11� P1il ,!11 .111()11" ydcrabad l Dellii l Bbopa) I Punc l Bhubaneswarl Lucknowl Patnal Bcnga)uru i Chennai lVliayawadal V17.3gi�pari I Kukatpa)ly l Kolkata
: 967 : Se que nci ng

02.
Five Marks Solutions Sol: Se quen ce by Johnson 's Rule is:
6-3-4- 1-2-5
01
Sol: Se quen ce by Joh nson 's Rule is: DENTING PAINTING
E- F- B- A- D- C- G CAR Out Out
In time In time
time time
Milling MIC Drilling MIC Idle time 6 0 1 1 7
In Out In Out 3 l 3 7 12
Job
time time time time 4 3 8 12 16
E 0 6 6 20 6 1 8 12 16 19
F 6 13 20 29 2 12 22 22 24
B 13 21 29 41 5 22 28 28 30
A 21 33 41 51
D 33 47 51 59 Minimum Make Span = 30
C 47 56 59 65 1
G 56 66 66 71

Utili zation o f Milling MIC=


66
x100 = 9 2.9 %
71

Utili zation o f Drilling MIC =


64
x100 = 90.1 %
71

\( I I 11•.,111t t rlli� 1'1dd1t ,1111111-. �ydcrabadl Dclhi j Bhopalj Punc l Bhubancswarl I..ncknow l Patna l Bcngalwu l Chcnnai lVuayawadalV,zag ITuupati I Kuka!pallyj Kolkala I
Ctt Assignment

One Marks Questions Two Marks Questions

01 . T he tota l n umber of d ecision va riables in 01. A firm is required to procure three items
the obj ective fun ction of an assignmen t (P, Q an d R) . T he prices quoted for these
problem of siz e n x n ( n j obs an d n items (in Rs. ) by suppliers S1, S2, an d S3 a re
machin es) is given in table. T he management policy
(a ) n2 (b) 2n (c) 2n-1 (d ) n requires that ea ch item has to be supplied by
(GATE-ME-1 4) on ly on e supplier an d on e supplier supply
on ly on e item. T he min imum tota l cost (in
02. Solution to the ba lan ced assignmen t problem Rs. ) of procuremen t to the firm is
is bina ry d ue to (GATE - PI-15) (GATE-ME-06)
S uppli ers
It em
(a ) linea r formula tion
(b) n on-empty feasible region S1 S2 S3

(c) approx imation a lgorithms p 110 120 130


(d ) uni-modula rity property Q 115 140 140
R 125 145 165

(a) 350 (b) 360 (c) 385 (d) 395

\( I I 1 1 .:,1 1 ,t 1 1 1 11.:, P11l,l1< .111(111, �yderabad I Dcllii I Bhopal I Punel Bhubaneswarl Lucknow! Patna! Bengaluru l Chennai IVrjayawada IVtt.ag jT=pati I Kukatpal)yl Kolkata I
SOLUTIONS

One Mark Solutions Two Mark Solutions

01. Ans: (c)

01. Ans: (a) Sol:


Sr S2 . S3 _ S1 S2 S3
p 1 1 0 1 20 1 30 �
02. Ans: (d)
V Colurnn

Sol: Only one job can be allotted to one


Transaction
Q 1 15 1 40 1 40 ) 15 5

machine.
R 1 25 1 45 1 65 ) 1 0 20

p 0 1 0 20 Row 5 e 6
Q 0 25 25
Transactior
e l0 e
R 0 20 40 e 5 15

P-S 2 - 1 20
Q- S 3 - 1 40 } 385�
R-S 1 - 1 25

lltl•ih§jjjgq.jjj4Rftftifflmil!� yderabad I Delhi I Bhopal I Pune I Bhubaneswarl Lucknow! Patna I Benga)uru I Chennai I Vrjayawada I Vizag I Tirupari I Kukatpally I Kolkata
c12 Material Requirement
Planning

Two Marks Questions


One Mark Questions
01. The product structure of an assembly P is
01. For planning the procurement or production shown in the figure.
of dependent d emand items, the technique E stimated d emand fo r end product P is as
most suitable is__ (GATE-ME-95) fo llows

02. In an MR P system, component d emand is W eek 1 2 3 4 5 6


(GATE-ME-06) Demand 1000 1000 1000 1000 1200 1200
(a) Forecasted
Ignore lead times fo r assembly and sub­
(b) E stablished by the master production
assembly. Production capacity (per week)
schedule
fo r component R 1 s the bottleneck
(c) Calculated by the MR P system from the
operation. Starting with z ero inventory, the
master production schedule
smallest capacity that will ensure a feasible
(d) Ignored
production plan up to week 6 is
(GATE-ME-08)
03. M aterial R equirements planning DOES
NOT include (GATE - PI-15)
(a) M aterial price Assembly

(b) B ill of material


(c) Inventory level Sub-assembly

(d ) Production schedule

(a) 1000 (b) 1200 (c) 2200 (d) 2400

yderabad I Delhi I Bhopal I Pune !Bhubaneswarl Lucknow I Patna! Benga)uru I Chennai IVgayawadajVu.ag - j Tuupati I Kukatpal)yl Kolkata
: 97 1 : MRP

Five Marks Questions

01 . The product structure of a product P i s


shown i n Fi gure. The assembly of 2 uni ts of
4
B and 3 uni ts of C to produce 1 uni t of A
takes 1 week, li st the assembly of 1 uni t of
A and 4 uni ts of D to produce 1 uni t of
product P takes 2 weeks. The orderi ng lead Determi ne the net requi rements of i tem C i f
ti mes fo r B , C and D are 2, 1 and 2 weeks the i ni ti al i nventory of A, B , C and D i s
respecti vely . The M aster schedule fo r 1 0, 20, 1 5 and 50 respecti vely . Also,
product P i s as fo llows: (GATE-ME-96) determi ne the ord eri ng schedule under a lot
fo r lot orderi ng poli cy .
W eek 6 7 8
Demand 20 1 0 50

\( I J ! ]� ] I I { ( ! I l l � 1 '1 i ! ,l 1 t , t 1 H " I " I I I I I I I I I


�ydcrabad Delhi Bhopal Pune Bhubaneswarl Lucknow Patna Bcngaluru Chennai Vuayawada l V,zag Tirupari I Kukatpally I Kolkata I
SOLUTIONS
One Mark Solutions Five Marks Solutions

01. 01.
Sol: MRP. Sol : O level

02. Ans: (c)


LT=2 weeks

l leve� l level D- 1

03. Ans: (a) LT= l week


LT=2 weeks
2levelct] C- 1 2level

i 3 ' 5 ' 1 I

....
Dl8CBIPUOl'f DIICRIP?IOl.'I l
Two Marks Solutions p Pl. 20 10 58
'l E C 20
0 1-i
LF L OHi - - - - - -- --
58

01. Ans: (c)


SS = O P O it [i) '20 10 58
LT = 2 W

Sol: Every P requires 2R


A P B. '20 10 51
I.EC
t Le'ftl
-- -
lO 10 St

Let production be x per period


LFL OBI 10 10 lO --
S S =O , oil!!] lO 10 50
LT= l 'W
D P l. .,JO 40 20)
20)
1 2 3 4 5 6
l lnel
LFL
I.EC
OBI 50 ,0 ,0 - -- --40

li] JO 40
Week S S=O POB. 200
Demand l OOC 1 000 1 000 1 000 1 200 1 200 LT=2'W
B P B. 20 '20 IOI
x- 2x- 3 x- 4x- 5 x- 6x- 2 1cftl I.EC ·- '20 UNI
Ending
LFL ORI f! 20 20 ·- - -
Inventory 200C 4000 6000 8000 1 040C 1 2800 S S=O P O I. 20 100
LT •2W
10

6x - 12800 = 0 for minimum capacity condition ts


C P B. 10 1511
2 1nel
LF L
I.EC
ORI IS .. .. 1:50
lS
..
JO
S S=O P O I. � IS JO 150

PR = Projected Requirement
LT = I 'W

= 12800 =
2133 ;::; 2200
6 REC = Receipts
X

OHI = On Hand Inventory


POR = Planned Order Release.

IMl•lh§jjji4.jji4Rflnjffi\d.jj+yderabadI Dcllri I Bhopal I Pune I BhubancswarI LucknowI Patna I BengaluruI Chennai I Vuayawada IVizag I TJrupari I Kukatpa)JyI Kolkat,j
Ct3 Miscellaneous
04. If standard production is 20 units, a
One Mark Questions worker's actual output is 1 8 units, piece rate
is Rs. 500 per unit, and over - achievement
01. For an assembly made of n components, the rate is Rs. 750 per unit, then the wage paid
dimensions on each component i follow a to the worker, (in Rs.), as per Taylor's
normal distribution and have tolerance Ti. differential price rate wage incentive plan,

(GATE - PI -15)
Overall dimension of the assembly is La lS ----

(GATE - PI-15)
with tolerance Ta· The relationship between

(GATE - PI-15)
(b) T = �L�=I T;2 05. Ishikawa diagram represents
3

(a) Different types of quality defects


(b) Quantitative relation between the extent
02. As per the principles of motion economy, of defect and a process parameter
which one of the following is NOT a pivot (c) Relation between defects and their

(GATE - PI-15)
for a classified movement of human body ? causes
(d) Prioritized quality defects
(a) Knee (b) Elbow
(c) Torso (d) Wrist
06. Which one of the following is NOT in the

(GATE - PI-15)
scope of Enterprise Resource Planning
03. For air travel over a distance of 500 km, the
(ERP) system ?
ticket price is Rs. 4000. The comfort of the
air travel can be monetized at Rs.3000, and (a) General ledger entries
the monetary value of time saved because of (b) Materials management system
air travel is Rs. 3000. The value of air travel (c) Order management system

(GATE - PI-15)
is ------ (d) Employee promotion policy

!IJIIQ@jjjl4fmQRflGiM\hih+iyderabad I Delhi I Bhopal I Pune I Bhubaneswarl Lucknow I Patna I Benga)uru I Chennai IVuayawada I V,zag ITuupari I Kukatpally I Kolkata I
: 974 : IM&OR

07. The facility layout technique that uses (b) directly proportional to its functions
relationship (REL) chart is and inversely proportional to its cost
(GATE - PI-16) (C) inversely proportional to its functions
(a) CRAFT. (b) Travel chart. as well as its cost
(c) Partial Set Covering. (d) ALDEP. ( d) directly proportional to its functions as
well as its cost
08. For a random variable, X, let Xbe the
sample average. The sample size is n. The 11. The operating characteristic curves of three
mean and the standard deviation of X are µ single sampling plansX, Yand Z with same
and cr, respectively. The standard deviation lot size and acceptance number are shown
of X is (GATE - PI-16) in the figure.
(a) ncr (b) cr
cr
(c ) - (d) �
8 1
§
c. 0.8
fn II)
C)

� 0.6
.....
09. ST and NT denote the standard time and the
;E' 0.4
normal time, respectively, to complete a job.
� 0.2

Allowance = LLx ST, where O < LL<l .


£ O L---------=�-------=-..1
0.00 0.02 0.04 0.06 0.08 0. 1 0 0. 1 2 0. 1 4 0. 1 6

Which one of the following relationships is Proportion nonconfirming

correct?
Considering the above operating
(GATE - PI-16)
NT
(a) ST = characteristic curves, the correct
(1-LL)
relationship of the plans with respect to
(b) ST=NT(l +LL)
sample size is
NT
(GATE- PI- 17)
(c) ST = (a) Sample size ofX< sample size ofY<
(l + LL)
sample size of Z
(d) ST=NT(l-LL)
(b) Sample size ofX= sample size ofY=
sample size of Z
1 0. In Value Engineering approach, the value of
(c) Sample size ofX> sample size ofY>
the product is
sample size of Z
(GATE- PI- 17)
(a) inversely proportional to its functions
(d) Sample size ofX> sample size ofY<
and directly proportional to its cost
sample size of Z

!1i•1ih§jjji4.jjgjjnnj@jjii!.\j ydcrabad I Delhi I Bhopal I Pune I Bhubaneswarl Lucknow I Patna l Bengaluru I Chennai I Vuayawada J V,z.ag JTirupati I Kuk.atpally J Kolkat,.
= 975 :
�.. :·�·��
:!'.ffll
�5·�=&�..,,�·- ���· �·::=========�:��
Nmn :::::.::
===========��·::ll:
t." ACE Misce ane ou s

12. Match the ASME process chart symbols 02. A machine is bought for 25,00,000. The
with their correct description organization follows a declining balance
Symbols Description method of depreciation with a depreciation
P. 0 1. STORAGE charge of 25%. If the machine is sold at Rs.
1750000 at the end of second year, then the
Q. => 2. TRANSPORTATION
profit on the book, m Rs. . , · is
R. D 3. OPERATION (GATE - PI �lS)
S. V 4. DELAY
T. D 5. INSPECTION 03. There are three alternatives to meet the
(GATE-PI-17) demand of a product
(a) P-3, Q-4, R-1, S-5, T-2 Alternative I:
(b) P-4, Q-2, R-5, S-1, T-3 Manufacture using a process P -
(c) P-3, Q-2, R-5, S-1, T-4 Alternative II:
(d) P-1' Q-5' R-3' S-2' T-4 Manufacture using a process Q
Alternative III:
13. A Shewhart X-chart was developed for an Buy the product from a vendor
in-control process. Considering the The costs associated with each alternative is
probability of a point falling outside the 3cr given below.
control limits as 0.0026, the value of
average run length for this chart is
Alternative Alternative Alternative
Cost

(GATE-PI-17)
I II III
Fixed cost Rs. 1 ,00,000 Rs. 1,90,000
Variable
Rs. 75 Rs.60
cost(per unit)
Purchase price
Two Marks Questions (per unit)
Rs.87.50

Alternative I is cheaper compared to


01. In a time study experiment, observed time is
alternative II when the demand is
15 minutes, operator rating is 90, personal
(GATE - PI-15)
need allowance is 4%, fatigue allowance is
(a) 8500 (b) above 8000
3%, contingency allowance for work is 3%
(c) 6500 (d) below 6000
and contingency allowance for delay is 2%.
The total work content, in minutes, is__
(GATE - PI-15)
yderabad I Delhi I Bhopal I Pune I Bhubaneswar I wcknowI .Patna I Bengalmu I Chennai I Vijayawada I V=g I Tuupari I Kukatpally I Kolkata
: 976 : IM &OR

04. In a manufacturing process, 24 samples For the mam study, the minimum sample
each of size 50 items were inspected and a size for the sample mean time of any work
total of 52 defective items were observed . element to be within 0.1 minutes of its true
The lower and upper control limits set for mean time with 95% confidence
the p-chart should, respectively, be (corresponding standard normal value,
(GATE - PI-15) Zo.02s = 1.96 is ------
(a) (0.043, 0.12) (b) (-0 .043, 0 .086) (GATE - PI-1 6)
(c) (-0.043, 0.10) (d) (0,0.13)
0 7. Consider a system with 10 identical
05. A quality control engineer has collected 5 components connected in series. The time to
samples, each of size 30. The numbers of failure of each component is exponentially
defective items in the samples are given in distributed with a failure rate of 0 .10 per 500
the table below. days. The reliability of the system after 400
Sample Number I II III IV V days of operation is __
Number of defective items 3 2 4 1 5 (GATE - PI-1 6)

The upper three-sigma (3cr) control limit for


08. For a process, the quality loss coefficient is
the proportion of defective items in any
5. The target value on the dimension to be
sample is __ (GATE - PI-1 6)
attained through the process is 50 mm. If the
maximum loss permissible (in monetary
06. A job consists of two work elements, P and
terms) is INR 80, the maximum allowable
Q . Completion time (in minutes) of each
deviation (in mm) from the target is
work element was measured. A pilot study
(GATE - PI-1 6)
involved collecting a sample of 40
observations. The results of this pilot study 1
(a) - (b) (c) 4 (d) Jw
are summarized in the table below.
4 vw
{T

Work Mean completion Standard 09. In a given year, a restaurant earned INR
element time (in minutes) deviation 38,500 in revenues. In that year, total
(in minutes) expenses incurred were INR 30,000 and the
p 1 0.50 depreciation amount was INR 3,200. At 40%
Q 1 0.05 tax rate, the net cash flow (in INR) for that
year was __ (GATE - PI-1 6)

!ISl#ii@jjj§§jjjijQftelM\hi¥�-Iyderabad l Dellii l Bhopal l Pune l Bhubaneswarl Lucknowl PatnaJ Bengaluru j Chennai j Vtjayawada j V,z;ig Tuupali I Kukatpa))yJ Kolkalal
"�
. •{F�PnJ,lirai,n
�••
" . ACE
. . . : 977 : Misc e lla ne ous

10. An air con d itioner un it is expected to run 150. The fi xed cost of production is Rs.
con tin uously . The mean time between 80,000 an d variable cost of production per
failures (MTBF) for this un it is 2,000 hours un it is Rs. 70. If the compan y wishes to
an d the mean time to repair (MTTR) is 48 achieve a profi t of Rs. 20,000 durin g the
hours. The availability of the ai r calendar y ear, then the min imum quan tity to
con d ition in g un it is ___ (up to three be produced is__ (GATE - PI - 1 7)
d ecimal places) . (GATE-Pl-1 7)
13. A fi rm manufactures capacitors using a
11. The ann ual d eman d of wrist watches specializ ed process. The d esired
produced on an assembly lin e is 1,03,125 specifi cation for the capacitan ce is 40± 10
un its. The lin e operates 50 weeks/y ear, 5 picofarad s (pF). The process used is in
shifts/ week an d 7. 5 hours/shift. The uptime statistical con trol. If the process mean is 41
effi cien cy of the line is 99% . The cycle pF and the estimated standard deviation is 3
time (Tc ) of the assembly lin e (in pF, then the process capability in dex ½k is
minutes/un it) is __ (up to two d ecimal (GATE-PI - 1 7)
places). (GATE-PI-1 7)

12. A hose couplin g manufacturing company


has production capacity of 2,500 un its per
y ear. The un it sellin g price of the item is Rs.

14. Quality con trol d epartmen t of a company main tain s ' c' chart to assess the quality of laptops. In
this process, twen ty laptops are examin ed randomly . The number of non conformities observed per
laptop is given in the Table.
L aptop n umber 1 2 3 4 5 6 7 8 9 10 11 12 13 14 15 16 17 18 19 20
N umber of
1 3 7 4 10 6 1 5 4 3 6 4 2 7 4 2 9 8 5 2
n on conformities
B ased on the data, the upper con trol limit for the ' c' chart is ____ (up to two d ecimal places) .
(GATE-PI- 1 7)

\( ( I 11.:.111t t l l l1.:_ P1 i l ,I H ,tlit )lh �ydcrabad l Delhi l BhopaI I Punc l Bhubaneswar l w:know l P'atna l BcngaJuru l Chcnnai l Vuayawada l Vi.zag I Tirupati I Kukatpallyl Kolkata I
·s o LOTIONS
08. Ans: (d)

Sol: Standard deviation, S = -


One Mark Solutions
fn
01. Ans: (b)
09. Ans: (a)
02. Ans: (a) Sol: RA = Relaxation allowance = LL x ST
ST = NT + RA
03. Ans: 1 .5 ST = NT + LL X ST
worth 3000 +3000 ST (1 - LL) = NT
Sol: Value = = = 1.5
cos t 4000 NT
ST =
1- LL
0 4. Ans: 9000
Sol: Standard output = 20 units 10. Ans: (b)
Actual output = 18 units WORTH
Sol: VALUE=
Actual output < Standard output COST
Hence low piece rate. Standard or more than
standard high piece rate. 11. Ans: (c)
Wage = 18x500 = 9000
12. Ans: (c)
05. Ans: (c)
Sol: Ishikawa diagram 1s cause effect 13. Ans: 3 8 4.6
relationship. Sol: Average run length,
(or) It is also a fish bone diagram. l
ARL = .!_ = 384.6
p 0.0026
06. Ans: (d)
P(ONE POINTOUTSIDE CONTROL LIMIT)
=----------------
07. Ans: (d)
Sol: ALDEP BASED ON REL
REQUIREMENTS

jat4l4j@jjj4UijjiAflbiM\@j.1�fyderabadI Delhi I Bhopal I Pune I Bhubaneswar I Lucknow I Patna I Bengaluru I Chcnnai I VrjayawadaJ Vizag JTirupari I Kukatpa1J'y I Kolkata I
t . A:CE . .
'!..:-.��PnNiariooa : 979: Misce llane ous
� � ======================================
= 468750
Two Marks Solutions B ook value at end of second y ear
= 1875000 -468750 = 1406250

01. Ans: 1 4. 85 min (or) 15 min Profi t = 1750000 -1406250 = Rs. 343750
Sol: Observed time= OT = 15 minutes
Ope rator rating = 90% 03. Ans: (d)
Contingency allowance for delay is not
Sol: q1 = Fz -Fi
consid ered for calculation of work content. V1 -Vz
. OTxR
B as1c T'1me = --- = 190000 -100000 = 90000 _ 6000 units
SR 75-60 15
= 15x90 = 13.5min Quantity < 6000 units lower fixe d cost
=

100 machine F I is prefe rred


Total work content Quantity > 6000 units = higher fix ed cost
= BT( l + PWA+ FA+ CWA) machine F2 is prefe rred
100 At 6000 any machine Alternative I or
= 13.5( 1+ 4+ 3+ 3) Alternative II.
100
= 13.5 x 1.1= 14.85 minutes 0 4. Ans: (d)

Sol:
(or) _ = Total number of d efe ctives
p -----------
Total work content no.of samplesx items per sample
BT 52
- = = 0.04333
PWA+ FA+ CWA 24x 50
( l+ )
100 CL = 0.0433
13. 5 =
=- .
- 15 mmute s U CL � CL + 3�p(i: p)
0.9

02. Ans: 3 437 49 to 3 43751 = 0.0433+ 3 0. 0433 1-0.0433


Sol: Cost = 2500000
50
= 0.1297
De preciation in fi rst year = 2500000x0.25
l
= 62 5000 LCL = CL - 3�p{ : p)
B ook value after fi rst ye ar = 1875000
De pre ciation in second ye ar = 1875000x0. 25 = 0_ 0433_ 3 / 0.0433(1-0.0433) = 0
� 50
!IJl••@hliiih40inj@ij/§j�)Hyderabad I Delhi I Bhopal I Pune I Bhubaneswar I Lucknow I Patna I Bengalwu I Chennai I Vijayawada I ':,zag ITirupati I Kukatpally I Kolkata I
.., ,....:.. ....Fii · ACE
... 9=
· ========= : = M=
80 : ==============I= &=OR
;.. ... =�tgm
�;;;;;;;;;,g�;,:;•;;;;';;;;;;
W,'l
nn Pl it,liatinns
= = =

h,
- ( z:s')'
O. l l 1--------- UCL
0.09
= (1.96x0. 5 ) = 96.04 obser vations
2

0.07 0.1
Standard deviation = s2 = 0.0 5 minutes
0.05
0.03
Error ofstudy = h = 0.1 minutes
0.01 1---------
CL Zo.ozs = 1.96
O
0. 1 0.2 0.3 0.4
x LCL
h,
- ( z:s, )'
05. Ans: UCL = 0.26 43
= (1 ·96x0.0 5 ) = 1 observations
2

Sol: p = -----------
sum ofde fectives 0.1
samplesi ze x number ofsamples Larger ofthe t wo values is 96.
= 3--
+2 +4 +1 + 5
--- Hence study should be continued upto 96
5x30 obser vations.
= � = _!._ =0.1
07. Ans: 0. 4 49
t
5x30 10
Standard error ofproportion Sol: R = e-nA.
s

Fa 1i· ure rate, /\, = -


p(l - p) = /o.lx (l-0.1) 0.1
cr p = �
'I

n v 30 500
t =400 days , n = 10 in series
= �O.lx0.9 =0.0 54 772
R s =e 500
30 -10x�x400
=0.449
Central line = p = 0.1
Upper control limit = p +3 cr P 08. Ans: (c)

Sol: R =
= 0.1 + 3x0.0 54 772 = 0.2643 Ao
Lo wer control limit = p -3 cr P (�o )2
R = Quality loss coefficient
= 0.1 -3x0.0 54 772 = 0
Ao = Economic consequence
06. Ans: (96) �o = Functional limit or customer tolerance
Sol: Standard deviation = s 1 = 0. 5 minutes R = 5 , Ao = 80,
Error ofstudy = h = 0.1 minutes
vs
�0 = � = [so = -J16 =4
Zo.ozs = 1.96 VR
!M1•Qjj§jjji44hQ13inj@jd@ij>Hyderabad l Delhi l Bhopal l Pune l Bhubancswar l Lucknow l Patna l BengaJuru l Chennai j Vijayawada j Vuag ITirupati I Kukatpally J Kolkata I
"
�-·���
'!.,. .
. . .
" . ACE : 981 : Misce llane ous

09. Ans: 63 80 12. Ans: 1250


Sol: Sol: s = Rs 150/-
Particulars Amount F = Rs 80,000
(INR) v = Rs 70/-
Ye arly reve nue 38500 P = Rs 20,000
(-) Total ex pe nses 30000 M arginal Costing Equation,
(-) De pre ciation 3200 q(s- v) = F+ P
-
Earn ing before taxe s (EBT) 5300 -
F + P 80,000 + 20,000
q=
(-) Tax rate @ 40% 2120 s-v 150- 70
Earn ings afte r taxe s (EAT) 3180 = 1.00,000= 1250 units
(+ ) De pre ciation 3200 80
63 80
13. Ans: 1
Net Cash flow = (EAT + Depreciation)

Sol:
10. Ans: 0.976
Sol: MTBF = 2000 hours
MTTR = 48 hours
. MTBF
Ava11ab·t·
1 1ty = LSL =30pf µ = 4 1 pf USL=50pf
MTBF +MTTR
= 2000 = 0.976
2000+48 USL = 50 pf , LSL = 30 pf, cr = 3 pf

1 1. Ans: 1.08
USL -LSL
Proce ss Capab.1.
1 1ty = ----
Sol: N umbe r of units re quire d,
6cr
50-30 =
N = 1,03,125 units = 1.11
6x 3
Available time for production,
USL-µ = 50-41=
T = N o.of hours/shift x N o. of shifts/ wee k (CPU ) = +l
3cr 3X 3
x N o. of wee k/ye ar x 1l1ine(uptime )
µ -LSL = 41-30 =
T = 7. 5 x 5 x 50 x 0.99 x 60 minute s (CPL) = 1. 22
3cr 3X 3
= 111375 minutes
Proce ss capability index ,
Cy cle Ti me = C = _!. Cpk = min (CPU ; CPL) = 1
N
111375 .
= = 1.08 mmute s
103125

llfll4jj§jjji4jjjj4dfiiiijjfij.ffydcrabad I Delhi I BhopalI PuneI BhubaneswarI wcknowj PatnaI Bcngaluru I ChcnnaiI VtiayawadaIVu.ag jTuupali I Kukatpa))yI Kolkata I
: 982: IM & OR

1 4. Ans: 1 1.11
Sol: LC = 93 , n = 20

c = LC = 93 = 4. 65
n 20
Cont rol limit , CL = C = 4.65
U pper Control Limit ,
U CL = C+3.Jc = 4.65 + 3.J 4.65 = 1 1 .1 1

ubaneswarl Lucknow j Patna j Bengaluru l Chennai j Vrjayawada l Vizag I Tirupati I Kukatpallyj Kolkala I
!IJl•i!l§jjj4iljjjQAflGiMl!Nh+1yderabad j Dclhi ! Bhopal j Pune l Bh
Page No. 983

Material Science
(Questions)
Page No. 984

CONTENTS
- �r- - '-"'--- --

Chapter Q ue stions Solutions


e
-1 �c ��g
Name of the Chapter
No.
11

No. Page No.

01 Material Science 985 - 996 997 - 1004


Material Science
05. Aging of aluminium alloy leads to
One Mark Questions (GATE -Pl- 8 7)
(a) Loss of hardness with time
01. A linearly elastic and perfectly plastic (b) Loss of toughness with time
material 1s loaded slightly above the (c) Increase in hardness with time
proportionality limit. This will lead to__ (d) Appearance of the oxide scales.
(GATE -ME-8 7)
(a) Fracture (b) Small deformation 06. Formation of white cast iron is promoted by
(c) Large deformation (d) Cracking (GATE -PI-8 7)
(a) A slow rate of cooling
02. Structural constituents of equilibrium
(b) Fast rate of cooling
cooling of plain carbon steel (0.4%C) at
(c) A high carbon equivalent
room temperature are (GATE -Pl-8 7)
(d) The addition of calcium carbonate.
(a) Austinite and ferrite
(b) Ferrite and pearlite
07. Aluminium alloy 1s employed for
(c) Cementite and pearlite
transportation applications primarily
(d) Ferrite and martensite
because of its (GATE -PI-88)
03. A steel with higher hardenability primarily (a) Low density
enables the following to be achieved (b) High strength
(GATE -PI- 8 7) (c) High strength to weight ratio
(a) Higher hardness at the surface (d) High toughness
(b) Greater depth of hardening
(c) Lower depth of hardening 08. Carbon content of mild steel can be
(d) Reduced Soaking time (GATE -PI-88)
(a) 0.15% {b) 0.51%
04. Hardness of piece of brass can be increased (c) 0 .87% (d) 1.8%
by (GATE -Pl- 8 7)
(a) Pack carburizing 09. Tool steels are quenched and tempered to
(b) Induction hardening impart (GATE -PI-88)
(c) Cold working (a) High hardness
(d) Nitriding (b) High toughness
j Delhi j Bhopa! I Pune j Bhubaneswarl Luclmowj Patna j Bengaluru j Chcnnai l V\iayawadaj V,zag ITuupati I Kukatpallyl Kolkata I
Fydcrabad
\( 'I I 11g111< , 1 111g 1'11hlu .111<111,
" . ACE
. . .
.....����
., : 986 : ME_GATE_Previous solutions

(c) Combined hard ness and toughness 14. The powd er metallurgy technique for the
(d ) High strength production of precision components is
characteriz ed mainly by reduction in
10. The final heat treatment given to a forging (GATE -PI-88)
die is (GATE -PI-88) (a) M aterial cost (b) M achining cost
(a) Process annealing (c) Equipment cost (d ) Tool related cost
(b) Hard ening and tempering
(c) N ormalising and stress relieving 15. W hich of the following is / are not generally
(d ) Homogenising employ ed for shaping of thermoplastic
materials (GATE -PI-88)
11. The hard ness testing method which does not
(a) Inj ection molding
involve resistance to plastic d eformation is
(GATE -PI-88)
(b) Compression molding
(c) B low mold ing
(a) Shore scleroscope (b) B rinell
(d ) Extrusion
(c) Rockwell (d) Vickers

12. The method most widely used for 16. W hich of the following commonly used
items are not prod uced using thermosetting
(GATE -PI-88)
production of metal powders for use in
powder metallurgy (GATE -PI-88) plastic
(a) Crushing using impact (a) House hold buckets
(b) B all mill (b) Electric switches used in d omestic
(c) Liquid metal spray wm ng
(d ) Electroly tic deposition (c) Dining table tops
(d ) M elamine dinner set
13. During sintering of a powder metal compact
the following process takes place
(GATE -PI-88)
17. M etals which can be plastically d eformed
very easily have unit cells of
(a) All the pore reduce in size (GATE -PI-89)
(b) The powder particles fu se and j oin (a) B .C .C structure
together (b) F .C.C structure
(c) The powder particles do not melt but a (c) H. C.P. structure
bond is formed between them (d ) Cubic structure
(d ) Some of the pores gro'Y

\( ( l 11�IIH t ! Jill!, P11 hl1( ,l[h lJI'\ �yderabad j Delhi j Bhopal j Pnne j Bhubaneswar j LucknowlPatna!Bengaiwu!ChcnnailVtjayawada j Vizag I Tirupati I Kukatpally j Kolkata I
'!.
:.t : Fflf)PCClml Pubtiraiooa
. .t\CE . . : 987: Material Science
� � ======================================
18. The temperature required for heating (a) Density (b) Hardness
hypereutectoid steel for normalizing is (c) Yield strength (d) Elastic modulus
(GATE -PI-89)
(a) Equal to that for annealing 23. PVC sheets are produced by GATE -PI-90)
(b) Greater than that for annealing (a) Melt spinning process (

(c) Lesser than that for annealing (b) Injection molding process
(d) Equal to that for tempering (c) Calendaring process
(d) Roto molding process
19. On analysis a piece of steel was found to
contain 0.3%C, 12.0¾Cr, 12.0% V. It is a 24. Green strength in powder metallurgy refers
(GATE -PI-89) to the strength of (GATE -PI-90)
(a) High speed steel (b) Carbon steel (a) The original material
(c) Low alloy steel (d) High alloy steel (b) The powder before compaction
(c) The powder after compaction
20. Thermosetting plastics are joined by (d) The product after sintering and
(GATE -PI-89) compaction
(a) Hot air welding
(b) Friction welding 25. Ass ertion (A): Thermosetting plastic are
(c) Ultrasonic method gradually being substituted by
(d) Adhesive bonding thermoplastics for many engmeenng
applications
21. Plain carbon steel is called hypoeutectoid R eason (R): Thermosetting plastics are
steel if it has carbon percentage difficult to process (GATE -PI-90)
(GATE -PI-90) (a) Both A and R are true and R explains A
(a) Less than 0.8 (b) Both A and R are true and R does not
(b) Equal to 0.8 explains A
(c) Between 0.8 & 2.0 (c) A is true and R is false
(d) Between 2.0 and 4.3 (d) A is false and R is true

22. The following property of a crystalline 26. At 1000° C the crystallographic structure of
material depends on crystal defects 1ron 1s ____ (GATE -PI-91)
(GATE -PI-90)

!IHIQb§IIIIP@QRflftiM\iiijj� yderabadI Delhi I Bhopal I Pune I BhubaneswarI Lucknow! Patna I Bengalwu I Chennai I Vijayawada I Viz.ag I Tirupati I Kukatpally I Kolkata
27. Increase in ca rbon content in pla in ca rbon (a ) G lass (b) Ca rbon
steel ra ises its (GATE -PI-91) (c) Aluminium (d ) Magnesium
(a ) Ductility and UTS
(b) Tensile strength a nd ma llea bility 32. Electrica l sw itch es mad e out of th ermoset
(c) Tensile strength a nd ha rdness ma terial a re prod uced by (GATE -PI-92)
(d ) Ductility a nd melting point (a ) Compression mold ing
(b) Tra nsfe r mold ing
28. In pow d er meta llurgica l process of (c) Inj ection mold ing
ma nufacturing, max imum tempera ture is a (d ) Vacuum forming
a ssocia ted w ith (GATE -PI-91)
(a) B riquetting (b) Sintering 33. Pow d er meta llurgica l components hav e
(c) Pre-sintering (d ) B lending (GATE -PI-92)
(a ) Sa me density a s that of cast products
29. When 1. 0 percent of ca rbon steel is slow ly (b) High er d ensity than cast products
cooled from molten sta te to 740° C th e (c) Low er density than cast products
resulting structure w ill conta in (d ) High er d ensity tha n forged products
(GATE -PI-91)
(a) Austenite and Ferrite 34. Ma tch th e poly mer w ith th e a ppropriate
(b) Austenite and Cementite product. (GATE -PI-93)
(c) Ferrite a nd Cementite List - I
(d ) Pea rlite a nd Cementite (P) Poly ester resin
(Q) M ethyl methacry la te
30. The percenta ge of ca rbon content in w rough t (R) Polyu retha ne
iron is (GATE -PI-92) (S) Polyv iny l ch loride
(a) less tha n 0. 01
(b) between 0.1 and 1. 0 List - II
(c) between 1.0 a nd 3. 0 (1) M old ed luggage
(d ) b etw een 3 .0 and 5.0 (2 ) Refri gera tor insula ti on
(3) FRP
31. The most w id ely used reinforcement in (4) Contact lenses
modem day FRP (Fibre reinforcement ( 5) Floor tiles
Plastic) tennis racket. (GATE -PI-92)

llfl•ijj§jfj44ijj,MJMni4m.jj..
/Iyderabad I Delhi I Bhopal I Pune I Bhubaneswar I Lucknow I Patna I Bengaluru I Chennai I Vijayawada J Vmg I Tuupati I Kukalpally I Kolkata I
" " ACE . .
'!.�-�:�PnNiranms
. . : 989 : Material Science
� � ======================================
Codes: 38. Inoculating cast iron melt with magnesium
(a) P-4, Q-5, R-2, S-3 gives (GATE -PI-94)
(b) P-3, Q-4, R-1, S-5 (a) Gray cast iron
(c) P-2, Q-1, R-4, S-5 (b) White cast iron
(d) P-4, Q-5, R-1, S-3 (c) Spheroidal graphite iron
(d) Malleable iron
35. Match the following. (GATE -PI-93)
List - I 39. FRP is a (GATE -PI-95)
(P) Strengthening of A/-4% cu alloy (a) Thermoplastic (b) Thermoset
(Q) Improvement in machinability (c) Composite (d) Elastomer
(R) Wear resistance of 0.45% low alloy steel
(S) Redrawing of cold worked copper wires 40. The iron-carbon diagram and the TTT
List - II curves are determined under
(1) Normalizing (GATE -ME-96)
(2) Precipitation hardening (a) equilibrium and non-equilibrium
(3) Process annealing conditions respectively
( 4) Induction hardening (b) non-equilibrium and equilibrium
(5) Tempering conditions respectively
Codes: (c) equilibrium conditions for both
(a) P-4, Q-5, R-2, S-3 (d) non-equilibrium conditions for both.
(b) P-3, Q-2, R-1, S-4
(c) P-2, Q-5, R-4, S-3 41. The iron carbon diagram and the TTT
(d) P-4, Q-5, R-1, S-3 curves are determined under
(GATE-ME-96)
36. Full annealing is a process in which the part (a) Equilibrium and non-equilibrium
can be heated above___ Temperature conditions respectively
and cooled in a turned off furnace. (b) Non-Equilibrium and equilibrium
(GATE -PI-94) conditions respectively
(c) Equilibrium conditions for both
37. The common measures employed to (d) Non-equilibrium conditions for both
quantify ductility are percent elongation and
percent __ Area. (GATE -PI-94)

l1t11i!i§iliiNIIIIYAdddM!iiii.fryderabadI DelhiI BhopalIPuncI BhubaneswarI Lucknowl PatnaI Bcngaluru I Chcnnai !Vuayaw.ida.!Vmg I Tlnlpati I KukatpallyI Kolkala I
: 990: ME GATE Previous solutions

42. In powder metallurgy, the sintering 1s 47. The capacity of a material to absorb energy
carried out in (GATE -PI-01) when deformed elastically, and to release it
(a) Oxidising atmosphere back when unload is termed as
(b) Inert atmosphere (GATE -PI-08)
(c) Reducing atmosphere (a) Toughness (b) Resilience
(d) Air ( c) Ductility (d) Malleability

43. The main purpose of spheroidising treatment 48. Which of the following process is used to
is to improve (GATE -PI-06) manufacture products with controlled
(a) Hardenability of low carbon steels porosity (GATE -PI-08)
(b) Machinability of low carbon steels (a) Casting (b) Welding
( c) Mardenability of high carbon steels (c) Forming ( d) Powder metallurgy
( d) Machinability of high carbon steels
49. In powder metallurgy, sintering of a
44. If a particular Fe-C alloy contains less than component
0.83% carbon, it is called (GATE -PI-07) (a) Improves strength and reduces hardness
(a) high speed steel (b) Reduces brittleness and improves
(b) hypo-eutectoid steel strength
(c) hyper eutectoid steel (c) Improves hardness and reduces
(d) cast iron toughness
(d) Reduces porosity and improves
45. Which one of the following cooling method brittleness
is best suited for converting austenite steel
into very fine pearlite steel (GATE -PI-07) 50. Which one among the following statements
(a) Oil quenching (b) Water quenching are TRUE ?
( c) Air cooling (d) Furnace cooling (a) Thermoplastic polymers have cross
linked chain structure
46. A typical Fe3 C alloy containing greater than (b) Thermosetting polymers have covalent
0.8% C is known as (GATE -PI-08) bonded three dimensional structure
(a) Eutectoid steel ( c) Polyethelene 1s a thermosetting
(b) Hypoeutectoid steel polymer
(c) Hypereutectoid steel ( d) Thermosetting polymers harden on
( d) Mild steel heating and soften on cooling

!li11i!i§idi4jjjjiRflbiMi@j.l� yderabad l Delhi l Bhopal l Pune ! Bhubanes,...;. I Lucknow l Patna l BengaJuru! Chennai! Vtiayawada l Vmg I Tirupati I Kukatpa]lyl Kolkata
: 991 : Material Science

5 1 . Crystallographic structure of austenite is 55. Match the heat treatment processes (Group
(GATE -ME-11) A) and their associated effects on properties
(a) BCC (b) FCC (Group B) of medium carbon steel
(c) CPH (d) Simple cubic
Group A
52. The crystal structure of austenite is (P) Tempering
(GATE-ME-11) (Q) Quenching
(a) body centered cubic (R) Annealing
(b) face centered cubic (S) Normalizing
(c) hexagonal closed packed Group B
(d) body centered tetragonal (I) Strengthening and grain refinement
(II) Inducing toughness
53. During normalizing process of steel, the (III) Hardening
specimen is heated (GATE -ME-12) (IV) Softening (GATE -14-SET-4)

Cod es :
(a) Between the upper and lower critical
temperature and cooled in still air
(b) Above the upper critical temperature (a) P - III , Q - IV, R - II, S - I
and cooled in furnace (b) P - II , Q - III, R - IV, S - I
(c) Above the upper critical temperature (c) P - III , Q - II, R - IV, S - I
and cooled in still air (d) P - II , Q - III, R - I, S - IV
(d) Between the upper and lower critical
temperature and cooled in furnace 56. For a metal alloy, which one of the
following descriptions relates to the stress
54. The process of reheating the martensitic relief annealing process? (GATE -PI-14)
steel to reduce its brittleness without any (a) Heating the work piece material above
significant loss in its hardness is its recrystallization temperature,
(GATE -14-SET-1) soaking and then cooling in still air
(a) Normalizing (b) Heating the workpiece material below
(b) Annealing its recrystallization temperature,
(c) Quenching holding for some time and then furnace
(d) Tempering cooling.

\( I I 11�111t ( 1 1 11� P11ld1t ,llll n h yderabad I Delhi I Bhopal I Punc I Bhubaneswarl Lucknowl Patnal Bcnga)uru I Chcnnai IVuayawadalV17.3g I Tirupati I Kukalpally I Kolkata
"� . "" Fflll"C"l'Jq Pm,li,ramm
. . ME_GATE_Previous solutions
:.t
"
ACE . . : 992:

(c) Heating the workpiece material up to 59. The atomic packing factor for a material
its recrystallization temperature and with body centered cubic structure is __
then rapid cooling. (GATE-15-S et 2)
(d) Heating the workpiece up to its
recrystallization temperatue and 60. The strain hardening exponent n of stainless
cooling to room temperature alternately steel SS 304 with distinct yield and UTS
for a few cycles . values undergoing plastic deformation is
(GATE-15-S et 3)
57. Which one of the following methods is NOT (a) n < 0 (b) n=O
used for producing metal powders? (c) O < n < l (d) n= l
(GATE -PI-1 4)
(a) Atomization 61. The "Jominy test" is used to find
(b) Compaction (GATE-1 6-SET-l)
(c) Machining and grinding (a) Young's modulus
(d) Electrolysis (b) Hardenability
(c) Yield strength
58 . Which of the following types of stress strain (d) Thermal conductivity
relationship best describes the behaviour of
brittle materials, such as ceramics and 62 . In the phase diagram shown in the figure,
thermosetting plastics, (cr stress and i:: = four samples of the same composition are
strain)? (GATE-15-S et 1) heated to temperatures marked by a, b, c and
d
(a)

cr c (b)

cr
� 650

_j
700

crc. crL
600

j
( )
550
g_ 500 a

c
( .-. 450 -+-------+-------+----�
d) 5 10 15
Composition (Arbitrary Units)

\( l I 11t.!,11 1t , 1 1 1 1 1.!, P1 1 l ili, .1111 1 1 1 ..., yderabad J Delhi J Bhopal J Pune J Bhubaneswar J Lucknow J Patna I Bengaluru J Chennai I Vtjayawada I Vizag I Tirupati I Kukatpally J Kolkata
: 993: Material Science

At which temperature will a sample get 68. In powder metallurgy, the process

(GATE-1 6-SET-2) (GATE - PI-1 7)


solutionzed the fastest? 'atomization' refers to a method of

(a) a (b) b (c) C (d) d (a) producing powders


(b) compaction of powders

(GATE - PI-1 6)
63. The elastic modulus of a rigid perfectly (c) sintering of powder compacts
plastic solid is (d) blending of metal powders
(a) 0 (b) 1 (c) 1 00 (d) infinity
69. The ideal stress-strain behavior for a

(GATE - PI-16)
64. Which one of the following is a natural
completely brittle material during tensile
polymer?
testing up to failure is described by
(a) Cellulose (b) Nylon
(c) Polyester (d) Polyvinyl chloride (A) (B)
Stress Stress

(GATE - PI-1 6)
65. In powder metallurgy, sintering of the
component
(a) increases density and reduces ductility Strain

(b) increases porosity and reduces density (C) (D)


(c) increases density and reduces porosity Stress Stress
(d) mcreases porosity and reduces
brittleness
Strain

(GATE - 1 7 - SET-2) (GATE-Pl-1 7)


66. The crystal structure of aluminium is

(a) body -centred cubic


(b) face - centred cubic 70. With reference to Iron-Carbon equilibrium
(c) close - packed hexagonal phase diagram, the crystal structure of 0.3%
plain carbon steel at 1 , 1 00 ° C is
(GATE-PI-1 7)
(d) body - centred tetragonal

67. Given the atomic weight of Fe is 56 and that


(a) HCP (b) BCT
of C is 12, the weight percentage of carbon
(c) BCC (d) FCC

(GATE-1 7-SET-2)
in cementite (Fe3 C) is----

\( I l 1 1�111t l 1 111!.!, P1 1l i\11 ,tl)( )IJ", yderabad I Delhi I Bhopal I Pune I Bhubaneswar I Lu�know I Patna I Bengalwu I Chennai I Vrjayawada I Vttag I Tuupati I Kuk.atpally I Kolkata
: 994 : ME GATE Previous solutions

71. In Glass Fiber Reinforced Plastic (GFRP) 03. Copper-zinc proportion in brasses used for
composites with long fibers, the role of cartridges and drawn tubes is
matrix is to (GATE-ME-91)
(P) Support and transfer the stresses to the (a) 90: I 0 (b) 85: 1 5
fibers (c) 70: 30 (d) 50: 50
(Q) Reduce propagation of cracks
(R) Carry the entire load 04. Poisson's ratio for an incompressible
(S) Protect the fibers against damage material is (GATE-ME-91)
The correct statements are (a) always less than 0.5
(GATE - PI - 17) (b) sometimes less than 0.5
(a) P, Q and R (b) Q, R and S (c) always equal to 0.5
(c) P, Q and S (d) P, R and S (d) never equal to 0.5

05. Nitriding of a steel shaft improves its


(GATE-ME-91)
(a) machinabillity (b) fatigue strength
Two Marks Questions
(c) torsional stiffness (d) surface finish

0 1 . Spheroidal graphite cast iron is produced by 06. Match the terms used in connection with
(GATE- ME-91) heat-treatment of steel with the micro
(a) Spheroidizing gray cast iron structural/physical characteristics.
(b) Tempering white cast iron (GATE-ME-92)
(c) Adding spheroidal graphite to cast iron List - I
melts (P) Pearlite
(d) Inoculating gray cast iron melts (Q) Martensite
(R) Austenite
02. Austempering of steel is done for obtaining (S) Eutectoid
(GATE-ME-91)
List - II
(a) Bainite structure
( 1 ) Extremely hard and brittle phase
(b) Martensitic structure
(2) Cementite is finely dispersed in ferrite
(c) Austenitic structure
(3) Alternate layers of cementite and
(d) Ferritic structure
ferrite
(4) Can exist only above 723 °C
\( I I ll!..!,llH t lllll.!, P11hl u ,1t1cm, �ydcrabad l Delhi l Bhopal l Pune j Bhubaneswa( j Lucknow l Patna ! BcngaJuru j Chcnnai j VliayawadaJYizag I Tirupati I Kukatpally l Kolkata I
: 995: Material Science

(5) Pertaining to state of equilibrium List- II


between three solid phases (1) Surface hardening
(6) Pertaining to state of equilibrium (2) Relieving stresses
between one liquid and two solid (3) Refining grain size
phases (4) Hard and brittle structure.

Codes: Codes:
(a) P-4 Q-6 R-2 S-3 (a) P-3 Q-2 R-4 S-1

(b) P-3 Q-2 R-1 S-4 (b) P-1 Q-3 R-4 S-2

(c) P-2 Q-1 R-4 S-5 (c) P-2 Q-4 R-3 S-1

(d) P-4 Q-5 R-1 S-3 (d) P-2 Q-3 R-4 S-1

07. Assertion(A): Powder metallurgy products 09. Match the following. (GATE-PI-06)
are relatively weaker as compared to List- I
wrought iron products made from the same (P) Degree of polymerization
material. (Q) Polymer degradation
Reason (R): In powder metallurgy, (R) Vulcunisation
controlled atmosphere is necessary during (S) Tempering
sintering (GATE-PI-93) List- II
(a) Both A and R are true and R explains (1) Toughness (2) Melting point
A (3) Stabilizer (4) Sulphur
(b) Both A and R are true and R does not
explains A Codes:

(c) A is true and R is false (a) P-2 Q-3 R-4 S-1

(d) A is false and R is true (b) P-1 Q-3 R-4 S-2


(c) P-2 Q-4 R-1 S-3
08. Match the following . (GATE-PI-06) (d) P-3 Q-2 R-1 S-4

List- I
10. Match the following . (GATE-PI-06)
List- I
(P) Annealing
(Q) Normalising
(P) Composite (Q) Lead
(R) Martempering
(R) Vanadium (S) Titanium oxide
(S) Nitriding

!M•1ii@jjj4UijjjiQjjijlM\jj.jj� yderabad I Delhi I Bhopal I Pune I Bhubaneswarj Lucknow I Patna I Bengaluru J Chennai I Vgayawada J Vizag I Tirupafi I Kukal!;>ailY I Kolk.ala
: 996: ME GATE Previous solutions

List- II the volume fraction of the fibres will be


(1) Grain refinement (GATE-PI-14)
(2) High strength
(3) Crystalline glass 13. Consider a glass-fiber reinforced polymer
(4) Machinability material. The stress-strain curves of the
fiber, matrix and composite are plotted in
Codes:
the figure. Which one of the following
(a) P-2 Q-3 R-4 S-1
statements is TRUE ?
(b) P-1 Q-3 R-4 S-2
(c) P-2 Q-4 R-1 S-3 "'"'
(d) P-3 Q-2 R-4 S-1
·2
11. Match the following (GATE-PI-12)
0
0
·oo
C R
Product �
C

P. Gears Q. Helmets
Engineering Strain
R. Lenses S. Food packing
(GATE - PI -15)
Process
(a) Curve P represents the composite, Curve
1. Polymethylemethacrylate
Q the matrix and Curve R the fiber.
2. Polyamides
(b) Curve Q represents the composite, curve
3. Polyethylene
R the matrix and Curve P the fiber.
4. Acrylonitrile-butadiene-styrene
(c) Curve R represents the composite, curve
Codes:
P the matrix and Curve Q the fiber.
(a) P-3 Q-1 R-4 S-2
(d) Curve P represents the composite, curve
(b) P-3 Q-1 R-2 S-4
R the matrix and Curve Q the fiber.
(c) P-1 Q-4 R-3 S-2
(d) P-4 Q-1 R-2 S-3
14. In a binary system of A and B, a liquid of
20% A (80% B) is coexisting with a solid of
12. Elastic moduli of a fibre reinforced plastic
70% A (30%B). For an overall composition
composite and fibres are 200 GPa and 400
having 40% A, the fraction of solid is
GPa. respectively. The longitudinal fibres
(GATE - 16 - SET-2)
are taking up 50% of the load. Assuming the
(a) 0.40 (b) 0.50 (c) 0.60 (d) 0.75
area fraction equal to the volume fraction,

\() I llgllll'l'lllli.:: P11lil11 ,IIHllJ', �yderabad Delhi


I I Bhopal I Pune I Bhubaneswar I Lucknow I Patna I Benga)wu I Oiennai I Vuayawada I V123g I T1n1pari I Kukatpa)ly I Kolkata I
SOLUTIONS
06. Ans: (b)
One Mark Solutions Sol: By increasing the cooling rate the chances of
formation white cast iron will increase.
01. Ans: (c)
Sol: In case of perfectly plastic material, once the 07. Ans: (c)
material 1s loaded beyond the Sol: Even-though the strength of aluminum is
proportionality limit, the material will low but due to less density, the strength to
experience large amount of deformation. weight ratio is high.

08. Ans: (a)


02. Ans: (b)
Sol: Mild steel means the presence of carbon is
Sol: At 0.8% carbon pure pearlite is forming,
very low that is equal to 0.15%.
below which ferrite and pearlite will be
forming and above which pearlite and 09. Ans: (c)
cementite will forms. Sol: Due to quenching the hardness of the steel
will increase but toughness reduces hence to
03. Ans: (b) increase toughness the tempering will be
Sol: Hardenabilty is defined as the depth upto done after quenching process.
which the required hardness is achieved.
Hence higher hardenebility means that depth 10. Ans: (b)
of hardness obtained higher. Sol: Because the forging die is the tool for
forging process, it should have high
04. Ans: (c) hardness and toughness therefore it has to be
Sol: Generally heat treatment is applicable only hardened and tempered.
for steels due to the presence of austenite,
hence hardness of brass can be improved by 11. Ans: (a)
cold working process. Sol: In shore scleroscope the hardness 1s
measured based on the amount rebounding
05. Ans: (c) of the ball when it is dropped on the
Sol: In case of aluminium, with agemg the material. This is generally used for
hardness will increase. measurement of hardness of very soft
materials like rubber.
!lfli4j@jjji1jjji4RfiniM\jjj@�yderabad I Delhi I Bhopal I Pune I Bhubaneswar I Lucknow I Patna I Bengaluru I Chcnnai I Vtjayawada I Vu.ag ITuupati I Kukatpa))y I Kolkata I
: 998: ME_GATE_Previous Solutions

12. Ans: (b) 18. Ans: (b)


Sol: The commonly used method for producing
powder required for PM process is ball 19. Ans: (d)
mills. Sol: Steel having total alloying element less than
5% is called low alloy steels, 5 to 10% is
13. Ans: (c) medium alloy steels and above 10% is called
Sol: During sintering process, when the product high alloy steels.
is heated to high temp the liquid added in
the mixing process will get evaporate and 20. Ans : (d)
the bond formation will takes place between Sol: Plastics are joined by adhesive bonding
the powder particles. method.

14. Ans: (b) 21. Ans: (a)


Sol: No machining is required on the products Sol: Steels with carbon less than 0.8% is called
produced by powder metallurgy process as hypoeutectoid steels and above 0.8%
because the required shape and size can be carbon is called hypereutectoid steels.
produced directly.
22. Ans: (c)
15. Ans: (b) Sol: Yield strength of material depends on crystal
Sol: Compression molding used for shaping of defects.
thermosetting plastic materials but not for
thermoplastic materials. 23. Ans: (c)
Sol: Plastic sheets are generally produced by
16. Ans: (a & c)
calendaring process.
Sol: House hold buckets and dining table tops
will not be produced by using thermosetting
24. Ans: (c)
plastics because of no reusability of
Sol: The strength of the powder metallurgical
thermosetting plastics.
component after the compaction is called
17. Ans: (d) green strength. This is produced · due to
Sol: Cubic structured metals will have high mechanical interlocking of the powder
ductility hence they can be plastically particles and presence of liquid in the
deformed easily. powder.

!IS11i@jjjiUijjjjRfln5iii1jj� yderabad I Delhi I Bhopal I Pune I Bhubaneswar I Lucknow I Patna I Bengah1ru I Chennai I Vijayawada I V,zag I Tuupati I Kukatpal)y I Kolkala
ACE
�-��=======================================
J
E� J'nhliraiml : 999: Material Science

25. Ans: (c) 31. Ans: (b)


Sol: Thermosetting plastic are gradually being Sol: The density of Carbon fiber reinforced
substituted by thermoplastics for many plastics (CFRP) is less than the glass fiber
engineering applications because reinforced plastics (GFRP), hence the CFRP
thermosetting plastic are not echofriendly or is most common
they are not biodegradable materials and
processing of thermosetting plastics are not 32. Ans: (b)
difficult.
33. Ans: (b)
26. Sol: Due to large amount of compact forces, the
Sol: At 1000°C, the iron will present in the form density of powder metallurgical parts is
of y-iron that is austenite and it has FCC higher.
structure.
34. Ans: (b)
27. Ans: (c) Sol: Polyester resin is used in FRP, polyurethane
Sol: With increase of carbon content, the strength is used for making foam in moded luggage.
and hardness of the steel will increase.
35. Ans: (c)
28. Ans: (b)
Sol: In sintering the powder metallurgical 36. Sol: Above upper critical temp in case of
component is heated to higher temp. hyp o-eutectoid steels and above lower
critical temp in case of hyper-eutectoid
29. Ans: (b) steels .
Sol: Because the 740° C is greater than the 723°C,
structure produced 1s austenite and 37. Sol: Percent Reduction in area
cementite. If it is below 723°C then it
produces pearlite and cementite. 38. Ans: (d)
Sol: Tiny amounts of magnesium or cenum
30. Ans: (a) added to these alloys slow down the growth
Sol: Wrought iron has the carbon content very of graphite precipitates by bonding to the
low which is less than 0.1%. edges of the graphite planes. Along with
careful control of other elements and timing,

\( J I [!�lilt t I Ill� P11lilt( ,Ill< )fl",


Fyderabad I Delhi I Bhopal I Pune I Bhubaneswarl Lucknow! PatnalBengaiuru I Chcrmai !Vtjayawada!Vizag ITuupati I Kukarpa)ly! Kolkata I
" " . ACE
. . . ME_GATE_Previous Solutions
4

� :&,pwrqPwiliratiooa : 1000:
, ......
".

this allows the carbon to separate as 44. Ans: (b)


spheroidal particles as the material Sol: Steels with carbon less than 0.8% is called
solidifies. The properties are similar to as hypo-eutectoid steels and above 0.8%
malleable iron, but parts can be cast with carbon is called hypereutectoid steels.
larger sections.
45. Ans: (c)
39. Ans: (b) Sol: When the steel is cooled in stand-still air
Sol: FRP (Fiber Reinforced Plastics) are always called as normalizing and it produces fine
made by using thermoset plastics only. pearlite.

40. Ans: (a) 46. Ans: (c)


Sol: Fe-C diagram is drawn under equilibrium Sol: Steels with carbon less than 0.8% is called
conditions but T-T-T diagram is drawn as hypoeutectoid steels and above 0.8%
under non-equilibrium conditions. carbon is called hyp ereutectoid steels.

41. Ans: (a) 47. Ans: (b)

42. Ans: (b) 48. Ans: (d)


Sol: The sintering is done in the inert atmosphere Sol: In powder metallurgy process the porosity
to avoid the oxidation of metals at high can be controlled by varying the pressure in
temp. the compacting stage.

43. Ans: (d) 49. Ans: (c)


Sol: In spheroidising heat treatment process, the Sol: By removing liquid m the powder the
cementite in the form of granular (globular) toughness is reduced and by forming the
form is produced in the structure of steel. bond the hardness will increase.
This process causes the agglomeration of all
carbides in the steel in the form of small 50. Ans: (b)
globules or spheroids and this process is
applicable for high carbon steels which will 51. Ans: (b)
improve the machinability of high carbon Sol: When liquid molten metal of iron is cooled
steels. to 1539° C it forms 8-iron having BCC

!11•14h40iiiiUQ@niMhih� yderabad I Dellii I Bhopal I Pune I Bhubaneswar I Lucknow I Patna I Bengaluru I Chennai I Vuayawada I Vizag I Tirupari I Kukatpallyl Kolkata
: 1001: Material Science

structure, on further cooling it converts into In tempering, the component is heated to


y-iron having FCC structure called as below lower critical temperature (low
austenite and on further cooling it get temperature) followed by air cooling,
converts into a-iron having BCC structure produces relatively small size of grains
called as ferrite. compared to annealing. Hence, brittleness is
reduced and little ductility enhances without
52. Ans: (b) significant loss of hardness.
Sol: Liquid molten metal of iron on cooling will
become solid at l 539°C (8 - iron) having 55. Ans: (b)
BCC, on further cooling at 14 1o0c 8- iron Sol: Tempering induces toughness/ ductility

is converted to y - iron called as austinite without loss of hardness


Quenching is a rapid cooling process
having FCC and on further cooling it counts
produces smallest grains and improve
to a - iron called ferrite at 9 10° C.
hardness
Annealing reduces the hardness of the steel
53. Ans: (c)
completely and improve toughness/complete
Sol: During normalizing process, both hypo and
softening process
hyper eutectoid steels are heated to a
Normalizing is obtaining pearlite phase
temperature above upper critical temperature
(ferrite with grain boundary of cementite)
and cooled at stand still air.

56. Ans :(b)


54. Ans: (d)
Sol: Because the stress reliving will be taking
Sol: Normalizing and quenching are used for as
place at below recrystallization temperature
it is produced condition of steel whereas
and annealing means, the metal is cooled in
annealing and tempering 1s used for
furnace cooling.
hardened steels (which 1s m martensite
phase).
57. Ans: (b)
In annealing the component is heated to
Sol: Compaction is the one of the step used in
above upper critical temperature (high
manufacturing of a part with powder
temperature) foll�ed by furnace cooling
metallurgy process.
produces largest size of grains. Hence
achieves highest ductility due to significant
loss of hardness.
liuiNJIJ:1ffww1111$Ufib1J1D1111� yderabadjDelhijBhopaljPunejBh ubaneswar I Lucknow I Patna I BengalurujChennaijVyayawadajVuag jTirupati I Kukalpallyj Kolkata
.----·-···--····----·····-!Z'
: 1002: ME_GATE_Previous Solutions

58. Ans: (d) 62. Ans: (c)


Sol: Ceramics and thermosets are brittle Sol: For aluminium alloy solution hardening
materials which does not undergo plastic process will be used to increase strength and
strain. Therefore the curve will end up to hardness. In this process component will be
elastic point (where stress is proportional to heated to 550°C above temperature so that
strain) and then it fails. solute particles can penetrate into the lattice
easily.
59. Ans: 0.68
Sol: No of atoms in b.c.c unit cell = 2 63. Ans: (d)
atoms Vatom _ 2 ( 4 I 3)m
3
Sol:
APF = N
vcrystal - (4r 1-JJ}
= 0.68 = 68%

60. Ans: (c)


Sol: Formula a = Ks 0, 64. Ans: (a)

a represents the applied stress on the Sol: cellulose is made by C, H,O atoms can be
material, s is the strain, extracted from trees.

65. Ans: (c)


K is the strength coefficient.
The value of the strain hardening exponent

66. Ans: (b)


lies between O and 1.
If n=O means that a material is a perfectly
plastic
If n=1 means it is elastic solid. 67. Ans: 6.67

For SS304 steeV any metal n value between Sol: The percentage of carbon (Fe3C)
0.10 and 0.50. 12
= -----x100=6.67%
56x3+12xl

61. Ans: (b)


68. Ans: (a) 69. Ans: (b)
Sol: The depth upto which the required hardness

70. Ans: (d) 71. Ans: (c)


is obtained is called as hardenability and it is
determined by using jomney end quench
test.

\C I I t1t4"111t t 1111g P11hlu ,llH>II\ �ydcrahadlDelhilBhopallPuncjBhubancswarl LucknowlPatnalBcngaiwulChcnnailVuayawadalVizag ITirupali I Kukatpallyl Kolkat,I


: 1003: Material Science

08. Ans: (d)


Two Marks Solutions Sol: Annealing IS used for relieving stresses,
normalizing used for producing fine grains,
martepering is used for producing matensite
0 1. Ans: (b) and give high brittleness and nitriding is the
surface hardeneing heat treatment process.
02. Ans: (a)
Sol: Austempering heat treatment IS used for 09. Ans: (a)
obtaining bainite structure. Sol: Tempering is for improving toughness.

03. Ans: (c) 04. Ans: (c) 10. Ans: (c)


Sol: Composite has high strength to weight ratio
05. Ans: (b) and lead is used to improves machinability.
Sol: By nitriding heat treatment process, the
composition in the surface of the component 1 1. Ans: (a)
will get change hence the surface will
become harder and interior is softer. 12. Ans 0.25
A
Sol: Let x = volume fraction of fiber = r ,
06. Ans: (c) Ac
Sol: Pearlite is produced with 0.8% carbon as whereAr = area fraction of fiber and
cementite is finely dispersed in ferrite, Ac = total area of composite ,
martensite is very hard and brittle material, E r, Ee, Em = elastic modules of fiber,
austenite exists only above 723°C and composite and matrix respectively
eutectoid is a equilibrium state of 3 solid
A1 soAc =Ar+ Am ,-=--, and -
Ar x
phases. A m 1- x'
Load shared by fiber = 50% means Pt!Pm = 1
07. Ans: (b)
Sol: PM parts are weaker due to high brittleness Based on the same strain induced in the
and controlled atmosphere is required to matrix, fiber and composite
avoid oxidation of metal powder during �=Ar Er =-x- E r l
sintering process. Pm Am Em (1- x) Em
=

� xE f = (1- X)Em
!IJ•l4jj§jj.4Uijj/4Rflni&ijjii!� yderabad I Delhi I Bhopal I Pune I Bhubaneswar I Lucknow I Patna I Bengaluru I Chennai I Vijayawada I Vizag I Tirupati j Kukatpally I K.olkata
: 1004: ME_GATE_Previous Solutions

Also E e = xEr + (1- x)E m WA = 0.2 Wt+ 0.7 (WT- Wt)


200 = xE r + xE r = 2xEr WA = 0.7 WT - 0.5Wt

200 =2x X 400 => X = 200 /800 = 0.25 wA = 0.7-0.5 wt


WT WT

13. Ans: (b) 0.4 = 0.7 - 0.5 (% Wt)


(0.5) (% Wt) = 0.3
14. Ans: (a)
(% Wt) = i5
Sol: Weight of the liquid of A = WLA = la x Wt
= 0.2 x Wt (Where W1 is Weight of (% Ws) = 1- -
3 = 0.4
total liquid)
5

Weight of solid of A
= WsA = Sa (WT- Wt) = 0.7 x (WT- Wt)

jlJ11ijj§jjji4.jjijRflftj@jjjjjj+yderabad I Delhi I Bhopal I Pune I Bhubaneswar I wcknow I Patna I Benga)uru I Chennai I Vtjayawada IVizag I Ttrupari I Kukatpally I Kolkata I
General Aptitude
(Questions)
Page No.1006

CONTENTS
-- - --
- - -�- -
- ---- - - - - --

Chapter f h h - � �
Questions ', Solutions
Name o t- e C apter
l
1

No. Page No. Page No. I


L ----------==-=----=-- --- • - - - --- �
- - -- - --- -- -- - - -
-

01 Numerical Ability 1007 - 1027 1028 - 1057

02 Verbal Ability 1058 - 1080 1081 - 1096


Ct Numerical Ability
04. If (1.001 ) 1 259 = 3.52 and (1.001)2062 = 7.85,
One Mark Questions then (1.001)332 1 = (EC/EE/INST-12)
(a) 2.23 (b) 4.33
01. 25 persons are in a room 15 of them play (c) 11.37 (d) 27.64
hockey 17 of them play football and 10 of
them play both hockey and football. Then 05. The cost function for a product in a firm is
the number of persons playing neither given by 5q2 , where q is the amount of
hockey nor football is: production. The firm can sell the product at a
(ALLBRANCHES-IO) market price of '50 per unit. The number of
(a) 2 (b) 17 (c) 13 (d) 3 units to be produced by the firm such that
the profit is maximized is
02. There are two candidates P and Q in an (ME/CE/CSE/PI-12)
election. During the campaign, 40% of the
(a) 5 (b) 10 (c) 15 (d) 25
voters promised to vote for P, and rest for Q.
However, on the day of election 15% of the
06. In the summer of 2012, in New Delhi, the
voters went back on their promise to vote for
mean temperature of Monday to Wednesday
P and instead voted for Q 25% of the voters
was 41°c and of Tuesday to Thursday was
went back on their promise to vote for Q and
43°C. If the temperature on Thursday was
instead voted for P. Suppose, P lost by
15% higher than that of Monday, then the
2votes, then what was the total number of
temperature in °c on Thursday was
voters? (EC/EE/INST-11)
(EC/EE/INST-13)
(a) 100 (b) 10 (c) 90 (d) 95
(a) 40 (b) 43
(c) 46 (d) 49
03. If Log(P) = (1/2)Log(Q) =( l /3)Log (R) then
which of the following options is TRUE?
07. What will be the maximum sum of 44, 42,
(ME/CE/CSE/PI-11)
(a) p2 = Q2 R2 (b ) Q 2 = P R 40, . . ...? (ME/CSE/PI-13)
(a) 502 (b) 504
(c) Q2 = R3 P (d) R = P2 Q2
(c) 506 (d) 500

l1i•••@jjj4Uijji4pnnftMjj.jj� ydcrabad I Delhi I Bhopal I Punc I Bhubaneswar l Lucknow! Palna I Bcngaluru I Chcnnai I Vijayawada I Vizag ITuupati I Kukatpally I Kolkata
.., ,....:&gioeeriog
�......
.. ACE Pnbliratioos : 1 008: General Aptitude
� ����=�===========��===============�==
08. The statistics of runs scored in a series by four 13. In which of the following options will the
batsmen are provided in the following table. expression P < M be definitely true?
Who is the most consistent batsman of these (GATE-EC/ME-14-Set-3)
four? (a) M < R > P > S (b) M > S < P < F
Batsman Average Standard deviation (c) Q < M < E < P (d) P=A < R < M
K 31.2 5.2 1
L 46.0 6.35 14. Let f(x, y ) = x n y m = P . If x is doubled and
M 54.4 6.22 y is halved, the new value of f is
N 17.9 5.90 (GATE-EC/ME-14-Set-4)
(GATE-EC/ME-14-Set-l) (a) 2 n-m p (b) 2 m-n p
(a) K (b) L (c) M (d) N (c) 2(n - m)P (d) 2(m - n)P

09. What is the next number in the series?


15. In a sequence of 12 consecutive odd
12 35 81 173
357 --- numbers, the sum of the first 5 numbers is
(GATE-EC/ME-14-Set-1)
425. What is the sum of the last 5 numbers
in the sequence? (GATE-EC/ME-14-Set-4)
10. A regular die has six sides with numbers 1 to
6 marked on its sides. If a very large number
16. Ram and Ramesh appeared in an interview
of throws show the following frequencies of
for two vacancies in the same department.
occurrence: 1 � 0. 167; 2�0. 167; 3�0. 152; The probability of Ram's selection is 1/6 and
4�0. 166;5� 0. 168;6�0. 180. We call this that of Ramesh is 1/8. What is the
die (GATE-EC/ME-14-Set-2) probability that only one of them will be
(a) irregular (b) biased selected? (GATE-ME/EC - 15_Set-1)
(c) Gaussian (d) insufficient (a) 47/48 (b) 1/4
(c) 13/48 (d) 35/48
1 1. Fill in the missing number in the series
2 3 6 15 --- 157.5 630 17. An electric bus has onboard instruments that
(GATE-EC/ME-14-Set-2) report the total electricity consumed since
the start of the trip as well as the total
12. The next term in the series 8 1, 54, 36, 24, ...
distance covered . During a single day of
operation, the bus travels on stretches
lS____

(GATE-EC/ME-14-Set-3)
M,N,O and P, in that order. The cumulative
!li11ii@jjj§§.OijQflalMJ11jjj+yderabad l Dellii l Bhopal l Pune l Bhuban� I f..ucknowl Patna l Bengahuu l Chennai lVtiayawada ! Vizag I Tuupati I Kukatpal!yl Kolkata I
.,
,, ,....

......
'!.,.
. �.
:&ipwm,gPnln:alma
" ,
CE
...
. . : 1009 : Nurnerical Ability

distances travelled and the corresponding 20. Five teams have to compete in a league, with
electricity consumption are shown in the every team playing every other team exactly
Table below:- once, before going to next round. How many
Stretch Cumulative Electricity matches will have to be held to complete the
distance (km) used (kWh) league round of matches?
M 20 12 (GATE-ME/INST/Pl - lS_Set-3)

N 45 25 (a) 20 (b) l O (c) 8 (d) 5

0 75 45
p 100 57
2 1. In a huge pile of apples and oranges, both
ripe and unripe mixed together, 15% are
The stretch where the electricity
unripe fruits . Of the unripe fruits, 45% are
consumption per km is minimum is
apples. Of the ripe ones, 66% are oranges. If
(GATE-ME/EC - 15_Set-l)
the pile contains a total of 5692000 fruits,
(a) M (b) N (c) O (d) P
how many of them are apples?
(ME/EC - 16)(Set 1)
18. If x>y>l , which of the following must be
(a) 2029198 (b) 2467482
true?
(c) 2789080 (d) 3577422
(i) /n x > /n y
(ii) ex > eY
22. Michael lives 1 0 km away from where I live.
(iii) yx > xY
Ahmed lives 5km away and Susan lives 7
(iv) COS X > COS y
km away from where I live. Arnn is farther
(GATE-ME/EC - 15_Set-2)
away than Ahmed but closer than Susan
(a) (i) and (ii) (b) (i) and (iii)
from where I live. From the information
(c) (iii) and (iv) (d) (ii) and (iv)
provided here, what is one possible distance
(in km) at which I live from Arun's place?
(ME/EC - 16)(Set1)
19. Find the missing sequence in the letter series
below:
(a) 3.00 (b) 4.99 (c) 6.02 (d) 7.01
A, CD, GHI, ? , UVWXY
(GATE-ME/EC - 15_Set-2)
23. A window is made up of a square portion
(a) LMN (b) MNO and an equilateral triangle portion above it.
(c) MNOP (d) NOPQ
The base of the triangular portion coincides
with the upper side of the square. If the

\( l I l l� l ! l t t ! l i t :._ P1il,l1t . 1 1 1 1 1 1 1-. �ydcrabad I Dcllii I Bhopal I Pune I Bhubaneswar I Lucknow I Patna I Bengaluru I Chennai I Vuayawada I Vu.ag I Tllllpati I Kukatpal)y I Kolkata I
perimeter of the window is 6m, the area of 27. The sum of the digits of a two digit number
the window in m2 is is 12. If the new number formed by
(ME - 16) (Set 2) reversing the digits is greater than the
(a) 1.43 (b) 2.06 original number by 54, find the original
(c) 2.68 (d) 2.88 number. (GATE - PI - 16)
(a) 39 (b) 57 (c) 66 (d) 93
24. Given (9 inches) 112 = (0.25 yards) 112 , which
one of the following statements is TRUE? 28. In the summer, water consumption is known
(ME/EC - 16)(Set 3) to decrease overall by 25%. A water Board
(a) 3 inches = 0.5 yards official states that in the summer household
(b) 9 inches = 1.5 yards consumption decreases by 20% while other
(c) 9 inches = 0.25 yards consumption increases by 70%.
(d) 81 inches = 0.0625 yards (GATE-PI - 17)
Which of the following statements is
25. S, M, E and F are working in shifts in a term correct?
to finish a project. M works with twice the (a) The ratio of household to other
efficiency of others but for half as many . . 8
days as E worked. S and M have 6 hour
consumpt10n 1s -
17
shifts in a day, whereas E and F have 12 (b) The ratio of household to other
hours shifts. What is the ratio of contribution . . 1
of M to contribution of E in the project?
consumption 1s -
17
(ME/EC - 16)(Set 3) (c) The ratio of household to other
(a) l :1 (b) 1 :2 . . 17
consumption 1s -
(c) 1:4 (d) 2:1 8
(d) There are errors m the officials
26. (x% ofy) + (y¾ of x) is equivalent to __ statement.
(GATE - PI - 16)
(a) 2% of xy 29. 40% of deaths on city roads may be
(b) 2% of (xy/100) attributed to drunken driving. The number of
(c) xy% of 100 degrees needed to represent this as a slice of
(d) 100% of xy a pie chart is (GATE-PI - 17)
(a) 120 (b) 144 (c) 160 (d) 212

\( l I 11l!,111t t 1 1 11� P11!1lu .111,.,i-. �yderabad l Dclhi l Bhopa! J Punc J BhubaneswarJ Lucknow I PatnaJ Bcngalurul Chcnnai JVgayawadal Vu.ag JT=pati I KukatpallyJ Kolkata I
30. Some tables are shelves. Some shelves are (a) Rs. 20,000 (b) Rs. 30,000
chairs. All chairs are benches. Which of the (c) Rs. 32,300 ( d) Rs. 40,000
following conclusions can be deduced from
the preceding sentences? 33. P, Q and R talk about S ' s car collection. P
(i) At least one bench is a table states that S has at least 3 cars. Q believes
(ii) At least one shelf is a bench that S has less than 3 cars. R indicates that
(iii)At least one chair is a table to his knowledge, S has at least one car.
(iv)All benches are chairs Only one of P, Q and R is right. The number
(GATE-PI - 17) of cars owned by S is
(a) Only I (b) Only ii (GATE-ME - 17)(SET -l)
( c) Only ii and iii (d) Only iv (a) 0 (b) 1
(c) 3 (d) Cannot be determined
31. A right-angled cone (with base radius 5 cm
and height 12 cm), as shown in the figure 34. P looks at Q while Q looks at R. P is
below, is rolled on the ground keeping the married, R is not. The number of pairs of
point P fixed until the point Q (at the base of people in which a married person is looking
the cone, as shown) touches the ground at an unmarried person is
agam. (GATE-ME - 17)(SET -2}
(a) 0 (b) 1
(c) 2 (d) Cannot be determined
?

35. If a and b are integers and a - b is even,


which of the following must always be
Ground

By what angle (in radians) about P does the even? (GATE-ME - 17)(SET -2)
cone travel? (GATE-ME - 17)(SET -1} (a) ab (b) a2 + b2+ l
Sn Sn 24n 10n (c) a2 + b+ l (d) ab - b
(a) (b) (c) (d)
12 24 5 13

36. A couple has 2 children. The probability


32. In a company with 100 employees, 45 earn
that both children are boys if the older one is
Rs. 20,000 per month, 25 earn Rs. 30,000,
a boy is (GATE-ME - 17)(SET -2)
20 earn Rs. 40,000, 8 earn Rs. 60,000, and 2
(a) 1/4 (b) 1/3 (c) 1/2 (d) 1
earn Rs. 1,50,000. The median of the salaries
lS (GATE-ME - 17)(SET -1)
Jl"S•li!@jjj§i.jji4Rflij5IO,jj� 'yderabad I Delhi I Bhopal I Pune I Bhubaneswar I Lucknow I Patna I Bengaluru I Chennai I Vrjayawada I V,zag I Tirupari I Kukatpally I Kolkata
�•EJL-n : 1012: General Aptitude
� � ==================================
civilian populations. Chemical agents that do
Two Marks Questions their work silently appear to be suited to
such warfare; and regretfully, there exist
01. Hari (H), Gita (G), lrfan (I) and Saira (S) are people in military establishments who think
siblings (i,e. brothers and sisters). All were that chemical agents are useful tools for their
born on 1st January. The age difference cause. (ALLBRANCHES-10)
between any two successive siblings (that is
born one after another is less than 3 years.
Which of the following statements best sums
(ALLBRANCHES-10)
up the meaning of the above passage:
Given following facts: (a) Modem warfare has resulted in civil
(i) Hari's age + Gita's age > lrfan's age + strife
Saira's age (b) Chemical agents are useful in modem
(ii) The age difference between Gita and warfare
Saira is 1 year. However, Gita is not the (c) Use of chemical agents in warfare
..oldest and saira is not the youngest would be undesirable
(iii) There are no twins (d) People in military establishments like to
In what order were they born (oldest first)? use chemical agents in war
(a) HSIG (b) SGHI
(c) IGSH (d) IHSG 04. Given digits 2,2,3,3,3,4,4,4,4 how many
distinct 4digit numbers greater than 3000 can
02. 5 Skilled workers can build a wall in 20 be formed (ALLBRANCHES-10)
days, 8 semi-skilled workers can build a wall (a) 50 (b) 51 (c) 52 (d) 54
in 25 days; 10 unskilled workers can build a
wall in 30 days. If a team has 2 skilled 6 05. If 137+276=435, how much is 731 +672=1
semi-skilled and 5 unskilled workers, how
(ALLBRANCHES-10)
long will it take to build the wall?
(a) 534 (b) 1403 (c) 1623 (d) 1513
(ALLBRANCHES-10)
(a) 20 days (b) 18 days
06. There friends, R, S and T shared toffee from
(c) 16 days (d) 15 days
a bowl. R took 113rd of the toffees, but
returned four to the bowl. S took 114th of
03. Modem warfare has changed from large
what was left but returned three toffees to
scale clashes of armies to suppression of
the bowl. T took half of the remainder but
�yderabad J Dclhi J Bhopal J Pune l Bhubanes�,..I Lucknowl Patnal Bengaiuru J ChennaiJVuayawadalVizag JT11Upari I Kuka!pa)Jyj Kolkata I
: 1013 : Numerical Ability

returned two back into the bowl. If the bowl 10 . The fuel consumed by a motorcycle during a
had 17 toffees left, how many toffees were journey while traveling at various speeds is
originally there in the bowl? indicated in the graph below.
(EC/EE/INST-11) 120
(a) 38 (a) 31 (c) 48 (d) 41

07. Given that f(y) = I YI / y , and q is any non­ •

zero real number, the value of I f { q) - f { -q) I



lS (EC/EE/INST-11)

(a) 0 (b) - 1 (c) 1 (d) 2 0 15 30 45 60 7 5 90


08. The sum of n terms of the series
Speed

4+44+444+ ... .is


(kilometers per hour)

(EC/EE/INST-11)
The distances covered during four laps of the
(a) (4/81) ( l o -9n - 1]
n+ l
journey are listed in the table below
(b) (4/81) ( 10°- 1 -9n - 1]
(c) (4/81) ( l On+ l -9n - 10] Distance Average speed
Lap
(d) (4/81) [ 10° -9n - 10] (kilometers) (kilometers per hour)

15 15
09. The horse has played a little-known but very
p
75 45
important role in the field of medicine.
Q
40 10
Horses were injected with toxins of diseases
R

until their blood built up immunities. Then a


s 10 10

serum was made from their blood. Serums to From the given data, we can conclude that
fight with diphtheria and tetanus were the fuel consumed per kilometer was least
developed this way. It can be inferred from during the lap (EC/EE/INST-11)
the passage, that horses were (a) p (b) Q (c) R (d) s
(EC/EE/INST-11)
(a) given immunity to diseases 1 1 . A transporter receives the same number of
(b) generally quite immune to diseases orders each day . Currently, he has some
(c) given medicines to fight toxins pending orders (backlog) to be shipped. If
(d) given diphtheria and tetanus serums he uses 7 trucks, then at the end of the 4th
�ydcrabad I Delhi I Bhopal I Pune I Bhubaneswar I. hickn9� Patna I Bengaluru I Chennai I Vtjayawada I Vizag I Tuupari I Kukatpally I Kolkata I
'!i
:.t . .t\CE . .
��Pnhticmoos : 1014 : General Aptitude
� � =========================================
day he can clear all the orders. Alternatively, (a) p (b) Q (c) R (d) s
if he uses only 3 trucks, then all the orders
13. A container originally contains1 Olitres of
are cleared at the end of the 10th day. What
pure spirit. From this container 1 litre of
is the minimum number of trucks required so
spirit is replaced with 1 litre of water,
that there will be no pending order at the end
(ME/CE/CSE/Pl-11)
Subsequently, l litre of the mixture is again
of the 5th day.
replaced with I litre of water and this process
(a) 4 (b) 5 (c) 6 (d) 7
is repeated one more time. How much spirit
12. P, Q, R and S are four types of dangerous is now left in the container?
microbes recently found in a human habitat. (ME/CE/CSE/PI-11)
The area of each circle with its diameter
14. The variable cost (V) of manufacturing a
printed in brackets represents the growth of a
product varies according to the equation
single microbe surviving human immunity
V = 4q, where q is the quantity produced.
system within 24 hours of entering the body.
The fixed cost (F) of production of same
The danger to human beings vanes
product reduces with q according to the
proportionately with the toxicity, potency
equation F =100/q. How many units should
and growth attributed to a microbe shown in
(ME/CE/CSE/Pl-11)
be produced to minimize the total cost (V+F)
(ME/CE/CSE/PI-11)
the figure below:

15. Few school curricula include a unit on how


to deal with bereavement and grief, and
yet all students at some point in their lives
suffer from losses through death and parting.
Based on the above passage which topic
would not be included in a unit on
bereavement? (ME/CE/CSE/PI-11)
Potency (a) how to write a letter of condolence
(Probability that microbe will overcome (b) what emotional stages are passed
human immunity system) through in the healing process
A pharmaceutical company is contemplating (c) what the leading causes of death are
the development of a vaccine against the (d) how to give support to a grieving friend
most dangerous. Which microbe should the
company target in its first attempt?
!M11ih§jjji4@NRflCilmjjj� yderabad I Delhi I Bhopal I Pune I Bhuban� I.m:knowl Patna! Bengaluru I Chennai !Vuayawada!Vizag ITirupati I K�� Kolkata
16. The data given in the following table 19. There are eight bags of rice looking alike,
summarizes the monthly budget of an seven of which have equal weight and one is
average household. slightly heavier. The weighing balance is of
unlimited capacity. Using this balance, the
Category Amount (Rs.)
minimum number of weighings required to
Food 4000
identify the heavier bag is
Clothing 1200
(EC/EE/INST-12)
Rent 2000
Savings 1500 (a) 2 (b) 3 (c) 4 (d) 8
Other expenses 1800
20. One of the legacies of the Roman legions
The approximate percentage of the monthly was discipline. In the legions, military law
budget NOT spent on savings is prevailed and discipline was brutal.
(EC/EE/INST-12) Discipline on the battlefield kept units
(a) 10% (b) 14% (c) 81% (d) 86% obedient, intact and fighting, even when
the odds and conditions were against
17. A and B are friends. They decide to meet them.
between 1 PM and 2 PM on a given day. Which one of the following statements best
There is a condition that whoever arrives sums up the meaning of the above passage?
first will not wait for the other for more than (EC/EE/INST-12)
15 minutes. The probability that they will (a) Thorough regimentation was the main
meet on that day is (EC/EE/INST-12) reason for the efficiency of the Roman
(a) 1 /4 (b) 1/16 legions even in adverse circumstances.
(c) 7/16 (d) 9/16 (b) The legions were treated inhumanly as
if the men were animals.
18. Raju has 14 currency notes in his pocket (c) Discipline was the armies' inheritance
consisting of only Rs. 20 notes and Rs. from their seniors.
1Onotes. The total money value of the notes (d) The harsh discipline to which the
is Rs. 230. The number of Rs. 10 notes that legions were subjected to led to the
Raju has is (EC/EE/INST-12) odds and conditions being against them.

(a) 5 (b) 6 (c) 9 (d) 10

\( I l 1 1....,::H t r 11 1!.!, Pt1hl1t ,llJ<Jri, �yderabad I Delhi I Bhopal I Pune I Bhubaneswar I �owl Patnal Bengaluru I Chennai I VtjayawadaJ Vu.ag I T=pati I Kukalpally I KolkataI
: 1016 : General Aptitude

2 1. An automobile plant contracted to buy shock 24. A political party orders an arch for the
absorbers from two suppliers X and Y, X entrance to the ground in which the annual
supplies 60% and Y supplies 40% of the convention is being held. The profile of the
shock absorbers. All shock absorbers are arch follows the equation y= 2x-O.l x2 where
subjected to a quality test. The ones that pass y is the height of the arch in meters. The
the quality test are considered reliable. Of maximum possible height of the arch is
X's shock absorbers, 96% are reliable. Of (ME/CE/CSE/PI-12)
Y's shock absorbers, 72% are reliable. (a) 8 meters (b) 10 meters
The probability that a randomly chosen (c) 12 meters (d) 14 meters
shock absorber, which is found to be
reliable, is made by Y is 25. What is the chance that a leap year, selected

(ME/CE/CSE/PI-12) at random, will contain 53 Saturdays?


(a) 0.288 (b) 0.334 (EC/EE/INST-13)
(c) 0.667 (d) 0.720 (a) 2/7 (b) 3/7 (c) 1/7 (d) 5/7

26. The set of values of p for which the roots of


22. Which of the following assertions are the equation 3x2 + 2x + p(p- 1) = 0 are of
CORRECT? opposite sign is (EC/EE/INST-13)
P: Adding 7 to each entry in a list adds 7 (a) (-co, 0) (b) (0, 1)
to the mean of the list
(c) ( 1, co) (d) (0, co)
Q: Adding 7 to each entry in a list adds 7
to the standard deviation of the list 27. A car travels 8 km in the first quarter of an
R: Doubling each entry in a list doubles hour, 6 km in the second quarter and 16 km
the mean of the list in the third quarter. The average speed of
S: Doubling each entry in a list leaves the the can in km per hour over the entire
standard deviation of the list Journey 1s (EC/EE/INST-13)
. .

unchanged (ME/CE/CSE/PI-12) (a) 30 (b) 36 (c) 40 (d) 24

(a) P,Q (b) Q,R (c) P,R (d) R.S 28. Find the sum to n terms of the series
10+ 84+ 734+ . ....... (EC/EE/INST-13)
23. Given the sequence of terms, AD CG FK JP,
( + 1) 9 (9 n - 1)
(a) 9 9
n

the next term is (ME/CE/CSE/PI-12) +1 (b) +1


10 8
(a) OV (b) OW (c) PV (d) PW
9(9 - 1) Il ( ° - 1)
(d) 9 9
°
(C) ------'-+ + n2
8
\( 'l I 11g11a t 1 111g P11hli( .111011 .... �yderabad J Delhi J Bhopal J Punc J Bhubancswar[ I..ucknowJ Patna J Bcngaluru J Chennai J VijayawadaJ Vizag J Tirupari J Kukatpally J Kolkata I
.,
':....
. ACE
� �...."��PnNiraioos
..,, . . . : l017 : Nwnerical Ability

29. Out of all the 2-digit integers between 1 and 33. Find the odd one from the following group:
100, a 2-digit number has to be selected at W,E,K,O I,Q,W,A
random. What is the probability that the F,N,T,X N,V,B,D
selected number is not divisible by 7? (GATE-EC/ME-14-Set-l )
(ME/CSE/PI-13) (a) W,E,K,O (b) 1,Q,W,A
(a) 13/90 (b) 12/90 (c) F,N,T,X (d) N,V,B,D
(c) 78/90 (d) 77/90
34. For submitting tax returns, all resident males
30. A tourist covers half of his journey by train with annual income below Rs 10 lakh should
at 60 Km/h, half of the remainder by bus at fill up Form P and all resident females with
30 km/h and the rest by cycle at 10 km/h. income below Rs 8 lakh should fill up From
The average speed of the tourist in km/h Q. All people with incomes above Rs 10
lakh should fill up Form R, except non
during his entire journey is
(ME/CSE/PI-13)
residents with income above Rs15 lakhs,
who should fill up Form S. All other should
(a) 36 (b) 30 (c) 24 (d) 18 fill Form T. An example of a person who
should fill Form T is
31. Find the sum of the expression (GATE-EC/ME-14-Set-1)
I I I I
(a) a resident male with annual income Rs
.[i + ..fi +
..fi + .Ji .Ji + .[4
+ +
......
+
Jw + .Jsj
(ME/CSE/Pl-13)
9 lakh
(b) a resident female with annual income
(a) 7 (b) 8 (c) 9 (d) 1 0 Rs 9 lakh
(c) a non-resident male with annual
32. The current erection cost of a structure is Rs. income Rs 16 lakh
13,200. If the labour wages per day increase (d) a non-resident female with annual
by 1/5 of the current wages and the working income Rs 16 lakh
hours decrease by 1/24 of the current period,
then the new cost of erection in Rs. is
(ME/CSE/PI-13)
35. A train that is 280metres long traveling at a
uniform speed, crosses a platform in 60sec
(a) 16,500 (b) 15,180 and passes a man standing on the platform in
(c) 1 1 ,000 (d) 1 0, 1 20
20sec. what is the length of the platform in
metres? (GATE-EC/ME-14-Set-l)

!@1ii@Oii4i!i1Nflldiii@.Hyderabadl Delhi I Bhopal I Pune I Bhubanell\Wl"I LJ.JCknowl Patna! Benga)uru I Chennai I Vtjayawadaj Vizag I Tirupati I I
Kukalpallyj Kolkala
: 1018 : General Aptitude

36. The exports and imports (in crores of Rs.) of (a) Q, W, Z, B (b) B, H, K, M
a country from 2000 to 2007 are given in the (c) W, C, G, J (d) M, S, V, X
following bar chart. If the trade deficit is
defined as excess of imports over exports, in 39. The sum of eight consecutive odd numbers
which year is the trade deficit 115th of the is 656. The average of four consecutive even
exports? (GATE-EC/ME-14-Set-1) numbers is 87. What is the sum of the
smallest odd number and second largest even
number? (GATE-EC/ME-14-Set-2)
120

100
40. The total exports and revenues from the
exports of a country are given in the two
80

charts shown below. The pie chart for


60

exports shows the quantity of exported as a


40

percentage of the total quantity of exports.


20

0
The pie chart for the revenues shows the
percentage of the total revenue generated
(a) 2005 (b) 2004 through export of each item. The total
(c) 2007 (d) 2006 quantity of exports of all the items is 500
thousand tones and the total revenues are
37. You are given three coins: one has heads on 250 crore rupees. Which item among the
both faces, the second has tails on both following has generated the maximum
faces, and the third has a head on one face revenue per kg? (GATE-EC/ME-14-Set-2)
and a tail on the other. You choose a coin at
Exports Revenues
random and toss it, and it comes up heads.
The probability that the other face is tails is
(GATE-EC/ME-14-Set-l)
(a) 1/4 (b) 1/3 (c) 1/2 (d) 2/3
Item 3
38. Find the odd one in the following group 22% 19%
Q, W, Z, B B, H, K, M
W, C, G, J M,S, V,X (a) Item 2 (b) Item 3
(GATE-EC/ME-14-Set-2) (c) Item 6 (d) Item 5

\( l J 1 1..., l l l l ( I 1111.,: 1'11lilll ,11Jt 111, Fydcrabad j Delhi j Bhopa) I Punc l Bhubaneswarl Lucknowj Patna j BcngaJwu l Cbcnnai l VuayawadajVizag I T1111pali I Kukatpallyl Kolkala I
... ....... ACE
.,.
�.. �5� P.iNirariooa : 1019 : Numerical Ability
===================================
41. It takes 30 minutes to empty a half-full tank (GATE-EC/ME-14-Set-3)
by draining it at a constant rate. It is decided (i) Butterflies are birds
to simultaneously pump water into the half­ (ii) There are more tigers in this forest than
full tank while draining it. What is the rate at red ants.
which water has to be pumped in so that it
(iii) All reptiles m this forest are either
gets fully filled in 10 minutes?
(GATE-EC/ME-14-Set-2)
snakes or crocodiles.
(iv) Elephants are the largest mammals m
(a) 4 times the draining rate
(b) 3 times the draining rate this forest.
(c) 2.5 times the draining rate (a) (i) and (ii) only
(d) 2 times the draining rate (b) (i), (ii), (iii) and (iv)
(c) (i), (iii) and (iv) only
42. Find the next term in the sequence. 7G, 1 l K, (d) (i), (ii) and (iii) only
13M, ____
(GATE-EC/ME-14-Set-3) 44. A man can row at 8 km per hour in still
(a) 15Q (b) 17Q (c) 15P (d) 17P water. If it takes him thrice as long to row
upstream, as to row downstream, then find
43. The multi-level hierarchical pie chart shows the stream velocity in km per hour.
the population of animals in a reserve forest. (GATE-EC/ME-14-Set-3)
The correct conclusions from this
information are: 45. A firm producing air purifiers sold 200 units
in 2012. The following pie chart presents the
share of raw material, labour, energy, plant
& machinery, and transportation costs in the
total manufacturing cost of the firm in 2012.
The expenditure on labour in 2012 is Rs.
4,50,000. In 2013, the raw material expenses
increased by 30% and all other expenses
increased by 20%. If the company registered
a profit of Rs. 10 lakhs in 2012, at what price
(in Rs.) was each air purifier sold?

!Iii1ih§jji§4ijjjiRflblM\iljjj� yderabad I Delhi I Bhopal I Pune I Bhubaneswar I Lucknow I Patna I Bengaluru I Chennai IVuayawada I VIZII! I Tirupari I Kukaipally I Kolkata
•t" • ACE : 1020: General Aptitude
�.. .... &,pw,,t:q Pobliramm
u • • •

49. Industrial consumption of power doubled


from 2000-2001 to 2010-20 11. Find the
Plant and Machinery

annual rate of increase in percent assuming it


30%

to be uniform over the years.


(GATE-EC/ME-14-Set-4)
(a) 5.6 (b) 7.2
ransportation 1 0% Raw
Material, 20%
Labour, 1 5%
(c) 10.0 (d) 12.2

(GATE-EC/ME-14-Set-3) 50. A firm producing air purifiers sold 200 units


in 2012. The following pie chart presents the
46. A batch of one hundred bulbs is inspected by share of raw material, labour, energy, plant
testing four randomly chosen bulbs. The & machinery, and transportation costs in the
batch is rejected if even one of the bulbs is total manufacturing cost of the firm in 2012.
defective. A Batch typically has five The expending on labour in 2012 is Rs.
defective bulbs. The probability that the 4,50,000. In 2013, the raw material expenses
current batch is accepted is___ increased by 30% and all other expenses
(GATE-EC/ME-14-Set-3) increased by 20%. What is the percentage
mcrease in total cost for the company in
47. Find the next term in the sequence: 13M, 2013?
17Q, 19S,
(GATE-EC/ME-1 4-Set-4)
(a) 21W (b) 21V
(c) 23W (d) 23V

48. If 'KCLFTSB' stands for 'best of luck ' and


'SHSWDG ' stands for 'good wishes ' , which
of the following indicates 'ace the exam'?
(GATE-EC/ME-14-Set-4)
(a) MCHTX (b) MXHTC
(c) XMHCT (d) XMHTC (GATE-EC/ME-14-Set-4)

!MM•i@jjjl4ljj4Pdftl@jj.jj4 yderabad I Delhi I Bhopal I Pune I Bhubaneswar I Lucknow! Patna I Bengaluru I Chennai I Vijayawada I Vu.ag I Tirupari I Kukalpal)y I Kolkata
.,... . .
, ......." ACE
':. ,.� �Niliratims
. . : 1021 : Numerical Ability

51. A five digit number is formed using the above information, which one of the
digits 1,3,5,7 and 9 without repeating any of following is TRUE?
them. What is the sum of all such possible (GATE-ME/INST/PI - lS_Set-3)
five digit numbers? (a) X and Y are not independent
(GATE-EC/ME-14-Set-4) (b) Y and Z are dependent
(a) 6666660 (b) 6666600 (c) Y and Z are independent
(c) 6666666 (d) 6666606 (d) X and Z are independent

52. Given below are two statements followed by


55. Given below are two statements followed by
two conclusion. Assuming these statements
two conclusions. Assuming these statements
to be true, decide which one logically
to be true, decide which one logically
follows. Statement:
follows.
Statements: I. No manager is a leader.
I. All film directors are playback singers. II. All leaders are executives.
II. All film directors are film stars. Conclusions:
Conclusion : I. No manager is an executive.
I. All film directors are playback singers. II. No executive is a manager.
II. Some film stars are film directors. (GATE-ME/INST/Pl - lS_Set-3)
(GATE-ME/EC - lS_Set-1) (a) Only conclusion I follows.
(a) Only conclusion I follows. (b) Only conclusion II follows.
(b) Only conclusion II follows. (c) Neither conclusion I nor II follows.
(c) Neither conclusion I nor II follows. (d) Both conclusions I and II follow.
(d) Both conclusions I and II follow.
56. In the given figure angle Q is a right angle,
53. log tanl O + log tan 2° + ......... log tan 89°
1s. .. . .. (GATE-ME/EC - lS_Set-2)
PS:QS = 3:1, RT:QT = 5:2 and PU:UR= l : 1
If area of triangle QTS is 20cm2 , then the
(a) 1 (b) 1/i (c) 0 (d) -1 area triangle PQR in cm2 is___
R
54. A coin is tossed thrice. Let X be the event
that head occurs in each of the first two
tosses. Let Y be the event that a tail occurs
P ------�
on the third toss. Let Z be the event that two
S Q
(GATE-ME/INST/PI - lS_Set-3)
tails occur in three tosses. Based on the
\( I I llj..;lllt t I Ill� Ptihlt{ ,llllll\', yderabad I DelhiJ Bhopal ! Punc I Bhubancswar J Lucknow I Patna I Bengaluru I Chcnnai J VtjayawadaJ Vizag J Tuupati I Kukatpally J Kolkatl
...
� ,....
... ACE
���
·ru
�·�·�========�:�����:==========�������p��
General A u de
·-

&�,gm
..EE= :·���1Nirarims
:erq
1 0 22
�..

57. A person moving through a tuberculosis evaluated out of 200 marks. It was observed
prone zone has a 50% probability of that the mean of group P was 105, while that
becoming infected. However, only 30% of of group Q was 85. The standard deviation
infected people develop the disease. What of group P was 25, while that of group Q
percentage of people moving through a was 5. Assuming that the marks were
tuberculosis prone zone remains infected but distributed on a normal distribution, which
does not shows symptoms of disease? of the following statements will have the
(ME/EC - 16)(Set1) highest probability of being TRUE?
(a) 15 (b) 33 (c) 35 (d) 37 (ME - 16) (Set2)
(a) No student in group Q scored less
1 1 1 marks than any student in group P.
58. If q-a - and r-b = - and s-C = - , the
r s q (b) No student in group P scored less marks
-

value of abc is __. (ME/EC - 16)(Set1) than any student in group Q.


(a) (rqsr 1 (b) 0 (c) Most students of group Q scored marks
(c) 1 (d) r+ q+s in a narrower range than students in
group P.
59. P, Q, R and S are working on a project. (d) The median of the marks of group P is
Q can finish the task in 25 days, working 100.
alone for 12 hours a day. R can finish the
task in 50 days, working alone for 12 hours 61. Find the missing sequence in the letter series

per day. Q worked 12 hours a day but took B,FH,LNP,-----­ (ME - 16) (Set2)
sick leave in the beginning for two days. R (a) SUWY (b) TUVW
worked 18 hours a day on all days. What is (c) TVXZ (d) TWXZ
the ratio of work done by Q and R after 7
62. The binary operation O is defined as a Ob =
days from the start of the project?
(ME/EC - 16)(Set1) ab + (a + b), where a and b are any two real
(a) 10: 11 (b) l l : 10 numbers. The value of the identity element
(c) 20:21 (d) 21 :20 of this operation, defined as the number x
such that a Ox = a, for any a, is _.
60. Students taking an exam are divided into two
groups, P and Q such that each group has the (ME - 16) (Set2)
same number of students. The performance (a) 0 (b) l (c) 2 (d) 10
of each of the students in a test was
!IJjl4jj§jjji4@iRfl§j@ufjj� yderabadl Delhi I Bhopal I Punel Bhuban�warl Luclmowl Patna I Bengaluru I Chennai I VtiayawadaJ Vmg ITirupati I K�y l Kolk.ata
: 1023 : Numerical Ability

63. Which of the following curves represents the 64. The Venn diagram shows the preference of
the student population for leisure activities.
function y = ln(le�sinijxJ�ll) forlxl < 21t ? Here, x
(ME - 16)(Set 3)
represents the abscissa and y represents the
Read books Watch TV
ordinate. (ME - 16) (Set 2)

(A)

Play sports

From the data given, the number of students


who like to read books or play sports is _.
(a) 44 (b) 51 (c) 79 (d) 1 08
(B) 65. Two and a quarter hours back, when seen in
a mirror, the reflection of a wall clock
without number markings seemed to show
1 :30. What is the actual current time shown
by the clock? (ME - 16)(Set 3)
(a) 8: 1 5 (b) 1 1 : 1 5 (c) 1 2: 1 5 (d) 1 2:45
(C)
66. M and N start from the same location. M
travels 1 0 km East and then 1 0 km North­
East. N travels 5 km South and then 4 km
South-East. What is the shortest distance (in
km) between M and N at the end of their
travel? (ME - 16)(Set 3)
-27t

(D)
(a) 18.60 (b) 22.50 (c) 20.61 (d) 25.00

67. A wire of length 340 mm is to be cut into


two parts. One of the parts is to be made into
a square and the other into a rectangle where
sides are in the ratio of 1 :2. What is the
-lt 0 7t length of the side of the square (in mm) such
\( l I 1 1 .... 1 1 1 1 1 r 1 1 1 � P1 il d11 . 1 1 1 t 1 1 1 " �ydcrabad J Dclhi J Bhopal J Punc J BhubaneswarJ LuclmowJ Patna J Bcngaluru I Chcnnai J Vtjayawada J Vizag J T1n1pati J Kukatpally J Kolkala I
, ' , ",." . ACE
.:.
.., . . . : 1 024: General Aptitude
':, l�PnNionoaa
that the combined area of the square and the (a) II only (b) III only
rectangle is a MINIMUM? (c) I, II and III (d) I only
(ME - 16)(Set 3)
(a) 30 (b) 40 (c) 120 (d) 180 70. A square pyramid has a base perimeter x,
and the slant height is half of the perimeter.
68. Two finance companies, P and Q, declared What is the lateral surface area of the
fixed annual rates of interest on the amounts pyramid? (GATE - PI - 16)
invested with them. The rates of interest (a) x 2
(b) 0.75 x2
offered by these companies may differ from (c) 0.50 x2 (d) 0.25 x2
year to year. Year-wise annual rates of
interest offered by these companies are 71. Ananth takes 6 hours and Bharath takes 4
shown by the line graph provided below. hours to read a book. Both started reading
copies of the book at the same time. After
-- p how many hours is the number of pages to
be read by Ananth, twice that to be read by
----,- Q

Bharath? Assume Ananth and Bharath read


all the pages with constant pace.
I I I
2000 2001 2002 2003 2004 2005 2006

If the amounts invested in the companies, P (GATE - PI - 16)


and Q, in 2006 are in the ratio 8: 9, then the (a) 1 (b) 2 (c) 3 (d) 4
amounts received after one year as interests
from companies P and Q would be in the 72. S, T, U, V, W, X, Y, and Z are seated
ratio: (GATE - PI - 16) around a circular table. T's neighbours are
(a) 2:3 (b) 3:4 (c) 6:7 (d) 4:3 Y and V. Z is seated third to the left of T
and second to the right of S. U's neighbours
69. Fact 1: Humans are mammals. are S and Y; and T and W are not seated
Fact 2: Some humans are engineers. opposite each other. Who is third to the left
Fact 3: Engineers build houses of V? (GATE-PI - 17)
If the above statements are facts, which of
the following can be logically inferred?
(a) X (b) W (c) U ( d) T

I. All mammals build houses. 73. A contour line joins locations having the
II. Engineers are mammals. same height above the mean sea level. The
III. Some humans are not engineers. following is a contour plot of a geographical
(GATE - PI - 16)
\( I I 1 1 �1 1 1 1 1 1 1 1 1 :.... P1 1 ! ,l1t .1111 ,1 1, ydcrabadl Delhi! Bhopal! Punc I Bhubaneswarl I..ncknowl Patna! Bcnplurul Chcnnai I Vuayawadaj Vmg I Tuupati I Knbtpaily J Kolkala
: 1025 : Numerical Ability

region. Contour lines are shown at 25 m 76. The growth of bacteria (lactobacillus) in
intervals in this plot. milk leads to curd formation. A minimum
bacterial population density of 0.8 (in
suitable units) is needed to form curd. In the
graph below, the population density of
lactobacillus in 1 litre of milk is plotted as a
function of time, at two different

(GATE-PI - 17) temperatures, 25° C and 37° C.


The path from P to Q is best described by
(a) Up-Down-Up-Down
1 .0

i
0.9

(b) Down-Up-Down-Up
,8 0.8
0.1
j.
(c) Down-Up-Down
0.7

(d) Up-Down-Up
f5 0.S

---- ···-·!! ·-·-'!·-··-··!1-··+I


t • t l

J! C.<1
1
- ·- . . . . -r· · · 1 ..... .,-.. t·
a a.3
74. Trucks (10 m long) and cars (5 m long) go e_ : I
0.2 ••- : • ! •I f•

on a single lane bridge. There must be a gap


0.1
0.0 '

of at least 20 m after each truck and a gap of


I

0 � � � M ioo t � 1� ™ iM �

at least 15 m after each car. Trucks and cars Time (min)


travel at a speed of 36km/h. If cars and Consider the following statements based on
trucks go alternately, what is the maximum
the data shown above:
number of vehicles that can use the bridge
The growth in bacterial population stops
in one hour?
1.

earlier at 37° C as compared to 25°C


(GATE-PI - 17)
(a) 1440 (b) 1200
(c) 720 (d) 600 11. The time taken for curd formation at
25° C is twice the time taken at 37° C
75. There are 3 Indians and 3 Chinese m a Which one of the following options 1s
group of 6 people. How many subgroups of correct? (GATE-ME - 17)(SET -1)
this group can we choose so that every (a) Only i (b) Only ii
subgroup has at least one Indian?
(c) Both i and ii (d) Neither i nor ii
(GATE-PI - 17)
(a) 56 (b) 52 (c) 48 (d) 44

\( I I 11!.!,Htt 1 1 1 111.:, P11lilit , 1 1 1 1 , 1 1 , ydcrabad I Delhi I Bhopal I Pune I Bhubancswar I Lucknow I Patna I Bcngaluru I Chennai I Vijayawada I Vizag I T'U'Upati I Kukalpally I Kolkata
: 1026 : General Aptitude

77. What is the sum of the missing digits in the P says "Both of us are knights". Q says
subtraction problem below? "None of us are Knaves".
5 Which one of the following can be logically
-4 8 8 9 inferred from the above?
11 11 (GATE-ME - 17)(SET -2)
(GATE-ME - 17)(SET -l ) (a) Both P and Q are knights
(a) 8 (b) 10 (b) P is a knight; Q is a knave
(c) 11 (d) Cannot be determined (c) Both P and Q are knaves
(d) The identities of P, Q cannot be
78. Let S 1 be the plane figure consisting of the determined
points (x,y) given by the inequalities lx-11 �
2 and ly+21 � 3. Let S2 be the plane figure 81. In the graph below, the concentration of a
particular pollutant in a lake is plotted over
given by the inequalities x - y � -2, y � 1,
(alternate) days of a month in winter
and x � 3. Let S be the union of S 1 and S2 .
(average temperature l 0° C) and a month in
The area of S is
(GATE-ME - 17)(SET -l) summer (average temperature 30° C).
(a) 26 (b) 28 (c) 32 (d) 34 ..-. 1 1
E
g
10
9
79. There are 4 women P, Q, R, S, and 5 men V,
W, X, Y, Z in a group. We are required to �
i 6 ,.... .... .., .. . . ····-· ,. \·
T: ... . ;,
form pairs each consisting of one woman u 5 ; . . . (... ;.. . ., . .. ....... )
a
+ -� ; .
·1· .•• .,,.
i. j t r

0 .. ! . i · · · ! ., .. , ., ..... -'0•"
and one man. P is not to be paired with Z,
i ; : .
....... 3

i . :
�•••�-•1•-•-1"- • •0•• • ••-••n•• ;-•o-••• •....• •••-;..-. .,, ,... ,,...._
•..,_._,,,,.,.
i i •

and Y must necessarily be paired with � 2 .

; :· ;
I
I

someone. In how many ways can 4 such � :. .., ; : .: : . ... ... -···· ·
pairs be formed? o 2 " o e 10 1 2 14 10 1e 20 22 24 2e 2a 30

(GATE-ME - l 7)(SET -2)


Day of the month

(A) 74 (B) 76 (C) 78 (D) 80 Consider the following statements based on


the data shown above:
1. Over the given months, the difference
80. All people in a certain island are either
' Knights' or Knaves' and each person knows between the maximum and the
every other person's identify. Knights mm1mum pollutant concentrations is
NEVER lie, and knaves ALWAYS lie. the same in both winter and summer.

\( I I 111.!lJH l'llll� P11lilH ,llltlfl',


Fyderahad I Delhi IBhopal I Punc IBhubaneswarl Lucknow! Patna I Bcngaluru I Oiennai IVuayawadal Viz.ag ITirupati I .Kukalpally I Kolkal:l I
. .t\CE . .
'!i�-�t Fllp)C'll21Dg PnNicdiooa : 1 027 : Numerical Ability
� � ===========================================
11. There are at least four days in the 82. X bullocks and Y tractors take 8 days to
summer month such that the pollutant plough a field. If we halve the number of
concentrations on those days are within bullocks and double the number of tractors,
1 ppm of the pollutant concentrations it takes 5 days to plough the same field.
on the corresponding days in the winter How many days will it take X bullocks alone
month. to plough the field?
Which one of the following options is (GATE-ME - 17)(SET -2)
correct? (a) 30 (b) 35 (c) 40 (d) 45
(GATE-ME - 1 7)(SET -2)
(a) Only i (b) Only ii
(c) Both i and ii (d) Neither i nor ii

!11•1i@iiiiiih/iRflbftihijj� !d�rabad I Delhi I Bhopal I Pwie I Bhubaneswar I Lucknow I Patna I Bengaiuru I Chennai I Vijayawada I Vmg I T1rt1pari I Kukatpally I Kolkala
SOLUTIONS
03. Ans: (b)
One Mark Solutions Sol: Given P = Q½ = R½
01. Ans: (d)
Sol:
Q2 = P4

Q2 = p . p 3 p3 = ( R ½ J = R
hockey players
Q2 = P . R
For above the logical venn diagram shows as
follows
04. Ans: (d)
Total No. of players = hockey only+ foot ball
Sol: (1.001)12 = 3.52 and (1.001)2° = 7.85
59 62

only+ both hockey and foot ball


(1.00 1) = (1.001} X (1.001)
1 259 2062
= 5+ 10+ 7
3321

= 22 = 3.52 X 7.85 = 27.63 �27.64


Total No. of persons = 25
:. Neither hockey nor football players 05. Ans: (a)
= Total No. persons - Total No. of players Sol: The cost function for a product
= 25 - 22 = 3 In a firm = 5q2
q = The amount of production
02. Ans: (a) = The number of units to be produced by
Sol: P : Q the firm
8 cost price (C.P) = 5q2
40 : 60 15% of 40 =- x 40 = 6
100 selling price (S.P) = 50q
-6 6 25 profit = S.P - C.P
- .+
- 25% of 60 =- x 60 = 15
34 66 100
25 q S.P = (SOq) C.P = Sq2 Profit
15% Of 40 = X 60 = 15
100 1 50( 1 ) = 50 5( 1 )" = 5 45
2 50(2) = 1 00 5(2)2 = 20 80
The difference between P and Q is 2
3 50(3) = 1 50 5(3)2 = 45 1 05
4 50(4) = 200 5(4) = 80 1 20
:. The total No. of voters are 100
2

\( f l ,11�IJH { I Jill-; P11lilH ,IIIOII' �ydcrabadl Dclhi l Bhopal J Pune J Bhubaneswarl wclmowl Patna l Bengaiuru l Chennai l VuayawadalV17.ag ITirupati I Kukatpally l Kolk.at, I
".,...
'!,.. .. .
..- -......"' . ACE . .
�&JgjnmqPnhlionooa : 1029 : Nwnerical Ability

5 50(5) = 250 5(5)2 = 125 125 15


- x Mon = 60
6 50 (6) = 300 5(6) = 1 80 1 20
100
2

7 50 (7) = 350 5(7)l = 245 1 05


8 50 (8) = 400 5(8)2= 320 80
:. Temperature on Monday = 40°C
Temperature on Thursday = 40°+ 6°
= 46°C

:. Profit = 50q - 5q2is maximum at q = 5 07. Ans: (c)


:. The profit is maximized at q = 5 Sol: The maximum sum of the sequence
Alternative method 44, 42, 40 ........2
Sum of ' n' terms of anA.P.
Selling price = 50q
Cost price = 5q2 Sn = ; (2a+ (n - l}d)
Profit(y) = 50q - 5q2
Where n = 22, a = 44, d = -2
y' = 50 -l Oq = 0 => l Oq = 50 => q = 5
22
Sn =
y" = -10 < 0 => y"(5) < 0 2 (2 x 44+ (22 - 1X- 2))
:. y is maximum at q = 5 = 11 [88+ (-42)] = 11[46] = 506
Maximum profit = y(5) Alternative method:
= 50(5) -5(s)2 Given sequence is even decreasing maximum
= 250 - 125 = 125 n(n+ 2 ) 44(46)
sum =
- = 506 (n = 44)
06. Ans:(c) 4 4
Sol: Mean temperature of Monday to Wednesday 08. Ans: (c)
is 4 1°C Average
Sol: Index = ____::;____
Stan dard deviation
Sum of the temperature of Mon + Tue+ Wed
3 X 41°C = 123°c 31.2
Index of K = -- = 5.98
=
5.2 1
=> Mean temperature of Tuesday to Thursday
is 43° sum of the temperature of Tue+ Wed+ Index of L = � = 7.244
6.35
Thu
54.4
= 3 X 43° C = 129°c Index of M = = 8.745
6.22
Therefore Thursday - Monday = 6°C
17 9
Temperature on Thursday was 15% higher Index of N = · = 3.033
5.90
than that of Monday Batsman M will have more Index.
(i.e) 15% of Mon = 6
°
:. The most consistent Batsman is ' M '.
!N•ijj§jjj4Ujjjji@GiMJhlhti yderabad j Delhi j Bhopal j Ptme j Bhubaneswarl Lucknow j Patna j Benga)uru l Chennai l Vuayawada l Vizag jTuupati I K.ukatpally j Kolkala
: 1 030: General Aptitude

09. Ans: 725 Option(c) F = p


Sol:
x 2+ 11 x2+ 11 x2+ 11 x2+ 11 x2+ 11
M P > M � not True
/\(\(\(\(\ Q
12 35 81 173 357 @IJ
Option (d)
10. Ans:(b)
M

Sol: Six faces frequencies are different. So, it is


R
P < M � True
biased .
P=A

11. Ans: 45 14. Ans: (a)


Sol: Sol: f (x,y) =xn yrn =p
If x is doubled and y is halved, then

- 2 n .x n . L
m

12. Ans: 16 2m
-

Sol: 81 54 36 24 16
\__}\_A_)\__)
2 2 2 2 = 2n -m_f( x, y)
x- x- x- x-
3 3 3 3
15. Ans: 495
13. Ans: (d) Sol: x, x+2, x+4, x+6, x+8 . . . . . . .
M 5x + 20 =425
Sol: Given
p
5x = 405
Option (a) R p
- s X = 81
M � not True Hence 12 odd numbers
81, 83, 85, 87, 89, 9 1, 93, 95, 97, 99, 101, 103
_e

Sum of last 5 numbers =495


s

Option (b) F
£ F 16. Ans: (b)
£ F Sol:
A: Ram selected ( A ): Ram not selected
M � not True
.e
B: Ramesh selected ( B ) : Ramesh not selected
s
F

!IS11i@jjj4iijjjiRflbjiijjjjj� yderabad I Delhi I Bhopal I Punc I Bhubancswar I Lucknow I Patna I Bcngaluru I Chennai I Vijayawada I Vt.zag j Tirupati I Kukatpally I Kolkata
ACE
� �E�PnNaiool
-:,�• : 1031: Nwnerical Ability
==================================
=> P ( A ): = 1 - P(A) = 1-
-
(ii) ex > eY
6
1 1 5
P(A) = =
6 6 e3 = 20.0�
P(B) = ½ => P( B ): = 1 - P(B) = 1- ½ = ; e2 = 7.38

P (only one of them will be selected)


-
= P(A) X p ( B ) + P(B) X P( A )
-
(iii) f > Xy
1 7 1 5 7+ 5 12 1 (2)3 = 8
= - X - + - X - = -- = - = - (3)2 = 9
6 8 8 6 48 48 4
f > xY is false
17. Ans: (b) (iv) COSx > COSy
Sol: The electricity consumption per km of COS3 = 0.998
stretches M, N, 0 and P are COS2 = 0.999
Stretch M per km = .!3..
20
= 0.60 COSx > COSy is false
25
Stretch N per km = = 0.55 19. Ans: (c)
45
45 Sol: In the alphabetical sequence, the m1ssmg
Stretch O per km = = 0.60
75 word is as follows

\J\j\
Stretch P per km = 57 = 0.57
1 00
Therefore in the stretch 'N', the electricity
consumption per km is minimum.
B EF JKL QRST
(+I ) (+2) (+3) (+4)
18. Ans: (a)
Sol: For whole numbers, greater the value greater Therefore, the missing word in the sequence
will be its log and same logic for power of e is 'MNOP'
Example:
20. Ans: (b)
Assume x = 3, y = 2
Sol: For a match to be played, we need 2 teams
(i) lnx > lny No. of matches = no. of ways of selections 2
ln3 = 1.0986 teams out of 5 teams
ln2 = 0.6931 51
5C 2 -
-
lnx > lny is true 2 !><(5 - 2)!
5x4x3x2xl
= = 10
2xlx3x2xl
\Cl } llgllH't Tlllg P11hlH .111011, yderabadj Dclhi j Bhopal j Pune j Bhubaneswarl Lucknow j Patnaj Bengaluru j Chennai jVtjayawada j Vizag ITirupati I Kukalpallyj Kolkata
" "
..:F�PnNirPD
�-�
. ACE
. . . : 1032: General Aptitude

21. Ans: (a)


Sol: Total no. of fruits = 5692000
Unripe type of apples
= 45% of 15% of 5692000

45
=X � X 5692000 = 384210
X X

100 100
Ripe type of apples
34 85 X 5692000 = 1644988
X

=
100 100 The perimeter of the window = 5x = 6 m
X

: . Total no. of apples = 384210 + 1644988


= � = 1.20 m
= 2029198 5
X

Area of the window


22. Ans: (c) = Area of square + Area of !l.1e

Sol: From given data, the following diagram is .Ji


possible = 1.2 X 1.2 + - X (1.2)
2
4
___7 km___ ,@] = 1.44 + 0.623 = 2.06 m2
____ 6.02 m......____,.,�
5 km 24. Ans: (c)
LiJ--__�@!1
-
AH_..____-'"'1(Ml
Sol: Given (9 inches/ 12 = (0.25 yards) 112
------- 10 km -------+l 9 inches = 0.25 yards

I = I live 25. Ans: (b)


AH = Ahmed lives; M = Michael lives Sol: M efficiency = 2 [ efficiency of S,E, and F]
S = Susan lives; A= Arun lives
Contribution of M in the project
� Arun lives farthes away than Ahmed
= x days x 6 hrs x2
means more than 5 km but closer than
Contribution of E in the project
Susan means less than 7 km, from given
alternatives, option 'C' only possible.
= 2x days x 12 hrs x 1
Contribution of M : Contribution of E
23. Ans: (b) X X 6 X 2 : 2X X 12 X 1
Sol: From the given data, the following window 1:2
is possible

\( I l 11�1111 t 1 1111.!. P11hl1< ,1t1< ,11.., �yderabad I Delhi I Bhopal I Pune I Bhubaneswar I Lucknow I Patna I Bengalwu I Chennai I Vuayawada j Vizag I Tirupati I Kukatpally I Kolkata I
-:,:.t
. �CE . .
:&p,erqPnNiariool
: 1033 : Numerical Ability
... � ==================================
26. Ans: (a) 80H 170P 75
+ = (H + P )
100 100 100
Sol: (x¾ of y) + (y¾ of x) = 2- x y + L x x
100 100 80 H + 170 P = 75 H + 75P
xy + xy 80 H - 75 H = 75 P - 170P
100 5 H = - 95 P
2xy There is a negative ratio so, there are errors
= - = 2% of xy
100 in the official's statement .
27. Ans: (a)
29. Ans: (b)
Sol: The new number formed by reversing the
Sol: Sum of angles in a pie chart = 360°
digits is greater than the original number is
The relation between angle and percentage is
possible in options A and B only.
100 % = 360°
Options 'a':
% = 3.6°
Sum of two digits in the number = 3+ 9 = 12
After reversing the two digits number = 93 : . 40% = ?
The difference between the new number 40 X 3.6 = 144°
formed and original number = 93 - 39 = 54
30. Ans: (b)
:. option 'A' is correct.
Sol: From given statements the following Venn
Option 'b':
diagrams are possible
original number = 57
T = tables, S = shelves,
After reversing, the number is formed = 75
C = chairs and B = benches
The difference between these two numbers
= 75 - 57 = 18

:. It is not.

28. Ans: (d)


Sol: H = house hold consumption
P = other consumption
House hold consumption decreases by 20%
80
= H
100
Other consumption increases by 70%
= 170 P
100
!l1••4i@jjj44ijjjijmfij@jj.jj� yderabad I Delhi I Bhopal I Pune I Bhubancswar l Lucknow I Patna I Bengaluru I Chennai IVuayawada j V17.ag I Tirupari I Kukatpally I Kolkata
: 1 034 : General Aptitude

33. Ans: (a)


Sol: P states that S has atleast 3 cars=� 3
Q believes that S has less than 3 cars = < 3
R indicates that S has atleast one car = � 1
P's and Q's statements are exactly opposite
in nature and R's statement is proportional
From all of the above diagrams, conclusion to P's statement.
(ii) only deduced from the statements. From the given data, only one person
statement is right as it mean that two person
31. Ans: (d)
statements are wrong. i.e., P and R when S
Sol: From the given data, base radius = 5 cm,
has zero cars.
Height of the cone=12 cm,
Slant height of the cone (!) = �(hf + r 2 34. Ans: (b)
=�(12)2 +(5)2 = .JI 44+25 = M9 = 13 cm Sol: p � Q� R
(married) (un married)

It is rolled on the ground through the fixed P is a married person is looking Q


point 'P' But Q is married (or) unmarried is not given
We have the relation r =� xR so,
21t : . Cannot be determined
Where, r = radius of the cone ,
R = radius on the ground = slant height 35. Ans: (D)
0 = Angle about P does the cone travel Sol: According to the given relation of a-b =
0 101t even, there is a possibility of odd-odd (or)
5=-x13 => 0 = -
21t 13 even-even is equal to even
32. Ans: (B)
From the given option (D)
Sol: Among the group of 100 employees middle
odd x odd - odd (or) even x even - even �
persons are 50th and 5 1 st persons,
is always even number
their salaries are Rs. 30,000 each
odd = odd number, even = even number
30000 + 30000
Median = =30000 all other options are not satisfied this
:. Option (B) is correct.
condition.

yderabad I Dellii I Bhopal I Punc I Bhubaneswar I I.J.icknow I P:itna I Bengahuu I Qieonai I Vuayawada I Vwg I Tuupali I Kukalpally I Kolka!a
.!'.�
"
.... ._.,wli'�

36. Ans: (c)


ACE
.........,.nog PIJbticatiooa
---
: 1035 :

02. Ans: (d)


Nwnerical Ability

Sol: 5 Skilled workers can build a wall in 20 days 1


Sol: Probab 1. 11ty
no.of favorable cases
. = ---------
total no. of possible cases skilled workers 5 x 20 = 100 days
Among two children's (boys), the older one 1
1 day work of skilled worker = --
is a boy = 1 and two children's are boys 100
8 semi-skilled workers can build-wall = 25
only.
1 semi-skilled worker = 8x25= 200 days
:. Probability = _!_ 1
2 1 day work of semi-skilled worker = --
200
10 unskilled workers can build a wall = 30 days
1 unskilled workers = 10 x 30 = 300 days
Two Marks Solutions 1
1 day work of unskilled worker = --
300
:. 2 skilled + 6 semi-skilled + 5 unskilled
01. Ans: (b)
= 2(
_1 )+ 6 (_
1 + -
) 5(
1
Sol:
)
S - 5 years 100 200 300
G - 4 years =
1 3 1 6+ 9+5 1
- + - + - --- =
H - 3 years 50 100 60 300 15
I - 1 year 2 skilled + 6 semi + skilled + 5 unskilled
Assume, the age of Irfan is one year, Hari is 3 can build a wall = 15 days

years, Gita is 4 years and Saira is 5 years


03. Ans: (c)
Sol: The question is about summary. It means we
. ·. The age difference between all of them is not
more than 3 years
need to find out the intension of writer
Hari's age + Gita's age = 3 + 4 = 7 years (a) Strife means conflict, this option is not the
Irfan's age + Saira's age = 1+5 = 6 years right choice because chemical agents attacked
:. 7 years > 6 years civilians but not resulted in civil strife
The age difference between Gita and Saira (b) This option only talks about positive usage of
is 1 years chemical agents in modem warfare. Which is
not the intension of writer
:. The order of they born from oldest is SGHI

!M•li@hiiOlhiM@flj@jj.jj�_ yderabad j Delhi j Bhopal j Pune j Bhubaneswarj Lu�ow j Patna j Bengalwuj Chennai j Vrjayawada j \Cu.ag j T11Upari I .K�yl Kolkala
ACE
��-� :F.ngiocr.riog P.,blic:ariooa : 1036 :
� - � ==================================
General Aptitude

( c) The key word here is "regretfully" the writer 12--------- 4


feels that chemical agents in the warfare is not 13 5
desirable. This is the right choice. 14 6
(d) Even this option talks about the positive usage 15 7
of chemical agents (i,e) military establishment
encourages the usage of them which is not the 16_____(_
2)___ 0
writers intension. 17_________ 1
18 2
04. Ans: (d) 19 3
Sol: More than 3000, 4 digit number means, the
First digit may be 3 (or) 4, the second third
CD<D CD
and fourth digits are three in each (i,e) 137 731

I2 13 13 13 I 276
435
672
1623
= 2 x 3x 3 x 3 = 54

05. Ans: (c) 06. Ans: (c)


Sol: This is Octa numbers addition operation Sol: The total No. of toffees in a bowl = x

= 2x+ 12 - ( 2x - 24 )
General Number Octa numbers 3 12
0 0 8x+ 4 2x+ 24 -
� for R� = \� ( ; 4)
l l
6x+ 72
2 2
12
= ---
3 3
The remaining toffees in a bowl
4 4
5 5
6 6
� 3x - x+ 12
7 7 = x- +4=
3 3
( 2x+ I 2)
8 1 0 =
9 1
� for S �
10 2
11 3

.1ij@jjji4.jji@6iMhifh� I I I I I I I I I
yderabad Delhi Bhopal Pune Bhubaheswar i Lucknow Patna Bengaluru Chennai j Vtjayawada Vil.ag T"ll\lpati I I
Kukatpally Kolkata
t.\ ��::1;����;:�. .t\CE . . 7 =
.l!lFti..,.W'J11111 Puibili.rati-oo• =======:;,..;l�0�3,:..
-:,.. ..:'= Numerical Abili
=========�::::;�:���;::;;;ty� ,;,,,
2x + 12 36
( 3 ) for r S => - = 9 => 36-9= 27+ 3 =30
-'------'--- - 2 => 2x + 12 _ 3 = 2x + 12 - 36 4
4 12 12 30
for T => - =15 => 30-15 = 15 + 2 =17
2x - 24
= --- 2
12 :. Ans 48 . (d)
The remaining toffees in a bowl
2x + l2 2x - 24
= -( )
07. Ans: (d)
3 12 Sol: Given that f(y) = 141/4
-
= 8x + 48 2x + 24 = --
6x + 72 Q = any non-zero real number
12 12 = +q (or) -<1
6x + 72 If q is the
( 12 ) 6x + 72
� for T => 2= 2 q
2 24 f(q) = l l = 1
q
6x + 72 - 48 --
= ---- = 6x + 24 -q
24 24 f(-<I) = I i = -1
-q
The remaining toffees in a bowl
6x + 72 6x + 24 :. I f ( q ) - f ( -q )I = 1 1 - ( - I )I = 11 +11 = 2
= -( )
12 24 If q is -ve
12x + l 44 - 6x - 24 6x + l20 -q
f(-v = I i = -1 => f(-v
24 24
= =
-q
6x +120 =
l7 -q
24 = I i = -l
-q
6x + l 20 = 17 x 24 = 408
q
6x = 408 - 120 = 288 f (-q) = l l = 1
q
288 =
j f ( q ) - f ( -q )I = 1 1 - ( - l )I = 1- 1 - 11 = 2
X= 48

Short cut Method


08. Ans: (c)
Among all given alternatives, divisible by 3 is Sol: The sum of n terms of the senes
48 only 4+44+444+ .....is in the given alternatives if n
for R => -=16 => 48-16=32+4 =36
48 = 1 , which will give 4 and if n = 2 which
3 will give sum of first two numbers (i, e) 4 +
44 = 48 is our answer.
\( I l 11�l'll t I l!!� P,1lil1t ,t!J{)Jh �yderabad l Dellii l Bhopal l Pune l Bhubaneswar l Lucknowl Patna l Bengaiwu l Chennai l Vuayawada l Viz.ag ! Tuupati , Kuk.alpallyl Kolkala 1·
ACE : 1 038 :
':.:...:�Pnbtiraima General Aptitude
..- .. ====::::::::==================================
Ex: If n = 1 09. Ans: (b)

(a) :l [10n -l - 9n - 1 J Sol: Inference questions should deal with


conclusion of the paragraph that is not clearly
mentioned in the paragraph so, according to
= � [10 1+1 - 9(1) - 1]
81
(a) Horses were given immunity to disease but
= - [100 - 9 - 1] = - x 90 according to paragraph immunity is naturally
4 4
81 81
developed by the horses
(b) This is the right choice, because Horses have
'A ' is not. natural capacity to develop immunity to
deceases. That is the reason when Horses
[
(b) 84 10 -1 - 9n - 1
1
n
J were injected toxins they built up immunities
(c) Horses were not given medicines to fight
= � [10 - - 9n - 1] = � [1- 9 ( 1) - 1]
1 1
toxins. This is not the fact according to the
81 81
passage.
= (d) Diphtheria cannot be given a diseases
4
81(-9)
It is not
10. Ans: (b)
:.

Sol: From the given graph, we can conclude that


(C) 841 [10 +I - 9n - 1 0J
n
the fuel consumed per kilometer was least
during the lap ' Q ' because it will have
= � [101+ 1 9 ( 1) - 1 0] = � [ 100 - 9 - 10] average speed is 45 krnph but it covered 75
81 81
-

kilometres of distance (maximum)


= - x 81 = 4 and
4
81
11. Ans: (c)
If n = 2
Sol: 7 trucks can clear all the orders at the end of
� [10 2+1
- 9 ( 2) - 10] = � [1000 - 18 - 10] 4th day
81 81
3 trucks can clear all the orders at the end of
= [972 ] 10th day
841
Transporter receives every day same number
It is true orders currently, he has some pending orders
(backlog)

!li11i@jjjl4imQAflft5ijj.jj.ydcrabad I Delhi I Bhopal I Pune I Bhubaneswar I Lucknow I Patna I Bcngalwu I Chennai I Vrjayawada I Vizag fTuupati I Kukatpally( Kolkata I
.. � ..
"., ......,
'!.
. �.CE . .
�F�NW'Jfiml : 1039 : Numerical Ability
.. � ===================================
What is the minimum number of trucks 12. Ans: (d)
required so that there will be no pending order Sol: P, Q, R and S are four types of dangerous
at the end of the 5th day? microbes recently found in a human habitat
1 day = 7 x 4 = 28 No. of trucks In the graph
1 day 3 x 10 30 No. of trucks
= =
� on X-axis represents probability that microbe
L.C.M of 28, 30 = 420 will over come human immunity system and
Assume everyday received orders = 105 � on Y-axis represents Toxicity (in milligrams
First case of microbe required to destroy half of the
420orders+ back log (b) body mass in kilograms
1 day =
28 Microbe 'S' will have 80% of probability that
420 microbe will overcome human immunity
= 105
system and less weight of milligrams of
microbe required to destroy half of the body
Second case
mass in kgs.
10 days = 1050 :. Microbe 'S' is danger to human beings.
1050orders+ back log (b)
1 day = 13. Ans: (d)
30
420 + b 1050+ b Sol:
--- = JO lit
28 30 of pure spirit

12, 600 + 30b = 29400 + 28b r. 1 lit of spirit is replaced

2b = 16, 800 9 lit + I lit water


90000 ml + 1 000 l Op = J 000 ml
b = 8400
8820 = 9
1 day =
28
315 -900 - J OO
� 1 lit of water add
8 1 00 1 900
For 5 days case 81 19

1 day = 105 X 5 + 8400 = 8925 - 810 : -l water



No. of trucks required on 5th day spirit + 7290 = 271 0 I
8925
= = 5.66 2: 6
315 x 5 :. Spirit left in container = 7.290 lit
Minimum number of trucks required without Shortcut:
pending orders at the end of 5th day = 6
10 x _2_ x _2_ x _2_ = 7.29 lit.
10 10 10
\( I I 11g111t t 11n12, P11hlH.1t1(111, I I I I I I I I I I I
ydcrabad Delhi Bhopal Pune Bhubaneswar Lucknow Patna Bengaluru Chennai Vtjayawada Vtz.ag Tuupari I Kukatpallyl Kolkata
: 1040 : General Aptitude

14. Ans: (a) Explanation: It is clear that we are supposed

Sol: V = 4q ,
l OO to find out the choice that is not related to
F=
death or situation before the death. Its about
100 healing process a person goes through after
(V + F) = 4q + -
losing a relative
(a): ' 5 '
100 16. Ans: (d)
Sol: The total monthly budget of an average
= 4x5 + - = 20 + 20 = 40

(b): ' 4 ' household


100 = 4000+1200+2000+1500+1800
= 4x4 + - = 16 + 25 = 41
= Rs. 10500
(c): ' 7 '
Percentage of the monthly budget spent on
100
= 4x7 + - = 28 + 14.28 = 42.28 . savings amount
7 savmgs = -----"--­
(d): '6'
Total expenses

l OO = l 500 x l OO = 14.285%
= 4x6 + = 24 + 16.66 = 40.66 10500
6
No. of units should be produced to . ·. The approximate percentage of the
minimize the total cost (V + F) = ' 5 ' = 100 - 14.285 = 85.714 ::::: 86%
Monthly budget NOT spent on savings
15. Ans: (c)
Sol:
a) Letter of condolence is obviously related to 17. Ans: (c)
bereavement and grief because it is written to Sol: Two friends A and B
the family which is going through pain and
grief
b) This option even related to the situation and
mental agony a family goes through after a
death in the family
c) The leading causes of death are clearly not
related to bereavement and parting of
deceased person. This is the right choice 1 . 1 5 1 .30 1 .45 2.00
d) Giving support to a grieving friend means ( 1 . 1 5, 1 .00)
� Person A
dealing with situation after death

l11•1il/4ihilihiNRtifliMiiiih+1yderabad I Dellii I Bhopal I Puncl Bhubancswarl Lucknow I Patnal Bcngaluru IChcnnai jVuayawadaj V.;...rrJJ'Upali I Kulwpa))yl Ko1ka1a !

" . .
.,·•·:&p,,,,mgNilii»n
ACE . . : 1 04 1 : Numerical Ability

The probability that they will meet on that 19. Ans: (a)
day Sol: Let us consider eight rice bags are A, B,C, D,
E, F, G and H
= 1-2 [!2 X
60 60 J
45 X 45
Case - I
(Area of ABCD) First weighing

_ l --
- 16
_ 16
9 - --
16
-9 -
_- 7
16
8 A, B & C
8
D, E &F

18. Ans: (a) If A, B & C (set 1) and D, E and F (set 2) are


Sol: Total No. of currency notes = 14 equal, in second weighing
Rs. 20 notes = x either G (or) H are heavier.

Rs. 1 0 notes = y Case - II


x + y = 14 . . ........ . . (i) First weighing
Money value of Rs. 20 notes =20 x
Money value of Rs. 10 notes =10 y
The total money value of the notes
= Rs. 230
8 A, B & C if:
8
D, E & F

20 X + 1 0 y = 230 . . . . . .. . .. (ii)
solve (i) and (ii) If A,B & C (set 1) and D, E & F (set 2) are
X + y = 14 . . . . . . . . ...... (i) x20 not equal among set 1 and set 2 any one them
20 X +10 y = 230 . . . . . . . . .... (ii) is heavier than other.
If set 2 (D, E & F) is heavier than set 1, In
20 X + 20 y = 280 second weighing D Vs E
20 x + 10 y = 230 � if D and E are equal, than F is heavier
1 0 y = 50 � if D and E are unequal, then higher side one is
50 heavier.
y = = 5
10 From case I and case II, the minimum number
:. The number of Rs.10 notes that has is 5. of weighings required to identify the heavier
bag is '2'.
I Delhi I Bhopal I Punc I Bhubaneswar I Lucknow I Patna I Bcnga)uru I Chcnnai I Vuayawada J V11.ag I Tuupati I Kukatpally I Kolkata I
�ydenbad
: 1 042 : General Aptitude

20. Ans: (a) Alternative Method


Sol: The question is about sum up. In sum up
100
questions we have to make sure that all the
points mentioned in the paragraph should be
reflected in the answer choice.
X y
(a) This is the right choice. 60% 40%
(b) This is irrelevant as there is no mention of i
inhuman treatment.
reliable 96% 72%
(c) This is a general statement, not reflecting the
given points in the paragraph. 60 x 96 40 x 72
(d) The interpretation of the sentence 1s 100 100
completely opposite to the given paragraph 0.576 0.288

The probability that a randomly chosen shock


21. Ans: (b)
absorber, which is found to be reliable, 1s
Sol: X supplies = P(x) = 0.6
made by Y is
Y supplies = P(y) = 0.4
0.288
= ----- = 0.334
0.576 + 0.288
X's shock absorbers reliable = P (�) = 0.96

22. Ans : (c)


Y's shock absorbers reliable = P (�) = 0.72
Sol:
P: is correct, by adding any constant value to
p(y) = P(Y n R) each term of the sequence, the mean increases
R P(R) to the same extent.
P(Y ) P (�) Q: is incorrect, by adding any constant value to
each term of the sequence do not change the
-

P(X) P(; ) + P(Y) P(; ) standard deviation


R: is correct, doubling each entry m a list
(o.4)x (0.72) doubles the mean of the list
=
(0.6 )(0.96) + (0.4) (0.72)
S: is incorrect,
0.288 = 0.288 = 0.334 :. P and R assertions are CORRECT
0.576 + 0.288 0.864

\( I l 11i..,111l l 1 1 11� P11lilu .111011, �ydcrabadJ Delhi_l Bhopa)J Punc J Bhubancswarl Lucknow I Patna J Bcngaluru J Chcnnai J VuayawadaJ Vizag JTuupali I KnkalpallyJ Kolkala I
: 1043 : Numerical Ability

23. Ans: (a) 25. Ans: (a)


Sol: Sequence of terms AD, CG FK JP, the next Sol: Total No. of days in leap year = 366 days
term? In which there are 52 weeks and 2 no. of odd
days
The probability of being Saturday of these

+3 +4 +5 +6 two days of the week is �


7

24. Ans : (b) 26. Ans: (b)


Sol: The profile of the arch y = 2.x - O. lx2 Sol: The given equation is a quadratic equation to
y = height of the arch in meters get the roots of opposite sign, the product of
Assume x=2, 4, 6, 8, 10, 12, 14, 16 p(p - l)
roots must be negative (i . e) is less
y = lx - O.t.x2 3
than zero. Therefore it is possible only
X

2 Y=2 X 2 - 0.1 X (2)2 = 3.6


4 y = 2 x 4 -0.1 (4)2 = 6.4 p E (0, 1)
6 y = 2 X 6 - 0. 1 X (6)2 = 8.4
8 Y=2 X 8 - 0. 1 X (8)2 = 9.6 27. Ans: (c)
10 y = 2 xlO - O.lx(10)2 = 10
Sol: � A car travels 8 km in the first quarter of
an hour (i.e 15 min)
12 y=2 X 1 2 -0. 1 ( 1 2)2 = 9.6
14 y = 2 X 1 4 - 0. 1 ( 14)2 = 8.4
� next it travels 6 km in the second
16 y = 2 x 1 6 - 0. l x ( l 6)2 = 6.4
quarter of the hour (i.e 15 min)
� next it travels 16 km in the third quarter of
the hour (ie 15 min)

Time Distance
:. The maximum possible height of the arch is at 15 min 8 km
x = 10 15 min 6 km
Alternative method 15 min 16 km
y = 2x - O. lx2
y' = 2 - 0.1 X 2.x = 0 => X = 10 = 45 min = 30 km
y" = - 2 x 0 . 1 => y"(l O) = - 0.2 < 0 3
= - hrs
=> y is max @ x = 10

jljjli@jjj§§@IRflbftjmjjj� yderabad J Delhil Bhopal J Pune J Bhubaneswarl wcknow J Patnal BengaJuru J Chennai J Vrjayawada J Vmg J T,rupati I Kukatpal)y J Kolkata
ACE Gene titude
�-�
� ":E
= ·m

b�· •:
:·�·���j�
:- ·
·�
1 044
=·�=======�:�::� �A
: ==========:�;ral �p: :;::=

Total distance 30. Ans: (c)


:. Average speed of the car = ---­
Total Time Sol: A tourist covers half of his journey by train =
30 = 30 x 4 60 kmph = S1
40 kmph
One - fourth of the distance = 30 kmph = S2
= =
3 3
4 One - fourth of the distance = 10 kmph = S3
Firstly the average speed of Bus and Cycle
28. Ans: (d) 2xy 2S2 x S3
= -- = -.c.....____..c....
Sol: Given series
10 + 84 + 734 + . . . . . . . . . .. n terms 2 x 30 x 10 = S
A = l kmph
It can be written as I 30 + 10
(9 + 1 ) + (92 + 3) + (93 + 5) + . . . .. n terms Next the average speed of train and average
(9+92+ 93 + .. n terms) + ( 1+3+5+ . . . n terms) speed of Bus and Cycle
2 x S1 1 = 2 x 15 x 60
(A i )= = 24 kmph
A

SI + l 15 + 60
a(r n - 1)
A

Sn + Sn=
n (2a (n 1)d
+ - )
r _1
=
2
Where Where 31. Ans: (b)
Sol: Given expression
a = 9, r = 9 a = l, d = 2

�[
9
t� !)] + [� (2 x I + (n - l)x 2)]
I

..fi + .fi
+
I

-h + -h
+
I

./J + .{4

1
+
......
+
I

.fso + .fsi

Let the first term


-v l + -vr;:;2
� [9�: - 1)H�(Zn)] � [9(9:- !)J + n '
11

= I x fi - 1 = ..fi - I = fi Ji
-
fi + Ji fi - 1 2-1
29. Ans: (d) Similarly
Sol: From 1 to 100, 2-digit integers are possible fi - Ji + .Ji - fi + -.J4 - -J3 + ...... Jsi - Fso
from 10 to 99 is 90 = .Jsi - Ji = 9 - 1 = 8
From 10 to 99 divisible by 7 are 13 numbers
are not divisible by 7 are 77 numbers 32. Ans: (b)
The probability that the selected number is Sol: Current erection cost of a structure
. . 'ble by 7 = -77 = Rs. 13,200
not d1v1s1
90 The erection cost of a structure
= wages x time = x x y

\( I l 11�11 1 l t 1 11 1 .., P1ilil1l ,1!1c,11-.


�yde
rabad I Delhi I Bhopal I Pune l Bhubaneswarl Lucknow! Patna! Bcngaluru i Chennai j VijayawadajVu.ag !Tlfllpati I Kukatpallyi Kolkata I
: 1045: Numerical Ability

1
The labour wages per day increased by - of
34. Ans: (b)
5 Sol: Option (a) people can fill Form P
the current wages Option (c) people can fill Form S
1 6 Option (d) people can fill Form S
The new labour wages per day = 1 + - = - th
5 5 :. Option (b) people can fill Form 'T'
of current wages

The working hours decrease by - of the


1 35. Ans: 560
24 Sol: Let us find out the speed of train first given
current period, the train crosses a man standing on a plot
1 23 form is 20 seconds.
The new working hours = 1- - =- th of
24 24
D 280 => = 280 =14m / sec
the current period T= =20= S
S S 20
The new erection cost of the Structure Speed of the train = 14 m/sec
6 23
= -x-xl3,200 =Rs.15,180 Now let us suppose the length of platform
5 24 = 'P'mts
D 280 + p
33. Ans: (d) T= = 60= => 280 +P = 840
Sol: S 14
W E K 0 P = 840 - 280 = 560 mts
23 5 11 15
"-.J�\_J
+8 +6 +4 36. Ans: (d)

I Q W A
Sol:
9 17 23 1 Year Trade deficit 1
-(Exports)
"-.J�\_J (Imp-Exp) 5
+8 +6 +4
2005 20 1
14=-x70
F N T X
-::f.

5
6 14 20 24 2004 10 1
"-.J�\_J 14=-X 70
-::f.

+8 +6 +4 5
2007 10 1
N V B D
-::f.

5
22=-X 110

=
1� ,22 2 4
2006 20
"---/�\_J 20=-xlOO
1
+8 +6 +2 5

N V B D does not follow the series order.

!11•11@jjj4ii0!4AflniM\jjfjj+yderabad I Delhi I Bhopal I Pune I Bhubaneswar I ��w I Patna I Bengalwu I �hennai I VijayawadajVmg !Tirupati I Kukatpally I Kolkata I
: 1046: General Aptitude

37. Ans: (b) 39. Ans: 163


Sol: Sol: Let us suppose 8 consecutive odd numbers
.. .. Favourable Pr obability be X, X + 2, X + 4 ............. X +1 4.
Cond1tlonalPr obab111ty =---------=-
Total Pr obability Given that their Sum = 656
1 1
-x- 8x + 56 = 656

½)
3 2
=--------- 8(x+7) = 656
(½x1 ) +(½xO ) +(½x 7=
656
= 82
X+
8
1
- 1 x = 75 (also smallest odd number )
1 6 1
6 = 6 =-X- =- Now let us consider fore even numbers be
1 1 --
2+ 1 6 3 3
-+ O + - x , x+2 , x+ 4, x+6
3 6 6
Given that Average = 87
38. Ans: (c) 4x +1 2
=87
Sol:
4 x +3
( ) =87 � X = 84
Q W Z B
17 23 26 2 second largest even number = x + 4 = 88
'---/
+6
�'---/
+3 +2 sum ofsmallest odd number & second largest
even number = 75 + 88 = 1 63
B H K M
2 8 11 13 40. Ans: (d)
��'---/
+6 +3 +2
Sol: 250 crore rupees = 25x 10 8 rupees
500 thousand tonnes = 5 x 10 8 rupees
W C G J Revenue per Kg
23 3 7 10

+6
'---/�
+4 +3
Item 2=
20%of25xl0 8
20%of5xl0 8
=5

23%of25xl0 8 = 23x5 1 1 5
Item 3= = =6_0 5
M S V X 19%of5X10 8 19 9
13 19 22 24
'---/
+6
'---/�
+3 +2
Item 6 =
19%of25xl0 8 19 x5
= =5_9 3
1 6%of5X10 8 16
20%of25xl0 8 100 25
Item 5= = = = 8_33
1 2%of5X10 8 12 3

IIJ11ii@jji1.ji4Afln1ii\iiii� yderabad I Delhi I Bhopal I Pune I Bhubaneswarf Lucknow I Patna I Bengaluru I Chermai jVijayawadajV,17.3g·1'I;irupati I _!(�I �olkata
: 1 047: Numerical Ability

41. Ans: (a) 10 4


Sol: Procedure 3
-=-
X

Given there is a filling pipe and drain pipe. 30


X= = 7. 5
� Let us suppose the filling pipe fills half­ 4
full tank in ( x) minutes x = 7. 5 hours which is 4 times the rate of
drain pipe.
In one minute .!.th of half-full tank filled
X

� Given drain pipe empties half full tank is 42. Ans: (b)
l th Sol: 7G, 1 lK, 1 3M, -11Q
3 0min. In one minute, - of half full
30 A sequence of letters whose place values are
tank is emptied. pnme.
� The purpose is to fill the remain half full
tank in 10 min. 43. Ans: (d)
Sol: By studing the pie-chart we can clearly say
When both the pipes are open. that all the 4 options are true.
th
So in one minute they can fill up - of half­
l
IO 44. Ans: 4
full tank . Sol: Let the velocity of stream be x kmph
1 Up stream speed = ( 8 + x) kmph
! +- =
( 30) 1� Down stream speed = (8 - x) kmph
1 1 1
-= It is given that time taken in upstream
- +-
x 10 30 = 3x time in downstream

1 3 +-�-=-
1 4 2 d d
_ _= 3x ['d'is distance]
x 30 3 0 15 8-x (8 + x)
-=-

So the filling can fill the half full tank in � hour ( 8 + X ) = 3 (8- x)
2 x = 4 kmph
= 7. 5 hours which 4 times the rate of drain pipe.
Short -cut 45. Ans: 20000
10 �=
- 1
X 30 46. Ans: 0.80 to 0.82
10= _!_= 3+1
1+
X 3 3

!111•41Miiiii•i!4@MIM!+iyderabad I Delhi !Bhopal I Pune JBhubaneswarJ Lucknow! Pa!nal Benga)urul Otcnnai JVijayawadaJVizag JTuupali I Kukalpallyj Kolkala I
ACE
��-· 048 = 1 General A tirude
E�&�p�·W'l:-:·���N�b�n�l'#i�:.W.:·�
... '"= ========�;::;;;,.;,.:==========��;�;:,:.�p�;:��

47. Ans: (c) Where ,


Sol: 13M, 17Q, 1 9S, 23W Pn = 2x => consumption in 2010-2011 year
The respective positions of the letters are Pn-l = x => consumption in 2000- 2011 year
consecutive prime Numbers starting with 13 k = 100 % = 1
tn- 1 = 0 = Present
48. Ans: (b) tn =?
Sol: K C L F T S B - B�ST QF LyCK 2x = x e 1 (i.-o)
S H S W D G - GooD WiSH�S tn = 7.38::; 7.2
? - aCe THe eXaM
(M X H T C) 50. Ans: 22%
Move from back direction without vowels Sol: The expense on labour in 2012 is 4,50,000
Labour = 15% oftotal cost = 4,50,000
49. Ans: (b)
15
Sol: Consumption is doubled is 10 years =-x total cos t = 4,50,000
100
i.e. 100 became 200 Total cost = 30,00,000
Since the annual increase is same. Expense on Raw material in 2012
10
= 20% of3000000 = 6,00,000
100(2-) =200
100 Expense on all other expenses in 2012
( )IO = 80% of 3000000 = 240000
X -2
100 Expense on Raw material in 2013
= 6000000 + 30% of6000000
XIO
--=2 =
10010 780000
x 10 = 2 x100 10 Exp on all other exp in 2013
X = 2 X (102) 10
IO = 2400000+20% of2400000
= 2880000
XIO = 2 X 1020
20 Total cost in 2013
X =�2x l 0 =7.177
= 7 .2 approximately 780000 + 2880000 = 36,60,000
=

Percentage increase in the total cost for


Another method company in 2013
For Exponential growth 3660000 - 3000000
= X lOO=22%
p =p ek(1.-1._1) 3000000
n n-1

�ydcrabadlDclhilBhopaJ!Punc!Bhubaneswarl LucknowlPatna!BcngalwuiChcnnailVijayawadalVizag ITirupari I Kukatpallyi Kolkata I


....t"
l.,
w • • ACE
• •
,. ..,&:ipw,qPnNirafima : 1049: Numerical Ability

51. Ans: (b) 54. Ans: (d)


Sol: The sum of all 'n' digit numbers that can be Sol: X= { HHT, HHH}
formal by using 'n' distinct non zero digits. Y depends on X
= (n-1)! x(sum ofdigits)x(l 11....n times) Z = {TTH, TTT}
= (5-1)! X (1+3+5+7+9) X (11111) D is the correct choice.
= 4! X 25 X 11111
= 6666600 55. Ans: (c)
Sol: From given statements, the following
52. Ans: (d) diagrams are possible
Sol: The possible Venn diagrams from the given M = Manager
statements L = Leader
E = Executives
Playback singers

Film directors
II First Choice
Film Stars

From this diagram, the conclusions I and II


are Logically follows. Second Choice
Therefore Both conclusions I and II follows.

53. Ans: (c)


Sol:
log tan 1°+ log tan 2° + ............ + log tan 8 9°
log tan 1° + log tan 8 9° = log (tan 1° +tan 8 9°)
= log (tan 1° +cos ( 90 - 8 9)) • From the first choice of diagram, Both
= log (tan 1° +cot 1°) conclusions I and II are follows but from
= log 1° • second choice of diagram the manager may
=O be executive
Using this same logic total sum is 'O' Therefore option 'C' is correct.

\( 'I l 11g1 i Illll!, P111i 1 1 1 �ydcrabad I Delhi I Bhopal I Punc I Bhubancswarl Lucknow! Patna! Bcnga)uru I Chcrmai JVuayawadaJVizag I Tarupati I Kukaq,ally I Kolkat& I
: 1050: General Aptitude

56. Ans: 280 57. Ans: (c)


Sol: PS : QS = 3:1 Sol:
Tuberculosic
RT :QT = 5: 3 I +
PU : UR = 1:1 Afected Infected
The area of QTS triangle = 20 cm2 50% 50%

Develop the disease Does not


30% of50% Develop the disease
70% of50%

70 50 35
= - x- = - = 351/o
0

100 100 100

58. Ans: (c)


Let area of triangle P Q R be 'A' 1 1 1 � a
l q -a =-
S0: q =r
qa r
�-=-
SQ 1 1
1 1 =1 � = b
PQ 1 + 3 4 r =- �- - s r
-=--=-
-b

s rb s
QT 2 2
S =-1 �-=-
1 1 �S c =
QR 2 +5 7 q
-=--=-
q Sc q
-C

:. Area of /l1e QTS = _!_ x SQxQT q a =r �(s = r �s ac =r


Cr
2
(sac)b = s
!
= �x( xPQ) x(f QR) Sabe = S I

= .!._x�x xPQxQR]
4 7 2
[.!.. :. abc = 1

59. Ans: (c)


= _!_xArea of /l 1e
PQR Sol: Q can finish the task = 25 days, 12 hrs/day
14
1
= 300 hrs, 1 hr = -th
= _!_ x A = 20 cm2 300
14 R can finish the task = 50 days, 12 hrs/day
A = 280 cm2 = 50 X }2
1
= 600 hrs, 1hr= -th
600
Q working hour��(7 - 2) x 12 = 60 hrs

\( I l r11
.
.;1!1t, !II!..., P1ddu.1!J1)/1-, �ydcrabadlDclhilBbopailPunelBh�bancswarl wcknowlPatnalBengaiurulChennailiV� wadalV�I
>"'
"
�.,
':i.
" . ACE . . .
�,,.,,...,..Nllirmrm : 1051: Nwnerical Ability

R working hours => 7 x 18 = 126 hrs µ2 -cr2 = 85 - 5 = 80


After 7 days, the ratio of work done by Q µ2 + cr2 = 85 + 5 = 90
and R Range of Q in one standard deviation is 80 to
Q R 90
60 126
300 600 Distribution of 'Q'
20 : 21 68% within one standard
deviation of Q
60. Ans: (c)
Sol: Group 'P'
Mean (µ) = 105 80 85 90
Standard deviation (cr 1) = 25 68% within one standard deviation of Q is
Pr (µ - cr � x � µ + cr) ;;:; 0. 6827 narrower
:. 68% within one standard deviation . ·. 68% within one standard deviation of Q
µ, - cr 1 = 105 - 25 = 80 means most students of group Q.
µ 1 + cr 1 = 105 + 25 = 130 :. Most students of group 'Q' scored marks
:. range = 80 to 130 in a narrower range than students in group
'P'

Distribution of P:
61. Ans: (c)
68% within one standard deviation of P Sol: The following letter series is in the order of
even letters series
+4 +2 +4 +2 +2 +4

B, F H, L N P,------- TVXZ

62. Ans: (a)


Sol: The binary operation D is defined
GroupQ
Mean (µ2) = 85 => a D b = ab + (a+b)
Standard deviation (cr 2) = 5 aOx = a
:. From the equation 'b'is the variable
Pr (µ - cr � x � µ + cr) ;;:; 0.6827
Option A: x = 0
. ·. 68% within one standard deviation
a Do = axO + (a + 0) = 0 + a = a
.
"".,,,.,.� ACE
.
':.. ��Poblii::Pwll
. . : 1052: General Aptitude

Option B: x = 1 66. Ans: (c)


aD 1 = a x 1 + (a + 1) = a + a + 1 = 2a + 1 Sol: From the given data, the following diagram
Option C: x = 2 is possible
AD 2 => ax 2 + (a + 2) = 2a + a + 2 = 3a+ 2
Option D: x = 10
a D 10 =>ax 10 + (a + 10) = 10a + a + 10 +-- -10km
= l l a + lO A - C---

.·. Option 'A'only True. :s1cm


5km

63. Ans: (c)

64. Ans: (d)


"
Di - 45cr--------�E

4km
:
:

Sol: Read books = n(R) = 12 + 44 + 7 + 13 = 76


��

DE
Play sports = n(s) = 44 + 7 + 17 +15 = 83 cos45 ° =-
4
n (R nS) = 44 + 7 = 51 DE = cos45°x 4 = 2.828 km
n (R u S) = n (R) + n (S) - n (R nS) . EN
sm 450 =-
= 76 + 83- 51 = 108
EN = sin 45° x 4 = 2.828 km
65. Ans: (d)
CN = NE + CE = 2.828 + 5
Sol: .Time back = 2 : = 2 hrs 15 min = 7.828 km

Clock time (C.T) + Mirror Time (M.T) = 12 CB = AB - AC = 10 -2.828


,J, = 7.171 km
69' 1.30 (NB) = (NC) + (BC)2
2 2

:. C.T 12.00
= = (7.828)2 + (7.171)2
1.30
10.30 :. NB = �(7.828) + (7.171)
2 2

= 10.616 km
:. NM = NB + BN = 10.616 + 10
.·. The actual time shown by the clock
= 20.61 km
= 10.30 + 2.15 = 12.45
: 1053: Nwnerical Ability

67. Ans: (b)


Another method:
Sol: Length of the wire = 3 40m
Elimination procedure from alternatives
2x option [C] and [D] are not possible because
3
area may be ma ximum.
X
3
Option (a)
Side of the square = x = 30mm
Square Rectangle Perimeter of the square = 3 0+ 3 0+ 3 0 + 3 0
= 1 20mm

2x . ·. Perimeter of the rectangle = 3 40- 1 20


Perimeter of rectangle = 2[; + J
3 = 220mm
= 2x 2x + 2 x 2x = 220
Perimeter of square = 3 40- 2x x = 37
Side of square = 3 40- 2x 2x = 37 x 2= 7 4
4 Area of square = x2 = (30)2 = 9 00
Total area = Area of square + Area of Area of rectangle = x x 2x = 37 x 7 4= 27 38
rectangle Total area = 9 00+ 27 38 = 36 38 mm2

r
=[3 40-2x] +� x 2x
2

4 3 3 Option (b)
340 2x 2 ' Side of the square = x = 40mm
Perimeter of the square = 3 40- 16 0
-[ ; + ;
= 18 0 mm
Combined area of square + rectangle
= minimum 2x + 2x 2x = 18 0mm
f(
' x) = 0 6 x = 18 0mm
3 40- 2x 2 x = 3 0mm
f(X ) = [--- ]
2

Area of the square = 40x 40= 16 00mm2


2
+-X
4 9
4 3 40- 2x = Area of the rectangle = 3 0x 2 x 3 0
f '( x )= x 2 _ 0 = 18 00mm2
9 4
4 2= :. Total area = 16 00+ 18 00= 3 400mm2
_!_ [3 40- 2x]
:. 3 400mm2 < 36 38 mm2
X
9 4
= x = 90 Option B is correct.
. 3 40-2x
Side of square = ---= 40mm
4
!M�l!MMYIIMM���l�IBhopallPupeJBh��l�
. �urul<llcnnailVuaya�l�� IT�!J{ukalpa)lyl,� !.
:.'\

'!,�
I

. .t\CE . .
: 1054: General Aptitude
=======================================
�:�Publir*n
1
=- xpx/
68. Ans: (d) X

Sol: The amounts invested in the companies of, P 2 2


and Q in 2006=8 : 9 1 X

The rate of interest of company 'P' in 2006


=-X X X-
2 2
=6%
The rate of interest of company 'Q' in 2006 4
=4% =0.25x2

x/4 /
I 1/"'
x/4
� �
The amounts received after one year by P
and Q companies in 2006 year. 71. Ans: (c)
p Q Sol: Ananth takes 6 hours to read a book
6% of 8 4% of 9 Bharath takes 4 hours to read a book
6 4 L.C.M=12
-x8 -x9
100 100 The number of pages read by Ananth and
4 3 Bharath must be 12 (or) multiple of 12 only.
If Ananth read 12 number of pages in 6 hrs
69. Ans: (b) 12
1 hr= - =2 pages
Sol: From given facts, the following Venn 6
diagram is possible. If Bharath read 12 number of pages in 4 hrs
H=Humans 12
1 hr= - =3 pages
M=Mammals 4
E=Engineers
BH=Build houses Option 'a'
:. From above diagram, statement III is true. I hrs

Ananth Bharath
70. Ans: (d)
Sol: Base perimeter of square pyramid= x=p
/\
Read Not read
/\
Read Not read
2 pages IO pages 3 pages 9 pages

Slant height = �=l


2 10: 9

Lateral surface area of pyramid


=_!_x Base perimeter (p) x slant height (l) :. it is not
2

\( I l 11.!,ttu t 1111!.!. P11hl1t ,1t1,111-.


: 1055: Numerical Ability

Option 'b' 2hrs


73. Ans: (c)
Sol: Contour lines can be observed to cross
region with height from P to Q is as follows
Ananth

A
Bharath

Read Not read


A
Read Not read
4 pages 8 pages 6 pages 6 ages
(P)550 500 575 550(Q)
8 : 6

:. it is not
4: 3
Down Up Down

Options 'c'
3 hrs
:. The path from P to Q is Down-Up-Down
option ( c) is satisfies this path
Ananth Bharath

Read not Read 74. Ans: (a)


Sol: Length of truck + gap required=10 + 20
Read not Read
6 pages 6 pages 9 pages 3 pages

2 :I =30 m
Length of car + gap required =5 + 15
After 3 hours is the number of pages to be
=20 m
read by Ananth, twice that to be read by
Total distance is need for truck and car for
Bharath.
passing alternatively=30 + 20=50 m
72. Ans: (a) Given, speed=36 kmph
Sol: From the given data, eight persons are seated =36x2-=10 m/sec
around a circular table as follows 18
Let 'x' be the number of repetitions of
Y T V S U Y (Truck + car) in one hour
(or) (or) 50xx =
lO m/s
V T Y Y U S 60x60
10x60x60
x =----
S-Z--T y
50
=720 numbers of (Trucks + cars)

. ·. The maximum number of vehicles


=720 + 720=1440

:. X is third to the left of V


75. Ans: (a) 77. Ans: (d)
Sol: Sub group has at least one Indian means Sol: From the given data
minimum one Indian and maximum three 50000 5 0100
(or) more -48889 (or) - 48989
1111 0 1111
Sub groups containing only Indians
The sum of the missing digits m
= 3C,+3C +3C = 3 +3 + 1 = 7
2 3
subtraction = 0 + 0 + 0 +O+ 8 = 8
In the sub group one Indian and remaining (or) :. The sum of the missing digits m
are Chinese subtraction = 0 + 1 + 0 + O+ 9 = 10
= 3C [3C +3C +3C ] = 3 [3 + 3 + 1]
1 1 2 3
Both option (a) & (b) are possible. That's
= 3x7 = 21 why option (d) is correct.

In the sub group two Indians and remaining 78. Ans: (c)
are Chinese Sol: I x -11 � 2
= 3C2 [3C,+3C +3C ] = 3 [3 + 3 + 1]
2 3 => x-1 � 2 or x-1�-2
= 3x 7 = 21 => x � 3 or x � -1 ---------(1)
In the sub group three Indians and l y + 21 � 3
remaining are Chinese => y + 2 � 3 or y +2 � -3
= 3C3 [3C,+3C2+3C3 ]
or y �-5 --------(2)
= 1 [ 3 +3 + 1 ] = 7
S 1 plane can be obtained as area bounded by
lines x = 3, x = -1, y = 1 & y = -5
:. Total number of sub groups
= 7 +21 +21 + 7 = 56
Similarly, x=3

x-y � -2
76. Ans: (a)
1.e. y � x+2
Sol: From the graph statement (i) is correct
: . Area bounded by y y= I
• the time taken for curd formation @ 25 °C
= 1 20 min = x+2, y = 1 and x = 3 X

gives S2
• the time taken for curd formation @ 37°C
= 80 min
S = S, + S2
y = -5

:. Statement (ii) is not correct. 4x6+_!_4x4 = 32


2
=

\( l l ! J...,llll t I lllL: P11lib ,IIJcJIJ.., Fyderabad I Delhi I Bhopal I Punilitetmbaneswai:J Lucknow! Patna I Bcngaluru I Oiennai I Vuayawada I V�pati I KukatpallyJ Kolkata I
: 1057 : Numerical Ability

79. Ans: (c)


Sol: P, Q, R, S � Women 82. Ans: (a)
V, W, X, Y, Z � Men Sol: No. of bullocks = x

P is not to be paired with Z No. of tractors = y,


From the given data,
Y must necessarily be paired with some one.
(x + y) = 8 days,
Let Y be paired with P. The remaining makes
1
can be assigned as = 4C 1 x 3 C 1 x 2C 1 = 24 1 day (x+y) = - ------ (i)
8
Y is paired with Q
( ; + 2y) = 5 days,
Here Z cannot be paired with P.
There will be two more cases where Y is
paired with R and S.
1 day ( ; + 2y ) = ¼ ------ (ii)

The total no. of pairs 1


X +y=- ---- (i) X 2
8
= 24 + 1 8 + 1 8 + 1 8 = 78
1
2x + 2y = -
80. Ans: (d) 4
X 1
- + 2y = -
81. Ans: (b) 2 5
Sol: From the given graph ,
The difference between the maximum and X 1
2x - - = -
the minimum pollutant concentrations in the 2 20
winter = 8 - 0 = 8 ppm 4x - x
---= 1
The difference between the maximum and 20
the minimum pollutant concentrations in the 1
3x = -
10
summer = 1 0.5 - 1 .5 = 9 ppm
Over the given months, these differences are 1 th
X=-
not equal. 30
:. Therefore statement (i) is not correct. 1
One day work of x bullocks = - th
30
From the given graph, the statement (ii) is
:. x bullocks alone to plough the field = 30 days
correct.

!MIIMH9hi44hh4RbbiMiiii .yderabad j Delliij Bhopal j Pune j Bhubancswar l J�knowl Patna ! Bengaiuru j Chennai l Vuayawada j Vizag ! Ttrupali I Kubtpallyj Kolkala I
Chapter
2 Verbal Ability
04. Choose the most appropriate word from the
One Mark Questions options given below to complete the
following sentence: (ALL BRANCHES-IO)
O I . Which of the following options is the closest His rather casual remarks on politics __
in meaning to the word below: his lack of seriousness about the subject.
(ALL BRANCHES-IO)
(a) masked (b) belied
Circuitous
(c) betrayed (d) suppressed
(a) cyclic (b) indirect
(c) confusing (d) crooked 05. Choose the word from the options given
below that is most nearly opposite in meaning
02. The question below consists of a pair of to the given word. (EC/EE/INST-II)
related words followed by four pairs of Frequency
words. Select the pair that best expresses the (a) periodicity (b) rarity
relation in the original pair. (c) gradualness (d) persistency
(ALL BRANCHES -10)
UNEMPLOYED : WORKER : :
06. Choose the most appropriate word from the
(a) fallow land options given below to complete the
(b) unaware sleeper following sentence.
(c) wit jester Under ethical guidelines recently adopted
(d) renovated house by the Indian Medical Association, human
genes are to be manipulated only to correct
03. Choose the most appropriate word from the diseases for which ----- treatments
options given below to complete the are unsatisfactory. (EC/EE/INST-11)
following sentence: (a) similar (b) most
If we manage to our natural (c) uncommon (d) available
resources, we would leave a better planet for
our children. (ALL BRANCHES-IO) 07. Choose the most appropriate word from the
(a) uphold (b) restrain options given below to complete the
(c) cherish (d) conserve following sentence.

I I I I I I I I I
yderabad Delhi Bhopal Punc Bhubahcswar Lucknow Patna Bcnga)uru Cheonai Voayawada r'!{ 1 TIIUpati I Kukatpallyt;xoncata
: 1059 : Verbal Ability

It was her view that the country's Amalgamate


problems had been ____ by foreign (a) merge (b) split
technocrats, so that to invite them to come (c) collect ( d) separate
back would be counter-productive.
(EC/EE/INST-11) 11. Choose the most appropriate word( s) from the
(a) identified (b) ascertained options given below to complete the
( c) exacerbated ( d) analyzed following sentence.
(ME/CE/CSE/PI-11)
08. The question below consists of a pair of I contemplated ____ Singapore for
related words followed by four pairs of my vacation but decided against it.
words. Select the pair that best expresses the (a) to visit (b) having to visit
relation in the original pair: ( c) visiting ( d) for a visit
(EC/EE/INST-11)
GLADIATOR ARENA : : 12. Which of the following options is the closest
(a) dancer stage in meaning to the word below:
(b) commuter train (ME/CE/CSE/PI-11)
(c) teacher classroom Inexplicable
(d) lawyer courtroom (a) incomprehensible (b) indelible
( c) inextricable ( d) infallible
09. Choose the most appropriate word from the
options given below to complete the 13. Which one of the following options is the
following sentence. closest in meaning to the word given below?
If you are trying to make a strong (EC/EE/INST-12)
impression on your audience, you cannot Latitude
do so by being understated, tentative or (a) Eligibility (b) Freedom
(ME/CE/CSE/PI-11) ( c) Coercion ( d) Meticulousness
(a) hyperbolic (b) restrained
( c) argumentative (d) indifferent 14. One of the parts (A, B, C, D) in the sentence
given below contains an ERROR. Which one
10. Choose the word from the options given of the following is INCORRECT?
below that is most nearly opposite in meaning (EC/EE/INST-12)
to the given word. (ME/CE/CSE/PI-11)

�yderabad ! Delhi l Bhopal l Pune! Bhubaneswar l Lucknow! Patna l BengaJuru! Chennai lV\iayawada!Vizag ITllUpati ,Kukatpally l � ,
!lii•f@!lii@ijjflGiiii@4*
: 10 60 : General Aptitude

I requested that he should be given the 1 8. Choose the grammatically INCORRECT


driving test today instead of tomorrow. sentence: (ME/CE/CSE/PI-12)

(a) requested that (a) They gave us the money back less the

(b) should be given service charges of Three Hundred rupees.


(c) the driving test (b) This country's ex penditure is not less than
(d) instead of tomorrow that of Bangladesh.
(c) The committee initially asked for a
1 5. Choose the most appropriate alternative from funding of Fifty Lakh rupees, but later
the options given below to complete the settled for a lesser sum.
following sentence: (d) This country's ex penditure on educational
(EC/EE/INST-12) reforms is very less.
If the tired soldier wanted to lie down, he
19 . Choose the most appropriate alternative from
__ the mattress out on the balcony.
the options given below to complete the
(a) should take (b) shall take
following sentence: (ME/CE/CSE/Pl-12)
(c) should have taken (d) will have taken
Suresh' s dog is the one ___ was hurt in

16. Choose the most appropriate word from the the stampede.

options given below to complete the (a) that (b) which


following sentence: (c) who (d) whom

Given the seriousness of the situation that


20 . Choose the most appropriate alternative from
he had to face, his __ was impressive.
the options given below to complete the
(EC/EE/INST-12)
following sentence: (ME/CE/CSE/PI-12)
(a) beggary (b) nomenclature
Despite several __the mission succeeded
(c) jealousy (d) nonchalance
in its attempt to resolve the conflict.

1 7. Which one of the following options is the (a) attempts (b) setbacks
closest in meaning to the word given below? (c) meetings (d) delegations
(ME/CE/CSE/PI-12)
Mitigate 21 . They were requested not to quarrel with
(a) Diminish (b) Divulge others.
(c) Dedicate (d) Denote Which one of the following options is the
closest in meaning to the word quarrel?

l1ti•4i@jjji4i!i44Mi@hih+Yderabad I Delhi I Bhopal I Pune I Bhubanesw.u- 1 Lucknow! Patna I Bengalwu I Chennai I Vliayawadal V� I Tuupari I Kukatpally I _ Kolkata I
ACE
��-�:&,inr,:rq Pnbliariooa
� �============= : 1061 : Verbal Ability
=====================
(EC/EE/INST-13) 26. Were you a bird, you ____ in the sky.
(a) make out (b) call out (ME/CSE/PI-13)
(c) dig out (d) fall out (a) Would fly (b) Shall fly
(c) Should fly (d) Shall have flown
22. Complete the sentence:
Dare ____ mistake. (EC/EE/INST-13) 27. Which one of the following options is the
(a) commit (b) to commit closest in meaning to the word given below?
(c) committed (d) committing (ME/CSE/PI-13)
Nadir
23. Choose the grammatically CORRECT (a) Highest (b) Lowest
sentence: (EC/EE/INST-13) (c) Medium (d) Integration
(a) Two and two add four
(b) Two and two become four 28. Choose the grammatically INCORRECT
(c) Two and two are four sentence: (ME/CSE/PI-13)
(d) Two and two make four (a) He is ofAsian origin.
(b) They belonged to Africa.
24. Statement: You can always give me a ring (c) She is an European.
whenever you need. (d) They migrated from India to Australia.
Which one of the following is the best
inference from the above statement? 29 . Choose the most appropriate phrase from the
(EC/EE/INST-13) options given below to complete the
(a) Because I have a nice caller tune. following sentence.
(b) Because I have a better telephone facility The aircraft ___ take off as soon as its
(c) Because a friend in need is a friend indeed fight plan was filed.
(d) Because you need not pay towards the (GATE-EC/ME-14-Set-l)
telephone bills when you give me a ring (a) is allowed to
(b) will be allowed to
25. Complete the sentence: (c) was allowed to
Universalism 1s to particularism as (d) has been allowed to
diffuseness is to (ME/CSE/PI-13)
(a) Specificity (b) Neutrality 30. Choose the statement where underlined word
(c) Generality (d) Adaptation is used correctly. (CSIT - 2015)

yderabad I Delhi I Bhopal I Pune I Bhubaneswar I Lucknow I Patna I Bengaluru I Chennai I Vijayawada ! Vizag I Tuupati I· �y I Kolkata
" v
':.-� •
ACE
• _r_ • : 1062:
� �============
�� ==============================
Pnhnnma General Aptitude

(a) The industrialist had a personnel jet. 34. Choose the most appropriate pair of words
(b) I write my experience in my personnel from the options given below to complete the
diary. following sentence.
(c) All personnel are being given the day off. She could not__the thought of __the
(d) Being religious is a personnel aspect. election to her bitter rival.
(GATE-EC/ME-14-Set-2)
31. Choose the most appropriate word from the (a) bear, loosing (b) bare, loosing
options given below to complete the (c) bear, losing (d) bare, losing
following sentence.
Many ancient cultures attributed disease to 35. "India is a country of rich heritage and
supernatural causes. However, modern cultural diversity".
science has largely helped ___ such Which one of the following facts best
notions. supports the claim made in the above
(GATE-EC/ME-14-Set-1) sentence?
(a) impel (b) dispel (GATE-EC/ME-14-Set-3)
(c) propel (d) repel (a) India is a union of 29 states and 7 union
territories.
32. Choose the most appropriate word from the (b) India has a population ofover 1 .1 billion.
options given below to complete the (c) India is home to 22 official languages and
following sentence. thousands ofdialects.
Communication and interpersonal skills are (d) The Indian cricket team draws players
__important in their own ways. from over ten states.
(GATE-EC/ME-14-Set-2)
(a) each (b) both (c) all (d) either 36. The value of one U.S. dollar is 65 Indian
Rupees today, compared to 60 last year. The
33. Which of the options given below best Indian Rupee has___
completes the following sentence? (GATE-EC/ME-14-Set-3)
She will feel much better ifshe (a) depressed (b) depreciated
(GATE-EC/ME-14-Set-2) (c) appreciated (d) stabilized
(a) will get some rest
(b) gets some rest 37. 'Advice' is -----
(c) will be getting some rest (GATE-EC/ME-14-Set-3)
(d) is getting some rest (a) A verb (b) A noun
(c) An adjective (d) Both a verb and a noun
\( l l 11�11wt 1111c, Pul,lu .1111111, yderabad I Delhi I Bhopal I Pune I Bhubaneswar I Lucknow I Patna I Bengaluru I Chennai I Vijayawada I ;\f'izag Tirupari I Kukatpal]y j 1¥,lkata
ACE : 1063: Verbal Ability
� E�PuNirat',ooa
��-�
'====================================
38. Which of the following options is closest in 42. Choose the most appropriate word from the
meaning to the word underlined in the options given below to complete the
sentence below? following sentence.
In a democracy, everybody has the freedom to He could not understand the judges awarding
disagree with the government. her the first prize, because he thought that her
(GATE-EC/ME-14-Set-4) performance was quite___
(a) dissent (b) descent (a) superb (b) medium
(c) decent (d) decadent (c) mediocre (d) exhilarating

39. After the discussion, Tom said to me, 'Please 43. In a press meet on the recent scam, the
revert! ' . He expects me to _____ minister said, "The buck stops here". What
(GATE-EC/ME-14-Set-4) did the minister convey by the statement?
(a) retract (b) get back to him (GATE-14-Set-l)
(c) move in reverse (d) retreat (a) He wants all the money
(b) He will return the money
40. While receiving the award, the scientist said, (c) He will assume final responsibility
"I feel vindicated". Which of the following is (d) He will resist all enquiries
closest in meaning to the word 'vindicated'?
(GATE-EC/ME-14-Set-4) 44. Choose the most appropriate phrase from the
(a) punished (b) substantiated options given below to complete the
(c) appreciated (d) chastened following sentence. (GATE-14-S2)
India is a post-colonial country because
(a) It was a former British colony
4 1 . Which of the following options is the closest
(b) Indian Information Technology
in meaning to the phrase underlined in the
professionals have colo�i41P!tf-Wlj
sentence below?
( c) India does not follow any colonial
It is fascinating to see life forms cope with
practices
varied environmental conditions.
(d) India has helped other countries gam
(a) adopt to (b) adapt to
freedom
(c) adept in (d) accept with

!11i1ijj§jjjl4.jjdAfll!Mijj.jjlj yderabad I Delhi I Bhopal I Pune I Bhubaneswar I Lucknow I Patna I Bengalwu I Cbcnnai ! Vuayawada I Vizag I Tuupa1{ 1' Kukatpal)y I Ko1b1a
45. Who ___ was coming to see us this (a) Contentment (b) Ambition
evening? (GATE-14-S2) (c) Perseverance (d) Hunger
(a) you said (b) did you say
( c) did you say that (d) had you said 49. Which of the following options is the closest
in meaning to the sentence below?
46. Match the columns. (GATE-14-S2) " As a woman, I have no country"
Column 1 Column 2 (GATE -14-CE/IN/PI- set 2)
1 . Eradicate P. misrepresent (a) Women have no country.
2. Distort Q. soak completely (b) Women are not citizens of any country.
3. Saturate R. use (c) Women's solidarity knows no national
4. Utilize S. destroy utterly boundaries.
(d) Women of all countries have equal legal
Codes:
rights.
(a) 1 : S, 2 : P, 3 : Q, 4 : R
(b) 1 : P, 2 : Q, 3 : R, 4 : S
50.
(c) 1 : Q, 2 : R, 3 : S, 4 : P
The professor ordered to the students to go out of the class
(d) 1 : S, 2 : P, 3 : R, 4 : Q I II III IV
Which of the above underlined parts of the
4 7. Choose the most appropriate word from the
sentence is grammatically incorrect?
options given below to complete the
(GATE - 13)
following sentence.
(GATE -14 -CE/IN/PI- Set 2) (a) I (b) II (c) III (d) IV
A person suffering from Alzheimer's disease
____ short-term memory loss. 5 1 . Which of the following options is the closest
(a) experienced (b) has experienced in meaning to the word given below: Primeval
(c) is experiencing (d) experiences (GATE - 13)
(a) modern (b) Historic
48. Choose the most appropriate word from the (c) Primitive (d) Antique
options given below to complete the
following sentence. 52. Friendship, no matter how ___it is, has
(GATE -14 -CE/IN/PI- set 2) its limitations. (GATE - 13)
___ is the key to their happiness; they are (a) cordial (b) intimate
satisfied with what they have. (c) secret (d) pleasant

\( I I ! l l.!, l l l l I r 1 1 1� P1il,l11 .111 1 11 1 .., yderahad ! Dclhi j Bhopal ! Pune j Bhubaneswar l Lucknowj Patna! Bengaiwu ! Chennai l Vtjayawada j Vizag T1I11pati I Kukatpally l Kolkata
'"'
:..\w •ACE • • •
"
"� ," r..,.,...,..PnNir.afiooa : 1065 : Verbal Ability

53. Select the pair that best expresses a (a) Rajan has decided to work only in a
relationship to that expressed in the pair: group.
(GATE - 13) (b) Rajan and Sajan were formed into a group
MEDICINE HEALTH .. against their wishes.
(a) Science Experiment (c) Sajan had decided to give in to Raj an's
(b) Wealth Peace request to work with him.
(c) Education Knowledge (d) Rajan had believed that Saj an and he
(d) Money Happiness would be working together.

54. A student is required to demonstrate a high 57. Choose the statement where underlined word
level of comprehension of the subject, is used correctly. (CE - 2015)
especially in the social sciences. (a) The minister insured the victims that
The word closest m meaning to everything would be all right.
comprehension is (GATE -14- set 1) (b) He ensured that the company will not
(a) understanding (b) meaning have to bear any loss
(c) concentration (d) stability (c) The actor got himself ensured against any
accident
55. Choose the most appropriate word from the (d) The teacher insured students of good
options given below to complete the results
following sentence.
One of his biggest __ was his ability to 58. Which word is not a synonym for the word
forgive. (GATE -14- set 1) vernacular? (CE - 2015)
(a) vice (b) virtues (a) regional (b) indigenous
(c) choices (d) strength (c) indigent (d) colloquial

56. Rajan was not happy that Sajan decided to do 59. Choose the most appropriate word from the
the project on his own. On observing his options given below to complete the
unhappiness, Sajan explained to Rajan that he following sentence. (CE - 2015)
preferred to work independently. The official answered ___ that the
Which one of the statements below is complaints ofthe citizen would be looked into
logically valid and can be inferred from the (a) respectably (b) respectfully
above sentences? (GATE -14- set 1) (c) reputably (d) respectively

IM•li!@Oii4@QAflflfMh1h.j yderabad I Delhi I Bhopal I Pune I Bhubaneswar I Lucknow! Pama ! Bengalwu I Chennai I Vtjayawada I Vizag I T� }(�Y I �
: 1066 : General Aptitude

60. The word similar in meaning to 'dreary' is 64. Which of the following options is the closest
(CE - 2015) in meaning to the sentence below?
(a) cheerful (b) dreamy (EE/CSIT - 2015)
(c) hard (d) dismal She enjoyed herself immensely at the party.
(a) She had a terrible time at the party
6 1 . Select the paIT that best expresses a (b) She had a horrible time at the party
relationship similar to that expressed in the (c) She had a terrific time at the party
pair: (CE, CSIT - 2015) (d) She had a terrifying time at the party
Children : Pediatrician
(a) Adult : Orthopaedist 65. Which one of the following combinations is
(b) Females : Gynaecologist incorrect?
(c) Kidney : Nephrologist (EE/CSIT - 2015)
(d) Skin : Dermatologist (a) Acquiescence - Submission
(b) Wheedle - Roundabout
62. Extreme focus on syllabus and studying for
( c) Flippancy - Lightness
tests has become such a dominant concern of
(d) Profligate - Extravagant
Indian students that they close their minds to
anything ___ to the requirements of the 66. Didn't you buy when you went
exam. (CE, CSIT-2015) shopping? (EE/CSIT - 2015)
(a) related (b) extraneous (a) any paper (b) much paper
(c) outside (d) useful (c) no paper (d) a few paper

63. The Tamil version of --- John Abraham-


67. Select the alternative meaning of the
starrer Madras Cafe cleared by the
underlined part of the sentence.
Censor Board with no cuts last week, but the
(EE/CSIT - 2015)
film's distributors ___ no takers among
The chain snatchers took to their heels when
the exhibitors for a release in Tamil Nadu
the police party arrived.
this Friday.
(a) took shelter in a thick jungle
(CE, CSIT- 2015)
(a) Mr., was, found, on (b) open indiscriminate fire
(b) a, was, found, at (c) took to flight
(c) the ,was, found, on (d) unconditionally surrendered
(d) a, being , find, at

!IJ11iMjjji4.jji@njjjjjfjj� yderabad l Dellri l Bhopal ! Pune l BhubaneswaFJ Lucknow�P:unaJ BengaJuru l Chennai l Vi,iayawadajV� J'"lfllpati I Kukatpally l,Kplkata
: 1067 : Verbal Ability

68. We ____ our friend's birthday and we (a) momento (b) memento
____ how to make it up to him. ( c) momentum (d) moment
(EE/CSIT -2015)
(a) completely forgot -- don't just know 72. Choose the appropriate word/phrase, out of
(b) forgot completely - - don't just know the four options given below, to complete the
( c) completely forgot -- just don't know following sentence:
(d) forgot completely-- just don't know Frogs____ (EC-2015)
(a) Croak (b) Roar
69. A generic term that includes various items of
(c) Hiss (d) Patter
clothing such as a skirt, a pair of trousers and
a shirt is (EE/CSIT -2015)
7 3. Choose the word most similar in meaning to
(a) fabric (b) textile
the given word: (educe) (EC-2015)
(c) fibre (d) apparel
(a) Exert (b) Educate
( c) Extract ( d) Extend
70. Out of the following four sentences, select the
most suitable sentence with respect to
74. The following question presents a sentence,
grammar and usage: (EE/CSIT - 2015)
part of which is underlined. Beneath the
(a) Since the report lacked needed
sentence you find four ways of phrasing the
information, it was of no use to them.
underlined part. Following the requirements
(b) The report was useless to them because
of the standard written English, select the
there were no needed information in it.
answer that produces the most effective
( c) Since the report did not contain the
sentence.
needed information, it was not real useful
Tuberculosis, together with its effects, ranks
to them.
one of the leading causes of death in India.
( d) Since the report lacked needed
information, it would not had been useful
(EC-2015)
(a) ranks as one of the leading causes of
to them.
death.
71. Choose the most appropriate word from the (b) rank as one of the leading causes of death.
options given below to complete the ( c) has the rank of the leading causes of
following sentence. (EC-2015) death.
The principal presented the chief guest with a ( d) are one of the leading causes of death.
____, as token of appreciation.
�yderabad l Delhi l Bhopal ! Pune j Bhubaneswarl Lucknowl Patna j Bengaluru j Chennai l VtiayawadajVizag ITiru� f Kukatpallyi Kolkalal
.,...
� �.... . ACE
. . . : 1068 : General Aptitude
�.. _ :F.Dgioeamg Pnbliranona

75. Choose the word most similar in meaning to 79 . Choose the most suitable one word substitute
the given word? (EC-2015) for the following ex pression: (EC-2015)
Awkward Connotation ofa road or way
(a) Inept (b) Graceful (a) Pertinacious (b) Viaticum
(c) Suitable (d) Dreadful (c) Clandestine (d) Ravenous

76. Choose the appropriate word/phrase, out of 80. Choose the most appropriate word from the
the four options given below, to complete the options given below to complete the
following sentence: (EC-2015) following sentence. (EC-2015)
Dhoni, as well as the other team members of Ifthe athlete had wanted to come first in the
Indian team, _____.present on the race, he ___ several hours every day.
occasion (a) should practise
(a) were (b) was (b) should have practised
(c) has (d) have (c) practised
(d) should be practising
77. What is the adverb for the given word below?
(EC-2015)
81. Choose the correct verb to fill in the below:
MISOGYNOUS
(EC-2015)
(a) Misogynousness (b) Mysogynity
Let us ---
(c) Misogynously (d) Misogynous
(a) introvert (b) alternate
78. In the following sentence certain parts are (c) atheist (d) altruist
underlined and marked P, Q, and R. One of
the parts may contain certain error or may not 82. Ram and Shyam shared a secret and promised
be acceptable m standard written to each other it would remain between them.
communication. Select the part containing an Ram ex pressed himself in one of the
error. Choose D as your answer ifthere is no following ways as given in the choices below.
error. (EC-2015) Identify the correct way as per standard
English. (EC-2015)
The student corrected all the errors that the
(a) It would remain between you and me.
instructor marked on the answer book.
(b) It would remain between I and you.
p Q R
(c) It would remain between you and I.
(a) p (b) Q (c) R (d) No Error
(d) It would remain with me.

IM11iii§Oii4jjji4Rffb#iijjj� yderabad I Delhi I Bhopal I Pune I Bhubaneswari Lucknow l f,una I Bengaluru I � I Vtjayawacla fVizag-J Tirupati I Kukatpally I Kolkata
ACE : 1069 :
��-t:�Poolicatm Verbal Ability
� � ===================== ===============
83. Didn't you buy ____ when you went 87. Select the appropriate option in place of
shopping? (EE-2014) underlined part ofthe sentence.
(a) any paper (b) much paper
Increased productivity necessary reflects
(c) no paper (d) a few paper
greater efforts made by the employees.
(IN/ME-2015)
84. Choose the appropriate word/phrase, out of
(a) Increase in productivity necessary
the four options given below, to complete the
(b) Increase productivity is necessary
following sentence: (IN/ME-2015)
(c) Increase in productivity necessarily
Apparent lifelessness ----dormant life (d) No improvement required.
(a) harbours (b) leads to
(c) supports (d) affects 88. Choose the appropriate word/phrase, out of
the four options given below, to complete the
85. Choose the statement where underlined word following sentence:
is used correctly. (ME-2015)
(IN/ME-2015) Dhoni , as well as the other team members of
(a) When the teacher eludes to different Indian team, present on the
authors, he is being elusive. occas10n.
(b) When the thief keeps eluding the police, (a) were (b) was
he is being elusive. (c) has (d) have
(c) Matters that are difficult to understand,
identify or remember are allusive. 8 9. Choose the word most similar in meaning to
(d) Mirages can be allusive. but a better way the given word: (ME-2015)
to express them is illusory. AWKWARD
(a) Inept (b) Graceful
86. Fill m the blanks with the correct (c) Suitable (d) Dreadful
idiom/phrase. (IN/ME-2015)
That boy from the town was a ___ in the 90. In the following sentence certain parts are
sleepy village. underlined and marked P,Q and R. One of the
parts may contain certain error or may not be
(a) dog out ofherd
acceptable m standard written
(b) sheep from the heap
communication. Select the part containing an
(c) fish out ofwater
error. Choose D as your answer if there is no
(d) bird from the flock
error. (ME-2015)
. -
.•
.

I �,. yderabad I Delhi I Bhopal I Punc I Bhu�i;s;..... I Lucl/nqw I Patna! Bengaluru I Chennai I Vrjayawada l Vu.ag I Tiru'pati i Kukatpally I Kolkata
n!M@jjef
Iltl•l!i§Oii4ihjj@
ACE : 1070 : General Aptitude
:.�
'!i :F� PoNiat'ffll
� �==================================
94. The policeman asked the victim of a theft,
The student corrected all the errors that the
"What did you __?" (ME -16) (Set 1)
instructor marked on the answer book.
(a) loose (b) lose
p Q R
(c) loss (d) louse
(a) p (b) Q (c) R (d) No error
95. Despite the new medicine's ____ 1Il
9 1 . Choose the most suitable one word substitute treating diabetes, it is not ___ widely
for the following expression: (ME - 16) (Set 1)
(ME-2015) (a) effectiveness ---- prescribed
Connotation of a road or way (b) availability ---- used
(a) Pertinacious (b) Viaticum (c) prescription ---- available
(c) Clandestine (d) Ravenous (d) acceptance ---- proscribed

92. Choose the most appropriate word from the 96. The volume of a sphere of diameter 1 unit is
options given below to complete the than the volume of a cube of side 1
following sentence unit.
If the athlete had wanted to come first in the (ME -16) (Set 2)
race, he ____ several hours every day. (a) least (b) less
(ME-2015) (c) lesser (d) low
(a) should practise
97. The unruly crowd demanded that the
(b) Should have practiced
accused be without trial.
(c) Practised
(ME - 16) (Set 2)
(d) Should be practicing
(a) hanged (b) hanging
(c) hankering (d) hung
93. Which of the following is CORRECT with
respect to grammar and usage?
98. Choose the statement(s) where the
Mount Everest is __ (ME -16) (Set 1)
underlined word is used correctly:
(a) the highest peak in the world
(i) A prone is a dried plum.
(b) highest peak in the world
(ii) He was lying prone on the floor.
(c) one of highest peak in the world
(iii) People who eat a lot of fat are Prone to
(d) one of the highest peak in the world
heart disease.
(ME -16) (Set 2)
: 1071: Verbal Ability
(a) (i) and (iii) only 101. The students the teachers on teachers
(b) (iii) only day for twenty years of dedicated teaching.
(c) (i) and (ii) only (ME - 16) (Set 3)
(d) (ii) and (iii) only (a) facilitated (b) felicitated
( c) fantasized ( d) facillitated
99. Fact: If it rains, then the field is wet.
Read the following statements: 102. After India's cricket world cup victory in
(i) It rains 1985, Shrotria who was playing both tennis
(ii) The field is not wet and cricket till then, decided to concentrate
(iii) The field is wet only on cricket. And the rest is history.
What does the underlined phrase mean in
(iv) It did not rain
this context?
Which one of the options given below is
(ME - 16) (Set 3)
(a) history will rest in peace
NOT logically possible, based on the given
(b) rest is recorded in history books
fact? (ME -16) (Set 2)
( c) rest is well known
(a) If (iii), then (iv)
( d) rest in archaic
(b) If (i), then (iii)
( c) If (i), then (ii)
103. If I were you, I___ that laptop. It's
(d) If (ii), then (iv)
much too expensive (GATE - PI - 16 )
(a) won't buy (b) shan' t buy
1 00. Based on the given statements, select the
( c) wouldn' t buy (d) would buy
appropriate option with respect to grammar
and usage. (ME - 16) (Set 3) 104. He turned a deaf ear to my request. What
Statements does the underlined phrasal verb mean?
(i) The height of Mr. X is 6 feet. (GATE -PI - 16 )
(ii) The height of Mr. Y is 5 feet. (a) Ignored (b) appreciated
(a) Mr. X is longer than Mr. Y. (c) twisted ( d) returned
(b) Mr. X is more elongated than Mr. Y.
105. Choose the most appropriate set of words
(c) Mr. X is taller than Mr. Y.
from the options given below to complete
the following sentence.
(d) Mr. X is lengthier than Mr. Y.

--- is a will, --- is a way.


(GATE - PI - 16 )
!Ml•!Miliilihi4Wbi¥iiLl4 yderabad I Delhi I Bhopal I Pune I Bhubane,swar I �wl Patna I Bengaluru I Chennai I Vijayawada I Vizag I Tiru�1 � r� '
.,
�-�
" . �CE . . . : 1072 : General Aptitude

� �==========================================
�� Poblir,noos

(a) Wear, there, their 1 1 0. If you choose plan P, you will have to
(b) Were, their, there ___ plan Q, as these two are mutually
(c) Where, there, there
(d) Where, their, their (GATE-ME - 17)(SET -2}
(a) forgo, exclusive
1 06. I made arrangements had I (b) forget, inclusive
----- informed earlier. (c) accept, exhaustive
(GATE - PI - 17) (d) adopt, intrusive
(a) could have, been
(b) would have, being 1 1 1 . The ways in which this game can be played
(c) had, have __ potentially infinite.
(d) had been, been (GATE-ME - 17)(SET -2)
(a) is (b) is being
1 07. She has a sharp tongue and it can (c) are (d) are being
occasionally tum ____
(GATE - PI - 17)
(a) hurtful (b) left Two Marks Questions
(c) methodical (d) vital
0 1 . Wanted Temporary, Part-time persons for the
1 08. As the two speakers became increasingly post of Field Interviewer to conduct personal
agitated, the debate became ---- interviews to collect and collate economic
(GATE-ME - 17)(SET -1} data. Requirements: High School-pass, must
(a) lukewarm (b) poetic be available for Day, Evening and Saturday
(c) forgiving (d) heated work. Transportation paid, expenses
reimbursed.
1 0 9. He was one of my best ____ and I felt Which one of the following is the best
his loss ---- inference from the above advertisement?
(GATE-ME - 17)(SET -1) (ME/CE/CSE/PI-12)
(a) friend, keenly (a) Gender-discriminatory
(b) friends, keen (b) Xenophobic
(c) friend, keener (c) Not designed to make the post attractive
(d) friends, keenly (d) Not gender-discriminatory

llfllii@h/il@YRflflj@jj/jjj� ydcrabad I Delhi I Bhopal I Pune I Bhubaneswar I LucknowJ Patna I Bengaluru I Chennai I Vyayawada. I Vu.ag JTirupati I Kukatpal)y J Kolkara
: 1073 : Verbal Ability

02. Statement: There were different streams of (a) Failure is the pillar ofsuccess.
freedom movements in colonial India carried (b) Honesty is the best policy.
out by the moderates, liberals, radicals (c) Life begins and ends with- adventures.
socialists, and so on. (d) No adversity justifies giving up hope.
Which one of the following is the best 04. Lights of four colors (red, blue, green,
inference from the above statement? yellow) are hung on a ladder. On every step
(EC/EE/INST-13) ofthe ladder there are two lights. If one ofthe
(a) The emergence of nationalism in colonial lights is red, the other light on that step will
India led to our independence always be blue. Ifone ofthe lights on a step is
(b) Nationalism in India emerged in the green, the other light on that step will always
context ofcolonialism be yellow. (GATE-EC/ME-14-Set-2)
(c) Nationalism in India is homogeneous Which of the following statements is not
(d) Nationalism in India is heterogeneous necessarily correct?
(a) The number of red lights is equal to the
03. After several defeats in wars, Robert Bruce number ofblue lights
went in exile and wanted to commit suicide. (b) The number of green lights is equal to the
Just before committing suicide, he came number ofyellow lights
across a spider attempting tirelessly to have (c) The sum of the red and green lights is
its net. Time and again, the spider failed but equal to the sum of the yellow and blue
that did not deter it to refrain from making lights
attempts. Such attempts by the spider made (d) The sum ofthe red and blue lights is equal
Bruce curious. Thus, Bruce started observing to the sum ofthe green and yellow lights
the near-impossible goal of the spider to have
the net. Ultimately, the spider succeeded in 05. The Palghat Gap (or Palakkad Gap), a region
having its net despite several failures. Such about 30 km wide in the southern part of the
act of the spider encouraged Bruce not to Westem Ghats in India, is lower than the hilly
commit suicide. And then, Bruce went back terrain to its north and south. The exact
again and won many a battle, and the rest in reasons for the formation of this gap are not
history. clear. It results in the neighbouring regions of
Which one of the following assertions is best Tamil Nadu getting more rainfall from the
supported by the above information? South West monsoon and the neighbouring
(ME/CSE/PI-13) regions of Kerala having higher summer
temperatures.
yderabad I Delhi I Bhopal I Pune I Bhubaneswar I Lucknow I Patna! Bengaluru I Chcnnai I Vtjayawadaj Viz.ag I Tuupati I Kukalpa)ly I Kolkala
ACE
':.�-��,,......,,.PoNirJnoos
II 'I/fl • • • •
: 1074 : General Aptitude

What can be inferred from this passage? 07. The old city of Koenigsberg, which had a
(GATE-14-Sl) German majority population before World
(a) The Palghat gap is caused by high rainfall War 2, is now called Kaliningrad . After the
and high temperature in southern Tamil events of the war, Kaliningrad is now a
Nadu and Kerala Russian territory and has a predominantly
(b) The regions in Tamil Nadu and Kerala Russian population. It is bordered by the
that are near the Palghat Gap are low­ Baltic Sea on the north and the countries of
lying Poland to the south and west and Lithuania to
(c) The low terrain of the Palghat Gap has a the east respectively.
significant impact on weather patterns in Which of the statements below can be
neighbouring parts of Tamil Nadu and inferred from this passage?
Kerala (GATE-14-S2)
(d) Higher summer temperatures result in (a) Kaliningrad was historically Russian in its
higher rainfall near the Palghat Gap area ethnic make up
(b) Kaliningrad is a part of Russia despite it
06. Geneticists say that they are very close to not being contiguous with the rest of
confirming the genetic roots of psychiatric Russia
illnesses such as depression and (c) Koenigsberg was renamed Kaliningrad, as
schizophrenia, and consequently, that doctors that was its original Russian name
will be able to eradicate these diseases (d) Poland and Lithuania are on the route
through early identification and gene therapy. from Kaliningrad to the rest of Russia
On which of the following assumptions does
the statement above rely? (GATE-14-Sl) 08. The number of people diagnosed with dengue
(a) Strategies are now available for fever (contracted from the bite of a mosquito)
eliminating psychiatric illnesses in north India is twice the number diagnosed
(b) Certain psychiatric illnesses have a last year. Municipal authorities have
genetic basis concluded that measures to control the
(c) All human diseases can be traced back to mosquito population have failed in this
genes and how they are expressed region.
(d) In the future, genetics will become the Which one of the following statements, if
only relevant field for identifying true, does not contradict this conclusion?
psychiatric illnesses (GATE-14-S2)

Fyder.wad I
'>'
\('l. l 11i::;t11t t 1 111g P11hli1 ,1t11111 ... I Delhi I Bhopal I Pune I Bhuhaneswar I Lucknow I Patna I Bengaluru I Chennai I Vgayawada I Vmg IT11Upati I ��,'Kolkara
: 1075 : Verbal Ability

(a) A high proportion of the affected (GATE-14 - CE/IN/PI- Set 2)


population has returned from
neighbouring countries where dengue is 10. Alex ander turned his attention towards India,
prevalent since he had conquered Persia.
(b) More cases of dengue are now reported Which one of the statements below is
because of an increase in the Municipal logically valid and can be inferred from the
Office's administrative efficiency above sentence?
(c) Many more cases of dengue are being (a) Alex ander would not have turned his
diagnosed this year since the introduction attention towards India had he not
ofa new and effective diagnostic test conquered Persia.
(d) The number of people with malarial fever (b) Alex ander was not ready to rest on his
(also contracted from mosquito bites) has laurels, and wanted to march to India.
increased this year (c) Alex ander was completely in control of
his army and could command it to move
09 . Moving into a world of big data will require towards India.
us to change our thinking about the merits of (d) Since Alex ander's kingdom ex tended to
ex actitude. To apply the conventional mindset Indian borders after the conquest of
of measurement to the digital, connected Persia, he was keen to move further.
world of the twenty-first century is to miss a (GATE-15 - CE/CSIT)
crucial point. As mentioned earlier, the
obsession with ex actness is an artifact of the 11 . Most ex perts feel that in spite of possessing
information-deprived analog era. When data all the technical skills required to be a
was sparse, every data point was critical, and batsman ofthe highest order, he is unlikely to
thus great care was taken to avoid letting any be so due to lack of requisite temperament.
point bias the analysis. He was guilty of throwing away his wicket
From "BIG DA TA " Viktor Mayer- several times after working hard to lay a
Schonberger and Kenneth Cukier strong foundation. His critics pointed out that
The main point ofthe paragraph is: until he addressed this problem, success at the
(a) The twenty-first century is a digital world. highest level will continue to elude him.
(b) Big data is obsessed with exactness Which of the statement(s) below is/are
(c) Ex actitude is not critical in dealing with logically valid and can be inferred from the
big data above passage? (GATE-15 - CE/CSIT)
(d) Sparse data leads to a bias in the analysis
ydcrabad l Delhi ! BhoJJ3! 1 Pime ! Bhubancswarl Lucknow ! Patna ! Bcngaiuru ! Chcnnai !Vliayawada!Vu.ag l 'Iirupati I Kukalpally l Kolkala
: 1076 : General Aptitude

(i) He was already a successful batsman at 13. Humpty Dumpty sits on a every day while
the highest level. having lunch. The wall sometimes breaks. A
(ii) He has to improve his temperament m person sitting on the wall falls if the wall
order to become a great batsman breaks.
(iii)He failed to make many of his good Which one of the statements below is
starts count. logically valid and can be inferred from the
(iv)Improving his technical skills will above sentences? (GATE-15 -EC)
guarantee success. (a) Humpty Dumpty always falls while
having lunch.
(a) (iii) and (iv) (b) (ii) and (iii)
(c) (i), (ii) and (iii) (b) Humpty Dumpty does not fall sometimes
(d) (ii) only
while having lunch.
(c) Humpty Dumpty never falls during
12. The given statement is followed by some
dinner.
courses of action. Assuming the statement to
(d) When Humpty Dumpty does not sit on the
be true, decide the correct option.
(GATE-15 -CSIT/EE)
wall, the wall does not break.
Statement:
There has been a significant drop in the water 14. Read the following paragraph and choose the
correct statement.
level in the lakes supplying water to the city.
Climate change has reduced human security
Course of action:
and threatened human well being. An ignored
I. The water supply authority should impose
reality of human progress is that human
a partial cut in supply to tackle the
security largely depends upon environmental
situation.
security. But on the contrary, human progress
II. The government should appeal to all the
seems contradictory to environment security.
residents through mass media for minimal
To keep up both at the required level is a
use ofwater.
challenge to be addressed by one and all. One
III. The government should ban the water
ofthe ways to curb the climate change may be
supply in lower areas.
suitable scientific innovations, while the other
(a) Statements I and II follow
may be the Gandhian perspective on small
(b) Statement I and III follow
scale progress with focus on sustainability.
(c) Statements II and III follow
(GATE-15 - EC)
(d) All statements follow

crabad I Delhi I Bhopal I Pune I Bhubaneswar I Lucknow I Patna I Bengaluru I Chennai I Vuayawada I yiz.ag I TlfUpati Kolkata I
\ ( l l 111.�111l l 1 1 1 1g Pt1hlil ,1t11 111"
�yd I �I
ACE
: 1077 :
'!.�-�:&,pw.rq Pohliatioos Verbal Ability
J �======================= ===========�
(a) Human progress and security are 16. Ms. X will be in Bagdogra from 01/05/2014
positively associated with environmental to 20/05/2014 and from 22/05/2014 to
security. 31/05/2014. On the morning of 21/05/2014,
(b) Human progress is contradictory to she will reach Kochi via Mumbai.
environmental security. (GATE-15 -EC/ME)
(c) Human security 1s contradictory to Which one of the statements below is
environmental security. logically valid and can be inferred from the
(d) Human progress depends upon above sentences?
environmental security. (a) Ms. X will be in Kochi for one day, only
in May.
15. Lamenting the gradual sidelining of the arts in
(b) Ms. X will be in Kochi for only one day in
school curricula, a group of prominent artists
May.
wrote to the Chief Minister last year, asking
(c) Ms. X will be only in Kochi for one day in
him to allocate more funds to support arts
May.
education in schools. However, no such
(d) Only Ms. X will be in Kochi for one day
increase has been announced in this year's
in May.
Budget. The artists expressed their deep
anguish at their request not being approved,
17. In the following question, the first and the last
but many of them remain optimistic about
sentence of the passage are in order and
funding in the future.
numbered 1 and 6. The rest of passage is split
(GATE-15 -EC/ME)
Which of the statement(s) below is/are into 4 parts and numbered as 2, 3,4 and 5.
logically valid and can be inferred from the These 4 parts are not arranged in proper order.
above statements? Read the sentences and arrange them in
(i) The artists expected funding for the arts to logical sequence to make a passage and
increase this year choose the correct sequence from the given
(ii) The Chief Minister was receptive to the options. (GATE-15 -EC/ME)
idea of increasing funding for the arts 1. On Diwali, the family rises early in the
(iii)The Chief Minister is a prominent artist morning
(iv)Schools are giving less importance to arts 2. The whole family, including the young
education nowadays. and old enjoy doing this.
3. Children let off fireworks later in the night
(a)(iii) and (iv) (b) (i) and (iv)
with their friends.
(c) (i), (ii) and (iv) (d) (i) and (iii)
\( I l 11g111t t J Iil� P11hl1t ,i!Jt))\', Fyderabad i Delhi I Bhopal I Punc I Bhubaneswar l Lucknow! Patna I Bcngaluru I Chennai !Vgayawada!V,zag !Tirupati I Kukatpally i Kolkala I
: 1078 : General Aptitude

4. At sunset, the lamps are lit and the family all three like to play chess. Pavithra wins
performs various rituals. more often than Leela does.
5. Father, mother and children visit relatives Which one ofthe following statements must
and exchange gifts and sweets. be TRUE based on the above?
6. Houses look so pretty with lamps all (ME - 16) (Set 1)
around. (a) When Shiva plays chess with Leela and
(a) 2, 5, 3, 4 (b) 5, 2, 4, 3 Pavithra, he often loses.
(c) 3, 5, 4, 2 (d) 4, 5, 2, 3 (b) Leela is the oldest ofthe three.
(c) Shiva is a better chess player than
18. In a world filled with uncertainty, he was Pavithra
glad to have many good friends. He had (d) Pavithra is the youngest ofthe three
always assisted them in times of need and
was confident that they would reciprocate. 20. A smart city integrates all modes of
However, the events ofthe last week proved transport, uses clean energy and promotes
him wrong. sustainable use of resources. It also uses
Which of the following inference(s) is/are technology to ensure safety and security of
logically valid and can be inferred from the the city, something which critics argue, will
above passage? lead to a surveillance state.
(i) His friends were always asking him to Which of the following can be logically
help them inferred from the above paragraph?
(ii) He felt that when in need of help, his (i) All smart cities encourage the
friends would let him down. formation ofsurveillance states
(iii) He was sure that his friends would help (ii) Surveillance is an integral part of a
him when in need. smart city.
(iv) His friends did not help him last week. (iii) Sustainability and surveillance go hand
(ME -16) (Set 1) in hand in a smart city.
(a) (i) and (ii) (b) (iii) and (iv) (iv) There is a perception that smart cities
(c) (iii) only (d) (iv) only promote surveillance.
(ME - 16) (Set 2)
1 9. Leela is older than her cousin Pavithra, (a) (i) and (iv) only (b) (ii) and (iii) only
pavithra's brother Shiva is older than Leela. (c) (iv) only (d) (i) only
When Pavithra and shiva are visiting Leela,

\( I l 1 n., 1r1t l 1 1 11� P11iilH ,1t1c )J1, �ydcrabad l Dclhi l Bhopal l Pune l Bhubancswarl Lucknow l Patna l Bcngaiuru l Cbcnnai l Vtjayawada l Vtzag I TlfUpati I Kukatpallyl Kolkala I
: 1079 : Verbal Ability

21. Social science disciplines were in existence Which of the following can be logically
in an amorphous form until the colonial inferred from the above sentences?
period when they were institutionalized. In (GATE - PI - 16 )
varying degrees, they were intended to (a) Emperors who do not leave significant
further the colonial interest. In the time of sculpted evidence are completely
globalization and the economic rise of forgotten.
postcolonical countries like India, (b) Ashoka produced stone carved edicts to
conventional ways of knowledge production ensure that later historians will respect
have become obsolete. him.
Which of the following can be logically (c) Statues of kings are reminder of their
inferred from the above statements? greatness.
(i) Social science disciplines have become (d) A king's greatness, as we know him
obsolete. today, is interpreted by historians.
(ii) Social science disciplines had a pre­
colonial origin 23. "If you are looking for a history of India, or
(iii) Social science disciplines always for an account of the rise and fall of the
promote colonialism British Raj, Or for the reason of the cleaving
(iv) Social science must maintain of the subcontinent into two mutually
disciplinary boundaries antagonistic parts and the effects this
(ME - 16) (Set 3) mutilation will have in the respective
(a) (ii) only sections, and ultimately on Asia, you will
(b) (i) and (iii) only not find it in these pages; for though I have
(c) (ii) and (iv) only spent a lifetime in the country, I lived too
(d) (iii) and (iv) only near the seat of events, and was too
intimately associated with the actors, to get
22. Today, we consider Ashoka as a great ruler the perspective needed for the impartial
because of the copious evidence he left recording of these matters".
behind in the form of stone carved edicts. Here, the word 'antagonistic' is closest in
Historians tend to correlate greatness of a meaning to (GATE - PI - 17 )
king at his time with the availability of
(a) impartial (b) argumentative
evidence today.
( c) separated (d) hostile

jM11i@j!ji4.jjAJmiji1hifjj�)iyderabad j Delhij Bhopall Pune l Bhubaneswarl I..ucknowl Patnal BengaJurul Chennai lV\iayawada jVmg IT11Upati I Kukatpa)lyj Kolkata l
: 1080 : General Aptitude

24. "Here, throughout the early 1820s, Stuart James replied, "Maybe not, but at least I am
continued to fight his losing battle to allow the best player in my own family."
his sepoys to wear their caste-marks and Which one of the following can be inferred
their own choice of facial hair on parade, from this conversation?
being again reprimanded by the commander­ (GATE-ME - 17)(SET -l)
in-chief. His retort that 'A stronger instance (a) Mark was known to play better than
than this of European prejudice with relation James
to this country has never come under my (b) Steve was known to play better than
observations' had no effect on his Mark
superiors." (c) James and Steve were good friends
According to this paragraph, which of the (d) James played better than Steve
statements below is most accurate?
(GATE-ME - 17)(SET -1) 26. "If you are looking for a history of India, or
(a) Stuar's commander-in-chief was moved for an account of the rise and fall of the
by this demonstration of his prejudice. British Raj, or for the reason of the cleaving
(b) The Europeans were accommodating of of the subcontinent into two mutually
the sepoys' desire to wear their caste­ antagonistic parts and the effects this
marks. mutilation will have in the respective
(c) Stuart's ' losing battle' refers to his sections, and ultimately on Asia, you will
inability to succeed in enabling sepoys not find it in these pages; for though I have
to wear caste-marks. spent a lifetime in the country, I lived too
(d) The commander-in-chief was exempt near the seat of events, and was too
from the European prejudice that intimately associated with the actors, to get
dictated how the sepoys were to dress. the perspective needed for the impartial
recording of these matters."
25. Two very famous sportsmen Mark and Steve Which of the following is closest in meaning
happened to be brothers, and played for to ' cleaving'? (GATE-ME - 17)(SET -2)
country K. Mark teased James, an opponent (a) deteriorating (b) arguing
from country E, "There is no way you are (c) departing (d) splitting
good enough to play for your country. "

\( I I 11�11u < 1 111� P11hlu .1111111, �yderabad 1 Dellii 1 Bhopa) I Pune l Bhubaneswarl Lucknowl PaJna l BengaJuru l Chennai l Vuayawadal Vizag I Tirupari I Kukatpallyl Kolkatal
SOLUTIONS
which certain kinds of treatments are
One Mark Solutions unsatisfactory.
(a) If treatments are similar to those based
01. Ans: (b) on manipulation of human genes
Sol: Circuitous means of a route or journey, unsatisfactory then there is no point in
long and not direct which means indirect. trying to correct diseases in this way.
(b) If most treatments are unsatisfactory,
02. Ans: (a) then at least some treatments are
Sol: Analogy satisfactory. Therefore, these
A worker who is unemployed is un satisfactory treatments may be used to
productive just as a land which is fallow is correct diseases.
un productive. (c) To state that uncommon treatments are
03. Ans: (d) unsatisfactory would mean common
Sol: The latter sentence ' we would leave a better treatments are satisfactory, then there
planet for our children' supports the former is no need to manipulate human genes
sentence. The right word is ' conserve' to correct diseases.
(d) Correct: If available or other types of
04. Ans: (c) treatments are unsatisfactory, then
manipulation of genes could be
05. Ans: (b) allowed.
Sol: Frequency means the rate at which
something happens or is repeated. Its
07. Ans: (c)
Sol: The question refers to the country's
opposite is rarity (the quality of being rare).
problems. To call foreign technocrats back
who had been in the country before,
06. Ans: (d)
according to the author would be
Sol: The national Institute of Health has adopted
counterproductive. This implies that the
ethical guidelines regarding manipulation
foreign technocrats had not solved the
of human genes. The human genes are to be
countries problems and they did just the
manipulated only to correct diseases for
opposite.

\( l I 1i�111t t 1111 .... P11lil11 .1t1c 111, ydcrabad I Delhi I Bhopal I Punc I Bhubancswar I Lucknow I Patna I Bcngaluru I Chennai I Vijayawada I Vizag I Trrupari I Kukalpally I Kolka!a
" "" . ACE
. . .
'!.�-��FJWIIC'ft'PI Pooliacaml : 1082 : General Ability

(a) If problems are identified, then (d) Lawyer fights (argues) in the court
problems should be solved but the room
word counter productive gives ex actly Lawyer is one who argues in the court
opposite meaning. room. The obvious analogical relationship is
(b) If the foreign technocrats have gladiator fights in an Arena just as lawyer
ascertained what the countries fights in the court room.
problems were then they would be Gladiator : Arena : : Lawyer : Court Room
ex pected to have some idea of
countries problems. To state that they 09. Ans: (b)
should not be invited back does not Sol: The given sentence is a contra sentence. If
make sense. you observe the first part of the sentence
(c) If the foreign technocrats had you see an ex pression strong impression. In
ex acerbated or worsened the countries the second part of the sentence you see
problems than improving the situation understated, tentative. When you compare
then calling those foreign technocrats these two words with the ex pression 'strong
would certainly be counterproductive. impression' we understand its a 'contrast'
(d) To analyze is to separate into parts in model sentence. Moreover, the given option
order to understand. If the foreign should be synonymous to tentative,
technocrats had analyzed the countries understated. Obviously a linking word
problems then that is not necessarily a should be closer to the meanings of
reason why they should not be called Understated (mute, inex pressive, tentative
back. (not confident). So it is clear that indifferent
is the word that goes with these words. 'D'
08. Ans: (d) is the right answer.
Sol: The analogical relationship is between
person and workplace. 10. Ans: (d)
Gladiator: (man trained to fight for public Sol: Amalgamate means to unite ( two or more
entertainment in Ancient Rome) things such as two business) into one thing.
Arena: (Level area in centre of sports Amalgam; especially: to merge into a single
stadium.) body.
Q.Gladiator fights in an Arena. (a) merge (means to unite) is a synonym
(a) Dancer doesn't fight on the stage. (c) collect is also a synonym
(b) Commuter doesn't fight in the train Obviously, there is a slight difference
(c) Teacher doesn't fight in the class room between split and separate.
!lfllil@lliiiillAlibtMilliiiL� I I I I I I I I I I
ydcrabad Delhi Bhopal Pune Bhubaneswar Lucknow Patna Bcnga)uru Chennai Vuayawacb , 17.ag Tirupati I �I Kolkata
: 1083 : Verbal Ability
Split means to tear or rend apart (2) If + V2 - would/should (type 2)
Separate means to breakdown or break up. (3) If + had + V3 - would/should have + V3
Which is clearly antonym to amalgamate. (type 3)
'D'is the right choice. The verb in the if clause is in the past tense
i.e., V2(wanted). So the main clause should
11. Ans: (c) be would/should (type-2).
Sol: After verbs like stop, suggest, enjoy
contemplate take V 1 + ing form. So the 16. Ans: (d)
choice is (c) i.e., Visiting. Sol: This is a sentence in which one part of the
sentence contradicts what is said in the other
12. Ans: (a) part of the sentence. The blank is in the
Sol: Inexplicable means which can not be second part of the sentence. The first part of
explained or can not be understood. So the sentence contains a content word
clearly the choice IS 'A' 1.e. , 'seriousness' and the opposite of this word
incomprehensible. is 'nonchalance' which means an air of
Indelible means which cannot easy, unconcern or indifference.
erased/removed (a) beggary means poverty
Inextricable means which cannot be solved (b) nomenclature means name
Infallible means incapable of error (c) jealousy means envy

13. Ans: (b) 17. Ans: (a)


Sol: Latitude means freedom of action, choice or Sol: mitigate
space. (a) To cause to become less harsh, hostile or
Eg: Students are allowed considerable mollify
latitude in choosing courses. Ex: (i) aggressiveness may be mitigated
or channeled
14. Ans: (b) (ii) to make less severe, painful or
Sol: The error IS'he should be given'. The alleviate
correct way of writing the sentence is 'he (iii) extenuate
should give'
Synonyms of mitigate: allay, alleviate,

15. Ans: (a) assuage, ease, help, mollify, palliate,


Sol: The given sentence is a conditional sentence relieve, sooth and diminish
(1) If + V 1 - wi�l (type 1) So, the right choice is 'A'.

�yderabadl Delhi I Bhopal I Pune I Bhubaneswarl Lucknow I Patna I Bengaluru I Chcnnai j Vtjayawada jVizag ITuupari I Kukatpally I Kolkala I
: 1084 : General Ability

18. Ans: (d) 22. Ans: (a)


Sol: The country's ex penditure on educational Sol: When dare means "to have enough courage
reforms is very low. or to challenge some one to do something".
It can be used either as a modal verb or as a
19. Ans: (b) & (a) main verb. As a modal verb it takes "bare
Sol: The relative pronoun 'that' is used in infinitive"
preference to 'who', 'which', and after some In the given question "Dare" is used as a
words such as 'all', 'same', 'any', 'none', modal verb and it goes with bare infinitive
'nothing', 'one', 'and', 'the only', Here the (i.e.) commit.
antecedent is 'Suresh's dog' which refers to
only animal so 'that' should be used. 23. Ans: (d)
Sol: "Two and Two make four".
20. Ans: (b) The word "make" means "amount to" Two
Sol: The preposition 'despite' indicates that it is and two (amount to) or (make) four. "Two
contrasting sentence. So the sentence falls and two are four suggests that they are
under the category of contrast sentence. In
separate numbers.
the second part of the sentence the word
Grammatically both options (c) and (d) are
'succeeded' is the content word. The blank
used in different sense. Hence, the general
is obviously opposite of the content word.
The answer is 'set back' usage is Two and Two make four.·

21. Ans:(d) 24. Ans: (c)


Sol: The meaning of quarrel is fall out which Sol: Inference is an act of drawing a conclusion
means to cut off relations over a quarrel. from the given information
(a) Make out means to discover the (a) you cannot infer any information about
meaning of, to understand, to see, to the caller tune as it is a need
hear, etc. clearly.
(b) It doesn't imply anything about
(b) Call out means to summon someone to
telephone facility
leave his house to deal with a situation
(c) Correct: you can infer from this
outside.
(c) Dig out means to find something that statement as it is a need

has been hidden or forgotten for a (d) Whether you pay the telephone bills or
long time or unearth. not is the concern but it is the need
(d) Fall out means to quarrel
!M11Q@jjj4iibiMAflftiM\jjfjj� yderabad I Delhi I Bhopal I Pune I Bhubaneswar i Lucknow I Patna I Bengalwu I Chennai I Vijayawada I Vm,g ITirupati I Kukatpally I Kolkata
: 1085 : Verbal Ability

25. Ans: (a) 28. Ans: (c)


Sol: Analogy: Antonymous relationship Sol: Article
Both Universalism and particularism are The word 'European' is a noun. Articles a,
Antonyms. The obvious analogical an, are indefinite articles, that are decided
relationship 1s Universalism 1s to with the initial 'sound' of the noun.
particularism just as diffuseness (ill "European" starts with a phonetic sound Liul.
organized) is to Specificity (the condition of It is a consonant sound so it takes 'a' not
being particular individual or group of 'an'.
organism) She is a European.
Diffuseness (ill organized) The opposite to
this word is specificity (The condition of 29. Ans: (c)
being particular individual or group of Sol: The rule ofthe tense is (Ifthe main clause is
organism) in the past tense the subordinate clause also
should be in the past tense)
26. Ans: (a) Here, the latter part ofthe sentence is in the
Sol: Conditional sentence (Type - 2) past tense 'as soon as its flight plan was
Ifyou were a bird, you would fly in the sky field'. Hence, the main clause also should
(or) be in the past tense- 'was allowed to'.
Were you a bird, you would fly in the sky
"were" (past tense) = would (past tense) 30. Ans: (c)
Sol: 'personnel' means employees of an
27. Ans: (b) organization.
Sol: Nadir means:
31. Ans: (b)
(i) The lowest point
Sol: It is a contrasting sentence. These are the
(ii) The point of the celestial sphere that is
sentences in which one part of the sentence
directly opposite to the zenith (Which contradicts or contrasts what is stated in the
means the highest point) and vertically other part of the sentence. Here, the word
downward from the observer 'However' in the latter part ofthe sentence'
Ex: The relationship between the two Modem Science' contradicts what is stated
countries reached a Nadir in the 19 20 's in the former part of the sentence 'Many
Synonyms: ancient cultures'. Among the choices, it is
Bed rock, bottom, depth, Rock bottom, zero 'dispel' which has this meaning, and is the
answer.

l1i•i4ii4jjji4ijji4RfiijitWj@j*)ayderabad I Delhi I Bhopal I Pune I Bhubaneswar I ll!cknow J Patna J Bengaluru I Chennai I VijayawadaJV17.ag I Tll1lpati I Kukatpal)y I Kolkata I
: 1086 : General Ability

32. Ans: (b) (c) This statement supports the claim made
Sol: "Both" refers to two people or things and in the above statement. 'Rich heritage
saying that something is true about each of and cultural diversity has influence over
them. 'Both .. .. And' expressions go languages and thousands of dialects'.
together. Therefore, since the sentence is (d) Unwarranted (out of scope)
referring to two things i.e., communication
and interpersonal skills. 36. Ans: (b)
Option (B) is the befitting word. Sol: The sentence is with regard to fall in value
of US dollar and Indian Rupee. To meet the
33. Ans: (b) requirement of the comparison to a fall in
Sol: Conditional sentence type 1. value the right word is 'depreciated' which
In a type 1 conditional tense, the verb in the means to lower in estimation.
'if clause' is in the present tense and the
verb in the main clause is in the simple 37. Ans: (b)
future. The main clause normally has Sol: Words usually ending in 'ice' denote state,
(will/shall/can/may/must + base infinitive). action or result of an action
The modal verb is often will, which Advice is a noun. It is a suggestion or
expresses the idea of probability. The recommendation. Advise is a verb. It means
subordinate clause should be in simple to give advice and therefore refers to the act
present tense. Therefore, the 'if clause' of giving advice.
should be option 'B'.
Advice Advise
,!, ,!,
34. Ans: (c)
Sol: The right word is bear which means
Noun verb
something difficult to do or deal with and ,J, ,!,
losing means to miss from one's possession C s
or from a customary or supposed place. Pronunciation S Pronunciation Z
Thus 'she could not bear the thought of
losing the election to her bitter rival'. 38. Ans: (a)
Sol: Disagree means to fail to agree, to differ in
35. Ans: (c) opinion, to cause discomfort or distress. Its
Sol: (a) Out of scope closest meaning is dissent which means to
(b) Out of scope with hold.

!1Dl4jjajjj44ijjijjnfiiM\hM� I I I I I I I I
ydcrabad Delhi Bhopal Punc Bhubaneswar Lucknow Patna Bcngaluru Chennai ! Vtjayawada I I
11.3g Tnupati I Kukatpallyj Kolkata
: 1087 : Verbal Ability

39. Ans: (b) 43. Ans: (c)


Sol: Revert means to come or go back (as to a Sol: If you say 'The buck stops here' or 'The
former condition, period or subject). buck stops with me' you mean that you have
Therefore, the right choice is 'B' get back to to take responsibility for something and will
him. not try to pass the responsibility onto some
one else. Therefore the right option is 'C'.
40. Ans: (b) He will assume final responsibility.
Sol: Vindicated means to set free, to free from
allegation, confirm, substantiate, to provide 44. Ans: (a)
justification or defense for, justify. Its Sol: (a) Correct: It is the right answer as India
closest meaning is substantiated. Therefore, indeed was a British colony.
B is the right option. (b) Unwarranted (out ofscope)
(c) Contradicting the phrase
41. Ans: (b) (d) Unwarranted (out ofscope)
Sol: Cope with means to satisfy, to fulfill or to
meet a need. Therefore, the closest in 45. Ans: (b)
meaning to the phrase cope with is adapt to. Sol: The question has to be framed with
auxiliary verb.
42. Ans: (c) (a) This is a statement therefore, it can not
Sol: This is a contrasting sentence in which one be a question
part of the sentence contradicts what is said (b) Correct: It is the right answer as it
in the other part of the sentence. The starts with did ...
sentence states, 'He could not understand (c) In questions the word 'that' should be
why the judges were awarding her the first avoided
prize' which means he is expressing (d) Irrelevant construction
'doubt'. The first prize is usually awarded to
an excellent, high grade, first rate or top 46. Ans: (a)
notch. Here the situation is contrast to the Sol: (a) Eradicate means destroy utterly
thought expressed. Her performance may (b) Distort means misrepresents
not be up to the standard. So, the right (c) Saturate means soak completely
option is 'C' mediocre which means of (d) Utilize means use
moderate or low quality, value, ability, or Therefore the right match is option 'A'
performance, ordinary, so - so.

!ID11ijj§jjj4iiOSAflftii@jj+yderabad I Delhi I Bhopal I Pune I Bhubaneswar I Lucknow I Patna I Bcngaluru I Oiennai I Vtjayawada I Vizag I Tllllpati I Kukatpally I Kolk.at, I
"':,... . .
. ' �.," ACE
:�PoNirmn
. . : 1088 : General Ability

47. Ans: (d) C. It is the right answer. As it is


Sol: This sentence expresses ' present simple illustrated above the sentence is quite
tense'. One of the usages of simple present metaphysical in its sense.
is stating established facts and things in D. One nation of woman rights are
general it is also used for habitual activities different from the other nations.
or routines. Hence, the answer should be Sentence has nothing to do women's
A person suffering from Alzheimer's rights.
disease experiences (simple present tense)
short - term memory loss. 50. Ans: (b)
Sol: Remove "to" after verb "order".
48. Ans: (a)
Sol: 'This is sentence _completion ' 51. Ans: (c)
These are kinds of sentences in which one Sol: Primeval means from the earliest period of
part of the sentence strengthens, the history of the world, very ancient: Ex:
complements, supports or corroborates what Primeval forests.
is said in the other part of the sentences. The = of or relating to the earliest ages (as of the
latter part of the sentence states ' they are world or human history)
satisfied with what they have?' , the semi - Primitive is the closet meaning. Modem is
colon between the two sentences has the opposite. Don't get confused with historic
same meaning as the conjunctions because (= important) and historical (related to
' The word satisfied' reflects or supports the history)
former sentence. Therefore, the equivalent Antique means object of ancient time.
word to satisfied should be contentment
which means the quality or state of being 52. Ans: (b)
contended. Hence, the right option is (a). Sol: Friendship has its __limitation.
The subordinate clause "No matter how
49. Ans: (c) __ it is'' stands in apposition to
Sol: ' As a woman I have no country' ' friendship' (antecedent) so "Intimate" goes
A. It doesn't mean that women have no with friendship.
country. The expression here, it is
quite literal in its sense. 53. Ans: (c)
B. How women are not citizens of any Sol: Hint: Analogy
country? It is not taking about Medicine gives health just as
citizenship (a) Science gives experiment
l1t114!@0!14104Rfifliiih•h+yderabad I Delhi I Bhopal I Pune I Bhubaneswarl Lucknow I Patna I Bengaluru I Chennai I Vuayawada I Vizag I T111.1pati I Kukatpally I Kolkata I

ACE
�-��Ii'�
��
PublicatioDs
;_� ��
: 1089 : Verbal Ability
=====================================
(b) Wealth gives peace 58. Ans: (c)
(c) Education gives knowledge Sol: Indigent means very poor the other three
(d) Money gives Happiness words are similar m meanmg to
The relation between Medicine and health is 'vernacular'.
Thing and Purpose. Science does not give
experiment. Wealth may not give peace. 59. Ans: (b)
Money may not give Happiness but Sol: 'respectfully' means showing respect.
Education definitely gives knowledge. The
obvious analogical relationship is medicine 60. Ans: (d)
gives health just as education gives Sol: Dreary means dismal or dull
knowledge.
61. Ans: (b)
Sol: Pediatrician 1s a doctor who studies and
54. Ans: (a)
treats the diseases of children just as
Sol: Comprehension means the ability to
gynecologist is a doctor who studies and
understand.
treats the medical conditions and diseases of
woman.
55. Ans: (b)
Sol: One of the biggest virtues was his ability to 62. Ans: (b)
forgive. Grammatically after "one of the Sol: 'They close their minds' means reject or
expressions "phrase takes plural subject, so refuse to learn anything other than
A and D are eliminated straight away. Since requirement for the exam. The word
"ability to forgive" is a positive quality. 'B' 'outside' m the options should not be
is the right answer. followed by 'to'.

56. Ans: (d) 63. Ans: (c)


Sol: A. This is a contrasting statement Sol: In the first blank the word 'the' refers to the
B. out of scope film 'madras cafe' in which John Abraham
C. This is a contrasting statement acted
D. This is the right answer
64. Ans: (c)
Sol: The given sentence gives a positive sense
57. Ans: (b)
and ' she had a terrific time' means enjoyed
Sol: 'ensured' means 'make sure' or 'assure'.
herself. The other three sentences are
negative in meaning.

!M1•@i@jjj§4ijjiji,1jflftij11ih� yderabad I Delhi I Bhopal I Pune I Bhubaneswari Lucknow J Patna I Bengaluru I Chennai I Vijayawada I Viz.ag I 11I1lpati I Kukatpally I Kolkata
: 1090 : General Ability

65. Ans: (b) 74. Ans: (a)


Sol: 'wheedle' means to get (some one) to do Sol: Tuberculosis is a singular number and the
something by gentle urging special attention word 'ranks' should be followed by 'as'.
or flattery and 'round about means not
straight forward or direct. The other three 75. Ans: (a)
combinations exactly match with each other. Sol: 'Awkward'means clumsy, unskilled, inept.

66. Ans: (a) 76. Ans: ( b)


Sol: The other three are grammatically wrong. Sol: The sentence should be in simple past tense
and the verb should agree with the first
67. Ans: (c)
subject 'Dhoni'.
Sol: 'Took to their heels' or 'took to flight'
means run away.
77. Ans: (c)
68. Ans: (c) Sol: 'A' and 'B' are nouns and 'D' 1s an
Sol: In the first blank 'adverb + verb' and in the adjective
second blank 'just + don't' are the correct
verbal structures.
78. Ans: (b)
Sol: In part 'Q' it should be 'the instructor had
69. Ans: (d) marked' because the first action of the past
Sol: 'apparel' means all types of clothing where should be in past perfect tense.
as 'textile'means cloth.
79. Ans: (b)
Sol: pertinacious means determined
70. Ans: (a)
Sol: Only this sentences is grammatically correct
Clandestine means secret
and conveys the meaning properly.
Ravenous means very hungry

71. Ans: (b) 80. Ans: (b)


Sol: 'memento'is a souvenir or gift. Sol: 'If had' 1s followed by 'should
have'.
72. Ans: (a)
Sol: The sound made by frogs is croak.
81. Ans: (b)
Sol: 'A,C and D' are nouns. 'Let us' is followed
73. Ans: (c)
by a verb.
Sol: 'educe' means takeout or extract.

\( l l 1 \ .,, '!('{ I l l l� P1i!d1t ,l!Jt)Jj', yderabad I Delhi I Bhopal I Pune I Bhubaneswar I Lucknow I Patna I Bengalwu I ateimai I Vtjayawada I V',r.agJT,rupari I Kukalpa)ly.J l(.olkala
.,...
�.,
, ' .,.," . ACE
:F�PuNirP:D
. . . : 1091 : Verbal Ability

82. Ans: (a) 91. Ans: (b)


Sol: 'Between' is followed by pronoun in object Sol: Pertinacious means determined
form. Clandestine means secret
Ravenous means very hungry
83. Ans: (a)
Sol: The other three are grammatically wrong. 92. Ans: (b)
Sol: 'If. . . had' is follow�d by 'should have'
84. Ans: (b)
Sol: 'Leads to' is appropriate because it means 93. Ans: (a)
'resulting in' Sol: Before superlative article 'the' has to be
used. 'one of ' the expression should take
85. Ans: (b) plural noun and so option 'C' and 'D' can't
Sol: 'elude/elusive' means escape or avoide. be the answer.

86. Ans: (c) 94. Ans: (b)


Sol: 'fish out of water' means feel extremely Sol: ' lose' is verb.
uncomfortable and out of place.
95. Ans: (a)
87. Ans: (c) Sol: 'effectiveness' is noun and 'prescribed' is
Sol: ' Increase m productivity' should be verb. These words are apt and befitting with
followed by an 'adverb'. the word 'medicine.'

88. Ans: (b) 96. Ans: (b)


Sol: The sentence should be in simple past tense Sol: Less means not so much: smaller in amount
and the verb should agree with the first or number.
subject Dhoni.
97. Ans: (a)
89. Ans: (a) Sol: 'hanged' means death by hanging 'hung' is
Sol: 'Awkward' means clumsy, unskilled, inept. used only with things and not with people.

90. Ans: (b)


98. Ans: (d)
Sol: In part 'Q' it should be 'the instructor had
Sol: ' lying prone' means lie down flat. 'Prone to'
marked' because the first action of the past
means vulnerable to.
should be in past perfect tense.

\C I I 11�11u t 1 111� P1 i l ilH .1 1,,:1-.. yderabad I Delhi J Bhopal I Punc I Bhubancswar I Lucknow I Patna I Bcngaluru I Chennai I V\jayawada I Vtzag I Tlfllpall I '� I Kolkata
: 1092 : General Ability

99. Ans: {c) 107. Ans: {a)


Sol: Statements i and ii are not logically possible Sol: Hurtful � It is a supporting sentence. The
based on the given fact. word 'sharp tongue' strengthens the latter
part of the sentence 'it can occasionally
100. Ans: {c) tum hurtful'
Sol: In degrees ofcomparison Mr. X is taller
than Mr. Y is apt. 108. Ans: {d)
Positive degree tall Sol: It is a strengthening sentence. One part of
Comparative degree taller the sentence i.e., "Two speakers became
Superlative degree tallest increasingly agitated" strongly supports the
latter part of the sentence "the debate
101. Ans: {b)
became heated"
Sol: Felicitate means honour.

109. Ans: {d)


102. Ans: {c)
Sol: 'One of the ex pression always takes
Sol: 'rest is history' is an idiomatic ex pression
'plural noun' that is 'friends' and 'keenly'
which means 'rest is well known
is an adverb which modifies the verb felt.
So the right option is 'D'.
103. Ans: {c)
Sol: In if clause (type2) 'were' is in the past
110. Ans: {a)
tense so the so main clause should be in the
Sol: Choosing plan 'P', you will have to forgo
conditional clause. Therefore 'C' is the
plan 'Q'. Both are different plans so they
best answer
are ex clusive.

104. Ans: {a)


Sol: 'turned a deafear' means ignored 111. Ans: {c)
Sol: Subject verb agreement 'Two ways' is
105. Ans: {c) plural so the verb also should be plural
Sol: Where there is a will there is a way. It is a (are)
quotation

106. Ans: {a)


Sol: Conditional tense Type 3 - Past perfect
(could have) + perfect conditional {had +
V3 )

lffl•li@jjji4.jji1RflnjMijjjjj+yderabad I I I I I I I I I
Delhi Bhopal Pune Bhubaneswar l Lucknow Patna Benga)uru Chennai j Vtiayawada Vizag Tuupali I Kukatpal)y I Kolkala I
t ACE
.!''!i..:'�Pubincm : 1093 :
� � =====================================
Verbal Ability

03. Ans: (d)


Two Marks Solutions Sol: Option A
Failure is the pillar of success
01. Ans: (d) It is probably true but it doesn't support
Sol: The question 1s about inference which completely the above information.
means the reasoning involved in drawing a Option (B)
conclusion or making a logical judgment on Honesty is the best policy. We can not
the basis of circumstantial evidence. assert this statement from the given passage
as it doesn't talk about "Honesty".
(a) There is no mention of gender in the
Option C
given advt. so no question of gender
Life begins and ends with adventures
discrimination.
This statement doesn't go with the passage
(b) xenophobic means fear of foreigners.
as it doesn't mention about adventures.
(c) Day, Evening and Saturday work
indicate that it is designed to make Option D
attractive. Correct: No adversity justifies g1vmg up
(d) Correct: The Advt.is not talking about hope
gender. The passage talks about 'Robert Bruce' and
spider's not giving up hopes in adverse
02. Ans: (d) situations. It can be inferred that no
Sol: "Inference" 1s an act of drawing a adversity justifies giving up hope.
conclusion from the given information. We
04. Ans: (d)
can draw the conclusion clearly from the
Sol: According to the given paragraph, it is not
words 'different streams- moderates,
mandatory that the number of steps of the
liberals, radials, socialists, and so on' .
ladder should be an even number which
(A) It doesn't support the above statement
results an equal number of (Red-Blue) and
(B) It doesn't support the above statement
(Green-Yellow) lights. So 'D' is not
(C) It is definitely false as it is just
necessarily true.
opposite to the argument.
(D) It can be inferred from the given 05. Ans: (c)
information. i.e., different streams is Sol:
equal to heterogeneous. (a) The passage states that the exact
reasons for the formations of the
Palghat gap are not clear but the
!M11i@jjj44ijjiAflajiM\jjijj+yderabad l DclhilBhopail Pime! Bhubaneswarl Lucknow l Patnal Bcngaluru i ChennailVijayawada lV17.ag ITirupati I Kukalpally i Kolkata l
: 1094 : General Ability

options states the Palght gap is caused (b) Correct: This is the right inference and
by high rainfall and. ... It 1s now it is a part ofRussia
contradicting the statement. (c) Kaliningrad is not original Russian name
(b) It is the Palghat gap that is low-lying (d) Out ofscope (unwarranted)
not the regions in Tamilnadu and
Kerala. 08. Ans: (d)
(c) Correct: The Palghat gap as Sol: It is a strengthen the argument question
significant impact on weather patterns (a) Unwarranted (out of scope) it doesn't
in neighbouring parts of Tamilnadu support the conclusion. The Passage
and Kerala. doesn't mention 'neighbouring
(d) The cause of the Palghat gap 1s not countries'
known. (b) Contradicts the conclusion
(c) Contradicts the conclusion
06. Ans: (b) (d) Correct: This is the right answer as it
Sol: Assumption is an un stated idea that supports the conclusion. Municipal
supports the conclusion. authorities have concluded that
(a) The sentence doesn't support the measures to control the mosquito
conclusion as it contradicts the population have failed in this region.
statement i.e. the passage states the
strategies are now available 09. Ans: (c)
(b) Correct: It supports the conclusion. Sol: (a) It is a general statements of twenty first
Therefore, this is the right assumption. century but the paragraph's main point is
(c) The word 'All human diseases' is the merits ofexactitude
ambiguous as the passage mentions (b) Big data changes our thinking.
only two diseases namely 'depression Contradicting the statement.
and schizophrenia'. It leads to fallacy (c) It is the right answer. This is the main
called generalization. point ofthe agreement.
(d) The word 'only' 1s ambiguous. (d) This is not the main point of the
Therefore, it doesn't support the argument. It is partial information.
conclusion.
10. Ans: (a)
07. Ans: (b) Sol: The keyword in the statement 'since' which
Sol: (a) Contradicting the passage - Kaliningrad in this context means 'because'
was never historically Russian.
!M11i@jjj44ijj/QRflbfti\d.jj� ydcrabad J Dellii J Bhopal J Punc J Bhubaneswar J Lucknow J Pama J Bcngaluru I Chennai J Vrjayaw.ida I VJZag f Tllllpati I Kukatpal)y J Kolka.ta
: 1095 : Verbal Ability

11. Ans: (b) 17. Ans: (b)


Sol: Statement (i) and (iv) cannot be inferred Sol: This paragraph is based on time sequence.
from the passage because he was not Hence, the sentences should be organized in
already a successful bats man and a chronological order clock wise.
improving his skills may not guarantee his
success. 18. Ans: (b)
Sol: The words 'was confident that they would
12. Ans: (a) reciprocate' and 'last week proved him
Sol: (I) and (II) are correct because the course wrong' lead to statements iii and iv as
of action taken by the government logically valid inferences.
should be positive and should not cause
any inconvenience to the people. 19. Ans: (d)
Sol: From given data, the following arrangement
13. Ans: (b) is possible
Sol: The key word to solve this question 1s Shiva
'sometimes' Leela
Pavithra
14. Ans: (d) Among four alternatives, option D 1s
Sol: Please see the second sentence "an ignore TRUE.
reality . . . security".
20. Ans: (c)
15. Ans: (b)
Sol: A close reading of the passage would show 21. Ans: (a)
that only statements (i) and (iv) are logically Sol: · Until the colonial period means pre-colonial
valid. origin. Other options can't be inferred.

16. Ans: (b) 22. Ans: (d)


Sol: Ifyou look into the given statement there is Sol: 'Today, historians correlate greatness of a
a gap of one day between 20/5/ 1 4 and king at his time with the availability of
22/5/20 1 4. This means Ms. X will be in evidence.' This statement leads to the best
Kochi for only one day in may. Hence, the inference option 'd'.
answer is 'B'.

!11•1iii§jjjiiih4Pffbftimijj� yderabad I Delhi I Bhopal I Pune I Bhubaneswar I Lucknow I Patna I Bengaluru I Chennai I Vijayawada I Vizag !Tlfllpati I Kukatpally I Kolkata
ACE : 1096 : General Ability
� ���
�������============================�
�•.�...............Publicalicm

, 23. Ans: (d) 25. Ans: (b)


Sol: 'Antagonistic' means showing dislike or Sol: When James says to Mark that at least I am
opposition. So the word closest in meaning best player in m family, it implies that Mark
is 'hostile' (not friendly, having or showing is not best player in his family. Which
unfriendly feelings, unpleasant or harsh) means Steve played better than Mark.

24. Ans: (c) 26. Ans: (d)


Sol: The key word is 'losing batter' Sol: Cleaving means to divide by or to separate
into distinct parts.

!IJ•IM..YihMhiiRflfiiMihih� yderabad I Delhi I Bhopal I Pune I Bhubaneswarl uicknow l Pa!nal Bengaluru I Chennai I Vuayawada l Vu.ag ITllllpati I Kukatpally l Kolkata
No. of selections 17
H EARTY CO N G RATU LATI O N S TO OU R

I ES-201 5 RAN K E RS
- Total No.of selections in IES 2015 - EC:52 EE:36 CE:24 M E:28 -
From Pain to Prosperity . . .
The Journey of Life
- G.K Sir

A science teacher was taking a biology class. She brought two silk-worm cocoons and told
the students to observe how the silk-worm gradually emerges from the cocoon. She went
out of the classroom for sometime. The students were keenly watching the emergence of
the silkworms from the cocoons. It is a very slow and laborious process for a silkworm to
come out of the cocoon. So, the two worms were labouring to come out. One of the students took pity on
them and though the others tried to prevent him, he went on to slowly snap the strings of the cocoon. He
thought that he was making it easy for the silkworm.
At that moment, the teacher returned to the classroom and when she came to know about what had
happened, she asked all the students to keep observing for some more time. Oddly, the silkworm which
was helped by the student died, and other one, which went through a lot trouble to come out of the cocoon
was moving about actively.

The teacher then explained to the students how slow and painful it is for silkworms to come out of
the cocoon. It takes a lot of time for them to adjust to the conditions outside. As they keep snapping the
silk threads that hold them back, their body gains strength and energy. But this process takes a very long
time and evokes pity in the onlookers. Even then, we should allow them to use their own strength, and
trying to help them would put their lives in danger. The teacher then cited Darwin's Theory "survival of
the fittest" to say that only the capable can survival in this world and seeking sympathy from others will
only weaken us and destroy our self image.
Life is not about sympathy-seeking or self-pity. Life is all about possessing a tenacious mind-set
to achieve success. They can conquer who believe they can. Let us recollect our childhood days. When we
were learning how to walk, we fell down many times. But did't we get up and walk again? If we had not
tried to getup, all of us would be crawling today. In many sports and games, the sportspersons get injured
several times. But it is their commitment and determination that makes them come back successfully and
become a Tendulkar, a P.T. Usha, a Saina or a MaryKom. Let us not seek pity from others. Let us
empower ourselves. Learn to put in your best and take every disappointment as a challenge. There is no
elevator or escalator to success. There are only steps. Start climbing them and don't look back. Any man
of achievement is highly respected because of his moral courage and hard work. The future beckons you.
Go and Enjoy it.
Remember - Life is like a rose. Every rose has a thorn: but aren't the roses sweet?
*** GOOD LUCK ***
"Every great success is preceded by a long
period of hard work with clear Goal in mind"

"The tragedy in life doesn 't lie in not


reaching the goal, The tragedy lies in having
no goal to reach "

You might also like